You are on page 1of 964

AIR 2008 SUPREME COURT 2585 "Indian Bank v.

Godhra Nagrik Co-operative Credit


Society Ltd."
(From : Gujarat)*
Coram : 2 S. B. SINHA AND LOKESHWAR SINGH PANTA, JJ.
Civil Appeal No. 3303 of 2005 with C. A. Nos. 3336, 3337, 3338 and 3304-3335 of 2005,
D/- 16 -5 -2008.
Indian Bank v. Godhra Nagrik Co-operative Credit Society Ltd. and Anr.
(A) Constitution of India, Art.226 - WRITS - BANKING - CONTRACT - Writ
jurisdiction - Nationalised Banks - Not amenable to writ jurisdiction as regards
enforcement of contracts.
It is one thing to say that the Public Sector Banks having regard to the provisions of the
Banking Companies (Acquisition and Transfer of Undertakings) Act (1970) should
discharge their functions keeping in mind the larger public interest but ordinarily in the
matter of enforcement of contract, they are to be governed by the terms thereof, which
would not be amenable to writ jurisdiction of the High Court unless the actions of the
Banks are found to be wholly arbitrary and unreasonable. (Para 10)
(B) Constitution of India, Art.226 - WRITS - BANKING - Writ petition - Against refusal
by Bank to encash FDRs on maturity - Encashment refused as Bank suspected fraud to
have been committed against it - Question whether its officers alone or holders of FDR
also were responsible for fraud, in dispute - Entertainment of writ petition in
circumstances was improper - Court ought to have considered question whether action of
Bank was so arbitrary as to invoke public law jurisdiction - Considering disputes
involved and right of genuine FDR holder - Necessary directions issued to Bank.
L.P.A. No. 258 of 2000, D/-01-10-2003 (Guj). Reversed. (Paras 16, 17, 19, 21)
Cases Referred : Chronological Paras
2005 AIR SCW 2676 : AIR 2005 SC 2821 (Ref.) 17
@page-SC2586

2005 AIR SCW 4535 : AIR 2005 SC 3454 (Ref.) 17


(2004) 3 SCC 553 (Ref.) 17
2003 AIR SCW 6039 : AIR 2004 SC 561 (Rel. on) 17
1995 AIR SCW 2834 : AIR 1995 SC 1811 (Ref.) 17
(1995) 2 AC 500 : (1995) 3 All ER 918 10
AIR 1993 SC 1407 17
AIR 1991 SC 247 : 1990 All LJ 872 (Disting.) 17
AIR 1973 SC 205 (Ref.) 17
1915 AC 705 : 113 LT 195 (HL) 10
P. P. Rao, Ajay Kr. Mishra, Sr. Advocates, Pramod B. Agarwala, Ms. Praveena Gautam,
Darshan Partkh, Nitin Kant Setia, Himanshu Munshi, N. R. Parikh, Rajesh Kr. Chaurasia
and Vikram, for Appellant; C. A. Sundaram, Sr. Advocate, B. S. Patel, Aniruddha P.
Mayee, Ms. Rucha Mayee, K. S. Rathod, Abhishek Gupta, Shabu Sreedharari, B. V.
Balaram Das, V. Shekhar, S. Wasim A. Quadri and Gaurav Agarwal, for Respondents.
* L.P.A. No. 258 of 2000, D/- 1-10-2003 (Guj).
Judgement
1. S. B. SINHA, J. :- These appeals involve an interesting question as regards the power
of judicial review of a Superior Court.
2. Respondents herein are Co-operative Societies registered under the Co-operative
Societies Act and/or their Members.
They deposited certain amounts in cash in fixed deposits of Banks wherefor Fixed
Deposit Receipts (FDRs) were to be issued. Such deposits were made through some so-
called Commission Agents of the Banks on payment of huge commission which is
ordinarily not allowed by the Nationalized Banks.
3. Applications for grant of loans by various persons were filed before the prescribed
authorities of the Banks on the basis of the said FDRs. Allegedly a large number of
officers of the banks were involved in a scam whereby unofficial investments of the said
amount were being made.
4. As and when the FDRs matured, the investors requested the Banks for their
encashment. The Banks refused to accede thereto stating that the amount under the FDRs
had already been paid by way of loans and, thus, no further amount was payable. It was
contended that a fraud on the banks has been practiced to which the depositors and the
officers of the Banks were parties.
5. Writ petitions were filed. A learned Single Judge of the High Court opined that serious
disputed questions of fact being involved in the said writ petitions, no relief can be
granted to the writ petitioners.
6. Despite the same, the learned single Judge relying on the provisions contained in
Section 35A of the Banking Regulations Act, 1949 directed constitution of a Committee
under the Chairmanship of the Deputy Governor of Reserve Bank of India or his nominee
to go into the matter in great details. Various powers were delegated in favour of the
Committee including the one that the decision of the Committee shall be final and
binding upon the parties.
A Division Bench of the said Court in an intra-court appeal preferred thereagainst,
however, stayed only the operation of some of the clauses of the said order. The
Committee, however, was allowed to function.
A special leave petition filed thereagainst has been dismissed by this Court with certain
observations.
7. The Committee submitted its report. It was found that principally the officers of the
Banks were involved in the matter of commission of the alleged fraud on the Banks.
Members of the Committee, however, differed in their opinion as to whether, having
regard to the limited scope of the enquiry, any positive direction could be issued.
8. Relying on and/or on the basis of the report of the Committee, the Division Bench of
the High Court opined that as the writ petitioners were not parties to the fraud, subject to
any other or further orders that may be passed in the criminal case, appellant-Banks
should be directed to pay the amounts under the FDRs to the depositors.
9. Appellants are, therefore, before us.
On 5th April 2004, a limited notice was issued by this Court, which is to the following
effect :-
"Issue notice on the special leave petition limited to the question as to whether the High
Court should have directed payment having regard to the fact that the Committee itself
had not finally resolved the question of liability as far as the disputed amount was
concerned.
Issue notice on the prayer for interim relief also."
This Court in its order dated 10th December, 2004 explained the said order stating :-
"The issue which is now required to be resolved is a narrow one viz. whether the
Committee had finally decided that the
@page-SC2587
amounts payable by the Bank (a) were the liability of the Bank, and (b) if so, what was
the quantum if any, payable by the Bank to the deposits. Learned counsel appearing on
behalf of the respondent prays for time till after the vacation.
Let the matter appear two weeks after reopening on a miscellaneous day.
There will be interim order staying the operation of the impugned order." However, by an
order dated 9th May, 2005, upon hearing the counsel for the parties, 'Leave' was granted,
as a result whereof all the contentions of the parties are now open.
10. Mr. P. P. Rao, learned Senior Counsel appearing on behalf of Bank of Baroda would
submit :-
(i) In a writ petition involving private dispute, no direction for payment of money in
favour of the writ petitioners should have been issued by the High Court;
(ii) As the writ petitions involved serious disputed questions of fact, the High Court
should not have entertained the same;
(iii) Having regard to the fact that the Central Bureau of Investigation has since submitted
a charge-sheet wherein not only the officers of the Banks but also the commission agents
have been found to be guilty of an offence of conspiracy in committing fraud on the
Bank, the judgment and direction of the High Court should be set aside.
Mr. Sundaram, learned Senior Counsel, appearing on behalf of the respondents, on the
other hand, would contend :
(a) Although the writ petitions were filed for enforcement of a contract, as the same
involved public law character, the writ petitions were maintainable.
(b) Appellants being 'State' within the meaning of Article 12 of the Constitution of India,
they were amenable to writ jurisdiction of the High Court.
(c) The writ petitions having not been dismissed in limine, the High Court was entitled to
go into the merit of the matter for the purpose of arriving at a finding as to whether any
case has been made out for issuance of any writ, direction or order.
(d) In a case of this nature the High Court is entitled to convert a private dispute into a
public interest litigation, the same having a public ramification by appointing a
Committee and act thereupon, as a consequence whereof, relief in favour of the writ
petitioners/respondents could be granted.
The propositions of law which are undisputed are :-
i) Writ Petitions against the Banks being 'State' within the meaning of Article 12 of the
Constitution of India were maintainable;
ii) Writ Petitions involving serious disputed questions of fact, ordinarily should not be
entertained although the High Court in some cases may enter into disputed questions of
fact.
The question, however, is as to whether the learned Single Judge, despite holding that the
writ petitions were not maintainable, could have issued direction for constitution of the
Committee.
The powers and functions delegated to the Committee were wide, by reason whereof for
all intent and purport, even judicial power were delegated.
Let us now consider the question as to whether direction to constitute such a Committee
was legally permissible. Indisputably, the authorities of the Reserve Bank of India in
exercise of their statutory powers conferred upon them under Section 35A of the Banking
Regulations Act could issue directions for initiating an enquiry into the affairs of the
Banks. The Banks being public sector undertakings could themselves do so and have in
fact done so.
It is one thing to say that the Public Sector Banks having regard to the provisions of the
Banking Companies (Acquisition and Transfer of Undertakings) Act, 1970 should
discharge their functions keeping in mind the larger public interest but ordinarily in the
matter of enforcement of contract, they are to be governed by the terms thereof, which
would not be amenable to writ jurisdiction of the High Court unless the actions of the
banks are found to be wholly arbitrary and unreasonable.
The core question which arises for consideration in the writ petitions was as to whether,
keeping in view the apprehension in the mind of the Bank that it has been subjected to
fraud by its own officers as also the apprehension in their mind that the writ petitioners or
their agents might have conspired with the officers of the Banks, was it unfair and
unreasonable in its decision to refuse to make payment? The answer to that question
prima facie must be rendered in
@page-SC2588
the negative. If, however, it is found as of fact that the writ petitioners-respondents were
not parties to the fraud, whether even in a lis involving private law domain, namely
contract qua contract, as a trustee of the investors' money, they may be held to be liable to
refund the amount, is the question?
Indisputably, whether as a public sector undertakings or otherwise the Banks cannot
refuse to accede to the just demand of the investors to pay any amount lawfully due to
them inter alia on the premise that their officers are guilty of commission of any fraud.
It is one thing to say that fraud has been committed by their officers to cause wrongful
loss to the Bank but it is another thing to say that the banks are constructively liable for
the acts of their officers.
In given cases, the employers are constructively liable for acts of negligence on the part
of their employees.
The Alter Ego approach adopted in the theory of Corporate liability which has been
applied by the House of Lords in Lennard's Carrying ? Ltd. v. Asiatic Petroleum Co. Ltd.
[?] AC 705 HL, is one such instance. The facts of the case concerned a cargo claim which
Lennards sought to defend by contending that Section 502 of the Merchant Shipping Act
2894 exonerated the owner from losses arising without his actual fault. The House of
Lords held that they could not rely on that defence since the fault of the appropriate organ
such as the Board of Directors or Managing Director could be attributed to the company.
In Farrar's Company Law, 4th Edn. Page 147, it is stated :
"An employee who acts for the company in the course of his or her employment will
usually bind the company and his or her knowledge will be attributed to the company
because he or she is the company for the purpose of the transaction in question.
This is so even if the employee is acting dishonestly or against the interests of the
company or contrary to orders but it is not so where the company is the victim. This is to
avoid an obvious contradiction." Another instance of the application of the theory of
Corporate Liability is the 'Attribution Approach' as adopted by the Judicial Committee of
the Privy Council in Meridian Global Funds Management Asia Ltd. v. Securities
Commission [1995] 2 AC 500, [1995] 3 All ER 918.
In that case, two employees of Meridian, had improperly used their authority to purchase
in the name of the company a substantial interest in Euro-National Corp. Ltd., a New
Zealand listed company. Under the New Zealand Securities Amendment Act, 1988,
Meridian was required to give notice of its acquisition to ENC and the Stock Exchange.
The two employees knew this but the Board and the Managing Director of Meridian did
not. No notice was given. The Privy Council upheld the New Zealand Court's decision
holding that Meridian had contravened the law, on the premise that the knowledge of the
employee would be attributed to Meridian.
We will, thus, assume for the purpose of this case that the Banks are constructively liable
for acts of their employees. We will also assume that the Banks are liable to pay the
amount under the contract for which the FDRs were issued.
11. The main question, however, would still remain as to whether in a case of this nature
(which would in turn depend on the finding of fact) as to whether the petitioners
themselves or their agents being party to the fraud, any direction in the public law domain
can be issued. The larger question would be as to whether the writ petitions having not
been found to be maintainable being purely of private law character, the High Court
could treat it to be one involving a public law domain and could still have private law
remedy available to the writ petitioners?
A writ petition indisputably would be maintainable even in relation to a matter arising out
of contract qua contract.
12. As has been submitted by Mr. Sundaram that some cases may start on a private
interest but if the court finds involvement of a public law element therein concerning a
large number of people, it may proceed on the basis as if it was a public interest litigation
and appoint a Committee and then grant relief in favour of the writ petitioners. Whether
such an extraordinary case has been made out herein is the question.
Respondents are co-operative societies. They at the instance of some agents or the
middlemen thought it expedient to invest in Fixed Deposit Receipts (FDRs) in Bank of
Baroda and Indian Bank. The modus operandi
@page-SC2589
appears to be that the brokers/intermediaries lured a few co-operative banks/ credit
societies for placing deposits with the branches of Bank of Baroda/Indian Bank in and
around Surat as also at Bharuch wherefor a handsome commission/incentive ranging
from 3.8 to 25 per cent used to be given. The Co-operative Societies themselves did not
approach the Bank. All acts were done through the agents. The documents seem to have
been sent through brokers who also delivered to them the Maturity Value Certificates in
lieu of original FDRs or the FDRs/Xeroxed copies, in addition to delivery of drafts for
commission. The commission used to be paid in cash. In some cases, the original FDRs
were retained by the Banks. However, the original documents sent by the Co-operative
Societies to the Banks in some cases were not available in the offices of the bank but a
different set of documents were replaced in the files. Loans were raised against FDRs on
the basis of such documents mainly for further investment by private individuals. Funds
were withdrawn in cash either directly from their current accounts or by issuing cheques
in favour of some individuals who reportedly discounted their cheques.
13. The fact that the officers of the Banks were involved in the entire dealings is not in
dispute. It is furthermore not in dispute that some brokers/commission agents were also
involved. To what extent, the authorities of the co-operative banks and/or co-operative
societies were involved and/or in know thereof, however, is not very certain.
Respondents-Co-operative Societies prayed for issuance of a writ of or in the nature of
mandamus or any other appropriate writ, order or direction quashing and/ or setting aside
the letter at Annexure-A dated 2-4-98 of respondent No. 1. It was furthermore prayed that
the respondent Nos. 1 and 2 be directed to honour the FDRs copies of which were
annexed thereto with the interest accrued thereon and to restrain the respondent No. 1 to
appropriate the proceeds of the said FDR and also be further directed to honour the FDR
with interest accrued thereon on such terms and conditions which the Hon'ble Court
deems fit.
The learned single Judge issued the following directions :
"I. A Committee is hereby constituted as under :-
(i) The Deputy Governor of the Reserve Bank of India himself or his nominee as the
Chairman of the Committee.
(ii) One member to be nominated by the Deputy Governor of the Reserve Bank of India
who shall be the officer of the highest rank in the Reserve Bank of India but subordinate
to Dy. Governor.
(iii) One member from the Bank of Baroda/Indian Bank to be appointed by the Board of
Directors of the concerned Bank preferably a Chief General Manager or an officer not
below the rank of General Manager.
II. The member from Bank of Baroda would participate in the meeting of this Committee
only when the cases relating to the Bank of Baroda are taken up and the member from the
Indian Bank shall participate in the meeting of the Committee only when the cases
relating to Indian Bank are taken up.
III. The Committee may evolve and follow its own procedure and will also have the
power to examine summon or examine the witnesses.
IV. This Committee shall examine each and every case on its own merits with reference
to the records desired to be made available and will give its findings with regard to the
amount due and payable to the concerned petitioners/parties and the rate of interest. The
Bank of Baroda and Indian Bank shall inform the Committee in writing about the
undisputed amount with full details. This Committee shall also go into the question of the
rate of interest payable in case of the undisputed amount, which is directed to be paid
under this order. It will also be open for the Committee to opine as to who were the
officers/employees/party responsible for this conspiracy and fraud.
V. The findings as may be given by the Committee shall be binding on both the sides and
shall be implemented forthwith.
VI. The petitioners/parties on receipt of any amount as a result of the finding of the
Committee as aforesaid, shall also give an undertaking to the concerned Bank before the
amount is withdrawn, that in case as a result of CBI inquiry, it is found and held by the
concerned Court after the trial that any amount had been withdrawn by any such party, as
a part of the aforesaid conspiracy, etc. they will return such amount to the concerned
Bank.
@page-SC2590
VII. The Committee shall decide all these cases within a period of three months from the
date the certified copy of this order is produced before the Dy. Governor of the Reserve
Bank of India.
VIII. It will be open for the petitioners/ parties to agitate their grievance, if any, against
the orders which may be passed as a result of the findings of the Committee constituted
under this Court's order as aforesaid.
IX. These directions are in addition to and not in derogation of any legal remedy, which
any party may seek after the report of this Committee.
X. For the time being, the concerned Banks, i.e. Bank of Baroda/Indian Bank shall
disburse the undisputed amount, if any, with interest at the rate of Savings Bank Account
to the respective petitioners/parties within 15 days from the date the copy of this order is
produced before the concerned Bank. The payment shall be made through Demand Drafts
in the name of the petitioners Societies/Banks/Depositors. However, if the Committee
decides any rate of interest higher than the Savings Bank Account, the consequences shall
follow."
The learned single Judge opined that there existed disputed questions of fact in respect
whereof no definite finding could be arrived at having regard to the modus operandi of
the persons involved.
14. The private dispute between the parties were, thus, sought to be converted into a
public interest litigation for the purpose of making an enquiry into the affairs of the Bank
by a Committee.
The Committee was consisted of the following members :

"(a) Mr. JR Prabhu Banking, Ombudsman, Mumbai - Chairman


(b) Mr. VS Das, Regional Director, RBI, Ahmedabad - Member
(c) Mr. RV lyar General Manager, Bank of Baroda Or - Member
(d) Mr. S. Arunachalam,General Manager, Indian Bank. - Member

As per the judgment of the learned single Judge, the Committee consisted of three
members. When the Committee dealt with the cases of Bank of Baroda, only the
representative of Bank of Baroda acted as a member and when the Committee dealt with
the cases of Indian Bank, only the representative of the Indian Bank acted in the said
capacity. The other members of the Committee were Shri JR Prabhu, Banking
Ombudsman, Mumbai and Mr. VS Das, Regional Director, RBI, Ahmedabad.
15. We have been taken through the report submitted by the Committee. The Committee
did a yeoman job. It went into various aspects of the matter. It tried to cover as much
ground as possible. It noticed the facts leading to setting up of the Committee. It
considered the written submissions as also the oral submissions of the appellant and the
submissions made by the respective Banks, the officers of the Banks, the intermediaries
as also the actions taken by the Banks concerned.
Whereas general observations and recommendations by all the members appear to be
unanimous, no unanimity however could be reached in regard to the question as to what
direction could be issued in the matter.
Whereas the Chairman and Regional Director, RBI were of the opinion that the Banks
should refund the disputed amounts of deposit to the depositors; other two members
representing the Banks were of the view that the return of the deposits may amount to
double payments to the depositors.
It was opined :
"The Committee is required to examine each and every case on its own merits with
reference to the records and its findings in regard to amount due and payable to the
concerned petitioner-borrowers in terms of the terms of reference based on the orders of
the Gujarat High Court. The modus operandi in perpetrating the fraud in respect
@page-SC2591
of all the petitioner-depositors has been the same. There have been only minor variations
here and there. Couple of petitioner-depositors has mentioned during their deposition
before the Committee that some of them had gone to the Banks' branches along with the
middlemen/intermediaries for placing the deposits. The photographs and specimen
signature of all the authorized signatories of the co-operative societies/co-operative
Banks were not available on the Banks' record. The Banks' records also do not reveal that
the authorized signatories had signed the account opening forms and the loan documents
in the presence of the officials of the Banks. The resolutions purported to have been
passed for the purpose of availing of loans by the co-operative societies/ co-operative
banks have not been on their letterheads in almost all cases. In view of these reasons it
has not been considered necessary to differentiate the cases of the petitioners and the
Committee's recommendations are uniformly applicable to all the petitioner - depositors.
A Statement indicating the disputed and undisputed amounts submitted by the two Banks
to the Committee is given in Annexure-3. The undisputed amount deposited with the
Gujarat High Court by Bank of Baroda amounts to Rs. 3.16 crore and interest thereon at
Savings Bank rate Rs. 29.02 lakh. An undisputed amount of Rs. 16.20 lakh has been
remitted to the Income Tax Authority. The disputed amount of deposits aggregate Rs.
20.00 crore. In the case of Indian Bank, the undisputed and disputed amounts of deposits
aggregate Rs. 72.85 lakh and Rs. 4.45 crore respectively. The Bank has stated that it has
deposited the undisputed amount of Rs.72.85 lakh with interest of Rs. 1.66 lakh in the
Court.
The refund of the deposits to the petitioner-depositors by the Banks should, however, be
subject to certain terms and conditions which are as under :
a) The co-operative societies/co-operative banks should execute necessary documents as
per the Banks' procedure.
b) The Banks could take indemnity bonds from persons acceptable to them apart from the
co-operative societies/co-operative banks before effecting the refund of the disputed
amount of deposits.
c) In case at a later date for any reason the amounts are required to be refunded to the
Banks by the co-operative societies/cooperative banks they will have to pay interest at the
prime lending rates of the Bank concerned compounded at half yearly rest from the date
of refund of the deposits by the Banks till the date of repayment by the co-operative
societies/co-operative banks to the two Banks."
Unanimity, however, was arrived at that no interest on the amount of deposits would be
payable. It opined that the officers of the Banks were primarily responsible for
perpetration of fraud.
Mr. Rao had taken us through various parts of the report as also the charge-sheet
submitted by the C.B.I. to contend that involvement of the brokers/commission agents
appointed by some of the co-operative societies has also been found both by the
Committee as also by the C.B.I. It was also submitted that in any event most of them
were aware of the illegal transactions which had been going on and that they had directly
or indirectly connived with the officers of the Banks in respect of their activities, which
would be apparent from the fact that they used to get a large amount by way of interest, a
portion of which was paid in cash and which was not even accounted for.
Our attention has also been drawn to the charge-sheet filed by the Central Bureau of
Investigation, wherein Jyotiben, the agent of the co-operative societies/bank has also
been made an accused' One of the co-operative societies, it was pointed out, had even
authorized her to enter into negotiations with the Bank.
In the said charge-sheet, it was stated :
"It has been stated by the executive and office-bearers of the society that, we have
appointed Jyotiben as the agent for the purpose of depositing and withdrawing the F.D. at
Surat and to give loan on the F.D. Receipts etc. It is the say of this Jyotiben that, since
there is a big lobby of industrialists and builders in Surat city, if they are given the loans
on our deposits receipts, they are paying us 24% interest and on the other side, we are
getting 11% interest, and thus, since we have been getting 35% interest, the deposits were
made through them." Mr. Sundaram, on the other hand, submitted that the fact that it was
only the officers of the Banks who have been found to be primarily liable and their
modus operands was
@page-SC2592
to grant loan utilizing the said FDRs wherefor the co-operative societies had no role to
play.
It was urged that in view of the fact that the officers of the Banks have been found to be
liable, the co-operative societies should not be punished.
16. A writ court exercising the power of judicial review has a limited jurisdiction. A writ
petition would lie against a State within the meaning of Article 12 of the Constitution of
India. Indisputably, exercise of jurisdiction by the High Court is permissible in a case
where action of the State is found to be unfair, unreasonable or arbitrary. The question
which should have been posed by the High Court was as to whether the action of the
Bank was so arbitrary so as to invoke the public law jurisdiction. If the answer to the said
question was to be in the negative, the High Court should have refused to exercise its
jurisdiction.
A fraud has been practiced on the Banks. Primary accused may be the Bank officers but a
conspiracy with them by the outsiders has also been alleged. The original FDRs only in
some cases are available; in most of the cases they are not. Even the Committee could not
decide for as to which one was the original FDR and which was not. It could not
distinguish between an original FDR and the Xerox copy thereof.
Opinion of the expert thereon might have been received, but the final verdict thereupon in
the cases initiated by the C.B.I. is still awaited.
17

. The law as regards application of the power of judicial review, inter alia, in the
contractual field stands covered by a large number of decisions. (See LIC of India and
Anr. vs. Consumer Education and Research Centre and Ors., [(1995) 5 SCC 482];
Sanjana M. Wig (Ms.) vs. Hindustan Petroleum Corpn. Ltd. [(2005) 8 SCC 242]; ABL
International Ltd. and Anr. vs. Export Credit Guarantee Corporation of India Ltd. and
Ors. [(2004) 3 SCC 553]; The D.F.O, South Kheri and ors. vs. Ram Sanehi Singh [(1971)
3 SCC 864]. We, however, do not think that facts involved in each case and the law laid
down therein need to be discussed at length as there does not exist any dispute in regard
to basic principles laid down therein. 1995 AIR SCW 2834
2005 AIR SCW 4535
AIR 1973 SC 205

In M/s. Hyderabad Commercials vs. Indian Bank and Ors. [1991 Supp (2) SCC 340], this
Court held : AIR 1991 SC 247, Para 5

"Since the basic facts regarding the unauthorized transfer of the disputed amount from the
appellant's account as well as the Bank's liability was admitted, there was no justification
for the High Court to direct the appellant to file suit on ground of disputed questions of
fact. The respondent Bank is an instrumentality of the State and it must function honestly
to serve its customers.
Would the ratio laid down therein apply in the instant case? We do not think so. The
question as to whether fraud has been committed by the officers of the Bank is pending
consideration before a competent criminal court. There are other various disputed
questions which are required to be gone into in the said proceeding. The role played by
some of the writ petitioners-respondents is also in issue. Such a seriously disputed
questions of fact, in our opinion, could not have been gone into by the writ court.

We would accept the proposition of law as propounded by this Court in Guruvayoor


Devaswom Managing Committee and Anr. vs. C.K. Rajan and Ors. [(2003) 7 SCC 546].
In that case it was, inter alia, observed that public interest litigation procedures may be
adopted in a case where initially the writ petition was filed as a private interest litigation.
(See also Ashok Lanka and Anr. vs. Rishi Dixit and ors. (2005) 5 SCC 598 at page 618).
2003 AIR SCW 6039
2005 AIR SCW 2676

We may in this behalf notice development of law in other jurisdiction.


Abram Chayes in his article on "The Role of the Judge in Public Law Litigation" Harv.
Law. Rev. Vol. 89 (1976) at Pg. 1281, opines that "Traditionally, adjudication has been
understood to be a process for resolving disputes among private parties which have not
been privately settled." He thus emphasizes the need for a "Public Law" model wherein
"the traditional adversary relationship is suffused and intermixed with negotiating and
mediating processes at every point. The Judge is the dominant figure in organizing and
guiding the case, and he draws for support not only on the parties and their counsel, but
on a wide range of outsiders-masters, experts and oversight personnel." He goes on to
give examples of school desegregation, employment discrimination, and

@page-SC2593
prisoners' or inmates' rights cases as also antitrust, securities fraud and other aspects of
the conduct of the corporate business, bankruptcy and reorganizations, union governance,
consumer fraud, housing discrimination, electoral reapportionment. environmental
management-fields that display in varying degrees the features of public law litigation.
According to him, the public law litigation model Inter alia has the following features :
"7. The Judge is not passive, his function limited to analysis and statement of governing
legal rules; he is active, with responsibility not only for credible fact evaluation but for
organizing and shaping the litigation to ensure a just and viable outcome.
8. The subject-matter of the lawsuit is not a dispute between private individuals about
private rights, but a grievance about the operation of public policy."

In Krishna Swami v. Union of India and another with Raj Kanwar v. Union of India and
another, (1992) 4 SCC 605, a constitutional Bench of this Court had to decide upon the
maintainability of a writ petition filed under Article 32 against the removal of a Supreme
Court Judge without impleading the Judge himself as a party to the proceedings. AIR
1993 SC 1407

The court on the role of an investigation committee opined : Para 56 of AIR

"The investigation done by the Committee, thus is to find whether the alleged
misbehaviour incapacity has been proved. Undoubtedly, the public law litigation often
contradicts the premise behind those of private law. In public law wider public interest is
involved over and beyond the contending parties. It concerns the future and private law
litigation is retrospective in operation. What the court could do? It could appoint a
Committee. But the decision of the Committee would not have been decisive. The
Division Bench appears to have applied its mind on the report, but in the absence of any
categorical finding that it was the officers of the Banks alone who were liable, no
direction as has been done in the instant case should have been issued. It may be that in
appropriate cases, the court may find the recommendations made by the Committee
acceptable.
18. But it is. in our opinion, not a public interest litigation in that sense of the term.
The report, however, was not unanimous. The opinion of the Committee was a divided
one on the crucial issue. Two members of the Committee were of the opinion that
whether the amount deposited by the cooperative banks was received back by them or
not, was yet to be ascertained. We are, therefore, of the opinion that it cannot be said that
the fact-finding body, assuming that the same could be constituted, made such
recommendations which could be accepted by the Court without going into the merit
thereof. It is also not a case where any mandatory relief could be granted in favour of the
respondents.
19. Having however said so, we must pose unto ourselves a further question. Could those
co-operative societies which had absolutely no role to play in the entire episode should
suffer in any manner whatsoever ? The co-operative societies/co-operative banks for the
purpose of their day-today functioning, require the amount which they have invested in
FDRs on their maturity. Should they wait till the criminal cases are over? Should they be
pushed to institute civil suits ? They can indisputably be compensated by grant of interest.
What, however, happens if in the meanwhile in the absence of the requisite funds being
available to them, they find it difficult to run the day-to-day affairs ?
20. Answers thereto may be difficult to find but it is not a wholly impossible task. We
think that the appellant-Bank being a 'State' within the meaning of Article 12 of the
Constitution of India with the assistance of officer(s) of the Central Bureau of
Investigation should make all attempts to ascertain as to which of the co-operative
societies/co-operative banks are in no way involved with the scam, and subject to such
precautions as may be found necessary to be taken, release the amount in their favour.
21. In any event, the quantum of the amount which all the depositors would have
otherwise received, in the event their investment in FDRs is found to be genuine, should
be informed thereabout. Once the liability of the Bank is determined, the Bank may
invest the said amount in its own account and issue fresh FDRs therefor. Whereas the
Bank may keep the original FDRs with itself, it may issue the duplicate copies thereof to
the eligible co-operative bank. Such an exercise should be completed within a period of
four weeks from date.
@page-SC2594
22. In the event, the co-operative society intending to avail loan facilities from the banks
for running their business, may approach them which may apart from usual conditions
release the same on a further condition that the amount of FDR would remain with them
and on that basis, loans may be granted of such amount. The usual precautions in regard
thereto may also be taken by the Bank(s).
23. We, while saying so, do not intend to lay down any law. These directions should not
be treated to be precedent. We are issuing these directions keeping in view that the factual
scenario obtaining in the case and that non-release of the amount is likely to enure
hardships that may be faced by the co-operative societies. We would also direct the
criminal court to dispose of the criminal cases pending before them with utmost
expedition. These appeals are allowed with the aforementioned directions. There shall,
however, be no order as to costs.
Appeals allowed.
AIR 2008 SUPREME COURT 2594 "State Bank of India v. S. N. Goyal"
(From : Punjab and Haryana)*
Coram : 2 H. K. SEMA AND R. V. RAVEENDRAN, JJ.
Civil Appeal Nos. 4243-4244 of 2004, D/- 2 -5 -2008.
State Bank of India and Ors. v. S.N. Goyal.
(A) Civil P.C. (5 of 1908), S.100 - APPEAL - Second Appeal - Substantial question of
law - Question of law which affects final decision of case - Is substantial question of law
- Question already decided - Can also be a substantial question in certain contingencies.
The word 'substantial' prefixed to 'question of law' does not refer to the stakes involved in
the case, nor intended to refer only to questions of law of general importance, but refers
to impact or effect of the question of law on the decision in the lis between the parties.
'Substantial questions of law' means not only substantial questions of law of general
importance, but also substantial question of law arising in a case as between the parties.
In the context of Section 100, CPC, any question of law which affects the final decision
in a case is a substantial question of law as between the parties. A question of law which
arises incidentally or collaterally, having no bearing on final outcome, will not be a
substantial question of law. (Para 9.1)
Even if there is a clear enunciation of law by Supreme Court or by the concerned High
Court, but the lower Court had ignored or misinterpreted or misapplied the same, and
correct application of the law as declared or enunciated by Supreme Court or the
concerned High Court would have led to a different decision, the appeal would involve a
substantial question of law as between the parties. Even where there is an enunciation of
law by Supreme Court or the concerned High Court and the same has been followed by
the lower Court, if the appellant is able to persuade the High Court that the enunciated
legal position needs reconsideration, alteration, modification or clarification or that there
is a need to resolve an apparent conflict between two view- points, it can be said that a
substantial question of law arises for consideration. (Para 9.1)
(B) Civil P.C. (5 of 1908), S.100, O.41, R.1, R.2 - APPEAL - APPELLATE COURT -
Second Appeal - Entertainment - Errors often committed by appellate Court - Enumerated
- Courts need to be careful since such lapses or technical errors lead to injustice and
prolongation of litigation. (Para 9, 3)
(C) Specific Relief Act (47 of 1963), S.14 - CONTRACT - Contract of personal service -
Suit for specific enforcement - Maintainable in case of employees of statutory bodies -
Wherein declaration is claimed that termination is in violation of statutory rules and he is
deemed to continue in service. (Para 12)
(D) Civil P.C. (5 of 1908), O.6, R.1 - Specific Relief Act (47 of 1963), S.14 -
PLEADINGS - CONTRACT - REMOVAL FROM SERVICE - Pleadings - Necessity -
Suit to declare order of removal from service illegal - No plea raised that order of
punishment was passed by considering extraneous material - Order impugned cannot be
set aside on such ground - Fact that evidence has been adduced to such effect, immaterial.
In the absence of appropriate pleading on a particular issue, there can be no adjudication
of such issue in a civil suit. Adjudication of a dispute by a civil Court is significantly
different from the exercise of power
@page-SC2595
of judicial review in a writ proceedings by the High Court. In writ proceedings, the High
Court can call for the record of the order challenged, examine the same and pass
appropriate orders after giving an opportunity to the State or the statutory authority to
explain any particular act or omission. In a civil suit parties are governed by rules of
pleadings and there can be no adjudication of an issue in the absence of necessary
pleadings. (Para 14)
Charge of financial irregularity was framed against the Branch Manager of State Bank of
India. Inquiry was conducted and finally an order of removal from service came to be
passed against the Branch Manager. A civil suit was filed by the Branch Manager for a
declaration that the order of removal as also the orders of the Appellate Authority and
Reviewing Authority were arbitrary and illegal. In the plaint, the challenge to the order of
removal was on the ground that the enquiry by the Enquiry Officer was opposed to
principles of natural justice, that is : (i) the charge was vague and not established; (ii) he
was not given reasonable opportunity to defend himself; (iii) material witnesses were not
examined; (iv) documents relied on were not formally proved; (v) burden of proof was
wrongly placed on him; (vi) findings in the enquiry report were based on surmises and
conjectures; and (viii) the enquiry officer was prejudiced. The plaintiff-respondent had
also averred that the Appointing Authority had approved the recommendation made by
the Disciplinary Authority for imposition of penalty of removal, without application of
mind and without giving him a hearing. He alternatively contended that the punishment
imposed was severe and disproportionate to the gravity of the proved charge. But there
was absolutely no plea with reference to the advice/recommendation of the Chief
Vigilance Officer of the Bank. Even though, during the examination of the Bank's witness
it was elicited that the Disciplinary Authority had put up a recommendation to impose the
penalty of reduction of pay by four stages by taking a lenient view; that the Appointing
Authority had by his note accepted the said recommendation; that subsequently the
Appointing Authority had informed the Chief Vigilance Officer of the Bank about the
enquiry and proposed punishment; and that after receiving the comments of the Chief
Vigilance Officer, the Appointing Authority on the recommendations of the disciplinary
authority had reconsidered the question of punishment and imposed the penalty of
removal, the respondent-plaintiff did not amend the plaint to include the averments and
grounds to challenge the order of removal on the said additional ground. No issue was
framed in that behalf. No amount of evidence on a plea that was not put forward in the
pleadings can be looked into. In the absence of necessary pleading and issue, neither the
trial Court nor the appellate Court could have considered the contention and recorded a
finding thereon. If a party should be permitted to rely on evidence led on an issue/aspect
not covered by pleadings, the other side will be put to a disadvantage. (Para 13)
(E) State Bank of India Act (23 of 1955), S.43 - State Bank of India Officers Service
Rules (1992), R.50, R.68(3) - BANKING - MISCONDUCT - DISCIPLINARY
AUTHORITY - SERVICE MATTERS - Misconduct - Financial irregularities -
Disciplinary Authority recommending lesser punishment - Accepted by Appointing
Authority but not communicated - Punishment changed to dismissal on fresh
recommendation of Disciplinary Authority made after receipt of opinion of Chief
Vigilance Officer - Appointing Authority cannot be said to have become functus officio
before passing order.
R.S.A. Nos. 4183-4184 of 2002, D/-11-12-2003 (P and H), Reversed.
Under provisions of Civil P.C., a Judge becomes functus officio when he pronounces,
signs and dates the judgment (subject to S. 152 and power of review). The position is
different with reference to quasi-judicial authorities. While some quasi- judicial tribunals
fix a day for pronouncement and pronounce their orders on the day fixed, many quasi-
judicial authorities do not pronounce their orders. Some publish or notify their orders.
Some prepare and sign the orders and communicate the same to the party concerned. A
quasi-judicial authority will become functus officio only when its order is pronounced, or
published/notified or communicated (put in the course of transmission) to the party
concerned. When an order is made in an office noting in a file but is not pronounced,
published or communicated, nothing prevents the Authority from correcting it or altering
it for valid reasons. But once the order is pronounced or
@page-SC2596
published or notified or communicated, the Authority will become functus officio.
(Para 19)
In the present case on an inquiry against Branch Manager he was found to have
committed financial irregularities. The disciplinary authority recommended punishment
of reduction of pay. The recommendation was accepted by Appointing Authority. This
order made on an office note was not communicated to the delinquent. Subsequently on
the observations of Chief Vigilance Officer that the case deserves no leniency, the
disciplinary authority made fresh recommendations to pass order of removal. The
recommendation was accepted by Appointing Authority and order of removal was
communicated. This order of removal cannot be said to have been passed after
Appointing Authority has become functus officio. (Paras 20, 21)
The Vigilance Department merely gave its comment or view that it was not a fit case for
showing leniency and left it to the concerned authority to take a decision on the
punishment to be imposed. The order of punishment passed is not based on extraneous
considerations.
R.S.A. Nos. 4183-4184 of 2002, D/-11-12-2003 (P and H), Reversed. (Para 23)
(F) Constitution of India, Art.133 - APPEAL - PLEA - REMOVAL FROM SERVICE -
SERVICE MATTERS - New plea - Suit for declaration that order of removal from
service was invalid - Challenge to validity of inquiry report raised in suit given up in first
appeal - Delinquent cannot revive plea in appeal to Supreme Court. (Para 27)
(G) State Bank of India Act (23 of 1955), S.43 - State Bank of India Officers Service
Rules (1992), R.50 - BANKING - SERVICE MATTERS - MISCONDUCT - Misconduct
- Financial irregularity by Branch Manager - Misappropriation by him even if temporary
is serious misconduct - Punishment of removal cannot be reduced. (Para 28)
Cases Referred : Chronological Paras
1995 SCC (L and S) 1011 (Disting., Pnt. E) 26.4
(1994) 5 SCC 118 (Disting., Pnt. E) 26.3
1992 AIR SCW 3353 : AIR 1993 SC 1 197 : 1992 Lab IC 2585 (Disting., Pnt. E) 26.2
1991 AIR SCW 1263 : AIR 1991 SC 1507 (Disting. Pnt. E) 26.1
AIR 1987 SC 1422 : 1987 Lab IC 1012 (Rel. on, Pnt. C) 11
AIR 1981 SC 122 : 1980 All LJ 1070 (Rel. on, Pnt. C) 11
AIR 1976 SC 888 : 1976 Lab IC 576 (Rel. on, Pnt. C) 11
AIR 1973 SC 855 : 1973 Lab IC 453 (Rel. on, Pnt. C) 11
AIR 1970 SC 1244 : 1970 Lab IC 1044 (Rel. on, Pnt. C) 11
AIR 1958 SC 1050 (Rel. on, Pnt. C) 11
(1941) 3 All ER 417 (Ref.) 17
R. F. Nariman, Sr. Advocate, Sanjay Kapur, Mrs. Shubra Kapur and Miss Arti Singh, for
Appellants; Dr. K. S. Chauhan, Dr. Indra Pratap Singh, Chand Kiran, Ms. Gyan Mitra and
P. K. Jayakrishnan, for Respondent.
* R.S.A. Nos. 4183-4184 of 2002, D/- 11-12-2003 (P and H).
Judgement
R. V. RAVEENDRAN, J. :- Theses appeals by special leave are filed by a defendant-
employer (State Bank of India) against the judgment dated 11.12.2003 of the Punjab and
Haryana High Court in R.S.A. No. 4184 of 2002.
2. A charge-sheet dated 28.4.1994 was issued by the Appellant-Bank to the respondent
alleging that when he was posted as the Branch Manager of appellant's Kalanwali
Branch, Sirsa, Haryana, he had received cash payments tendered by two customers of the
Bank, for being credited to their loan accounts, and temporarily misappropriated such
amounts and had belatedly deposited them to the borrowers' accounts (after about five
months in one case and two and half months in another). The said acts amounted to a
misconduct, violative of Rule 50(4) of the State Bank of India Officers Service Rules
('Service Rules' for short). An enquiry was held in regard to the said charge. The Enquiry
Officer submitted his report dated 11.11.1994 holding that the charge was proved. The
Disciplinary Authority furnished a copy of the said report to the respondent and gave him
an opportunity to show cause in the matter.
3. Rule 68(3) of the Service Rules required, where the Disciplinary Authority was of the
opinion that a major penalty is to be imposed, and where he was lower in rank to the
Appointing Authority (in respect of the category of officers to which the delinquent
officer belonged), that he should submit to the Appointing Authority, the records of the
enquiry together with his recommendations regarding the penalty that may be imposed,
@page-SC2597
and the Appointing Authority should make the order imposing the penalty, which in his
opinion was appropriate. In view of the above rule, the Disciplinary Authority after
considering the inquiry records and the representation of the respondent, made a
recommendation on 2.5.1995 to the Appointing Authority to impose the penalty of
'removal from service' on the respondent. The Appointing Authority considered the entire
material and concurred with the recommendation of the Disciplinary Authority and made
an order dated 3.5.1995 imposing the penalty of removal from service, which was
communicated to the Respondent by letter dated 30.6.1995 of the Disciplinary Authority.
4. The Appeal and Revision (Review) filed by the Respondent were dismissed on
29.11.1995 and 27.11.1996. The respondent thereafter filed Civil Suit No. 158 of 1998 on
the file of the Civil Judge, Senior Division, Jind, for a declaration that the order of
removal dated 30.6.1995 as also the orders of the Appellate Authority and Reviewing
Authority were arbitrary and illegal. He also prayed that the said orders be set aside with
a direction to take him back into service with all consequential benefits. The suit was
resisted by the appellant-Bank. After trial, the suit was decreed on 19.4.2003. The Trial
Court found that there was no violation of principles of natural justice in conducting the
enquiry and the order holding the respondent guilty of misconduct was proper. The trial
court however found that the Disciplinary Authority, by his earlier note dated 18.1.1995,
had recommended imposition of the penalty of reduction of pay of respondent by four
stages in his time scale and the Appointing Authority had agreed with the said
recommendation on 18.1.1995. According to the trial court, the said order was a final
order of punishment by the Appointing Authority; and the Appointing Authority had
thereafter sought the advice of the Bank's Chief Vigilance Officer, and acting on such
advice, had changed his earlier decision and imposed a higher punishment by way of
removal from service, by order dated 3.5.1995, (communicated on 30.6.1995). The trial
court was of the view that the second order imposing penalty was passed by the
Appointing Authority "on extraneous reasons after taking advice of the Chief Vigilance
Officer", and that rendered the order of removal illegal, null and void. The trial court
therefore set aside the order of removal dated 30.6.1995 as also the orders dated 29.11.95
and 27.11.1996 of the Appellate Authority and Reviewing Authority affirming the order
of removal. It directed the appellant-Bank to reinstate the respondent with continuity of
service and all consequential benefits except back-wages. The Trial Court reserved liberty
to the appellant to pass a fresh order imposing appropriate penalty on the respondent,
other than the penalty of dismissal or removal from service.
5. Feeling aggrieved, both parties filed appeals. Before the First Appellate Court, the
respondent did not challenge the finding of the trial court that the domestic enquiry was
fair and proper and that his guilt was established. He limited his challenge only to the
quantum of punishment (that is, reservation of liberty to the employer to pass a fresh
order imposing appropriate penalty) and the denial of back-wages. The appellant, in its
appeal, contended that the Trial Court, having found that the enquiry was fair and proper
and the finding of guilt was justified, ought not to have set aside the order imposing
penalty. The two appeals were heard and disposed of by the Additional District Judge,
Jind, by a common judgment dated 20.7.2002. The First Appellate Court upheld the
decree of the Trial Court, but in addition held that the respondent was entitled to full
back-wages with interest thereon at 9% per annum. Consequently, the First Appellate
Court dismissed the appeal by the appellant and allowed in part the appeal of the
respondent.
6. Feeling aggrieved the Bank filed the second appeal, which was dismissed by the
judgment under appeal. The judgment is short. After referring to the prayer in the suit and
the judgments rendered by the courts below, it contains the following reasoning :

"It is not in dispute that originally the punishment proposed against the plaintiff was to
bring him lower by four steps. Subsequently on directions issued by the Chief Vigilance
Commissioner of the Bank, the punishment was converted to that of dismissal. The
plaintiff made a complaint that the aforesaid orders and the material placed before the
Chief Vigilance Commissioner were never put to him and as such the order of
punishment was violative of principles of natural justice.
@page-SC2598
The learned courts below found that the contention of the plaintiff was duly substantiated
from the record. Accordingly, the punishment orders were set aside with a liberty as
noticed above.
Nothing has been shown that the findings recorded by the learned courts below suffer
from any infirmity or are contrary to law in any manner.
No question of law, much less any substantial question of law, arises in this appeal."
7. We find that the High Court misread the findings of the courts below. The Trial Court
held that the Appointing Authority passed the order of removal after taking the advice of
the Chief Vigilance Officer. The first appellate court held that the Appointing Authority
imposed the penalty of removal on the recommendations of the Chief Vigilance Officer.
But the High Court observed that 'on the directions of the Chief Vigilance Commissioner
of the Bank, the punishment was converted to that of dismissal'. This observation
contains three errors-firstly the penalty of removal was read as dismissal; secondly the
communication from the Chief Vigilance Officer, termed as "advice/recommendation" by
the courts below, was wrongly read as 'directions'; and thirdly, the Chief Vigilance Officer
of the Bank was wrongly referred to as the Chief Vigilance Commissioner. The High
Court also erroneously assumed that plaintiff (respondent herein) had pleaded that the
Appointing Authority had placed certain material which was never put to him (the
plaintiff), before the Chief Vigilance Commissioner and as such the order of punishment
was violative of principles of natural justice. There was no such plea, nor did the courts
below record a finding on any such plea.
8. We also find that the High Court completely missed the real points arising for
determination. After a cursory wrong reference to the findings of the court below, the
High Court wrongly held that the second appeal did not give rise to any substantial
question of law, ignoring the several substantial questions of law arising for consideration
of the High Court, which were clearly specified in the memorandum of second appeal.
We find that the second appeal gave rise to several substantial questions of law including
the following :
(i) Whether a direction by the Civil Court to reinstate the respondent, amounted to
granting specific performance of a contract of personal service which is barred by section
14 of Specific Relief Act, 1963 ?
(ii) In the absence of a pleading that the order imposing penalty was invalid because the
Appointing Authority acted on the advice or recommendation of the Chief Vigilance
Officer, and in the absence of any issue in that behalf, could the Courts below hold that
the order imposing punishment was Illegal on that ground ?
(iii) Whether an order recorded by the Appointing Authority on an office note, to impose
the penalty of reduction in pay, which was neither pronounced, published or
communicated, is a final decision which could not be reconsidered or altered, by the
Appointing Authority ?
(iv) Whether the decision of the Appointing Authority imposing penalty can be said to
have been influenced by extraneous material, merely because the Chief Vigilance Officer
of the Bank requested him to re-examine the proposed penalty ?
(v)Whether the Appointing Authority ought to have communicated the
advice/recommendation of the Chief Vigilance Officer to the respondent and given him
an opportunity to show cause before imposing punishment?
If questions (iii) to (v) or any of them is answered in the affirmative and as a consequence
if it has to be held that the order of removal was illegal or invalid, then, the second appeal
would give rise to several further substantial questions of law. One question would have
been whether the civil court could direct the authority empowered to impose penalty, to
restrict the punishment to something other than dismissal/removal. Another question
would have been whether full back-wages with interest could be awarded where the court
accepts that the employee was guilty of misconduct of misappropriation. Be that as it
may.
9. Before examining the merits of the matter, we may briefly refer to the scope of second
appeals as also the procedure for entertaining them, as laid down in section 100 of the
Code of Civil Procedure.
What is a substantial question of law ?
9.1Second appeals would lie in cases which involve substantial questions of law. The
word 'substantial' prefixed to 'question
@page-SC2599
of law' does not refer to the stakes involved in the case, nor intended to refer only to
questions of law of general importance, but refers to impact or effect of the question of
law on the decision in the lis between the parties. 'Substantial questions of law' means not
only substantial questions of law of general importance, but also substantial question of
law arising in a case as between the parties. In the context of section 100, CPC, any
question of law which affects the final decision in a case is a substantial question of law
as between the parties. A question of law which arises incidentally or collaterally, having
no bearing in the final outcome, will not be a substantial question of law. Where there is a
clear and settled enunciation on a question of law, by this Court or by the High Court
concerned, it cannot be said that the case involves a substantial question of law. It is said
that a substantial question of law arises when a question of law, which is not finally
settled by this court (or by the concerned High Court so far as the State is concerned),
arises for consideration in the case. But this statement has to be understood in the correct
perspective. Where there is a clear enunciation of law and the lower court has followed or
rightly applied such clear enunciation of law, obviously the case will not be considered as
giving rise to a substantial question of law, even if the question of law may be one of
general importance. On the other hand, if there is a clear enunciation of law by this Court
(or by the concerned High Court), but the lower court had ignored or misinterpreted or
misapplied the same, and correct application of the law as declared or enunciated by this
Court (or the concerned High Court) would have led to a different decision, the appeal
would involve a substantial question of law as between the parties. Even where there is
an enunciation of law by this court (or the concerned High Court) and the same has been
followed by the lower court, if the appellant is able to persuade the High Court that the
enunciated legal position needs reconsideration, alteration, modification or clarification
or that there is a need to resolve an apparent conflict between two viewpoints, it can be
said that a substantial question of law arises for consideration. There cannot, therefore, be
a strait-jacket definition as to when a substantial question of law arises in a case. Be that
as it may.
Procedure relating to second appeals
9.2 We may next refer to the procedure relating to second appeals as evident from section
100 read with Order 42, Rules 1 and 2, of the Code of Civil Procedure :
(a) The appellant should set out in the memorandum of appeal, the substantial questions
of law involved in the appeal.
(b) The High Court should entertain the second appeal only if it is satisfied that the case
Involves a substantial question of law.
(c) While admitting or entertaining the second appeal, the High Court should formulate
the substantial questions of law involved in the case.
(d) The second appeal shall be heard on the question's of law so formulated and the
respondent can submit at the hearing that the second appeal does not in fact involve any
such questions of law. The Appellant cannot urge any other ground other than the
substantial question of law without the leave of the court.
(e) The High Court is at liberty to reformulate the substantial questions of law or frame
other substantial question of law, for reasons to be recorded and hear the parties or such
reformulated or additional substantial questions of law.
9.3. It is a matter of concern that the scope of second appeals and as also the procedural
aspects of second appeals are often ignored by the High Courts. Some of the oft-repeated
errors are :
(a) Admitting a second appeal when it does not give rise to a substantial question of law.
(b) Admitting second appeals without formulating substantial question of law.
(c) Admitting second appeals by formulating a standard or mechanical question such as
"whether on the facts and circumstances the judgment of the first appellate court calls for
interference" as the substantial question of law.
(d) Failing to consider and formulate relevant and appropriate substantial question/ s of
law involved in the second appeal.
(e) Rejecting second appeals on the ground that the case does not involve any substantial
question of law, when the case in fact involves substantial questions of law.
(f) Reformulating the substantial question of law after the conclusion of the hearing,
@page-SC2600
while preparing the judgment, thereby denying an opportunity to the parties to make
submissions on the reformulated substantial question of law.
(g) Deciding second appeals by re-appreciating evidence and interfering with findings of
fact, ignoring the questions of law. These lapses or technical errors lead to injustice and
also give rise to avoidable further appeals to this court and remands by this court, thereby
prolonging the period of litigation. Care should be taken to ensure that the cases not
involving substantial questions of law are not entertained, and at the same time ensure
that cases involving substantial questions of law are not rejected, as not involving
substantial questions of law.
10. In this case, the failure on the part of the High Court to take note of the substantial
questions of law involved, has led to unwarranted dismissal of the second appeal and
calls for interference. One alternative available to us is to remand the matter to the High
Court for formulating the substantial questions of law and then hear and dispose of the
appeal. But that is likely to delay the matter further. The questions arising for decision are
questions of law. These had been raised in the memorandum of second appeal before the
High Court and again referred to in the special leave petition. Elaborate arguments have
been addressed on those questions (extracted in para 8 above) by both sides. We are,
therefore, of the view that instead of remanding the matter, we should ourselves consider
the several questions of law that ought to have been considered by the High Court and
decide the matter finally.
Re : Question (i) - Enforcement of a contract of personal service.
11. Where the relationship of master and servant is purely contractual, it is well settled
that a contract of personal service is not specifically enforceable, having regard to the bar
contained in section 14 of the Specific Relief Act. 1963. Even if the termination of the
contract of employment (by dismissal or otherwise) is found to be illegal or in breach, the
remedy of the employee is only to seek damages and not specific performance. Courts
will neither declare such termination to be a nullity nor declare that the contract of
employment subsists nor grant the consequential relief of reinstatement. The three well
recognized exceptions to this rule are :
(i) where a civil servant is removed from service in contravention of the provisions of
Article 311 of the Constitution of India (or any law made under Article 309);
(ii) where a workman having the protection of Industrial Disputes Act, 1947 is wrongly
terminated from service; and
(iii) where an employee of a statutory body is terminated from service in breach or
violation of any mandatory provision of a statute or statutory rules.

There is thus a clear distinction between public employment governed by statutory rules
and private employment governed purely by contract. The test for deciding the nature of
relief - damages or reinstatement with consequential reliefs - is whether the employment
is governed purely by contract or by a statute or statutory rules. Even where the employer
is a statutory body, where the relationship is purely governed by contract with no element
of statutory governance, the contract of personal service will not be specifically
enforceable. Conversely, where the employer is a non-statutory body, but the employment
is governed by a statute or statutory rules, a declaration that the termination is null and
void and that the employee should be reinstated can be granted by courts. (Vide : Dr. S.
Dutt vs. University of Delhi, AIR 1958 SC 1050; Executive Committee of U.P. State
Warehousing Corporation Ltd. vs. Chandra Kiran Tyagi, 1970 (2) SCR 250; Sirsi
Municipality vs. Cecelia Kom Francies Tellis. 1973 (3) SCR 348; Executive
Committee of Vaish Degree College vs. Lakshmi Narain, 1976 (2) SCR1006; Smt. J.
Tiwari vs. Smt. Jawala Devi Vidya Mandir. AIR 1981 SC 122; and Dipak Kumar Biswas
vs. Director of Public Instruction, AIR 1987 SC 1422). AIR 1970 SC 1244
AIR 1973 SC 855
AIR 1976 SC 888

12. In this case the appellant is a statutory body established under the State Bank of India
Act. 1955 and the contract of employment was governed by the State Bank of India
Officers Service Rules, which are statutory rules framed under section 43(1) of the said
Act. The respondent approached the civil court alleging that his removal from service
was in violation of the said statutory rules. When an employee of a statutory body whose
service is terminated, pleads that such termination is in violation of statutory
@page-SC2601
rules governing his employment, an action for declaration that the termination is invalid
and that he is deemed to continue in service is maintainable and will not be barred by
section 14 of the Specific Relief Act.
Re : Question (ii) - Effect of absence of pleading.
13. The plaint did not contain any plea that the order of removal by the Appointing
Authority (Chief General Manager) was vitiated on account of his consulting and acting
on the advice of the Chief Vigilance Officer of the Bank. Nor did it contain any allegation
that the Appointing Authority acted on extraneous material in passing the order of
removal. In the plaint, the challenge to the order of removal was on the ground that the
enquiry by the Enquiry Officer was opposed principles of natural justice that is : (i) the
charge was vague and not established; (ii) he was not given reasonable opportunity to
defend himself; (iii) material witnesses were not examined; (iv) documents relied on were
not formally proved; (v) burden of proof was wrongly placed on him; (vi) findings in the
enquiry report were based on surmises and conjectures; and (viii) the Enquiry Officer was
prejudiced. The respondent had also averred that the Appointing Authority had approved
the recommendation made by the Disciplinary Authority for imposition of penalty of
removal, without application of mind and without giving him a hearing. He alternatively
contended that the punishment imposed was severe and disproportionate to the gravity of
the proved charge. But there was absolutely no plea with reference to the
advice/recommendation of the Chief Vigilance Officer of the Bank. However, during the
examination of the Bank's witness DW-1 (T.S. Negi, Deputy Manager) it was elicited that
on 18.1.1995, the Disciplinary Authority had put up a recommendation to impose the
penalty of reduction of pay by four stages by taking a lenient view; that the Appointing
Authority had by his note dated 18.1.1995 accepted the said recommendation; that
subsequently, on 2.2.1995, the Appointing Authority had informed the Chief Vigilance
Officer of the Bank about the enquiry and proposed punishment; and that after receiving
the comments of the Chief Vigilance Officer, the Appointing Authority on the
recommendations of the Disciplinary Authority had reconsidered the question of
punishment and imposed the penalty of removal. The respondent- plaintiff did not amend
the plaint to include the averments and grounds to challenge the order of removal on the
said additional ground. No issue was framed in that behalf. No amount of evidence on a
plea that was not put forward in the pleadings can be looked into. In the absence of
necessary pleading and issue, neither the trial court nor the appellate court could have
considered the contention and recorded a finding thereon.
14. The learned counsel for the respondent submitted that the order of removal was
challenged on the ground that it was opposed to principles of natural justice, and the
averments in the plaint were sufficient to enable the plaintiff to establish any ground in
support of it and it was not necessary to separately plead each and every fact or ground in
support of his contention that the order of removal was vitiated. While there is no need to
plead evidence, the grounds of challenge and the facts in support of each ground, will
have to be pleaded. In this case, the minimum pleading that was necessary was that the
Appointing Authority acted on extraneous material in arriving at the decision or acted on
the advice or recommendation of an Authority who was not concerned with the Enquiry.
In the absence of appropriate pleading on a particular issue, there can be no adjudication
of such issue. Adjudication of a dispute by a civil court is significantly different from the
exercise of power of judicial review in a writ proceedings by the High Court. In a writ
proceedings, the High Court can call for the record of the order challenged, examine the
same and pass appropriate orders after giving an opportunity to the State or the statutory
authority to explain any particular act or omission. In a civil suit parties are governed by
rules of pleadings and there can be no adjudication of an issue in the absence of necessary
pleadings. The learned counsel for the respondent submitted that the respondent was
unaware of the earlier order dated 18.1.1995 or about the consultation with the Chief
Vigilance Officer when he filed the suit and therefore, could not make necessary
averments in the plaint in that behalf. But that is no answer. Code of Civil Procedure
contains appropriate provisions relating to interrogatories, discovery and inspection
(Order XI, Rules 1, 12 and 15) to gain access to relevant material available with the other
party. A party to a suit
@page-SC2602
should avail those provisions and if any new ground becomes available on the basis of
information secured by discovery, a party can amend his pleadings and introduce new
facts and grounds which were not known earlier. The difficulty in securing relevant
material or ignorance of existence of relevant material will not justify introduction of
such material at the stage of evidence in the absence of pleadings relating to a particular
aspect to which the material relates. If a party should be permitted to rely on evidence led
on an issue/aspect not covered by pleadings, the other side will be put to a disadvantage.
For example, in this case, if there had been a plea and issue on the question whether
extraneous material was taken into account, the Bank could have examined the
Appointing Authority to explain the context in which he informed the Chief Vigilance
Officer about the matter or explain how his decision was not dependant upon any
extraneous material. Therefore, the courts below committed a serious error in holding that
the order of removal was based on an extraneous material (the advice/recommendation of
Chief Vigilance Officer) and, therefore, invalid.
15. Where the enquiry was found to be fair and proper and the finding of guilt in the
enquiry in respect of a serious charge was found to be valid, in the absence of any other
valid ground of challenge, the courts below ought to have held that the penalty of
removal from service did not warrant any interference and dismissed the suit. Be that as it
may. We will now consider the matter on merits, on the assumption that the averments in
the plaint were sufficient to enable the court to consider this issue.
Re : Question (iii) - When did the Appointing Authority became functus officio.
16. Ex.P24 is the note dated 18.1.1995 by which the Disciplinary Authority accepted the
finding of guilt recorded arrived at by the Enquiry Officer in regard to the charge against
the respondent that he temporarily misappropriated the funds of the customers of the
Bank. The Disciplinary Authority though of the view that the respondent deserved a
severe punishment, felt that having regard to the length of his service, he should be
shown leniency, and therefore, recommended imposition of a lesser punishment of
reduction of pay by four stages in the time scale. The Appointing Authority made a note
on the same day (18.1.1995) agreeing with the said recommendation. But the said order
was not communicated to the respondent. On the other hand, the Disciplinary Authority
on reconsideration of the matter put up a fresh note dated 2.5.1995 recommending the
penalty of removal and that was accepted by the Appointing Authority on 3.5.1995 and
communicated to the respondent on 30.6.1995.
17. The learned counsellor respondent contended that the Appointing Authority became
functus officio once he passed the order dated 18.1.1995 agreeing with the penalty
proposed by the Disciplinary Authority and cannot thereafter revise/review/ modify the
said order. Reliance was placed on the English decision Re : VGM Holdings Ltd.,
reported in 1941 (3) All ER page 417, wherein it was held that once a Judge has made an
order which has been passed and entered, he becomes functus officio and cannot
thereafter vary the terms of his order and only a higher court, tribunal can vary it. What is
significant is that decision does not say that the Judge becomes functus officio when he
passes the order, but only when the order passed is 'entered'. The term 'entering judgment'
in English Law refers to the procedure in civil courts in which a judgment is formally
recorded by court after it has been given.
18. It is true that once an Authority exercising quasi-judicial power, takes a final decision,
it cannot review its decision unless the relevant statute or rules permit such review. But
the question is as to at what stage, an Authority becomes functus officio in regard to an
order made by him. P. Ramanatha Aiyar's Advance Law Lexicon (3rd Edition, Vol.2
Pages 1946-47) gives the following illustrative definition of the term 'functus officio' :
"Thus a Judge, when he has decided a question brought before him, is functus offlcio, and
cannot review his own decision."
Black's Law Dictionary (Sixth Edition Page 673) gives its meaning as follows :
"Having fulfilled the function, discharged the office, or accomplished the purpose, and
therefore, of no further force or authority."
19. We may first refer to the position with reference to civil courts. Order XX of Code of
Civil Procedure deals with judgment and decree. Rule 1 explains when a judgment is
pronounced. Sub-rule (1) provides that the Court, after the case has been heard, shall
@page-SC2603
pronounce judgment in an open court either at once, or as soon thereafter as may be
practicable, and when the judgment is to be pronounced on some future day, the court
shall fix a day for that purpose of which due notice shall be given to the parties or their
pleaders. Sub-rule (3) provides that the judgment may be pronounced by dictation in an
open court to a shorthand writer (if the Judge is specially empowered in this behalf). The
proviso thereto provides that where the judgment is pronounced by dictation in open
court, the transcript of the judgment so pronounced shall, after making such corrections
as may be necessary, be signed by the Judge, bear the date on which it was pronounced
and form a part of the record. Rule 3 provides that the judgment shall be dated and signed
by the Judge in open court at the time of pronouncing it and when once signed, shall not
afterwards be altered or added to save as provided by section 152 or on review. Thus
where a judgment is reserved, mere dictation does not amount to pronouncement, but
where the judgment is dictated in open court, that itself amounts to pronouncement. But
even after such pronouncement by open court dictation, the Judge can make corrections
before signing and dating the judgment. Therefore, a Judge becomes functus officio when
he pronounces, signs and dates the judgment (subject to section 152 and power of
review). The position is different with reference to quasi judicial authorities. While some
quasi-judicial tribunals fix a day for pronouncement and pronounce their orders on the
day fixed, many quasi-judicial authorities do not pronounce their orders. Some publish or
notify their orders. Some prepare and sign the orders and communicate the same to the
party concerned. A quasi judicial authority will become functus officio only when its
order is pronounced, or published/notified or communicated (put in the course of
transmission) to the party concerned. When an order is made in an office noting in a file
but is not pronounced, published or communicated, nothing prevents the Authority from
correcting it or altering it for valid reasons. But once the order is pronounced or published
or notified or communicated, the Authority will become functus officio. The order dated
18.1.1995 made on an office note, was neither pronounced, nor published/notified nor
communicated. Therefore, it cannot be said that the Appointing Authority became functus
officio when he signed the note dated 18.1.1995.
20. Let us next consider whether the decision taken on 18.1.1995 is a final decision. A
careful examination shows that the order dated 18.1.1995 was intended only to be
tentative and not final. Firstly, the said decision was not communicated to the respondent,
nor was any letter or order issued to the respondent imposing the penalty mentioned in
the order dated 18.1.1995. Secondly, the Appointing Authority by letter dated 2.2.1995
(Ex.P23) informed the Chief Vigilance Officer of the Bank about the enquiry against
respondent, his decision accepting the findings of the Enquiry Officer, and the proposal to
show leniency by imposing only a punishment of reduction of pay by four stages. The
Chief Vigilance Officer sent a reply dated 7.2.1995 (Ex.D2) wherein he observed that "by
pocketing the money of the customers Sri Goyal has exposed the Bank's faith reposed in
him" and there was no ground for showing leniency. He also expressed the view that the
respondent deserved a more severe punishment and requested the appointing authority to
re-examine whether respondent should be continued in the post. Thereafter the
Disciplinary Authority reconsidered the entire issue again and put up another note dated
23.3.1995/2.5.1995 to the Appointing Authority proposing the punishment of removal
from service. After considering the said recommendation, the Appointing Authority
passed the following order on the said note on 3.5.1995 :
"On a dispassionate and objective evaluation of the facts, circumstances of the case,
inquiry proceedings and evidence available, I concur with the recommendations of the
Disciplinary Authority mentioned at serial No. 4 of the note and have come to the
conclusion that the penalty of "removal from Bank's service" proposed to be inflicted on
Sri S.L. Goyal. Officer JMGS-I, is just and appropriate and I. therefore, order imposition
of this penalty on the official."
21. It is thus clear that on 18.1.1995, the Appointing Authority had only tentatively
approved the proposal of the disciplinary authority that a lenient view be taken by
imposing a penalty of reducing the pay by four stages in the time scale; and that on
3.5.1995, a final decision was taken in regard to the penalty and that final order was
communicated to the respondent as per letter dated 30.6.1995. Therefore, the contention
@page-SC2604
that the Appointing Authority had earlier passed a final order on 18.1.1995 and had
become functus officio and therefore, he could not change the said order dated 18.1.1995
is liable to be rejected.
Re : Questions (iv) and (v) -Whether the Appointing Authority was influenced by
extraneous material.
22. A perusal of the letter dated 2.2.1995 sent by the Appointing Authority to the Chief
Vigilance Officer clearly demonstrates that the Appointing Authority did not seek any
guidance or advice or directions from the Vigilance Department and that the letter was
only by way of 'intimation' of factual position. For convenience, we extract below the
said letter entirely :
"The Chief Vigilance Officer,
State Bank of India, Central Office, Bombay.
Dear Sir,
Staff : Supervising
Shri S.N. Goyal : Officer JMGS I, Kaluana Branch
Disciplinary Action.
Further to our letter No.CO/VIG/4266, dated the 19th November, 1994, we advice that
the Disciplinary Authority has examined the enquiry proceedings and findings of the
Inquiring Authority in the case initiated against Shri S. N. Goyal, Officer JMGS I, and
has agreed with the same.
2. In this connection, copies of the following documents are enclosed for your perusal
and record :
(i) Charge-sheet issued to the official
(ii) Enquiry proceedings
(iii) Findings of the Inquiring Authority
(iv) Tabular statement showing the charges levelled against the official, findings of the
Inquiring Authority, official's submissions on the findings and Disciplinary Authority's
comments thereon.
(v) Note put up by the Disciplinary Authority to the Appointing Authority viz., the Chief
General Manager.
(vi) Bio-data of the Official.
3. In view of the seriousness of the charge proved against Shri S.N. Goyal. Officer JMGS
I, he deserves a severe punishment. The Disciplinary Authority is, however, inclined to
take a lenient view in the matter considering the length of service put in by the official in
the Bank and also to provide him a chance to reform himself. The Disciplinary Authority
is of the view that the ends of justice will be met if the official is brought down by four
stages in his time scale in terms of Rule 67 (e) of State Bank of India Officers Service
Rules to which I concur in the capacity as the Appointing Authority of the official.
Yours faithfully,
Chief General Manager."
23. The reply dated 7.2.1995 from the Chief Vigilance Officer also makes it clear that he
neither issued any direction to the Appointing Authority to impose a higher punishment
nor altered the finding regarding guilt. He merely gave his opinion that the gravity of the
proved charge did not warrant leniency and therefore, suggested that the quantum of
penally may be examined again. The subsequent note put up by the disciplinary authority
on 2.5.1995 and the order passed thereon by the appointing authority on 3.5.1995
imposing the penalty of removal, show that they were on independent consideration of
the question. Neither the note dated 2.5.1995 nor the order dated 3.5.1995 refer to the
opinion or the view expressed by the Chief Vigilance Officer of the Bank. Nor is there
any material to show that the order imposing punishment was on the dictates of the Chief
Vigilance Officer. There was no mechanical acceptance of any suggestion or advice by
the Chief Vigilance Officer nor consideration of any extraneous material as assumed by
the courts below. The Appointing Authority is required to inform the vigilance
department in regard to cases involving vigilance angle. The Appointing Authority did so.
But he did not seek any instruction, direction, suggestion or advice from the Vigilance
Department. There was also no direction or circular or instruction requiring the
Appointing Authority to accept or act upon the suggestions or views of the Chief
Vigilance Officer. The Vigilance Department merely gave its comment or view that it was
not a fit case for showing leniency and left it to the concerned authority to take a decision
on the punishment to be imposed. So long as the decision was not on the dictates of the
Vigilance Department or other outside authority, but on independent consideration, the
order of removal cannot be faulted. It cannot be said that either the act of intimating the
Vigilance Department about the enquiry or independently re-considering
@page-SC2605
the issue of penalty after receiving the views of the Vigilance Department amounted to be
acting on extraneous material, or acting on the advice or recommendation or direction of
the Chief Vigilance Officer.
24. The assumption made by the High Court that the Appointing Authority had placed
some undisclosed additional material before the Chief Vigilance Officer is without any
basis. The Enquiry Officer had found the respondent guilty of the charge on consideration
of the evidence. The finding of guilt was accepted by the Disciplinary Authority and the
Appointing Authority. This is not a case where any evidence or other material was sent to
the vigilance department seeking their decision or views on the question of guilt of the
respondent. The issue relating to the respondent's guilt was neither referred to the
Vigilance Department nor did the Vigilance Department give any finding on the question
of guilt. When the Disciplinary Authority and the Appointing Authority accepted the
finding of guilt recorded by the Enquiry Officer on examining the facts, even before the
matter was informed to Vigilance Department, it cannot be said that the said decision was
influenced by any extraneous advice from Vigilance Department. The issue on which the
Vigilance Department made its comment was on the limited ground whether any leniency
should be shown in imposing punishment. No additional facts or material were placed by
the Appointing Authority before the Vigilance Department for this purpose. Further the
Vigilance Department merely expressed the view that the gravity of the charge did not
warrant leniency and the authority should examine the matter. Therefore the assumption
by the High Court that the Appointing Authority had placed some material not put to the
respondent, before the Chief Vigilance Officer and that the Chief Vigilance Officer had
issued any direction to the Appointing Authority on the basis of such material, is baseless.
25. The Disciplinary Authority made available the Enquiry Report to the respondent to
enable him to make his submissions on the findings of the Enquiry Officer. The
respondent made his submissions in regard to the Enquiry Report. The correspondence
between the Appointing Authority and Chief Vigilance Officer of the Bank was not
'material' on which the finding regarding guilt/ misconduct was based. Such
correspondence was subsequent to the Enquiry Report. There was no compulsion or
requirement that the Appointing Authority should consult the Chief Vigilance Officer or
act as per his recommendations or directions. Nor was there any direction by the Chief
Vigilance Officer to Impose any specific direction. Therefore non furnishing of copies of
the correspondence between the Appointing Authority and the Chief Vigilance Officer to
the respondent, did not violate principles of natural Justice nor vitiate the order of
penalty.
26. The decisions relied on by the respondent do not lay down any proposition of law
which requires us to take a different view in the matter.
26.1

In Nagaraj Shivarao Karjagi vs. Syndicate Bank -1991 (3) SCC 219, this Court
considered a case where the employer Bank referred the matter to the Chief Vigilance
Commissioner (for short 'CVC') for advice and the Commissioner made a specific
recommendation that the employee may be compulsorily retired from service by way of
punishment. The impugned directive of the Ministry of Finance directed that the
disciplinary authority and appellate authority could not impose a lesser punishment than
what was suggested by CVC without its concurrence. The Bank accordingly imposed the
penalty of compulsory retirement. This Court held that the advice tendered by the CVC
was not binding on the punishing authority and it was not obligatory upon the punishing
authority to accept the advice of the CVC. This Court held that no third party like CVC or
Central Government could dictate the disciplinary authority or the appellate authority as
to how they should exercise their power and what punishment they should impose on the
delinquent officer. This Court also held that the Finance Ministry directive that a
punishment lesser than what was recommended by the CVC could not be imposed, was
without jurisdiction and contrary to the statutory regulations governing disciplinary
matters. The said decision is of no assistance in this case, as there is no directive that the
recommendation of the Vigilance Department is mandatory and should be followed while
imposing punishment; nor has the Vigilance Department directed the punishing authority
to impose any specific punishment; nor has the appointing authority 1991 AIR
SCW 1263

@page-SC2606
acted on the dictates of the Vigilance Department.
26.2

The next decision relied upon by the respondent is the decision rendered by this Court in
State Bank of India vs. D.C. Aggarwal [1993 (1) SCC 13]. In that case, the Enquiry
Officer recommended exoneration of the employee. Instead of acting on the
recommendation, the Bank directed the Enquiry Officer to submit the report through
CVC. The CVC disagreed with the finding of the Enquiry Officer and recorded a finding
of guilt and recommended the imposition of major penalty of removal. A copy of the
CVC's recommendation was not furnished to the employee. The disciplinary authority
acting on the recommendation of the CVC and agreeing with CVC's finding of guilt,
passed an order but imposed a punishment lesser than what was directed by CVC. This
Court held that the order of the disciplinary authority imposing punishment was vitiated
as it violated the principles of natural justice by denying the copy of the recommendation
of the CVC which was prepared behind his back. The said decision therefore related to
CVC examining the facts of the case and arrived at a finding relating to guilt contrary to
the finding of the Enquiry Officer and such finding being accepted by the Disciplinary
Authority without giving opportunity to the employee to comment upon the CVC Report
finding him guilty. In this case as noticed above, the Enquiry Report relating to guilt was
not referred to the opinion of the Vigilance Department at all. The Vigilance Department
neither expressed any view in regard to the finding of guilt recorded by the Enquiry
Officer nor did it re-assess the evidence or arrive at a finding different from that of the
Enquiry Officer. It merely opined that the case was not a fit one for showing leniency
while imposing punishment and left it to the Appointing Authority to take his own
decision in the matter. Therefore, this decision is also of no assistance. 1992 AIR
SCW 3353

26.3 Reference was next made to the decision of this Court in Mohd. Quaramuddin
(dead) By LRs. vs. State of AP [1994 (5) SCC 118]. In that case, the Chief Vigilance
Commissioner's report which formed part of the report of the enquiry and which was
taken into consideration by the disciplinary authority was not supplied to the employee. It
was held that the omission has vitiated the order of dismissal. The said decision is also of
no assistance.
26.4 The last decision relied on by the respondent was UP State Agro Industrial
Corporation Ltd. Vs. Padam Chand Jain 1995 SCC (L and S 1011). In that case, the
report of the Enquiry Officer was in favour of the employee exonerating him of all
charges. The Disciplinary Authority invited the comments of the Accounts Officer and
relying on the basis of the adverse comments made by such officer, held the employee
guilty and terminated him from service. This Court upheld the view of the High Court
that the decision of the Disciplinary Authority was vitiated on account of the same being
influenced by some extraneous material in the form of adverse comments of the Accounts
Officer. That is not the case here.
27. The learned counsel for respondent submitted that as the order of removal was set
aside and as the employer's second appeal was rejected, he should be permitted to support
the decision of the courts below by demonstrating that the Enquiry Officer had violated
the principles of natural justice and therefore, the order of removal deserves to be set
aside. This is not permissible. Though in the suit, the respondent had challenged the
enquiry as being opposed to principles of natural justice, and the finding guilt recorded
by the Enquiry Officer as being erroneous, he gave up those contentions before the first
appellate court, and restricted the challenge to the quantum of punishment and non-grant
of back wages. He cannot therefore be permitted to revive the contention that the Enquiry
Officer violated the principles of natural justice in conducting the enquiry.
28. At the relevant point of time the respondent was functioning as a Branch Manager. A
Bank survives on the trust of its clientele and constituents. The position of the Manager
of a Bank is a matter of great trust. The employees of the Bank in particular the Manager
are expected to act with absolute integrity and honesty in handling the funds of the
customers/borrowers of the Bank. Any misappropriation, even temporary, of the funds of
the Bank or its customers/borrowers constitutes a serious misconduct, inviting severe
punishment. When a borrower makes any payment towards a loan, the Manager of the
Bank receiving such amount is required to credit it immediately to the borrower's
account. If the matter is to be viewed lightly or leniently it will encourage
@page-SC2607
other Bank employees to indulge in such activities thereby undermining the entire
banking system. The request for reducing the punishment is misconceived and rejected.
29. In view of the above we allow these appeals and set aside the judgments and decrees
of the courts below and dismiss the respondent's suit.
Appeals allowed.
AIR 2008 SUPREME COURT 2607 "Yunis Ali v. Khursheed Akram"
(From : 2002 (2) WLC 206 (Raj))
Coram : 2 C. K. THAKKER AND LOKESHWAR SINGH PANTA, JJ.
Civil Appeal No. 1901 of 2003, D/- 28 -5 -2008.
Yunis Ali (Dead) Thru his L.Rs. v. Khursheed Akram.
Civil P.C. (5 of 1908), S.115 - REVISION - CONCURRENT FINDINGS - EVICTION -
HOUSES AND RENTS - concurrent findings of facts - Interference with - Eviction salt -
Non-payment of arrears of rent at enhanced rate - Trial Court determined provisional rent
and directed tenant to deposit arrears in Bank - Order upheld by first appellate Court -
Interference with well merited concurrent findings by High Court in revision simply
because another view of evidence is possible - Not proper - Moreso when final decision
in matter is yet to come.
Rajasthan Premises (Control of Rent and Eviction) Act (17 of 1950), S.13(3).
2002 (2) WLC 206 (Raj), Reversed. (Paras 15, 16)

Ajay Choudhary, for Appellant; Abhijeet Sinha and Ejaz Maqbool, for Respondent.
Judgement
LOKESHWAR SINGH PANTA, J. :- The subject-matter of the challenge in this appeal is
a judgment of the High Court of Rajasthan passed in S.B. Civil Revision Petition
No.669/2001 on 18th July 2001 setting aside the judgments of the Courts below where
both the trial court as well as the lower Appellate Court determined the provisional rent
under Section 13(3) of the Rajasthan Premises (Control of Rent and Eviction) Act, 1950
(hereinafter referred to as the Act of 1950) at the rate of Rs.400/- p.m. for the shop in
question with effect from 01.06.1994 to 01.07.1999. The High Court in revision re-
appreciated the evidences and reversed the concurrent findings of the Courts below and
held as under :-
"Thus, the provisional determination of rent by both the courts below at the rate of 400/-
p.m. appears on the very face to be illegal and both the courts below have committed
jurisdictional error in determining the rent at such rate and they should have determined
the rent at the rate of Rs.200/-p.m. as it was the rent last paid by the defendant-petitioner
to the plaintiff-respondent.
For the reasons stated above, it is held that the orders dated 16.02.2001 passed by the
learned Additional District Judge No.5. Jaipur City, Jaipur and 13.07.1999 passed by the
learned Additional Civil Judge (J.D.) No.3, Jaipur City, Jaipur are contrary to the
provisions of Section 13(3) of the Act of 1950 and interference becomes inevitable for
ends of justice and they are liable to be set aside and this revision is liable to be allowed.
Accordingly, this revision petition filed by the defendant-petitioner Khursheed Akram is
allowed and the impugned orders dated 16.02.2001 passed by the learned Addl. District
Judge No.5, Jaipur City, Jaipur and 13.07.1999 passed by the learned Addl.Civil Judge
(J.D.) No.3, Jaipur City, Jaipur are set aside. The learned Addl.Civil Judge (J.D.) No.3,
Jaipur City, Jaipur is directed to determine provisional rent at the rate of Rs.200/-p.m. It
is made clear that this provisional rent is not final, but is only interim till the final
decision of the court and subject to adjustment as provided under Section 13(8) of the Act
of 1950. No order as to costs."
2. Brief facts, leading to the filing of this appeal, are as under :-
Yunus Ali, the appellant (now represented through his legal representatives as the
appellants) was owner of shop No. 2 situated at House No. 242 at Nahari Ka Naka behind
Khetri House, Madina Masjid Road, Jaipur. By an oral agreement, he leased the shop on
rent to Khursheed Akram, tenant-respondent herein, on a monthly rent of Rs. 300/-
excluding electricity and water charges. The respondent-tenant also paid an advance
amount of Rs. 5,000/- which was to be adjusted against the instalments of rent or to be
returned when tenancy will expire. On 22.01.1993, a rent deed was duly executed
between the parties on a stamp paper of Rs. 10/- giving effect to the agreed terms and
conditions of the earlier oral agreement of monthly rental of Rs. 300/-. The agreement
was executed in the presence of the witnesses
@page-SC2608
and attested by a Notary.
3. In the month of March 1994, the original landlord at the request of the respondent-
tenant, made addition of a platform in front of the shop with stone floor and erection of
shutter over the shop. There was agreed marginal Increase of the rent amount after the
renovation was over. A fresh rent deed was duly executed in favour of the original
landlord on 01.04.1994 whereunder rent was increased to Rs. 400/- per month payable
w.e.f. April 1994. It was also agreed that the advance amount of Rs. 5000/- shall be
adjusted in 14 monthly instalments of the rent due upto 31.03.1994 @ Rs. 300/- per
month and thereafter upto 31.5.1994 @ Rs. 400/- per month and other terms remained
unchanged and unaltered. Though, in spite of repeated requests and demands of the
original-landlord, the respondent-tenant failed to pay rent even after the execution of rent
deed on 01.4.1994. Arrears of the rent started accumulating since 01.06.1994 after
adjustment of the advance amount of Rs.5,000/-. The landlord left with no other remedy
except to file a suit for eviction on 29.01.1997 before the Court of Additional Civil Judge
(S.D.) No. 3, Jaipur City, Jaipur inter alia contending that apart from various other
grounds mentioned in the plaint, the respondent-tenant has defaulted in payment of the
arrears of rent since 01.06.1994.
4. The respondent-tenant in his written statement pleaded that the agreed rent of the shop
was Rs.200/- per month and he had paid an advance amount of Rs. 10,000/-and later on
the rent was never agreed to be increased to Rs.300/- per month and further to Rs. 400/-
per month w.e.f. 01.04.1994 as claimed in the plaint. It was admitted by him that he had
paid rent upto April, 1995 @ Rs.200/- per month and in support thereof, he placed rent
receipts on the record.
5. On 13.07.1999, the learned Civil Judge, after considering the fact that the rent was
being paid by the respondent-tenant @ Rs. 200/- in support of which he had produced
cash receipts but since a fresh rent deed was executed between the parties, which prima
facie would reveal that the rent was agreed to be charged @ Rs. 400/- per month and on
the premise of the agreed rate of rent, the trial court determined the provisional rent under
Section 13(3) of the Act of 1950 @ Rs. 400/- per month from 01.06.1994 to 01.07.1999.
6. Being aggrieved by the above-said order of the learned Civil Judge, the respondent-
tenant preferred a Civil Miscellaneous Appeal before the learned District Judge, Jaipur
City, Jaipur which was transferred to the learned Additional District Judge No. 5, Jaipur
City, Jaipur, for trial. During the course of hearing of the appeal, the respondent-tenant
produced a copy of compromise deed dated 30.03.1993 before the Appellate Court in
support of his defence that the rent of the shop was Rs. 200/- per month and not Rs.400/-
per month. The learned Additional District Judge, on careful perusal of the said
document, arrived at the conclusion that the said deed pertained to shop No. 5 and not
shop No. 2 which was the subject-matter of the suit. The learned Additional District
Judge has found no error or illegality in the order of the learned Additional Civil Judge
and, accordingly, rejected the appeal of the respondent-tenant.
7. Thereafter, the respondent-tenant filed S.B. Civil Revision Petition No. 669/2001
before the High Court of Rajasthan, Bench at Jaipur, under Section 115 of the Civil
Procedure Code. The learned Single Judge of the High Court, as noticed earlier, allowed
the revision.
8. Hence, the original landlord filed this appeal by way of special leave. During the
pendency of this appeal, the landlord died and his legal representatives have been brought
on record as appellants.
9. Mr. Ajay Choudhary, learned counsel appearing on behalf of the appellants, submitted
that the impugned order is perverse, erroneous and illegal as the High Court has exceeded
its jurisdiction under Section 115 CPC to replace concurrent findings of facts with its
own findings as if it was exercising the jurisdiction of the Appellate Court. He contended
that the High Court has exercised its jurisdiction contrary to the well-settled law laid
down by this Court in a series of decisions that the High Court should not interfere in the
findings of facts recorded by the courts below based upon proper and reasonable
appreciation of evidence.
10.On the other hand. Mr. Abhijeet Sinha, learned counsel appearing on behalf of the
respondent-tenant, in support of the order, has submitted that the High Court in exercise
of its revisional powers vested in it under Section 115 of CPC has rightly interfered with
the erroneous and unsustainable orders of both the courts below and this
@page-SC2609
Court normally under Article 136 of the Constitution should not interfere with the well-
reasoned order of the High Court.
11. Having heard the learned counsel for the parties and having examined the orders of
the courts below, we are of the opinion that the High Courts order on the face of it does
not stand legal scrutiny and deserves to be set aside.
12. We do not think it proper and necessary to embark upon the facts of the present case
in detail. Suffice it to notice that the predecessor-in-interest of the appellants had entered
into oral agreement with the respondent-tenant to lease out shop No. 2 situated at House
No. 242 at Nahari Ka Naka behind Khetri House, Madina Masjid Road, Jaipur, to the
respondent-tenant at the rate of Rs.300/- per month as rent. The expenses of electricity
and water were also agreed to be paid separately as pleaded by the original landlord in the
plaint. In pursuance of the oral agreement, a rent deed was executed on 22-01-1993
incorporating all the agreed terms and conditions therein. The respondent-tenant also paid
an amount of Rs.5,000/- as advance to the predecessor-in-interest of the appellants, which
was agreed to be adjusted against the non-payment of rent by the respondent-tenant or
shall be returned to him at the time of vacation of the shop. The predecessor-in-interest of
the appellants pleaded in the suit that the respondent-tenant failed to pay the rent at the
agreed rate since 01-02-1993 onwards and it was desired by him that the rental amount
should be adjusted from the advance amount of Rs.5,000/- deposited by the respondent-
tenant as security with the predecessor-in-interest of the appellants. Accordingly, rent of
14 months from 01-02-1993 to 31-03-1994 was stated to have been adjusted. Some
addition was made to the shop by the predecessor-in-interest of the appellants at the
request of the respondent-tenant and thereafter rent @ Rs.400/- per month was agreed to
be paid by the respondent-tenant w.e.f. 01-04-1994, for which Rent Deed dated 10-04-
1994 was executed in the presence of the witnesses and duly attested by a Notary. The
amounts of two months rent from 01-04-1994 to 31-05-1994 @ Rs.400/- per month were
also adjusted out of the advance amount and the balance amount of rent was not paid by
the respondent-tenant despite repeated requests and demands made by the predecessor-in-
interest of the appellants.
13. The respondent-tenant pleaded before the trial court that the shop in question was
given to him on rent by the predecessor-in-interest of the appellants @ Rs.200/- per
month and he had paid Rs.5,000/- as advance against the amount of rent. On 30-03-1993,
a rent deed was executed and he had paid the rent upto January 1997, but the predecessor-
in-interest of the appellants did not give rent receipts after April 1995. The respondent-
tenant denied the claim of the predecessor-in-interest of the appellants that the monthly
rent of the shop was ever increased from Rs.300/- to Rs.400/- and he reiterated and
reasserted that the agreed rent was Rs.200/-per month.
14. The learned trial court, during the pendency of the eviction suit and on examination of
the rent deeds produced before him, determined the provisional rent of the shop @
Rs.400/- per month under Section 13(3) of the Act of 1950 payable by the respondent-
tenant for a period of 61 months, i.e. from 01-06-1994 to 01-07-1999, and the total
amount of arrears of rent works out to be Rs.24,400/- on which an interest at the rate of
6% has been imposed. The trial court directed the respondent-tenant to deposit the arrears
of rent together with interest in the bank account of the predecessor-in-interest of the
appellants and also directed the respondent-tenant to pay the rent, as determined,
regularly in the account of the predecessor-in-interest of the appellants. The order of the
trial court has been upheld by the first Appellate Court.
15. It is well-settled position in law that under Section 115 of the Code of Civil Procedure
the High Court cannot re-appreciate the evidence and cannot set aside the concurrent
findings of the Courts below by taking a different view of the evidence. The High Court
is empowered only to interfere with the findings of fact if the findings are perverse or
there has been a non-appreciation or non-consideration of the material evidence on record
by the courts below. Simply because another view of the evidence may be taken is no
ground by the High Court to interfere in its revisional jurisdiction.
16. Considering all the facts and circumstances as noticed above, we are constrained to
hold that the order of the High Court cannot be sustained and as such we set aside the
same. The High Court has acted in exercise
@page-SC2610
of its jurisdiction with material irregularity to interfere with the well-merited concurrent
findings and reasonings recorded by two courts below. The High Court, with respect to it,
has lost sight of the important fact that it was provisional determination of the amount of
rent by the trial court and the eviction suit is still pending before it for final decision.
17. For the foregoing reasons, the impugned judgment is set aside and the appeal is
allowed, but in the facts and circumstances of the case, without any order as to costs.
Appeal allowed.
AIR 2008 SUPREME COURT 2610 "Dharampal Satyapal Ltd. v. State of Bihar"
(From : Patna)*
Coram : 2 S. B. SINHA AND LOKESHWAR SINGH PANTA, JJ.
Civil Appeal Nos. 5779 with 5782 and 5785 of 2005, D/- 14 -5 -2008.
M/s. Dharampal Satyapal Ltd. and Anr. v. State of Bihar and Ors.
WITH
M/s. Agricultural Produce Market Committee, Muzzafarpur v. M/s. Prabhat Zarda
Factory (India) Ltd. and Ors.
AND
M/s. Prabhat Zarda Factory (India) Ltd. and Anr. v. State of Bihar and Ors.
Bihar Agricultural Produce Markets Act (16 of 1960), S.4A, S.39, S.3, S.4 -
AGRICULTURAL PRODUCE - HIGH COURT - Notification "No. 6/Misc. 13/91-6386,
S.O. 220 dated 31-6-1991 - Market fee - Levy on Zafrani Zarda, a manufactured item -
Petition challenging levy so also vires of notification dated 31-6-1991 including Zardas in
schedule - High Court disposing of petition by upholding levy from 31-8-1992 - Vires of
notification however not considered - Order of H.C. liable to be set aside - Matter
remitted. (Paras 7, 8, 9)
Cases Referred : Chronological Paras
1994 Supp (2) SCC 514 (Ref.) 2
Rakesh Dwivedi, Anoop G. Chaudhary, Vijay Hansaria, S. B. Sanyal, R. G. Padia, Mathai
M. Paikeday, Mrs. Shobha Dikshit, K. K. Mohan, Amit Kr. Singh, Shantanu Krishna, Ms.
Mukti Chowdhary, Anant Prakash. Ms. Sneha Kalita, Sanjai K. Pathak, Vivek Narayan,
Ajit Kumar Pande, Sunil Roy, Ms. Mandhurima Tatia, Anil Katiyar, V. K. Verma, Shishir
Pinaki, Amit Singh, P. L. Jose, Pradeep Misra, Daleep Dhayanti, Gopal Singh, Anukul
Raj, Manish Kr. and Chandan Kr. with them, for appearing parties.
* C.W.J.C. No. 10145 of 2001, D/- 7-10-2002 (Pat).
Judgement
S. B. SINHA, J. :- These three appeals involve a question as to whether the levy of
market fee on Zafrani Zarda in terms of the provisions of Bihar Agricultural Produce
Markets Act, 1960 (for short the Act) is valid which arise out of a common order dated
7th October, 2002 passed by the High Court of Patna in three writ petitions filed by the
appellants.
2. Writ petitioners are manufacturers of Zafrani Zarda. It is indisputably a manufactured
form of tobacco.
The question as to whether Zafrani Zarda is manufactured item of tobacco or not came up
for consideration before this Court in Agricultural Produce Market Committee and others
v. M/s. Prabhat Zarda Factory and another, [1994 Supp (2) SCC 514]. This Court noticed
that the definition of agricultural produce' has undergone a change so as to include the
'manufactured goods' therefrom by way of amendment on 30th April, 1982. This Court
noticed that the judgment of the High Court did not take into consideration the changed
definition of 'agricultural produce' with effect from 30th April, 1982 but upheld the
judgment of the High Court in respect of levy of market fee prior thereto.
3. Indisputably again with effect from 30th April, 1982 Zafrani Zarda was inserted in the
Schedule appended to the said Act, by reason of a Notification dated 31st July, 1991,
issued by the State of Bihar which reads thus :-
"No. 6/Misc. 13/91 - 6386 .... Governor of Bihar in exercise of its powers conferred under
Section 39 of Bihar Agricultural Produce Markets Act, 1960 (Bihar Act 16 of 1960)
makes the following amendment in the Schedule of the abovementioned Act :-
AMENDMENT
In the said list - Heading 11 - following heads should be added or included after head 1
under Narcotic - Tobacco.
2. Zarda
3. Zafrani Zarda etc.
@page-SC2611
The effect of this amendment shall be considered to be in force from the date of
implementation of Bihar Agricultural Produce Marketing Act, 1960.
By order of Governor of Bihar
Nagendra Tiwari
Joint Secretary to Govt."
4. The State Government issued another Notification being S.O. 220 dated 31st August,
1992 for regulation of sale, purchase, storage and process of all agricultural produce
mentioned in the Schedule of the Act in the areas covering 122 market committees
including the respondent Market Committee.
5. Questioning the orders of assessment of market fee relying on and on the basis of the
said G.Os., by the Assessing Authority, the Writ Petitioners filed writ applications before
the Patna High Court inter alia for the following reliefs :-
"i) issue Rule NISI calling upon the respondent to show cause as to why the notice
No.569 dated 08-06-2001, 638 dated 26-06-2001 and 770 dated 24-07-2001 as contained
in Annexures 6, 7, 9 and issued by the respondent Secretary, Agricultural Produce Market
Committee should not be set aside and quashed and upon return of the rule and after
hearing the parties make the rule absolute.
ii) Issue rule in the nature of mandamus restraining the respondents from enforcing the
provisions of Bihar Agriculture Produce Market Committee Act and the Rules thereunder
in respect of the petitioners dealing in zafrani zarda and spices and demanding any fee in
respect thereof and upon return of the rule and after hearing the parties make the rule
absolute.
iii) Issue Rule NISI in the nature of writ of mandamus calling upon the respondents to
show cause as to why the notification issued under section 39 published in the Bihar
Gazette Extra Ordinary dated 31-07-2001 (Annexure-2) be not declared invalid, illegal
without jurisdiction and unenforceable and upon return of the rule and after hearing the
parties make the rule absolute.
iv) Issue such other writ(s) order(s) direction(s) as your Lordships may deem fit and
proper."
6. In the said writ petitions, inter alia it was contended on behalf of the respondent-
Market Committee, that the market fee would be assessed from the date of coming into
force of the Amendment i.e. 30th April, 1982 from which date the definition of
'agricultural produce' had been amended.
7. The High Court by reason of the impugned judgment did not go into other contentions
raised by the parties. It was held that the market fee would be leviable with effect from
31st August, 1992. It failed to take into consideration the important question raised by the
petitioners that the Notifications were ultra vires the Act and/or would have no
application in relation to Zafrani Zarda. It also did not take into consideration the
contention of the Market Committee that having regard to the provisions of Section 4-A
of the Act, Sections 3 and 4 thereof were not required to be complied with.
8. The questions raised by the parties are of significance. They should have been dealt
with by the High Court.
9. We, therefore, are of the opinion that the impugned judgment cannot be sustained.
They are set aside accordingly and the matters are remitted to the High Court for
consideration and disposal thereof afresh as expeditiously as possible and preferably
within a period of six months from the date of receipt of a copy of this order.
If any application is filed by the parties hereto raising additional contentions, the same
may be considered by the High Court on their own merits.
10. The appeals are disposed of accordingly with no order as to costs.
Order accordingly.
AIR 2008 SUPREME COURT 2611 "Sulochana v. Rajendra Singh"
(From : Madhya Pradesh)
Coram : 2 S. B. SINHA AND LOKESHWAR SINGH PANTA, JJ.
Civil Appeal No. 3636 of 2008 (arising out of SLP (C) No. 21407 of 2006), D/- 16 -5
-2008.
Smt. Sulochana v. Rajendra Singh.
M.P. Accommodation Control Act (41 of 1961), S.45, S.23J - ACCOMMODATION
CONTROL - EVICTION - CIVIL COURT - EQUALITY - INTERPRETATION OF
STATUTES - Bar to jurisdiction of Civil Court - Landlady, a widow - Composite suit for
eviction filed on different grounds including bona fide requirement - Averments in plaint
not showing that suit is filed in capacity of
@page-SC2612
'specified landlord' - Jurisdiction of civil Court does not stand excluded - Moreso when
title of landlady was disputed.
Civil Second Appeal No. 260 of 2004, D/-28-09-2006 (MP), Reversed.
Civil P.C. (5 of 1908), S.9.
Constitution of India, Art.14.
Interpretation of Statutes - Exclusion provision.
Chapter III-A provides for special provisions. It is confined to eviction of tenants on
grounds of bona fide requirement of different classes of landlords specified therein. A
summary procedure is provided for. Recourse thereto can be taken only by the specified
landlord within the meaning of the provision of Section 23-J of the Act which means a
'landlord who is a widow or divorced wife' amongst others. Only a landlord who comes
within the purview of the said definition is entitled to file suit on the ground of his or her
bona fide requirement. Moreover, the definition of 'specified landlord' as contained in
Section 23-J of the Act is not as broad as the definition of the same term as contained in
Section 2(b) thereof. A statute must be read, keeping in view the Constitutional scheme of
equality as adumbrated in Article 14 of the Constitution of India. Once a special benefit
has been conferred on a special category of landlord, the same must receive strict
construction. Even otherwise, it is well settled, that an exclusion provision must be
construed strictly. A statute ousting jurisdiction of the civil Court should also be strictly
construed. (Paras 16, 36)
Moreover, the jurisdiction of civil Court is also to be determined having regard to the
averments contained in the plaint. Therefore, where appellant did not proceed on the basis
that she was a 'specified landlord' within the meaning of Section 23-J of the Act though
she was widow and a composite suit for eviction was filed, i.e., not only on the ground of
bona fide requirement but also on the ground of default of payment of rent as also denial
of relationship of landlord and tenant and respondent in his written statement not only
denied and disputed the title of the appellant but also denied and disputed that he was in
default, apart from the contention raised as regards the bona fide requirement of the
appellant, the issue as regards title over a property can be decided only by a civil Court
and, therefore, the suit as framed was maintainable. It was moreso when, not only a
decree for eviction was passed, a decree for payment of arrears of rent, which otherwise
could not have been granted by the Rent Controller, was also passed. Moreover the
premises were purchased by the appellant where the respondent was already a tenant she
was, thus, not a landlord within the meaning of Section 23-J of the Act. The relevant date
for claiming the special benefit of Chapter III-A was the date of her becoming a widow.
Civil Second Appeal No. 260 of 2004, D/-28-9-2006 (MP), Reversed. (Paras 21, 25,
26, 30, 37, 44)
Cases Referred : Chronological Paras
2006 (1) MPLJ 231 (Disting.)9, 34
2005 AIR SCW 2548 : AIR 2005 SC 2392 (Foll.) 20
2002 AIR SCW 1124 : AIR 2002 SC 1310 (Disting.) 9, 35, 42
2002 AIR SCW 2873 : AIR 2002 SC 2572 (Disting.) 9, 41
2002 (1) LSC (2) 22 (Ref.) 26
2002 (1) MPS 5 (Ref.) 26
1998 MP ACJ 162 (Ref.) 27
(1990) 1 SCC 324 (Foll.) 40
1990 MP ACJ 88 (Ref.) 26
(1988) 1 SCC 656 (Foll.) 39, 40
AIR 1984 SC 458 (Foll.) 38, 39, 40
1982 MPRCJ 62 (Ref.) 26
Sushil Kumar Jain, Puneet Jain, Sarad Kumar Singhania and Ms. Pratibha Jain, for
Appellant; Pramit Saxena, Amit Yadav and S.Y. Deshpande, for the Respondents.
Judgement
1. S. B. SINHA, J. :-Leave granted.
2. Whether the civil court has jurisdiction to entertain a composite suit filed by the
appellant herein for eviction of the tenant is the question involved in this appeal which
arises out of a judgment and order dated 28th September, 2006 passed by a learned Single
Judge of the High Court of Madhya Pradesh at Indore in Second Appeal No. 260 of 2004,
whereby and where-under while allowing the appeal filed by the respondent, the suit filed
by the appellant for eviction of the respondent was dismissed.
3. The basic fact of the matter is not in dispute.
4. The premises in dispute is a shop located on the ground floor of House No.370-D,
Parasi Mohalla, Neemuch Cantt, in the State of Madhya Pradesh. Appellant purchased the
property in question on 23rd
@page-SC2613
March, 1996 from Smt. Anntu Jenra w/o Sh. Turab Bhai. Respondent was a tenant under
the predecessor-in-interest of the appellant on a monthly rent of Rs.200/- per month. By a
letter dated 29th July, 1996 the appellant informed the respondent in regard to the
purchase of the property by her and requested the respondent for payment of monthly rent
to her. Since, despite the service of the said letter, the respondent failed and/or neglected
to make payment of rent, the appellant terminated the tenancy of the respondent and
requested him to vacate the tenanted premises. It was also mentioned that the shop in
question was required by the, appellant bona fide so as to enable her son to can out
business therein. Respondent, while denying any liability to pay any rent to the appellant,
also denied her title contending that he has not been informed of the sale of the property
by the original landlord in favour of the appellant.
5. Appellant thereafter filed a composite suit for eviction of the respondent on the
grounds of :- (i) default in payment of rent, (ii) her bona fide requirement; and (iii) denial
of her title on the part of the respondent.
6. The trial court considered the merit of the suit for passing a decree on the ground of
bona fide requirement as also on arrears of rent. A decree for mesne profits was also
passed. In regard to denial of title, the trial court noticed that since the earlier landlord did
not give any notice of transfer to the respondent, the title of the appellant was although
denied at that time but the tenant now accepted his title.
It was furthermore held that since the court had condoned the delay for deposit/ payment
of rent and allowed the respondent time to pay the rent, the delay in deposit of the same
cannot form the basis for grant of a decree for eviction on that count. However, as stated
earlier, the court decreed the suit on the ground of bona fide requirement on the part of
the landlord and directed the respondent to handover vacant possession within two
months.
7. An appeal, being Regular Civil Appeal No. 1A of 2004 filed by the respondent before
the District Judge was dismissed by a judgment and order dated 17th February, 2004.
8. Respondent preferred a second appeal before the High Court of Madhya Pradesh,
which as stated earlier, was allowed by reason of the impugned judgment and the suit
filed by the appellant was dismissed on that ground alone.
9

. The High Court in its judgment, relying on or on the basis Nandlal v. Nangibai [2006 (1)
M.P.L.J. 231], held that the civil court has no Jurisdiction as the suit was decreed only on
the ground of bona fide requirement on the part of the appellant. Nandlal (supra) relied on
two decisions of this Court in Dhannalal vs. Kalawatibai and others, [(2002) 6 SCC 16]
and Ashok Kumar Gupta vs. Vijay Kumar Agarwal. [(2002) 3 SCC 717]. 2002 AIR
SCW 2873
2002 AIR SCW 1124

10. Mr. Sushil Kumar Jain, learned counsel appearing on behalf of the appellant, would
submit that keeping in view the pleadings of the parties as also the findings of the learned
trial judge, the High Court must be held to have committed a serious error in holding that
the civil court had no jurisdiction to pass a decree for eviction. It was pointed out that the
respondent-tenant was inducted as a tenant in the suit premises as far as back in 1978 by
the previous owner and as the appellant purchased the suit property on 23rd March, 1996
whereas she became a widow, much earlier, i.e., on 9th July, 1990 and in that view of the
matter Chapter IIIA of Madhya Pradesh Accommodation Control Act, 1961 (in short, 'the
Act') will not be applicable.
11. Mr. Pramit Saxena, learned counsel appearing on behalf of the respondent, on the
other hand, drew our attention to the provisions of Section 45 of Act to contend that the
civil court's jurisdiction is completely ousted.
12. Before adverting to the rival contentions raised, we would notice some of the
provisions of the said Act.
The Act was enacted for giving protection to tenants belonging to the weaker section of
society who were incapable of building their own houses. Tenant has been defined in
section 2(i) to mean :-
"'tenant' means a person by whom or on whose account or behalf the rent of any
accommodation is, or, but for a contract express or implied, would be payable for any
accommodation and includes any person occupying the accommodation as a sub-tenant
and also, any person continuing in possession after the termination of his tenancy
@page-SC2614
whether before or after the commencement of this Act; but shall not include any person
against whom any order or decree for eviction has been made."
13. Eviction of the tenant is governed by Chapter III of the Act. Section 11-A of the Act
excludes applicability to certain categories of landlords as specified in Chapter III-A of
the Act and as defined in Section 23-J. Section 12, however, starts with a non obstante
clause; it specifies the grounds only on the basis whereof the landlord may file a suit for
eviction of tenant from any accommodation.
14. Admittedly, denial of relationship of landlord and tenant, arrears of rent and the bona
fide requirement are some of the grounds on the basis whereof a suit for eviction can be
filed.
15. Section 45 of the Act excludes the jurisdiction of the civil court stating :-
"45. Jurisdiction of Civil Courts barred in respect of certain matters.- (1) Save as
otherwise expressly provided in this Act, no Civil Court shall entertain any suit or
proceeding in so far as it relates to the fixation of standards rent in relation to any
accommodation to which this Act applies or to any other matter which the Rent
Controlling Authority is empowered by or under this Act to decide, and no injunction in
respect of any action taken or to be taken by the Rent Controlling Authority under this
Act shall be granted by any Civil Court or other authority.
(2) Nothing in sub-section (1) shall be construed as preventing a Civil Court from
entertaining any suit or proceeding for the decision of any question of title to any
accommodation to which this Act applies or any question as to the person or persons who
are entitled to receive the rent of such accommodation."
Sub-section (6) of Section 13 of the Act, however, provides for the benefit of protection
against eviction, stating :-
"13. When tenant can get benefit of protection against eviction.-
(6) If a tenant fails to deposit or pay any amount as required by this Section, the Court
may order the defence against eviction to be struck out and shall proceed with the hearing
of the suit, appeal or proceeding, as the case may be."
16. Chapter III-A provides for special provisions. It is confined to eviction of tenants on
grounds of bona fide requirement of different classes of landlords specified therein. A
summary procedure is provided for. Recourse thereto can be taken only by the specified
landlord within the meaning of the provision of Section 23-J of the Act which means a
'landlord who is a widow or divorced wife' amongst others. Amongst others a servant of
any Government including a member of defence services, would also fall within the
purview of the said definition. Only a landlord who comes within the purview of the said
definition is entitled to file suit on the ground of his or her bona fide requirement.
17. Section 23-H provides for deposit of rent pending proceedings for eviction or for
revision.
18. The jurisdiction of the civil court can thus be excluded only if the matter comes
within the purview of Section 45 of the Act of Chapter III thereof. It is beyond any cavil
that the application for eviction contemplated by Chapter III-A relates to an eviction of
the tenant by the landlord as defined in Section 23-J of the Act.
19. Ex facie Section 45 of the Act has no application to the facts and circumstances of
this case. Section 45 is subject to the other provisions contained therein; one of them,
indisputably is Section 12 which confers jurisdiction upon the civil court to entertain a
suit for eviction of the tenants subject, of course, to the case falling under one or more
grounds specified therein.
20

. It is now well settled that the provision excluding jurisdiction of the civil court are to be
strictly construed. They are not to be inferred readily. [See Swamy Atmananda and Others
v. Sri Ramakrishna Tapovanam and Others (2005) 10 SCC 51] 2005 AIR SCW 2548

21. The jurisdiction of civil court is also to be determined having regard to the averments
contained in the plaint. Appellant did not proceed on the basis that she was a 'specified
landlord' within the meaning of Section 23-J of the Act. Furthermore a composite suit for
eviction was filed, i.e., not only on the ground of bona fide requirement but also on the
ground of default of payment of rent as also denial of relationship of landlord and tenant.
22. It was explained as to why the civil court had the requisite jurisdiction.
@page-SC2615
23. Requisite averment as regards the cause of action for the said suit was made in
paragraph 10 of the plaint which reads as under :-
"(10) That, despite communicating information through notice to the defendant about
having purchased the disputed shop by the plaintiff, and about bona fide and reasonable
necessity of the suit/disputed shop along with possession of excess area than that of
tenanted portion, along with the arrears of the rent thereof, for opening of the shop for
medicines by her son Rajesh Kumar, and because of denying by the defendant to
recognize the plaintiff as the owner of the disputed shop, as also because of denial by the
defendant to pay the arrears of the rent as well as handing over possession of the shop,
the plaintiff has been compelled to file this suit."
24. It is also relevant to notice the prayers made in the said suit, which are :-
"13) That the plaintiff prays for the following relief against the defendant :-
a) That a decree of eviction may be passed in favour of the plaintiff and against the
defendant, to vacate the municipal house No.370-D, in whose north is public road, in
south is the house No.370-F; in east is the house No.370-D; and in west is joint gali and
house No.370-E are located, and in which the defendant is in occupation against rent and
is carrying on his business by the name and style of M/s. Rathore and Sons; and its vacant
possession be peacefully awarded to the plaintiff from the defendant.
b) That, the plaintiff be awarded arrears of the rent from the defendant amounting to
Rs.3000/- and decree may be passed in favour of the plaintiff and against the defendant,
and from the date of institution of the suit till the date of its remittance interest at the rate
of Rs.2/- per hundred per month may also be awarded by way of compensation and
belated payment against use and utilization.
c) That, from the date of the institution of the suit till receipt of vacant possession of the
disputed premises, compensation be awarded at the rate of Rs.200/- per month against the
use and utilization of the disputed premises by the defendant.
d) That, the total expenses of the suit be awarded to the plaintiff from the defendant along
with interest at the rate of Rs. 2/- per hundred per month, from the date of the institution
of the suit till its recovery.
e) That, any other justified relief which may be considered to be eligible by the plaintiff
may also be awarded from the defendant."
25. Respondent in his written statement not only denied and disputed the title of the
appellant but also denied and disputed that he was in default, apart from the contention
raised as regards the bona fide requirement of the appellant, inter alia stating :-
"1) That the contents of paragraph 1 of the plaint are not admitted. The ex-owner/
landlord of the house Antu Jehara wife of Shri Turab Bhai (H.M. Fazal Hussain) resident
of Bombay has not communicated any information uptill today to the defendant about
transfer of proprietorship of the disputed premises; nor has appraised about this fact that
presently the defendant has become tenant of the plaintiff. In the notice issued by the
plaintiff, the date of purchase of the disputed premises has been shown as 29th of March,
1996 whereas in the plaint, the date of purchase has been shown as 23rd of March, 1996,
and due to this reason it is not known as to on which date the plaintiff has become the
owner/landlord of the disputed premises. The plaintiff has mentioned entire contents in
paragraph No. 1 of the plaint as false and illusionary. The plaintiff should prove the
proprietorship of the disputed premises."
26. Indisputably, the issue as regards title over a property can be decided only by a civil
court and, therefore, there cannot be any doubt whatsoever that the suit as framed was
maintainable. Learned trial Judge however, in regard to the issue of denial of relationship
of landlord and tenant opined :-
"20) But, the defendant has revealed the reason about denial of the title of the plaintiff
due to non-communication of any information by his ex landlord Antu Jehara; and it has
been made clear in para 26 of his cross-examination that when he had received the notice
of the plaintiff, then he was not admitting the plaintiff as its landlord. But now he admits
the plaintiff to be the landlord and is also remitting the rent. Therefore, in such a
circumstance, the defendant has disclosed the reason about the denial of the title of the
plaintiff. Therefore, in this case, he is not found liable to be evicted on the basis of denial
of title, when
@page-SC2616
he is accepting the title of the plaintiff. As regards the ground of default, the trial court
held :-
"21) The plaintiff has also advanced an argument that the defendant has not deposited the
rent within the prescribed period. He has not deposited the rent within a period of one
(month) since receipt of the notice, then any benefit would not be accruable to the
defendant by depositing the rent later on, and simply on the basis of having withdrawn
and having received the rent through court, it would not be an abdication by the plaintiff
to have left the ground under Section 12(A); whereas the plaintiff himself does not
abdicate this right of her own self. On the aforesaid point, following rulings have been
cited on behalf of the plaintiff :-
i) Hiralal v/s. Harisingh, 1990 M.P.A.C.J. 88;
ii) R.C. Tambrakar and Others v/s. Nidhi Lekha, 2002 (1) L. S.C. (2) 22.
iii) Sushil Srivastava v/s. Nafis Ahamad, 2002 (1) M.P.S. 5; and
iv) Kalyansingh v/s. Ramswarup, 1982 M.P.R.C.J. 62.
But in these citations it is also mentioned that if permission is granted by the court for
depositing the rent belatedly, i.e. delay is condoned, then simply on this ground, eviction
should not be allowed."
It was, therefore, not a case that no cause of action had arisen to file the suit for eviction
on the ground of default or denial of title, but they were negatived having regard to the
subsequent events.
27. One of issues which arose for consideration of the learned trial Judge was the
jurisdiction of the civil court. The learned Judge held :
"24) During the course of the arguments, the defendant has also raised an objection to the
effect that the plaintiff being a woman is widowed and on the basis of necessity, proper
forum is not a civil court, but is the Rent Control Authority, and in support of this
argument has cited the ruling of Narayan Rao v/s. Parvatibai, 1998 M.P.A.C.J. 162.
25) In the aforesaid ruling, the suit was filed for eviction simply on the ground of bona
fide necessity i.e. was filed for obtaining possession, in which the point about the arrears
of the rent was not found; but in the present case the plaintiff has since beginning has
instituted this suit for recovery of arrears of the rent amounting to Rs.3000/-and for
eviction; and this issue has been framed being disputed amongst the rival parties, and
therefore, in such a situation the facts of this case and the citation being different, any
relief is not available to the defendant from the aforesaid ruling, and in this respect the
objection of the defendant is rejected."
28. So far as the ground of bona fide necessity on the part of the appellant is concerned, it
was admittedly held in her favour.
29. The reliefs granted by the civil courts in favour of the appellant are as under :- "a) The
defendant should hand over the vacant possession of the disputed premises of House
No.370-D, Parsi Mohalla, Neemuch Cantt. to the plaintiff within two months of the date
of the judgment.
b) The defendant should pay rent to the plaintiff in respect of the disputed premises from
23rd of March, 1996 to 22nd of June, 1997 at the rate of Rs.200/- per month, and
thereafter uptil handing over vacant possession, should pay at the rate of Rs.200/- per
month against its use and utilization. In this context, the plaintiff would be entitled to
recover the rent deposited by the defendant in the court."
30. It is, therefore, evident that not only a decree for eviction was passed, a decree for
payment of arrears of rent, which otherwise could not have been granted by the Rent
Controller, was also passed.
31. Before the first appellate court, inter alia, an application was filed for rejection of the
plaint. It was rejected. The first appellate court held :-
"43. Because the plaintiff has filed the suit apart from the necessity for the business of her
son, on the grounds of denial of title and default in payment of rent; therefore such a suit
falls within the jurisdiction of a civil court. Therefore, the amendment which has been
proposed by the plaintiff, the same is unnecessary and is not bona fide. Due to the reason
of such a situation, there is no necessity to dismiss the suit also."
32. In the second appeal, however, a purported substantial question of law was framed
which reads, thus :-
"Whether, in the facts and circumstances of the case, Civil Court had the jurisdiction
@page-SC2617
to entertain a composite eviction suit filed by a landlord covered by section 23(J) on
various grounds including 12 (1)(f) of the Act."
33. As noticed hereinbefore the said substantial question of law has been answered in
favour of the respondent.
34. The High Court proceeded on the basis that the civil court's jurisdiction would stand
ousted if the provisions contained in Sections 11, 12, 23-A, 23-J and 45 of the Act are
conjointly read stating :-
"After having heard learned counsel and going through material available on record, we
do not think that learned counsel for the appellant is right in submitting that courts below
had the jurisdiction to entertain the composite suit for eviction in the facts of the present
case. The point and controversy raised in this appeal stands decided by this court in
Nandlal case supra. No contrary view of larger bench or Supreme Court was brought to
notice. No doubt as a general rule, in all types of civil disputes, civil courts have
jurisdiction unless a part of cause of action is carved out from such jurisdiction, expressly
or by implication. In such a situation, it does not amount to splitting of cause of action.
On a conjoint reading of relevant provisions of the Act and Code of Civil Procedure, to us
it is clear that Civil Court's jurisdiction is barred in respect of composite claim for
eviction on bona fide need set up by the special category landlord covered by Section
23(J) of the Act. In view of the above discussions, we have no hesitation in holding that
in the facts of the case in hand, civil court acted without jurisdiction while granting an
eviction decree on the grounds of bona fide need set up by the plaintiff who is
indisputably covered by Section 23-J of the Act."
35
. With respect, the learned Single Judge failed to notice that the definition of 'landlord' as
contained in Section 2(b) and Section 23-J are different. The learned Judge furthermore
failed to notice the limited application of Chapter III-A of the Act. Some decisions have
been noticed by the learned Single Judge, including Ashok Kumar Gupta vs. Vijay Kumar
Agarwal, [(2002) 3 SCC 717] to which we would refer to a little later. 2002 AIR
SCW 1124

36. The definition of 'specified landlord' as contained in Section 23-J of the Act is not as
broad as the definition of the same term as contained in Section 2(b) thereof. A statute
must be read, keeping in view the constitutional scheme of equality as adumbrated in
Article 14 of the Constitution of India. Once a special benefit has been conferred on a
special category of landlord, the same must receive strict construction. Even otherwise, it
is well settled, that an exclusion provision must be construed strictly. A statute ousting
Jurisdiction of the civil court should also be strictly construed.
37. Appellant has purchased the premises on 23rd March, 1996 whereas the respondent
was inducted as tenant of the premises way back in 1978. It is, therefore, not a case where
the respondent was inducted as a tenant by the appellant. She was, thus, not a landlord
within the meaning of Section 23-J of the Act. The relevant date for claiming the special
benefit of Chapter III-A was the date of her becoming a widow.
38

. An identical question came up for consideration in Winifred Ross and another Evi
Fonseca and others, vs. [(1984) 1 SCC 288 wherein application of a pari materia
provision contained in Section 13-A1 of the Bombay Rents, Hotel and Lodging House
Rates Control Act, 1947 fell for consideration of this Court. AIR 1984 SC 458

Plaintiff therein was an officer of the armed forces. This Court while lauding the object of
the Act, however, held :- Para 5 of AIR

"Even the widows of such landlords may under clause (b) of Section 13-A1 can recover
possession of such buildings if they satisfied the conditions mentioned therein. An
analysis of clause (a) of Section 13-A1 shows that the person who wishes to claim the
benefit of that section should be a landlord of the premises while he is a member of the
armed forces of the Union and that he may recover possession of the premises on the
ground that the premises are bona fide required by him for occupation by himself or any
member of his family on the production of the required certificate either while he is still
in service or after his retirement. The essential requirement is that he should have leased
out the building while he was a member of the armed forces. His widow can also recover
the premises of which she is or has become the landlord under clause (b) subject to
fulfilment of the conditions. Having regard to the object and purposes of the
@page-SC2618
Act and in particular Section 13-A1 it is difficult to hold that Section 13-A1 can be
availed of by an ex-member of the armed forces to recover from a tenant possession of a
building which he acquires after his retirement. Acceptance of this argument will 'expose
the very Section 13-A1 of the Act to a successful challenge on the ground of violation of
Article 14 of the Constitution for if that were so, a retired military officer who has no
house of his own can purchase any building in the occupation of a tenant after his
retirement, successfully evict a tenant living in it on the ground that he needs it for his
use, then sell it for a fancy price and again because he has no house of his own, he can
again acquire another building and deal with it in the same way. There appears to be no
restriction on the number of times he can do so. It was argued that he would not be able
to get the requisite certificate under the Act more than once. A reading of Section 13-A1
of the Act shows that the certificate should show that the person concerned has been a
member of the armed forces and that he does not possess any other suitable residence in
the local area where he or members of his family can reside. Those conditions being
satisfied the certificate cannot be refused. A liberal construction of Section 13-A1 of the
Act as it is being pressed upon us would also enable unscrupulous landlords who cannot
get rid of tenants to transfer their premises to ex-military men, as it has been done in this
case in order to avail of the benefit of the said section with a private arrangement between
them. It is also possible that a person who has retired from the armed forces may after
retirement lease out a premises belonging to him in favour of a tenant and then seek his
eviction at his will under Section 13-A1 of the Act."
39. Winfred Ross and various other decisions came up for consideration again before this
Court in Dr. D.N. Malhotra vs. Kartar Singh, [(1988) 1 SCC 656]. Following Winfred
Ross (supra), it was held :-

"12. On a conspectus of the decisions referred to hereinbefore more particularly the


decision rendered by this Court in the case of Mrs. Winifred Ross v. Mrs. Ivy Fonseca it
is well settled that in order to get the benefit of eviction of the tenant in a summary way
the ex-serviceman must be a landlord qua the premises as well as the tenant at the time of
his retirement from service. The ex-serviceman is not competent to make an application
to the Rent Controller to get possession of his house by evicting the tenant in a summary
way unless and until he satisfies the test that he is a landlord qua the premises and the
tenant at the time of his retirement or discharge from service." AIR 1984 SC 458

40. The question yet again came up for consideration before a three Judge Bench of this
Court in Bhagwat Dutt Rishi vs. Raj Kumar, [(1990) 1 SCC 324]. The ratio laid down in
Winfred Ross (supra) and Dr. D.N. Malhotra (supra) was upheld stating :-

"10. In Malhotra case, this Court was called upon to consider Section 13-A1 of the very
Act with which we are now concerned. On the basis of the ratio in Winifred Ross case,
this Court came to the conclusion that until the landlord satisfied the test that he was a
landlord qua the premises and the tenant at the time of his retirement or discharge from
service, he would not be entitled to the benefit of Section 13-A of the Act. AIR 1984 SC
438

11. It is not disputed that the appellant retired on September 30, 1981. On the finding the
appellant is right in his submission that this was not a case of transfer with an oblique
motive but as the property belonged to a Mitakshara father, upon his death the property
has come to his hands. This feature which is different from the facts appearing in the two
reported decisions, however, would not persuade us to give a different meaning to the
definition in Section 2 (hh). In both the cases, for good reason this Court came to the
conclusion that the public officer should have been a landlord of the premises in question
while in service. Admittedly, the appellant was not the landlord before he superannuated."
41

. We may now examine the decision whereupon reliance has been placed by the High
Court, i.e., Dhannalal (supra). In Dhannalal (supra) the question which arose for
consideration was as to whether a specified landlord could file a composite suit along
with others for whose bona fide requirement the eviction of the tenant was sought for.
Holding that in such a case even a suit by a co-owner alone would be maintainable, it was
opined :- 2002 AIR SCW 2873

"17. It follows that a widow, who is a co-owner and landlady of the premises can in
@page-SC2619
her own right initiate proceedings for eviction under Section 23-A(b), as analysed
hereinbefore, without joining other co-owners/ co-landlords as party to the proceedings if
they do not object to the initiation of proceedings by such landlady, because she is the
owner of the property and requires the tenanted accommodation for the purpose of
continuing or starting the business of any of her major sons. The major sons though co-
owners/co-landlords may not have been joined as party to the proceedings but it would
not adversely affect the maintainability of the proceedings. It would also not make any
difference if they are also joined as party to the proceedings. Their presence in the
proceedings is suggestive of their concurrence with the widow landlady maintaining the
proceedings in her own right."
On the aforementioned narrow context of the factual matrix involved therein, it was
held :-
"19. .....The requirement pleaded is the requirement of a widow landlady for continuing
or starting the business of her major sons. In proceedings for eviction of a tenant it is
permissible for all the co-owner landlords to join as plaintiffs. Rather, this is normally
done. Now, if they all file a claim before the civil court, an objection may possibly be
raised on behalf of the defendant tenant that the widow landlady being one of the
claimants for eviction she must go to the Rent Controlling Authority under Chapter III-A.
If they collectively join in initiating the proceedings for eviction of the tenant before the
Rent Controlling Authority under Chapter III-A the defendant tenant may object that the
requirement being that of the major sons who are themselves applicant landlords the
claim should have been filed before the civil court, as is the plea before us. How can such
dilemma be resolved ?
20. Both the learned Senior Counsel for the parties stated that there is no specific
statutory provision nor a binding precedent available providing resolution to the problem
posed. Procedural law cannot betray the substantive law by submitting to subordination
of complexity. Courts equipped with power to interpret law are often posed with queries
which may be ultimate. The judicial steps of the Judge then do stir to solve novel
problems by neat innovations. When the statute does not provide the path and precedents
abstain to lead, then they are the sound logic, rational reasoning, common sense and urge
for public good which play as guides of those who decide. Wrong must not be left
unredeemed and right not left unenforced. Forum ought to be revealed when it does not
clearly exist or when it is doubted where it exists. When the law - procedural or
substantive - does not debar any two seekers of justice from joining hands and moving
together, they must have a common path. Multiplicity of proceedings should be avoided
and same cause of action available to two at a time must not be forced to split and tried in
two different fora as far as practicable and permissible."
The said decision, therefore, in our opinion, cannot be said to have any application to the
present case.
42

. Ashok Kumar Gupta (supra) in fact runs counter to the contention of the respondent.
Noticing Sections 12, 23-A, 23-J and Section 45 of the Act it was held : 2002 AIR
SCW 1124, Para 9

"10. The position after 16-1-1985 is that only in respect of the aforementioned categories
of the landlords the Rent Controlling Authority has jurisdiction to order eviction of a
tenant on grounds of bona fide requirement under Section 23-A. A conjoint reading of
Sections 11-A, 12, 23-A, 23-J and Section 45 would show that in regard to the bona fide
personal requirement of the landlord who does not fall within the specified categories in
Section 23-J, the civil court has jurisdiction to entertain a suit and pass decree under
clause (e) of sub-section (1) of Section 12 of the Act. It follows that the civil court rightly
entertained counter-claim under Section 12(1)(e) of the Act so the decree passed by it is
not vitiated for want of jurisdiction."
43. Thus, any matter which stricto sensu does not come within the purview of Chapter
III-A would be entertainable by a civil court. This ratio of the decisions, in our opinion,
was wrongly applied.
44. We have, therefore, no hesitation to hold that the decision of the High Court is
unsustainable. The same is set aside accordingly. The appeal is allowed with no order as
to costs.
Appeal allowed.
@page-SC2620
AIR 2008 SUPREME COURT 2620 "New India Assurance Co. Ltd. v. M/s. Hira Lal
Ramesh Chand"
Coram : 3 B. N. AGRAWAL, PRAKASH PRABHAKAR NAOLEKAR AND R. V.
RAVEENDRAN, JJ.
Civil Appeal No. 4306-4307 of 2003, D/- 13 -6 -2008.
New India Assurance Co. Ltd. v. M/s. Hira Lal Ramesh Chand and Ors.
(A) Marine Insurance Act (11 of 1963), S.4 - INSURANCE - WAREHOUSE - Insurance
cover - Extending "warehouse to warehouse" - Thus, it cannot be said that insurance
cover is available only in regard to maritime perils.
In view of the insurance cover extending 'warehouse to warehouse' the consignments are
covered by insurance not only during the sea journey, but beyond as stated in the policy.
Therefore, the plea by the insurer that the insurance cover is available only in regard to
martime perils that is perils relating to or incidental to the navigation of the sea may not
be correct. Having regard to Section 4 of the Marine Insurance Act and the terms of the
policy undertaking insurance cover against wider risks, the policy of insurance would
cover the loss not only while goods are navigating the sea but also any loss or damage
during transit from the time it leaves the consignor's warehouse till it reaches the
consignee's warehouse. The cover against risks will, however, cease on the expiry of 60
days after discharge of the consignment from the vessel at the final port of discharge, if
the goods do not reach the consignee's warehouse or place of storage for any reason
within the said 60 days. (Para 17)
There is a difference between marine insurance policies which extend cover only against
marine losses or maritime perils (as enumerated) and marine 'extra' insurance policies
which extend cover against all risks from consignor's warehouse to consignee's
warehouse which include not only the sea journey but also the land journey at either end.
(Para 19)
(B) Marine Insurance Act (11 of 1963), S.4 - Consumer Protection Act (68 of 1986),
S.2(g) - INSURANCE - CONSUMER PROTECTION - Claim against insurer -
Consignment covered by insurance - There is no averment or evidence that consignments
were lost or damaged - Nor it was averred that holder of documents of title applied for
delivery of consignments and delivery was refused - No allegation that consignments
were wrongly delivered within stipulated period of sixty days - Liability of insurer came
to end - Complaint against insurer - Not maintainable.
The complainants had booked the consignments showing the consignee as 'unto order'
thereby indicating that the goods covered by the Bills of lading should be delivered only
to the holder /endorsee of the Bills of Lading. There is also no doubt that the original
documents were not cleared/retired by the buyer 'Atlanta Rugs Inc.' and that the original
documents were ultimately returned by the foreign correspondent bank to Punjab
National Bank, bank of complainant and they are lying with Punjab National Bank. There
is also no doubt that the consignments were insured against all risks of loss and damage.
There is no averment or evidence that the consignments were lost or damaged. Nor is
there any averment that the holder of the documents of title applied for delivery of the
consignments and was denied or refused delivery on account of nonavailability of the
consignments either due to pilferage, loss or misdelivery. When there is no allegation or
proof of Sun Trust Bank (foreign Correspondent Bank) having applied for delivery and
refusal of delivery, it is inconceivable how the complainants can maintain a claim against
the insurer. (Paras 31, 32)
Failure of the buyer to make payment and take delivery is not a 'loss' of consignment
which is covered by the Insurance Policy. The complainant should make out a case of
actual 'loss' of the consignment covered by the contract of insurance or non-delivery of
the consignment, that is refusal to meet a demand for delivery. The question is whether
the complainants have proved such loss or non-delivery. The case of the complainant as
put forth in the complaint and reiterated in the affidavit is that they have dispatched the
consignment to Atlanta, foreign country, that the consignments were insured against all
risks; that the buyer did not retire the documents by making payment; that they do not
know what happens to the consignments and that therefore, the Insurer ought to have paid
them the value of the consignment and failure to do so amounted to deficiency in service
as contemplated under the Consumer Protection Act, 1986. These are not allegations or
proof of loss or non-delivery sufficient to foist any liability on the insurer. (Para 33)
@page-SC2621
(C) Consumer Protection Act (68 of 1986), S.2(g) - CONSUMER PROTECTION -
Deficiency in service - Claim against insurer - Not lodged in writing - Complainants did
not produce any document showing lodging of claim - It was on mere oral intimation to
insurer the investigation by surveyor was set in motion - Contents of report already
notified to complainants by surveyor in telexes - In facts failure to furnish copy of
Surveyor's report to complainant - Cannot be termed as deficiency in service. (Para
41)
Cases Referred : Chronological Paras
2007 AIR SCW 902 (Ref.) 19
1993 AIR SCW 2116 : AIR 1993 SC 2054 (Disting.) 18, 19
ILR (1974) 2 Ker 649 18, 19
Raju Ramachandran, Sr. Advocate, M.K. Dua, Kishore Rawat, Debasis Misra, for
Appellant Nagendra Rai, Harish Chander and Vivek Krishna Tankha, Sr. Advocate, M. T.
George, N. Ganpathy, Subramonium Prasad, Imtiaz Ahmed, Ms. Naghma Imtiaz, Farukh
Rashid, V.N. Raghupathy, J.R. Midha, Pranab Kumar Mullick, S.K. Pattnaik and Anshul
Raj (For M/s. Equity Lex Associates), for Respondents.
Judgement
1. R. V. RAVEENDRAN, J. :-These appeals under Section 23 of the Consumer Protection
Act, 1986, are filed against the common order dated 31-1-2003 passed by the National
Consumer Disputes Redressal Commission, New Delhi ('Commission' for short) allowing
in part OP No. 45 of 1997 and OP No. 49 of 1997. OP No.45 of 1997 was filed by M/s.
Hira Lal Ramesh Chand and its partner Rajender Kumar Jain (respondents 1 and 2 in CA
No.4306/2003). OP No.49 of 1997 was filed by M/s. Ratan Chand Deep Chand and its
two partners (respondents 1 to 3 in CA No.4307/2003).
2. As the ranks of parties differ in the two appeals and as some parties were given up
before the Commission, for convenience, we will also refer to the parties as follows :
New India Assurance Co. Ltd., as 'Appellant' or 'Insurer'; M/s. Hira Lal Ramesh Chand
and its partners (Respondents 1 and 2 in the first matter) and M/s. Ratan Chand Deep
Chand and its partners (Respondents 1 to 3 in the second matter) as the 'complainants';
M/s. Niranjan Shipping Agency Pvt. Ltd., (third Respondent in the first matter and fourth
Respondent in the second matter) as 'Niranjan Shipping'; Punjab National Bank (fourth
Respondent in the first matter and fifth Respondent in the second matter), and Bank of
Baroda (sixth Respondent in the second matter) by their names; Atlanta Rugs Inc. as the
'Buyer'; and Overseas Container Lines Inc. (the Non-Vessel Owning Common Carrier
acting as shipping Agent) as 'NVOCC' or 'Overseas Container'.
3. The case of the complainants, in brief, is as follows : Complainants are manufacturers
of Rugs and Durries, carrying on business at Mirzapur, U.P. In pursuance of orders placed
by Atlanta Rugs Inc., Atlanta (for short the 'buyer'), M/s. Hira Lal Ramesh Chand
dispatched 17 consignments of rugs and durries of the value of US $ 4,06,096 between
15-3-1995 and 29-6-1995; and M/ s. Ratan Chand Deep Chand dispatched 38
consignments of the value of US $ 8,87,973 between 23-8-1994 and 4-7-1995. The
consignments were entrusted to M/s. Overseas Container Line Inc., a non-vessel owning
shipping Agent represented by its Agent Niranjan Shipping Agency (P) Ltd., for
transhipment from Mumbai to Atlanda (USA). The Bill of Lading issued by Overseas
Container in regard to each of the consignment showed the consignee as "Unto order"
and party to be notified as "Atlanta Rugs Inc.". All the consignments were insured by the
consignors, with the New India Assurance Co. Ltd. The original documents relating to the
consignments were forwarded by Niranjan Shipping to the Bankers of complainant -
Punjab National Bank. The complainants obtained credit facilities from Punjab National
Bank by discounting the Bills and endorsed the Bill of Lading in favour of the said Bank.
The said Bank, in turn, forwarded the original documents of title to its agent Sun Trust
Bank (earlier known as Trust Company Bank) Atlanta, for collection, by endorsing the
documents in their favour. The buyer (Atlanta Rugs Inc.) did not make payment and
obtain release of the documents of title. They therefore made efforts to contact the buyer
and the shipping Agent-Overseas Container. They were also not able to locate them. Nor
were they able to find out the whereabouts of the consignments. Therefore they
telephonically lodged an oral claim with the insurer on 2-2-1996 seeking payment of the
value of the consignments. The insurer directed them to get in touch with their Surveyor-
cum-Claim Settlement Agent at Atlanda - M/s. Toplis
@page-SC2622
and Hoarding Inc. They accordingly requested the said Surveyor to inquire and
investigate the matter and issue necessary certificates. The surveyor submitted their
reports to the Insurer, but failed to furnish copies thereof to the complainants. Their claim
was not settled by the Insurer for more than a year in spite of reminders. Such failure
amounted to deficiency in service and consequently the insurer became liable to pay the
value of the consignments and the other amounts claimed, as compensation.
4. The complainants sought a direction to the New India Assurance Co. Ltd. (appellant) to
pay the following amounts as compensation :

Particulars
Claim in OA 45/1997 Amount (in US $)
Claim in OA 49/1997 Amount (in US $)
a) Insurance amount (unrealized value of shipments) : 406096 887973

b) Increase in cost of goods (10%) : 40609 88797


c) Compensation for mental agony : 100000 100000
d) Business loss for one year : 58091 100000
e) Expenses for pursuing the claim : 10000 10000
______________ ______________
TOTAL : 614796 1186770
f) Interest at 24% p.a. with quarterly rests on the unrealized value of shipments from
the date of claim to date of complaint 108253 233072
g) Pendente lite and future interest at 24% p.a. from the date of complaint to date of
payment.

5. The Appellant - Insurer, the opposite party-Respondent in the two complaints, resisted
the said claims. It contended that the claim was not maintainable as none of the
consignments were lost or damaged in transit. According to them, the investigation report
of the surveyor disclosed that one Kumar Chaudhary was the common President of M/s.
Overseas Container Lines Inc. (the shipping Agent) and M/s. Atlanta Rugs Inc. (the
buyer) and that the said Kumar Chaudhary had admitted to the surveyors that the
consignments had all been received by the buyer. If the buyer, having taken delivery of
all the consignments, failed to pay the value of the consignments, such nonpayment of
price by the buyer or non-realisation of the price by the seller, will not be a maritime peril
giving rise to a claim against the Insurer under a Marine Insurance Policy. When the
insured consignments had been delivered to the buyer, it cannot be said that there is a loss
of the consignments. The claims were repudiated on 4-3-1997. The reasons for
repudiation were furnished to the complainants by the Insurer as also by the surveyors.
There was thus no deficiency in service. It was also pointed out that when the goods are
entrusted to a sea going vessel, a master bill of lading is issued by the vessel/shipping line
showing the particulars of consignments and the names of the consignees and the said
Master Bill of Lading was not part of the documents of title. The failure on the part of the
complainants to take any action against the buyer and the manner in which the
transactions were conducted, gave room for doubt that there has been a collusion between
the complainants and the buyer to foist false claims against the insurer. The Insurer also
prayed that the detailed reasons for repudiation given in its letter dated 4-3-1997 be read
as part of its written statement. The relevant portions of the letter of repudiation are
extracted below :
"The buyer had taken delivery of all the consignments but has not paid your Company for
the same. Non-payment of the price by the buyer is not an insured peril under the
captioned policies and hence your claim falls beyond the scope and ambit of the policies
issued by our company. xxxx
Instead of taking up the matter with your
@page-SC2623
buyer for the payment of the price of the said consignments, you thought it fit to take no
action whatsoever against your buyer. xxxxxx
D) You have failed and neglected to act with reasonable dispatch as required by Clause
18 of the Institute cargo clauses (A) to which the above policies were made subject to.
Clause 18 reads as under :
'It is a condition of this Insurance that the Assured shall act with reasonable dispatch in
all circumstances within their control'.
The consignments were shipped from Bombay to Atlanta between the months of August
1994 to July 1995. However, your company has informed our company and our aforesaid
Surveyors of the alleged loss allegedly suffered by you only in the first week of February,
1996. As you have failed to act with reasonable dispatch as required by the said policies,
our company is not liable to pay any amount under the captioned policies.
E) As you are aware, it is a condition of the policy that a certificate of loss/damage should
be obtained from our Surveyors who are the company's agents at the port of discharge. It
is an admitted position that no such certificates has been obtained by your company. In
view of breach of the aforesaid condition our company is not liable to pay any amount to
your company under the captioned policies.
G) We have to state that under the aforesaid policies, we had agreed to insure that
consignment subject to Institute Cargo Clause (A). Clause I of the said Institute Cargo
Clause (A) clearly stipulates as under
'This insurance covers all risks of loss or damage to the subject matter.....'
As is pointed out by the surveyors, that the consignment under the captioned policies
have been received by the buyers. In view thereof, there is no loss and/or damage to the
subject matter i.e. the said consignments entitling your company to seek an indemnity
from our company.
H) Records in our possession show that you kept on sending consignments to your buyer
without caring to ascertain if the buyer was financially solvent and would make payment
in respect of the consignments shipped to you. In view thereof, we have to state that the
alleged loss is also attributable to the aforesaid wilful misconduct on the part of your
company under provisions of Institute Cargo Clause (A) -clause 4.1 and Section 55 of the
Marine Insurance Act, 1963.
I) You will appreciate that our company has agreed to indemnify you for the loss and/or
damage suffered to the consignments during its journey from your company's warehouse
to the buyers warehouse due to the insured perils. Financial insolvency and/or refusal of
the buyer to pay for the price of the said consignments is not one of the perils insured
under the captioned policies and hence your company cannot seek any indemnity under
the captioned policies."
6. Initially, there were four common respondents in both the complaints. The Divisional
Office, Regional Office and Head Office of New India Assurance company Ltd. were
respondents 1 to 3. Overseas Container Lines Inc., was the fourth Respondent. By order
dated 10-11-2000, the Commission directed the complainants to implead M/s. Atlanta
Rugs Inc. (Buyer), M/s. Niranjan Shipping Agency Pvt. Ltd. (Forwarding Agent of the
Complainants, as also the agent of Overseas Container Lines Inc.), Punjab National Bank
and Bank of Baroda (Bankers of complainant), Sun Trust Bank, earlier known as Trust
Company Bank (the foreign correspondent Bank of Punjab National Bank) as they were
proper and necessary parties. Accordingly, M/s. Atlanta Rugs Inc., Niranjan Shipping
Agency (P) Ltd., Punjab National Bank and Sun Trust Bank, were impleaded as
respondents 5 to 8 in OP No.45 of 1997. However, subsequently respondents 4, 5, and 8
were given up by the complainants as service could not be effected and their names were
deleted from the array of parties and consequently, when the matter was heard, the
respondents in OP No.45/1997 were the insurer, M/s. Niranjan Shipping Agency (P) Ltd.,
and the Punjab National Bank. Similarly, the said four persons as also Bank of Baroda
were impleaded as respondents 5 to 9 in OP No.49 of 1997. But later, the three
respondents who could not be served were given up and deleted and at the time of
hearing the respondents were the insurer, M/s. Niranjan Shipping Agency (P) Ltd.,
Punjab National Bank and Bank of Baroda.
7. The parties did not lead any oral evidence nor mark any document as exhibits, but
produced some documents. The complainants
@page-SC2624
and the insurer filed affidavits supporting the complaint and the written statement. On the
basis of the pleadings, affidavits, copies of documents produced and arguments, the
Commission decided the complaints by its common order dated 31-1-2003. The
Commission held that the complainants held valid marine insurance cargo policies which
gave insurance cover against all risks of loss. The bills of lading showed the consignee as
"Unto order" and they were endorsed in favour of Sun Trust Bank. The original
documents were not retired by the buyer and were returned by Sun Trust Bank to Punjab
National Bank. As delivery could be taken only after obtaining original bills of lading
which were with the Sun Trust Bank and as the original documents had been returned,
and as there was no evidence on record about the fate of the shipments, the shipments
should be covered under the term "loss" and insurer will have to indemnify the
complainants for such loss. Even if Atlanta Rugs Inc. had managed to receive the
consignments by fraudulent means without legitimate endorsement or transfer of the
original documents of title which were in the custody of the Sun Trust Bank, that
amounted to loss of goods. As the consignee was shown as "Unto order" in the bills of
lading, the complainants had lien and control over the consignments and consequently,
property in the goods did not pass to the buyer and in such circumstances, delivery to
Atlanta Rugs Inc. would still amount to loss of goods. As the goods were lost, the insurer
was liable to indemnify the complainants for such loss and failure to do so was a
deficiency in service. It therefore allowed both the complaints in part.
8. In OP No.45/1997, the Commission directed the insurer to indemnify the complainant -
M/s. Hira Lal Ramesh Chand by paying US $ 367311 (equivalent to Rs. 1,32,78,293 at an
exchange rate of Rs.36.15 per US Dollar) with interest at 12% p.a. from the date of
complaint till the date of payment, and pay the said amount to Punjab National Bank so
that the said Bank could adjust the said amount against the amount due from the
complainants. In OP No.49/1997 the Commission directed the insurer to indemnify the
complainant - M/ s. Ratan Chand Deep Chand to an extent of Rs.30,00,000/- with interest
at 12% p.a. from the date of complaint to the date of payment, and pay the said amount
equally to the complainants' bankers, Punjab National Bank and Bank of Baroda,
Mirzapur.
9. Feeling aggrieved the insurer has filed these two appeals. On the contentions raised,
the following questions arise for our consideration in these appeals :
(i) What is the scope of the policies of insurance issued by the Insurer to the insured?
(ii) Whether the complainants had proved that there has been loss of consignments falling
within the risks covered by the Marine insurance policies ?
(iii) Whether the Commission was justified in holding the insurer liable ? Re : Question
(i)
10. M/s. Hira Lal Ratan Chand had taken two marine policies (cargo) which are in the
nature of open covers, from the Insurer. They are open cover Policy Nos.2142 11000
8745 dated 29-4-1994 (which was in force from 29-4-1994 to 28-4-1995) and No.2142
11000 9032 dated 19-5-1995 (which was in force from 19-5-1995 to 18-5-1996) each
with an assured limit of Rs.50 lakhs. The assured limit of Open Cover No.2142 11000
8745 was extended by another Rs.50 lacs with effect from 22-2-1995 and by another Rs.7
lacs with effect from 27-4-1995 the total assured amount being Rs.1.07 crore. The Open
Cover constituted the contract under which the insurer agreed to issue separate Marine
Insurance Certificates as and when the insured made declarations of each shipment. The
terms of the open cover are extracted below (from Policy No.2142 110008745) :
Subject matter Insured :
On consignment said to contain of Indian handmade knotted woolen carpets/ Durries
packed in alkathene paper and double new hessiar cloths, dispatched vide
Road/Rail/Approved Vessel from warehouse Mirzapur to anywhere in world, from time
to time. Insured will declare each and every consignment within 15 days or one month (as
per insured) and dispatch with its value packing marks etc. Total sum insured under this
policy is Rs.50 lakhs.
Risk covered : All risks 0.1525%; war and SRCC Rs.0.0275%. On receipt of declarations,
certificate of insurance will be issued for each consignment. This policy will remain in
force for one year i.e. 29-4-1994 to 28-4-1995 unless previously exhausted by way of
declaration. If no declaration is received from the insured the minimum premium will be
retained by the company.
@page-SC2625
Special conditions and warranties
Covered against : All risks (ICC), War and SRCC from consignor's warehouse to
consignee's warehouse : Inland transit clause (A) Institute cargo clause (A), Institute War
Clause (Cargo), Institute strike clause (Cargo) Lorry warranty as attached hereto.
The attached clauses and endorsements form part of this policy.
Survey and claims settlement :
In the event of loss or damage which may result in a claim under this insurance
immediate notice be given to carrier at destination.
Certificate of loss/damage be obtained from our surveyor, who are the company's agents
at port of discharge in order that they may examine the goods and issue a survey report.
Where the company has no agent, the notice must be given to Lloyd's agents.
Payable as per declaration.
Similar were the terms of Open Cover No.2142 11000 9032 as also the Open Cover
Policies issued to M/s. Ratan Chand Deep Chand.
11. As and when the complainants entrusted the consignments to OCL, they made
declarations and the Insurer issued Marine Insurance Certificates in respect of each
consignment. Each of these Marine Insurance Certificates specified the particulars of the
consignment, and the value thereof and confirmed that the consignment was "covered
against : All risks (ICC), war and SRCC from consignor's warehouse to consignee's
warehouse." The certificates required that in the event of loss or damage which may
involve a claim under the certificate, notice of loss or damage should be given to its
surveyors M/s. Toplis and Harding Inc., Atlanta (USA).
12. Clause (1) of the Institute Cargo clause (A) forming part of the Insurance Policy
stipulated that the insurance covered all risks of loss of or damage to the subject matter
insured (except as provided in clauses 4, 5, and 7 therein). Clause (1) of the Inland
Transit clause (A) stipulated that the insurance covered all risks or loss or damage to the
subject matter insured (except as provided in clauses 2, 3 and 4 therein). Clause (1) of the
Institute War Clause (Cargo) stipulated that the insurance covered loss or damage to
subject matter insured caused by war, civil war etc. (except as provided in clauses 3 and 4
therein). Clause (1) of the Institute Strikes clause (cargo) stipulated that the insurance
covered loss or damage to the subject matter insured caused by strikes, lock outs etc.
(except as provided in clauses 3 and 4 therein). The duration of insurance cover is
specified in clause (8) of the Institute Cargo Clause (A), and clause (5) of the Inland
Transit Clause (A). They are extracted below :
Clause (8) of Institute Cargo Clause (A)
Duration.
8.1 This insurance attaches from the time the goods leave the warehouse or place of
storage at the place named herein for the commencement of the transit, continues during
the ordinary course of transit and terminates either
8.1.1 on delivery to the Consignee's or other final warehouse or place of storage at the
destination named herein,
8.1.2 on delivery to any other warehouse of place of storage, whether prior to or at the
destination named herein, which the Assured elect to use either
8.1.2.1 for storage other than in the ordinary course of transitor
8.1.2.2 for allocation or distribution,
or
8.1.3 on the expiry of 60 days after completion of discharge overside of the goods hereby
insured from the overseas vessel at the final port of discharge, whichever shall first occur.
Clause (5) of Inland Transit (Rail or Road) Clause (A)
Duration
"5. This insurance attaches from the time the goods leave the warehouse and/or the store
at the place named in the policy for the commencement of transit and continues during
the ordinary course of transit including customary transhipment, if any,
(i) until delivery to the final warehouse at the destination named in the policy or
(ii) in respect of transits by Rail only or Rail and Road until expiry of 7 days after arrival
of the railway wagon at the final destination railway station or
(iii) in respect of transits by Road only until expiry of 7 days after arrival of the vehicle at
the destination town named in the policy whichever shall first occur."
@page-SC2626
Similar duration clauses are found Institute War Clause (Cargo) and Institute Strike
Clause (Cargo).
13. The Marine Insurance Act, 1963 governs the law relating to marine insurance, Section
3 defines marine insurance as under :
"3, Marine insurance defined. - A contract of marine insurance is an agreement whereby
the insurer undertakes to indemnify the assured, in the manner and to the extent thereby
agreed, against marine losses, that is to say, the losses incidental to marine adventure."
13.1 The term 'marine adventure' is defined in section 2(d). The term 'maritime peril'
referred to in the definition of 'marine adventure' is defined in section 2(e). The said two
definitions are extracted below :
(d) "marine adventure" includes any adventure where -
(i) any insurable property is exposed to maritime perils;
(ii) the earnings or acquisition of any freight, passage money, commission, profit or other
pecuniary benefit, or the security for any advances, loans, or disbursements is endangered
by the exposure or insurable property to maritime perils;
(iii) any liability to a third party may be incurred by the owner of, or other persons
interested in or responsible for, insurable property by reason of maritime perils;
(e) "maritime perils" means the perils consequent on, or incidental to, the navigation of
the sea, that is to say, perils of the seas, fire, war perils, pirates, rovers, thieves, captures,
seizures, restraints and detainments of princes and people, jettisons, barratry and any
other perils which are either of the like kind or may be designated by the policy;"
13.2 Section 4 makes it clear that a contract of marine insurance may, by its express
terms, or by usage of trade, be extended so as to protect the assured against losses on
inland waters or on any land risk which may be incidental to any sea voyage. The
provisions of Marine Insurance Act are therefore subject to the terms of the policy of
insurance.
13.3 Section 57 provides that where the subject matter insured is destroyed, or so
damaged as to cease to be a thing of the kind insured, or where the assured is irretrievably
deprived thereof, there is an actual total loss.
14. Marine Insurance is a contract whereby the insurer undertakes to indemnify the
assured in the manner and to the extent thereby agreed, against marine losses, that is to
say losses incident to marine adventure. The instrument in which the contract of marine
insurance is generally embodied is called a policy. The thing or property insured in called
the subject matter of insurance and the assured's interest in that subject matter is called
his insurable interest. That which is insured against is the loss arising from maritime
perils and casualties, and these are called the perils insured against or the losses covered
by the policy. When the insurer's liability commences under the contract, the policy is
said to attach; or in other words, the risk is said to attach or to begin to run from that time.
A marine insurance cover applies to the shipment and if the shipment reaches the
destination, in a safe and sound condition, no claim can arise against the insurer. A
contract of marine Insurance may, however, by its express terms or by trade usage, be
extended so as to protect the assured against losses on Inland waters or against any land
risk which may be incidental to a sea voyage. (Vide sections 3 and 4 of Marine Insurance
Act, 1963 and Halsbury's Laws of England, 4th Edition, Vol. 25 paras 216 and 218).
15. The insurers offer different types of insurance cover. There are three standard types of
Institute Cargo Clauses (and Inland Transit Clauses) denoted as A, B, and C, providing
Insurance cover of varying extents. Institute Cargo Clause (C) provides the basic
minimum cover as enumerated. Institute Cargo Clause (B) offers a cover against wider
range of enumerated risks. The Insurer will also cover certain 'extraneous risks' like theft,
pilferage and/or non-delivery in addition to the risks covered by Institute Cargo Clause
(B) on payment of extra premium. While Institute Cargo Clause (C) and (B) specify and
enumerate the risks covered, Institute Cargo Clause (A) which offers the widest cover,
does not specify or enumerate the risks covered. Institute Cargo Clause (A) provides
insurance cover against all risks of loss and damage to the subject matter insured except
those excluded by clauses (4) to (7) thereof which are extracted below :
"4. In no case shall this insurance cover
@page-SC2627
4.1 loss, damage or expenses attributable to wilful misconduct of the Assured
4.2 ordinary leakage, ordinary loss in weight or volume, or ordinary wear and tear of the
subject matter insured
4.3 loss, damage or expense caused by insufficiency or unsuitability of packing or
preparation of the subject-matter insured (for the purpose of this Clause 4.3 "packing"
shall be deemed to include stowage in a container or lift-van but only when such stowage
is carried out prior to attachment of this insurance or by the Assured or their servant)
4.4 loss, damage or expense caused by inherent vice or nature of the subject matter
insured
4.5 loss, damage or expense proximately caused by delay, even though the delay be
caused by a insured against (except expenses payable under Clause 2 above)
4.6 loss, damage or expense arising from Insolvency or financial default of the owners,
managers, charterers of operators of the vessel
4.7 loss, damage or expense arising from use of any weapon of war employing atomic or
nuclear fission and/or fusion or other like reaction of radioactive force or matter
5.5.1 In no case shall this insurance cover loss, damage or expense arising from
unseaworthiness of vessel or craft.
Unfitness of vessel craft conveyance container or liftvan for the safe carriage of the
subject-matter insured.
Where the Assured or their servants are privy to such unseaworthiness or unfitness, at the
time the subject-matter insured is loaded therein,
5.2 The Underwriters waive any breach of the implied warranties of seaworthiness of the
ship and fitness of the ship to carry the subject-matter insured to destination, unless the
Assured or their servants are privy to such unseaworthiness or fitness.
6. In no case shall this insuration cover loss, damage or expense caused by
6.1 war, civil war, revolution, rebellion, insurrection, or civil strife arising therefrom, or
any hostile act by or against a belligerent power
6.2 capture, seizure arrest, restraint or detainment (piracy excepted), and the
consequences thereof or any attempt thereat
6.3 derelict mines, torpedoes, bombs or other derelict weapons of war
7. In no case shall this insurance cover loss, damage or expense
7.1 caused by strikers, locked out work-men, or person taking part in labour disturbances,
riots or civil commotions
7.2 resulting from strikes, lock-outs, labour disturbances, riots or civil commotions
7.3 caused by any terrorist or any person acting from a political motive."
The exclusions under clauses (6) and (7) get deleted when Institute War Clause (Cargo)
and Institute Strikes Clause (Cargo) are included. Exclusion under sub-clause (1) of
clause (5) virtually gets deleted by sub-clause (2) of clause (5). As a result, an insurance
policy with ICC(A), ITC(A) and SRCC, providing cover against "all risks" of loss or
damage to the insured consignment from consignor's warehouse to consignee's
warehouse, provides a very wide coverage. All risks except those mentioned in clause (4)
of Institute Cargo Clause (A) are covered. Theft, pilferage or non-delivery of the
consignment are therefore risks covered by an insurance policy with ICC (A) and
ITC(A). If the insured goods are not delivered by the shipping company or shipping
Agent, who issued the Bill of Lading to the assured, due to theft, pilferage, loss or non-
availability, occurring within the duration of insurance cover, the insurer will be liable
under the policy of insurance.
16. Having regard to clause (8) of Institute Cargo clause (A) relating to duration, the
insurance cover attaches from the time the goods leave the warehouse of the assured and
terminates either on delivery at the consignee's final warehouse or store at the destination
named in the policy (or on delivery to any other warehouse or place of storage, which the
assured elects to use) or on the expiry of 60 days after completion of discharge of the
goods from the vessel at the final port of discharge. The insurance cover commences
when the consignment leaves the warehouse or place of storage at the place named in the
policy for commencement of transit and the cover continues during transit and continues
until the consignment reaches the final destination specified in the policy as the
consignee's warehouse. Where the consignment is temporarily stored on arrival at an
interim destination (as for
@page-SC2628
example on the dock, or in the shipping lines warehouse or in the custom warehouse,
pending onward journey to the consignee's warehouse mentioned as final destination in
the policy), the cover would remain only for a period of 60 days from the discharge of the
consignments from the ship irrespective of whether consignment is put on onward
journey to the consignee's warehouse/storage place, or not.
17. In view of the insurance cover extending 'warehouse to warehouse' the consignments
are covered by insurance not only during the sea journey, but beyond as stated in the
policy. Therefore the contention of the insurer that the insurance cover is available only in
regard to maritime perils that is perils relating to or incidental to the navigation of the sea
may not be correct. Having regard to Section 4 of the Marine Insurance Act and the terms
of the policy undertaking insurance cover against wider risks, the policy of insurance
would cover the loss not only while goods are navigating the sea but also any loss or
damage during transit from the time it leaves the consignor's warehouse till it reaches the
consignee's warehouse. The cover against risks will however cease on the expiry of 60
days after discharge of the consignment from the vessel at the final port of discharge, if
the goods do not reach the consignee's warehouse or place of storage for any reason
within the said 60 days.
18

. The learned counsel for the Appellant relied on two decisions, the first being a decision
of this Court in Bihar Supply Syndicate v. Asiatic Navigation [1993 (2) SCC 639] and the
second being a decision of the Kerala High Court in Concord of India Insurance Co. v.
Ravi Thokassaria [ILR 1974 Kerala 649). In Bihar Supply Syndicate, this Court was
concerned with a marine voyage policy with Institute Cargo clause (FPA) covering
"warehouse to warehouse". This Court held that the expression 'warehouse to warehouse'
in the policy merely denotes the time during which the policy would remain in force and
cannot be interpreted as covering each and every risk. This Court held that under a typical
marine voyage policy with Institute Cargo Clauses (FPA), in the absence of loss due to
perils of the sea, the insurance company was not liable, and the onus was on the plaintiff
to prove as a fact that the cargo was lost due to the perils of the sea. In Concord of India
Insurance Co., the Kerala High Court considering marine risk policy, held that non-
delivery may be a good ground against the shipping company, but not against the
insurance company as non-delivery was not a maritime peril. 1993 AIR SCW 2116

19

. There is a difference between marine insurance policies which extend cover only against
marine losses or maritime perils (as enumerated) and marine 'extra' insurance policies
which extend cover against all risks from consignor's warehouse to consignee's
warehouse which include not only the sea journey but also the land journey at either end.
the decision in Concord of India Insurance Co. (supra) of the Kerala High Court and the
decision of this Court in Bihar Supply Syndicate (supra) relate to marine insurance
policies and not to mixed sea/land risks policies or to marine 'extra' insurance policies.
They are not of assistance while considering the scope of a policy covering all risks
including ICC(A), ITO(A), IWC(Cargo), ISC (Cargo). In fact in Peacock Plywood (P)
Ltd. v. Oriental Insurance Co. Ltd. [2006 (12) SCC 673], this Court held that where the
policy contained a wider term of risk coverage, the decision in Bihar Supply Syndicate
(supra) will not apply. In Peacock Plywood, the extended warranty clause in the
insurance policy specifically extended the coverage to include the risks of theft, pilferage
and non-delivery. In view of it, this Court held that a claim by way of constructive total
loss on account of a ship being stranded on sea on account of its unseaworthiness was
maintainable, although the goods themselves were not damaged. In that case when the
ship carrying the goods got stranded at a port due to its unseaworthiness, the assured took
steps to recover the value of the cargo with a view to minimize its total loss due to non
delivery, but found that the cost of recovering and getting the cargo back to the
destination port would be more than the value of the goods. Therefore the assured
effected sale of the insured goods at the port where ship was stranded. Insurer was found
liable to pay the insured value of the goods (less the amount actually recovered by such
sale). 2007 AIR SCW 902

Re : Questions (ii) and (iii) :


20. The complainants were manufacturers and exporters of carpets and durries, having
their principal place of business and manufacturing unit at Mirzapur. They allege that
M/s. Atlanta Rugs Inc. based at Atlanta
@page-SC2629
(USA) placed orders on them for supply of rugs/durries. The orders are not placed on
record. The complainants allege that they sent several shipments of Rugs/durries to
Atlanta (USA) for the buyer. The two complaints relate to 17 consignments sept by M/s.
Hira Lal Ramesh Chand and 38 consignments sent by M/s. Ratan Chand Deep Chand, for
which they did not receive payment. The common procedure adopted for supply is set out
below.
20.1 The complainants handed over the consignment with an invoice made out in the
name of Atlanta Rugs Inc., to their Forwarding Agent at Mumbai, namely, M/s. Niranjan
Shipping Agency Pvt. Ltd. The said Niranjan Shipping was also the shipping agent of
Overseas Container Lines Inc., which was a Non Vessel Owning Common Carrier
registered in USA. Niranjan Shipping as Forwarding Agent of the complainant entrusted
the goods to Overseas Container Line Inc. (also represented by Niranjan Shipping as
Shipping Agent) for transhipment from Mumbai to Atlanta. Overseas Container Line Inc.,
represented by Niranjan Shipping issued a Bill of lading in regard to each consignment.
The Bill of lading showed the complainant as the consignor and mentioned "unto order"
in the consignee column. Atlanta Rugs Inc. was shown as the buyer who should be
notified by the collecting Bank. Overseas Container Line Inc. (represented by Niranjan
Shipping) in turn entrusted the consignment to a shipping line which actually carried the
consignment and the Shipping Line would issue a Master Bill of Manifest also known as
Master Bill of Lading, showing the details of all consignments loaded in the container for
being carried from the load port to the foreign destination port.
20.2 In regard to each consignment, M/ s Niranjan Shipping as Custom House Agent
issued a 'Shipping Bill for Export of Goods under Claim for Duty Drawback' showing the
complainant as the Exporter and Atlanta Rugs Inc. as the Consignee, giving the
particulars of the consignment and its value as also the name of carrier (vessel). The said
Shipping Bill contained an endorsement certified by the Customs Officer giving the
particulars of the vessel and date of sailing.
20.3 The original documents of title comprising (i) the Bill of Lading issued by Overseas
Container, represented by Niranjan Shipping as Agent, (ii) the invoice issued by
complainants, (iii) the packing list and (iv) the shipping bill issued by Niranjan Shipping
for claiming duty drawback, were sent by Niranjan Shipping to the Complainant's
Bankers- Punjab National Bank. The copy of the Master Bill of Lading issued by the
shipping line/ship was not Included by Niranjan Shipping as part of the documents of
title.
20.4 The complainant had a foreign Out Bill Purchase Account with Punjab National
Bank. The Bank used to purchase/ discount the bills, and the Bill of Lading (Negotiable
copy) was endorsed by the complainant in favour of Punjab National Bank or its order.
The Punjab National Bank in turn endorsed the Negotiable copy of Bill of Lading in
favour of its foreign correspondent Bank namely Trust Company Bank, Atlanta
(subsequently known as Sun Trust Bank) and forwarded the documents of title to the said
Trust Company, Bank for collection. The foreign correspondent Bank would intimate the
buyer about the receipt of the documents. On the buyer making payment of the Invoice
amount, the foreign Bank would endorse the documents in favour of the buyer to enable
the buyer to take delivery of the consignment. If the buyer did not make payment and
retire documents within 90 days, the foreign correspondent Bank would return the
documents to the Punjab National Bank.
20.5 The complainant would make a declaration under the marine insurance open cover
issued by the appellant Insurance Company, in respect of each consignment as and when
entrusted for shipment, for which the appellant would issue a Marine Insurance
Certificate.
21. OP No.45/1997 filed by M/s. Hira Lal Ramesh Chand related to 17 consignments of
the total value of US$ 406,096. According to complainant, 12 consignments were
covered by Marine Insurance Certificates issued under marine open policy cover No.2142
11000 8745 and 5 consignments were covered by certificates issued under Marine open
Policy Cover No.2142 11000 9032. The seventeen invoices were made between 9.3.1995
and 13.6.1995 and corresponding Bills of Lading were dated between 15.3.1995 to
29.6.1995.
22. OP No.49/1997 filed by M/s. Ratan Chand Deep Chand related to 38 consignments of
the total value of US$ 8, 87, 973. According to complainants, 33 consignments were
covered by Marine Insurance Certificates issued under Marine Open Cover
@page-SC2630
Policy No.2142 11000 8749 (validity period 4.5.1994 to 3.5.1995), two consignments
were covered by certificates issued under Marine Open Policy Cover No.2142 11000
9038 (validity period 23.5.1995 to 22.5.1996) and three consignments were covered by
specific policies No.2142 11000 8868, 2142 11000 8869 and 2142 11000 8870. The
invoices in regard to these 38 consignments were made between 28.7.1994 to 25.6.1995
and corresponding Bills of Lading were dated between 23.8.1994 to 4.7.1995. According
to the complainants, the original insured value of the two marine open cover policies was
Rs.20 lakhs and Rs.10 lakhs, but on account of extra endorsements in respect of the open
covers the extent of cover in respect of the two open covers stood increased to Rs.3,
70,00,000 and Rs. 90,00,000, and the three specific policies were for Rs. 4,06,300, Rs.
2,91,000 and Rs. 2,81,700 in all Rs. 4,69,79,000/-. It is contended that the value of 38
consignments which was US$ 887973 (equivalent to Indian Rs. 3,21,00,224/-) was well
within the insurance cover amount and the commission erred in taking the total extent of
cover as only Rs.30 lakhs. The complainants (respondents 1 to 3 in CA 4307/ 2003) have
therefore filed cross-objections seeking increase in the amount awarded for Rs.30 lakhs
to Rs. 3,21,00,224/-. Be that as it may.
23. When the consignments were entrusted to Overseas Container, the complainants were
not aware of its address, as the Bill of Lading did not indicate any address. On their
request, Niranjan Shipping provided the address of Overseas Container on 8.8.1995. The
complainants claim to have written the following letter on 8.8.1995 to the Overseas
Container Lines Inc. (NVOCC)
"We have to transfer our goods to other buyers in USA. We have to convey the position
of our cargoes to them and ask our bankers to transfer the documents in their name. For
this reason we need the current position of our all the cargoes.
Please let us know the position of our cargoes carried by you which had been booked
with you for onward sea journey through forwarding agent M/s Niranjan Shipping
Agency Pvt. Ltd., Bombay.
Please treat the matter most urgent."
There was no reply in spite of a reminder dated 9.11.1995.
24. The complainants claim to have instructed their Bankers (Punjab National Bank) on
15.8.1995 to store the consignments sent to Atlanta in a Bonded Warehouse duly insured.
But apparently neither Punjab National Bank nor its Foreign Correspondent Bank took
any steps in the matter nor complied with the instructions.
25. On 25.1.1996, the foreign collecting Bank informed the Punjab National Bank and
complainants by telex message as follows :
"We have contacted the drawee on several occasions and on each occasion they promised
to make payment but as of date they have not make good on their promises. We have
exhausted all efforts to obtain payment and do not wish to continue our fruitless efforts in
pursuing these transactions. Please instruct us to turn the documents over to your agent
for them to pursue as we do not offer the services you are requesting us to do and our
policy is to return document after we exhausted efforts to collect payment and also not to
hold documents over 90 days.
If we do not receive your instructions to forward documents to your agent are returned
them yourself by latest February 10, 1996.
All documents will be returned to you and we will close our files on these transactions. It
is not our practice to investigate how thegoods was released nor to obtain any warehouse
merchandise. We can assure you that we did not issue any form of guarantee to the
drawee nor release any of documents to them. Please remit our charges of US $ 3930.
presenting our collection charges of US $ 85 each, US $ 5 each postage, cable charges
US $ 50 and courier US $ 195."
(Emphasis supplied)
26. Thereafter, the complainants telephonically informed the insurer on 2.2.1996 about
the non-realization of the Bills and claimed the value of the consignments. The insurer
instructed the complainants to get in touch with their Surveyor and Claim settlement
Agent - M/s Toplis and Harding Inc., Atlanta, to investigate into the matter and give their
report/certificate. Accordingly, the complainant sent a letter on 6.2.1996 to the said
Surveyor requesting for an inquiry and investigation in regard to the consignments. In
that letter the complainant
@page-SC2631
stated :
"In the above reference, we have to bring your kind attention that we failed to trace out
the consignment as per details enclosed. We made the correspondence through our
Bankers in India, that foreign Bank (Trust Company Bank now known as Sun Trust
Bank) to whom the documents were endorsed and sent for collection of payments. Now
foreign Bank has confirmed that the original documents are with them. We presumed that
the following consignments are either lost or shipping company has done some fraud
with our consignments with them."
(Emphasis supplied)
The complainant also sent reminder dated 19.2.1996 to the Surveyor. The Surveyor sent a
Fax reply dated 4.3.1996 to the complainant stating that the various shipments had been
delivered to and/or picked up by its customer - Atlanta Rugs Inc., with probable collusion
from Overseas Container Line Inc., as Mr. Kumar Chaudhary was the President of both
companies. In its letter dated 20.3.1996 to the Surveyor, the complainant acknowledged
the information that the goods had been delivered by Overseas Container Line Inc., to
someone other than Sun Trust Bank (who was holding the documents) and stated that
such act on the part of Overseas Container Line Inc. in releasing the goods has resulted in
loss to them. The complainants therefore sought the following information from the
surveyor :
(a) The basis for the surveyor's finding that the Trust Company Bank was not the
consignee.
(b) Name and description of the person who actually took the delivery of the
consignments.
(c) The particulars of documents on the basis of which releases were made by the custom
services.
(d) Name and description of the authority giving delivery/release.
The surveyor sent a telex dated 1.4.1996 to the complainant reiterating that its inquiry
revealed that the President of Overseas Container Lines Inc. and President of M/s. Atlanta
Rugs Inc. was one and the same person namely Kumar Chaudhary and answered the four
queries as follows :
(a) In the steamship line bill of lading (Master Bill of lading) Overseas Container Lines
Inc. was probably named/shown as the Consignee.
(b) M/s. Atlanta Rugs Inc. was apparently the person who actually took the delivery with
the help of Overseas Container Lines Inc. which was named as the Consignee on the
steamship line bill of lading.
(c) As the complainant had not furnished the details of the customhouse broker who
cleared the goods through US Customs, the answer could be provided by Kumar
Chaudhary.
(d) The release was probably by steamship line/custom warehouse. The surveyor also
suggested that the complainant should contact S.K. Verma of M/s. Niranjan Shipping as
he was the signatory of the bills of lading issued by Overseas Container Lines Inc. and as
his role in the matter was not clear. The surveyor also informed the complainant as the
Marine Insurance cover applied to shipments and as the shipments had reached the
destination and were delivered without loss or damage, as per the insurance contract, the
claim was not maintainable.
27. The complainants also contacted the Federal Meritime Commission, Washington,
seeking their help to ascertain whether their consignments were lying at port or had been
released and if lying at the port under whose custody they were lying or if they were
released, when and to whom they had been released and under what conditions. The
Federal Maritime Commission sent a reply dated 1.3.1996 to the complainant stating that
it had been unable to locate either Overseas Container Lines Inc. or Atlanta Rugs Inc and
advising the complainant to seek legal advice. The Commission also informed the
complainant that Overseas Container Lines Inc. as a Non-vessel operating common
carrier (NVOCC) had maintained a bond for US $ 50000 and if any judgment is rendered,
against it, the complainant could present a copy to Inter-Cargo Insurance Co., Illinois
who were the insurer of Overseas Container Lines Inc. for payment.
28. Thereafter the complainant wrote a letter dated 6.5.1996 to Overseas Container Line
Inc., stating as follows :
"It is really strange that all our communications that is letters dated 8.8.1995 and
9.11.1995 have been un-replied. We are not able to contact you on phone and fax. We
have to come that all the goods have been
@page-SC2632
released to you. How and why you have done causing this huge lose to us. We will file a
claim against you in proper forum at your entire risks."
(Emphasis supplied)
29. The insurer informed the complainant that they were awaiting the report from the
Surveyor M/s. Toplis and Harding Inc. and as soon as they receive the report, they will
attend to the claims. The surveyor sent two reports dated 27.6.1996 in respect of the
consignments of the two complaints. The relevant portions of the said report which are
identical are extracted below :
".... We noted that the original bills of lading on hand at Sun Trust Bank were issued by
Overseas Container Line Inc. which were the NVOCC involved in arranging of the
various shipments at origin. The authorized signatory of the Overseas Container Line Inc.
ocean bill of lading was Mr. S. K. Verma of Niranjan Shipping Agency Pvt. Ltd. We
advised all concerned parties to contact Mr. Verma for further information but we did not
receive confirmation if this was carried out."
Master Bills of lading should have been issued by the steamship line but none were in
Sun Trust Bank's possession. We noted that the Overseas Container Line Inc. bills of
lading were consigned to the order of Trust Company Bank with the notified party as
Atlanta Rugs Inc. At no time, have we been shown the master bills of lading issued by the
steamship line.
Mr. Chang at Sun Trust Bank stated that he met Mr. Kumar Chaudhary, President of
Atlanta Rugs Inc., on a number of occasions and he stated that to his knowledge Mr.
Chaudhary was going through financial difficulties in both his personal and business life.

On February 23, 1996, we attended at the premises of Atlanta Rugs, Inc., and initially met
with a person by the name of Rajesh Shorie, who said he was a new employee and could
not assist our investigations. However, during our visit, Mr. Chaudhary arrived and was
willing to speak with us but not to issue a signed statement.
Mr. Chaudhary understood the reason for the shipper's concern and stated unequivocally
that he had received all the shipments and that very few pieces remained although he
would not allow us to inspect his storage facility. He also did not give specifics as to how
the cargo was cleared into his possession. He referred us to an attorney in Atlanta,
Georgia, a Marshall Siegel. However, later Mr. Siegel informed us that he had only
recently been contacted by Mr. Chaudhary and had not been retained by him as counsel.
During this time, we were contacted by another shipper, insured through a different
under-writer, who also informed us that their shipments had not been paid for. We had
been given a customs house clearing agents name in Atlanta - C.H. Powell and Company
and we contacted their import manager, Mr. Wick. Mr. Wick informed us that he had
acted as Atlanta Rugs, Inc. agent in 1994 and still had $40000 of fees outstanding. We
requested particular details of the steamship lines involved and he volunteered that
Neptune Orient Line was one of the carriers. Mr. Wick stated that on the NOL bill of
lading the consignee and notify party were listed as Overseas Container Lines.
This confirmed our earlier suspicion that probably Overseas Container Lines were using
the steamship line master bill of lading to obtain and clear the shipments.
We later became aware that Mr. Kumar Chaudhary is the registered President of Overseas
Container Lines, Inc. We contacted Bureau of Enforcement at the Federal Maritime
Commission (FMC) and we discussed that Overseas Container Lines address is registered
at the same location as Atlanta Rugs, Inc. They also have a $50000 bond with intercargo
at Schaumberg, Illinoise since 1993. They could not act as a Custom House Broker to
clear the goods through US Customs as they would need to post a much larger bond with
FMC. Ratan Chand Deep Chand did not furnish us with details of their customs house
broker, although we did request this information on a number of occasions.
In May, 1996 we received a request from Punjab National Bank stating that they had
purchased the 'Bills' pertaining to the shipments and that the claim amount should be
remitted directly to themselves (see enclosed correspondence).
At this time, we believe that Atlanta Rugs, Inc, has been dissolved and that Mr.
Chaudhary is operating a different company. Based upon all the available information, at
this present' time, it would appear that all shipments were picked up by Atlanta Rugs,
@page-SC2633
Inc. who used fraudulent methods to obtain the shipments without payment through the
bank. We believe that the involvement of the steamship lines will be necessary to prevent
further acts and to obtain more information on the stolen shipments. We are issuing our
report to document all the facts as known at this present time and it is our intention to
issue an addendum should our further involvement be necessary. Please note that we do
not comprehend, and have received no explanation as to why the shipments were
continually being sent even when the shipper was not receiving any proceeds."
30. The copies of the reports were not furnished to complainants. As the Insurer did not
settle their claim, the complainants filed complaints (OP Nos.45 and 49/1997) before the
Commission on 25.2.1997 alleging that the conduct of the Insurer in riot settling the
claim amounted to a deficiency in service and consequently, claiming compensation from
the Insurer in regard to the value of the consignments and other losses (as detailed in para
3 above). The Insurer sent letters of repudiation dated 4.3.1997 to complainants giving
reasons for repudiation, relevant portions of which have been extracted above.
31. We have considered the detailed submissions made by learned counsel. We have also
considered the material that was placed before the Commission. We find on a careful
consideration that the Commission has not addressed itself to the relevant issues. It is no
doubt true that the complainants had booked the consignments showing the consignee as
'unto order' thereby indicating that the goods covered by the Bills of lading should be
delivered only to the holder/endorsee of the Bills of Lading. There is also no doubt that
the original documents were not cleared/retired by the buyer 'Atlanta Rugs Inc.' and that
the original documents were ultimately returned by the foreign correspondent Bank to
Punjab National Bank and they are lying with Punjab National Bank. There is also no
doubt that the consignments were insured against all risks of loss and damage.
32. The basic and fundamental averment and proof required in a case of this nature is that
the consignments had been lost or damaged in transit or that when the holder of the
documents applied for delivery, the goods were not delivered on account of the same
being irretrievably lost that is having been pilfered, stolen, lost or misdelivered. But there
is no such averment or evidence that the consignments were lost or damaged. Nor is there
any averment that the holder of the documents of title applied for delivery of the
consignments, and was denied or refused delivery on account of non-availability of the
consignments either due to pilferage, loss or misdelivery. When there is no allegation or
proof of Sun Trust Bank having applied for delivery and refusal of delivery, it is
inconceivable how the complainants can maintain a claim against the insurer.
33. Failure of the buyer to make payment and take delivery is not a 'loss' of consignment
which is covered by the Insurance Policy. The complainant should make out a case of
actual 'loss' of the consignment covered by the contract of insurance or non-delivery of
the consignment, that is refusal to meet a demand for delivery. The question is whether
the complainants have proved such loss or non-delivery. The case of the complainant as
put forth in the complaint and reiterated in the affidavit is that they have dispatched the
consignment to Atlanta, that the consignments were insured against all risks; that the
buyer did not retire the documents by making payment; that they do not know what
happens to the consignments and that therefore the Insurer ought to have paid them the
value of the consignment and failure to do so amounted to deficiency in service as
contemplated under the Consumer Protection Act, 1986. We are afraid that these are not
allegations or proof of loss or non-delivery sufficient to foist any liability on the insurer.
34. Another question that arises for consideration is who was responsible for
transshipment and delivery. Admittedly Overseas Containers was only a non-vessel
owning common carrier (NVOCC) and not the actual shipping line. Necessarily therefore
Overseas Container had to entrust the consignment to an actual shipping line for
transportation and the Master Bill of Lading given by the shipping line would show the
Overseas Containers as the consignee entitled to receive the delivery. The very fact that
the Master Bill of Lading is not given to the consignors/complainants and the fact that the
complainants did not demand for the same shows that they did not intend to apply for
delivery directly from the shipping line that carried the consignments but only intended
@page-SC2634
that delivery should be from Overseas Containers which had issued the Bill of lading. As
the contract for carriage was between complainants and Overseas Containers and as the
Bills of Lading issued by the Overseas Container showed that the consignments were
deliverable to the order of the complainants, necessarily Overseas Containers were
expected to take delivery of the consignment at Atlanta from the shipping line which
actually transported the consignments and then deliver it to the holder of the documents
of title who seeks delivery. If that is so, the Overseas Containers were entitled to take
delivery from the shipping line which transported the consignment and there was nothing
collusive, clandestine or Irregular about delivery of consignments being taken by
Overseas Containers from the shipping line, as it had entrusted the consignment to the
shipping line for transportation.
35. There is no averment or proof that the consignments did not reach the destination
namely the Port at Atlanta or that the goods were not taken delivery by the Overseas
Containers. There is no averment as to whether the holders of the documents namely the
Sun Trust Bank applied for delivery or attempted to take delivery of the consignment and
store them in a bonded warehouse and whether they were refused delivery within the
insurance cover period.
36. The insurance cover was in regard to all risks from consignor's warehouse to
consignee's warehouse. There is no dispute as to what is the consignor's warehouse as the
complainant is clearly shown as the consignor. The difficulty arises about the consignee's
warehouse. If the consignee is treated as Atlanta Rugs Inc., on delivery to Atlanta Rugs
Inc., the insurer is discharged of any liability for risks. If the consignee is shown as 'unto
order' whose warehouse is to be treated as consignee's warehouse? It does not obviously
refer to complainants' warehouse as admittedly the complainants were not having any
warehouse in Atlanta (USA), nor were they the holders of the documents. The 'consignee'
at the relevant time could only be the Sun Trust Bank in whose favour the documents had
been endorsed. But it is not the case of the complainant that Sun Trust Bank as the holder
of the documents of title sought delivery of the consignments from the custom's
warehouse or steam ship line or from Overseas Containers. Where there is no effort on
the part of the 'consignee' to take delivery from the shipping line/customs warehouse, the
duration of insurance cover cannot be infinite or indefinite. In such circumstances the risk
cover would terminate on the expiry of 60 days after completion of discharge overside of
the insured shipment from the Overseas vessel at the final port of discharge at Atlanta
having regard to clause 8 of Institute Cargo clause. It is not the case of the complainant
that within that time (of 60 days), delivery was sought by the holder of the documents
and that such delivery was refused. It is also not the case of the complainants that the
consignments were unauthorizedly delivered to Atlanta Rugs or that such delivery was
within 60 days of the landing of the consignments at Atlanta. In the absence of any
averment or evidence as to when the consignments were discharged from the ship at
Atlanta and an averment that within 60 days of the landing of the consignments at Atlanta
the holder sought delivery and delivery was refused, the question of the Insurer being
made liable for nondelivery does not arise. After 60 days of the landing of a consignment
even if the consignment is destroyed, lost or misdelivered, it is no concern of the Insurer.
37. It should be noted that a claimant insured in a marine insurance claim has to plead
and prove the following (i) his position - whether he is the assured or an assignee; (ii) his
insurable interest; (iii) the type or kind of the insurance policy and its relevant terms; (iv)
the duration of the cover; (v) the nature of risk/loss; and (vi) the risk loss is covered by
the policy. In the absence of necessary averments and evidence to establish a marine
insurance claim, a claim against the insurer is liable to be rejected. It is unfortunate that
the Commission disposed of the matter without examining the terms of the policy and
obligations undertaken by the insurer. In fact, having regard to the nature of issues
involved, this was more appropriately a matter for civil court. Be that as it may.
38. Another significant aspect is that the consignments were being continuously sent from
August, 1994 to July, 1995 by M/s. Ratan Chand Deep Chand and from March to June,
1995 by M/s. Hira Lal Ramesh Chand without making any effort to ascertain the fate of
the earlier consignments. Even when they learnt none of the documents
@page-SC2635
relating to the consignments had been retired by the buyer, they merely gave the oral
intimation to the insurer that too on 2.2.1996 about the non-retirement of the documents.
No claim was lodged with the insurer in writing. The allegation that orally a claim for
'loss of goods' was made on 2.2.1996 cannot be true as according to complainants
themselves till that date they had no knowledge that the consignments had been either
lost or wrongfully delivered. In fact the complainant have not produced even a single
document making a claim on the insurer on the ground that the goods had been lost or not
delivered.
39. The complaints, affidavits and the documents are also significantly silent about the
following aspects :
(i) The particulars of the ships by which the consignments were transported and the dates
on which the ships sailed, and whether any loss or damage was caused to any of the
consignments while in transit. [Though the Xerox copies of the 'Shipping Bills for export
of goods under claim for duty drawback' give the name of ship and date of sailing, they
are not authenticated or supported by the evidence of the persons who prepared those
shipping Bills or the Customs officers who certified them].
(ii) If the consignments were not damaged or lost in transit, when did the ships discharge
the consignments in Atlanta Port; on such discharge whether the consignments were with
the shipping line or were shifted to a customs warehouse; or whether they were
immediately delivered to Overseas Containers and if so when.
(iii) In their rejoinder, the complainants stated that by letter dated 15.8.1995 they asked
their Bankers to take delivery of the consignments and store them in a bonded warehouse
duly insured. This assumes that the consignments had arrived safely and soundly at
Atlanta Port. The Sun Trust Bank was holding the original documents at that time. There
is no averment as to whether they complied with the said instructions and took delivery
of the consignment and place them in a warehouse in its control. If they did not do so, the
reasons ought to have been given.
(iv) Nothing is said as to whether the Bank holding the documents of title apply for
delivery by producing them. The complainants did not obviously apply. If no one ever
applied or sought delivery and if there was no refusal, there is no question of loss or the
question of liability on the part of the Insurer.
(v) There is no averment as to whether the complainants or the holders of documents of
title ever complained or reported loss of the consignment, or about any wrong delivery or
misdelivery to any one, in writing.
(vi) The consignments were entrusted to Niranjan Shipping, who was acting as the
forwarding agent of complainants as also the agent of a non-vessel owning shipping
agent (NVOCC) namely Overseas container. The Bills of Lading were issued by a
NVOCC. In those circumstances the reason as to why the complainants did not demand
for the supply of the copies of the Master Bills of Lading issued by the shipping
line/vessel which actually transported the consignments, is not disclosed. The failure of
complainants to pursue the matter with Niranjan Shipping to whom they had delivered
the consignments, is also strange.
(vii) None of the Bills of lading issued by the Overseas Container Lines Inc., contains
their address. The complainants did not care to seek their address, even though it would
not have been possible to take delivery without knowing the address. The complainants
did not also care to enquire with Niranjan Shipping, about the absence of address in the
Bills of Lading. It was only long after all the consignments were dispatched, when the
documents were not retired by the buyer, the complainants woke up and requested
Niranjan Shipping to furnish the address of the Overseas Container and Niranjan
Shipping provided the address by communication dated 8.8.1995. This strange behaviour
is not explained. This indicates either willful negligence or willful misconduct by way of
collusion.
(viii) In regard to all the consignments, Niranjan Shipping had issued 'shipping bills for
export of goods under claim for duty drawback' showing the complainant as the exporter
and the Atlanta Rugs Inc. as the consignee. If the Bills of lading showed the consignee as
"unto order" (that is the endorsee of the documents of title) and if the property in the
goods had not passed by the buyer (Atlanta Rugs Inc.), then the reason why the
complainant did not protest against the issue of such shipping bill showing Atlanta Rugs
Inc. as the consignee, is not stated.
@page-SC2636
40. The complainants have failed to plead and make out a case of loss, in respect of each
and every consignment, either during transit or within 60 days of the consignments being
discharged from the ship at Atlanta Port. They merely proceed the assumption that the
Insurer is liable when the documents are not retired by the buyer, which, to say the least,
is untenable. As there is no averment or proof that the consignor or the foreign
correspondent Bank holding the documents of title or any person authorized by the said
Bank applied for delivery within 60 days of the goods being discharged, and as there is
no averment or proof that the consignments were lost or wrongly delivered within the
said period of 60 days, the liability and responsibility of the insurer under the policy of
insurance came to an end with reference to each of those consignments. Consequently the
claim of the complainants against the insurer is liable to be rejected.
41. The Commission has referred to the delay of nine months on the part of the Insurer in
repudiating the claim, after receiving the surveyor's Report, and the failure to furnish a
copy of the reports to the complainants as deficiency in service. But what is overlooked is
that the complainants did not lodge any claim in writing. At all events, they did not
produce any document showing the lodging of claim. It was on a mere oral intimation on
2.2.1996, the investigation by surveyor was set in motion. Further the contents of the
report had already been notified to the complainants by the surveyor in the telexes dated
4.3.1996 and 1.4.1996. Therefore, the finding of deficiency in service was not warranted.
42. In this view of the matter, it is unnecessary to consider the counter claim of the
respondents 1 to 3 in CA No.4307 of 2003. Nor is it necessary to consider whether such
counter claim is maintainable in an appeal under section 23 of the Consumer Protection
Act, 1996.
43. In view of the foregoing, the appeals are allowed and the order of the National
Consumer Redressal Commission is set aside and OP No.45/1997 and 49/1997 before the
Commission stand dismissed. Parties to bear their respective costs.
Appeals allowed.
AIR 2008 SUPREME COURT 2636 "Sudhir S. Mehta v. Custodian"
(From : Bombay)*
Coram : 2 S. B. SINHA AND V. S. SIRPURKAR, JJ.
Civil Appeal Nos.5690-5697 of 2007, D/- 16 -5 -2008.
Sudhir S. Mehta and Ors. v. Custodian and Anr.
(A) Special Court (Trial of offences Relating to Transactions in Securities) Act (27 of
1992), S.3 - SPECIAL COURT - SECURITY TRANSACTION - APPEAL - SUPREME
COURT - Sale of shares of notified persons by custodian - Order by Special Court -
Completely confirmed by Supreme Court - Advertisement for sale of shares by custodian
- Direction by Special Court to custodian to consult and obtain opinion of Committee of
experts on various aspects - Appeal - Plea by notified appellants, shareholders that shares
cannot be sold, raised for first time - Not tenable. (Para 34)
(B) Special Court (Trial of Offences Relating to Transactions in Securities) Act (27 of
1992), S.3 - SPECIAL COURT - SECURITY TRANSACTION - SALE - Sale of
property, shares - Application for denotification by appellants, notified persons found to
be already withdrawn - Thus, no application for denotification was pending - Therefore
plea by them that since there are chances of their denotification, the shares belonging to
them should not be sold - Not tenable. (Paras 35, 36)
(C) Special Court (Trial of Offences Relating to Transactions in Securities) Act (27 of
1992), S.3 - SPECIAL COURT - SECURITY TRANSACTION - SUPREME COURT -
APPEAL - Sale of shares of notified persons by custodian - Order by Special Court as to
- Confirmed by Supreme Court - Same is binding - Conducting sale by custodian -
Appeal against - Raising issues that individual liability of such a notified person vis-a-vis
group liability should be arrived at first - Not permissible - Moreso when no prejudice
shown to have been caused to said notified persons - Furthermore no loss shown to have
been caused in earlier sale of part of shares. (Paras 37, 38, 40, 46, 47)
(D) Special Court (Trial of Offences Relating to Transactions in Securities) Act (27 of
1992), S.3 - SPECIAL COURT - SECURITY TRANSACTION - SALE - Sale of shares
belonging to notified persons - Decision
@page-SC2637
of custodian stand frustrated because of lapse of time - Custodian directed to repeat
whole procedure. (Para 48)
Cases Referred : Chronological Paras
2006 AIR SCW 243 : AIR 2006 SC 795 : 2006 CLC 178 (Ref) 7, 9, 30, 31, 35, 36,
39, 42, 44, 46
2004 AIR SCW 6868 : AIR 2005 SC 1209 : 2005 CLC 117 (Ref) 26
1998 AIR SCW 2193 : AIR 1998 SC 2291 (Ref) 13, 18, 26
(1993) 1 SCC 78 14
Mahesh Jethmalani, Sr. Advocate, Ms. Kamini Jaiswal, for Appellants; Arvind Kumar
Tewari and Abhishek Tewari, for Respondents.
* Misc. Appln. Nos. 272 to 279 of 2007, D/- 2-11-2007 (Bom)
Judgement
1. V. S. SIRPURKAR, J. :- These appeals are by way of a challenge to the order dated
02.11.2007 passed by the Special Court of Bombay constituted under the Special Court
(Trial of Offences Relating to Transactions in Securities) Act (hereinafter referred to as
'the Act'). By the impugned common order, Miscellaneous applications filed by Mr.
Sudhir S. Mehta, Ms. Deepika A. Mehta, Mr. Ashwin B. Mehta, Gromore Research
Assets Management Ltd., Ms. Jyoti S. Mehta and Mr. Hitesh S. Mehta, as also Ms.
Pratima Mehta were disposed of by the learned single Judge. In that order, the Special
Court directed the Custodian under the Act to refer two questions for the opinion of
Disposal Committee. They were :
(i) What would be the appropriate time to be given to the bidders for submitting bids after
publication of the notice inviting bids?
(ii) Whether it will be admissible to break up the shares into appropriate groups and to
give options to the bidders to bid either for whole lot or for a limited number of groups?
The Custodian was further directed to take legal and professional opinion in relation to
the liability to pay Capital Gains Tax. The learned Judge further directed that if the
notified parties wanted to make any submission on the above questions, the same may be
submitted to the Custodian within the period of 4 days and such submissions would be
transmitted by the Custodian to the Disposal Committee for its consideration, and after
the opinion of the Disposal Committee is given, the Custodian shall take steps in
accordance with the said opinion, as also in accordance with the legal and professional
advice. The learned Judge further directed that the Custodian would be free to approach
the Court and seek appropriate orders. With these directions, the learned Judge disposed
of the report of the Custodian, as also the applications made by the parties. All the
applicants are the family members of late Sh. Harshad S. Mehta, while respondent No. 1
herein, is the Custodian appointed under Section 3(1) of the Act. The respondent No. 2 is
the Disposal Committee in all the appeals, which are filed under Section 10 of the Act.
Each of the individual appellant is in close relation with late Sh. Harshad S. Mehta and
have filed 6 appeals, while Gromore Research Assets Management Ltd. have filed 2
appeals. The questions are common and the learned counsel also apprised us treating all
the questions involved, to be common. Hence, the appeals are being disposed of by this
common Judgment.
2. All the common questions have arisen on account of the advertisements issued by the
Custodian dated 28.10.2007 for the sale of the shares of Reliance Industries Ltd. As many
as 24, 26, 376/- shares belonging to the individual appellants and 1, 75, 316 shares
belonging to Fairgrowth Financial Services Ltd. and 5,300/- shares belonging to Mr. N.K.
Aggarwal were covered by these advertisements. It was stated in the advertisements that
these shares would be sold in bulk categories and the offers were to be submitted on or
before 1.11.2007. Accordingly, the offers were received by the Custodian and the same
were considered by the Disposal Committee, and the Custodian submitted his report for
sale of the shares in favour of the Life Insurance Corporation of India (LIC of India), as
the LIC of India had offered the highest price at the rate of Rs.2,701/- per share.
3. At this stage, the objections were raised by way of the Miscellaneous Applications
before the Special Court at the instance of the appellants herein. The common grievances
made in these objections cum applications were:
(i) that the time given in the advertisements for making offers was too short for the
intended investors considering the huge number of shares and the prevailing market price
of the shares.
(ii) that the shares could fetch more price if the Custodian had divided the shares into
appropriate groups and given the option to
@page-SC2638
the offerers to make offer for the whole lot or one or more groups.
(iii) that in fact, this was not an appropriate time to sell the shares considering the
prevailing market conditions.
(iv) that if the shares were to be sold privately as wag being done, the Capital gains tax
would be required to be paid and, therefore, the shares should have been sold at the stock
exchange.
4. The Special Court dealt with all the 4 objections. It firstly noted its order dated
17.08.2000, whereby, a scheme was framed for the sale of the attached shares and a
Committee of experts known as Disposal Committee was constituted and the sale of
shares was conducted under the supervision of that Committee. The learned Judge also
took the stock of the arguments before him that before issuing advertisements inviting the
offers, the Custodian had not consulted the said Disposal Committee regarding the
appropriate time to be given for submitting the offers. The learned Judge also took the
notice of the earlier order dated 23.8.2001 passed by this Court regarding sale of shares,
wherein, this Court had permitted even the private parties to submit their offers for the
purchase of shares and, therefore, the learned Judge observed that the Custodian should
have sought the opinion of the Disposal Committee.
5. Further, the learned Judge also observed that in the order passed by this Court, it was
expressed that there was no provision for breaking up the bulk shares into groups and for
selling each group separately, so as to invite the best price. The learned Judge, therefore,
held that the opinion of the Disposal Committee was bound to be sought on the question
as to "whether if the option is given to the buyers to bid for one or more groups instead of
putting the bids for entire bulk, it would fetch more price?". The learned Judge, further
observed that it was not for the notified parties to decide as to what would be the
appropriate time, nor could the Court go by the opinion of the notified parties regarding
the appropriate time for sale of the shares. The learned Judge, therefore, came to the
conclusion that since, the Custodian had taken the opinion of the Disposal Committee on
this aspect and since the Disposal Committee had opined that it was the opportune time
for selling the shares, the objection raised regarding the opportune moment could not be
accepted and that the opinion of the Disposal Committee on that behalf would be final. In
short, the objection regarding the time of the sale was overruled, Lastly, as regards the tax
liability, since the Custodian represented before the Court that the legal and professional
advice regarding the tax liability would have to be obtained, the learned Judge permitted
the Custodian to obtain such legal and professional advice. In this view, the learned Judge
wanted to know from the LIC of India, whether they were willing to keep their offer open
till the opinion of the Disposal Committee was obtained. It was noted that the
representative of the LIC of India was not willing to keep their offer open. The learned
Judge, therefore, decided not to accept the report of the Disposal Committee
recommending the sale in favour of the LIC of India and issued the directions which we
have already mentioned above.
6. It is, therefore, obvious that, firstly, there is no immediate possibility of the sale of the
shares as was intended by the Custodian unless the directions given by the learned Judge
are complied with. So also, since as many as 6 months have elapsed, the whole situation
regarding the market has drastically changed and, therefore, the Disposal Committee
would again be required to decide afresh as to whether the Reliance Shares should be
sold and/or when they should be sold. It is also an admitted position that the legal
opinions regarding the tax liability has also not been obtained by the Custodian and,
therefore, the matters have not been crystallized and are still in a fluid state.
7

. However, by these appeals, the basic objection is being raised to the effect that the
Custodian or the Special Court have not examined nor given a finding with respect to the
involvement of the appellants with late Sh. Harshad S. Mehta, nor has the Custodian
examined the claim inter se between the entities within the so called group. In short, the
appellants have challenged the very concept of the sale of shares. The further contention
raised now is that the assets of the appellants were appreciating, therefore, it would not be
advisable to effect the sale of the assets. The appellants suggested that in the past also, the
Custodian had sold the shares, the value of which were appreciating and, therefore, loss
of Rs.6,500 Crores was caused to the appellants. The 2006 AIR SCW 243

@page-SC2639
further objection raised in the appeal is that the Special Court is acting contrary to the
directions issued by this Court, whereby, it was mandated that Special Court should
arrive at a firm conclusion as regards the involvement of the individuals with late Sh.
Harshad S. Mehta. The appellants dubbed the impugned order as a step towards the sale
of assets of the appellants without any liabilities having been established against the
appellants. It is also said that if the assets of the appellants are mere than the liabilities,
there would absolutely be no reason or warrant for the sale of appreciating assets of the
appellants. The appellants have also raised the question mark against the so called illegal
and exaggerated demands of revenue and according to them, there are adequate liquid
balances, which can meet any eventuality of further liability. They further point out that
though 21 months have elapsed after the order of this court in Ashwin S. Mehta and Anr.
Vs. Custodian and Ors. reported in 2006 2 SCC page 385, the Custodian had neither
preferred their claim nor had examined the inter se liability between the so-called group
of individuals (meaning his relatives who are the appellants). The appellants objected to
the entire group being considered as one legal entity. Lastly, the aforementioned
judgment of this Court in Ashwin Mehta's Case and some observations therein are
heavily relied upon.
8. As against this, the Custodian has justified the sale of the assets as has been decided by
the Special Court on various grounds. Our attention has been invited by the Custodian to
the various provisions of the Act, as also the earlier orders passed by the Special Court
and this Court including the last judgment in 2006 (cited supra). It is firmly suggested
that there was no question of doing anything contrary to the judgments of this Court nor
could it ever be said that the Custodian in any manner failed to do anything that was
expected of him. Further, the Custodian had asserted that the appellants are trying to
wake up the dead issues and non-issues without there being any occasion for the same.
9. Shri Jethmalani appearing for the appellants mainly stressed on the judgment of this
court in Ashwin's case. Heavy reliance was placed on paragraphs 36, 41, 42, 46, 47, 50,
51, 52 and 77 and it was expressed that all these directions were never complied with by
the Special Court nor were the individual liabilities were ever considered as was directed
by this Court in that judgment.
10. Some basic facts were brought before us.
BASIC FACTS
11. After the huge seam broke out in respect of the shares and securities, which was
almost of oceanic proportion, the Central Government came out with the aforementioned
Act.
12. Section 3 of the Act provides for the appointment and functions of the Custodian. The
Custodian, on being satisfied that any person is involved in any offence relating to the
transactions of securities between the period 1.4.1991 and 6.6.1992, can notify the name
of such person in the Official Gazette. Section 3(3) provides that any property movable or
immovable or both belonging to any person notified under Section 3(2) stands attached
simultaneously with the issue of the notification, and such attached properties would be
dealt with by the Custodian in such a manner as the Special Court may direct. Under
Section 4(1), the Custodian is empowered to cancel any contract or agreement entered
into between two aforementioned dates by the notified person. Section 4(2) provides for
hearing as regards the correctness or otherwise of the notification under Section 3
notifying any person, on an application being made within 30 days of the issuance of
notification. The Special Court is established under Section 5 and has exclusive
jurisdiction conferred upon it under Section 7 for any prosecution pending in any court
and such prosecution stands transferred to the Special Court under that provision. The
Special Court is also conferred with the jurisdiction in respect to the civil matters, more
particularly, specified in Section 9A. Section 11 is the crux of this Act, which reads as
under :-
"11. Discharge of liabilities :- (1) Notwithstanding anything contained in the Code and
any other law for the time being in force, the Special Court may make such order as it
may deem fit directing the custodian for the disposal of the property under attachment.
(2) The following liabilities shall be paid or discharged in full, as far as may be, in the
order as under :
(a) all revenues, taxes, cesses and rates due from the persons notified by the Custodian
@page-SC2640
under sub-section (2) of Section 3 to the Central Government or any State Government or
any local authority;
(b) all amounts due from the person so notified by the Custodian to any bank or financial
institution or mutual fund;
(c) any other liability as may be specified by the Special Court from time to time."
13

. Accordingly, on 8.6.1992, a notification was issued notifying the appellants and some
other entities and obviously as per the mandatory need of the provision on and from that
date any property movable or immovable or both, belonging to the notified
persons/entities simultaneously and automatically stood attached. This property, which
stood attached belonging to the notified persons and entities, included vast number of
shares held by late Sh. Harshad S. Mehta, as also the other close relatives of late Sh.
Harshad S. Mehta like the appellants, so also the other entities including the one which is
before us today, i.e. the Fairgrowth Financial Services Ltd. The shares belonging to late
Sh. Harshad Mehta, as also the appellants herein and the entities were of various
companies. On 20.02.1995, in Misc. Application No. 107 of 1993 and other similar Misc.
Applications, the Special Court formulated certain questions. On the interpretation of
Section 11 of the Act, more particularly, in respect of the priorities created under that
Section, the learned Judge presiding the Special Court directed the Custodian to move to
the Supreme Court and hence, the appeal being Civil Appeal No. 5525 of 1995 came to
be filed by the Custodian before this Court. In the same appeal all the notified persons
were joined as the parties and they also filed their say. Not only that, but the notified
parties also filed Civil Applications before this Court which were clubbed together and all
these Civil Applications were disposed of by an order dated 11.03.1996 passed by this
Court. By the said order, this Court directed a scheme to be drafted in respect of the sale
of shares from time to time. The Custodian was directed to forward the scheme to the
Union of India for approval and on such approval being obtained, the said scheme was
directed to be placed before this Court again. In compliance of the order dated
11.03.1996, the scheme for the sale of attached shares was proposed. Civil Appeal No.
5225 of 1995 was heard along with the other allied appeals like Civil Appeal No. 5326 of
1995, 5147, 5325, 6080 of 1995, 12574 of 1996 and T.C. (C) No. 5 of 1998 (the
transferred writ petition) were disposed of by this Court by a judgment dated 13.05.1998
in Harshad Shantilal Mehta Vs. Custodian and Others in 1998 (5) SCC 1. The transfer
case was in respect of constitutional validity of Section 11 of the Act by a writ petition
filed before the Delhi High Court which was got transferred by this Court itself. In its
judgment disposing of all these appeals, this Court considered the 3 questions formulated
by the Special Court :- 1998 AIR SCW 2193

"1. Whether the priority created by Section 11 of the Special Court (Trial of Offences
Relating to Transactions in Securities) Act, 1992 is only in respect of amounts due prior
to the date of notification and/or whether the priority would also apply to amounts due
after the date of the notification.
2. Whether the phrase 'taxes' as used in Section 11 of the Special Court (Trial of Offences
Relating to Transactions in Securities) Act, 1992 can only mean amounts due as and by
way of taxes or whether it would also include penalties and interest, if any.

3. Whether penalty and/or interest can be levied on or charged to notified parties after the
date of notification."
14. This Court then took notice of the whole Act including the Statement of Objects and
Reasons with special reference to Section 11. It was observed in paragraphs 12 and 13 :-
"12. Before the Special Court makes any order under Section 11(1), the Special Court
must be satisfied that the property which is attached and is being disposed of, is the
property belonging to the notified person. If any person other than the notified person has
any share, or any right, title or interest in the attached property on the date of notification
under Section 3, that right of a third party cannot be extinguished. There is no provision
in the Special Court Act which extinguishes the right, title and interest of a third party in
any property which is attached as a consequence of a notification under Section 3. The
only right which the Custodian has, in respect of the rights of third parties in such
properties, is conferred by Section 4 under which, if the Custodian is satisfied that any
contract or agreement which was entered into by the notified party within the "statutory
period" in relation to
@page-SC2641
an attached property, is fraudulent or entered into for the purpose of defeating the
provisions of the Special Court Act, he can cancel such contract or agreement. There is no
other provision under the Special Court Act which affects the existing rights of a third
party on the date of attachment, in the property attached. The attached property also does
not vest in the Custodian. In this regard, the position of a Custodian is different from that
of an official liquidator of a company in winding up. Had the Act provided for the
extinguishment of any subsisting rights of other persons in the attached property, the Act
could well have been considered as arbitrary or unconstitutional (vide C.B. Gautam v.
Union of India).
13. The directions, therefore, for disposal under Section 11 (1) can be given only after the
Special Court has satisfied itself that the property under attachment is the property which
belongs to the notified person. The directions for disposal can only be in respect of the
right, title and interest of the notified person in the attached property. If, therefore, any
application is filed before the Special Court by a third party claiming the property so
attached and/or for releasing the right, title and interest of a third party in the property
from attachment, the Special Court will have to decide the application before proceeding
under Section 11."
15. In paragraph 15, this Court took the note of the words in Section 11 (2) "in order as
under" and held that before the amounts can be paid to banks or financial institutions
under Section 11(2)(b), the liabilities under Section 11(2)(a) are required to be
discharged. This Court reframed the questions framed by the Special Court in paragraph
16, which are :-
"1. What is meant by revenues, taxes, cesses and rates due? Does the word "due" refer
merely to the liability to pay such taxes etc., or does it refer to a liability which has
crystallized into a legally ascertained sum immediately payable?
2. Do the taxes [in clause (a) of Section 11(2)] refer only to taxes relating to a specific
period or to all taxes due from the notified person?
3. At what point of time should the taxes have become due?
4. Does the Special Court have any discretion relating to the extent of payments to be
made under Section 11(2)(a) from out of the attached funds/property?
5. Whether taxes include penalty or interest?
6. Whether the Special Court has the power to absolve a notified person from payment of
penalty or interest for a period subsequent to the date of his notification under Section 3.
In the alternative, is a notified person liable to payment of penalty or interest arising from
his inability to pay taxes after his notification?"
16. In paragraph 35, this Court observed that the Special Court can decide how much of
the tax liability will be discharged out of the funds in the hands of the Custodian. It
further observed, that the payment in full may or may not be made by the Special Court
depending upon various circumstances. For this purpose, it can examine whether there is
any fraud, collusion or miscarriage of justice in assessment proceedings. It was observed
that where the assessment is based on proper material and pertains to the "statutory
period", the Special Court may not reduce the tax claimed and pay it out in full. In
paragraph 40, the Court made reference to the order dated 11-3-1996, whereby, the Court
had directed the Custodian to draft a scheme in respect of the shares held by the
Custodian whereby such shares can be sold from time to time. The Court further noted
that the Custodian was directed to forward the scheme for the approval of the Union of
India and after the approval, the final scheme incorporating the modifications by the
Union of India was filed in this Court. The Court specifically directed that the Special
Court shall consider the scheme and the appropriate orders may be passed by the Special
Court in respect of the scheme so submitted. The Court also upheld the constitutional
validity of Section 11 read with Section 3(3) of the Act, and that is how, the earlier
appeals were disposed of.
17. In pursuance of the order, to consider the scheme, the Special Court came out with an
order dated 17-8-2000.
The learned Judge then considered the whole scheme in extenso. Arguments were raised
challenging the sale of the shares at that point of time. It was noted that the main
objection of the notified parties was that the time for distribution of assets had not yet
arrived and, therefore, the scheme of the sale of shares should neither be framed nor
implemented. The learned Judge took the note of this scheme and the suggestions
@page-SC2642
made therein on setting up a Disposal Committee. It was also noted that under the
scheme, modality of sale of shares was provided for. Three questions were framed by the
learned Judge, they being :-
A. Whether the time to frame the Scheme for sale of attached shares belonging to the
notified parties has arrived?
B. Challenge to the validity of scheme?
C. Implementation of the scheme?
18

. It was specifically urged before the learned Judge that until the date of distribution is
imminent, there would be no question of selling the attached shares, which though
attached, remain the property of the notified parties. It was also contended that the sale
and distribution were not separate events and, therefore, the sale of assets could not take
place on a particular day and the distribution after 5 years. And, therefore, the sale of the
attached assets being solely for the purpose of distribution can only take place at the time
of distribution and after the question of nexus of the attached assets with the illegal
security transactions is considered on merits. Even the judgment reported in Harshad
Shantilal Mehta vs. Custodian and Others, reported in (1998) 5 SCC 1 (cited supra) was
relied upon, more particularly, paragraph 27 thereof, and lastly, it was contended that the
order dated 20-2-1995 related only to Harshad Mehta Group and Fairgrowth Financial
Service Ltd. It was reiterated that in its order dated 11-3-1996 passed by this Court,
which was the interim order in Civil Appeal No. 5326 of 1995, this Court had also stated
that the time for distribution of assets in possession of the Custodian was drawing very
near, and this was with reference to only 2 notified parties, viz., Harshad Mehta Group
and Fairgrowth Financial Services Ltd. It was further contended that the time for
distribution of assets of Dhanraj Mills had still not arrived. A reference was given to the
order dated 20-2-1995 of the Special Court by Hon. Variava, J. (as he then was), which
according to the learned counsel, ultimately, shows that even according to the learned
Judge, the distribution of assets was required to be made only in respect of Harshad
Mehta Group and Fairgrowth Financial Services Ltd. and, therefore, the scheme should
not be applied to Dhanraj Mills. The learned Judge then observed :- 1998 AIR
SCW 2193
"The above arguments were adopted by Mr. Jethmalani, the learned counsel for
Respondent Nos. 3 to 27 (Harshad Mehta Group). The said arguments were also adopted
by the counsel for other notified parties. However, Mr. Jethmalani added that in the Order
of Variava, J. (as he then was) dated 20th February, 1995, three questions of law were
settled. In the said ruling the learned Judge has set out the time for distribution under
Section 11 and, it was on that basis, that the Supreme Court proceeded to give its interim
order for drafting the Scheme of sale of shares (see Order dated 11th March, 1996 being
the interim order in Civil Appeal No. 5326 of 1995). The Supreme Court till then had not
considered the question as to when the stage for distribution arises under Section 11. It
was contended that the said issue was settled finally when the Supreme Court delivered
the judgment on 13th May, 1998 in the case of Harshad Mehta vs. Custodian, reported in
(1998) 5 SCC pg. 1. Therefore, at the time of delivering the interim order on 11th March,
1996, the question as to when the sale and distribution took place, remained unanswered.
However, he contended that in view of the final judgment of the Supreme Court in the
above case of Harshad Mehta, this Court should proceed on the basis of the final
judgment of the Supreme Court. The learned counsel contended that in view of the
judgment of the Supreme Court, the stage for sale and distribution of the assets under
Section 11 of the Act arise only after completion of examination of all civil claims under
Section 9A of the Special Court Act and only after the assessment orders of the Revenue
Department reached finality and only after they became binding i.e. when the assessee
has exhausted all statutory remedies under the Act and since the process of examination
of claims under Section 9A has not commenced, the Scheme is premature. It was
contended that on a proper and legal assessment, the actual tax liability of Harshad Mehta
Group would be marginal and a large portion of the amounts would have to be refunded
by the revenue. He contended that in case of Harshad Mehta Group, the demands made
by the Department are based on the best judgment assessments, which are highly
exaggerated. He contended that the assessment orders are ex parte in nature. He
contended that Harshad Mehta 1998 AIR SCW 2193

@page-SC2643
Group is contesting the demands before the Appellate Authorities. That significant reliefs
have been given by the tax department and, therefore, no sale should take place so that a
reasonable opportunity is given to Harshad Mehta Group to bring down the demands to
realistic levels.........."
(Emphasis supplied).
19. When answering the issue, the learned Judge of the Special Court Hon. S.H. Kapadia,
J. (as he then was), firstly, found as a preface that 3 Chartered Accountants Firms were
appointed by the Special Court for preparing the submission of accounts of 9 notified
parties including Harshad Mehta Group and Dhanraj Mills. A complaint was made by the
3 firms of Chartered Accountants that they had not received the relevant documents from
Harshad Mehta Group. The learned Judge deduced that no progress was made in the
matter of accounting, as there was opposition for the sale of shares. The learned Judge
referred to the large number of shares in possession of the Custodian as of date being 6.65
crores, out of which Harshad Mehta Group only controlled 2.88 crores of shares, apart
from the benami and unregistered shares. The learned Judge then noticed that there were
in all 6.65 crores of shares. The learned Judge then observed :
"the position which has emerged is that the notified parties have not brought before the
authorities/accountants appointed by the Court the relevant documents. It is for this
reason that even the income-tax department has ultimately proceeded to assess some of
the assessee-notified parties under best judgment assessment. During the said period none
of the notified parties have come before the Court claiming that the assets are more than
the liabilities. Their only contention is that the liabilities have not been crystallized. Their
only contention is that till final adjudication, is carried out by all the authorities under the
Income-tax Act by way of appeals, the assessment is not final and binding."
20. After noting these preface facts, the learned Judge examined the judgment of this
Court and noted that the only condition prescribed by the judgment vide paragraph 13
was the satisfaction of the Special Court before it gives directions for disposal to the
effect that the attached property belonged to the notified parties. The learned Judge,
therefore, held that there was a dichotomy between the sale and distribution, which was
accepted by this Court. The learned Judge then noted that the scheme was not for
distribution and it was not under Section 11(2). The scheme instead was under Section
11(1) for sale and, therefore, the arguments regarding the sale being premature, as the
distribution point had not arrived at, was liable to be rejected. The learned Judge then
went on to hold, on the basis of this Court's judgment, that Section 11(2) could not be
restrictive only to the tax liability during the statutory period and it covers all assessed
taxes due for pre-statutory period and post-statutory period. Further, the learned Judge
observed :
"However, in answering the last contention of the notified parties that the liability should
have been ascertained on the date of distribution, the Supreme Court observed that the
date of distribution arrives when the Special Court completes the examination of claims
under Section 9A and any tax liability for the statutory period is finally assessed and the
assessment is final and binding, then such liability will be considered for payment under
Section 11(2)(a) of the Act. As stated hereinabove, the pre-condition for sale of the
property is that the attached property belongs to the notified parties whereas, the
precondition of distribution is completion of examination of claims under Section
9A......"
There is no principle of law shown to this Court that sale cannot take place till
completion of examination of all claims under Section 9A of the Act."
The learned Judge, therefore, recorded his conclusions as under :
"CONCLUSIONS ON POINT A
(a) That sale is different from distribution. The Scheme placed before this Court is for
sale of shares. The Scheme is not for distribution of assets. Therefore, Section 11(1) of
the Act applies to the Scheme and Section 11(2) which deals with distribution does not
come into picture at this stage of the matter.
(b) That Sections 11(2)(a), (b) and (c) cover claims for pre-statutory period, statutory
period and post-statutory period.
(c) On scaling down, in appropriate cases as held by the Supreme Court, the liability of
the assessee of the balance tax would subsist and the taxing authorities would be entitled
to realize the remaining liabilities
@page-SC2644
including penalty and interest from the assessee under Section 11(2)(c). Therefore, there
is no merit in the contention that the funds of some of the notified parties with the
Custodian are far in excess of the tax demand and, therefore, they should not be brought
within the Scheme for sale of shares.
(d) In view of the provisions of the Special Court Act, it is not necessary for this Court to
postpone, in any event, the sale of shares till the claims against the notified parties are
finally adjudicated upon. Looking to the income-tax demands, the decrees passed in
various suits by this Court, it is clear that the liabilities of the notified parties exceed the
attached assets and, therefore, one need not wait till all pending claims are finally
adjudicated upon.
(e) The words 'taxes due' in Section 11(2)(a) only refers to the liability, which is
completed in accordance with the provisions of the income-tax Act. In other words, the
expression 'taxes due' would mean assessed tax, which are presently payable. The said
expression does not contemplate taxes as finally payable."
21. By way of a second question, which pertained to the challenge of the validity of
scheme, the learned Judge held the scheme to be valid and further considered the
objections raised against the scheme and rejected the same. The objections were more
particularly related to the modality to be adopted for sale of some shares. The learned
Judge then decided the norms in respect of the bulk shares : (1) Norms for preparation of
lots of bulk shares; (2) Norms for Sale of bulk shares; (3) Norms for lot preparation in
respect of controlling block of shares; (4) Norms for sale of controlling block of shares;
(5) Norms in respect of routine shares. The learned Judge also decided upon procedure to
be followed by the Custodian for registration/dematization of shares before
implementation of the above norms.
22. Ultimately, the learned Judge approved the scheme with the modifications.
Undoubtedly, the following points are clear from the above judgment :
(1) That the existence and the treatment of Sh. Harshad Mehta and his relatives and some
concerns as Harshad Mehta Group was neither objected to nor contradicted and the
learned Judge was addressed by all those entitites as the Harsahd Mehta Group.
(2) That a clear dichotomy was there in the matters of sale of shares and the distribution
of assets.
(3) The scheme for the sale of shares which was ordered by the interim order of this
Court and was finalized in the 1998 judgment, was approved with some modifications.
(4) That the total liabilities are more than the total assets of the notified parties.
23. There was an appeal filed against this judgment, which appeal was disposed of by this
Court by its judgment dated 23-8-2001. The opening words of this judgment are very
telling. They are :
"In these appeals, the only question relates to the Scheme devised by the Special Court
for the sale of shares of the appellant-Apollo Tyres Ltd."
This Court noted that the Special Court had categorized the shares into 3 classes. They
being : (1) Routine Shares, (2) Bulk Shares, (3) Controlling Block of Shares. The Court
constituted a Disposal Committee and had issued directions in respect of those shares.
The Court observed :
"The whole emphasis, and in our opinion rightly so, of the Special Court has been to
ensure that maximum price is realized from the sale of the said shares. Keeping this in
view, we do not find that the Special Court has erred in issuing the aforesaid directions.
After hearing the counsel for the parties, we affirm the said directions with minor
changes."
24. This Court then gave certain directions for the sale of bulk shares and modified the
order dated 17-8-2000, holding that it would be more appropriate that the offer of the
shares be not restricted only to the institutional buyers, and the non-institutional buyers
including the management of the company may also be offered the shares of all the
appellant-company. It observed that in that way, the best price would be realized. As
regards the controlling block of shares, the Court directed that it would be open to the
Special Court to decide whether to have the sale of the controlling block of shares either
by inviting bids for purchase of controlling block as such or by selling the said shares
according to the norms fixed for the sale of bulk shares or by the norms fixed in respect
of routine shares. With these words, the order of the learned Special Judge was totally
confirmed. It is only on the
@page-SC2645
basis of this order, that ultimately, the advertisements came to be issued. However, there
is one more development, which we must refer to.
25. On 26-4-1999, the Custodian had filed an application being Misc. Application No. 41
of 1999, seeking permission of the Special Court for sale of the residential premises
commonly known as Madhuli of eight notified entities. A miscellaneous application being
Misc. Application No. 4 of 2001 was filed by the Custodian praying for the sale of
commercial premises. Some of the notified persons filed several miscellaneous
applications for lifting of attachment on their residential premises on the ground that the
same had been purchased much prior to 1-4-1999 and the same had no nexus with any
illegal transactions in securities. All these applications were disposed of by the Special
Judge by his judgment dated 17-10-2003, who held that if an undertaking is given by the
adult members of the family of late Sh. Harshad S. Mehta (by then Sh. S. Harshad Mehta,
as already expired), in the Special Court within a period of 4 weeks to vacate the flat
occupied by them and hand over peaceful possession thereof to the Custodian within a
period of 4 weeks from the date on which the Custodian sends them communication, the
Custodian shall permit the members of family of late Sh. Harshad S. Mehta to occupy the
flats during the time that the process of the sale of the flats goes on. This was challenged
before this Court on the following grounds :
"(i) Some of the entities having assets much more than actual liability, the impugned
judgments are unsustainable. There was no occasion for the Custodian to club all the
notified entities in one block so as to be termed as Harshad Mehta Group and/or to club
their assets and liabilities jointly. Although in relation to a body corporate incorporated
and registered under the Companies Act, the doctrine of lifting the corporate veil would
be applicable, but the same cannot be applied in case of individuals.
(ii) Having regard to the fact that only three entities out of eight were involved in the
offences, the liability of Harshad Mehta could not have been clubbed for the purpose of
directing attachment and consequent sale of the properties which exclusively belong to
them.
(iii) The liabilities of Harshad Mehta, who was a sui generis, could have been recovered
from the properties held and possessed by him or from the companies floated by him but
not from the individual entities; at least two of whom being medical practitioners have
their income from other sources.
(iv) The books of accounts and other documents on the basis whereof the auditor's report
had been made having not been allowed to be inspected by the appellants herein on the
plea that they had the knowledge thereabout, the same could not have been taken. into
consideration for the purpose of passing of the impugned order or otherwise.
(v) The appellants having preferred appeals against the income-tax orders of assessment
passed by the authority and the same having been set aside, no liability to pay income-tax
by the appellants as of now being existing, the residential properties could not have been
sold.
(vi) Drawing our attention to a representative chart showing the discrepancies in the
accounts of Mrs. Deepika A. Mehta as shows in (a) affidavit by the custodian; (b) books
of accounts maintained by the appellants; and (c) auditor's report, it was submitted that
the auditor's report could not have been relied upon.
(vii) A copy of the auditor's report having only been supplied during pendency of these
appeals, the learned Special Judge committed a serious error in passing the impugned
judgment relying on or on the basis thereof.
26

. On behalf of the respondents, it was pointed out that all properties belonging to the
notified persons, could be applied for discharge of the joint liabilities of the Harshad
Mehta Group in terms of Section 11 of the Act in view of the ruling reported in (2004) 11
SCC 456. It was secondly, contended that applications for denotification by the appellants
were already withdrawn and, therefore, they could not raise the contention that they were
not liable in terms of the provisions of the Act, and they could not also file fresh
applications for denotification, as such applications would be barred by time. It was
further contended that the tax liability have become final. It was also suggested that the
appellants apart from the corporate entities, had received large loans, advances and
credits from the Harshad Mehta Group and that there had been intermingling of the assets
to the tune of crores of rupees, and as such, they could not escape their liabilities into
2004 AIR SCW 6868
1998 AIR SCW 2193
2004 AIR SCW 6868

@page-SC2646
the Act and, therefore, liabilities exceed the assets. Some other grounds on merits were
also raised. Lastly, it was contended that the sale of commercial property had only been
seriously contested by the appellants and a contention was raised that if the commercial
properties were sold, there would be no need to sell the residential properties. They
pointed out that even before this Court, the sale of commercial properties was not
questioned. This Court after analyzing the various provisions of the Act, referred to the
ruling in (1998) 5 SCC 1 (cited supra) in extenso. It also referred to the other ruling in L.
S. Synthetics Ltd. vs. Fairgrowth Financial Services Ltd. in (2004) 11 SCC 456. The
Court formulated 5 issues :
(i) Whether the appellants being not involved in offences in transactions in securities
could have been proceeded against in terms of the provisions of the Act.
(ii) Whether individual liabilities of the appellants ought to have been separately
considered by the Special Court as not a part of Harshad Mehta Group.
(iii) Whether the tax liabilities could not have been held to be due as the order of
assessments did not become final and binding.
(iv) Whether the commercial properties could have been sold in auction.
(v) Whether the residential properties should have been released from attachment.
27. In paragraph 30 of the judgment, this Court expressed that barring Harshad S. Mehta,
Ashwin S. Mehta and Sudhir S. Mehta, the denotification applications were filed by
individual and corporate appellants, and by order dated 14-7-2000, those applications
were permitted to be withdrawn with the permission to refile the same. In paragraph 31,
the Court expressed that the said applications were pending for consideration before the
Special Court and since those applications were to be decided by the Special Court
particularly in respect of the limitation and jurisdiction etc., the Court will refrain itself
from adverting to the said question. In paragraph 41, the Court observed that it was open
to the appellants to show that even if they continue to be notified, the Custodian was not
right in clubbing all the individual members of the family as a single entity styled as
Harshad Mehta Group. The Court noted that a property belonging to the mother of
Harshad Mehta was released from attachment. The Court then went on to consider the
liabilities against the notified parties as also the valuation of immovable properties. The
Court also disapproved the acceptance by the learned Judge, of the figures mentioned in
the affidavit of the Custodian and stated that the learned Judge had relied upon the same
without discussing the contentions and arguments raised on behalf of the appellants. The
Court observed that it was necessary to give another opportunity of hearing. In para 51,
the Court observed that if any notified party had no connection with late Sh. Harshad
Mehta, they could not have been proceeded against for meeting the liabilities of late Sh.
Harshad Mehta jointly or severally and a clear finding was required to be arrived at. The
Court, further, observed :
"It was, thus, necessary for the learned Special Court to arrive at a firm conclusion as
regards the involvement of the individuals with Harshad Mehta, if any, and the extent of
his liability as such."
28. In paragraph 55, the Court noted the judgment dated 17-8-2000 passed by the Special
Court by Hon. Kapadia, J. (as he then was), as also the fact that the appeal against the
same was dismissed by this Court. The question of sale of commercial properties was
considered from paragraphs 67 to 73 and that of the sale of residential properties in
subsequent 3 paragraphs i.e. paragraph Nos. 74 to 76. In paragraph 73, however, it was
observed that the Court was not to interfere with that part of the order, whereby, the
auction-sale as regards the commercial property had been directed by the learned Judge.
Lastly, the Court recorded its conclusions with paragraph 77. Some of the relevant
conclusions are to be found as below :
"(i) The contention of the appellants that they being not involved in offences in
transactions in securities could not have been proceeded in terms of the provisions of the
Act cannot be accepted in view of the fact that they have been notified in terms thereof.
(ii) The appellants being notified persons, all their personal properties stood
automatically attached and any other income from such attached properties would also
stand attached. The question as to whether the appellants could have been considered to
be part of Harshad Mehta Group by the learned Special Court need not be determined
@page-SC2647
by us, as, at present advised, in view of the fact that appropriate applications in this
behalf are pending consideration before the learned Special Court. The question as
regards intermingling of accounts by the appellants herein with that of the Harshad Mehta
Group and/or any other or further contentions raised by the parties hereto before us shall
receive due consideration of the learned Judge, Special Court afresh in the light of the
observations made hereinbefore.
xxxxxxxxxxxxxxxxxx
(vi) We direct the Custodian to permit the appellants to have inspection of all the
documents in his power or possession in the premises of the Special Court in the presence
of an officer of the court. Such documents must be placed for inspection for one week
continuosly upon giving due notice thereof to the appellants jointly. As the appellants
have been represented in all the proceedings jointly, only one of them would be
nominated by them to have the inspection thereof. The appellants shall be entitled to take
the help of a chartered or cost accountant and may make notes therefrom for their use in
the pending proceeding.
(vii) The appellants shall file their objections to the said report, if any, within ten days
thereafter. The Custodian may also take assistance and/or further assistance from a
chartered accountant of his choice. A reply and/or rejoinder thereto shall be filed within
one week from the date of the receipt of the copy of the objection. The parties shall file
their respective documents within one week thereafter. Such documents should be
supported by affidavits. Both the parties shall be entitled to inspect such documents and
file their responses thereto within one week thereafter. The parties shall file the written
submissions filed before this Court together with all charts before the learned Special
Judge, Special Court within eight weeks from date.
(viii) The learned Judge, Special Court shall allow the parties to make brief oral
submissions with pointed reference to their written submissions. Such hearing in the
peculiar facts and circumstances of this case should continue from day to day.
(ix) The learned Judge, Special Court while hearing the matter in terms of this order shall
also consider as to whether the auction-sale should be confirmed or not. It will also be
open to the learned Judge, Special Court to pass an interim order or orders, as it may
think fit and proper, in the event any occasion arises therefor.
(x) We would, however, request the learned Special Judge, Special Court to complete the
hearings of the matter, keeping in view the fact that auction sale in respect of the
residential premises is being considered, as expeditiously as possible and not later than
twelve weeks from the date of the receipt of the copy of this order. Save and except for
sufficient or cogent reasons, the learned Judge shall not grant any adjournment to either
of the parties.
(xi) The learned Judge, Special Court shall take up the matter relating to confirmation of
the auction sale in respect of the commercial properties immediately and pass an
appropriate order thereupon within four weeks from the date of receipt of copy of this
order. If in the meanwhile the orders of assessment are passed by the Income Tax
Authorities, the Custodian shall be at liberty to bring the same to the notice of the learned
Special Court which shall also be taken into consideration by the learned Judge, Special
Court."
29. As has been stated by us in paragraph 9 of this judgment, the learned counsel for the
appellants based his contentions more or less on the observations made in the
aforementioned judgment and it is, therefore, that we have dealt with that judgment
extensively.
30. The contentions raised by Sh. Jethmalani based on the aforementioned judgment are:

(1) that the Custodian and the Special Court have failed to comply with the directions
given by this Court in the aforementioned judgment dated 3.1.2006 in Civil Appeal No.
667-81 of 2004 (hereinafter called Ashwin Mehta's case) and more particularly, in
paragraphs 41, 42, 46, 47, 51, 52 and 53, the whole arguments turns practical on this very
issue. 2006 AIR SCW 243

(2) that there was no reason for the Special Court to have ordered the sale of shares, and
the Custodian as well as the Special Court have failed to justify the decision to put the
shares on auction and distribute the liabilities.
(3) that such decision is arbitrary and the sale of the shares shall lead to serious loss to the
notified persons. The liabilities were
@page-SC2648
only of late Sh. Harshad S. Mehta and not of the other notified parties and since the assets
of the notified parties can meet their liabilities, the sale of the shares by auction was not
justified. This is all the more true in view of the fact that the Custodian has not yet found
the inter se liabilities of the notified parties, when their applications for denotification are
not decided and pending before the Special Court.
(4) it was also submitted by the learned counsel that because of the earlier sale of the
shares, the parties were put to the loss of 6500 crores and that though the objections for
denotifications were pending before the Special Court, the same have not yet been
disposed of and, therefore, the decision to sell the shares belonging to the notified parties
is wholly incorrect.
(5) that the whole decision to put the shares for sale by auction is jurisdiction-ally,
procedurally, as well as financially not correct.
31. It will be, therefore, our task to test these propositions on the anvil of the judgment in
Ashwin Mehta's case, which is treated to be the backbone of the arguments of the
appellants herein.
32. First of all, we must point out even at the cost of repetition that the decision to sell the
shares was taken in the last part of the year 2007. The notice itself was issued in the
month of October. That was of course, on the basis of advice by the Disposal Committee.
On the objections having been taken, the learned Judge had given certain directions, but
before giving those directions, the learned Judge has practically wiped out the effect of
the auction. It must be remembered that in that auction, the LIC of India had made an
offer of Rs. 2,701/- per share, which offer was accepted. However, the learned Judge
found and in our view, rightly stated that something more was required to be done
procedurally, as well as, by way of a policy. We have extensively quoted the directions
given by the learned Judge, on the basis of the four formulated objections. In order to
comply with the directions given by the learned Judge, it was necessary to put the auction
proceedings on hold. However, the LIC of India was not prepared to keep its offer open
and, therefore, the learned Judge had relieved the LIC of India of its obligations on the
basis of its offer.
33. The learned Judge rightly felt that since the Custodian had not consulted the Disposal
Committee regarding the appropriate time to be given for submitting the offers, the
Custodian should do that first. The learned Judge, thereafter, held that that opinion of the
Disposal Committee was bound to be sought for even on the question as to whether if the
option is given to the buyers to bid for one or more groups instead of putting the bid for
the entire bulk, it could fetch more price. As regards the opportune time, the learned
Judge, however, held that the shares were being sold at the proper time. Lastly, as regards
the tax liability also, the learned Judge had directed the Custodian to seek legal and
professional advice, and for that purpose, practically wiped out the effect of the auction
by directing the compliance of his directions. Thus, for all the practical purposes, one
thing was certain that the shares were not to be sold unless all the directions were
complied with. However, the appellants did not wait and rush to this Court even before
the Disposal Committee had given its opinion on the various issues and even before the
Custodian was able to get the legal and professional opinions regarding the tax liability.
34. It is obvious that the Disposal Committee would now have to again take a decision
whether at this point of time, the shares should be sold or not. The Disposal Committee
consists of the experts who would know best, whether the shares should at all be sold at
this point of time. However, the appellants have come before this Court insisting that the
shares should not be sold at all, which stand was conspicuously absent when the matters
were argued firstly before the Special Court or even before the this Court, as we do not
find any trace of the said contentions in the arguments before the Special Court. The old
theory of not selling the shares at all unless individual liabilities were fixed one way or
the other was wreaked up in this appeal and very surprisingly, though the order of the
learned Special Judge was completely confirmed by this Court which also meant that the
shares were bound to be sold. Completely giving a go-by to the judgment of this Court
dated 23-8-2001, by which the judgment of the Special Court was confirmed, the
appellants are now saying that the shares cannot be sold. This would be impermissible
now.
35

. While doing so, some factual 2006 AIR SCW 243

@page-SC2649
incorrect statements have also been made before us, as they were made before this Court
in Ashwin Mehta's case, that the applications for denotifications were pending. In fact,
the argument which was pressed in service in the Court was that since the denotification
applications were pending, and had not been finally decided upon, the properties
belonging to the notified persons should not be sold. We have before us, the current status
of such applications from which it is seen that each and every notified appellant herein
had already withdrawn his/her denotification application, some of them in 1997 and rest
of them in January, 2000. Thus even on the day, when the matters were being argued
before this Court in Ashwin Mehta's case, excepting those by Mrs. Rasila S. Mehta and
Mrs. Rina S. Mehta (who are not covered here), no application for denotification was
pending. True it is that permission was given to withdraw their applications with a liberty
to file fresh petitions after the criminal trials, if any, are over but no application has been
filed. Thus, the main stay of the arguments on the part of the appellants is knocked out on
the basis of this fact, and it is not open to the appellants to say that since there are chances
of their denotification, the shares belonging to them should not be sold.
36

. When the provisions of the act are seen in the light of conclusions drawn and more
particularly, the first conclusion in Ashwin Mehta's case by this Court, the properties of
the notified persons like the appellants would stand automatically attached and any other
income from such attached properties would also stand attached. It is obvious that on the
day when Ashwin Mehta's Judgment was delivered, there were no applications pending
ponsideration before the Special Court nor are any such applications pending today also.
Hence, an objection to the sale, on that ground, must be rejected. This answers contention
No. 3. 2006 AIR SCW 243

37. The second aspect, which we would like to consider is the objection which is now
raised to the nomenclature 'Harshad Mehta Group'. We do not want to go to that aspect,
because for the decision of this case, that is not a relevant aspect whether any appellant is
referred to as a group or not. The very fact that such appellant is a notified person would
be enough for the attachment of his/ her property because of the Section 3(2) of the Act.
In our opinion, there would be no necessity to consider the individual liability of any such
appellant being a notified person. Unless any appellant is denotified, there would be no
question of raising of these defences regarding individual liability. It is obvious, that the
notification covers all the properties including the shares and securities of the notified
persons and, therefore, comes into the hands of the Custodian. There would, therefore, be
no question of raising the issues that the individual liability of such a notified person
should be arrived at first. We say this, particularly, because the claim of the notified
persons that their assets exceeds the liability, is also not correct. That is a clear-cut
finding given by Hon. Kapadia, J. in his judgment dated 17-8-2000, which is later on
confirmed by this Court. The Custodian argues before us and not without any reasons that
the tables prepared by the Custodian shows that the liability of the notified persons does
exceed assets, we shall not go into that aspect at this juncture. However, the fact remains
that there would be no question of any individual liability being arrived at before the
shares are sold. The judgment of the learned Special Judge for selling the shares having
been confirmed by this Court, whereby, the decision to sell the shares has been confirmed
by the three-Judge Bench of this Court concluding the issue. The same is binding.
Therefore, it cannot be said at this juncture at least, that on that account, the sale of the
shares should be postponed, till such time, as the question of individual liability viz-a-viz
Harshad Mehta is decided upon.
38. We have already shown that there is a clear dichotomy arrived at by Hon. Kapadia, J.
in his aforementioned judgment dated 17-8-2000, that there is no question of waiting for
the distribution and on that account, the sale cannot be stopped. The judgment having
been confirmed by the three Judge Bench of this Court, that question will not be opened.
39

. This takes us to the aforementioned paragraphs heavily relied upon by the learned
counsel in the judgment of Ashwin Mehta's case (cited supra). In paragraph 41, it was
stated that it was open to the appellants to show that even if they continued to be notified,
the Custodian was not right in clubbing all the individual members of the family as a
single entity styled as Harshad Mehta Group. We 2006 AIR SCW 243

@page-SC2650
do not find that there was any attempt on the part of the appellants to disassociate
themselves from Harshad Mehta Group. When we see the judgment dated 17-8-2000
passed by the Special Court, it is obvious that the learned counsel arguing that matter had
argued it on behalf of the Harshad Mehta Group. It is for this purpose that we have
quoted in the argument before the learned Special Judge in extenso. We will only quote a
sentence which forms a part of the argument :
"it was contended that on a proper and legal assessment, the actual tax liability of
Harshad Mehta Group would be marginal and a large portion of the amounts would have
to be refunded by the revenue. He contended that in case of Harshad Mehta Group, the
demands made by the Department are based on the best judgment assessments, which are
highly exaggerated. He contended that the assessment orders are ex parte in nature. He
contended that Harshad Mehta Group is contesting the demands before the Appellate
Authorities."
40. It was, therefore, obvious that at that juncture, when the question was as to whether
the shares should be sold or not, the move was objected to by the appellants formulating
themselves as Harshad Mehta Group. No such objection to form and treat the relatives as
a group was raised before the Special Court in the year 2000 when the question of sale of
shares fell for consideration for the first time. At any rate, unless it is shown as to what
prejudice would be caused by treating them to be a group, this contention has no basis.
We, therefore, do not think that the argument in this behalf has any basis.
41. In paragraphs 46 and 47, this Court has criticized that the Special Court should have
analysed the respective contentions of parties in greater details and in particular, in regard
to the assets and liabilities of the separate entities, having regard to the contentions raised
by them that they are not part of Harshad Mehta Group and their individual liabilities can
be met from the assets held and possessed by them separately. We must immediately
point out that these observations did not relate to the sale of shares. These observations
obviously related to the sale of the immovable properties, regarding which the appeal was
filed. The proceedings in that case emanated out of miscellaneous application No. 41 of
1999, seeking permission of the Special Court for sale of residential premises commonly
known as Madhuli of eight notified entities, as also for the sale of the commercial
premises and the only objection raised there was that the attachment should be lifted on
the ground that the same properties had been purchased prior to 1-4-1991 and the same
had no nexus with any illegal transactions in securities. It was also objected to on the
ground that the asset base was greater than genuine liabilities, and hence, the residential
premises should be released from attachment. We may, at this juncture, point out that this
Court in the aforementioned judgment has specifically held with respect to commercial
properties in paragraph 73 that the Court was not to interfere with the sale of the
commercial properties and the Special Court was even allowed to pass appropriate orders
regarding the confirmation of the sale of such properties. It was only in respect of the
residential property that this Court had directed the Special Court to deal with the matter
afresh.
42
. We must repeat at this juncture, that the judgment in Ashwin Mehta's case did not
concern the shares in the name of the appellants/ notified parties and the sale thereof
which question was already decided finally by this Court while confirming the judgment
dated 17-8-2000 passed by the Special Court. As regards the observations in paragraphs
51 and 52 again these pertains to the subject of the Group. It is expressed that a question
may further arise as to whether the learned Judge was correct in considering the
individual liability of the notified parties as the liabilities of the Group. It is then
expressed : 2006 AIR SCW 243

"if there were certain individuals, who had no connection with Sh. Harshad Mehta, they
could not have been proceeded against for meeting the liabilities of Sh. Harshad Mehta
jointly or severally and a clear finding was required to be arrived at. Only because there
had been large intermingling and flow of funds from Sh. Harshad Mehta and inter se
within the group, the same by itself may not justify the conclusion that all of their assets
were required to be sold irrespective of their individual involvement and it was, therefore,
necessary for the learned Special Court to arrive at a firm conclusion as regards the
involvement of the individuals with Sh. Harshad Mehta, if any, and the extent of his
liability as such."
@page-SC2651
No such argument seems to have been advanced before the Special Judge at all in respect
of the shares and the securities in respect of which this Court had finalized the issue.
43. Paragraph 52 refers so-called contradictory stand taken by the Custodian regarding
the liabilities which were treated to be joint liabilities of the Harshad Mehta Group and
further, inconsistency on the part of the Custodian to treat the liabilities of the notified
entities also as their separate liabilities. Such question was not addressed in the Special
Court. We have already shown that the observations would not apply to the sale of shares
as the issue was concluded by this Court on 23-8-2001.
44. This Court had directed the Custodian in Ashwin Mehta's case to permit the
appellants to have inspection of all the documents in his power or possession in the
premises of the Special Court in the presence of an officer of the court. In compliance
thereof, the Custodian argues before us, that such inspection was to be allowed for one
week continuously and all the documents in possession of the Custodian were laid open
for a period of one week. The Custodian further points out that the directions in these
paragraphs 41, 42, 46, 47, 51, 52 and 53 were dealt with by the Custodian in his affidavits
dated 1-3-2006 and 22-3-2006 filed in Miscellaneous Petition No. 49 of 1999, and in
these affidavits, the Custodian had taken into account the assets and liabilities position of
each of the notified entities as on 31-12-2005 and those statements were also annexed to
those affidavits. It is further pointed out that in each case, the liability was more than
assets. The Custodian argues before us that late Sh. Harshad S. Mehta had siphoned off
money from Banks and Financial Institutions and distributed the same to his family
members and various corporate entities by transferring the money to their accounts by
purchasing shares in their names through 3 brokerage firms : (1) M/s. Harshad S. Mehta;
(2) M/s. Jyoti S. Mehta; (3) M/s. Ashwin S. Mehta and all the transactions of purchase
and sale of 25 notified entities were debited and credited in their mutual interest.
45. It is the further case of the Custodian that the notified parties had shown in their
accounts, that these siphoned off monies were received by them as loan, borrowings and
advances, and also shown that they were paying interest thereon to Sh. Harshad Mehta
with the sole idea to show that they were running their own business with their own funds
and that the monies borrowed by them. The Custodian has taken a stand before us that in
the affidavit dated 1-3-2006, efforts have been made to show clearly as to how much
money is transferred in cash to his relatives and corporate bodies and also how much
siphoned off money was utilized for the purchase of shares in the name of various
notified entities including the appellants. The affidavit dated 22-3-2006 is filed before us.
It is the stand of the Custodian that he has already worked out the position of the assets
and liabilities separately for individual members of the family and it is reflected in the
affidavit dated 1-3-2006. The Custodian further submits that these accounts show that for
the present, all the notified entities of the Harshad Mehta Group are in excess of their
assets. It is thus, pointed out that all these materials were already available before the
Special Court passed the orders. The Custodian further argues that these facts are known
to the appellants, and there is an attempt to mislead the Special Court as well as this
Court on the part of the appellants. It is then submitted that all the accounts, which are
audited and reviewed by the Chartered Accountants have been prepared by the notified
parties themselves and it is, therefore, that the liabilities shown therein, have been taken
as admitted liabilities. In our opinion, this argument on the part of the Custodian must be
accepted. It has already been shown in the earlier part of the judgment that all these
contentions were only raised before the Special Court, particularly, when the objections
were raised. We do find some traces of these objections in the petition, but it is obvious
that these questions were never pressed into service before the Special Court, perhaps
because the appellants knew the futility thereof. We, therefore, leave the matters at that,
in view of the final order that we propose to pass.
46

. This is apart from the fact that before us also, not even a distant reference was made to
these affidavits dated 1-3-2006 and 22-3-2006. A bald statement was made that there was
no compliance of this Court's order in Ashwin Mehta's case. We are certain that if these
arguments had been addressed 2006 AIR SCW 243

@page-SC2652
before the Special Court, the Special Court would have taken note thereof. The Special
Court chose to go-by the judgment of this Court confirming the earlier judgment
regarding the sale of shares passed by the Special Court [Hon. Kapadia, J. (as he then
was)] and in our opinion, that was a right approach since the controversy involved,
related to the sale of shares and securities. At this juncture, we cannot ignore the fact that
in 2005 itself, in pursuance of the judgment dated 17-8-2000 and the confirming
judgment of this Court, the majority of the shares have already been sold. It is only in
respect of the Reliance Shares that the present sale was contemplated. It is really
surprising that when the major shares were sold way back in 2005, the appellants did not
think it proper either to challenge the same or to raise this argument of the individual
liabilities viz-a-viz, the group liabilities or the second argument that unless the
denotification applications were decided upon, there should be no sale of shares. We have
already pointed out the hollowness of the argument regarding the denotification
applications, which were claimed to be pending before the Special Court, which claim is
also baseless. Therefore, on both these counts, there would be no question of finding fault
with the impugned order of the Special Court.
47. This takes us to another contention raised more particularly, in point numbers 2, 4 and
5. Apart from the fact that the contentions were never raised before the Special Court, it is
pointed out that as in the earlier case of the sale of shares in 2005, the Disposal
Committee which was formulated under the orders of the Special Court as also this
Court, had found that it was opportune time for the sale of Reliance Shares. The
Custodian argues that the instant sale of Reliance Shares was being carried out strictly in
compliance with the procedure laid down by the Special Court and this Court in the
earlier-referred judgments. It is pointed out by the Custodian that between 12-12-2000 to
1-11-2007, 12.12 crores shares valued at Rs. 1792.77 crores were sold. Out of these, the
shares worth Rs. 1463.96 crores belonged to the various entities of Harshad Mehta Group
including the appellants. The learned counsel very surprisingly did not refer to these facts
during his arguments, instead, it was suggested that there was a loss of 6500 crores of
rupees because of the sale. Such figure apart from being imaginary, has no basis. We
cannot ignore that the Special Court is dealing with the scam which shook the whole
financial world of India. We again cannot ignore the fact that the decision to sell the
shares in 2005 was taken by the Disposal Committee, which consisted of the experts of
the financial world who were well-experienced in the sale of shares and securities and
who had a thorough study of the share markets. No mala fides were ever alleged against
the Disposal Committee. Under the circumstances, we find no reason to accept the
argument that the earlier sale caused huge loss and, therefore, the shares should not be
sold. In our opinion, the Special Court was right in confirming the advice and accepting
the report filed before him on behalf of the Custodian justifying the sale of shares.
However, all that exercise, we are afraid, would have to be repeated again, particularly,
because more than six months have elapsed after that decision and the sale has yet not
taken place. The Special Court has referred back the matter and has passed the directions
for obtaining the legal and expert advice to deal with the taxes. We are told at the Bar that
such exercise had already been completed. It would, therefore, be proper for the Disposal
Committee to again decide as to whether the shares should be sold at all and when. That
would depend upon the market conditions and so many other factors which are certain to
be considered by the Disposal Committee. The Custodian has referred all the happenings
during the pendency of this appeal and has relied on the report dated 27-11-2007. We
need not go into the question, since, it would be for the Disposal Committee to decide
upon the proper time and the manner in which the sale is to be executed, and it would be
for the Special Court to further decide on the matter.
48. In view of what we have stated above, we are convinced that the appeals have no
merits. However, one thing is certain that the sale, as well as the decision to make the sale
at a particular time, stand frustrated because of the lapse of time. The whole procedure for
sale of shares will have to be repeated now, meaning thereby, that the Disposal
Committee would have to take a fresh decision in the light of the directions given by the
Special Court, which are the correct directions. That shall be done at the opportune time.
If the appellants so feel, they
@page-SC2653
would be at liberty to put their objections subject to what has already been said in this
judgment. The appeals are dismissed with the above observations under the
circumstances. The cost is quantified at Rs. 2 lakhs.
Appeals dismissed.
AIR 2008 SUPREME COURT 2653 "Maghendra Pal Tyagi v. Jayant Davar"
(From : Bombay)
Coram : 2 C. K. THAKKER AND LOKESHWAR SINGH PANTA, JJ.
Civil Appeal Nos.3034-3036 of 2005, D/- 5 -6 -2008.
Maghendra Pal Tyagi v. Jayant Davar and Ors.
Special Court (Trial of Offences Relating to Transactions in Securities) Act (27 of 1992),
S.3, S.7 - SPECIAL COURT - SECURITY TRANSACTION - Transfer of shares - 200
duplicate shares of Company were found to be obtained by appellant by
misrepresentation - Said 200 shares plus 50 Bonus shares issued thereon were attached by
C.B.I. in proceedings initiated against deceased, notified person - Said shares could not
be transferred to any party - Transfer of shares during stipulated period covered under Act
- Direction by Special Court to transferor to pay the purchase amount to purchaser with
interest at rate of 18% till date of realisation - No interference. (Paras 17, 18)
Cases Referred : Chronological Paras
2006 AIR SCW 243 : AIR 2006 SC 795 : 2006 CLC 178 (Ref.) 16
2004 AIR SCW 6868 : AIR 2005 SC 1209 : 2005 CLC 117 (Ref.) 15
1996 AIR SCW 3826 : AIR 1997 SC 442 (Ref.) 16
Abhishek Vikas Singh, Yunus Malik, Ravi Kishore, Ravindra Tyagi and Himinder Lal,
for Appellant; T. V. S. Raghavendra, Nikhil Nayyar, Rohit Aggarwal, Jagjit Singh
Chhabra and Subramanium Prasad, for Respondents.
Judgement
1. LOKESHWAR SINGH PANTA, J. :- These appeals are directed against the judgment
and order dated 10-10-2003 in Misc. Application No. 186 of 2000 (impugned order-1);
order dated 14-7-2004 in Misc. Application No. 178 of 2004 in M. A. No. 186/ 2000
(impugned order-2) and order dated 18-08-2004 in Misc. Application No.263 of 2004 in
M. A. No. 186/2000 (impugned order-3) passed by the Special Court constituted under
The Special Courts (Trial of Offences Relating to Transactions in Securities) Act, 1992
[hereinafter referred to as "the Act"] at Bombay.
2. The short facts leading to the present proceedings are as under :-
The appellant herein held 200 shares of Hero Honda Company - fourth respondent-
company herein. In and around September 2003, the appellant desired to dispose of the
said 200 shares, but he allegedly lost the same. On 21-09-1993, the appellant got a police
report registered in the Sihani Gate Police Station, Ghaziabad. On or about 22-09-1993,
the appellant approached and requested the fourth respondent-company for issue of
duplicate Certificates in lieu of his lost shares along with all supporting documents,
indemnity bonds and affidavits, etc.
3. On 05-01-1994, the fourth respondent-company got an Advertisement/Public Notice
published in Newspapers calling upon to file objections, if any, against issue of duplicate
Share Certificates to the appellant and also striking a note of caution to the public at large
not to deal with the shares so specified in the advertisement. Having not received any
objection from any one, the fourth respondent-company on 03-02-1994 issued duplicate
Share Certificates to the appellant. The appellant transferred his shares in favour of
Jayant Davar the first respondent herein, which were registered in his name by the fourth
respondent-company on 18-10-1994. The first respondent had been offered 50 Bonus
Shares by the fourth respondent-company, which offer was profitably availed by him. The
first respondent sold/transferred 200 shares which he got from the appellant and 50
Bonus Shares consequently acquired by him, but the fourth respondent-company did not
register the said transfer. This was done on the asking of the CBI, who was investigating
the Share Transfer Scam, and advised the first respondent and the transferee to approach
the Custodian - second respondent herein. The fourth respondent-company asked the
appellant to enforce indemnity bond, but the appellant did not agree as the fourth
respondent-company had not suffered any loss, etc. as a result of the transaction.
4. The case of the first respondent before the Special Court was that he purchased 800
shares of fourth respondent-company
@page-SC2654
through its broker M/s. Jamnadas Morarjee and Co. during the months of July-August,
1994 and thereafter the said shares were sent for transfer to the Registrar and Share
Transfer Agent of respondent No. 3, i.e. MCS Limited. The shares were finally
transferred in his name on the basis of valid instruments of transfer and, accordingly, a
ledger folio No. 141982 has been allotted to him. The second respondent filed
Miscellaneous Application No. 186/2000 before the Special Court claiming 200 shares
which were transferred in his name from the appellant and 50 bonus shares in the ratio of
1:4 as issued by third respondent against those 200 shares. The first respondent stated
before the Special Court that he had sold 250 shares in the open market through his share
broker M/s. TRC Securities Pvt. Ltd. in the month of May/June, 1997. Upon lodgment of
the said 250 shares with MCS Limited, they, vide their letter dated 26-06-1997 refused to
transfer/register the shares in the name of the lodger i.e. Morgan Stanley Assets
Management Inc., A/c Morgan Stanley Institutional Fund Inc. Emerging Markets
Portfolio. Subsequently, the 250 shares were returned to first respondent as 'Bad Delivery'
under two different covering letters dated 26-06-1997 and 10-07-1998 respectively. MCS
Limited received a letter bearing No. 5696/Cus/Mob/UR-CBI/96 (533B) dated 29-02-
1996 from the second respondent-Custodian regarding stop transfer of the shares in
favour of any person without permission of the Custodian. The MCS Limited also
enclosed copy of transfer deeds, share certificates and Custodian's letter dated 29th
February, 1996 along with their letter to the second respondent who on going through the
same, came to know that 117335 shares of fourth respondent- company belonged to the
Notified Persons of the group of Late Harshad S. Mehta which were seized by CBI and
remained in their custody. The letter also revealed that Late Harshad S. Mehta and his
group were notified by the Custodian on 8-6-92 under the provisions of the Act and all
properties belonging to them stood attached simultaneously with the issue of the
notification and the fourth respondent-company was informed not to deal with those
shares in any manner including transfer, pledge, issue of duplicate etc. and all corporate
benefits admissible on these shares may be held in abeyance till the orders passed by the
learned Special Judge.
5. In view of the above stated circumstances, the first respondent requested the fourth
respondent-company to transfer the shares in the name of the buyer who purchased the
same in the open market. The fourth respondent-company vide their letter dated 8th
October, 1997 informed the first respondent that one Mr. Mahendra Pal Tyagi - appellant
herein was holding the said 200 shares under Ledger Folio No. 128027 bearing Share
Certificate Nos. 58193, 46706, 179855 which he claimed having been lost and requested
the fourth respondent-company to issue duplicate shares in lieu of the aforesaid original
lost share certificates. The appellant also submitted the Police Report, indemnity bond,
affidavit along with the request letter to the fourth respondent-Company who issued
duplicate shares to the appellant under Share Certificate Nos. 191549-191552. The
duplicate shares subsequently were lodged by the first respondent for transfer in his name
and, accordingly, the fourth respondent-company transferred the said shares on 18th
October, 1994 in the name of the first respondent.
6. The fourth respondent-company thereafter received a letter dated 29th February, 1996
from the office of the second respondent-Custodian whereby the Custodian asked the
fourth respondent-company to "stop transfer" of certain shares including the shares which
are the subject matter of these proceedings and also held in abeyance all the benefits
accruing on those shares as the said shares were seized by the CBI at the time of raid laid
on the places of Late Harshad S. Mehta. The letter of 8th October, 1997 revealed that the
list furnished by the second respondent-Custodian includes original shares of the
appellant which he allegedly lost. The fourth respondent-company, therefore marked
"stop transfer" against the duplicate shares which were transferred in the name of the first
respondent and advised the first respondent to approach the stock exchange through
whom the first respondent purchased those shares so that through proper channel, the
introducing broker as well as the share holder, i.e. the appellant could be asked to replace
the said shares with good shares. The first respondent admitted that he purchased 200
shares from the open market through their share broker and paid the consultation thereof
and thereafter the shares were also
@page-SC2655
registered in his name by the fourth respondent-company as per the provisions of the
Companies Act, 1956 and he had absolutely no knowledge about the duplicate shares
being issued in the name of the appellant by the fourth respondent-company. He claimed
that in the facts and circumstances narrated in the application, first respondent is the real
and only owner of these shares and accordingly, all corporate benefits accrued thereon
since the date of registration of the 250 shares in his name, be paid to him in the interest
of justice. By reasons of the impugned order dated 10-10-2003, the learned Special Judge
allowed Misc. Application 186 of 2000 filed by the first respondent. It was directed :-
"This application relates to 250 shares of respondent No.3 company. It appears that the
respondent No.4 who was holding these shares had sold the shares on the Stock
Exchange which were purchased by the notified party. Taking advantage of the fact that
the notified party because of the notification could not apply for transfer of the shares, the
respondent No.4 applied for duplicate shares by making a misrepresentation that he has
lost the shares and received from the Company the duplicate shares. Those duplicate
shares were again sold and they were now purchased by the applicant. The principal
prayer in the application is for lifting of attachment on these 250 shares. It is obvious that
these 250 duplicate shares have been issued by the Company because of
misrepresentation made by the respondent No.4. By an order dated 16th July, 2003, the
respondent No.4 was directed to deposit in this Court an amount of Rs.6,00,000/-. The
respondent No.4 has not obeyed this order. In the affidavit filed by the respondent No.4,
the explanation that has been given by him is incapable of being accepted. There are no
documents produced in support of that explanation. It is thus clear that there is no
question of attachment of 250 shares of the respondent No.3 company being lifted. The
relief to which the applicant would be entitled is to recover from the respondent No.4 the
value of the shares. It is clear from the report submitted by the Custodian, that these
shares were purchased by the Applicant in the month of June 1994 and payment for it was
made by cheque dated 6th July, 1994 and the amount was Rs.2,94,400/-. The applicant
therefore would be entitled to a decree against the respondent No.4 in this amount. The
application therefore is disposed off in the following terms.
The respondent No.4 is directed to pay to the applicant an amount of Rs.2,92,400/-with
interest at the rate of 18% p.a. from 6th July, 1994 till realization. Application is disposed
of."
7. Being aggrieved, the appellant filed Review Application being Misc. Application No.
178 of 2004 under clause (f) of sub-section (5) of Section 9-A of the Act before the
learned Special Judge. The said application came to be rejected on 14-07-2004 vide
order, which reads as under :-
"Called for hearing and Final Disposal. None for the applicant.
Mr. Modi i/b Yogesh Thakur for Respondent No. 1.
Mr. J. Chandran i/b M/s. P.M. and Mithi and Co. for the Custodian/Respondent No.2.
Mr. V.M. Singh i/b Arun Mehta for Respondent No.4.
Coram D.K. Deshmukh, J.
Judge, Special Court
Dated 14th July, 2004.
P.C.
Matter called twice. None present for the applicant. Application rejected."
8. Again, the appellant preferred Misc. Application No.263 of 2004 for restoration of the
Review Petition, which was dismissed and the following order came to be passed on 18-
08-2004 :-
"Even assuming that due to mistake of the lawyer, lawyer could not remain present and
therefore, the review petition was rejected, after having heard the learned counsel
appearing for the Applicant on the review application, I find that there is no reason to
review the order dated 10th October, 2003. Applicant was Respondent No.4 in Misc.
Application No. 186 of 2000. By order dated 16th July, 2003, he was directed to deposit
an amount of Rs.6 lakh in the court. He did not obey that order. Therefore, the Applicant
is not entitled to any indulgence from this court.
Misc. Application disposed of."
9. Hence, the appellant has assailed the above-said three orders before this Court in these
appeals preferred under Section 10 of the Act.
10. During the pendency of the appeals
@page-SC2656
in this Court, the legal representatives of late Harshad Mehta are substituted as
respondents Nos. 3(i), (ii) and (iii).
11. Mr. Abhishek Vikas Singh, learned counsel appearing on behalf of the appellant, in
assailing the orders of the learned Special Judge, inter alia, contended that the learned
Special Judge did not appreciate the fact that the original shares were not valid and legal
and had come to the hands of the notified person (deceased Harshad Mehta) in illegal and
wrongful manner and were never transferred and registered in his name in accordance
with law and as such, the appellant could not have been penalized for the acts and deeds
of a third person, who had acquired the shares in illegal and clandestine manner. He
submitted that the action of the appellant being bona fide and reasonable, he had faced
loss at last stage, even when the duplicate shares were already stood transferred in his
name in due course after following all legal procedures and due application of law. The
learned counsel then contended that the orders of the learned Special Judge impugned in
these appeals have resulted in manifest error and miscarriage of justice to the appellant,
which deserve to be set aside.
12. Mr. Rohit Aggarwal, learned counsel appearing on behalf of the first respondent, on
the other hand, would inter alia submit that the learned Special Judge passed an order
based upon the material on record which would reveal that the appellant had committed a
fraud of selling 200 shares on the Stock Exchange and thereafter applying to the fourth
respondent-company for duplicate shares on the plea that the said shares had been stolen.
He also submitted that the learned Special Judge had not burdened the appellant with
payment for the entire amount of 800 shares as alleged, but in fact has directed payment
of Rs.2,92,400/ - with interest thereon, which is the value of 250 shares only.
13. Mr. Subramanium Prasad, learned counsel appearing on behalf of second respondent-
Custodian, would contend that the appellant had sold the shares in question to late
Harshad Mehta, a notified person under Section 3(2) of the Act and deceased Harshad
Mehta could not apply for transfer of those shares, the appellant, on a misrepresentation
that he had lost the shares, applied for and got duplicate shares from the fourth
respondent-company, which were also sold by the appellant to first respondent. Learned
counsel for the respondents, in nutshell, supported the orders of the learned Special Judge
which, according to them, cannot be found faulty or invalid on any grounds whatsoever
as alleged by the appellant.
14. We have given our thoughtful and anxious consideration to the respective contentions
of the learned counsel for the parties and perused the material on record. The contentions
of the learned counsel for the appellant at the first blush sound attractive, yet we are
afraid to accept the same.
15

. The undisputed facts are that the first respondent purchased 800 shares including 200
shares (the subject matter of the proceedings) of fourth respondent-company in open
market in the months of July and August, 1994 through its share broker M/ s. Jamnadas
Morarjee and Co., C-4 Defence 1 Colony, New Delhi-24. The fourth respondent-
company allotted 50 bonus shares to him against the said 200 shares in the ratio of 1:4. In
all, the dispute before the learned Special Judge was limited to 250 shares. Late Harshad
S. Mehta, who was a party -third respondent herein, is represented through his legal
representatives Nos. 3(i), (ii) and (iii) respectively. Indisputably, deceased Harshad Mehta
was a notified person under sub-Section (2) of Section 3 of the Act and the appellant
transacted the said shares with the deceased Harshad S. Mehta entered after the first day
of April, 1991 and on or before 1st June, 1992, the stipulated period covered under the
Act. Claim submitted by the first respondent before the learned Special Judge would arise
out of the transaction of the said 250 shares between Late Harshad S. Mehta and the
appellant during the aforesaid period. The entire properties belonging to the notified party
on the day of notification would stand attached in terms of Section 3(2) of the Act. The
appellant knowing fully well that he has already sold the shares to late Hashad S. Mehta,
he made a false representation to the fourth respondent-company that as the appellant had
lost original shares, therefore, duplicate shares were allotted to him which stood in his
name since late Harshad S. Mehta had not applied for change of the name. The 2004
AIR SCW 6868

@page-SC2657
whole exercise was done by the appellant on the basis of his mis-representation. This
Court in L. S. Synthetics Ltd. v. Fairgrowth Financial Services Limited and Anr. (2004)
11 SCC 456, held that Section 3(3) of the Act should be literally construed and all
properties belonging to the notified party on the date of notification would stand attached.
16

. In terms of the provisions of sub-section (3) of Section 3 of the Act, the properties
belonging to deceased Harshad S. Mehta being a notified person stood attached. Such
attachment being automatic, no finding was required to be arrived at that the same had
been acquired during the notified period. In Tejkumar Balakrishna Ruia v. A.K. Menon
(1997) 9 SCC 123, this Court held that the terms of sub-section (3) Section 3 are clear
that the property that belongs to a notified person stands attached simultaneously with the
issue of notification that makes him a notified party. It is said that the words 'on or from
the date of notification' indicate the point of time at which the attachment takes effect;
this is reiterated by the words 'shall stand attached simultaneously with the issue of the
notification'. Further that this also indicates that no special notification or order in regard
to the attachment is necessary. In the latest judgment of this Court in Ashwin S. Mehta
and Ors. v. Union of India and Ors. (2006) 2 SCC 385, this Court reiterated that property,
be it shares, dividends and bonus and rights shares that belongs to a notified person
would also be attached property. 1996 AIR SCW 3826
2006 AIR SCW 243

17. In this view of the matter, learned Judge of the Special Court has rightly concluded
that 200 duplicate shares were obtained by the appellant by misrepresentation. The said
200 shares plus 50 Bonus shares were attached by the CBI in proceedings initiated
against deceased Harshad S. Mehta, therefore, the attached shares of the fourth
respondent-company could not be transferred to any party. The record of second
respondent-Custodian would reveal that 250 shares were purchased by the appellant in
the month of June, 1994 and payment of Rs. 2,92,400/- was made by cheque dated 6th
July, 1994. In these circumstances, the learned Special Judge directed the appellant to pay
to the first respondent an amount of Rs. 2,92,400/- with interest at the rate of 18% per
annum from 6th July, 1994 till the date of realization.
18. In the backdrop of the facts and circumstances and in the light of the provisions of
law, in our view, the orders of the learned Special Judge Impugned in these appeals do
not suffer from any infirmity or illegality warranting interference in exercise of appellate
power.
19. For the reasons aforementioned, we do not find any merit in these appeals which are
dismissed, accordingly. Parties are left to bear their own costs.
Appeals dismissed.
AIR 2008 SUPREME COURT 2657 "National Insurance Co. Ltd. v. General Insurance
Dev. Officers Assocn."
(From : Punjab and Haryana)*
Coram : 2 Dr. A. PASAYAT AND P. SATHASIVAM, JJ.
Civil Appeal Nos. 2438 to 2442, 2450, 2454, 2456, 2437, 2444-49, 2453, 2451-52, 2455
of 2008 (arising out of SLP (C) Nos. 8115, 8117, 8118, 8324, 8325, 12693, 12438 and etc
etc. of 2003) T. C. (C) Nos. 60 to 64, 73 of 2004, 42 and 47 of 2005, D/- 3 -4 -2008.
National Insurance Co. Ltd. v. General Insurance Dev. Officers Assocn. and Ors.
General Insurance Business (Nationalisation) Act (57 of 1972), S.17A, S.16(1)(g) -
General Insurance (Rationalisation of Pay Scales and Other Conditions of Service of
Development Staff) Amendment Scheme (2003), Cl.1 - INSURANCE - SERVICE
MATTERS - Modifications in Scheme - Done for purpose of rationalisation - Held, not
liable to be interfered with - However, it would be in interests of officers and insurance
companies not to transfer Development Officers who work within cost ratio unless
transfer is required to be done in public interest - Draft policy formulated for wage
revision, to be finalised within period of three months. (Para 21)
Cases Referred : Chronological Paras
2001 AIR SCW 1309 : AIR 2001 SC 1493 (Ref.) 19, 21
AIR 1984 SC 1130 : 1984 Lab IC 691 (Ref.) 19
Vikas Singh, ASG, Ranjit Kumar, L. N. Rao, Mahabir Singh, Sr. Advocates, R. C.
Mishra. Dr. Meera Agarwal, Dinesh Mathur,
@page-SC2658
H. K. Puri, S. C. Dhand, V. M. Chauhan, S. K. Puri, Ms. Priya Puri, R. Santhan Krishnan,
Ms. K. Radha Rani, P. Vijaya Kumar, Praveen K. Pandey, D. Mahesh Babu, Sunil Kumar
Jain, S. Borthakur, Ajay K. Bhatia, Manish Pitale, Chander Shekhar Ashri, Mrs. Sunita
Sharma, Ms. Sushma Suri, Romy Chacko, Arpit Gupta, Dinesh Verma and A. P. Mohanty,
for the appearing parties.
* C.W. P. No. 3204 of 2003, D/- 31-3-2003 (P and H).
Judgement
Dr. ARIJIT PASAYAT, J. :- Leave granted.
2. These appeals are taken up along with Transfer Case (Civil) Nos.60-64/2004, 73/ 2004,
42/2005 and 47/2005.
3. In all these cases the basic issue is the legality of General Insurance (Rationalisation of
Pay Scales and Other Conditions of Service of Development Staff) Amendment Scheme,
2003 (in short '2003 Scheme').
4. The present scheme purports to amend the earlier scheme framed under Section 17A of
the General Insurance Business (Nationalisation) Act, 1972 (in short the 'Act'). The
principal scheme was framed in 1976 in exercise of powers under Section 16(1)(g) of the
Act. The scheme was amended earlier in the years 1987, 1990 and 1996 and 2000. The
principal scheme of 1976 was challenged but the challenge was turned down and legality
of the scheme was upheld by this Court. Several writ petitions have been filed by
Development Officers questioning legality of the scheme on the ground that there was
unilateral change of service conditions of the Development Officers in Class II category.
The declaration sought for in the writ petitions was that administrative guidelines dated
5-2-2003 were without power, jurisdiction and legal sanctity. It was pointed out that
while changing service conditions of the Development Officers in Class II category the
service conditions of other employees in Class I, III and IV were not touched. According
to the Development Officers the following stipulations affected them :
"Cost Norms: As per 2 (c) in the amendment, the proviso of clause 7 of the original
scheme of 1976 as amended in 1990 was omitted.
The proviso inserted as per 1990 amendment is as follows :
"Provided that for the purposes of Paras 11, 11A and 13 cost shall mean gross
emoluments paid to the development officer during a performance year".
The Development Officer Marketing governed by cost norms has to perform within
stipulated cost ratio. As per the pre amended scheme he gets the benefit of two tier cost
system i.e.
1. For the purpose of increment.
2. For the purpose of incentives.
5. Now by the 2003 amendment single cost system has been introduced whereby the cost
system for the purposes has been withdrawn by deleting the proviso to clause 7.
The comparison table is as follows :

Development Officer Operating at City/townApplicable in relation to increment


As per 2003 Cost ratio Existing cost ratio Applicable in relation to incentives.
As per 2003 cost ratio Existing cost ratio
A Cities 7% 8% 7% 7%
B Cities/Towns 8% 9% 8% 8%
C Other Centres 10% 11% 10% 10%

Existing scheme was amended in 1996.


"Cost ratio" is the ratio expressed as percentage of cost incurred on a person of the
development staff to the scheduled premium income procured through him during the
concerned year.
6. Cost relaxation was done from time to time by amending the scheme. The 2003
amendment brought down the cost ratio by 1% in all centers thereby increasing the cost
ratio beyond stipulated limits. This resulted in monetary loss by way of decrement. This
would not only lead to reduction in salary but would ultimately result in termination of
service.
7. Through the following illustration it is demonstrated that as to how the consequence of
the 2003 amendment adversely affects a development officer having a basic pay of Rs.
13,630/-
@page-SC2659

SALARY COST RATIO PREMIUM TO BE PROCURED


EXISTING BASIC. DA, HRA, CCA Rs. 2,55,096 8% Rs. 31,88,000/-
REVISED BASIC, DA, HRA, CCA Rs. 2,55,096 Add : Non-Core allowance Rs.
54,000 Conveyance + Entertainment + Phone + TE Rs. 3,09,096 7% Rs.
44,15,000/-

8. The above illustration shows how a development officer put on constrain to maintain
his cost in revised norms he has to procure an additional premium of rs. 12,27,000/- in
this competitive market scenario or otherwise he will directly lose the monetary benefits
proportionate to his premium income.
9. The core benefits that a Development Officer gets is as indicated in the original
scheme in the shape of "gross emoluments" which is an aggregate of basic pay, dearness
allowance, hill station allowance, house rent allowance and city compensatory allowance.
10. The Non-Core benefits such as Conveyance, Entertainment, Telephone allowance,
Travelling Expenses incurred to procure premium, are exempted from Income Tax as per
CBDT Rules. But through the 2003 Amendment the respondents have added the entire
non core benefits to the cost ratio. Thereby as per the above illustration the development
officer who was procuring a business of Rs. 32,00,000/- premium has to now procure a
business of Rs. 43,36,000/- to maintain the cost ratio and to make himself eligible for an
increment.
11. Deletion of ASPI Provision.
As per the original scheme of 1976 para 12 indicates that a development officer shall
have to procure a minimum premium income out of all or any of the following types of
business namely :
1. All risk insurance, 2. baggage insurance, 3. cash-in-transit insurance, 4. cattle
insurance, 5. insurance of pump sets and lifts, 6. machinery breakdown insurance, 7.
pedal cycle insurance, 8. personal accident insurance for individuals including the janata
personal accident policies, 9. shop keepers or house holders comprehensive insurance,
10. any other class of insurance notified by the Central Government from time to time in
this behalf. This has been omitted by 2003 Amendment.
The premium earned in this category is called Adjusted schedule premium income. If a
development officer procures premium on this count the same is credited to his account
with double benefit. Such a premium earned by a development officer gives him the
benefit of adjusted premium income that is ASPI as the specified business prescribed by
the company from time to time. If premium is not procured under this category the
schedule premium income earned shall be notionally reduced by an amount equal to the
shortfall and such reduction shall not be deemed as penalty.
Withdrawal of para 12 through the 2003 amendment pushes the development officer into
an extreme difficulty in achieving the premium targets and fulfilling the cost norms. This
not only results in monetary loss in the form of non core allowance but also leads to
decrements thereby adversely affecting the service conditions.
12. Change in incentive Scheme
Through paras 14, 14A and 15 a Development Officer would get Cost based Growth
Incentive and Profit Incentive. The growth incentive and cost saving profit incentive is
based on the performance of the development officer. The margin in cost ratio as
provided in the amended scheme 1987 are withdrawn and replaced by one single
incentive scheme which is totally based on the profitability as per the 2003 amendment.
This incentive is directly related to the claims arising due to accidents, and natural
calamities which are beyond the control of a development officer. This is arbitrary as in
any industry it is universally accepted that the incentives to the marketing staff shall be
@page-SC2660
linked to their sales p erformance.
13. No career prospects :
The 2003 amendment gives a development officer an option to take a voluntary
retirement or in the alternative to opt to be in the administration. But the scheme is silent
in regard to career prospects of a development officer who opts to work in the
administration. Without specifying as to what would be the promotional avenues for a
person opting for working in administration. Such an option would be meaningless and
the amended scheme would arbitrarily push the development officer out of the company.
14. Transfer :
A Development Officer who works in a particular area invests his time and energy to
familiarize himself with the market conditions and thereafter starts procuring business for
the Company. Now by the 2003 amendment the respondent has brought in transfer policy
where a Development Officer can be transferred to totally a new place even to a different
State also. This would not only make the life of a Development Officer difficult but he
would not be in a position to procure business for a company immediately. This action of
the respondent virtually amounts to killing of the insurance business.
It has been pointed out that because of the introduction of the scheme not only the
Development Officers suffered financial loss but there shall be great deal of
inconvenience caused because of the transfer modes.
A Development Officer because of his personal efforts nourishes the locality and with his
personal touch attracts more persons for being covered by insurance coverage. It is also
submitted that though there is provision for being transferred to administrative posts, it is
not clear as to what are the promotional prospects.
15. In response, learned counsel for the respondents submitted that Section 17A provides
for framing, amending, adding to and altering schemes governing the conditions of
service of the various classes of employees in various Public Sector General Insurance
Companies. Section 17A(4) provides a copy of every such scheme is required to be laid
before each House of Parliament. Section 17A(6) provides that every such scheme shall
have effect notwithstanding any other law, award, instruments etc. The Central
Government has power under Section 17A(2) to amend a scheme under Section 16(1)(g).
The Impugned amendment scheme was made taking into consideration the
recommendations made by Malhotra Committee in its report which is known as
"Malhotra Committee Report on Reforms in the Insurance Sector".
16. It is the stand of the respondents that as a matter of fact the report was foundation for
introduction of the Insurance Regulatory and Development Authority Act, 1999 (in short
'IRDA Act'). On the basis of the recommendations amendments were made to the Act,
Life Insurance (Business Nationalization) Act, 1956 and the Insurance Act, 1938 (in short
the 'Insurance Act'). The Malhotra Committee examined the state of the insurance
industry and gave specific suggestions regarding the working of Development Officers
and other reforms inter alia necessary for the growth of the insurance industry.
17. It is pointed out by learned counsel for the respondents that it is not correct to say that
in every case in routine manner transfers will be affected. The cost ratio, it is pointed out,
is the same as was in 1976. It is stated that normally a Development Officer who
functions within the cost ratio will not be transferred. Presently, the practice is to transfer
within 150 kms. It is also stated that promotional norms for Class I, III and IV category
officers have been finalized. In case of Class II officers because of order of status quo
passed by some High Courts the same is at the draft stage and the same shall be finalized
after disposal of these cases. It is also pointed out that there is scope for wage revision on
a five year basis. The periods to which these cases relate are 1st August, 2002 and 1st
August, 2007. The revision has not been effected because of status quo order passed by
this Court and various High Courts which are the subject matter of challenge in the
Special Leave Petitions where leave has been granted.
18. It is true as contended by learned counsel for the writ petitioners that a personal factor
has a role to play notwithstanding the overall importance of the entity. With opening of
economy there is a remarkable change in the various sectors including the insurance
sector. Since modifications appear to have been done for the purpose of rationalization,
there is no scope for interference because essentially a policy decision is immune
@page-SC2661
from judicial review unless it is founded on no rational basis or material to justify the
change in policy.
19

. It is to be noted that initially the Central Government had amended the scheme under
Section 16(1)(g) which was struck down by a three-Judge Bench of this Court in Ajoy
Kumar Banerjee and Ors. v. Union of India and Ors. (1984 (3) SCC 127). Thereafter the
Act was amended in the year 1985 w.e.f. the appointed day under the Act i.e. 1-1-1973.
By virtue of this amendment a new Section 17A was introduced in the Act and the
Central Government was empowered to amend the scheme under Section 16(1)(g) and
the authority was upheld in Kishan Prakash Sharma and Ors. v. Union of India and Ors.
(2001 (5) SCC 212). It was inter alia observed in the said case as follows : AIR 1984 SC
1130
2001 AIR SCW 1309

"2. The Preamble to the Act explains the purpose of the Act as to provide for the
acquisition and transfer of shares in the Indian insurance companies and undertakings of
other insurers in order to serve better the needs of the economy in securing development
of general insurance business in the best interest of the community and to ensure that the
operation of the economic system does not result in concentration of wealth to the
common detriment for the regulation and control of such business and for matters
connected therewith or incidental thereto. Section 2 declared that it was for giving effect
to the policy of the State towards securing the principles specified in Article 39(c) of the
Constitution and under Section 3(a) "acquiring company" has been defined as any Indian
insurance company and where a scheme had been framed involving the merger of one or
more insurance companies in another or amalgamation of two or more such companies
means the Indian insurance company in which any other company has been merged or the
company which has been framed as a result of amalgamation. Section 4 provides that on
the appointed day all the shares in the capital of every Indian insurance company shall be
transferred to and vested in the Central Government free of all trusts, liabilities and
encumbrances affecting these. Section 5 provides for transfer of the undertakings of other
existing insurers. Section 6 provides for the effect of transfer of undertakings. Section 8
provides for provident fund, superannuation, welfare or any other fund existing. Section
9 stipulates that the Central Government shall form a government company in accordance
with the provisions of the Companies Act to be known as "General Insurance Corporation
of India" for the purpose of superintending, controlling and carrying on the business of
general insurance. Section 10 stipulates that all shares in the capital of every Indian
insurance company which shall stand transferred to and vested in the Central Government
by virtue of Section 4 shall immediately on such vesting, stand transferred to and vested
in the Corporation. Chapter 4 deals with the amounts to be paid for acquisition. Chapter 5
of the Act deals with the scheme for reorganisation of general insurance business.
Sections 16 and 17 are important, to which we will advert to later and by amendment of
the Act by an Ordinance issued in 1984 and subsequently replaced by an Act in 1985, the
said provisions have been amended and a fresh provision was introduced as Section 17-A
to which we will advert later in detail. After the Act came into force, several schemes
have been framed by the Board of Directors and two Schemes, one dated 30-7-1977
amending the provisions regarding sick leave and another Scheme pertaining to the
payments to be made to the provident fund were challenged before this Court in the case
of Ajoy Kumar Banerjee v. Union of India. The main ground of attack in that writ petition
is that the amended notification altering the conditions of service is illegal as the Central
Government has no power to issue it under Section 16 of the Act and as such the
notification framing the scheme is ultra vires Section 16(1) of the Act. It was contended
that once the merger of the Indian companies had taken place and the process of
reorganisation was complete on 1-1-1974 as stated before by forming the 4 insurance
companies by 4 Schemes framed in 1973, there could be no further reorganisation of the
general insurance business and the merger of more insurance companies inasmuch as in
the amended Scheme there was no merger or reorganisation contemplated unlike the
1974 Scheme. Mere amendment of the terms and conditions of service of the employees
unconnected with or not necessitated by reorganisation of the business or merger or
amalgamation of the companies could not fall within Section 16(1)(g) of the Act. It was
also noticed by AIR 1984 SC 1130

@page-SC2662
this Court that under the Life Insurance Corporation Act and the Banking Companies Act
provisions have been made to frame regulations independently of the reorganisation and
there is no such comparable power under the Act and, therefore, the Schemes impugned
herein are made without authority of the law. This contention found favour with this
Court. On interpretation of the provisions it was held that the power under Section 16(1)
(g) to frame scheme for rationalising the provisions regarding pay scales and other terms
and conditions of service of officers and other employees wherever necessary if unrelated
to the object envisaged in sub-section (2) of Section 16 of the Act will not fall within the
scope of exercise of powers and it would fall outside the same if the power exercised is
beyond delegation and in view of the fact that the Scheme of 1980 so far as it does not
relate to the amalgamation or merger of the insurance company is not warranted by
Section 16(1) of the Act. Ultimately, this Court held that the Amended Scheme of 1980
was bad as beyond the scope of the authority of the Central Government under the Act.
Further it was also made clear that the parties will be at liberty to adjust their rights as if
the Scheme had not been framed and it was further made clear that this order will not
prevent the Government, if so advised, to frame any appropriate legislation or make any
appropriate amendment giving power to the Central Government to frame any scheme as
it considers fit and proper.
xx xx xx
6. At this stage, we may notice the following amendments effected to the Act :
(a) In the definition clause in Section 3(o), the expression "scheme" was altered to mean
not only one framed under Section 16(1) but also "a scheme framed under Section 17-A".
(b) Section 16 of the principal Act was amended by introducing an additional subsection
(8) after sub-section (7) to the effect that the power to frame a scheme under subsection
(1), and the power conferred under sub-section (6) to add to, amend or vary any scheme
framed under this section, shall include the power to frame such scheme with
retrospective effect from a date not earlier than the appointed day.
(c) Section 17-A is introduced in which a validation clause and some consequential
amendments have been added which we reproduce hereunder :
"17-A. (1) The Central Government may, by notification in the Official Gazette, frame
one or more schemes for regulating the pay scales and other terms and conditions of
service of officers and other employees of the Corporation or of any acquiring company.
(2) A scheme framed under sub-section (1) may add to, amend or vary any scheme
framed under Section 16 including any addition, amendment or variation made therein by
notification under sub-section (6) of Section 16 with respect to rationalisation or revision
of pay scales and other terms and conditions of service of officers and other employees of
the Corporation or of any acquiring company, to provide for further rationalisation or
revision of such pay scales and other terms and conditions of service notwithstanding that
such further rationalisation or revision is unrelated to, or unconnected with, the
amalgamation of insurance companies or merger consequent on nationalisation of general
insurance business.
(3) The Central Government may, by notification, add to, amend or vary any scheme
framed under this section.
(4) The power to frame a scheme under sub-section (1), and the power conferred by sub-
section (3) to add to, amend or vary any scheme framed under this section, shall include
the power to frame such scheme, or, as the case may be, to make such addition,
amendment or variation in any scheme framed under this section, with retrospective
effect from a date not earlier than the appointed day.
(5) A copy of every scheme, and every amendment thereto, framed under this section
shall be laid, as soon as may be after it is made, before each House of Parliament.
(6) The provisions of this section and of any scheme framed under it shall have effect
notwithstanding anything to the contrary contained in any other law or any agreement,
award or other instrument for the time being in force.
(7)(1) Notwithstanding anything contained in any judgment, decree or order of any court,
tribunal or other authority or in any other law, agreement, award or other instrument for
the time being in force,
@page-SC2663
every scheme framed or purporting to have been framed with retrospective effect under
sub-section (1) of Section 16 of the principal Act and every notification made or
purporting to have been made with retrospective effect under sub-section (6) of that
section before the commencement of the General Insurance Business (Nationalisation)
Amendment Ordinance, 1984 shall be, and shall be deemed always to have been, for all
purposes, as valid and effective as if the amendment made in the said Section 16 by
Section 3 of this Ordinance had been part of that section and had been in force at all
material times.
(2) Notwithstanding anything contained in any judgment, decree or order of any court,
tribunal or other authority or in any other law, agreement, award or other instrument for
the time being in force, -
(a) every scheme framed, or purporting to have been framed, by the Central Government
under sub-section (1) of Section 16 of the principal Act; and
(b) every notification made, or purporting to have been made by the Central Government
under sub-section (6) of the said Section 16,
before the commencement of the General Insurance Business (Nationalisation)
Amendment Ordinance, 1984, insofar as such scheme or notification provides (whether
with or without retrospective effect) for any rationalisation or revision of pay scales or
other terms and conditions of service of officers and other employees of the Corporation
or of any acquiring company, otherwise than in relation to, or in connection with,
amalgamation of insurance companies of merger consequent on nationalisation of general
insurance business shall be, and shall be deemed always to have been, for all purposes, as
valid and effective as if Section 17-A, as inserted in the principal Act by Section 4, of this
Ordinance had been part of the principal Act, and had been in force at all material times
and such scheme or notification insofar as it provides as aforesaid had been framed or
made, under the said Section 17-A :
Provided that nothing in this section shall apply to, or in relation to, the notification dated
the 30th day of September, 1980, framing the General Insurance (Nationalisation and
Revision of Pay Scales and Other Conditions of Service of Supervisory, Clerical and
Subordinate Staff) Second Amendment Scheme, 1980.
Explanation. - In this section, the expressions "acquiring company" and "Corporation"
shall have the meanings respectively assigned to them in the principal Act."
Xx XX XX
10. Prior to 1972, there were about 106 general insurance companies both of Indian and
foreign origin. The conditions of service of the employees of the said insurance
companies were governed by the respective contracts of service between the companies
and the employees. The set-up, working, management and employment of staff by the
erstwhile insurance companies showed no uniformity. The erstwhile companies were
managed in diverse managerial systems and no uniform pattern of management could be
discovered by the Central Government after the nationalisation. There was a pronounced
disparity between one company and the other at all levels in the matter of remuneration
and designations for similar posts. Employees of different companies were holding
different designations and were paid differently for the same kind of work at the same
station. Some companies gave very high-sounding designations and paid salaries which
were not commensurate with the work. So the necessity for rationalisation of the entire
structure of general insurance business, including designations, pay scales and other
conditions of service arose.
Xx xx xx
17. The challenge now to the enactment is that this Court having held, the expression
"scheme for reorganisation of general insurance business" will not include a scheme
made after the reorganisation is complete; that no further schemes, except in connection
with the reorganisation of the general insurance business and merger of more insurance
companies could be effected and the impugned Scheme did not involve any such merger;
that therefore, this Scheme is ultra vires the Act; that the provision enabling the Central
Government to frame the Scheme is bad and the provision which gives retrospectivity to
the said enactment is equally bad as there are no guidelines in Section 17-A. Though
there can be no limitation regarding providing better terms and conditions of service the
same cannot be modified to the detriment of the workmen. The power that has been
conferred upon the
@page-SC2664
Central Government to frame the Scheme without guidelines is bad and the guidelines
have to be read into the provisions in such a manner that the benefit which is already
given to the workmen should not be taken away and there should be enough scope for
collective bargaining particularly in the absence of consultation and when there is no
limitation on upward revision, the conferment of the power upon the authority concerned
is bad.
18. So far as the delegated legislation is concerned, the case-law will throw light as to the
manner in which the same has to be understood and in each given case we have to
understand the scope of the provisions and no uniform rule could be laid down. The
legislatures in India have been held to possess wide power of legislation subject,
however, to certain limitations such as the legislature cannot delegate essential legislative
functions which consist in the determination or choosing of the legislative policy and of
formally enacting that policy into a binding rule of conduct. The legislature cannot
delegate uncanalised and uncontrolled power. The legislature must set the limits of the
power delegated by declaring the policy of the law and by laying down standards for
guidance of those on whom the power to execute the law is conferred. Thus the
delegation is valid only when the legislative policy and guidelines to implement it are
adequately laid down and the delegate is only empowered to carry out the policy within
the guidelines laid down by the legislature. The legislature may, after laying down the
legislative policy, confer discretion on an administrative agency as to the execution of the
policy and leave it to the agency to work out the details within the framework of the
policy. When the Constitution entrusts the duty of law-making to Parliament and the
legislatures of States, it impliedly prohibits them to throw away that responsibility on the
shoulders of some other authority. An area of compromise is struck that Parliament
cannot work in detail the various requirements of giving effect to the enactment and,
therefore, that area will be left to be filled in by the delegatee. Thus, the question is
whether any particular legislation suffers from excessive delegation and in ascertaining
the same, the scheme, the provisions of the statute including its preamble, and the facts
and circumstances in the background of which the statute is enacted, the history of the
legislation, the complexity of the problems which a modern State has to face, will have to
be taken note of and if, on a liberal construction given to a statute, a legislative policy and
guidelines for its execution are brought out, the statute, even if skeletal, will be upheld to
be valid but this rule of liberal construction should not be carried by the court to the
extent of always trying to discover a dormant or latent legislative policy to sustain an
arbitrary power conferred on the executive. These very tests were adopted in Ajoy Kumar
Banerjee case also to examine whether there is excessive delegation in framing schemes
and reading the preamble, the scheme and the other provisions of the enactment taking
note of the general economic situation in the country, the authorities concerned had to
frame appropriate schemes. Therefore, it is not open to the petitioners to contend that
there is excessive delegation in relation to the enactment to frame schemes.

19. In Ajoy Kumar Banerjee case this Court after holding that there is no excessive
delegation observed that the Scheme framed was ultra vires the enactment for the Scheme
could only be framed once. Now the argument is that once a scheme is framed no further
scheme should be allowed to be framed. If the legislature recognises the fact the
rationalisation resulting from the merger of several companies are not yet over and on
that basis enacts a law to enable the Government to frame appropriate schemes, we do not
think that such step by the legislature is arbitrary or irrational as to be violative of Article
14 of the Constitution. In Ajoy Kumar Banerjee case this Court pointed out that though
there is power in the Government to revise the pay scales, it cannot exercise the power
more than once at the time of merging different companies for the purpose of
rationalisation and this power could have been exercised no further. But now the
enactment itself specifically provides that every scheme framed or purporting to have
been framed by the Central Government under Section 16(1) of the principal Act and
every notification made or purporting to have been made thereunder insofar as such
scheme or notification provides for rationalisation or revision of pay scales or other terms
and conditions of the officers and other employees of the Corporation are deemed always
to have been for all purposes AIR 1984 SC 1130

@page-SC2665
as valid and effective as made under Section 17-A of the Act. The retrospective effect
given to the scheme is only to overcome the difficulty pointed out by this Court in Ajoy
Kumar Banerjee case. That lacuna having been overcome, it is not open to the petitioners
to contend that retrospective effect given is violative of Articles 14, 19 and 21 of the
Constitution. Validation of invalid rule by amending the main enactment under which it is
made is a well-known legislative device approved by this Court.
Xx xx xx
24. The Central Government, in exercise of the powers conferred under Section 16(l)(g)
of the Act, framed three Schemes for three different categories of employees relating to
(i) supervisory, clerical and subordinate staff; (ii) officers; and (iii) development staff.
The Schemes also provided, inter alia, various provisions like fixation of pay on
promotion, increments, provident fund and gratuity, etc. When the process of
categorisation and rationalisation was in progress, it was noticed that as per the 1974
Scheme, contribution to the provident fund was @ 8 per cent of the basic salary and
dearness allowance with an equal contribution of GIC or any of its subsidiaries. However,
LIC and nationalised banks were giving provident fund at different rates. So as to keep
parity with other similar organisations, the Scheme was corrected by an amending
notification issued on 1-6-1976 and it was provided that the provident fund shall be
contributed by every employee at the rate of 10% of the basic pay plus personal pay and
special pay, if any, in place of 8% of the basic salary and dearness allowance.
Xx xx xx
26. The stand of the respondents is that amendments were made while the process of
rationalisation of pay scales and other service conditions were still in progress and the
process had not been finally completed to achieve uniformity and inter se rationalisation
in terms and conditions of service of different categories of employees of merged
companies. In 1977 various labour unions presented a charter of demands in relation to
revision of pay scales and service conditions. The Scheme of 1974 contained a provision
to the effect that the provisions of the Scheme relating to scales of pay, dearness
allowance etc. will continue to be in force till the Government modified the same. After
considering the demands of the unions and the view of the management, the Government
formulated guidelines and requested the management to hold consultations and
discussions with the unions so that final views of the unions may be known and may be
taken into account by the Government before modifying the pay scales, etc. But this
course will not indicate that there was an obligation cast on the Government to formally
negotiate with the unions. However, in keeping with the democratic tradition and to
maintain harmonious industrial relations the management had several rounds of
discussions with the four major registered unions. The procedure of consultations and
discussions was adopted in order to narrow down the differences to the minimum and to
ensure that the viewpoint of the employees was kept in mind before any scheme was
finalised by the Government.
20. It was further clarified that if the scheme is prima facie discriminated it is open to
challenge.
21

. In para 28 it was held that there was no need for any consultation with the employees.
When the changes introduced by the scheme are considered in the background of the
position in law and the decision of this Court by a Constitution Bench in Prakash
Sharma's case (supra) there is no scope for interference in these appeals. However, it
would be in the interests of the officers and the insurance companies if the Development
Officers who work within the cost ratio are not transferred unless the transfer is required
to be done in public interest. So far as the promotional prospects and the wage revision
are concerned, a draft policy stated to have been formulated for the latter be finalized
within a period of three months. The writ petitions filed in different High Courts stand
dismissed because of this judgment. Consequentially, the interim orders passed which
form the subject matter of challenge in the appeals are vacated subject to the directions
given supra. 2001 AIR SCW 1309

22. The appeals are allowed. The transfer petitions stand disposed of.
Order accordingly.
@page-SC2666
AIR 2008 SUPREME COURT 2666 "Bhura Ram v. State of Rajasthan"
(From : 2004 (2) Cri LR (Raj) 1495)
Coram : 2 PRAKASH PRABHAKAR NAOLEKAR AND V. S. SIRPURKAR, JJ.
Criminal Appeal No. 587 of 2008 (arising out of SLP (Cri.) No.79 of 2006), D/- 2 -4
-2008.
Bhura Ram and Ors. v. State of Rajasthan and Anr.
Criminal P.C. (2 of 1974), S.177, S.178 - CRIMINAL COURT - CRUELTY BY
HUSBAND OR HIS RELATIVE - COMPLAINT - MAGISTRATE - Jurisdiction of
Court - All alleged acts as per complaint relating to offence u/S.408-A. I.P.C. took place
in State of Punjab - Complaint filed in Court in State of Rajasthan where complainant
residing with her maternal relations - No came of action arose in Rajasthan - Magistrate
at that place had no jurisdiction to deal with said matter.
2004 (2) Cri LR (Raj) 1495, Reversed. (Para 4)
Cases Referred : Chronological Paras
2004 AIR SCW 4788 : AIR 2004 SC 4286 : 2004 Cri LJ 4180 (Ref.) 3
K. V. Vishwanathan and Naresh Kumar, for Appellant; Milind Kumar and Aruneshwar
Gupta, for Respondents.
Judgement
P. P. NAOLEKAR, J. :- Leave granted.
2. The complainant Rajeshwari lodged a complaint on 4-9-2001 before the learned
Additional Chief Judicial Magistrate, Sri Ganganagar against the appellants. The
complaint under Section 156(3) of the Code of Criminal Procedure was sent to the Police
Station, Sadar Sri Ganganagar for investigation on which FIR No. 246 of 2001 was
registered against the appellants for offences under Sections 498A, 406 and 147 of the
Indian Penal Code (IPC). Challan was filed against the appellants in the Court of learned
Additional Chief Judicial Magistrate, Sri Ganganagar. The charges were framed against
the appellants for offences under Sections 498A and 406 IPC. The appellants made a
prayer before the Court that the Court of Additional Chief Judicial Magistrate had no
jurisdiction to try the offences as the cause of action accrued within the jurisdiction of the
other court. The application was rejected. The Revision Petition before the learned
Sessions Judge, Sri Ganganagar was also rejected. The High Court dismissed the S.B.
Criminal Miscellaneous Petition preferred by the appellants holding that although the
marriage was solemnized at Village Ramsara, Tehsil Abohar, District Ferozpur, and right
from the marriage, the complainant and her husband Ravindra Kumar were living in
Punjab with her in-laws and her husband had died, and that she is now residing in Sri
Ganganagar District in Rajasthan along with her maternal relations, but still offence
under Section 498A IPC, being a continuing one, the complaint cannot be dismissed on
the ground that it was time barred; and that the offence of cruelty being a continuing
offence is still continuing with the local area of Rajasthan, where at present the
complainant is living and, therefore, the Additional Chief Judicial Magistrate, Sri
Ganganagar had jurisdiction to try the case. The Court has found that all the allegations
regarding the offences charged with have been committed at the previous residence of the
complainant.
3

. It is contended by the learned counsel for the appellants that the question involved is
squarely covered by the decision of this Court in Y. Abraham Ajith and Others vs.
Inspector of Police, Chennai and Another, (2004) 8 SCC 100, wherein this Court has held
that cause of action having arisen within the jurisdiction of the court where the offence
was committed, could not be tried by the court where no part of offence was committed.
2004 AIR SCW 4788

4. The facts stated in the complaint disclose that the complainant left the place where she
was residing with her husband and in-laws and came to the city of Sri Ganganagar, State
of Rajasthan and that all the alleged acts as per the complaint had taken place in the State
of Punjab. The Court at Rajasthan does not have the jurisdiction to deal with the matter.
On the basis of the factual scenario disclosed by the complainant in the complaint, the
inevitable conclusion is that no part of cause of action arose in Rajasthan and, therefore,
the Magistrate concerned has no jurisdiction to deal with the matter. As a consequence
thereof, the proceedings before the Additional Chief Judicial Magistrate, Sri Ganganagar
are quashed. The complaint be returned to the complainant and if she so wishes she may
file the same in the appropriate court to be dealt with in accordance with law.
5. The appeal is accordingly allowed.
Appeal allowed.
@page-SC2667
AIR 2008 SUPREME COURT 2667 "Nikhil Kanchanlal Vakharia v. S. E. B. I."
Coram : 2 TARUN CHATTERJEE AND DALVEER BHANDARI, JJ.
Civil Appeal No.4210 of 2006, with C.A Nos. 2951, 3004, 3008 to 3010, 3015 to 3017,
3058 and 3082 of 2006, D/- 15 -5 -2008.
Nikhil Kanchanlal Vakharia v. S. E. B. I. and Anr.
Securities and Exchange Board of India Act (15 of 1992), S.12 - SECURITIES AND
EXCHANGE - STOCK EXCHANGE - Benefit of fee continuity - Registration fee - Fee
structure - First five years of operation of broker, quantum of fee linked to turn-over of
stock-broker - From sixth financial year, during each block period of five years, broker
required to pay flat rate of Rs. 5000/- - Appellant's father registered as member of Stock
Exchange and charged for first five years on basis of turnover - He was doing business in
partnership with appellant and others - Appellant nominated as member by his father due
to ill health - Claim by him for 'fee continuing benefit' i.e. charge on flat rate of Re.
5000/- - Cannot be allowed - No provision of succession to registration is permissible.
SEBI (Stock Brokers and Sub-Brokers) Rules and Regulations (1992), Regn.10.
SEBI (Stock Brokers and Sub-Brokers) Rules and Regulations (1992), R.4.
Bombay Stock Exchange Rules (1979), R.179. (Para 18)

C. A. Sundaram, Shyam Divan, Sr. Advocates, Mrs. Nisha Bhaksi, Mrs. Shashi M.
Kapila, Vikas Mehta, Ms. Bina Gupta, Varuman Khandelwal, M. K. S. Menon, Thomas J.
Arackaparamban and M. K. Michael, for Appellants; Altaf Ahmad, Sr. Advocate,
Bhargava V. Desai, Rahul Gupta, Ms. Reema Sharma, Rajeev Kumar, Ms. Jyoti
Mendiratta and Ambhoj Kumar Sinha, for Respondents.
Judgement
DALVEER BHANDARI, J. :- This batch of appeals involve the similar issue, therefore,
all these appeals are disposed of by this common Judgment. For the sake of convenience,
the facts of Civil, Appeal No. 4210 of 2006 are recapitulated.
2. This statutory appeal under section 15Z of the Securities and Exchange Board of India
Act, 1992 (hereinafter referred to as "the Act") is directed against the order dated 12th
May, 2006 passed by the Securities Appellate Tribunal, Mumbai in Appeal No.221 of
2004.
3. The impugned order is a one line order which makes a reference to the detailed order
passed on 12th May, 2006 in a companion matter being Appeal No.211 of 2004 titled as
Kamlesh Ramanlal Shah v. SEBI and Another.
4. The question which calls for adjudication in this case is regarding "fee continuity
benefit". Under the SEBI (Stock Brokers and Sub-Brokers) Regulations, 1992 (for short
"the Regulations") a fee is required to be paid by the stock brokers. Broadly, the fee was
structured in two distinct phases. In the first five years of operation of a broker, the
quantum of the fee was linked to the turnover of the stock broker. Greater the turnover,
higher the fee.
5. The second phase comprised blocks of five years from the sixth financial year after the
grant of initial registration. During each block period of five years, the stock broker was
required to pay a flat rate of Rs.5000/-in order to keep the registration in force. The flat
fee had no link to the turnover.
6. The appellant claims that whenever the event of transmission occurs within five years,
they should be given the fee continuity benefit and should not be made to pay the
turnover basis fee for the remainder of initial period of five years. The appellant is
claiming that on account of transmission, since the business and trade continues in the
same name or entity and the Stock Exchange permits continuation of the same
membership under the same number and clearing code, they should also be given the
benefit under the same registration of the earlier Stock-Broker and thus grant the benefit
of fee continuity.
7. According to the appellant, the present case involves a situation where at all material
times the stock broking firm was a partnership firm carrying on business in the name and
style of M/s. Kanchanlal and Sons. The appellant along with his son, wife and daughter-
in-law constituted a partnership firm. Late Shri Kanchanlal K. Vakharia because of his ill
health decided to nominate the appellant in his place as a member of Stock Exchange,
Mumbai (respondent No.2). The appellant claimed that he is a partner of M/s. Kanchanlal
and Sons and, therefore, now the Security Exchange Board
@page-SC2668
of India (for short SEBI) should give the benefit of fee continuity as for the first five
years they have already been charged from the partnership on a turnover basis, therefore,
they must now charge on a flat rate of Rs.5000/- per annum for the registration. The
appellant claims on account of transmission since the business and trade continued in the
same name or entity and the stock exchange permits continuation of the same
membership under the same number and clearing code. They should also be given the
benefit under the same registration of the earlier stock broker and the benefit of fee
continuity.
8. Mr. Altaf Ahmed, learned senior counsel appearing for the SEBI submitted that there is
no provision in the SEBI Act, Rules and/or Regulations of the SEBI in this behalf which
recognizes the registration of stock-brokers by inheritance and/or transmission for the
purpose of granting fee continuity benefit. The appellant who is son of Late Shri
Kanchanlal K. Vakharia on transmission can be registered only as a new stock broker
with SEBI in accordance with the Act, Regulations and the SEBI (Stock-Brokers and
Sub-Brokers) Rules, 1992 (for short "the Rules") and subject to payment of registration
fee for a new stock-broker as per the schedule fixed in the Regulations. He further
submitted that there is no provision for grant of fee continuity benefit in cases of such
transmission. The only situation under which fee continuity benefit is granted is under
para 4 of Schedule III under Regulation 10 of the Regulations, which reads thus :
"4. Where a corporate entity has been formed by converting the individual or partnership
membership card of the exchange, such corporate entity shall be exempted from payment
of fee for the period for which the erstwhile individual or partnership member, as the case
may be, has already paid the fees subject to the condition that the erstwhile individual or
partner shall be the wholetime Director of the corporate member so converted and such
Director will continue to hold minimum 40% shares of the paid-up equity capital of the
corporate entity for a period of at least three years from the date of such conversion.
Explanation. - It is clarified that the conversion of individual or partnership membership
card of the exchange into corporate entity shall be deemed to be in continuation of the old
entity and no fee shall be collected again from the converted corporate entity for the
period for which the erstwhile entity has paid the fee as per the regulations."
9. Mr. Ahmed further contended that it was an incentive for corporatisation since a
corporate entity is required to maintain all records under law and as such it facilitates
regulating of the stock brokers. Under no other circumstances fee continuity benefit is
available under the statutory regulations and hence the appellant cannot be granted
benefit of fee continuity on account of transmission.
10. Mr. Ahmed also submitted that every stock-broker who wants to deal in securities in
the securities market is required to be a member of a stock exchange and then get himself
registered with SEBI under section 12 of the Act in accordance with the procedure as
provided in the Regulations subject to the payment of registration fee for a new stock-
broker under rule 4 of the Rules and Regulation 10 of the Regulations on the rates
mentioned in Schedule-III.
Rule 4 of the Rules reads thus : "4. Conditions for grant of certificate to stock-broker. -
The Board may grant a certificate to a stock-broker subject to the following conditions
namely :-
(a) he holds the membership of any stock exchange;
(b) he shall abide by the rules, regulations and bye-laws of the stock exchange or stock
exchanges of which he is a member;
(c) in case of any change in the status and constitution, the stock-broker shall obtain prior
permission of the Board to continue to buy, sell or deal in securities in any stock
exchange;
(d) he shall pay the amount of fees for registration in the manner provided in the
regulations; and
(e) he shall take adequate steps for redressal of grievances of the investors within one
month of the date of the receipt of the complaint and keep the Board informed about the
number, nature and other particulars of the complaints received from such investors."
Regulation 10 of the Regulations reads thus :
"10. Payment of fees and the consequences of failure to pay fees. - (1) Every applicant
eligible for grant of a certificate
@page-SC2669
shall pay such fees and in such manner as specified in Schedule III;
Provided that the Board may on sufficient cause being shown permit the stock-broker to
pay such fees at any time before the expiry of six months from the date on which such
fees become due.
(2) Where a stock-broker fails to pay the fees as provided in regulation 10, the Board may
suspend the registration certificate, whereupon the stock-broker shall cease to buy, sell or
deal in securities as a stockbroker.
11. Mr. Ahmed contended that in order to become a member of the stock exchange, the
person is required to be qualified as per rule 8 of the Securities Contracts (Regulations)
Rules, 1957. This right is also not inheritable, since every person on transmission may
not even be qualified to become a member of a particular stock exchange. It is pertinent
to mention here that membership of a stock exchange is a privilege and not a matter of
right and thus this cannot be claimed as inheritable.
12. Mr. Ahmed also contended that SEBI has no discretion in implementation of the Act,
Rules or Regulations and has to strictly adhere to the provisions as laid down and,
therefore, has no power to waive the said requirement. It may also be relevant to mention
that out of the 19 stock brokers who prayed for waiver of the fresh registration or new
entities upon transmission, only 9 or 10 have come to challenge the same before this
court and balance have accepted the judgment of the learned Tribunal.
13. Mr. Ahmed further submitted that the SEBI has applied the turnover regime for the
period 1992-93 to 1996-97 and, therefore, charged on the flat rate basis. Clause I(1)(c) of
Schedule III of the Regulations reads thus :
"after the expiry of five financial years from the date of initial registration as a stock-
broker, he shall pay a sum of rupees five thousand for every block of five financial years
commencing from the sixth financial year after the date of grant of initial registration to
keep his registration in force."
14. Learned senior counsel also submitted that, under section 12 of the Act, no person can
deal in securities in the securities market without being registered with the SEBI. In the
present case, admittedly, Late Shri Kanchanlal K. Vakharia, father of the appellant, was a
member of the stock exchange and not the firm M/s. Kanchanlal and Sons. Ordinarily, if
M/s. Kanchanlal and Sons is not a member of the stock exchange, the firm would not be
entitled to deal with securities in securities market in the Bombay Stock Exchange. The
Bombay Stock Exchange does not enroll partnership firm as members. As such, Late Shri
Kanchanlal K. Vakharia alone was the member of the stock exchange and he alone was
thus entitled to deal in securities in the Bombay Stock Exchange. However, under rule
179 of the Bombay Stock Exchange Rules, an individual member can do business in
partnership with certain categorized relations and, therefore, the Bombay Stock Exchange
permits trading by the individual in the name of the partnership firm. Rule 179 of the
Bombay Stock Exchange reads thus :
"179. No partnership shall be formed except -
(i) between two or more members of the Exchange; or
(ii) between a member of the Exchange and his father or mother or wife or his son or sons
or daughter or daughter-in-law or daughters-in-law or father's brother or brothers or
unmarried sister or sisters or brother's or brother's son or sons; or
(iii) between two or more members of the Exchange and their father, mothers or wives or
son or sons or daughter or daughters or daughter-in-law or daughters-in-law or brother or
brothers or father's brother or brothers or unmarried sister or sisters or brother's or
brothers' son or sons;
Provided that a son or daughter or son's son or brother or father's brother or unmarried
sister of brother's shall not be taken into partnership unless he or she be in all respects
eligible for membership of the Exchange."
15. It was contended by Mr. Ahmed that Late Shri Kanchanlal K. Vakharia alone was a
member and through his partnership, the entire partnership firm was allowed to trade on
the Bombay Stock Exchange, the entire turnover of trade on the Bombay Stock Exchange
is relatable to the individual member Late Shri Kanchanlal K. Vakharia as otherwise the
partnership firm and non-member partners would not have been able to deal in securities
on the Bombay Stock Exchange. Consequently, this partnership firm could also not deal
with securities unless
@page-SC2670
the member of the stock exchange namely the individual member Late Shri Kanchanlal
K. Vakharia gets registered with SEBI. It is through that individual member Late Shri K.
Vakharia that the partnership firm and registered partners are able to deal in securities on
the Bombay Stock Exchange. Even otherwise, the entire turnover of the partnership firm
on the stock exchange is on securities and, therefore, relatable to the registered member
i.e. Late Shri Kanchanlal K. Vakharia under whose membership of Bombay Stock
Exchange and registration of SEBI, such trading is permitted.
16. It was also submitted on behalf of the SEBI that the appellant wants only his turnover
to be considered as a member of the Exchange and the other partners being non-member
partners want to be outside the purview of the registration of the SEBI since they cannot
be registered but at the same time want to deal in securities on the exchange under the
membership and registration of Late Shri Kanchanlal K. Vakharia.
17. According to the learned counsel for the SEBI, the entire dealing in securities by the
non-member partners would be illegal and contrary to section 12 of the Act and liable to
all such consequences in law. In fact, if the stand taken is correct then the partnership
firm is also the non-member partnership and cannot deal in securities but are dealing in
securities in breach of law.
18. We have heard the learned counsel for the parties at length and carefully analysed the
provisions of the Act, Rules and Regulations. By clear interpretation of the Regulations, it
is abundantly clear that no provision of succession to registration is permissible. Nikhil
K. Vakharia son of Late Shri Kanchanlal K. Vakharia in order to operate in the stock
exchange has to obtain a fresh registration from the SEBI and for the first five years, he
would be required to pay the quantum of fee linked to the turnover and thereafter at the
flat rate of Rs.5000/-in order to keep the registration in force.
19. In view of the provisions of the Act, Rules and Regulations, we have no difficulty in
arriving at the conclusion that the appeal is devoid of any merit and is accordingly
dismissed.
CIVIL APPEAL Nos.2951, 3004, 3008, 3009, 3010, 3015, 3016, 3017, 3058, 3082 of
2006.
20. In view of our decision in Civil Appeal No.4210 of 2006, these appeals also stand
disposed of accordingly.
21. In the facts and circumstances of the case, we direct the parties in all the appeals to
bear their own costs.
Order accordingly.
AIR 2008 SUPREME COURT 2670 "Syndicate Bank v. New Look Rubbers (P) Ltd."
(From : Kerala)*
Coram : 2 TARUN CHATTERJEE AND HARJIT SINGH BEDI, JJ.
Civil Appeal No.8491 of 2001, D/- 3 -4 -2008.
Syndicate Bank v. New Look Rubbers (P) Ltd. and Ors.
Civil P.C. (5 of 1908), O.21, R.90, S.115 - SALE - REVISION - EXECUTION -
Execution sale - Objection - Execution of money decree against property of M. D. of
Industrial Unit - Objection raised to sale of property of M.D. on ground of applicability
of S.22(1) of SICA and S.18 FH of Regulation Act - Revision against rejection of
objection - Revisional Court though finding that objection was not tenable setting aside
sale in interest of revival of unit - Revisional Court held exceeded its jurisdiction by
setting aside orders of civil Court which have attained finality.
C. R. P. No. 2315 of 1998 (G), D/-11-04-2001 (Ker), Reversed. (Para 7)

A.B. Dial, Sr. Advocate, Gopal Trivedi and Rajiv Nanda, for Appellant; M.C. Jose,
Respondent-in-person.
* CRP No. 2315 of 1998 (G), D/- 11-4-2001 (Ker)
Judgement
1. HARJIT SINGH BEDI, J. :- This appeal by special leave arises out of the following
facts.
2. The Syndicate Bank, the appellant herein, sanctioned various credit limits to the
respondent No. 1 company including an overdraft limit of Rs.1,00,000/- and work ing
capital term loan of Rs.1,00,000/- on certain prescribed conditions. These loans were
granted after cancellation of the earlier limits with a view to nurse respondent No.1 which
was a sick unit. In order to secure the advance, the Managing Director of respondent
No.1, that is respondent No.2 and the other Directors executed several documents as
securities and respondent No.2 also mortgaged his property to the
@page-SC2671
Bank as a collateral security. As the respondent No. 1 defaulted in the repayment of the
loan, the Bank filed a suit for recovery (O.S.No.732/1987) in the Trichur Civil Court
seeking a decree for Rs. 1,19,832.63 with interest @ 12.5% per annum in the Term Loan
Account and Rs.2,09,120.75 in the Overdraft Account with interest @ 16% per annum
compounded quarterly. The following issues were framed in the suit :
1) What is the correct amount that is due to the Plaintiff?
2) Whether the defendants are entitled to the benefit of the direction given by the Reserve
Bank of India on sick units?
3) What is the correct rate of interest?
4) Reliefs and costs.
3. The trial court observed that the Bank had not been harsh or arbitrary in dealing with
the defendants and that it appeared that the defendants were not prepared to repay the
loan despite the agreements that had been executed. It was also observed that the
defendants were not entitled to the benefit of any scheme framed for the rehabilitation of
sick units which had been formulated by the Reserve Bank of India. The suit was
accordingly decreed on 9th April 1990 and the defendants were given a year's time for
payment. No appeal was filed against this judgment with the result that it attained finality.
An application for execution of the decree (EP No. 819/1991) was filed by the Bank on
20th December 1991 and while the matter was still pending, the judgment-debtors (the
defendants in the above suit) filed civil suit (No. 1340/1993) for injunction seeking an
order prohibiting the execution of the decree rendered in O.S. No.732/1987 alleging that
it was a nullity. The Bank contested the suit which was ultimately dismissed by Judgment
and Order dated 1st March 1996 and as no appeal was filed thereagainst, this decision too
attained finality. In April 1993, the judgment-debtors also filed a Pauper Petition (No.
19/1993) in the Sub-Court, Trichur claiming damages of about Rs.30 lacs from the Bank
with 18% and 100% exgratia payment on several grounds. This suit was dismissed for
non-prosecution on 15th November 1995 and an application for its restoration was also
dismissed on 6th June 1997. As no further proceedings were taken by the judgment-
debtors, these orders attained finality as well. It appears that while this spate of litigation
was continuing, the judgment-debtor filed various objections (during the year 1994-97) in
E.P.No.819/1991 alleging that the decree was not executable, and amongst others, two
objection petitions being E.A.No. 847/1997 and 1197/1997 were filed claiming the
protection and benefits available under section 22(1) of the Sick Industrial Companies
(Special Provisions) Act, 1985 (hereinafter called "SICA") and under section 18FH of the
Industries Development and Regulation Act, 1951 (hereinafter called the "Regulation
Act") praying that the execution proceedings be kept in abeyance till such time the
proceedings initiated by the Government of Kerala for the revival of the unit were going
on. It appears that while the objections were pending, the mortgaged property of
judgment-debtor No.2 was sold in auction by the executing court for a sum of Rs.3.50
lacs which was deposited in Court and later released to the Bank and the sale was also
confirmed and possession of the property handed over to Antony the auction purchaser.
Vide order dated 21st July 1998 the execution application Nos.847/1997 and 1197/97
were dismissed. This order was challenged by the respondent Nos. 1 and 2, the original
defendants, by way of CRP No.2315/1998 alleging that the execution proceedings ought
to have been stayed pending the decision of the proceedings under section 18FH of the
Regulation Act and section 22 of SICA. The respondent No. 1 Company also filed a writ
petition on 7th April 2000 under Article 226 of the Constitution of India (No. O.P. 11862
of 2000) in the Kerala High Court against the Government of Kerala, the Kerala
Financial Corporation, the Syndicate Bank and Antony the auction purchaser alleging that
the judgment-debtors' unit had been rendered sick on account of the arbitrary actions of
the Bank and the Kerala Financial Corporation and that the proceedings in O.S. 732/1987
were without jurisdiction and, therefore, null and void and consequently the decree in EP
819/ 1991 too was not enforceable. The Civil Revision and the Civil Writ Petition were
heard together and both were allowed vide order dated 11th April, 2001 with the High
Court observing that though the benefits of section 18FH of the Regulation Act and
section 22 of the SICA were not available to the judgment-debtor but they were entitled
to succeed on other issues and concluded that :
"The petitioner has filed CRP 2315/98
@page-SC2672
challenging the common order passed by the execution court in E.A.847/97 and
E.A.1197/97 in E.P.819/91 in O.S.732/87. The petitioner was the judgment debtor in
O.S.732/87, a suit instituted by the present 3rd respondent bank for realization of the
entire arrears due from the petitioner. The decree holder filed E.P.819/91 and the property
and the residential building belonging to the Managing Director of the company placed as
security was sold in auction for an amount of Rs.3.5 lakhs. The petition filed by the
petitioner for setting aside the sale was also dismissed. Later the petitioner filed
E.A.847/97 and E.A. 1197/97 u/S. 18FH of the IDR Act Sec.22 of the SICA and Sec. 151
CPC for annulling the court sale and also for staying further execution proceedings. Both
the petitions were dismissed by the execution court. I have already found that Sec.22 of
SICA or sec.18FH of the IDR Act have any application in the present case and as such the
above order dismissing E.A.847/ 97 and E.A. 1197/97 has only to be upheld in the
ordinary course. As the suit was instituted and proceeded in collusion with the KFC and
as both the above financial institutions have agreed before the 1st respondent for the
revival of the industry granting concessions and packages (as evident from Ext.P.24), I
think it just and proper to set aside the sale of the property of the Managing Director of
the company in execution of the decree in O.S.732/87. In fact the KFC was strangulating
the industry from one side where as the Bank was doing the same thing on the Managing
Director of the company from the other side which was shocking to judicial conscience.
In the above circumstances for the ends of justice this court is constrained to interfere and
to undo the injustice caused to the petitioner and to save both the industry and the
Managing Director of the industry by setting aside the order of taking over the industry
and the court sale of the property. The entire amount deposited by the 4th respondent
towards price of the auctioned property with 5% of the above amount and interest at 6%
from the date of deposit till return shall be paid by the 3rd respondent bank to the 4th
respondent. The bank also will have to consider the grant of interest holiday for the
period during which the industry had been under the possession of the KFC in pursuance
to the take over. Hence for the proper administration of justice, I think it proper to set
aside the court sale and to allow C.R.P.2315/ 98."
4. As the order dated 11th April 2001 was a comprehensive one, taking within its ambit
the Civil Revision as well as the Writ Petition, two LPAs, one by the Bank and the other
by the judgment-debtor, were filed and are pending in the Kerala High Court against the
order in the writ petition whereas the present appeal has been filed by the Bank against
the order in the Civil Revision.
5. Mr. A.B.Dial, the learned senior counsel for the appellant has pointed out that the High
Court had overstepped its jurisdiction as after giving a positive finding that the
respondent was not entitled to any relief under section 22 of the SICA or under section
18FH of the Regulation Act, it had virtually set aside all the effective orders of the Civil
Courts which had been made in favour of the appellant Bank and had attained finality. It
has further been pleaded that the further direction of the High Court as to the entitlement
of the respondent to the benefit of the re-settlement schemes for revival was not the issue
before it as the executing court or the High Court could not go beyond the decree itself
and hold that the suit which had led the decree was bad having been filed in collusion
with one or the other party. Respondent No.2 appearing in person has, however, argued
that the High Court had proceeded on the basis that the respondent had been gravely
wronged by the Bank and the Kerala Financial Corporation and has also submitted a large
number of documents in support of this submission. He has also pointed out that the State
Government had taken steps towards the revival of his unit and as such, there was no
justification in interfering with the order of the High Court.
6. Before we go to the issues raised, we reproduce here the final directions of the High
Court :
"1. The order of the 2nd respondent KFC taking possession of the industrial unit New
Look Rubbers (P) Ltd. (petitioner company) is set aside and the KFC is directed to
surrender possession of the industrial unit to the petitioner forthwith.
2. The 2nd respondent (KFC) shall grant interest holiday for the period from the date of
taking possession of the industry till it is handed over to the petitioner.
3. The court sale of the property of the
@page-SC2673
Managing Director of the petitioner company in pursuance to the decree in OS 732/87 of
Sub-Court, Trichur, shall stand set aside and the 3rd respondent shall return the entire sale
amount with a sum equal to 5% of the above amount and with 6% interest on the sale
amount from the date of deposit till the date of return to the 4th respondent."
7. We are of the opinion that the High Court has clearly overstepped its jurisdiction. The
facts of the case show that the appellant had filed civil suit O.S. No.732/ 1987 against the
respondent for recovery of the sums advanced as loan plus interest. This suit was decreed
on 9th April 1990 with the positive finding that there was no harsh or arbitrary behaviour
in the proceedings against the respondent. Admittedly, no appeal was filed against the
judgment dated 9th April 1990 and it attained finality. The appellant Bank thereafter filed
E.P. No.819/ 1991 for the realization of the decretal amount on which the respondent
filed O.S. No. 1430/93 praying that the appellant Bank be injuncted from executing the
decree in O.S. No.732/1997. This suit too was dismissed on 1st April 1996. Another
Pauper Petition No. 19/1997 was filed by the respondent claiming damages from the
Bank and this too was dismissed. Admittedly all these matters have attained finality. Two
sets of objections in execution petition, E.A. No. 847/97 and E.A. No. 1197/97 had been
filed with regard to the applicability of section 22(1) of SICA and section 18 FH of the
Regulation Act and in the meanwhile, the property mortgaged was sold in execution of
the decree. The aforesaid objection petitions were dismissed on 21st July 1998. It is
against the order in E.A. No. 1197/1997 that the present revision was filed. We are
therefore of the opinion that in the light of the issues decided by the Civil Court in the
various litigations which were allowed to become final and as the decree had already
been executed in as much that the mortgaged property has been sold, the directions issued
by the High Court are clearly not warranted. We are also of the opinion that there can be
no objection to the revival of the respondent's unit but that is a matter between the
respondent and the Kerala Financial Corporation or other Governmental Agencies and
cannot in any manner affect the legal rights that have accrued to the appellant as a
consequence of a series of orders/judgments. We accordingly allow this appeal, set aside
the order of the High Court in so far as the decision in the Civil Revision is concerned,
but with the rider that should the Government or its agencies choose to provide any
succour to the respondent they would be at liberty to do so, but without in any manner
affecting the rights or interest of the appellant Bank. There will be no order as to costs.
Appeal allowed.
AIR 2008 SUPREME COURT 2673 "Union Public Service Commission v. A. K. Salim"
(From : Kerala)
Coram : 2 TARUN CHATTERJEE AND J. M. PANCHAL, JJ.
Civil Appeal No. 3648 of 2008 (arising out of SLP (C) No. 21416 of 2007), D/- 16 -5
-2008.
Union Public Service Commission and Anr. v. A.K. Salim and Ors.
Constitution of India, Art.16 - Indian Forest Services (Appointment by Promotion)
Regulations (1966), Regn.1 - EQUALITY IN PUBLIC EMPLOYMENT - FOREST -
APPOINTMENT - ADMINISTRATIVE TRIBUNAL - APPEAL - Appointment to Indian
Forest Service (IFS) - Respondent, Asstt. Conservator of Forest - Included in Select List -
Non-consideration for appointment due to lack of proposal from State Government to fill
vacancy - Administrative Tribunal directing UPSC to hold selection meeting before
respondent retires - Meeting, however, not convened - Order passed in contempt petition
that UPSC should hold selection meeting without waiting for further proposals - Appeal
against - Respondent accepting proposal of UPSC that he may be considered along with
others - Appeal dismissed in view of consensus - Direction issued that if respondent is
selected respondent though has retired would be given all monetary benefits. (Paras
10, 11)

Ms. Binu Tamta, for Appellants; D.S. Mahra, Dr. Sumant Bharadwaj, Ms. Mridula Ray
Bharadwaj and Ms. P. Gupta, for Respondents.
Judgement
1. J. M. PANCHAL, J. :-Leave granted.
2. The instant appeal is directed against judgment dated October 18, 2007 rendered by the
High Court of Kerala at Ernakulam in W.P.(C) No. 30695 of 2007 by which direction
dated September 11, 2007, given by the Central Administrative Tribunal, Ernakulam
Bench in M.A. No. 649 of 2007,
@page-SC2674
which was filed in O.A. No. 26 of 2007, given to UPSC, New Delhi to convene the
Selection Committee meeting based on the proposal already received from the State
Government vide letters dated May 10, 2007 and June 15, 2007 for considering the case
of the applicant for promotion to the Indian Forest Service, without waiting for further
proposals in respect of additional vacancies, is upheld.
3. The respondent No. 1, i.e., Mr. A.K. Salim, was appointed as a Forest Range Officer on
November 1, 1977. He was thereafter promoted as Assistant Conservator of Forest on
January 5, 1995. He was also promoted as Deputy Conservator of Forest (non-cadre). The
appointment to Indian Forest Service is governed by the provisions of Indian Forest
Services (Appointment by Promotion) Regulations, 1966. The respondent No. 1 was
confirmed in the cadre post of Assistant Conservator of Forest with effect from July 1,
2001. In the seniority list of Assistant Conservator of Forest he was placed at serial No.
24 and was also granted integrity certificate for consideration of his appointment to
Indian Forest Service. The name of the respondent No. 1 was included in the select list
prepared in terms of the Regulations of 1966 for the years 2004 and 2005. Since the
respondent No. 1 was confirmed in the cadre post of Assistant Conservator of Forest with
effect from July 1, 2001 and had completed eight years of service in the year 2003, his
name was considered and included for the four vacancies that arose during the year 2004.
Similarly his name for promotion to Indian Forest Service was included in the year 2005
for the two vacancies which had arisen in that year. However, he was found to be
ineligible for being considered for the vacancies for the years 2004 and 2005 since he had
not completed the prescribed eight years' service. Likewise, he was found ineligible for
being considered for the sole vacancy of the year 2006. The Selection Committee for
Indian Forest Service met on December 22, 2006 and selected candidates for eight
vacancies for the years 2003, 2004 and 2005. However, the Selection Committee did not
consider filling up the sole vacancy of the year 2006. The reason for not considering
filling up of the said vacancy was lack of proposal from the State Government and the
Principal Secretary Forest, Government of Kerala. The respondent No. 1 made
representations for his promotion to Indian Forest Service but of no avail. He, therefore,
moved O.A. No. 26 of 2007 before the Central Administrative Tribunal, Ernakulam
Bench and prayed to direct the appellants to convene meeting of the Selection Committee
for considering his case for selection and appointment to Indian Forest Service cadre
against the substantive vacancies available as on January 1, 2006.
4. On service of notice, the appellants contested the claim of the respondent No. 1 by
filing reply. The Central Administrative Tribunal, Ernakulam Bench, by judgment dated
March 9, 2007, directed the State of Kerala and the Principal Secretary Forests,
Government of Kerala, to submit the consolidated proposal for considering the selection
to Indian Forest Service for the year 2006 to the UPSC within a period of three weeks
from the date of receipt of copy of the said order. The Tribunal further directed that on
receipt of such proposal, the UPSC, New Delhi and the Selection Committee for selection
to Indian Forest Service shall convene a meeting well before the date of retirement of the
respondent No. 1, which was May 31, 2007 in accordance with the Rules.

5. The record indicates that the above mentioned directions, were not complied with by
the appellants. Therefore, the respondent No. 1 moved M.A. No. 649 of 2007 in O.A. No.
26 of 2007 and prayed to initiate contempt proceedings against the appellants for non-
compliance of directions dated March 9, 2007 given by the Central Administrative
Tribunal, Ernakulam Bench in O.A. No. 26 of 2007.
6. The Tribunal issued notice to the appellants. On receipt of the notice, the appellants
filed their reply. Meanwhile, proposals were received from Principal Secretary Forests,
Government of Kerala and the State of Kerala. After hearing the learned counsel for the
parties, the Tribunal, by judgment dated September 11, 2007, directed the UPSC, New
Delhi to convene the Selection Committee meeting based on the proposals already
received from the State Government by letters dated May 10, 2007 and June 15, 2007 for
considering the case of respondent No. 1 for promotion to Indian Forest Service without
waiting for the further proposals in respect of additional vacancies which had arisen and
disposed of M.A. No. 649 of 2007.
7. Feeling aggrieved the appellants invoked
@page-SC2675
extraordinary jurisdiction of High Court of Kerala at Ernakulam under Article 226 of the
Constitution by filing W.P. (C) No. 30695 of 2007 and prayed to set aside the directions
given by the Tribunal in M.A. No. 649 of 2007, which was filed in O.A. No. 26 of 2007.
8. The High Court of Kerala at Ernakulam has dismissed the petition filed by the
appellants vide judgment dated October 18, 2007, giving rise to the instant appeal.
9. This Court has heard the learned counsel for the parties at length and in great detail.
This Court has also considered the documents forming part of the Instant appeal.
10. One of the grounds of challenge urged in the memorandum of special leave petition is
that no harm or prejudice would be caused to the respondent No. 1, i.e., Mr. A.K. Salim if
the select list is prepared for all the three vacancies because according to the appellants,
the Tribunal has already directed to regularize services of the respondent No. 1 till the
date of consideration of his case For promotion to Indian Forest Service by the Selection
Committee, if he is ultimately found to be eligible though he has already retired on May
31, 2007 and, therefore, the judgment of the High Court, impugned in appeal, should be
set aside. On service of notice, the respondent No. 1 has filed counter affidavit on January
11, 2008 wherein he has stated that he has no objection if selection proceedings for the
three vacancies for the year 2006 are initiated. During the course of hearing of instant
appeal Mr. Ranjit Kumar, learned senior counsel for the appellants as well as Dr. Sumant
Bhardwaj, learned advocate for the respondent No. 1 and Mr. B. Dutta, learned
Additional Solicitor General for Government of India, have stated at the Bar that the
instant appeal may be disposed of by directing the appellant No. 2, i.e., the Selection
Committee through UPSC to convene the meeting for considering the case of respondent
No. 1 and others for the three vacancies which had arisen in the year 2006 and that the
case of the respondent No. 1 be considered in the said meeting.
11. In view of the consensus arrived at between the parties, the appellant No. 2, i.e., the
Selection Committee through UPSC, is directed to convene the meeting for considering
the case of the respondent No. 1 and other eligible candidates for promotion to Indian
Forest Services within two months from today. In case the respondent No. 1 is selected
for induction into Indian Forest Service, the intervening period shall be regularized
notionally with effect from June 1, 2007 and the respondent No. 1 shall be accorded all
benefits including monetary benefits. The appeal is allowed only to the extent indicated
hereinabove.
12. There shall be no orders as to costs.
Order accordingly.
AIR 2008 SUPREME COURT 2675 "Vimalben Ajitbhai Patel v. Vatslaben Ashokbhai
Patel"
(From : Gujarat)*
Coram : 2 S. B. SINHA AND V. S. SIRPURKAR, JJ.
Civil Appeal Nos.2003 of 2008 with Cri. A. No.502 of 2008 (arising out of SLP (C) No.
1061 of 2007 with SLP (Crl.) No. 213 of 2007), D/- 14 -3 -2008.
Vimalben Ajitbhai Patel v. Vatslaben Ashokbhai Patel and Ors.
(A) Hindu Adoptions and Maintenance Act (78 of 1956), S.18 - MAINTENANCE -
Maintenance of wife - Personal obligation of husband - Such obligation cannot be met
from property of mother-in-law.
Obligation to pay maintenance to wife, during subsistence of marriage, is on the husband.
It is a personal obligation. The obligation to maintain a daughter-in-law arises only when
the husband has died. Such an obligation can also be met from the properties of which the
husband is a co-sharer and not otherwise. For invoking the said provision, the husband
must have a share in the property. The property in the name of the mother-in-law can
neither be a subject matter of attachment nor during the lifetime of the husband, his
personal liability to maintain his wife can be directed to be enforced against such
property. Sections 18 and 19 prescribe the statutory liabilities in regard to maintenance of
wife by her husband and only on his death upon the father-in-law. Mother-in-law, thus,
cannot be fastened with any legal liability to maintain her daughter-in-law from her own
property or otherwise.
AIR 1977 SC 1944, Disting. (Paras 21, 24)
(B) Criminal P.C. (2 of 1974), S.82, S.85 - ATTACHMENT - TENANCY - Absconding
accused - Attachment
@page-SC2676
of property - Property attached cannot be sold - Has no effect on right of tenant.
Section 82 was enacted to secure the presence of the accused. Once the said purpose is
achieved, the attachment shall be withdrawn. Even the property which was attached,
should be restored. The provisions of Criminal P.C. do not warrant sale of the property
despite the fact that the absconding accused had surrendered and obtained bail. Once he
surrenders before the Court and the Standing Warrants cancelled, he is no longer an
absconder. The purpose of attaching the property comes to an end. It is to be released
subject to the provisions of the Code. If the property is to be sold, it vests with the State
subject to any order passed under Section 85 of the Code. It cannot be a subject matter of
execution of a decree, far less for executing the decree of a third party, who had no right,
title or interest thereon. An order of attachment of a property has nothing to do with the
right of tenancy. The terms and conditions of tenancy, being governed by statute, the
tenant cannot be evicted except in accordance with law. (Paras 32, 33)
(C) Criminal P.C. (2 of 1974), S.439(2) - BAIL - Bail - Cancellation - Ailing in-laws of
complainant granted bail on conditions - Conditions complied with - Cancellation of, on
mis-statement by complainant that passports had not been surrendered - Improper -
Consideration of purported suffering of complainant - Was irrelevant - Sympathy or
sentiment should not allow Court to have any effect in its decision making process.
(Paras 38, 41, 47)
(D) Constitution of India, Art.21, S.300A - RIGHT TO LIFE - RIGHT TO PROPERTY -
Human rights - Right to property - Facet of human right - For deprivation of property
procedure prescribed by law ought scrupulously to be followed. (Para 42)
(E) Constitution of India, Art.133. S.134 - APPEAL - PRACTICE AND PROCEDURE -
Practice and procedure - Arguments complete - Counsel seeking to withdraw from case
thereafter - And for allowing party to argue in person - Not expected from a counsel
practising in Supreme Court or from a party, who herself is an Advocate - Practice
deprecated. (Para 49)
Cases Referred : Chronological Paras
2007 AIR SCW 1088 : AIR 2007 SC 1118 (Rel. on) 28
2007 AIR SCW 6679 : AIR 2008 SC 251 : 2008 (1) ALJ 40 (Ref.) 40
2007 AIR SCW 6692 : AIR 2008 SC 261 (Rel. on) 42
2006 AIR SCW 1991 : AIR 2006 SC 1806 : 2006 (3) AIR Kar R 320 (Ref.) 47
2005 AIR SCW 323 : AIR 2005 SC 716 : 2005 Cri LJ 883 (Ref.) 39
2000 AIR SCW 1518 : AIR 2000 SC 1908 (Rel. on, Pnt. A) 26
(19961) 4 SCC 479 (Rel. on, Pnt. A) 25
AIR 1987 SC 149 : 1987 Cri LJ 157 (Ref.) 45
AIR 1984 SC 372 : 1984 Cri LJ 160 (Ref.) 40
AIR 1984 SC 1767 (Disting.) 43
AIR 1982 SC 1555 : 1982 Lab IC 1793 (Disting.) 47
AIR 1978 SC 179 : 1978 Cri LJ 129 (Rel. on, Pnt. C) 39
AIR 1977 SC 1944 (Disting. Pnt. A) 29
AIR 1927 Mad 1187 25
Rohit M. Alex, P. S. Sudheer and Ms. Anne Mathew, for Appellant; Mayur Shah, Ms.
Sonal Patel, Shankar Divate, Nikhil Goel, Mrs. Sheela Goel, Ms. Hemantika Wahi, Ms.
Shivangi and Ms. Sangeeta Singh, for Respondents.
* L.P.A. No. 1792 of 2005 in Spl. Civil Appln. No.15377 of 2004, D/- 21-11-2006 (Guj).
Judgement
1. S. B. SINHA, J. :-Leave granted in both the matters.
2. These two appeals being inter related were taken up together for hearing and are being
disposed of by the common judgment.
3. Vimlaben Ajitbhai Patel (Appellant in Civil Appeal is the mother-in-law of Sonalben
Rameshchandra Desai-respondent No. 3 in Civil Appeal and respondent No. 2 in
Criminal Appeal) while she along with her husband are the appellants in the Criminal
Appeal. For the purpose of disposal of these appeals, Vimalben Ajitbhai Patel is being
described as Appellant No. 1 while her husband Ajitbhai Revandas Patel is being
described as Appellant No. 2.
4. Sonalben Rameshchandra Desai was married to Jitendra Ajitbhai Patel (son of the
appellants) on 4th May, 1992. The couple fell apart. In 1993 a complaint petition was
filed by the 3rd respondent against her husband and the appellants alleging commission
of an offence under Sections 406 and 114 of the Indian Penal Code. In the said complaint
the 3rd respondent accepted that
@page-SC2677
her husband had incurred huge losses in the business in United States. Appellants were
granted bail subject to the condition that they would not leave India without prior
permission of the Court. Allegedly oh the premise that Appellant No.2 requires medical
treatment, an application for permission was filed in October, 1997 but they left India
without obtaining the same from the Court.
5. An application was filed for cancellation of the bail which was rejected by the
Metropolitan Magistrate as also by the Sessions Judge. The 3rd respondent filed an
application before the High Court being Special Criminal Application No. 1360 of 1997.
The said application was allowed by the High Court by its order dated 18th November,
1997 cancelling the bail of the appellants. The learned Metropolitan Magistrate was
directed to issue Standing Warrant of arrest against the appellants as and when they
returned to India.
6. On an application filed by the 3rd respondent on 24th April, 1998 the husband of the
appellant was declared an absconder and a public proclamation was issued in terms of
Section 82(2) of the Code of Criminal Procedure attaching her properties if she did not
present before the Learned Magistrate within 30 days from the issuance of the said
publication. There is nothing on record to show that the said order was served on the
appellants. It, however, is not disputed that on their failure to remain present within a
period of 30 days their properties were subjected to order of attachment under Section 85
of the Code of Criminal Procedure. By an order dated 5th January, 2004 the District
Magistrate was asked by the Leaned Metropolitan Magistrate to take further action in
terms of Section 85 of the Code of Criminal Procedure by holding a public auction of the
said properties. In the said order it was wrongly stated that the properties belonged to the
appellants and husband of the 3rd respondent, whereas in fact Appellant No. 1 alone was
the owner thereof.
7. The 1st Respondent (tenant) and the subsequent auction purchaser filed an application
before the High Court of Gujarat which was marked as Special Civil Application No.
15377 of 2004 against the Mamlatdar. A learned Judge of the High Court by an order
dated 5th April, 2005 directed :-
"8. In view of the above, I am inclined to pass the following order :
8.1) Rule. By interim order it is directed that the Mamlatdar - Respondent No. 1 shall
proceed with the auction of the premises in question on condition that the auction which
may be held shall be subject to the further condition that -
i) the possession of the premises shall be handed over by the Mamlatdar to the auction
purchaser, after the conclusion of the proceedings as ordered hereinafter by the ULC
Authority against the petitioner as well as respondent No.3;
ii) after the auction, it would be open to the Mamlatdar to notify the said aspect regarding
the transfer effected by auction in the conspicuous part of the premises and such an
intimation may also be given to the concerned local authority;
iii) it is further directed that the Mamlatdar-Respondent No. 1 herein shall make reference
to competent Authority under ULC Act to examine the aspects as to whether the
transaction between the absconder and Respondent No.3 can be said in breach of the
condition of Scheme under Section 21 of the ULC Act and he shall also make reference
on the aspects to the competent authority under ULC Act as to whether the action of the
absconder and subsequently rectification by respondent No.3 to give the possession of the
petitioner as tenant can be said in breach of the conditions of the Scheme under Section
21 on the basis of which the premises came to be allotted to the absconder-original
allottee. Such reference shall be made within a period of two weeks from today and the
Mamlatdar shall request the concerned authority to decide the reference within a period
of three months from the date of receipt of the reference. In the event it is found by the
competent authority under ULC Act that the action of absconder of entering into
transaction with respondent No. 3 and for handing over the possession to the petitioner as
unlawful, the Mamlatdar shall be at liberty to take possession of the premises in question
from the petitioner and thereafter he shall further be at liberty to hand over the vacant
possession of the premises to the auction purchaser.
iv) It is further directed that until the aforesaid reference is made and is decided by the
Mamlatdar, the petitioner shall deposit the amount at the rate of Rs. 1,500/-
@page-SC2678
per month with the Mamlatdar without prejudice to the proceedings of the reference and
the said amount shall remain as deposited with the Mamlatdar. In the event it is found by
the competent authority under ULC Act as an outcome of the reference and the inquiry
that the possession is unlawful of the petitioner and the transaction is unlawful of the
absconder with respondent No. 2, the Mamlatdar shall be at liberty to refund the amount.
However, in the event it is found that the possession is lawful and there is no breach of
the condition of allotment as per the scheme under Section 21 of the ULC Act, the
Mamlatdar shall be at liberty to appropriate the amount in accordance with law."
v) The aforesaid exercise of holding auction shall be completed within a period of five
weeks from today.
9. The report of the proceedings and the outcome of the reference shall also be made by
the Mamlatdar to this Court."
8. However, the appellant and her husband were not made parties therein. Against the
said order, an LPA, which was marked as LPA No. 1792 of 2005, was filed by the 1st
respondent and a Division Bench of the High Court directed that the amount of rent
deposited by him with the Mamlatdar be deposited in the High Court and the 3rd
respondent will be permitted to withdraw the same without prejudice to the rights and
contentions of the parties. The said order was passed, purported to be on the premise, that
the 3rd respondent had placed reliance on an order dated 13th May, 2005 passed by
another Bench of the High Court in First Appeal No. 2626 of 2004 whereby her husband
was directed to deposit a sum of Rs. 10,000/- per month towards arrears of maintenance
and to continue to deposit the same.
9. By an order dated 25th January, 2006 the 1st respondent was asked to deposit a sum of
Rs. 4 lakhs (as he expressed his intention to purchase the said property) apart from a sum
of Rs. 10,000/- per month which was to be deposited with the Registrar by him from 10th
February, 2006. It was furthermore directed :-
"6.2 The withdrawal of Rs. 10,000/- by respondent No. 2-Sonalben Rameshchandra
Desai shall be adjusted against any amount which may be payable to her by Jitendra
Ajitbhai Patel under any orders in First Appeal No. 2626 of 2004, in any other
matrimonial proceedings or in any civil or criminal case between her, her husband and
her mother in law."
10. Appellant No. 1 made an application to get herself impleaded as a party but her
application was dismissed by the High Court on 11.9.2006. On or about 21st November,
2006 the High Court directed the first respondent to pay a sum of Rs. 17 lakhs to
respondent No.3 in regard to the auction sale of the property in question. Respondent No.
2, Mamlatdar, was also directed to execute the deed of conveyance and register the same
in the name of the 1st respondent upon full payment.
11. Appellant and her husband returned to India. They filed an application for
cancellation of the said Standing Warrants. By an order dated 27th June, 2006 the said
application was allowed directing :-
"Application is granted and warrant against both the applicant accused are ordered to be
cancelled with a fine of Rs. 3,000/-(Rupees three thousand only) each and with condition
to submit one new surety of Rs. 10,000/- (Ten thousand) and on executing the bond of
such like amount.
CONDITIONS
1. Accused shall not leave India, without prior permission of the court.
2. Accused shall surrender his passport before the court."
12. Indisputably pursuant to the said order the Passports were deposited on 28th June,
2006 by them.
13. The 3rd respondent filed an application for setting aside the said order by filing a
Criminal Miscellaneous Application before the Sessions Judge, Ahmedabad inter alia
contending that the Passports had not been deposited by the accused pursuant to the said
order of 27th June, 2006. The learned Additional Sessions Judge set aside the said order
dated 27th June, 2006 and non-bailable warrants were directed to be issued against the
appellants herein. Aggrieved thereby she filed Criminal Misc. Application No. 14340 of
2006 before the High Court on 13th December, 2006 which by reason of the impugned
judgment and order dated 27th December, 2006 has been dismissed.
14. The questions which arise for consideration are :-
(i) Whether in the facts and circumstances
@page-SC2679
of the case, the property of Appellant No. 1 could have been sold in auction? and
(ii) Whether in a case of this nature, the bail granted to the appellants should have been
directed to be cancelled?
15. Submissions of learned counsel appearing on behalf of the appellants are :
i) Having regard to the provisions of the Hindu Adoptions and Maintenance Act, 1956
duty to maintain a wife being on the husband and not on her mother-in-law, the impugned
judgments are wholly unsustainable;
ii) The property of a person who is no longer absconding, cannot be subjected to
continuous attachment or sale thereof.
iii) Appellants having surrendered their Passports and having been attending the Court
subsequently, the High Court committed a manifest error in directing cancellation of their
bail without appreciating that the factors relevant for interfering with the order granting
bail and directing cancellation of bail are distinct and different.
16. Submissions of Mr. Mayur Shah, learned counsel appearing on behalf of the 3rd
respondent, are :-
i) That her husband being the only son of his parents and the properties having been
acquired through ancestral funds and there being no assertion that the properties are self
acquired properties, she has a right of maintenance out of the Joint Family Property in
terms of Section 18 of the Hindu Adoptions and Maintenance Act.
ii) In terms of Section 84 of the Code of Criminal Procedure, keeping in view the fact that
her husband had been directed to pay maintenance @ Rs. 10,000/- per month and which
having not been paid, respondent No. 3 could have prayed for realization of the said
amount of maintenance from the sale proceeds of the auction sale.
iii) Even an offer was made that one residential property would be transferred in her
name, apparently goes to show that the properties are Joint Family Properties. She,
having been denied her right of maintenance, could initiate the proceeding before the
Metropolitan Magistrate as also before the High Court.
iv) The Metropolitan Magistrate committed a serious error in granting bail upon
cancellation of Standing Warrants as appellants have breached the conditions for grant of
bail. They had, although placed a large number of documents and in particular medical
certificates to show that they were ill, there is nothing on record to show that they were
bed ridden and not permitted to move out.
v) Their near relatives in India would be deemed to have knowledge of the pendency of
the said proceeding and in that view of the matter neither under the guise of the medical
certificates nor on the ground of age, they deserve any sympathy of the Court.
17. Mr. Nikhil Goel, learned counsel appearing on behalf of the 1st respondent (Auction
Purchaser) would submit :
i) The tenant has a right to reside in the property irrespective of the order of attachment
and the same could not have been interfered with by Mamlatdar under the orders of the
Learned Metropolitan Magistrate or the District Magistrate.
ii) The 1st respondent had deposited a sum of Rs. 10,000/- (Rupees ten thousand only)
each month for a period of ten months which have been withdrawn by the 3rd respondent.
Out of the total auction amount of Rs. 17 lakhs, the 1st respondent had deposited Rs. 4
lakhs which has been invested in a short term deposit, besides a sum of Rs. 1 lakh. He has
also deposited a further sum of Rs. 12 lakhs which sum have, however, since been
refunded. The learned counsel would contend that in this view of the matter the amount
deposited by him should be directed to be refunded with interest.
18. Sonalben Rameshchandra Desai is an Advocate. She filed a large number of cases
against her husband and in-laws. She initially filed a Complaint Petition before the
Metropolitan Magistrate, Ahmedabad, under Section 498A of the Indian Penal Code
which was registered as Case No. 1662 of 1996. It was transferred to the Court of Chief
Judicial Magistrate, Baroda. It has since been dismissed for default. She initiated another
criminal proceeding against the appellants and their family members under Sections 323,
452, 427, 504, 506 and 114 of the Indian Penal Code, the same proceeding has also been
dismissed as withdrawn. Another criminal case was initiated by her against appellant No.
2, his son and another, being Case No.47 of 1996 under Section 406, 420, 468 and 114 of
the Indian Penal Code, which is still pending. Another case, being No. 2338 of 2006 was
filed by her under Section 500 of the Indian Penal Code. Another
@page-SC2680
case under Section 406 of the Indian Penal Code being Case No. 2145 of 1993 was filed
against the appellants.
19. Before embarking on the questions of law which arise for our consideration, we may
notice some statutory provisions.
20. The matter relating to grant of maintenance are now governed by the provisions of
Hindu Adoptions and Maintenance Act, 1956. Sections 3 (b), 18 and 19 of the said Act
read as under :-
"3. (b) "Maintenance" includes -
(i) in all cases, provision for food, clothing, residence, education and medical attendance
and treatment;
Section 18 - Maintenance of wife
(1) Subject to the provisions of this section, a Hindu wife, whether married before or after
the commencement of this Act, shall be entitled to be maintained by her husband during
her life time.
Sub-section (2) of Section 18 thereof, however, lays down certain exceptions therefor.
Sub-section (3) of Section 18 reads :-
"(3) A Hindu wife shall not be entitled to separate residence and maintenance from her
husband if she is unchaste or ceases to be a Hindu by conversion to another religion."
Section 19 - Maintenance of widowed daughter-in-law
(1) A Hindu wife, whether married before or after the commencement of this Act, shall be
entitled to be maintained after the death of her husband by her father-in-law:
Provided and to the extent that she is unable to maintain herself out of her own earnings
or other property or, where she has no property of her own, is unable to obtain
maintenance -
(a) from the estate of her husband or her father or mother, or
(b) from her son or daughter, if any, or his or her estate.
(2) Any obligation under sub-section (1) shall not be enforceable if the father-in-law has
not the means to do so from any coparcenary property in his possession out of which the
daughter-in-law has not obtained any share, and any such obligation shall cease on the
remarriage of the daughter-in-law."
21. Maintenance of a married wife, during subsistence of marriage, is on the husband. It
is a personal obligation. The obligation to maintain a daughter-in-law arises only when
the husband has died. Such an obligation can also be met from the properties of which the
husband is a co-sharer and not otherwise. For invoking the said provision, the husband
must have a share in the property. The property in the name of the mother-in-law can
neither be a subject-matter of attachment nor during the life time of the husband, his
personal liability to maintain his wife can be directed to be enforced against such
property.
22. Wholly un-contentious issues have been raised before us on behalf of Sonalben
(wife). It is well settled that apparent state of affairs of state shall be taken a real state of
affairs. It is not for an owner of the property to establish that it is his self-acquired
property and the onus would be on the one, who pleads contra. Sonalben might be
entitled to maintenance from her husband. An order of maintenance might have been
passed but in view of the settled legal position, the decree, if any, must be executed
against her husband and only his properties could be attached therefor but not of her
mother-in-law.
23. Sections 4 and 28 of the Hindu Adoptions and Maintenance Act read as under :-
"4. Overriding effect of Act
Save as otherwise expressly provided in this Act, -
(a) any text, rule or interpretation of Hindu law or any custom or usage as part of that law
in force immediately before the commencement of this Act shall cease to have effect with
respect to any matter for which provision is made in this Act;
(b) any other law in force immediately before the commencement of this Act shall cease
to apply to Hindus insofar as it is inconsistent with any of the provisions contained in this
Act.
28. Effect of transfer of property on right to maintenance
Where a dependant has a right to receive maintenance out of an estate, and such estate or
any part thereof is transferred, the right to receive maintenance may be enforced against
the transferee if the transferee has notice of the right or if the transfer is gratuitous; but
not against the transferee for consideration and without notice of the right."
@page-SC2681
24. Section 4 provides for a non obstante clause. In terms of the said provision itself any
obligation on the part of in-laws in terms of any text, rule or interpretation of Hindu Law
or any custom or usage as part of law before the commencement of the Act, are no longer
valid. In view of the non obstante clause contained in Section 4, the provisions of the Act
alone are applicable. Sections 18 and 19 prescribe the statutory liabilities in regard to
maintenance of wife by her husband and only on his death upon the father-in-law,
Mother-in-law, thus, cannot be fastened with any legal liability to maintain her daughter-
in-law from her own property or otherwise.
25. In Unnamalai Ammal vs. F.W. Wilson : AIR 1927 Madras 1187 the obligation to
maintain wife by a husband has been held to be a personal obligation. This Court in
Kirtikant D. Vadodaria vs. State of Gujarat : (1996) 4 SCC 479 has held as under :-
"8. We have given serious thought and consideration to the submissions made above by
the learned counsel for the appellant and notice that Dhayalal Hirachand, the husband of
Respondent 2 Smt. Manjulaben, has been found to be a person of sufficient means and
income. It is also true that there are 5 natural born sons of Respondent 2 besides 2
daughters, who are all major. It is also a fact that Dalip one of the sons had contested the
Municipal Election and two other sons are carrying on various businesses. According to
the Law of the Land with regard to maintenance, there is an obligation of the husband to
maintain his wife which does not arise by reason of any contract express or implied but
out of jural relationship of husband and wife consequent to the performance of marriage.
Such an obligation of the husband to maintain his wife arises irrespective of the fact
whether he has or has no property, as it is considered an imperative duty and a solemn
obligation of the husband to maintain his wife."
It was, furthermore, observed :-
"Further, according to Section 20 of the Hindu Adoptions and Maintenance Act, 1956, a
Hindu is under a legal obligation to maintain his wife, minor sons, unmarried daughters
and aged or infirm parents. The obligation to maintain them is personal, legal and
absolute in character and arises from the very existence of the relationship between the
parties. But the question before us is whether a step-mother can claim maintenance from
the step-son under Section 125 of the Code. In other words, whether Section 125 of the
Code includes within its fold the step-mother also as one of the persons to claim
maintenance from her stepson."
26

. We may notice that in Balwant Kaur vs. Chanan Singh : (2000) 6 SCC 310, this Court
reiterated the said principle in the following words :- 2000 AIR SCW 1518

"21. This provision clearly indicates that if the widowed daughter-in-law is a destitute
and has no earnings of her own or other property and if she has nothing to fall back upon
for maintenance on the estate of her husband or father or mother or from the estate of her
son or daughter, if any, then she can fall back upon the estate of her father-in-law. This
provision also indicates that in case of a widowed daughter-in-law of the family if she has
no income of her own or no estate of her husband to fall back upon for maintenance, then
she can legitimately claim maintenance from her father or mother. On the facts of the
present case, therefore, it has to be held that Appellant 1, who was a destitute widowed
daughter of the testator and who was staying with him and was being maintained by him
in his lifetime, had nothing to fall back upon so far as her deceased husband's estate was
concerned and she had no estate of her own. Consequently, as per Section 19(1)(a) she
could claim maintenance from the estate of her father even during her father's lifetime.
This was a pre-existing right of the widowed daughter qua testator's estate in his own
lifetime and this right which was tried to be crystallised in the Will in her favour after his
demise fell squarely within the provisions of Section 22(2) of the Maintenance Act."
27. The Domestic Violence Act provides for a higher right in favour of a wife. She not
only acquires a right to be maintained but also thereunder acquires a right of residence.
The right of residence is a higher right. The said right as per the legislation extends to
joint properties in which the husband has a share.
28

. Interpreting the provisions of the Domestic Violence Act this Court in S.R. Batra vs.
Taruna Batra : (2007) 3 SCC 169 held that even a wife could not claim a right of
residence in the property belonging to her mother-in-law, stating : 2007 AIR SCW 1088

@page-SC2682
"17. There is no such law in India like the British Matrimonial Homes Act, 1967, and in
any case, the rights which may be available under any law can only be as against the
husband and not against the father-in-law or mother-in-law.
18. Here, the house in question belongs to the mother-in-law of Smt. Taruna Batra and it
does not belong to her husband Amit Batra. Hence, Smt Taruna Batra cannot claim any
right to live in the said house.
19. Appellant 2, the mother-in-law of Smt. Taruna Batra has stated that she had taken a
loan for acquiring the house and it is not a joint family property. We see no reason to
disbelieve this statement."
29

. Reliance placed by Mr. Goel on V. Tulasamma and others vs. Sesha Reddy (Dead) by
L.Rs. : [1977] 3 SCR 261 is wholly misplaced. The question which arose for
consideration therein was the nature of the right, a widow acquires in the property in
which she had been in possession in lieu of maintenance. Interpreting sub-section (1) of
Section 14 of the Hindu Succession Act this Court held that the term "possessed" should
receive a wide meaning. It is in this context this Court noticed the authorities from Sastric
Hindu Law whereupon our attention has been drawn :- AIR 1977 SC 1944

"Similar observations have been made by the learned author at p. 528 of the book which
may be extracted thus :
'According to both the schools, the lawfully wedded wife acquires from the moment of
her marriage a right to the property belonging to the husband at the time and also to any
property that may subsequently be acquired by him so that she becomes a co-owner of
the husband, though her right is not co-equal to that of the husband, but a subordinate one
owing to her disability founded on her status of perpetual or life long tutelage or
dependence.
This right of the wife to maintenance from her husband is not lost even if the husband
renounce Hinduism.
This right subsists even after the husband's death although her husband's right as
distinguished from hers may pass by survivorship or by succession to sons or even to
collaterals; these simply step into the position of her husband, and she is required by
Hindu law to live under their guardianship after her husband's death.'"
30. The orders passed by the High Court which are impugned before us are, thus, wholly
unsustainable. They suffer from total non-application of mind.
31. The said orders might have been passed only on consideration that Sonalben is a
harassed lady, but the fact that the appellant is also a much harassed lady was lost sight
of. She has more sinned than sinning. Appellant and her husband are old. They suffer
from various diseases. They have been able to show before the Court that they had to go
to the United States of America for obtaining medical treatment. They, We would assume,
have violated the conditions of grant of bail but the consequence therefore must be kept
confined to the four corners of the statutes.
32. The provisions contained in Section 82 of the Code of Criminal Procedure were put
on the statute book for certain purpose. It was enacted to secure the presence of the
accused. Once the said purpose is achieved, the attachment shall be withdrawn. Even the
property which was attached, should be restored. The provisions of the Code of Criminal
Procedure do not warrant sale of the property despite the fact that the absconding accused
had surrendered and obtained bail. Once he surrenders before the Court and the Standing
Warrants cancelled, he is no longer an absconder. The purpose of attaching the property
comes to an end. It is to be released subject to the provisions of the Code. Securing the
attendance of an absconding accused, is a matter between the State and the accused.
Complainant should not ordinarily derive any benefit therefrom. If the property is to be
sold, it vests with the State subject to any order passed under Section 85 of the Code. It
cannot be a subject-matter of execution of a decree, far less for executing the decree of a
third party, who had no right, title or interest thereon.
33. The learned Metropolitan Magistrate had, in his order dated 5th January, 2004
wrongly asked the District Magistrate to put the said properties on auction sale stating
that to be belonging to the appellants and their son. The Mamlatdar appears to have
exceeded his jurisdiction in trying to evict the 1st respondent. His right as a tenant could
not have been affected by reason of any order of attachment. An order of attachment of a
property has nothing to do with the right of tenancy. The terms and conditions
@page-SC2683
of tenancy, being governed by statute, the tenant cannot be evicted except in accordance
with law. It is a matter of grave concern that an independent right was also sought to be
interfered with at the instance of Sonalben.
34. Right to object in terms of Section 84 of the Code to which reliance has been placed
by Mr. Mayur Shah, could not have been invoked by the wife as she has no independent
claim over the property. The said provisions also could not have been invoked for the
purpose of execution of a decree.
35. It is in the aforementioned context that we may now consider the impugned judgment
of the High Court directing cancellation of bail of the appellants.
36. The fact that they have surrendered is not in dispute. They are of old age as also the
fact that they have been suffering from various diseases has also not been disputed.
37. The contention of Sonalben that the passports had not been deposited, appears to be
wholly incorrect. Ajitbhai Revandas Patel was the holder of U.S. Passport. The same
having expired another Passport bearing No. 217921248 was issued. It is that passport
which was deposited. This is the current Passport. Allegations that they are having other
passports and may leave the country appears to be wholly without any basis. They have
been attending the courts. The observation made by the Metropolitan Magistrate that they
had not come of their own is unfortunate. Nobody wants to come to court of law and that
too as an accused, of his own.
38. The High Court committed a manifest illegality in directing cancellation of bail in so
far as it failed to take into consideration that the factors relevant for setting aside an order
granting bail and directing cancellation of bail are wholly distinct and different. An
application for cancellation of bail must be premised on the factors envisaged under sub-
section (2) of Section 439 of the Code of Criminal procedure. The learned Metropolitan
Magistrate in passing the order dated 27th June, 2006 while granting bail took into
consideration all the relevant factors. He imposed a fine on them. Even the passports had
been surrendered. Application for cancellation of bail was filed on a mis-statement that
the passports had not been surrendered. Various contentions, as noticed hereinbefore, in
regard to purported suffering of the wife appears to have been taken into consideration
which were wholly irrelevant. We have noticed hereinbefore that such contentions have
also been raised before us not on the basis that there exists any legal principle behind the
same but as an argument of desperation.
39

. In Gurcharan Singh and others vs. State (Delhi Administration) : 1978 (2) SCR 358 this
Court held : AIR 1978 SC 179

"24. Section 439(1) Cr. P.C. of the new Code, on the other hand, confers special powers
on the High Court or the Court of Session in respect of bail. Unlike under Section 437(1)
there is no ban imposed under Section 439(1), Cr. P.C. against granting of bail by the
High Court or the Court of Session to persons accused of an offence punishable with
death or imprisonment for life. It is, however, legitimate to suppose that the High Court
or the Court of Session will be approached by an accused only after he has failed before
the Magistrate and after the investigation has progressed throwing light on the evidence
and circumstances implicating the accused. Even so, the High Court or the Court of
Session will have to exercise its judicial discretion in considering the question of granting
of bail under Section 439(1) Cr. P.C of the new Code. The overriding considerations in
granting bail to which we adverted to earlier and which are common both in the case of
Section 437(1) and Section 439(1) Cr. P.C. of the new Code are the nature and gravity of
the circumstances in which the offence is committed; the position and the status of the
accused with reference to the victim and the witnesses; the likelihood, of the accused
fleeing from justice; of repeating the offence; of jeopardising his own life being faced
with a grim prospect of possible conviction in the case; of tampering with witnesses; the
history of the case as well as of its investigation and other relevant grounds which, in
view of so many valuable factors, cannot be exhaustively set out.
25. The question of cancellation of bail under Section 439(2) Cr. P.C. of the new Code is
certainly different from admission to bail under Section 439(1) Cr. P.C. The decisions of
the various High Courts cited before us are mainly with regard to the admission to bail by
the High Court under Section 498 Cr. P.C. (old). Power of the High Court or of the
Sessions Judge to admit persons
@page-SC2684
to bail under Section 498 Cr. P.C. (old) was always held to be wide without any express
limitations in law. In considering the question of bail justice to both sides governs the
judicious exercise of the Court's judicial discretion."

[See also Bhagirath Singh s/ o. Mahipat Singh Judeja vs. State of Gujarat : [1984] 1 SCR
839 and Jayendra Saraswathi Swamigal vs. State of Tamilnadu : 2005 (2) SCC 13].
AIR 1984 SC 372
2005 AIR SCW 323

40

. We may notice that recently a Bench of this Court considered the consequence of
issuance of warrant of arrest at some length in Inder Mohan Goswami and another vs.
State of Uttaranchal and others : (2007) 12 SCALE 15. It was held :- 2007 AIR
SCW 6679

"26. Before parting with this appeal, we would like to discuss an issue which is of great
public importance, i.e. how and when warrants should be issued by the Court? It has
come to our notice that in many cases that bailable and non-bailable warrants are issued
casually and mechanically. In the instant case, the court without properly comprehending
the nature of controversy involved and without exhausting the available remedies issued
non-bailable warrants. The trial court disregard the settled legal position clearly
enumerated in the following two cases."
It was furthermore observed
"51. In complaint cases, at the first instance, the court should direct serving of the
summons along with the copy of the complaint. If the accused seem to be avoiding the
summons, the court, in the second instance should issue bailable-warrant. In the third
instance, when the court is fully satisfied that the accused is avoiding the court's
proceeding intentionally, the process of issuance of the non-bailable warrant should be
resorted to. Personal liberty is paramount, therefore, we caution courts at the first and
second instance to refrain from issuing non-bailable warrants."
41. Keeping in view the entirety of the facts and circumstances of the case we are of the
opinion that gross injustice has been caused to the appellant. She did not deserve such
harsh treatments at the hands of the High Court. Respondent No. 3 speaks of her own
human rights, forgetting the human rights of the appellant, far less the fundamental right
of life and liberty conferred on an accused in terms of Article 21 of the Constitution of
India.
42

. The right of property is no longer a fundamental right. But still it is a constitutional


right. Apart from constitutional right it is also a human right. The procedures laid down
for deprivation thereof must be scrupulously complied with [See-Devinder Singh and
Ors. vs. State of Punjab and Ors. : JT 2007 (12) SC 256]. 2007 AIR SCW 6692

43

. Last but not the least, a plea of equity has been raised by Mr. Shah stating that this Court
should issue some directions keeping in view the equitable principles. Reliance has been
placed on Chandra Bansi Singh vs. State of Bihar : (1984) 4 SCC 316, wherein it was
observed :- AIR 1984 SC 1767

"16. On an analysis of the various steps taken by the parties and others in the taking of
possession, there is undoubtedly a delay of about 1 year and for the purpose of
calculation and convenience when rounded off, the delay may be taken to be of two
years. So far as this delay is concerned, the appellants have undoubtedly a case for
payment of some additional compensation in equity though not under law and as this
Court is not only a Court of law but a Court of equity as well, it will be impossible for us
to deny this relief to the appellants. After taking into consideration the various shades and
aspects of the case we are clearly of the opinion that apart from compensation which may
be awarded by the Collector or enhanced by the Judge or a higher Court, the appellants
should get an equitable compensation in the form of interest calculated at the rate of 7 per
cent per annum for two years on the value of land owned by each land-owner. This
equitable compensation has been awarded in the special facts of this case and will not be
the subject-matter of appeal, if any, under the Act on the amount of compensation."
44. The said case arose out of a proceeding under the Land Acquisition Act which has no
relevance to the issues involved in these appeals.
45
. On cancellation of bail Mr. Shah has relied upon a decision of this Court in Raghubir
Singh vs. State of Bihar : (1986) 4 SCC 481 wherein this Court observed : AIR 1987 SC
149

"22. The result of our discussion and the


@page-SC2685
case-law is this: An order for release on bail made under the proviso to Section 167(2) is
not defeated by lapse of time, the filing of the charge-sheet or by remand to custody
under Section 309(2). The order for release on bail may however be cancelled under
Section 437(5) or Section 439(2). Generally the grounds for cancellation of bail, broadly,
are, interference or attempt to interfere with the due course of administration of justice, or
evasion or attempt to evade the course of justice, or abuse of the liberty granted to him.
The due administration of justice may be interfered with by intimidating or suborning
witnesses, by interfering with investigation, by creating or causing disappearance of
evidence etc. The course of justice may be evaded or attempted to be evaded by leaving
the country or going underground or otherwise placing himself beyond the reach of the
sureties. He may abuse the liberty granted to him by indulging in similar or other
unlawful acts. Where bail has been granted under the proviso to Section 167(2) for the
default of the prosecution in not completing the investigation in 60 days, after the defect
is cured by the filing of a charge-sheet, the prosecution may seek to have the bail
cancelled on the ground that there are reasonable grounds to believe that the accused has
committed a non-bailable offence and that it is necessary to arrest him and commit him to
custody. In the last mentioned case, one would expect very strong grounds indeed."
46. A bare perusal of the decision of this Court demonstrates that the ratio laid therein
runs counter to the submissions of the learned counsel.
47

. Reliance has also been placed on I. J. Divakar and others vs. Govt. of Andhra Pradesh
and another : (1982) 3 SCC 341. The said decision was rendered under the Industrial
Law. AIR 1982 SC 1555

Regularization was directed to be provided to the workmen. A Constitution Bench of this


Court in Secretary, State of Karnataka and others vs. Umadevi and others : (2006) 4 SCC
1 opined that all such decisions shall stand overruled. 2006 AIR SCW 1991

Sympathy or sentiment, as is well known, should not allow the Court to have any effect
in its decision making process. Sympathy or sentiment can be invoked only in favour a
person who is entitled thereto. It should never be taken into consideration as a result
whereof the other side would suffer civil or evil consequences.
48. We are at a loss to understand as to on what premise such a contention has been
raised. If we accept the contention of the learned counsel the same would mean that we
send the old couple to jail or deprive them of their lawful right of a valuable property
and/or ask them to meet obligations which statutorily are not theirs. Such a direction, in
our opinion, should also not be passed, keeping in view the conduct of the 3rd
respondent. She not only filed a large number of cases against her in-laws, some of which
have been dismissed for default or withdrawn but also have been filing applications for
cancellation of their bail on wholly wrong premise.
49. We may also notice that after the arguments were over, a strange submission was
made before us. Learned counsel for respondent No. 3 submitted that he may be
permitted to withdraw from the case and the 3rd respondent be allowed to argue in
person. Such a submission was not expected from a counsel practicing in this Court or
form a party, who herself is an Advocate. We deprecate such practice.
50. Having regard to the facts and circumstances of this case we are of the opinion that
the interest of justice shall be subserved if the impugned judgments are set aside with the
following directions :-
i) The property in question shall be released from attachment.
ii) The 3rd respondent shall refund the sum of Rs. 1 lakh to the respondent with interest
@ 6% per annum.
iii) The amount of Rs. 4 lakhs deposited by the 1st respondent shall be refunded to him
immediately with interest accrued thereon.
iv) The 3rd respondent should be entitled to pursue her remedies against her husband in
accordance with law.
v) The Learned Magistrate before whom the cases filed by the 3rd respondent are
pending should bestow serious consideration of disposing of the same, as expeditiously
as possible
vi) The 3rd respondent shall bear the costs of the appellant which is quantified at Rs.
50,000/- (Rupees fifty thousand) consolidated.
@page-SC2686
51. The appeals are allowed with the aforesaid directions. I.A. for direction Dismissed.
Order accordingly.
AIR 2008 SUPREME COURT 2686 "Venkateshappa v. State of Karnataka"
(From : Karnataka)*
Coram : 2 Dr. A. PASAYAT AND P. SATHASIVAM, JJ.
Civil Appeal No.1867 of 2008 (arising out of SLP (C) No. 25804 of 2004), D/- 10 -3
-2008.
Venkateshappa v. State of Karnataka and Ors.
Karnataka Land Reforms Act (10 of 1962), S.48A - LAND REFORMS - TRIBUNALS -
Land Tribunal - Jurisdiction - Dispute whether land in question was inam land and falls
outside applicability of Act - Tribunal passing order without deciding question of its
jurisdiction - Tribunal's order upheld by High Court - Order liable to be set aside.
W.A. No. 935 of 2004 (LR), D/-27-05-2004 (Kant.), Reversed. (Paras 9, 10)
Cases Referred : Chronological Paras
1991 AIR SCW 2932 : AIR 1992 SC 212 5
Shantha Kr. Mahale and Rajesh Mahale, for Appellant; S. N. Bhat, Sanjay R. Hegde,
Vikrant Yadav, Amit Kumar, Aral Varma and Mrs. K. Sharda Devi, for Respondents.
* W.A. No. 935 of 2004 (LR), D/- 27-5-2004 (Kar)
Judgement
Dr. ARIJIT PASAYAT, J. :- Leave granted.
2. Challenge in this appeal is to the order of the Division Bench of the Karnataka High
Court dismissing the writ appeal filed under Section 4 of the Karnataka High Court Act.
Challenge in the writ appeal was to the order passed by a learned Single Judge. The
dispute relates to applicability of the Karnataka Land Reforms Act, 1961 (in short the
'Act') in the background of Mysore (Personal and Miscellaneous) Inam Abolition Act,
1954 (in short 'Inam Act') as amended by the Karnataka Inams Abolition Laws
(Amendment) Act, 1979 (in short 'Amendment Act').
3. The factual controversy lies in a very narrow compass.
Appellant had filed the writ petition No. 32930 of 1996 which was disposed of by orders
dated August 4, 2000 and August 24, 2000. By the latter order the following directions
were given :
"Even with regard to the question as to whether the lands in question are Inam lands or
not, it is impossible for me to form a correct impression because each of the learned
Advocates is making a different statement. The Tribunal shall first ascertain whether, the
lands in question are inams lands and if the answer is in the affirmative, then the Tribunal
shall forward the records to the Special Deputy Commissioner who shall give notice to
the parties, hear them and decide the case. If however, the Tribunal does have Jurisdiction
in law to entertain the proceeding insofar as, if the lands are not inam lands then the
Tribunal shall proceed to do so."
4. It is the case of the appellant that the Land Tribunal did not consider this aspect and did
not also record any finding and came to an abrupt conclusion as follows :
"The Gattarlahally was the jodi village, after abolition, it is vest to the Government and
not a Inam land."
5. Before the learned Single Judge the specific stand relating to the jurisdiction was
disposed of with the following observations :

"After hearing the learned counsel for the parties, I have examined the correctness of the
findings and reasons recorded in the impugned order by the Land Tribunal on the
contentious points. In my considered view, none of the contentions urged in this petition
warrant interference with the impugned order for the reason that, the order passed by the
Special Deputy Commissioner under the Act of 1954 does not bind third respondent as he
was not party to the proceedings. Further, in view of sub-section (1) of Sec. 44 of Act
notwithstanding the order of the Special Deputy Commissioner, Act of 1/ 74 has come
into force, it is a tenanted land and therefore it will statutorily vests with the State
Government. Thereafter, consequences as enumerated under sub-section (2) of Sec. 44
will come into operation. Further, the contention urged that Form No. 7 application is not
maintainable as urged above in this petition are wholly untenable in law for the reason
that submissions made on behalf of third respondent is well founded in place reliance
upon the provisions of KLRF Act and also in view of Muniyellapa vs. B.M. Krishna
Murthy reported in AIR 1992 SC 205 and the same is accepted. Therefore, contention
urged on 1991 AIR SCW 2932 : AIR 1992 SC 212

@page-SC2687
behalf of petitioner in this regard placing reliance upon the decisions of this Court are
wholly untenable in law and the same is rejected. Further the reliance placed upon
Rangaiah's case is wholly inapplicable to the fact situation and is misconceived. Hence,
reliance placed upon the said Judgment are misplaced and the contention in this regard is
rejected."
6. Learned Single Judge only observed that since the respondent was not party to the
proceeding, the order passed by the Special Deputy Commissioner under the 1954 Act
was of no consequence and even otherwise the consequences as enumerated in Section
44(2) came into operation. The Division Bench did not analyse the issue in detail and
upheld the view of the learned Single Judge.
7. The specific ground has been raised in this appeal that question of jurisdiction of the
Land Tribunal to reopen a case and decide by the Special Deputy Commissioner for
Inams Abolition has not been dealt with. Reference has been made to Section 141 of the
Act.
8. Learned counsel for the respondent submitted that though it has not been specifically
dealt with, the factual scenario clearly shows that no relief has been granted to the
appellant.
9. On the earlier occasion, the learned Single Judge has specifically stated that the
question of jurisdiction of the Tribunal has to be dealt with as quoted above. This
apparently has not been done by the Land Tribunal, and learned Single Judge and the
Division Bench lost sight of these relevant aspects.
10. In the circumstances, the impugned orders of the learned Single Judge and the
Division Bench of the High Court are quashed and the matter is remitted to the learned
Single Judge to deal with the issue in accordance with law.
11. The appeal is disposed of with no order as to costs.
Appeal allowed.
AIR 2008 SUPREME COURT 2687 "State of M. P. v. Bharat Singh Bhati"
(From : Madhya Pradesh)*
Coram : 2 TARUN CHATTERJEE AND HARJIT SINGH BEDI, JJ.
Civil Appeal No.2351 of 2008 (arising out of SLP (C) No. 11968 of 2007), D/- 1 -4
-2008.
State of M. P. and Ors. v. Bharat Singh Bhati and Ors.
Constitution of India, Art.226 - WRITS - PRACTICE AND PROCEDURE - Procedure -
Opportunity to file return to petition - Not afforded to respondents - Order allowing
petition - Liable to be set aside. (Para 6)

B. S. Bantia and Vikas Upadhyay, for Appellant; Shail Kumar Dwivedi, Satish Kumar
and Ashwani Bhardwaj, for Respondents.
* MCC No. 1241 of 2005 and WA No. 80 of 2007, D/- 11-8-2006 and 25-1-2007 (MP)
respectively.
Judgement
JUDGMENT :- Leave granted.
2. In our view, this appeal can be disposed of on a very short point. The respondents,
Bharat Singh Bhati and Ors. filed a writ application before the High Court of Madhya
Pradesh at Indore claiming relief that a minimum of pay-scale from the respective dates
of their initial appointment be paid to them as had been paid to other similarly situated
persons and for other incidental reliefs.
3. This writ petition was disposed of by directing payment of the minimum pay-scale
from the respective dates of their initial appointment. From the order of the High Court, it
would be evident that the High Court had noted that return to the writ petition was filed
by the State of Madhya Pradesh, appellant herein, although, admittedly, no such return
was filed.
4. An application for review of the order of the High Court allowing the writ petition of
the respondents was filed, which was also rejected. A writ appeal against the original
order of the learned Single Judge of the High Court allowing the writ application was
also filed. The said appeal was also dismissed by a Division Bench of the High Court of
Madhya Pradesh holding that in view of the
@page-SC2688
rejection of the Review Application, the writ appeal could not be entertained.
5. Feeling aggrieved by the aforesaid orders of the learned Single Judge as well as of the
Division Bench of the High Court, this Special Leave Petition has been filed, which was
heard on grant of leave in presence of the learned counsel for the parties.
6. Having heard the learned counsel for the parties and considering the fact that return
was in fact not filed by the appellants, we are of the view that the High Court was not
justified in allowing the writ application without affording any opportunity of filing
return to the writ application filed by the respondents. In that view of the matter, we set
aside all the orders passed by the learned Single Judge in the writ petition as well as in
the Review Application and also the order passed in the writ appeal. The appellants shall
file their return to the petition within six weeks from the date of supply of a copy of this
order to the High Court and the High Court, thereafter, shall dispose of the writ
application after giving hearing to the parties within a period of three months from the
date of filing the return to the writ application by passing a reasoned and speaking order
in accordance with law.
7. Accordingly, the appeal is allowed to the extent indicated above. There will be no order
as to costs.
Appeal allowed.
AIR 2008 SUPREME COURT 2688 "Oswal Woolen Mills Ltd. v. Punjab State
Electricity Board"
Coram : 2 S. B. SINHA AND H. S. BEDI, JJ.
I.A. No. 91 In Civil Appeal No.2335 of 2006, D/- 11 -1 -2008.
Oswal Woolen Mills Ltd. v. Punjab State Electricity Board and Anr.
Electricity (Supply) Act (54 of 1948), S.48, S.49 - ELECTRICITY - APPEAL - Levy of
surcharge - Till conversion of supply line to 33KV - Cutoff D/-13-05-1992 fixed in
2006(5) Scale 21 - Clarification issued that Company is liable to pay surcharge with
effect from the date of expiry of one year from 13-05-1992 and not from 13-05-1992
itself.
Constitution of India, Art.137. (Para 3)

Mahindra Anand, Sr. Advocate, Neeraj Kumar Jain, Bharat Singh, Sanjay Singh, Sandeep
Chaturvedi and Ugra Shankar Prasad, with him for Appellant; Mrs. Ruchi Gour Narula,
Ms. Ruchi Kohli and Rajiv Nanda, for Respondents.
Judgement

ORDER :- This application has been filed for clarification of a judgment and order dated
28.04.2006 passed by a Division Bench of this Court in Civil Appeal No. 2335 of 2006.
reported in 2006 (5) Scale 21
2. The clarification has been sought for on the issue as to whether 13.05.1992 was the
cut-off date for commencement of the period of one year for converting supply from 11
KV to 33 KV or higher voltage or the surcharge was actually payable with effect from the
said date.
3. Having heard the learned counsel for the parties, we are of the opinion that the
Company is liable to pay the surcharge with effect from the date of expiry of one year
from 13.05.1992 and not from 13.05.1992 itself. The relevant paragraph of the judgment
of this Court, accordingly, may be substituted by the following :
'For the reasons aforementioned, although Mr. R.K. Jain, the learned Senior Counsel
appearing for the company, may be right in his submission that the Board has no
jurisdiction to levy surcharge after 29.01.1992, but as the said contention had not been
raised and furthermore as notice was issued by the court on a limited question, we are of
the opinion that the company is liable to pay the surcharge with effect from the date of
expiry of one year from 13.05.1992. We may furthermore notice that the actual amount of
surcharge payable from that date has already been paid by the company to the Board.
However, in view of our findings aforementioned, there cannot be any doubt that the
surcharge @ 17½% was not required to be paid in terms of the tariff notification dated
01.02.1994.'
The I.A. is disposed of accordingly.
Order accordingly.
@page-SC2689
AIR 2008 SUPREME COURT 2689 "N. Lokananandham v. Chairman, Telecom
Commission"
(From : Andhra Pradesh)*
Coram : 2 S. B. SINHA AND V. S. SIRPURKAR, JJ.
Civil Appeal No. 2896 of 2008 (arising out of SLP (C) No. 16640 of 2004), D/- 22 -4
-2008.
N. Lokananandham v. Chairman, Telecom Commission and Ors.
Administrative Tribunals Act (13 of 1985), S.14 - ADMINISTRATIVE TRIBUNAL -
TRIBUNALS - PROMOTION - ESTOPPEL - Jurisdiction of Tribunal - Departmental
test for promotion - Some questions asked in particular paper were out of syllabus -
Direction by Tribunal to award appellant minimum qualifying marks in that paper on
supposition that appellant did not prepare themselves well owing to some
misunderstanding in regard to extent of syllabus - Without jurisdiction - It there is
ambiguity in prescribed syllabus it can be decided by expert body - Further, if it is found
that any question has been put out of syllabus, only those who could not answer the same
might have been entitled to any relief - Moreso, as appellant appeared in subsequent
exam. without any demur and failed therein also.
Evidence Act (1 of 1872), S.115. (Paras 17, 18, 19)
Cases Referred : Chronological Paras
2006 AIR SCW 4703 : 2006 (6) AIR Bom R 132 : 2006 (6) AIR Kar R 96 (Disting.)
14
(2005) 11 SCC 192 (Disting.) 14
AIR 1983 SC 1230 : 1983 All LJ 1220 (Disting.) 12
M. N. Rao, Sr. Advocate, T. N. Rao, Ms. Manjeet Kirpal and Paramjeet Singh, for
Appellant; Ms. Shalini Kumar and Ms. Neeru Vaid, for Respondents.
* W. P. No. 20914 of 2004, D/- 16-6-2006 (AP)
Judgement
1. S. B. SINHA, J. :-Leave granted.
2. Appellant was at all material times a Junior Accounts Officer. He had passed Junior
Accounts Officer Part-I examination held in 1998. For further promotion, he was required
to appear in Junior Accounts Officer Part-II examination. It was held in December, 2000.
The syllabus for the said examination consisted of nine papers.
Paper IX thereof was a theory paper with the following syllabus :
"1. P. and T FHB Volume I (General Principles and cash)
2. P. and T FHB Volume III (Parts I, II and III)
3. P. and T FHB Volume IV
4. P. and T Manual Vol-X
5. P. and T Manual Vol-XTV
6. Telecom Accounts Manual (Chapters 1 to 7 and 11)
7. Books of Accounts Officer Forms, Vol. I
8. Book of P and T Accounts Forms"
3. Allegedly, questions in respect of 65 out of 100 marks were framed out of the
prescribed syllabus. It is stated that question Nos. 1(b), 2(a), 2(b), 3 and 5 of paper IX
were covered in F.B.H. Vo1-1 under different chapters which were not prescribed in the
syllabus.
Our attention in this behalf has been drawn to the following chart :

"Question No. Chapter No. /Name Rules Marks Allotted

1(b) XI-Contingent charges 344 5


2(a) XIII-Loans and Advances to Govt. servants GID 10
under
Rule 410
2(b) XIII-Loans and Advances to Govt. servants 424 10
3. XIII-Loans and Advances to Govt. servants GID 20
(GID-1 payment of Advance to contractors) under
(GID-2 Advance payment to private Firms for supply of stores/Services and
Maintenance of Machines) Rule 390
5. VII-Revenue and Miscellaneous Receipts
Under D.G.Orders 20

Total Marks out of Syllabus 65"


4. Appellant did not pass the said examination. He made a representation for declaration
of his results by a letter dated 25.9.2002.
It is stated that the Government issued directions pursuant whereto they became entitled
to obtain six grace marks in any one of the subjects for being declared qualified in JAO-
Part II examination. Despite grant of grace marks, he did not qualify.
5. Appellant filed an original application before the Central Administrative Tribunal
questioning the right of the respondents to prescribe questions in paper IX of JAP Part II
examination 2000 out of the syllabus. A direction was sought for against the respondents
to award minimum qualifying marks in that paper.
By reason of a judgment and order dated 23.4.2004, the Tribunal allowed the said
application directing :
"Now since sufficient time has passed after the examination held in December 2000 and
we are told by the ld. Counsel for the respondents that another examination has already
taken place for JAO Part-II, in which all the applicants have appeared for all the papers
including Paper-IX, it would not be advisable to hold any re-examination in Paper-IX at
this stage. However, in the interest of justice, it would be proper to award minimum
qualifying marks to each of the seven applicants, namely, 33 marks in Paper-IX. Since as
per rules, 33% marks are to be obtained by the applicants in each of
@page-SC2690
the papers and 35 per cent marks in the aggregate, the result would be her only 3
candidates, namely, N. Lokanadham, K. Subrahamanyeswara Rao and I. Lakshmi would
pass all the papers, after being given minimum pass marks in paper-IX and they will clear
the JAO Part-II Examination, but the remaining four applicants of the two OAs who are
short of the minimum qualifying marks in other papers besides Paper-IX, would as a
result not get the benefit of passing the JAO Part-II Examination, after being given the
minimum pass marks in Paper-IX and they will have to appear again for the JAO Part-II
Examination in future to clear all the papers.
In the result, both the OAs stand disposed of with the direction to the respondents to
award all the seven applicants the minimum qualifying marks in Paper-IX i.e. 33 per cent
marks within two weeks from the date of receipt of a copy of this order and to declare
their results accordingly."
6. Respondents sought extension of the said time which by an order dated 8.7.2004 was
allowed.
Respondent filed a writ petition before the High Court of Judicature of Andhra Pradesh at
Hyderabad questioning the said order dated 23.4.2006. By reason of the impugned order
dated 16.6.2006, the said writ petition has been allowed.
7. Mr. M.N. Rao, learned senior counsel appearing on behalf of the appellant, drew our
attention to the syllabus contending that the first paper of Paper-IX being P and T FHB
Volume I comprises of General Principles and Cash and, according to the learned
counsel, General Principles are dealt with in Chapter 2 and the syllabus, relating to cash
is dealt with in Chapter 5 and in that view of the matter, no question out of the
aforementioned syllabus should have been framed.
8. Ms. Shalini Kumar, learned counsel appearing on behalf of the respondents, on the
other hand, would submit that the words 'General Principles and Cash' must be
understood in its etymological sense and not in the context of their contents as contained
in Chapter 2 and Chapter 5 thereof. It was pointed out that wherever the State intended to
prescribe a particular Chapter, the same had categorically been mentioned in the syllabus.
9. Our attention in this behalf has been drawn to Item No.6 of the syllabus. The syllabus
contains reference to the books which includes Financial Handbook. It may be true that
the words 'General Principles and Cash' have been mentioned but it might not be intended
to be kept confined to Chapter 2 and Chapter 5 of the said manual, as no other question
was to be framed from the said volume.
10. The core question, however, would be as to whether the Tribunal had the jurisdiction
to issue the impugned directions. The Tribunal, in terms of the provisions of the
Administrative Tribunals Act, 1985 exercises a limited jurisdiction. Indisputably, a
candidate, in order to qualify in the JAO Part II
@page-SC2691
examination is required not only to secure 33 per cent marks in each paper but also to
secure 35 per cent marks in aggregate. Rules of examination have been prescribed in
terms whereof only a candidate who secures 60 per cent of the marks in any of the papers
would be exempted from appearing in that particular paper in the subsequent
examinations.
The marks obtained by the appellant are as under :

"P-VII P-VIII P-IX P-X P-XI


1. N. Lokandaham Out of 100 100 100 100 100
35 56 20 35 98"

11. Respondents contend that the duties of the Accounts Officer have been laid down in
Rule 17 of Part-I of Financial Handbook Volume III which reads as under :
"17. The functions of Accounts Officers are threefold :
(a) For the correct compliance of the Accounts of the Division in accordance with the
prescribed rules.
(b) For applying preliminary checks to initial accounts, vouchers etc., and pre check of
claims.
(c) To render general assistance and advice to the Divisional Engineer in all matters
relating to accounts and budget estimates or to the operation of financial rules."
12

. Reliance has been placed by Mr. Rao on Kanpur University, Through Vice-Chancellor
and Ors. v. Samir Gupta and Ors. ((1983) 3 SCC 309], wherein multiple choice objective
type test was conducted. Key-answers had been supplied by the paper-setter. It is in that
connection this Court opined : AIR 1983 SC 1230
Paras 16 and 17

"We agree that the key answer should be assumed to be correct unless it is proved to be
wrong and that it should not be held to be wrong by an inferential process of reasoning or
by a process of rationalization. It must be clearly demonstrated to be wrong, that is to say,
it must be such as no reasonable body of men well-versed in the particular subject would
regard as correct. The contention of the University is falsified in this case by a large
number of acknowledged text-books which are commonly read by students in U.P. Those
text-books leave no room for doubt that the answer given by the students is correct and
the key answer is incorrect."
It was held :
"If this were a case of doubt, we would have unquestionably preferred the key answer.
But if the matter is beyond the realm of doubt, it would be unfair to penalize the students
for not giving an answer which accords with the key answer, that is to say, with an
answer which is demonstrated to be wrong."
13. This decision itself demonstrates to show the limited jurisdiction of the Superior
Court. It does not advance the case of the appellant.
14. P. K. Velson and Ors. v. Union of India and Ors. [(2005) 11 SCC 192] was rendered in
the peculiar facts and circumstances of the case. It was a case where several objective
type questions were asked in the screening test which were confusing and were not
according to the model question paper distributed earlier to them.
For the self same reasons, the said decision also does not render any assistance to us.

Manish Ujwal and Ors. v. Maharishi Dayanand Saraswati University and Ors. [(2005) 13
SCC 744] has been relied upon by Mr. Rao to contend that the Central Administrative
Tribunal has the requisite jurisdiction to issue a direction for consideration of the
representation of the aggrieved persons. 2006 AIR SCW 4703

That case was one of those where again the key answers were provided.
15. The Court arrived at a finding that the key answers were palpably and demonstrably
erroneous. It was in that context stated :
"For the present, we say no more because there is nothing on record as to how this error
crept up in giving the erroneous key answers and who was negligent. At the same time,
however, it is necessary to note that the University and those who prepare the key
answers have to be very careful and abundant caution is necessary in these matters for
more than one reason. We mention few of those; first and paramount reason being the
welfare of the student as a
@page-SC2692
wrong key answer can result in the merit being made a casualty. One can well understand
the predicament of a young student at the threshold of his or her career if despite giving
correct answer, the student suffers as a result of wrong and demonstrably erroneous key
answers; the second reason is that the courts are slow in interfering in educational matters
which, in turn, casts a higher responsibility on the University while preparing the key
answers; and thirdly, in cases of doubt, the benefit goes in favour of the University and
not in favour of the students. If this attitude of casual approach in providing key answers
is adopted by the persons concerned, directions may have to be issued for taking
appropriate action, including disciplinary action, against those responsible for wrong and
demonstrably erroneous key answers, but we refrain from issuing such directions in the
present case."
16. We are herein dealing with the cases of experienced employees. They knew the
significance of the departmental examination. They were aware what was expected of
them. They knew the purport and object of holding departmental tests. It is furthermore
not a case where key answers had been provided which were found to be palpably wrong.
17. For performing their functions in a more responsible position, they were required to
be thorough with the rules pertaining to all receipts and expenditure of the Department
and it is in that sense, the First Volume of Financial Handbook deals with such
transactions in general. Indisputably, the cadre of Junior Accounts Officer is the feeder
post for promotion to the cadre of Assistant Accounts Officer, Accounts Officer and
Senior Accounts Officers. Officers belonging to the said cadre indisputably must have
sufficient knowledge as regards the procedure pertaining to the Account Code, Treasury
Rules, Financial Rules etc. Furthermore, assuming there was some ambiguity in the
prescribed slabs, it would have been for an expert body to clear the same and in the event
it is found that any question has been put out of syllabus, only those who could not
answer the same might have been entitled to any relief. No relief in a case of this nature
could have been granted on assumptions. It was not for the Tribunal to pass an order only
on the supposition that the appellant did not prepare themselves well owing to some
misunderstanding in regard to the extent of syllabus.
18. We may furthermore notice that the appellant herein without any demur whatsoever
appeared in the subsequent examination. He even did not qualify therein. The principle of
estoppel would, therefore, apply in this case.
Tribunal had, thus, exceeded to its jurisdiction in passing its order dated 23.4.2004.
19. There is, therefore, no merit in the appeal. It is dismissed accordingly with no orders
as to costs.
Appeal dismissed.
AIR 2008 SUPREME COURT 2692 "State of Karnataka v. Chikkahottappa"
(From : 2000 (1) Kant LJ 17)
Coram : 3 Dr. A. PASAYAT, P. SATHASIVAM AND AFTAB ALAM, JJ.
Criminal Appeal No. 313 of 2001, D/- 16 -5 -2008.
State of Karnataka v. Chikkahottappa @ Varade Gowda and Ors.
(A) Penal Code (45 of 1860), S.149 - UNLAWFUL ASSEMBLY - COMMON OBJECT -
Unlawful assembly - Common object - Provision has its foundation on constructive
liability which is sine qua non for its operation - Emphasis is on common object and not
on common intention. (Para 6)
(B) Penal Code (45 of 1860), S.149 - UNLAWFUL ASSEMBLY - COMMON OBJECT -
Unlawful assembly - Mere presence in unlawful assembly cannot render a person liable
unless he was actuated by common object. (Para 6)
(C) Penal Code (45 of 1860), S. 149 - UNLAWFUL ASSEMBLY - COMMON OBJECT
- Unlawful assembly - Common object - Not proved - Accused cannot be convicted with
help of S.149. (Para 6)
(D) Penal Code (45 of 1860), S.149, S.141 - UNLAWFUL ASSEMBLY - Unlawful
assembly - Member of - Pre-requisites - Person should have understood that assembly
was unlawful - And was likely to commit any of acts which fall within purview of S.141.
(Para 6)
(E) WORDS AND PHRASES - COMMON OBJECT - Words and Phrases - Common
object - Word 'object' means the purpose or design and, in order to make it 'common' - It
must be shared by all. (Para 6)
(F) Penal Code (45 of 1860), S.149 - UNLAWFUL ASSEMBLY - COMMON OBJECT -
@page-SC2693
Unlawful assembly - Common object - May be formed by express agreement after
mutual consultation - But that is by no means necessary. (Para 6)
(G) Penal Code (45 of 1860), S.149 - UNLAWFUL ASSEMBLY - Unlawful assembly -
It may be formed at any stage by all or a few members - It may be modified or altered or
abandoned at any stage. (Para 6)
(H) Penal Code (45 of 1860), S.149 - UNLAWFUL ASSEMBLY - COMMON OBJECT -
WORDS AND PHRASES - Unlawful assembly - Common object - Expression 'in
prosecution of common object' as appearing in S.149 have to be strictly construed as
equivalent to 'in order to attain the common object' - It must be immediately connected
with common object by virtue of nature of object. (Para 6)
(I) Penal Code (45 of 1860), S.149 - UNLAWFUL ASSEMBLY - COMMON OBJECT -
Unlawful assembly - Common object - Effect of S.149 may be different on different
members of same assembly. (Para 6)
(J) Penal Code (45 of 1860), S.149 - UNLAWFUL ASSEMBLY - COMMON OBJECT -
COMMON INTENTION - Unlawful assembly - 'Common object' - Is different from a
'common intention' - It does not require a prior concert and a common meeting of minds
before the attack. (Para 7)
(K) Penal Code (45 of 1860), S.149 - UNLAWFUL ASSEMBLY - COMMON OBJECT -
Unlawful assembly - Common object - Is determined keeping in view nature of assembly,
arms carried by members, and behaviour of members at or near scene of incident. (Para
7)
(L) Penal Code (45 of 1860), S.149, Expln., S.141 - UNLAWFUL ASSEMBLY -
COMMON OBJECT - Unlawful assembly - Unlawful common object - It is not
necessary in all cases that same must be translated into action or be successful.
It is not necessary under law that in all cases of unlawful assembly, with an unlawful
common object, same must be translated into action or be successful. Under the
Explanation to Section 141, an assembly which was not unlawful when it was assembled,
may subsequently become unlawful. It is not necessary that the intention or the purpose,
which is necessary to render an assembly an unlawful one comes into existence at the
outset. The time of forming an unlawful intent is not material. An assembly which, at its
commencement or even for some time thereafter, is lawful, may subsequently become
unlawful. In other words it can develop during the course of incident at the spot eo
instante. (Para 7)
(M) Penal Code (45 of 1860), S.149 - UNLAWFUL ASSEMBLY - OBJECT OF AN ACT
- Scope - It consists of two parts - Distinction between two parts cannot be ignored or
obliterated.
Section 149, IPC consists of two parts. The first part of the section means that the offence
to be committed in prosecution of the common object must be one which is committed
with a view to accomplish the common object. In order that the offence may fall within
the first part, the offence must be connected immediately with the common object of the
unlawful assembly of which the accused was member. Even if the offence committed is
not in direct prosecution of the common object of the assembly, it may yet fall under
Section 141, if it can be held that the offence was such as the members knew was likely
to be committed and this is what is required in the second part of the section. The
distinction between the two parts of S. 149 cannot be ignored or obliterated. In every case
it would be an issue to be determined, whether the offence committed falls within the first
part or it was an offence such as the members of the assembly knew to be likely to be
committed in prosecution of the common object and falls within the second part.
AIR 1956 SC 731 ; 2004 AIR SCW 2954, Rel. on. (Para 8)
(N) Penal Code (45 of 1860), S.300, S.148, S.149, S.323 - MURDER - UNLAWFUL
ASSEMBLY - HURT - Unlawful assembly and murder - Proof - Accused persons
allegedly formed unlawful assembly and assaulted deceased with machus, sticks - There
were three injuries on head - Most of injuries were deep incised wounds of varying sizes -
Additionally, multiple fractures on base of occipital bone was noticed - Intention of
assailants as established by evidence of witnesses was to cause death of deceased and not
to cause grievous injury - Accused persons are, therefore, liable to be convicted u/S.302
r/w.149 instead of S.326 r/w.149, u/S.148.
2000 (1) Kant LJ 17, Reversed. (Paras 10, 11, 12)
@page-SC2694
Cases Referred : Chronological Paras
2004 AIR SCW 2954 : AIR 2004 SC 2451 : 2004 Cri LJ 2536 : 2004 All LJ 1871 (Rel.
on) 8
AIR 1956 SC 731 : 1956 Cri LJ 1365 (Rel. on) 8
Sanjay R. Hegde, Amit Kr. Chawla and A. Rohan Singh, for Appellant; N. D. B. Raju,
Ms. Bharathi Raju, Guntur Prabhakar and N. Ganpathy, for Respondents.
Judgement
Dr. ARIJIT PASAYAT, J. :- Challenge in this appeal is to the judgment of a Division
Bench of the Karnataka High Court partially allowing the appeal filed by the respondent
who were convicted for offence punishable under Sections 148, 302 read with Section
149 of the Indian Penal Code, 1860 (in short the 'IPC'). By the impugned judgment the
High Court held that the respondents were to be convicted under Section 148 and Section
326 read with Section 149 IPC.
2. Background facts as projected by the prosecution in a nutshell are as follows :
Eight persons faced trial for allegedly committing murder of one Rajanna (hereinafter
referred to as the 'deceased') on 13.7.1992. It was also alleged that they committed
offence punishable under Sections 143, 147 and 148 IPC. First Information Report (in
short the 'FIR') was lodged on 13.7.1992 at about 9.30 p.m.
It was alleged that in an incident that took place at Bandihalli at about 7 p.m. on
13.7.1992, the eight accused who were members of an unlawful assembly had assaulted
the deceased Rajanna with machus, sticks and a wooden reaper and as a result of the
injuries sustained by him, he died shortly thereafter. The accused are all inter-related and
there was some rivalry between the two groups which is of a long standing nature and
that this was the real reason for the incident. The mother of the deceased Ningamma
(P.W. 1) stated that the accused persons had come to her house shortly before the incident
and some of them were armed with machus and remaining persons had clubs and a
wooden reaper with them. They asked her as to where her son Rajanna was. She informed
the persons who had come there that Rajanna had gone out and she bolted the door
because they were in an aggressive mood. According to her, they threw stones on the
house and once again enquired about Rajanna and since she told them that he was not in
the house, they left the place stating that they would finish him. Shortly after this, she
went in the direction in which these persons have proceeded and saw Rajanna
approaching from the opposite side. On seeing the accused persons, he tried to escape
from them but the accused caught hold of him and severely assaulted him. Rajanna fell
on the ground with several injuries on his head and different parts of the body and the
lower limbs and that he was bleeding. The accused left the place with the weapons stating
that Rajanna was finished. Attempt was made to take the injured person to the hospital at
Huliyurdurga in a car. Rajanna died on the way and ultimately, the body was taken to the
Police Station and from there to the hospital. The complainant Ningamma (PW1) lodged
the complaint at 9.30 p.m. and this complaint which has been treated as the F.I.R. was
ultimately sent to the J.M.F.C., Kunigal, which reached him at 7.30 am, the next morning.
Accused No.4-Lokesh had also sustained two injuries of considerable seriousness on his
left thigh and right leg respectively and he came to be admitted to the hospital at
Huliyurdurga on the same evening at about 7.30 p.m. A-4 had lodged a complaint with
the police to the effect that deceased Rajanna and two other persons had assaulted him
near his house at about 5.30 p.m. on 13.7.1992 and that he had sustained the injuries in
the course of that incident. Ultimately, the Police filed a report in respect of this
complaint. As far as the complaint lodged by Ningamma is concerned, the Police
registered an offence being Crime No. 92/1992 under Section 302 IPC read with Section
149 IPC and after completion of the investigation, put up eight accused for trial. The
learned trial Judge found the eight accused persons guilty of the offences punishable
under Sections 302 read with 149 IPC and convicted all of them and sentenced them to
suffer R.I. for life under the main charge along with a fine of Rs. 5,000/- in default, to
undergo further R.I. for a period of one year. The accused were also convicted of offence
punishable under Section 148 IPC and imposed fine of Rs.500/- in default, to undergo
simple imprisonment for three months.
3. In appeal, the High Court found that A4 i.e. Lokesh was not guilty but the rest of the
accused persons were responsible for the death of the deceased. But, however altered the
conviction as noted above. The High
@page-SC2695
Court for the purpose of altering the conviction noted as follows :
"On behalf of the appellants, it was pointed out to us that on the basis of the oral
evidence, it has not been established as to which accused dealt which blow and the
number of blows that each of the accused had inflicted. Secondly, on a careful scrutiny of
the medical evidence, we find that there is a serious lacunae is in so far as the doctor has
not indicated as to which of them are not. Of the twenty injuries that were found on the
person of deceased Rajanna, it is true that two of them are on the head, the majority of
them are aimed at the lower part of the body and the limbs and consequently, having
bestowed our very serious attention to the cumulative effect of this record. We find that it
was incorrect on the part of the trial court to have invoked the provisions of Section 302
IPC. Having regard to the weapons used and the nature of injuries that have been
inflicted, the accused would be liable to be convicted of the offence punishable under
Section 326 read with 149 IPC."
4. In support of the appeal learned counsel for the appellant submitted that the reasons
indicated by the High Court are palpably wrong and cannot be sustained. With reference
to the injuries sustained it was submitted that the intention of the unlawful assembly is
clear from the weapons held by the assailants. The injuries inflicted on the eye-witnesses
i.e. Ningamma (PW1), Appaji (PW3), Sivappa (PW6) clearly described the role of the
accused persons in surrounding and assailing the deceased. It is submitted that the High
Court has wrongly held that there were only two injuries on the head and that the rest of
the injuries on the lower part of the body and limbs. It is not factually correct on a bare
reading of the injury report. In fact, there were three injuries on the head. Additionally,
the injuries 6 and 7 clearly show the force with which the injuries were inflicted, and in
fact, injury No. 7 shows that a hand was severed. The doctor's evidence also shows that
there were multiple fractures of base of the occipital bone.
5. Learned counsel for the respondent on the other hand shows that the acquittal of A4
because of non-explanation of injuries on him shows the falsity of the prosecution case. It
was stated that the occurrence took place in the course of free fight and therefore the
High Court's judgment does not warrant any interference. It was submitted High Court's
judgment shows that Section 149 IPC was ruled out.
6. The pivotal question is applicability of Section 149 IPC. Said provision has its
foundation on constructive liability which is the sine qua non for its operation. The
emphasis is on the common object and not on common intention. Mere presence in an
unlawful assembly cannot render a person liable unless there was a common object and
he was actuated by that common object and that object is one of those set out in Section
141. Where common object of an unlawful assembly is not proved, the accused persons
cannot be convicted with the help of Section 149. The crucial question to determine is
whether the assembly consisted of five or more persons and whether the said persons
entertained one or more of the common objects, as specified in Section 141. It cannot be
laid down as a general proposition of law that unless an overt act is proved against a
person, who is alleged to be a member of unlawful assembly, it cannot be said that he is a
member of such an assembly. The only thing required is that he should have understood
that the assembly was unlawful and was likely to commit any of the acts which fall
within the purview of Section 141. The word 'object' means the purpose or design and, in
order to make it 'common', it must be shared by all. In other words, the object should be
common to the persons, who compose the assembly, that is to say, they should all be
aware of it and concur in it. A common object may be formed by express agreement after
mutual consultation, but that is by no means necessary. It may be formed at any stage by
all or a few members of the assembly and the other members may just join and adopt it.
Once formed, it need not continue to be the same. It may be modified or altered or
abandoned at any stage. The expression 'in prosecution of common object' as appearing in
Section 149 have to be strictly construed as equivalent to 'in order to attain the common
object'. It must be immediately connected with the common object by virtue of the nature
of the object. There must be community of object and the object may exist only up to a
particular stage, and not thereafter. Members of an unlawful assembly may have
community of object up to certain point beyond which they may differ in their objects
and
@page-SC2696
the knowledge, possessed by each member of what is likely to be committed in
prosecution of their common object may vary not only according to the information at his
command, but also according to the extent to which he shares the community of object,
and as a consequence of this the effect of Section 149, IPC may be different on different
members of the same assembly.
7. 'Common object' is different from a 'common intention' as it does not require a prior
concert and a common meeting of minds before the attack. It is enough if each has the
same object in view and their number is five or more and that they act as an assembly to
achieve that object. The 'common object' of an assembly is to be ascertained from the acts
and language of the members composing it, and from a consideration of all the
surrounding circumstances. It may be gathered from the course of conduct adopted by the
members of the assembly. What the common object of the unlawful assembly is at a
particular stage of the incident is essentially a question of fact to be determined, keeping
in view the nature of the assembly, the arms carried by the members, and the behaviour of
the members at or near the scene of the incident. It is not necessary under law that in all
cases of unlawful assembly, with an unlawful common object, the same must be
translated into action or be successful. Under the Explanation to Section 141, an
assembly which was not unlawful when it was assembled, may subsequently become
unlawful. It is not necessary that the intention or the purpose, which is necessary to
render an assembly an unlawful one comes into existence at the outset. The time of
forming an unlawful intent is not material. An assembly which, at its commencement or
even for some time thereafter, is lawful, may subsequently become unlawful. In other
words it can develop during the course of incident at the spot co instante.
8

. Section 149, IPC consists of two parts. The first part of the section means that the
offence to be committed in prosecution of the common object must be one which is
committed with a view to accomplish the common object. In order that the offence may
fall within the first part, the offence must be connected immediately with the common
object of the unlawful assembly of which the accused was member. Even if the offence
committed is not in direct prosecution of the common object of the assembly, it may yet
fall under Section 141, if it can be held that the offence was such as the members knew
was likely to be committed and this is what is required in the second part of the section.
The purpose for which the members of the assembly set out or desired to achieve is the
object. If the object desired by all the members is the same, the knowledge that is the
object which is being pursued is shared by all the members and they are in general
agreement as to how it is to be achieved and that is now the common object of the
assembly. An object is entertained in the human mind, and it being merely a mental
attitude, no direct evidence can be available and, like intention, has generally to be
gathered from the act which the person commits and the result therefrom. Though no hard
and fast rule can be laid down under the circumstances from which the common object
can be culled out, it may reasonably be collected from the nature of the assembly, arms it
carries and behaviour at the time of or before or after the occurrence. The word 'knew'
used in the second limb of the section implies something more than a possibility and it
cannot be made to bear the sense of 'might have been known'. Positive knowledge is
necessary. When an offence is committed in prosecution of the common object, it would
generally be an offence which the members of the unlawful assembly knew was likely to
be committed in prosecution of the common object. That, however, does not make the
converse proposition true; there may be cases which would come within the second part
but not within the first part. The distinction between the two parts of Section 149 cannot
be ignored or obliterated. In every case it would be an issue to be determined, whether the
offence committed falls within the first part or it was an offence such as the members of
the assembly knew to be likely to be committed in prosecution of the common object and
falls within the second part. However, there may be cases which would be within the first
part but offences committed in prosecution of the common object would also be
generally, if not always, be within the second part, namely, offences which the parties
knew to be likely to be committed in the prosecution of the common object. (See
Chikkarange Gowda and others v. State of Mysore AIR 1956 SC 731). These aspects
were also recently highlighted in Chandra and Ors. v. State of U.P. and Anr. [2004 (5)
SCC 141]. 2004 AIR SCW 2954

@page-SC2697
9. The injuries which were inflicted by the accused persons on the deceased as noted by
the High Court are as follows :
1. One contusion 4" x 4" on occipital region of head.
2. Deep incised wound 2" x 1" x 2" right side of the pariatal region of head.
3. Incised wound 6 x 3" x 1" on the left side of the fore head. Bone exposed on both
bounds. Blood stains seen on all the wounds.
4. Contusion 3"x4" on right shoulder joint.
5. Deep incised wound 5"-x 2" x1" on middle 1/3rd of right arm humerous exposed.
6. Incised wound 4"x3" x2" on right axilla in horizontal direction, pleura and lung
exposed.
7. Deep incised cut injury on right wrist joint. Right hand detached from the body at the
level of wrist joint, only skin flap is connecting, all the bones are exposed.
8. Deep incised wound 2" x 2" on dorsum of the left hand bones exposed wound is in
horizontal direction.
9. Incised wound 2" x 1" x 1" on left elbow joint, horizontal direction, bones exposed,
Radial artery and small blood vessels cut open.
10. Deep incised wound upper 1/3rd of the left fore arm 2"x2"x1" just below the elbow
joint, bones exposed.
11. Deep incised wound 4" x 3" x 2" on middle, 1/3rd and the left thigh, shaft of left
femur exposed and blood vessels cut open. Wound is in a horizontal direction.
12. Deep incised wound 3" x 2" x 1" on the left knee joint, pattellar tendon are cut and
bone exposed. Horizontal direction femeral artery and sophanus vain cut open._
13. Deep incised wound 3" x 2" x 2" right knee joint, pattella bone exposed 2" x 2" x 2"
blood vesels are cut open, horizontal direction.
14. Deep incised wound 6" x 3" just below the left knee joint. Head of the tibia exposed.
Blood vessels cut open, horizontal direction.
15. Incised wound 3" x 2" x 3" on lateral aspect of right thigh, shaft of femur exposed,
blood vessels cut open, horizontal in direction, blood clots seen on the wounds.
16. Deep incised wound 6" x 4" x 5" middle 1/3rd of right leg, tibia and febul also cut
superficially, horizontal in direction, femoral ortery sophenus vain cut open.
17. Left ankle joint and tendo achulus tender also cut into pieces.
18. Deep incised wound 3"x2"x1" on the left popleteal fosa, all the blood vessels cut
open, horizontal in direction.
19. Incised wound 6"x4" on right leg on the medial aspect, all the muscles and blood
vessels cut open, horizontal in direction.
20. Incised wound 2"x 2" x1" on dorsum of right foot, horizontal in direction.
10. As noted above, there were three injuries on the head and injury No.6 was of such
nature that the plura and lung were exposed. Injury No. 7 was a deep incised cut injury on
right wrist joint. Right hand was detached from the body at the level of wrist joint.
Further, rest of the injuries were not on the lower parts of the body as noted by the High
Court. In fact the injury No.4 was an injury on the right shoulder joint and injury No.5
was a deep incised wound 5"x 2"x 1", on middle 1/3rd of right arm humerous exposed.
11. Most of the injuries were deep incised wounds of varying sizes. Additionally, as noted
above, the multiple fractures on the base of the occipital bone was noticed. The intention
of the assailants as established by the evidence of the witnesses was the cause of death of
the deceased and not to cause grievous injury.
12. Above being the position the High Court's judgment is clearly unsustainable and is set
aside. The respondents are convicted for offence punishable under Section 302 read with
Section 149 IPC instead of Section 320 read with Section 149 IPC as held by the High
Court. The sentences imposed by the Trial Court stand restored. Respondents shall
surrender to custody forthwith to serve remainder of sentences.
13. Appeal is allowed.
Appeal allowed.
AIR 2008 SUPREME COURT 2697 "Sajjan Textile Mills Ltd. v. ICICI Bank Ltd."
(From : Madras)*
Coram : 2 TARUN CHATTERJEE AND HARJIT SINGH BEDI, JJ.
Civil Appeal No. 573 of 2005, D/- 16 -5 -2008.
Sajjan Textile Mills Ltd. v. ICICI Bank Ltd. and Ors. @page-SC2698
Recovery of Debts Due to Banks and Financial Institutions Act (51 of 1993), S.25 -
Constitution of India, Art.226, Art.14 - RECOVERY OF DEBT - WRITS - EQUALITY -
ATTACHMENT - SALE - Recovery of loan by Bank - Attachment and sale of property
of appellant debtor ordered by Debt Recovery Tribunal - Extension of time granted to
bidder to deposit bid money - And, thereafter, to comply with conditions of payment -
Writ petition filed by Bank for direction to DRT to proceed with recovery of amount due
- Once appellant was made party to writ proceedings. Bank cannot take plea that
appellant was not entitled to hearing as sale proceedings had attained finality.
W. P. No. 14090 of 2004, D/- 2-8-2004 (Mad), Reversed. (Para 7)

Dr. Abhishek Manu Singhvi, Sr. Advocate, R. Parthasarathy, Amit Bhandari and Ajit
Kumar Sinha, with him for Appellant; T. R. Andhyarujina, Sr. Advocate, Naveen Chawla,
Ms. Manjula Gupta and Ambrish Kumar, with him for Respondents.
* W. P. No. 14090 of 2004, D/- 2-8-2004 (Mad).
Judgement
HARJIT SINGH BEDI, J. :- In the light of the order we intend making, only the skeletal
facts are necessary. They are as under :
2-3. The appellant, Sajjan Textile Mills Ltd. was sanctioned a loan of Rs.3 Crore by the
respondent Bank on 15th July, 1992. As the appellant committed default in making
repayment, a civil suit for recovery was filed. The Bombay High Court also passed an
order dated 18th December 1997, appointing the Court Receiver, as the Receiver for the
movable and immovable properties of the appellant. The appellant, however, made an
application before the BIFR on 23rd June, 1998 for being registered as a sick industrial
unit and it was registered as such on 15th July, 1998 and was declared a sick unit by order
dated 6th August, 1998. Two suits were thereafter filed by the respondent Bank in the
Bombay High Court, one against the appellant for recovery of a sum of Rs.8,36,15,087/-
and the other against the guarantors. The aforesaid civil suits as also suits filed by the
Central Bank of India and the State Bank of Travancore also for recovery of loans
advanced were all transferred to the DRT, Bombay Bench, and an inventory of all the
machinery available in the appellant's mill was made and a report submitted to the
Tribunal. The BIFR also passed an order on 5th October, 2001 that as the appellant
company could not be resusticated, it was just and equitable that it be wound up. In the
meanwhile, the respondent Bank (in April 1999) also filed an application for the recovery
of the principal amount of Rs.3 Crore. The DRT in its order allowed the application and
passed an order for the recovery of the aforesaid amount from the appellant. A notice of
demand dated 31st December, 2002 was issued by the Recovery Officer and recovery
proceedings were initiated. The Recovery Officer issued a certificate against the appellant
and the guarantors making them jointly and severally responsible for a payment of
Rs.5,90,32,753/-. As the properties were situated within the jurisdiction of the DRT,
Coimbatore, Tamil Nadu, the recovery proceedings were accordingly transferred to
Coimbatore on an application made by the Bank. As the appellant and the guarantors did
not pay the amounts due, an order for the attachment of the movable property of the
appellant was issued, and despite several objections taken by the appellant, a
proclamation for sale by way of tender was issued on 19th January, 2004. The five
tenders received were opened on 3rd February, 2004 and Sri Maruthi Textiles was found
to be the highest bidder with a bid of Rs.2,50,99,999/-. The Recovery Officer thereupon
directed the successful tenderer to deposit the balance money within 15 days which was
further extended by 30 days up till 27th February, 2004. The aforesaid amount was,
however, not paid on which the Recovery Officer in his order dated 17th March, 2004
directed that the offer of the 2nd respondent, Sri Vairalakshmi be accepted on payment of
the amount tendered by Sri Maruthi Textiles. This offer was accepted by respondent No.2
and it undertook to deposit the bid money, but vide application dated 25th March 2004,
sought an extension of time for doing so, which too was granted. It appears that on 1st
March 2004, the Office of the Recovery Officer and DRT Coimbatore fell vacant and the
Presiding Officer DRT, Chennai was appointed to hold the dual charge of both places. On
16th April 2004, respondent No.2 once again applied for more time for complying with
the conditions of payment and the Presiding Officer, DRT, Chennai, who was holding the
post as an additional charge made the following order on 7th May, 2004 :
"Since no regular Recovery Officer is available in this Tribunal, the EMD deposited
@page-SC2699
by the second bidder has to be returned. Further, the matter should be returned to DRT-
III, Mumbai for further N.A."
4. On 11th May 2004, the respondent Bank filed a writ petition before the Madras High
Court for a direction to the Presiding Officer, DRT, Chennai to proceed with the recovery
of the amounts due. In this petition, Sajjan Textiles, the present appellant, was shown as
respondent No.3 and Sri Vairalakshmi and Co. was shown as the second respondent. The
appellant was duly served and a vakalatnama also filed by an Advocate on its behalf on
21st July, 2004. It appears, however that the High Court by its order dated 2nd August,
2004 disposed of the writ petition in the absence of the appellant's counsel by giving yet
more time to respondent No.2 to pay the balance amount either in full or in instalments
within a time frame of 2 months and the Presiding Officer, DRTC was directed to
complete the sale transaction in terms mentioned in the body of the order. It is this order
which has been impugned in this appeal.
5. Mr. Singhvi, the learned senior counsel for the appellant, has at this stage raised only
one argument before us. He has pointed out that the appellant was respondent No.3 in the
writ proceedings in the High Court and though a Vakalatnama had been filed by a counsel
on its behalf, the name of the counsel had not appeared in the cause list on the 2nd
August, 2004 nor on the date preceding that date with the result that the appellant had
suffered serious prejudice on account of remaining unrepresented on being unaware of
the proceedings.
6. Mr. Andhyarujina, the learned senior counsel for the respondent Bank has, however,
taken us through the entire sequence of events and the history of the litigation, and
pointed out that the appellant was only a formal party in the writ proceedings, and as all
the orders pertaining to the recovery and the sale had become final as some had not been
challenged by the appellant and in some others, the challenge had failed, no useful
purpose would be served in interfering in this matter. He has also pointed out that the
property had been sold and removed from the appellant's mill under the orders of this
Court and nothing now remained to be decided and that the efforts of the appellants to
prolong the litigation any further should be discouraged.
7. We have heard the learned counsel for the parties and gone through the record very
carefully. It is true that the litigation has had a chequered career in several forums
including this Court. However in the present proceedings, we are not called upon to take
a decision on the ultimate effect of the earlier set of proceedings as Mr. Singhvi has
limited his claim to the fact that the appellant had not been heard at the time when the
High Court had made the impugned order on 2nd August 2004. We are unable to accept
Mr. Andhyarujina's plea that as the sale proceedings had attained finality, there was no
need to have heard the appellant/Company as it had been impleaded as a mere formality.
We feel that once having made the appellant a party in the writ proceedings, it does not
lie on the Bank to contend that the appellant was not entitled to a hearing. We,
accordingly, set aside the order of the High Court dated 2nd August 2004, and remit the
case for a fresh decision in accordance with law. We also direct that till such time the
High Court takes its decision in the matter, the status quo order passed by this Court on
29th October, 2004 will continue to operate. We also request the High Court, in the
background that the matter has been pending for a very long time, to render its decision
as expeditiously as possible.
8. The appeal is allowed. There will, however, be no order as to costs.
Appeal allowed.
AIR 2008 SUPREME COURT 2699 "Bakemans Industries Pvt. Ltd., M/s. v. M/s. New
Cawnpore Flour Mills"
(From : Delhi)
Coram : 2 S. B. SINHA AND V. S. SIRPURKAR, JJ.
Civil Appeal Nos. 3628 with 3629 of 2008 (arising out of SLP (C) Nos. 12616 with
14427 of 2007), D/- 16 -5 -2008.
M/s. Bakemans Industries Pvt. Ltd. v. M/s. New Cawnpore Flour Mills and Ors.
(A) Companies Act (1 of 1956), S.437, S.457 - State Financial Corporations Act (63 of
1951), S.29 - COMPANY - FINANCIAL CORPORATION - AUCTION SALE - Powers
of Company Court - Company in liquidation - Auction sale of its property - Company
Judge could exercise jurisdiction only in terms of Companies Act and not in terms of S.29
of 1951 Act - Financial Corporation, SICOM having statutory power to conduct auction
sale, waived its right and submitted to jurisdiction of Company Judge - Bid was held only
at its instance
@page-SC2700
- Company Judge thus can exercise supervision of sale of assets of company on behalf of
SICOM in terms of S.29 of 1951 Act - Claims shown to be made by other creditors and
workmen of company also - Involvement of Official Liquidator, therefore, cannot be
ignored merely because provisional and not regular official liquidator was appointed -
Sale to be held afresh. (Paras 40, 42, 46, 47, 48, 53, 56, 62, 63, 64, 65, 67)
(B) State Financial Corporations Act (63 of 1951), S.29 - Companies Act (1 of 1956),
S.446 - FINANCIAL CORPORATION - COMPANY - OBJECT OF AN ACT - Scope -
Financial Corporation intending to exercise statutory power u/S.29 - Same will prevail
over general powers of Company Judge under Companies Act, 1956 - Proceedings u/S.29
of 1951 Act would prevail over winding up proceedings before company Judge.
2003 AIR SCW 1524, Foll. (Paras 39, 40)
(C) Companies Act (1 of 1956), S.450, S.457 - COMPANY - Provisional liquidator -
Powers and functions of - Are same as that of official liquidator subject to limitations
imposed by Court. (Paras 55, 58)
Cases Referred : Chronological Paras
2006 AIR SCW 113 : AIR 2006 SC 755 : 2006 CLC 15 (Ref.) 37, 40
2006 AIR SCW 2361 : AIR 2006 SC 2088 : 2006 CLC 709 : 2006 (4) AIR Kant HCR 54
(Ref.) 40
2005 AIR SCW 1608 : AIR 2005 SC 1814 : 2005 CLC 531 (Ref.) 42
2005 CLC 1602 : 2005 AIR Kant HCR 2679 (SC) (Ref.) 47
2003 AIR SCW 1524 : AIR 2003 SC 2012 : 2003 CLC 441 : 2003 AIR-Kant HCR 1002
(Foll.) 40
(2003) UKHL 49 60
2000 AIR SCW 1347 : AIR 2000 SC 1535 : 2000 CLC 913 (Ref.) 42
2000 AIR SCW 2804 : AIR 2000 SC 2642 : 2000 CLC 1628 (Ref.) 47
1989 AC 755 61
(1987) 1 All ER 114 61
(1986) 4 ACLC 215 61
(1983) Ch 258 61
1974 Tax LR 2406 (Mad) 58
1964 (1) Ch D 19 59
1888 (39) Ch. D. 88 54
P. V. Kapur, P. H. Parekh, Sr. Advocates, Viks Pahwa, Abhinit Das, Nitin, Ms. Chetna,
Ms. Marie, Prem Malhotra and Rishi Malhotra, with them for Appellant; Rajiv Shekhar,
G. A. Sundaram, Sr. Advocates, Chinnoy, Pradip Sharma, Dr. Kailash Chand, P. C. Sen,
Ms. Rohini Musa, Pallav Kumar and R. C. Kaushik, with them for Respondents.
Judgement
S. B. SINHA, J. :- Leave granted in both the matters.
2. Whether power of a Company Court to sell the property of a company vis-a-vis the
power of the Financial Corporation can be merged is the question involved in these
appeals which arise out of the judgments and orders dated 2nd July, 2007 and 6th July,
2007 passed in Company Appeal No. 27 of 2004 and Company Appeal No.2 of 2007
respectively passed by the Division Benches of the Delhi High Court.
3. Certain basic facts are not in dispute which are as under :
SICOM Ltd. (SICOM in short) advanced a loan of Rs.17 crores to the appellant (M/s.
Bakemans Industries Pvt. Ltd.). It became a defaulter, SICOM issued a notice under
Section 29 of the State Financial Corporations Act (1951, Act in short) on 22nd January,
2003. Another notice was issued for taking over possession of the properties of the sister
concern of the appellant, viz. Captain Hygiene Products Ltd. Appellant and its sister
concern filed two writ petitions in the Punjab and Haryana High Court at Chandigarh.
They were dismissed as withdrawn on 10th February, 2003.
4. 1st respondent and fourteen others filed fifteen applications before the Delhi High
Court for winding up of the appellant-company. Notices were issued thereupon. SICOM
issued a second notice under Section 29 of the 1951 Act on 6th June, 2003.
5. Indisputably the factory of the appellant was an ongoing concern. SICOM took over
the possession of the appellant's factory at Patiala on 18th July, 2003. It was at that time
in operation. It had finished bakery products which were perishable in nature. Allegedly
the operations were shut down and the factory was locked.
6. We may notice here that different proceedings were initiated either at the instance of
the appellant or at the instance of some of the respondents.
7. Appellant evidently took recourse to a proceeding which was unknown to law. A
@page-SC2701
purported agreement was entered into by and between the appellant and one NRI Lead
Bank. We are not aware as to what were the disputes about between them. The said
purported disputes were referred to Arbitral Justice Tribunal of ADR Arbitration, a body
said to have been recognized by the Government of India in terms of Section 21 of the
Arbitration and Conciliation Act, 1996. A purported reference of disputes in terms of a
purported arbitration agreement contained in a composite instrument dated 14th August,
2003 was referred on 16th August, 2003. It was accepted by the Tribunal on 18th August,
2003 and notices were issued. The majority of the Tribunal opined that there was no
genuine arbitration agreement. The arbitration proceeding was closed on 23rd August,
2003.
8. A new set of Arbitrators was constituted by the Tribunal who rendered an award on
16th August, 2003 upon holding a day's sitting only opining that (i) taking over of the
unit was illegal and (ii) a direction was issued to handover possession to Bakemans.
9. A purported execution petition was filed by NRI Lead Bank before the Delhi High
Court seeking execution of a purported written agreement/settlement dated 16th August,
2003 passed by the Board of Conciliation in the said proceedings.
10. The execution petition was filed not only against the appellant and its sister concern,
Captain Hygiene Products Pvt. Ltd. but also against SICOM. Industrial Development
Bank of India, Industrial Finance Corporation of India, HUDF Bank, State Bank of
Patiala, and Punjab State Industrial Development Corporation Ltd. were also impleaded
as parties therein.
11. We shall deal with the factual matrix thereabout a little later.
12. However, in the meantime, another proceeding by way of an application under
Section 9 of the Arbitration and Conciliation Act, 1996 was filed before the Tis Hazari
Courts, Delhi. It was registered as Misc. Suit No. 139 of 2003. Inter alia, a prayer was
made therein to appoint a receiver. However, it appears that another Bank initiated a
proceeding before the Debt Recovery Tribunal for recovery of its dues. A Receiver was
appointed by the said Tribunal in respect of the perishable goods on 1st September, 2003.
13. Possession of the said perishable goods lying in the factory was taken from SICOM.
A spot report was prepared.
14. Appellant in the meantime relying on or on the basis of the said purported Award of
the Board of Conciliation took forcible possession of the factory premises on 14th
September, 2003.
15. SICOM filed an application in the said purported execution proceeding seeking for
the following directions :
i) to withdraw the proceeding before the learned Additional District Judge;
ii) to vacate and handover the premises;
iii) to grant prohibitory injunction; and
iv) to stay the operation of the Arbitration Award.
16. An order of status quo which had been passed earlier was directed to be maintained
by the parties by the High Court on 15th September, 2003.
17. An application for modification of the order dated 15th September, 2003 was filed by
SICOM on 16th September, 2003.
18. Appellant also filed an application for permission to sell all perishable goods lying in
the factory. Allegedly, the Receiver was asked to sell the perishable goods.
It also directed the appellant to pay some amount to show its bona fide. Appellant
furthermore filed an application for vacation of the order dated 15th/16th September,
2003. On 28th November, 2003 an assurance was also given to the Court that the
appellant will come with a definite proposal for payment to the creditors. By an order
dated 18th December, 2003 the High Court directed the appellant to deposit a sum of
Rupees two crores failing which SICOM was given a liberty to proceed with the statutory
remedies available to it under the Act for sale of the properties. An undertaking was given
to the Court by the Managing Director of the appellant in the following terms :-
"Mr. Rajiv Kumar Gupta, Managing Director of judgment debtor No. 1 and Director of
judgment debtor No.2, who is present in Court, undertakes to the Court that on or before
7.2.2004, a sum of Rs.2 crores would be deposited with judgment debtor No.3, to be
apportioned towards the liability of judgment debtor Nos. 3, 4 and 5. Judgment debtor
Nos. 1 and 2 shall also give a proposal for settlement, setting out a firm payment schedule
for consideration of judgment
@page-SC2702
debtor Nos. 3, 4 and 5. In the event the payment of Rs. 2 crores is not made on the date
stipulated, judgment debtor No. 3 would be at liberty to avail of statutory remedies
available at law for sale of the property.
Counsel for the parties also pray that the modalities of restoration of possession be got
done under the supervision of officers of this Court, so as to avoid unseemly
controversies and a clear account of the equipments, machinery and the assets, of which
possession is taken over at the factory premises is available. Considering the quantum of
work required, counsel for the parties pray that at least three Local Commissioners be
appointed. Accordingly, I appoint Mr. D.K. Batra, Joint Registrar of this Court, Mr. S. P.
Tara, Deputy Registrar of this Court and Mr. Anil Kumar Arora, Sr. Personal Assistant of
this Court, as the Local Commissioners to visit the Factory Area, Village Rasulpur
Saidan, Tehsil and District Patiala, State of Punjab. The Local Commissioners shall make
a complete inventory of the equipment, machinery, assets, raw materials, finished, semi
finished products, if any. The possession of factory and assets be handed over to the
representatives of respondent No.3. Inventory be also got signed by the parties. The Local
Commissioners may in their discretion also make any observation with regard to the
condition or state of equipment, assets etc. The Local Commissioners to execute the
commission on 23.12.2003 at 11.00 a.m. The fee of the Local Commissioners, Mr. D. K.
Batra is fixed as Rs.22,000, Mr. S. P. Tara is fixed as Rs.20,000/- and Mr.Anil Kumar
Arora is fixed as Rs. 18,000/-, exclusive of out of pocket, travel and lodging expenses.
Learned counsel for judgment debtor Nos.1 and 2 submit that upon payment of Rs.2
crores and a firm schedule being given for repayment, as acceptable to the financial
institutions, the Court should grant repossession to judgment debtor No.2. This aspect
would be considered upon the payment of Rs.2 crores having been made and firm
schedule for repayment having been given and accepted. Counsel for judgment debtor
Nos.1 and 2 state that, in the meanwhile, they would not proceed further with the
arbitration proceedings, initiated before the ADR, Arbitral Tribunal No.3. Mr. Arun
Bhardwaj, counsel for judgment debtor No. 1, further states that judgment debtor No. 1
would not proceed with Suit No. 139/2003, pending in the Court of Sh. S. K. Sarvaria,
A.D.J., Delhi."
19. In the meantime, SICOM obtained a valuation report in respect of the factory from a
Public Sector Organization known as Northern India Technical Consultancy Organization
Ltd. (NITCOL). In the said proceeding, SICOM had also moved an application for
direction to permit them to publish an advertisement for sale of the moveable properties
of the appellant and to invite bids for sale.
20. We may now deal with the process of sale of assets of the company. The factory of
the appellant was situated in village Rasulpur, District Patiala in the State of Punjab. The
land measured 30, 544 sq. yards. The building comprised of three floors having RCC
construction. There were plants and machineries. There was also unpacked material
which had been imported from abroad. Pursuant to the permission granted by the Court to
SICOM to make an advertisement, one was issued in Economic Times(All Editions),
Business Standard (All Editions), Tribune (Chandigarh Edition) and Dainik Bhaskar
(Chandigarh and Patiala Editions). As the appellant failed to deposit the said sum of
Rupees two crores and never submitted the definite proposal in terms of the order dated
28th November, 2003, SICOM was given the liberty to proceed with the sale.
21. On or about 15th March, 2004, respondent No.4, Ceylon Biscuits Pvt. Ltd. filed an
application seeking direction that they be also permitted to inspect the factory on the
premise that they had held negotiations with the appellant for taking over the entire unit.
Counsel who was representing the appellant also represented Ceylon Biscuits Pvt. Ltd.
22. A question was raised in regard to the jurisdiction of the executing court to proceed
with the matter of sale of the properties. By reason of an order dated 16th March, 2004,
the Court noticed the bids submitted by the ITC Limited and Britannia Industries Ltd. not
only on the entire plant but also on item wise basis. The Court rejected the contention of
the appellant both in regard to its jurisdiction as also its valuation report inter alia opining
that it had failed to deposit a sum of Rupees two crores and submitted the repayment
schedule in terms of its earlier order as such there was
@page-SC2703
no other option but to proceed with the sale process.
In regard to the offer of M/s. Ceylon Biscuits Ltd. it was directed :-
"They shall file their bid positively before 23.3.2004. It is also made clear that if there
could be any other interested bidder, he/it could submit a bid in accordance with the
requirements, which shall be considered. It shall also be open to the Judgment debtor
Nos. 1 and 2 to obtain other/better offers from any other bidder. It is made clear that in all
the offers/bids which shall be submitted by any other bidder, the bidders shall have to
comply with the formalities and the terms that have been advertised on 23.2.2004."
23. Ceylon Biscuits Pvt. Ld. on or about 24th March, 2004 offered the bid price at Rs.
12.5 crores. It also deposited the earnest money of Rs. 25 lakhs. There was another bidder
M/s. Longful Trading (India) Pvt. Ld. who had made a bid of Rs. 11.7 crores. It had also
deposited the earnest money of Rs. 25 lakhs. In regard to the valuation of the properties
both in respect of the factory of the appellant as also its sister concern Captain Hygiene
Products Pvt. Ltd. the Court noticed :-
"It is, however, pointed out by the counsel appearing for Bakemans Industries Pvt. Ltd.
and Captain Hygiene Products Pvt. Ltd. that valuation of the said plant and machineries,
and land and building would be much higher than what is shown in the valuation report.
A valuation report is placed on record wherein it is stated that the realisable value of the
aforesaid assets is Rs.8,42,43,000/-. Counsel appearing for M/s. Bakemans Industries Pvt.
Ltd., however, disputes the aforesaid valuation. In order to ascertain the valuation of the
aforesaid assets, it would be appropriate to pass an order directing for re-evaluation of the
entire aforesaid assets of the said company. M/s. SICOM Ltd. is directed to get the entire
assets re-evaluated by appointing an approved valuer. The said valuation report shall be
submitted before the next date. The approved valuer shall visit the factory premises on
March 29, 2004 at 11.00 A.M. when the representative of M/s. Bakemans Industries Pvt.
Ltd. could also be present at the site for the purpose of assisting and giving appropriate
guidance to the approved valuer in ascertaining real value of the assets. The necessary
papers of the plant and machineries and other connected records shall be produced by
M/s. Bakemans Industries Pvt. Ltd. before the approved valuer in order to assist him in
evaluating the aforesaid property. It shall also be open for the approved valuer to collect
informations in respect of various assets from other sources as well like custom
authorities, Director General Foreign Trade and such like authorities. He shall also give a
separate valuation report for un-installed plant and machinery, if any, so as to enable this
Court to ascertain the break-up value of the various plants and machineries and to
facilitate the process of sale by this Court.
It shall be open to any other willing purchasers also to submit their fresh bids, if so
desired, on or before the next date."
24. Allegedly, the appellant filed an application before the Executing Court with a prayer
to decide its jurisdiction at the first instance. It is stated at the Bar that neither there is any
record in respect thereof in the High Court nor any order appears to have been passed
thereon.
25. We may now notice the proceeding before the learned Company Judge.
26. The Company Applications were admitted by an order dated 6th April, 2004. A
Provisional Liquidator was appointed. It was directed to take charge of the properties and
books of accounts of the company. On an application made by SICOM, however, the
learned Company Judge by order dated 16th April, 2004 directed that its possession may
not be disturbed.
27. As the Provisional Liquidator had been appointed, the Executing Court transferred the
petition to the Company Judge by an order dated 19th April, 2004.
28. Some correspondences appear to have passed between the Advocate of the appellant
Official Liquidator and SICOM as regards the effect of the provisions of the Companies
Act viz-a-viz Section 29 of 1951 Act.
29. Appellant, thereafter filed an application on 12th July, 2004 for restraining SICOM
from taking any further action for the sale/auction of the properties and also asked for an
order of status quo to be maintained by the parties. No order on the said application was,
however, passed. In its order dated 17th July, 2004 the learned Company Judge observed
that the offer of Ceylon Biscuits did not appear to be improper.
@page-SC2704
However, appellant was given an opportunity to bring a better offer. Second report of
NITCON as regards valuation was also accepted.
30. Before the learned Company Judge a valuation report of a Chartered Accountant was
submitted which was rejected stating that they were not the approved valuers and they
had only taken into account the book value and not the market value of the assets.
31. The matter was posted for hearing on 22nd July, 2004. On that date, proceedings
before the learned Company Judge were in two sessions - one before lunch and another
after lunch. Before recess, appellant was granted one more opportunity to bring any other
bid and the Judge adjourned the matter to 4th August, 2004. However, after recess on a
purported request made by the learned counsel for M/s. Ceylon Biscuits the case was
preponed to 28th July, 2004. Learned counsel for the appellant was not present, although
it was mentioned that he had been informed. On the next date, i.e. 28th July, 2004 the
Court recorded a statement that the respondent-company was negotiating with some
buyers. An affidavit of the prospective buyer and its Managing Director was directed to
be filed in this behalf along with an undertaking to honour the bid quoted by the
prospective buyer. The matter came up before the learned Company Judge on 30th July,
2004. A prayer for adjournment was made. An affidavit of the Ex-Managing Director of
the appellant was filed. However, adjournment was refused. The affidavit was called from
the Registry and the matter was heard. The Court is said to have waited for the learned
counsel to appear till 4.00 O' clock and then took up the matter for hearing at 4.45 p.m. In
its order the learned Company Judge noticed the earlier proceedings at some length. It
was held :-
" No affidavit is filed of any prospective buyer. Affidavit of Managing Director of the
respondent-company is filed. It does not offer any bid of any buyer. On the contrary, what
is stated is that the Managing Director has been able to tie up finances with the various
associates and the first instalment would be received on or before 5th August, 2004 on
which date a pay order of Rs. 50 lacs shall be produced in the court. It is also stated that
the management and associates thereafter would be definitely for the welfare of all the
financial institutions and workers and would be a far better than which is being offered by
the bidder. This affidavit, obviously, is not in compliance with the directions contained in
the earlier orders and Mr. Chhabra's own statement to the effect that the respondent-
company had negotiated with a buyer who was willing to offer more than the amount
offered by M/s. Ceylon Biscuits Ltd. such attempt had been made earlier but failed. The
arrangement offered in the affidavit does not inspire confidence and it is only a delaying
tactic. He offer to deposit Rs. 50 lacs, in the first instance when the total liability of
secured creditors itself is more than Rs. 50 crores, is a pittance. The respondent-company
has also not stated as to in what manner and within how much time it would be in a
position to discharge the entire liability. It is also not stated as to from where it would
generate the resources/finances for this purpose. It is, thus, clear that in spite of giving
various opportunities to the respondent-company and its Managing Director the
respondent- company has not been able to produce better bid.
Property in question, which is subject-matter of sale, has been valued at Rs. 10 crores.
Bid of Rs. 12.50 crores of M/s. Ceylon Biscuits Ltd. is, therefore, reasonable more
particularly when other bidders whose bids were not only lesser have already withdrawn
from the bidding process, this bid is hereby accepted.
Let balance payment be made by the successful bidder strictly in terms with the bidding
conditions and the amount would be deposited in the court. The amount so deposited
should be kept in FDR initially for a period of six months."
32. An intra-court appeal was preferred against the orders dated 17th July, 2004, 27th
July, 2004 and 30th July, 2004. The matter was listed on 26th August, 2004. Before the
appellate court also an offer was made by the appellant to bring a higher offer of Rs. 15
crores. Pursuant to an order made in this regard, a sum of Rs. 50 lakhs was directed to be
deposited. The Division Bench also directed maintenance of status quo in the meantime.
33. In the meantime, SICOM and Ceylon Biscuits both filed applications for possession
of the factory to be handed over. Such permission was granted on 13th October, 2004.
@page-SC2705
34. Various applications were filed before the Division Bench and/or this Court. Except
noticing that in the meantime another valuation report was filed on 21st November, 2006
in regard to the intangible assets of the company as being Rs. 35.88 crores which had
been sold by SICOM in favour of Ceylon Biscuits for a sum of Rs. 10 crores, we need not
take note of any other fact. By reason of the impugned judgment dated 2nd July, 2007 the
Letters Patent Appeal preferred by the appellant was dismissed and by an order dated 6th
July, 2007 the sale certificate was directed to be issued to M/s. Ceylon Biscuits.
It is these orders which are in question before us.
35. Mr. Kapur, the learned senior counsel appearing on behalf of the appellant inter alia
would submit :-
i) The learned Company Judge while proceeding to direct sale committed a serious
illegality in not directing a fresh valuation of the assets of the company and upon taking
into consideration the interest of other creditors as also that SICOM itself before
accepting the offer of M/s. Ceylon Biscuits.
ii) When a Provisional Liquidator was appointed, his involvement in the process of sale
was imperative in character.
iii) Provisions of Sections 441, 456, 450 and 457 read with Rule 293 of the Companies
Act show that the involvement of Official Liquidator was absolutely mandatory and the
Court could not, in the name of supervision over the sale, substitute itself in the place of
the Official Liquidator.
iv) The learned Company Judge completely disregarded the law laid down by this Court
in a series of decisions in each and every respect concerning the sale of the assets of a
company, in so far as :-
a) it did not issue any fresh advertisement;
b) the advertisement issued being in small print and no guidelines having been issued, the
same was irrelevant;
c) the Company Court did not fix any reserve price;
d) the Company Court did not make any attempt to secure the best possible market price
which was its duty to do for the sake of the general body of creditors including workmen
and other secured creditors.
v) The Company Court on the one hand appointed an independent valuer for valuing
appellant's intangible assets; on the other it simply relied upon two valuation reports
made by NITCON without application of mind about its correctness or otherwise.
vi) SICOM's action is mala fide as even it should not have been averse to the process of
sale of the factory of the appellant at a higher price, particularly when a memorandum of
agreement entered into by and between the appellant and Ceylon Biscuits show that the
actual value of the factory was very high as per the Ceylon Biscuits' own valuation report
dated 9th September, 2005.
vii) The learned Company Judge as also the Division Bench of the High Court proceeded
to determine the entire dispute only on the conduct of the appellant both in respect of
obtaining the Award of the Board of Conciliators as also its failure to secure a better price
and not on the basis of the legal principles involved in sale of assets of the company in
liquidation.
viii) As the Company was an ongoing concern, the Company Judge without involving the
Official Liquidator committed a serious error in directing sale of the assets of the
company at an early stage of the winding up proceeding without applying its mind that a
Scheme for revival of the Company was possible to be filed in terms of Section 391 of
the Companies Act.
36. Mr. Rajiv Shakdher, learned senior counsel appearing on behalf of SICOM, on the
other hand, urged :-
i) SICOM had never been averse to obtaining any higher price as would appear from the
proceedings before the High Court both in Execution Proceeding as also the Winding-up
Proceeding.
ii) SICOM had all along exercised its right to sell the mortgaged assets in exercise of its
statutory powers under Section 29 of the 1951 Act which being in consonance with the
principles and guidelines laid by this Court, could not have been interfered with.
iii) The appellant having questioned the action of SICOM in invoking its statutory
powers under Section 29 of 1951 Act by filing two writ applications and having
withdrawn the same, it was entitled to take possession of the properties which it did on
18th July, 2003.
iv) The appellant with a view to get back the possession of the factory forged a settlement
@page-SC2706
agreement to deceive SICOM in purported execution of the award of the Board of
Arbitration.
v) It took recourse to adventurous litigations not only by getting the aforementioned case
filed but also filing an application under Section 9 of the Arbitration and Conciliation
Act, 1996 with a view to get a Receiver appointed, although it did not succeed in that
attempt.
vi) It is not correct to contend that a Receiver was appointed by the Court in the
Arbitration proceeding but the Receiver was appointed by Debt Recovery Tribunal in
respect of perishable articles only.
vii) The Executing Court at the initial stage and subsequently the learned Company
Judge, merely supervised the sale with a view to bring about transparency in the entire
process.
viii) That when a sale is held by a Financial Institution in terms of Section 29 of the 1951
Act, opportunities are granted to the debtors to purchase the property at the price for
which the sale had been held or to bring a higher offer.
ix) With a view to satisfy the set norms, the High Court not only permitted Ceylon
Biscuits and another to take part in the bidding process but also gave opportunities after
opportunities to the appellant to bring a better offer which it failing and/or neglected to
comply with.
x) Appellant having undertaken to pay a sum of Rupees two crores and having failed to
comply with the same, it was not entitled to raise any objection in regard to the legality or
otherwise of the sale, particularly when it was on their suggestions, other bidders were
permitted to bid and the said bids were opened in the Court itself.
xi) The advertisement issued by SICOM was in accordance with the usual practice and it
is not correct to contend that no guideline was issued or bidders were not permitted to bid
(in accordance with the norms).
xii) NITCON is a Public Sector Organization with which SITCOM has no concern, thus
it would not be correct to contend that the second valuation report should not have been
obtained by it, particularly when the said valuation was in relation to the uninstalled
machinery lying at the factory premises in respect whereof the appellant moved the
learned Company Judge.
37. Mr. Sundaram, learned counsel appearing on behalf of respondent No.4 (Ceylon
Biscuits), would submit :-
i) SICOM had all along exercised its powers under Section 29 of the 1951 Act and the
Court merely supervised exercise of such powers and in that view of the matter the
appellant has not been prejudiced at all inasmuch as the same merely provided for
additional safeguard for fetching a proper price for the assets.

ii) In view of the decision of this Court in Rajasthan Fiancnial Corporation Ltd. and
another vs. The Official Liquidator, (2005) 8 SCC 190, the involvement of the Official
Liquidator is necessary only to sell the assets of the company in liquidation and as no
winding up order has been passed, involvement of Official Liquidator was not necessary.
2006 AIR SCW 113

iii) The Company Court exercised its jurisdiction in terms of Rule 293 of the Companies
(Court) Rules which permitted it to sell the assets itself or through an agent.
iv) If the learned Company Judge thought that SICOM should act as an agent, no
illegality can be set to have been committed by reason thereof.
v) Respondent No.4 being a bona fide purchaser, pursuant to an offer, it would be highly
prejudiced if the auction sale is set aside at this stage.
38. The core issues which arise for our consideration in view of the rival contentions of
the leaned counsel are :-
1) Whether in the facts and circumstances of the case the Executing Court and
consequently the Company Judge could have supervised the purported sale of the assets
of the appellant on behalf of SICOM having regard to the provisions of Section 29 of the
1951 Act?
2) Whether in a case of this nature and particularly having regard to the fact that SICOM
submitted itself to the jurisdiction of the executing court and company court, can now
turn around and contend that in effect and substance it had exercised its statutory powers
under Section 29 of the Act and allowed the same only to be supervised by the learned
Company Judge?
3) Whether the statutory powers of a Financial Corporation as envisaged under Section
29 of the 1951 Act would prevail over the proceedings before a Company Judge in
@page-SC2707
a winding up proceeding?
4) Whether involvement of the Official Liquidator in the facts and circumstances of the
case and particularly in view of the fact that Official Liquidator brought to the court's
notice claims of other creditors, the Company Judge ought to have dealt with the same in
the manner laid down in the Companies Act and/or the Rules framed thereunder and/or
the decision of this Court?
5) Whether the High Court while exercising its powers under Section 433 of the
Companies Act read with other provisions could ignore the claims of the other creditors,
and in particular the workmen, having regard to the provisions of Section 529A thereof.

6) Whether the High Court while exercising its jurisdiction both in the execution
proceeding as also winding up proceeding can, in the fact-situation obtaining herein, be
said to have adopted a fair procedure.
7) Whether in any event the High Court could have Ignored the legal requirements as
regards the conduct of sale of the assets of the appellant only on the basis of : (1)
wrongful conduct on the part of the appellant in obtaining an award from the Conciliation
Tribunal; and (2) its failure to bring a better offer from another bidder.
39. The 1951 Act indisputably is a special statute. If a financial corporation intends to
exercise a statutory power under Section 29 of the 1951 Act, the same will prevail over
the general powers of the Company Judge under the Companies Act.
40

. There cannot be any doubt whatsoever that the proceedings under Section 29 of the
1951 Act would prevail over a winding up proceeding before a Company Judge in view
of the decision of this Court in International Coach Builders Ltd. v. Karnataka State
Financial Corporation [(2003) 10 SCC 482] wherein it has been held : 2003 AIR
SCW 1524
2006 AIR SCW 113

"26. We do not really see a conflict between Section 29 of the SFC Act and the
Companies Act, at all, since the rights under Section 29 were not intended to operate in
the situation of winding up of a company. Even assuming to the contrary, if a conflict
arises, then we respectfully reiterate the view taken by the Division Bench of this Court
in A.P. State Financial Corpn. case. This Court pointed out therein that Section 29 of the
SFC Act cannot override the provisions of Sections 529(1) and 529-A of the Companies
Act, 1956, inasmuch as SFCs cannot exercise the right under Section 29 ignoring a pari
passu charge of the workmen...."

The view taken therein was reiterated by a three-Judge Bench of this Court in Rajasthan
State Financial Corporation and Anr. v. Official Liquidator and Anr (2005) 8 SCC 190,
wherein it was stated :2006 AIR SCW 113

"18. In the light of the discussion as above, we think it proper to sum up the legal position
thus :
(i) A Debts Recovery Tribunal acting under the Recovery of Debts Due to Banks and
Financial Institutions Act, 1993 would be entitled to order the sale and to sell the
properties of the debtor, even if a company-in-liquidation, through its Recovery Officer
but only after notice to the Official Liquidator or the Liquidator appointed by the
Company Court and after hearing him.
(ii) A District Court entertaining an application under Section 31 of the SFC Act will have
the power to order sale of the assets of a borrower company-in-liquidation, but only after
notice to the Official Liquidator or the Liquidator appointed by the Company Court and
after hearing him.
(iii) If a financial corporation acting under Section 29 of the SFC Act seeks to sell or
otherwise transfer the assets of a debtor company-in-liquidation, the said power could be
exercised by it only after obtaining the appropriate permission from the Company Court
and acting in terms of the directions issued by that court as regards associating the
Official Liquidator with the sale, the fixing of the upset price or the reserve price,
confirmation of the sale, holding of the sale proceeds and the distribution thereof among
the creditors in terms of Section 529-A and Section 529 of the Companies Act.
(iv) In a case where proceedings under the Recovery of Debts Due to Banks and
Financial Institutions Act, 1993 or the SFC Act are not set in motion, the creditor
concerned is to approach the Company Court for appropriate directions regarding the
realisation of its securities consistent with the relevant provisions of the Companies Act
regarding distribution of the assets of the company-in-liquidation."
@page-SC2708

[See also ICICI Bank Ltd. v. SIDCO Leathers Ltd. and Ors. 2006 (5) SCALE 27] 2006
AIR SCW 2361

But, in this case, the sale in favour of Ceylon Biscuits Pvt. Ltd. having not taken place in
terms of Section 29 of the 1951 Act, the said question cannot have any application
whatsoever.
It is, however, a case where the learned Company Judge was not authorized to exercise its
power under Section 29 of the 1951 Act. It purported to exercise its power only under the
Companies Act. SICOM submitted itself to its jurisdiction. It allowed the Company Judge
to conduct the sale. The sale that was conducted was purported to be in terms of the
Companies Act. We have noticed hereinbefore that when a provisional liquidator was
appointed, the High Court instead of exercising its writ jurisdiction referred the matter to
the Company Judge. It was the Company Judge, therefore, who proceeded in the matter.
The Company Judge could exercise its jurisdiction only in terms of the Companies Act
and not in terms of Section 29 of the 1951 Act. If it did not have the power under the
1951 Act, any decision purported to have been taken by it would be a nullity. SICOM
indisputably has a statutory power but it could waive the same. It preferred the conduct of
the auction at the hands of the Company Judge in stead and place of carrying on the same
by itself. It submitted itself to the jurisdiction of the Company Judge. Not only it took
part in the proceedings without any demur whatsoever, it actively participated therein. It
is only at its instance that the bid was held. The other bidders were also brought in.
It is, therefore, not a case where the learned Company Judge had no jurisdiction to
exercise supervision of sale of the assets of the appellant on behalf of SICOM in terms of
the provisions of Section 29 of the 1951 Act or otherwise. Respondents even never
insisted to get the question of jurisdiction determined as a preliminary issue, although
raised by it specifically. It, thus, for all intent and purport waived its right.
41. It is in the aforementioned situation, we must consider the question as to whether in
the facts and circumstances of this case, the involvement of Official Liquidator was
imperative.
42

. The Official Liquidator brought to the court's notice the claims of the other creditors.
The Company Judge having been exercising its jurisdiction under Section 433 of the
Companies Act was, thus, under a statutory obligation to consider the cases of all
creditors of the Company simultaneously. For the said purpose, the learned Company
Judge was bound to follow the provisions of the Companies Act and/or the Companies
(Court) Rules. The jurisdiction of a Company Court extends only to those matters which
are specified in the Companies Act and apart therefrom it had no jurisdiction. It also has a
duty to see that the claims of all creditors be dealt with, particularly having regard to the
provisions of Section 529A of the Companies Act. We are informed that the workers had
also filed their claims. Their claims having regard to a series of decisions of this Court
could not have been ignored. [See Allahabad Bank v. Canara Bank (2000) 4 SCC 406 and
Andhra Bank v. Official Liquidator and Ann (2005) 5 SCC 75]. 2000 AIR SCW 1347
2005 AIR SCW 1608

43. The claim of the workmen having regard to the special provision as contained in
Section 529A of the Companies Act is pari passu to the secured creditors of the Company.
Clause (11) of Section 2 of the Companies Act, 1956 provides for the definition of 'the
court'. In A. Ramaiya, 16th Edn. 2004, the learned author opines that the jurisdiction of a
companies court extends only to those matters which are specified in the Act and apart
from those matters it has no Jurisdiction.
44. The matter might have been otherwise if SICOM had remained outside the winding
up proceedings. If it attained, disposal of the assets of the Company would be subject to
pari passu claim of unpaid workmen in terms of Section 529A of the Companies Act.
45. The sale has been effected by the court treating SICOM as an agent. Factually the
court did not do so. Even otherwise, it is impermissible. It exercised its own jurisdiction.
It was bound to do so. There cannot be any doubt whatsoever that in the matter of control
over the assets of a company in liquidation, the courts exercise a wide jurisdiction. It may
not only take recourse to the sale of the assets of the company whether before or after it is
wound up, but also would be entitled to, nay obligated to, if the situation so warrants to
attempt to
@page-SC2709
rehabilitate the company itself.
While doing so, it exercises its parens patriae power. It safeguards not only the interest of
the mortgages, but also the interest of the mortgagor. It has a statutory obligation to
safeguard the interest of the workmen as also other non-secured creditors.
It is one thing to say as to how the assets shall be distributed but it is another thing to say
that while exercising the power to cause the sale of the assets of the company, it would
ignore the statutory provision. It must, while exercising its power, take into consideration
all relevant factors. The mode and manner as to how a sale would be conducted is one
thing but it is another thing that before putting the assets of the company to sale, the court
will undertake certain obligations which are inherent in exercise of its jurisdiction under
the provisions of the Companies Act.
46. We will assume that the court could appoint SICOM as an agent but apart from the
fact that it, in fact, did not do so, we are inclined to hold that the stand of the learned
counsel is mutually destructive. On the one hand, it is stated that SICOM was exercising
its statutory power to cause sale of the assets of the mortgagor through the agency of the
court but it is also contended that the sale was effected by the court through SICOM.
Such a contradictory or inconsistent stand, in our opinion, is impermissible in law.
47

. In NGEF Ltd. v. Chandra Developers Pvt. Ltd. and Anr., [(2005) 8 SCC 219], this Court
opined : 2005 CLC 1602

The Company Judge moreover will have to bear in mind the provisions contained in
Section 529A of the Companies Act in terms whereof the dues of the workman and the
debts due to the secured creditors to the extent such debts rank in clause (c) of the proviso
appended to sub-section (1) of Section 529 pari passu therewith and shall have a priority
over all other debts."

In A.P. State Financial Corporation v. Official Liquidator [(2000) 7 SCC 291], this Court
held : 2000 AIR SCW 2804, Para 10

"Under the proviso to sub-section (1) of Section 529, the liquidator shall be entitled to
represent the workmen and force the above pari passu charge. Therefore, the Company
Court was fully justified in imposing above conditions to enable the Official Liquidator
to discharge his function properly under supervision of the Company Court as the new
Section 529A of the Companies Act confers upon a Company Court a duty to ensure that
the workmen's dues are paid in priority to all other debts in accordance with provisions of
the above Section. The Legislature has amended the Companies Act in 1985 with a social
purpose viz. to protect dues of the workmen. If conditions are not Imposed to protect the
right of the workmen there is every possibility that secured creditor may frustrate the
above pari passu right of the workmen."
At this stage we may also notice a decision of Three-Judge Bench of this Court in Andhra
Bank (supra) wherein this Court had to consider the correctness of the decision in
Allahabad Bank (supra). The questions therein, inter alia, to be decided were : 2005
AIR SCW 1608
2000 AIR SCW 1347

"Whether after a winding-up order is passed under Section 446(1) of the Companies Act
or a provisional liquidator is appointed, whether the Company Court can stay proceedings
under the RDB Act, transfer them to itself and also decide questions of liability, execution
and priority under Section 446(2) and (3) read with Sections 529, 529-A and 530 etc. of
the Companies Act or whether these questions are all within the exclusive jurisdiction of
the Tribunal?"
This court after referring to the provisions of Sections 529 and 529-A stated the law in the
following terms :
"In terms of the aforementioned provisions, the secured creditors have two options (i)
they may desire to go before the Company Judge; or (ii) they may stand outside the
winding up proceedings. The secured creditors of the second category, however, would
come within the purview of Section 529-A(1)(b) read with proviso (c) appended to
Section 529(1). The 'workmen's portion' as contained in proviso (c) of sub-section (3) of
Section 529 in relation to the security of any secured creditor means the amount which
bears to the value of the security in the same proportion as the amount of the workmen's
dues bears to the aggregate of (a) workmen's due, and (b) the amount of the debts due to
all the creditors."
Thus, the High Court could not have dis-regarded
@page-SC2710
regarded the pari passu charge of the workmen upon the company's assets.
48. The role of the Official Liquidator in a situation of this nature assumes great
importance.
49. Chapter II of the 1956 Act deals with winding up of a company by the court. Section
433 provides for winding up, inter alia, by two modes. One, if the company has by
special resolution resolved that it should be wound up by the court; or (2) if the company
is unable to pay its debts.
An application for winding-up is to be filed in terms of Section 431 of the Act. Section
441 provides that winding up of a company by the court shall be deemed to commence at
the time of presentation of petition for winding up. The provision has since been omitted
by Companies (Amendment) Act, 2002. Section 442 provides for the power of the court
to stay or restrain proceedings against the company, Section 443 envisages power of the
court on hearing petition. Section 446 provides for stay of all suits. Sub-section (3) of
Section 446 reads as under :
"S. 446. Suits stayed on winding up order.-
(1) ...
(2) ...
(3) Any suit or proceeding by or against the company which is pending in any Court other
than that in which the winding up of the company is proceeding may, nothwithstanding
anything contained in any other law for the time being in force, be transferred to and
disposed of by that court."
50. The Execution Court being a co-ordinate court (as the Execution Petition was filed in
the High Court itself) transferred the same to the Company Judge having regard to the
fact that a provisional liquidator was appointed. Sub-section (4) of Section 446, therefore,
has no application as the proceedings before the Executing Court was not a matter which
came up in appeal from a judgment and order of another Court. Section 447 provides for
the effect of winding up order.
51. Section 448 provides for appointment of 'Official Liquidator'. An Official Liquidator
would be a liquidator on a winding up order being made in respect of a company. Section
450 provides for appointment and powers of provisional liquidator; sub-sections (1), (2)
and (3) whereof read as under :
"Section 450- Appointment and powers of provisional liquidator- (1) At any time after the
presentation of a winding up petition and before the making of a winding up order, the
[Tribunal] may appoint the Official Liquidator to be liquidator provisionally.

(2) Before appointing a provisional liquidator, the Tribunal shall give notice to the
company and give a reasonable opportunity to it to make its representations, if any,
unless, for special reasons to be recorded in writing, the Tribunal thinks fit to dispense
with such notice.
(3) Where a provisional liquidator is appointed by the Tribunal, the Tribunal may limit
and restrict his powers by the order appointing him or by a subsequent order, but
otherwise he shall have the same powers as a liquidator."
52. Section 456 envisages that when a winding up order has been made or where a
provisional liquidator has been appointed, the liquidator or the provisional liquidator, as
the case may be, shall take into his custody nay his control of the property, assets and
actionable claims to which the company is or appears to be entitled. It is true that the
Court had not permitted the provisional liquidator to take over the assets. It protected the
possession of SICOM. But the same by itself would not mean that the provisional
liquidator was denied from performing its other functions.
Section 457 provides for the powers of liquidator. It is in two parts, one which had to be
exercised with the sanction of the tribunal and the other which had to be exercised by
itself. A liquidator, in terms of clauses (c) and (ca) is entitled to sell the moveable and
immoveable property. Exercise of such jurisdiction by a provisional liquidator, therefore,
shall not be denied of his powers only because it did not obtain possession of the
properties. Section 529 of the Act which occurs in Chapter V provides for application of
insolvency rules in winding up proceeding of the insolvent companies.
Section 529A expressly saves the rights of the workmen. It contains a non obstante
clause. A statutory pari passu charge is created in support of the dues of the workmen
being equivalent to the dues of a secured creditor for the purpose of enforcing the
insolvency rules as contained in clause (c) of sub-section (1) of Section 529.
Section 538 of the Companies Act provides
@page-SC2711
for offences by officers of companies in liquidation.
53. The rights, jurisdiction and powers of the provisional liquidator may not be the same
as that of an Official Liquidator.
But in a case of this nature, only because the financial institution stands outside the
winding up proceedings, would it mean that the Court shall, for all intent and purport,
ignore its officer and concentrate on the interest of the financial institution alone? Can it
be said that supervision of the court is necessary only in a post-winding scenario and not
prior to it? The question which should be addressed, in our opinion, by the Company
Court is that the ultimate interest of both secured and non-secured creditors must be kept
in mind. Should Court have exercised its jurisdiction for directing the sale of the prime
property and, in fact, the essence of the assets of the appellant at the initial stage. The
answer, in our opinion, should be rendered in the negative.
54. The Chancery Division in Re : Dry Docks Corporation of London [1888 (39)
Chancery Division 88], wherein Fry J. held
"But then there are circumstances which, in my opinion, vary the rights of the parties. On
the 8th of March a provisional liquidator had been appointed. Now the provisional
liquidator's appointment is not only provisional, but contingent in this sense, that it
operates to protect the property for an equal distribution only in the event of an order for
compulsory winding-up being made; and if no such order be made, then his appointment
ought not to interfere with the rights of third persons. He was in the position of a receiver,
whose appointment might interfere with the rights of third persons. Now with regard to
that, the practice of the Court is perfectly plain, as was stated by Lord Truro, in the case
of Russel v. East Angilan Railway Company n(1), in very clear terms. He said : "I
apprehend then it may be taken as a rule that, though this Court may have issued a
process or have made an order which may interfere with the supposed rights and interests
of other parties not parties to the cause, it is always competent for such parties to make an
application to the Court for relief; and it is not to be presumed or doubted, but that justice
will be duly administered to them on that application."
55. The courts in India have to keep in mind different considerations. The concept of
right of property which was existing in 19th Century in England would not stand the test
of the act and the interpretation it deserves keeping in view the object and purport of the
1956 Act. In India, the Company Courts have a statutory duty to protect and rights of
workmen keeping in view the pari passu charge created in their favour in terms of Section
529A of the Act. Power and functions of a provisional liquidator subject to the limitations
imposed by the court are the same as that of an Official Liquidator.
56. It is furthermore not a case where the rights of third persons were involved. We have
held hereinbefore that SICOM failed to keep itself outside the winding up proceedings. It
has become a parry to it and, thus, when a sale is held by a Company Judge, it should not
keep a provisional liquidator out of its purview. It may be true that the provisional
liquidator could not sell the property without the sanction of the Court, but then feed back
of the provisional liquidator by the Company Court was necessary for the purpose of
having a complete picture before it.
The Official Liquidator has informed us that about 373 claims have been filed. The
amount of claim is about 100 crores; amongst the claimants, there are banks in whose
favour also deeds of mortgages have been executed. Provident Fund dues and other dues
of statutory claims are also subject-matter of the claim petition. They also have a priority.
The claim of the provident fund is on behalf of the workmen. For scrutiny of the said
claims, a Committee has been constituted and we had been informed that except the
properties which have been sold in liquidation, there is hardly any other asset upon which
the creditors can bank upon for the purpose of realization of their dues.
57. It is true that in a liquidation petition, secured creditors ought to be differently treated.
A third party who has an independent right would not be affected by reason thereof.
Ordinarily, even the statutory power of the said financial corporation would also not be
affected.
58. We, however, are not in a position to agree with the submissions of Mr. Sundaram
that provisional liquidators have no statutory powers in relation to effecting sale of a
moveable or immoveable property. Indisputably, it is subject to the direction of the Court
but, as indicated hereinbefore, the Court while undergoing the process of winding up
@page-SC2712
and, in any event, resorting to sale of the assets of the company under winding up
proceeding could not have ignored the involvement of the provisional liquidator for any
purpose whatsoever.
At the cost of repetition, it is reiterated that the discretion of the Court for selecting the
mode and manner of sale has nothing to do with the process required to be gone into for
the said purpose.
It must have before it all these facts and figures so as to enable it to pass a final order one
way or the other. In so doing, the Court must keep in mind that it is not only determining
an issue by and between the mortgagor and one mortgagee only but could also be
determining the issue between a debtor and a vast number of creditors; whether secured
or non-secured.

The ratio of the decision of the Madras High Court in Sri Chamundi Theatre Mysore
Talkies Ltd. v. S. Chandrasekara Rao [1975 (45) Company Cases 60] whereupon reliance
has been placed by Mr. Sundaram may be noticed. In that case, an advocate was
appointed as a provisional liquidator. The distinction between appointment of an Official
Liquidator as a provisional liquidator and an advocate as a provisional liquidator must be
viewed differently. When an Official Liquidator is appointed as a provisional liquidator,
the purpose is that he must become aware of all the processes of winding up leading to
exercise of his statutory power, if ultimately the courts find it just and equitable to direct
the winding up of a company. In that case, the application for winding up was not pressed
by the petitioner-creditor. 1974 Tax LR 2406

Provisional liquidator, however, was directed to continue unless he hands over the charge
to the Managing Director to be elected in terms of the order passed by the learned
Company Judge. The provisional liquidator, in view of the orders of the court, ceased to
be in judicial control or statutory control over the properties of the company.
Interpretation of Section 450 as opined by the learned Judges of the Madras High Court
must be viewed from the aforementioned factual matrix in mind.
It is not the law nor has such a proposition been canvassed before us that the properties
vested in the provisional liquidator, as was the submission in that case. But then,
however, the learned Judges opined that the appointment and power of an Official
Liquidator is controlled by the instrument which appoints him and that his office is not in
equation to that of an Official Liquidator, the same, however, would not mean that even
when there does not exist such limitation, the services of provisional liquidator shall not
be resorted to.
59. Strong reliance has been placed on in Re : A.I. Levy (Holdings) Ltd. [1964 (1)
Chancery Division 19].
60. We may at this stage notice the statutory provisions as regards the provisional
liquidator in the United Kingdom. The Insolvency Act, 1986 governs the winding up
proceedings in England and Wales.
Briefly stated the scheme of the said Act is as under :
The expression "office-holder" is defined in Section 234(1). It means the administrator,
the administrative receiver, the liquidator or the provisional liquidator, as the case may
be. For the purposes of Section 236 the expression includes, in the case of a company
which is being wound up by the court in England and Wales, the official receiver,
whether or not he is the liquidator.
Under the heading "The liquidator's functions" Section 143 of the Insolvency Act
describes the general functions of the liquidator in a winding up by the court as follows :
"General functions in winding up by the Court
(1) The functions of the liquidator of a company which is being wound up by the court
are to secure that the assets of the company are got in, realised and distributed to the
company's creditors and, if there is a surplus, to the persons entitled to it.
(2) It is the duty of the liquidator of a company which is being wound up by the court in
England and Wales, if he is not the official receiver -
(a) to furnish the official receiver with such information,
(b) to produce to the official receiver, and permit inspection by the official receiver of,
such books, papers and other records, and
(c) to give the official receiver such other assistance, as the official receiver may
reasonably require for the purposes of carrying out his functions in relation to the
winding up."
In Official Receiver (Appellant) v. Wadge Rapps and Hunt (a firm) and another and two
@page-SC2713
other actions [2003] UKHL 49, the question which was to be decided by the House of
Lords was whether the official receiver can have recourse to the powers conferred by
Section 236 of the Insolvency Act, 1986 ("the Insolvency Act") for the sole purpose of
obtaining evidence for use in disqualification proceedings against a former director.
Observing the functions of the liquidator vis-a-vis disqualification proceedings envisaged
under Section 236 of the Act, Lord Millett opined :
"The first of these strands proceeds from the premise that the powers conferred by
Section 236 are conferred on a liquidator "for the better discharge of his functions in the
winding up". These words are not derived from the express terms of the Section but are
evidently considered to be implicit in it. The unspoken assumption is that a liquidator's
"functions in the winding up" are limited to the collection and distribution of the
company's assets. I agree that the bringing of disqualification proceedings is not a
function which is conferred on the official receiver "in the winding up"; if it were, the
costs of the proceedings would be payable out of the assets of the estate. It is not
necessary to consider whether the gathering of evidence for the purpose of such
proceedings is part of "his functions in the winding up", for this formulation is unduly
narrow. The liquidator's functions in relation to the company which is being wound up
are not and never have been limited to the recovery and distribution of the company's
assets. It would be very odd if the liquidator of a company in voluntary liquidation could
apply to the court to direct a public examination in the wider public interest but could not
invoke Section 236 to order a private examination in the same interest. In practice the
liquidator would usually prefer to invite the official receiver to make the application; and
even where the application was made by the liquidator the court would be disposed to
invite the views of the official receiver. But it is impossible to say that the liquidator
would be acting outside his proper role in the one case and not in the other.
Section 236 contains no express limitation on the purpose for which it may be invoked.
Of course it may be invoked only for a legitimate purpose in relation to the company
which is being wound up, and the Court, which has discretion to make or refuse an order,
should be astute to see that the powers conferred by the Section are not abused. It would
plainly be an abuse to use those powers for a purpose which is foreign to the functions of
the applicant in relation to the company which is being wound up. But I reject the
unspoken assumption that the functions of a liquidator are limited to the administration of
the insolvent estate. This is only one aspect of an insolvency proceeding; the
investigation of the causes of the company's failure and the conduct of those concerned in
its management are another. Furthermore such an investigation is not undertaken as an
end in itself, but in the wider public interest with a view to enabling the authorities to take
appropriate action against those who are found to be guilty of misconduct in relation to
the company. If the investigation yields information material to the Secretary of State's
decision to bring or continue disqualification proceedings, it must be reported."
It was furthermore opined :
"In my opinion, the only limitation which is implicit in Section 236 is that it may be
invoked only for the purpose of enabling the applicant to exercise his statutory functions
in relation to the company which is being wound up. Whether the applicant is the official
receiver or the liquidator or other office-holder these include the provision of information
to the Secretary of State or the official receiver which is relevant to the bringing or
continuing of disqualification proceedings."
61. Interestingly, Mr. Rajiv Shakdher has made extensive reference from Farar's
Company Law, Third Edition to contend that as the appellant had defaulted in payment of
its dues to various secured and non-secured creditors including SICOM, it was admittedly
heading towards insolvency and in that view of the matter, the assets of the company
were really in a practical sense their assets and not the assets of the creditors. We may
notice the observations made by the learned author :
"As we have seen, directors do not owe duties to shareholders as such. Neither do they
owe duties to the company's creditors. The orthodox position being as stated by Dillon LJ
in Multinational Gas and Petrochemical Co. v. Multinational Gas and Petrochemical
Services Ltd. [1983 Ch 258] directors owe fiduciary duties to the company though not to
the creditors, present or future, or individual shareholders.
@page-SC2714
Winkworth v. Edward Baron Development Co. Ltd. [(1987) 1 All ER 114], a House of
Lords decision, might suggest that there has been a change to that position with Lord
Templeman stating :
'...a company owes a duty to its creditors, present and future. The company owes a duty
to its creditors to keep its property inviolate and available for repayment of its debts. The
conscience of the company, as well as its management, is confided to its directors. A duty
is owed by the directors to the company and to the creditors of the company to ensure
that the affairs of the company are properly administered and that its property is not
dissipated or exploited for the benefit of the directors themselves to the prejudice of the
creditors'."
The learned author furthermore observed :
"Support here for this approach can be found in West Mercia Safetywear Ltd. v. Dodd
[(1986) 4 ACLC 215] where Dillon LJ approved the following statement of the position
by the New South Wales Court of Appeal in Kinsela v. Russell Kinsela Pry Ltd. [(1989)
AC 755] :
'In a solvent company the proprietary interests of the shareholders entitle them as a
general body to be regarded as the company when questions of the duty of directors arise.
If as a general body, they authorize or ratify a particular action of the director, there can
be no challenge to the validity of what the directors have done. But where a company is
insolvent, the interests of the creditors intrude. They become prospectively entitled
through the mechanism of liquidation, to displace the power of the shareholders and
directors to deal with the company's assets. It is in a practical sense their assets and not
the shareholders' assets that through the medium of the company are under the
management of the directors pending either liquidation, return to solvency, or the
imposition of some alternative administration'."
62. This is the meet of the matter. If the property which has been put to auction was the
prime property over which the fate of the creditors depended, be they secured or non-
secured ones, the company Court, in exercise of its equity jurisdiction could not have
obliterated it from its mind the cases of the others. If the assets belong to the creditors,
that must mean the whole body of the creditors and not only one of the secured creditors.
The inconsistency is self-evident, as, on the one hand, it is stated that the property of the
company does not vest in the Court or the Official Liquidator, on the other hand, it is
stated that it is vested in the body of the creditors and not only in SICOM.
63. The High Court, therefore, could not have ignored the Official Liquidator only on the
ground that a provisional official liquidator was appointed and not a regular Official
Liquidator. The power and functions of the provisional official liquidator for all intent
and purport would be the same as that of the Official Liquidator and, therefore, it was not
necessary for the Company Judge to wait till the Company was wound up.
64. If the jurisdiction of a Company Judge is limited, any substantial deviation and
departure therefrom would result in unfairness. When an order is passed in total disregard
of the mandatory provisions of law, the order itself would be without jurisdiction. In this
case, however, even otherwise a fair procedure was not adopted. We, however, very much
appreciate the anxiety on the part of the Court to see that otherwise just dues of SICOM
be realized. Conduct of a party plays an important role in the matter of grant of a relief.
However, only because the conduct of a party was not fair, the same, by itself, cannot be
a ground to adopt a procedure which is unjust or unfair, particularly, when by reason
thereof, not only the Company itself but also other creditors are seriously prejudiced. We
fail to see any reason as to why the hearing of the case was to be preponed. Why even a
day's time could not have been granted when a prayer for adjournment was made. The
jurisdiction of the Company Court is vast and wide. It can mould its reliefs. It may
exercise one jurisdiction or the other. It may grant a variety of reliefs to the parties before
it. The parties before the Company Judge are not only the Company or the creditors who
had initiated the proceedings but also others who have something to do therewith. Even
in a given case a larger public interest may have to be kept in mind. The court may direct
winding up. It may prepare a scheme for its restructuring.
65. We, therefore, are of the opinion that the Company Judge was not correct in its view
and passed the impugned judgments only having regard to the wrongful conduct
@page-SC2715
on the part of the appellant in obtaining an award from the conciliation tribunal or failure
to bring a better offer from another bidder.
66. The question which is really an intricate one is what relief can be granted. On the one
hand, the Company has committed wrongs, on the other, its property has been sold in
auction. Even a part of the property has been permitted by us to be taken out of the
country. The factory, we are told, has started operation. It has employed a large number of
workmen. Would that itself mean that we should refrain ourselves from granting any
relief? Direction issued by this Court in a case of this nature need not be a narrow one.
The Court has to take into consideration the fate of not only those workmen who are
working but also those who have a claim against the Company. We must also take into
consideration the fate of the other creditors.
67. We, therefore, are of the opinion that interest of justice would be subserved if while
allowing the appeal, the learned Company Judge is requested to go into the question
afresh in accordance with the provisions of the Companies Act and hold a fresh auction.
While doing so, indisputably, Ceylon Biscuits Pvt. Ltd.'s offer would be considered. The
Company Judge may consider the question of grant of some preference to Ceylon
Biscuits Pvt. Ltd. but while an auction is to be held, there should be a proper valuation of
all the assets of the Company both movable and immovable.
The Court, indisputably, may consider the question of framing an appropriate scheme if it
is found that there is a possibility of revival of the Company. In other words, we leave all
options open to the learned Company Judge as are available in terms of the provisions of
the Companies Act including adjustment of equities amongst the parties.
Till, however, a final order is passed, Ceylon Biscuits Pvt. Ltd. would continue to
function not as an auction-purchaser but as a Receiver of the Company Court. Ceylon
Biscuits Pvt. Ltd. shall file all statement of accounts in regard to the amounts which it had
invested and all other requisite statements including the valuation of machinery it had
taken out of the country before the Court. The Court may appoint a Chartered Accountant
to verify the said statements.
The court, if it thinks fit and proper, may, apart from the provisional liquidator, appoint
another person to supervise the works and functioning of Ceylon Biscuits Pvt. Ltd. as a
receiver of the Court. As Ceylon Biscuits Pvt. Ltd. is being appointed as a receiver, it
goes without saying that it shall act strictly under the supervision of the court and abide
by the orders which may be passed by it from time to time.
68. For the reasons aforementioned, the appeals are allowed to the aforementioned extent.
In the facts and circumstances of the case, however, there shall be no order as to costs.
Order accordingly.
AIR 2008 SUPREME COURT 2715 "Purushottam Das Bangur v. B. Majumdar
Samajpati and Sons Hotel (P) Ltd."
(From : Calcutta)
Coram : 2 TARUN CHATTERJEE AND HARJIT SINGH BEDI, JJ.
Petition for Special Leave to Appeal (C) No. 16193 of 2007, D/- 25 -3 -2008.
Purushottam Das Bangur and Ors. v. B. Majumdar Samajpati and Sons Hotel (P) Ltd.
Transfer of Property Act (4 of 1882), S.108 - LEASE - TENANCY - SPECIAL LEAVE
PETITION - Tenant unauthorisedly using terrace of tenanted property - Claim by
landlord for payment of additional amount by tenant - Rejected by High Court - S.L.P.
against - Court without going into question of legality of user of terrace by tenant -
Directed tenant to deposit Rs. 25,000/- p.m. in Court. (Para 6)
Cases Referred : Chronological Paras
2004 AIR SCW 7349 : AIR 2005 SC 1444 (Disting.) 8
Brij Bhusan, for Appellant; Jaideep Gupta, Sr.Advocate, D. N. Ray and Mrs. Sumita Ray,
with him for Respondent.
Judgement
1. ORDER :- This special leave petition is filed against the order dated 24th of April,
2007 passed by the High Court at Calcutta in APOT No. 44 of 2007 whereby the High
Court had dismissed the appeal filed by the appellant challenging an order passed by a
learned Judge sitting in the original side jurisdiction of the High Court.
2. The petitioners are trustees of a trust named Gouri Devi Trust, which at all times was
and still is the absolute owner of premises situated at 95A Chittaranjan Avenue, Calcutta-
700 073 (in short the disputed premises).
@page-SC2716
The disputed premises consists of a three storied building and comprises an area of 1
Bigha 4 Cottahs of land and is situated in the heart of the city. A portion of this building
comprising 11 rooms on the 1st floor and 11 rooms on the 2nd floor including 2 kitchen
rooms was let out by the then trustees of the said Trust to one M/s. B. Majumdar
Samajpati at a monthly rent of Rs. 1610/- In the year 1992, at the request of the said B.
Majumdar Samajpati, the tenancy in respect of the tenanted portion was transferred by
the then trustees in favour of B. Majumdar Samajpati and Sons Hotels Pvt. Ltd. which is
now respondent before us in this petition on the same terms and conditions contained in a
letter dated 18th of November, 1992 with effect from December, 1992, On 5th of
October, 2005, the petitioners had filed the instant suit in the original side of the High
Court for a decree of Rs. 12 lakhs and for mandatory and perpetual injunction and for
recovery of possession, and for damages of mesne profit which have been described in
the prayer portion of the plaint and for other incidental reliefs.
3. In this suit on an application filed before a learned Judge, an advocate Commissioner
was appointed to make inventory of the demised premises and also passed an order of
status quo. The advocate Commissioner after due notice to the parties inspected the
demised premises and made a detailed inventory of the same in the presence of both the
parties and on 15th of November, 2005 submitted a detailed report which is already on
record. On a perusal of the said report, it reveals that :
(1) The demised premises is used for running hotel in the name and style of "Hotel
Avenue Club".
(2) In the third floor :
(i) a makeshift store room of the western side of the terrace (made up to bamboos and
canvas) are being used as a makeshift store room in which fridge and freezers and other
utensils dumped on the north western corner of the said makeshift of the store room.
From the said report, it also appears that a makeshift mezzanine made of plywood having
canvases and other utensils dumped on top of it.
(ii) On the eastern side of the terrace, there is a room which is also used as a store room
having a number of racks inside it with books, papers and soft drink bottles. (10' x 6')
(iii) The rear wall of the room on the eastern side is a common wall of another room
adjacent to the front room.
The rear room is also used as a store room and it is filled with utensils for buffet. (8' x 5')
iv) 2 rooms adjacent to the front room (both 14" x 10')
v) There is a small passage of terrace on the eastern side of the terrace, which leads to the
southern wall of the premises.
vi) There is one small room for ironing clothes and two small toilets. From there if one
turns to the northern side and walks towards the stair case, one will find a kitchen which
is approximately 30' x 15'. (2 rooms made into one by demolishing the partition wall
inside).
vii) A huge satellite TV antenna having a diameter of 14 feet mounted on top of the room,
which is in front of the stair case on the eastern side.
4. A perusal of the report of the Advocate Commissioner, as noted herein above, would
show that he found only one room in front of the staircase on the eastern side and two
rooms on the southern side. He also noted that almost the entire terrace was converted by
the respondent for the purpose of the hotel business and the character of the terrace had
been changed considerably.
5. From the aforesaid report of the Advocate Commissioner, it would also be apparent
that the respondent has unauthorisedly occupied terrace of the demised premises and has
been using by making makeshift arrangement for the purpose of running the hotel. In that
view of the matter, an application was filed by the petitioners before the learned single
Judge of the High Court to direct the respondent to deposit additional amount as the
respondent has wrongfully and illeglly trespassed the property of the petitioners and not
paying any amount with regard thereto. The trial Court as well as the appellate Court on a
prima facie finding did not come to a conclusion in favour of the petitioners and the
application has been rejected by the learned single Judge as well as by the Division
Bench of the High Court. This special leave petition has been filed by the petitioners
against the aforesaid orders of rejection which has been heard in the
@page-SC2717
presence of the learned counsel for the parties after issuing notice.
6. In our view, the only order that can be passed in the special leave petition is to direct
the petitioners to deposit a sum of Rs. 25,000/- every month in Court without going into
the merits as to whether the respondents are entitled to use the terrace and also the
makeshift arrangement made by him on the same for the purpose of running the hotel in
addition to their tenancy. It is no doubt true that the Court will decide at the final stage
whether the construction or the makeshift arrangement made in the terrace and other
portions of the demised premises by the respondents were unauthorized or not and
whether the terrace and other places where makeshift arrangement has been made by the
respondent for the purpose of running the hotel would be considered at the time of final
decision of the suit. Therefore, after hearing the learned counsel appearing for the parties
and after going through the allegations made in the application and the objections filed
therein and the report of the Advocate Commissioner, in our view, it would be fit and
proper that by an interim measure a direction must be given to the respondent to deposit
Rs. 25,000/- every month in Court within 15th of each succeeding month, the first of such
deposit shall be made by 15th of April, 2008 and in default of any of the two deposits, it
would be open to the petitioners to recover the amount from the respondent in accordance
with law.
7. We make it clear that we have not gone into the merits as to whether the tenancy
consists of terrace and the makeshift arrangement of rooms in the terrace would be
included in the tenancy agreement of the respondent which shall be decided at the final
hearing of the suit. We also make it clear that direction to deposit the rent of Rs. 25,000/-
per month to use the terrace rooms and other makeshift arrangement would not mean that
the appellants are not the tenants of the said rooms which are now being used by them for
running the hotel business and we keep it open for the parties to lead evidence on that
question which shall be decided after taking evidence at the final hearing of the suit.
8

. Before parting with this order, it would be necessary for us to refer to a decision of this
Court in the case of Metro Marins and Anr. v. Bonus Watch Co. (P) Ltd. and Ors., ((2004)
7 SCC 478) which was strongly relied by Mr. Jaideep Gupta learned senior counsel
appearing for the respondents. Since, we are not granting any order of injunction against
the respondent excepting that a direction has been made on the respondent to deposit Rs.
25,000/-per month, we are not in a position to rely on this judgment for the purpose of
deciding this special leave petition. 2004 AIR SCW 7349

9. With the above direction, the special leave petition is disposed of and the order of the
High Court is modified to the extent indicated above.
Order accordingly.
AIR 2008 SUPREME COURT 2717 "State Bank of India v. Mela Ram"
(From : 2001 (1) Shim LC 335 (Him Pra))
Coram : 2 TARUN CHATTERJEE AND HARJIT SINGH BEDI, JJ.
Civil Appeal No. 7520 of 2001, D/- 27 -2 -2008.
State Bank of India v. Mela Ram and Ors.
Constitution of India, Art.136 - SPECIAL LEAVE PETITION - DECREE - Special leave
petition - Discretionary jurisdiction - Petition against decree passed in favour of
appellant-petitioner - Decree already satisfied and execution case disposed of - Court
refused to exercise discretionary powers under Art.136. (Para 2)

Anil Kumar Sangal, Ms. Nalin Sangal and Deba Prasad Mohanty, for Appellant; Gaurav
Jain and Ms. Abha Jain, for Respondents.
Judgement
JUDGMENT :- Heard learned counsel for the parties.
2. In our view, in the facts and circumstances of the present case, we are not inclined to
exercise our power under Article 136 of the Constitution of India because we find from
the record that the decree passed in favour of the appellant has already been satisfied and
the execution case has already been disposed of. Accordingly this appeal is dismissed.
There will be no order as to costs.
Appeal dismissed.
@page-SC2718
AIR 2008 SUPREME COURT 2718 "Arjan Singh v. Punit Ahluwalia"
(From : Punjab and Haryana)*
Coram : 2 S. B. SINHA AND LOKESHWAR SINGH PANTA, JJ.
Civil Appeal No. 3573 of 2008 (arising out of SLP (C) No. 605 of 2007), D/- 14 -5 -2008.
Arjan Singh v. Punit Ahluwalia and Ors.
(A) Civil P.C. (5 of 1908), O.23, R.3 - COMPROMISE - DECREE - AGREEMENT TO
SELL - IMMOVABLE PROPERTY - Consent decree - Validity - Two suits for specific
performance of agreement of sale of same property by two purchasers, one of them was
appellant - Two purchasers impleaded in each other's suit - Consent decree passed in
terms of compromise in other purchaser's suit - Appellant not being party in such
compromise was not bound by a consent decree - Compromise also hit by S.52 of
Transfer of Property Act and S.19 of Specific Relief Act - Court can pass discretionary
order u/S.20 of Specific Relief Act, in suit filed by appellant.
C.R. No. 947 of 2006, D/-10-10-2006 (PandH), Reversed.
Transfer of Property Act (4 of 1882), S.52.
Specific Relief Act (47 of 1963), S.19, S.20.
Where two suits for specific performance of agreement for sale were filed in respect of
same suit property and the two purchasers - appellant and other purchaser were
impleaded in suits filed by each other and a consent decree was passed in suit filed by
other purchaser according to terms of compromise between him and owner of property
which was not signed by appellant, the compromise could be said to have entered under
first part of R. 3 of O. 23 and would not be binding on appellant. The appellant had a
rival claim. The suit filed by him was required to be considered together. The Court could
exercise its discretionary jurisdiction in one of the suits or the other, having regard to S.
20 of the Specific Relief Act, 1963. By reason of a compromise or otherwise, the claim of
the appellant could not have been defeated. Moreover, the suit would be hit by the
doctrine of lis pendens, as adumbrated under Section 52 of the Transfer of Property Act.
The said deed of sale would not come in the Court's way in passing a decree in favour of
the appellant. Its validity or otherwise would not be necessary to be considered as the
appellant was not bound thereby. Said other purchaser would be deemed to be aware of
the pendency of the suit. Even Section 19 of the Specific Relief Act will be attracted.
C.R. No. 947 of 2006, D/-10-10-2006 (PandH), Reversed. (Paras 8, 11, 16, 20)
(B) Civil P.C. (5 of 1908), O.39, R.2A - INJUNCTION - AGREEMENT TO SELL -
DECREE - SALE DEED - Breach of injunction - Suit for specific performance of
agreement for sale - Interim injunction against defendant from selling property for limited
period - Application for extension filed - No order was passed - Consent decree passed in
another suit filed by another purchaser in respect of same property against same
defendant on basis of compromise entered into between parties of that suit - Sale deed as
per compromise cannot be held to be in violation of interim order.
Where in a suit for specific performance of agreement for sale, an interim order for
injunction for restraining defendant from selling property was passed for a particular
period and though an application for extension of period was filed by the plaintiff,
however, no order was passed thereon, the consent decree passed in another suit for
specific performance filed in another suit filed by another purchaser in respect of same
property against same defendant cannot be held to be in violation of interim order passed
as aforesaid. It cannot be said in such a case that plaintiff cannot be made to suffer for
violation of the order of the Court. As no order of injunction was operative, the Court
cannot pass an order of injunction with retrospective effect so as to take away the right of
the parties created for the said purpose. To the said effect, the Court must make a
distinction between an incidental proceeding and a supplemental proceeding. An order of
injunction can be passed in terms of the provisions of the supplemental proceedings
contained in Section 94 of the Code of Civil Procedure. An express order must be passed
while giving effect to the supplemental proceedings which is additional to the incidental
power of the Court. The distinction is fine but real. (Para 20)
Cases Referred : Chronological Paras
2008 AIR SCW 298 : AIR 2008 SC 901 : 2008 (2) AIR Kar R 146 (Foll., Pnt. B) 18
@page-SC2719

2006 AIR SCW 3549 : AIR 2006 SC 2628 : 2006 (5) AIR Kar R 249 (Foll., Pnt. A)
11
2004 AIR SCW 2684 : AIR 2004 SC 2227 (Ref.) 19
2001 AIR SCW 4193 : AIR 2001 SC 3041 (Ref.) 19
1995 AIR SCW 3830 : AIR 1996 SC 135 (Foll., Pnt. B) 17
AIR 1989 Orissa 148 (Ref.) 15
(1987) 91 Cal WN 1078 (Ref.) 15
1973 Pun LJ 84 (Ref.)15
(1952) 2 All ER 567 (Ref.) 19
J. L. Gupta, Sr. Advocate, Ashok K. Mahajan, with him for Appellant; Dhruv Metha, O.
S. Walia, Dhiraj, Mrs. Reeta Dewan Purl and P. N. Puri, for the Respondents.
* C.R. No. 947 of 2006, D/- 10-10-2006 (PandH).
Judgement
S. B. SINHA, J. :- Leave granted.
2. Dr. S. R. Bawa was the owner of a property bearing House No. 169, Section 11-A, in
the town of Chandigarh. Two suits for specific performance of contract in respect of the
said property were filed in the Court of Civil Judge, Chandigarh; one of them filed by the
appellant on the basis of a purported oral agreement for sale entered into on or about 20-
6-1995 for a consideration of Rs. 32,00,000/- in terms whereof allegedly a sum of Rs.
3,20,000/- was deposited in his account through Banker's Cheque on 22-6-1995. The said
agreement of the appellant was repudiated by Dr. S. R. Bawa in or about October 1995. A
suit for specific performance was filed on 20-11-1995. An ex parte order of injunction
was issued passed therein for a limited period but was admittedly extended from time to
time, the last one having been extended upto 16-10-1996. An application for extension
was filed but no order was passed.
Relying on or on the basis of a purported agreement dated 20-6-1995, Sanjeev Sharma
also filed a suit on 1-2-1996 for specific performance of contract in respect of the suit
property which also stood repudiated by Dr. Bawa. Even in that suit, the Trial Court
granted an injunction. Sanjeev Sharma applied for and was impleaded as a defendant in
the suit filed by the appellant by an order dated 14-10-1997. Similarly, the appellants
application for being impleaded as a defendant was allowed by an order dated 18-12-
1997.
The issues in both the suits being identical, parties led similar evidence in both the suits.
The principal defendant, Dr. Bawa, however, did not lead evidence.
3. The suit filed by Mr. Sanjeev Sharma was referred to the Lok Adalat. However, no
settlement was arrived at.
A purported compromise was, therefore, entered into by and between Dr. Bawa and
Sanjeev Sharma. On or about 19-2-2003, a consent decree was passed, pursuant whereto
or in furtherance whereof, a deed of sale was executed by Dr. Bawa in favour of one
Puneet Ahluwalia, a nominee of Sanjeev Sharma. Appellant filed an application under
Order 23 Rule 3 for the recall of the order dated 19-2-2003. Punit Ahluwalia was also
impleaded in the said suit as Defendant No.3. By reason of an order dated 21-1-2006, the
said application for recall was allowed by the learned trial Judge opining :
(1) The consent decree purported to have been entered into by and between Dr. Bawa and
Sanjeev Sharma being in terms of the second part of Order 23 Rule 3 of the Code of Civil
Procedure, the same was not valid as deed of sale executed pursuant thereto; and
(2) Although the order of interim injunction passed in the case of the appellant was not
extended beyond 16-10-1996. as an application had been filed therefor and as the
appellant could not suffer owing to an act of the Court and the same being violative of the
order of injunction, the deed of sale was invalid in law.
4. The High Court, however, while exercising its revisional jurisdiction by reason of the
impugned judgment, set aside the said order of the learned Judge on the premise that it is
the first part of Order 23 Rule 3 which was applicable in the case. It was opined that as
the interim order was not extended, the question of execution of any deed of sale in
violation of the said order of injunction did not arise.
5. Mr. J. L. Gupta, learned senior counsel appearing on behalf of the appellant, would
contend :
(1) The High Court committed a manifest error in passing the impugned order insofar as
it failed to take into consideration that it was a case where the first part of Order 23 Rule
3 would apply; and
(2) In view of the well known legal principle that any party cannot suffer owing to the
fault on the part of the Court, the deed of sale dated 25-3-2003 must be held to be
@page-SC2720
bad in law.
6. Mr. Dhruv Mehta, learned counsel appearing on behalf of the respondent, on the other
hand, would urge :
(1) From the order sheet dated 4-3-2003, it would appear that the willingness of the
parties to the suit to enter into a compromise was expressed and, thus, although the
appellant was aware thereof, he did not raise any objection to the recording of the
compromise dated 19-2-2003.
(2) The purported application for recall dated 31-7-2003 was not maintainable as an
appeal against the order dated 19-2-2003 lay before the higher Court.
(3) An order of injunction having been made operative upto 16-1-1996, it is wrong to
contend that the order of injunction by reason of a legal fiction or otherwise could
continue beyond and, in any event the appellant having not taken any step to obtain an
order of injunction thereafter, the impugned order cannot be found fault with.
(4) Assuming that there was any breach of injunction, the consequence thereof having
been provided in terms of Order 39 Rule 2A of the Code of Civil Procedure, thus, the
Court could have taken recourse only thereto, thus, its power under Section 151 of the
Code of Civil Procedure could not be taken recourse.
(5) The contention of the appellant that the sale deed became inoperative in law is
fallacious.
7. Order 23 Rule 3 of the Code of Civil Procedure reads thus :
"3. Compromise of suit- Where it is proved to the satisfaction of the Court that a suit has
been adjusted wholly or in part by any lawful agreement or compromise in writing and
signed by the parties, or where the defendant satisfies the plaintiff in respect of the whole
or any part of the subject matter of the suit, the Court shall order such agreement,
compromise or satisfaction to be recorded, and shall pass a decree in accordance
therewith so far as it relates to the parties to the suit, whether or not the subject-matter of
the agreement, compromise or satisfaction is the same as the subject matter of the suit:
Provided that where it is alleged by one party and denied by the other that an adjustment
or satisfaction has been arrived at, the Court shall decide the question; but no
adjuournment shall be granted for the purpose of deciding the question, unless the Court,
for reasons to be recorded, thinks fit to grant such adjournment.
Explanation.- An agreement or compromise which is void or viodable under the Indian
Contract Act, 1872 (9 of 1872), shall not be deemed to be lawful within the meaning of
this rule."
8. The said provision indisputably is in two parts. The first part applies where the parties
to the suit enters into a compromise in terms whereof a decree may become executable on
the basis of the compromise. The second part, however, shall apply in a case where the
claim of the plaintiff stands satisfied and no further action is required to be taken by the
parties in terms of the consent decree.
It may be true that parties to the suit signed the compromise petition. But, indisputably,
the appellant herein has a rival claim. The suit filed by him, vis-a-vis, the one of Sanjeev
Sharma was required to be considered together. The Court could exercise its discretionary
jurisdiction in one of the suits or the other, having regard to Section 20 of the Specific
Relief Act, 1963. By reason of a compromise or otherwise, the claim of the appellant
could not have been defeated. When a compromise is entered into, the Court has a duty to
see as to whether the same meets the requirements of law. A compromise decree which
does not satisfy the requirements of law is not legal. It would be unlawful. It, therefore,
cannot be recorded.
The terms of the compromise may, briefly be noticed :
"(d) That the defendant No. 1 admits the claim of the plaintiff and the said defendant No.
1 has no objection if the suit of the plaintiff for specific performance is decreed in favour
of the plaintiff and against the defendant No. 1 and the defendant No. 1 shall get the Sale
Deed executed in favour of the plaintiff or in the name of the nominee(s) on or before
31-3-2003 subject to the balance payment of consideration price amounting to Rs. 24.75
lacs by way of Banker's cheque /draft in the name of the defendant No.1 and the
defendant No. 1 shall hand over the physical vacant possession of first floor and the
symbolic possession of the tenanted portion of the house in question.
(e) That the defendant No. 1 is to go back
@page-SC2721
to States and the execution of decree is mandatory to be performed within the above
mentioned time and in the case of plaintiff fails to pay the balance amount of Rs. 24.75
lacs With the said stipulated period the defendant No. 1 shall not be entitled to sale deed
by way of its execution and it will be presumed that the plaintiff had no funds to purchase
the property.
xxx xxx xxx xxx xxx xxx
(i) That the defendant No. 2 Arjan Singh has also filed a suit against the defendant No. 1
in respect of the same property i.e. House No. 169, Sector 11-A, Chandigarh alleging the
contract having taken place on 21-6-1995 after the contract of the plaintiff with defendant
No. 1. The said suit is also pending in the Court. The plaintiff is a pro forma defendant in
the said suit and no relief has been claimed against the plaintiff in the said suit. The
outcome of the said suit shall be the sole responsibility of the defendant No. 1 and the
liability in the said suit qua the earnest money, damages, interest shall be the
responsibility of defendant No. 1 alone."
9. It is only pursuant to or in furtherance of the said purported terms of settlement, the
deed of sale was executed on 25-3-2003.
The settlement entered into by and between the parties proceeded on the assumption that
no decree for specific performance would be passed in the case of the appellant. It
wrongly recorded that the appellant is only a pro forma defendant in the suit.
10. The learned Trial Judge, while recording the compromise, categorically held :
"Ld. Counsel for the plaintiff has stated that the compromise has been effected between
the parties and compromise Ex. C-1 has been placed on the file. Both the parties i.e. the
plaintiff and the defendant No. 1 had got their statements recorded separately that they
have agreed that the suit of the plaintiff is decreed as per the compromise. In view of the
compromise Ex. C- 1 no claim has been made against the defendant No. 2.
Heard.
The suit of the plaintiff is decreed as the defendant No. 1 has admitted the claim of the
plaintiff for specific performance and he had agreed that he shall get the sale deed
executed in favour of the plaintiff or in the name of the nominee(s) on or before 31-3-
2003 subject to the balance payment of consideration price amounting to Rs. 24.75 lacs
and the defendant No. 1 shall handover the physical vacant possession of the first floor
and the symbolic possession of the tenanted portion of the house in question.
The plaintiff shall complete formalities and inform defendant No. 1 or his counsel 3 days
before the execution and registration of the sale deed.
There is no order as to costs. Suit of the plaintiff is decreed in view of the compromise
Ex. C-1 which is to be read as part of the decree. Decree sheet be prepared accordingly
and file be consigned to the record room after due compliance."
11

. One of the questions, therefore, which arose for consideration, is as to whether the first
part or second part of Order 23 Rule 3 of the Code of Civil Procedure would apply. The
Trial Judge, in our opinion, has rightly held that it was a case where the first part of Order
23 Rule 3 would apply. As the appellant was not a parry to the settlement, the same was
not binding on him. The issue is directly covered by a judgment of this Court in Pushpa
Devi Bhagat (Dead) through L. R. Sadhna Rao (Smt.) v. Rajinder Singh and Ors. ((2006)
5 SCC 566) wherein two questions which had ben framed were : 2006 AIR SCW 3549

"(i) Whether the appeal filed by Pushpa Devi under Section 96 of the Code of Civil
Procedure against the consent decree was maintainable.
(ii) Whether the compromise on 23-5-2001 resulting in a consent decree dated 18-7-2001
was not a valid compromise under Order 23 Rule 3 CPC."
The said questions were answered in the following terms, opining :
"19. What is the difference between the first part and the second part of Rule 3 ? The first
part refers to situations where an agreement or compromise is entered into in writing and
signed by the parties. The said agreement or compromise is placed before the Court.
When the Court is satisfied that the suit has been adjusted either wholly or in part by such
agreement or compromise in writing and signed by the parties and that it is lawful, a
decree follows in terms of what is agreed between the parties. The
agreement/compromise spells out the agreed terms by which the claim is admitted or
adjusted by mutual concessions or promises, so that the parties thereto can be held to
@page-SC2722
their promise(s) in future and performance can be enforced by the execution of the decree
to be passed in terms of it. On the other hand, the second part refers to cases where the
defendant has satisfied the plaintiff about the claim. This may be by satisfying the
plaintiff that his claim cannot be or need not be met or performed. It can also be by
discharging or performing the required obligation. Where the defendant so "satisfies" the
plaintiff in respect of the subject-matter of the suit, nothing further remains to be done or
enforced and there is no question of any "enforcement" or "execution" of the decree to be
passed in terms of it."
12. The compromise, in our opinion, was unlawful. What would be its effect is the
question. But before we advert thereto, another finding of the learned Trial Judge may
also be noticed.
13. The learned Trial Judge passed an interim order on 2-2-1996, which was periodically
extended. Indisputably, by reason thereof, Dr. Bawa was restrained from transferring the
property. A similar order of injunction was passed in Sanjeev Sharma's case which was
made absolute on 28-5-1997.
14. It is, however, again beyond any dispute that the said order of injunction continued
from time to time. It was operative till 16-10-1996. It has been noticed by the learned
Trial Judge that an application for extension was filed. However, because the Presiding
Officer was on leave on 16-10-1996 and later the matter was transferred to another Court,
the interim order was neither extended nor vacated.
15. Was the order of injunction operative so as to attract the provisions of Rule 2A of
Order 39 of the Code of Civil Procedure or invoking the inherent jurisdiction of the Court
under Section 151 thereof ?
The learned Trial Judge opined that it was so because it was for the Court to pass an
appropriate order thereunder. The High Court, however, differed with the aforementioned
finding of the learned Trial Judge to hold that no order of injunction was operative. It,
furthermore, held that any transaction carried out in violation of the order of the Court is
void; it would be a nullity. The decision of the High Court is based on the decisions of
different High Courts including Pranakrushna and others v. Umakanta Panda and others
(AIR 1989 Orissa 148), Phani Bhushan Dey v. Sudhamoyee Roy and Anr. (91 Calcutta
Weekly Notes 1078) and Harbalas and others v. The State of Haryana and others, 1973
Punjab Law Journal, 84).
16. We agree with the High Court on this issue. If the order of injunction was operative
upto a particular date, technically the order of injunction shall not remain operative
thereafter. The owner of the land Dr. Bawa and the defendant No. 2 Sanjeev Sharma,
thus, could have entered into the compromise.
The effect thereof would be that the said deed of sale was not binding on the appellant. It
would be hit by the doctrine of lis pendens, as adumbrated under Section 52 of the
Transfer of Property Act. The said deed of sale would not come in the Court's way in
passing a decree in favour of the appellant. Its validity or otherwise would not be
necessary to be considered as the appellant is not bound thereby. Sanjeev Sharma and
consequently Puneet Ahluwalia would be deemed to be aware of the pendency of the suit.
Even Section 19 of the Specific Relief Act will be attracted.
17

. Reliance has been placed by Mr. Gupta on Surjit Singh v. Harbans Singh, (AIR 1996 SC
135 : (1995) 6 SCC 50) wherein this Court opined : 1995 AIR SCW 3830

"4...........In defiance of the restraint order, the alienation/assignment was made. If we


were to let it go as such, it would defeat the ends of justice and the prevalent public
policy. When the Court intends a particular state of affairs to exist while it is in seisin of a
lis, that state of affairs is not only required to be maintained, but it is presumed to exist
till the Court orders otherwise. The Court, in these circumstances has the duty, as also the
right, to treat the alienation/assignment as having not taken place at all for its purposes..."
18

. There cannot be any dispute with regard to the aforementioned proposition of law. This
decision answers the questions rised by Mr. Mehta that the consequences of violating the
order of injunction must be kept confined only to Rule 2A of Order 39 of the Code of
Civil Procedure.We must also take notice of the fact that even a Court in exercise of its
inherent jurisdiction under Section 151 of the Code of Civil Procedure, in the event of
coming to the conclusion that breach to an order of restraint had taken 2008 AIR
SCW 298

@page-SC2723
place, may bring back the parties to the same position as if the order of injunction has not
been violated. (Gurunath Manohar Pavaskar and Ors. v. Nagesh Siddappa Navalgund and
Ors., (2007 (14) Scale 283).
19

. Furthermore, in a given case, the Court, may also invoke the rule as adumbrated in
Hadkinson v. Hadkinson ((1952) 2 All ER 567). The said principle, however, has been
explained by this Court in Pravin C. Shah v. K. A. Mohd. Ali and another ((2001) 8 SCC
650) stating : 2001 AIR SCW 4193

"21. The observations can apply to the Courts in India without any doubt and at the same
time without impeding the disciplinary powers vested in the Bar Councils under the
Advocates Act.
{See also Bar Council of India v. High Court of Kerala, ((2004) 6 SCC 311)} 2004
AIR SCW 2684

20. However, as in this case, no order of injunction was breached, the said principle has
no application.
The deed of sale, therefore, need not be set side. It will have its own effect having regard
to Section 52 of the Transfer of Property Act and Section 19 of the Specific Relief Act.
The learned Trial Judge, however, was right in holding that the purported compromise
was bad in law. It was unlawful being without any written consent of all the parties. We
need not go into the question as to whether the same was fraudulent or not, but
indisputably not only the same was not binding on the parties, the Court in a case of this
nature while considering the appellant's case shall not take note of the fact that any deed
of sale has been executed pursuant thereto. Respondent No. 3, as logical corollary of
these findings, would not be entitled to set up the plea of being bona fide purchaser for
value without notice. The Court may also pass such other order or orders, as it may deem
fit and proper keeping in view its discretionary jurisdiction under Section 20 of the
Specific Relief Act, 1963. To that extent the judgment of the learned Trial Judge must be
upheld and that of the High Court must be set aside.
We, however, do not agree that the appellant cannot be made to suffer for violation of the
order of the Court but as the legal principle stated in the judgment of the learned Trial
Judge in that behalf is not correct, the same would not apply in this case. As no order of
injunction was operative, the Court cannot pass an order of injunction with retrospective
effect so as to take away the right of the parties created for the said purpose. To the said
effect, the Court must make a distinction between an incidental proceeding and
supplemental proceeding. An order of injunction can be passed in terms of the provisions
of the supplemental proceedings contained in Section 94 of the Code of Civil Procedure.
An express order must be passed while giving effect to the supplemental proceedings
which is additional to the incidental power of the Court. The distinction is fine but real.
21. For the reasons aforementioned, the impugned judgment is set aside to the
aforementioned extent. The appeal is allowed in part. In the facts and circumstances of
the case, respondent Nos. 2 and 3 must bear the costs of the appellant. Counsel's fee
assessed at Rs. 50,000/- (Rupees fifty thousand only).
Appeal partly allowed.
AIR 2008 SUPREME COURT 2723 "Ashok Kumar v. State of Bihar"
(From : Patna)
Coram : 2 TARUN CHATTERJEE AND J. M. PANCHAL, JJ.
Civil Appeal No. 3243 of 2008 (arising out of SLP (C) No. 10445 of 2007), D/- 2 -5
-2008.
Ashok Kumar v. State of Bihar and Ors.
Constitution of India, Art.226 - WRITS - Writ petition - Delay and laches - Petition filed
4 yean after order impugned - Delay due to pendency of review/representation before
State Govt. - Sufficient ground to condone delay - Dismissal of petition on ground of
delay and laches improper.
L.P.A. No. 1348 of 2005, D/-24-01-2007 (Pat.), Reversed. (Para 3)
H. L. Agrawal, Sr. Adv., Gaurav Agrawal and Dr. Kailash Chand, with him for Appellant;
Gopal Singh and Manish Kumar, for Respondents.
Judgement
JUDGMENT :- Leave granted.
2. This appeal is directed against an order dated 24th of January, 2007 passed by a
Division Bench of the High Court of Judicature
@page-SC2724
at Patna in LPA No. 1348 of 2005 by which the writ petition filed by the writ
petitioner/appellant was dismissed not on merits but on the ground of delay and laches.
3. On grant of leave, we have heard the learned counsel for the parties. Having heard the
learned counsel for the parties and after examining the orders of the High Court, viz., the
order of the Division Bench impugned in this appeal and the order of the learned Single
Judge, we are of the view that the Division Bench as well as the learned single Judge of
the High Court were not justified in rejecting the writ petition as well as the writ appeal
on the ground of delay and laches as the writ petitioner i.e. the appellant had moved the
writ petition before the High Court against the decision of the State Government only in
1996, i.e. after 4 years from the date of passing of such order. The Division Bench as well
as the learned Single Judge, in our view, had committed an error in holding that the
pendency of the review/ representation of the writ petitioner/appellant could not be taken
to be a ground for condoning the delay after 4 years of the decision of the State
Government. In our view, the High Court had fallen into error in not holding that the
appellant had sufficiently explained why the writ petition could not be moved or why it
was moved after 4 years of the decision of the State Government. Since the appellant had
filed a representation/review of the decision of the State Government, it was expected by
him that an order should be passed on the said representation/review. Therefore, in our
view, the delay in moving the writ application against the decision of the State
Government was sufficiently explained by the appellant and, therefore, the writ petition
ought not to have been dismissed on the ground of delay and laches. Accordingly, we set
aside the impugned orders of the Division Bench as well as of the learned Single Judge.
The writ petition is, accordingly, restored to its file. The High Court is requested to
decide the writ petition on merits in accordance with law after giving opportunity of
hearing to the parties and after permitting the parties to exchange their affidavits, if not
already exchanged in the meantime. Accordingly, the appeal is allowed to the extent
indicated above. There will be no order as to costs.
Appeal allowed.
AIR 2008 SUPREME COURT 2724 "Sudarsha Avasthi v. Shiv Pal Singh"
(From : Allahabad)*
Coram : 2 A. K. MATHUR AND ALTAMAS KABIR, JJ.
Civil Appeal No. 6807 of 2005 with C.A. No. 1108 of 2007, D/- 16 -5 -2008.
Sudarsha Avasthi v. Shiv Pal Singh.
Representation of the People Act (43 of 1951), S.123, S.83 - ELECTION - Election
petition - Corrupt practice - Bribery etc. - Allegations not precise to show that those are
serious allegations to be tried in election petition - Mostly unbelievable and impracticable
sequence of events - Petition lacking in particularity in any allegations of bribery - Liable
to be dismissed.
The election petition is a serious matter and it cannot be treated lightly or in fanciful
manner nor is it given to a person who uses this as a handle for vexatious purpose. The
allegations made in the petition as disclosed in the election petition found to be totally a
cock and bull story. It is unbelievable that a real brother of the returned candidate will
give a sum of Rs. 10,000/- in the office of the Returning Officer i.e., Assistant
Commissioner for appearing in the election or contesting the election or a sum of Rs.
30,000/- to another candidate for withdrawing in full public view of all and sundry. And
these allegations have been alleged in order to make a ground for somehow or the other
to put pressure on the respondent. These allegations are thus not precise allegations so as
to show that these are serious allegations to be tried in the election petition. These
allegations are most unbelievable and impracticable sequence of events. It is easy to
allege without giving the detail particulars whether the whole thing transpired in a
hearing distance. Therefore, this election petition lacks in particularity any allegation of
bribery as contained in S. 123 read with S. 83 of the Act, and would therefore, liable to be
dismissed. (Paras 7, 9)
Cases Referred : Chronological Paras
2006 AIR SCW 4 : AIR 2006 SC 713 (Ref.) 8
2004 AIR SCW 6205 : AIR 2005 SC 22 (Ref.) 8
1999 AIR SCW 1156 : AIR 1999 SC 1359 (Ref.) 8
@page-SC2725

1991 AIR SCW 1492 : AIR 1991 SC 1557 (Ref.) 8


AIR 1982 SC 1559 (Ref.) 8
Sudarsha Avasthi (Appellant-in-person) and Dinesh Kumar Garg, for Appellant;
Ardhendumauli K. Prasad, T. N. Singh, V. K. Singh, Ms. Taruna A. Prasad and Kishan
Datta, for Respondent.
* Election Petn. No. 3 of 2002, D/- 28-4-2005 (All) (L.B.).
Judgement
1. A. K. MATHUR, J. :-Both these appeals involve common question of fact and law,
therefore, they are disposed of by this common order. For convenient disposal of both the
cases, the facts given in C.A.No. 6807 of 2005 (Sudarsha Avasthi v. Shiv Pal Singh) are
taken into consideration.
2. This civil appeal is directed against the order passed by learned Single Judge of the
High Court of Judicature at Allahabad, Lucknow Bench, Lucknow by which three
election petitions were disposed of one by the appellant, Jitendra Nath Pandey and Sharad
Tiwari by the common order. The appellant before us, Sudarsha Avasthi filed an election
petition being Election Petition No. 3 of 2002 for declaration of Shiv Pal Singh's election
to the Uttar Pradesh Legislative Council to be void on various grounds. The appellant was
an elector in the Electoral Roll for election to the Legislative Council of Uttar Pradesh
from Lucknow Division Graduates' Constituency and his name was mentioned at Serial
No. 1595 of Part No. 190 Aliganj Ward Lucknow. The election was held on 2.5.2002 and
the result was declared on 7.5.2002. The respondent-Shiv Pal Singh was declared elected.
The election of the elected representative-respondent was challenged on the ground that
the result of the election had been materially affected by improper acceptance of
nomination paper of respondent. Respondent committed corrupt practice by giving
money directly to Ram Pratap Singh and Pradeep Kumar with a view to induce them to
contest as candidates in the said election. The respondent also committed corrupt practice
by giving money to S.P. Singhal with the object of inducing him to withdraw his
nomination. Lastly, the respondent committed corrupt practice of procuring assistance in
furtherance of his prospects in the election from the Additional Commission
(Administration) , Lucknow Division who was the Assistant Returning Officer in the said
election. A detailed affidavit was filed by the appellant disclosing the material facts of the
corrupt practice. The election petition was contested by the returned candidate-the
respondent, Shiv Pal Singh. It was pleaded on behalf of the respondent that the election
petition did not disclose any cause of action, pleadings are vague, frivolous and
vexatious. The concise statement of material facts and the full particulars of the
allegations of corrupt practices had not been disclosed. Therefore, the election petition
was liable to be dismissed for non-compliance of the provisions of Sections 82 and 83 of
the Representation of the People Act, 1951 (hereinafter to be referred to as the Act). An
application was also filed under Order VI Rule 16 read with Order VII, Rule 11 of the
Code of Civil Procedure (hereinafter to be referred to as the C.P.C.) praying for dismissal
of the election petition. The respondent moved an application and prayed that the
preliminary issues pertaining to the maintainability of the election petition and the other
that the election petition lacked material facts and disclosed no cause of action. Two
issues were framed as preliminary issues which read as under :
" 1. Whether the election petition preferred by the petitioner is liable to be dismissed as it
was presented before the Registrar and not before the Judge of the High Court dealing
with the election matter ?
2. Whether the election petition deserves to be dismissed as it does not disclose any cause
of action and material facts and the particulars are lacking as alleged at this stage ?
It was also referred by learned Single Judge that during the course of the proceedings,
only learned counsel for the respondent-Shiv Pal Singh in Election Petition No.3 of 2002
and the appellant in person submitted their arguments on the preliminary issues as
reproduced above. No issues were framed nor any arguments were advanced in the other
two election petitions. Therefore, learned Single Judge observed that the order passed in
election petition No. 3 of 2002 will have no bearing on the other two election petitions.
3. So far as the issue No. 1 was concerned, learned Single Judge after review of the
pleadings, held that it was wrong to say that the election petition was not properly
represented. So far as issue No. 2 was concerned, learned single Judge after review of
pleadings and arguments made in the petition came to the following conclusion.
@page-SC2726
"The long and short of above discussions is that the petitioner suppressed the material
facts which could disclose the cause of action and there being no cause of action, which
might have accrued to him, I am of the decisive opinion that this election petition being
devoid of merit deserves to be dismissed."
4. Incidentally, it may be mentioned that the tenure of the present office of Legislative
Council has already expired and new election has already been ordered and they were
held during the pendency of this petition and the results are awaited. Therefore, it was
contended on behalf of the respondent that it is purely an academic issue as the fresh
election has already been held and the term of the present office is already over.
Therefore, no useful purpose will be served in going into the merit of the case. The
appellant submitted that he has made serious allegations of corrupt practice and in case he
succeeds in establishing that there was corrupt practice then the incumbent could be
debarred from contesting the election. Therefore, the appellant insisted that the civil
appeal should be decided on merits.
5. We have heard the appellant in person and learned counsel for the respondent and
perused the record. So far as issue No. 2 is concerned, whether the election petition
deserves to be dismissed as it does not disclose any cause of action and material facts and
the particulars are lacking. In order to appreciate the contention we may refer to
necessary provisions of the Act pertaining to pleadings. Section 83 of the Act says what
should be the contents of the election petition. Section 83 of the Act reads as under :
"83. Contents of petition. - (1) An election petition -
(a) shall contain a concise statement of the material facts on which the petitioner relies;
(b) shall set forth full particulars of any corrupt practice that the petitioner alleges
including as full a statement as possible of the names of the parties alleged to have
committed such corrupt practice and the date and place of the commission of each such
practice; and
(c) shall be signed by the petitioner and verified in the manner laid down in the Code of
Civil Procedure, 1908 (5 of 1908) for the verification of pleadings :
Provided that where the petitioner alleges any corrupt practice, the petition shall also be
accompanied by an affidavit in the prescribed form in support of the allegation of such
corrupt practice and the particulars thereof. (2) Any schedule or annexure to the petition
shall also be signed by the petitioner and verified in the same manner as the petition."
As per Section 83 of the Act, it is the duty of the person who files the election petition
and levels the allegation of corrupt practice, he has to disclose the material facts on which
he relies and that should set forth the full particulars of a corrupt practice that the
petitioner alleges including the full statement as far as possible disclosing the names of
the parties alleged to have committed such corrupt practice and the date and place of
commission of each such practice and the same shall be filed by the petitioner and
verified in the manner as laid down in the Code of Civil Procedure. Apart from this, he
has to file an affidavit in prescribed form in support of the allegation of such corrupt
practice and he should disclose the particulars thereof. If he wants to rely on any
document then it should be annexed to the petition signed by the petitioner and verified in
the same manner as the petition. Section 123 of the Act deals with the corrupt practice.
What shall be the corrupt practice have been enumerated in Section 123 of the Act, like;
bribery which has been defined that any gift, offer or promise by a candidate or his agent
or by any other person with the consent of a candidate or his election agent of any
gratification, to any person whomsoever, with the object, directly or indirectly of
including a person to stand or not to stand as, or to withdraw or not to withdraw from
being a candidate at an election or an elector to vote or refrain from voting at an election,
or as a reward to a person for having so stood or not stood, or for having withdrawn or
not having withdrawn his candidature; or an elector for having voted or refrained from
voting. Therefore, the detailed particulars are required to be given that how a person is
being bribed by various modes. All these particulars have to be given in the manner
provided in Section 83 of the Act.
6. Now, keeping in background these provisions we may advert to the facts that what
were the pleadings made by the appellant
@page-SC2727
in his election petition and disclosure made therein. So far as the question of nomination
is concerned, the appellant in person has not seriously agitated because he submitted that
the election was held and that the term has almost come to an end. Therefore, he
concentrated primarily on the question of corrupt practice disclosed by him in his election
petition. The appellant tried to persuade us that sufficient material particulars have been
disclosed and the view taken by learned single Judge that sufficient material particulars
were not disclosed was not correct. Therefore, what are the material particulars disclosed
by the appellant in his election petition have to be considered. So far as the allegation of
corrupt practice is concerned, the same are contained in paragraph 5 B, C and D of the
election which read as under :
"5B. Because the Returned Candidate Respondent No. 1 committed the corrupt practice
of Bribery for gratification (i.e. giving money) directly to induce Respondent Nos. 2 and
3 to stand as candidates in the aforesaid election.
C. Because the Returned Candidate, Respondent No. 1 committed the corrupt practice of
Bribery by giving money to Respondent No.4 with object of inducing directly the
Respondent No.4 to withdraw from the contest from the aforesaid election.
D. Because the Returned Candidate the Respondent No. 1 committed the corrupt practice
of procuring assistance for the furtherance of his prospects in the aforesaid election, from
the Additional Commissioner (Administration), Lucknow Division, Shri A. K. Mishra,
who was also the Assistant Returning Officer in the aforesaid election, and wrongly
allowed an application of the Respondent No. 1 for correction of his name on the ballot
paper in violation of the applicable statutory provision and disallowed, with utter
disregard to the directions given by the Election Commission of India to the Returning
Officers for dealing with such applicants, a similar application of Km. Vibha Avasthi,
another candidate to the aforesaid election, on the basis of extraneous material privately
supplied by the Respondent No. 1 to the said Assistant Returning Officer." So far as the
ground B is concerned, the allegations have been set out in paragraph 16 and the relevant
allegations start from paragraphs 19 and 20 which read as under :
"19. The Respondent No. 2 Ram Pratap Singh s/o Late Ram Lal Singh is real brother of
the Respondent No. 1. The Respondent inducing directly Respondent No. 2 to stand as a
candidate in the aforesaid election. On 11.04.2002 at about 12.45 p.m. gave Rupees Ten
Thousand (Rs. 10,000.00) to Respondent No. 2 inside the courtroom of the
Commissioner, Lucknow Division, for filing his (i.e. of Respondent No. 2) nomination
papers to become a candidate in the aforesaid election to U.P. Legislative Council from
Lucknow Division Graduates' Constituency. The Respondent No.2 thereafter deposited
the security money and filed his nomination papers before the Assistant Returning Officer
in the courtroom of the Commissioner, Lucknow, in presence of the Respondent No. 1.
20. That Respondent No. 1, while inducing Respondent No. 3 directly to stand as a
candidate in the aforesaid election, on 11.04.2002 at about 1.00 p.m. gave Rupees Ten
Thousand (Rs. 10,000.00) to Respondent No. 3 Pradeep Kumar s/o Late Behari Lal in the
Verandah which is outside the courtroom of the Commissioner, Lucknow Division, for
filing his (i.e. of Respondent No. 3) nomination papers to become a candidate in the
aforesaid election to the Uttar Pradesh Legislative Council from the Lucknow Division
Graduates' Constituency. The Respondent No. 1, thereafter, delivered the nomination
papers to the Respondent No.3 and made him to put his signature on them at the table
inside the courtroom of the Commissioner, Lucknow Division, while the Respondent No.
1 was standing by his side indicating the places for putting the signatures. The
Respondent No.3 thereafter deposited the security money and filed his nomination papers
before the Assistant Returning Officer in the courtroom of Commissioner, Lucknow
Division."
So far as the allegations contained in Paragraph 5 C are concerned, those are contained in
paragraph 21 onwards. Relevant allegations are in paragraphs 23 and 24 which read as
under :
"23. That Respondent No. 4 S. P. Singhal s/o Ram Chandar Singhal had filed his
nomination paper on 06.04.2002 in the aforesaid election and his nomination papers were
found valid and accepted by the Returning Officer on 12.04.2002.
24. That Respondent No. 1 gave Rupees Thirty Thousand (Rs. 30,000.00) to Respondent
@page-SC2728
No. 4 inside the compound of the Commissioner's Office (Lucknow Division) near the
exit gate, on 15.04.2002 at about 2.00 p.m. to withdraw from being a candidate at the
aforesaid election. Thereafter, the Respondent No. 4 withdraw his candidature by giving
Notice in writing addressed to the Returning Officer on 15.4.2002."
So far as the allegations contained in Paragraph 5 D, those are set out at paragraph 25
onwards. Relevant portions contained in paragraphs 27 to 29 which read as under :
"27. That on 15.04.2002 at 2.25 p.m. the petitioner had submitted an application from
Km. Vibha Avasthi, a candidate in the aforesaid election, for addition of her popular name
on the ballot paper, by also including her alias "Dr. Vibha Harikrishna Avasthi", and at
that time the respondent No. 1 and his counsel were pleading before the Assistant
Returning Officer Shri A.K. Mishra for allowing to move a third application for
correction of his name in a manner as to distance/distinguish it on the ballot paper from
the other two candidates with similar name viz. Shiv Pal Singh and the Petitioner tried to
object to such a request, which was improper and beyond the scope of the provision in
this regard, but a visibly annoyed Assistant Returning Officer Shri A. K. Mishra advised
the Petitioner to confine his submissions on the application moved by the petitioner, but
when requested to inform the objection, if any, or the reasons of dissatisfaction of the
Returning Officer as to the genuineness of the request, the said Assistant Returning
Officer announced that time for withdrawing the nominations and making applications
was over, and left the room without pronouncing any decision on the applications before
him.
28. That at about 3.45 p.m. on the same day, the petitioner went to the chamber of the
Assistant Returning Officer Shri A. K. Mishra, the doors of his room were half closed,
and the petitioner was not allowed entry by the Home Guard on duty outside the room,
informing that the Additional Commissioner was busy with another candidate and while
the petitioner was waiting outside his room he could hear a conversation inside the room,
with Mr. A. K. Mishra inquiring from Mr. Raees, the genuineness of a sample ballot
paper given by S. P. Singh, and the reply was in affirmative.
29. That thereafter the doors of the room opened and the Petitioner saw the Respondent
No. 1 and his counsel coming out of room bowing and profusely thanking the ARO Shri
A. K. Mishra; and then the petitioner was allowed entry inside the room of Shri A. K.
Mishra and saw Mr. Raees Ahmad, the Senior Clerk of Lucknow District Election Office
sitting inside the room and when the Petitioner asked the fate of the application moved by
him on behalf of Km.Vibha Avasthi, Shri A. K. Mishra the ARO replied that the same
was rejected, but refused to give reasons for his decision, even on asking by the
Petitioner."
Learned Single Judge has taken into consideration these grounds of corrupt practice along
with the allegations contained in the election petition as reproduced above and came to
the finding that these all appear to be cock and bull story. It was observed that it is not
believable that Ram Pratap Singh who is the real brother of the respondent would accept
illegal gratification in public view, that too inside the Courtroom of the Commissioner. It
was further observed that the appellant did not disclose the name of any witness who was
present inside the courtroom or outside when the money was handed over to the brother
of the respondent. Similarly on the analogy that the theory of giving Rs. 30,000/- as bribe
to the Respondent No. 4 in office of Commissioner appears to be nothing but figment of
imagination. Lastly regarding ground D it was observed that Ku. Vibha Avasthi wanted
some alteration in her name. She did not move any application for addition of surname of
her father. But the copy of this application for alteration of the name was not filed and it
was deliberately concealed that he was the agent of Kumari Vibha Avasthi and it was also
found that it is difficult to understand when the Office of the Assistant Commissioner
cum Returning Officer, Shri A. K. Mishra was cordoned off by the Home Guards how did
he enter inside the room of Shri A. K. Mishra and it is unbelievable that the respondent
went inside the room of Shri A. K. Mishra and influenced him in one way or the other
and secondly it was also held that it was not wrong on the part of the candidate to have
entered the office of Shri A. K. Mishra and it is difficult to believe that other person
standing outside could know what transpired between the Officer and the person inside.
Therefore, learned single
@page-SC2729
Judge found that this is nothing but a cock and bull story. On the basis of these pleadings
learned single Judge ultimately concluded that there was no material facts disclosing the
cause of action and consequently, dismissed the election petition.
7. The election petition is a serious matter and it cannot be treated lightly or in fanciful
manner nor is it given to a person who uses this as a handle for vexatious purpose. The
allegations made in the petition as disclosed in the election petition appear to us to be
totally a cock and bull story. We are in agreement with the view taken by the learned
Single Judge that a brother will give a sum of Rs. 10,000/- in the office of the Returning
Officer i.e. Assistant Commissioner for appearing in the election or contesting the
election or a sum of Rs. 30,000/- to another candidate for withdrawing in full public view
of all and sundry. On the face of it appears to be unbelievable and these allegations have
been alleged in order to make a ground for somehow or the other to put pressure on the
respondent. Learned Single Judge after review of the allegations contained therein has
rightly concluded that these allegations are not precise allegations so as to show that these
are serious allegations to be tried in the election petition. Learned Single Judge has gone
through all these allegations and it also appears to us to be most unbelievable and
impracticable sequence of events. It is easy to allege without giving the detail particulars
whether the whole thing transpired in a hearing distance. We are in full agreement with
the view taken by learned single Judge and we are of opinion that the learned single
Judge has correctly appreciated that this case lacks in particularity any allegation of
bribery as contained in Section 123 read with Section 83 of the Act.
8. The appellant in person has taken us through various decisions of this Court. The
followings are the list of cases cited by the appellant.
i) AIR 1982 SC 1559, Roop Lal Sathi v. Machhattar Singh

ii) (1991) 3 SCC 375, F.A. Sapa and Ors. v. Singora and Ors. 1991 AIR SCW 1492

iii) (1999) 4 SCC 274, T.M. Jacob v. C.Poulose and Ors. 1999 AIR SCW 1156

iv) (2004) 11 SCC 196, Sardar Harcharan Singh Brar v. Sukh Darshan Singh and Ors.
2004 AIR SCW 6505

v) (2005) 13 SCC 511, Harkirat Singh v. Amrinder Singh. 2006 AIR SCW 4

9. No useful purpose will be served by referring to all these cases. The ratio of all these
cases is the election petition should contain the allegation of bribery in a concise manner
with material particulars. Material particulars disclosed in the present as mentioned
above, are not sufficient to be gone into for trial. We are in full agreement with the view
taken by learned Single Judge. Therefore, we find no merit in the appeal and the same is
dismissed. No order as to costs.
Civil Appeal No. 1108 of 2007.
10. The election petition before the High Court was dismissed for want of prosecution as
nobody appeared. The learned Judge has recorded in his order that the matter was
adjourned number of times and nobody appeared. Therefore, it is not necessary to restore
this petition as petitioner was not serious and present term has already expired. This
appeal is dismissed with no order as to costs.
Appeal dismissed.
AIR 2008 SUPREME COURT 2729 "Oriental Insurance Co. Ltd. v. Sudhakaran K.V."
(From : Kerala)
Coram : 2 S. B. SINHA AND LOKESHWAR SINGH PANTA, JJ.
Civil Appeal No. 3634 of 2008 (arising out of SLP (C) No. 19562 of 2006), D/- 16 -5
-2008.
Oriental Insurance Co. Ltd. v. Sudhakaran K.V. and Ors.
Motor Vehicles Act (59 of 1988), S.147 - MOTOR VEHICLES - INSURANCE -
CONTRACT - Liability of Insurance Company - Contract of insurance covering risk of
third party but not that of owner or pillion rider of two-wheeler - Death of pillion rider
not due to rash and negligent driving on part of driver of another vehicle - Liability of
Insurance Company not extended to death of pillion rider.
M.F.A. No. 536 of 1999, D/-22-03-2006 (Ker.), Reversed.
Where the contract of insurance covered risk of third party but not that of owner or
pillion rider of two wheeler, the liability of the Insurance Company in a case of this
nature was not extended to a pillion rider of the motor vehicle unless the requisite amount
of premium is paid for covering his/
@page-SC2730
her risk. The legal obligation arising under Section 147 of the Act cannot be extended to
an injury or death of the owner of vehicle or the pillion rider. The pillion rider in a two
wheeler was not to be treated as a third party when the accident has taken place owing to
rash and negligent riding of the scooter and not on the part of the driver of another
vehicle.
M.F.A. No. 536 of 1999, D/- 22-3-2006 (Ker.), Reversed. (Para 19)
Cases Referred : Chronological Paras
2008 AIR SCW 533 : AIR 2008 SC 989 (Foll.) 18
2008 AIR SCW 2035 (Foll.) 18
2007 AIR SCW 1505 : AIR 2007 SC 1334 : 2007 (3) ALJ 193 (Ref.) 13
2007 AIR SCW 2279 : AIR 2007 SC 1563 : 2007 (3) AIR Kar R 389 (Ref.) 13
2007 AIR SCW 2362 : AIR 2007 SC 1609 : 2007 (3) AIR Kar R 437 : 2007 (3) ALJ 596
(Ref.) 13
2007 AIR SCW 3734 : AIR 2007 SC 1971 (Ref.) 17
2007 AIR SCW 7280 : AIR 2008 SC 460 : 2008 (1) AIR Bom R 252 (Foll.) 17
2006 AIR SCW 1822 : AIR 2006 SC 1576 : 2006 (3) AIR Kar R 371 (Ref.) 5, 12
2002 AIR SCW 5259 : AIR 2003 SC 607 (Ref.) 11, 12
1999 (2) Ker LT 886 (Ref.) 4
Ms. Aanchal Jain, Santosh Paul and M. J. Paul, for Appellant; Ms. Purnima Bhat and
Mrs. K. Sarda Devi, for Respondents.
Judgement
S. B. SINHA, J. :- Leave granted.
1. This appeal is directed against a judgment and order dated 22.3.2006 passed by the
High Court of Kerala at Ernakulam in M.F.A. No. 536 of 1999 whereby and whereunder
the appeal preferred by the appellant herein from the judgment and award dated
31.10.1998 passed by the Motor Accident Claims Tribunal, Perumbavoor awarding a sum
of Rs. 1,18,900/- (Rupees One lakh eighteen thousand and nine hundred only) together
with interest thereon at the rate of 12% p.a. from the date of the filing of the claim
petition till date of realization of the amount against the appellant as also against the
owners of the vehicle was dismissed.
2. The basic fact of the matter is not in dispute. Thankamani (hereinafter referred to as the
deceased) was travelling as a pillion rider on, a scooter on 20.10.1993. She fell down
from the scooter and succumbed to the injuries sustained by her. In regard to the said
accident, a claim petition was filed.
Appellant having been served with a notice, in its written statement, inter alia, raised a
contention that she being a pillion rider and, thus, a gratuitous passenger, the insurance
policy did not cover the risk of injury or death of such a passenger and, thus, it was not
liable to reimburse the owner of the scooter therefor.
It was, furthermore, contended that the accident had taken place at a private place.
By reason of the impugned award, the tribunal, however, opined :
(i) The accident had taken place due to rash and negligent riding of the scooter by
Sebastian P.V. respondent No. 1 to the claim petition;
(ii) Keeping in view the monthly income of the deceased which was estimated at Rs.
1200/- per month as also age of the deceased assessed at 50 years; claimants were entitled
to compensation for a sum of Rs. 1,05,600/-. A sum of Rs. 5,000/- was allowed towards
compensation for pain and suffering; a sum of Rs. 100/- was allowed towards damage of
clothing and articles, a sum of Rs. 5,000/- was allowed towards loss of love and affection
and a sum of Rs. 1,000/- was allowed towards mental shock and agony.
3. As regards liability of the appellant it was held as the existence of the insurance policy
in respect of the offending scooter is admitted, it was also liable.
4. Aggrieved by the said award, the appellant filed an appeal before the High Court of
Kerala under Section 173 of the Motor Vehicles Act, 1988 (for short "the Act"). On the
question as to whether the Insurance Company would be liable in a case of this nature,
the Division Bench opined as under:
" 1. The appellant is the third respondent in O.P.(MV) 119/94 on the file of the Motor
Accident Claims Tribunal, Perumbavoor. Appellant was directed to pay compensation for
the death of the pillion rider of a motor cycle. The vehicle was insured with the appellant.
2. It was contended that the pillion rider would not come within the coverage of the Act
policy. The Tribunal repelled that contention. Hence this appeal.
@page-SC2731
3. The question whether the pillion rider is covered by an Act policy stands settled by the
decision of the Full Bench of this Court in Oriental Insurance Co. Ltd. vs. Ajay Kumar
(1999 (2) KLT 886. Hence the appellant cannot successfully take up a contention contrary
to the above proposition in this appeal..."
5. Ms. Aanchal Jain, learned counsel appearing on behalf of the appellant, submits that as
the deceased was in a vehicle which was not covered by the contract of insurance must be
held to be a gratuitous passenger and as such the impugned judgment cannot be
sustained.

Strong reliance, in this behalf, has been placed on United India Insurance Co. Ltd.,
Shimla v. Tilak Singh and Ors. [(2006) 4 SCC 404]. 2006 AIR SCW 1822

6. Mrs. Purnima Bhat and Mrs. K. Sarada Devi, learned counsel appearing on behalf of
the respondents, on the other hand, would urge :
(i) the principles of law deduced by this Court as regards gratuitous passenger should not
apply in a case of this nature;
(ii) in any event this Court should exercise its jurisdiction under Article 142 of the
Constitution of India directing the appellant to pay the claimed amount to the claimants
and recover the same from the owner of the scooter.
7. Before embarking on the rival contentions, we may notice the insurance policy. The
contract of insurance was entered into on or about 2.12.1992. It was 'A policy for act
liability' meaning thereby a third party liability.
The relevant clauses of the said contract of insurance are as under :
"1. Subject to the limit of liability as laid down in the Motor Vehicles Act the Company
will indemnify the insured in the event of accident caused by or arising out of the use of
Motor Vehicle anywhere in India against all sums including claimant's costs and expenses
which the insured shall become legally liable to pay in respect of death or bodily injury to
any person and/or damage to any property of Third Party.
Exception
Except so far as necessary to meet the requirements of the Motor Vehicles Act the
Company shall not be liable in respect of death arising out of and in the course of
employment of person in the employment of the insured or in the employment of any
person who is indemnified under this Policy or bodily injury sustained by such person
arising out of and in the course of such employment."
8. In terms of Section 147 of the Act only in regard to reimbursement of the claim to a
third party, a contract of insurance must be taken by the owners of the vehicle. It is
imperative in nature. When, however, an owner of a vehicle intends to cover himself
from other risks; it is permissible to enter into a contract of insurance in which event the
insurer would be bound to reimburse the owner of the vehicle strictly in terms thereof.
9. The liability of the insurer to reimburse the owner in respect of a claim made by the
third party, thus, is statutory whereas other claims are not.
10. The only question which, therefore, arises for our consideration is as to whether the
pillion rider on a scooter would be a third party within the meaning of Section 147 of the
Act.
Indisputably, a distinction has to be made between a contract of insurance in regard to a
third parly and the owner or the driver of the vehicle.
11

. This Court in a catena of decisions has categorically held that a gratuitous passenger in a
goods carriage would not be covered by a contract of insurance entered into by and
between the insurer and the owner of the vehicle in terms of Section 147 of the Act. [See
New India Assurance Co. Ltd. v. Asha Rani (2003) 2 SCC 223] 2002 AIR SCW 5259

12

. A Division Bench of this Court in United India Insurance Co. Ltd., Shimla v. Tilak
Singh and Ors. [(2006) 4 SCC 404] extended the said principle to all other categories of
vehicles also, stating as under : 2006 AIR SCW 1822, Para 21

"In our view, although the observations made in Asha Rani case were in connection with
carrying passengers in a goods vehicle, the same would apply with equal force to
gratuitous passengers in any other vehicle also. Thus, we must uphold the contention of
the appellant-Insurance Company that it owed no liability towards the injuries suffered by
the deceased-Rajinder Singh who was a pillion rider, as the insurance policy was a
2002 AIR SCW 5259

@page-SC2732
statutory policy, and hence it did not cover the risk of death of or bodily injury to a
gratuitous passenger."
13

. The submission of Mrs. Bhat, learned counsel, however, is that this Court should not
extend the said principle to the vehicles other than the goods carriage. As at present
advised, we may not go into the said question in view of some recent decisions of this
Court, viz., National Insurance Co. Ltd. v. Laxmi Narain Dhut [(2007) 3 SCC 700];
Oriental Insurance Co. Ltd. v. Meena Variyal [(2007) 5 SCC 428] and New India
Assurance Co. Ltd. v. Ved Wati [(2007) 9 SCC 486]. 2007 AIR SCW 2279
2007 AIR SCW 2362
2007 AIR SCW 1505

14. The provisions of the Act and, in particular, Section 147 of the Act were enacted for
the purpose of enforcing the principles of social justice. It, however, must be kept
confined to a third party risk. A contract of insurance which is not statutory in nature
should be construed like any other contract.
15. We have noticed the terms of the contract of insurance. It was entered into for the
purpose of covering the third party risk and not the risk of the owner or a pillion rider. An
exception in the contract of insurance has been made, i.e., by covering the risk of the
driver of the vehicle. The deceased was, indisputably, not the driver of the vehicle.
16. The contract of insurance did not cover the owner of the vehicle, certainly not the
pillion rider. The deceased was travelling as a passenger, stricto sensu may not be as a
gratuitous passenger as in a given case she may not be a member of the family, a friend or
other relative. In the sense of the term which is used in common parlance, she might not
be even a passenger.
In view of the terms of the contract of insurance, however, she would not be covered
thereby.
It is not necessary for us to deal with large number of precedents operating in this behalf
as the question appears to be covered by a few recent decisions of this Court.
17

. In United India Insurance Company Ltd. v. Serjerao and Ors. [2007 (13) SCALE 80], it
was held as under : 2007 AIR SCW 7280

"7. . . . When a statutory liability has been imposed upon the owner, in our opinion, the
same cannot extend the liability of an insurer to indemnify the owner, although in terms
of the insurance policy or under the Act, it would not be liable therefor.
17. In a given case, the statutory liability of an insurance company, therefore, either may
be nil or a sum lower than the amount specified under Section 140 of the Act. Thus, when
a separate application is filed in terms of Section 140 of the Act, in terms of Section 168
thereof, an insurer has to be given a notice in which event, it goes without saying, it
would be open to the insurance company to plead and prove that it is not liable at all.
18. Furthermore, it is not in dispute that there can be more than one award particularly
when a sum paid may have to be adjusted from the final award. Keeping in view the
provisions of Section 168 of the Act, there cannot be any doubt whatsoever that an award
for enforcing the right under Section 140 of the Act is also required to be passed under
Section 168 only after the parties concerned have filed their pleadings and have been
given a reasonable opportunity of being heard. A Claims Tribunal, thus, must be satisfied
that the conditions precedent specified in Section 140 of the Act have been substantiated,
which is the basis for making an award.
19. Furthermore, evidently, the amount directed to be paid even in terms of Chapter-X of
the Act must as of necessity, in the event of non-compliance of directions has to be
recovered in terms of Section 174 of the Act. There is no other provision in the Act which
takes care of such a situation. We, therefore, are of the opinion that even when objections
are raised by the insurance company in regard to it liability, the Tribunal is required to
render a decision upon the issue, which would attain finality and, thus, the same would be
any award within the meaning of Section 173 of the Act."
It was furthermore held as under :

"8. So far as the question of liability regarding labourers travelling in trollies is


concerned, the matter was considered by this Court in Oriental Insurance Company Ltd.
vs. Brij Mohan and Ors. (2007) 7 SCALE 753, and it was held that the Insurance
Company has no liability . . ."2007 AIR SCW 3734

18

. Yet again in Ghulam Mohammad Dar v. State of JandK and Ors. [(2008) 1 SCC 422],
this Court opined that the words "injury to any 2008 AIR SCW 533
2008 AIR SCW 2035

@page-SC2733
person" as inserted by reason of the 1994 Amendment would only mean a third party and
not a passenger travelling on a goods carriage whether gratuitous or otherwise. [See also
The New India Insurance Company v. Darshana Devi and Ors., 2008 (2) SCALE 432].

19. The law which emerges from the said decisions, is: (i) the liability of the insurance
company in a case of this nature is not extended to a pillion rider of the motor vehicle
unless the requisite amount of premium is paid for covering his/her risk; (ii) the legal
obligation arising under Section 147 of the Act cannot be extended to an injury or death
of the owner of vehicle or the pillion rider; (iii) the pillion rider in a two wheeler was not
to be treated as a third party when the accident has taken place owing to rash and
negligent riding of the scooter and not on the part of the driver of another vehicle.
20. For the views we have taken, it is not necessary to refer to a large number of
decisions cited at the Bar as they are not applicable in a case of this nature.
21. For the reasons aforementioned, the impugned judgment cannot be sustained. It is set
aside accordingly. The appeal is allowed. No costs
Appeal allowed.
AIR 2008 SUPREME COURT 2733 "Kesarwani Zarda Bhandar, M/s. v. State of U.P."
(From : 2003 (6) All WC 5441)
Coram : 2 S. B. SINHA AND LOKESHWAR SINGH PANTA, JJ.
Civil Appeal No. 5760 with 5761 of 2005, D/- 14 -5 -2008.
M/s. Kesarwani Zarda Bhandar v. State of U. P. and Ors.
U.P. Krishi Utpadan Mandi Adhiniyam (25 of 1964), S.17, S.2(a) - AGRICULTURAL
PRODUCE - Market fee - Eligibility - Zafrani Zarda - Is a manufactured product - Not a
processed agricultural produce - Not exigible to market fee - Manufacture and processing
- Distinction.
2003 (6) All WC 5441, Reversed.
Market fee is leviable on specified agricultural produce, not on agricultural produce
simpliciter. Zarda is not a specified agricultural produce. It can be subjected to payment
of market fee provided it is held to be 'tobacco'. Zafrani Zarda, as an agricultural produce
for the purpose of market fee must answer the description of 'specified agricultural
produce' as defined in Section 2(a). If it is held that Zafrani Zarda is merely a processed
form of tobacco, it would be subjected to levy of market fee, but if it is manufactured, it
would not. Zafrani Zarda being a 'manufactured tobacco' would not answer the
description of processed tobacco. No market fee can, therefore, be levied.
2003 (6) All WC 5441, Reversed. (Paras 15, 16, 18)
The distinction between 'manufactured' and 'processed' may not in all situation depend
upon the nature of the Statute involved. It must pass the requisite test, namely, as to
whether it is a completely new item. Raw material of a manufactured produce has to be
distinguished from the manufactured product. Zarda is used by a class of consumers. It is
used for a specific purpose. Tobacco in a processed form is used for many purpose, by
many persons and in many ways. Tobacco in raw form or in any other processed form is
not commercially known as Zarda. The common parlance test may have to be applied for
the purpose of finding out as to whether the product in question is manufactured goods or
not. (Paras 17, 18)
Cases Referred : Chronological Paras
2008 AIR SCW 1981 (Ref.) 17
2005 AIR SCW 2526 : AIR 2005 SC 4043 (Rel. on) 11
2003 AIR SCW 6696 : 2004 All LJ 1 (Ref.) 12
2001 AIR SCW 757 : AIR 2001 SC 931 : 2001 All LJ 498 13
1999 AIR SCW 3074 : AIR 1999 SC 3125 (Ref.) 12
1994 Supp (2) SCC 514 (Rel. on) 9
AIR 1984 SC 1870 (Ref.) 12
AIR 1966 SC 1000 (Rel. on) 8
Rakesh Dwivedi, Anoop G. Chaudhary, Vijay Hansaria, S. B. Sanyal, R. G. Padia, Mathai
M. Paikeday, Mrs. Shobha Dikshit, Sr. Advocates, K. K. Mohan, Amit Kr. Singh,
Shantanu Krishna, Ms. Mukti Chowdhary, Anant Prakash, Ms. Sneha Kalita, Sanjal K.
Pathak, Vivek Narayan, Ajit Kumar Pande, Sunil Roy, Ms. Madhurima Tatia, Mrs. Anil
Katiyar, V. K. Verma, Shishir Pinaki, Amit Singh, P. I. Jose, Pradeep Misra, Daleep
Dhayani, Gopal Singh, Anukul Raj, Manish Kr. and Chandan Kr., for the Appearing
Parties.
@page-SC2734

Judgement
1. S. B. SINHA, J. :-Levy of market fee on Zafrani Zarda in terms of the provisions of the
U.P. Krishi Utpadan Mandi Adhiniyam, 1964 (for short, 'the Act') is in question in these
appeals which arise out of a common judgment and order dated 21st July, 2003.
2. The Act was enacted to provide for the regulation of sale and purchase of agricultural
produce and for the establishment, superintendence and control of markets therefor in
Uttar Pradesh.
3. Before embarking on the question, we may notice some of the relevant provisions of
the Act.
Sections 2(a), 2(d), 6, 8, 17(iii)(b) read as under :
2(a) 'Agricultural produce' means such items of produce of agriculture, horticulture,
viticulture, apiculture, sericulture, pisciculture, animal husbandry or forest as are
specified in the Schedule, and includes admixture of two or more of such items, and also
includes any such item in processed form and further includes gur, rab, shakkar,
khandsari and jaggery;
2(a-1) "Board" means the State Agricultural Produce Markets Board constituted under
Section 26-A;
2(d) "Central Warehousing Corporation" means the Central Warehousing Corporation
established or deemed to be established under the Warehousing Corporations Act, 1962;
2(d-1) "Collector", in relation to a Committee for a Market Area means the Collector of
the District where the principal Market Yard of that Market Area is situated, and includes
such other officer as may be authorized by him in that behalf;
6. Declaration of Market Area. - On the expiry of the period referred to in Section 5, the
State Government shall consider the objections received within the said period and may
thereupon declare, by notification in the Gazette, and in such other manner as may be
prescribed, that the whole or any specified portion of the area mentioned in the
notification under Section 5 shall be the Market Area in respect of such agricultural
produce, and with effect from such date as may be specified in the declaration.
8. Alteration of Market Area and Modifications of the List of Agricultural produce. - (1)
The State Government, where it considers necessary or expedient in the public interest so
to do, may, by notification in the Gazette, and in such other manner as may be prescribed
and with effect from the date specified in the notification, -
(a) include any agricultural produce in, or exclude any agricultural produce from, the list
of agricultural produce specified in the notification under Section 6;
(b) include any area in, or exclude any area from, the Market Area specified in the
notification under Section 6;
(c) divide a Market Area specified in the notification under Section 6 into two or more
separate Market Areas;
(d) amalgamate two or more Market Areas specified in the notification under Section 6
into one Market Area; or
(e) declare that a Market Area specified in the notification under Section 6 shall cease to
be such area :
Provided that before action under this sub-section is taken, the State Government shall
invite and consider, in the prescribed manner, objections, if any, against the proposed
action.
(2) When during the term of a Market Committee the limits of the Market Area for which
it is established are altered under clause (b), clause (c) or clause (d) of subsection (1), the
following consequences shall, with effect from the date specified in the notification,
follow :
(a) the Market Committee shall stand dissolved and its members shall vacate their offices
as such members;
(b) a new Market Committee shall be constituted for the modified or newly created
Market Area in accordance with the provisions of Section 13;
(c) all property and assets, all rights, liabilities and obligations of the dissolved Market
Committee in respect of civil or criminal proceedings, contracts, agreements or other
matter or thing arising in relation to any part of the Market Area of a dissolved Market
Committee shall be vested in and stand transferred to the new Market Committee having
jurisdiction over that part.
(3) Where a Market Area ceases to be such area under clause (e) of sub-section (1), the
following consequences shall, with effect from the date specified in the notification,
follow :
(a) the Market Committee shall stand dissolved
@page-SC2735
and its members shall vacate their offices as such members;
(b) the Principal Market Yard and Sub-Market Yards, if any, established therein shall
cease to be such;
(c) the unspent balances of the Market Committee Fund and other assets and liabilities of
the Market Committee shall vest in the State Government :
Provided that the liability of the State Government shall not extend beyond the assets so
vested.
17. Powers of the Committee. - A Committee shall, for the purposes of this Act, have the
power to -
(i) and (ii) ....
(iii) Levy and collect -
(a) such fees as may be prescribed for the issue or renewal of licences; and
(b) market fee which shall be payable on transactions of sale of specific agricultural
produce in the market area at such rates, being not less than one percentum and not more
than two and a half percentum of the price of the agricultural produce so sold as the State
Government may specify by notification, and development cess which shall be payable
on such transactions of sale at the rate of half percentum of the price of the agricultural
produce so sold, and such fee or development cess shall be realized in the following
manner :-
(1) If the produce is sold through a commission agent, the commission agent may realize
the market fee and the development cess from the purchaser and shall be liable to pay the
same to the Committee;
(2) if the produce is purchased directly by a trader from a producer, the trader shall be
liable to pay the market fee and development cess to the Committee;
(3) if the produce is purchased by a trader from another trader, the trader selling the
produce may realize it from the purchaser and shall be liable to pay the market fee and
development cess to the Committee :
Provided that notwithstanding anything to the contrary contained in any judgment, decree
or order of any court, the trader selling the produce shall be liable and be deemed always
to have been liable with effect from June 12, 1973 to pay the market fee to the Committee
and shall not be absolved from such liability on the ground that he has not realized it
from the purchaser :
Provided further that the trader selling the produce shall not be absolved from the liability
to pay the development cess on the ground that he has not realized it from the purchaser;
(4) in any other case of sale of such produce, the purchaser shall be liable to pay the
market fee and development cess to the Committee :
Provided that no market fee or development cess shall be levied or collected on the retail
sale of any specified agricultural produce where such sale is made to the consumer for his
domestic consumption only :
Provided further that notwithstanding anything contained in this Act, the Committee may
at the option of, as the case may be, the commission agent, trader or purchaser, who has
obtained the licence, accept a lump sum in lieu of the amount of market fee or
development cess that may be payable by him for an agricultural year in respect of such
specified agricultural produce, for such period, or such terms and in such manner as the
State Government may, by notified order specify :
Provided also that no market fee or development cess shall be levied on transactions of
sale of specified agricultural produce on which market fee or development cess has been
levied in any market area if the trader furnishes in the form and manner prescribed, a
declaration or certificate that no such specified agricultural produce market fee or
development cess has already been levied in any other market area."
4. In the Schedule appended to the Act, tobacco is mentioned at Item No. V.
The question which arises for consideration is as to whether Zafrani Zarda is a processed
form or manufactured form prepared from the raw material tobacco.
The process undertaken to manufacture Zafrani Zarda is admitted. It was stated by the
appellant in their writ petition to be as under :-
"7. That the petitioner used to purchase the raw tobacco/processed tobacco outside the
Mandi Samiti, Allahabad. However, it is stated that the Zafrani Zarda or Zafrani Patti is
used for chewing which is prepared from the raw tobacco. The Jaggery Juice is sprinkled
on the raw Tobacco and then it is cut into small pieces by shearing machine. The resulting
tobacco is called as "nice tobacco". The "Nice Tobacco", is allowed to dry
@page-SC2736
for few days and then flavouring essence are being sprinkled oh it and at this stage, this
tobacco is known as "Chewing tobacco". Then, thereafter, menthol, Geru, Lime and
spices etc. are being homogeneously mixed with the same, either from electric machine
or by the manually operated machine. These items get un-separately mixed with the
processed tobacco and the resulting tobacco is called as "Zafrani Zarda" and "Zafrani
patti". In this way, the raw tobacco loses its original identity and its physical and
chemical properties are changed. It is a different commodity in the commercial world as
well as amongst the consumers. The quality, cost and liking amongst the consumers are
vary according to the material mixed with it."
5. Mr. Rakesh Dwivedi, learned senior counsel appearing on behalf of the appellant,
would submit that the question, i.e., it is a manufactured form of agricultural produce, is
no longer res integra in view of several decisions of this Court.
6. On the other hand, Ms. Shobha Dikshit, learned senior counsel appearing on behalf of
the respondents, submits that Zafrani Zarda is a processed form of tobacco.
7. It is not in dispute that having regard to the definition of 'agricultural produce' as
contained in Section 2(a) of the Act as also Section 2(d) therein, market fee would be
leviable only in the event Zafrani Zarda is held to be a processed form of tobacco and not
the manufactured form.
8

. The question came up for consideration before this Court in State of Madras v. Swastik
Tobacco Factory, [ (1966) 3 SCR 79 ] wherein, upon taking into consideration the
provisions of Section 5(l)(i) of the Madras General Sales Tax (Turnover and Assessment)
Rules, 1939, it was held that Zafrani Zarda is a manufactured form of tobacco. AIR
1966 SC 1000
9. The issue apparently is directly covered by a decision of this Court in Agricultural
Produce Market Committee v. Prabhat Zarda Factory, [ 1994 Supp (2) SCC 514], in
relation to the definition of 'agricultural produce' as contained in Bihar Agricultural
Produce Markets Act, 1960, which is pari materia with the definition of 'agricultural
produce' under 2(a) of the Act, which reads :-
" 'Agricultural produce' includes all produce, whether processed or non-processed of
agriculture, horticulture, animal husbandry, and forest specified in the Schedule."
Later, however, the said definition was amended by the Amending Act of 1982 effective
from April 30, 1982, the English rendering of which is as under :
" 'Agricultural produce' means all produce whether processed or non-processed,
manufactured or not, product of agriculture, horticulture, plantation, animal husbandry,
forest, sericulture, pisciculture, livestock or poultry, as specified in the Schedule."

10. This Court clearly affirmed the finding of the Patna High Court that 'Zarda' is a
variety of manufactured tobacco and not in its processed or non-processed form.
11

. Similar reasonings have been adopted by this Court in Dharampal Satyapal v. CCE,
[(2005) 4 SCC 337 ] wherein in relation to levy of central excise, it was held :- 2005
AIR SCW 2526

"19. Applying the above tests to the facts of this case, we find that sada kimam was
bought by the assessee as a raw material which was then blended with saffron, perfumes,
menthol, etc. to form a compound which was then packed in "balties" and cleared to the
above three licensed units at Okhla Industrial Estate, Phase II, New Delhi, Noida (U.P.)
and Barotiwala (H.P.), where Tulsi Zafrani Zarda was manufactured. That, the assessee
used to buy a similar compound (Lucknowi kimam) from the market from time to time
and use it in the manufacture of their final product. That, the compound (kimam)
prepared by the assessee at 96, Okhla Industrial Estate, Phase III, New Delhi and at E-1,
Maharani Bagh, New Delhi, in the highly concentrated form, was cleared therefrom and
taken to the above three licensed factories where it was diluted and used in the
manufacture of Tulsi Zafrani Zarda. In their reply to the show-cause notice, the assessee
admitted that the said "compound" was not capable of being used for any purpose, other
than for manufacture of branded chewing tobacco. (Underline supplied by us) This
statement of the assessee in reply to the show-cause -notice establishes that the said
compound (kimam) was not edible, it was not capable of consumption as such, however,
it was used as preparation in the manufacture of Tulsi Zafrani Zarda which was a branded
chewing tobacco manufactured in the licensed factories of the assessee at Okhla
Industrial
@page-SC2737
Estate, Phase II, New Delhi, Noida (U.P.) and Barotiwala (H.P.). Further, from time to
time, the assessee herein bought from the market a similar compound (Lucknowi kimam)
and used it in the manufacture of the final product which indicated that on blending of
sada kimam with saffron, spices, menthol, etc., the compound in question (kimam) which
emerged was a distinct, identifiable product, known to the market as kimam. Hence, we
do not find any infirmity in the impugned judgment of the Tribunal which has held that
the said compound (kimam) was marketable and classifiable as chewing tobacco or a
preparation for chewing tobacco under Chapter Sub-Headings 2404.49/ 2404.40."
12

. Indisputably an agricultural produce has to be a specified one for the purpose of levy of
market fee. [See KUMS v. Ganga Dal Mills, [(1984) 4 SCC 516 ; Belsund Sugar Co. Ltd.
v. State of Bihar and others, [(1999) 9 SCC 620 ] and Krishi Utpadan Mandi Samiti and
others v. Pilibhit Pantnagar Beej Ltd. and another, [(2004) 1 SCC 391. AIR 1984 SC
1870
1999 AIR SCW 3074
2003 AIR SCW 6696

13

. Ms. Shobha Dikshit, however, relied upon the decision of this Court in Park Leather (P)
Ltd. v. State of U.P. and others, [(2001) 3 SCC 135] wherein a Division Bench opined
that leather is only a processed form of hides and skins. Process undertaken for making
leather from hides and skins has been noticed therein. 2001 AIR SCW 757

14. It is not in dispute that when a new form comes into being and in the market parlance
it is considered to be a new product; the same would be deemed to be manufactured
goods as contradistinguished from processed goods.
15. Market fee is leviable on specified agricultural produce, not on agricultural produce
simpliciter. Zarda is not a specified agricultural produce. It can be subjected to payment
of market fee provided it is held to be 'tobacco'. Zafrani Zarda, as an agricultural produce
for the purpose of market fee must answer the description of 'specified agricultural
produce' as defined in Section 2(a) of the Act. If it is held that Zafrani Zarda is merely a
processed form of tobacco, it would be subjected to levy of market fee, but if it is
manufactured, it would not.
16. In this case, Zafrani Zarda, in view of the decision of this Court in Prabhat Zarda
(supra), must be held to be a manufactured product.
17. The distinction between 'manufactured' and 'processed' may not in all situation
depend upon the nature of the Statute involved. It must pass the requisite test, namely, as
to whether it is a completely new item. Raw material of a manufactured product has to be
distinguished from the manufactured product.

The distinction between 'processing' and 'manufacturing' is well known. When a new
thing comes into being, the steps which are taken for manufacture may be relevant but
may not be decisive. {[See Commissioner of Central Excise, Tamil Nadu v. Vinayaga
Body Building Industries Ltd. (2008) 3 SCC 666]}. 2008 AIR SCW 1981

18. Zafrani Zarda being a 'manufactured tobacco' would not answer the description of
processed tobacco. It is used by a class of consumers. It is used for a specific purpose.
Tobacco as a processed form is used for many purposes, by many persons and in many
ways. Tobacco in raw form or in any other processed form is not commercially known as
Zarda. The common parlance test may have to be applied for the purpose of finding out
as to whether the product in question is manufactured goods or not.
19. The High Court, unfortunately, had not considered this aspect of the matter. The
impugned judgment cannot, thus, be sustained. It is set aside accordingly. The appeals are
allowed. In the facts and circumstances of the case, there shall be no order as to costs.
Appeals allowed.
AIR 2008 SUPREME COURT 2737 "Khoday India Ltd. v. Scotch Whisky Association"
(From : 2008 (36) PTC 315 (Madras))
Coram : 2 S. B. SINHA AND LOKESHWAR SINGH PANTA, JJ.
Civil Appeal No. 4179 of 2008 (arising out of SLP (C) No. 21367 of 2007), D/- 27 -5
-2008.
Khoday India Ltd. v. Scotch Whisky Association and Ors.
(A) Trade and Merchandise Marks Act (43 of 1958), S.56 - TRADE AND
MERCHANDISE - TRADE MARK - LIMITATION - Trade mark - Rectification of
register - Application - Limitation - Art.137 of 1963 Act does not apply since Registrar is
not Court -
@page-SC2738
Plea that even otherwise 3 years should be taken as upper limit for rectification - Not
tenable - Reasonableness of period depend not only on nature of action initiated but also
on purport and object of statute.
Limitation Act (36 of 1963), Art.137.
AIR 1988 SC 1279, Foll. (Paras 30, 32)
(B) Trade and Merchandise Marks Act (43 of 1958), S.56 - TRADE AND
MERCHANDISE - TRADE MARK - Register of Trade Marks - Rectification -
Application for - Delay, acquiescence or waiver can in no case be ground for rejection -
Not the correct proposition.
The power of the Registrar in terms of Section 56 is wide. Sub-section (2) of Section 56
use the word "may" at two places. It enables a person aggrieved to file an application. It
enables the tribunal to make such order as it may think fit. It may not, therefore, be
correct to contend that under no circumstances the delay or acquiescence or waiver or any
other principle analogous thereto would apply. Purity of register as also the public interest
would indisuptably be relevant consideration. But, when a discretionary jurisdiction has
been conferred on a statutory authority, the same although would be required to be
considered on objective criteria but as a legal principle it cannot be said that the delay
leading to acquiescence or waiver or abandonment will have no role to play. (Paras
41, 42)
(C) Trade and Merchandise Marks Act (43 of 1958), S.56 - TRADE AND
MERCHANDISE - TRADE MARK - SUPREME COURT - Register of Trade Marks -
Rectification - Delay - Respondent-applicant taking action against every party throughout
world using Trade Mark evocating Scotland - No action, however, taken against appellant
for years even though a notice was issued - Explanation that they waited for decision of
S.C. on same issue - Hollow, since action was initiated even before S.C. decision was
passed - Application liable to be dismissed on principles of acquiescence, waiver. (Paras
51, 53, 55, 81)
(D) Trade and Merchandise Marks Act (43 of 1958), S.46 - TRADE AND
MERCHANDISE - TRADE MARK - DOCTRINES - Register of Trade Marks -
Rectification - Delay in making application - Court may refuse to rectify register -
Doctrine of continuing wrong has no application. (Para 76)
(E) Trade and Merchandise Marks Act (43 of 1958), S.46 - TRADE AND
MERCHANDISE - TRADE MARK - Register of Trade Marks - Rectification - Ground
deceptive similarity - Tests to be applied to decide deception - Use of brand name "Peter
Scot" by whisky manufacturer - Cannot deceive the class of buyers of scotch whisky -
Application for rectification liable to be dismissed.
2008 (36) PTC 315 (Mad.), Reversed.
The tests which are required to be applied to decide question of deceptive similarity in
trademarks in each case would be different. Each word must be taken separately. They
should be judged by their look and by their sound. Must consider the goods to which they
are to be applied. Nature and kind of customers who would likely to buy goods must also
be considered. Surrounding circumstances play an important factor. What would likely to
happen if each of those trade marks is used in a normal way as a trade mark of the goods
of the respective owners of the marks would also be a relevant factor. Where the class of
buyers is quite educated and rich, the test to be applied is different from the one where
the product would be purchased by the villagers, illiterate and poor. Ordinarily, again they
like tobacco, would purchase alcoholic beverages by their brand name. When, however,
the product is to be purchased both by villagers and town people, the test of a prudent
man would necessary be applied. It may be true that the tests which are to be applied in a
country like India may be different from the tests either in a country of England, United
States of America or Australia. (Paras 108, 110)
Thus, where rectification of Trade name Peter Scot registered by an Indian manufacturer
of whisky was sought on the ground that the use of words 'Scot' may cause deception in
the mind of purchasers that the whisky is of Scottish origin, considering the class of
people who buy Scotch Whisky the application was liable to be dismissed. The buyers are
supposed to know the value of money, the quality and content of Scotch Whisky. They
are supposed to be aware of the difference of the process of manufacture, the place of
manufacture and their origin.
2008 (36) PTC 315 (Mad), Reversed. (Para 125)
(F) Geographical Indication of Goods (Registration and Protection) Act (48 of 1999),
S.20(2), S.26(2) - TRADE MARK - APPLICABILITY OF AN ACT -
@page-SC2739
Protection to certain Trade Marks - Rectification of Trade Mark register - Ground that
Trade mark registered creates confusion as to place of manufacture in minds of buyers as
to take place of manufacture of product - Plea as to applicability of Act - Held not tenable
in view of S.22, S.26. (Para 127)
Cases Referred : Chronological Paras
(2008) Civ. Appeal No. 3639 of 2008, D/-16-5-2008 (SC) (Rel. on) 82
(2008) FCA 73 90
2007 (2) Scale 135 (Disting) 79
(2007) 143 Del LT 321 106
2007 (35) PTC 59 (Delhi) 73
2007 (5) AIR Kar R 318 : AIR 2007 (NOC) 2284 : 2007 AIHC 3606 74
2006 AIR SCW 4988 : AIR 2006 SC 3304 : 2006 CLC 1464 67, 68, 69
2006 (32) PTC 656 (Delhi) 48
2006 (33) PTC 527 48
2001 AIR SCW 1411 : AIR 2001 SC 1952 : 2001 CLC 564 107
(1999) 6 MLJ 280 (Malaysia) 48
1997 AIR SCW 1522 : AIR 1997 SC 1398 (Disting.) 78
1997 (17) PTC 134 48
1994 AIR SCW 2760 66
AIR 1994 Bom 231 104
AIR 1992 Bom 294 48, 61
AIR 1985 SC 1279 (Foll.) 30
(1984) 748 F2d 669 : 223 USPQ 1281 103
(1981)2 WLR 554 62
(1981) 2 WLR 576 63
(1980) RPC 31 123
(1980) 1 WLR 1265 62
ILR (1972) Del 124 48
AIR 1964 SC 1006 (Disting.) 31, 32
1961 (5) RPC 116 116
1960 MPC 304 31
1960 (1) RPC 16 110
AIR 1959 SC 798 77
AIR 1958 Bom 56 (Ref.) 42, 57
(1952) 86 CLR 536 95
(1952) HCA 15 (Aus) 96
(1947) 332 US 809 : 92 L Ed 386 99
1943 (60) RPC 29 120
(1906) 23 RPC 774 95, 108
(1895) AC 587 114
1889 (6) RPC 531 122
(1884) 26 Ch D 398 : 50 LT 784 66
(1880) 15 Ch D 96 : 49 LJ Ch 792 62, 72
(1866) LR 1 HL 129 : 14 WR 926 (HL) 63
(1860) 28 Beay 303 : 54 ER 382 66
(1847) 136 ER 816 : 17 LJCP 52 66
958 F 2d 594 101
476 F 2d 1357 99
162 F 2d 280 99
148 F 2d 1373 102
R. F. Nariman, Sr. Advocate, Lakshmi Narayana, Sunder Srinivas, Gopal Jain, Ajay
Bhargava, Mrs. Vanita Bhargava, Amit Verma, Giridharan P. and Ms. Nupur Mukherjee
(for M/s. Khaitan and Co.), with him for Appellant; Ashok H. Desai, Sr. Advocate, Zubin
Morris, Ms. Shruti Chanduary, Ms. Maneka Guruswamy, Ms. Mamta Tiwari, Ms. Swati
Sinha and Ms. Jayasree Singh (for M/s. Fox Manlal and Co.) with him for Respondents.
Judgement
S. B. SINHA, J. :- Leave granted.
2. This appeal by special leave is directed against the judgment and order dated 12th
October, 2007 passed by a Division Bench of the High Court of Judicature at Madras in
Trade Mark Second Appeal (TMSA) No. 2 of 1998 affirming the judgment and order
dated 25th September, 1998 passed in T.M.A. No.3 of 1989 whereby and whereunder an
appeal preferred by the appellant herein under Section 109 of the Trade and Merchandise
Marks Act, 1958 arising out of an order dated 12th May, 1979 by respondent No.3 was
dismissed.
3. Appellant is a company incorporated under the Companies Act, 1956. It manufactures
whisky under the mark 'Peter Scot'. Manufacture of the said product allegedly was started
by the company in May, 1968. An application was filed by it for registration of its mark
before the respondent No.3. Appellant was informed that its application was accepted and
allowed to proceed with the advertisement, subject to the condition that the mark would
be treated as associated with Reg. T.M. No.249226-B.
4. A proceeding was initiated as regards registration of the trade mark. No opposition was
filed by the respondent. Only one M/s. Mohan Meakins filed an opposition. The said
trade mark was registered.
5. Respondent Nos. 1 and 2 came to know of the appellant's mark on or about 20th
September, 1974. They filed an application for rectification of the said trade mark on 21st
April, 1986. We may also notice that a suit for passing off has also been filed by the 1st
respondent and others in the Bombay High Court being C.S. No. 1729 of 1987, which is
stated to be still pending.
6. Appellant having been called upon
@page-SC2740
showing its cause, by way of an affidavit affirmed by one Mr. Petern Warren explaining
the manner in which the word 'Peter Scot' was coined, stating :-
"While I was employed in Khoday, the whisky we produced was sold under at least two
marks, namely RED KNIGHT and PETER SCOT. The brand name 'Peter Scot' was
coined primarily with my father in mind i.e. using his forename, 'Peter', and his
nationality, 'Scot'. Another factor behind the coining of this brand name was the
internationally known British explorer, Captain Scott, and his son Peter Scott, who is
widely known as an artist, naturalist and Chairman of the World Wildlife Fund. Although
the name 'Scott' is spelt with two 't's, it is phonetically the same as 'Scot'.
7. Before the 3rd respondent, respondents 1 and 2 filed affidavits affirmed by 20 different
persons in support of their application for modification. One of the affidavits to which,
we would advert to a little later was affirmed by Ian Barclay, who is an inhouse Solicitor
of the said respondents.
8. Several issues were framed by the 3rd respondent in the said proceedings which were
as under -
"(1) Whether the applicants are "persons aggrieved" under Section 56;
(2) Whether the Application for rectification is not maintainable due to any mis-joinder of
Applicants;
(3) Whether the impugned mark was not distinctive of the goods of the registered
proprietors at the commencement of the rectification proceedings;
(4) Whether the impugned registration contravenes Section 11 at the commencement of
rectification proceedings;
and
(5) Whether the mark is liable to be rectified and if so, in what manner."
9. While opining that the 1st respondent has no locus standi to maintain the said
application for rectification, it upheld the locus standi of the 2nd respondent. As regards
Issue No.2 is concerned, it held that the mis-joinder of the applicants is not fatal. Issue
No.3 which was to the effect that whether the impugned mark was not distinctive of the
goods of the registered proprietors at the commencement of the rectification proceedings,
it was answered in favour of the appellant.
10. Issue No.4 which primarily concern us was discussed by the 3rd respondent at some
details. It was held that the mark has been used deceptively for long time and, although
there is also unexplainable and inexcusable delay on the part of the respondents in filing
the rectification application, the registered proprietors failed to file any evidence nor did
they raise their little finger to rebut the evidence filed by the applicants/ respondents. The
plea of acquiescence/delay raised by the appellant was negatived on the ground that the
plea of deceptive element in the impugned mark having neither been displaced nor
rebutted by evidence on the part of the registered proprietors, the pleas of delay and
acquiescence cannot be allowed in favour of the registered proprietors.
11. On the affidavit evidence filed on behalf of the respondents, respondent No.3,
although opined, that the same was not satisfactory but held the respondents plea that the
impugned registration contravenes Section 11 of the Act, stating :-
"Nonetheless, the evidence gives an impression that some customers are being persuaded
into thinking that PETER SCOT brand Whisky is also a Scotch Whisky. This is on
account of the presence of two factors, namely (1) the presence of the word Scot in the
PETER SCOT mark and (2) the presence of some slogan on the Whisky bottles under
PETER SCOT brand. Unfortunately, there is absolutely no evidence whatever from the
registered proprietors to assail or counteract or rebut the Applicants evidence. In the
absence of any material or evidence in rebuttal or reply from the registered proprietors,
the evidence of Applicants stands unquestioned, unrebutted, unassailed and even
unmitigated and I have no other go except to receive the voluminous affidavit and
documentary evidence filed by Applicants on its face value."
12. On the aforementioned premise, the application for rectification was allowed.
13. An appeal was preferred thereagainst by the appellant before the High Court in terms
of Section 109 of the Act.
14. A learned Single Judge of the High Court dismissed the said appeal. As regards the
plea of acquiescence it was held :-
"The acquiescence if it is to be made a ground for declining to rectify, must be of such a
character as to establish gross-negligence
@page-SC2741
on the part of the applicant or deliberate inaction which had regulated in the appellant
incurring substantial expenditure or being misled into the belief that the respondents
though entitled to, had deliberately refrained from taking any action and were unmindful
of the use of the mark by the person in whose name it was registered.
The facts of this case are not such as to warrant the conclusion that there has been
acquiescence."
15. Noticing that the appellant had neither adduced any evidence nor cross-examined the
deponents of the affidavits, it was held :-
"It is unfortunate that the appellant had chosen to refrain from placing any material
before the authority to refuse the claim for rectification. Apparently, appellant did not
have great faith in the validity of the registration and did not consider it worthwhile even
to cross-examine the witnesses who had filed the affidavits in support of the application
for rectification. It is evident that adoption of the mark 'Peter scot' by the appellant was
for the reasons mentioned in the affidavit of Peter Jeffery Warren. The mark was adopted
apparently with a view to take advantage of the good will associated with Scotch whisky
by using the word 'Scot' as part of the trade mark for the whisky manufactured by the
appellant."
16. As regards the quality of the affidavits it was held :-
"The criticism leveled against the deponents of some of the affidavits filed by the
respondents as evidence though justified to some extent does not help the appellant to
avoid rectification. The statutory standard is not the actual confusion and deception but
likelihood thereof. The term 'Scot' when used in association with whisky of non-Scottish
origin is inherently capable of and is likely to cause confusion and deception."
17. Feeling aggrieved, an intra court appeal was preferred thereagainst by the appellant. A
Division Bench of the High Court, as noticed hereinbefore, dismissed the said appeal.
The Division Bench noticed at some length the submissions made by the parties to inter
alia hold :-
"We have carefully considered the abovesaid submissions made by the counsel on either
side. In our considered view, the use of the device "Lion Rampant" and the abovesaid
description especially the description "Distilled from the Finest Malt and Blended with
the Choicest Whiskeys by Scotch Experts under Government Supervision" is definitely
intended to lead the consumers to believe that the whiskey manufactured by the appellant
is scotch whiskey. Though specific averments as above said have been made in the
affidavit on Ian Barclay, the same have not been rebutted by the appellant by adducing
rebuttal evidence. The appellant has also not chosen to cross-examine Ian Barclay on the
averments contained in the affidavit. Therefore, we are of the considered view that both
the third respondent and the learned single Judge have considered all the relevant
materials available on record and have exercised their discretion properly and as such we
do not find any reason to interfere with the judgment of the learned single Judge."
18. Mr. R.F. Nariman, learned senior counsel appearing on behalf of the appellant,
submitted :-
1. Respondent No. 3 as also the High Court committed a serious error insofar as they
failed to take into consideration that in view of the statement made in the affidavit
affirmed by Ian Barclay that the respondents were aware of infringement of marked as far
back in 1974 but as no action was taken in relation thereto till 1986, the application for
rectification was barred under the principles of waiver and acquiescence.
2. Long delay of 14 years caused in filing the said application for rectification should
have been held to be fatal having regard to the fact that the same caused immense
prejudice to the appellant as in the meantime, the sale of the appellant had gone many
folds.
3. Acquiescence on the part of the respondents would amount to waiver, if not
abandonment, of their right as any order passed for rectification of the mark being
unconscionable and inequitable, the same should not have been allowed.
4. The word 'Peter Scot' allegedly being evolved of Scotland cannot be held to be a
subject-matter of passing off as :-
a) Purchasers of the same are discerning;
b) They are aware of the brand which they would be purchasing;
c) Rich and wealthy people would only
@page-SC2742
ordinarily purchase goods like Scotch Whisky and they being literate cannot possibly be
pulled by the word 'Scot' knowing that they are purchasing Scotch Whisky, particularly in
view of the fact that in the label as also in the box, it has categorically been mentioned
that the product is a 'PRIDE OF INDIA' and is manufactured at Bangalore in India;
5. The Division Bench committed a serious error insofar it failed to take into
consideration the aforementioned arguments of the appellant, although categorically
noticed by it, so far as if the label is to be looked by in its entirety, the emblem of
Rampant Lion with the words 'Distilled from the Finest Malt and Blended with the
Choicest Whiskies by Scotch Experts under Government Supervision' must be read with
the word "PRIDE OF INDIA" and the names of the appellant and the fact that it was
manufactured at Bangalore.
6. The Division Bench of the High Court committed a serious error insofar as it failed to
take into consideration that having regard to the provisions contained in Section 26 of the
Geographical Indication of the Goods Act, 1999 (for short "the 1999 Act"), the rights of
trade marks which had been acquired through use in good faith were protected and
thereby committed a serious error in not allowing the appellant to raise the said
contention, on the premise that the same was being raised for the first time before it.
19. Mr. Ashok H. Desai, learned Senior Counsel appearing on behalf of the respondent
Nos. 1 and 2, on the other hand submitted :-
(i) The findings of fact arrived at by the respondent No. 3 and as affirmed by the learned
Single Judge and the Division Bench of the High Court should not be interfered with as
they cannot be characterized as extraneous or perverse being based on no evidence.
(ii) An application for rectification is the only remedy in respect of a registered trade
mark which is likely to deceive or cause confusion for the purpose of maintaining the
purity of register of trade marks wherefor the extended concept of passing off action
should be recognized.
(iii) The basic distinction between the trade mark infringement and passing off action is
that whereas the former flows from a statutory right, the latter is action in tort being in the
realm of unfair competition.
(iv) Courts in India and abroad having consistently protected Scotch Whisky, any mark
carrying words such as SCOT, Glen and Highland should not be allowed to continue.
Appellant by using its product as Peter Scot, which is an Indian Whisky, intended to be
seen as a manufacturer of Scotch Whisky, as would be evident from the affidavit of Mr.
Peter J. Warren.
(v) If there is a fraud at inception in adoption of the name, the court should discourage
such fraud in sternest way. It is not correct to contend that the delay, if any, on the part of
the respondents would amount to acquiescence or waiver as the purpose of filing an
application for rectification is to maintain the purity of register and public interest.
(vi) There is significant evidence in the present case as regards confusion concerning
whisky bearing the name of Peter Scot which having not been controverted or rebutted
would clearly go to show that any ordinary consumer would tend to believe that Peter
Scot is a Scotch and not an Indian Whisky.
(vii) The label used by the appellant clearly suggests that it is a Scotch Whisky and not of
Indian origin.
(viii) The provisions of the 1999 Act are not applicable to the facts and circumstances of
the present case.
20. Before adverting to the rival contentions we may notice some of the provisions of the
Act.
21. Sections 2(1)(d) ; 11, 27(2); 56 and 109 of the Act read as under :
"2. Definitions and interpretation - (1) In this Act, unless the context otherwise requires, -
(a) to
(c) .........................
(d) "deceptively similar" :- A mark shall be deemed to be deceptively similar to another
mark if it so nearly resembles that other mark as to be likely to deceive or cause
confusion;
11. Prohibition of registration of certain marks - A mark -
(a) the use of which would be likely to deceive or cause confusion; or
(b) the use of which would be contrary to any law for the time being in force; or
@page-SC2743
(c) which comprises or contains scandalous or obscene matter; or
(d) which comprises or contains any matter likely to hurt the religious susceptibilities of
any class or section of the citizens of India; or
(e) which would otherwise be disentitled to protection in a court,
shall not be registered as a trade mark.
27. No action for infringement of unregistered trade mark. -
(1)............
(2) Nothing in this Act shall be deemed to affect rights of action against any person for
passing off goods as the goods of another person or the remedies in respect thereof.
56. Power to cancel or vary registration and to rectify the register. - (1) On application
made in the prescribed manner to a High Court or to the Registrar by any person
aggrieved, the tribunal may make such order as it may think fit for cancelling or varying
the registration of a trade mark on the ground of any contravention, or failure to observe a
condition entered on the register in relation thereto.
(2) Any person aggrieved by the absence or omission from the register of any entry, or by
any entry made in the register without sufficient cause, or by any entry wrongly
remaining on the register, or by any error of defect in any entry in the register, may apply
in the prescribed manner to a High Court or to the Registrar, and the tribunal may make
such order for making, expunging or varying the entry as it may think fit.
(3) The tribunal may in any proceeding under this section decide any question that may
be necessary or expedient to decide in connection with the rectification of the register.
(4) The tribunal, of its own motion, may, after giving notice in the prescribed manner to
the parties concerned and after giving them an opportunity of being heard, make any
order referred to in sub-section (1) or sub-section (2).
(5) Any order of the High Court rectifying the register shall direct that notice of the
rectification shall be served upon the Registrar in the prescribed manner who shall upon
receipt of such notice rectify the register accordingly.
(6) The power to rectify the register conferred by this section shall include the power to
remove a trade mark registered in Part A of the register to Part B of the register.
109. Appeals. - (1) No appeal shall lie from any decision, order or direction made or
issued under this Act by the Central Government or from any act or order of the Registrar
for the purpose of giving effect to any such decision, order or direction.
(2) Save as otherwise expressly provided in sub-section (1) or in any other provision of
this Act, an appeal shall lie to the High Court within the prescribed period from any order
or decision of the Registrar under this Act or the rules made there-under.

(3) Every such appeal shall be preferred by petition in writing and shall be in such form
and shall contain such particulars as may be prescribed.
(4) Every such appeal shall be heard by a single Judge of the High Court :
Provided that any such Judge may, if he so thinks fit, refer the appeal at any stage of the
proceeding to a Bench of the High Court.
(5) Where an appeal is heard by a single Judge, a further appeal shall lie to a Bench of the
High Court.
(6) The High Court in disposing of an appeal under this section shall have the power to
make any order which the Registrar could make under this Act.
(7) In an appeal by an applicant for registration against a decision of the Registrar under
section 17 or section 18 or section 21, it shall not be open, save with the express
permission of the court, to the Registrar or any party opposing the appeal to advance
grounds other than those recorded in the said decision or advanced by the party in the
proceedings before the Registrar, as the case may be, and where any such additional
grounds are advanced, the applicant for registration may, on giving notice in the
prescribed manner, withdraw his application without being liable to pay the costs of the
Registrar or the parties opposing his application.
(8) Subject to the provisions of this Act and of the rules made thereunder, the provisions
of the Code of Civil Procedure, 1908 (5 of 1908), shall apply to appeals before a High
Court under this Act."
22. Although a large number of issues have been raised by the learned counsel for the
parties, we are of the opinion that the principal issues which arise for our consideration
@page-SC2744
are :
(i) Whether the delay on the part of the respondent Nos. 1 and 2 in filing the application
for rectification would amount to acquiescence and/or waiver?
(ii) Whether the respondent No. 3 as also the learned Single Judge and the Division
Bench of the High Court have failed to apply the correct tests and, thus, misdirected
themselves in law.
23. It is not in dispute that the appellant manufactures whisky under the brand name of
'Peter Scot'. The box of the carton contains the emblem of 'Rampant Lion'. It is a malt
whisky. On one side of the box it is stated "PRIDE OF INDIA" and on the other
"KHODAY DISTILLERIES PRIVATE LIMITED". Apart from the said information on
the right hand side of the label it is stated 'Distilled from the Finest Malt and Blended
with the Choicest Whiskies by Scotch Experts under Government Supervision'.
24. In terms of Section 56 of the Act, the Registrar exercises a quasi judicial power
wherefor he has to take into consideration the provisions of Sections 31 and 21 of the
Act. A presumption can be raised in terms of Section 31 in all the legal proceedings that
the trade mark is valid. The jurisdiction can be invoked if the mark is not distinctive or
the mark used falls under any of the clause envisaged in Sections 32 of the Act.
Furthermore if a mark had remained in existence for 7 years or more from the date of
registration, it would be presumed to be valid, unless :-
a) It had been obtained by fraud.
b) It is contrary to Section 11 ; and
c) The mark had not become distinctive.
25. In this case no plea of fraud has been raised. Registrar held in favour of the appellant
that the mark had not become distinctive. The respondents' case is based on the premise
that the same was contrary to Section 11 of the Act.
26. In support of their application, the respondents raised the following grounds :
1) The mark is not distinctive.
2) It was not capable of distinguishing itself as the goods of the appellant.
3) The use of the mark is likely to deceive or confuse.
4) Non-user of the mark.
27. They have succeeded before the learned Magistrate only on the third ground.
28. The principal question which arises for consideration is as to whether the term 'Scot'
would itself be a sufficient ground to opine that the mark 'Peter Scot' is deceptive or
confusing. Indisputably the onus of proof would be open the respondents. The question
arises is as to whether they have discharged the same or not.
RE : ISSUE NO. 1
29. Ian Barclay is admittedly the in-house Solicitor of the respondents. They have not
only been filing actions against several persons infringing the said mark in India but also
in several other countries like Australia and United Sates of America. Ian Barclay in his
affidavit stated :-
"The first applicant received notice of the advertisement of the said mark PETER SCOT
in the Trade Mark Journal when it received a routine report from Wildbore and Gibbons
dated 20th September, 1974. Regrettably, the first Applicant did not lodge opposition
with the time allowed..."
30. Respondents, therefore, were well aware that the appellant had filed an application for
registration. One of the questions which was raised before respondent No.3 as also before
the High Court was as to whether Article 137 of the Limitation Act, 1963 would apply to
the rectification proceedings. Keeping in view the decision of this Court in Sakur v.
Tanaji [AIR 1985 SC 1279], evidently the same has to be rejected as the Registrar is not a
court.
31

. The submission of Mr. Nariman, however, is that the period of three years provided for
should be taken to be the upper limit as an equitable jurisdiction is to be invoked. Our
attention in this behalf has been drawn to the decision of this Court in State of Madhya
Pradesh v. Bhailal Bhai and others, [1964 (6) SCR 261 at 271] wherein it was held :-
AIR 1964 SC 1006, Paras 20 and 21

"It was necessary for the High Court to consider this question of delay before any order
for refund was made. It does not appear however that any attention was paid to this
question. In making the orders for refund in each of these cases the High Court merely
said this :
"The present case is governed by Bhailal Bhai case. Learned Government Advocate
formally raised the question of the remedy open to the petitioner for refund of tax in
@page-SC2745
order to keep the point open in the Supreme Court. We accordingly allow this petition
and issue a writ directing the opponents to refund to the applicant firm the amount of tax
collected from it during the above mentioned period."
The learned Judges appear to have failed to notice that the delay in these petitions was
more than the delay in the petition made in Bhailal Bhai case out of which Civil Appeal
No. 362 of 62 has arisen. On behalf of the respondents-petitioners in these appeals (CAs
Nos. 861 to 867 of 1962) Mr. Andley has argued that the delay in these cases even is not
such as would justify refusal of the order for refund. We argued that assuming that the
remedy of recovery by action in a Civil Court stood barred on the date these applications
were made that would be no reason to refuse relief under Article 226 of the Constitution.
Learned counsel is right in his submission that the provisions of the Limitation Act do not
as such apply to the granting of relief under Art 226. It appears to us however that the
maximum period fixed by the legislature as the time within which the relief by a suit in a
Civil Court must be brought may ordinarily be taken to be a reasonable standard by
which delay in seeking remedy under Article 226 can be measured. The court may
consider the delay unreasonable even if it is less than the period of limitation prescribed
for a civil action for the remedy but where the delay is more than this period, it will
almost always be proper for the court to hold that it is unreasonable. The period of
limitation prescribed for recovery of money paid by mistake under the Limitation Act is
three years from the date when the mistake is known. If the mistake was known in these
cases on or shortly after January 17, 1956 the delay in making these applications should
be considered unreasonable. If, on the other hand, as Mr. Andley seems to argue, that the
mistake discovered much later this would be a controversial fact which cannot
conveniently be decided in proceedings. In either view of the matter we are of opinion the
orders for refund made by the High Court in these seven cases cannot be sustained."
32

. We are unable to accept this submission. What would be the reasonable period in a
given case would not depend upon the nature of action initiated before a statutory
authority but also upon the purport and object of the statute. If the reliefs sought for
before the superior courts are the same which could be sought for in a suit, the period of
limitation specified for in the Limitation Act may be taken to be the criteria for
consideration as to whether the same should be treated as a reasonable period, but not
otherwise. In Bhailal Bhai and others (supra) a writ petition was filed for refund of the
tax which was legally collected. It was essentially a money claim and in that context the
aforementioned observations were made. AIR 1964 SC 1006

33. It was next contended that the appellant suffered a great prejudice by reason of delay
as during the said period of 18 years the sales figures had gone up. Our attention has been
drawn to a chart filed before the learned Single Judge on 6th July, 1988. Such a stand
apparently had not been taken before the respondent No. 1. No application appears to
have been filed in terms of Order 41 Rule 27 of the Code of Procedure or principles
analogous thereto nor the learned Single Judge appears to have permitted them to
produce additional evidence before it. The sales figures as contained in the said chart,
therefore, cannot be taken into consideration.
34. We would, however, assume that such a delay ex facie causes prejudice as a
manufacturer of a popular brand. It must have arranged its affairs in such a manner so
that it may not have to withdraw the brand or the mark after it gains popularity.
35. Respondent No. 3 as also the High Court refused to entertain the plea of delay leading
to application of the principles of estoppel and/or waiver only on the premise that the
appellant is guilty of adopting a mark which is deceptively similar and/or causes
confusion.
36. Contention raised before us by Mr. Desai is that having regard to the provisions of the
Act, no application for rectification should be rejected on the ground of delay.
37. The said contention is based on the premise that Section 11 of the Act provides for a
prohibition of registration of certain marks, the use of which would likely to deceive or
cause confusion.
38. Section 56 of the Act provides for filing of application and application for
rectification and, thus, should be considered only from that point of view. Registration of
a trade mark is governed by the provisions of the Act. Section 9 thereof provides for
restrictions
@page-SC2746
for registration in Parts A and B of the Register unless the essential particulars laid down
therein are satisfied. It provides for a detailed enquiry. An advertisement has to be issued.
Objections are to be called for. Only upon proof of existence of one or the other essential
particulars therein and upon consideration of the objections which may be raised therein,
the registration of mark may be allowed. Sections 10 to 14 provide for prohibitions. The
prohibitory provisions, therefore, are required to be taken into consideration for the
purpose of registration of the mark. The question as to whether the use of a mark would
likely to deceive or cause confusion so as to disable the Registrar from registering the
mark as a trade mark would be necessary to be considered only in the course of the
enquiry conducted therefor. A Registrar of Trade Mark is not supposed to know that there
exist other marks which are registered or which would deceive or cause confusion with
any other established mark. In a given case, the Registrar of Trade Mark may be aware
thereof. But, in some cases, he may not be. It may, therefore, not be correct to contend
that Section 11 of the Act prohibits the Registrar for registration of marks which would
likely to deceive or cause confusion.
39. Indisputably, the purity of the Registrar is to maintain the register. Indisputably again,
the public interest has to be kept in view. An application for registration has also to be
considered keeping the public interest in view. What is therefor, necessary for the
Registrar is to arrive at a conclusion as regards registration of mark, is as to whether
having regard to the nature of the mark sought to be registered and the use thereof as also
the class of bias, would be deceived or confused with the mark registered or not.
40. An application for rectification and correction of the register may be entertained if
any of the grounds specified therein exists, viz., contravention or failure to observe the
condition entered on the register or in relation thereto. An application for rectification and
correction of the register would also be maintainable if a person is aggrieved by the
absence or omission from the register of any entry or by any entry made in the register
without sufficient cause or by any entry wrongly remaining on the register or by any error
or defect in any entry in the register.
41. The power of the Registrar in terms of Section 56 of the Act is wide. Sub-section (2)
of Section 56 of the Act used the word "may" at two places. It enables a person aggrieved
to file an application. It enables the Tribunal to make such order as it may think fit. It may
not, therefore, be correct to contend that under no circumstances the delay or
acquiescence or waiver or any other principle analogous thereto would apply.
42. Purity of register as also the public interest would indisputably be relevant
consideration. But, when a discretionary jurisdiction has been conferred on a statutory
authority, the same although would be required to be considered on objective criteria but
as a legal principle it cannot be said that the delay leading to acquiescence or waiver or
abandonment will have no role to play. [See Ciba Ltd. Basle Switzerland v. M.
Ramalingam and S. Subramaniam trading in the name of South Indian Manufacturing
Co., Madura and another AIR 1958 Bombay 56]
43. In determining the said question, therefore, conduct of the person aggrieved in filing
the application for rectification would be relevant.
44. For the aforementioned purpose, whether it is a class or group action or a private
action although would be relevant but may not be decisive. It is one thing to say that class
or group action will receive special attention of the statutory authority vis-a-vis a private
action. But, in both types of cases, public interest should remain uppermost in the mind of
the authority. The question which is required to be posed therefor would be as to whether
the public in general or the class of bias would be deceived or be confused if the existing
mark is allowed to remain on the register. Thus, deceptively similar or confusion is the
principal criteria for determining applications both for registration as also for
rectification.
45. We have noticed hereinbefore that it is not the case of the respondents that any fraud
was practised by the appellant.
46. Appellant started manufacturing of the product as far back as in the year 1968. It
marketed two brand names, viz., Peter Scot and Red Knight. It is said to have been done
for making the name attractive. It was a period when there were restrictions on import of
Whisky. The custom duty was high. The price of a genuine Scotch Whisky was
prohibitive for a large section of the consumers.
@page-SC2747
Appellant applied for registration in the year 1971.
47. Ian Glen Barclay affirmed an affidavit in support of the respondents. He, in his
affidavit which otherwise remained uncontroverted, admitted that the respondents were
aware of registration of the mark through an advertisement made in a trade mark journal
at the instance of the appellant as also a report received from Wildbore and Gibbons
dated 20.09.1974. Respondent No. 1 had issued a notice. It also issued a notice in respect
of an application filed by the appellant for registered proprietors trade mark 'Hogmanay'
which led to withdrawal of the said application by the appellant before hearing. He also
affirmed that there are provisional trade mark agencies about the trade marks
advertisements in all the countries in classes 32 and 33 which inter alia have British or
Scottish connotations.
48. On their own showing a large number of applications were filed against those who
were using trade marks which have some relation with use of a label, mark or insignia
which reminds the customers of the products of Scotland which included the word such
as Scot, Glen and Highland. Respondents have referred to as many as 19 judgments
rendered by different High Courts in India and four decisions rendered by the Courts of
France, Italy, Illinois and Malaysia to contend that all oppositions made and/or litigations
filed ended in success. It is relevant to place on record the decisions which have been
placed before us by Mr. Desai and the marks which were being used by different
companies :
1. Mohan Meakin Breweries Ltd. Vs. The Scotch Whisky Association, PTC (Suppl) (1)
352 (Del) (DB) Para 6, Paras 17 and 18
2. Scotch Whisky Association and Ors. Vs. Golden Bottling Ltd., 2006 (32) PTC 656
(Del.) Paras : 22, 23, 24, 25, 27 and 28.
3. Srilab Breweries Pvt. Ltd. Vs. Scotch Whisky Association, 2006 (33) PTC 527 (Reg.)
Para 13
4. SWA and Anr. Vs. Mohan Meakin Ltd. (Royal Scot) (Bombay High Court)
5. Scotch Whisky Association and Ors. Vs. Golden Bottling Ltd. (Red Scot) 2006 (32)
PTC 656 (Del.)
6. SWA and Anr. Vs. Royal Distillery (Royal's Scot) (Bombay High Court)
7. SWA and Ors. Vs. The Brihan Maharashtra Sugar Syndicate Ltd. (Brihan's Old Scot)
(Bombay High Court).
8. SWA and Ors. Vs. The Ugar Sugar Works Ltd. (Appeal Order) (Bombay High Court).
9. SWA and Anr. Vs. Rangar Breweries Ltd. (Grand Scot) (Bombay High Court)
10.SWA and Anr. Vs. Swaroop Vegetable Products Industries Ltd. and Ors. (Final Order)
(Delhi High Court)
11. Scotch Whisky Association Vs. Dyer Meakin Breweries Ltd. (Highland Chief) ILR
1972 Delhi 124
12. Dyer Meakin Breweries Ltd. (now known as Mohan Meakin Breweries Ltd.) Vs. The
Scotch Whisky Association PTC (Suppl) (1) 352 (Del) (DB)
13. SWA and Anr. Vs. M/s Rheea Distillers (Scotch Terrier) (The Court of Addl. Civil
Judge, Sr. Division Margao, Goa).
14. William Grant and Sons Ltd. Vs. McDowell and Co. Ltd. (Glenfiddich Case) 1997
(17) PTC 134
15. Srilab Breweries Pvt. Ltd. Vs. Scotch Whisky Association (Rare Blend) 2006 (33)
PTC 527 (Reg.) WA and Anr. Vs. Forbes Camphell and Co. Ltd. (Glen Forbes, Blended
with the finest Scotch) (Bombay High Court)

16. SWA and Anr. Vs. Pravara Sahakar Shakar Karkhana Ltd. (Drum Beater, Gold
Tycoon) (Bombay High Court) AIR 1992 Bom 294

17. SWA and Anr. Vs. Scottish Distilleries and Ors. (Macqueen and Black Kilt) (Bombay
High Court)
18. SWA Vs. Silver Oak Blenders and Bottlers Private Ltd. and Anr. (Windsor Club)
(Bombay High Court).
19. SWA Vs. Maharashtra Manufacturing Corporation and Ors. (Black Skipper, White
Scot and Salute India) (Delhi High Court)
Decisions of Foreign Courts :
20. SWA Vs. Societe d' Importation e de Distribution des Grandes Marques (Judgment
dated 23rd January, 1992 of the Commercial Court of Saint Etienne, France)
21. SWA and Anr. Vs. Pollini Liquori Spa (Judgment dated 19th March, 1980 passed by
the Court of Rome, Italy)
22. SWA and Ors. Vs. Bartaon Distillery Co. (Judgment dated 12th November, 1973
passed by the United States Court of Appeal, Illinois)
23. SWA and Anr. Vs. Ewein Winery (M)
@page-SC2748
Sdn. BhD [1999] 6 MLJ 280 (Malaysia)"
49. The details of the cases aforementioned would clearly go to show that not only
oppositions were made, but also actions were initiated against the persons who used the
word 'Scot' as also against those who used the words 'Highland Chief, Scotch Terrier,
Glenfiddich, Rare Blend', etc.
50. In Australia and United States of America, the respondent No. 1 initiated actions
almost on the self-same cause of action but failed.
51. It is, therefore, evident that whereas actions had been taken against each and every
party throughout the world whenever a mark evocating Scotland or any other brand
which remained in the minds of the buyer of the Scotland had been opposed, the
appellant was singularly left out. They opposed to the registration of the mark and in fact
issued a notice. If it had issued a notice then there is absolutely no reason as to why they
did not pursue the same. A notice was issued in relation to an attempt made by the
appellant to get the name Hogmanay registered. If such an opposition had been made
even in relation to the registered proprietors trade mark Peter Scot, as it did in the case of
Hogmanay, it could have withdrawn its application. It would have known its position as
to where it stood. It could have started manufacturing Whisky of the same quality with a
different brand name. We may furthermore place on record that not only in respect of
Hogmanay, opposition was also made in relation to 'Old Angus' in Class 32 which bears
the name of a county in Scotland and, therefore, evocative of Scottish origin although the
product was a 'Rum' and not 'Whisky'. If the respondent No. 1 took such actions in
respect of trade marks 'Hogmanay' and 'Old Angus' in 1974 and 1979, one fails to
understand as to why a similar action was not taken in relation to Peter Scot.
52. Barclay, in his affidavit, attempted to give an explanation therefor stating :-
"43. Proceedings in respect of the registration of the mark "HIGHLAND CHIEF" were
pending in the Indian Courts at the relevant time. The 1st Applicant succeeded in these
proceedings before the Delhi Court in 1971 but thereafter the matter was taken to the
Division Bench of the Delhi High Court where the 1st Applicant also succeeded.
However, the matter was thereafter taken to the Supreme Court. The 1st Applicant did not
appreciate, that it would take so long to obtain a final judgment. The Judgment from the
Supreme Court would be binding on all tribunals in India. At issue in the said Appeal was
registration of words, or devices or marks or labels or descriptions evocative of Scotland
for use on Indian Whisky. In this connection I crave leave to refer to and rely upon the
judgment of the Delhi High Court (see Exhibit X-4). I say that the Appeal to the Supreme
Court from the same judgment had not yet been decided.
44. I say that the 1st Applicant, having failed to lodge opposition to the 'Peter Scot'
application within the prescribed period, was awaiting the decision of the Supreme Court
before adopting proceedings against the registered proprietor for rectification of the mark
PETER SCOT. However, in recent years, the 1st Applicant learned that various other
parties were attempting to adopt the word 'SCOT as part of their trade marks, and/or other
marks with Scottish Connotations. Therefore, the 1st Applicant decided that it could not
afford to wait for the decision of the Supreme Court and proceedings were adopted by
filing passing off actions which are referred to in the annexures referred to hereinabove,
as well as the rectification proceedings against the registered proprietor. Thus, the delay is
due to the aforesaid facts."
53. Ex facie, the said explanation is wholly unacceptable.
54. Even before us, the result of the litigation before this Court has not been disclosed.
When action had been taken by the respondent No. 1 in so many matters although one of
the matters was pending before this Court, the explanation offered appeared to be
absolutely hollow, particularly when even on its own showing the respondent No. 1
succeeded even in the said proceedings.
55. Moreover, if the said explanation is to be accepted, then why an application for
rectification should have been filed even in 1986? They should have waited till the
decision in the case then pending was rendered by this Court. Nothing has been stated in
the said affidavit as to what was the occasion to file such an application in 1986 but
prompted them to wake up after such a long time is beyond anybodys comprehension.
56. Once the facts are admitted the legal
@page-SC2749
inference shall ensue. Respondent No. 3 as also the learned Single Judge and the Division
Bench of the High Court failed and/or neglected to advert to this aspect of the matter.
They did not pose unto themselves the question as to whether an application for
rectification can be dismissed on the ground of principle of delay, acquiescence, waiver
or abandonment. It is one thing to say that such principles can never be applied but it is
another thing to say that whether they should be applied in a fact situation obtaining in a
particular case or not. If they have any application, the same should have been considered
by the respondent No. 3 as also by the High Court on their own merit.
57

. Mr. Desai himself has relied upon a decision of Chagla, C.J. of the Bombay High Court
in Ciba Ltd. Basle Switzerland (supra). The learned Chief Justice therein recorded his
opinion, thus : AIR 1958 Bom 56

"10. The second contention in which there is much more force is that the appellants have
come to file this application under Section 46 after considerable delay and no relief
should be granted to them. It is true that the granting of relief under Section 46 is a
discretionary relief, but it is equally true that the Legislature has not laid down any period
of limitation for making an application under Section 49. Kerly at p. 271 states : "The
delay of the applicant in coming to the Court to ask for rectification of the register is not
of itself any bar to his application, and the marks have been removed or varied after being
registered for long periods to the knowledge of the applicant....." But at p. 264 the learned
author points out that the demerits of the applicant in any such case are irrelevant to the
question whether rectification should be ordered or not, for the question is not between
the applicant on the one hand and the respondent on the other, but between public and the
respondent. But the learned author further points out at p. 272 that where the objection
alleged to a mark is that it to the same as that of the applicant, or that it has such
resemblance to his as to be calculated to deceive, it will be some evidence against the
applicant, on whom the burden lies of showing that the registration was made without
sufficient cause, if he had stood by and allowed the registered proprietor to use the mark
objected to for a length of time, especially if no case of actual deception is proved. But
the learned author also points out that in such a case two things have got to be
considered; first, what was the reason of the delay in the application to expunge; and,
secondly, whether any substantial injury has been caused by the delay to the person who
has registered. Therefore, the question of delay must be approached from this point of
view whether the applicant stood by and thereby caused substantial injury to the
respondent and the injury was so substantial that that injury would outweigh the interest
of the public which the Court must consider where a trade mark is likely to deceive."
58. In this case, the respondent stood by the mark.
59. A distinction may be made between a 'fraudulent trade mark' and a 'deceptive trade
mark'. In both the cases evidences have to be adduced.
60. Whether a misrepresentation is made or not is essentially a question of fact. It will
depend upon many factors.
61. The principles of waiver and acquiescence in a case of this nature are applicable.
Apart from the ordinary rule of waiver of a right expressly provided for in a case of
passing off, the court has consistently been noticing development of law in this field. The
principles in that behalf were laid down by way of provender in Scotch Whiskey
Association and another v. Pravara Sahakar Shakar Karkhana, [AIR 1992 Bom. 294].
62. The development of law was also noticed by the Court of Appeal in Habib Bank Ltd.
v. Habib Bank, A.G. Zurich, [ [1980] (1) W.L.R. 1265 ] at 1283 1284, in the following
terms :-
"We were again referred to many authorities on this subject and to the debate which has
taken place as to whether, in order to succeed in a plea of acquiescence, a defendant must
demonstrate all the five probanda contained in the judgment of Fry J. in Willmott v.
Barber [(1880) 15 Ch.D.96 : see the recent judgment of Robert Goff J. in Amatgamated
Investment and Property Co. Ltd. v. Texas Commerce International Bank Ltd. [(1981) 2
WLR 554]. Whether all five of those probanda are necessary or not, Mr. Aldous submits
that to succeed HBZ must at least establish three things. They must show, first, that HBZ
have been acting under a mistake as to their legal rights. That, in the instant case, must
mean that they
@page-SC2750
were unaware that what they were doing (that is to say, carrying on their business under
the name in which they had been incorporated with the active assistance of the plaintiffs'
predecessors), constituted any invasion of the plaintiffs' rights. Secondly, they must show
that the plaintiffs encouraged that course of action, either by statements or conduct.
Thirdly, they must show that they have acted upon the plaintiffs representation or
encouragement to their detriment."
63. Noticing various other decisions, Oliver, L.J., noticing a decision in Taylor Fashions
Ltd. v. Liverpool Victoria Trustees Co. Ltd. [ (Note) [1981) 2 W.L.R.] 576 opined :-
"Furthermore the more recent cases indicate, in my judgment, that the application of the
Ramsden v. Dyson, L.R. 1 H.L. 129 principle - whether you call it proprietary estoppel,
estoppel by acquiescence or estoppel by encouragement is really immaterial - requires a
very much broader approach which is directed rather at ascertaining whether, in particular
individual circumstances, it would be unconscionable for a party to be permitted to deny
that which, knowingly, or unknowingly, he has allowed or encouraged another to assume
to his detriment than to inquiring whether the circumstances can be fitted within the
confines of some preconceived formula serving as a universal yardstick for every form of
unconscionable behaviour."
64. It was held :
"I have to acknowledge my indebtedness to counsel on both sides for some illuminating
arguments, but at the end of them I find myself entirely unpersuaded that the judge erred
in any material respect. He concluded his judgment in this way on the question of
estoppel :
"Of course, estoppel by conduct has been a field of the law in which there has been
considerable expansion over the years and it appears to me that it is essentially the
application of a rule by which justice is done where the circumstances of the conduct and
behaviour of the party to an action are such that it would be wholly inequitable that he
should be entitled to succeed in the proceeding."
That, to my mind, sufficiently appears on the facts of this case."
65. Thus, in cases involving equity or justice also, conduct of the parties has also been
considered to be a ground for attracting the doctrine of estoppel by acquiescence or
waiver for infringement.
66

. This Court also in M/s. Power Control Appliances and others v. Sumeet Research and
Holdings, [(1994) 2 SCC 448] held :-1994 AIR SCW 2760, Para 29

"26. Acquiescence is sitting by, when another is invading the rights and spending money
on it. It is a course of conduct inconsistent with the claim for exclusive rights in a trade
mark, trade name etc. It implies positive acts; not merely silence or inaction such as is
involved in laches. In Harcourt v. White Sr. John Romilly said : "It is important to
distinguish mere negligence and acquiescence". Therefore, acquiescence is one facet of
delay. If the plaintiff stood by knowingly and let the defendants build up an important
trade until it had become necessary to crush it, then the plaintiffs would be stopped by
their acquiescence. If the acquiescence in the infringement amounts to consent, it will be
a complete defence as was laid down in Mouson (J.G.) and Co. v. Boehm. The
acquiescence must be such as to lead to the inference of a licence sufficient to create a
new right in the defendant as was laid down in Rodgers v. Nowill.
67

. The question again came up for consideration before this Court in Ramdev Food
Products (P) Ltd. v. Arvindbhai Rambhai Patel and others, [(2006) 8 SCC 726] wherein it
was held :- 2006 AIR SCW 4988, Para 105

"103. Acquiescence is a facet of delay. The principle of acquiescence would apply


where : (i) sitting by or allowing another to invade the rights and spending money on it;
(ii) it is a course of conduct inconsistent with the claim for exclusive rights for trade
mark, trade name, etc."
68

. The delay by itself, however, may not be necessarily a ground for refusing to issue
injunction. It was opined :- 2006 AIR SCW 4988, Para 108

"106. The defence of acquiescence, thus, would be satisfied when the plaintiff assents to
or lays by in relation to the acts of another person and in view of that assent or laying by
and consequent acts it would be unjust in all the circumstances to grant the specific
relief."
69. It was furthermore observed :-

108. Specific knowledge on the part of the 2006 AIR SCW 4988, Para 110

@page-SC2751
plaintiff and prejudice suffered by the defendant is also a relevant factor. (See Spry on
Equitable Remedies, 4th Edn., p. 433.)
70. In Halsbury's Laws of England, Fourth Edition, Vol. 16, para 1505, it is stated :
"Where a person has by words or conduct made to another a clear unequivocal
representation of fact, either with knowledge of its falsehood or with the intention that it
should be acted upon, or has so conducted himself that another would, as a reasonable
man, understand that a certain representation of fact was intended to be acted on, and that
the other has acted on the representation and thereby altered his position to his prejudice
an estoppel arises against the party who made the representation, and he is not allowed to
aver that the fact is otherwise than he represented it to be."
71. Delay would be a valid defence where it has caused a change in the subject matter
and action or brought about a situation in which justice cannot be done.
72. Mr. Desai relied upon Willmott v. Barber [15 Ch D 96] wherein it is stated :
"The equitable doctrine of acquiescence is founded on there having been a mistake of
fact; can it be repelled by showing that there was constructive notice of the real facts? In
every case in which a man acts under the mistaken belief that he is entitled to land, he
might, if he had inquired, have found out that he had no time. And yet the Courts appear
always to have inquired simply whether a mistake has been made, not whether the
plaintiff ought to have made it."
73. However, in Pfizer Products Inc. v. Rajesh Chopra and Ors. [2007 (35) PTC 59
(Del)], injunction was granted despite delay, stating :-
"Where infringement is deliberate and wilful and the defendant acts fraudulently with
knowledge that he is violating plaintiff's rights, essential elements of estoppel are lacking
and in such a case the protection of plaintiff's rights by injunctive relief never is properly
denied. The doctrine of estoppel can only be invoked to promote fair dealings.
31. It would appear to me that where there is an honest concurrent user by the defendant
then inordinate delay or laches may defeat the claim of damages or rendition of accounts
but the relief of injunction should not be refused. This is so because it is the interest of the
general public, which is the third party for such cases, which has to be kept in mind. In
the case of inordinate delay or laches, as distinguished from the case of an acquiescence,
the main prejudice which may be caused to the defendant is that by reasons of the
plaintiff not acting at an earlier point of time the defendant has been able to establish his
business by using the infringing mark. Inordinate delay or laches may be there because
the plaintiff may not be aware of the infringement by the defendant or the plaintiff may
consider such infringement by the defendant as not being serious enough to hurt the
plaintiff's business. Nevertheless, if the Court comes to the conclusion that prejudice is
likely to be caused to the general public who may be misled into buying the goods
manufactured by the defendant thinking them to be the goods of the plaintiff then an
injunction must be issued. The Court may, in appropriate cases, allow some time to the
defendants to sell off their existing stock but an injunction should not be denied."
74

. In Me Donald's Corporation and Another v. Sterlings Mac Fast Food Represented by its
Partner John Mathew [ILR 2007 Karnataka 3346], the Karnataka High Court held :
2007(5) AIR Kar R 318

"9. Though the plaintiff adopted its registered trade mark BIG MAC in 1968 in USA, in
paragraph 7 of memorandum of appeal it is stated as under :-
'7. The first appellant is the proprietor in India of other McFamily trade marks having
prefix or suffix Mc/MAC and it has also applied for registration and most of them were
registered during the pendency of the suit and the trademarks are used in India since
October, 1996.'
From the above, it is clear that the plaintiffs are using their trade marks in India since
October 1996. However, in the very next paragraph it is stated as under :
'8. The first appellant was given approval on February 15, 1993 by the Government of
India to operate a chain of restaurants in India. The second appellant was incorporated
and registered with the Registrar of Companies on August 30, 1993 vide (Ex. P-30)'.
It is thus clear that plaintiffs got registered in India in the year 1993 only. On the
@page-SC2752
other hand, the defendant has been carrying on with its business since 1983. Having
started business by the plaintiffs much later than the defendant in India, the plaintiffs
cannot found fault with the defendant in using its trademark in its business."
75. A contention is sought to be raised that the purported wrong committed by the
appellant being a continuing one would not attract the doctrine of laches, acquiescence or
waiver.
76. The doctrine of continuing wrong has nothing to do with the refusal on the part of a
statutory authority or a court of law to exercise its discretionary jurisdiction on the
ground of laches, acquiescence or waiver.
77. In Balakrishna Savalram Pujari Waghmare and others v. Shree Dhyaneshwar Maharaj
Sansthan and others [AIR 1959 SC 798], this Court, while dealing with a question of
applicability of Articles 124 and 120 of the Limitation Act, 1908, made a distinction
between a wrongful act causing an injury which is complete and a wrong which creates a
continuing source of injury rendering the doer of the act responsible and liable for the
continuance of the said injury.
78

. Reliance placed by Mr. Desai on Bengal Waterproof Limited v. Bombay Waterproof


Manufacturing Company and Another [(1997) 1 SCC 99] cannot be said to have any
application herein whatsoever. Therein, this Court principally was dealing with a petition
as regards application of Order II, Rule 2(3) of the Code of Civil Procedure. Holding that
the cause of action filed in the suit in question was not the same on which the earlier suit
was passed and further opining that the cause of action for filing the suit was a
continuous and recurring, infringement of the plaintiffs trade mark by the respondents
continuously till the date of filing of the second suit, it was held that the presence in the
register of a mark was a continuous wrong. However, it is not necessary to delve deep
into the matter any further as we have held heretobefore that the provisions of the
Limitation Act, 1963 will have no application in the instant case. 1997 AIR SCW 1522

79. It is also not a case where a court has been conferred power to exercise a suo motu
jurisdiction. Reliance placed by the appellant in State of Punjab and Ors. v. Bhatinda
District Coop. Milk P. Union Ltd. [2007 (2) SCALE 135], therefore, cannot be said to
have any application.
80. Whereas on the one hand Mr. Desai objects to the evidence that was produced before
the learned Single Judge with regard to the increase in the volume of sale of Peter Scot,
on the other hand, it was urged that if a comparison is made of the Indian whisky and
Scotch Whisky it would appear that some Indian whiskies are costlier than some of the
Scottish brands. The stand taken by the respondents is self contradictory. We think that
their stand is not fair.
81. We, therefore, in the peculiar facts and circumstances of this case, are of the opinion
that action of the respondents is barred under the principles of acquiescence and/or
waiver.
82. The power conferred on the Registrar is discretionary in nature. In a given case, the
Registrar may not exercise its jurisdiction. [See Kabushiki Kaisha Toshiba v. Tosiba
Appliances Co. and Ors. [Civil Appeal No. 3639 of 2008 decided on 16th May, 2008]
RE : ISSUE NO. 2
83. Another principal question which arises for consideration is as to whether the use of
the term 'Scot' would itself be a sufficient ground to form an opinion that the mark 'Peter
Scot' is deceptive or confusing. Indisputably, the onus of proof therefor would be on the
respondents. Whether they have discharged the said onus is the question?
84. The nature of a passing off action visa-vis the action for infringement of a registered
trade mark may lead to ground of the same reliefs, but we, at the outset, may notice that
in Modern Law Trade Marks by Morcom, it is stated :
"Passing off the group or class type action the necessary characteristics.
14.64 A trade name may come to denote goods of a particular type or class, Traders who
sell these goods do not have the exclusive rights in the name. The goodwill in the name is
shared by all of the traders who sell this type or class of goods. A passing off action may
be brought by one or more of these traders to protect the goodwill in the name."
85. A passing off action may be brought by those who fulfil the following requirements :
@page-SC2753
(i) the claimant himself owns or has a sufficient proprietary interest in the requisite
goodwill, and
(ii) the goodwill so owned must be the goodwill which is really likely to be damaged by
the alleged misrepresentations.
86. The decision of the respondent No. 3 that the respondent No. 1 has no locus standi to
file an application under Section 56 of the Act has attained finality. Locus has been found
only in favour of the respondent No. 2. It comes within the purview of a foreign
manufacturer whose goods are sold in this country. It indisputably have acquired a
protectable goodwill in his trade name or trademarks. There will then be the question,
which in principle would involve pure question of fact, is whether the goodwill is that of
the foreign manufacturer or the importer. [See Kerly's Law of Trade Marks and Trade
Names, Fourteenth Edition, para 15-067, page 456]
87. The Law in this regard has been specifically stated in Kerly's Law of Trade Marks
and Trade Names, Thirteenth Edition, pg. 600 in the following terms:
"(3) If the goods are expensive or important to the purchasers and not of a kind usually
selected without deliberation, and the customers generally educated persons, these are all
matters to be considered."
88. Respondent No. 1 initiated some proceedings in Australia and United States of
America.
89. We may notice some precedents operating in Australia and United States of America.
AUSTRALIA
90. In Scotch Whisky Association v. Marton De Witt, [(2008) FCA 73] Federal Court of
Australia, was concerned with the question as whether the mark 'GLENN OAKS' was
deceptively similar to trade marks that, before the priority date of the Application, had
acquired a reputation in Australia for scotch whisky and scotch whisky-based liquors as
envisaged under Section 60 of the Trade Marks Act, 1995. In that proceeding also Ian-
Barclay had affirmed an affidavit in favour of respondent No.1-Association. As regards
the market (s) and consumer knowledge, it was held :-
"33. There are two classes of consumer in the scotch whisky and bourbon markets : the
involved consumer and the uninvolved consumer. The knowledge of these consumers
may be obtained in several ways including labels, customer salespeople, word of mouth
and past experiences."
91. Noticing the details involving the manufacture of scotch whisky that it is ordinarily
made from barley; it originates in Scotland ; if there are two forms of whisky : blended
and malt; it has various brands and its alcoholic content is at least 40 % by volume. It has
further been noticed:
35. All the applicants expert witnesses would fall within this class of consumer. Given the
market is quite small, product distinction between scotch whisky and other products is
more readily discerned by consumers in this class. They would also be aware that scotch
whisky is sold in bottles and in pre-mixed drinks (scotch whisky products)."
92. It was also held :-
"38. The purchase of bourbon and scotch whisky products is not one of impulse. The
habits of alcohol purchasers differs from those of purchasers of soft-drinks or sports
drinks, who are usually teenagers and children. Are, pricing and product quality are all
factors that make for more discerning consumers of scotch whisky and bourbon.
39. The uninvolved purchaser buying a present or someone else will usually ask a
salesperson, who will assist in product distinction between bourbon products and scotch
whisky products. The salesperson in a liquor store is a means by which consumers
become knowledgeable about different products. By contrast, consumers would not
typically ask a salesperson what a bottle of coke or a new soft drink tastes like. They
would purchase the product and try it themselves. Consumers would also read product
labels, see that "scotch whisky" is written on the container, and infer that scotch whisky is
made in Scotland."
93. The tests which have been laid down therein are as under :-
"38. The purchase of bourbon and scotch whisky products is not one of impulse. The
habits of alcohol purchasers differs from those of purchasers of soft-drinks or sports
drinks, who are usually teenagers and children. Are, pricing and product quality are all
factors that make for more discerning consumers of scotch whisky and bourbon.
39. The uninvolved purchaser buying a
@page-SC2754
present or someone else will usually ask a salesperson, who will assist in product
distinction between bourbon products and scotch whisky products. The salesperson in a
liquor store is a means by which consumers become knowledgeable about different
products. By contrast, consumers would not typically ask a salesperson what a bottle of
coke or a new soft drink tastes like. They would purchase the product and try it
themselves. Consumers would also read product labels, see that "scotch whisky" is
written on the container, and infer that scotch whisky is made in Scotland."
94. The Court held that both involved and uninvolved purchasers of bourbon and scotch
whisky products could not be confused save and except those who are exceptionally
stupid or careless.
95. As regards deceptive similarity test, it was held :-
"66.The key issue under S.60 is a comparison between the GLENN OAKS mark and
marks used before its priority date. The approach in Pianotist Co.'s Application (1906) 23
RFC 774 at 777 is oft cited. There, Parker J said :
"You must take the two words. You must judge of them, both by their look and by their
sound. You must consider the goods to which they are applied. You must consider the
nature and kind of customer who would be likely to buy those goods. In fact, you must
consider all of the surrounding circumstances; and you must further consider what is
likely to happen if each of those trade marks are used in a normal way as a trade mark for
the goods of the respective owners of the marks."
67. That passage was cited with approval in Cooper Engineering Co. Pty. Ltd. v. Sigmund
Pumps Limited (1952) 86 CLR 536 at 538 and in Woollen Mills at 658, where Dixon and
McTiernan JJ described the comparative analysis as follows :
"In deciding this question, the marks ought not, of course, to be compared side by side.
An attempt should be made to estimate the effect or impression produced on the mind of
potential customers by the mark or device for which the protection of an injunction is
sought. The impression or recollection which is carried away and retained is necessarily
the basis of any mistaken belief that the challenged mark or device is the same. The effect
of spoken description must be considered. If a mark is in fact or from its nature likely to
be the source of some name or verbal description by which buyers will express their
desire to have the goods, then similarities both of sound and of meaning play an
important part. The usual manner in which ordinary people behave must be the test of
what confusion or deception may be expected. Potential buyers of goods are not to be
credited with any high perception or habitual caution. On the other hand, exceptional
carelessness or stupidity may be disregarded. The course of business and the way in
which the particular class of goods are sold gives, it may be said, the setting, and the
habits and observation of men considered in the mass affords the standard. Evidence of
actual cases of deception, is forthcoming, is of great weight."
96. The High Court of Australia in Cooper Engineering Co. Pty. Ltd. v. Sigmund Pumps
Ltd., [ (1952) HCA 15] was considering the question as to whether the word
'RAINMATER' is deceptively similar to 'RAIN KING' in respect of spray nozzles,
sprinklers and their parts. It was opined that the same word 'RAIN' suffixing the word
'MASTER' and 'KING' differs in appearance and sound and those marks as a whole make
them quite distinct, opining :-
"There is not a single common letter in master and in King. The two words are so unlike
to the eye and to the ear that counsel for the appellant was forced to rely on the likelihood
of deception arising from the two words conveying the same idea of the superiority or
supremacy of the article as a mechanism for making a spray similar to falling rain or
artificial rain as it was called during the argument. But it is obvious that trademarks,
especially word marks, could be quite unlike and yet convey the same idea of the
superiority or some particular suitability of an article for the work it was intended to do.
To refuse an application for registration on this ground would be to give the proprietor of
a registered trademark a complete monopoly of all words conveying the same idea as his
trademark. The fact that two marks convey the same idea is not sufficient in itself to
create a deceptive resemblance between them, although this fact could be taken into
account in deciding whether two marks which really looked alike or sounded alike were
likely to deceive. As Lord Parker said in the passage cited, you
@page-SC2755
must consider the nature and kind of customer who would be likely to buy the goods. A
purchaser of spray nozzles and sprinklers would not be likely to be lacking in
discernment. He would not be in a hurry to buy. He would not be likely to pay any
attention to the presence of a common word like rain in the combination. That prefix
already appears in other trademarks for goods of the same description sold on the
Australian market such as Rainwell, Rainmaker, Rain Queen, and Rainbow. The learned
registrar was right in holding that the only similarity between the two marks is the
common prefix 'Rain' and that this similarity is not sufficient to create a reasonable
likelihood of deception when the remaining portions of the marks are so different. (at
p539)."
97. Our attention has also been drawn to an order of a Delegate of the Registrar of Trade
Marks in Re :- Opposition by Southcorp Wines Pty. Ltd. to the registration of trade mark
application number 749793 in the name of Kemeny's Food and Liquor Pty. Limited for
the trade mark, comprising the words DEVIL'S RIDGE and device, in Class 33 wherein
on the question of similarity of the words 'DEVIL'S LAIR' and 'DEVILS RIDGE' both
being brand names of wines, it was observed :-
"Despite Ms. Williamson's concerns about wines being purchased with labels unseen on
wine lists, or in a hurry at bottle shops, I do not think that there would be a great deal of
confusion between the marks here because of the common word in them. I believe that
ordinary Australian wine drinkers are becoming increasingly knowledgeable and
sophisticated in their drinking habits and take more care with their selection than Ms.
Williamson believes they do."
98. The two marks were held to be not substantially identical with, or deceptively similar
to each other.
UNITED STATES OF AMERICA
99. In Application of E.I. DuPont DeNemours and Co.[476 F.2d 1357], it was stated:
"The Decisional Process
The ultimate question of the likelihood of consumer confusion has been termed a
question of fact. Coca-Cola Company v. Snow Crest Beverages, Inc., 162 F.2d 280 (1st
Cir. 1947), cert. den. 332 U.S. 809, 68 S.Ct. 110, 92 L.Ed. 386 (1947). If labelled a mixed
question or one of law, it is necessarily drawn from the probative facts in evidence. As so
often said, each case must be decided on its own facts. There is no litmus rule which can
provide a ready guide to all cases.
In testing for likelihood of confusion under Sec. 2(d), therefore, the following, when of
record, must be considered :
1. The similarity or dissimilarity of the marks in their entireties as to appearance, sound,
connotation and commercial impression.
2. The similarity or dissimilarity of and nature of the goods or services as described in an
application or registration or in connection with which a prior mark is in use.
3. The similarity or dissimilarity of established, likely-to-continue trade channels.
4. The conditions under which and buyers to whom sales are made, i.e. "impulse" vs.
careful, sophisticated purchasing.
5. The fame of the prior mark (sales, advertising, length of use).
6. The number and nature of similar marks in use on similar goods.
7. The nature and extent of any actual confusion.
8. The length of time during and conditions under which there has been concurrent use
without evidence of actual confusion.
9. The variety of goods on which a mark is or is not used (house mark, "family" mark,
product mark).
10. The market interface between applicant and the owner of a prior mark :
(a) a mere "consent" to register or use.
(b) agreement provisions designed to preclude confusion, i.e. limitations on continued use
of the marks by each party.
(c) assignment of mark, application, registration and goodwill of the related business.
(d) laches and estoppel attributable to owner of prior mark and indicative of lack of
confusion.
11. The extent to which applicant has a right to exclude others from use of its mark on its
goods.
12. The extent of potential confusion, i.e., whether de minimis or substantial.
13. Any other established fact probative of the effect of use."
@page-SC2756
100. As regards public interest, the learned Judge held :
"Whether offered in response to a right-to-use argument or against any of the evidentiary
considerations listed above, citation of "the public interest" as a basis for refusal of
registration is a bootless cry. [fn8] We need add little to the shattering of that shibboleth
in the concurring opinion in National Distillers, supra, and in the dissents in Ultra-White,
Zildjian and Continental Baking, supra. Writers and scholars listed in those reported
opinions have also shown the fallacy in the notion that the Patent Office is somehow
guarding the public against confusion when it refuses a registration. After a likelihood of
confusion is found (and the case thus decided) citation of the public interest is
unnecessary.
The Patent Office does have a guardianship role under Sec. 2(d). It lies not in a negative,
nay-saying of refusal alone, but in the protection of a mark by registering it and then
rejecting later improper attempts, of which the registrant is unaware, to register it or a
similar mark. Refusal to register cannot prevent confusion. At most, it might discourage
further use. [fn9] Refusal can, under certain circumstances, encourage potential
confusion. Absence of a registration of RALLY for auto cleansers in the present case
may, for example, lead others to adopt and use that or a similar mark for auto cleansers.
Granting a registration will not produce confusion. Use alone can do that and neither we
nor the Patent Office can grant or deny a right to use.
Presumably, everything the Patent Office and this court does is in the public interest. We
find no place for "the guardianship of the public interest" as support for refusals to
register under Sec. 2(d)."
101. Respondent No. 1 herein brought out an action against Majestic Distilling Company
in The Scotch Whisky Association v. Majestic Distilling Company [958 F.2d 594]. The
Dupont test was applied to hold :
"Although the dictionary defines Black Watch as a Scottish infantry regiment, SWA
produced no evidence that the public would understand it as such. We simply do not
believe this creates an issue of material fact, especially considering that the labels clearly
indicate the products are made in the United States. Moreover, the labels contain no
express reference to Scotland nor were the products ever advertised as being of Scottish
origin. More importantly, gin, blended whiskey, and vodka are not characteristically
products of Scotland.
102. Yet again in Champagne Louis Roederer, S.A. v. Delicato Vineyards, [148 F. 3d
1373] the United States Court of Appeals for the Federal Circuit applying the DuPont
(supra) test in a case of wine stated
"Like regulatory agencies or other executive tribunals in their subject areas, the
Trademark Trial and Appeal Board has acquired a high level of expertise in evaluating
the DuPont factors and counter-weighing these factors to reach its ultimate conclusion -
the likelihood vel non of confusion between competing marks. Nonetheless, the Board
too should explain with reasonable "precision" not only its factual findings but the
"theory underlying" its final conclusion. We need to be told and not be "compelled to
guess at the theory" the Board applied to compare its conflicting findings and decide
likelihood of confusion. Without such explanation we are hard-pressed to review
independently, as we must, whether the Board's ultimate conclusion is legally correct. In
such circumstances, we cannot assure ourselves that the Board did indeed "engage in
reasoned decision-making."
103. It was observed :-
"Because the Board in this case failed to "supply a synthesis" of its conflicting findings
that would enable us to "discern the path" to its ultimate conclusion, I initially wondered
whether its conclusion of no likelihood of confusion in this case was correct. As our per
curiam opinion notes, however, it is perfectly lawful for the Board to determine in an
appropriate case that one DuPont factor outweighs all others and thus disposes of the
question of whether competing marks are confusingly similar. It was not immediately
clear to me from the Boards opinion, however, that this is such a case. See Specialty
Brands v. Coffee Bean Distributors, Inc., 748 F.2d 669, 671, 223 USPQ 1281, 1282 (Fed.
Cir. 1984) (holding that, in some instances, the appearance, sound, and significance of the
marks may be dispositive, "but the similarity between words in the respective marks is
only part of the inquiry into likelihood of confusion"). Thus, the difficulty here arises, in
my view, because the Board explicitly found that four
@page-SC2757
of the DuPont factors weighed in Roederer's favor, but concluded nonetheless that the
dissimilarities of the marks in appearance, sound, significance, and commercial
impression weighed dispositively in favor of Delicato. I for one am left wondering how
the Board justified its disregard or diminution in weight of the four DuPont factors that it
found supported Roederer's Opposition, for the Board has failed to "bring together the
disparate elements" to explain the basis for its conclusion of no likelihood of confusion."
INDIA
104. We may also notice some Indian decisions operating in the field. In Carew Phipson
Limited v. Deejay Distilleries Pvt. Ltd., [AIR1994 Bom. 231], the Bombay High Court
stated the law thus :-
"6. Even on comparison of the trade mark of the plaintiffs with the defendants' trade
mark, it is difficult to hold that the two marks are deceptively similar and are likely to
create any confusion in the minds of the customers."
105. It was furthermore observed :-
"On comparison of the two marks bearing the aforesaid principles in mind, it is difficult
to appreciate as to how there is even a remote possibility of any customer being misled.
In my opinion, when a customer goes to a shop to buy the plaintiffs' product, he will not
ask for "Duet" or "Gin N Lime" or "Gin N Orange" but he will ask for a "Blue Riband
Gin N Lime" or "Blue Riband Tango Gin N Orange". Further having regard to the fact
that the customer who is likely to buy the products of the plaintiffs and the defendants
will be normally educated and discerning type, it is impossible to hold that there is any
likelihood of confusion. It is pertinent to note that the plaintiffs have failed to cite even a
single instance showing that there was confusion in the minds of the customers. The
absence of evidence of actual deception is a circumstance which definitely weighs in
favour of the defendants. It is also necessary to mention that the defendants have
produced on record voluminous evidence showing that such premixtures are sold by
several other companies under the names "American Extra Dry Gin with Lime Duet
Plus", "Forbes Two in One Dry Gin and Lime", "Rainbow Gemlet Gin and Lime" etc. In
my opinion, there is no possibility of any confusion in this case at all."
106. Yet again in Diageo North America, Inc. and another v. Shiva Distilleries Ltd., [143
(2007) DLT 321] a learned Single Judge of the Delhi High Court held as under :-
"14. So much for the second syllable. As regards the first syllable, I find that there is no
similarity between SMIR and BRIS. Although the learned Counsel for the plaintiffs had
submitted that all the letters are common except the letter and M in SMIR and the letter B
in BRIS, this, to my mind, is of no consequence because the arrangement of the letters is
entirely different, as is the phonetic and visual result. I also agree with the submission
made by the learned Counsel for the defendant that the intending purchasers of the
competing products are literate persons belonging to the affluent class of society and who
would be in a position to easily distinguish SMIRNOFF from BRISNOFF particularly
when the eyebrow device and the colour combination is sought to be given up by the
defendant. The average person with imperfect recollection would have to be from
amongst the sub-set of such persons i.e., discerning consumers of vodka. My prima facie
view is that the trade mark BRISNOFF is not deceptively similar to, nor can it be
confused with the Plaintiffs' trade mark SMIRNOFF."
107

. This Court in Cadila Health Care Ltd. v. Cadila Pharmaceuticals Ltd., [(2001) 5 SCC
73] inter alia laid down the law in the following terms :- 2001 AIR SCW 1411

"35. Broadly stated, in an action for passing-off on the basis of unregistered trade mark
generally for deciding the question of deceptive similarity the following factors are to be
considered :
(e) The class of purchasers who are likely to buy the goods bearing the marks they
require, on their education and intelligence and a degree of care they are likely to exercise
in purchasing and/or using the goods."
108. The tests which are, therefore, required to be applied in each case would be
different. Each word must be taken separately. They should be judged by their look and
by their sound. Must consider the goods to which they are to be applied. Nature and kind
of customers who would likely to buy goods must also be considered. Surrounding
circumstances play an important factor. What would likely to happen if each of those
trade marks is used in a normal way as a trade mark of the goods of the respective
@page-SC2758
owners of the marks would also be a relevant factor. [See Pianotist Co. Application, Re ,
(1906) 23 RPC 774].
109. Thus, when and how a person would likely to be confused is a very relevant
consideration.
110. Where the class of buyers, as noticed hereinbefore, is quite educated and rich, the
test to be applied is different from the one where the product would be purchased by the
villagers, illiterate and poor. Ordinarily, again they, like tobacco, would purchase
alcoholic beverages by their brand name. When, however, the product is to be purchased
both by villagers and town people, the test of a prudent man would necessary be applied.
It may be true that the tests which are to be applied in a country like India may be
different from the tests either in a country of England, United Sates of America or
Australia. We however, do not mean to suggest that in a case of this nature, the
Heightened Scrutiny Test should be applied as urged on behalf of the appellant. Bollinger,
J. and Others v. Costa Brava Wine Coy., Ld. [1960 (1) RPC 16], whereupon Mr. Desai
has strongly relied upon, makes such a distinction. Bollinger, J. (supra) was a case on
demurrer. It was concerned with sale of Spanish Champagne. In that case, in paragraph 4
of the application, the applicant stated :
"Then in Para 4 they deny that this name "Spanish Champagne" is a false description, and
they continue : "The defendants deny that the said section imposes any statutory duty on
the defendants or any statutory duty owed by the defendants to the plaintiffs.
Alternatively, if the said section does impose any such statutory duty the same is not
actionable at the suit of any of the plaintiffs or at all."
111. The court proceeded on certain assumptions which are :
"(1) the Plaintiffs carry on business in a geographical area in France known as
Champagne;
(2) the Plaintiffs wine is produced in Champagne and from grapes grown in Champagne;
(3) the Plaintiffs wine has been known in the trade for a long time as "Champagne" with a
high reputation;
(4) Members of the public or in the trade ordering or seeing wine advertised as
"Champagne" would expect to get wine produced in Champagne from grapes grown
there; and
(5) The Defendants are producing a wine not produced in that geographical area and are
selling it under the name of "Spanish Champagne".
112. It was noticed :
"The well-established action for "passing-off" involves the use of a name or get-up which
is calculated to cause confusion with the goods of a particular rival trader, and I think it
would be fair to say that the law in this respect has been concerned with unfair
competition between traders rather than with the deception of the public which may be
caused by the Defendant's conduct, for the right of action known as a "passing-of action"
is not an action brought by the member of the public who is deceived but by the trader
whose trade is likely to suffer from the deception practised on the public but who is not
himself deceived at all."
113. Before the learned Judge, the plaintiffs claimed that their goodwill in the name or
description "Champagne" is injured by the Defendants conduct to which the counsel for
the defendants did not contest the correctness of the statement.
114. The learned Judge, referring to Mayor of Bradford v. Pickles [1895 AC 587] and
laying down the principles of injuries, noticed the argument of the counsel that before a
person can negative the argument of the defence counsel, the person can recover for loss
or it must be shown that his case falls within the class of actionable wrongs stating :
"But the law may be thought to have failed if it can offer no remedy for the deliberate act
of one person which causes damage to the property of another. There are such cases, of
course, but they occur, as a rule, when the claims of freedom of action outweigh the
interests of the other persons who suffer from the use which a person makes of his own
property."
115. It was in the aforementioned fact situation, the learned Judge proceeded to determine
as to whether the description "Spanish Champagne" is calculated to deceive holding that
the plaintiff has a right to bring any action.
116. J. Bollinger and Others v. The Costa Brava Wine Company Limited (for short
"Bollinger II") [1961 (5) RPC 116], however,
@page-SC2759
clearly shows as to what was the test applied. The inference deduced from the evidence
adduced was noticed in the following terms :
"In particular, it appeared from the evidence that Champagne is a wine specially
associated with occasions of celebration so that (in addition to sales to persons who
regularly buy wine) it is purchased on such occasions from time to time by many persons
who are not in the habit of buying wine for consumption and are not educated in the
nature or qualities of different kinds of wine".
117. Holding that "Spanish Champagne" may lead to ordinary belief into thinking that
wine so described was the real thing, it was observed:
"Moreover, when the case is tried in an atmosphere of educated persons, many of whom
are well acquainted with the qualities of various wines, it may seem absurd that persons
should be deceived by what may appear to be a transparent impersonation. It was argued
indeed that Champagne was so well known that everyone except a trifling minority of
ignorant persons (who were not to be considered, especially in regard to what was termed
a luxury article) would not be deceived."
118. It was held :
"Champagne, on the other hand, is a French word, and it is wrong if not dishonest to
apply it to anything but the product of Champagne, the area round Rheims in France.
Others must call themselves Sparkling Moselle, Sparkling Californian White, or what
they please. There is some still champagne, a pleasant hard white wine, which you can
drink in France, and even some dull still red wine; and on pages 79-80 of the same book
it is said : It mayn't be forgery to describe a wine as, say, Australian Chablis; it is
deplorable and shows the maker has no proper pride in his product, but the adjective
'Australian' is a sort of warning. In a previous edition (the 6th) of the same book, the
words in the same passage are "is a fair warning".
119. The learned Judge proceeded to hold :
"All the lovers of wine who gave evidence before me deplored this practice by which the
names of well-known wines have been debased. Some of the witnesses, employed in the
more practical side of the wine trade, referred to the convenience of thus using the name
of the real wine to indicate a type. But it appeared that a number of the wine merchants
who dealt in such wines were careful in their price lists to list such wines under titles or
descriptions which showed that they were of a type but not from the original district."
120. Referring to Kerly on Trade Marks, which we have referred to hereinbefore, the
learned Judge said:
"And it has been said that regard should not be had to "unusually stupid people, fools or
idiots". Moreover, "if the goods are expensive and not of a kind usually selected without
deliberation and the customers generally educated persons these are all matters to be
considered". (That is also a quotation from the same book.) Various other judicial
statements are collected in the judgment of the Assistant-Registrar in George Angus and
Co.'s Application (1943) 60 R.P.C. 29, at pp. 31-32, to which I was referred.
121. In arriving at the said decision, the following was specifically noticed :
"Mr. Munday, whose wine business was in Swansea, when asked "How far do you think
the class of customers that you deal with know the origin of Champagne"? replied:
"Limited. Some would know. The first category I mentioned would know a fair amount
about it. In the second category some. But there would be a considerable number in my
area who would know nothing about it except that it was a wine they wanted for a special
occasion or for something in their life they wanted to celebrate with. They would then
want that. That is how much they would know about it - just a general outline"."
122. It was furthermore held :
There is thus, in my view, a considerable body of evidence that persons whose life or
education has not taught them much about the nature and production of wine, but who
from time to time want to purchase Champagne, as the wine with the great reputation, are
likely to be misled by the description "Spanish Champagne."
Something was said on the subject of the burden of proof. Well, burden of proof is
something which may shift in the course of an action. It appears to me that when the
plaintiffs have shown that a description used by the defendants contains an untruthful
statement that a wine which is not Champagne
@page-SC2760
is Champagne, they have gone some way to establishing their case, and the Court might
require to be satisfied that such an untrue statement was so clearly qualified as to be not
likely to mislead. But, however, that may be, I am satisfied on the evidence that a
substantial portion of the public are likely to be misled. And as Lord Justice Lindley said
in Slazenger and Sons v. Feltham and Co. (1889) 6 R.P.C. 531 at p. 537 : "One must
exercise one's common sense, and, if you are driven to the conclusion that what is
intended to be done is to deceive if possible, I do not think it is stretching the imagination
very much to credit the man with occasional success or possible success. Why should we
be astute to say that he cannot succeed in doing that which he is straining every nerve to
do?"
123. Bellinger test was not only applied in Warnick (Erven) Besloten Vennootschap v. J.
Townend and Sons (Hull) Ltd. [1980 RPC 31], but in all the case which have been
referred to by Mr. Desai to which the different High Courts of India as also to which we
have taken note of.
124. However, tests laid down in Australia and United States in respect of self-same
goods are noticed hereinbefore are somewhat different.
125. But then we are concerned with the class of buyer who supposed to know the value
of money, the quality and content of Scotch Whisky. They are supposed to be aware of
the difference of the process of manufacture, the place of manufacture and their origin.
Respondent No.3, the learned Single Judge as also the Division Bench of the High Court,
therefore, failed to notice the distinction, which is real and otherwise borne out from the
precedents operating in the field. [See - Kerlys Law of Trade Marks and Trade Names,
Thirteenth Edition pg. 600].
126. Had these tests been applied the matter might have been different. In a given case
probably we would not have interfered but we intend to do so only because wrong tests
applied led to a wrong result.
127. So far as the applicability of the 1999 Act is concerned, having regard to the
provisions of Sections 20(2) and 26(2), we are of the opinion that the 1999 Act will have
no application.
128. For the reasons aforementioned, the impugned judgment is set aside. The appeal is
allowed. No costs.
Appeal allowed.
AIR 2008 SUPREME COURT 2760 "Subha B. Nair v. State of Kerala"
(From : Kerala)
Coram : 2 S. B. SINHA AND LOKESHWAR SINGH PANTA, JJ.
Civil Appeal No. 4176 with 4177 and 4178 of 2008 (arising out of SLP (C) No. 1826
with 1227 and 3596 of 2007), D/- 27 -5 -2008.
Subha B. Nair and Ors. v. State of Kerala and Ors.
(A) Kerala Co-operative Societies Act (21 of 1969), S.109 - Kerala Co-operative
Societies Rules (1969), R.188 (as amended by Amendment Rules of 2004) - Constitution
of India, Art.320 - CO-OPERATIVE SOCIETIES - PUBLIC SERVICE COMMISSION -
RECRUITMENT - Recruitment - Posts in Co-opertive Bank - Such recruitment most be
made by Committee in terms of Statutory Rules. (Para 9)
(B) Kerala Co-operative Societies Act (21 of 1969), S.109 - Kerala Co-operative
Societies Rules (1969), R.188 (as amended by Amendment Rules of 2004) - CO-
OPERATIVE SOCIETIES - Classification of branches by Co-operative Bank - Cadre
strength of Bank would depend upon its classification - For determining cadre strength,
its financial health would be relevant factor - Statement for classifying branches is,
therefore, required to be prepared scientifically - In case true picture of financial position
is not reflected therein, Registrar will have say in matter.
2000 Lab IC 3724 (Ker), Relied on. (Paras 12, 13, 14)
(C) Kerala Co-operative Societies Act (21 of 1969), S.109 - Kerala Co-operative
Societies Rules (1969), R.188 (as amended by Amendment Rules of 2004) - Constitution
of India, Art.226 - CO-OPERATIVE SOCIETIES - WRITS - RECRUITMENT -
POLICY DECISION - Recruitment - Posts in co-operative Bank - Filling up of 201 posts
out of approved 208 posts - Policy decision taken by Bank not to fill up existing
vacancies when some persons did not join posts - Merely because Bank had sent
requisition or had been making recruitments in other categories of staff, by itself may not
be a ground for issuance of writ of mandamus, in view of expiry of validity of ranked list.
2007 AIR SCW 7311; 2007 AIR SCW
@page-SC2761
3211; 1991 AIR SCW 1583; 2008 AIR SCW 322, Relied on. (Paras 7, 16, 18)
Cases Referred : Chronological Paras
2008 AIR SCW 322 (Rel. on) 17
2007 AIR SCW 3211 : 2007 Lab IC 2495 (Rel. on) 8, 19
2007 AIR SCW 7311 (Rel. on) 7, 19
2000 Lab IC 3724 (Rel. on) 4, 10
1991 AIR SCW 1583 : AIR 1991 SC 1612 : 1991 Lab IC 1460 (Rel. on) 17
AIR 1987 SC 169 : 1987 Lab IC 34 17
AIR 1984 SC 1850 17
AIR 1973 SC 2216 : 1973 Lab IC 1212 17
C. N. Sree Kumar, P. R. Nayak, Thomas Joseph, A. D. Parashar and Ms. Malini Poduval,
for Appellant; Vipin Nair, S. Balaji, P. B. Suresh, Vivek Sharma (for M/s. Temple Law
Firm), Roy Abraham, Mrs. Seema Jain, Himinder Lal, P. V. Dinesh and Sindhu T. P., for
Respondents.
Judgement
S. B. SINHA, J. :- Leave granted.
2. Appellants are before us aggrieved by and dissatisfied with a judgment and order dated
18.07.2006 passed by the Kerala High Court in Writ Appeal Nos. 2275, 2527 and 2622 of
2005 affirming the judgment and order dated 7.9.2005 passed by a learned single judge of
the said Court.
3. Recruitment to the posts is made by the respondent No. 4 Commission. A requisition
was made for filling up of 214 posts. Allegedly, the respondent No.2 approved only 208
posts. 201 vacancies were filled up. Contending inter alia that the Respondent-
Cooperative Bank could fill 16 more vacancies, a writ petition was filed. A learned single
judge of the High Court opined that having regard to the approved vacancy position, six
more vacancies could be filled up and one vacancy having arisen due to non-joining of
the same could also be filled up. A direction was, therefore, issued to fill up seven more
vacancies. An intra-court appeal was preferred there against, which by reason of the
impugned judgment has been dismissed opining :
"The vacancies already stand reported to the Public Service Commission in
implementation of the interim order passed on 10.12.2004 and this report shall be deemed
to have been in respect of the vacancies occurred before the expiry of the list, limited to
the vacancies available. There cannot have any dispute on that. In Ext.P10, the Registrar
had approved the staff strength sanctioning only 208 posts of clerk/cashier. Out of that,
only 201 are in position. Therefore, there are remaining 7 vacancies. These 7 vacancies
shall be taken as, as mentioned above, reported before 31.12.2004 and the candidates
shall be advised as if the report had been received before the expiry of the said date,
following the appropriate ratio and communal rotation as applicable to the post. As the
Public Service Commission had received the report regarding the Non Joining Duty
vacancy only on 10.5.2005, they need consider it as one in respect of the vacancies that
had arisen after the expiry of the list. When the Registrar had sanctioned only 208 posts,
whatever be the resolution in Ext.R4(g) or the contentions raised in the counter affidavit,
the bank is disabled from appointing persons any more than what is contained in the
order of the Registrar. Moreover, because of the computerization of the branches and
other modern facilities introduced in the banking business, necessarily there may be
reduction in the number of vacancies. Therefore, the view taken by the learned Single
Judge to direct 7 vacancies to be advised, cannot be said to be unjustified to invite
interference in these appeals."
4

. Mr. Sree Kumar, learned counsel appearing on behalf of the appellants submits that the
High Court committed a serious error in so far it failed to take into consideration that
having regard to the decision of the said Court in Elampal Service Coop. Bank Ltd. v.
Government of Kerala, [2000 (3) KLT 389], approval of Registrar in such matters was
not necessary and in any event profitability or otherwise of the Cooperative Bank being
not a relevant factor for determining the cadre strength and, thus, the impugned judgment
cannot be sustained. 2000 Lab IC 3724

5. Mr. Roy Abhraham; Mr. P.V. Dinesh and Mr. Vipin Nair, learned counsel appearing on
behalf of the Respondent Cooperative Bank, Kerala Public Service Commission and the
State of Kerala, on the other hand, submit that the ranked list having expired on 31st
December, 2004 the High Court could not have issued any writ on the petition filed by
the appellants herein, particularly in view of the fact that the Bank had taken a policy
decision not to fill up any other or further post. It was furthermore submitted that in terms
of Rule 182 of the Kerala Cooperative Societies Rules, 1969, the approval of the
Registrar is imperative.
@page-SC2762
6. Indisputably requisition was made by the respondent-Bank to the Commission for
appointment of 220 clerk-cum-cashier in the Bank. After the processes were gone into
ranked list was prepared on or about 18th September, 2001. Its validity expired on or
about 31st December, 2004. Some persons did not join the posts. The vacancies remained
unfilled.
Another requisition for filling up of the vacancies position was notified again on or about
10th May, 2005 by which date the validity of the earlier ranked list expired. It is now
accepted at the Bar that pursuant or in furtherance thereof the Commission has already
conducted an examination on 10th May, 2008, and in terms thereof a fresh rank list
would be prepared.
7

. A decision on the part of an employer whether to fill up the existing vacancies or not is
within its domain. On this limited ground in absence of discrimination or arbitrariness, a
writ court ordinarily would not interfere in such matters. This has been so held by this
Court in Deepa Keyes v. Kerala State Electricity Board, [(2007) 6 SCC 194] observing
that the rank list having expired and the validity having not been extended, no relief could
be granted to the appellants therein. 2007 AIR SCW 7311

. Similar view has also been expressed by this Court in K. Thulaseedharan v. Kerala State
Public Service Commission, Trivandrum and others, [(2007) 6 SCC 190] 2007 AIR
SCW 3211

9. Recruitment to a post having regard to the provisions contained in Article 320 of the
Constitution of India must be made by the Committee in terms of the Statutory Rules.
Rule 188 of the 1969 Rules provides for a staff pattern in the following terms :
"188. Staff Pattern. - Every society shall adopt the staff pattern indicated in Appendix III
to these rules, according to the type and class to which it belongs :
Provided that where any society cannot adopt such staff pattern due to its financial
position, the members of the committee may work; in an honorary capacity in lieu of
appointing any paid employee :
Provided further that where any society is in need of any change in the pattern of staff
including the scale of pay under special circumstances the same may be made by the
society with the prior approval of the Registrar of Co-operative Societies."
10

. A Division Bench of the Kerala High Court in Elampal Service Cooperative Bank Ltd.
(supra) opined that the power of the Registrar either to rescind a resolution or
reclassification made by the Cooperative Society can be exercised so as to enable him to
set them aside, stating :- 2000 Lab IC 3724

"If reclassification made by the petitioner Bank is incorrect, the Registrar is not helpless
in setting aside the resolution passed."
11. Rules appear to have undergone a change in 2004 by way of Kerala Cooperative
Societies (Amendment) Rules, 2004. Note 4 Appended to Appendix II under Serial No.
2m under Class IV of the 1969 Rules reads as under :-
Note 4 :- Classification made by the Societies should be got certified and approved by the
registrar of Co-operative Societies before being implemented.
12. Thus, it may not be entirely correct that the Registrar will have no say in the matter.
Financial health of a bank is a relevant factor.
13. In this case from the profit and loss account it appeared that the statement for
classifying the branches were not prepared scientifically and hence a true picture of the
financial position was not reflected therein. The Registrar has the responsibility to see
that the Cooperative Societies function effectively and efficiently. A Cooperative Bank,
according to the guidelines issued by NABARD, should be in a position to maintain the
cost of management to working fund at the optimum level of 2%. The cost of
management, however, of the Cooperative Bank in question was found to be 8%, which
according to the Registrar, was at an alarming level as has been observed by the Auditor.
14. The cadre strength of a Cooperative Society would depend upon its classification
thus, although the Registrar may not have anything to do therewith directly, but the same
would follow as a necessary corollary.
15. It furthermore appeared that the classification norms which were prescribed by the
Government long time back may not be valid in the present day situation having regard to
the computerization programme resorted to by the Bank.
In its counter-affidavit the State of Kerala averred:-
@page-SC2763
"The Central Banks Conference, which is the forum constituted to take stock of the
progress and to review the problem faced by the credit structure in the State consisting
the representatives of District Co-operative Banks, Kerala State Co-operative Bank,
Government, Registrar of Co-operative Societies, has of the view that the present
classification norms needs changes in par with the present credit scenario and slow
growth of the movement, threat posed by the new generation banks in the wake
globalization and liberalization, a committee has been constituted to study the aspect and
the committee recommended to change the classifications of District Co-operative Bank
urgently, lest the very existence of the banks will be in peril. The proposal is under active
consideration of the Government, and it is expected that the classification norms will be
revised soon."
16. In the absence of any material it is difficult for a court to arrive at a firm conclusion
that having regard to the fact that over a period of time the Cooperative Bank had opened
many more new branches or the volume of its operation had increased requiring
appointments of more persons. What would be the effect of computerisation is also not
known. Only because the Bank has sent a requisition or had been making recruitments in
other categories of staff by itself may not be a ground for issuance of a writ of or in the
nature of mandamus, although the Cooperative Bank had adopted a policy decision not to
fill up more than 201 vacancies which stands filled up to 208 vacancies in terms of the
judgment of the learned Single Judge of the High Court.
17

. In Shankarsan Dash v. Union of India [(1991) 3 SCC 47], this Court held : 1991
AIR SCW 1583

"7. It is not correct to say that if a number of vacancies are notified for appointment and
adequate number of candidates are found fit, the successful candidates acquire an
indefeasible right to be appointed which cannot be legitimately denied. Ordinarily the
notification merely amounts to an invitation to qualified candidates to apply for
recruitment and on their selection they do not acquire any right to the post. Unless the
relevant recruitment rules so indicate, the State is under no legal duty to fill up all or any
of the vacancies. However, it does not mean that the State has the licence of acting in an
arbitrary manner. The decision not to fill up the vacancies has to be taken bona fide for
appropriate reasons. And if the vacancies or any of them are filled up, the State is bound
to respect the comparative merit of the candidates, as reflected at the recruitment test, and
no discrimination can be permitted. This correct position has been consistently followed
by this Court, and we do not find any discordant note in the decisions in State of Haryana
v. Subhash Chander Marwaha, Neelima Shangla v. State of Haryana, or Jatendra Kumar
v. State of Punjab. AIR 1973 SC 2216
AIR 1987 SC 169
AIR 1984 SC 1850

[See also Jitendra Kumar and Ors. v. State of Haryana and Anr., [2008 (2) SCC 161].
2008 AIR SCW 322

18. The question as to whether there existed 7 vacancies or 16 vacancies in the


aforementioned situation looses all significance. We would assume that as per the
requisition, 9 more vacancies could be filled up but it is trite that if the employer takes a
policy decision not to fill up any existing vacancy, only because a person's name is found
in the select list, the same by itself would be a ground to compel the bank to fill them up.
19

. Rules of the Cooperative Societies as has been interpreted by this Court in Deepa Keyes
(supra) and K. Thulaseedharan (supra) clearly show that after the expiry of rank list,
vacancies should not be directed to be filled up. 2007 AIR SCW 7311
2007 AIR SCW 3211

20. This Court furthermore cannot issue a direction only on sentiment/sympathy.


21. For the reasons aforementioned no relief can be granted to the appellants. The appeals
fail and are dismissed. However, in the facts and circumstances of the case, there shall be
no order as to costs.
Appeals dismissed.
AIR 2008 SUPREME COURT 2763 "Swami Shankaranand v. Mahant Sri Sadguru
Sarnanand"
(From : 2006 (3) UC 1425 (Uttaranchal))
Coram : 2 S. B. SINHA AND LOKESHWAR SINGH PANTA, JJ.
Civil Appeal No. 4175 of 2008 (arising out of SLP (C) No. 19185 of 2006), D/- 27 -5
-2008.
Swami Shankaranand (D) By L.R. v. Mahant Sri Sadguru Sarnanand etc. and Ors.
Civil P.C. (5 of 1908), S.92, S.104 - PUBLIC TRUST - TRUST - APPEAL -
@page-SC2764
Permission to sell property of Math - Appeal - Locus standi - Person in charge of another
Math - Cannot be said to have no locus to file appeal - Cannot he said that no person who
is third party to application would be a person aggrieved - Order dismissing appeal
however not interfered with since purchaser as well as seller trusts have put up
constructions. (Paras 9, 11)
Cases Referred : Chronological Paras
2008 AIR SCW 3546 (Rel. on.) 10
A. K. Ganguli, Sr. Advocate, Atishi Dipankar and Santosh Kumar, for the Appellants; S.
R. Singh Sr. Advocate Abhisth Kumar, D. N. Dubey, Ms. Archana Singh, Makarand D.
Adkar, S. D. Singh, Vijay Kumar and Vishwajit Singh, for the Respondents.
Judgement
S. B. SINHA, J. :- Leave granted.
2. Whether a disciple attached to a Mahant in one of the establishments run by a
Religious Trust will have locus standi to maintain an appeal from an order of the District
Judge allowing an application filed by the Trust under Section 92(1)(f) of the Code of
Civil Procedure, 1908 (for short, the Code) is the short question which arises for
consideration in this appeal.
3. One Swami Sarupanand was the founder of the Math. He was disciple of Swami
Advaitanand. The latter was a religious preceptor of great learning and had a large
following. Swami Sarupanand took his Samadhi at Meerut in March 1936 and according
to his wishes Swami Atmavivekanand became the Mahant. He was succeeded by Swami
Harsewanand who in turn was succeeded by Swami Harshankaranand. Swami
Harshankaranand died on 22.02.1993. He had three disciples; Sarnanand, Premanand and
Smt. Tapesara. Premanand died on 10.06.2005. He was succeeded by Swami
Shankaranand. Appellant is said to have succeeded Swami Shankaranand. Appellant
contends that succession to the office of Mahant is by nomination. Any person so
nominated adopts the life of a sanyasi. He leads the life of celibacy and religious
mendicancy.
4. A dispute in regard to the office of the Mahant after the death of Swami
Atmavivekanand arose in between one Swami Harsewanand on the one side and Sri
Krishna Singh on the other.
This Court held Swami Harsewanand to be the successor of Swami Atmavivekanand.
After his death, Swami Harshankaranand was substituted in place of Swami
Harsewanand in the aforementioned litigation before this Court. Whether he would
acquire the status as a successor Mahant or not was left open. One Sri Krishna Singh filed
Suit No. 153/80 questioning the status of Swami Harshankaranand as a Mahant of the
Math in question commonly known as Garhwaghat Math. The question in regard to the
holder of the office is still pending before this Court in Civil Appeal No. 5550 of 2003.
5. Mahant Satguru Sarananand who also was a disciple of Swami Harshankaranand was
incharge of the Garhwaghat Math. He entered into an agreement for sale with the
respondent No.3 which is also a Public Trust. A sum of Rs. 35,50,000/- was the agreed
amount of consideration for the said land. Out of the said amount Rs. 33,00,000/- was
paid in advance. An application for grant of permission to sell the said property was filed
on 02.07.1990. It was advertised in two local newspapers. No objection having been
received, permission as sought for was granted by the learned District Judge by an order
dated 13.10.1992. Name of the respondent No. 3 was mutated in the revenue records on
31.1.1994. An application was filed by the respondent No. 3 before the Hardwar
Development Authority for grant of sanction of building plans on 15.10.1993. It was
allowed by an order dated 2.6.1994. Huge constructions have since been raised by the
respondent No. 3. Respondent No. 3 popularly known as Gayatri Pariwar Shanti Kunj on
the said land has developed: (1) A Research Laboratory known as Brahma Varchas Shodh
Sansthan, and (2) Dev Sanskriti Vishwavidyalaya.
It has also a network of 4000 Shakti Peeths, 25,000 Pragya Sansthans and 30,000
Swadhyaya Mandals etc., which regularly organize Satsang, Discourses, Inspiring songs,
and discussions on various problems in their areas to advance the noble cause of the
mission. In fact, these serve as local centres of which Shantikunj is the Headquarter.
6. Appellant preferred an appeal before the High Court of Allahabad on 15.11.1994 only,
which was transferred to the Uttaranchal High Court. By a reason of a judgment and
order dated 19.5.2006, the said appeal has been dismissed, inter alia, on the ground that
the appellant was not a
@page-SC2765
person aggrieved to maintain the same.
7. Mr. A.K. Ganguli, learned Senior Counsel appearing on behalf of the appellant would
submit that the High Court committed a serious error insofar as it failed to take into
consideration that the appellant had no knowledge about coming of existence of the said
agreement for sale as also the order granting sanction by the District Judge. The learned
counsel would contend that the minimum valuation of the property as per the report of
the valuer should have been Rs. 72 lakhs and not Rs. 35,50,000/- as has been found by
the learned District Judge. In any event, it was urged that having regard to the fact that
the dispute in regard to Mahantship between the interested parties being pending
consideration before this Court, the High Court committed a serious error in passing the
impugned judgment.
8. Mr. S.R. Singh, learned Senior Counsel appearing on behalf of the respondent No. 3
and Mr. Adkar, learned counsel appearing on behalf of the other respondent, on the other
hand, supported the impugned judgment.
9. Section 92 of the Code of Civil Procedure provides for special power of the District
Judge in regard to Public Trusts both charitable and religious. An application for sale of
the Trust property must be filed before the District Judge and only on his approval the
same can be effected.
In a case of this nature judiciary exercises the jurisdiction of parens patriae and, thus,
when an objection is filed for grant of sanction in terms of Section 92(1)(f) of the Code,
the same should receive serious consideration. The High Court thus may not be entirely
correct in opining that the appellant had no locus standi to maintain an appeal. It is true
that the appellant is said to be in-charge of a Math situated at Varanasi. However, it is
contended that he really stays at Mirzapur. According to the respondents, he has nothing
to do with the Math in question. But, that is to say, no person being a third party to the
application, would not be a person aggrieved, in a case of this nature cannot be sustained,
if the appellant establishes that he is otherwise interested in the welfare of the Trust.
10. The High Court in its judgment noticed :
"10. Even according to the case of the appellant Swami Har Shankaranand was the
Mahant of the Math. The appellant in the injunction application filed before the Appellate
Court has claimed himself to be succeeding Mahant of the Math Garhwa Ghat on the
death of Swami Har Shankaranand. The Counter affidavit has been filed by the
respondents before the High Court as Annexure C.A. 10 to the counter affidavit, which is
judgment dated 3.5.1991 passed by X Addl. District Judge, shows that Swami Sarananad,
respondent No.1 has become Mahant after death of Swami Har Shankaranand. The
aforesaid order dated 3.5.1991 also shows that appellant Premanand was not declared to
be the Mahant. Again in view of Annexure C.A. 8 to the counter affidavit it reveals that
the competent authority in mutation proceeding vide order dated 15.6.1993 came to the
conclusion that Chadar Mahanthi was given to Swami Satguru Sarananad and not to
appellant Premanand on the death of Swami Har Shankaranand. Further paragraph 4(g) to
(r) and (s) of the aforesaid counter affidavit reveal that Suit No. 153/1980 which was
sought to be converted was initially filed challenging the status of Swami Har
Shankaranand as Mahant of Math Garhwa Ghat and the High Court vide judgment dated
15.5.2002 passed in W.P. No. 46291 of 2000 has quashed the entire proceedings of suit
No. 153/1980. Therefore at present Suit No. 153/1980 is not pending, hence under the
aforesaid circumstances it is quite clear that the appellant Premanand is not Mahant of
Math "Garhwa Ghat" hence he cannot be said to be an aggrieved party as well as does not
have any locus to maintain the aforesaid appeal. Swami Premanand who had filed the
appeal is now dead and there is a dispute regarding the succession of Swami Premanand,
but once it is held that Swami Premanand has no locus or grievance to maintain the
aforesaid appeal, therefore after the death of Swami Premanand who is the appellant in
this case, there is nothing on record to indicate as to how the person claiming succession
to late Swami Premanand is aggrieved by the impugned order."

It is also not in dispute that Swami Premanand was not a party to the proceedings before
the learned District Judge. It, however, did not mean that a person who was not a party to
the proceedings cannot prefer an appeal. The question in regard to the extension of locus
standi of a person to reported in 2008 AIR SCW 3546

@page-SC2766
prefer an appeal has recently been considered in Machindranath Kernath Kesar vs. D.S.
Mylarappa and Ors. (C.A. No. 3041 of 2008) disposed of on 29.04.2008, wherein it was
held that in a case where a person's right to obtain compensation may be defeated by a
judgment passed in a connected matter, he would have a right of appeal.
Unfortunately, the High Court has not considered this aspect of the matter. But in view of
the order proposed to be passed, it is not necessary to deal with this aspect of the matter
any further.
11. The learned counsel for the parties, however, have taken us through the entire records.
From the affidavit filed by the third respondent, it appears that it is running a Trust which
serves a larger public interest. A large number of constructions have already been made.
Constructions started in the year 1994 and have been completed in 1995. Various
activities have been going on at that place.
Respondent Nos. 1 and 2 also have purchased an alternative land and raised constructions
thereon at a cost of Rs. 10 lakhs. In this view of the matter, in our opinion, no useful
purpose would be served in entertaining the appeal. Furthermore, the appellant cannot be
permitted to prefer an appeal only because he is interested in the result of Civil Appeal
No. 5550 of 2003 which is pending before this Court. He is not a party thereto. He is not
claiming Mahantship in his individual capacity in respect of the establishment at
Hardwar. It is accepted at the Bar that the said Civil Appeal has got nothing to do with the
property in question. Furthermore, the nature of the property when sold was not Abadi
but was a jungle land. It is also not in dispute that the name of Sadguru Sarnanand was
also mutated in the revenue records pursuant to the order dated 26.9.1983 in revenue
proceedings.
12. We are, therefore, of the opinion that it is not a fit case for exercise of extraordinary
jurisdiction under Article 136 of the Constitution of India. The appeal is dismissed
accordingly. There shall be no order as to costs.
Appeal dismissed.
AIR 2008 SUPREME COURT 2766 "Sarbdeep Singh Virk v. State of Punjab"
(From : Punjab and Haryana)
Coram : 2 TARUN CHATTERJEE AND J. M. PANCHAL, JJ.
Civil Appeal No. 3649 of 2008 (arising out of SLP (C) No. 12440 of 2008), D/- 16 -5
-2008.
Sarbdeep Singh Virk v. State of Punjab and Ors.
Constitution of India, Art.311 - TERMINATION OF SERVICE - DISCIPLINARY
AUTHORITY - SERVICE MATTERS - Disciplinary authority - Deputationist an I.P.S.
Officer repatriated to parent State - Central Government would be appropriate
disciplinary authority - Direction issued to borrowing State to remit entire matter relating
to the disciplinary proceedings initiated against deputationist to the Central Government,
proper - Stay granted by High Court liable to be set aside - Order of repatriation to be
given effect notwithstanding suspension order passed by borrowing State. (Para 6)

Pramjit Singh Patwalia, Sr. Advocate, R. Shetty and S. Chandra Shekhar, with him for
Appellant; Soli J. Sorabjee, L. N. Rao, Sr. Advocates, Ajay Pal, Nikhil Jain,
Ardhendumauli K. Prasad and Ravindra K. Adsure, with them for Respondents.
Judgement
J. M. PANCHAL, J. :- Leave granted.
2. The instant appeal is directed against interim order dated April 25, 2008, rendered by
the Division Bench of the High Court of Punjab and Haryana at Chandigarh in CWP No.
6821-CAT of 2008 staying the order dated April 3, 2008 passed by the Central
Administrative Tribunal, Chandigarh Bench in Original Application No. 692-CH of 2007
(1) holding that the repatriation of the appellant to the State of Maharashtra under order
of Union of India dated April 10, 2007 as also his joining in his parent cadre under the
State of Maharashtra is valid, (2) quashing the order of suspension dated April 4, 2007 as
well as holding that final authority to take disciplinary action after termination/expiry of
the period of deputation vests with the Central Government and (3) directing the State of
Punjab to remit the entire matter relating to the disciplinary proceedings against the
appellant to the Central Government for taking a final decision.
3. The appellant is a 1970 batch IPS Officer of Maharashtra cadre. In the year 1984 he
was specially sent to Punjab to combat
@page-SC2767
militancy. It is the case of the appellant that the single minded devotion to get the State of
Punjab free from militancy bore fruits and today the State has become one of the most
peaceful and prosperous States in India. The appellant was promoted as Director General
of Police, Punjab, and he took several measures for public good. He issued several
instructions to the police force such as (i) not to accept Diwali gifts, (ii) not to heed to
any political interference and follow the rule book (iii) not to bow to pressures in cases of
land grabbing even if political leaders were involved, (iv) to adopt a professional attitude,
etc. According to him because of his commitment to duty he earned wrath of certain
sections of politicians and, therefore, false and frivolous allegations were levelled against
him by the respondent Nos. 5 and 7. After the formation of new Government the
respondent Nos. 5 and 7 requested through proper channel for pre-mature termination of
the repatriation of the appellant. The appellant also sought pre-mature termination of
repatriation. The Government of Punjab did not object to the request of the appellant for
pre-mature termination of his repatriation. On March 23, 2007 a First Information Report
being FIR No. 98 of 2007 was lodged in which one Vijay Pal Singh was named as an
accused. It is the case of the respondent that during police interrogation, said Vijay Pal
Singh allegedly stated that he had purchased some land for the appellant. On March 23,
2007 the investigating agency had moved an application before the competent court
seeking discharge of accused Vijay Pal Singh from FIR No. 98 of 2007. On the basis of
the statement made by Vijay Pal Singh during his interrogation, a departmental inquiry
was sought to be initiated against the appellant and the appellant was placed under
suspension by order dated April 4, 2007. Apprehending arrest in a false case the appellant
moved an application seeking anticipatory bail with reference to FIR No. 98 of 2007. On
notice being served, the Investigating Officer made a statement before the court that the
appellant was not required with reference to the said case. The appellant moved Criminal
Miscellaneous case No. 54610-M of 2007 seeking transfer of investigation of the
criminal case to CBI. The respondent State again made a statement on January 16, 2008
that the appellant was not required in connection with FIR No. 98 of 2007. The appellant
was served with article of charges. Meanwhile, the Government of Maharashtra gave its
no objection certificate to the Central Government for pre-mature termination of
repatriation of the appellant. The Government of Maharashtra also sent a copy of letter
dated March 28, 2007 to the Government of Punjab, but no objection was raised by the
Government of Punjab. For the first time on April 12, 2007 the Government of Punjab
wrote to the Central Government that by an order dated April 4, 2007, issued by the
Principal Secretary to the Government of Punjab, the appellant was put under suspension.
The Central Government, by an order dated April 10, 2007, ordered premature
termination of the repatriation of the appellant from Punjab to his parent cadre
Maharashtra. The order dated April 10, 2007 was neither reviewed nor recalled and is still
in force. On April 12, 2007 the Government of Punjab raised an objection to the
premature termination of the repatriation of the appellant from Punjab to Maharashtra on
the ground of his alleged suspension from service by order dated April 4, 2007. A case of
possession of disproportionate assets was registered by the Punjab Vigilance Bureau
against the appellant and he was arrested on September 9, 2007. Before effecting arrest of
the appellant neither the Delhi Police nor the Maharashtra Government nor the Central
Government was informed. Before registration of the said case no explanation or
comment was sought for from the appellant. As the appellant was of the opinion that
order suspending him as well as registering a case against him for possessing
disproportionate assets were illegal, he moved Central Administrative Tribunal,
Chandigarh Bench for quashing of those orders. The Tribunal, by order dated April 3,
2008, partly allowed the Original Application moved by the appellant and held that his
repatriation to the State of Maharashtra under order of Union of India dated April 10,
2007 as well as his joining parent cadre under the State of Maharashtra was valid. The
Tribunal further held that the order of suspension dated April 4, 2007 was bad in law and
quashed the same. It was also held by the Tribunal that final authority to take disciplinary
action after termination/expiry of the period of deputation was the Central Government
and directed the State of Punjab to remit the entire matter relating to the disciplinary
proceedings initiated against the
@page-SC2768
appellant to the Central Government for taking a final decision.
4. Feeling aggrieved by the above mentioned findings and directions given by the
Tribunal, the State of Punjab has invoked extraordinary jurisdiction of the High Court
under Article 226 of the Constitution of India by filing CWP No. 6821-CAT of 2008. The
High Court of Punjab and Haryana, by order dated April 25, 2008, has stayed the
operation of the order dated April 3, 2007 passed by the Central Administrative Tribunal,
Chandigarh Bench, Chandigarh, in Original Application No. 692-CH of 2007, giving rise
to the instant appeal.
5. This Court has heard the learned counsel for the parties at length and in great detail.
This Court has also considered the documents forming part of the appeal.
6. As noticed earlier the Union Government, by order dated April 10, 2007, has
repatriated the appellant to the State of Maharashtra whereas the order of suspension
dated April 4, 2007 is quashed by the Tribunal. Prima facie this Court is of the opinion
that after termination/ expiry of the period of deputation the final authority to take
disciplinary action against the appellant would be the Central Government. The
respondent No. 3 herein, i.e., the State of Maharashtra had filed written statement before
the Central Administrative Tribunal. In the said written statement it was mentioned that
the appellant had reported for duty in the State of Maharashtra on April 27, 2007 and was
allowed to join the duties in his parent cadre after his repatriation to Maharashtra by the
Central Government. It was further mentioned in the reply that the appellant had joined
the Government of Maharashtra on April 27, 2007 and as no post in the rank of Director
General of Police was vacant he was made to wait compulsorily. The statements made by
the State of Maharashtra in its written statement filed before the Central Administrative
Tribunal are reiterated before this Court by the learned counsel for the State of
Maharashtra. As the appellant has already joined duties in parent cadre pursuant to the
order issued by the Central Government, this Court is of the opinion that the High Court
was not justified in staying the declaration made by the Tribunal that repatriation of the
appellant from the State of Punjab to the State of Maharashtra was valid. The order dated
April 10, 2007 repatriating the appellant to the State of Maharashtra will have to be given
effect notwithstanding the order of suspension dated April 4, 2007. The declaration made
by the Tribunal that the Central Government is competent to take disciplinary action
against the appellant and directing the State of Punjab to remit the entire matter relating
to the disciplinary proceedings initiated against the appellant to the Central Government
should not have been stayed by the High Court.
7. On the facts and in the circumstances of the case this Court is of the opinion that the
interim order dated April 25, 2008, passed by the High Court of Punjab and Haryana in
CWP No. 6821-CAT of 2008, deserves to be modified by clarifying that it would be open
to the State of Maharashtra to give posting to the appellant on his repatriation to the State
of Maharashtra from the State of Punjab.
8. For the foregoing reasons the appeal partly succeeds. It is clarified that in view of
repatriation of the appellant to the State of Maharashtra under order dated April 10, 2007
of the Union of India it would be open to the State of Maharashtra to give posting to the
appellant as Director General of Police. This Court is informed by the learned counsel for
the parties that CWP No. 6821-CAT of 2008, filed by the State of Punjab, is listed for
final disposal before the High Court of Punjab and Haryana at Chandigarh on May 21,
2008. Having regard to the facts of the case the High Court of Punjab and Haryana is
requested to dispose of CWP No. 6821-CAT of 2008 finally on or before May 31, 2008.
The appeal is allowed to the extent indicated hereinabove.
9. There shall be no order as to costs.
Order accordingly.
AIR 2008 SUPREME COURT 2768 "P. S. Gopinathan v. State of Kerala"
(From : (2007) ILR (3) Ker 601)
Coram : 2 S. B. SINHA AND PRAKASH PRABHAKAR NAOLEKAR, JJ.*
Civil Appeal No. 3477 of 2008 (arising out of SLP (C) No. 18307 of 2007), D/- 9 -5
-2008.
P.S. Gopinathan v. State of Kerala and Ors.
(A) Constitution of India, Art.233 - Kerala State Higher Judicial Service Rules (1961),
R.6 - APPOINTMENT - JUDICIAL SERVICE - SENIORITY - ESTOPPEL -
@page-SC2769
Seniority - Petitioner, claimant appointed as District Judge by Governor on 14-1-1992 -
Posting order issued by High Court on 29-2-1992 treating him to be temporary appointee
- No objection raised by petitioner and readily accepting posting order joined service on
7-3-1992 - Fresh appointment order was issued again on 15-7-1992 by Governor and
petitioner was posted on 31-7-1992 treating bis first appointment order as temporary
appointment - No protest was made by him - Act and action of petitioner in accepting his
appointment as temporary one amounts to his assent to temporary appointment - Not
entitled to claim permanent appointment from 14-1-1992 to claim seniority on post.
Evidence Act (1 of 1872), S.115. (Paras 31, 32, 5)
(B) Constitution of India, Art.233 - Kerala State Higher Judicial Service Rules (1961),
R.6 - APPOINTMENT - JUDICIAL SERVICE - PROMOTION - SENIORITY -
Selection Grade and Super-selection Grade - Orders of promotions in terms of Rules were
granted on basis of merit and ability - Seniority was considered to be relevant only where
merit and ability were approximately equal - Inter se seniority, therefore, not the sole
criteria. (Per S.B. Sinha, J.) (Paras 3, 4)
Cases Referred : Chronological Paras
AIR 1976 SC 2428 : 1976 Lab IC 1546 (Ref) 5
(1880)15 Ch D 96 : 43 LT 95 30
(1860) 3 LT 130 : 3 HLC 829 31
P. S. Patwalia, Sr. Advocate, Jayanth Muthraj and K. V. Mohan, with him for Appellant;
Parag P. Tripathi, AAG, Jawahar Lal Gupta, L. N. Rao, T. L. V. Iyer, Sr. Advocates, Romy
Chacko, Arpit Gupta, B. S. Verma, G. Prakash, Ms. Vismai Rao and T. G. Narayanan
Nair, with them for Respondents.
* The Judgments are printed in the order in which they are given in the certified
copy....Ed.
Judgement
S. B. SINHA, J :- Although I respectfully agree with the opinion of my learned brother
Naolekar, J., I would like to add a few words.
2. Respondent Nos. 3 to 5 herein were granted selection grade much prior to the
appellant. They have also been granted super-selection grade. They have been thus placed
in Category -I of the services. For all intent and purport they were promoted much prior
to the appellant herein.
3. Such orders of promotion in terms of the Rules were granted on the basis of merit and
ability. Seniority was considered to be relevant only where merit and ability were
approximately equal. The inter se seniority, therefore, does not remain the sole criteria.
4. The case of the appellant for the purpose of grant of selection grade as also super-time
scale must have been considered alongwith the said respondents. They must have been
found to be more meritorious. In that view of the matter, it is, in our opinion, not a fit
case where we should even exercise our jurisdiction under Article 136 of the Constitution
of India.
5

. No doubt the Governor is the appointing authority of the District Judges in the State.
However, the same in terms of the constitutional provisions, was required to be done in
consultation with the High Court. The High Court keeping in view the amendments made
in the Rule treated the appointment of the appellant as temporary. Apart from the fact that
the appellant accepted his posting orders without any demur in that capacity, his
subsequent order of appointment dated 15th July, 1992 issued by the Governor had not
been challenged by the appellant. Once he chose to join the mainstream on the basis of
option given to him, he cannot turn back and challenge the conditions. He could have
opted not to join at all but he did not do so. Now it does not lie in his mouth to clamour
regarding the cutoff date or for that matter any other condition. The High Court,
therefore, in our opinion, rightly held that the appellant is estopped and precluded from
questioning the said order dated 14th January, 1992. The application of principles of
estoppel, waiver and acquiescence has been considered by us in many cases, one of them
being Dr. G. Sarana v. University of Lucknow and others, [AIR 1976 SC 2428] stating :-
Para 15 of AIR

"He seems to have voluntarily appeared before the Committee and taken a chance of
having a favourable recommendation from it. Having done so, it is not now open to him
to turn round and question the constitution of the Committee."
6. P. P. NAOLEKAR, J. :- Leave granted.
7. The appellant was directly recruited to the post of Munsiff and was later promoted to
the post of Subordinate Judge. Thereafter, the appellant was promoted to the Kerala State
Higher Judicial Service, whereas the respondents 3 to 5 were directly
@page-SC2770
recruited to the Higher Judicial Service.
8. The subordinate judiciary in the State of Kerala consisting of District Judges, Chief
Judicial Magistrates, Subordinate Judges, Munsiffs, Judicial Magistrates of the First
Class, Judicial Magistrates of the Second Class, was functioning in the matter of
appointment and promotions as two separate wings, (1) consisting of the Kerala State
Higher Judicial Service and the Kerala Civil Judicial Service and (2) the Kerala Criminal
Judicial Service. The Higher Judicial Service consisted of District Judges. The Kerala
Civil Judicial Service consisted of Subordinate Judges and Munsiffs, whereas the Kerala
Criminal Judicial Service consisted of Chief Judicial Magistrates, Judicial Magistrates of
the First Class and Judicial Magistrates of the Second Class. The Civil Judicial Service
and the Criminal Judicial Service came into being as a result of the orders issued under
G.O.(Ms) 24/73/Home dated 12-2-1973 and with that the subordinate judiciary was also
bifurcated for the first time into Civil Wing and Criminal Wing. The Higher Judicial
Service all along constituted one separate Service to which after the implementation of
the G.O. dated 12-2-1973, only the Civil Judicial Service Officers are eligible to be
considered. Thus, there was subordinate Civil Judicial Service and the Criminal Judicial
Service. Promotion to the Higher Judicial Service was available from the Kerala Civil
Judicial Service only.
9. The Higher Judicial Service was constituted under the Kerala State Higher Judicial
Service Rules published on 18-7-1961. It consisted of two categories : Category 1 :
Selection Grade District and Sessions Judge and Category 2 : District and Sessions Judge
(including Additional District and Sessions Judge). Appointment to Category 2 was to be
made by transfer (promotion) from the category of Civil Judicial Service or by direct
recruitment from the Bar. The number of posts to be filled up by direct recruitment is
1/3rd of the permanent posts in Categories (1) and (2) taken together. While under the
said Rules, a select list of subordinate Judges (Civil category) was prepared on 8-1-1991
and approved by the Administrative Committee of the High Court, was also approved by
the Full Court on 11-1-1991. The Government also approved the select list of
Subordinate Judges by its order dated 19-11-1991 for appointment as District and
Sessions Judges. The appellant was ranked No. 2 in the said list. A letter dated 10-12-
1991 was addressed by the Registrar, High Court, to the Commissioner and Secretary to
the Government in Home Department, wherein it was stated that the sanction had been
accorded for the establishment of three Special Courts at Thiruvananthapuram for the
trial of mark list cases in the light of the Supreme Court direction dated 20-8-1991; the
Government had also sanctioned three posts of District Judges; as per the Government
Order, the Government had approved 11 Sub-Judges for appointment as District Judges in
the Kerala State Higher Judicial Service and in order to provide three District Judges in
the new Special Courts to be established at Thiruvananthapuram, the actual appointment
as District Judges of the following three officers was considered necessary to be made: (i)
P.S. Gopinathan; (ii) K.S. Gopinathan Pillai; and (iii) M.V. Viswanathan; and, therefore,
the Government orders and notifications appointing the above three officers as a panel of
District Judges in the Kerala State Higher Judicial Service may be issued immediately.
10. In place of the Kerala Civil Judicial Service and the Kerala Criminal Judicial Service,
a common Service was brought into force by the Kerala Judicial Service Rules, 1991
published in the Gazette on 31-12-1991 and amendment to the Kerala Higher Judicial
Service Rules, 1961 vide G.O.(P) No. 47/92/Home dated 28-2-1992 was given
retrospective effect from 1-1-1992, the date of coming into force of the Kerala Judicial
Service Rules. The common Service constituted under the said Rules consisted of the
following categories of officers :
Category 1 : Subordinate Judges/Chief Judicial Magistrates
Category 2 : Munsiffs/Magistrates.
11. After the enactment of the Kerala Judicial Service Rules, 1991, the Governor in
exercise of the powers conferred by clause (1) of Article 233(1) appointed three Sub-
Judges, including the appellant, as District and Sessions Judge in the Kerala State Higher
Judicial Service on 14-1-1992, without prejudice to the claims of candidates to be
recruited from the Bar to satisfy the provisions in Rule 2(b) of the Kerala State Higher
Judicial Service Rules, 1961.
12. Before the appointment order of the
@page-SC2771
appellant was issued on 14-1-1992, the new Rules integrating two lower subordinate
services had come into force and the question arose whether the vacancies in the three
courts created as per the direction of the Supreme Court would be filled up by temporary
appointments and the order of the Governor issued could be treated as appointing the
officers temporarily without there being any claim of seniority by the officers who had
been appointed to fill up those vacancies. The Administrative Committee of the High
Court approved and made the following recommendations among other matters :
"(1) The panel of Sub Judges prepared by the High Court and approved by the
Government be annulled except in the case of those already appointed from the panel;
(2) Even in the case of those appointed from the panel after 1-1-1992, their appointment
may be treated as temporary without probationary rights. Their seniority in the category
be decided later after a fresh panel is prepared;
(3) Steps be taken to prepare a fresh panel for appointment as District Judges from among
Sub Judges/Chief Judicial Magistrates in service on 1-1-1992;
(4) While taking such steps, the case of Shri K.N. Balakrishna Panicker, the senior most
Sub Judge who had been superseded last time, be considered after assessing his
judgments."
The Committee recommended that the case of K.N. Balakrishna Panicker (respondent
No. 6), the senior-most Sub-Judge who had been superseded last time be also considered
after assessing his judgments. The Full Court approved the recommendations made by the
Administrative Committee.
13. From the Official Memorandum issued by the High Court dated 9-9-2005, the reason
for taking the decision for posting the appellant on a temporary basis appears to be that
the select list was prepared on the basis of the seniority list which had been prepared
before the integration of the two Services, i.e. Civil Judicial Service and Criminal Judicial
Service. After the integration of the two Services, the select list was prepared and the
appointment order was issued on the said basis, whereas before the appointment order
was issued on 14-1-1992, the rules were amended and the two Services were integrated.
Therefore, there was a necessity to prepare the fresh seniority list in the light of the
integration of the Civil and Criminal Wings of the State Subordinate Judiciary w.e.f. 1-1-
1992. Pending preparation of the combined seniority list, a posting order was issued
without any probationary rights.
14. On 29-2-1992, the High Court passed the order with regard to the posting of the
appellant who had been appointed as a District Judge in the Kerala State Higher Judicial
Service and he was posted as an Additional District Judge, Kozhlkode and was directed
to hand over his charge to the Additional Sub-Judge, Vadakara. The posting of the
appellant along with other officers was made temporary without probationary rights and
their seniority in the category of the District Judges was to be determined later. By this
order, the High Court posted the appellant as Additional District Judge on temporary
basis without probationary rights and seniority was left open to be considered on a later
date.
15. The appellant joined service on 7-3-1992 as per the posting order. Respondents 3 to 5
were the direct recruits from the Bar to the Higher Judicial Service.
16. On 29-2-1992, Rule 2(b) of the Rules was substituted by providing for the category of
Chief Judicial Magistrates also to be a feeder category for appointment to Category (2) of
the Kerala State Higher Judicial Service under the Service Rules, which came into effect
on 1-1-1992. While so, respondents 3 to 5 were directly appointed as District Judges by
the Governor. Respondents 3 and 4 were appointed on 31-3-1992 and they took charge on
2-4-1992, whereas respondent 5 was appointed by order dated 30-5-1992 and he took
charge on 1-6-1992.
17. After the introduction of Service Rules, 1991, which came into effect on 1-1-1992,
the High Court drew a fresh panel of Sub-Judges and Chief Judicial Magistrates for
promotion as District and Sessions Judges from the integrated seniority list of Sub-Judges
and Chief Judicial Magistrates. The fresh panel was approved by the Full Court on 21-2-
1992.
18. On 15-7-1992, the Governor of Kerala approved the panel of Sub-Judges and Chief
Judicial Magistrates for appointment as District and Sessions Judges in the Kerala State
Higher Judicial Service, without prejudice to the claim of the candidates to be recruited
@page-SC2772
from the Bar as provided in Rule 2(b) of the Kerala State Higher Judicial Service Rules,
1961. In the said panel, respondent 6 (K.N. Balakrishna Panicker) was shown at Sl.No. 1,
whereas the appellant was at Sl.No.2. On the same day, the Governor of Kerala was
pleased to appoint the Sub-Judges and Chief Judicial Magistrates at Sl.Nos. 1 to 8 in the
panel approved in the Government Order as District and Sessions Judges in the Kerala
State Higher Judicial Service without prejudice to the claims of the candidates to be
recruited from the Bar. This appointment order indicates respondent 6 at Sl. No. 1 and the
appellant at Sl.No.2.
19. On 31-7-1992, the High Court passed an order whereunder the appellant who was
posted in the Motor Accident Claims Tribunal, Kozhikode (now temporary District
Judge) who had been appointed as District and Sessions Judge in the Kerala State Higher
Judicial Service as per the Government Order, was allowed to continue in the present post
as regular District Judge. This order indicates that the appellant was appointed
temporarily as District Judge and by a later appointment order issued by the Governor he
was treated as a regular District Judge and was allowed to continue on the same post.
20. On the basis of the office memorandum of High Court dated 29-9-1992, it was
proposed to show the seniority of the appellant below the 6th respondent. The appellant
made a representation on 28-10-1992 to the High Court claiming that his appointment as
District and Sessions Judge should not be treated as a temporary one and to treat him
senior to respondents 3 to 6 in the category of District and Sessions Judges. On 16-8-
1994, a draft seniority list of Selection Grade District and Sessions Judges was published.
The appellant was shown below respondents 3 to 6. While the appellant was shown at
Sl.No. 60, respondents 3 to 6 were shown at Sl.Nos. 56, 57, 58 and 59. The appellant
again submitted a representation on 25-10-1994 challenging the assignment of seniority
to the respondents above the appellant. On 18-1-1995, the High Court issued an order
declaring completion of the probation period by the appellant. In the said order, the
commencement of the appellant's probation was shown as on 31-7-1992 (the date when
the second posting order was issued). As per the representation, the commencement of
the probation of the appellant as on 31-7-1992 could not have been shown. The appellant
was appointed as District and Sessions Judge as per the order and notification dated 14-1-
1992 and joined duty on 7-3-1992 and continued to be in service without any break and,
thus, the commencement of the period of probation should have been from that date. The
appellant submitted representation to that effect on 17-4-1995. Thereafter, a reminder
representation on 7-2-1998 and another representation on 20-5-2000 were made by the
appellant. On 18-12-2003, the High Court issued order permitting respondents 3 to 5 as
Selection Grade District and Sessions Judges in preference to the appellant. On 8-3-2004,
the appellant filed representation challenging the seniority list in the category of District
and Sessions Judges in Selection Grade given to the respondents in preference to the
appellant. On 12-3-2004, the High Court sent a communication to the appellant stating
that his representations for re-fixation of his seniority in the cadre of District and Sessions
Judge were under consideration. The appellant's representation for advancing the date of
his promotion as Selection Grade District Judge would also be considered. Thereafter on
1-9-2004, the High Court gave permission to respondents 3 to 5 and the appellant to the
category of Selection Grade District Judges with retrospective effect from 3-7-2000, 12-
7-2000, 12-7-2000 and 12-7-2000 respectively. In the said order, the appellant was
shown at Sl.No. 14, whereas respondents 3 to 5 were shown as Sl.Nos. 11, 12 and 13
respectively. The appellant was shown junior to respondents 3 to 5 in the category of
Selection Grade District Judges. Therefore, the appellant again submitted a representation
on 15-9-2004. On 2-11-2004, the High Court again permitted respondents 3 to 5 as Super
Time Scale District and Sessions Judges w.e.f. 13-10-2004, 14-10-2004 and 28-10-2004
respectively, whereas the appellant was retained as Selection Grade District and Sessions
Judge. Respondent 6 had already left the Judicial Department while holding the post of
District and Sessions Judge and he was appointed as Railway Claims Tribunal.
21. On 22-5-2005, the appellant filed a writ petition before the High Court of Kerala
challenging the draft seniority list dated 16-8-1994, order dated 18-1-1995 (declaration of
the probation of the appellant), order
@page-SC2773
dated 18-12-2003 (promotion of respondents 3 to 5 to the post of Selection Grade District
and Sessions Judges) and order dated 1-9-2004 [proceedings of the High Court (2nd
respondent) permitting respondents 3 to 5 as Super Time Scale District and Sessions
Judges]. On 9-9-2005, during pendency of the writ petition, the High Court rejected the
appellant's representation for re-fixation of his seniority in the category of District and
Sessions Judges. On 13-10-2005, the appellant's representation challenging the draft
seniority list was also rejected. The appellant challenged the Office Memorandum dated
13-10-2005 by amendment of the writ petition. On 19-2-2007, the learned Single Judge
of the High Court referred the case to be heard by a Bench of two Judges. It would be
pertinent to note that in the writ petition the appellant had not challenged the
Memorandum dated 15-7-1992 whereby the Governor of Kerala had approved the fresh
panel of Sub-Judges and Chief Judicial Magistrates for appointment as District and
Sessions Judges, the order of appointment of the appellant from the panel as District and
Sessions Judge in the Kerala State Higher Judicial Service by order dated 15-7-1992 or
the posting order dated 31-7-1992 whereby the appellant was shown to be a temporary
District Judge and was posted as regular District Judge after the issuance of the order of
appointment on the post of District Judge by order dated 15-7-1992.
22. The Division Bench of the High Court has taken into consideration various aspects
which had arisen for determination of the court, namely, as to which order, i.e. order
dated 14-1-1992 or order dated 15-7-1992 shall be taken to be the first appointment
order which is relevant for Rule 2(b) of the Kerala Higher Judicial Service Rules;
whether it is open to 2nd respondent- High Court to treat the order dated 14-1-1992
passed by the Governor under Article 233 to have given the appellant status of a
temporary employee appointed in the Higher Judicial Service and not to treat that period
of service as on probation. The High Court further considered the effect of not
challenging the orders passed by the Governor on 15-7-1992 and 31-7-1992, by the
appellant. Whether the appointment order issued by the Governor under Article 233 can
be pronounced as having no legal effect or illegal on account of violation of the rules
which provide for the integration of Civil and Criminal Wings and the order of
appointment being issued without taking into consideration the integrated service on the
feeder post.
23. The High Court held that the Governor is the appointing authority of the District
Judges in the State which shall be done by the Governor in consultation with the High
Court. The High Court while issuing the order of posting treated the appellant as
temporary and acted entirely bona fide since the amendment including Chief Judicial
Magistrates in the feeder category for promotion as District Judges, was not only on the
anvil but, in fact, it was published on the same day with retrospective effect from 1-1-
1992. Since Rule 6 of the Kerala Higher Judicial Service Rules declares that the seniority
will be determined on the first order of appointment, the High Court has committed an
error in treating the appellant's appointment as temporary appointment, particularly when
there is no indication in the order of the appointing authority, namely, the Governor, that
the appointment of the appellant was temporary. However, since the appellant has
accepted the posting order treating him to be a temporary employee while joining duty,
the subsequent orders issued approving the fresh panel consisting both of Sub-Judges and
Chief Judicial Magistrates in purported compliance of the mandate of the Rules as
amended, the appointment order of the Governor and thereafter the posting order
indicating him to be a temporary District Judge and appointing him as regular employed
District Judge have not been challenged. Thus, the High Court was of the view that since
the appointment order dated 15-7-1992 issued by the competent authority, namely, the
Governor, had not been challenged by the appellant, he cannot challenge the previous
order since there can only be one appointment order with reference to which seniority can
be ascertained under Rule 6 and the High Court took the appointment order dated 15-7-
1992 as the order of appointment of the appellant to the post of District Judge in the
Higher Judicial Service and consequently dismissed the writ petition filed by the
appellant herein.
24. It is submitted by Shri P. S. Patwalia, learned senior counsel appearing for the
appellant that the appellant's seniority has to be counted from the date of his appointment
on 14-1-1992 made by the Governor in exercise
@page-SC2774
of the powers under Article 233 in consultation with the High Court. Merely because the
rule has been amended with retrospective effect from 1-1-1992 whereby the feeder post
to the transfer (appointment) to the category of District and Sessions Judges (including
Additional District and Sessions Judges) was made from the category of Subordinate
Judges/Chief Judicial Magistrates of the Kerala Judicial Service instead of the category of
Subordinate Judges only, it is urged by the learned senior counsel that the post being
available prior to the rule being amended, the appellant's transfer (appointment) to the
post of District and Sessions Judge could not have been treated as temporary appointment
without probationary rights.
25. It is urged by Shri Jawahar Lal Gupta, learned senior counsel appearing for
respondent No. 3 that the appellant was although appointed on 14-1-1992 his
appointment for all purposes and intent as expressly mentioned in the posting order dated
29-2-1992 had been treated as temporary appointment without probationary rights which
was accepted by the appellant by accepting the fresh appointment order dated 15-7-1992
as permanent appointment on the post of Category (2) District and Sessions Judge and,
thus, the appellant cannot now contend that the order dated 14-1-1992 was his
appointment on the permanent basis on the cadre post of Category (2) District and
Sessions Judge. That apart, it is submitted by the learned senior counsel that the appellant
having not challenged the order dated 15-7-1992 or order dated 31-7-1992 issued by the
High Court posting him, is not entitled to get the seniority on the basis of the order passed
on 14-1-1992.
26. Shri L.N. Rao, learned senior counsel appearing for respondents 4 and 5 has
submitted that although the post had accrued when the old rules were in operation, the
appointment has to be treated under the old rules but authority can certainly say that the
appointment shall not be made under the old rules and can defer the appointment until the
new rules came into force.
27. Shri T.L.V. Iyer, learned senior counsel appearing for respondent No. 2 - Kerala High
Court has supported the decision taken by the High Court in treating the appointment of
the appellant dated 14-1-1992 as temporary appointment.
28. The relevant rules of the Kerala State Higher Judicial Service Rules, 1961, which
shall govern the appointment and seniority of Category (2) : District and Sessions Judges,
read as under :
"1. Constitution.- The service shall consist of the following categories, namely :-

Category (1) Selection Grade District and Sessions Judge.


Category (2) District and Sessions Judge (including Additional District and Sessions
Judge).

2. Method of appointment.- (a) Appointment to category (1) shall be made by the High
Court by promotion from category (2).
(b) Appointment to category (2) shall be made by transfer from category 1, Subordinate
Judges/Chief Judicial Magistrates of the Kerala Judicial Service or by direct recruitment
from the Bar, provided that the number of posts in category (2) to be filled up or reserved
to be filled up by direct recruitment shall be one-third of the permanent posts in
categories (1) and (2) taken together.
(c) Appointment by promotion to category
(1) and appointment by transfer to category
(2) shall be made on the basis of merit and ability, seniority being considered only where
merit and ability are approximately equal.
3. Qualification.- (1)(a) No person appointed to category (2) either by transfer or by
direct recruitment shall be eligible for promotion to category (1) unless he is an approved
probationer in category (2) on the date of occurrence of the vacancy.
(b) Omitted
(2) A candidate for appointment to category (2) from the Bar shall satisfy, the following
general conditions namely :-
xxx xxx xxx
4. Probation.- (a) Every person appointed to category (2) shall, from the date on which he
joins duly, be on probation for a period of two years on duty within a continuous period
of three years.
(b) There shall be no probation for category (1).
5. Appointing Authority.- (1) All appointments to category (1) shall be made by the High
Court.
(2) All first appointments to category (2) whether by direct recruitment or by transfer,
shall be made by Governor in consultation with the High Court.
@page-SC2775
5A. Postings and Transfers.- All postings and transfers of persons appointed to categories
(1) and (2) shall be made by the High Court.
6. Seniority.- (1) The seniority of a person appointed either to category (1) or category (2)
shall, unless he has been reduced to a lower rank as punishment, be determined with
reference to the date of the order of his first appointment to the said category :
xxx xxx xxx
Under Rule 1, the cadre of District and Sessions Judges is in two categories: Category (1)
relates to Selection Grade District and Sessions Judges, whereas Category (2) relates to
District and Sessions Judges (including Additional District and Sessions Judges). Rule
2(b) provides for appointment by transfer (promotion) to Category (2) i.e. District and
Sessions Judges (including Additional District and Sessions Judges). Feeder post from 1-
1-1992 shall be Subordinate Judges/Chief Judicial Magistrates of the Kerala Judicial
Service or by direct recruitment from the Bar. Under clause (c) of Rule 2, the eligibility
criteria for transfer/promotion to the post of District and Sessions Judge shall be on the
basis of merit and ability and seniority shall be taken into consideration only where the
merit and ability of the promotee officer are approximately equal. Thus, the promotion to
the post of District and Sessions Judge in Category (2) from the post of Subordinate
Judge/Chief Judicial Magistrate would be on the basis of merit-cum-seniority. Under
Rule 4, every person appointed to Category (2) shall be on probation for a period of two
years within the continuous period of his service for three years from the date of his
joining duty on the promoted post. By virtue of sub-rule (2) of Rule 5, the first
appointment to Category (2), whether by direct recruitment or by transfer, shall be made
by the Governor in consultation with the High Court. Rule 6 on which the emphasis is
laid by the appellant is in regard to the seniority of a person appointed either to Category
(1) or Category (2), i.e. District and Sessions Judges, says that unless the person
appointed is reduced to a lower rank as punishment, his seniority shall be determined
with reference to the date of the order of his first appointment to the said category.
Therefore, for the purposes of ascertaining the seniority of an officer, the date of the order
of his first appointment will have a relevant consideration under the rules.
29. It is an admitted fact that on 14-1-1992 the appellant was appointed on the post of
District and Sessions Judge by the. Governor in exercise of the power conferred by clause
(1) of Article 233, without prejudice to the claim of candidates to be recruited from the
Bar to satisfy the provisions of Rule 2(b) of the Kerala State Higher Judicial Service
Rules, 1961. However, as there was an integration of the Civil Judicial Service and
Criminal Judicial Service, the matter was referred to the Administrative Committee and
the Committee recommended that the entire panel prepared of Sub-Judges (which does
not include the Chief Judicial Magistrates) and approved by the Government be annulled
except in the case of those already appointed from the panel. The Administrative
Committee further recommended that the appointment from the panel after 1-1-1992
shall be treated as temporary without probationary rights and their seniority in the
category be decided later after a fresh panel is prepared and the directions were issued for
preparation of the fresh panel. This recommendation of the Administrative Committee
was accepted by the Full Court. The reason for taking this decision was that the select list
was prepared on the basis of the seniority list which had been prepared before integration
of two Services. After the integration of two Services w.e.f. 1-1-1992, the appointment
order was issued by the Governor on 14-1-1992 without taking into consideration the
integrated service of the Sub-Judges and the Chief Judicial Magistrates. The appointment
order of the appellant was issued on 14-1-1992. In pursuance thereof and the decision
taken by the High Court, a posting order was issued on 29-2-1992. The posting order
clearly specified that the appellant was being posted as Additional District Judge on
temporary basis without probationary rights in the category of District Judges and his
seniority in the category of District Judges will be determined on a later date. The
appellant took charge of the post on 7-3-1992 without any demur or objection. When the
appellant was continuing on the post, respondents 3 to 5 were appointed as District and
Sessions Judges in the quota of direct recruits. Later on, a fresh panel for the
transfer/promotion was prepared by the High Court which was approved by the Governor
and a fresh appointment order issued on 15-7-1992
@page-SC2776
without prejudice to the claim of the candidates recruited from the Bar. In pursuance of
the appointment order issued by the Governor on 15-7-1992, on 31-7-1992 the appellant
was posted on the same post where he was serving on the post of Motor Accident Claims
Tribunal. The posting order categorized him as 'now temporary District Judge' and he was
allowed to continue on the post as a regular District Judge. The posting order treated the
appellant as a temporary District Judge till that date and he was treated as a regular
District Judge from the date of posting, i.e. 31-7-1992, in pursuance of the order issued
on 15-7-1992. All along by posting order dated 29-2-1992 as well as by posting order
dated 31-7-1992, the appellant's appointment on the post of District and Sessions Judge
has been treated on temporary basis. Yet, the appellant kept silence, accepted the orders
and worked on the post as temporarily appointed and posted District and Sessions Judge.
It is after lapse of considerable period he made a representation on 28-10-1992
complaining that he was appointed by appointment order dated 14-1-1992 and in
pursuance of his posting orders he had joined the duty on 7-3-1992; thereafter he had
been continuously working on the post; later on, direct recruitment was made whereby
three District and Sessions Judges were appointed but he was being proposed to be
ranked below K.N. Balakrishna Panicker; he cannot be treated as junior to Panicker in the
light of the definition of the term 'appointed to service' as occurring under the rules and
other provisions governing service and seniority he is entitled to be ranked above
Panicker; in view of the vested rights which accrued to him on the basis of his legal
entitlement in the light of the inclusion of his name in the select list, the admitted vacancy
available and the order of appointment passed by the Governor, his appointment as
District and Sessions Judge cannot at all be treated as temporary.
30. The law of equitable estoppel by acquiescence has been clearly stated by Fry, J. in
Wilmott v. Barber, (1880) 15 Ch D 96, 105 : 43 LT 95. It has been said therein that the
acquiescence which will deprive a man of his legal rights should amount to fraud. A man
is not to be deprived of his legal right unless he has acted in such a way as would make it
fraudulent for him to set up those rights. What, then, are the elements or requisites
necessary to constitute fraud of that description, are stated thus :
(i) The plaintiff (i.e. the party pleading acquiescence) must have made a mistake as to his
legal rights;
(ii) The plaintiff must have expended some money or must have done some act (not
necessarily upon the defendant's land) on the faith of the mistaken belief;
(iii) The defendant, the possessor of the legal right, must know of the existence of his
own right which is inconsistent with the right claimed with the right claimed by the
plaintiff. If he does not know of it, he is in the same position, as the plaintiff, and the
doctrine of acquiescence is founded upon conduct with a knowledge of your legal rights;
(iv) The defendant, the possessor of the legal right, must know of the plaintiffs mistaken
belief of his rights. If he does not, there is nothing which calls upon him to assert his own
rights; and
(v) The defendant, the possessor of the legal right must have encouraged the plaintiff in
his expenditure of money, or in the other acts which he has done, either directly or by
abstaining from asserting his legal right. Where all these elements exist, there is fraud of
such a nature as will entitle the court to restrain the possessor of the legal right from
exercising it, but nothing short of this will do. These principles were followed and
applied in many cases in India.
31. The appellant was appointed by the Governor by transfer/appointment order issued on
14-1-1992 and his seniority was to be considered as provided under Rule 6 of the Rules
which says that the seniority of a person appointed shall be determined with reference to
the date of the order of his first appointment to the category. Thus, apparently when the
order dated 14-1-1992 was issued by the Governor, it would be the first appointment for
the determination of the seniority of the appellant. There is nothing in the order dated 14-
1-1992 on the basis of which it can be treated to be an appointment on temporary basis
made by the Governor. It is apparent from the posting order dated 29-2-1992 that the
High Court, because of the integration of two Services w.e.f. 1-1-1992 prior to the
issuance of the order of appointment on 14-1-1992, has treated the order of appointment
as a temporary one and, therefore, the posting order specifically
@page-SC2777
mentioned that the appellant's appointment would be temporary without there being any
probationary rights. Thereafter on 21-2-1992, a fresh panel was prepared by the High
Court for the purposes of transfer/promotion to Category (2) : District and Sessions Judge
and the proposal for promotion of the officers in that list has been accepted by the
Governor by issuance of the order of appointment including that of the appellant on 15-7-
1992. On 31-7-1992, the appellant's posting order was issued. Posting order clearly
indicated that the appellant was working as a temporary District Judge and by virtue of
the order issued on 15-7-1992 he shall be treated as permanent District Judge. While
issuing the posting order of the appellant in pursuance of the order dated 14-1-1992, the
High Court has committed a mistake in treating it to be an order of temporary
appointment of the appellant when there was nothing to this effect in the appointment
order. When the posting order was issued on 29-2-1992, the appellant was well aware of
the order of his appointment dated 14-1-1992 whereby he was appointed on permanent
basis on the post of District and Sessions Judge, yet when the posting order was issued
treating him to be a temporary appointee which was inconsistent with the order dated 14-
1-1992, the appellant did not raise any objection and readily accepting the posting order
joined the service on 7-3-1992 as temporary Additional District Judge. The posting order
dated 29-2-1992 specifically mentioned that he has been posted as a temporary
Additional District Judge without any probationary rights and thus the appellant was well
aware of the mistaken belief of the High Court in appointing and posting him as a
temporary employee. As there was no objection and protest by the appellant, a fresh panel
prepared, recommended and fresh order of appointment of appellant was issued by the
Governor. Again when the fresh appointment order was issued on 15-7-1992 by the
Governor and the appellant was posted on 31-7-1992 treating his first appointment order
as a temporary appointment, no protest was made by him. In view of the fact that the
Governor issued an order dated 15-7-1992 even when order of 14-1-1992 was in
existence, it is apparent that the appointing authority has also treated the first order dated
14-1-1992 as an order of appointment on temporary basis. It is, therefore, apparent from
the second appointment order that the appointing authority as well as the posting
authority have all along treated the appellant as a temporary District Judge, but the
appellant did not object on both occasions when he Joined on 7-3-1992 and on 31-7-1992
of he being treated as temporary District Judge. The act and action of the appellant in
accepting his appointment as temporary one amounts to his assent to the temporary
appointment and the appellant throughout till he raised an objection on 29-10-1992 has
slept on his right of being appointed permanently on the post of District and Sessions
Judge. By his conduct at the time of the issuance of the order by the High Court on 29-2-
1992 and thereafter issuance of the second appointment order on 15-7-1992 with full
knowledge of his own right and the act of the High Court which infringes it, led the High
Court to believe that he has waived or abandoned his right. Lord Campbel in Cairncross
v. Lorrimer, 3 LT 130 held that "generally speaking if a party having an interest to
prevent an act being done had full notice of its being done, and acquiesce it, so as to
induce a reasonable belief that he consents to it and the position of the others is altered by
their giving credit to his sincerity, he has no more right to challenge the act to their
prejudice than he would have had if it had been done by his previous license."
32. The aforesaid facts clearly make out an acquiescence of the appellant of accepting
order dated 14-1-1992 being treated as temporary appointment order on the post of
District and Sessions Judge and he cannot now be permitted to change his position and
claim the permanent appointment from 14-1-1992 to claim seniority on the post. Besides
this, the High Court has rightly held that in the absence of the challenge to the second
appointment order dated 15-7-1992 from the fresh panel dated 21-2-1992, that order will
stand, though later in time, and has to be given effect to as an order of appointing the
appellant on permanent basis under Rule 6 of the Rules.
33. For the aforesaid reasons, the appeal is dismissed. However, there shall be no order as
to costs.
Appeal dismissed.
@page-SC2778
AIR 2008 SUPREME COURT 2778 "Baijnath Jha v. Sita Ram"
(From : Patna)*
Coram : 2 Dr. A. PASAYAT AND PRAKASH PRABHAKAR NAOLEKAR, JJ.
Criminal Appeal Nos. 403 with 404 and 405 of 2000, D/- 12 -6 -2008.
Baijnath Jha v. Sita Ram and Anr.
(A) Criminal P.C. (2 of 1974), S.482 - INHERENT POWERS - Inherent powers -
Exercise of - No infelexible rule can be laid down which would govern exercise of
powers. (Para 3)
(B) Criminal P.C. (2 of 1974), S.482 - INHERENT POWERS - APPEAL - REVISION -
Inherent powers - Exercise of - Not as a Court appeal or revision. (Para 3)
(C) Criminal P.C. (2 of 1974), S.482 - INHERENT POWERS - Inherent powers - Are
very wide - However, Courts have to exercise them sparingly, carefully and with caution.
(Para 3)
(D) Criminal P.C. (2 of 1974), S.482 - INHERENT POWERS - EVIDENCE - Inherent
powers - Exercise of, to quash proceedings - Distinction between a case where there is no
legal evidence and a case where there is legal evidence, stated.
AIR 1992 SC 604; AIR 1960 SC 866, Relied on. (Para 5)
(E) Criminal P.C. (2 of 1974), S.482 - INHERENT POWERS - Inherent powers - Should
not be exercised to stifle a legitimate prosecution.
1993 AIR SCW 248; AIR 1964 SC 1, Relied on. (Para 6)
(F) Criminal P.C. (2 of 1974), S.482 - INHERENT POWERS - THEFT - ELECTRICITY
- Quashing of proceedings - Theft of electricity - Compliant lodged against member of
raiding party alleging that he demanded illegal gratification - No date of such alleged
demand was indicated in complaint - Background clearly showed that proceedings
instituted on complaint were mala fide, based on vague assertions - And were initiated
with mala fide intents - Proceedings are therefore liable to be quashed.
AIR 1992 SC 604, Rel on.
Cri. Misc. No. 10561 of 1998, D/-23-11-1008 (Pat.), Reversed. (Paras 7, 8)
Cases Referred : Chronological Paras
1993 AIR SCW 248 : AIR 1993 SC 892 : 1993 Cri LJ 608 (Rel on) 6
AIR 1992 SC 604 : 1992 Cri LJ 527 (Rel on) 5, 7
AIR 1964 SC 1 : 1964 (1) Cri LJ 1 (Rel on) 6
AIR 1960 SC 866 : 1960 Cri LJ 1239 (Rel on) 4
Pramod Swarup, for Appellant; Manish Kapur and Gopal Singh, for Respondents.
* Cri. Misc. No. 10561 of 1998, D/- 23-11-1998 (Pat.)
Judgement
Dr. ARIJIT PASAYAT, J. :- In these appeals challenge is to the order passed by a learned
single Judge of the Patna High Court dismissing the application filed under Section 482
of the Code of Criminal Procedure, 1973 (in short the 'Code'). The validity of the order
taking cognizance in complaint Case No. 40 of 1994 pending in the Court of Judicial
Magistrate, 1st Class, Patna City was questioned in the petition filed before the High
Court.
2. Background facts as highlighted by the appellant in a nutshell are as follows :
A raid was conducted on 4-1-1994 in the premises of respondent No. 1 by four officers of
the Bihar State Electricity Board. The appellant in each of the Criminal Appeal was a
member of the raiding party along with others and one Ravindra Kumar Singh who was
then functioning as Executive Engineer. Respondent No. 1 was arrested on the basis of
First Information Report was lodged with the police officials. Allegation against
respondent No. 1 was that he had committed theft of electricity attracting penal
consequences under Section 379 of the Indian Penal Code, 1860 (in short the 'IPC') and
Sections 39 and 44 of the Indian Electricity Act, 1910 (in short the 'Electricity Act'). The
respondent No. 1 was released on bail on 7-1-1994 and on 11-2-1994 respondent No.1
filed a complaint before the learned Judicial Magistrate alleging that the appellant in each
case and aforesaid Ravindra Kumar Singh had demanded illegal gratification.
Interestingly, no date of such demand was indicated in the complaint. When the learned
Judicial Magistrate took the cognizance of the offences punishable under Sections 347,
161, 167, 385 and 511 IPC the appellants filed petitions before the High Court
questioning correctness of the order. A specific stand was that the complaint was a
counter blast by respondent No. 1 to the action taken against him and no explanation was
offered
@page-SC2779
as to why the complaint was filed on 11-2-1994 without even indicating as to when the
alleged demand for illegal gratification was made. The said Ravindra Kumar Singh had
filed an application (Crl. Misc. No. 1973 of 1995) for quashing the prosecution. The same
was dismissed on 20-7-1995. The High Court dismissed the application filed by Ravindra
Kumar Singh while the application filed by present appellants was pending. He moved
this Court in SLP (Crl.) No. 3045 of 1996. This Court granted stay on 23-9-1996 and
therefore proceedings before the trial Court remained stayed. The special leave petition
was disposed of on 17-7-1997. Since a statement was made before this Court by the
complainant that he did not want to proceed against the petitioner in the special leave
petition but he would like to continue the proceedings so far as the others are concerned,
the proceedings were accordingly quashed qua the petitioner in the SLP.
So far as the present appellants are concerned the High Court was of the view that this
Court had interfered because of the concession made and, therefore, no case for
interference was made out. The High Court did not attach importance to the fact that the
complaint was lodged in February, 1994 and there was no date indicated as to when the
alleged demand was made.
Learned counsel for the appellant in each case submitted that the proceedings were sheer
abuse of the process of the Court. The appellants and Ravindra Kumar Singh stood on
similar footing. His case was disposed of by this Court and earlier in his case the High
Court had declined to interfere and he had moved this Court. It was submitted that in the
background facts it is clear that with a view to harass the appellants for taking action
against him, the respondent No. 1 had lodged the complaint. There is no appearance on
behalf of the respondent No. 1.
Learned counsel for respondent No.2 State supported the order of the High Court.
3. Exercise of power under Section 482 of the Code in a case of this nature is the
exception and not the rule. The section does not confer any new powers on the High
Court. It only saves the inherent power which the Court possessed before the enactment
of the Code. It envisages three circumstances under which the inherent jurisdiction may
be exercised, namely, (i) to give effect to an order under the Code, (ii) to prevent abuse of
the process of Court, and (iii) to otherwise secure the ends of justice. It is neither possible
nor desirable to lay down any inflexible rule which would govern the exercise of inherent
jurisdiction. No legislative enactment dealing with procedure can provide for all cases
that may possibly arise. The Courts, therefore, have inherent powers apart from express
provisions of law which are necessary for proper discharge of functions and duties
imposed upon them by law. That is the doctrine which finds expression in the section
which merely recognises and preserves inherent powers of the High Courts. All courts,
whether civil or criminal possess, in the absence of any express provision, as inherent in
their constitution, all such powers as are necessary to do the right and to undo a wrong in
the course of administration of justice on the principle "quando lex aliquid alicui
concedit, concedere videntur et id sine quo res ipsae esse non potest" (when the law gives
a person anything it gives him that without which it cannot exist). While exercising
powers under the section, the Court does not function as a court of appeal or revision.
Inherent jurisdiction under the section though wide has to be exercised sparingly,
carefully and with caution and only when such exercise is justified by the tests
specifically laid down in the section itself. It is to be exercised ex debito justitiae to do
real and substantial justice for the administration of which alone the courts exist.
Authority of the Court exists for advancement of justice and if any attempt is made to
abuse that authority so as to produce injustice, the Court has power to prevent abuse. It
would be an abuse of the process of the Court to allow any action which would result in
injustice and prevent promotion of justice. In exercise of the powers the Court would be
justified to quash any proceeding if it finds that initiation/continuance of it amounts to
abuse of the process of Court or quashing of these proceedings would otherwise serve the
ends of justice. When no offence is disclosed by the complaint, the Court may examine
the question of fact. When a complaint is sought to be quashed, it is permissible to look
into the materials to assess what the complainant has alleged and whether any offence is
made out even if the allegations are accepted in toto.
@page-SC2780
4

. In R. P. Kapur v. State of Punjab (1960 (3) SCR 388) this Court summarised some
categories of cases where inherent power can and should be exercised to quash the
proceedings (SCR p. 393) : AIR 1960 SC 866

(i) where it manifestly appears that there is a legal bar against the institution or
continuance e. g. want of sanction;
(ii) where the allegations in the first information report or complaint taken at its face
value and accepted in their entirety do not constitute the offence alleged;
(iii) where the allegations constitute an offence, but there is no legal evidence adduced or
the evidence adduced clearly or manifestly fails to prove the charge.
5

. In dealing with the last case, it is important to bear in mind the distinction between a
case where there is no legal evidence or where there is evidence which is clearly
inconsistent with the accusations made, and a case where there is legal evidence which,
on appreciation, may or may not support the accusations. When exercising jurisdiction
under Section 482 of the Code, the High Court would not ordinarily embark upon an
enquiry whether the evidence in question is reliable or not or whether on a reasonable
appreciation of it accusation would not be sustained. That is the function of the trial
Judge. Judicial process should not be an instrument of oppression, or, needless
harassment. The Court should be circumspect and judicious in exercising discretion and
should take all relevant facts and circumstances into consideration before issuing process,
lest it would be an instrument in the hands of a private complainant to unleash vendetta to
harass any person needlessly. At the same time the section is not an instrument handed
over to an accused to short circuit a prosecution and bring about its sudden death. The
scope of exercise of power under Section 482 of the Code and the categories of cases
where the High Court may exercise its power under it relating to cognizable offences to
prevent abuse of process of any Court or otherwise to secure the ends of justice were set
out in some detail by this Court in State of Haryana v. Bhajan Lal (1992 Supp (1) SCC
335). A note of caution was, however, added that the power should be exercised sparingly
and that too in the rarest of rare cases. The illustrative categories indicated by this Court
are as follows : (SCC pp. 378-79, para 102) AIR 1992 SC 604, Para 108

"102. (1) Where the allegations made in the first information report or the complaint,
even if they are taken at their face value and accepted in their entirety do not prima facie
constitute any offence or make out a case against the accused.
(2) Where the allegations in the first information report and other materials, if any,
accompanying the FIR do not disclose a cognizable offence, justifying an investigation
by police officers under Section 156(1) of the Code except under an order of a Magistrate
within the purview of Section 155(2) of the Code.
(3) Where the uncontroverted allegations made in the FIR or complaint and the evidence
collected in support of the same do not disclose the commission of any offence and make
out a case against the accused.
(4) Where, the allegations in the FIR do not constitute a cognizable offence but constitute
only a non-cognizable offence, no investigation is permitted by a police officer without
an order of a Magistrate as contemplated under Section 155 (2) of the Code.
(5) Where the allegations made in the FIR or complaint are so absurd and inherently
improbable on the basis of which no prudent person can ever reach a just conclusion that
there is sufficient ground for proceeding against the accused.
(6) Where there is an express legal bar engrafted in any of the provisions of the Code or
the Act concerned (under which a criminal proceeding is instituted) to the institution and
continuance of the proceedings and/or where there is a specific provision in the Code or
the Act concerned, providing efficacious redress for the grievance of the aggrieved party.
(7) Where a criminal proceeding is manifestly attended with mala fides and/or where the
proceeding is maliciously instituted with an ulterior motive for wreaking vengeance on
the accused and with a view to spite him due to private and personal grudge."
6

. As noted above, the powers possessed by the High Court under Section 482 of the Code
are very wide and the very plenitude of the power requires great caution in its exercise.
The Court must be careful to see that its decision in exercise of this power is based on
sound principles, the inherent 1993 AIR SCW 248
AIR 1964 SC 1

@page-SC2781
power should not be exercised to stifle a legitimate prosecution. The High Court being
the highest Court of a State should normally refrain from giving a prima facie decision in
a case where the entire facts are incomplete and hazy, more so when the evidence has not
been collected and produced before the Court and the issues involved, whether factual or
legal, are of magnitude and cannot be seen in their true perspective without sufficient
material. Of course, no hard and fast rule can be laid down in regard to cases in which the
High Court will exercise its extraordinary jurisdiction of quashing the proceeding at any
stage. (See Janata Dal v. H. S. Chowdhary (1992 (4) SCC 305) and Raghubir Saran (Dr.)
v. State of Bihar (1964 (2) SCR 336). It would not be proper for the High Court to
analyse the case of the complainant in the light of all probabilities in order to determine
whether a conviction would be sustainable and on such premises arrive at a conclusion
that the proceedings are to be quashed. It would be erroneous to assess the material
before it and conclude that the complaint cannot be proceeded with. In a proceeding
instituted on complaint, exercise of the inherent powers to quash the proceedings is called
for only in a case where the complaint does not disclose any offence or is frivolous,
vexatious or oppressive.
7

. The backgrounds clearly show that the proceedings instituted were mala fide, based on
vague assertions and were initiated with mala fide intents and constitute sheer abuse of
process of law. No reason was shown before the High Court as to why the complainant
chose not to proceed and one of the four persons initially named. The cases at hand fit in
with category (7) of Bhajan Lal's case (supra). AIR 1992 SC 604

8. The appeals are allowed and the proceedings in complaint case No. 40 of 1994 in the
Court of Judicial Magistrate, First Class, Patna City stand quashed.
Appeal allowed.
AIR 2008 SUPREME COURT 2781 "Monica Kumar v. State of Uttar Pradesh"
(From : Allahabad)*
Coram : 2 S. B. SINHA AND LOKESHWAR SINGH PANTA, JJ.
Criminal Appeal No. 968 of 2008 (arising out of SLP (Cri) No. 5593 of 2006), D/- 27 -5
-2008.
Dr. Monica Kumar and Anr. v. State of U.P. and Ors.
Constitution of India, Art.142 - SUPREME COURT - CRIMINAL PROCEEDINGS -
Powers to do complete justice - Quashing of criminal proceedings - Appellants while
studying M. B. B. S. Course had problems with Principal and College staff - Facing
criminal proceedings - Allegations and counter allegations by parties in various
complaints - In peculiar facts and circumstances, considering future career of appellants
who now joined medical profession, Supreme Court quashed criminal proceedings
against appellants in exercise of its powers under Art.142 - Appellants directed to file
written apology.
Though there is no provision like Section 482 of the Cr. P. C. conferring express power
on the Supreme Court to quash or set aside any criminal proceedings pending before a
criminal Court to prevent abuse of process of the Court, but the inherent power of
Supreme Court under Article 142 coupled with the plenary and residuary powers under
Articles 32 and 136 embraces power to quash criminal proceedings pending before any
Court to do complete justice in the matter before Supreme Court. If the Court is satisfied
that the proceedings in a criminal case are being utilised for oblique purposes or if the
same are continued on manufactured and false evidence or if no case is made out on the
admitted facts, it would be in the ends of justice to set aside or quash the criminal
proceedings. Once Supreme Court is satisfied that the criminal proceedings amount to
abuse of process of Court, it would quash such proceedings to ensure justice. Supreme
Court's power under Article 142(1) to do "complete justice" is entirely of different level
and of a different quality. What would be the need of "complete justice" in a cause or
matter would depend upon the facts and circumstances of each case and while exercising
that power the Court would take into consideration the express
@page-SC2782
provisions of a substantive statute. Any prohibition or restriction contained in ordinary
laws cannot act as a limitation on the constitutional power of Supreme Court. Once
Supreme Court has seisin of a cause or matter before it, it has power to issue any order or
direction to do "complete justice" in the matter. (Para 34, 35)
In the instant case, the appellants during their study of MBBS Course had some problems
with the principal; some staff of the College and the then SHO of P. S. whose daughter
was also studying in the same College. The record would reveal that both the appellants
being NRI candidates have undergone physical and mental agony and torture during their
student's career in pursuing the MBBS Course. They had lost most of their precious time
in litigation in the courts fighting for their genuine and legitimate claims. They have been
facing criminal proceedings for the past about 3 years. Looking to the entire backdrop of
the peculiar facts of countless incidents having faced by the appellants during their
primary life as MBBS students and the nature of the offences alleged against them in the
criminal cases and allegations and counter allegations in various complaints made by the
parties against each other and coupled with the tenor and contents of the apology
tendered by the appellants, and also taking into consideration the future career of the
appellants who by this time might have joined the noble medical profession, the Supreme
Court found that it is a fit case where it should exercise its jurisdiction under Art. 142, so
as to bring the dispute between the parties to an end. However, as the appellants, at the
relevant time, were students, no amount of compensation be directed to be paid. They
must, however, file a written apology in the Courts where the proceedings are pending.
(Paras 34, 37, 39)
Cases Referred : Chronological Paras
2008 AIR SCW 1003 : AIR 2008 SC 1126 : 2008 Cri LJ 1610 : 2008 (2) AIR Kar R 415
(Ref.) 22, 32
(2005) 13 SCC 540 (Ref.) 22, 31
2004 AIR SCW 6185 : AIR 2005 SC 9 : 2005 Cri LJ 92 (Ref.) 30, 31
2000 AIR SCW 2647 : AIR 2000 SC 2573 (Ref.) 36
1996 AIR SCW 15 : AIR 1996 SC 722 : 1996 Cri LJ 1127 (Ref.) 31
1996 AIR SCW 422 : AIR 1996 SC 977 : 1996 Cri LJ 1354 (Ref.) 22
1995 AIR SCW 1448 : 1995 Cri LJ 2107 (Ref.) 36
1995 AIR SCW 3488 : AIR 1995 SC 2348 : 1995 Cri LJ 3994 (Ref.) 36
1993 AIR SCW 248 : AIR 1993 SC 892 : 1993 Cri LJ 608 (Ref.) 30
1992 AIR SCW 1444 : AIR 1992 SC 1379 : 1992 Cri LJ 1956 (Ref.) 25
AIR 1992 SC 604 : 1992 Cri LJ 527 (Ref.) 20, 25, 29
AIR 1980 SC 326 : 1980 Cri LJ 98 (Ref.) 31
AIR 1977 SC 2229 : 1977 Cri LJ 1900 (Ref.) 30
AIR 1964 SC 1 : 1964 (1) Cri LJ 1 (Ref.) 30
AIR 1960 SC 866 : 1960 Cri LJ 1239 (Ref.) 28
Harish N. Salve, Uday U. Lalit, Sr. Advocates, K. Pavan Kumar and Siddharatha
Chowdhury, with them for Appellant; A. Sharan, A. S. G., C. S. Vaidyanathan, P. P. Rao,
Sr. Advocates, Amit Pawan, B. K. Prasad, P. Parmeswaran, K. K. Venugopal, Ms. Liz
Mathew, S. Chandra Shekhar, Manoj N. George, Pramod Dayal, Nikunj Dayal, Shail
Kumar Dwivedi, Ms. Vandana Mishra, Ms. Vibha Dwlvedi, Kamalendra Mishra and C.
D. Singh, with them for Respondents.
* Cri. Misc. Appln. Nos. 7791 and 7792 of 2006, D/-24-08-2006, reported in 2006 (3) All
Cri R 2760 : 2006 (6) ALJ (NOC) 1217.
Judgement
LOKESHWAR SINGH PANTA, J. :- Leave granted.
2. Challenge in this appeal is to the final judgment and order dated 24-08-2006 passed by
the High Court of Judicature at Allahabad whereby and whereunder the High Court has
dismissed Criminal Miscellaneous Applications bearing Nos. 7792 of 2006 and 7791 of
2006 filed by the appellants under Section 482 of the Code of Criminal Procedure [for
short 'Cr.P.C.'] in Case Crime No. 412 of 2005 under Sections 452, 323, 504, 506 and 427
of the Indian Penal Code [for short 'the IPC'] and in Case Crime No. 21 of 2006 under
Sections 452, 323, 336, 504, 506, 420 IPC respectively registered against them at Police
Station. Vijay Nagar; District Ghaziabad and seeking for entrustment of further
investigation of the aforesaid cases to the Central Bureau of Investigation [for short 'the
CBI'].
3. This case would reveal a chequered history of legal battle being fought by the
appellants-the students of Santosh Medical College on one hand and the authorities of the
College on the other hand.
4. Dr. Narendra Kumar, the father of the
@page-SC2783
appellants, is presently working as Professor/Medical Director of Neonatal Intensive Care
Unit [NICU] and also performing medical practice at 2917, Middleboro Place, Modesto,
California. Both the appellants were born in California and completed their schooling in
USA. They decided to get admission in MBBS course for the academic session 1996-97
in Santosh Medical College, Ghaziabad [for short 'College'] against NRI quota after
remitting US $50,000 and US $49,700 respectively towards capitation fees and additional
hostel fees of Rs. 75,000 and Rs. 45,000 and security deposits for one year. That apart,
the College took a loan of Rs. 25 lakhs on Interest @ 11.5% p.a. from the father of the
appellants and its payment was assured by a handwritten slip. The disputes and
differences arose after the father of the appellants demanded repayment of the loan from
Dr. P. Mahalingam - the second respondent herein, Chairman and Managing
Director/Trustee of Maharaji Educational Trust and Santosh Medical College and
Hospital, Pratap Vihar, Vijay Nagar, Ghaziabad. In April 2001, the matter was reported to
the Additional District Magistrate, Ghaziabad, for taking appropriate steps to get the loan
amount refunded. Dr. P. Mahalingarn the second respondent in his letter dated 9-4-2001
acknowledged the liability and had also assured to refund the entire loan amount. It was
alleged that the second respondent with vindictive attitude started harassing the
appellants and in the result declared in July 2000, Dr. Monica Kumar the first appellant
was got failed in both theory papers of Pharmacology and she was not allowed to appear
in two subsequent supplementary examinations as well as in Final Professional MBBS
Part-I Examination.
5. The first appellant filed a Writ Petition No. 9150 of 2001 in the High Court wherein
vide order dated 14-3-2001, the second respondent was directed to permit the first
appellant to appear in the final Professional MBBS Part-I Examination. In compliance of
the High Courts order, the first appellant was permitted to appear in the examination, but
her result was deliberately withheld for an oblique motive which compelled the first
appellant to approach the High Court of Allahabad by way of Miscellaneous Application
in the pending Writ Petition No. 9150/ 2001 for issuing necessary directions for
declaration of her result. The High Court vide order dated 19-9-2001 directed the College
authorities to declare the results of MBBS Final Professional Part-I Examination, 2001
and the result of the scrutiny of Pharmacology of Second Professional Examination, 2000
and further to permit the first appellant to appear in Final MBBS Part-II Examination and
to declare the result of the said examination as well. By Orders dated 7-01-2002/16-01-
2002, the High Court directed the college authorities to produce answer books of
Pharmacology of the first appellant. On 4-03-2002, the High Court got the answer books
of the first appellant re-examined by the Head of Department of Pharmacology of Motilal
Nehru Medical College, Allahabad in the court itself. On re-examination of the papers,
the first appellant secured good marks in both the papers and accordingly, the college
authorities were directed to declare her results forthwith. It appears that the
orders/directions of the High Court were not complied with which gave rise to the first
appellant to file contempt of court proceedings against the college authorities. The High
Court vide order dated 9-4-2002 directed the College authorities to be present personally
in the Court but in the meantime on 22-04-2002 the result was declared and for no valid
reasons, the first appellant was declared failed in Surgery Practical Examination. The first
appellant left with no other remedy, but to approach the High Court by means of another
writ petition. The High Court directed the second respondent to produce before it the
tabulation chart of Surgery Practical Examination of all the students including the first
appellant. The High Court on 12-11-2002 having noticed serious allegations of mala fide,
restrained Dr. P. Mahalingam the second respondent from interfering in and conducting
examination of the first appellant and further directed that the practical examination of
the first appellant be got conducted through Agra Medical College in which the first
appellant was declared pass with 70% marks.
6. The appellants alleged that having miserably failed in all attempts to ruin the career of
the first appellant, Dr. Mahalingam - the second respondent on 04-04-2003 got a false and
frivolous report lodged under Sections 504 and 506 IPC through his yes-man and
associate - Dr. Anil Tomar against all the members of the appellants' family whereupon
Case Crime No. 286 of 2003 was
@page-SC2784
registered against them. Both the appellants and their parents filed Writ Petition No. 1923
of 2003 seeking for quashing the said criminal case and the High Court vide order dated
17-4-2003, stayed the arrest of the appellants and their parents during the investigation of
the above-said FIR. The Police rushed to file charge-sheet without making any fair and
effective investigation against which Criminal Miscellaneous Application No. 8542 of
2003 under Section 482 Cr.P.C. was filed by the parents of the appellants in the High
Court for quashing the charge-sheet and the High Court vide its order stayed further
proceedings pending before the trial Court.
7. On a complaint made by the father of the appellants and on intervention of the District
Magistrate and S.S.P., Ghaziabad, Dr. P. Mahalingam the second respondent on 28-02-
2003 allegedly, gave 5 cheques for Rs. 5 lakhs each against the loan amount and two
demand drafts of Rs. 2.5 lacs each on account of payment of the accrued interest. It was
stated that one cheque was dishonoured on 18-10-2003 and the father of appellants
preferred a Criminal Case No. 7272 of 2003 against Dr. Mahalingam - the second
respondent under Section 138 of the Negotiable Instruments Act in the Court of
Additional Chief Judicial Magistrate, Ghaziabad wherein by order dated 24-1-2004 the
second respondent and others were summoned as accused persons. On filing of the above
criminal case by the father of the appellants, the second respondent got infuriated and
became more and more vindictive against the appellants and their family members.
8. The appellants then stated that the taperecorded conversation held between the father
of the appellants and Dr. M.K. Shrivastava, Principal of the College, would clearly reveal
that Dr. P. Mahalingam - the second respondent is the main person instrumental in
victimisation and harassing of the appellants. On 2-5-2004 and 2-6-2004 the appellants
were allegedly assaulted mercilessly by the second respondent, Anil Somania, Station
Officer, P.S. Vijay Nagar and their drivers. The first appellant was molested and she had
been threatened to be kidnapped, raped and even murdered whereas Dr. Manish Kumar-
the second appellant, brother of the first appellant was assaulted with kicks, fists, shoes
and sticks. They got themselves medically examined at the Government M.M.G.
Hospital, Ghaziabad and on refusal to register their FIR by the Police of Police Station,
Ghaziabad, the appellants proceeded to file an application under Section 156(3) Cr.P.C.
before Chief Judicial Magistrate, Ghaziabad seeking direction to the police to register the
FIR and hold proper investigation in the case. Though the said application was initially
rejected by the Chief Judicial Magistrate, but in view of the order of the IIIrd Additional
District and Sessions Judge, Ghaziabad, the Chief Judicial Magistrate by order dated 3-
10-2005 directed the concerned Police Station Officer to register the case against the
culprits.
9. Aggrieved thereby, Dr. P. Mahalingam - the second respondent filed a Criminal Writ
Petition before the High Court which was dismissed vide order dated 9-11-2005. In
pursuance of the order of the Chief Judicial Magistrate dated 03-10-2005 and subsequent
order of the High Court dated 9-11-2005, FIR at the instance of Dr. Monica bearing
Crime No. 425 of 2005 was registered on 28-11-2005 under Sections 147/
323/342/352/354/427/504 and 506 IPC at the Police Station against Dr. P. Mahalingam
and other persons named in the complaint.
10. The appellants stated that as citizens of the United States of America, they sent a
representation to the President of USA whereupon White House responded and sent a
letter dated August 16th 2004 informing the appellants that White House had decided to
send the petition to the Department of State to address the grievances of the appellants.
Further, by letter dated August 30, 2004 the appellants were also informed about the steps
having been taken by the USA.
11. The appellants also stated that even on issuance of satisfactory completion certificate
of internship to the first appellant duly signed by all the Professors and Heads of
Departments, Medical Officers and Dean of Faculty on 18/19-3-2004, the Principal of the
College who was simply required to countersign internship completion certificate,
deliberately for no valid reason entered the word 'unsatisfactory' by ante-dating it as 16-
01-2004 at the behest of Dr. P. Mahalingam - the second respondent as a result thereof the
first appellant could not get the MBBS Degree for getting herself enrolled with Medical
Council of India nor she
@page-SC2785
could appear in any Post Graduation Examination. Aggrieved by the action of the
Principal of the College, the first appellant preferred Civil Writ Petition No. 19069 of
2004 in the High Court of Allahabad and the High Court vide its order dated 11.01.2005,
recorded that the certificate issued by the competent authorities was deliberately
antedated. The learned single Judge of the High Court by order dated 17.2.2005 disposed
of the said writ petition as counsel for Dr. P. Mahalingam - the second respondent
produced a fresh certificate reporting therein that the first appellant had completed her
internship satisfactorily and therefore was eligible for MBBS Degree. The first appellant
being aggrieved against the order by which other reliefs prayed for in the writ petition
were declined, filed a Special Appeal in the High Court which was allowed on 31.3.2005,
directing the authorities concerned to issue other required certificates, i.e. character
certificate, pass certificate and attempt certificate to the first appellant.
12. In compliance to the order of the High Court Dr. P. Mahalingam - the second
respondent issued character certificate, pass certificate and attempt certificate to the first
appellant but with wrong dates and incomplete particulars. The first appellant again was
forced to file Contempt Petition No. 4057 of 2005 against the second respondent praying
for taking legal proceedings against him for violation of the court's order. The High Court
on 23.12.2005 recorded the following order :-
"This court without going into the controversy is not issuing any notice on the contempt
application at this stage and disposes of this application with a direction to the opposite
party to consider the request made by the applicant in her representation within three
weeks from the date of the production of a certified copy of this order. If the grievance of
the applicant is found to be genuine, in that event, fresh certificates shall be issued
immediately."
13. The appellants stated that Dr. P. Mahalingam - the second respondent found a good
ally in Anil Somania the then Station Officer of P.S. Vijay Nagar whose daughter was
also studying in the same College and thus was able to intensify the harassment of the
appellants and got initiated criminal proceedings against them under Sections 107/116
Cr.P.C. This time again on being approached by the appellants, the High Court by order
dated 25.11.2005 stayed those proceedings.
14. Having failed in all earlier attempts to harm the careers of the appellants, the second
respondent allegedly in collusion with Anil Somani, SHO, instigated Dr. I.M. Sharma,
Warden of Girls Hostel of the College the third respondent herein and got a false and
frivolous FIR No. 297/2005 (Case Crime No. 412/2005) dated 5.10.2005 registered
against the appellants under Section 452/323/504 and 506 IPC at P.S. Vijay Nagar. The
appellants filed Criminal Miscellaneous Petition No. 11192 of 2005 in the High Court
and the High Court on 7.11.2005 passed the following order :-
"Having regard to the facts and circumstances of the case, the arrest of the petitioners for
the offences indicate above is stayed till the submission of the report on the following
conditions :-
1. That the petitioners will not be arrested in respect of the said crime number during the
pendency of the investigation provided they co-operate with the investigation.
2. The stay of arrest will operate only if certified copy of this order along with one self
attested copy of the writ petition is served upon the investigation officer within fifteen
days from today.
3. The stay of arrest will cease to operate if it is decided to submit a charge-sheet after
investigation.
4. Because the complainant has not been head at this stage, therefore, it will be open to
the complainant or the investigation officer who has not been given opportunity to file
counter affidavit or any other party aggrieved to apply in this writ petition for
recall/modification of this order, if any misstatement is found in the material facts stated
in the writ petition or other legally valid ground which may be available to the party so
applying.
5. The investigating officer will make all possible efforts to conclude the investigation
within three months of the date on which a certified copy of this order is served upon
him. The SSP Ghaziabad is directed to hand over investigation of this case to a Gazetted
Officer not below the rank of Deputy Superintendent of Police."
15. Leaving no stone unturned to fulfil his vengeance and revengeful attitude against the
appellants, the second respondent got
@page-SC2786
one more frivolous FIR bearing Crime No. 21 of 2006 dated 14.1.2006 registered against
them under Sections 452/323/336/ 504/506 and 427 IPC at P.S. Vijay Nagar at the behest
of Rajendra Kuntal - Head Security Guard of Dr. P. Mahalingam. The complaint of
Rajendra Kuntal was sent through Ram Murti Mani Kandan, Personal Manager of the
second respondent, to the Police Station. The appellants were arrested on 15.01.2006
from their house and lodged in jail. They were released on bail by the learned Sessions
Judge on 31.01.2006.
16. Owing to constant threats and victimisation by Dr. P. Mahalingam in collusion with
the local police, the appellants filed Miscellaneous Writ Petition No. 1947 of 2006
praying for CBI investigation into the matter. On 22.07.2006, the High Court passed the
following order :-
"Considering the facts and circumstances of the case, we are of the view that the correct
position of the investigation of the cases be also brought on record. Therefore, learned
AGA is directed to file counter affidavit annexing the copies of the statement of the
prosecution witnesses recorded under Section 161 Cr.P.C. in the cases referred to above.
The case be listed on 27.3.2006.
The Senior Superintendent of Police, Ghaziabad shall look into the grievances of the
petitioners regarding the ill-treatment/ humiliation/harassment etc. by the local police and
the respondents/accused concerned and ensure adequate security to them and their family
members in all respects and submit the compliance report on or before 10th March, 2006.
Let a copy of this order be furnished to the learned AGA free of cost by tomorrow for
intimating the authority concerned."
17. The appellants alleged that Sub-Inspector J.K. Gangwar ought not to have conducted
the investigation of the cases, as in the earlier proceedings, the High Court made
observations that he was under the influence of SHO Anil Somani. S.I. Gangwar without
proper and fair investigation hurriedly prepared and filed charge-sheet in the trial court on
the basis of which the learned Chief Judicial Magistrate proceeded to take cognizance of
the offences against the appellants.
18. The appellants then preferred two separate petitions referred to above under Section
482, Cr.P.C. for quashing of the said FIRs and entrusting further investigation of the cases
to CBI. The High Court by impugned order dated 24.08.2006 dismissed the petition.
Hence, the appellants are before us in this joint appeal by way of special leave.
19. We have heard the learned counsel for the parties and meticulously examined the
entire material on record.
20. Shri Harish N. Salve, learned senior counsel appearing for the appellants, assailed the
judgment of the High Court inter alia contending that the First Information Reports and
further proceedings initiated thereto by the trial court against the appellants are vitiated
on the following grounds :-
(1) that the allegations made in the FIRs and evidence collected during investigation on
their face are so absurd and Inherently improbable that no prudent person can ever arrive
at a conclusion that there are sufficient grounds for proceeding against the appellants-
students of MBBS course for the commission of the alleged offences registered at the
instance of the employees of the College;

(ii) that the appellants' case is fully covered by the principles laid down by this Court in
the case of State of Haryana v. Bhajanlal (1992) Suppl (1) SCC 335 and the High Court
has misapplied the ratio of the said case in the facts of the present case; AIR 1992 SC
604

(iii) that the High Court has failed to appreciate that there was an apparent nexus between
Dr. P. Mahalingam - the second respondent and two informers, who in collusion with the
local police, have launched two false and vexatious criminal cases against the appellants
in continuation of series of acts of victimization and harassment first to spoil their future
career;
(iv) the High Court has failed to appreciate that the material on record leaves no room of
doubt that the Criminal Cases were initiated at the instance of Dr. P. Mahalingam - the
second respondent due to mala fide intention, vengeance and animosity in continuation of
his designs to misappropriate the loan amount advanced to him by the father of the
appellant for establishment of the College;
(v) that the High Court has failed to appreciate that Dr. P. Mahalingam who happens to be
the Chairman and Sole Trustee of the Maharaj Ji Educational Trust which
@page-SC2787
is running the Santosh Medical College for imparting medical education at the Under
Graduate Level, which is one of the noblest professional qualification that one can impart
on human beings, has been indulging in acts of omissions and commissions which are
wholly unexpected of him. After having failed in all his repeated attempts to spoil the
career of the appellants, the second respondent adopted a vindictive attitude towards the
appellants and the facts of the present case clearly establish that the allegations made in
the complaints are not only false but are the result of mala fides of Dr. P. Mahalingam.
21. In opposition, Mr. Amarendra Sharan, learned Additional Solicitor General appearing
on behalf of CBI the fourth respondent, urged before us that acceptability of mala fides
against second respondent is a matter of trial and that it is not a case where charge-sheet
prima facie does not disclose commission of offences and that the defence pleaded by the
appellants, is in fact, has to be considered by the courts below during the trial of the cases
pending against them.
22

. Shri Shall Kumar Dwivedi, learned counsel appearing on behalf of the State of U.P.,
supported the contention of the learned counsel for CBI and submitted that since the
investigation of the cases having been completed, charge-sheets filed and charges framed
by the trial court against the appellants, therefore, now the trial is completely in the
domain of the trial court and certainly it is not a proper stage of quashing the FIRs and
charge-sheets filed under Section 173, Cr.P.C. In support, reliance is placed on Som
Mittal v. Government of Karna-taka (2008) 3 SCC 753; State of H.P. v. Prithi Chand
(1996) 2 SCC 37 and State of Orissa and Anr. v. Saroj Kumar Sahoo (2005) 13 SCC 540,
to contend that the inherent power of the High Court for quashing criminal proceedings
should be exercised very sparingly and with circumspection and that too in the rarest of
rare cases and that the present case does not fall in that category. 2008 AIR SCW 1003

23. Shri K.K. Venugopal, learned senior counsel appearing on behalf of Dr. P.
Mahalingam - the second respondent, vehemently contended that the appellants have
concocted contradictory stories in different proceedings regarding their alleged
harassment by the College authorities inasmuch as they initially took the plea that they
were allegedly being harassed by Dr. M.K. Srivastava - Principal of the College because
he wanted the second appellant to marry his daughter but in subsequent proceedings the
appellants changed their earlier stand and took up another false plea that they were being
harassed at the behest of the second respondent from whom their father demanded the
return of loan amount of Rs. 25 lakhs. He submitted that it was proved on record that loan
amount of Rs. 25 lakhs has already been paid to the father of the appellants by way of
cheques and bank drafts way back in the year 2003 itself. Mr. Venugopal has brought to
our notice the relevant paragraphs of counter-affidavit filed by the second respondent in
opposition to the present appeal in which the allegations of the appellants that they were
intentionally got failed in the examinations at the instance of the second respondent has
been categorically denied. The second respondent stated that the first appellant had failed
repeatedly due to her poor performance in the examination and definitely not due to any
amount of harassment or acts of victimization by him as alleged by the first appellant,
whereas the second appellant could successfully complete his MBBS Course and,
accordingly, all certificates like Internship Certificate, Passing Certificate and Character
Certificate were issued to him on successful completion of the course. He submitted that
the record of the College would reveal that despite a series of complaints regarding the
act of indiscipline of the first appellant, he always took a lenient view so that she should
not suffer in her studies and he has always extended full support to every student
including the first appellant. He submitted that the FIRs were registered against the
appellants by the employees of the College in their personal capacity for the commission
of the alleged offences by them and their allegations that the said cases were registered at
his behest, are absolutely false, baseless and unfounded. He then submitted that the High
Court in exercise of its jurisdiction under Section 482, Cr.P.C. has found prima facie case
against the appellants and recorded well-reasoned order which is based upon proper
appreciation of the settled proposition of law, this Court, therefore, shall restrain itself
from interfering with the impugned order of the High Court in exercise of jurisdiction
under Article 136 of the Constitution of India.
@page-SC2788
24. Shri P.P. Rao, learned senior counsel appearing for Dr. M.K. Shrivastava - Principal
of the College and Dr. I.M. Sharma - Warden of Girls Hostel (an informant of Case Crime
No. 412 of 2005), contended that not only the police found prima facie case but the court
below also found sufficient material against the appellants on the basis of which
cognizance of the offences alleged against them in Case Crime No. 412 of 2005 was
taken and the trial court now has framed charges against the appellants. He next
contended that the mere fact that senior police officer's daughter was a student of the
respondent-College by itself would not lead to the conclusion that investigation of the
cases registered against the appellants was tainted or not being conducted properly and
fairly by the Investigation Officer. He supported the order of the High Court which,
according to him, is valid and legal both on facts and law.
25. Having heard the learned counsel for the parties and having noticed and considered
the proposition of law laid down by this Court in a number of decisions, the learned
Single Judge of the High Court observed :
"The contents of the report registered as Case Crime No. 412 of 2005 under Sections,
452, 323, 504, 506, 427, IPC at Police Station Vijay Nagar, Ghaziabad, transpires that on
5.10.2005 at about 6 p.m. Dr. Monica Kumar and Dr. Manish Kumar entered in the house
of Dr. Indra Mohinl Sharma, H. No. 14, Sector-12, Pratap Vihar, Ghaziabad with knife
and brick bats. They started hurling abuses to her saying that she is much close to Dr. P.
Mahalingam. She was also slapped and was also threatened that her children would be
kidnapped and killed. On her cries, security men namely, Rajveer, Prempal and some of
the students of the College, came for her rescue. Some of the household goods were also
damaged by them. The victim (Dr. Indra Mohini Sharma) who is teacher in Santosh
Medical College under Section 161 of the Code supported the F.I.R. version and
mentioned that both the accused threatened and slapped her. She was rescued by the
security men Rajveer and Prem Pal. Police also recorded statement of these two security
men also of Gaurav Pandey, student of the College who reiterated about the incident. For
the other incident dated 14.1.2006 report was lodged as Case Crime No.21 of 2006 under
Sections 452, 323, 336, 504, 506, 420, IPC at Police Station Vijay Nagar, Ghaziabad,
against Dr. Monica Kumar and Dr. Manish Kumar as they are said to have beaten the
security man Rajendra Kuntal and also damaged the College properties. The Investigating
Officer has recorded the statement of Rajendra Kuntal and other security personnel
namely Prempal and Manoj Kumar. Both the witnesses have supported the FIR version.

It may be mentioned that in exercise of the proceedings under Section 482 of the Code,
this Court has to prima facie ascertain about the existence of the sufficient ground for
proceeding against the accused. For limited purpose the court can evaluate the material
and documents on record but it cannot appreciate the evidence so as to access the
credibility of the statement of the witnesses recorded in the course of investigation.
Further it is not required to appreciate the evidence to find out whether the materials
produced are sufficient or not for convicting the accused. In the case of Chand Dhawan v.
Jawahar Lal (1992) 3 SCC 317, it was observed by the Apex Court that when the material
relied upon by a party are required to be proved, no inference can be drawn on the basis
of materials to conclude the FIR/complaint version to be unacceptable. The scope of
exercise of the power under Section 482 of the Code and categories of the cases where
High Court may exercise its power under it relating to cognizable offences to prevent the
abuse of the process of court or otherwise to secure the ends of justice were set in detail
by the Apex Court in the case of [State of Haryana v. Ch. Bhajan Lal 1992 Suppl (I) SCC
335] they have been enumerated as under :- AIR 1992 SC 604

(1) Where the allegations made in the first information report or the complaint, even if
they are taken at their face value and accepted in their entirety do not prima facie
constitute any offence or make out a case against the accused.
(2) Where the allegations in the first information report and other materials, if any,
accompanying the FIR do not disclose a cognizable offence, justifying an investigation
by police officers under Section 156(1) of the Code except under an order of a Magistrate
within the purview of Section 155(2) of the Code.
(3) Where the uncontroverted allegations made in the FIR or complaint and the evidence
@page-SC2789
collected in support of the same do not disclose the commission of any offence and make
out a case against the accused.
(4) Where, the allegations in the FIR do not constitute a cognizable offence but constitute
only a non-cognizable offence, no investigation is permitted by a police officer without
an order of a Magistrate as contemplated under Section 155(2) of the Code.

(5) Where the allegations made in the FIR or complaint are so absurd and inherently
improbable on the basis of which no prudent person can ever reach a just conclusion that
there is sufficient ground for proceeding against the accused.
(6) Where there is an express legal bar engrafted in any of the provisions of the Code or
the concerned Act (under which a criminal proceeding is instituted) to the institution and
continuance of the proceedings and/or where there is a specific provision in the Code or
the concerned Act, providing efficacious redress for the grievance of the aggrieved party.
(7) Where a criminal proceeding is manifestly attended with mala fide and/or where the
proceeding is maliciously instituted with an ulterior motive for wreaking vengeance on
the accused and with a view to spite him due to private and personal grudge.
Here allegations made in the report and the evidence so collected in the course of
investigation construe a cognizable offence, it would not fall in any category of the case
enumerated above, call for the exercise of extraordinary powers or inherent power
quashing the charge-sheet submitted in the above-noted cases."
26. The special leave petition came up before this Court on 20.11.2006 on which date it
was ordered :
"Issue notice.
On an oral prayer made by the learned counsel, issue notice to Union of India confined to
the question as to whether the investigation be done by the C.B.I, in the event the Court
finds it necessary to do so.
The Court below may frame charges wherefor the petitioners shall make them available
on the next date fixed. Thereafter, further proceedings shall remain stayed.
Four weeks' time is granted for filing counter-affidavit. Two weeks time, thereafter, is
granted for filing rejoinder."
27. The parties have exchanged their counter affidavits and rejoinders. Indisputably, there
is no quarrel with the well-settled principles of law that while exercising powers under
Section 482, Cr.P.C., the High Court does not function as a court of appeal or revision.
Inherent jurisdiction under the Section though has to be exercised sparingly, carefully and
with caution and only when such exercise is justified by the tests specifically laid in the
Section itself. It is to be exercised ex debito justitiae to do real and substantial justice for
the administration of which courts exist. When the complaint is sought to be quashed it is
permissible to look into the materials to assess what the complainant has alleged and
whether any offence is made out even if the allegation are accepted in toto.
28

. In R.P. Kapur v. State of Punjab (1960) 3 SCR 388, this Court summarises some
categories of cases in which inherent power can and should be exercised to quash the
proceedings :- AIR 1960 SC 866

(i) Where it manifestly appears that there is a legal bar against the institution or
continuance e.g. want of sanction;
(ii) Where the allegations in the first information report or complaint taken at their face
value and accepted in their entirety do not constitute the offence alleged.
(iii) Where the allegations constitute an offence, but there is no legal evidence adduced or
the evidence adduced clearly or manifestly fails to prove the charge.
29

. The scope of exercise of power under Section 482, Cr.P.C. and the categories of cases
where the High Court may exercise its power under it relating to cognizable offences to
prevent abuse of process of any court or otherwise to secure the ends of justice were set
out in some detail by this Court which has been dealt with by the High Court in State of
Haryana v. Bhajan Lal (1992) 2 Suppl (1) SCC 335. In the said case, a note of caution to
the effect was, however, added that the power should be exercised very sparingly and
with circumspection and that too in the rarest of rare cases. The illustrative categories
indicated by this Court are earlier extracted in the order of the High Court. AIR 1992 SC
604
30

. We may reiterate and emphasise that the powers possessed by the High Court under
Section 482, Cr.P.C. are very wide and the very plenitude of the power requires great
caution in its exercise. Court must be careful 1993 AIR SCW 248
AIR 1964 SC 1
AIR 1977 SC 2229
2004 AIR SCW 6185

@page-SC2790
to see that its decision in exercise of this power is based on sound principles. The inherent
power should not be exercised to stifle a legitimate prosecution. The High Court being
the highest court of a State should normally refrain from giving a prima facie decision in
a case where the entire facts are incomplete and hazy, moreso when the evidence has not
been collected and produced before the Court and the issues involved, whether factual or
legal, are of magnitude and cannot be seen in their true perspective without sufficient
material. Of course, no hard and fast rule can be laid down in regard to cases in which the
High Court will exercise its jurisdiction of quashing the proceeding at any stage. [See
Janata Dal v. H.S. Chowdhury (1992) 4 SCC 305; Raghubir Saran Dr. v. State of Bihar
1964 (2) SCR 336; Kurukshetra University v. State of Haryana (1977) 4 SCC 451; and
Zhandu Pharmaceuticals Works Limited and others v. Mohd. Sharaful Haque and another
2005 (1) SCC 122).
31

. In fact, the question of mala fides in a case like the present is not at all relevant. If the
complaint which is made is correct and offence has been committed which will have to be
established in a court of law, it is of no significance that the complainant is a person who
is inimical or that he is guilty of mala fides. If the ingredients which establish the
commission of the offence or misconduct exist then, the prosecution cannot fail merely
because there was an animus of the complainant or the prosecution against the accused.
Allegations of mala fides may be relevant while judging the correctness of the allegations
or while examining the evidence. But the mere fact that the complainant is guilty of mala
fides, would be no ground for quashing the proceedings. [See State of Maharashtra v.
Ishwar Piraji Kalpatri (1996) 1 SCC 542; Zhandu Pharmaceuticals Works Limited and
others v. Mohd. Sharaful Haque and another, 2005 (1) SCC 122; State of Bihar and Anr.
v. J.A.C. Saldanha (1980) 1 SCC 544; State of Orissa v. Saroj Kumar Sahoo, 2005 (13)
SCC 540]. There may be some exceptions to the said rule but we are not concerned with
such a case. 1996 AIR SCW 15
2004 AIR SCW 6185
AIR 1980 SC 326

32

. This Court in the latest decision has held that where investigation was completed,
charge-sheet had been filed and charges are framed, the High Court should not ordinarily
embark upon an enquiry as to the reliability of offences to sustain the allegations made in
the complaint which is the function of the trial court, [see Som Mittal v. Government of
Karnataka (2008) 2 SCC 753] 2008 AIR SCW 1003

33. Having given our careful consideration to the submissions made by the learned
counsel for the parties and in the backdrop of the facts and in the light of principles of
law highlighted above, we have examined the entire material placed on record by the
parties in the case on hand. Indisputably, both the appellants took admission in the MBBS
course in the session 1996-97 in Santosh Medical College and Hospital under the NRI
quota against handsome payments of US $50,000 and US$ 49, 700 each in addition to
hostel charges and security deposit, i.e. Rs. 75,000/- and Rs. 45,000/-respectively for one
year. In April 2000, Dr. P. Mahalingam the second respondent, Chairman/Managing
Director of the Medical College took a loan of Rs. 25 lakhs on interest at the rate of 11.5
% from Dr. Narendra Kumar, father of the appellants. As noticed in the earlier part of this
judgment, a series of civil writ petitions and criminal proceedings besides contempt
proceedings were initiated by the appellants in which allegations of mala fides, acts of
victimization and physical and mental harassment were alleged against the second
respondent in his personal capacity and also as a Chairman/Managing Director of the
College Trust. The appellants filed application under Section 156(3), Cr.P.C. before the
Chief Judicial Magistrate on the basis of which case under Section 347/502/506/
342/352/354 and 427, IPC has been registered against the defaulters. The second
respondent is said to have challenged the order of the Magistrate but he could not
succeed. The appellants are facing trial of Case Crime No. 412/2005 lodged against them
by Dr. Indra Mohini Sharma, third respondent, under Sections 458/323/504/506 IPC and
Case Crime No. 21/2006 registered on 14.01.2006 in Police Station Vijay Nagar at the
behest of Rajender Kuntal-respondent, Security Guard of the institution, under Sections
452/323/336/504/506 and 427, IPC. The above-stated cases pertained to the period when
the appellants were students and studying MBBS Course in the College.
@page-SC2791
The trial of the said cases at present is at initial stage and further proceedings thereof are
stayed by this Court. The record would reveal that during the investigation of Case Crime
No. 412/2005 for offences punishable under Sections 453, 323, 504, 506, IPC, the
appellants were arrested by the police on 15.01.2006 from their house and were lodged in
jail. Their bail applications were adjourned four times by the learned Magistrate, who had
called for case diary and medical reports, which the prosecution did not produce. It was
only on the adjourned date, i.e. 18.01.2006 when written medical report of doctor from
Batra Hospital, New Delhi, was filed before the learned Magistrate reporting some
injuries have received by complainant Rajender Kuntal and on the basis of the said
medical report, Section 308, IPC came to be added in the said Crime Case. The learned
Magistrate rejected the bail application of the second appellant in that case. The second
appellant filed a bail application before the learned Sessions Judge on 21.01.2006 which
was adjourned to 31.01.2006 on that date the second appellant could be released on bail.
The first respondent-State of U.P. has filed with their affidavit translated true copies of
apology letters dated 13.04.2004 and 02.05.2004 respectively said to have been written
by the appellants and addressed to the SHO, Vijay Nagar P.S., Ghaziabad, the contents
whereof read as under :-
"Tomorrow morning 9 a.m., myself and my daughter Monica Kumar and Manish Kumar
will go to SP City office. So we won't go to the Mess of Santosh Medical College and we
won't abuse any employee. We apologize for the quarrel happened today evening in the
Mess with Mr. Krishnamoorthy.
Sd/-
Monica Kumar
Sd/
Manish Kumar
Sd/-
Savitri
K-8 Sector 12,
Pratap Vihar
Dt. 13.04.04
Sd/-
G. Krishnamoorthy
Dt. 13.04.04
Sd/-
R. Manikandan
Dt. 13.04.04"
"SO, Vijay Nagar Police Station, Ghaziabad.
Ref.
From today onwards we won't stand in front of the police car. Neither open our lights.
I am sorry for misconduct today.
Sd/-
Monica Kumar
Dt. 2-5-2004
Sd/-
Manish Kumar
Dt. 2-5-2004"
34. The entire details of the facts of the present case do indicate that the appellants during
their study of MBBS Course had some problems with the second respondent; some staff
of the College and the then SHO of P. S. Vijay Nagar, whose daughter was also studying
in the same College. The record would reveal that both the appellants being NRI
candidates have undergone physical and mental agony and torture during their students'
career in pursuing the MBBS course. They had spent most of their precious time in
litigation in the courts fighting for their genuine and legitimate claims. They may be
lacking in some indiscipline activities in the College for which they have been facing
criminal proceedings for the past about 3 years. Looking to the entire backdrop of the
peculiar facts of countless incidents having faced by the appellants during their primary
life as MBBS students and the nature of the offences alleged against them in the above-
mentioned crime cases lodged by Mrs. Indra Mohini Sharma and Rajender Kuntal in
Police Station Vijay Nagar, Ghaziabad and allegations and counter-allegations in various
complaints made by the parties against each other and coupled with the tenor and
contents of the apology tendered by the appellants, we are of the view that it is a fit case
where we should exercise our jurisdiction under Article 142 of the Constitution of India.
We are conscious of the well-settled law laid down by this Court in the above-referred
decisions and many more that in case of persons against whom prima facie case is made
out and charge-sheet is filed in the competent court, it is that court which will then deal
with the case on merits in accordance with law and the High Court should not except in
extraordinary circumstances exercise its jurisdiction under Section 482, Cr.P.C. so as to
quash the prosecution proceedings after they have been lodged.
@page-SC2792
35. Under Article 142 of the Constitution this Court in exercise of its jurisdiction may
pass such decree or make such order as is necessary for doing complete justice in any
'cause' or 'matter' pending before it. The expression "cause" or "matter" would include
any proceeding pending in court and it would cover almost every kind of proceeding in
court including civil or criminal. Though there is no provision like Section 482 of the
Criminal Procedure Code conferring express power on the Supreme Court to quash or set
aside any criminal proceedings pending before a criminal court to prevent abuse of
process of the court, but the inherent power of this Court under Article 142 coupled with
the plenary and residuary powers under Articles 32 and 136 embraces power to quash
criminal proceedings pending before any court to do complete justice in the matter before
this Court. If the court is satisfied that the proceedings in a criminal case are being
utilised for oblique purposes or if the same are continued on manufactured and false
evidence or if no case is made out on the admitted facts, it would be in the ends of justice
to set aside or quash the criminal proceedings. Once this Court is satisfied that the
criminal proceedings amount to abuse of process of court, it would quash such
proceedings to ensure justice. This Court's power under Article 142(1) to do "complete
justice" is entirely of different level and of a different quality. What would be the need of
complete justice in a cause or matter would depend upon the facts and circumstances of
each case and while exercising that power the Court would take into consideration the
express provisions of a substantive statute. Any prohibition or restriction contained in
ordinary laws cannot act as a limitation on the constitutional power of this Court. Once
this Court has seisin of a cause or matter before it, it has power to issue any order or
direction to do "complete justice" in the matter.
36

. While considering the nature and ambit of its own power under this Article, this Court
observed that it was advisable to leave its power undefined and uncatalogued so that it
remains elastic enough to be molded to suit the given situation; even where no alternative
remedy is efficacious due to lapse of time. [see Delhi Development Authority v. Skipper
Construction Co. (P) Ltd. [(1996) 4 SCC 622] relying on Re: Vinay Chandra Mishra
(1995) 2 SCC 584 and Kerala State Electricity Board v. Kurien E. Kalathil (2000) 6 SCC
293). The power to do complete justice under this Article is. in a way, corrective power,
which gives preference to equity over law. It is a residuary power, supplementary and
complementary to the powers specially conferred by the statutes to do complete justice
between the parties whenever it is just and equitable to do so. It is intended to prevent any
obstruction to the stream of justice. 1995 AIR SCW 1448
1995 AIR SCW 3488
2000 AIR SCW 2647

(Emphasis supplied)
37. In this view of the matter, in order to do complete Justice to the parties in the
backdrop of the peculiar facts of this case and other circumstances noticed hereinbefore
and also taking into consideration the future career of the appellants who by this time
might have joined the noble medical profession and owing to the reasons and
observations above-stated, this appeal is allowed as a result thereof the order of the High
Court impugned in this appeal is set aside subject to the directions contained herein.
38. Mr. K. K. Venugopal and Mr. P. P. Rao at one stage of the hearing very fairly
suggested that keeping in view the relationship of a teacher and taught and having regard
to the peculiar facts and circumstances of the case, the concerned respondents would be
satisfied if an apology is tendered and some amount of compensation is awarded in
favour of the third respondents-informants of Crime No. 412 of 2005 and of Crime No.
21 of 2006.
39. We, having regard to the peculiar facts and circumstances of this case, are of the
opinion that it is a fit case where we should exercise our discretionary jurisdiction under
Article 142 of the Constitution of India so as to bring the dispute between the parties to
an end. We, however, are of the opinion that as the appellants, at the relevant time, were
students, no amount of compensation be directed to be paid. They must, however, file a
written apology in the courts where the proceedings are pending.
40. Consequently, criminal proceedings arising out of Case Crime No. 412/2005
registered at the behest of Dr. Indra Mohini Sharma under Sections 452, 323, 504, 506
and 427, IPC and proceedings of Case Crime No. 21/2006 under Sections 452, 323, 336,
504, 506 and 427, IPC filed by Rajender Kuntal at Police Station Vijay Nagar,
@page-SC2793
Ghaziabad and charges said to have been framed by the trial court based upon the above-
said criminal cases against the appellants shall also stand quashed.
41. Before parting with this judgment, we make it clear that any observations made by us
in this judgment may not be construed as an expression of opinion on the genuineness,
authenticity, validity and legality of the allegations and counter-allegations levelled by
the parties against each other in different proceedings and we have closed the proceedings
of the above-mentioned FIRs initiated against the appellants mainly in exercise of our
jurisdiction under Article 142 of the Constitution.
Appeal allowed.
AIR 2008 SUPREME COURT 2793 "V. R. Dalal v. Yogendra Naranji Thakkar"
(From : Bombay)*
Coram : 2 S. B. SINHA AND HARJIT SINGH BEDI, JJ.
Criminal Appeal No. 925 of 2008, D/- 16 -5 -2008.
V.R. Dalai and Ors. v. Yogendra Naranji Thakkar and Anr.
(A) Criminal P.C. (2 of 1974), S.482 - INHERENT POWERS - COMPLAINT -
Quashing of proceedings - Criminal complaint - Dispute between parties civil in nature -
Court may not allow criminal proceedings to go on - But no law as such can be laid down
as in a given case both proceedings would be maintainable.
(2005) 10 SCC 336, Ref. (Para 12)
(B) Penal Code (45 of 1860), S.405, S.420 - BREACH OF TRUST - CHEATING -
Criminal breach of trust and cheating - Proof - Appellants, partners of firm in which
Complainant was also a partner, allegedly conspired and cheated him to deprive him of
share in profit by dissolving firm behind his back - Partnership firm in question was
constituted on premises that four persons of a Company would represent firm - Said
partnership did not however take off - Partnership stood dissolved at very inception as it
was never acted upon - In view thereof, question of any wrongful act on part of accused
does not arise - As essential ingredients of S.405 and S.420 were missing, no offence is
made out against appellants.
2006 AIR SCW 3830; 2008 AIR SCW 1956, Rel. on.
Cri. Writ Petn. No. 315 of 2004. D/-13-10-2006 (Bom.), Reversed. (Paras 13, 14, 15)
(C) Constitution of India, Art.142 - SUPREME COURT - Powers of Court - Proceeding
found to be an abuse of process of Court - Supreme Court may not allow it to continue.
2008 AIR SCW 11, Rel. on. (Para 16)
Cases Referred : Chronological Paras
2008 AIR SCW 11 : AIR 2008 SC 787 : 2008 Cri LJ 1375 (Rel. on) 16
2008 AIR SCW 1956 : AIR 2008 SC 1683 (Rel. on) 15
2006 AIR SCW 3830 : AIR 2006 SC 2780 (Rel. on) 14
(2005) 10 SCC 336 (Ref.) 11
Suresh Kumar J. Panicker, Pravin Satale and Naresh Kumar, for Appellants; Jatin Zaveri,
for Respondents.
* Cri. Writ Petn. No. 315 of 2004, D/- 13-10-2006 (Bom).
Judgement
1. S. B. SINHA, J. :-Leave granted.
2. A complaint petition was filed by the respondent No. 1 herein in the Court of
Metropolitan Magistrate, 30th Court at Kurla. It was registered as Case No. 271/M of
2002; Accused Nos. 1 to 6 thereof were partners of M/s. N.M. Raiji and Company and
Accused No. 7 was its employee. Appellants herein who were arrayed as the Accused
Nos. 8 to 13 were partners of another firm known as M/s. Gandhi Dalai and Shah. The
said firm was earlier known as Dalai and Shah. Out of the said accused, Accused No. 8
Mr. Y.C. Amin has expired. In the aforementioned complaint petition, allegations were
made that all the accused persons conspired with each other so as to deprive the
complainant from deriving the benefits of a firm by dissolving the firm N.M. Raiji and
Company behind his back.
3. It appears that the firm 'M/s. Gandhi Dalai and Shah' which was constituted with effect
from 1.02.2000 was cancelled from the very inception as the same is said to have not
been acted upon. The name M/s. Gandhi Dalai and Shah was surrendered to the
Chartered Accountants of India. Indisputably, in the complaint petition itself, it has been
accepted that the said M/s. Gandhi Dalai and Shah is no longer in existence. It has further
not been disputed that one Mahendra Thakkar also signed in the original
@page-SC2794
deed of partnership dated 10.02.2001. In relation to the cancellation of the said
partnership, it is alleged :
The Complainant states that they have with some dishonest intention have not prepared
any Deed of dissolution which is mandatory for cancellation of any Deed of partnership.
Merely by cancelling Deed on piece of paper has no meaning in the eye of law and it is
misguiding but in law the Deed of partnership will remain in force till it is dissolved by
Deed of dissolution. The Accused have again played fraud upon the complainant by
misrepresenting him that they have cancelled the Deed of partnership. They have also
filed false documents with Institute of Chartered Accountants in order to commit fraud."
4. The role of the appellants herein are said to be that of conspirators. The complainant
stated :
"The Accused persons in criminal conspiracy with each other intending to kicked out the
complainant from the said firm by adopting intellectual tactics, which is evident from the
conduct and act of the accused. The complainant is putting his full time for the prosperity
of the said firm and he has contributed Lion Share in creating good reputation and
goodwill of the said Firm among the Corporate Sector and other business communities.
The Complainant is not having control over the income of the firm. In spite of the
demand of the Complainant for giving him accounts of the firm the accused have failed
and neglected to do so as such the complainant is not aware at present exact amount
misappropriated by the Accused persons. The complainant states that it is not only
misappropriation but it amounts to theft of the valuable property of the complainant."
5. The firm M/s. N.M. Raiji and Company is a firm of Chartered Accountants. Some
business allegedly had been transferred. It is stated that the accused have committed
criminal breach of trust in respect of income and goodwill of the firm. According to the
complainant, his income from the said firm would have been 5% from the profit of the
said firm which was estimated at 30% wherefrom he has allegedly been deprived, as he
was not made a partner in the new firm.
6. Appellants filed a writ petition before the Bombay High Court which was marked as
Criminal Writ Petition No. 315 of 2004. A separate writ application was also filed by
Accused Nos. 1 to 7 which was marked as Writ Petition No. 542 of 2003. The said writ
petition was dismissed. The High Court, by reason of the impugned judgment, dismissed
the writ petition filed by the appellants also stating :
"5. Mr. Panicker appearing for the petitioners, does not dispute that such an order is
passed. However, he submits that the case of the present petitioners stand on a slightly
different footing, although they are accused in the same criminal case, as far as they are
concerned, they are not the partners of M/s. N.M. Raiji and Company/This N.M. Raiji
and Company was a partnership firm in which all accused 1 to 7 and the complainant
were partners. The petitioners are partners of distinct firm which is known as "Dalai and
Shah" and later on "M/s. Gandhi Dalai and Shah". In such circumstances, it would not be
proper to rely upon the order passed in the other writ petition. These are disputes between
persons, who are partners, during the course of administration and management of the
business of the firm. It is a purely civil dispute. Allowing criminal proceeding according
to Shri Panicker would be abuse of process of the court."
7. Mr. Sureshkumar J. Panicker, learned counsel appearing on behalf of the appellants,
would submit that the dispute between the parties being a civil dispute, if the proceeding
is allowed to continue, it would amount to an abuse of process of law.
8. Mr. Jatin Zaveri, learned counsel appearing on behalf of the respondents, on the other
hand, would contend that as allegations made in the complaint petition constitute an
offence, this Court should not interfere with the impugned judgment.
9. The short question which arises for consideration is as to whether the complaint
petition given its face value and taken to be correct in its entirety constitutes an offence.
M/s. Gandhi Dalai and Shah admittedly was constituted as a partnership firm on
1.12.2000. The said partnership firm was constituted on the premise that four partners of
M/s. N.M. Raiji and Company were representing the firm Dalai and Shah. As indicated
hereinbefore, an outsider was also included therein. Once the said partnership did not
take off and the partnership deed was cancelled as it had never been acted
@page-SC2795
upon, the question of any wrongful act on the part of the appellants did not arise. It is one
thing to say that there exists a dispute amongst the partners inter se but it is another thing
to say that by constituting another firm wherein two firms would be represented by their
respective nominees together with an outsider would itself indicate an act of conspiracy.
Once it has been accepted as of fact that the said partnership has been cancelled, the
question of relying thereupon for any purpose would not arise.
10. The learned counsel appearing on behalf of the respondents, however, has drawn our
attention to a letter dated 28.02.2001 addressed by Arun R. Gandhi, Mahendra N.
Thakkar, Jayesh M. Gandhi, Vinay D. Balse and Sujal A. Shah to Yogendra N. Thakkar
wherein one of the sequence of events was stated to be as under :
"(3) On getting you letter dated February 7, 2001, Mr. Arun Gandhi had discussed the
matter with Mr. Mahendra Thakkar on 9th and 10th February, 2001 who had informed
Mr. Arun Gandhi that he need not worry about it and that everything would be sorted out
by meeting with you, and that we should go ahead with the signing of the partnership
deed (of Gandhi Dalai and Shah). On that basis, the said partnership deed was signed on
10th February, 2001, by five signatories. Mr. Mahendra Thakker could not sign on that
day as he had some urgent work to attend."
This letter does not take us anywhere. It merely shows that internal dispute leading to
abandonment of the concept of starting a new firm.
11. Mr. Panicker has relied upon a decision of this Court in Uma Shankar Gopalika v.
State of Bihar and another [(2005) 10 SCC 336] wherein it has been held that where the
dispute is pure civil in nature an offence under Section 420 or Section 120-B of Indian
Penal Code cannot be said to have been made out.
12. It may be true that in the event the court finds that the dispute between the parties is
civil in nature, it may not allow the criminal proceedings to go on. But, no law, in our
opinion, as such can be laid down as in a given case both civil suit and criminal
complaint would be maintainable although the cause of action for both the proceedings is
the same.
13. We, however, in this case are satisfied that the appellants by no stretch of imagination
can be said to have committed an offence particularly when admittedly the new firm has
been cancelled from its very inception. If the new firm has not derived any income, the
question of depriving the claimant therefrom does not arise. Whether the constitution of
the said firm was illegal, or mala fide, thus, need not be gone into as by reason thereof the
respondent No. 1 cannot be said to have suffered any loss.
14

. We may notice that as regards commission of an offence in terms of Section 405 of


Indian Penal Code, this Court in Indian Oil Corpn. v. NEPC India Ltd. and others [(2006)
6 SCC 736] held that where the first ingradient of criminal breach of trust, that is,
entrustment is missing, the same would not constitute a criminal breach of trust. 2006
AIR SCW 3830

As regards essential ingredients of the offence of cheating, it was stated :


"(i) deception of a person either by making a false or misleading representation or by
other action or omission, (ii) fraudulent or dishonest inducement of that person to either
deliver any property or to consent to the retention thereof by any person or to
intentionally induce that person to do or omit to do anything which he would not do or
omit if he were not so deceived and which act or omission causes or is likely to cause
damage or harm to that person in body, mind, reputation or property."
15

. The aforementioned ingredients of Section 420 of Indian Penal Code are also absent in
the instant case. [See also Suryalakshmi Cotton Mills Ltd. v. Rajvir Industries Ltd. and
Ors. 2008 (1) SCALE 331] 2008 AIR SCW 1956

16

. When a proceeding is found to be an abuse of the process of court, this Court in exercise
of its jurisdiction under Article 142 of the Constitution of India may not allow it to
continue. For the said purpose, the fact of the matter can be looked into. It was so done
recently in Sanapareddy Maheedhar and another v. State of Andhra Pradesh and another
[2007 (14) SCALE 321) wherein upon noticing a large number of decisions of this Court,
it was held : 2008 AIR SCW 11

"We are further of the view that in the peculiar facts of this case, continuation of
proceedings of CC No.240/2002 will amount
@page-SC2796
to abuse of the process of the Court. It is not in dispute that after marriage, Shireesha
Bhavani lived with appellant No. 1 for less than one and a half months (eight days at
Hyderabad and about thirty days at New Jersey). It is also not in dispute that their
marriage was dissolved by the Superior Court at New Jersey vide decree dated
15.12.1999. Shireesha Bhavani is not shown to have challenged the decree of divorce. As
a matter of fact, she married Sri Venkat Puskar in 2000 and has two children from the
second marriage. She also received all the articles of dowry (including Jewellery) by
filing affidavit dated 28.12.1999 in the Superior Court at New Jersey. As on today a
period of almost nine years has elapsed of the marriage of appellant No.1 and Shireesha
Bhavani and seven years from her second marriage. Therefore, Page 0086 at this belated
stage, there does not appear to be any justification for continuation of the proceedings in
CC No.240/2002. Rather, it would amount to sheer harassment to the appellant and
Shireesha Bhavani who are settled in USA, if they are required to come to India for
giving evidence in relation to an offence allegedly committed in 1998-99. It is also
extremely doubtful whether the Government of India will, after lapse of such a long time,
give sanction in terms of Section 188, Cr.P.C."
17. For the reasons aforementioned, the impugned judgment cannot be sustained. It is set
aside accordingly. The summons issued by the learned Magistrate against the appellants
is quashed. The appeal is allowed.
Appeal allowed.
AIR 2008 SUPREME COURT 2796 "Kusumam Hotels (P) Ltd., M/s. v. Kerala State
Electricity Board"
(From : Kerala)*
Coram : 2 S. B. SINHA AND LOKESHWAR SINGH PANTA, JJ.
Civil Appeal No. 101 with 102 to 106 and 3309 of 2007, D/- 16 -5 -2008.
M/s. Kusumam Hotels (P) Ltd. v. Kerala State Electricity Board and Ors.
(A) Constitution of India, Art.162 - POLICY DECISION - STATE LEGISLATURE -
Policy decision - Can be amended or rescinded retrospectively.
The State would be entitled to alter, amend or rescind its policy decision. Such a policy
decision, if taken in public interest, should be given effect to. In certain situations, it may
have an impact from a retrospective effect but the same by itself would not be sufficient
to be struck down on the ground of unreasonableness if the source of power is referable
to a statute or statutory provisions. (Para 26)
(B) Electricity (Supply) Act (54 of 1948), S.78A - ELECTRICITY - STATE - Electricity
tariffs - Concession given to Hotels by Electricity Board in view of Directive issued by
State - Subsequent notification issued by State limiting concession to 5 years - Cannot
have retrospective effect - State in issuing notification was exercising statutory powers of
Issuing directions to Board under Act - Issuance of retrospective directions not
contemplated by Act.
W.A. No. 624 of 2005, D/-18-05-2005 (Ker). Reversed.
By reason of a policy decision adopted by the Central Government, 'tourism' was
declared to be an 'industry'. The State of Kerala adopted the said policy of the Central
Government. Pursuant to the said policy decision, various incentives were to be granted.
It was declared that "Tourism" will be treated as an 'Industry'. The State in view of its
policy decision gave directive to State Electricity Board to apply industrial tariff to
certain Hotels certified by Director Tourism. Subsequently, the State of Kerala issued a
Government Order on or about 26-9-2000 stating that the concession on electricity tariff
shall be limited only to five years by the Department of Tourism, Government of Kerala.
The concession was not to be extended for any further period. The G.O. Order provided
that the G.O. would be effective from an anterior date. It further provided that, the
tourism units, which have received certificate of eligibility for tariff concession from
Director, Department of Tourism, have to produce a certificate from the Kerala State
Electricity Board regarding the total period for which they have enjoyed the concessional
tariff. They will be eligible for concessional tariff only for a period of five years including
the period for which already enjoyed. Demand and disconnection notices were thereupon
issued by Board to the Hotels. The legality of the G.O. and particularly its retrospectivity
was challenged by Hotels receiving the concession.
Held, the State of Kerala in this case did
@page-SC2797
not grant any concession by itself. It issued directions which were binding upon the
Board having regard to the provisions contained in Section 78-A of the 1948 Act. The
Board was bound thereby. While issuing the impugned notifications the State was not
exercising the power under any statute to grant or withdraw the concession. It was
exercising its statutory power of issuing direction. It is, therefore, a statutory authority.
The 1948 Act does not authorize the State to issue a direction with retrospective effect.
The Board, therefore, could only give prospective effect to such directions in absence of
any clear indication contained therein. By reason of withdrawal of concession with
retrospective effect, the accrued right of the Hotels had been affeted. The impugned G.O.
dated 26-9-2000 must be held to have a prospective operation and not a retrospective
operation. That view would save it from being vulnerable to the challenge of being hit by
Article 14 of the Constitution of India.
W.A. No. 624 of 2005, D/-18-05-2005 (Ker), Reversed. (Paras 27, 29)
(C) Electricity Act (36 of 2003), S.56(2) - ELECTRICITY - LIMITATION - Recovery of
electricity tariff - Limitation - S.56(2) has no application to dues arising under 1948 Act.
(Para 30)
Cases Referred : Chronological Paras
2008 AIR SCW 599 : AIR 2008 SC 1032 : 2008 (2) ALJ 186 (Ref.)19
2007 AIR SCW 3752 : AIR 2007 SC 1984 (Rel. on) 19, 30
2007 AIR SCW 7814 : AIR 2008 SC 693 : 2008 (1) ALJ 701 19
2006 AIR SCW 3627 : AIR 2006 SC 2652 (Ref.) 25, 27
2006 AIR SCW 6304 : AIR 2007 SC 401 : 2007 (1) AIR Bom R 730 28
2006 AIR SCW 6320 : AIR 2007 SC 414 (Ref.) 25
(2005) 1 SCC 625 (Ref.) 25
2003 AIR SCW 6130 : AIR 2004 SC 297 19
1998 AIR SCW 186 : AIR 1998 SC 591 (Ref.) 19, 24
1997 AIR SCW 3839 : AIR 1997 SC 3910 : 1997 All LJ 2202 (Ref.) 25
(1997) 3 SCC 398 (Ref.) 19, 24
(1996) 1 SCC 597 32
1995 AIR SCW 680 : AIR 1995 SC 874 (Ref.) 19, 20
AIR 1987 SC 1727 (Rel. on) 18
AIR 1986 SC 806 : 1986 Tax LR 2002 23
AIR 1985 SC 421 : 1985 Tax LR 353 (Rel. on) 18
AIR 1980 SC 1285 23
AIR 1979 SC 621 : 1979 All LJ 368 22, 24
(1964) 3 All ER 556 : (1964) 1 WLR 1326 24
P. S. Patwalia, R. Venkataramani, P. Krishnamoorthy, Sr. Advocates, Ankur Chawla, Ms.
Pallavi Langer, Prithvi Singh Sidhu, Mrs. Manik Karanjawala, A. K. Joseph, K. Rajeev
and Romy Chacko, for Appellant; M. T. George and R. Sathish, for Respondents.
* W.A. No. 624 of 2005, D/- 18-5-2005 (Ker).
Judgement
1. S. B. SINHA, J. :-These appeals involving similar questions of facts and law were
taken up for hearing together and are being disposed of by this common judgment.
2. Appellants herein are owners of hotels situated at different parts of the State of Kerala.
By reason of a policy decision adopted by the Central Government, 'tourism' was
declared to be an 'industry'. The State of Kerala adopted the said policy of the Central
Government. Pursuant to the said policy decision, various incentives were to be granted.
It was declared that "Tourism" will be treated as an "Industry" and the concessions
available to the' 'tourism industry' were :
"(i) Subsidy for preparation of feasibility/ project report.
(ii) Investment subsidy limited to 10% thereof.
(iii) Incentive for training local manpower.
(iv) Augmenting availability of funds from State Financial Corporations.
(v) Concession in electricity and water charges.
(vi) Allocation of land at concessional rate.
(vii) Exemption from building tax levied by the Revenue Department. (Action to amend
the Kerala Buildings Tax Act, 1975 will be taken separately)."
3. Apart from the concession in electricity and water charges and payment of building tax
to be levied by the Revenue Department which was open ended in nature, other
concessions were to be granted on a one time measure.
4. A new policy for grant of investment subsidy was also floated.
Classified hotels (One to Five Stars) came within the purview thereof. In terms of the
@page-SC2798
said policy decision, the Kerala State Electricity Board (the Board) was directed to grant
tariff concessions to the classified hotels and motels consequent on the said declaration of
Government of Kerala and Government of India. The concessions to be granted thereby
were :
"(1) The electricity tariff applicable to the categories listed above will be hti-industrial
tariff/l.t. Iv industrial tariff depending on the type of supply from 1.4.1987
(2) The tariff as indicated above will be applied to the institutions either on production of
proper certificate from the Director of Tourism or based on list of institutions eligible for
the concessional tariff furnished by the director of tourism to the Secretary, Kerala
Electricity Board. The certificates/ communications should be given by the Director of
Tourism himself.
(3) In the case of institutions in the above categories applying for power connection
hereafter tariff as above will be applied by the Kerala State Electricity Board on receipt of
necessary certificate from the Director of Tourism.
(4) Regarding the admissibility of the concession to any particular unit the matter will be
referred to the Director of Tourism and the report on the matter will be accepted by the
Kerala State Electricity Board."
5. Indisputably, the appellants had set up or upgraded their hotels and motels. The
Government of Kerala classified the hotels in question in several categories for which
they became entitled to from the year 1990.
The Board, allegedly, had been suffering losses. The Government of Kerala, however,
issued a Government Order on or about 25.8.1997 adopting the mode of grant of subsidy,
inter alia, to the industrial sector, the relevant portions whereof read as under :
"In the Government Order read as first paper above it was ordered that the actual cost of
electricity concessions allowed to Industries in the State, as part of Industrial policy will
be reimbursed to Kerala State Electricity Board to the extent necessary to reach 3% Rate
of Return (ROR) starting with the accounting year 1986-87, by adjusting the amount of
concession against the dues payable to Government by Kerala State Electricity Board.
2. The Chairman, Kerala State Electricity Board in his letters read above has reported that
the loss sustained by the Kerala State Electricity Board due to concessional electricity
tariff allowed to Industries during the last ten years comes to Rs. 60.3 crores and that the
loss for the year 1995-96 alone is Rs. 24 crores.
xxx xxx xxx
4. Having considered the entire issue in detail, Government are pleased to issue the
following orders :
(i) The Industries and the Agricultural Departments in Government will find the funds
from their respective Budget required for giving subsidy to Industries and farmers for the
year 1997-1998 by re-appropriation. The above departments will also provide required
amounts in their department budget from the financial year 1998-99 onwards.
(ii) The subsidy for electricity tariff admissible to Industrial consumers and farmers will
be disbursed to the beneficiaries by the concerned Departments from the financial year
1998-99 onwards."
6. By an order dated 11.10.1999, the industrial tariffs granted to the hotels in the State
stood cancelled w.e.f 15.10.1999. It was ordered that industrial tariff already granted by
various officers of the Board from 15.5.1999 would be suspended by an order dated
8.11.1999, stating :
The Board hereby orders that the institutions which were already enjoying industrial tariff
prior to 15.5.99 on the strength of certificate issued by Director of Tourism shall continue
to be charged at the industrial tariff until further orders. This is subject to the final
decision of the Government on payment of subsidy. From 15.5.99 new applications for
granting industrial tariff will not be sanctioned to such institutions. The field officers of
the Board shall not grant industrial tariff from 15.5.99 to the institutions certified by
Director of Tourism."
7. The hotels of the appellants were reclassified in the year 1999 keeping in view the
investment made by them.
Appellants, however, were served with demand-cum-disconnection notices on the basis
of bills raised on commercial tariffs on or about 9.4.2000.
8. A writ petition was filed thereagainst. In the meantime, the State of Kerala issued a
Government Order on or about 26.9.2000 stating that the concession on electricity
@page-SC2799
tariff shall be limited only to five years by the Department of Tourism, Government of
Kerala. The concession was not to be extended for any further period. Clause (3) of the
said GO reads, thus :
"These orders will be operative from 15.5.1999, the effective date from which Kerala
State Electricity Board has withdrawn the concessional tariff offered to tourism units. The
tourism units, which have received certificate of eligibility for tariff con-cession from
Director, Department of Tourism, have to produce a certificate from the Kerala State
Electricity Board regarding the total period for which they have enjoyed the concessional
tariff. They will be eligible for concessional tariff only for a period of five years including
the period for which already enjoyed the concession, i.e., if the tourism unit has already
enjoyed tariff for a period of three years prior to 15.5.1999, they will be eligible for
concessional tariff for a further period of two years only. This period will be counted
from the effective date originally certified by the Director of Tourism, Government of
Kerala for granting concessional tariff for three years. If any tourism unit has already
enjoyed concessional tariff for a period of five years or more prior to 15.5.1999, it will
not be eligible for any extension of the period of concession."
9. The writ petition filed by the appellants was disposed of by an order dated 4.8.2004
directing that commercial tariff may be charged w.e.f. 15.5.1999 onwards. After the
aforementioned Government Order dated 26.9.2000 was issued, demand-cum-
disconnection notices were issued again. Representations were made by the appellants
which were rejected.
10. They preferred an intra-court appeal.
Fresh writ petitions were filed, inter alia, praying for quashing of the bill and the said
Government Order as also for further classification of the hotel, as industrial units.
By reason of a judgment and order dated 16.2.2005, the said writ petition was disposed of
directing that 18% interest instead of 24% would be charged, if the demanded amount is
paid till 31.5.2005.
11. Intra-court appeals were preferred thereagainst and by reason of the impugned
judgment, the same have been dismissed.
12. Mr. Patwalia, Mr. Venkataramani and Mr. Krishnamoorthy, learned senior counsel
appearing on behalf of the appellants, would submit :
(i) The concessions granted to the appellants should not have been withdrawn from an
anterior date.
(ii) The Board could not have directed application of commercial tariff despite the fact
that the hotels are still considered to be an industry.
(iii) In view of the provisions in sub-section (2) of Section 56 of the Electricity Act, 2003,
no bill could have been raised after a period of two years.
13. Mr. George, learned counsel appearing on behalf of the State Electricity Board and
Mr. Sathish, learned counsel appearing on behalf of the State of Kerala, would submit :
(a) 2003 Act is not applicable in relation to the bills raised under the Electricity (Supply)
Act, 1948.
(b) The impugned order dated 26.9.2000 is not retrospective in operation. In any event,
the State has the requisite jurisdiction to stop grant of concession even with retrospective
effect.
(d) No foundational fact having been laid to establish the plea of promissory estoppel, the
same is not available to the appellants particularly when they had entered into a contract
with the Board for which the bills were to be raised on the basis of commercial tariff.
(e) Appellants having filed writ petitions after a long time, the impugned judgment should
not be interfered with.
14. Indisputably, by reason of the impugned Government Order, the benefit of one of the
concessions made available to the appellants by reason of the Government Order dated
11.7.1996 had been taken away. The core question which arises for our consideration is
whether the said Government Order dated 26.9.2000 is reasonable having been given
retrospective effect and retroactive operation.
15. Tourism was declared to be an industry. The wide range of concessions as noticed
hereinbefore, inter alia, covered electricity and water charges. It is not a case where some
exemptions or concessions were to be given for a specific period or as a one time
measure. No time limit was fixed for applicability in respect of the policy decisions.
Pursuant thereto long term investments
@page-SC2800
might have been made. It is not based on a principle of giving benefit with a view to
facilitate the initial growth of the industry. It was not based on any formula or criteria to
evaluate the realization of the object of grant of such concession over a period. It was an
open ended offer. It must, therefore, be held that the Government was satisfied that the
need was to grant concession if not permanently, at least for a long time.
16. There cannot be any doubt whatsoever that a policy decision can be reviewed from
time to time. It is also beyond any doubt that the concessions granted can be withdrawn
in public interest.
Indisputably, the State is also entitled to change or alter the economic policies. Appellants
do not have any vested right to enjoy the concessions granted to them forever,
particularly when the Board is constituted and incorporated under the provisions of
Electricity (Supply) Act, 1948. Any policy decision adopted by the State would not be
binding on the Board, save and except provided for in the Act. The Board being an
independent entity, the duties and functions of the Board vis-a-vis the State are
enumerated in the Act. The Board, however, would be bound by any direction issued by
the State Government on questions of policy. A dispute which may arise as to whether a
question is or not a question of policy involving public interest, Central Government is
the final arbiter. The policy decision adopted by the State on the basis whereof the Board
felt obligated to grant electrical connection in favour of the appellants on the basis of
industrial tariff must, therefore, be understood in the context of Section 78-A of the 1948
Act. What is binding on the Board is the policy of the State. The direction of the State
was to apply a particular category of tariff to the appellants. Such directions could have
been withdrawn while making another tariff. The State indisputably has the power to
grant subsidy from its own coffer instead of directing the Board to grant concession.
17. It is now a well settled principle of law that the doctrine of promissory estoppel
applies to the State. It is also not in dispute that all administrative orders ordinarily are to
be considered prospective in nature. When a policy decision is required to be given a
retrospective operation, it must be stated so expressly or by necessary implication. The
authority issuing such direction must have power to do so. The Board, having acted
pursuant to the decision of the State, could not have taken a decision which would be
violative of such statutory directions.
15.5.1999 was fixed as the cut-off date by the Board. It, by itself, could not have done so.
But the State for issuing the GO dated 26.9.2000 could have fixed the said cut-off date on
its own. We although do not agree that by granting retrospectivity to the said order, the
entirety of the Government Order should be set aside the same or per se would be held to
be unreasonable, but what we mean to say is that it could be given effect to only from the
date of the order, i.e., prospectively and not from an anterior date, i.e., retrospectively.
18

. It was held in Lohia Machines Ltd. and Anr. v. Union of India (UOI) and Ors. [(1985) 2
SCR 686] : AIR 1985 SC 421, Para 81

"On the other hand it is quite clear that if the relief granted is to be withdrawn with
retrospective operation from 1972 the assessees who have enjoyed the relief for all those
years will have to face a very grave situation. The effect of the withdrawal of the relief
with retrospective operation will be to impose on the assessee a huge accumulated
financial burden for no fault of the assessee and this is bound to create a serious financial
problem for the assessee. Apart from the heavy financial burden which is likely to upset
the economy of the undertaking, the assessee will have to face other serious problems. On
the basis that the relief was legitimately and legally available to the assessee, the assessee
had proceeded to act and to arrange its affairs. If the relief granted is now permitted to be
withdrawn with retrospective operation, the assessee may be found guilty of violation of
provisions of other statutes and may be visited with penal consequences..."
Yet again in M/s. Indian Metals and Ferro Alloys Ltd. and Anr. v. State of Orissa and Ors.
[(1987) 3 SCC 189], it was opined : AIR 1987 SC 1727, Para 23

"25...we hold that the High Court was not right in observing that the orders under Section
22-B of the Act imposing restrictions on consumption of power could not legally and
validly be passed by the Government "with retrospective effect" in the middle of a water
year. But the position regarding disallowance of clubbing stands on an entirely
@page-SC2801
different footing. If a consumer had been allowed the benefit of clubbing previously, that
benefit cannot be taken away with retrospective effect thereby saddling him with heavy
financial burden in respect of the past period where he had drawn and consumed power
on the faith of the orders extending to him the benefit of clubbing..."
19

. It is not necessary for us to notice a large number of decisions on promissory estoppel as


the principle thereof has recently been noticed by this Court in Southern Petrochemical
Industries Co. Ltd. v. Electricity Inspector and E.T.I.O. and Ors. [(2007) 5 SCC 447]
wherein it was stated :2007 AIR SCW 3752

"We are also unable to agree with Mr. Andhyarujina that exemption from tax is a mere
concession defeasible by the Government and does not confer any accrued right to the
recipient. Right of exemption with a valid notification issued gives rise to an accrued
right. It is a vested right. Such right had been granted to them permanently. "Permanence"
would mean unless altered by statute. Thus, when a right is accrued or vested, the same
can be taken away only by reason of a statute and not otherwise. Thus, a notification
which was duly issued would continue to govern unless the same is repealed."
It was further held :

"126. This Court distinguished its earlier decision in Kasinka Trading v. Union of India,
whereupon Mr. Andhyarujina placed strong reliance, in the following terms : 1995
AIR SCW 680

"40. The case of Kasinka Trading v. Union of India cited by the appellant is an authority
for the proposition that the mere issuance of an exemption notification under a provision
in a fiscal statute such as Section 25 of the Customs Act, 1962, could not create any
promissory estoppel because such an exemption by its very nature is susceptible to being
revoked or modified or subjected to other conditions. In other words, there is no
unequivocal representation. The seeds of equivocation are inherent in the power to grant
exemption. Therefore, an exemption notification can be revoked without falling foul of
the principle of promissory estoppel. It would not, in the circumstances, be necessary for
the Government to establish an overriding equity in its favour to defeat the petitioner's
plea of promissory estoppel. The Court also held that the Government of India had
justified the withdrawal of exemption notification on relevant reasons in the public
interest. Incidentally, the Court also noticed the lack of established prejudice to the
promises when it said :
The burden of customs duty, etc. is passed on to the consumer and therefore the question
of the appellants being put to a huge loss is not understandable.'

(See also Shrijee Sales Corpn. v. Union of India and STO v. Shree Durga Oil Mills) We
do not see the relevance of this decision to the facts of this case. Here the representations
are clear and unequivocal"." 1998 AIR SCW 186

In LML Ltd. v. State of U.P. and Ors. [2007 (14) SCALE 469], this Court opined : 2008
AIR SCW 599

"38. Those suppliers, who keeping in view of their capacity to supply uninterrupted
electrical energy had made a representation and pursuant thereto the consumers had
altered their position, cannot be permitted to take a different stand as the doctrine of
promissory estoppel would apply against them. The said doctrine is premised on the
conduct of party making a representation to the other so as to enable him to arrange its
affairs in such a manner as if the said representation would be acted upon. It provides for
a cause of action. It need not necessarily be a defence."

Yet again in U.P. Power Corporation Ltd. and Anr. v. Sant Steel and Alloys (P) Ltd. and
Ors. [2007 (14) SCALE 36], it was held : 2007 AIR SCW 7814

"In this background, in view of various decisions noticed above, it will appear that the
Court's approach in the matter of invoking the principle of promissory estoppel depends
on the facts of each case. But the general principle that emerges is that once a
representation has been made by one party and the other party acts on that representation
and makes investment and thereafter the other party resiles, such act cannot stated to be
fair and reasonable. When the State Government makes a representation and invites the
entrepreneurs by showing various benefits for encouraging to make investment by way of
industrial development of the backward areas or the hill areas, and thereafter the
entrepreneurs on the representations so made bona fidely make investment and thereafter
if the State Government
@page-SC2802
resile from such benefits, then it certainly is an act of unfairness and arbitrariness.
Consideration of public interest and the fact that there cannot be any estoppel against a
Statute are exceptions."

In State of Orissa and Ors. v. Mangalam Timber Products Ltd. [(2004) 1 SCC 139], a
three-Judge Bench of this Court, held : 2003 AIR SCW 6130, Para 4

"... The State Government having persuaded the respondent to establish an industry and
the respondent having acted on the solemn promise of the State Government, purchased
the raw material at a fixed price and also sold its products by pricing the same taking into
consideration the price of the raw material fixed by the State Government and supplied;
the State Government cannot be permitted to revise the terms for supply of raw material
adversely to the interest of the respondent and effective from a back date and place the
respondent in a situation which it will not be able to resolve. The respondent could not
have revised its price from a back date and recovered it from innumerable consumers to
whom its finished products were supplied at a fixed price."
20

. Our attention, however, has been drawn to a decision of this Court in Kasinka Trading
and Anr. v. Union of India and Anr. [(1995) 1 SCC 274]. Therein the power of the State to
change its policy decision in public interest was emphasized. It was held that the power
which can be used for grant of concession, namely, Section 25(1) of the Customs Act
itself is the source to rescind the earlier notification, stating : 1995 AIR SCW 680,
Para 22

"Since, the notification had been issued under Section 25(1) of the Act, the very same
power was available to the authority for rescinding or modifying that notification and
appellant ought to have known that the said notification was capable of or liable to be
revoked, modified or rescinded at any time even before the expiry of 31-3-1981 if the
'public interest' so demanded. To hold that after the Government had issued the
Notification No.66 of 1979 indicating that it was to remain operative till 31-3-1981, it
could not be rescinded or modified before the expiry of that date would amount to
prohibiting the Government from discharging its statutory obligation under Section 25(1)
of the Act, if it was satisfied that it was in the 'public interest' to withdraw, modify or
rescind the earlier notification. The plain language of Section 25 of the Act is indicative
of the position that it is the public interest and public interest alone which is the dominant
factor. It is not the case of the appellants that the withdrawal of Notification No.66 of
1979 by the impugned notification was not in 'public interest'. Their case, however, is that
relying upon the earlier notifications they had acted and the Government should not be
permitted to go back on its assurance as otherwise they would be put to huge loss. The
courts have to balance the equities between the parties and indeed the courts would bind
the Government by its promise 'to prevent manifest injustice or fraud'."
It was further held :
"23. The appellants appear to be under the impression that even if, in the altered market
conditions the continuance of the exemption may not have been justified, yet,
Government was bound to continue it to give extra profit to them. That certainly was not
the object with which the notification had been issued. The withdrawal of exemption "in
public interest" is a matter of policy and the courts would not bind the Government to its
policy decisions for all times to come, irrespective of the satisfaction of the Government
that a change in the policy was necessary in the "public interest". The courts, do not
interfere with the fiscal policy where the Government acts in "public interest" and neither
any fraud or lack of bona fides is alleged much less established. The Government has to
be left free to determine the priorities in the matter of utilisation of finances and to act in
the public interest while issuing or modifying or withdrawing an exemption notification
under Section 25(1) of the Act."
21. We are not concerned with the exercise of a statutory power in this case. We are
concerned with issuance of a direction by the State which is binding on the Board as also
how and to what extent it can be rescinded.
22
. We may, however, notice that in Motilal Padampat Sugar Mills v. State of U.P. [(1979) 2
SCR 641, this Court held : AIR 1979 SC 621, Para 25

"Public bodies are as much bound as private individuals to carry out representations of
facts and promises made by them, relying on which other persons have altered their
@page-SC2803
position to their prejudice
***
If our nascent democracy is to thrive different standards of conduct for the people and the
public bodies cannot ordinarily be permitted. A public body is, in our judgment, not
exempt from liability to carry out its obligation arising out of representations made by it
relying upon which a citizen has altered his position to his prejudice."
23

. Another Bench in Jit Ram v. State of Haryana [(1980) 3 SCR 689] took a different view.
Jit Ram was overruled in Union of India v. Godfrey Philips India Ltd. [(1985) 4 SCC
369]. AIR 1980 SC 1285
AIR 1986 SC 806

24. If the doctrine of promissory estoppel applies for the purpose of enforcing the
concession granted in favour of entrepreneurs, it can be withdrawn, inter alia, in public
interest. Despite absence of an overriding public interest, however, although a different
policy decision can be taken but therefor adequate notice should be given. It was so held
in Shrijee Sales Corporation and Anr. v. Union of India [(1997) 3 SCC 398] in the
following terms :

"Once public interest is accepted as the superior equity which can override individual
equity, the principle should be applicable even in cases where a period has been
indicated. The Government is competent to resile from a promise even if there is no
manifest public interest involved, provided, of course, no one is put in any adverse
situation which cannot be rectified. To adopt the line of reasoning in Emmanuel Ayodeji
Ajay v. Briscoe quoted in M.P. Sugar Mills even where there is no such overriding public
interest, it may still be within the competence of the Government to resile from the
promise on giving reasonable notice which need not be a formal notice, giving the
promisee a reasonable opportunity of resuming his position, provided of course, it is
possible for the promisee to restore the status quo ante. If, however, the promisee cannot
resume his position, the promise would become final and irrevocable." AIR 1979 SC
621

The same principle was reiterated in Sales Tax Officer and Anr. v. Shree Durga Oil Mills
and Anr. [(1998) 1 SCC 572]. 1998 AIR SCW 186

25
. In Pawan Alloys and Casting Pvt. Ltd. v. U.P. State Electricity Board and Ors. [(1997) 7
SCC 251], it was held : 1997 AIR SCW 3839, Para 53

"60. So far as Point No. 3 is concerned the appellants are on a weaker footing. It is true
that by earlier notifications dated 29-10-1982, 13-7-1984 and 28-1-1986 the scheme of
incentives by way of development rebate of 10% was continued to be offered to new
industries to be established in the plains of State of U.P. Identically worded Item 9 in the
earlier notifications and Item 8 in the last notification dated 28-1-1986 had continued the
said incentive scheme. By virtue of the last notification of 28-1-1986 it was clearly laid
down by the Board that all new industries which might be established on and after 28-1-
1986 will earn this development rebate for the three years' period from the date of
commencement of supply of electricity. It was also provided that all the existing new
industries which might have earlier been established before 28-1-1986 and which had still
some part of unexpired period of three years of development rebate available with them
also were given the continued benefit of the development rebate for the unexpired period
from 1-2-1986. What the impugned notification of 31-7-1986 sought to do was to delete
this first para of Item 8 of the notification of 28-1-1986. The result was that from 1-8-
1986 whatever unexpired period for getting development rebate of 10% was available
with the new industries covered by the sweep of the said notification, got withdrawn. It
could not be said and it is also not the case of the respondent-Board that in the light of the
notification of 31-7-1986 whatever development rebate was granted to these new
industries earlier as per the then existing scheme would stand withdrawn or any recovery
would be effected against them for the said amount. The case of the Board is that despite
any unexpired period for earning the incentive rebate of 10% was available to the existing
new industries on 31-7-1986, they would lose that benefit of development rebate for the
rest of the unexpired period with effect from 1-8-1986 onwards. Hence it is not possible
to agree with the contention of learned counsel for the appellants that the said notification
had any retrospective effect. It was purely prospective and had resulted into two
consequences - (i) any new industry which entered into an agreement with the Board for
supply of electricity for the first time on and after 1-8-1986
@page-SC2804
could not get the benefit of incentive of 10% development rebate; and (ii) all existing
new industries which were armed with the guarantee of 10% development rebate under
the earlier notifications and had unexpired period out of the three years from the date of
earlier commencement of supply of electricity to their concerns lost the benefit for that
unexpired period which otherwise would have been available to them from 1-8-1986
onwards till the entire three years' period which had already commenced would have
been over. Both these effects of the notification of 31-7-1986 were purely prospective in
character and had no retrospective effect. Consequently it cannot be said that the said
notification was liable to be struck down on the score of being retrospective in nature.
The third point for consideration, therefore, is answered in the negative."

Similar view has been taken in Bannari Amman Sugars Ltd. v. Commercial Tax Officer
and Ors. [(2005) 1 SCC 625]; Kuldeep Singh v. Govt. of NCT of Delhi [(2006) 5 SCC
702]; and M.P. Mathur and Ors. v. DTC and Ors. [(2006) 13 SCC 706]. 2006 AIR
SCW 3627
2006 AIR SCW 6320

26. The law which emerges from the above discussion is that the doctrine of promissory
estoppel would not be applicable as no foundational fact therefor has been laid down in a
case of this nature. The State, however, would be entitled to alter, amend or rescind its
policy decision. Such a policy decision, if taken in public interest, should be given effect
to. In certain situations, it may have an impact from a retrospective effect but the same by
itself would not be sufficient to be struck down on the ground of unreasonableness if the
source of power is referable to a statute or statutory provisions. In our constitutional
scheme, however, the statute and/or any direction issued thereunder must be presumed to
be prospective unless the retrospectivity is indicated either expressly or by necessary
implication. It is a principle of rule of law. A presumption can be raised that a statute or
statutory rules has prospective operation only.
27. The State of Kerala in this case did not grant any concession by itself. The Central
Government took a larger policy of treating the tourism as an industry. A wide range of
concessions were to be granted by way of one time measure; some of them, however, had
a recurring effect. So far as grant of benefits which were to be recurring in nature, the
State exercises its statutory power in the case of grant of exemption from payment of
building tax wherefor it amended the statute. It issued directions which were binding
upon the Board having regard to the provisions contained in Section 78A of the 1948 Act.
The Board was bound thereby. The Board, having regard to its financial constraints,
could have brought its financial stringency to the notice of the State. It did so. But the
State could not have taken a unilateral decision to take away the accrued or vested right.
The Board's order dated 11-10-1999 in law could not have been given effect to. The
Board itself kept the said notification in abeyance by reason of order dated 8-11-1999.

Appellants, indisputably, continued to derive the benefits in terms of the original order.
They obtained certificates of classification. It is on the aforementioned context, the
question as regards construction of the impugned notification dated 26-9-2000 arises. Ex
facie, the said policy decision could not be given a retrospective effect or retroactive
operation. The State was not exercising the power under any statute to grant or withdraw
the concession. It was exercising its statutory power of issuing direction. It is, therefore, a
statutory authority. The 1948 Act does not authorize the State to issue a direction with
retrospective effect. The Board, therefore, could only give prospective effect to such
directions in absence of any clear indication contained therein. By reason of withdrawal
of concession with retrospective effect, the accrued right of the appellants had been
affected. In Kuldeep Singh v. Govt. of NCT of Delhi. [(2006) 5 SCC 702], this Court held
: 2006 AIR SCW 3627, Para 36

"In a case of this nature, where the State has the exclusive privilege and the citizen has no
fundamental right to carry on business in liquor, in our opinion, the policy which would
be applicable is the one which is prevalent on the date of grant and not the one, on which
the application had been filed. If a policy decision had been taken on 16-9-2005 not to
grant L-52 licence, no licence could have been granted after the said date."
We, however, are not concerned with a similar situation.
@page-SC2805
28

. However, in Ramchandra Murarilal Bhattad and Ors. v. State of Maharashtra and Ors.
[(2007) 2 SCC 588], it was held : "64. It is not a case where the court is called upon to
exercise its equity jurisdiction. It is also not a case where ex facie the policy decision can
be held to be contrary to any statute or against a public policy. A policy decision may be
subject to change from time to time. Only because a change is effect, (sic) the same by
itself does not render a policy decision to be illegal or otherwise vitiated in law." 2006
AIR SCW 6304, Para 63

29. We, therefore, are of the opinion that the impugned GO dated 26-9-2000 must be held
to have a prospective operation and not a retrospective operation. That view would save it
from being vulnerable to the challenge of being hit by Article 14 of the Constitution of
India.
30. We, however, are not in a position to accept the contention that the Bills could not
have been issued having regard to subsection (2) of Section 56 of the Act. Appellants
herein have incurred liabilities.
Sub-section (5) of Section 185 of the Electricity Act, 2003 reads, thus :
"(5) Save as otherwise provided in subsection (2), the mention of particular matters in
that section, shall not be held to prejudice or affect the general application of section 6 of
the General Clauses Act, 1897 (10 of 1897), with regard to the effect of repeals."

Whereas the bills are issued only in respect of the dues arising in terms of the law as was
applicable prior to the coming into force of 2003 Act. Sub-section (2) of Section 56 shall
apply after the said Act came into force. The Board could have even framed a tariff in
terms of the provisions appended to Section 61 of the Act. Appellants incurred liability to
pay the bill. The liability to pay electricity charges is a statutory liability. The Act
provides for its consequences. Unless, therefore, the 2003 Act specifically introduced, the
bar of limitation as regards the liability of the consumer incurred prior to corning into
force of the said Act. In our opinion, having regard to Section 6 of the General Clauses
Act, the liability continues. [See Southern Petrochemical Industries Co. Ltd. v. Electricity
Inspector and E.T.I.O. and Ors. [(2007) 5 SCC 447].2007 AIR SCW 3752

31. We, therefore, are of the opinion that the High Court was not correct in its view to the
aforementioned extent. The judgment of the High Court is, thus, set aside to the
aforementioned extent. The appeals are allowed with costs. Counsel fee assessed at
25,000/- (Rupees five thousand only) in each appeal.
CIVIL APPEAL No. 106 OF 2005
32. Board has preferred this appeal only against grant of instalments in favour of the
respondents. The contention of Mr. George that the High Court could not have waived the
provisions of interest on the delayed payment under the tariff cannot be accepted. In all
other cases, the High Court directed that 18% interest would be payable following the
decision of the Court in Kerala State Electricity Board through its Special Officer
(Revenue) and Anr. v. M.R.F. Ltd. [(1996) 1 SCC 597]. The same principle would apply
in this case also but the bill having been raised only in 2003, the question of charging any
interest thereupon from a retrospective date would not arise.
33. This appeal is, thus, dismissed. However, there shall be no order as to costs.
Order accordingly.
AIR 2008 SUPREME COURT 2805 "Tata Motors Ltd. v. Pharmaceutical Products of
India Ltd."
(From : Bombay)
Coram : 3 S. B. SINHA, LOKESHWAR SINGH PANTA AND MARKANDEY
KATJU, JJ.
Civil Appeal No. 3640 of 2008 (arising out of SLP (C) No. 20289 of 2006), D/- 16 -5
-2008.
Tata Motors Ltd. v. Pharmaceutical Products of India Ltd. and Anr.
(A) Sick Industrial Companies (Special Provisions) Act (1 of 1986), S.26, S.32 -
COMPANY - OBJECT OF AN ACT - HIGH COURT - SICK INDUSTRIAL
UNDERTAKING - Scope - Provisions under Act - Would prevail over provisions under
Companies Act, 1956 - Reference to BIFR in respect of sick Company - Scheme for its
revival/rehabilitation was pending under consideration - Company presented Scheme of
arrangement u/S.391 of Companies Act before Company Court/ High Court - Sanction of
Scheme by High Court - Not permissible.
Appeal No. 725 of 2006 in Company Petn. No. 470 of 2005 with Company Appln. No.
282 of 2005, D/-16-10-2006 (Bom.), Reversed.
@page-SC2806

Sick Industrial Companies (Special Provisions) Act (SICA) is a special statute. It is a self
contained Code. The jurisdiction of the Company Judge in a case where reference had
been made to BIFR would be subject to the provisions of SICA. SICA furthermore was
enacted to secure the principles specified in Art. 39 of the Constitution of India. It seeks
to give effect to the larger public interest. It should be given primacy because of its higher
public purpose. S. 26 of SICA bars the jurisdiction of the Civil Courts. (Paras 20, 22,
31)
What scheme should be prepared by the operating agency for revival and rehabilitation of
the sick industrial company is within the domain of BIFR. S. 26 not only covers orders
passed under SICA but also any matter which BIFR is empowered to determine. The
jurisdiction of civil Court is, thus, barred in respect of any matter for which the appellate
authority or the Board is empowered. The High Court may not be a civil Court but its
jurisdiction in a case of this nature is limited.
Appeal No. 725 of 2006 in Company Petn. No. 470 of 2005 with Company Appln. No.
282 of 2005, D/- 16-10-2006 (Bom.), Reversed. (Paras 22, 23, 26)
(B) Companies Act (1 of 1956), S.391, S.392, S.393, S.394 - COMPANY - OBJECT OF
AN ACT - SICK INDUSTRIAL UNDERTAKING - Scope - Scheme of arrangement in
respect of sick Company - It is not possible to harmonize provisions of S.391 to S.394
with provisions of SICA, 1985.
Sick Industrial Companies (Special Provisions) Act (1 of 1986), S.26, S.32. (Para
26)
(C) Companies Act (1 of 1956), S.391 - COMPANY - SICK INDUSTRIAL
UNDERTAKING - Scheme of arrangement - Made by Company u/S.391, Companies
Act, 1956 - Scheme provides for not only entering into arrangement as regards repayment
of debts to secured creditors and unsecured creditors - But also provides for a merger -
Such a Scheme could not be placed for approval before BIFR under SICA
Sick Industrial Companies (Special Provisions) Act (1 of 1986), S.18. (Para 28)
(D) Sick Industrial Companies (Special Provisions) Act (1 of 1986), S.18 - SICK
INDUSTRIAL UNDERTAKING - APPEAL - APPELLATE AUTHORITY -
Rehabilitation scheme - Approval - Granted by BIFR in view of observations made by
Appellate Authority - Nothing to show that order of BIFR is fair or reasonable or meets
the requirements of law - Appeal - Disposed of by AAIFR only in terms of order of High
Court in Company matter - Order of BIFR and AAIFR liable to be set aside - Matter
remitted to BIFR so as to enable it to proceed in accordance with provisions of Act,
afresh. (Paras 30, 31)
Cases Referred : Chronological Paras
(2008) Company Petn. No. 108 of 2006, D/- 25-1-2008 (reported in 2008 (3) Bom CR
78) 26
2007 AIR SCW 350 : AIR 2007 SC 683 : 2007 CLC 347 (Ref.) 21
2006 AIR SCW 1392 : AIR 2006 SC 1489 : 2006 (3) AIR Bom R 164 (Ref.) 22
2005 CLC 1602 : 2005 AIR - Kant HCR 2679 (SC) (Ref.) 13, 21
2003 AIR SCw 3088 : AIR 2003 SC 2743 26
2003 AIR SCW 3328 : AIR 2003 SC 2696 26
AIR 1998 Kant 195 21
AIR 1966 SC 135 (Ref.) 25
AIR 1962 SC 403 (Ref.) 24
R. F. Nariman, C. A. Sundaram, Abhishek M. Singhavi, Shyam Divan, Sr. Advocates,
Birendra Saraf, Amar Dave, Mrs. Nandini Gore, Ms. Sonia Nigam, S. Jayaram, Ms.
Rohni Musa, Sudanshu Batra, Abhishek Gupta, Pramit Saxena, Amit Yadav, S. V.
Deshpande, S. M. Jadhav, P. K. Manohar, Arvind Kumar Gupta and Bipin Bihari Singh,
with them for the appearing parties.
Judgement
S. B. SINHA, J. :- Leave granted.
Introduction
2. Interpretation/application of the provisions of the Sick Industrial Companies (Special
provisions) Act, 1986 (SICA) vis-avis the Companies Act, 1956 (1956 Act) is in question
in this appeal which arises out of a judgment and order dated 16th October, 2006 passed
by a Division Bench of the High Court of Judicature at Bombay in Appeal No. 725 of
2006 arising out of a judgment and order dated 13th February, 2006 passed by a learned
Single Judge of the Bombay High Court approving a Scheme filed by the respondent
herein in Company Petition No. 470 of 2005 which was under Section 391 of the 1956
Act.
Background Facts :
3. First respondent is a company registered
@page-SC2807
and incorporated under the 1956 Act. It took loan from Tata Finance Ltd., predecessor-in-
interest of the appellant on interest @ 18% per annum. Disputes and differences arose
between the parties, which were referred to arbitral tribunal. An award was passed on
30th July, 2002 in the Arbitration proceedings for a sum of Rs. 1,51,36,795/- together
with interest @ 18% per annum till payment and/or realization. It is stated that the total
amount due to the appellant from the respondent would be near about 5.7 crores of
rupees. There were other secured and unsecured creditors also.
Proceedings under SICA
4. Respondent being unable to pay the dues made a reference in terms of Section 15 of
SICA before the Board for Industrial and Financial Reconstruction (BIFR). The BIFR
appointed Industrial Development Bank of India (IDBI) as an operating agency. It
purported to have considered various schemes. However, as Unit Trust of India (UTI)
raised an objection for giving up any of its dues and there were six secured creditors and
large number of unsecured creditors, BIFR on or about 27th October, 2004 passed an
order recommending winding up of the respondent. An appeal was preferred thereagainst
before the Appellate Authority for Industrial and Financial Reconstruction (AAIFR).
5. The AAIFR granted stay of operation of the order of BIFR dated 27th October, 2004
by an order dated 13th September, 2005. Before the AAIFR two separate Schemes were
framed, one of them related to an arrangement between the respondent and M/s. Wanbury
Ltd. It agreed to settle the outstanding dues of the creditors of PPIL. But before doing so,
it thought it fit to settle all the large creditors being Financial Institutions and Banks. The
scheme envisaged payment to a class of creditors.
It was also envisaged :
"In addition, two immovable properties of the company (which were its primary and
main assets) were to be sold and the unsecured creditors were to be paid a proportion of
the sale proceeds. The balance of the sale proceeds were to be paid over to the secured
creditors.
Upon payment of the cash consideration, Wanbury was to get complete control over the
Respondent including all its assets subject to the approval of the merger before the
appropriate forum.
The scheme was to become effective upon approval of overall settlement including an
order for merger or any other mode of acquisition of assets of PPIL by Wanbury or such
scheme of PPIL by BIFR/AAIFR."
Appellant was kept outside the said Scheme. The scheme involved some selective
secured creditors and some selective unsecured creditors.
Company Court Proceedings
6. Respondent, however, filed an application before the High Court of Judicature at
Bombay purported to be in terms of Section 391 of the 1956 Act during the pendency of
the said appeal on or about 29th April, 2005. A Scheme was presented before the
Company Judge purported to be involving about 80 per cent of the creditors, most of
them being banks, financial institutions. Allegedly, even at that stage, it was not disclosed
before the Company Court that unsecured creditors listed in the Scheme were only a
selected few creditors, as a result whereof a large number of creditors had been excluded.
7. Before the Company Judge, the appellant filed an application for intervention. It filed
an objection to the said Scheme primarily on the grounds :-
"That the revival/rehabilitation of the company was under consideration of a specialized
body formed under the Sick Industries Act which is a special legislation and would
prevail over the provisions of the Companies Act.
That the non obstante clause contained in the Sick Industries Act will have the effect of
overriding and excluding the provisions of the Companies moreso where there is an
overlapping between the two Acts.
That considering the scheme of the Sick Industries Act, the revival/restructuring of the
company cannot be considered by two separate forums separately.
That the scheme involved financial reconstruction, sale of assets of the company and
merger/take over by Wanbury. These issues expressly fall within the domain of the BIFR
under Section 18 of the Sick Industries Act.
That a scheme could not be presented only in respect of selected unsecured creditors to
the exclusion of the other similarly placed unsecured creditors such as the Petitioners.
@page-SC2808
That the entire scheme was nothing but a fraud which was being played whereby the
company and its assets were being transferred to Wanbury which was associated with the
company itself."
UTI also filed an objection.
8. The said contentions of the appellant, however, were rejected by a learned Single
Judge of the High Court by his order dated 13th April, 2006 and the Scheme was
approved.
Order of the AAIFR
9. In view of the aforementioned order of the High Court, AAIFR also on or about 1st
June, 2006 approved the said Scheme opining :-
"5. Learned counsel for the Appellant Company states that the scheme of Compromise
and Arrangement approved by the Bombay High Court have been incorporated in the
scheme of revival-cum-merger submitted to IDBI (Operating Agency) in pursuance of
direction given by us on 9-11-2005.
6. In view of IDBI's recommendation of the revival-cum-merger proposal submitted by
PPIL, which is in accordance with Bombay High Court's order dated 13-2-2006, we set
aside the impugned order dated 27-10-2004 and direct BIFR to consider the scheme
vetted by the OA within a period of three months from the date of this order and take
necessary further steps for the revival of the appellant company in accordance with law."
10. An intra court appeal was preferred thereagainst by the appellant on or about 3rd
August, 2006. By reason of the impugned judgment the said Letters Patent Appeal has
been dismissed, stating :-
"2. The Appellant claims to be an unsecured creditor to the extent of Rs. 1.51 crores as set
out in the award dated 30-7-2002 with further interest at the rate of 18% per annum. It is
not in dispute that the Scheme of Arrangement approved by the learned Company Judge
between Pharmaceutical Products of India Ltd. and its unsecured creditors and Wanbury
does not affect the rights of the appellant as the appellant, though an unsecured creditor,
is not specified in Schedule-I, appended to the Scheme. In this backdrop, the impugned
order cannot be faulted. However, it is clarified that whatever objections the appellant
may have against the revival scheme pending before the BIFR, pursuant to the order
dated 1-6-2006 passed by the AAIFR, they may place their objections before the BIFR
and obviously upon such objections being placed the BIFR shall consider the revival
scheme of the respondent-Company on its own merits, keeping in view all relevant facts
and circumstances, including the objections of the appellant."
Subsequent Events
11. We may also take note of some subsequent events. In view of the order of AAIFR
dated 1st June, 2006, BIFR issued notice on 1st February, 2007 to consider the Scheme-
cum-merger with M/s. Wanbury Ltd. propounded by the respondent company returnable
on 29th March, 2007. On the said date, all the interested parties including the appellant
were heard. By an order dated 1st May, 2007, BIFR is said to have sanctioned the
Scheme-cum-merger of M/s. Wanbury Ltd. with the respondent.
12. We may also place on record that inter alia on the premise that the said Scheme of
merger was approved in gross violation of this Court's order dated 15th December, 2006,
a contempt petition was filed. We are not concerned with the said Contempt Petition
herein.
Contentions
13. Mr. R.F. Nariman, learned Senior Counsel appearing on behalf of the appellant, in
support of this appeal would submit :-
1. SICA being a special statute, the provision thereof shall prevail over the provisions of
the 1956 Act.
2. The High Court committed a manifest error in entertaining the respondent's application
for merger under Sections 391 to 394 of the Act, although the matter was pending before
the AAIFR.

3. The High Court failed to notice the binding precedent of this Court in NGEF Ltd. vs.
Chandra Developers (P) Ltd., (2005) 8 SCC 219 wherein it has clearly been held that
SICA will prevail over the 1956 Act. 2005 CLC 1602

4. The Division Bench of the High Court has failed to consider that the Company Judge
had no jurisdiction to entertain any proceeding.
5. Section 26 of the SICA bars the jurisdiction of the Company Judge.
14. Mr. C.A. Sundaram, learned senior counsel appearing on behalf of the respondent,
@page-SC2809
on the other hand would urge :-
1. The operation of the order of BIFR having been stayed, the Company Petition was
maintainable at the instance of the respondent.
2. Section 19 of SICA will have no application as it speaks of financial assistance by the
persons specified therein.
3. Section 22 of SICA must be read in the context of Section 19 thereof.
4. Section 26 or any other provision of SICA do not oust the jurisdiction of the Company
Court.
5. SICA as interpreted by this Court in NGEF Ltd. (supra) would prevail over 1956 Act
only if the provisions of the latter are inconsistent with the provisions of SICA and not
otherwise.
6. The Scheme in question being subject to approval by BIFR and that BIFR by a reason
of its order dated 1st May, 2007 had granted approval thereof, the legal requirements
must be held to have been complied with.
STATUTORY PROVISIONS SICA
15. SICA was enacted to make, in the public interest, special provisions with a view to
securing the timely detection of sick and potentially sick companies owning industrial
undertakings, the speedy determination by a Board of experts of the preventive,
ameliorative, remedial and other measures which need to be taken with respect to such
companies and the expeditious enforcement of the measures so determined and for
matters connected therewith or incidental thereto.
16. Section 15 of SICA provides for making reference by the Board of Directors of the
Company on becoming an industrial company, a sick industrial company, to the Board for
determination of the measures to be adopted with respect to the company. Section 16
provides for making inquiry into the working of sick industrial company by the Board
after receiving reference. Section 17 provides for powers of Board to make suitable order
on the completion of inquiry. Sub-section (3) thereof reads as under :-
"17. Powers of Board to make suitable order on the completion of inquiry.
(3) If the Board decides under sub-section (1) that it is not practicable for a sick industrial
company to make its net worth exceed the accumulated losses within a reasonable time
and that it is necessary or expedient in the public interest to adopt all or any of the
measures specified in section 18 in relation to the said company it may, as soon as may
be, by order in writing, direct any operating agency specified in the order to prepare,
having regard to such guidelines as may be specified in the order, a scheme providing for
such measures in relation to such company."
17. Section 18 provides for preparation and sanction of Scheme. Sections 18(1)(c), 18(3)
and 18(6A) read as under :-
"Section 18 - Preparation and sanction of Schemes
(1) Where an order is made under sub-section (3) of section 17 in relation to any sick
industrial company, the operating agency specified in the order shall prepare, as
expeditiously as possible and ordinarily within a period of ninety days from the date of
such order, a scheme with respect to such company providing for any one or more of the
following measures, namely :-
(c) the amalgamation of -
(i) the sick industrial company with any other company, or
(ii) any other company with the sick industrial company;
(hereafter in this section, in the case of sub-clause (i), the other company, and in the case
of sub-clause (ii), the sick industrial company, referred to as "transferee company");
(3) (a) The Scheme prepared by the operating agency shall be examined by the Board and
a copy of the scheme with modification, if any, made by the Board shall be sent, in draft,
to the sick industrial company and the operating agency and in the case of amalgamation,
also to any other company concerned, and the Board shall publish or cause to be
published the draft scheme in brief in such daily newspapers as the Board may consider
necessary, for suggestions and objections, if any, within such period as the Board may
specify.
(b) The Board may make such modifications, if any, in the draft scheme as it may
consider necessary in the light of the suggestions and objections received from the sick
industrial company and the operating
@page-SC2810
agency and also from the transferee industrial company and any other company
concerned in the amalgamation and from any shareholder or any creditors or employees
of such companies :
Provided that where the scheme relates to amalgamation the said scheme shall be laid
before the company other than the sick industrial company in the general meeting for the
approval of the scheme by its shareholders and no such scheme shall be proceeded with
unless it has been approved, with or without modification, by a special resolution passed
by the shareholders of the company other than the sick industrial company.
(6A) Where a sanctioned scheme provides for the transfer of any property or liability of
the sick industrial company in favour of any other company or person or where such
scheme provides for the transfer of any property or liability of any other company or
person in favour of the sick industrial company, then, by virtue of, and to the extent
provided in, the scheme, on and from the date of coming into operation of the sanctioned
scheme or any provision thereof, the property shall be transferred to, and vest in, and the
liability shall become the liability of, such other company or person or, as the case may
be, the sick industrial company."
18. Section 19 provides for rehabilitation by giving financial assistance; sub-sections (1),
(2) and (4) whereof reads as under :-
"Section 19 - Rehabilitation by giving financial assistance.- (1) Where the scheme relates
to preventive, ameliorative, remedial and other measures with respect to any sick
industrial company, the scheme may provide for financial assistance by way of loans,
advances or guarantees or reliefs or concessions or sacrifices from the Central
Government, a State Government, any scheduled bank or other bank, a public financial
institution or State level institution or any institution or other authority (any Government,
bank, institution or other authority required by a scheme to provide for such financial
assistance being hereafter in this section referred to as the person required by the scheme
to provide financial assistance) to the sick industrial company.
(2) Every scheme referred to in sub-section (1) shall be circulated to every person
required by the scheme to provide financial assistance for his consent within a period of
sixty days from the date of such circulation or within such further period, not exceeding
sixty days, as may be allowed by the Board, and if no consent is received within such
period or further period, it shall be deemed that consent has been given.
(4) Where in respect of any scheme consent under sub-section (2) is not given by any
person required by the scheme to provide financial assistance, the Board may adopt such
other measures, including the winding up of the sick industrial company, as it may deem
fit."
Sections 20, 26 and 32 of SICA read as under :-
"Section 20 - Winding up of sick industrial company.- (1) Where the Board, after making
inquiry under section 16 and after consideration of all the relevant facts and
circumstances and after giving an opportunity of being heard to all concerned parties, is
of opinion that the sick industrial company is not likely to make its net worth exceed the
accumulated losses within a reasonable time while meeting all its financial obligations
and that the company as a result thereof is not likely to become viable in future and that it
is just and equitable that the company should be wound up, it may record and forward its
opinion to the concerned High Court.
(2) The High Court shall, on the basis of the opinion of the Board, order winding up of
the sick industrial company and may proceed and cause to proceed with the winding up
of the sick industrial company in accordance with the provisions of the Companies Act,
1956 (1 of 1956).
(3) For the purpose of winding up of the sick industrial company, the High Court may
appoint any officer of the operating agency, if the operating agency gives its consent, as
the liquidator of the sick industrial company and the officer so appointed shall for the
purposes of the winding up of the sick industrial company be deemed to be, and have all
the powers of, the official liquidator under the Companies Act, 1956 (1 of 1956).
(4) Notwithstanding anything contained in sub-section (2) or sub-section (3), the Board
may cause to be sold the assets of the sick industrial company in such manner as it may
deem fit and forward the sale
@page-SC2811
proceeds to the High Court for orders for distribution in accordance with the provisions
of section 529A, and other provisions of the Companies Act, 1956 (1 of 1956).
Section 26 - Bar of jurisdiction.- No order passed or proposal made under this Act shall
be appealable except as provided therein and no civil court shall have jurisdiction in
respect of any matter which the Appellate Authority or the Board is empowered by, or
under, this Act to determine and no injunction shall be granted by any court or other
authority in respect of any action taken or to be taken in pursuance of any power
conferred by or under this Act.
Section 32 - Effect of the Act on other laws.- (1) The provisions of this Act and of any
rules or schemes made thereunder shall have effect notwithstanding anything inconsistent
therewith contained in any other law except the provisions of the Foreign Exchange
Regulation Act, 1973 (46 of 1973) and the Urban Land (Ceiling and Regulation) Act,
1976 (33 of 1976) for the time being in force or in the Memorandum or Articles of
Association of an industrial company or in any other instrument having effect by virtue of
any law other than this Act.
(2) Where there has been under any scheme under this Act an amalgamation of a sick
industrial company with another company, the provisions of section 72A of the Income-
tax Act, 1961 (43 of 1961), shall, subject to the modifications that the power of the
Central Government under that section may be exercised by the Board without the
Central Government under that section may be exercised by the Board without any
recommendation by the specified authority referred to in that section, apply in relation to
such amalgamation as they apply in relation to the amalgamation of a company owning
an industrial undertaking with another company."
The Companies Act, 1956
Section 391 of the Companies Act, 1956 reads as under :-
Section 391 - Power to compromise or make arrangements with creditors and members.-
(1) Where a compromise or arrangement is proposed -
(a) between a company and its creditors or any class of them; or
(b) between a company and its members or any class of them, the Tribunal may, on the
application of the company or of any creditor or member of the company or, in the case
of a company which is being wound up, of the liquidator, order a meeting of the creditors
or class of creditors, or of the members or class of members, as the case may be to be
called, held and conducted in such manner as the Tribunal directs.
(2) If a majority in number representing three-fourths in value of the creditors, or class of
creditors, or members, or class of members as the case may be, present and voting either
in person or, where proxies are allowed under the rules made under section 643, by proxy,
at the meeting, agree to any compromise or arrangement, the compromise or arrangement
shall, if sanctioned by the Tribunal, be binding on all the creditors, all the creditors of the
class, all the members, or all the members of the class, as the case may be, and also on
the company, or, in the case of a company which is being wound up, on the liquidator and
contributories of the company :
Provided that no order sanctioning any compromise or arrangement shall be made by the
Tribunal unless the Tribunal is satisfied that the company or any other person by whom
an application has been made under sub-section (1) has disclosed to the court, by affidavit
or otherwise, all material facts relating to the company, such as the latest financial
position of the company, the latest auditor's report on the accounts of the company, the
pendency of any investigation proceedings in relation to the company under sections 235
to 351, and the like.
(3) An order made by the Tribunal under sub-section (2) shall have no effect until a
certified copy of the order has been filed with the Registrar.
(4) A copy of every such order shall be annexed to every copy of the memorandum of the
company issued after the certified copy of the order has been filed as aforesaid, or in the
case of a company not having a memorandum, to every copy so issued of the instrument
constituting or defining the constitution of the company.
(5) If default is made in complying with sub-section (4), the company; and every officer
of the company who is in default, shall be punishable with fine which may extend to one
hundred rupees for each copy in respect
@page-SC2812
of which default is made.
(6) The Tribunal may, at any time after an application has been made to it under this
section stay the commencement or continuation of any suit or proceeding against the
company on such terms as the Tribunal thinks fit, until the application is finally disposed
of."
Interpretation of the Statutory Provisions
19. It was conceded by Mr. Sundaram SICA being a special law vis-a-vis the 1956 Act, it
shall prevail over the latter. The learned counsel, however, qualifies his submission by
contending that SICA only excludes the provisions of the Companies Act when they are
inconsistent with each other.
The provisions of a special Act will override the provisions of a general Act. A latter of it
will override an earlier Act. 1956 Act is a general Act. It consolidates and restates the law
relating to companies and certain other associations. It is prior in point of time to SICA.
Wherever any inconsistency is seen in the provisions of the two Acts, SICA would
prevail. SICA furthermore is a complete Code. It contains a non obstante clause in
Section 32.
20. SICA is a special statute. It is a selfcontained Code. The jurisdiction of the Company
Judge in a case where reference had been made to BIFR would be subject to the
provisions of SICA.
We may, at this stage, notice the effect of SICA vis-a-vis the other Acts, as has been
noticed by this Court in some of its judgments
21

. In NGEF Ltd. vs. Chandra Developers (P) Ltd., (2005) 8 SCC 219, in regard to the
jurisdiction of the Company Court it was held :- 2005 CLC 1602

"20. Mr. K.K. Venugopal, the learned Senior Counsel, would submit that having regard to
sub-section (2) of Section 536 of the Companies Act, the High Court has the jurisdiction
to permit sale of assets of the Company even before passing of the winding-up order, in
relation whereto Section 20(4) of SICA will have no application.
23. The provisions relating to winding up by the courts occur in Chapter II of the
Companies Act, 1956. Section 433 of the Act enumerates the circumstances in which the
company may be wound up by the court including the inability on the part of the
company to pay its debts. Section 441 of the Act specifies as to when the proceeding for
winding-up of a company by the court shall commence at the time of the presentation of
the petition for the winding-up.
In a case, however, where winding-up proceedings are initiated in terms of
recommendations made by BIFR or AAIFR, as the case may be, no such petition is
required to be presented. Section 443 lays down the power of a court on hearing petition;
clause (d) of sub-section (1) whereof provides for a power to make an order for winding
up of the company with or without costs or any other order that it thinks fit. Section 444
lays down the consequences of the winding-up order. In terms of Section 446 of the Act,
in the event of passing of a winding-up order or appointment of liquidator as Provisional
Liquidator, no suit or legal proceeding would commence or if pending at the date of the
winding-up order, shall not be proceeded with against the company except by leave of the
court and subject to such terms as the court may impose. Sub-section (2) of Section 446
provides for a non obstante clause, in terms whereof the Company Court shall have
jurisdiction to entertain or dispose of any suit or proceedings specified therein. Section
451 lays down general provisions as to liquidators. Section 457 specifies the power of the
liquidator which is required to be exercised with the sanction of the court. Sub-section (2)
of Section 536 reads as under :
"536. Avoidance of transfers, etc., after commencement of winding up.- (1)
***
(2) In the case of a winding up by the Tribunal, any disposition of the property (including
actionable claims) of the company, and any transfer of shares in the company or
alteration in the status of its members, made after the commencement of the winding up,
shall, unless the Tribunal otherwise orders, be void."
In regard to jurisdiction of the Company Court it was held :-
"39. The provisions of SICA contain non obstante clauses. It is a special statute. It is a
complete Code in itself. The jurisdiction of the Company Court in such matters would
arise only when BIFR or AAIFR, as the case may be, has exercised its jurisdiction under
@page-SC2813
Section 20 of SICA recommending winding up of the Company upon arriving at a finding
that there does not exist any chance of revival of the Company." It was furthermore held :

"40. Mr. Venugopal has placed reliance upon a decision of a learned Single Judge of the
Karnataka High Court in Karnataka State Industrial Investment and Development Corpn.
Ltd. v. Intermodal Transport Technology Systems, for the proposition that despite the fact
BIFR retains jurisdiction to get the assets of a sick company sold in terms of sub-section
(4) of Section 20 of SICA; still the leave of the Company Court, therefor would be
required. The said decision, however, has been reversed by the Division Bench of the
Karnataka High Court in BPL Ltd. v. Intermodal Transport Technology Systems
(Karnataka) Ltd. holding that the Company Court has no such jurisdiction. We generally
accept the views of the Division Bench. AIR 1998 Kant 195
41. It is difficult to accept the submission of the learned counsel appearing on behalf of
the respondents that both the Company Court and BIFR exercise concurrent jurisdiction.
If such a construction is upheld, there shall be chaos and confusion. A company declared
to be sick in terms of the provisions of SICA, continues to be sick unless it is directed to
be wound up. Till the company remains a sick company having regard to the provisions
of sub-section (4) of Section 20, BIFR alone shall have jurisdiction as regards sale of its
assets till an order of winding up is passed by a Company Court.
42. Apart from the fact that sub-section (4) of Section 20 contains a non obstante clause
and, thus, it shall prevail over the provisions contained in sub-section (2). The said Act is
also a latter statute.
43. The provisions of SICA would prevail over the provisions of the Companies Act.
Section 20 of SICA relates to winding up of the sick industrial company. Before BIFR or
AAIFR, as the case may be, makes a recommendation for winding up of the Company, an
enquiry is made in terms of Section 16 thereof wherefor all relevant facts and
circumstances are required to be taken into consideration. Before an opinion is arrived at
in that behalf, the parties are given an opportunity of hearing. The satisfaction arrived at
by BIFR that the Company is not likely to become viable in future and it is just and
equitable that the Company should be wound up must be based on objective criteria. The
High Court indisputably on receipt of such recommendation of BIFR would initiate a
proceeding for winding-up in terms of Section 433 of the Companies Act. Subsection (2)
of Section 536 ipso facto does not confer any jurisdiction upon the Company Court to
direct sale of the assets of the sick company. It has to exercise its power thereunder
subject to the provisions of the special statute governing the field. Despite the fact that
the procedures laid down under the Companies Act would be applicable therefor but they
must be read with subsection (4) of Section 20 of SICA which contains a non obstante
clause and in terms thereof, BIFR is authorised to sell the assets of the sick industrial
company in such a manner as it may deem fit. By reason of the said provision, BIFR is
also empowered to forward the sale proceeds to the High Court for orders for distribution
in accordance with Section 529-A and other provisions of the Companies Act which in no
uncertain terms would mean that the distribution of the sale proceeds would be for the
purpose of meeting the claims of the creditors in the manner laid down therein. The
intention of Parliament in enacting the said provision becomes clear as in terms of
Section 22-A of SICA, BIFR is empowered to issue any direction in the interest of the
sick industrial company or its creditors or shareholders and direct the sick industrial
company not to dispose of its assets except with its assent. Section 32, as noticed
hereinbefore, again contains a non obstante clause. The scheme suggests that BIFR
retains control over the assets of the Company and in terms of the aforementioned
provisions may either prevent any sale or permit any sale of the assets of the sick
industrial company. Such a power in BIFR remains till a winding-up order is passed by
the High Court and a stage arrives for the High Court for issuing orders for distribution of
the sale proceeds.
44. SICA was furthermore enacted subsequent to the provisions of the Companies Act. It
is not, thus, possible to accept the submission that the High Court exercises a concurrent
jurisdiction."
@page-SC2814
It was ruled that the Company Court and the BIFR do not exercise concurrent
jurisdiction, holding :-
"45. It may be true that the High Court's jurisdiction is that of the Appellate Authority but
keeping in view the terminology contained in sub-section (4) of Section 20 read with
Section 32 of the Act, it leaves no manner of doubt that the provisions of SICA shall
prevail over the provisions of the Companies Act. For the aforementioned purpose, it was
not necessary for Parliament to mention specifically the provisions of sub-section (4) of
Section 20 that the same shall prevail over Section 536 of the Companies Act, as was
suggested by the learned counsel appearing for the first respondent. The construction of
the provisions of both the Acts, as suggested by the learned counsel, that both the
provisions of sub-section (4) of Section 20 and Section 536 should be read conjointly so
as to enable an applicant to obtain a sanction of both BIFR and the Company Court, thus,
do not appeal to us." The Court noticed the non obstante clause contained in clause (4) of
Section 20 as also Section 32 of SICA to hold that the High Court does not exercise
concurrent jurisdiction with BIFR The fact that SICA was enacted in 1984 had also been
taken into consideration.
The Court considered in details the exercise of the jurisdiction of the Company Court vis-
a-vis the BIFR to opine :-
"69. BIFR admittedly had the power to sell the assets of the Company but the High Court
until a winding-up order is issued does not have the same. BIFR in its order dated 24-8-
2002 might have made an observation to the effect that the Company may approach the
High Court in case it intended to dispose of its property by private negotiation but the
same would not mean that BIFR could delegate its power in favour of the High Court.
BIFR being a statutory authority, in the absence of any provision empowering it to
delegate its power in favour of any other authority had no jurisdiction to do so.
"Delegatus non potest delegare" is a well-known maxim which means unless expressly
authorised a delegatee cannot sub-delegate its power. Moreover, the said observations of
BIFR would only mean that the Company Court could exercise its power in accordance
with law and not dehors it. If the Company Court had no jurisdiction to pass the
impugned order, it could not derive any jurisdiction only because BIFR said so."

(See also Morgan Securities and Credit Pvt. Ltd. v. Modi Rubber Ltd. [AIR 2007 SC
683]) 2007 AIR SCW 350

22

. The principle laid down therein has been reiterated in Bombay Dyeing and
Manufacturing Co. Ltd. vs. Bombay Environmental Action Group, (2006) 3 SCC 434,
stating : 2006 AIR SCW 1392

"13. The 1993 Act was enacted to provide for and regulate the payment of interest on
delayed payments to small-scale and ancillary industrial undertakings and for matters
connected therewith.
14. The provisions of the 1993 Act, therefore, do not envisage a situation where an
industrial company becomes sick and requires framing of a scheme for its revival.
15. It is no doubt true that an award in relation to a claim of a small-scale industry if
made by the Council would be governed by the provisions of the Arbitration and
Conciliation Act, 1996 (for short "the 1996 Act")."
SICA furthermore was enacted to secure the principles specified in Article 39 of the
Constitution of India. It seeks to give effect to the larger public interest. It should be
given primacy because of its higher public purpose. Section 26 of SICA bars the
jurisdiction of the Civil Courts.
What scheme should be prepared by the operating agency for revival and rehabilitation of
the sick industrial company is within the domain of BIFR. Section 26 not only covers
orders passed under SICA but also any matter which BIFR is empowered to determine.
23. The jurisdiction of civil court is, thus, barred in respect of any matter for which the
appellate authority or the Board is empowered. The High Court may not be a civil court
but its jurisdiction in a case of this nature is limited.
24

. Our attention has been drawn to the decision of this Court in Jyoti Bhushan Gupta v.
Banaras Bank Ltd, [(1962) Supp 1 SCR 73] Where the question which arose for
consideration was as to whether Article 183 of the Limitation Act shall have any
application in regard to the applicability of the provisions of the Limitation Act, it was
stated : AIR 1962 SC 403, Paras 8 and 9

@page-SC2815
"By the Companies Act of 1913, the High Court was invested with jurisdiction to order
payment of the amounts due by debtors of companies ordered to be wound up. This
jurisdiction may be invoked as of right against all persons whose names are placed on the
list of contributories. The jurisdiction is ordinary : it does not depend on any
extraordinary action on the part of the High Court.
The jurisdiction is also original in character because the petition for exercise of the
jurisdiction is entertainable by the High Court as a court of first instance and not in
exercise of its appellate jurisdiction. Again by S. 187 no special jurisdiction is conferred.
The High Court adjudicates upon the liability of the debtor to pay debts due by him to the
Company : the jurisdiction is therefore civil. Normally, a creditor has to file a suit to
enforce liability for payment of a debt due to him from his debtor. The Legislature has by
S. 187 of the Companies Act empowered the High Court in a summary proceeding to
determine the liability and to pass an order for payment but on that account the real
character of the jurisdiction exercised by the High Court is not altered. Nor is there any
substance in the contention that the authority to order payment of a debt under S. 187 is
merely a power of the High Court and not its jurisdiction. By S. 3 read with S. 187 of the
Companies Act the High Court has jurisdiction to direct payment of the amount due by a
contributory : and an order passed for payment manifestly is an order passed in exercise
of the jurisdiction vested in the High Court by S. 3 read with S. 187 of the Companies
Act."
It was furthermore observed:
"The jurisdiction to deal with the claims of companies ordered to be wound up is
conferred by the Indian Companies Act and to that extent the Letters Patent are modified.
There is, however, no difference in the character of the original civil jurisdiction which is
conferred upon the High Court by Letters Patent and the jurisdiction conferred by special
Acts. When in exercise of its authority conferred by a special statute the High Court in an
application presented to it as a court of first instance declares liability to pay a debt, the
jurisdiction exercised is original and civil and if the exercise of that jurisdiction does not
depend upon any preliminary step invoking exercise of discretion of the High Court, the
jurisdiction is ordinary."
25. In Damji Valji Shah v. Life Insurance Corporation of India, [(1965) 2 SCR 665], the
question which arose for consideration was as to whether a similar provision made in the
Life Insurance Corporation Act, 1956 shall bar the jurisdiction of the Company Court in
terms of Section 446 (1) of the Companies Act. Referring to Section 41 of the Life
Insurance Corporation Act, 1956 it was stated that the Tribunal constituted under the LIC
Act will have exclusive jurisdiction. It was opined :-
"20. It is in view of the exclusive jurisdiction which sub-section (2) of S. 446 of the
Companies Act confers on the company Court to entertain or dispose of any suit or
proceeding by or against a company or any claim made by or against it that the restriction
referred to in sub-section (1) has been imposed on the commencement of the proceedings
or proceeding with such proceedings against a company after a winding-up order has
been made. In view of S. 41 of the LIC Act the company Court has no jurisdiction to
entertain and adjudicate upon any matter which the Tribunal is empowered to decide or
determine under that Act. It is not disputed that the Tribunal has jurisdiction under the Act
to entertain and decide matters raised in the petition filed by the Corporation under S. 15
of the LIC Act. It must follow that the consequential provisions of sub-section (1) of S.
446 of the Companies Act will not operate on the proceedings which may be pending
before the Tribunal or which may be sought to be commenced before it."
26. What in this case, however, has been contended is that BIFR had no jurisdiction to
make a scheme as envisaged under Section 391 of the Act. Even otherwise, 'civil court'
has a definite connotation. The jurisdiction of the Company Court is now vested in the
Tribunal. Therefore, it will be difficult to hold, in view of a changed situation, that
Section 26 ousts the jurisdiction of the Company Court in totality. The decision, however,
also says that the special statute shall prevail over the general rule.

Although it may not be very relevant, we may notice that this Court in Dwarka Prasad
Agarwal v. Ramesh Chander Agarwal, [(2003) 6 SCC 220] opined as under :- 2003
AIR SCW 3328

@page-SC2816

"22. The dispute between the parties was eminently a civil dispute and not a dispute
under the provisions of the Companies Act. Section 9 of the Code of Civil Procedure
confers jurisdiction upon the civil courts to determine all disputes of civil nature unless
the same is barred under a statute either expressly or by necessary implication. Bar of
jurisdiction of a civil court is not to be readily inferred. A provision seeking to bar
jurisdiction of a civil court requires strict interpretation. The court, it is well settled,
would normally lean in favour of construction, which would uphold retention of
jurisdiction of the civil court. The burden of proof in this behalf shall be on the party who
asserts that the civil court's jurisdiction is ousted. (See Sahebgouda v. Ogeppa). Even
otherwise, the civil court's jurisdiction is not completely ousted under the Companies Act,
1956." 2003 AIR SCW 3088

We are, therefore, of the opinion that the judgment of the High Court cannot be sustained.
We may furthermore notice that the decision of the learned single Judge has been
overruled by a Division Bench of the Bombay High Court in Ashok Organics Industries
Ltd. v. Dena Bank (Company Petition No. 108 of 2006, disposed of on 25-1-2008).
It is also not possible to harmonize the provisions of Sections 391 to 394 of the 1956 Act
with the provisions of SICA.
For the views we have taken, it is not necessary to consider the other contentions raised at
the bar.
27. The question, however, is what relief should be granted in view of the subsequent
events. Various intervention applications have been filed. We do not intend to make any
observation in regard thereto. We are, however, of the opinion that it is a fit case where
we should exercise our jurisdiction under Section 142 of the Constitution of India to meet
the object for which the Act has been enacted.
28. We have been taken through the Scheme. The Scheme provides for not only entering
into an arrangement as regards repayment of debts to secured creditors and unsecured
creditors but also provides for a merger, subject of course, to an appropriate order being
passed by BIFR. The question is as to whether such a Scheme could be placed for
approval before BIFR. We are of the view that it could not be. Before BIFR could
approve a scheme, the same must be drawn in terms of the provisions of the Act and not
dehors the scheme. It is required to apply its own mind. The operating agency is supposed
to make a scheme. The operating agency before the AAIFR took one stand; before us it
has taken another. According to it, it was not involved in the preparation of the Scheme. It
had no occasion to apply its own mind. Furthermore, after the learned Single Judge
passed its order, AAIFR disposed of the appeal only in terms of the order of the High
Court stating :-
"In view of IDBI's recommendation of the revival-cum-merger proposal submitted by
PPIL, which is in accordance with Bombay High Court's order dated 13-2-2006, we set
aside the impugned order dated 27-10-2004 and direct BIFR to consider the scheme
vetted by the OA within a period of three months from the date of this order and take
necessary further steps for the revival of the appellant-company in accordance with law."
29. The order of BIFR dated 1st May, 2007 also clearly show that it has granted its
approval in view of the observations made by the appellate authority. It might have done
so keeping in view the doctrine of judicial discipline in mind.
30. The order of BIFR is not an outcome of any pre-application of mind. There is no
finding that it has taken into consideration all the relevant facts. There is nothing to show
that such an order is fair or reasonable or meets the requirements of law.
31. We are, therefore, of the opinion that not only the judgment of the High Court but
also the orders of BIFR as also the AAIFR should be set aside and the matter should be
remitted to the BIFR so as to enable it to proceed in accordance with the provisions of
SICA afresh.
32. The appeal is allowed with the aforementioned observations and directions. In the
facts and circumstances of the case, there shall be no order as to costs.
Appeal allowed.
AIR 2008 SUPREME COURT 2817 "Benny Thomas v. Food Inspector, Kochi"
(From : Kerala)*
Coram : 2 Dr. A. PASAYAT AND P. SATHASIVAM, JJ.
Criminal Appeal No. 998 of 2008 (arising out of SLP (Cri.) No. 2226 of 2007), D/- 7 -7
-2008.
Benny Thomas v. Food Inspector, Kochi and Anr.
Prevention of Food Adulteration Act (37 of 1954), S.16, S.7 - Prevention of Food
Adulteration Rules (1955), R.17, R.18 - FOOD ADULTERATION - SALE -
EXAMINATION OF ACCUSED - SENTENCE IMPOSITION - Sale of adulterated food
- 'Sarbath' purchased from accused by Food Inspector - Found to be adulterated both by
Public Analyst and Central Food Laboratory - No irregularity found in manner of
dispatch of sample to Public Analyst and Local (Health) Authority - Plea by accused that
'Sarbath' was not sold - Has no substance since sale was admitted by accused in his
examination u/S.313, Criminal P.C. - Voucher issued by Food Inspector also proved sale -
Conviction of accused proper - Sentence of 6 months' simple imprisonment imposed not
harsh. (Paras 10, 11)

K. Rajeev, for Appellant; R. Sathish, for Respondents.


* Cri. Revn. Petn. No. 1917 of 2004, D/- 25-5-2006 (Ker).
Judgement
1. Dr. ARIJIT PASAYAT, J. :-Leave granted.
2. Challenge in this appeal is to the order passed by a learned Single Judge of the Kerala
High Court dismissing the Criminal Revision Petition which was filed questioning
correctness of the conviction for offence punishable under Section 16(1)(a)(i) read with
Section 7(1) and Section 2(ia)(m) of the Prevention of Food Adulteration Act, 1954 (in
short the 'Act') and also under Rule 5 read with Appendix B, Item A.07.08 and Rule 50 of
the Prevention of Food Adulteration Rules, 1955 (in short the 'Rules'). The appellant was
sentenced to undergo simple imprisonment for one year and to pay a fine of Rs. 2,000/-
with default stipulation as recorded by learned Judicial Magistrate, 1st Class, Kochi. The
learned IV Addl. Sessions Judge, Ernakulam in appeal modified the sentence and reduced
it to simple imprisonment for six months and a fine of Rs. 1,000/- with default
stipulation.
3. Background facts in a nutshell are as follows:
On 22.5.2000 at about 4.00 p.m., the Food Inspector, P.W. 1 inspected the shop of the
appellant by name "Bejoy Fruits and Vegetables". He found four bottles of Sarbath
(synthetic syrup) each of 700 ml. capacity, which were kept for sale. He bought one bottle
of synthetic syrup, on paying Rs. 40/-, Ex.P.4 being the voucher for payment. He sampled
it according to the procedure. After analysis, he obtained Ex.P. 12 report, which showed
that the sample did not conform to the standards prescribed under the rules and, therefore,
was adulterated. Accordingly, he proceeded against the appellant. Since accused abjured
guilt, trial was held.
4. Four witnesses were examined and 21 documents were marked on the side of the
prosecution and three documents were marked on the side of the defence. After
appreciation of the evidence, the appellant was found guilty, convicted and sentenced
accordingly. Appeal by appellant resulted only in reduction of sentence. The revision
petition did not bring any relief.
5. Stand before the High Court was that articles purchased by the Food Inspector (PW-1)
were not kept for sale and as such the same were not the food articles. It was further
submitted that there was no enquiry made by the Food Inspector as to whether these
articles were kept for sale. The High Court did not accept the stand. Referring to the
evidence of PW-1, it noted that PW-1 had introduced himself as Food Inspector and had
expressed willingness to purchase 700 ml. of 'synthetic syrup (Sarbath)' which was kept
for sale. He had purchased it after giving Rs. 40/-. The High Court noted that if the
articles were not kept for sale the question of selling it to the Food Inspector does not
arise. It found that the articles purchased were for human consumable and were kept for
sale and on analysis did not conform to the requirement. It held that there was ho
violation of Rules 17 and 18 of the Rules as claimed. Noting that the minimum sentence
has been imposed, revision petition was dismissed.
6. Learned counsel for the appellant submitted that no enquiry was conducted of the
samples as to whether articles were kept for sale. It was further submitted that the articles
were not meant for sale and, therefore,
@page-SC2818
the said Rules have no application.
7. Learned counsel for the respondent-State on the other hand supported the impugned
order. Admittedly, the sample was collected by the Food Inspector after effecting
purchase and had given the receipt. As rightly noted by the High Court the articles were
intended for sale.
8. The complainant, Food Inspector, Cochin Circle has given evidence as PW1. He has
spoken about Sarbath, the food article involved in this case from the shop of the accused
and also the various formalities done by him in sampling the same. The fact that the
Sarbath was purchased from him is not disputed by the accused. When he was questioned
under section 313 of the Code of Criminal Procedure, 1973 (in short the 'Cr.P.C.') he
conceded that he had sold sarbath to PW1. Further the sale of sarbath to PW1 is proved
by Ex. P4 voucher issued by the accused towards the purchase and acceptance of its cost
from PW1. He had also given Ex. P3 Form VI notice to the accused, the receipt of which
has been acknowledged by him as per Ex. P3 (a) endorsement and signature. As PW1, the
Food Inspector has stated that he had disclosed to the accused the intention of the
purchase of sarbath from him, what is contended by the accused is that the sarbath
purchased from him was not intended for sale as such. According to PW1, the purchased
sarbath was sampled by him at the spot as provided in the Rules, and one part of the
sample prepared by him was sent to the Public Analyst for analysis and the remaining
two parts of the sample were forwarded to the Local (Health) Authority, and received the
result of the analysis of the sample from the Public Analyst, through Local (Health)
Authority. Ex. P12 is the report of the Public Analyst, as per which the sample does not
conform to the standards prescribed for sarbath under the Rules and so the sample is
adulterated. On receipt of the intimation regarding the launching of prosecution against
him, the accused filed a petition before the court below seeking to send one part of the
sample kept with the Local (Health) Authority to the Central Food Laboratory for
analysis. Accordingly, one part of the sample was called for from the Local (Health)
Authority and sent to Central Food Laboratory. Ex. P17 is the report obtained from
Central Food Laboratory, as per which the sample does not conform the standard
prescribed for sarbath under the rules and is, therefore, adulterated. Thus the prosecution
has been able to establish that the sarbath purchased from the accused by PW1 is
adulterated.
9. One of the contentions of the appellant/accused is that the Food Inspector had violated
the mandatory provisions contained in Rules 17 and 18 of the Rules and so he is eligible
for an acquittal. Rules 17 and 18 of Rules are as follows:
"17. Manner of dispatching containers of samples :- The containers of the sample shall be
dispatched in the following manner, namely:
a) The sealed container of one part of the sample for analysis and a memorandum in
Form VII shall be sent in a sealed packet to the Public Analyst immediately but not later
than the succeeding working day by any suitable means:
b) The sealed containers of the remaining two parts of the sample and two copies of the
memorandum in Form VII shall be sent in a sealed packet to the Local (Health) Authority
immediately but not later than the succeeding working day by any suitable means:
(c) The sealed container of one of the remaining two parts of the sample and a copy of the
memorandum in Form VII kept with the Local (Health) Authority shall within a period of
7 days be sent to the Public Analyst on requisition made by him to it by any suitable
means:
Provided that in the case of a sample of food which has been taken from container
bearing Agmark seal, the memorandum in Form VII shall contain the following
additional information, namely:
a) Grade
b) Agmark Label No. /Batch No.
c) Name of Packing station
18. Memorandum and impression of seal to be sent separately: A copy of the
memorandum and specimen impression of the seal used to seal the packet shall be sent, in
a sealed packet separately to the Public Analyst by any suitable means immediately but
not later than the succeeding working day."
10. From the evidence of PW-1 it is clear that at one point of time the sample was handed
over to the Public Analyst on 23.5.2000 i.e. the succeeding day of taking the sample from
the shop of the accused. PW-1 also stated that other two parts of the
@page-SC2819
sample along with Form No.VII Memorandum and the specimen impression of the seal
used to seal the sample bottles were handed over to the Local (Health) Authority by PW-1
and copy of the information had been given to PW-2, the Local (Health) Authority. PW-2
stated in his evidence that he had received two parts of the sample along with Form VII
and the specimen impression of the seal used to seal the sample in separate sealed cover.
Therefore, as rightly held by the High Court there was no violation of Rules 17 and 18 of
the Rules.
11. The accused in his examination under Section 313 of the Cr.P.C. admitted that he had
sold the articles in question to PW-1. The plea that the articles were not intended for sale
has no substance as noted above. The sentence imposed as afore-noted is minimum and,
therefore, the plea, that the sentence is harsh, has no substance.
12. Looked at from any angle, the appeal is without merit, deserves dismissal, which we
direct.
Appeal dismissed.
AIR 2008 SUPREME COURT 2819 "Kusuma Ankama Rao v. State of A. P."
(From : Andhra Pradesh)*
Coram : 2 Dr. A. PASAYAT AND P. SATHASIVAM, JJ.
Criminal Appeal No. 185 of 2005, D/- 7 -7 -2008.
Kusuma Ankama Rao v. State of A.P.
(A) Evidence Act (1 of 1872), S.24 - CONFESSION - Extra-judicial confession -
Conviction on basis of - Permissible only after subjecting witness to whom it is made, to
a rigorous test of credibility.
An extra-judicial confession, if voluntary and true and made in a fit state of mind, can be
relied upon by the Court. The confession will have to be proved like any other fact. The
value of the evidence as to confession, like any other evidence, depends upon the veracity
of the witness to whom it has been made. The value of the evidence as to the confession
depends on the reliability of the witness who gives the evidence. It is not open to any
Court to start with a presumption that extra-judicial confession is a weak type of
evidence. It would depend on the nature of the circumstances, the time when the
confession was made and the credibility of the witnesses who speak to such a confession.
Such a confession can be relied upon and conviction can be founded thereon if the
evidence about the confession comes from the mouth of witnesses who appear to be
unbiased, not even remotely inimical to the accused, and in respect of whom nothing is
brought out which may tend to indicate that he may have a motive of attributing an
untruthful statement to the accused, the words spoken to by the witness are clear,
unambiguous and unmistakably convey that the accused is the perpetrator of the crime
and nothing is omitted by the witness which may militate against it. After subjecting the
evidence of the witness to rigorous test on the touchstone of credibility, the extra-judicial
confession can be accepted and can be the basis of a conviction if it passes the test of
credibility. (Para 18)
(B) Evidence Act (1 of 1872), S.24 - CONFESSION - Confession - Voluntariness -
Whether accused at time of making it was free man - Is of relevance.
The law is clear that a confession cannot be used against an accused person unless the
Court is satisfied that it was voluntary and at that stage the question whether it is true or
false does not arise. If the facts and circumstances surrounding the making of a
confession appear to cast a doubt on the veracity or voluntariness of the confession, the
Court may refuse to act upon the confession, even if it is admissible in evidence. One
important question, in regard to which the Court has to be satisfied with is, whether when
the accused made the confession, he was a free man or his movements were controlled by
the police either by themselves or through some other agency employed by them for the
purpose of securing such a confession. The question whether a confession is voluntary or
not is always a question of fact. (Para 17)
(C) Evidence Act (1 of 1872), S.3 - EVIDENCE - Circumstantial evidence - Conviction
on basis of - Conditions precedent for - Stated.
The conditions precedent before conviction could be based on circumstantial evidence
are :
(1) the circumstances from which the conclusion of guilt is to be drawn should be fully
established. The circumstances concerned
@page-SC2820
'must' or 'should' and not 'may be' established;
(2) the facts so established should be consistent only with the hypothesis of the guilt of
the accused, that is to say, they should not be explainable on any other hypothesis except
that the accused is guilty;
(3) the circumstances should be of a conclusive nature and tendency;
(4) they should exclude every possible hypothesis except the one to be proved; and
(5) there must be a chain of evidence so complete as not to leave any reasonable ground
for the conclusion consistent with the innocence of the accused and must show that in all
human probability the act must have been done by the accused. (Para 12)
(D) Evidence Act (1 of 1872), S.3 - EVIDENCE - Circumstantial evidence - Last seen
together - Time gap between point of time when accused and deceased were seen together
and deceased is found dead - Should not be large. (Paras 14, 15)
Cases Referred : Chronological Paras
2006 AIR SCW 1602 : AIR 2006 SC 1656 (Rel. on) 15
2005 AIR SCW 905 : AIR 2005 SC 1000 : 2005 Cri LJ 1428 : 2005 All LJ 885 (Rel. on)
14
(2005) 12 SCC 438 (Rel. on) 16
2003 AIR SCW 4097 : AIR 2003 SC 3601 : 2003 Cri LJ 3901 (Ref.) 13, 18
2003 AIR SCW 5180 : AIR 2003 SC 4377 : 2003 Cri LJ 5054 (Ref.) 13
2002 AIR SCW 3655 : AIR 2002 SC 3164 : 2002 Cri LJ 4664 (Rel. on) 15
1996 AIR SCW 2903 : AIR 1996 SC 3390 : 1996 Cri LJ 3461 6
1995 AIR SCW 3719 : AIR 1996 SC 66 : 1995 Cri LJ 4183 3
1993 AIR SCW 3883 : AIR 1994 SC 468 : 1994 Cri LJ 140 3
1992 AIR SCW 640 : AIR 1992 SC 840 : 1992 Cri LJ 1104 8
AIR 1990 SC 79 : 1990 Cri LJ 605 7
AIR 1989 SC 1890 : 1989 Cri LJ 2124 5
AIR 1987 SC 350 : 1987 Cri LJ 330 5
AIR 1985 SC 1224 : 1985 Cri LJ 1479 5
AIR 1984 SC 1622 : 1984 Cri LJ 1738 (Rel. on) 12
AIR 1983 SC 446 : 1983 Cri LJ 846 5
AIR 1977 SC 1063 : 1977 Cri LJ 639 5
AIR 1956 SC 316 : 1956 Cri LJ 559 5
AIR 1954 SC 621 : 1954 Cri LJ 1645 5
AIR 1952 SC 343 : 1953 Cri LJ 129 11
Sudhir Kulshreshtha, for Appellant; Debojit Borkakati and Mrs. D. Bharathi Reddy, for
Respondent.
Judgement
1. Dr. ARIJIT PASAYAT, J. :- Challenge in this appeal is to the judgment of a Division
Bench of the Andhra Pradesh High Court upholding the appellant's conviction for offence
punishable under Section 302 of the Indian Penal Code, 1860 (in short the 'IPC') for
committing murder of one Gottapu Adilakshmi (hereinafter referred to as the 'deceased')
by strangulating her with a towel on 22.2.2001. Learned VI Additional Sessions Judge
(Fast Track Court), Machilipatnam had found the accused guilty and convicted and
sentenced him to imprisonment for life and fine.
2. Prosecution case as unfolded during trial is as follows :
Kusuma Ankama Rao (hereinafter referred to as 'accused') was a resident of Pedaveedhi
of Gudivada Town. He was a fruit vendor. Sankara Rao (PW-1) and Rama Swamy (PW-
2) are the son and husband of the deceased respectively. The deceased stayed with her
family in the house of M. Simhachalam (PW-3) in Padamata Veedhi at Gudivada.
Accused was having illegal intimacy with the deceased. On 22.2.2001 at about 6.30 p.m.,
the accused met PW-1 (son of the deceased) and asked him to get a quarter bottle of
liquor and a beedi packet and paid Rs. 50/- for the purpose. Accordingly, PW-1 brought
the said items. Thereafter, the accused asked the whereabouts of the deceased. PW-1 took
the accused to Gopalakrishna (A.C.) theatre, where the deceased was working as a
labourer on that day. On their way to the theatre, they found the deceased and some
others coming in the opposite direction. At that point of time, the accused talked with the
deceased; and the accused, deceased and PW-1 went to the bypass road leading to Eluru
and thereafter they further went to the black gram field of one N. Narasimha Rao. At that
point of time the accused asked PW-1 not to follow them and to stop there. Accordingly,
PW-1 waited there for half an hour or so and as the deceased and accused did not return,
he returned to the hotel where he was working. Thereafter, he went to the house late in
the night. In the morning when he found that his mother had not returned home, he stated
@page-SC2821
the above facts to his father. In the meanwhile, they heard the people saying that there
was a dead body in the field of N. Narasimha Rao. Then PWs 1 and 2 went there and saw
the dead body of the deceased and PW-2 asked PW-1 to give complaint to the police.
Accordingly, PW-1 went to Town Police, Gudivada and gave Ex.P-1 report. On the basis
of the said report, FIR was registered by PW-11. The Investigating Officer (PW-12) on
receipt of the FIR went to the place of offence and conducted Panchanama of scene of
offence and thereafter held inquest over the dead body of the deceased. He also examined
the witnesses and seized the towel and other material objects. In the meanwhile, the
accused made an extra-judicial confession before PW-6, the Village Administrative
Officer to the effect that he had committed murder of the deceased by strangulation.
Immediately, thereafter PW-6 recorded the statement of the accused duly attested the
same by PW-8, the village servant. He took the accused to the Police Station along with
the report. The C.I. of police examined Village Administrative Officer. After completion
of investigation, charge-sheet was filed before the learned Additional Judicial First Class
Magistrate, Gudivada, who registered the same as P.R.C. No.30 of 2001. Since the
offence punishable under Section 302, IPC is exclusively triable by the Court of Sessions,
he committed the same to the Court of Session, Machilipatnam, who registered the case
as S. C. No. 211 of 2001. Thereafter, the case was made over to the learned VI Additional
District and Sessions Judge, Machilipatnam for trial and disposal in accordance with law.
In order to establish its version, prosecution examined 12 witnesses and marked as Exhs.
P-1 to P-14 documents and M.Os. 1 to 19 were also marked. The trial Court after
considering the evidence on record found the accused guilty and sentenced him as afore-
stated. The conviction was challenged before the High Court. The stand before the High
Court was that the prosecution case was based on circumstantial evidence and the
circumstances highlighted do not establish the guilt of the accused. The State on the other
hand referred to the evidence of PWs 1 and 2 and the extra- judicial confession made
before Village Administrative Officer (PW-6) to the effect that accused and the deceased
were last seen together, and the evidence clearly established the guilt of the accused. The
High Court accepted the stand of the State and dismissed the appeal.
3

. In support of the appeal, learned counsel for the appellant submitted that the last seen
concept is not applicable to the present case. The so-called extra-judicial confession was
before a stranger. There is no reason as to why the accused would make confession before
a stranger. Reliance is placed on a decision of this Court in State of Haryana v. Ved
Prakash (AIR 1994 SC 468) and Kailash Potlia v. State of Andhra Pradesh (AIR 1996 SC
66). 1993 AIR SCW 3883
1995 AIR SCW 3719

4. Learned counsel for the respondent-State on the other hand submitted that the three
witnesses i.e. PW 1 (son of the deceased), PWs 4 and 5 had seen the deceased and the
accused going together and, thereafter the dead body was recovered. The Village
Administrative Officer was not a stranger but he was in-charge of the village and was a
person of authority in that sense.
5. It has been consistently laid down by this Court that where a case rests squarely on
circumstantial evidence, the inference of guilt can be justified only when all the
incriminating facts and circumstances are found to be incompatible with the innocence of
the accused or the guilt of any other person. (See Hukam Singh v. State of Rajasthan
(AIR 1977 SC 1063); Eradu and Ors. v. State of Hyderabad (AIR 1956 SC 316);
Earabhadrappa v. State of Karnataka (AIR 1983 SC 446); State of U.P. v. Sukhbasi and
Ors. (AIR 1985 SC 1224); Balwinder Singh v. State of Punjab (AIR 1987 SC 350);
Ashok Kumar Chatterjee v. State of M.P. (AIR 1989 SC 1890)). The circumstances from
which an inference as to the guilt of the accused is drawn have to be proved beyond
reasonable doubt and have to be shown to be closely connected with the principal fact
sought to be inferred from those circumstances. In Bhagat Ram v. State of Punjab (AIR
1954 SC 621), it was laid down that where the case depends upon the conclusion drawn
from circumstances the cumulative effect of the circumstances must be such as to
negative the innocence of the accused and bring the offences home beyond any
reasonable doubt.
6

. We may also make a reference to a decision of this Court in C. Chenga Reddy and Ors.
v. State of A.P. (1996) 10 SCC 193, 1996 AIR SCW 2903, Para 20A

@page-SC2822
wherein it has been observed thus:
"In a case based on circumstantial evidence, the settled law is that the circumstances from
which the conclusion of guilt is drawn should be fully proved and such circumstances
must be conclusive in nature. Moreover, all the circumstances should be complete and
there should be no gap left in the chain of evidence. Further the proved circumstances
must be consistent only with the hypothesis of the guilt of the accused and totally
inconsistent with his innocence....".
7. In Padala Veera Reddy v. State of A.P. and Ors. (AIR 1990 SC 79), it was laid down
that when a case rests upon circumstantial evidence, such evidence must satisfy the
following tests:
"(1) the circumstances from which an inference of guilt is sought to be drawn, must be
cogently and firmly established;
(2) those circumstances should be of a definite tendency unerringly pointing towards guilt
of the accused;
(3) the circumstances, taken cumulatively should form a chain so complete that there is
no escape from the conclusion that within all human probability the crime was committed
by the accused and none else; and
(4) the circumstantial evidence in order to sustain conviction must be complete and
incapable of explanation of any other hypothesis than that of the guilt of the accused and
such evidence should not only be consistent with the guilt of the accused but should be
inconsistent with his innocence.
8

. In State of U.P. v. Ashok Kumar Srivastava, (1992 Cri LJ 1104), it was pointed out that
great care must be taken in evaluating circumstantial evidence and if the evidence relied
on is reasonably capable of two inferences, the one in favour of the accused must be
accepted. It was also pointed out that the circumstances relied upon must be found to
have been fully established and the cumulative effect of all the facts so established must
be consistent only with the hypothesis of guilt. 1992 AIR SCW 640

9. Sir Alfred Wills in his admirable book "Wills' Circumstantial Evidence" (Chapter VI)
lays down the following rules specially to be observed in the case of circumstantial
evidence: (1) the facts alleged as the basis of any legal inference must be clearly proved
and beyond reasonable doubt connected with the factum probandum; (2) the burden of
proof is always on the party who asserts the existence of any fact, which infers legal
accountability; (3) in all cases, whether of direct or circumstantial evidence the best
evidence must be adduced which the nature of the case admits; (4) in order to justify the
inference of guilt, the inculpatory facts must be incompatible with the innocence of the
accused and incapable of explanation, upon any other reasonable hypothesis than that of
his guilt, (5) if there be any reasonable doubt of the guilt of the accused, he is entitled as
of right to be acquitted".
10. There is no doubt that conviction can be based solely on circumstantial evidence but
it should be tested by the touch-stone of law relating to circumstantial evidence laid down
by this Court as far back as in 1952.
11

. In Hanumant Govind Nargundkar and Anr. V. State of Madhya Pradesh, (AIR 1952 SC
343), wherein it was observed thus: Para 10 of AIR

"It is well to remember that in cases where the evidence is of a circumstantial nature, the
circumstances from which the conclusion of guilt is to be drawn should be in the first
instance be fully established and all the facts so established should be consistent only
with the hypothesis of the guilt of the accused. Again, the circumstances should be of a
conclusive nature and tendency and they should be such as to exclude every hypothesis
but the one proposed to be proved. In other words, there must be a chain of evidence so
far complete as not to leave any reasonable ground for a conclusion consistent with the
innocence of the accused and it must be such as to show that within all human probability
the act must have been done by the accused."
12. A reference may be made to a later decision in Sharad Birdhichand Sarda v. State of
Maharashtra, (AIR 1984 SC 1622). Therein, while dealing with circumstantial evidence,
it has been held that onus was on the prosecution to prove that the chain is complete and
the infirmity of lacuna in prosecution cannot be cured by false defence or plea. The
conditions precedent in the words of this Court, before conviction could be based on
circumstantial evidence, must be fully established. They are:
@page-SC2823
(1) the circumstances from which the conclusion of guilt is to be drawn should be fully
established. The circumstances concerned 'must' or 'should' and not 'may be' established;
(2) the facts so established should be consistent only with the hypothesis of the guilt of
the accused, that is to say, they should not be explainable on any other hypothesis except
that the accused is guilty;
(3) the circumstances should be of a conclusive nature and tendency;
(4) they should exclude every possible hypothesis except the one to be proved; and
(5) there must be a chain of evidence so complete as not to leave any reasonable ground
for the conclusion consistent with the innocence of the accused and must show that in all
human probability the act must have been done by the accused.
13

. These aspects were highlighted in State of Rajasthan v. Rajaram (2003 (8) SCC 180),
state of Haryana v. Jagbir Singh and Anr. (2003 (11) SCC 261). 2003 AIR SCW 4097
2003 AIR SCW 5180

14

. So far as the last seen aspect is concerned it is necessary to take note of two decisions of
this court. In State of U.P. v. Satish [2005 (3) SCC 114] it was noted as follows: 2005
AIR SCW 905, Para 23

"22. The last seen theory comes into play where the time-gap between the point of time
when the accused and the deceased were seen last alive and when the deceased is found
dead is so small that possibility of any person other than the accused being the author of
the crime becomes impossible. It would be difficult in some cases to positively establish
that the deceased was last seen with the accused when there is a long gap and possibility
of other persons coming in between exists. In the absence of any other positive evidence
to conclude that the accused and the deceased were last seen together, it would be
hazardous to come to a conclusion of guilt in those cases. In this case there is positive
evidence that the deceased and the accused were seen together by witnesses PWs. 3 and
5, in addition to the evidence of PW-2."
15
. In Ramreddy Rajesh Khanna Reddy v. State of A.P. [2006 (10) SCC 172] it was noted as
follows: 2006 AIR SCW 1602

"27. The last-seen theory, furthermore, comes into play where the time gap between the
point of time when the accused and the deceased were last seen alive and the deceased is
found dead is so small that possibility of any person other than the accused being the
author of the crime becomes impossible. Even in such a case the courts should look for
some corroboration".

(See also Bodhraj v. State of JandK, (2002 (8) SCC 45).)" 2002 AIR SCW 3655

16. A similar view was also taken in Jaswant Gir v. State of Punjab [2005 (12) SCC 438].
17. Confessions may be divided into two classes i.e. judicial and extra-judicial. Judicial
confessions are those which are made before a Magistrate or a court in the course of
judicial proceedings. Extra-judicial confessions are those which are made by the party
elsewhere than before a Magistrate or court. Extra-judicial confessions are generally
those that are made by a party to or before a private individual which includes even a
judicial officer in his private capacity. It also includes a Magistrate who is not especially
empowered to record confessions under Section 164 of the Code of Criminal Procedure,
1973 (in short the 'Code') or a Magistrate so empowered but receiving the confession at a
stage when Section 164 does not apply. As to extra-judicial confessions, two questions
arise: (i) were they made voluntarily? and (ii) are they true? As the section enacts, a
confession made by an accused person is irrelevant in criminal proceedings, if the making
of the confession appears to the court to have been caused by any inducement, threat or
promise, (1) having reference to the charge against the accused person, (2) proceeding
from a person in authority, and (3) sufficient, in the opinion of the court to give the
accused person grounds which would appear to him reasonable for supposing that by
making it he would gain any advantage or avoid any evil of a temporal nature in
reference to the proceedings against him. It follows that a confession would be voluntary
if it is made by the accused in a fit state of mind, and if it is not caused by any
inducement, threat or promise which has reference to the charge against him, proceeding
from a person in authority. It would not be involuntary, if the inducement, (a) does not
have reference to the charge against the accused person; or (b) it does not proceed from a
person in authority;
@page-SC2824
or (c) it is not sufficient, in the opinion of the court to give the accused person grounds
which would appear to him reasonable for supposing that, by making it, he would gain
any advantage or avoid any evil of a temporal nature in reference to the proceedings
against him. Whether or not the confession was voluntary would depend upon the facts
and circumstances of each case, judged in the light of Section 24. The law is clear that a
confession cannot be used against an accused person unless the court is satisfied that it
was voluntary and at that stage the question whether it is true or false does not arise. If
the facts and circumstances surrounding the making of a confession appear to cast a
doubt on the veracity or voluntariness of the confession, the court may refuse to act upon
the confession, even if it is admissible in evidence. One important question, in regard to
which the court has to be satisfied with is, whether when the accused made the
confession, he was a free man or his movements were controlled by the police either by
themselves or through some other agency employed by them for the purpose of securing
such a confession. The question whether a confession is voluntary or not is always a
question of fact. All the factors and all the circumstances of the case, including the
important factors of the time given for reflection, scope of the accused getting a feeling of
threat, inducement or promise, must be considered before deciding whether the court is
satisfied that in its opinion the impression caused by the inducement, threat or promise, if
any, has been fully removed. A free and voluntary confession is deserving of the highest
credit, because it is presumed to flow from the highest sense of guilt. It is not to be
conceived that a man would be induced to make a free and voluntary confession of guilt,
so contrary to the feelings and principles of human nature, if the facts confessed were riot
true. Deliberate and voluntary confessions of guilt, if clearly proved, are among the most
effectual proofs in law. An involuntary confession is one which is not the result of the
free will of the maker of it. So where the statement is made as a result of harassment and
continuous interrogation for several hours after the person is treated as an offender and
accused, such statement must be regarded as involuntary. The inducement may take the
form of a promise or of a threat, and often the inducement involves both promise and
threat, a promise of forgiveness if disclosure is made and threat of prosecution if it is not.
(See: Woodroffe's Evidence, 9th Edn., p. 284,) A promise is always attached to the
confession alternative while a threat is always attached to the silence alternative; thus, in
one case the prisoner is measuring the net advantage of the promise, minus the general
undesirability of a false confession, as against the present unsatisfactory situation; while
in the other case he is measuring the net advantages of the present satisfactory situation,
minus the general undesirability of the confession against the threatened harm. It must be
borne in mind that every inducement, threat or promise does not vitiate a confession.
Since the object of the rule is to exclude only those confessions which are testimonially
untrustworthy, the inducement, threat or promise must be such as is calculated to lead to
an untrue confession. On the aforesaid analysis the court is to determine the absence or
presence of an inducement, promise etc. or its sufficiency and how or in what measure it
worked on the mind of the accused. If the inducement, promise or threat is sufficient in
the opinion of the court, to give the accused person grounds which would appear to him
reasonable for supposing that by making it he would gain any advantage or avoid any
evil, it is enough to exclude the confession. The words "appear to him" in the last part of
the section refer to the mentality of the accused.
18

. An extra-judicial confession, if voluntary and true and made in a fit state of mind, can be
relied upon by the court. The confession will have to be proved like any other fact. The
value of the evidence as to confession, like any other evidence, depends upon the veracity
of the witness to whom it has been made. The value of the evidence as to the confession
depends on the reliability of the witness who gives the evidence. It is not open to any
court to start with a presumption that extra-judicial confession is a weak type of evidence.
It would depend on the nature of the circumstances, the time when the confession was
made and the credibility of the witnesses who speak to such a confession. Such a
confession can be relied upon and conviction can be founded thereon if the evidence
about the confession comes from the mouth of witnesses who appear to be unbiased, not
even remotely inimical to the accused, and in respect of whom nothing 2003 AIR
SCW 4097

@page-SC2825
is brought out which may tend to indicate that he may have a motive of attributing an
untruthful statement to the accused, the words spoken to by the witness are clear,
unambiguous and unmistakably convey that the accused is the perpetrator of the crime
and nothing is omitted by the witness which may militate against it. After subjecting the
evidence of the witness to a rigorous test on the touchstone of credibility, the extra-
judicial confession can be accepted and can be the basis of a conviction if it passes the
test of credibility. (See State of Rajasthan v. Raja Ram (2003 (8) SCC 180)).
19. If the factual scenario is considered it is seen that the prosecution clearly established
the guilt of the accused. There is no infirmity in the judgment of the trial Court as
affirmed by the High Court. The appeal is without merit, deserves dismissal which we
direct.
Appeal dismissed. .
AIR 2008 SUPREME COURT 2825 "State of Maharashtra v. Zubair Haji Qasim"
(From : 2006 (6) AIR Bom R 69)
Coram : 2 A. K. MATHUR AND ALTAMAS KABIR, JJ.
Criminal Appeal No. 1064 with 1065 of 2008 (@ Spl. Leave Petn. (Cri.) No. 1224 with
1975 of 2007), D/- 11 -7 -2008.
State of Maharashtra and Ors. v. Zubair Haji Qasim.
Conservation of Foreign Exchange and Prevention of Smuggling Activities Act (52 of
1974), S.8 - Constitution of India, Art.22(3)(b) - FOREIGN EXCHANGE -
PREVENTIVE DETENTION - Preventive detention - Proceedings before Advisory
Board - Representation through lawyer - Not vested right of detenu - Detenu can,
however, make request to Board - Request made has to be considered on merits - Order
of Board rejecting prayer of detenu for legal assistance by observing, could not be
granted for obvious reasons - Not rejection on merits - Order liable to be set aside.
(Paras 15, 16)
Cases Referred : Chronological Paras
1985 Cri LJ 840 (Bom) (Approved) 4
AIR 1982 SC 710 : 1982 Cri LJ 340 (Ref.) 11
AIR 1981 SC 1641 : 1981 Cri LJ 1262 (Rel. on) 9, 14, 15, 16
AIR 1981 SC 2041 : 1981 Cri LJ 1501 (Rel. on) 15, 16
Ravindra Keshavrao Adsure and Gautam Godara, for Appellants; Naresh Kaushik,
Manish Kaushik and K.L. Janjani, for Respondent.
Judgement
1. ALTAMAS KABIR, J. :-Leave granted.
2. One Abu Baker Haji Qasim, the brother of respondent No. 1 herein, was placed under
detention after his arrest on 10th September, 2 2005, pursuant to an order of detention
dated 9th September, 2005, issued by the Principal Secretary (Appeals and Security) to
the Government of Maharashtra, specially empowered under Section 3(1) of the
Conservation of Foreign Exchange and Prevention of Smuggling Activities Act of 1974
(hereinafter referred to as "the COFEPOSA Act, 1974").
3. On 22nd September, 2005, the said Abu Baker Haji Qasim (hereinafter referred to as
"the detenu") filed Criminal Writ Petition No.2312 of 2005, through the respondent No. 1
herein, before the Bombay High Court for quashing and setting aside the detention order
dated 9th September, 2005. At the same time, in the proceedings, before the Advisory
Board, the detenu made a representation for permission to be represented in the
proceedings through a legal practitioner. Such representation was, however, rejected by
the Advisory Board on 28th October, 2005, on the ground that under the COFEPOSA Act
1974, a detenu was not entitled to be represented by a legal practitioner and consequently
it was not necessary to consider such prayer. In fact, the Advisory Board rejected the
prayer made on behalf of the detenu for permission to be represented by a legal
practitioner upon holding that such prayer could not be considered "for some obvious
reasons".
4

. When the writ petition came up for hearing, the High Court upon considering the
submissions made on behalf of the respective parties and upon placing reliance on a
Division Bench Judgment of the Bombay High Court in Kekalwa Samuele Kongwa vs.
Union of India [1985 (1) Bom CR 742] allowed the writ petition and quashed the
detention order holding that the prayer of the detenu for permission to be represented by a
legal practitioner was not rejected after proper consideration but on erroneous grounds. It
may not be out of place to mention that in Kekalwa Samuele Kongwa's case 1985
Cri LJ 840

@page-SC2826
(supra) the Division Bench of the Bombay High Court held as follows :-
"A request made by a detenu for being represented by a legal practitioner must be
considered on merits and cannot be turned down on the ground (i) that the law does not
give such a right to the detenue, or (ii) that it was the practice of the Board not to allow
representation of a detenue by a legal practitioner."
5. The said judgment and order of the Bombay High Court allowing the writ petition and
quashing the detention order is under challenge in this appeal.
6. Although, the life of the detention order came to an end on 9th September, 2006, Mr.
Adsure, appearing for the appellant-State of Maharashtra, submitted that the ground on
which the High Court had quashed the detention order was erroneous and was required to
be corrected.
7. Referring to clause (3)(b) of Article 22 of the Constitution, Mr. Adsure submitted that a
person placed under preventive detention was not entitled to be represented by a legal
practitioner before the Advisory Board. In this regard, Mr. Adsure also referred to Section
8(e) of the COFEPOSA Act, 1974, which reads as follows :-
"8(e) - a person against whom an order of detention has been made under this Act shall
not be entitled to appear by any legal practitioner in any matter connected with the
reference to the Advisory Board, and the proceedings of the Advisory Board and its
report, excepting that part of the report in which the opinion of the Advisory Board is
specified, shall be confidential;"
8. Mr. Adsure submitted that both Article 22(3)(b) of the Constitution as well as Section
8(e) of the COFEPOSA Act, 1974, makes it quite clear that a detenu under any of the
preventive detention enactments would not be entitled to be represented by a legal
practitioner before the Advisory Board.
9

. In support of his submissions, Mr. Adsure placed reliance on certain passages from the
decision of this Court in Smt. Kavita vs. State of Maharashtra and Ors. [1981 (3) SCC
558), where along with certain other questions, the question relating to a detenu's request
for being represented by lawyer before the Advisory Board in view of the provisions of
Section 8(e) of the COFEPOSA Act, 1974, fell for consideration. In the said case, a
similar question arose on account of the rejection by the Government of the detenu's
request to be permitted to be represented by a lawyer before the Advisory Board. The
detenu was informed by the Government that under the provisions of Section 8(e) of the
COFEPOSA Act, 1974, he was not entitled to be represented by a lawyer before the
Advisory Board, and, therefore, it was not possible to grant his request. The three-Judge
Bench, while considering the reply of the State Government, observed as follows :-
AIR 1981 SC 1641

"It is true that while Section 8(e) disentitles a detenu from claiming as of right to be
represented by a lawyer, it does not disentitle him from making a request for the services
of a lawyer."
10. It was observed further that without adequate legal assistance the personal liberty of
the detenu guaranteed by Article 21 of the Constitution could be jeopardized and
rendered meaningless. The request by a detenu for legal assistance would have to be
considered on its own merits in each individual case. However, since in the said case, the
detenu had not applied to the Advisory Board, it was held that it could not be said that the
detenu had been wrongly denied the assistance of counsel. What Mr. Adsure tried to
emphasise was that no legal right vested in the detenu for being allowed to be represented
by a legal practitioner before the Advisory Board.
11

. Mr. Adsure also referred to the Constitution Bench decision of this Court in A.K. Roy
vs. Union of India [1982 (1) SCC 271], wherein the questions raised in this appeal had
also been considered in detail in the light of the provisions of the Maintenance of Internal
Security Act, 1971 and the National Security Act and it was held, with regret, that the
detenu had no right to appear through a legal practitioner in the proceedings before the
Advisory Board. AIR 1982 SC 710

12. Various other decisions on the same issue were also referred to by Mr. Adsure in
support of his contention that the order of the High Court quashing the detention order
issued against the brother of respondent No. 1, was erroneous and was liable to be set
aside.
13. On behalf of the respondent No. 1, the views expressed before the High Court were
reiterated and it was urged that no interference
@page-SC2827
was warranted with the order of the High Court impugned in the instant appeal.
14
. Having considered the submissions made on behalf of the respective parties, we are of
the view that the order of the High Court impugned in this appeal does not warrant
interference. In Smt. Kavita's case (supra) on which reliance was placed by Mr. Adsure, it
was also laid down that, although, a detenu has no right under Section 8(e) of the
COFEPOSA Act, 1974, to legal assistance in proceedings before the Advisory Board, he
is entitled to make such a request to the Board and the Board is bound to consider such
request when made. AIR 1981 SC 1641

15

. In the instant case, the detenu's prayer for being allowed to be represented by a legal
practitioner was rejected by the Advisory Board upon observing that such prayer could
not be considered "for some obvious reasons". The said reasoning runs counter to the
decision of this Court in Smt. Kavita's case (supra) and cannot, therefore, be sustained.
The decision of the Division Bench of the Bombay High Court, relied upon by the High
Court in quashing the detention order, says much the same thing as has been stated by
this Court in Smt. Kavita's case (supra) and also in the case of Nand Lal Bajaj vs. State of
Punjab [1981 (4) SCCF 327]. It is quite clear that while rejecting the detenu's
representation the Advisory Board took refuge in vague verbiage without really applying
its mind as it was required to do, to the merits thereof. AIR 1981 SC 1641
AIR 1981 SC 2041

16

. Detention of a citizen under any of the preventive detention enactments is a serious and
severe invasion on the Fundamental Rights guaranteed to citizens under Articles 19 and
21 of the Constitution. Recognising such detention to be an evil necessity, various
safeguards have been placed at different stages of the detention proceedings to ensure that
such powers were not used indiscriminately to settle scores or to short-circuit the process
of investigation and trial of an alleged offence. In Smt. Kavita's case (supra) and also in
Nand Lal Bajaj's case (supra) this Court held that even if the detenu had no right to
appear through a legal practitioner in the proceedings before the Advisory Board he was
entitled to make a representation for the services of a lawyer to appear before the said
Board which was under an obligation to consider the same dispassionately in the facts of
the particular case. The representation made by a detenu for legal assistance before the
Advisory Board, has, therefore, to be considered not perfunctorily, as has been done in
this case, but with due application of mind, since in each case of detention the liberty of
an individual is involved. AIR 1981 SC 1641
AIR 1981 SC 2041

17. We, therefore, see no reason to interfere with the judgment and order of the High
Court impugned in this appeal and the appeal is accordingly dismissed.
18. This judgment would also govern Criminal Appeal No. 1605 (@ Special Leave
Petition (Criminal) No. 1975 of 2007) titled State of Maharashtra vs. Sheetal Manoj
Gore.
Appeals dismissed. .
AIR 2008 SUPREME COURT 2827 "State of Tamil Nadu v. R. Sasikumar"
(From : Madras)*
Coram : 3 Dr. A. PASAYAT, P. SATHASIVAM AND AFTAB ALAM, JJ.
Criminal Appeal No.465 of 2001, D/- 9 -7 -2008.
State of T.N. and Anr. v. R. Sasikumar.
(A) T.N. Prevention of Dangerous Activities of Bootleggers, Drug Offenders, Forest
Offenders, Goondas, Immoral Traffic Offenders and Slam Grabbers Act (14 of 1982), S.3
- PREVENTIVE DETENTION - Order of detention - Right to make representation -
Arises after order is passed - Representation made prior to passing of order - Non-
consideration by Detaining Authority - Not ground to set aside detention - Moreover,
presumption as to receipt of copy of representation by Detaining Authority - Cannot be
raised merely because representation has been received by two other addressees.
Habeas Corpus Petition No. 1262 of 1999, D/-24-03-2000 (Mad), Reversed.
Constitution of India, Art.22(5). (Paras 4, 5)
(B) T.N. Prevention of Dangerous Activities of Bootleggers, Drug Offenders, Forest
Offenders, Goondas, Immoral Traffic Offenders and Slam Grabbers Act (14 of 1982), S.3
- DETENTION - PLEA -
@page-SC2828
Order of detention - Validity - Plea of being based on stale incident - Order mentioning
several incidents - Last instance mentioned was a month prior to order of detention -
Order cannot be said to be based on stale incidents - Not liable to be set aside. (Para
6)
Cases Referred : Chronological Paras
2006 AIR SCW 5363 : 2006 (6) AIR Kar R 571 : 2007 Cri LJ 30 (Rel. on) 4
2006 (7) SCC 540 (Ref.) 6
2004 AIR SCW 6237 : AIR 2005 SC 160 : 2005 Cri LJ 105 6
2004 AIR SCW 6999 : AIR 2005 428 4
2003 AIR SCW 5423 : AIR 2003 SC 4622 : 2003 Cri LJ 4561 : 2004 AIR Jhar HCR 86
(Ref.) 4
2000 AIR SCW 582 : AIR 2000 SC 1023 : 2000 Cri LJ 1444 6
2000 AIR SCW 4496 : AIR 2001 SC 301 : 2001 Cri LJ 497 (Ref.) 4
1993 Supp (2) SCC 341 (Ref.) 4
AIR 1986 SC 356 : 1986 Cri LJ 493 : 1986 All LJ 397 (Ref.) 4
R. Sundaravaradan, Sr. Advocate, V.G. Pragasam, S.J. Aristotle and Prabhu
Ramasubramanian, for Appellants; K.K. Mani (A.C.), C.K.R. Lenin Sekar and Mayur R.
Shah, for Respondent.
* H.C.P. No.1262 of 1999, D/- 24-3-2000 (Mad).
Judgement
Dr. ARIJIT PASAYAT, J. :- Challenge in this appeal is to the judgment of a Division
Bench of the Madras High Court allowing the Habeas Corpus Petition filed by the
respondent questioning the order of detention i.e. Detention Order 519/BDFGIS/99 dated
9.7.1999 passed by the Commissioner of Police, Chennai.
2. Background facts in a nutshell are as follows:
The respondent (hereinafter referred to as the 'detenu') was detained under sub-section (1)
of Section 3 of Tamil Nadu Prevention of Dangerous Activities of Bootleggers, Drug
Offenders, Forest Offenders, Goondas, Immoral Traffic Offenders and Slum Grabbers
Act, 1982 (in short the 'Act'). The only point urged before the High Court was that an
order of detention was passed on 9.7.1999 and on 6.7.1999 the mother of the detenu had
sent a representation to the Chief Minister of Tamil Nadu. A copy of the representation
was marked to the Director General of Police, Chennai, the Advisory Board under the Act
as well as the Chief Justice of the High Court. It was, therefore, submitted that there was
evidence of dispatch of the representation and since it was not considered by the
detaining authority the order of detention was bad.
The stand of the detaining authority was that the representation was not sent to the
detaining authority and, therefore, there was no question of considering the same before
passing the order of detention.
The High Court found that since two of the authorities had received the representation it
must be presumed that the Director General of Police would have received the
representation in the usual course. A presumption was drawn that the Director General of
Police had been served the representation and accordingly it was held that the Director
General of Police must have received the representation and since that was not taken note
of, there was violation of Article 22(5) of the Constitution of India, 1950 (in short the
'Constitution'). Accordingly, the order of detention was quashed.
The State of Tamil Nadu and the detaining authority have challenged the correctness of
the order of the High Court. Notice was issued by this Court on 4.9.2000. When the
matter was taken up subsequently on 11.12.2000, it was noted that the detenu was not
represented and there was no appearance on behalf of the detenu, though he was served.
The Bench also noted that the period of detention was also over and the detenu had been
released. The Court further noted that it would be proper to appoint Mr. K. K. Mani,
Advocate as Amicus Curiae.
3. Learned counsel for the appellants submitted that the approach of the High Court is
clearly wrong. There was no question of any representation even before the order of
detention was passed and there was no question of sending it to the Advisory Board.
4

. Learned Amicus Curiae submitted that since the representations sent to the Chief
Minister and the Advisory Board amongst others had been received, the High Court
found that the Director General of Police is presumed to have received the notice.
Therefore, impugned order cannot be faulted. We find that the High Court's order
proceeds on presumption. Merely because two of the 2006 AIR SCW 5363, (Para
11)

@page-SC2829
addressees had received the representations that in no way shows that the Director
General of Police had received the representation. Additionally, as rightly submitted by
learned counsel for the appellant-State, before the order of detention was passed there is
no question of sending a representation to the Advisory Board. This appears to be clever
use to create evidence to contend non-application of mind. This is a classic case, (such
cases are increasing by leaps and bounds) where red-hearings are intentionally drawn to
deflect the course of justice. In Sri Anand Hanumathsa Katar v. Additional District
Magistrate and Ors. (2006 (10) SCC 725) it was observed by this Court as follows :
"11. At this juncture it would be relevant to take note of paras 17 to 19 of Union of India
v. Paul Manickam (2003 (8) SCC 342) They read as follows (SCC pp. 354-55) : 2003
AIR SCW 5423, (Paras 15 to 17)

"17. Coming to the question whether the representation to the President of India meets
with the requirement of law, it has to be noted that in Raghavendra Singh v. Supdt.,
District Jail, Kanpur (1986 (1) SCC 650) and Rumana Begum v. State of A.P. (1993
Supp. (2) SCC 341) it was held that a representation to the President of India or the
Governor, as the case may be, would amount to representation to the Central Government
and the State Government respectively. Therefore, the representation made to the
President of India or the Governor would amount to representation to the Central
Government and the State Government. But this cannot be allowed to create a
smokescreen by an unscrupulous detenu to take the authorities by surprise, acting
surreptitiously or with ulterior motives. In the present case, the order (grounds) of
detention specifically indicated the authority to whom the representation was to be made.
Such indication is also a part of the move to facilitate an expeditious consideration of the
representations actually made.
18. The respondent does not appear to have come with clean hands to the court. In the
writ petition there was no mention that the representation was made to the President;
instead it was specifically stated in paragraph 23 that the representation was made by
registered post to the first respondent on 11-5-2000 and a similar representation was
made to the second respondent.
Before the High Court in the writ petition the first and the second respondents were
described as follows:
'1. State of Tamil Nadu, rep. by its Secretary, Government of Tamil Nadu, Public (SC)
Department, Fort St. George, Chennai, 600 009.
2. Union of India, rep. by its Secretary, Ministry of Finance, Department of Revenue,
New Delhi.'
19. As noted supra, for the first time in the review application it was disclosed that the
representation was made to the President of India and no representation was made to the
State of Tamil Nadu or the Union of India who were arrayed in the writ petition as
parties. This appears to be a deliberate attempt to create confusion and reap an
undeserved benefit by adopting such dubious device. The High Court also transgressed its
jurisdiction in entertaining the review petition with an entirely new substratum of issues.
Considering the limited scope for review, the High Court ought not to have taken into
account factual aspects which were not disclosed or were concealed in the writ petition.
While dealing with a habeas corpus application undue importance is not to be attached to
technicalities, but at the same time where the court is satisfied that an attempt has been
made to deflect the course of justice by letting loose red herrings the court has to take
serious note of unclean approach. Whenever a representation is made to the President and
the Governor instead of the indicated authorities, it is but natural that the representation
should indicate as to why the representation was made to the President or the Governor
and not the indicated authorities. It should also be clearly indicated as to whom the
representation has been made specifically, and not in the manner done in the case at hand.
The President as well as the Governor, no doubt are constitutional Heads of the respective
Governments but the day-today administration at respective levels is carried on by the
Heads of the Departments/ Ministries concerned and designated officers who alone are
ultimately responsible and accountable for the action taken or to be taken in a given case.
If really the citizen concerned genuinely and honestly felt or was interested in getting an
expeditious consideration or disposal of his grievance, he would and should honestly
approach the real authorities concerned and would not adopt any
@page-SC2830
dubious devices with the sole aim of deliberately creating a situation for delay in
consideration and cry for relief on his own manipulated ground, by directing his
representation to an authority which is not directly/ immediately concerned with such
consideration."

12. Paras 17 to 19 of Union of India v. Chaya Ghoshal (2005 (10) SCC 97) are also
relevant. They read as follows (SCC pp. 106-07) : 2004 AIR SCW 6999, (Paras 15 to
17)

"17. While dealing with a habeas corpus application, undue importance is not to be
attached to technicalities, but at the same time where the court is satisfied that an attempt
has been made to deflect the course of justice by letting loose red herrings, the court has
to take serious note of unclean approach. Whenever a representation is made to the
President or the Governor instead of the indicated authorities, it is but natural that the
representation should indicate as to why the representation was made to the President or
the Governor and not to the indicated authorities. It should also be clearly indicated as to
whom the representation has been made specifically. The President as well as the
Governor, no doubt are constitutional Heads of the respective Governments but day-to-
day administration at respective levels is carried on by the Heads of the
Department/Ministries concerned and designated officers who alone are ultimately
responsible and accountable for the action taken or to be taken in a given case. If really
the citizen concerned genuinely and honestly felt or is interested in getting an expeditious
consideration or disposal of his grievance, he would and should honestly approach the
real authorities concerned and would not adopt any dubious devices with the sole aim of
deliberately creating a situation for delay in consideration and cry for relief on his own
manipulated ground, by directing his representation to an authority which is not
directly/immediately concerned with such consideration.
18. Where, however, a person alleging infraction of personal liberty tries to act in a
manner which is more aimed at deflecting the course of justice than for protection of his
personal right, the court has to make a deliberate balancing of the fact situation to ensure
that the mere factum of some delay alone is not made use of to grant relief. If a fraud has
been practised or perpetrated, that may in a given case nullify the cherished goal of
protecting personal liberty, which obligated this Court to devise guidelines to ensure such
protection by balancing individual rights and the interests of the nation, as well.

19. In R. Keshava v. M.B. Prakash ((2001 (2) SCC 145) it was observed by this Court as
follows: (SCC p. 154, para 17) 2000 AIR SCW 4496, (Para 17)
'17. We are satisfied that the detenu in this case was apprised of his right to make
representation to the appropriate Government/ authorities against his order of detention as
mandated in Article 22(5) of the Constitution. Despite knowledge, the detenu did not
avail of the opportunity. Instead of making a representation to the appropriate
Government or the confirming authority, the detenu chose to address a representation to
the Advisory Board alone even without a request to send its copy to the authorities
concerned under the Act. In the absence of representation or the knowledge of the
representation having been made by the detenu, the appropriate Government was justified
in confirming the order of detention on perusal of record and documents excluding the
representation made by the detenu to the Advisory Board. For this alleged failure of the
appropriate Government, the order of detention of the appropriate Government is neither
rendered unconstitutional nor illegal."
5. The question of making a representation to the Advisory Board arises only after the
order of detention had been passed and served on the detenu. The High Court therefore,
was clearly in error in quashing the order of detention.
6. Another point which has been urged is that the incidence referred to in the order of
detention is stale and could not have formed the foundation for the order of detention. We
find that several incidents have been referred to in the order of detention and the last of
such instances was of 22.6.1999. The detention order was passed on 9.7.1999 and,
therefore, it cannot be said to be relatable to stale incidents. The impugned order of the
High Court is therefore quashed. Since the impugned order of the High Court was passed
more than 8 years back, considering the nature of the order of detention which is
essentially preventive in character, it is appropriate for the State
@page-SC2831
Government and the detaining authority to consider whether there is any need to take the
detenu back to detention for serving the remainder of the period of detention which was
indicated in the order of detention. We express no opinion on that aspect. In State of T.N.
and Another v. Alagar [2006 (7) SCC 540] it was noted as follows:

"9. The residual question is whether it would be appropriate to direct the respondent to
surrender for serving remaining period of detention in view of passage of time. As was
noticed in Sunil Fulchand Shah v. Union of India (2000 (3) SCC 409) and State of T.N. v.
Kethiyan Perumal (2004 (8) SCC 780) it is for the appropriate State to consider whether
the impact of the acts, which led to the order of detention still survives and whether it
would be desirable to send back the detenu for serving remainder period of detention.
Necessary order in this regard shall be passed within two months by the appellant State.
Passage of time in all cases cannot be a ground not to send the detenu to serve remainder
of the period of detention. It all depends on the facts of the act and the continuance or
otherwise of the effect of the objectionable acts. The State shall consider whether there
still exists a proximate temporal nexus between the period of detention indicated in the
order by which the detenu was required to be detained and the date when the detenu is
required to be detained pursuant to the present order." 2000 AIR SCW 582
2004 AIR SCW 6237

7. The appeal is allowed to the aforesaid extent.


Order accordingly. .
AIR 2008 SUPREME COURT 2831 "State of U. P. v. Bhaiya Lal Verma"
(From : 2000 (41) All Cri C 669)
Coram : 3 Dr. A. PASAYAT, P. SATHASIVAM AND AFTAB ALAM, JJ.
Criminal Appeal No. 285 of 2001, D/- 7 -7 -2008.
State of U.P. v. Bhaiya Lal Verma.
Prevention of Corruption Act (2 of 1947), S.5(2) (since repealed) - Criminal P.C. (2 of
1974), S.386 - CORRUPTION - APPELLATE COURT - Bribery case - Allegation that
accused demanded money for stalling recovery from complainant - Trap laid - Demand
and recovery of marked currency notes from accused proved - Fact that recovery order
was already issued against complainant - Does not frustrate allegation of complainant
since recovery order contained signature of accused - Fact that accused asked police
about reasons of his arrest - Does not show his innocence when immediately thereafter he
blaimed complainant of betraying him - Interference by appellate Court with order of
conviction - Held, improper.
2000 (41) All Cri C 669, Reversed. (Paras 6, 8, 9)

S.G. Hasnianan, Sr. Advocate, Mohd. Fuzail Khan, Ms. Archana Singh and Anil Kumar
Jha, for Appellant; Anis Ahmed Khan and Shoaib Ahmad Khan, for Respondent.
Judgement
Dr. ARIJIT PASAYAT, J. :- Challenge in this appeal is to the order passed by a learned
Single Judge of the Allahabad High Court allowing the criminal appeal filed by the
respondent (hereinafter referred to as the 'accused'). The accused was convicted by
learned Special Judge (E.C. Act) Banda in Special Case No. 3 of 1985 for offence
punishable under Section 5(2) of the Prevention of Corruption Act, 1947 (in short the
'Act'). He was sentenced to undergo rigorous imprisonment for two years and was
directed to pay a fine of Rs.5,000/- with default stipulation. He was also convicted for
offence punishable under Section 161 of the Indian Penal Code, 1860 (in short the 'IPC')
and sentenced to rigorous imprisonment for two years. Both the sentences were directed
to run concurrently.
2. Background facts in a nutshell are as follows :
Sri Rajendra Singh Kushwaha, Assistant Agriculture Inspector of Rajkiya Krishi Sadhan
Purti Bhandar Badokhar Buzurg, Banda moved an application before the District
Magistrate, Banda confidentially on 2.4.1984 stating that Sri Bhaiya Lal Verma, the
accused who was the Accountant in the office of District Agriculture Officer and Project
Officer in collusion with the Project Officer (Agriculture) Sri. Lal Mani Ram was
harassing him by giving threats of recovery on the basis of fictitious bills. Sri Bhaiya Lal
Verma the Accountant had promised him that if he pays Rs.150/- to him no action will be
taken against him. Sri Bhaiya Lal Verma had called him in the office on 2-4-1984 and has
agreed to accept Rs. 150/- as bribe. Hence request was made that the Accountant Sri
Bhaiya Lal Verma may be
@page-SC2832
caught red handed while accepting Rs. 150/-as illegal gratification so that the applicant
may discharge his duties impartially. The then District Magistrate, Banda marked the said
application to the Additional District Magistrate (Finance and Revenue) and ordered him
to lay the trap. Thereafter, the then Additional District Magistrate (F), Banda Sri J.N.
Vishwakarma directed Sri. R.L. Gupta, the then Executive Magistrate and Additional
S.D.M./S.D.O. Naraini Banda to record the statement of the complainant and to prepare
the fard of currency notes which were to be given in the bribe and thereafter the papers
were to be handed over to C.O. Sadar for laying trap. In compliance with the order of
A.D.M. (F) Banda, Sri R.L. Gupta, Additional S.D.M. Banda recorded the statement of
the complainant Sri. Rajendra Singh on 2-4-1984 at 5.45 P.M. Sri Rajendra Singh
confirmed the facts mentioned in the application moved before the District Magistrate,
Banda. He further stated that he has brought one currency note of Rs. 100/-
denomination, and one currency note of Rs.50/- denomination, numbers of which are
AA/35 377745 and 3 DH 355826 respectively. Sri R.L. Gupta prepared the fard of the
said currency notes on 2-4-1984 at 6 P.M. He marked to currency notes with his initial
which were to be given in the bribe to Sri Bhaiya Lal Verma. Thereafter, the aforesaid
currency notes were handed over to the complainant Shri Rajendra Singh Kushwaha. The
fard was read over to him and his signatures were also obtained. Thereafter Sri R.L.
Gupta the Additional S.D.M. Banda called Sri. O.P. Kakkar the then C.O. Sadar in the
office of A.D.M. (F), Banda on 2-4-1984 at about 6 P.M., and handed over the application
dated 2-4-1984 of Sri Rajendra Singh Kushwaha containing orders of the District
Magistrate and Additional District Magistrate (F) for laying trap. He also handed over the
statement of Sri Rajendra Singh and the fard of currency notes, as referred to above, to
him. Sri O.P. Kakkar took Rajendra Singh with him and proceeded towards the spot. He
also took the then S.H.O. P.S. Kotwali Sri. Shiva Nandan and constables Rajendra Kumar
Tiwari, Dinesh Kumar and Gulab Singh from Kutchery Chauraha with him and
proceeded by jeep towards the office of District Agriculture Officer and Project Officer.
They left the jeep near the Telephone Exchange. From there they proceeded towards the
place of occurrence on foot. Sri Shiv Prasad Yadav and Sri Mithlesh Kumar Dwivedi met
him near the Telephone Exchange. He took them with him and gave information about
his purpose. They reached near the office and sent Rajendra Singh for giving illegal
gratification. They remained standing in the Verandah by the side of the eastern window
affixed in the northern wall of the big room lying in the middle of the office. At that time
light was on in the office and it was about 6.45 P.M. Sri Bhaiya Lal Verma was sitting on
the table and was talking with one person. The police party and the public witnesses
heard and saw from the window that Rejendra Singh informed Bhaiya Lal Verma that he
has brought Rs. 150/- which he had demanded for not making recovery from him and he
should accept the said amount. Thereafter Rajendra Singh offered Rs. 150/-to Bhaiya Lal
Verma, who accepted the currency notes of Rs.150/-. He took out purse from his pocket
and kept the said currency notes in the purse. Thereafter, he kept the purse in the left
pocket of his bush-shirt. On being satisfied that Bhaiya Lal has accepted bribe, the office
was raided immediately and Bhaiya Lal was apprehended inside the room. His personal
search was made in accordance with rules. One currency note of the denomination of
Rs.100/- having No.AA/35 377745 and one Currency note of the denomination of Rs.50/-
having No.3 DH 355826 with marked initials of Sri. R.L. Gupta were recovered from the
purse kept in the left pocket of his bush shirt. Besides it, two currency notes of the
denomination of Rs. 100/- each and two currency notes of the denomination of Rs.50/-
each were also recovered from the said purse. In the pUrse, photograph of the accused
was there. During search, the person with whom the accused was talking before taking
bribe escaped quietly from there. On interrogation, he disclosed his name as Bhaiya Lal
Verma son of Korey Lal Verma and stated his full address. The aforesaid currency notes
along With purse and the photo of the accused and the bush shirt which the accused was
wearing were taken into possession by the police at the spot and the aforesaid articles
were sealed at the spot. The recovery memo Ex. Ka. 1 was prepared at the spot by Sri
Shiv Nandan Singh at the dictation of Sri O.P. Kakkar. The recovery memo was read over
to the police personnel and the public witnesses and their signatures were obtained.
@page-SC2833
Thereafter, the accused Bhaiya Lal was brought to P.S. Kotwali and was lodged there.
The recovery memo and the other papers were also filed at P.S. Kotwali. On the basis of
the recovery memo the case was registered against the accused Bhaiya Lal Verma for
offences punishable under Section 161 I.P.C. and under Section 5 (2) of Act. The case
was investigated by Sri Akshay Kumar Singh, the then Deputy S.P. Babenu. During
investigation, he recorded the statements of the witnesses and prepared the site plan. The
sanction to prosecute the accused was granted by Sri Rishi Ram Sharma, Director
Agriculture, Uttar Pradesh on 14th January, 1985. After completion of the investigation,
the charge sheet was filed against the accused for commission of offences punishable
under Section 161 IPC and Section 5(2) of the Act.
The accused pleaded innocence. He admitted that he was posted as Accountant in the
office of District Agriculture and Project Officer Banda in April, 1984. In his statement
recorded under Section 313 of the Code of Criminal Procedure, 1973 (in short the
'Code'), it was his stand that Sri Ram Adhar Awasthi was dealing with files relating to
recovery. He had no concern with the files of recovery. He took the stand that there was
enmity between the then Plant Protection Officer, Banda who in collusion with Rajendra
Kumar Kushwaha registered the false case against him. He examined two witnesses to
substantiate his defence.
The prosecution examined Sri Mithlesh Kumar Dwivedi (PW.1), Shiv Prasad (PW2),
Rajendra Singh (PW3), Akshay Kumar Singh (PW4), Rishi Ram Sharma (PW5), Om
Prakash Kakkar (PW6), Munna Lal Katiyar (PW 7) and Ram Lakhan Gupta (PW8) in
support of its case. Sri Mithlesh Kumar Dwivedi (PW1) proved the recovery Memo
Ex.Ka.1. Sri Rajendra Singh (PW3) proved the application dated 2.4.1984 Ex.Ka.2. Sri
Akshay Kumar Singh (PW 4) proved the site plan Ex.Ka.3 and the charge sheet Ex.Ka.4.
Sri Rishi Ram Sharma (PW5) proved the sanction order Ex.Ka.5. Sri Om Prakash Kakkar
(PW6) also proved the recovery memo Ex.Ka. 1. Sri Munna Lal Katiyar (PW7) proved
the FIR Ex.Ka. 6 and the copy of G.D. Ex.Ka. 7. Sri Ram Lakhan Gupta (PW8) proved
the order of A.D.M.(F) dated 2.4.1984 Ex.Ka.8, order dated 2.4.1984 of District
Magistrate Ex.Ka.9, the statement of Rajendra Singh Ex.Ka. 10 and the Fard of currency
notes Ex.Ka.11. Sri Ram Adhar Awasthi (DW1) proved the initial of accused Bhaiya Lal
Ex.Ka. 12 on the Photostat copy of order Ex.Ka. 1. The currency notes Ka. 1 to Ex. 6, a
purse Ex. 7, bush shirt Ex.8, specimen of seal Ex. 9 and the photo of the accused Ex. 10
were produced in the evidence.
Placing reliance on the evidence of prosecution witnesses more particularly PWs. 1, 2
and 3, the trial court found the accused guilty and convicted and sentenced him as
aforestated. However, in appeal, the High Court directed acquittal.
3-4. In support of the appeal, learned counsel for the State submitted that the approach of
the High Court was clearly erroneous. Recovery of the money has been clearly
established. No reason has been indicated to discard the evidence of PW 3.
5. Learned counsel for the accused, on the other hand, submitted that the High Court on
analysis of the evidence came to hold that the prosecution has failed to establish the
accusations.
6. One of the reasons which has weighed with the High Court to direct acquittal is the
statement of PW 1. According to High Court his evidence clearly proved that recovery
order was issued against the complainant on 29.3.1984 and it contained signatures of the
superior officer as well as the accused. The High Court came to hold that once the
recovery order has been issued, the accused could not have frustrated the recovery on any
ground. The recovery can be frustrated only before the issuance of the recovery order.
The reasoning of the High Court is clearly fallacious in as much as it overlooked the fact
that the recovery order itself contained the signature of the accused. If that is so, there
was no question of his being not involved in recovery. The High Court noted that in the
personal search one currency note of Rs. 100/- having No. AA/35 377745 and the other
currency Note of Rs. 50/- having No. 3 DH 3555826 were recovered from him.
7. The High Court attached unnecessary importance to the evidence of PWs. 1 and 2 to
hold that they did not clearly hear the demand for bribe. It is contrary to the evidence on
record. The High Court's conclusion is that after examining the statement of these two
witnesses it is apparent that none of these witnesses had heard any conversation between
the appellant and the complainant.
@page-SC2834
Apart from it, it is further clear from their statements that it was not possible for anyone
to hear the conversation from that place where they were standing. Shiv Prasad (PW 2)
stated that appellant enquired from police party the reason for his arrest. This is a very
material piece of evidence which hints at an element of surprise in the conduct of the
appellant instead of an exhibition of shock. This clearly strikes at the root of prosecution
case of demand of a bribe by appellant for stalling the recovery. There was no reason
indicated to discard the evidence of PW 3.
8. It is not to understand how the root of the prosecution case of demand of bribe was
rendered vulnerable. Merely because the accused enquired from the police the reason for
his arrest, that does not establish the innocence. The question as to why he was being
arrested and then telling the complainant that he had not done a good thing to him and he
had deceived him rather goes to show that the accused was blaming the complainant for
having betrayed him. It was not a statement of innocence and on the contrary it was a
statement showing anguish that the complainant had got him caught. As noted above, the
recovery of the money has not been disputed. The evidence of PW 3 clearly establishes
the demand and acceptance of bribe and the recovery. The High Court had recorded
contradictory findings. On one hand it has noted that the recovery order contained the
signature of the accused but at another place it says that the copy of the recovery memo
was neither handed over to the accused nor his signature was obtained on that. With this
erroneous conclusion the High Court came to hold that the recovery memo was prepared
behind the back of the accused. According to the High Court's own conclusion it was not
really so. Interestingly, the suggestion made by the accused during the cross-examination
of PW 3 was that he had handed over the money to the accused stating that the amount is
the price of ghee for the Project Officer. This is an indirect way of accepting that money
has been received by him. In fact there was practically no denial of this aspect and the
recovery has also not been denied.
9. Above being the position, the order of the High Court is clearly indefensible and is set
aside. The order of conviction recorded by the trial court is restored. The custodial
sentence shall be one year which is the minimum sentence prescribed.
10. The appeal is allowed to the aforesaid extent.
Appeal allowed. .
AIR 2008 SUPREME COURT 2834 "N. Padmamma v. S. Ramakrishna Reddy"
(From : Andhra Pradesh)*
Coram : 2 S. B. SINHA AND LOKESHWAR SINGH PANTA, JJ.
Civil Appeal No. 3632 of 2008 (arising out of SLP (C) No. 19445 of 2006), D/- 16 -5
-2008.
N. Padmamma and Ors. v. S. Ramakrishna Reddy and Ors.
A.P. (Telangana Area) Abolition of Inams Act (8 of 1955), S.8 - ABOLITION OF
INAMS - PARTITION - Occupancy right granted in favour of one of family members -
Suit for partition for division of respective shares amongst members of a joint family -
Maintainability - Decision in 1995 AIR SCW 1843 not correctly decided - Matter referred
to larger Bench. (Paras 11, 16, 19)
Cases Referred : Chronological Paras
2005 AIR SCW 4796 : AIR 2005 SC 3520 (Ref.) 17
2003 (5) Andh LT 10816
1995 AIR SCW 1843 (Ref.) 6, 15, 19
AIR 1971 SC 2251 (Ref.) 6, 12, 15, 16
AIR 1957 SC 314 12
18 BHRC 52 16
T.L. Vishwanatha Iyer, Sr. Advocate, T. G.N. Nair, Rahul Shukla and R.V. Kameshwaran,
with him for Appellants; Dr. Rajiv Dhawan, Sr. Advocate, G. Ramakrishna Reddy (for
Mrs. D. Bharathi Reddy), with him for Respondents.
* L. P. A. No. 3 of 1993, D/- 11-8-2006 (A. P.).
Judgement
S. B. SINHA, J. :- Leave granted.
2. Whether the civil court has jurisdiction to entertain a suit for partition for division of
respective shares amongst the members of a joint family, when in respect of some of the
lands, occupancy right has been granted in favour of one of them in terms of the
provisions of the Andhra Pradesh (Telangana Area) Abolition of Inams Act, 1955 (for
short 'the Act') is the question involved herein.
3. The basic fact of the matter is not in dispute
@page-SC2835
4. One S. Ramakrishna Reddy was the owner of the properties. He had two sons, S.
Ramachandra Reddy and S. Anantharam Reddy. S. Ramachandra Reddy died in the year
1968. He had two wives. Plaintiffs-Appellants are the first wife and the daughter of S.
Ramachandra Reddy. The Defendants-Respondents are the son, second wife and the
daughter of said S. Ramachandra Reddy.
5. The Legislature of the State of Andhra Pradesh enacted the Andhra Pradesh (Telangana
Area) Abolition of Inams Act, 1955 which came into force on 20th July, 1955. In the year
1973, Section 8 was enacted in terms whereof a person in possession could be registered
as an occupant of the land from the date of vesting thereof. The first respondent was
granted occupancy right by the R.D.O., Hyderabad in terms of Section 8 read with
Section 10 of the Act. A suit for partition was filed on 3rd September, 1981. Some other
persons claiming right as 'Inamdars' filed an application before the District Collector
under Section 24 of the Act. However, the right of respondent No. 1 thereover was found
by the District Collector in terms of an order dated 5th August, 1985. A preliminary issue
was raised as regards the jurisdiction of the court which has been upheld by the impugned
judgment.
6

. Mr. T. L. Viswanatha Iyer, learned senior counsel appearing for the appellants, would
submit that the decisions of the courts below is based upon the decision of this Court in
Lokraj and others vs. Kishan Lal and others, [(1995) 3 SCC 291], which cannot be said to
have been correctly decided. Learned counsel pointed out that this Court in
Bhubaneshwar Prasad Narain Singh v. Sidheswar Mukherjee, [(1971) 1 SCC 556] upheld
the right of a co-sharer and recognized such right in the entire body of the co-sharers.
1995 AIR SCW 1843
AIR 1971 SC 2251

7. Dr. Rajiv Dhawan, learned senior counsel appearing on behalf of the respondents on
the other hand, would submit that the said Act is a complete Code itself. The contention
of the appellants that the registration of the land should not be granted in favour of
respondent No. 1 is not correct in view of the terminologies used in Section 8 of the Act.
It was contended that having regard to the provisions of Section 24 as also 29 of the Act,
the civil court has no jurisdiction to grant a decree of partition.

8. We, at the outset, may notice the relevant provisions of the Act.
Sections 3, 8, 10, 24 and 29 of the Act are as under :-
"Sec. 3 : Abolition and vesting of imams and the consequences thereof :- (1)
Notwithstanding anything to the contrary contained in any usage, settlement, contract,
grant sanad order or instrument, Act, regulation, rules or order having the force of law
and notwithstanding any judgment, decree or order of a Civil or Revenue or Atiyat Court,
and with effect from the date of vesting, all imams shall be deemed to have been
abolished and shall vest in the State.
(2) Save as expressly provided by or under the provisions of this Act and with effect from
the date of vesting the following consequences shall ensure, namely:
(a) xxxx
(b) all rights, title and interest vesting in the inamdar, kabiz-e-kadim, permanent tenant,
protected tenant and non-protected tenant in respect of the inam land, other than the
interests expressly saved by or under provisions of this Act and including those in all
communal lands, cultivated and uncultivated lands (whether assessed or not), waste
lands, pasture lands, forests, mines and minerals, quarries, rivers and streams, tanks and
irrigation works, fisheries and ferries, shall cease and be vested absolutely in the State
free from all encumbrances:
(c) to (f) xxxxxxxx
(g) the inamdar and any other person whose rights have vested in the State under clause
(b) shall be entitled only to compensation from the Government as provided for in this
Act;
(h) the relationship with regard to inam land as between the inamdar and kabiz-e-kadim,
permanent tenant, protected tenant or non-protected tenant shall be extinguished'
(i) xxxx
(3) xxxxxxxx
Sec. 8 : Registration of non-protected tenant as occupant :- (1) Every non-protected
tenant shall, with effect from the date of vesting subject to Section 37 of the A.P.
(Telangana Area) Tenancy and Agricultural Lands Act, 1950 be entitled to be registered
as an occupant of such inam lands in his possession as may be left over after the
allotment under Section 4 which, immediately
@page-SC2836
before the date of vesting, were under his personal cultivation and which, together with
any lands he separately owns and cultivates personally, are equal to four and a half times
the family holding.
(2) The non-protected tenant shall be entitled to compensation from the Government, as
provided under this Act in respect of inam lands in his possession in excess of the limit
prescribed in sub-section (1) whether cultivated or not.
(3) No non-protected tenant shall be registered as an occupant of any land under sub-
section (1) unless he pays to the Government as premium an amount equal to sixty-times
the land revenue for dry and twenty times for wet land. The amount of premium shall be
payable in not more than, ten annual instalments along with the annual land revenue and
in default of such payment, shall be recoverable as an arrears of land revenue due on the
land in respect of which it is payable.
Sec. 10 : Enquiry by Collector in certain cases: The Collector shall examine the nature
and history of all lands in respect of which an inamdar, Kabiz-e-kadim, permanent tenant,
protected tenant or non-protected tenant, claims to be registered as an occupant under
Sections 4, 5, 6, 7 and 8 as the case may be, and decide -
(a) in whose favour, and in respect of which inam lands, the claims should be allowed;
(b) the land revenue and the premium payable in respect of such lands.
Sec. 24 : Appeals from orders under Section 10 to prescribed authority :- (1) Any person
aggrieved by a decision of the Collector under Section 10 may, within 30 days from the
date of decision, or such further time as the prescribed authority may for sufficient cause
allow, appeal to the prescribed authority and its decision shall be final.
(2) If any question arises whether any building or land fails within the scope of Section 9
the same shall be referred to the prescribed authority whose decision shall be final.
Sec. 29. Savings :- Save as otherwise provided in this Act, no order passed by the
Collector or by Special Tribunal under this Act shall be liable to be cancelled or modified
except by the High Court as aforesaid or be questioned in any Court of law."
9. In terms of Section 3 of the Act all inam lands vest in the State of Andhra Pradesh with
effect from 20th July, 1955. Occupancy right as contemplated under Section 8 of the Act,
however, was to be granted with effect from 1st November, 1973. No doubt, grant of such
occupancy right is hedged with conditions as mentioned in Section 8 read with Section 10
of the Act; personal cultivation and possession inter alia being the relevant condition for
grant of such right.
10. What would be the meaning of the 'personal cultivation' and 'possession' is the
question. The properties were in possession of S. Ramachandra Reddy despite the vesting
of the land. Upon his death the parties hereto inherited his right, title and interest in the
properties. Respondent No. 1 being the only male member, assuming he had been
cultivating the said land, must be held to have been doing so for and on behalf of the
members of the joint family. There were 14 items of joint family properties. They were
living in a house. There is no dispute in regard to item Nos. 7 to 14. Item Nos. 1 to 6 of
the Schedule of the Plaint only were the subject matter of the said Act.
11. The said Act did not intend to deprive a co-sharer of his right to which he or she was
otherwise entitled to. The word 'person' cannot be given a limited meaning. It may be a
body of persons or association of person. When an occupancy right is granted in the name
of the Manager of the joint family it would enure for the benefit of the entire family. The
lands vested in the State. But as soon as the occupancy right is granted, in the event it is
held that the same inured to the benefit of the entire family, it becomes partible.
Occupancy right in favour of the first respondent has been granted on 24th October,
1978. In terms of Section 8 of the Act the same would be deemed to have been granted on
or from 20th July, 1955. The provisions, therefore, are required to be assigned proper and
effective meaning.
12

. This aspect of the matter has been considered in Bhubaneshwar Prasad Narain Singh v.
Sidheswar Mukherjee, [(1971) 1 SCC 556] wherein it was held AIR 1971 SC 2251,
Paras 8 and 9

"9. In our view the above decision is no authority for this broad proposition. In that case
the appellants who were mortgagees of an estate including Bakasht lands and other lands
filed a suit on their mortgage and tried
@page-SC2837
to follow up the preliminary decree which was obtained before the Act came into force by
a petition for passing a final decree. One of the questions before this Court was whether
the mortgage decree had become unexecutable in view of the provisions of the Act. It was
held that the net effect of Sections 3, 4 and 6 was that although on the vesting of the lands
in the State a settlement was deemed to be effected with the person in Khas possession in
law, there were two different transactions and the deemed settlement was in effect a
separate transaction creating new rights. The Court came to the conclusion that the only
remedy open to the decree-holders was that provided in Chapter IV of the Act i.e. a claim
under Section 14 before the Claims Officer for determining the amount of debt legally
and justly payable to each creditor in respect of his claim.

10. The Court was there dealing with the rights of the mortgage creditors after the Act
had come into force. Chapter IV of the Act made special provisions for dealing with the
rights of secured creditors and Section 4(1)(d) expressly provided for the abatement of all
suits and proceedings for the recovery of any money through proceedings which might be
pending on the date of vesting arising out of securities created by mortgage or a charge
on an estate or tenure. Here, however, we are not dealing with the claims of mortgagees
under Chapter IV. In this case we have to consider whether the appellants had laid a claim
which a co-sharer could not put forward except by pleading ouster or any other
independent ground. Even if they were in actual Khas possession within the meaning of
Section 2(k) of the Act it must be held that the plaintiff who was a co-sharer was in
constructive possession through the appellants as "under the law possession of one co-
sharer is possession of all the co-sharers". We see no reason to hold that the observations
of this Court to the above effect in P.L. Reddy v. L.L. Reddy are not applicable to the case
before us. The appellants do not claim to be trespassers on the property : neither did they
claim any title to the lands adversely to the plaintiff-respondent. The deeming provision
of Section 6 must therefore ensure for the benefit of all who in the eye of law would be
regarded as in actual possession. It follows that the plaintiff had not lost his share in the
Bakasht lands and had a right to them though not as tenure-holder or proprietor but
certainly as a Raiyat under the provisions of the Land Reforms Act. The appeal must
therefore be dismissed with costs." AIR 1957 SC 314 at P. 317

13. We will assume that the Act is a complete Code but its operation must be limited to
the purpose for which it was enacted. It is a well settled principle of law that a provision
in the statute ousting jurisdiction of the Court must receive strict construction.
14. The question, therefore, which arises for consideration is as to whether the civil
court's jurisdiction is completely ousted.
15

. In Lokhraj (supra) this Court referred to Bhubaneshwar Prasad Narain Singh (supra).
The judgment of this Court in Bhubaneshwar Prasad Narain Singh (supra) was, with
respect, not correctly read in Lokhraj (supra). Paragraph 4 of the said decision reads,
thus :- 1995 AIR SCW 1843
AIR 1971 SC 2251

"4. Consequent to the abolition, the preexisting right, title and interest of the inamdar or
any person having occupation of the inam lands stood divested and vested the same in the
State until re-grant is made. The inamdar, thereby lost the pre-existing right, title and
interest in the land. The right to partition itself also has been lost by the statutory
operation unless re-grant is made. We are not concerned with the consequences that
would ensue after re-grant of this appeal. Therefore, it is not necessary for us to go into
the question that may arise after the re-grant."
16

. The said decision, therefore, is not an authority for the proposition that only the person
in whose name occupancy right is granted became the sole beneficiary thereof.
Furthermore Bhubaneshwar Prasad Narain Singh (supra) was, in our opinion, again with
respect, had not been correctly applied. The Act contemplates resolution of dispute
between the Inamdar on the one hand and his lessees and assignees on the other. It does
not take into consideration the dispute, if any, inter se amongst the members of the joint
family, particularly when as on the date of grant of occupancy right there did not exist
any such dispute. The Act contemplates grant of decree for partition. It does not
contemplate a case where occupancy right is taken in the name of a person as
representing the entire joint family property. AIR 1971 SC 2251

@page-SC2838
Application of doctrine of trust is not contemplated in the said provision. Section 8 of the
Act must, therefore, be considered having regard to the provisions contained therein. The
Act contemplates registration of permanent tenants, protected tenants and non-protected
tenants. There are, thus, different types of tenants. Section 10 merely creates a forum for
determination of the entitlement under Sections 4 to 8 of the Act. It does not create a
forum for determination of the rights inter se between the parties claiming under the same
title.
Useful reference in this connection may be made to Shaik Sharfuddin alias Bukka
Sharfuddin vs. Joint Collector, R.R. District and Ors 2003 (5) A. L.T. 108.
Right of inheritance and succession is a statutory right. A right in a property which is
vested in terms of the provisions of the Hindu Succession Act cannot be taken away,
except in terms of provisions of another statute, which would have an overriding effect.
Such special statute should be a complete code. It shall ordinarily be a later statute.
Ordinarily again it must contain a non obstante clause.
Law of Primogeniture is no longer applicable in India. Such a provision may be held to
be unconstitutional being hit by Article 14 of the Constitution.
See Bhe and others v. Magistrate, Khayelistha and others [18 BHRC 52]
17

. Where the civil court's jurisdiction is barred expressly it must mean that the same would
be confined to the matters covered thereby or connected therewith. The right or the claim
must be necessarily required to be dealt with by the authorities under the Act. The
grievance/adjudicatory forum provided therein must be competent to resolve the dispute.
The right of property is a human right. The Act contemplates divesting of right of an
Inamdar. It does not contemplate cessation of a right of a co-sharer or recognition of a
right in favour of other co-sharer. The right has to be determined having regard to the
possession by way of personal cultivation. The word 'possession' in such cases should be
given a broader connotation. Possession of one sharer would be deemed to be the
possession of others. It is a legal concept. This legal concept cannot be held to have been
done away with under the Act. If a right of property is a human right as also a
constitutional right, the same cannot be taken away except in accordance with law.
Article 300 A of the Constitution protects such right. The provisions of the Act seeking to
divest such right, keeping in view of the provisions of Article 300 A of the Constitution of
India, must be strictly construed. (See Hindustan Petroleum Corpn. Ltd. v. Darius Shapur
Chenai, [(2005) 7 SCC 627]. 2005 AIR SCW 4796

18. The principle laid down in the said decision, having regard to concept of Article 300
A of the Constitution of India may be held to have some application in a case of this
nature. In terms of Hindu Succession Act, 1956 the right of succession is determined by
reason of the provisions thereof. It came into force with effect from 17th June, 1956. By
reason of a legal fiction created under the Act, the occupancy right is granted with effect
from 20th July, 1955. S. Ramachandra Reddy was alive then. What would be his status on
that date would be relevant. The legal fiction as is well known must be given its full
effect.
19
. We are, therefore, of the opinion that the decision of this Court in Lokhraj (supra) had
not been correctly rendered. The matter, therefore, requires consideration by a larger
Bench. It is directed accordingly. Let the records of the case be placed before the Hon'ble
the Chief Justice of India, 1995 AIR SCW 1843

Order accordingly. .
AIR 2008 SUPREME COURT 2838 "Tamil Nadu Electricity Board v. Status Spinning
Mills Ltd."
(From : Madras)*
Coram : 2 S. B. SINHA AND LOKESHWAR SINGH PANTA, JJ.
Civil Appeal No. 3940 of 2008, With C.A. Nos. 3941 to 3976, 4000 to 4015, 4080, 4081,
4083 and 4084 etc. etc. of 2008 (arising out of SLP (Civ) No. 23809 with 26309 etc. of
2005, 2026 and 21683 etc. of 2006 and 2385, 2391 of 2007, etc. etc.) D/- 16 -5 -2008.
Tamil Nadu Electricity Board and Anr. v. Status Spinning Mills Ltd. and Anr.
(A) INTERPRETATION OF STATUTES - Interpretation of Statutes - Exemption
notification - To be construed strictly as regards entitlement and broadly thereafter.
@page-SC2839

It may be true that the exemption notification should receive a strict construction but it is
also true that once it is found that the industry is entitled to the benefit of exemption
notification, it would receive a broad construction. (Para 28)
(B) Evidence Act (1 of 1872), S.115 - PROMISSORY ESTOPPEL - APPLICABILITY
OF AN ACT - Promissory estoppel - Applicability - Against statutes - Depends on nature
of statute and objects it seeks to achieve. (Paras 31, 32)
(C) T.N. Revision of Tariff Rates on Supply of Electrical Energy Act (1 of 1979), S.3, S.4
- INDUSTRY - WORDS AND PHRASES - G.O. Ms. No. 29, D/-31-01-1995 (as
amended on D/-04-02-1997) - Concessional Tariff - High Tension industries -
Amendment restricting benefit to industries set up before D/-15-02-1997 - Term 'set up' -
Defined as industry which is granted service connection - Giving literal construction may
result in anomaly - Provision has to be read down - Industries which have started
commercial production before cut-off date and had applied for connection - Held, would
be entitled to benefit of concession.
Words and Phrases - Set up.
The notification dated 31-1-1995 must be interpreted in a broad based manner, as a
promise was made to grant the concessional tariff not only for the new industries which
were to be set up thereafter but also to the pre-existing industries. By the 1997
amendment, the right accrued to them is sought to be taken away w.e.f. 15-2-1997. After
the 1997 amendment the concession is limited to industries set up before 15-2-1997. The
term 'set up' has been explained as industries to which service connection has been
granted. The proviso is an exception to the main clause whereas all industries which were
set up on or after 15th February become wholly ineligible for any tariff concession but
those who had set up prior thereto shall continue to avail themselves of the said tariff
concession. Legally, those who had not become consumer of electrical energy, but were
the potential consumers, they had not only applied for it but they were and, in fact, some
of them has also been gone into commercial production. Once they have set up the high
tension industries and who had gone up for commercial production must be held to have
set up the high tension industries. Once they have set up the high tension industries after
31st March, 1995, they became entitled to the benefit of concessional tariff for a period of
three years. Such concession was to be availed by them from the date of grant of service
connection. If they had already been granted service connection, they would continue to
avail themselves of the said tariff concession. However, the difficulty arises only in cases
where despite applying for grant of electrical communication, actual service connection
had not been granted. If a literal interpretation of the proviso is taken recourse to, the
same may result in an anomaly in the sense that in one case, connection may be granted
in one day and in another case, connection may not be granted for a long time. Because
of the acts of discrimination on the part of the officers of the Board or the State, the
entrepreneurs would suffer. It is in the aforementioned limited sense, the doctrine of
promissory estoppel will have application. If doctrine of promissory estoppel applies, the
right accrued in terms thereof cannot be withdrawn with a restospective effect. Some of
the industries had even installed generators. They had to do it. They inevitably had to do
it because the Board would not supply power. It would be too much to contend that even
those industries have not been set up as they have not become consumers. For the said
purpose, the proviso has to be read down. It must be made applicable to them who not
only had started commercial production before the said date, namely, 14-2-1997 but also
had applied and were otherwise ready to take electrical connections having deposited the
amount asked for. (Paras 35, 37, 45)
Cases Referred : Chronological Paras
2008 AIR SCW 1290 (Ref.) 41
2008 AIR SCW 3577 : 2008 (4) AIR Bom R 339 31
2007 AIR SCW 501 : AIR 2007 SC 797 (Rel. on, Pnt. A) 28, 31
2007 AIR SCW 531 : AIR 2007 SC 819 (Ref.) 23
2007 AIR SCW 3656 : AIR 2007 SC 2129 17
2007 AIR SCW 3752 : AIR 2007 SC 1984 (Rel. on, Pnt. B) 31, 37
2007 AIR SCW 6001 (Ref.) 40
2006 AIR SCW 1392 : AIR 2006 SC 1489 : 2006 (3) AIR Bom R 164 (Ref.) 23
2006 AIR SCW 1500 (Rel. on, Pnt. B) 31, 37
2006 AIR SCW 1642 (Rel. on, Pnt. A) 29
@page-SC2840

2006 AIR SCW 3627 : AIR 2006 SC 2652 (Ref.) 43


2006 AIR SCW 5272 (Ref.) 38
2006 AIR SCW 6304 : AIR 2007 SC 401 : 2007 (1) AIR Bom R 730 (Ref.) 43
2006 AIR SCW 6320 : AIR 2007 SC 414 (Ref.) 43
(2006) 1 SCC 746 (Ref.) 39
(2006) 12 SCC 753 (Ref.) 23
2005 AIR SCW 1781 : AIR 2005 SC 287 (Rel. on, Pnt. A) 28
2005 AIR SCW 6539 : AIR 2006 SC 674 (Rel. on) 16, 22
(2005) 1 SCC 625 (Ref.) 43
(2003) 2 All ER 615 32
2002 QB 35 : (2001) 4 All ER 138 32
(2000) Ch 162 32
1997 AIR SCW 3839 : AIR 1997 SC 3910 : 1997 All LJ 2202 (Rel. on, Pnt. B) 31
(1997) 3 SCC 398 (Ref.) 43
1995 AIR SCW 680 : AIR 1995 SC 874 (Rel. on) 43
1994 AIR SCW 3976 (Rel. on, Pnt. A) 28
AIR 1967 SC 509 (Rel. on) 16, 18, 22

Judgement
1. S. B. SINHA, J. :-Leave granted.
2. These appeals at the instance of the Tamil Nadu Electricity Board (for short "the
Board") and State of Tamil Nadu are directed against a judgment and order dated 19-7-
2005 passed by a Division Bench of the Madras High Court dismissing the writ appeals
filed by the appellants herein arising out of a judgment and order dated 23-4-1999 passed
by a learned Single Judge of the said Court.
3. The basic fact of the matter is not in dispute.
4. The State of Tamil Nadu despite the Parliamentary enactment of the Electricity
(Supply) Act, 1948 (for short "the 1948 Act") enacted the Tamil Nadu Revision of Tariff
Rates on Supply of Electrical Energy Act, 1978 (for short "the 1978 Act"); the relevant
provisions whereof are as under :
"3. Tariff rates for consumption of electrical energy
Notwithstanding anything contained in the Tamil Nadu Essential Articles Control and
Requisitioning (Temporary Powers) Act, 1949 (Tamil Nadu Act XXIX of 1949), the tariff
rates payable to the Tamil Nadu Electricity Board by any consumer on the electrical
energy supplied by the Board shall be as specified in the Schedule to this Act.
4. Power of the State Government to amend the Schedule -
The State Government may after taking into account the cost of production of energy, and
such other matters as may be prescribed by notification, amend the provisions of the
Schedule to this Act."
5. Pursuant thereto and in furtherance thereof, High Tension Supply tariff was prescribed.
The State issued a G. O. bearing No. G. O. Ms. No. 29 dated 31-1-1995 providing for
tariff concession for High Tension industries; the relevant portions whereof are as under :
"(a) In the case of new High Tension Industries to be set up in the areas other than the
Madras Metropolitan areas, the following concessional tariffs shall be charged for the
first three years from the date, the consumer is given service connection under high
tension tariff - I -

For the first year 60 per cent of the High Tension rates.
For the second year 70 per cent of the High Tension rates.
For the third year 80 per cent of the High Tension rates.
For the fourth year Full Tariff.

The above concession shall apply to both unit rates and maximum demand charges. This
concession shall not however, be applicable to an industry set up before the 3rd May,
1989. The concession shall not also be applicable to a consumer, who utilizes power from
his own generating units or makes other arrangements for production purposes and
utilizes the power supplied by the Board for auxiliary purposes only."
6. A memorandum was issued on or about 23-8-1995 to specify the time limit required for
dealing with the applications for grant of electrical connections as and when applications
therefor are filed which are in the following terms :
"Adverting to the above, the following further instructions are issued regarding disposal
of H. T. applications.
(i) The Superintending Engineers of all Elcy. Dist. Circle are requested to bestow their
efforts and thrive for clearing pending applications wherever possible.
(ii) The delays in processing the H. T. applications should be strictly avoided at all stages.
The revised flow chart showing the various stages of processing of the H. T. applications
is enclosed.
(iii) While scrutinizing the applications the defects observed in the application may be
conveyed to the applicants at the first instance itself and not on piece meal with a
@page-SC2841
view to speed up the disposal.
(iv) It is further to be reiterated that those H. T. applications for which supply could not
be extended within 18 months may be returned immediately to the applicants with a
request to renew after a specific date.
(v) If supply could be extended to applicants within 18 months but requires enhancement
of transformer capacity, improvement of existing lines etc., the load sanction may be
accorded pending execution of such improvement works stipulating the above conditions.
Depending on the readiness reported and also based on the merits of the each case of H.
T. extension, supply may be effected to H. T. applicants using the powers delegated to the
Chief Engineers (Distribution) vide memo. No. SE/IEMC/EE3/AEE1/802/91 dated 8-11-
1991.
All Superintending Engineers /Elecy. Distn. Circle are once again informed to ensure that
delays do not occur beyond a reasonable time for processing the H. T. applications as H.
T. services constitute major source of revenue for the Board and to bestow all efforts to
achieve the target fixed for the year 95-96 without fail".
7. The flow chart annexed thereto specified the maximum period of 18 month's time to be
taken from the date of filing of the application till the date of grant of connection.
8. The G. O. dated 31-1-1995 was amended on 14-2-1997 in the following terms :
"H. T. Tariff - I
There is increase in both demands and energy charges.
For consumption of energy during peak hours viz. 6.00 AM to 9.00 AM and 6.00 PM to
9.00 PM, the energy charges are to be billed at 20% extra. This will be implemented on
installation of 'Time of Day' meters.
New industries set up on or after 15-2-1997 are not eligible for any tariff concession.
However, in respect of H. T. industries set up in areas other than Chennai Metropolitan
area before 15-2-1997 shall continue to avail the tariff concession until the expiry of 3
years period reckoned from date of service concession."
By a G. O. Issued on 14-2-1997, the Schedule appended to the 1978 Act was amended as
under :
"(a) New High Tension Industries set up in any area on or after 15th February 1997, shall
not be eligible for tariff concession :
Provided that the High Tension Industries set up in any area other than Chennai
Metropolitan area before 15th February 1997 shall continue to avail themselves of the
said tariff concession until the expiry of the period of three years from the date on which
the consumer is given service connection.
(b) New industry to be set up in the areas other than the Chennai Metropolitan area which
will work night shift only and existing industry which has only night shift between 9.30
p.m. of a day and 5.30 a.m. of the next day, shall be given a concession of 40 per cent of
the appropriate rate for energy consumed during night shift only for a period of seven
months from July to January during a period of five years from the date of giving service
connection. This concession shall apply to energy rate arrived at after fixing the
concession if any:
Provided that in respect of those having one day shift and one night shift, the night shift
concession shall stand reduced from 40 per cent to 20 per cent."
9. By a letter dated 1-8-1997 addressed to the Chairman of the Board, the State
Government purported to clarify the meaning of the word "set up", stating :
"I am to state that the words "set up" would mean "obtained service connection". The
"New High Tension Industries set up in any area on or after 15th February, 1997 would
mean new High Tension Industries for which power service connection was actually
extended on or after 15-2-1997".
10. Questioning the validity of the said notification dated 14-2-1997 as also the letter
dated 1-8-1997, a large number of writ applications were filed contending that the same
was barred under the doctrine of promissory estoppel. Writ petitions were also filed
contending that the petitioners had set up their industries before 14-2-1997 and they were
entitled to the benefit of tariff concession in terms of the proviso appended to Clause (a)
of High Tension Tariff - I. By a judgment and order dated 23-4-1999, a learned single
Judge of the High Court disposed of the said batch of writ applications directing :
(a) Writ petitions which challenged the validity of the Notification dated 14-2-1997
withdrawing the concessions were dismissed.
@page-SC2842
(b) Those petitioners who had informed the Electricity Board on or before 14-2-1997
about their readiness for getting power connection or made applications to the Board
would be entitled to enjoy the tariff concessions for the full period of three years, from
the date of power connection as stipulated in the Notification dated 31-1-1995.
(c) Those writ petitioners who had not made applications or sent intimation to the
Electricity Board regarding their readiness to get power connection on or before 14-2-
1997 but have altered their position by establishing the industry on or before 14-2-1997
would be entitled to the tariff consessions as stipulated in the Notification dated 31-1-
1995 "provided if they are able to establish the same before the 2nd respondent -
Electricity Board."
11. Appeals were preferred thereagainst both by the Board and the State Government.
Some of the industries who have challenged the notification on the plea of applicability
of the doctrine of promissory estoppel also preferred appeals. Indisputably, during
pendency of the said appeals before the Division Bench of the High Court, the Schedule
to the 1978 Act was further amended by a notification dated 7-1-2000 whereby and
whereunder the following Explanations were added :
"Explanation 1. - For the purpose of this clause, an industry may be considered to be set
up on the date of obtaining High Tension Service Connection.
Explanation 2. - For the purpose of this clause "Existing Industry" means an industry,
which has not completed five years from the date, the consumer is given service
connection and which is still eligible to the concessional tariff rate."
12. The Division Bench of the High Court disposing of the said batch of appeals by a
judgment and order dated 19-7-2005 opined that it was not necessary to discuss the
individual fact of each of the respondents' case on the premise that the Secretary to the
Government could not clarify the amendments to the Schedule which were made by the
Governor. The said appeals were disposed of holding:
(i) ".......In the present case, the meaning of the words "set up is clear. These words mean
"erect or establish" as pointed out by the Supreme Court. If the high tension industry has
been erected or established before 15-2-1997, then it is entitled to the benefit of the tariff
concession, even though electricity connection may not have been given to it prior to 15-
2-1997.
(ii) We do not agree that it is the date on which the application for electricity supply is
received by the electricity board that is the relevant date. An application can be made
even before setting up the industry or it can be made after setting up the industry, but that
is wholly Irrelevant. It is the date when the industry has been set up that is the relevant
date.
(iii) We also do not agree that where the industries have not been set up, but are in the
process of being set up, yet, the benefit of the concession will be available, on the basis of
the principle of promissory estoppel. There is no question of promissory estoppel because
the language of the Notification is very clear and there is no estoppel against a Statute."
13. It is, thus, evident that two points were determined against the respondents. No appeal
has been preferred therefrom by the industries in question.
14. The core questions which arise for our consideration are as under :
(i) What is the connotation of the expression "set up" in the context of the concessional
tariff granted in favour of High Tension Industries for the first three years "from the date,
the consumer is given service connection ?.
(ii) Whether the expression "set up" appearing in the proviso to clause (a) of High
Tension Tariff would have to be construed in the context of the expression "shall continue
to avail themselves of the said tariff concession" appearing in the proviso ?
(iii) Whether the expression "set up" would also takes its colour from the context that the
continued tariff concession could be availed by the High Tension Industries "until the
expiry of the period of three years from the date on which the consumer is given service
connection ?
(iv) Whether the expression "set up" be not construed to ensure certainty amongst the
consumers who could continue to avail of the tariff concession and whether the date on
which the consumer is given a service connection provides such certainty ?
(v) Whether the Notification conferring tariff concessions could be construed liberally
@page-SC2843
to extend the benefit thereof by construction of the expression "set up" and by
introducing the element of uncertainty?
15. Mr. A. K. Ganguly, learned senior counsel appearing on behalf of the appellants
would submit that the State of Tamil Nadu intended to confer benefit on a class of people.
Such classes of people having been identified as consumers of High Tension Electrical
Energy, it was permissible for the State not only to fix a cut-off date but also to specify
the same so as to enable the industries to take the benefit only from the date when they
start consuming electrical energy. It was, however, submitted that there can be some
exceptional cases where despite filing of applications for grant of electrical connection,
there had been some avoidable delays on the part of the Board.
It was urged that for the purpose of grant of benefit of concession in tariff rate, the Board
must know who the beneficiaries were.
The learned counsel further submitted that the Division Bench of the High Court
committed a serious error insofar as it failed to take into consideration that a High
Tension industry gets ready to discharge the functions for which it was set up, only with
certain formalities as provided for in the Indian Electricity Act, 1910 (for short "the 1910
Act") and the Rules framed thereunder, viz, the Indian Electricity Rules, 1956 (for short
"the 1956 Rules") are complied with. An industry starts functioning, according to the
learned counsel, only when it is ready to operationalise its machinery which can only be
done when the power connection is granted. One of the essential pre-requisite therefor" is
certification and approval by the Inspector in terms of Section 37 of the 1910 Act and
Rule 63 of the 1956 Rules. The said Rule reads as under : "63. Approval by Inspector. -
(1) Before making an application to the Inspector for permission to commence or
recommence supply after an installation has been disconnected for one year and above at
high or extra high voltage to any person, the supplier shall ensure that the high or extra
high voltage electric supply lines or apparatus belonging to him are placed in position,
properly joined and duly completed and examined. The supply of energy shall not be
commenced by the supplier unless and until the Inspector is satisfied that the provisions
of rules 65 to 69 both inclusive have been complied with and the approval in writing of
the Inspector has been obtained by him :
Provided that the supplier may energise the aforesaid electric supply lines or apparatus
for the purpose of tests specified in rule 65 . . . . ."
Compliance of Rule 63 of the 1956 Rules, Mr. Ganguly submitted, is a definite parameter
to assess the readiness of industry to discharge the functions for which it had been set up.
The Government letter dated 1-8-1997 being clarificatory in nature, the same should be
given effect to. It was furthermore contended that the subsequent notification dated 7-1-
2000 whereby two Explanations were introduced is also clarificatory in nature as a bare
perusal of Explanation 1 would demonstrate that it merely clarified what was
inhered/implied in the said proviso which also reflects the undertaking of the government
which alone was competent to grant exemptions to concerned industries and/or to
withdraw or modify the same.
It was urged that the operative part of the order of the Division Bench should be suitably
modified to reflect that the setting up of an industry prior to 15-2-1997 implies that such
industry was ready to discharge all the functions for which it had been set up and that it
has already become a consumer having obtained High Tension service connection from
15-2-1997 and only those industries which have been set up before 15-2-1997 shall
continue to avail themselves of the said tariff concession until the expiry of the period of
three years from the date on which the consumer is given a service connection.
16

. Mr. K. Parasaran and Mr. S. Ganesh, learned senior counsel and other learned counsel
appearing on behalf of some of the respondents, on the other hand, would contend that
the amendments to the Schedule carried put by the notification are unreasonable being in
contravention of the statute. The word "set up" having been interpreted by this Court in
Commissioner of Wealth Tax v. Ramaraju Surgical Cotton Mills, Ltd. (1967 (1) SCR 761)
which in turn having been followed in Kabini Minerals (P) Ltd. and another v. State of
Orissa and others ((2006) 1 SCC 54), it must be held that the said word is in
contradistinction of the word "commence". Some AIR 1967 SC 509
2005 AIR SCW 6539

@page-SC2844
of the High Tension industries having already set up their business and having gone for
commercial production by reason of generators, it cannot be said that they had not been
set up. For the said purpose, not only the doctrine of promissory estoppel should be
applied, the wordings of the Section should be read down to mean that those industries
are entitled to the tariff concession who were not only granted connection but who ought
to have been granted connection.
It was pointed out that in most of the cases, not only the entrepreneurs altered their
position pursuant to or in furtherance of the promise made by the State in terms of the
notification dated 31-1-1995. They had applied for grant of connection much prior to the
cut off date fixed by the notification dated 14-2-1997. The entrepreneurs who had acted
pursuant to the promise made by the State should not be allowed to suffer for no fault on
their part.
17. The learned counsel in most of these matters have drawn our attention to the factual
matrix involved in each of the cases to contend that it is the Board and/or the State who is
responsible for delay in granting electric energy. They have not only set up the industry
and ready for commercial production but, in fact, some of them, pursuant to or in
furtherance of the permission granted by the State Electricity Board in terms of the
provisions of the Indian Electricity Rules, 1956 had set up diesel generating sets for the
purpose of running the factory which even stands accepted by the Board.

It was urged that although no appeals have been preferred from the judgment of the
Division Bench of the High Court, this Court in exercise of its jurisdiction under Order
41, Rule 33 of the Code of Civil Procedure may permit the respondents to raise the said
contention. Reliance in this behalf has been placed on UCO Bank v. Rajinder Lal Cooper
((2007) 6 SCC 694). 2007 AIR SCW 3656

18

. The High Court unfortunately did not go into the fact of each case. It proceeded on the
basis that the word "set up" should be given its dictionary meaning, i.e. erect or establish
in view of a decision of this Court in Ramaraju Surgical Cotton Mills Ltd. (supra). The
effect for a clarificatory order has also not been considered stating that the Secretary of
the Government cannot clarify an amendment to the Schedule which has been made by
the Governor. AIR 1967 SC 509

19. We wish that the Division Bench would have bestowed serious considerations on the
issues, as has been done by the learned single Judge.
20. The validity of the 1978 Act is not in question. It overrides the provisions of the 1948
Act. It empowered the State Government to amend the provisions contained in the
Schedule to the Act prescribing tariff rates payable by different classes of consumers for
supply of electrical energy by the Board taking into account the cost of production of
energy and such other matters as may be prescribed by notification. Indisputably, the
Schedule appended to the Act had been amended from time to time. The notification
dated 31-1-1995 was issued amending the Schedule. It for all intent and purport
substituted the then existing Schedule; Part A thereof dealt with tariff for High Tension
supply. It is only in that notification some concession, as noticed hereinbefore, had been
granted. The concession was to apply to both unit rates and maximum demand charges.
Certain limitations for grant of the said concession had also been specified.
21. The Schedule was amended by the notification dated 14-2-1997 reflecting the
operation of grant of concession tariff to those who had set up the new High Tension
industries on or after 15-2-1997. What is, therefore, significant for our purpose is the
meaning of the word "set up" vis-a-vis "commencement".
22

. A word cannot be assigned a meaning in vacuum. It has to be read in the context in


which it has been used. A decision which has been rendered on a different act dealing
with a different subject matter may not be apposite while construing the same term in
another statute. We may, however, at the outset notice the decision of this Court in
Ramaraju Surgical Cotton Mills Ltd. (supra) wherein this Court in the context of
interpretation of Section 5(1) (xxi) of the Wealth Tax Act held: AIR 1967 SC 509 at
P. 511

"..... A unit cannot be said to have been set up unless it is ready to discharge the function
for which it is being set up. It is only when the unit has been put into such a shape that it
can start functioning as a business or a manufacturing organization

@page-SC2845
that it can be said that the unit has been set up ......"
The decision centered round an exemption provision in respect of a portion of net wealth
of a company established with the object of carrying on an industrial undertaking in
India, as is employed by it in a new and separate unit set up after the commencement of
the Act. Although different terms "setting up" and "commencement" were used, this
Court opined that the word "set up" is equivalent to the word "established" and that a
business is established when it "is ready to commence business". It was furthermore
opined that an establishment could not be set up to be ready to commence business if it
not "set up".

The said view was reiterated by this Court in Kabini Minerals (P) Ltd. (supra) stating :
2005 AIR SCW 6539, Paras 10 and 11

"9. The expression "setting up" means, as is defined in the Oxford English Dictionary, "to
place on foot" or "to establish", and is in contradistinction to "commence". The
distinction is this that when business is established and is ready to commence business,
only then it can be said of that business that it is set up. But before it is ready to
commence business it is not set up. (See CWT v. Ramaraju Surgical Cotton Mills Ltd.)
AIR 1967 SC 509

10. In the said case, it was further held that the word "set up" is equivalent to the word
"established" but operations for establishment cannot be equated with the establishment
of the unit itself of (sic or) its setting up."
23

. What is necessary to take into consideration is that the Schedule appended to Section 3
of the 1978 Act is a part of the Act. It provides for High Tension tariff. It fixes up cut-off
dates. It is a piece of subordinate legislation. A subordinate legislation validly made may
have to be read in the same manner as if it is a part of the Act. For the said purpose,
although the same would be amenable to constitutional challenge on well-settled
principles of law, as noticed by this Court in Bombay Dyeing and Mfg. Co. Ltd. (3) v.
Bombay Environmental Action Group and Others, ((2006) 3 SCC 434), Vasu Dev Singh
and Others v. Union of India and Others, ((2006 12 SCC 753) and State of Kerala and
Others v. Unni and Another, ((2007) 2 SCC 365), the validity thereof is not under
challenge. 2006 AIR SCW 1392
2007 AIR SCW 531

24. The notification dated 31-10-1995 postulated concession to the new High Tension
industries "to be set up" for the first three years from the date the consumer is given a
service connection. It did not speak of commencement of production. It intended to attain
a certainty as to from which date such concession would be available. Grant of service,
connection was considered to be pre-requisite for grant of the concession. It is in the
aforementioned context, the impugned amendment will have to be construed. It fixes a
cut off date beyond which the concession shall not be available to the industries, viz.,
those who had set up in any area on or after 15-2-1997.
25. The proviso appended thereto, however, saves the cases of those who had availed
themselves of the said tariff concession and they who would continue to get the benefit
thereof until expiry of the period of three years from the date on which the consumer is
given service connection. The date on which the service connection is given, therefore,
plays an important role.
The clarification issued by the State during pendency of the appeals should have,
therefore, been considered by the High Court in its proper perspective. If it is clarificatory
in nature, it could be given a retrospective operation. Such a question, however, should
have been posed and answered. Furthermore, the letter dated 1-8-1997 was issued as
some confusion arose. When a subordinate legislation is made by the State Government,
it must be done in terms of the constitutional provision. An executive order is also issued
keeping in view the rules of executive business. It may not have the force of law but the
same may come within the purview of the well-known principle of contemporanea
exposito. Rules of executive construction are also relevant.
26. The Government placed the entire record before the learned single Judge. In his
judgment, the learned single Judge recorded :
"14. The learned Senior Counsel has produced the note file of the Government. From the
file, I am able to see that the Chairman, Electricity Board himself while constructing the
said expression 'set up' has stated that 'withdrawal of tariff' concession to the industries
set up with respect to the industries
@page-SC2846
during 1994 the date of load sanction was taken into account'. The Chairman has also
recommended to the effect that tariff concession which prevailed prior to 14-2-1997 can
be extended to consumers who have informed readiness on or before 14-2-1997 and
certified as such all Field Superintending Engineers on inspection of the industry after,
receipt of readiness report, and after 15-2-1997, due to the reason that it was not informed
prior to 14-2-1997."
27. It does not appear that the Chief Secretary of the State had issued the letter in
question upon following the procedure laid down in the Rules of Executive Business
framed under Article 166 of the Constitution of India. We are, however, not much
concerned therewith. It is not a case where the opinion of the Chief Secretary and/or for
that matter the State was decisive. In the matter of interpretation of statute, the Court has
the last say.
We have, therefore, to consider the issues raised before us independently.
28. Furthermore, concession is to be given in respect of payment of the charges for
electrical energy. When can it be given would be a question of fact. When it has been
given would be known to everybody. The bills are required to be paid only after electrical
energy is consumed. Question of availing the benefit of concession would not arise unless
a service connection is granted. For the said purpose, the definition of consumer, as
contained in Section 2 (1)(c) of the 1910 Act would be relevant. The benefit can be
availed by the consumer keeping in view the nature of concession granted. Exemption
notifications, therefore, require construction depending upon the tenor of the statute/
notification. Whether it should undergo a strict construction or a liberal construction is
one thing but it is another thing that whether a person is entitled to concession on a plain
reading of the notification.

It may be true that the exemption notification should receive a strict construction is has
been held by this Court in Novopan India Ltd., Hyderabad v. Collector of Central Excise
and Customs, Hyderabad, (1994 Supp (3) SCC 606), but it is also true that once it is
found that the industry is entitled to the benefit of exemption notification, it would
receive a broad construction. (See Tata Iron and Steel Co. Ltd. v. State of Jharkhand,
(2005) 4 SCP 272 and A. P. Steel Re-Rolling Mill Ltd. v. State of Kerala, (2007) 2 SCC
725). 1994 AIR SCW 3976
2005 AIR SCW 1781
2007 AIR SCW 501

A notification granting exemption can be withdrawn in public interest. What would be the
public interest would, however, depend upon the facts of each case.
29

. In State of Jharkhand v. Tata Cummins Ltd., ((2006) 4 SCC 57), this Court held : 2006
AIR SCW 1642, Para 16
"6. Before analysing the above policy read with the notifications, it is important to bear in
mind the connotation of the word "tax". A tax is a payment for raising general revenue. It
is a burden. It is based on the principle of ability or capacity to pay. It is a manifestation
of the taxing power of the State. An exemption from payment of tax under an enactment
is an exemption from the tax liability. Therefore, every such exemption notification has to
be read strictly. However, when an assessee is promised with a tax exemption for setting
up an industry in the backward area as a term of the industrial policy, we have to read the
implementing notifications in the context of the industrial policy. In such a case, the
exemption notifications have to be read liberally keeping in mind the objects envisaged
by the industrial policy and not in a strict sense as in the case of exemptions from tax
liability under the taxing statute."
30. The word "set up", therefore, was also required to be construed keeping in view the
provisions of the statute operating in the field, viz., the 1910 Act, the 1948 Act and the
1956 Rules.
31

. Validity of the notifications on the ground they are unreasonable has not been raised
before the High Court. We, therefore, cannot go into the issue. If that be so, it is difficult
to agree with Mr. Parasaran that we should undertake an exercise to interpret the
notifications in a manner which would not lead to unreasonableness. For the purpose of
declaring a statute unconstitutional, foundational facts have to be laid therefor. (See M/s.
Seema Silk and Sarees and Anr. v. Directorate of Enforcement and Ors., Criminal Appeal
@ SLP (Crl.) No. 6812 of 2007, decided on 12th May, 2008). Grounds are required to be
raised therefor. In absence thereof it would not be possible for us to enter into the debate
of constitutionality of the said provisions. The Division Bench of Reported in 2008 AIR
SCW 3577
2006 AIR SCW 1500
2007 AIR SCW 501
1997 AIR SCW 3839
2007 AIR SCW 3752

@page-SC2847
the High Court had rightly or wrongly opined that the doctrine of promissory estoppel has
no application. The fact that the said doctrine may apply even in relation to a statute is
beyond any dispute as has been held by this Court in Mahabir Vegetable Oils (P) Ltd. and
another v. State of Haryana and others, ((2006) 3 SCC 620); A. P. Steel Re-Rolling Mill
Ltd. (supra); Pawan Alloys and Casting Pvt. Ltd. v. U. P. State Electricity Board and
others ((1997) 7 SCC 251) and Southern Petrochemical Industries Co. Ltd. v. Electricity
Inspector and ETIO and others, ((2007) 5 SCC 447).
32. Strong reliance has been placed by Mr. Parasaran on Shah v. Shah, (2002 QB 35 :
(2001) 4 All ER 138) to contend that the doctrine of promissory estoppel is applicable
even in the field of the statute. Therein, it was held :
"In the Godden case (1997) NPC 1, an attempt was made to defeat by an estoppel the
provision in Section 2 (1) of the 1989 Act that "a contract, for the sale or other disposition
of an interest in land can only be made in writing; and only by incorporating all the terms
which the parties have expressly agreed in one document or, where contracts are being
exchanged, in each". Simon Brown LJ stated that the argument that "although Parliament
has dictated that a contract involving the disposition of land made otherwise than in
compliance with Section 2 is void, the defendants are not allowed to say so" was "an
impossible argument". Simon Brown LJ regarded the principle stated in Halsbury's Laws
as a "cardinal rule" the "absolute nature" of which cannot be "outflanked by one of the
equitable techniques or types of estoppels sought to be deployed in the present case".
Thorpe LJ and Sir John Balcombe agreed with Simon Brown LJ.
Yaxley v. Gotts (2000) Ch 162, was also concerned with Section 2 of the 1989 Act. An
oral agreement purporting to grant an interest in land, though void and unenforceable
under Section 2, was held still to be enforceable on the basis of a constructive trust under
Section 2 (5) which provides that "nothing in this section affects the creation or operation
of resulting, implied or constructive trusts". Robert Walker LJ stated, at p. 175 :
"Parliament's requirement that any contract for the disposition of an interest in land must
be made in a particular documentary form, and will otherwise be void, does riot have
such an obviously social aim as statutory provisions relating to contracts by or with
moneylenders, infants, or protected tenants. Nevertheless it can be seen as embodying
Parliament's conclusion, in the general public interest, that the need for certainty as to the
formation of contracts of this type must be general outweigh the disappointment of those
who make informal bargains in ignorance of the statutory requirement. If an estoppel
would have the effect of enforcing a void contract and subverting Parliament's purpose it
may have to yield to the statutory law which confronts it, except so far as the statutes
saving for a constructive trust provides a means of reconciliation of the apparent
conflict." Clarke LJ stated, at p. 182, that where a particular estoppel relied upon would
offend the public policy behind a statute it is necessary to consider the mischief at which
the statute is directed. Where a statute had been enacted as the result of the
recommendations of the Law Commission it is appropriate to consider those
recommendations. He stated that in his opinion : "the contents of that report (Transfer of
Land : formalities for Contracts of Sale etc. of Land (1987) (Law Com No 164)) will be
of the greatest assistance in deciding whether or not the principles of particular types of
estoppel should be held to be contrary to the public policy underlying the Act. In this
regard it seems to me that the answer is likely to depend upon the facts of the particular
case." Beldam LJ stated, at p. 191, that "The general principle that a party cannot rely on
an estoppel in the face of a statute depends upon the nature of the enactment, the purpose
of the provision and the social policy behind it."
The said decision has also been referred to Actionstrength Ltd. (trading as Vital
Resources) v. International Glass Engineering IN GL. EN SpA and another, (2003 (2) All
ER 615 at 619), but therein it was held that the doctrine of promissory estoppel may not
be applicable in case of a statute. As would appear from the discussions hereinafter,
applicability of the said doctrine would depend upon various factors including the nature
and purport of the Statute, the object it seeks to achieve, the purpose for grant of
concession/exemption etc.
@page-SC2848
33. It, therefore, depends on the nature of the statute as also applicability of the doctrine.
As noticed hereinbefore, even such a question had not been raised before the High Court.
34. The GOMs dated 31-1-1995 granted concession for the new high tension industries to
be set up. The eligibility for grant of concession therefore was for the industries which
were to be set up after the said date. The exemption does not stop at that. It is given
retrospective effect. It is extended to those industries which were set up after 3-5-1989.
However, exception therefor is sought to be curved out in respect of those industries who
had been utilizing power from their own generating units or making other arrangements
for production for the purposes and utilizing the power supplied by the Board in auxiliary
purposes only. The said notification therefore is a broad-based one. It not only is to apply
to those industries which were to be set up on or from 31-1-1995 but also to those which
were set up after 3-5-1989.
We may notice that concessional tariffs, however, were to be granted only for three years.
Those three years of concessional tariffs, therefore, were available to any industry which
had been set up after 3-1-1989 till the concession is withdrawn. Unlike other
notifications, no period is fixed. This Court, in a number of decisions, has considered the
effect of the notifications which were applicable for a fixed period, may be three years or
five years. The concession, although was to apply for a period of three years both in
respect of unit as also maximum demand charges, the same was meant to be applied to all
those industries who intended to set up their industries in the areas other than Madras
Metropolitan areas.
Indisputably, the respondents before us have started setting up their industries after the
said date. It is on the aforementioned backdrop, the impugned notification dated 14-2-
1997 requires interpretation. Those who had set up their industries have acquired a right,
viz. the right to obtain the tariff concession once a right is accrued in their favour. What
was promised to them was that they would be granted the tarif concession for a period of
three years @ 60%, 70% and 80% of the consumption charges.
35. Whether by reason of the said notification dated 14-2-1997, an accrued or vested right
has been taken away or not is the question. The core question, therefore, is as to whether
by reason of the said notification dated 31-1-1995, the entrepreneurs who had set up new
high tension industries after the said date have acquired any right pursuant thereto.
The notification dated 31-1-1995 must be interpreted in a broad-based manner, as a
promise was made to grant the concessional tariff not only for the new industries which
were to be set up thereafter but also to the pre-existing industries. The right accrued to
them is sought to be taken away w.e.f. 15-2-1997. Those who were eligible upto 14-2-
1997 to avail the benefit of the notification dated 31-1-1995 became ineligible. It is in the
aforementioned context, the proviso appended to clause (a) is required to be interpreted.
It has used the term 'set up' in any area other than Chennai Metropolitan area before 15-2-
1997. Should the rule of liberal interpretation be applied ? In our opinion, it should not
be. An accrued right ordinarily cannot be taken away with retrospective effect. It is not a
case where the notification has a retroactive operation. A person may apply on a
particular date for grant of electrical connection. He may get the electrical connection
within a few days or a few weeks or a few months. According to the State Electricity
Board, keeping in view the role played by all the three players, namely, the consumer, the
Board and the State, an outer limit of 18 mouths is taken for grant of supply.
36. There are cases before us wherefrom it appears that electrical connections had not
been provided owing to default on the part of the Electrical Inspector who is an officer of
the State and/or authorities of the Board, although prompt action had been taken in the
matter of depositing of money and/or complying with directions by the consumers.
37. A statute, even a subordinate legislation, may have to be construed reasonably. A
subordinate legislation ordinarily would not be given a retrospective effect. Retrospective
effect can be granted only if there exists any power in that behalf. There is nothing to
show that such a power has been conferred upon the State in terms of the Act. While
saying so, we are not oblivious of the situation that the State has a statutory power to fix
the tariff. It may also be true that when a statutory power is conferred, the State would
have power to amend,
@page-SC2849
alter, modify or rescind the same. The Court must also bear in mind that it may not cause
undue hardship. What we mean to say that if construction of a statute is possible as a
result of hardship is avoided, vis-a-vis, an undue hardship would be created, the Court
will prefer the former interpretation. The proviso is an exception to the main clause
whereas all industries which were set up on or after 15th February become wholly
ineligible for any tariff concession but those who had set up prior thereto shall continue to
avail themselves of the said tariff concession. Legally, those who had not become
consumer of electrical energy; but were the potential consumers, they had not only
applied for it but they were and, in fact, some of them has also been gone into
commercial production. Once they have set up the high tension industries and who had
gone up for commercial production must be held to have set up the high tension
industries. Once they have set up the high tension industries after 31st March, 1995, they
became entitled to the benefit of concessional tariff for a period of three years. Such
concession was to be availed by them from the date of grant of service connection. If they
had already been granted service connection, they would continue to avail themselves of
the said tariff concession. However, the difficulty arises only in cases where despite
applying for grant of electrical communication, actual service connection had not been
granted. If a literal interpretation of the proviso is taken recourse to, the same may result
in an anomaly in the sense that in one case, connection may be granted in one day and in
another case, connection may not be granted for a long time. Because of the acts of
discrimination on the part of the officers of the Board or the State, the entrepreneurs
would suffer. It is in the aforementioned limited sense, the doctrine of promissory
estoppel will have application. If doctrine of promissory estoppel applies, the right
accrued in terms thereof cannot be withdrawn with a retrospective effect.

(See Mahabir Vegetable Oils (P) Ltd. (supra); Southern Petrochemical Industries Co. Ltd.
(supra)) 2006 AIR SCW 1500
2007 AIR SCW 3752

38

. In MRF Ltd., Kottayam v. Asstt. Commissioner (Assessment) Sales Tax and others,
((2006) 8 SCC 702), this Court held :2006 AIR SCW 5272, Para 43

"In any event, the appeal preferred by the State of Kerala was dismissed and the
judgment of the High Court has therefore become final. Accordingly, it was held that
Section 10(3) does not confer the power to withdraw an exemption with retrospective
effect. Effect of this is that the amendment Notification SRO No. 38/98 has to be read so
as not to take away or disturb any manufacturer's pre-existing accrued right of exemption
for a period of 7 years. If SRO No. 38/98 is construed as now contended by the
respondent, then the inevitable consequence would be that SRO No. 38/08 would itself be
rendered ultravires Section 10(3) of the Act, and therefore, illegal, bad in law and null
and void ."
39. Yet again, in Tata Teleservices Ltd. v. Commissioner of Customs ((2006) 1 SCC 746),
this Court held :
"10. We are of the view that the reasoning of the Bombay Bench of the Tribunal as well
as that of the Andhra Pradesh High Court must be affirmed and the decision of the Delhi
Tribunal set aside insofar as it relates to the eligibility of LSP 340 to the benefit of the
exemption notification. The Andhra Pradesh High Court was correct in coming to the
conclusion that the Board had, in the impugned circular, predetermined the issue of
common parlance that was a matter of evidence and should have been left to the
Department to establish before the adjudicating authorities. The Bombay Bench was also
correction its conclusion that the circular sought to impose a limitation on the exemption
notification which the exemption notification itself did not provide. It was not open to the
Board to whittle down the exemption notification in such a manner. The exemption
notification merely reproduced the language of Entry 8525 20 17 and since the exemption
notification merely reproduced the tariff entry, the limitation sought to be imposed by the
Board would tantamount also to reading the limitation into the classification itself."
40

. In State of Orissa and others v. Tata Sponge Iron Ltd., ((2007) 8 SCC 189), this Court
held : 2007 AIR SCW 6001

"20. In view of the clear legal provision as also the aforementioned Notification dated 23-
9-1992, there cannot be any doubt whatsoever that the exemption in respect of deferment
of sales tax having been provided
@page-SC2850
for under the Orissa Sales Tax Act as also the notification issued thereunder, the High
Court, in our opinion, is correct in taking its view."
41

. In State of Kerala and Others v. Kurian Abraham (P) Ltd. and Another, ((2008) 3 SCC
582), it was opined : 2008 AIR SCW 1290

"23. Tax administration is a complex subject. It consists of several aspects. The


Government needs to strike a balance in the imposition of tax between collection of
revenue on one hand and business-friendly approach on the other hand. Today,
Governments have realized that in matters of tax collection, difficulties faced by the
business have got to be taken into account. Exemption, undoubtedly, is a matter of policy.
Interpretation of an Entry is undoubtedly a quasi-judicial function under the tax laws.
Imposition of taxes consists of liability, quantification of liability and collection of taxes.
Policy decisions have to be taken by the Government. However, the Government has to
work through its senior officers in the matter of difficulties which the business may face,
particularly in matters of tax administration. That is where the role of the Board of
Revenue comes into play. The said Board takes administrative decisions, which includes
the authority to grant Administrative Reliefs. This is the underlying reason for
empowering the Board to issue orders, instructions and directions to the officers under
it."
42. It is not case where decisions were altered pursuant to any representation made by the
State. Concessions in tariff had been granted by reason of a statutory provision. Such
concessions could also be withdrawn. If the appellants have not altered their position
pursuant to any promise, the doctrine of promissory estoppel would not apply. If that be
so, the question of any right being vested in the appellants would also not apply. In any
event, the reasonableness of the statute was not the subject-matter of the writ petition.
The provisions have not been sought to be declared ultra vires. Even otherwise, the State
while amending statute stated about the public interest necessitated the same. When a
statute is amended keeping in view the public interest even the concession can be
withdrawn with retrospective effect.
43

. In Kasinka Trading and Anr. v. Union of India and Anr., ((1995) 1 SCC 274), the power
of the State to change its policy decision in public interest was emphasized. It was held
that the power which can be used for grant of concession, namely, Section 25(1) of the
Customs Act itself is the source to rescind the earlier notification, stating : 1995 AIR
SCW 680, Paras 22 and 23

"Since, the notification had been issued under Section 25(1) of the Act, the very same
power was available to the authority for rescinding or modifying that notification and
appellant ought to have known that the said notification was capable of or liable to be
revoked, modified or rescinded at any time even before the expiry of 31-3-1981 if the
'public interest' so demanded. To hold that after the Government had issued the
Notification No. 66 of 1979 indicating that it was to remain operative till 31-3-1981, it
could not be rescinded or modified before the expiry of that date would amount to
prohibiting the Government from discharging its statutory obligation under Section 25(1)
of the Act, if it was satisfied that it was in the 'public interest' to withdraw, modify or
rescind the earlier notification. The plain language of Section 25 of the Act is indicative
of the position that it is the public interest and public interest alone which is the dominant
factor. It is not the case of the appellants that the withdrawal of Notification No. 66 of
1979 by the impugned notification was not in 'public interest'. Their case, however, is that
relying upon the earlier notifications they had acted and the Government should not be
permitted to go back on its assurance as otherwise they would be put to huge loss. The
Courts have to balance the equities between the parties and indeed the Courts would bind
the Government by its promise 'to prevent manifest injustice or fraud'."
It was further held :
"23. The appellants appear to be under the impression that even if, in the altered market
conditions the continuance of the exemption may not have been justified, yet,
Government was bound to continue it to give extra profit to them. That certainly was not
the object with which the notification had been issued. The withdrawal of exemption "in
public interest" is a matter of policy and the Courts would not bind the Government to its
policy decisions for all times to come,
@page-SC2851
irrespective of the satisfaction of the Government that a change in the policy was
necessary in the "public interest". The Courts, do not interfere with the fiscal policy
where the Government acts in "public interest" and neither any fraud or lack of bona fides
is alleged much less established. The Government has to be left free to determine the
priorities in the matter of utilisation of finances and to act in the public interest while
issuing or modifying or withdrawing an exemption notification under Section 25(1) of the
Act."

Kasinka Trading (supra) has been followed in many cases including, Shrijee Sales
Corporation and Anr. v. Union of India, ((1997) 3 SCC 398); Bannari Amman Sugars Ltd.
v. Commercial Tax Officer and Ors., ((2005) 1 SCC 625); Kuldeep Singh v. Govt. of
NCT of Delhi, ((2006) 5 SCC 702), M. P. Mathur and Ors. v. DTC and Ors., ((2006) 13
SCC 706) and Ramchandra Murarilal Bhattad and Ors. v. State of Maharashtra and Ors.,
((2007) 2 SCC 588). 1995 AIR SCW 680
2006 AIR SCW 3627
2006 AIR SCW 6320

44. A distinction must be made between a policy decision and a statute. Whereas prima
facie a policy decision may not have any retroactive operation, a statute may have. Only
because it affects a past transaction the same, by itself, would not come in the way of the
legislature in enacting an enactment or the executive government to exercise its power of
subordinate legislation.
45. We have noticed hereinbefore that some of the industries had even installed
generators. They had to do it. They inevitably had to do it because the Board would not
supply power. Would it not be too much to contend that even those industries have not
been set up as they have not become consumers? We think that for the said purpose, the
proviso has to be read down. It must be made applicable to them who not only had started
commercial production before the said date, namely, 14-2-1997 but also had applied and
were otherwise ready to take electrical connections having deposited the amount asked
for.
46. Those hard cases, even according to Mr. Ganguly, should be brought within the
purview of the proviso.
We, therefore, hold :
1. As the concession had been granted by the State, it had the power to withdraw the
same.
2. It is not a case where in view of the doctrine of promissory estoppel, the State could
not have in law amended the Schedule.
3. In view of existence of public interest the doctrine of promissory estoppel would have
no application.
4. Even otherwise the appellants having not preferred appeals against the judgment of the
Division Bench of the High Court, the said questions cannot be permitted to be raised
before us.
5. Proviso appended to the main provision should be read down as stated in paragraphs
44 and 45 supra.
6. In view of our findings aforementioned, we have not gone into the merit of the matter
involved in each case separately.
We direct accordingly. The matters would now be examined by the Appropriate Authority
of the Board, as directed by the High Court in individual cases. The appeals are allowed
with the aforementioned directions. No costs.
Order accordingly. .
AIR 2008 SUPREME COURT 2851 "A. P. S. R. T. C. v. K. Hemalatha"
(From : Andhra Pradesh)*
Coram : 2 Dr. A. PASAYAT AND P. SATHASIVAM, JJ.
Civil Appeal Nos. 3623-3626 of 2008 (arising out of SLP (C) Nos. 10950-10953 of
2005), D/- 16 -5 -2008.
A.P.S.R.T.C. and Anr. v. K. Hemalatha and Ors.
(A) Motor Vehicles Act (59 of 1988), S.168 - MOTOR VEHICLES - NEGLIGENCE -
Motor Accident - Composite negligence and contributory negligence - Distinction
between two stated.
'Composite negligence' refers to the negligence on the part of two or more persons.
Where a person is injured as a result of negligence on the part of two or more
wrongdoers, it is said that the person was injured on account of the composite negligence
of those wrongdoers. In such a case, each wrongdoer, is jointly and severally liable to the
injured for payment of the entire damages
@page-SC2852
and the injured person has the choice of proceeding against all or any of them. In such a
case, the injured need not establish the extent of responsibility of each wrongdoer
separately, nor is it necessary for the Court to determine the extent of liability of each
wrong-doer separately. On the other hand where a person suffers injury, partly due to the
negligence on the part of another person or persons, and partly as a result of his own
negligence, then the negligence on the part of the injured which contributed to the
accident is referred to as his contributory negligence. Where the injured is guilty of some
negligence, his claim for damages is not defeated merely by reason of the negligence on
his part but the damages recoverable by him in respect of the injuries stands reduced in
proportion to his contributory negligence. (Para 10)
(B) Motor Vehicles Act (59 of 1988), S.168, S.147 - MOTOR VEHICLES -
NEGLIGENCE - PRINCIPLES - Motor Accident - Composite vis-a-vis contributory
negligence - Accident between motor bike and bus - Deceased bike rider himself was
found partly liable in view of his negligence - Principle of 'composite negligence' will not
apply - Nor can there be automatic inference that negligence is 50 : 50 - By examining
extent of contributory negligence of bus driver, proportion can be fixed at 1 : 4 instead of
1 : 2 between deceased and Road Transport Corporation.
2008 AIR SCW 2045, Rel. on.
CMA Nos. 2913, 2925 of 2000, 283 and 551 of 2001, 28-10-2004 (AP), Reversed.
(Paras 8, 11)
Cases Referred : Chronological Paras
2008 AIR SCW 2045 (Rel. on) 12
R. Santhan Krishnan, Praveen K. Pandey and D. Mahesh Babu, for Appellants; K.
Maruthi Rao, Ms. K. Radha and Mrs. Anjani Aiyagari, for Respondents.
* CMA Nos. 2913, 2925 of 2000 and 283, 551 of 2001, D/- 28-10-2004 (AP).
Judgement
1. Dr. ARIJIT PASAYAT, J. :-Leave granted.
2. Challenge in these appeals is to the judgment of a learned single Judge of the Andhra
Pradesh High Court disposing of several appeals filed under Section 173 of the Motor
Vehicles Act, 1988 (in short the 'Act'). Appeals were filed by the claimants as well as the
present appellant-Corporation and its functionaries. By the impugned judgment the High
Court partly allowed the appeal filed by the claimant while dismissing the appeal filed by
the appellant-Corporation. One K. Lingam lost his life purportedly in a vehicle accident.
His widow and the minor children claimed compensation. Similarly his widow Smt. K.
Hemlatha also claimed compensation for about Rs. 8,00,000/- while the injured claimant
in respect of the same accident claimed compensation of Rs. 1,00,000/-. It was the case of
the claimants that on 19-3-1998 the deceased and injured claimant in O.P. No. 878 of
1998 were proceeding on motor bike bearing No. AP. 10J 5350 towards Yadagirigutta and
when they reached the RTC bus depot at Yadagirigutta, bus bearing No AP 9Z 3972
belong to APSRTC, came from back side and dashed the motor cycle. In the said
accident, the deceased and claimant suffered grievous injuries. At first instance, both
were admitted in Government Hospital, Bhongir and thereafter they were shifted to
Gandhi Hospital, Secunderabad. Considering the serious condition of the deceased he
was shifted to CDR Hospital, Hyderabad, where he succumbed to injuries on 24-3-1998.
On a complaint lodged to the police, a case in Crime No. 16 of 1998 was registered on
the file of the Police Station, Yadagirigutta. It was the further case of the claimants that
the deceased was a Class-I contractor and was an income-tax assessee and was doing
high magnitude civil contracts. Pleading that due to sudden and untimely death of the
deceased, they lost dependency, they claimed compensation which included non-
pecuniary damages on account of loss of estate, and loss of consortium. So far O.P. No.
878 of 1998 is concerned, the same was filed by the wife of the deceased who was also
injured in the same accident, claiming compensation on account of medical expenditure,
pain and suffering and disability. The said claim was resisted by the appellant Andhra
Pradesh State Road Transport Corporation (in short the 'Corporation') by filing counter-
affidavit before the Tribunal. It was the case and it was their specific case that the bus did
not hit the motor bike. Further, it was their case that on seeing the speeding bus the
deceased himself got puzzled and skidded of the road; as such, the deceased and claimant
suffered injuries. Precisely, it was the case of the Corporation that the bus of the
Corporation did not hit the motor bike at all; as such, there was no negligence on the part
of the driver of the
@page-SC2853
bus of the Corporation, to claim compensation from it.
3. The Tribunal in the two claim petitions framed issues. After taking note of the evidence
on record, it was held that the deceased was aged 41 years, his earning was about Rs.
5,000/- per month and after deducting 1/3rd for personal expenses the contribution to the
family was around Rs. 3,400/- p.m. The annual contribution was Rs. 40,800/-. After
applying multiplier of 11, compensation of Rs. 4,48,800/- was awarded. Additionally, a
sum of Rs. 70,000/- for medical expenses, transportation charges, funeral expenses and
the like was awarded. In other words in respect of claim for the death of the deceased Rs.
5,18,800/- was fixed as the amount of compensation. But since the Tribunal held that
there was contributory negligence, 1/3rd deduction was made. Interest at the rate of 12%
was awarded, from the date of claim. In the petition in respect of injuries a sum of Rs.
25,000/- was awarded but after making deduction of 1/3rd the amount was fixed as Rs.
16,666/- together with interest at the rate of 12% per annum.
4. Both the claimants and the Corportion filed appeal. As noted above the appeals filed by
the claimant was partially allowed while the appeal filed by the Corporation was
dismissed. Primarily the High Court came to hold that there was no question of any
contributory negligence.
5. In support of the appeal, learned counsel for the appellant submitted that the High
Court has misread the evidence on record. The Tribunal has referred to the evidence on
record to conclude that the deceased was also partially responsible for the accident and
therefore it clearly held that there was contributory negligence. However, the proportion
of 1 : 2 i.e. between the deceased and the Corporation, as fixed by the Tribunal, was not
correct. It is also pointed out that the rate of interest as awarded is extremely high.
6. Learned counsel for the respondent on the other hand supported the judgment of the
High Court.
7. To determine the question as to who contributed to the happening of the accident, it
becomes relevant to ascertain who was driving his vehicle negligently and rashly and in
case both were so doing who were more responsible for the accident and who of the two
had the last opportunity to avoid the accident. In case the damages are to be apportioned,
it must also be found that the plaintiff's fault was one of the causes of the damage and
once that condition is fulfilled the damages have to be apportioned according to the
apportioned share of the responsibility. If the negligence on the plaintiff's part has also
contributed to damage this cannot be ignored in assessing the damages. He can be found
guilty of contributory negligence if he ought to have foreseen that if he did not act as a
reasonable, reasoned man, he might be hit himself and he must take into account the
possibility of others being careless.
8. The Tribunal has noticed that the deceased was driving vehicle at high speed with a
view to attend the marriage function. Manner of the accident as deposed by the claimant's
witnesses indicate that the deceased was partially responsible for the accident. The High
Court was wrong in holding that the deceased had not contributed to the accident and
there was no contributory negligence. Taking into account the evidence of the witnesses it
can be certainly said that there was contributory negligence. The proportion can be fixed
at 1 : 4. From the compensation as awarded a sum of Rs. 1,00,000/- with round figures
needs to be deducted. Therefore, the compensation is fixed at Rs. 4,18,800/-. Considering
the date of the accident, the rate of interest should be 8%.
9. In an accident involving two or more vehicles, where a third party (other than the
drivers and/or owners of the vehicles involved) claims damages for loss or injuries, it is
said that compensation is payable in respect of the composite negligence of the drivers of
those vehicles. But in respect of such an accident, if the claim is by one of the drivers
himself for personal injuries, or by the legal heirs of one of the drivers for loss on account
of his death, or by the owner of one of the vehicles in respect of damages to his vehicle,
then the issue that arises is not about the composite negligence of all the drivers, but
about the contributory negligence of the driver concerned.
10. 'Composite negligence' refers to the negligence on the part of two or more persons.
Where a person is injured as a result of negligence on the part of two or more
wrongdoers, it is said that the person was injured on account of the composite negligence
@page-SC2854
of those wrongdoers. In such a case, each wrongdoer, is jointly and severally liable to the
injured for payment of the entire damages and the injured person has the choice of
proceeding against all or any of them. In such a case, the injured need not establish the
extent of responsibility of each wrongdoer separately, nor is it necessary for the Court to
determine the extent of liability of each wrong-doer separately. On the other hand where
a person suffers injury, partly due to the negligence on the part of another person or
persons, and partly as a result of his own negligence, then the negligence on the part of
the injured which contributed to the accident is referred to as his contributory negligence.
Where the injured is guilty of some negligence, his claim for damages is not defeated
merely by reason of the negligence on his part but the damages recoverable by him in
respect of the injuries stands reduced in proportion to his contributory negligence.
11. Therefore, when two vehicles are involved in an accident, and one of the drivers
claims compensation from the other driver alleging negligence, and the other driver
denies negligence or claims that the injured claimant himself was negligent, then it
becomes necessary to consider whether the injured claimant was negligent and if so,
whether he was solely or partly responsible for the accident and the extent of his
responsibility, that is his contributory negligence. Therefore where the injured is himself
partly liable, the principle of 'composite negligence' will not apply nor can there be an
automatic inference that the negligence was 50:50 as has been assumed in this case. The
Tribunal ought to have examined the extent of contributory negligence of the appellant
and thereby avoided confusion between composite negligence and contributory
negligence. The High Court has failed to correct the said error.
12

. The above position was highlighted in T. O. Anthony v. Karvarnan and Ors. (2008 (3)
SCC 748). 2008 AIR SCW 2045

13. Appeals are allowed to the aforesaid extent. The proportion in which the payment to
the claimants have to be made shall be the same as was fixed by the Tribunal.
Appeals allowed. .
AIR 2008 SUPREME COURT 2854 "State of U. P. v. M/s. Swadeshi Polytex Ltd."
(From : 2006 (5) ALJ 67)
Coram : 2 TARUN CHATTERJEE AND HARJIT SINGH BEDI, JJ.
Civil Appeal Nos. 3840 with 3839 of 2008 (arising out of SLP (C) Nos. 21002 with 3272
of 2006), D/- 16 -5 -2008.
State of U.P. and Ors. v. M/s. Swadeshi Polytex Ltd. and Ors.
U.P. Industrial Peace (Timely Payment of Wages) Act (5 of 1978), S.3(1) - U.P.
Zamindari Abolition and Land Reforms Rules (1952), R.285, R.283, R.246 - WAGES -
AUCTION SALE - LAND REFORMS - INDUSTRY - Recovery of wages as arrears of
land revenue - Failure of Company to pay wages due to its employees - Proclamation for
sale of property of Company issued - Properties found to be not properly valued -
Personal service of sale proclamation was made on Chowkidar of Company and not on
actual defaulter - Auction sale - Deposit of 25% of auction price on date of auction by
way of draft procured from Bank 460 Kms. away from that place - Was impossibility -
Remaining 75% amount not deposited within time - Company cannot be said to have not
suffered prejudice in auction proceedings - Setting aside of sale on ground of
irregularities in issuance of sale proclamation and conducting auction - Not interfered
with - However, direction to initiate action against concerned officers who conducted sale
- Not justified. (Paras 13, 15, 17, 20)
Cases Referred : Chronological Paras
(2005) W.P. No. 50571 of 2002, D/-13-01-2005 (Reported in 2005 (3) ESC 1676) (All)
3
2004 AIR SCW 2987 : AIR 2004 SC 2421 13, 14
2004 AIR SCW 3263 : AIR 2004 SC 3392 (Ref.) 13
2000 AIR SCW 500 : AIR 2002 SC 834 14
1995 AIR SCW 2566 : AIR 1995 SC 1632 14
1992 AIR SCW 3629 : AIR 1993 SC 935 : 1992 All LJ 1202 14
AIR 1957 SC 397 14
AIR 1954 SC 349 (Ref.) 16
Dinesh Dwivedi, Ashok H. Desai, R.F. Nariman, Rakesh Dwivedi, P.H. Parekh, Sr.
Advocates, D.K. Goswami, Manoj Kumar Dwivedi, Gunnam Venkateswara Rao,
Kamlendra Mishra, Ravi Prakash Mehrotra,
@page-SC2855
Mrs. Deepti R. Mehrotra, Garvesh Kabra, Ms. Ranjeeta Rohatgi, Sameer Parekh, Ms.
Deeksha Rai (For M/s. Parekh and Co.) and Shrish Kumar Misra, for the Appearing
Parties.
Judgement
1. HARJIT SINGH BEDI, J. :-Leave granted.
2. Respondent No.1, M/s. Swadeshi Polytex Limited (hereinafter referred to as "SPL") a
company registered under the Companies Act, 1956 and presently a sick unit has its
registered Office at Kavi Nagar, Industrial Area, Ghaziabad. Concededly approximately
33% of the shares of the SPL are held by Swadeshi Cotton Mills Limited, Kanpur (a unit
of the National Textile Corporation, a Government Enterprise) about 28% and 15% by
M/s. Paharpur Cooling Towers Limited and some financial institutions respectively, and
the remaining 23% or so by the general public. It is on record that the CMD of the
National Textile Corporation Ltd. is holding the charge of SPL and steps are underway
for the rehabilitation of the company. It appears that till year 1996-97, SPL was doing
reasonably well whereafter a financial crisis seems to have set in, forcing its closure on
30th September 1998. As SPL was unable to pay the wages due to its employees, several
applications were filed by its workmen under the provisions of the Uttar Pradesh
Industrial Peace (Timely Payment of Wages) Act, 1978 (hereinafter called the 1978 Act).
A recovery certificate was thereafter issued under sub-section (1) of Section 3 of the 1978
Act and pursuant thereto, the Company was called upon to make good the wages due to
the workmen and on its inability to do so, the authorities proceeded to recover the
amounts due as arrears of land revenue. A report was thereafter submitted by the Amin on
7th January 2005 which was endorsed by the Sub-Divisional Magistrate, Ghaziabad in
his communication dated 10th February 2005, whereupon an attachment notice in Form
73-D was issued and a proclamation for the sale of the property on 23rd February 2005
was also ordered. The proclamation was however cancelled by the SDM, Ghaziabad and
on reconsideration, an order dated 1st April 2005 was passed and the Tahsildar,
Ghaziabad was directed to hold the auction on 2nd May 2005 after giving wide publicity
and after the properties had been properly valued. A fresh proclamation was accordingly
issued by the Sub-Divisional Magistrate, Ghaziabad on 1st April 2005 itself, without
disclosing the details of the properties or their estimated value as also the date of the
auction. An auction notice was, however, published in "Amar Ujala" on the 22nd April
2005 indicating that the estimate value of the properties was about 27 Crores and that the
transfer of the property pursuant to the auction would be made on the terms and
conditions stipulated by the U. P. State Industrial Development Corporation (hereinafter
called the UPSIDC) the present appellant. It is also the case of the appellant herein that
the personal service of the sale proclamation was also made on the Chowkidar of the SPL
on 21st April 2005. The auction was in fact held on the stipulated day i.e. 2nd May 2005
and the UPSIDC was found to be the highest bidder. The recovery certificate issued by
the Deputy Labour Commissioner and the auction notice dated 22nd April 2005 was
challenged by SPL by way of Writ Petition No. 35005 of 2005 referring to the
irregularities in the issuance of the sale proclamation and the auction notice and it was
prayed that the proceedings be quashed. A reply was filed in response to the Writ Petition
but the petition was ultimately dismissed with the observation that repeated attempts to
recover the dues had failed on account of the recalcitrant attitude of SPL and that the
procedural defects which had been pointed out could be challenged by filing objections
under Rule 285(1) of the Uttar Pradesh Zamindari Abolition and Land Reforms Rules,
1952 (hereinafter called the "Rules"). Several objections were accordingly filed with
respect to the auction and the preceding events, but the Commissioner, Meerut Division,
in his order dated 24th June 2005 dismissed the objections. Aggrieved by the order dated
24th June 2005, SPL preferred a revision petition before the Board of Revenue under
Section 293 of the U. P. Zamindari Abolition and Land Reforms Act, 1950 (hereinafter
called the "Act") read with Section 219 of the Land Revenue Act but this petition too was
rejected by order dated 9th September 2005. This order was challenged before the
Lucknow Bench of the Allahabad High Court in Writ Petition No. 5160/2005 and it was
prayed, inter alia, that the aforesaid order and the order dated 24th June 2005 be set aside
and that the entire auction proceedings dated 2nd May 2005 be quashed. The High Court
in its interim
@page-SC2856
order dated 20th September 2005 directed the SPL to deposit a sum of Rs. 50 Lacs within
a period of 30 days and in the meanwhile, directed that the sale be not confirmed.
Aggrieved by the order dated 20th September 2005, the employees of the SPL filed a
Special Leave Petition and in its order dated 5th December 2005, this Court directed that
if the writ petition was not disposed of in the course of the week, the interim order passed
by the High Court would stand vacated. The High Court, however, in its judgment dated
3rd January 2006 allowed the writ petition with costs of Rs. 50,000/- and also passed
strictures against the officers of the State Government who had been instrumental in
arranging the auction. It is against this order that three Special Leave Petitions have been
filed which are SLP (Civil) No. 3272/2006 (U. P. State Industrial Development
Corporation and Anr. v. M/s. Swadeshi Plytex Ltd. and Ors.), SLP (Civil) No. 2858/2006
(M/s. Swadeshi Polytex Ltd. Karamchari Kalyan Sangh v. M/s. Swadeshi Polytex Ltd.
and Ors.) and SLP (Civil) No. 21002/2006 (State of U. P. and Ors. v. M/s. Swadeshi
Polytex Ltd. and Ors.) All these matters are being disposed of by this judgment with the
basic facts being taken from the first mentioned appeal.
3. The learned single Judge, at the very first instance, dealt with the preliminary
objections raised during the course of the hearing that in view of the Division Bench
judgments of the High Court dated 13th January 2005, 4th May 2005 and 26th May 2005,
it was not open to the SPL to contend at this stage that the recovery proceedings
including the procedure adopted was not maintainable in law. The Court observed that W.
P. No. 50571/2002 had been filed by M/s. Paharpur Cooling Towers Pvt. Ltd. and Ors. in
which SPL had been arrayed as respondent No. 6 and the proceedings relating to the
issuance of the recovery certificates by the Deputy Labour Commissioner under sub-
section (1) of Section 3 of the 1978 Act had been questioned, but the Division Bench had
dismissed the Writ Petition observing that as the petitioner therein i.e. M/s. Paharpur
Cooling Towers Pvt. Ltd. was pursuing the matter with the Company Law Board and had
availed of an alternative remedy, the writ petition was not maintainable. The Court also
observed that the matter had been taken by M/s. Paharpur Cooling Towers Ltd. to the
Supreme Court and an interim order dated 7th February 2005 had been made directing
the petitioner to deposit a sum of Rs. 5/- Crore in favour of the Registrar General of this
Court, but this amount had not been deposited and the Special Leave Petition had been
dismissed on 24th February 2005. The Court accordingly held that in this view of the
matter, it was clear that no order against SPL had been made by this Court in the above-
mentioned SLP. The learned Judge then went into the scope and effect of W. P. No.
35005/2005 filed by SPL impugning the auction notice dated 22nd April 2005 and
observed that this petition had been dismissed with the observation that it would be open
to SPL to avail of the alternative remedy available under rule 285(i) of the Rules. The
Bench also noted that the third writ petition, filed by one Jitendra Khaitan (Writ Petition
No. 36736/2005) once again challenging the validity of the auction notice dated 22nd
April 2005 had been filed and this writ petition too had been dismissed with the
observation, inter alia, that in the light of the order in Writ Petition No. 35005/2005, the
petitioner herein could also avail the alternative remedy by filing objections under rule
285(i) of the Rules. The Court accordingly rejected the prayer of the respondents before it
that in view of the aforesaid writ petitions, the writ petition was not maintainable. The
Court then examined the submission as to whether the procedure envisaged for recovery
of arrears under the Act and the Rules had been observed and in case they had been
breached, the effect thereof and after examining the various provisions threadbare, held
that the Act and Rules prescribed a procedure for the recovery of arrears of land revenue
and that before a recovery certificate could be issued, the defaulter was required to be
effectively served, that the Rules in question were mandatory and required strict
compliance and in conclusion highlighted that there was no material on record to show
that any attempt had been made to serve the demand notice on SPL, and service on the
Chowkidar was clearly not proper service on the defaulter. The Court also observed that
auction sale was liable to be set aside for the additional reason that a clear 30 days' notice
of the proposed auction had not been given even if the service on the Chowkidar was held
to be appropriate. The Court also held that the sale proclamation issued on 2nd May 2005
@page-SC2857
was not valid and did not comply with the provisions of rules 285, 286 and 283 of the
Rules and that the proclamation that had been issued was only of Rs. 1.10 Crores and did
not provide for the full amount as envisaged under rule 245, which provided an
opportunity to the defaulter to make good the payment so as to avoid the sale of the
property. The Court also held that on facts, it was impossible for the auction-purchaser
i.e., the UPSIDC to have procured the Bank Drafts from the Punjab National Bank,
Kanpur on the day of the auction so as to make the deposit of the 25% of the sale price at
the fall of the hammer as the auction had been conducted at Meerut, about 460 Kms.
away from Kanpur and that the balance 75% of the amount due on the auction had also
been deposited late i.e. on 18th May 2005 which again was contrary to rule 285-D of the
rules. The Court also observed that it appeared that the property had been sold at a price
far below its market price and in conclusion, passed strictures against the district
authorities which had conducted the auction and sale of the property in question thus
quashing the order dated 9th September 2005 passed by the Board of Revenue, the order
dated 24th June 2005 of the Commissioner as well as the auction sale proceedings dated
2nd May 2005 conducted by the Tahsildar, Ghaziabad with costs of Rs. 50,000/- and all
consequential relief, and a direction that it would be open for the State Government to
recover the costs from the salary of the officers who were responsible "for the auction of
the property in question in such unruly manner" by holding an enquiry and that the Chief
Secretary was advised to take appropriate action against the defaulting officers.
4. Before we embark on an examination of the contentions raised by the learned counsel
for the parties, we deem it appropriate to refer to certain supervening and material factors.
It is the admitted position that the workmen at whose instance the initial process of sale
of the property had been initiated, have entered into an agreement dated 3rd January 2008
with SPL and the entire due amount due to them and something more has since been paid.
It is also clear that the auction-purchaser is the UPSIDC, which is a Government agency
and is the owner of the land over which the superstructure of SPL has been built.
5. In this background, the learned counsel for the appellants has submitted that the
findings recorded by the High Court were erroneous as it was clear from the record that
despite numerous opportunities given to SPL to make the payments due to their own
workmen, no serious attempt had been made to do so and that on the contrary, every
attempt had been to forestall the payment. It has been pointed out that he had suddenly
woken up to its obligations and made full payment after the decision in the writ petition
to take advantage of the huge spurt in the price of real estate in Ghaziabad and the
surrounding areas. It has been pleaded that there was absolutely no irregularity in the
procedure relating to the auction and the finding of the High Court that the sale price
appeared to be undervalued was also not based on any relevant material. It has also been
pleaded that no substantial injury had been caused to SPL, as the recovery certificate
initially had been issued in the year 2002 and had been challenged by the associates of
SPL or by SPL itself and despite the fact that in the case of the SLP filed by M/s.
Paharpur Cooling Towers Ltd., this Court had directed that a sum of Rs. 5 Crore be
deposited before the Registrar General, the order had not been complied with and the SLP
had been dismissed proving a lack of intention on the part of the SPL or its associates to
make the payment. It has finally been pleaded that the UPSIDC had deposited a sum of
Rs. 32.20 Crores in May 2005 and the sale had thereafter been confirmed in its favour
and the possession transferred, and that if this amount plus interest of 18% was taken into
account, the amount now due to the appellant would be almost 48 Crores, in case the
order was to be set aside.
6. The learned counsel for the respondents have, however, supported the judgment of the
High Court. It has been especially emphasized that the workers due having been
discharged by SPL, it did not lie on the Sta.te Government or a State Government
Undertaking, the appellant herein, to still pursue the matter doggedly in this Court. It has
been reiterated that the property in question had not been properly valued, as provided by
rule 283 of the Rules and that the notice of the proclamation has also not been served on
the SPL or on any of its functionaries and had in fact been served to the Chowkidar and
that too, about a week before the auction whereas a minimum notice
@page-SC2858
period of 30 days ought to have been given. It has further been pleaded that the auction-
purchaser i.e., the UPSIDC had not deposited 25% of the sale price and/or the balance of
75% of the amount within 15 days, as required under rule 245 of the Rules and this too
was a ground which was relevant in determining the propriety of the sale of the auction.
7. As would be clear, the arguments pressed by the learned counsel for both the sides
pertain to the procedure adopted for the auction. Sub-section (1) of Section 3 of the 1978
Act provides that in case the occupier of an industrial establishment is in default of
payment of wages in excess of Rs. 50,000/-, the Labour Commissioner may forward to
the Collector a certificate under his signatures specifying the wages due from the
establishment concerned and that the Collector shall accordingly proceed to realize the
amounts due as arrears of land revenue. Admittedly the recovery certificate had been
issued and sent to the Collector, Ghaziabad by the Labour Commissioner under Section
3(1) and it is in this situation that the proceedings against the SPL had been set in motion.
Section 279 of the Act provides for the recovery of arrears of land revenue by various
methods including an attachment and sale of the immovable property of the defaulter in
respect of the arrears due. Section 280 stipulates that as soon as the land revenue had
become due, a writ of demand may be issued by the Tahsildar calling upon the defaulter
to pay the amount within a specified time and under Section 284, the property of the
defaulter may also be attached. Section 327 provides for the modes of service of the
notice on the defaulter and reads as under :
"327. Mode of service of notice.- Any notice or other document required or authorised to
be served under this Act may be served either-
(a) by delivering it to the person on whom it is to be served, or
(b) by leaving it at the usual or last known place of abode of that person, or
(c) by sending it in a registered letter addressed to that person at his usual or last known
place of abode, or
(d) in case of an incorporated company or body by delivering it or sending it in a
registered letter addressed to the Secretary or other principal functionary of the company
or body at its principal office, or
(e) in such other manner as may be laid down in the Code of Civil Procedure, 1908.
8. Section F of Chapter 10 of the Rules deals with the coercive procedure which can be
adopted by the Collector to recover the amounts as arrears of land revenue.
9. Rules 235 and 236 authorize the Tahsildar to issue citations, writs and warrants etc. as
per the prescribed form whereas Rr. 241 and 245 provide as to how the citation is to be
issued and writ of demand for the purpose of land revenue. Rule 246 provides for the
service of the writ or citation shall, if possible, be made on the defaulter personally, but if
service cannot be made on the defaulter, it can be made on the agent and sub-rule thereof
further postulates that personal service shall be made by delivery to the defaulter or the
agent of the foil of the writ or citation and with the sanction of the Collector such writs of
demand may also be served as registered post. Rule 247-A also refers to a warrant of
arrest which may be executed by a duly authorized person for the recovery of the arrears
and Rule 247-B (1) deals with the situation that where a defaulter at the time of his arrest
pays the entire amount of arrears specified in the warrant of arrest to the process-server or
to authorized officer, the defaulter will not be arrested. Rules 272, 272-A, 272-B, 273,
273-A, 278 and 285-C deal with the procedure for the attachment of the land which is
proposed to be sold and Rule 273-A postulates that the procedure envisaged in Order
XXI, Rule 54 of the Code of Civil Procedure must be followed at the time of attachment.
Rule 285-C also provides that in case the defaulter pays the arrears of land revenue in
respect of the land proposed to be sold on any day before the fixed day of the sale, the
sale officer on being satisfied shall stay the sale. Rules 282 and 283 when read togeher
provide that in the proclamation of sale to be issued in Form Z. A. 74, it will be
incumbent on the Collector to give the estimated value of the property calculated in
accordance with the rules in Chapter XV of the Revenue Manual.
10. We now examine the primary arguments in the background of the above provisions.
The question arises as to whether the provisions for the attachment and sale of the
property had been followed scrupulously, as would be necessary in such a case. We notice
that the learned Single Judge has
@page-SC2859
examined the matter and has concluded that there was no material on record to show that
proper procedures had been adopted. A positive finding has been arrived at on facts that
the Tehsildar or the Collector had even attempted to serve the demand notice personally
or by registered post on SPL, as called upon under Section 327 of the Act and Rule 246,
as the notice had been served on the Chowkidar who could not be said to be an agent of
SPL. It must also be noticed from a bare reading of the Rule 246 that the notice can be
served on the agent only if it is not possible to serve it on the actual defaulter. In the
present case, we find that no attempt whatever had been made to serve the notice to the
actual defaulter and had been served on the Chowkidar at the very initial stage.
11. There is yet another circumstance which indicates that the procedure for sale had not
been followed. It appears from the record that the notice of citations for appearance and
demand had been issued on 11th January 2005 and on 1st April 2005, the Sub-Divisional
Magistrate had passed an order for the valuation of the properties as well as for wide
publicity of the auction and sale of the property in question and the Tehsildar, Ghaziabad
had been appointed as the auction officer and the auction had been fixed for the 2nd May
2005. It is clear from the record that the sale proclamation had been issued on 1st April
2005 without any valuation of the properties and only the area of the vacant land had
been specified therein and it was this notice that had been served on the Chowkidar on
the 21st April 2005 and publication had been made in the newspaper "Amar Ujala" on the
22nd April 2005. There has, thus, been a clear violation of the Rules 283 and 285 ibid.
Rule 283 provides for the estimated value of the property to be determined under the
provisions contained in Chapter XV of the Revenue Manual. The said Chapter specifies
the procedure for valuation of the property in terms of other similar properties. It is,
however, clear from the record that the figure 27 Crores, the value of the property which
is mentioned in the advertisement in the "Amar Ujala", appears to have picked up without
any basis as it is not the case of the UPSIDC that the property had been valued in
accordance with the provisions of the Revenue Manual or by a valuer or expert in the
field.
12. Moreover, Rule 273-A makes the provision under Order XXI, Rule 54 of the CPC
applicable to proceedings for attachment and sub-rule (1-A) of Rule 54 specifically
provides for the judgment debtor to attend Court on a specified date to take notice of the
date which is fixed for setting the proclamation of the sale. Concededly, this procedure
had not been followed. The learned counsel for the respondent has also disputed the
valuation of the property by the UPSIDC and has referred us to the pleadings in the writ
petition and in particular to paragraphs 26 and 30 thereof. These paragraphs are
reproduced blow :
"That the auction was held on 2-5-2005 but in the furd neelam no time of auction was
provided. Even the description of the land sought to be auctioned, has not been provided
in the furd neelam. Six bidders, had participated in the auction, out of which only U. P.
State Industrial Development Corporation was the major bidder in competition with one
M/s. Suder Steel Pvt. Ltd. The property worth Rs. 56.00 Crores as per the rate list issued
by the U. P. State Industrial Development Corporation has been auctioned for petty
amount of Rs. 32.00 Crores and odd. In the open market, rate of the property in question
could fetch at least Rs. 100 Crores. This is apparent from the letter dated 26-4-2005
issued by Shri Mahak Singh, District Manager, UPSIDC. The circle rate of the land of the
area fixed by UPSIDC is effective from 9-4-2005 is Rs. 3000/- per sq. meter, by which
calculation, cost of the land in question comes to Rs. 56.42 Crores approx, whereas the
same has been sold only @ Rs. 1700/- per sq. meter, amounting to Rs. 32.20 Crores, to
the UPSIDC. A true copy of the furd neelam letter dated 26-4-2005 issued by District
Manager, UPSIDC are annexed herewith as Annexure Nos. 23A and 24 to this Writ
Petition.
That as per the newspaper clipping published in The Times of India property section,
New Delhi edition dated 25-6-2005, the land price in Kavi Nagar, Ghaziabad is Rs.
11,000/- per sq. mt. The petitioner has suffered a substantial injury and loss in the auction
held by the District Administration of a prime property situated at Kavi Nagar, Ghaziabad
Industrial Area, Ghaziabad @ Rs. 1700/- per sq. mt. a true copy of the newspaper
clipping dated 25-6-2005 is annexed herewith as Annexure No. 27 to this Writ Petition."
The replies to the paragraphs are given in SLP (C) No. 3272 of 2006.
@page-SC2860
"That in reply to the contents of paras 23 to 25 of the writ petition which are not correct,
hence denied. It is submitted that the petitioner had full knowledge about the auction and
the auction has not held much higher to the present circle rate. It is further submitted that
the rate offered and accepted by the deponent is of developed land whereas, the present
land is a lease land on 99 years of which already 35 years had expired and it is
undeveloped area for which maximum rate has been got in the auction held on 2-5-2005."
13

. Shri Nariman, the learned senior counsel for the respondents, therefore, appears to be
right in contending that the specific averments made by SPL in the writ petition have not
been denied by the respondent and it was therefore open to SPL to contend that the
property had not been properly valued and that the sum of Rs. 27 crores represents not
even half the market price. In Gajraj Jain v. State of Bihar (2004) 7 SCC 151 while
dealing with a case under the State Financial Corporations Act, this is what this Court had
to say : 2004 AIR SCW 3263, Paras 11 and 14

"In the light of the aforesaid judgment of this Court, the issue which arises for
determination is - whether Respondent 2 Corporation acted reasonably and in accordance
with Section 29 of the 1951 Act in transferring the assets of the Company on 19-3-2002
and in entering into agreement for sale with Respondent 4 on 26-4-2002. As stated above,
Respondent 2 Corporation had a paramount first charge on the assets of the flour mill
whereas Central Bank of India had the second charge thereon. There is a difference
between a charge and a mortgage. In the case of a charge under Section 100 of the TP
Act, there is no transfer of interest in the property. A charge is not a jus in rem. It is jus ad
rem. It creates a right of payment out of the property/fund charged with the debt or out of
proceeds of the realization of such property, a phrase used in Section 29(1) of the 1951
Act. A charge as defined under Section 100 of the TP Act may be enforced by sale (See
Mulla : Civil Procedure Code (15th Edn.), P. 2420). We have discussed the concept of
charge as it has a direct bearing on the interpretation of Section 29 of the 1951 Act.

In the present case, it has been urged that absence of valuation report and the reserved bid
does not vitiate the sale. We do not find merit in this argument. In the case of S. J. S.
Business Enterprises (P) Ltd. it has been held that the financial corporation, in the matter
of sale under Section 29, must act in accordance with the statute and must not act
unreasonably. In this case, the Corporation fails on both the counts. It has neither
complied with the provisions of subsections (1) and (4) of Section 29, nor has it acted
fairly. The test of reasonableness has been laid down in the above judgment in which it is
held that reasonableness to be tested against the dominant consideration to secure the best
price. Value or price is fixed by the market. In the case of a going concern, one has to
value the assets shown in the balance sheet (Datta, S. : Valuation of Real Property, p.
198). In our view, if the object of Section 29 of the Act is to obtain the best possible price
then the Corporation ought to have called for the valuation report. This has not been
done. There is no inventory of assets produced before us. The mortgaged assets of the
Company could be sold on itemized basis or as a whole, whichever is found on valuation
to be more profitable. No particulars in that regard have been produced before us. If
publicity and maximum participation is to be attained then the bidders should know the
details of the assets (or itemized value). In the absence of the proper mechanism the
auction sale becomes only a pretence. Further, in this case, the Corporation advanced Rs.
90 Lakh to the Company. At that time, it must have valued the assets. Not such report has
been produced. Lastly, in this case, the price of the assets is pegged to the dues of the
Corporation and Central Bank of India. The assets are agreed to be sold to Respondent 4
not for the market price but against repayment of dues of the Corporation plus a promise
to discharge the liability of Central Bank of India. Therefore, the Corporation,
Respondent 2, has not acted reasonably. It has not taken any steps to secure the best price.
In fact, it has failed to protect the interest of Central Bank of India, which is having the
second charge on the assets transferred to Respondent 4 as well as the mortgagor which
would be entitled to the balance of the sale proceeds, if any. It was contended that as the
bids were withdrawn, the offer of Respondent 4 was accepted. Even assuming for the
sake of argument, that 2004 AIR SCW 2987

@page-SC2861
there were no offers except the offer of respondent 4, it shows that value of the assets was
Rs. 198.85 lakhs (i.e. Rs. 28.85 lakhs + Rs. 170 Lakhs). No reason has been given why
Respondent 2 did not insist on downright payment of Rs. 198.85 lakhs."
14

. The question of valuation is to our mind of the utmost importance as it is designed to


ensure the best price for the property and it is essential in this circumstance that wide
publication and notice of the proposed sale should be given as per Rule 285-A which
postulates a notice of 30 days between the date of issuance of the sale proclamation and
the date of auction. It can hardly be over emphasized that the proper valuation of the
property and wide publicity of the proposed auction is intimately linked with the price
that the auction fetches. As already mentioned above, the auction had been held on 2nd
May 2005. The sale proclamation had been issued on the 1st April 2005, and served on
the Chowkidar on the 21st April 2005, the publication made in "Amar Ujala" on 22nd
April 2005 whereas rule 285 itself postulates a notice period of 30 days to be counted
from the date of issuance of the sale proclamation. While dealing with a similar situation,
this is what this Court had to say in S. J. S. Business Enterprises (P) Ltd. v. State of Bihar
(2004) 7 SCC 166 : 2004 AIR SCW 2987, Paras 17 and 18

"We are of the view that the sale effected in favour of Respondent 6 cannot be sustained.
It is axiomatic that the statutory powers vested in State Financial Corporation under the
State Financial Corporations Act, must be exercised bona fide. The presumption that
public officials will discharge their duties honestly and in accordance with the law may
be rebutted by establishing circumstances which reasonably probabilise the abuse of that
power. In such event it is for the officer concerned to explain the circumstances which are
set up against him. If there is no credible explanation forthcoming the Court can assume
that the impugned action was improper. (See Pannalal Binjraj v. Union of India, AIR at p.
409). Doubtless some of the restrictions placed on State Financial Corporations
exercising their powers under Section 29 of the State Financial Corporations Act, as
prescribed in Mahesh Chandra v. Regional Manager, U. P. Financial Corprn. Are no
longer in place in view of the subsequent decision in Haryana Financial Corpn. v.
Jagdamba Oil Mills. However, in overruling the decision in Mahesh Chandra this Court
has affirmed the view taken in Chairman and Managing Director, SIPCOT v. Contromic
(P) Ltd. and said that in the matter of sale under Section 29, State Financial Corporations
must act in accordance with the statute and must not act unfairly i.e. unreasonably. If they
do, their action can be called into question under Article 226. Reasonableness is to be
tested against the dominant consideration to secure the best price for the property to be
sold. AIR 1957 SC 397
1992 AIR SCW 3629
2000 AIR SCW 500
1995 AIR SCW 2566

"This can be achieved only when there is a maximum public participation in the process
of sale and everybody has an opportunity of making an offer. Public auction after
adequate publicity ensures participation of every person who is interested in purchasing
the property and generally secures the best price."
Adequate publicity to ensure maximum participation of bidders in turn requires that a fair
and practical period of time must be given to purchasers to effectively participate in the
sale. Unless the subject-matter of sale is of such a nature which requires immediate
disposal, an opportunity must be given to the possible purchaser who is required to
purchase the property on "as-is-where-is basis" to inspect it and to give a considered offer
with the necessary financial support to deposit the earnest money and pay the offered
amount, if required."
15. We must, therefore, repel Mr. Dwived's argument that as SPL had suffered no
prejudice in the auction proceedings, the sale should not be interfered with.
16

. There is yet another circumstance which vitiates sale. Rule 285-D of the Rules that 25%
of the amount of the auction money shall be deposited at the fall of the hammer and the
remaining 75% within 15 days. The case of the appellants is that the Bank draft for 7.80
Crores had been deposited by the auction purchasers on 2nd May 2005 i.e., the date of
auction but the learned single Judge has found that as the auction had been completed at
1.30 p.m., it would not have been possible to have received the Bank draft from Kanpur
460 Km. away on that date. This finding appears to be correct. We AIR 1954 SC 349,
Para 11

@page-SC2862
also find that the balance 75% of the amount that had been deposited by various Bank
drafts on 18th May 2005 was also beyond the 15 days permissible and the finding of the
learned single Judge based on the record is that though the drafts were dated 14th May
2005 but they had, in fact, had been handed over to the concerned authority only on the
18th May 2005. This Court in M. M. Shah v. S. S. A. S. Mahamad and Anr. 1954 SCR
108 has held as under :
"Having examined the language of the relevant rules and the judicial decisions bearing
upon the subject we are of opinion that the provisions of the rules requiring the deposit of
25% of the purchase-money immediately on the person being declared as a purchaser and
the payment of the balance within 15 days of the sale are mandatory and upon non-
compliance with these provisions there is no sale at all. The rules do not contemplate that
there can be any sale in favour of a purchaser without depositing 25% of the purchase-
money in the first instance and the balance within 15 days. When there is no sale within
the contemplation of these rules, there can be no question, of material irregularity in the
conduct of the sale. Non-payment of the price on the part of the defaulting purchaser
renders the sale proceedings as a complete nullity. The very fact that the Court is bound
to re-sell the property in the event of a default shows that the previous proceedings for
sale are completely wiped out as if they do not exist in the eye of law. We hold, therefore,
that in the circumstances of the present case there was no sale and purchasers acquired no
rights at all."
17. For this additional reason as well, the auction sale cannot be maintained.
18. In this view of the matter, we need not go into the argument raised by Mr. R. G.
Nariman that in the facts of the case we should not entertain this matter in the exercise of
the discretionary jurisdiction under Article 136 of the Constitution of India.
19. We also notice from the 1st paragraph of the judgment of the learned single Judge
appears to have been extremely annoyed with what he perceived to be gross irregularities
on the part of the officers of the State Government connected with the sale of the property
and he had accordingly directed as under :
"Subject to above writ petition is allowed with cost quantifies to Rs. 50,000/- petitioner
shall be entitled to withdraw Rs. 25,000/- and rest of Rs. 25,000/- shall be remitted to U.
P. State Legal Services Authorities to utilize for providing legal aid to the litigants
approaching Lucknow Bench of High Court. The cost shall be deposited within one
month from today in this Court by the District Magistrate, Ghaziabad. Registrar to ensure
compliance. It shall be open for the State Government to recover the cost from the salary
of the office/officers who are responsible to auction the property in question in such
unruly manner by holding an inquiry. The Chief Secretary Government of U. P. is further
directed to take appropriate action against the officers or employees who had acted in
arbitrary manner while proceeding with the auction and sale of the property in question.
Let a copy of the judgment be sent to the Chief Secretary, Govt. of U. P. by the office
within a week for appropriate action".
20. We are of the opinion, however, that High Court's direction that action should be
initiated against the concerned offices, is not justified and we accordingly expunge these
directions more particularly as SPL's conduct as well does it no credit. We, however,
maintain the directions insofar as the costs are concerned. With this minor modification,
the appeals are dismissed.
Appeals dismissed. .
AIR 2008 SUPREME COURT 2862 "State of Meghalaya v. Mecken Singh N. Marak"
(From : 2006 Gauhati L.T. (Supp) 316)
Coram : 2 ALTAMAS KABIR AND J. M. PANCHAL, JJ.
Civil Appeal No. 3471 of 2008 (arising out of SLP (C) No. 13673 of 2006), D/- 9 -5
-2008.
State of Meghalaya and Ors. v. Mecken Singh N. Marak.
(A) Constitution of India, Art.226 - WRITS - HIGH COURT - DISCIPLINARY
ENQUIRY - REMOVAL FROM SERVICE - Powers of High Court - Disciplinary
enquiry - Quantum of punishment - Police Officer found to be guilty of misconduct -
Removal from service - Interference by High Court by unreasoned order with further
direction to Appellate Authority to impose any punishment short of removal - Not
permissible.
2006 Gau LT (Supp) 316, Reversed.
@page-SC2863

The jurisdiction of High Court, to interfere with the quantum of punishment is limited
and cannot be exercised without sufficient reasons. The High Court, although has
jurisdiction in appropriate case, to consider the question in regard to the quantum of
punishment, but it has a limited role to play. The High Courts, in exercise of powers
under Article 226, do not interfere with the quantum of punishment unless there exist
sufficient reasons therefor. The punishment imposed by the disciplinary authority or the
Appellate Authority unless shocking to the conscience of the Court, cannot be subjected
to judicial review. In the impugned order of the High Court no reasons whatsoever have
been indicated as to why the punishment was considered disproportionate. Failure to give
reasons amounts to denial of justice. The mere statement that it is disproportionate would
not suffice. (Para 9)
In the instant case the delinquent, a Police Officer being a member of disciplined force
was supposed to carry out instructions given to him by his superior. Not only he flouted
the instructions but conducted himself in such a manner that he caused loss of part of pay
to be deposited with the exchequer and loss of service revolver with ammunition which
could be misused. When a statute gives discretion to the administrator to take a decision,
the scope of judicial review would remain limited. The proved charges clearly established
that the respondent, who was a police Officer failed to discharge his duties with utmost
integrity, honesty, devotion and diligence and his acts were prejudicial to the exchequer
and society. Even in cases where the punishment imposed by the disciplinary authority is
found to be shocking to the conscience of the Court, normally the disciplinary or the
Appellate Authority should be directed to reconsider the question of imposition of
penalty. The High Court in this case, has not only interfered with the punishment imposed
by the disciplinary authority in a routine manner but overstepped its jurisdiction by
directing the Appellate Authority to impose any other punishment short of removal. By
fettering the discretion of the Appellate Authority to impose appropriate punishment for
serious misconducts committed by the respondent, the High Court totally misdirected
itself while exercising jurisdiction under Article 226. Thus, the conclusion of the High
Court cannot be regarded as proper at all.
2006 Gau LT (Supp) 316, Reversed. (Paras 9, 10)
(B) Constitution of India, Art.133 - APPEAL - DISCIPLINARY ENQUIRY -
APPELLATE AUTHORITY - MISCONDUCT - Appeal - Finding of fact - Disciplinary
enquiry - Finding by Competent authority and appellate authority that grave misconduct
committed by delinquent was satisfactorily proved - Upheld by High Court on
reappreciation of evidence - Cannot be interfered with in appeal before Supreme Court.
(Para 8)

Ranjan Mukherjee and R.C. Ghosh, for Appellants; Parthis Goswami, Rajiv Mehta and
Biswanath Aggarwal, for Respondent.
Judgement
J. M. PANCHAL, J. :- Leave granted.
2. The instant appeal is directed against the Judgment dated March 7, 2006, rendered by
the Division Bench of the Gauhati High Court in Writ Appeal No. 282 of 2002 whereby
the decision of the learned Single Judge dated October 5, 2002 passed in Civil Rule No.
4048 of 1996, upholding the order of the Competent Authority, removing the respondent
from service, is set aside and the matter is remanded to the appellate authority, namely,
the Inspector General of Police to consider and inflict appropriate punishment, short of
removal from service, commensurate with the gravity of the proven misconduct.
3. The relevant facts emerging from the record of the case are as under. In the year 1967
the respondent was appointed as police constable by the Home Department, Government
of Meghalaya. During the course of time he was promoted to the post of Sub-Inspector of
Police and posted as Armed Branch Sub-Inspector, 2nd Meghalaya Police Battalion at
Goeragre. On May 5, 1995 he was directed to go to Shillong along with BNC Clyforth
Sangma to disburse the pay for the month of April 1995 to the Bn personnel posted at
Shillong. One 0.38 bore revolver bearing Number 787735 and 12 rounds of 0.38
ammunition were also issued to him for the purpose. He was specifically instructed to
proceed to Shillong in a vehicle belonging to the department with other police personnel
who were going to Shillong on platoon transfer with their arms and ammunition. The
respondent was further instructed to come back to Goeragre from Shillong in the same
vehicle after the
@page-SC2864
disbursement of pay etc. with other personnel who were to come back to Goeragre with
their arms and ammunition. The respondent, in the company of Clyforth Sangma left Bn
Headquarters in the morning of May 5, 1995 by Units vehicle No. ML-02 1038 at about
8.30 AM and arrived at Shillong at about 8.00 PM. On arrival at Shillong, the respondent
began to disburse the pay. He also spent the whole next day in disbursing the rest of the
pay. An amount of Rs. 17,314/-could not be disbursed by him and he was supposed to
deposit the same with the competent authority at the Headquarters. The respondent
permitted Clyforth Sangma to visit his wife who was not feeling well and, therefore, in
the morning of May 7, 1995 Clyforth Sangma left for his village Rajasimla and returned
back to Bn Headquarters on May 9, 1995.
4. The respondent disobeyed the instructions given to him to come back to Bn
Headquarters in the vehicle in which he had gone to Shillong. Instead he met and
instructed BNC 737 Emmanual Jalong to prepare to leave for Bn Headquarters by night
bus. On May 7, 1995 in the evening at about 4.30 p.m. he left the camp for the M. T. C.
Bus Station. At the bus station he did not see the constables who were supposed to travel
with him. Presuming that they must have proceeded in another bus, the respondent
boarded the MTC bus No. ML-03-0099 bound for Tura. He was seated in Seat No. 22.
There was a passenger in the seat next to him on Seat No. 21. At Jorabat the bus stopped
where the respondent took dinner. After the bus resumed its onward journey for Tura, his
co-passenger in Seat No. 21 started chatting with him. The co-passenger took out a
packet of biscuit and offered biscuits to the respondent. The respondent accepted the
biscuits and after eating the biscuit he dosed off. When he woke up, the bus had reached
somewhere near Anogri. He made enquiry about his belongings and found that his
revolver with ammunition and pouch containing an amount of Rs. 17,314/-, which was
undisbursed pay were missing. He also realized that his co-passenger in seat No. 21 was
not around.

5. When he reached Goreagre he looked for his belongings again but could not find them.
He then left for his residence. After sleeping for some time he woke up and went to Tura
with the intention of searching the bus once again. He met the Depot Manager who
informed that the bus had left for Nanggalbibra and would return only in the evening.
Therefore, he came back to his residence and went to the Commandant's office at 10.00 P.
M. to inform him about the loss of his revolver and money. When he reached the office of
the Commandant, too many people were present in the office and, therefore, he could
inform the Commandant about the incident in question at about 12.00 P.M.
6. On receipt of the information from the appellant, Respondent No.3 who is the
Commandant of the Battalion forwarded his report on May 8, 1995 to Tura Police Station
for registering a case and accordingly a case was registered at Tura Police Station under
the provisions of the Indian Penal Code. On May 12, 1995, the respondent was suspended
from service pending enquiry for loss of Rs. 17,314/- and service revolver with
ammunition. A preliminary enquiry was held with regard to the matter and thereafter the
competent authority decided to take steps to initiate a regular departmental enquiry
against the respondent. Accordingly, the respondent was served with statement of charges
and called upon to offer his explanation. He offered his explanation which was not found
to be satisfactory by the competent authority. The competent authority thereafter
appointed Inquiry Officer to conduct departmental inquiry against the appellant.
7. During the course of departmental enquiry several witnesses were examined in respect
of the charges levelled against the respondent. The respondent had also examined his
witnesses. At the conclusion of the departmental enquiry the enquiry officer submitted
report to the competent authority stating that the charges framed against the respondent
were duly proved. On careful consideration of the report as well as records, the
competent authority tentatively agreed with the findings of the enquiry officer. The
competent authority forwarded a copy of the report of the enquiry officer to the
respondent along with letter dated September 18, 1995, and called upon him to show
cause as to why he should not be discharged from service. On receipt of the show cause
notice, the respondent submitted his explanation. The competent authority by an order
dated January 1, 1996, removed the respondent from service. Feeling aggrieved, the
respondent preferred an appeal before
@page-SC2865
the Deputy Inspector General of Police, Western Range, Tura as provided by Rule 66 of
the Assam Police Manual - Part III. The Inspector General of Police (Training) dismissed
the appeal by an order dated May 13, 1996. Thereupon the respondent invoked
extraordinary jurisdiction of the High Court under Article 226 of the Constitution by
filing Civil Rule No. 4048 of 1996. The learned single Judge of the Gauhati High Court
dismissed the petition by order dated October 5, 2002. Aggrieved by the judgment
delivered by the learned single Judge, the respondent preferred an appeal before the
Division Bench of the Gauhati High Court. The Division Bench upheld the finding
recorded by the learned single Judge that misconduct by the respondent was satisfactorily
proved. However, the Division Bench was of the view that the version of the respondent
that he had to travel by Meghalaya State Road Transport Corporation night bus and lost
his consciousness after taking sweets offered by co-passenger was a mitigating
circumstance and, therefore, the punishment of removal from service imposed on him
was not commensurate with the gravity of the proven misconduct. In view of the said
conclusion the Division Bench has set aside the order removing the respondent from
service and remitted the matter to the appellate authority, namely, the Inspector General
of Police to consider and inflict appropriate punishment, short of removal from service,
commensurate with the gravity of the proven misconduct of the respondent, by the
impugned judgment, giving rise to the instant appeal.
8. This Court has heard the learned counsel for the parties at length and in great detail.
This Court has also considered the documents forming part of the instant appeal. The
competent authority as well as the first appellate authority have concluded that grave
misconduct committed by the respondent is satisfactorily proved. The said finding is
upheld by the learned single Judge of the Gauhati High Court while deciding the petition
filed by the respondent under Article 226 of the Constitution. On re-appreciation of
evidence adduced, during the course of the departmental inquiry initiated against the
respondent, the Division Bench has also recorded a finding of fact that the respondent
had committed serious misconduct. The said finding is a finding of fact which is not
liable to be interfered with in the instant appeal.
9. The next question which falls for consideration is whether the competent authority was
justified in removing the respondent from service and whether the Division Bench of the
High Court was right in remitting the matter to the Appellate Authority for passing
appropriate order of punishment short of removal. The record would indicate that the
respondent was a senior police officer. He was instructed by his Commandant to go to
Shillong to disburse the pay in a vehicle belonging to the department and along with him
another police officer was also deputed for safe carriage of pay to be disbursed to the Bn
personel posted at Shillong. Further, the respondent was issued 0.38 bore revolver with
12 rounds. It is an admitted position that the respondent was instructed to come back to
Bn headquarters by the vehicle of the department along with other police personnel but
the respondent disobeyed the instructions and travelled to Bn headquarters in a bus
wherein not only he lost cash of Rs. 17,314/- but also his service revolver with 12 rounds
of ammunition. Under the circumstances the question arises whether the Division Bench
of the High Court was justified in setting aside the order of removal of the respondent
from service and remitting the matter to the appellate authority, namely, the Inspector
General of Police to consider the question of imposition of appropriate punishment, short
of removal from service, commensurate with the gravity of the proven misconduct of the
respondent. A Court or a tribunal while dealing with the quantum of punishment has to
record reasons as to why it is felt that the punishment is not commensurate with the
proved charges. In the matter of imposition of sentence, the scope for interference is very
limited and restricted to exceptional cases. The jurisdiction of High Court, to interfere
with the quantum of punishment is limited and cannot be exercised without sufficient
reasons, The High Court, although has jurisdiction in appropriate case, to consider the
question in regard to the quantum of punishment, but it has a limited role to play. It is
now well settled that the High Courts, in exercise of powers under Article 226, do not
interfere with the quantum of punishment unless there exist sufficient reasons therefor.
The punishment imposed by the disciplinary authority or the Appellate Authority unless
shocking to the conscience of
@page-SC2866
the Court, cannot be subjected to judicial review. In the impugned order of the High
Court no reasons whatsoever have been indicated as to why the punishment was
considered disproportionate. Failure to give reasons amounts to denial of justice. The
mere statement that it is disproportionate would not suffice. While considering the
question of proportionality of sentence imposed on a delinquent at the conclusion of
departmental inquiry, the Court should also take into consideration, the mental set up of
the delinquent, the type of duty to be performed by him and similar relevant
circumstances which go into the decision making process. If the charged employee holds
the position of trust where honesty and integrity are inbuilt requirements of functioning, it
would not be proper to deal with the matter leniently. Misconduct, in such cases has to be
dealt with iron hands. The respondent belonged to a disciplined force. He was supposed
to carry out instructions given to him by his superior. Not only he flouted the instructions
but conducted himself in such a manner that he caused loss of part of pay to be deposited
with the exchequer and loss of service revolver with ammunition which could be
misused. When a statute gives discretion to the administrator to take a decision, the scope
of judicial review would remain limited. The proved charges clearly established that the
respondent, who was a police officer failed to discharge his duties with utmost integrity,
honesty, devotion and diligence and his acts were prejudicial to the exchequer and
society. Even in cases where the punishment imposed by the disciplinary authority is
found to be shocking to the conscience of the Court, normally the disciplinary authority
or the Appellate Authority should be directed to reconsider the question of imposition of
penalty. The High Court in this case, has not only interfered with the punishment imposed
by the disciplinary authority in a routine manner but overstepped its jurisdiction by
directing the Appellate Authority to impose any other punishment short of removal. By
fettering the discretion of the Appellate Authority to impose appropriate punishment for
serious misconducts committed by the respondent, the High Court totally misdirected
itself while exercising jurisdiction under Article 226. Judged in this background, the
conclusion of the Division Bench of the High Court cannot be regarded as proper at all.
The High Court has interfered with the punishment imposed by the competent authority
in a casual manner and, therefore, the appeal will have to be accepted.
10. For the foregoing reasons the appeal succeeds. The judgment rendered by the
Division Bench of the Gauhati High Court dated March 7, 2006 delivered in Writ Appeal
No. 282 of 2006 setting aside the order removing the respondent from service is quashed.
The direction given by the Division Bench to the appellate authority, namely, the
Inspector General of Police to consider and inflict punishment, short of removal from
service, commensurate with the gravity of the proven misconduct of the respondent is set
aside. The order passed by the competent authority removing the respondent from service
is restored. The appeal is accordingly allowed. There shall be no order as to costs.
Appeal allowed. .
AIR 2008 SUPREME COURT 2866 "Jaladi Suguna v. Satya Sai Central Trust"
(From : Andhra Pradesh)*
Coram : 2 R. V. RAVEENDRAN AND LOKESHWAR SINGH PANTA, JJ.
Civil Appeal No. 3375 of 2008 (arising out of SLP (C) No. 7818 of 2007), D/- 5 -5 -2008.
Jaladi Suguna (Dead) through L. Rs. v. Satya Sai Central Trust and Ors.
Civil P.C. (5 of 1908), O.22, R.5, R.4, R.11 - LEGAL REPRESENTATIVES - APPEAL -
Appeal - Death of respondent - L. Rs. of deceased have to be brought on record - Trial
cannot proceed till then - Any dispute as to who is legal representative has also to be
decided before trial proceeds.
Appeal Suit No. 294 of 2000, D/-19-09-2006 (A.P.), Reversed.
Filing an application to bring the legal representatives on record, does not amount to
bringing the legal representatives on record. When an L. R. application is filed, the Court
should consider it and decided whether the persons named therein as the legal
representatives, should be brought on record to represent the estate of the deceased. Until
such decision by the Court, the persons claiming to be the legal representatives have no
right to represent the
@page-SC2867
estate of the deceased, nor prosecute or defend the case. If there is a dispute as to who is
the legal representative, a decision should be rendered on such dispute. Only when the
question of legal representative is determined by the Court and such legal representative
is brought on record, it can be said that the estate of the deceased is represented. (Para
10)
Though Rule 5 does not specifically provide that determination of legal representative
should precede the hearing of the appeal on merits, Rule 4 read with Rule 11 make it
clear that the appeal can be heard only after the legal representatives are brought on
record. When a respondent in an appeal dies, the Court cannot, therefore, simply say that
it will hear all rival claimants to the estate of the deceased respondent and proceed to
dispose of the appeal. Nor can it implead all persons claiming to be legal representatives,
as parties to the appeal without deciding who will represent the estate of the deceased,
and proceed to hear the appeal on merits. The Court cannot also postpone the decision as
to who is the legal representative of the deceased respondent, for being decided along
with the appeal on merits. (Para 11)
If such a procedure is adopted it cannot be said that legal representatives were heard in
their capacity as person representing estate of deceased. If they are not heard there is no
hearing of appeal in eye of law.
Appeal Suit No. 294 of 2000, D/-19-9-2006 (A.P.), Reversed. (Para 13)

M.N. Rao, Sr. Advocate, Vedula Venkata Ramana, T.N. Rao, Ms. Kavita Yadav, Ms.
Manjeet Kirpal and Paramjeet Singh, with him for Appellant; S. S. Naganand, M.N.
Krishnamani, Sr. Advocates, Raghavendra S. Srivastsa, D. Bharat Kumar, Anand, Azim
H. Laskar, Ms. M. Indrani and Abhijit Sengupta, with them, for Respondents.
* Appeal Suit No. 294 of 2000, D/- 19-9-2006 (A. P.)
Judgement
R. V. RAVEENDRAN, J. :- Leave granted. Heard.
2. The appellants claim to be the legal representatives of one Jaladi Suguna. The said
Suguna had filed a suit in OS No. 658/ 1987 in the Court of Sub-ordinate Judge,
Vijayawada, seeking a declaration that the registered gift deed dated 27-3-1980 executed
by her in favour of the first Respondent Trust ('Trust' for short) in respect of the suit
property was null and void and for a consequential injunction restraining the said Trust
from interfering with her rights. The Trust was the first defendant and the tenant in
occupation of a portion of the suit property was the second defendant in the said suit. The
said suit was decreed by the Trial Court by Judgement and Decree dated 25-8-1999,
declaring that the said gift deed to be void and restraining the Trust from interfering with
her possession.
3. Feeling aggrieved, the Trust filed Appeal Suit No. 294/2000 in the High Court of
Andhra Pradesh. Suguna who as the first respondent in the said appeal, died during the
pendency of the appeal, on 21-3-2002. The Trust herein filed an application (CMP No.
10258/2002) to bring her husband (the third respondent herein) on record, as her legal
representative. The appellants, who are the nieces and nephews of Suguna filed an
application (CMP No. 13807/2002) seeking leave to come on record as her legal
representatives. The husband of the deceased claimed that she died intestate and he was
the sole legal heir. Incidentally, he also supported the case of the Trust in the litigation.
The appellants claimed that the deceased had bequeathed the suit property to them under
a will and they were interested in representing and safeguarding the estate of the deceased
which included the suit property and they should therefore be permitted to come on
record as the legal representatives of the deceased. Thus, there was a dispute as to who is
or are the legal representatives of the deceased Suguna. Therefore, the High Court
directed the Trial Court, under the proviso to Rule 5 of Order 22 of Code of Civil
Procedure ('CPC' for short) to try the said question and submit its finding.
4. The Trial Court accordingly, held an enquiry and submitted a report dated 28-11-2005
recording a finding that the deceased Suguna had executed two wills dated 27-4-1989 and
24-12-2002 in favour of the appellants under which the suit property was bequeathed to
them. On this finding, their application to come on record as legal representatives of the
deceased Suguna deserved to be accepted.
5. On the receipt of the said report, the High Court ought to have determined the question
as to who are the legal representatives of the deceased Suguna, as required
@page-SC2868
by Order 22 Rule 5 CPC. But it did not do so. Instead, it proceeded to hear the main
appeal itself as also the said two LR applications and rendered its judgment dated 19-9-
2006. In the judgment, it formulated the following two points as arising for its
consideration in the appeal : (i) whether the gift deed dated 27-3-1980 was void; and (ii)
whether the suit was barred by limitation. It considered the said two points and answered
them in the affirmative in favour of the Trust. Thereafter, it referred to the death of
Suguna during the pendency of the appeal and the dispute arising on account of two LR
applications. It considered the rival claims and the finding of the trial Court. It disagreed
with the finding of the trial Court and held that it was not satisfied that Suguna had
executed any will in favour of appellants. Consequently, CMP No. 10258/ 2002 filed by
the Trust to bring on record the third respondent as the legal representative of the
deceased Suguna was allowed and CMP No. 13807/2002 filed by the appellants herein
was dismissed. The High Court however clarified that its findings in regard to legal
representatives being summary in nature, the appellants can agitate their right in an
independent legal proceeding. In view of its finding on the two points relating to merits,
the High Court upheld the validity of the gift deed, allowed the appeal of the Trust, and
set aside the decree of the trial Court. As a result, the suit filed by Suguna stood
dismissed.
6. The said judgment and order of the High Court dated 19-9-2006 is under challenge in
this appeal by special leave. The appellants' challenge is three-pronged. Firstly, they
challenge the procedure adopted by the High Court in hearing the appeal without
bringing the legal representatives on record and deciding the appeal on merits first and
thereafter deciding the issue relating to legal representative. Secondly, they challenge the
decision on the question as to who are the legal representatives of Suguna. Thirdly, they
challenge the judgment on merits upholding the validity of the gift deed and dismissing
the suit.
7. We may refer to the rival contentions of the first question. According to the appellants,
the High Court ought to have decided the question of representation of the estate of the
deceased respondent first and only thereafter ought to have proceeded to hear the appeal.
They submit that the procedure adopted by the High Court has resulted in miscarriage of
justice as it did not afford them due opportunity to effectively contest the appeal on
merits. On the other hand, respondents 1 and 3 contend that there was no irregularity in
the procedure adopted by the High Court in deciding the appeal and the LR applications
together. It was submitted that the provision of Order 22, Rule 5 does not require the
question of legal representatives to be decided first before the appeal is heard. It was also
submitted that both the rival claimants to the estate, namely the husband (third
respondent) and the nephews and nieces (appellants), were represented by counsel and
were heard fully, both on the question of representation of the estate of the deceased and
on the merits of the appeal and therefore the appellants were in no way prejudiced.
Having heard the parties on the first point (relating to the procedure adopted by the High
Court), we are of the view that this appeal can be disposed of with reference to the said
preliminary point and it is not necessary to examine the other two points.
8. "Legal representative" according to its definition in Section 2(11) of the CPC, means a
person who in law represents the estate of a deceased person, and includes any person
who intermeddles with the estate of the deceased. Thus a legatee under a will, who
intends to represent the estate of the deceased testator, being an intermeddler with the
estate of the deceased, will be a legal representative. Order 22 CPC inter alia deals with
death of parties. Rule 4 relates to the procedure in case of death of one of several
defendants or of the sole defendant. Rule 5 relates to determination of question as to legal
representative. Rule 11 relates to application of Order 20 to appeals. The said rules, to the
extent relevant, are extracted below :
"4. Procedure in case of death of one of several defendants or of sole defendant :- (1)
Where one of two or more defendants dies and the right to sue does not survive against
the surviving defendant or defendants alone, or a sole defendant or sole surviving
defendant dies and the right to sue survives, the Court, on an application made in that
behalf, shall cause the legal representative of the deceased defendant to be made a party
and shall proceed with the suit.
(2) Any person so made a party may make any defence appropriate to his character as
@page-SC2869
legal representative of the deceased defendant. xxxxxxxxxxxxxxx
"5. Determination of question as to legal representative :- Where a question arise as to
whether any person is or is not the legal representative of a deceased plaintiff or a
deceased defendant, such question shall be determined by the Court :
Provided that where such question arises before an Appellate Court, that Court may,
before determining the question, direct any subordinate Court to try the question and to
return the records together with evidence, if any recorded at such trial, its findings and
reasons therefor, and the Appellate Court may take the same into consideration in
determining the question."
"11. Application of Order to appeals :- In the application of this Order to appeals, so far
as may be, the word 'plaintiff shall be held to include an appellant, the word 'defendant' a
respondent, and the word 'suit' an appeal."
(Emphasis supplied)
9. When a respondent in an appeal dies, and the right to sue survives, the legal
representatives of the deceased respondent have to be brought on record before the Court
can proceed further in the appeal. Where the respondent-plaintiff who has succeeded in a
suit, dies during the pendency of the appeal, any judgment rendered on hearing the appeal
filed by the defendant, without bringing the legal representatives of the deceased
respondent-plaintiff on record, will be a nullity. In the appeal before the High Court, the
first respondent therein (Suguna) was the contesting respondent and the second
respondent (tenant) was only a pro forma respondent. When first respondent in the appeal
died, the right to prosecute the appeal survived against her estate. Therefore it was
necessary to bring the legal representative/s of the deceased Suguna on record to proceed
with the appeal.
10. Filing an application to bring the legal representatives on record, does not amount to
bringing the legal representatives on record. When an LR application is filed, the Court
should consider it and decide whether the persons named therein as the legal
representatives, should be brought on record to represent the estate of the deceased. Until
such decision by the Court, the persons claiming to be the legal representatives have no
right to represent the estate of the deceased, nor prosecute or defend the case. If there is a
dispute as to who is the legal representative, a decision should be rendered on such
dispute. Only when the question of legal representative is determined by the Court and
such legal representative is brought on record, it can be said that the estate of the
deceased is represented. The determination as to who is the legal representative under
Order 22 Rule 5 will of course be for the limited purpose of representation of the estate of
the deceased, for adjudication of that case. Such determination for such limited purpose
will not confer on the person held to be the legal representative, any right to the property
which is the subject matter of the suit, vis-a-vis other rival claimants to the estate of the
deceased.
11. The provisions of Rules 4 and 5 of Order 22 are mandatory. When a respondent in an
appeal dies, the Court cannot simply say that it will hear all rival claimants to the estate
of the deceased respondent and proceed to dispose of the appeal. Nor can it implead all
persons claiming to be legal representatives, as parties to the appeal without deciding
who will represent the estate of the deceased, and proceed to hear the appeal on merits.
The Court cannot also postpone the decision as to who is the legal representative of the
deceased respondent, for being decided along with the appeal on merits. The Code clearly
provides that where a question arises as to whether any person is or is not the legal
representative of a deceased respondent, such question shall be determined by the Court.
The Code also provides that where one of the respondents dies and the right to sue does
not survive against the surviving respondents, the Court shall, on an application made in
that behalf, cause the legal representatives of the deceased respondent to be made parties,
and then proceed with the case. Though Rule 5 does not specifically provide that
determination of legal representative should precede the hearing of the appeal on merits,
Rule 4 read with Rule 11 make it clear that the appeal can be heard only after the legal
representatives are brought on record.
12. The third respondent, who is the husband of the deceased, wants to come on record in
his capacity as a sole legal heir of
@page-SC2870
the deceased, and support the case of the Trust that there was a valid gift by the deceased
in its favour. On the other hand, the appellants want to come on record as testamentary
legatees in whose favour the suit property was bequeathed by will, and represent the
estate of the deceased Suguna as inter meddlers. They want to continue the contest to the
appeal. When Suguna - the first respondent in the appeal before the High Court died, the
proper course for the High Court, was first to decide as to who were her legal
representatives. For this purpose the High Court could, as in fact it did, refer the question
to a Subordinate Court under the proviso to Rule 5 of Order 22 CPC, to secure findings.
After getting the findings, it ought to have decided that question, and permitted the
persons who are held to be the legal representatives to come on record. Only then there
would be representation of the estate of the deceased respondent in the appeal. The appeal
could be heard on merits only after the legal representatives of the deceased first
respondent were brought on record. But in this case, on the dates when the appeal was
heard and disposed of, the first respondent therein was dead, and though rival claimants
to her estate had put forth their claim to represent her estate, the dispute as to who should
be the legal representative was left undecided, and as a result the estate of the deceased
had remained unrepresented. The third respondent was added as the legal representative
of the deceased first respondent only after the final judgment was rendered allowing the
appeal. That amounts to the appeal being heard against a dead person. That is clearly
impermissible in law. We, therefore, hold that the entire judgment is a nullity and
inoperative.
13. We may look at it from yet another angle. The relief sought by Suguna in the suit was
one in regard to which the right to sue would have survived to her legal representatives if
she had died during the pendency of the suit. She successfully prosecuted the suit and
obtained the decree declaring the deed to be void. The said decree would continue to be
in force unless it is set aside in a manner known to law. It could be set aside in an appeal
filed by the aggrieved party, but only after hearing the plaintiff who had secured the
decree. Pronouncement of judgment in a case, can be only after the case has been heard.
(Vide section 33, Order 20 Rule 1 and Order 41 Rule 30 of CPC). When the respondent-
plaintiff died and his/ her estate remains unrepresented, it cannot be said that the appeal
was 'heard'. When the respondent-plaintiff died, the legal representatives who succeeded
to her estate will have to be brought on record and they should be heard in their capacity
as persons representing the estate of deceased plaintiff. If they are not heard, there is no
'hearing' of the appeal in the eye of law. Consequently the judgment of the trial Court
could not be disturbed or set aside by the appellate Court. Be that as it may.
14. We, accordingly, allow this appeal and set aside the judgment dated 19-9-2006,
restore the appeal to the file of the High Court, with the following directions :
(i) The High Court shall first decide the dispute between the husband of the deceased on
the one hand, and her nieces and nephews on the other, after considering the evidence and
findings dated 28-11-2005 recorded by the Trial Court and hearing the rival claimants.
(ii) After such determination, the persons determined to be the person/s entitled to
represent the estate of the deceased shall be brought on record as the legal representatives
of the deceased.
(iii) Thereafter, the appeal shall be heard on merits and disposed of in accordance with
law.
15. Having regard to the facts and circumstances, we request the High Court to dispose of
the appeal, preferably within a period of six months. Nothing stated above shall be
construed as expression of any opinion on the merits of the matter. We also make it clear
that the determination as to representation of the estate of the deceased, by the High
Court, will be only for the purposes of the appeal before the High Court and will not in
any way affect the rights of claimants to the estate of the deceased or the adjudication of
any dispute among them in any independent proceedings. Parties to bear the respective
costs.
Appeal allowed. .
@page-SC2871
AIR 2008 SUPREME COURT 2871 "United India Insurance Co. Ltd. v. Suresh K. K."
(From : 2006 (4) Ker. L. T. 333)
Coram : 2 S. B. SINHA AND LOKESHWAR SINGH PANTA, JJ.
Civil Appeal No. 2565 of 2008 (arising out of SLP (C) No. 18982 of 2006), D/- 4 -4
-2008.
United India Insurance Co. Ltd. v. Suresh K.K. and Anr.
(A) Motor Vehicles Act (59 of 1988), S.147 - MOTOR VEHICLES - INSURANCE -
Liability of insurer - Owner of goods travelling in goods carriage without goods - Travels
in capacity other than owner of goods - Insurer is not liable.
2006 (4) Ker LT 333, Reversed. (Paras 11, 12, 13)
(B) Motor Vehicles Act (59 of 1988), S.147 - MOTOR VEHICLES - INSURANCE -
Liability of insurer - Breach of conditions of Policy - 3 Wheeler goods carriage - Injured
sharing seat of driver at time of accident - Such travel is in violation of conditions of
Policy - Insurer cannot be held liable - However, considering fact that injured was coolie
worker insurer directed to pay claimant and realise same from owner. (Paras 13, 14,
15, 16)
Cases Referred : Chronological Paras
2004 AIR SCW 212 : AIR 2004 SC 1340 (Rel. on.) 12
K.L. Nandwani, S.K. Mishra, Raj Kishore Chaudhary and Debasis Mishra, for Appellant.
Judgement
JUDGMENT :- Leave granted.
2. Despite service of notice nobody has appeared on behalf of the respondent.
3. The core question which arises for consideration in this appeal is as to whether a
person who has hired a goods carriage vehicle would come within the purview of sub-
section 1 of Section 147 of the Motor Vehicles Act, 1988, although no goods as such were
carried in the vehicle.
4. The claimant/respondent was a 'coolie-worker'. He allegedly hired an auto rickshaw
which is a goods carriage vehicle bearing registration No. KL-8/M8568. The accident
occurred when he was sitting by the side of the driver. According to him the driver was
driving the vehicle in a most rash and negligent manner. When the vehicle reached at
Kandanchira, the driver turned it to the left side without applying brake, as a result
whereof, it overturned. The claimant allegedly suffered the following injuries :
"1. Compound fracture lower both bones of 1/3rd left leg and multiple abrasions. 2.
Lacerated wound (R) and (L) Legs."
5. He filed a claim application in terms of Section 166 of the Motor Vehicles Act for a
sum of Rs. 2.25 lakhs; the details whereof are as under :

"(a) Loss of earning from 13-8-99 to till now Rs. 15,000.00


(b) Partial loss of earnings from ...... to ............ at the net rate of Rs ...... a day/week
-" 10,000.00
(c) Transport to Hospital -" 3,000.00
(d) Extra nourishment -" 25,000.00
(e) Damage of clothing and article -" 2,000.00
(f) Other Medical Expenses -" 40,000.00
_______________
95,000.00
_______________
(g) Compensation for pain and suffering -Rs. 30,000.00
(h) Compensation for continuing or permanent disability, if any-Rs. 50,000.00
(i) Compensation for the loss of earning -Rs. 50,000.00
_______________
Rs. 1,30,000.00
_______________
Total Rs. 2, 25,000.00"

6. Appellant in his written statement raised the contention that although the vehicle in
question was insured, it is not liable to reimburse the owner of the vehicle as the injured
was not the owner of the alleged goods carried therein, and he was travelling as a
gratuitous passenger. Violation of conditions of policy was also alleged. By reason of the
award dated 23-1-2003, the
@page-SC2872
Tribunal held :
"10. I have already found that the accident had occurred due to the rash and negligent
driving of the goods auto rickshaw by the 1st respondent. That he was also the owner of
that vehicle, at the time of accident, is evident from Ext. A3, Report of Inspection of the
vehicle. Hence he is liable to pay the compensation to the petitioner. Admittedly his
goods auto rickshaw was insured with the 2nd respondent at the time of accident. Ext. D1
is copy of the insurance policy. Hence, they are liable to indemnify the 1st respondent.
Thus, the respondents 1 and 2 are liable to pay the compensation to the petitioner. Issue is
found accordingly."
7. A sum of Rs. 1,19,300/- was awarded in favour of the claimant with interest @ 9% per
annum. Appellant preferred an appeal before the High Court in terms of Section 173 of
the Motor Vehicles Act. The High Court negatived the contention of the appellant that the
word 'goods' was used in Section 147 of the Act, would not be referable to the word
'carried' stating :
"According to us, the language of the amended provision does not show that the owner or
the representative must accompany the goods or his representative who hires the vehicle
travels in the hired vehicle from the place of hiring to the place where the goods are to be
loaded into the vehicle and then proceeds to travel along with the goods. It is also
common that after unloading the goods such passengers travel in the same vehicle to the
place from where they commenced journey. The passenger does so and is allowed to do
so in his capacity as the owner of the goods or his representative who has hired the
vehicle for transporting goods. The amended provision makes it explicitly clear that the
word 'carried' qualifies the owner of goods or his representative and not the goods
carried. If goods are found inside the vehicle at the time of the accident, it is a clinching
circumstance to establish that the passenger who claims to be the owner of goods or the
owner's representative was travelling in that capacity. Chances of passengers or the
insured raising false claims in this regard cannot be safe method to ascertain the intention
of the Legislature. False claims can be disapproved by the insurer by adducing materials
and evidence and also by raising appropriate contentions. In our view, such issues are
matters of evidence and will not stand scrutiny while construing a beneficial provision
intended to compensate the loss caused to innocent victims of motor accidents. The party
who claims that the person who died or sustained injury as the owner of goods or the
representative of the owner of the goods shall discharge the burden cast on him. Merely
for the reason that the benefit granted will be misused, it will not be proper to give a
narrow interpretation to the above provision. We, therefore, hold that the owner or the
authorised representative need not invariably be shown to accompany the goods at the
time the goods carriage meets with accident causing injury to or resulting in the death of
the passenger who is either the owner of the goods or the authorised representative of the
owner of the goods."
8. Mr. Nandwani, learned counsel appearing on behalf of the appellant would urge that
the High Court committed a serious error in passing the impugned judgment insofar as it
failed to take into consideration that:
1. the vehicle in question being a goods vehicle, the driver could not have allowed
anybody to sit by his side.
2. the Tribunal as also the High Court did not arrive at the finding that the
claimant/respondent was the owner of goods particularly when no goods were found to
have been carried therein.
3. on a plain reading of sub clause (i) of clause (b) of the sub-section 1 of Section 147,
the words 'carried in the vehicle' must be held to be qualifying 'owner of the goods' or 'his
authorised representative'.
4. Section 147 (b)(i) reads as under :
"(i) against any liability which may be incurred by him in respect of the death of or
bodily injury to any person, including owner of the goods or his authorised representative
carried in the vehicle or damage to any property of a third party caused by or arising out
of the use of the vehicle in a public place."
5. Section 147 provides for mandatory insurance. The policy of insurance in terms of the
said provision must be in relation to the person or classes of person specified in the policy
sought to be insured. The insurance would be against any liability which the insured
incurs.
9. The insurance policy should, inter alia, be in respect of death or bodily injury of the
@page-SC2873
person carried in the vehicle. Such person may be the owner of the goods or his
authorised representative.
10. The High Court, therefore, may be correct that the owner of the goods would be
covered in terms of the said provision.
11. But the question which has not been adverted to by the High Court is as to whether
the policy contemplates the liability of the owner of the vehicle in respect of a person
who was in the vehicle in a capacity other than owner of the goods. If a person has been
travelling in a capacity other than the owner of the goods, the insurer would not be liable.
The purpose for which the provision had to be amended by Act No. 54 of 1994 was to
widen the scope of the liability of the insurance company.
12
. It is now well settled that the term 'any person' envisaged under the said provision shall
not include any gratuitous passenger (National Insurance Co. Ltd. v. Baljit Kaur 2004 (2)
SCC 1). 2004 AIR SCW 212

13. If the claimant had not been travelling in the vehicle as owner of the goods, he shall
not be covered by the policy of the insurance. In any view of the matter in a three wheeler
goods carriage, the driver could not have allowed anybody else to share his seat. No other
person whether as a passenger or as a owner of the vehicle is supposed to share the seat
of the driver. Violation of the condition of the contract of insurance, therefore, is
approved.
14. The Tribunal and the High Court, therefore, in our considered opinion, should have
held that the owner of the vehicle is guilty of the breach of the conditions of policy.
15. The question which arises for our consideration, however, is keeping in view the fact
that the accident took place on or about 13-8-1999, and further in view of the fact that the
claimant was a coolie worker as to whether he would be in a position to realise the dues
from the owner of the vehicle, we think not.
16. Keeping in view the aforementioned facts and circumstances into consideration, we
are of the opinion that with a view to do complete justice between the parties, a direction
should be given to the appellant to pay the amount to the claimant and realise the same
from the owner of the vehicle. Such a direction would, in our opinion, serve the ends of
justice.
17. We are passing this order in view of the fact that the appellant has already deposited
the amount pursuant to a direction issued by this Court dated 13-11-2006.
18. The appeal is allowed to the above extent.
19. No costs.
Order accordingly. .
AIR 2008 SUPREME COURT 2873 "C.E.S.C. Ltd. v. Sandhya Rani Barik"
(From : Calcutta)*
Coram : 2 Dr. A. PASAYAT AND P. SATHASIVAM, JJ.
Civil Appeal No. 7201 of 2005 with C.A. Nos. 7202 to 7206 of 2005, D/- 7 -7 -2008.
C.E.S.C. Ltd. v. Smt. Sandhya Rani Barik and Ors.
W.B. Land (Requisition and Acquisition) Act (2 of 1948), S.7(2)(a) - ACQUISITION OF
LAND - Market value - Determination - Acquisition of more than three bighas of land -
Plot acquired was wholly land locked - Comparison sought to be made with
comparatatively small piece of land sold - Percentage of price will have to be knocked off
- Award of compensation of Rs. 2.25 Lacs per cottah after making said deductions - Is
proper - Rate of interest as statutorily fixed shall be applicable. (Paras 18, 19, 22)

Shanti Bhushan, Shyam Diwan, Bhaskar Gupta, Sr. Advocates, K.V. Viswanathan,
Sanjeev K. Kapoor, Vishal Gupta, Vikram Bajaj (for M/s. Khaitan and Co.), Dhruv
Mehta, Chanchal Kr. Dutta, D. Bera, Partha Sil, Mrs. Sarla Chandra, Avijit Bhattacharjee
and Saumya Kundu, with them for the appearing parties.
*F.A. No. 12 of 2002, D/- 11-2-2003 (Cal).
Judgement
1. Dr. ARIJIT PASAYAT, J. :- These appeals have been filed by CESC Ltd. questioning
correctness of the judgment rendered by a Division Bench of the Calcutta High Court
disposing of three appeals filed by the present appellant challenging common judgment
and order of learned Land Acquisition Judge by which he disposed of three references
from the award of the Collector. The acquisition of these lands took place under the West
Bengal Land (Requisition and Acquisition) Act, 1948 (in short the 'West Bengal Act') and
not under the Land Acquisition
@page-SC2874
Act, 1894 (in short the 'Act').
2. Cross appeals have been filed by the claimants seeking enhancement of rent and higher
rate of interest.
3. The number of claimants was quite large and about 20. They were booked in three sets.
The land was acquired under the West Bengal Act in May 1995. The requisition was
made in 1995 and the Notification was issued on 3.5.1995. The total area was more than
3 bighas. One bigha is 20 cottahs and 1 cottah is 720 sq.ft. The land was part of 10 bigha
tract situated in 156A, Manicktala Main Road owned previously by a single common
ancestor namely, Kali Pada Barik who had since died. The Collector made the award
sometime in the year 1997 and thereafter the Land Acquisition Judge was approached on
reference by the Banks i.e., the respondents in the three appeals. The Collector's award
was initially on the basis of about Rs. 50,000/- per cottah as compensation but the Land
Acquisition Judge on the first occasion raised it to nearly Rupees 5 lakhs per cottah. At
that stage the State and the Bariks were opposite parties. Since the land was acquired for
the purpose of present appellant which was the requiring authority under the West Bengal
Act, acquisition has to be preceded by a requisition for maintenance of supplies essential
to public life and in this case supply of electricity was the service involved. A Sub-station
i.e. 132 K.V. had already been built over the acquired land. The Requiring Authority filed
a Writ Petition before the High Court taking the stand that the matter was decided in its
absence. On 3rd May, 2000 order was passed by a Division Bench of the High Court
whereby the matter was remanded to the Land Acquisition Judge directing the appellant
to be made a party. It also directed that persons who had already been examined before
the Land Acquisition Judge would be again presented for cross examination by the
appellant subject to their availability. The matter was considered again. The Land
Acquisition Judge again made assessment and after hearing the present appellant held
that the rale per cottah would be higher. But in view of the fact that earlier a lower rate
had been fixed, same was maintained.
4. The determination was challenged before the High Court.
5. The appellant referred to the evidence of some of the witnesses examined by Bariks.
One Chandra Nath Barik admitted that the acquired land was previously being used by
the washerman of the family for washing clothes and drying those. Evidence was led by
the appellant that at the time of requisition of the land in 1990 the land was in low lying
area and was filled with water hyacinths. Just outside the boundary of the tract acquired,
there was a big pond. The land had to be filled up by copious supply of fly ash which was
supplied by the appellant itself from its generation plant at Titagarh. The thickness of the
filling fly ash layer was of the order of 10 ft. The land development was undertaken by
the appellant by engagement of certain contractors. Their bills were exhibited and the
payment to the order of about Rs. 30 lakhs was claimed.
6. Bariks on the other hand took the stand that entire money was not spent on land
development, but a part of that was for putting up the structures of the appellant as well.
The High Court referred to some factual aspects and took note of the fact that the
appellant's main grievance was that large track of land was the subject matter of
acquisition and the rates applied for smaller plots cannot have any relevance. After
referring to certain factual aspects, the High Court disposed of the appeals in the
following manner:
"1. the market price of the land acquired is determined to be Rs. 2.25 lacs per cottah and
the referring claimants in all these cases do get an award of compensation of Rs. 2.25 lacs
per cottah.
2. In addition thereto, they are entitled to solatium at the rate of 30% on the said land
value.
3. They shall also be entitled to additional compensation of 12% per annum on the land
value, but not the solatium, from 3.5.95 to 27.3.97 (see : Sunder's case (2001) 7 SCC 211)
which deals with ultimate i.e. 5.28 interest on solatium but does not pronounce that
Section 23(1A) will apply on Section 23(2) also).
4. The respondents/referring claimants shall also be entitled to rent compensation at the
rate of 9% per annum from 16.10.90 to 2.5.95 on the land value computed at the rate (per
cottah) of Rs. 2.25 lac less 25%; we clarify that the land value is to be reduced by 25%
and rent compensation shall run thereon the said period 16.10.90 to 2.5.95.
5. On all the items 1, 2, 3 and 4 the respondents/
@page-SC2875
referring claimants shall be entitled to interest at the rate of 9% per annum from 27.3.97
until payment or payment into court; if such payment is not made within 17th February,
2004 the interest shall be thereafter at the rate of 15% per annum; and
6. Due credit shall be given in regard to the wiping of liability as regards the above heads,
on account of payments or payments into court already made by the appellants."
7. Stand of learned counsel for the appellant essentially was that the rate applicable for
acquired land cannot be as high as Rs. 5,32,000/- per cottah as was fixed by the
Reference Court. It is submitted that though the High Court fixed it to Rs. 2,25,000/- per
cottah, same was also high. The claimants relied upon sale deeds measuring about
cottahs. In the Cross Appeals filed, Bariks have taken the stand that the appreciation
aspect has not been taken note of. The reductions i.e. 25% for land locking, 15% for road
frontage and 5% for belting are irrational. A large number of claimants i.e. 22 are
involved and it is their undivided shares which had to be taken note of. It was pointed out
that each person on partition does not get more than 5 cottah. The purpose of acquisition
was construction of sub-station which required large area. The acquired land was
homestead urbanized industrial area.
8. It is pointed out that the statute i.e. West Bengal Act provides that the rate of interest
has to be 9% for one year and thereafter 15%.
9. It is submitted that the High Court took note of some common passage concept. There
was no lease and licence arrangement with Purbasa Housing Estate. So ingress and egress
facilities were known. Because of the locational advantage no deduction is called for any
largeness. It is pointed out that the High Court went wrong in not adding land value for
7½ years. The High Court while considering the largeness aspect has fixed 15 to 20%. It
is the stand of Bariks that largeness cannot be an issue in the present one. The Reference
Court rightly held that largeness question was not relevant. It was a case of acquisition of
contiguous land. Largeness has to be linked to the purpose. The belting method has no
application because it was an open area for particular purpose. Similarly, there was no
question of any land locking. The High Court observed, according to the Bariks,
erroneously that there was any access. The common passage was linked to the main road.
Since the purpose was to have protected area, largeness question was not of any
relevance. So far as the frontage factor is concerned, it was submitted that for the purpose
for which the land had been acquired frontage may not be necessary. It has to be
anywhere in the area.
10. Additionally, belting method is an obsolete method. Further, belting is not a proper
method as the land was situated in well defined development block.
11. It has been pointed out by learned counsel for the appellant that the purpose is really
irrelevant for determining market value. The potential has to be seen. It is a case of
willing buyer and willing seller. It is also pointed out that number of ultimate claimants is
really irrelevant because 20 sellers have to join if they have to sell the land and suit for
partition has to be filed.
12. The High Court has noticed the following factual position.
(i) The acquired plot of land is about 150 to 200 ft. away from the Manicktala Main
Road. It is the third belt away from the main road.
(ii) It is impossible to conclude that there is any frontage of the acquired land on the
southern side where Manicktala main road runs.
(iii) So far as northern and southern sides of the acquired plot are concerned it is admitted
that those are land bound. On the west there is other land of Bariks and on the north there
was a housing estate called Purbasa Housing Estate.
13. The case of the appellant was that the plot which was acquired for their use was
wholly land locked. This forms a very important factual issue which is important while
determining the compensation.
14. The issue was whether on the eastern side of the acquired land there was a frontage
on 40 ft. municipal public road. The case of Bariks that this was so. On the other hand the
appellant took the stand that there was no municipal road and it was a private land of
Purbasa Housing leading from the Manicktala Main road on the South into the Housing
Estate itself. The road runs south to north and belongs to Purbasa. That 40 ft
@page-SC2876
wide road is not a municipal road and it has no name. There are no premises numbers
attached to it.
15. Interestingly, the witnesses of the appellant were asked a question as to whether it had
taken permission from the Public Works Department for laying cables under the P.W.D.
road. No evidence was there to show that the road was a public road or it connected with
the Manicktala Main Road. From the plan it appears that the road goes into the Purbasa
Housing Estate and ends there. Therefore, the plot was wholly land locked. The High
Court ultimately therefore fixed the rates as noted above.
16. The armchair assessment of land value has to proceed with common sense and
circumspection. One should attempt to find out the just and reasonable compensation
without attempting any mathematical precision in that regard. For the purpose of
assessing compensation, the efforts should be to find out the price fixed for the similarly
land in the vicinity.
17. The difference in the land acquired and the land sold might take on various aspects.
One plot of land might be larger, another small, one plot of land might have a large
frontage and another might have none. There might be differences in land development
and location. There might be special features which have to be taken note of and
reasonably considered in the matter of assessing compensation.
18. Where a very large plot of land has been acquired and the comparison is sought to be
made with a comparatively small piece of land which has been sold or otherwise dealt
with, then in that event, a percentage of the price is to be knocked off because of the
largeness itself of the acquired land. Accordingly, the High Court made the deductions.
The High Court also dealt with the question of land locking and held that it was a special
feature which had to be taken note of.
19. We do not find any infirmity in the approach of the High Court. Therefore, the rate
fixed by the High Court does not suffer from infirmity. The appeals filed by the appellant-
CESC, therefore, stand dismissed.
20. Rate fixed by the High Court as questioned in the cross appeals does not warrant
interference. But there is substance in the plea regarding rate of interest.
21. Section 7(2)(a) of West Bengal Act is as follows :
"7(2)(a) - When the compensation has been determined under sub-section (1) the
Collector shall make an award in accordance with the principles set out in section 11 of
the Land Acquisition Act, 1894 and the amount referred to in sub-section (2) of section
23 of that Act shall also be included in the award:
Provided that interest at the rate of nine per centum per annum on the amount of
compensation under the award from the date of the publication of the notice under
subsection (1a) of section 4 until payment shall be included in the amount payable under
the award :
Provided further that if such compensation or any part thereof is not paid or deposited
within a period of one year from the date of publication of the notice under subsection
(1a) of section 4, interest at the rate of fifteen per centum per annum shall be payable
from the date of expiry of the said period of one year on the amount of compensation or
part thereof which has not been paid or deposited before the date of such expiry."
22. The rate of interest as statutorily fixed shall be applicable in place of rate fixed by the
Reference Court and the High Court.
23. The cross-appeals are allowed to that limited extent. There will be no order as to
costs.
Order accordingly. .
AIR 2008 SUPREME COURT 2876 "Murugan v. State of Tamil Nadu"
(From : 2006 (2) Mad LJ (Cri) 427)
Coram : 2 Dr. A. PASAYAT AND P. SATHASIVAM, JJ.
Criminal Appeal No. 997 of 2008 (arising out of SLP (Cri.) No. 4715 of 2007), D/- 7 -7
-2008.
Murugan v. State of T.N.
(A) Penal Code (45 of 1860), S.300 - MURDER - WITNESS - Murder - Eye-witness -
Unnatural conduct - Eye-witness, husband of deceased - After witnessing accused
strangulating his wife chose to chase accused - He also informed people found in nearby
church and thereafter came back to verify fate of his wife - Conduct cannot be said to be
unnatural - It is quite common for a person under shock to share his grief to
@page-SC2877
persons who are found close by. (Para 8)
(B) Penal Code (45 of 1860), S.300 - Evidence Act (1 of 1872), S.27 - MURDER -
EVIDENCE - Murder - Recovery evidence - Reliability - Accused working as servant in
shop of complainant - Residing in shop of complainant - Committing murder of
complainant's wife by strangulation after closing door of shop from inside - On being
seen by complainant running away - Recovery of blood stained clothes of accused on
basis of his confessional statement - Plea that accused who was spotted while doing act
would not have got time to conceal his clothes - Not tenable since it was not the case that
complainant opened shop door after spotting accused - But it was accused who opened
door and thereafter ran away. (Para 9)

Ms. Prasanthi Prasad, Advocate (SCLSC), for Appellant; R. Shunmugasundaram, Sr.


Advocate, S.J. Aristotle and V.G. Pragasam, with him for Respondent.
Judgement
1. Dr. ARIJIT PASAYAT, J. :-Leave granted.
2. Challenge in this appeal is to the judgment of a Division Bench of the Madras High
Court upholding the conviction of the appellant for offences punishable under Section
376(1) read with Section 511 of the Indian Penal Code, 1860 (in short the 'IPC') and
Section 302 IPC. The appellant was sentenced to undergo rigorous imprisonment for ten
years and Life Imprisonment for two offences. As noted above, fine was also imposed
with default stipulation.
3. Sans unnecessary details the prosecution version in a nutshell is as follows :
Tamilselvi (hereinafter referred to as the 'deceased') is none other than the wife of
Andrews (PW1). They had been blessed with three children. Except Romeo (PW 10) the
other two children were staying in a hostel.
PW 1 was carrying on groundnut cake business in the ground floor of his house at Gandhi
Nagar, Chennai. PW 1, the victim and their daughter Romeo were staying in the upstairs
of the said house. The accused Murugan was employed as an assistant in PW1's shop and
he was staying in the ground floor itself where the business was carried on. The deceased
used to get his ration of food from PW 1.
On 3.8.2000 at about 1.30 p.m. the victim went to the ground floor for the purpose of
handing over the ration of food to the accused. P.W. 1 waited for some time, but the
victim had not returned. He came down to the ground floor and heard an alarming noise.
When he attempted to push the outer door of the ground floor, he found that it was locked
from inside. P.W. 1 went around the house and peeped through the window. He found to
his shock that the accused, taking position on his wife who was lying on the ground,
attempted to strangulate her. Thereafter the accused opened the door from inside and sped
away from the scene of occurrence.
P.W. 1 gave a chase accompanied by Elumalia-P.W. 2. The accused took shelter in a
nearby bush. He went to the church and informed the people over there. He came down to
his house and found his wife dead. Thereafter P.W. 1 went to Kolathur Police Station and
lodged a complaint (Ex.P1) to the sub-Inspector of Police, P.W. 9, who was present over
there. The latter registered a case in Crime No. 1050/2000 for the offence under Section
302 IPC and prepared printed FIR Ex. P9 and despatched the same to the learned Judicial
Magistrate concerned and the copies thereof to the higher officials.
The Inspector of Police, Mr. Natrajan-P.W. 13, who was Incharge of the said police
station when Varadarajan, the regular Inspector of Police P.W. 14 was on leave, took up
the case for investigation on receipt of a copy of the FIR and rushed to the scene of
occurrence and prepared the rough sketch-Ex.P-12. He also prepared the observation
Mahazar-Ex. P-2 in the presence of Chellaiah, P.W.4 and another witness. He held inquest
on the dead body and prepared the inquest report, Ex. P13. At about 8.30 p.m. on the said
day, in the presence of the aforesaid witnesses, P.W. 13 recovered thali Chain M.O.4 and
packing material-M.O. 7 under relevant mahazar, Ex. P-3. He entrusted the dead body to
the Head Constable Mohan, P.W. 8 for the purpose of taking the same to the doctor for
conducting postmortem examination.
Dr. Deivasigamnai, P.W. 7, conducted autopsy on the dead body of the victim at about
11.40 a.m. on 4.9.2000 and found the following injuries and symptoms on the dead
body :
"A well defined incomplete oblique ligature
@page-SC2878
abrasions mark in front of the neck at the level of thyroid cartilage, 16 x 1 cms on the
front, the ligature abrasion was 6 cms below the chin and 6 cms about the suprasternal
noted and the ligature abrasion was absent on the back of the neck. The subcutaneous soft
tissues underlying the ligature abrasion were found congested.
2) Inward compression fracture of right horn of the hyoid bone found with extravasations
of blood in the surrounding soft tissues.
Heart : Intact, Normal. Trachea : Empty. Stomach contained 200 ml. of brown fluid with
partly digested cooked rice particles. No definite smell."
4. After investigation charge sheet was filed. As the accused pleaded innocence, he was
put on trial.
5. In order to establish the prosecution version 14 witnesses were examined. Placing
reliance on the evidence of PWs. 1 and 2, the trial court found the accused guilty and
convicted and sentenced. The High Court upheld the conviction and the sentence.
6. In support of the appeal, learned counsel for the appellant submitted that the defence
version has been erroneously discarded by the High Court. He has stated that the conduct
of PW 1 after allegedly having seen the accused with his wife unnatural and should not
have been relied upon. The presence of PW 2 at the spot had also not been explained.
7. Learned counsel for the respondent-State supported the judgment of the High Court.
8. PW 1 has chosen to chase the accused along with PW 2 and having found some people
in the church, which is nearby, informed them about the occurrence and thereafter came
back to his house to verify the fate of his wife. PW 1 obviously was in a state of shock
having seen the accused strangulating his wife. It is quite common for a person under
shock to share his grief to the persons who are found close by. It is not as if PW 1 rushed
straight to the police station after informing certain persons in the church without even
verifying the fate of his wife.
9. The accused had been arrested on 6.9.2000 and only on the basis of his confessional
statement his apparels had been recovered. It is contended by the learned counsel for the
accused that the accused, who was spotted committing the crime, would not have taken
some time to hide his apparels at a safe place. It is the case of the prosecution that PW 1
having witnessed the occurrence by peeping through the window came down to the
doorway with a view to open the door. The door was opened from inside by the accused.
It is not as if the door was opened by PW 1 immediately after witnessing the occurrence
through the window. The accused, who was inside the house, would have had time to
remove his apparels, which were found blood stained, to put it in a safe place in the
house. Further it will not take much of a time to remove the clothes by a person who was
in a hurry to escape from the scene of crime. In view of the above, there is nothing to
doubt the recovery of the apparels of the accused made by the investigating officer. The
recovery at the instance of the accused raises presumptions of guilt as against him.
10. It is an unfortunate case where the accused has come out with a repulsive counter
version that he had some affairs earlier with the victim, and when he was found
embracing the wife of PW 1, it was witnessed by PW 1. He has stated that he was not the
author of the murder. If at all the victim had an affair with a stranger residing at a far off
location, it would not have come to light. But stand of the accused, who lived in the
ground floor for five years to be having an affair without being noticed is too hollow to
be accepted.
11. Trial court and the High Court have analysed the evidence in great details and have
come to the right conclusion about involvement of the accused. We do not find any
infirmity in the reasoning of the trial court and the High Court to warrant any
interference.
12. The appeal fails and is dismissed.
Appeal dismissed. .
AIR 2008 SUPREME COURT 2878 "Radhe v. State of Chhattisgarh"
(From : 2007 (1) Cg LJ 96 (Chhat.)
Coram : 2 Dr. A. PASAYAT AND P. SATHASIVAM, JJ.
Criminal Appeal No. 999 of 2008 (arising out of SLP (Cri) No. 3018 of 2007), D/- 7 -7
-2008.
Radhe v. State of Chhattisgarh.
(A) Penal Code (45 of 1860), S.96, S.99 - PRIVATE DEFENCE - PLEA - Private
defence
@page-SC2879
Right of - Plea of, cannot be based on surmises and speculation. (Para 6)
(B) Penal Code (45 of 1860), S.96, S.99 - PRIVATE DEFENCE - Private defence - Right
of - Availability - Fact that person may have chance to inflict severe and mortal injury on
aggressor - Is not relevant. (Para 6)
(C) Penal Code (45 of 1860), S.96, S.99 - PRIVATE DEFENCE - Private defence -
Exercise of right - Entire incident must be examined with care and viewed in its proper
setting. (Para 6)
(D) Penal Code (45 of 1860), S.96, S.100, S.101 - PRIVATE DEFENCE - Right of
private defence - Claim extending to voluntarily causing death - Burden is on accused to
show that there were circumstances for apprehending grievous hurt or death to him.
(Para 6)
(E) Penal Code (45 of 1860), S.96, S.102, S.105 - PRIVATE DEFENCE - Right of
private defence of body and property - Commencement and continuance of - Right
commences as soon as reasonable apprehension of danger arises from an attempt or threat
- It lasts so long as reasonable apprehension of danger continues.
AIR 1963 SC 612; 2003 AIR SCW 469; 2003 AIR SCW 3984, Rel. on. (Para 7)
(F) Penal Code (45 of 1860), S.300(4), S.299(c) - MURDER - CULPABLE HOMICIDE
- Murder or culpable homicide not amounting to murder - Distinction between - Safest
way of approach is to keep in focus keywords used in S.299 and S.300.
AIR 1977 SC 45; 2002 AIR SCW 3463; AIR 1958 SC 465; 2006 (8) SCC 678, Rel. on.
(Paras 10, 15, 16)
(G) Penal Code (45 of 1860), S.300, S.304, Part I, S.96, S.99, S.101 - MURDER -
CULPABLE HOMICIDE - PRIVATE DEFENCE - Murder or culpable homicide - Proof
- Acccused persons allegedly assaulted deceased with Pharsa and Lathi - Homicidal death
of deceased was established - Plea of self defence raised by accused - However, no
credible evidence to establish that accused persons were under such grave apprehension
about safety of their life and property that retaliation to extent done was absolutely
necessary - Merely because there was a quarrel and accused persons claimed to have
sustained injuries, that does not confer a right of private defence extending to the extent
of causing death - Appellant had chopped both legs of deceased and caused other injuries
with weapon in brutal manner - Conviction of appellant for murder - Proper. (Paras
9, 19, 22)
Cases Referred : Chronological Paras
(2006) 8 SCC 678 (Rel. on) 18
2003 AIR SCW 469 : AIR 2003 SC 976 : 2003 Cri LJ 1226 (Rel. on) 8
2003 AIR SCW 3984 : AIR 2003 SC 3617 : 2003 Cri LJ 3876 (Rel. on) 8
2002 AIR SCW 3463 : AIR 2002 SC 2961 (Rel. on) 18
AIR 1977 SC 45 : 1977 Cri LJ 1 (Rel. on) 18
AIR 1963 SC 612 : 1963 (1) Cri LJ 495 (Rel. on) 7
AIR 1958 SC 465 : 1958 Cri LJ 818 (Rel. on) 11, 14, 15
Feroz Ahmed, (A.C.), for Appellant; Dharmendra Kumar Sinha and Anil Jha, for
Respondent.
Judgement
Dr. ARIJIT PASAYAT, J. :- Leave granted.
2. Challenge in this appeal is to the judgment of a Division Bench of the Chhattisgarh
High Court dismissing the appeal filed by the appellant (hereinafter referred to as the
'accused').
3. Challenge before the High Court was to the judgment of the learned Additional
Sessions Judge, Manendragarh. The trial Court had found the appellant guilty of offence
punishable under Section 302 of the Indian Penal Code, 1860 (in short the 'IPC') but
found the co-accused, namely, Kashi and Dev Kumar to be not guilty. The appellant was
sentenced to undergo RI for life and fine with default stipulation.
4. Prosecution version as unfolded during trial is as follows :
On 10.11.1997 Gyan Singh (hereinafter referred to as the 'deceased') went to
Ramkhilawan's house for collecting kanda (eatable bulb). When he did not come back till
evening, his father Heeralal went in search of the deceased to the house of Ramkhilawan
in the evening at around 7.00 p.m. Heeralal along with Ramkhilawan and his son Gyan
Singh were returning to his house. On the way, when they reached near
@page-SC2880
the house of Kashi, Kashi started scolding Ramkhilawan, who was refrained from doing
so. Therefore, a quarrel erupted. Appellant who was carrying pharsa and Kashi a lathi
started beating. Appellant gave a pharsa blow on the leg of the deceased. The leg was cut
and turned into two pieces. Gyan Singh fell down, thereafter Radhe chopped his other leg
and assaulted Gyan Singh with pharsa on his thigh and other parts of the body. Heeralal
came to rescue him. Dev Kumar assaulted him with a lathi on his head and also gave a
blow on his left shoulder. Heeralal fell down. When Ramkhilawan intervened, he was
assaulted by Dev Kumar. In the meantime, Beerbali, who is son of Ramkhilawan came
there. He was also beaten by Radhe with pharsa. Kashi assaulted Ramkhilawan with a
club. Gyan Singh instantaneously died at the spot and others were injured. Accused
appellant and his associates Kashi and Dev Kumar fled away from the place of
occurrence.
Heeralal gave intimation and lodged First Information Report. Both the documents were
recorded by Arjun Ram, Assistant Sub Inspector, Head Constable Jagsai conducted
inquest, prepared report and forwarded the dead body of Gyan Singh for autopsy to
Community Health Centre, Manendragarh. He collected blood stained and plain earth
from the spot. Dr. S.K. Chainpuria conducted autopsy. On examination, he found nine
injuries on the body of deceased and according to his opinion, the cause of death was
syncope due to shock and external haemorrhage caused by multiple injuries. All the
injuries found on the body of Gyan Singh were caused by hard and/or sharp objects
except one which was found to be abrasion and present below left knee. Death was
homicidal in nature. He prepared autopsy report and describing all the injuries found on
the body of the deceased forwarded the report to the concerned police station. Injured
Beerbali, Ramkhilawan and Heeralal were also sent for medical examination. On medical
examination, it was found that they have sustained various injuries. On the memorandum
statement of accused Kashi one club and one pharsa were recovered from the appellant
and seized. From Dev Kumar one club was seized. After post mortem examination, the
clothes found on the body of the deceased were also collected. The statements of
witnesses were recorded. The seized pharsa, clothes and earth were sent for chemical
examination and on examination stained earth, pharsa and clothes of Gyan Singh were
found to be stained with blood.
After completion of investigation, the charge sheet was filed in the Court of Additional
Chief Judicial Magistrate, Manendragarh, who committed the case to the Court of
Session for trial. Charges were framed against the appellant and co-accused. The
accused-appellant pleaded innocence and false implication.
5. Before the High Court the main stand was that the assault, if any done by the appellant
was in exercise of right of private defence and, therefore, conviction was not called for.
The learned counsel for the respondent on the other hand supported the judgment of the
trial Court. The High Court did not accept appellant's plea and dismissed the appeal.
Stands taken before the High Court were reiterated in this appeal.
6. A plea of right of private defence cannot be based on surmises and speculation. While
considering whether the right of private defence is available to an accused, it is not
relevant whether he may have a chance to inflict severe and mortal injury on the
aggressor. In order to find whether the right of private defence is available to an accused,
the entire incident must be examined with care and viewed in its proper setting. Section
97, IPC deals with the subject-matter of right of private defence. The plea of right
comprises the body or property (i) of the person exercising the right; or (ii) of any other
person; and the right may be exercised in the case of any offence against the body, and in
the case of offences of theft, robbery, mischief or criminal trespass, and attempts at such
offences in relation to property. Section 99 lays down the limits of the right of private
defence. Sections 96 and 98 give a right of private defence against certain offences and
acts. The right given under Sections 96 to 98 and 100 to 106, IPC is controlled by Section
99. To claim a right of private defence extending to voluntary causing of death, the
accused must show that there were circumstances giving rise to reasonable grounds for
apprehending that either death or grievous hurt would be caused to him. The burden is on
the accused to show that he had a right of private defence which extended to causing of
death. Sections 100 and 101, IPC define the limit and extent of right of private defence.
@page-SC2881
7. Sections 102 and 105, IPC deal with commencement and continuance of the right of
private defence of body and property respectively. The right commences, as soon as a
reasonable apprehension of danger to the body arises from an attempt, or threat, or
commit the offence, although the offence may not have been committed but not until that
there is that reasonable apprehension. The right lasts so long as the reasonable
apprehension of the danger to the body continues. In Jai Dev v. State of Punjab (AIR
1963 SC 612), it was observed that as soon as the cause for reasonable apprehension
disappears and the threat has either been destroyed or has been put to route, there can be
no occasion to exercise the right of private defence.
8

. The above position was highlighted in Rizan and Another vs. State of Chhattisgarh,
through the Chief Secretary, Govt. of Chhattisgarh, Raipur, Chhattisgarh (2003 (2) SCC
661) and Sucha Singh and Anr. v. State of Punjab (2003 (7) SCC 643). 2003 AIR
SCW 469
2003 AIR SCW 3984

9. Merely because there was a quarrel and accused persons claimed to have sustained
injuries, that does not confer a right of private defence extending to the extent of causing
death as in this case. Though such right cannot be weighed in golden scales, it has to be
established that the accused persons were under such grave apprehension about the safety
of their life and property that retaliation to the extent done was absolutely necessary. No
evidence much less cogent and credible was adduced in this regard. The right of private
defence as claimed by the accused has been rightly discarded.
10. The academic distinction between "murder" and "culpable homicide not amounting to
murder" has always vexed the courts. The confusion is caused, if courts losing sight of
the true scope and meaning of the terms used by the legislature in these sections, allow
themselves to be drawn into minute abstractions. The safest way of approach to the
interpretation and application of these provisions seems to be to keep in focus the
keywords used in the various clauses of Sections 299 and 300.
11. In Virsa Singh v. State of Punjab (AIR 1958 SC 465) Vivian Bose, J. speaking for the
Court, explained the meaning and scope of clause (3). It was observed that the
prosecution must prove the following facts before it can bring a case under Section 300
"thirdly". First, it must establish quite objectively, that a bodily injury is present;
secondly, the nature of the injury must be proved. These are purely objective
investigations. Thirdly, it must be proved that there was an intention to inflict that
particular injury, that is to say, that it was not accidental or unintentional or that some
other kind of injury was intended. Once these three elements are proved to be present, the
enquiry proceeds further, and fourthly, it must be proved that the injury of the type just
described made up of the three elements set out above was sufficient to cause death in the
ordinary course of nature. This part of the enquiry is purely objective and inferential and
has nothing to do with the intention of the offender.
12. The ingredients of clause "thirdly" of Section 300, IPC were brought out by the
illustrious Judge in his terse language as follows :
"12. To put it shortly, the prosecution must prove the following facts before it can bring a
case under Section 300 'thirdly';
First, it must establish, quite objectively, that a bodily injury is present;
Secondly, the nature of the injury must be proved; These are purely objective
investigations.
Thirdly, it must be proved that there was an intention to inflict that particular bodily
injury, that is to say, that it was not accidental or unintentional, or that some other kind of
injury was intended.
Once these three elements are proved to be present, the enquiry proceeds further and,
Fourthly, it must be proved that the injury of the type just described made up of the three
elements set out above is sufficient to cause death in the ordinary course of nature. This
part of the enquiry is purely objective and inferential and has nothing to do with the
intention of the offender."
13. The learned Judge explained the third ingredient in the following words (at page
468) :
"The question is not whether the prisoner intended to inflict a serious injury or a trivial
one but whether he intended to inflict the injury that is proved to be present. If he can
show that he did not, or if the totality of the circumstances justify such an inference,
@page-SC2882
then, of course, the intent that the section requires is not proved. But if there is nothing
beyond the injury and the fact that the appellant inflicted it, the only possible inference is
that he intended to inflict it. Whether he knew of its seriousness, or intended serious
consequences, is neither here nor there. The question, so far as the intention is concerned,
is not whether he intended to kill, or to inflict an injury of a particular degree of
seriousness, but whether he intended to inflict the injury in question; and once the
existence of the injury is proved the intention to cause it will be presumed unless the
evidence or the circumstances warrant an opposite conclusion."
14

. These observations of Vivian Bose, J. have become locus classicus. The test laid down
by Virsa Singh case (supra) for the applicability of clause "thirdly" is now ingrained in
our legal system and has become part of the rule of law. Under clause thirdly of Section
300, IPC, culpable homicide is murder, if both the following conditions are satisfied i.e.
(a) that the act which causes death is done with the intention of causing death or is done
with the intention of causing a bodily injury; and (b) that the injury intended to be
inflicted is sufficient in the ordinary course of nature to cause death. It must be proved
that there was an intention to inflict that particular bodily injury which, in the ordinary
course of nature, was sufficient to cause death viz. that the injury found to be present was
the injury that was intended to be inflicted. AIR 1958 SC 465
15

. Thus, according to the rule laid down in Virsa Singh case (supra) even if the intention of
the accused was limited to the infliction of a bodily injury sufficient to cause death in the
ordinary course of nature, and did not extend to the intention of causing death, the
offence would be murder. Illustration (c) appended to Section 300 clearly brings out this
point. AIR 1958 SC 465

16. Clause (c) of Section 299 and clause (4) of Section 300 both require knowledge of the
probability of the act causing death. It is not necessary for the purpose of this case to
dilate much on the distinction between these corresponding clauses. It will be sufficient
to say that clause (4) of Section 300 would be applicable where the knowledge of the
offender as to the probability of death of a person or persons in general as distinguished
from a particular person or persons - being caused from his imminently dangerous act,
approximates to a practical certainty. Such knowledge on the pan of the offender must be
of the highest degree of probability, the act having been committed by the offender
without any excuse for incurring the risk of causing death or such injury as aforesaid.
17. The above are only broad guidelines and not cast-iron imperatives. In most cases,
their observance will facilitate the task of the court. But sometimes the facts are so
intertwined and the second and the third stages so telescoped into each, that it may pot be
convenient to give a separate and clear cut treatment to the matters involved in the second
and third stages.
18

. The position was illuminatingly highlighted by this Court in State of A.P. v. Rayavarapu
Punnayya (1976 (4) SCC 382), Abdul Waheed Khan alias Waheed and Ors. v. State of
A.P. (2002 (7) SCC 175) and Raj Pal and Ors. v. State of Haryana (2006 (8) SCC 678).
AIR 1977 SC 45
2002 AIR SCW 3463

19. It is to be noted that Heeralal has stated that the appellant had assaulted both legs,
thigh and hands of the deceased with pharsa. He chopped both the legs of the deceased
who died instantaneously. Beerbali's (P.W.5) evidence was also to similar effect.
20. It is nobody's case that the appellant had assaulted any of the accused or that he had
participated in the quarrel.
21. Learned counsel for the appellant submitted that since he was present at the place of
occurrence, it is but natural on the part of the accused appellant to assume that he may
have assaulted him. Mere presence of a person at the place of quarrel is not sufficient to
show that he had any intention to cause any injury. In the instant case, even that intention
is not manifest and in any event, any intention to do an act cannot be counteracted by
actual assault. Even deceased was not armed.
22. In the instant case, in a brutal manner the appellant had chopped both legs of the
deceased and with the weapon caused other injuries on the body of the deceased. Above
being the position, there is no scope for interference in this appeal. The appeal is
dismissed.
Appeal dismissed. .
@page-SC2883
AIR 2008 SUPREME COURT 2883 "Krishi Utpadan Mandi Samiti, Ghaziabad v. M/s.
Metal Craft"
(From : 2001 All LJ 1947)
Coram : 3 Dr. A. PASAYAT, P. SATHASIVAM AND AFTAB ALAM, JJ.
Civil Appeal No. 8690 of 2001, D/- 7 -7 -2008.
Krishi Utpadan Mandi Samiti, Ghaziabad and Anr. v. M/s. Metal Craft and Ors.
U.P. Krishi Utpadan Mandi Adhiniyam (25 of 1964), S.17(iii)(b) - AGRICULTURAL
PRODUCE - SALE - Market fee - Levy of - Firm having its office in market - Purchased
rice from outside State of U.P. and exported the same in terms of agreement - Rice was
never brought or was in existence within market area - Thus, there was no transaction of
sale within market area - No market fee can be levied in respect of said transaction.
(Paras 9, 13, 16)
Cases Referred : Chronological Paras
AIR 1980 SC 1124 : 1980 All LJ 490 (Ref.) 10
AIR 1968 SC 741 (Ref.) 11
(1834) 2 Cr and M 530 : 149 ER 871 11
AIR 1961 SC 1214 (Ref.) 12
(1910) ILR 38 Cal 127 12
Pradeep Misra, for Appellants; T.N. Singh, for Respondents.
Judgement
Dr. ARIJIT PASAYAT, J. :- Challenge in this appeal is to the judgment of a Division
Bench of the Allahabad High Court holding that the appellant was not entitled to levy
market fee under Section 17(iii) (b) of the U.P. Krishi Utpadan Mandi Adhiniyam, 1964
(in short the 'Adhiniyam') if the agricultural produce is neither brought nor taken out of
the market place, and deciding in favour of respondent No. 1.
2. Background facts in a nutshell are as follows :
Respondent is a registered partnership firm having its business premises and office at 14,
Navyug Market, Ghaziabad, and it carried on the business of sale and purchase of iron
and steel and also export of rice. It wanted to purchase broken rice from the rice millers
of U.P. for the purpose of export to foreign countries and accordingly, made an
application on July 31, 1997, to Krishi Utpadan Mandi Samiti, Ghaziabad, for grant of a
licence. It was also stated in the application that the respondent had exported rice in
November, 1996 by purchasing it from places outside U.P. Appellant No. 1 asked the
respondent No. 1 to deposit the licence fees for the years 1995-96, 1996-97 and 1997-98,
which was done as per the demand. Thereafter, the appellant No. 1 sent a demand notice
to the respondent No. 1 on October 12, 1997, demanding market fee at the rate of 2 per
cent amounting to Rs. 12,94,860.00. The respondent No. 1 sent a reply on October 18,
1997, stating that it had never purchased any rice from inside the State of U.P. nor any
transaction of sale or purchase of rice was carried out within the State. It was accordingly
requested that the demand notice/order dated October 12, 1997, be rescinded. The
appellant No.1, however, initiated proceeding for recovery of the amount in question and
issued a citation dated December 6, 1997. The respondent No. 1 thereafter, filed C.M.
Writ Petition No, 43329 of 1997 in the High Court which was disposed of on December
17, 1997, with a direction to appellant No. 1 to decide the respondent No. 1's
representation within a month and the recovery proceeding were suspended for six
months. The respondent No.1 appeared before appellant No.1 on the date fixed, namely
January 14, 1998, along with the relevant records and submitted that the rice had been
purchased from places outside the State of U.P. and had been sent directly to the ports for
being exported to South Africa and as such, it was not liable to pay any market fee. The
appellant passed an order on January 25, 1998, holding that the transaction of sale of the
rice exported by the respondent No.1 firm took place within the market area of
Ghaziabad, and, accordingly, the market fee imposed by the order dated October 12, 1997
was valid and proper. Feeling aggrieved, the respondent No. 1 preferred a revision under
Section 32 of the Act before the Rajya Krishi Utpadan Mandi Parishad, Lucknow
(appellant No.2) which was dismissed by order dated March 9, 1998. The writ petition
under Article 226 of the Constitution of India, 1950 (in short the 'Constitution') was filed
for quashing the orders dated October 12, 1997 passed by appellant No. 1 and the order
dated March 9, 1998 passed by appellant No.2. The learned Single Judge, who heard the
petition, was of the opinion that the controversy raised involved a substantial question of
law of general importance and made a reference to larger Bench. That is how the matter
came before the Division Bench.
The case of the respondent No. 1 was that
@page-SC2884
the rice was exported by it because certain dealers in South Africa wanted to buy rice
from India. The respondent No. 1 quoted the rates and entered into negotiations. After the
deal was settled, the rice was purchased from rice millers in Haryana, Punjab, Madhya
Pradesh from where it was directly dispatched to the ports of Mumbai and Kandla and
clearing and forwarding agents of the respondent No. 1 loaded the same on the ship. After
the goods had been loaded a Bill of Lading was prepared and signed by the Master of the
ship in the capacity of carrier acknowledging the receipt of the goods. The Bill of Lading
was given to the clearing and forwarding agents and on receipt of the Bill of Lading by
the buyer through the respondent No.1's bankers, the rice were retired by the buyer in
South Africa. The sale price of the rice was received by the respondent No. 1 through its
banker viz. Oriental Bank of Commerce at Delhi. It is the specific case of the respondent
No. 1 was that the entire quantity of the exported rice was purchased from places outside
the State of U.P. and was directly sent to the ports without it ever coming within the
market area of Ghaziabad or in the State of U.P. It was also asserted that the sale was
effected only at the ports when the goods were loaded in the ship and the Bill of Lading
was handed over to the respondent No.1's clearing and forwarding agents.
The case of the present appellants was that the business establishment of the respondent
is at 14, Navyug Market Ghaziabad and the entire transaction was done from the said
place. The purchase order was received and accepted by it at Ghaziabad and the sale price
was also received there and therefore the transaction of sale took place in Ghaziabad. It
was also pleaded that the transport of the goods and how it was actually exported was
wholly irrelevant for ascertaining where the transaction of sale took place.
The High court did not accept the said stand and allowed the writ petition filed.
3. In support of the appeal, learned counsel for the appellants submitted that since the
transaction took place within the jurisdiction of the market area, the levy was justified
and the High Court was wrong in its view.
4. Learned counsel for the respondent No. 1 on the other hand supported the judgment of
the High Court.
5. It is to be noted that before the High Court the learned counsel for the appellant No. 1
had fairly admitted that rice exported by the appellant was never brought within the
market area of Mandi Parishad, Ghaziabad within the State of U.P.
6. Section 17(iii)(b) is the charging section which reads as follows:
"17. Powers of the Committee - A Committee shall, for the purposes of this Act, have the
power to -
(i)....................
(ii)..................
(iii) levy and collect :
(a) such fees as may be prescribed for the issue or renewal of licences, and
(b) market fee, which shall be payable on transactions of sale of specified agricultural
produce in the market area at such rates being not less than one percentum and not more
than two percentum of the price of the agricultural produce so sold as the State
Government may specify by notification, and such fee shall be realised in the following
manner -
(1) if the produce is sold through a commission agent may realise the market fee from the
purchaser and shall be liable to pay the same to the Committee;
(2) if the produce is purchased directly by a trader from a producer the trader shall be
liable to pay the market fee to the Committee;
(3) if the produce is purchased by a trader for another trader, the trader selling the
produce may realise it from the purchaser and shall be liable to pay the market fee to the
Committee : and
(4) in any other case of sale of such produce, the purchaser shall be liable to pay the
market fee to the Committees :
Provided that no market fee shall be levied or collected on the retail sale of any specified
agricultural produce where such sale is made to the consumer for his domestic
consumption only."
7. The object for which the Act was enacted is as follows :
"(i) to reduce the multiple trade charges, levies and exactions charged at present from the
produce-sellers;
(ii) to provide for the verification of accurate
@page-SC2885
weight and scales and see that the producer-seller is not denied his legitimate due;
(iii) to establish market committees in which the agricultural producer will have his due
representation;
(iv) to ensure that the agricultural producer has his say in the utilization of market funds
for the improvement of the market as a whole;
(v) to provide for fair settlement of disputes relating to the sale of agricultural produce.
(vi) to provide amenities to the producer-seller in the market;
(vii) to arrange for better storage facilities;
(vii) to stop inequitable and unauthorized charges and levies from the producer-seller;
and
(viii) to make adequate arrangements for market intelligence with a view to posting the
agricultural producer with the latest position in respect of the markets dealing with his
produce."
As the prefatory note and preamble clearly show the object of the Act is to save the
agricultural producer from innumerable charges, levies etc. and to enable them to have a
say in the proper utilization of amounts paid by him to reduce multiple charges levies,
exactions charged from the producer and seller and generally to help the agricultural
producer to sell his produce to his best advantage.
8. At the end of the Section there is an explanation which reads as follows :
"Explanation - For the purpose of clause (iii), unless the contrary is proved, any specified
agricultural produce taken out or proposed to be taken out of market area by or on behalf
of a licensed trader shall be presumed to have been sold within such area and in such case
the price of such produce presumed to be sold shall be deemed to be such reasonable
price as may be ascertained in the manner prescribed."
In exercise of the powers conferred by Section 40, Rules have been framed, which are
known as U.P. Krishi Utpadan Mandi Niyamavali, 1965 (hereinafter referred to as the
'Niyamavali') and Rules 66 and 68 reads as follows:
"(66) Market Fee (Section 17 (iii) - The Market Committee shall levy and collect market
fee in the Market Area in accordance with the provisions of sub-clause (b) of clause (iii)
of Section 17 of the Act at such rate as may be specified in the bye-laws :
Provided that no market fee shall be levied and charged prior to the date on which
provisions, Section 10 of the Act are enforced
Provided further that when the specified agricultural produce is presumed to have been
sold in accordance with the explanation given under clause (viii) of Section 17 of the
Uttar Pradesh Krishi Utpadan Mandi Adhiniyam, 1964 the price of such produce shall be
the price prevailed for that type of produce in that market just on the previous working
day.
(68) No market fee shall be levied more than once on any consignment of the specified
agricultural produce brought for sale in the Market Yard if the market fee has already
been paid on it in any Market Yard of the same Market Area and in respect of which a
declaration has been made and a certificate has been given by the seller in Form No. V."
9. A plain reading of Section 17(iii)(b) of the Act shows that the Committee is
empowered to levy and collect market fee which shall be payable on transaction of sale
of agricultural produce in the market area. The words "specified agricultural produce in
the market area" have great relevance. The manner of realization of market fee has been
enumerated in sub-clauses (1), (2), (3) and (4) of Section 17(iii)(b). Reference is to
"produce". This apparently shows that physical presence of the agricultural produce
within the market area is necessary for levy of market fee. The explanation to Section 17
(iii)(b) appended at the end of the Section lays down that unless the contrary is proved
any specified agricultural produce taken out or proposed to be taken out of a market area
by or on behalf of the licenced traders shall be presumed to have been sold within such
area. The explanation has application only if the agricultural produce is physically present
within the market area. The explanation becomes redundant if the stand of the appellant
that Section 17 (iii)(b) is applicable even in cases where agricultural produce is neither
physically brought nor is in existence within the market area.
10
. In Ram Chander Kailash Kumar and Co. v. State of U.P. (AIR 1980 SC 1124) it was
inter Para 29 of AIR

@page-SC2886
alia observed as follows :
"This point urged on behalf of the appellants is well founded and must be accepted as
correct. On the very wordings of Clause (b) of Section 17(iii) market fee is payable on
transactions of sale of specified agricultural produce in the market area and if no
transaction of sale takes place in a particular market area no fee can be charged by the
Market Committee of that area. If goods are merely brought in any market area and are
dispatched outside it without any transaction of sale taking place therein, then no market
fee can be charged. If the bringing of the goods in a particular market area and their
despatch therefrom are as a result of transactions of purchase and sale taking place
outside the market area, it is plain that no fee can be levied."
11

. In P.S.N.S. Ambalavana Chettiar and Company Ltd. v. Express Newspapers Ltd. (AIR
1968 SC 741) it was observed as follows : Para 11 of AIR

"Section 18 of the Sale of Goods Act provides that where there is a contract for the sale
of unascertained goods no property in the goods is transferred to the buyer unless and
until the goods are ascertained. It is a condition precedent to the passing of property
under a contract of sale that the goods are ascertained. The condition is not fulfilled
where there is a contract for sale of a portion of a specified larger stock. Till the portion is
identified and appropriated to the contract, no property passes to the buyer. In Gillett v.
Hill [(1834) 2 CandM 535 : 149 ER 871, 873], Bayley, B. said :
"Where there is a bargain for a certain quantify ex a greater quantity, and there is a power
of selection in the vendor to deliver which he thinks fit, then the right to them does not
pass to the vendee until the vendor has made his selection, and trover is not maintainable
before that is done. If I agree to deliver a certain quantity of oil as ten out of eighteen
tons, no one can say which part of the whole quantity I have agreed to deliver until a
selection is made. There is no individuality until it has been divided."
12

. Similarly, in Jute and Gunny Brokers Ltd. and Ors. v. The Union of India and Ors. etc.
(AIR 1961 SC 1214) it was held as follows : Para 14 of AIR

"The contention on behalf of the Union of India is that property in the goods cannot pass
in law to the holders of the pucca delivery orders till the goods are actually appropriated
to the particular order; therefore, as in this case it is not in dispute that no goods were
actually appropriated towards the pucca delivery orders concerned, the property in the
goods did not pass to the holders thereof but was still in the mills. Reliance in this
connection is placed on S. 18 of the Indian Sale of Goods Act, No III of 1930. That
section lays down that "where there is a contract for the sale of unascertained goods, no
property in the goods is transferred to the buyer unless and until the goods are
ascertained." In the present case, as we have already said it is not in dispute that the
goods covered by the pucca delivery orders are not ascertained at the time such orders are
issued and ascertainment takes place in the shape of appropriation when the goods are
actually delivered in compliance therewith. Therefore, till appropriation takes place and
goods are actually delivered, they are not ascertained. The contract therefore represented
by the pucca delivery orders is a contract for the sale of unascertained goods and no
property in the goods is transferred to the buyer in view of S. 18 of the Indian Sale of
Goods Act till the goods are ascertained by appropriation, which in this case takes place
at the time only of actual delivery. The appeal court in our opinion was therefore right in
holding that the property in the goods included in the pucca delivery orders did not pass
to the holders thereof in view of S. 18 of the Sale of Goods Act in spite of the decision in
the case of the Anglo-India Jute Mills Co. [(1910) ILR 38 Cal 127]. What that case
decided was that in a suit between a holder of a pucca delivery order - be he the first
holder or a subsequent holder who has purchased the pucca delivery order in the market -
and the mills, there will be an estoppel and the mill will be estopped from denying that
cash had been paid for the goods to which the delivery order related and that they held
the goods for the holder of the pucca delivery order. That case therefore merely lays
down the rule of estoppel as between the mill and the holder of the pucca delivery order
and in a suit between then the mill will be estopped from denying the title of the holder of
pucca delivery orders; but that does not mean that in law the title passed to the holder of
the pucca delivery order as soon as it was issued even though
@page-SC2887
it is not disputed that there was no ascertainment of goods at that time and that the
ascertainment only takes place when the goods are appropriated to the pucca delivery
orders at the time of actual delivery. The appeal court was in our opinion right in holding
that the effect of the decision in the case of Anglo-India Jute Mills Co. [(1910) ILR 38
Cal 127], was not that the property in the goods passed by estoppel and that that case only
decided that as between the seller and the holder of the pucca delivery order, the seller
will not be heard to say that there was no title in the holder of the delivery order. That
case was not dealing with the question of title at all as was made clear by Jenkins C.J. but
was merely concerned with estoppel. In the present case the question whether the
Government of India will be estopped is a matter which we shall consider later; but so far
as the question of title is concerned there can be no doubt in view of S. 18 of the Sale of
Goods Act that title in these cases had not passed to the holders of the pucca delivery
orders on September 30, 1946, for the goods were not ascertained till then, whatever may
be the position of the holders of the pucca delivery orders in a suit between them and the
mills to enforce them."
13. Under Section 17(iii)(b) the measure of levy of the fee is on the price of the goods
sold. It obviously means that there must be a complete transaction of sale or a concluded
sale. If there is only an agreement and the agreement fails, the remedy for the aggrieved
party is to suit for damages. Obviously, no fee can be charged on damages. The action for
levy of fee can arise only on a concluded sale and as the sale has not taken place within
the market area of Ghaziabad, no mandi fee can be levied.
14. The stand of the appellant is that the market fee is levied on "transaction of sale" and
not on "sale" only and, therefore, what is to be seen is where the transaction took place
and not the situs of the sale. If this argument is accepted then even an agreement to sale
without the presence or existence of the agricultural produce will come within the ambit
of the charging provision. It would also mean that if the agreement takes place outside the
boundaries of State of Uttar Pradesh, the provisions would still become applicable.
15. It is to be noted that the challenge in the writ petition was essentially to the revisional
order passed by the revisional authority under the Act. The revision was filed against the
order passed by the Mandi Samiti in respect of rice exported. A bare perusal of the
revisional order shows that the Samiti as well as the revisional authority proceeded on the
basis that since the contract for goods was entered into Ghaziabad and then goods were
sent through transport from Punjab, Haryana and Madhya Pradesh directly through ports,
therefore, the market fee was leviable.
16. The High Court rightly noted that the admitted position was that the rice was never
brought or was in existence within the market area, Mandi Samiti, Ghaziabad or for that
matter within the State of Uttar Pradesh. The High Court recorded a categorical finding
that the sale took place only when the rice was loaded on the sea at the port in terms of
the agreement. That being so, there was no transaction of sale within the market area of
the Mandi Samiti, Ghaziabad. Therefore, the High Court rightly held that the Mandi
Samiti was not entitled to levy any market fee. There is no merit in the appeal, which is
accordingly dismissed.
Appeal dismissed. .
AIR 2008 SUPREME COURT 2887 "M. C. Agrawal HUF v. M/s. Sahara India"
(From : Delhi)*
Coram : 2 TARUN CHATTERJEE AND HARJIT SINGH BEDI, JJ.
Civil Appeal No. 3007 of 2003 (arising out of SLP (C) No. 14462 of 2007), D/- 28 -4
-2008.
M.C. Agrawal HUF v. M/s. Sahara India and Ors.
Civil P.C. (5 of 1908), O.6, R.17 - AMENDMENT - PLEADINGS - EVICTION -
INJUNCTION - Amendment of plaint - Suit for eviction, mesne profits and for
mandatory injunction - Prayer to award mesne profits equivalent to rent payable in
respect of premises sought to be added by way of amendment - Amendment does change
nature of suit - It is necessary to adjudicate real controversies between parties as to
quantum of mesne profits - Refusal to grant permission on ground that claim for mesne
profits had to be dehors contract - Improper.
C.M. (M) No. 1025 of 2007, D/-02-08-2007 (Del.), Reversed. (Paras 4, 5, 6)
@page-SC2888

P. K. Aggarwal, Sanjay Kapur, Ms. Shubhra Kapur, Noor Alam and Ms. Arti Singh, for
Appellant; Aarohi Bhalla, Sunil Kumar Singh and Ms. Sujata Kurdukar, for Respondents.
* CM (M) No. 1025 of 2007, D/- 2-8-2007 (Del).
Judgement
1. ORDER :-By an order dated 27th of August, 2007, we issued notice in the present
special leave petition and granted interim stay of all further proceedings in Suit No. M-
73/2007 pending before the Additional District Judge, Delhi. In compliance with our
notice, the respondent has entered appearance. Counter affidavit has already been filed.
Learned counsel appearing on behalf of the petitioner submitted that no rejoinder
affidavit is required to be filed and the matter can be disposed of. Such being the stand
taken by the learned counsel for the parties, we grant leave and take up the hearing of the
appeal.
2. This appeal relates to rejection of an application for amendment of plaint filed at the
instance of the plaintiff/appellant in a suit for eviction, mesne profit and for mandatory
injunction. The suit was, however, decreed ex-parte and an application under Order 9
Rule 13 of the Code of Civil Procedure for setting aside the ex parte decree was rejected
by the trial Court as well as by the High Court, but the orders of the trial Court as well as
of the High Court were set aside by this Court and the suit was restored to file. When the
suit was restored to file, the plaintiff/appellant filed an application for amendment for
deletion of the prayer for delivery of air tickets and for consolidating the same with the
prayer of mesne profits as under :
"Award mesne profits equivalent to the rent payable in respect of the premises and the
value of the air tickets payable by defendant as determined by this Court."
3. Consequential amendment was also sought in paragraph 12 of the plaint. The trial
court declined the amendment solely on the ground that the relief for the delivery of air
tickets was earlier declined in the ex-parte decree and the petitioner, therefore, could not
ask indirectly what was declined to them directly. Feeling aggrieved, the appellant had
filed a revision application before the High Court which also affirmed the order of the
trial Court. The High Court by the impugned order while holding that the amendment
could not have been denied on the basis of the ex-parte decree which had already been set
aside, refused the prayer for amendment on the ground that contractual use and
occupation charges would not be necessary for the purpose of determination of the mesne
profits. Accordingly, the order of the trial Court was affirmed and the application for
amendment of the plaint was rejected.
4. Having heard the learned counsel for the parties and after going through the plaint as
well as the application for amendment of the plaint and the objections filed by the
respondent, we do not find any ground to refuse the prayer of the appellant to amend the
plaint in the manner they have prayed for. While rejecting the application for amendment
of the plaint, it was held by the High Court that the amendment was not necessary nor
germane to the controversy between the parties for the reason that claim for mesne
profits/damages had to be dehors the contract between the parties. It was further observed
that measure of mesne profits/damages would be the rental fetched by similar situated
properties in the vicinity over the period mesne profits was being claimed. Upon, these
observations, the prayer for amendment of the plaint was rejected. In our view, the
amendment of the plaint sought for by the plaintiff/appellant was necessary in deciding
the real controversy between the parties. It is always open by way of an amendment to
amalgamate the two reliefs in one suit. That apart, at the time of allowing or refusing to
amend the plaint, it is not open for the Court to decide the merits of the suit which can
only be gone into and decided by it at the time of decision of the suit. The
plaintiff/appellant is entitled to plead and prove the amount of rent and the equivalent
amount of benefit received out of the letting out of the property to show the contractual
rent of use and occupation charges. On the basis of the lease agreement, it is clear that the
mesne profit/ damages cannot be awarded less than the contractual rate of use and
occupation charges. Therefore, in the event of allowing the amendment of the plaint in
the aforesaid circumstances, the nature of the suit shall not be changed. Therefore, in our
view, there was no reason as to why the prayer for amendment of the plaint should not be
allowed. In our view also, the prayer for amendment of the plaint was necessary in order
to adjudicate the real controversies between the parties, i.e. with respect to the
@page-SC2889
quantum of the mesne profits/damages.
5. Accordingly, we do not find any reason why the prayer for amendment of the plaint as
prayed for by the appellant could be refused.
6. For the reasons aforesaid, the impugned orders are set aside and the application for
amendment of the plaint is allowed. The appellant is directed to file an amended plaint
within a period of two weeks from the date of supply of a copy of this order and the
defendant/respondent shall be entitled to file additional written statement within a
fortnight from the date of filing an amended plaint by the appellant.
7. In view of the discussions made herein above, the impugned orders are set aside and
the appeal is allowed to the extent indicated above. There will be no order as to costs.
Appeal allowed. .
AIR 2008 SUPREME COURT 2889 "Sidhartha Vashisht v. State (NCT of Delhi)"
(From : 2007 (1) Cur Cri R 44 (Delhi))
Coram : 2 C. K. THAKKER AND D. K. JAIN, JJ.
Criminal Misc. Petn. No. 1775 of 2007 in Cri. Appeal No. 179 of 2007, D/- 12 -5 -2008.
Sidhartha Vashisht alias Manu Sharma v. State (NCT of Delhi).
Criminal P.C. (2 of 1974), S.389 - SENTENCE SUSPENSION - BAIL - APPEAL - LIFE
IMPRISONMENT - Suspension of sentence and release on bail - Powers of Court -
Murder trial - Applicant though acquitted by trial Court but convicted by High Court and
sentenced for life imprisonment - Appeal, against - Already on board and awaiting final
hearing - Appeal thus, likely to be heard within 'measurable distance of time' -
Application for suspension of sentence and release on bail dismissed keeping in view
seriousness of offence, manner in which it was said to have been committed and gravity
of offence.
In considering the prayer for bail in a case involving a serious offence like murder
punishable under Section 302, IPC, the Court should consider all the relevant factors like
the nature of accusation made against the accused, the manner in which the crime is
alleged to have been committed, the gravity of the offence, the desirability of releasing
the accused on bail after he has been convicted for committing serious offence of murder,
etc. Normal practice in such cases
is not to suspend the sentence and it is only in exceptional cases that the benefit of
suspension of sentence can be granted. In such a case the mere fact that during the period
of trial, the accused was on bail and there was no misuse of liberty, does not per se
warrant suspension of execution of sentence and grant of bail. What really necessary is to
consider whether reasons exist to suspend execution of the sentence and grant of bail.
(Paras 32, 34)
Though the trial Court had acquitted the applicant-accused for the offences with which he
was charged, the High Court reversed the order of acquittal and convicted him under
Section 302, IPC and ordered him to undergo rigorous imprisonment for life and being
aggrieved by the said order, he had filed an appeal which had been admitted, and was
already on board and awaited final hearing. Hence, within 'measurable distance of time'
the appeal was likely to be heard. Consequently keeping in view the seriousness of
offence, the manner in which the crime was said to have been committed and the gravity
of offence, it was held that no case has been made out by the applicant-appellant for
suspension of sentence and grant of bail and the application deserved to be dismissed.
(Para 35)
Cases Referred : Chronological Paras
2008 AIR SCW 2475 : AIR 2008 SC 1793 : 2008 (3) AIR Bom R 451 (Ref.) 31
2004 AIR SCW 4266 : AIR 2004 SC 3936 : 2004 Cri LJ 3840 (Ref.) 31, 33
2004 AIR SCW 7409 : AIR 2005 SC 1481 : 2005 All LJ 1252 (Ref.) 31
(2002) 9 SCC 364 : 2002 Supp (1) JT (SC) 60 (Ref.) 31, 32
2000 AIR SCW 1783 : AIR 2000 SC 3404 : 2000 Cri LJ 2452 (Ref.) 27
2000 AIR SCW 3602 : AIR 2000 SC 3564 : 2000 Cri LJ 4992 (Ref.) 31
(2000) 6 SCC 461 (Ref.) 31
AIR 1978 SC 527 : 1978 Cri LJ 651 (Ref.) 27
AIR 1977 SC 2147 : 1977 Cri LJ 1746 (Ref.) 27, 29
AIR 1931 All 356 : 1931 (32) Cri LJ 1271 (Ref.) 22
Ram Jethmalani, Sr. Advocate, P.H. Parekh, Ms. Lata Krishnamurthy, E.R. Kumar, Lalit
Chauhan, Ajay Jha, Ms.Saurabh Ajay Gupta, Ms. Mary Mizty, Ms.
@page-SC2890
Rajdeep Banerjee, Ms. Joyeeta Banerjee, Ms. Bansuri Swaraj, Ms. Rukhmini Bobde,
Somandri Goud (for M/s. P. H. Parekh and Co.) with him for the Appellant; Gopal
Subramanium ASG, Ms.Mukta Gupta, Nikhil Nayyar, Ankit Singhal, T.V.S. Raghavendra
and Vibha Garg, with him for the Respondent; Ms. Mamta Kalra, In person for
Intervening Party.
Judgement
1. C. K. THAKKER, J. :-The present application is filed by the appellant-accused under
Section 389 of the Code of Criminal Procedure, 1973 (hereinafter referred to as 'the
Code') for suspension of sentence pending appeal in this Court and to release him on bail.
2. Since an appeal against an order of conviction and sentence recorded by the High
Court of Delhi is admitted by this Court and awaits final hearing, we will not enter into
larger questions and deal with the present application for suspension of sentence and bail.
3. Shortly stated, the case of the prosecution was that on April 29-30, 1999, a party was
organized at Tamarind Cafe' inside Qutub Colonnade. It was a private party where certain
persons were invited and liquor was served. Jessica Lal (since deceased) and one Shyan
Munshi were in charge of the bar. It was the allegation of the prosecution that appellant
Sidhartha Vashisht alias Manu Sharma along with his friends came there and asked for
liquor. Jessica Lal and Shyan Munshi did not oblige him by providing liquor since the bar
was closed. According to the prosecution, the appellant got enraged on refusal to serve
liquor, took out his 22 pistol and fired two rounds, first into the ceiling and the second at
Jessica Lal. Jessica Lal fell down as a result of the shot which proved fatal and she died.
According to the assertion of the prosecution, several persons witnessed the incident.
Beena Ramani who was present, stopped the appellant and questioned him as to why he
had shot Jessica Lal. She also demanded weapon from the accused but the accused did
not handover pistol and fled away.
4. FIR was lodged, a case was registered and investigation was carried out. At the trial,
more than 100 witnesses had been examined. The trial Court acquitted the accused
holding that it was not proved by the prosecution that the accused had committed the
offence with which he, along with other accused, was charged.
5. The State preferred an appeal against an order of acquittal recorded by the trial Court.
The High Court of Delhi held that the trial Court was wrong in acquitting the accused and
the prosecution was successful in proving the guilt against the appellant (as well as two
other accused) and accordingly recorded conviction inter alia for an offence punishable
under Section 302, Indian Penal Code (IPC) and imposed sentence of imprisonment for
life.
6. The High Court observed that it has "no hesitation in holding" that the appellant was
guilty of an offence punishable under Section 302 read with Sections 201 and 120B, IPC
and also under Section 27 of the Arms Act, 1959 for having committed murder of Jessica
Lal on April 29-30, 1999 at 'Tamarind Cafe' and ordered him to undergo rigorous
imprisonment for life and also imposed sentence for other offences.
7. With regard to the other two accused, however, the Court held that they were guilty for
committing an offence punishable under Sections 201 and 120B, IPC.
8. The appellant-applicant approached this Court by instituting an appeal under Section 2
(a) of the Supreme Court (Enlargement of Criminal Appellate Jurisdiction) Act, 1970 as
also under Section 379 of the Code. The appeal was placed for admission. On March 7,
2007, the appeal was admitted and notice was issued on application for bail. Counsel
appeared on behalf of the respondent and accepted the notice. It was ordered to be listed
in the first week of April, 2007, meanwhile, counter affidavit, if any, was to be filed.
9. On April 2, 2007 when the matter appeared on Board, the Court passed orders of bail
in respect of other accused, but in the instant case (Crl. M.P. No. 1775 of 2007), the Court
fixed final hearing of the matter. It, however, appears that the appeal could not be heard.
On January 24, 2008, the Court ordered listing of appeals along with bail applications
"before any other appropriate Bench" on 12th February, 2008. The matter was thus placed
before this Bench.
10. In view of several other matters, however, the appeal could not be taken up for
hearing. Mr. Ram Jethmalani, learned senior advocate, appearing for the appellant-
accused, no doubt, requested the Court to take up the matter out of turn. He alternatively
@page-SC2891
tively submitted that if the appeal is not heard, the application for bail may be heard as
according to him, he did not press for bail earlier when the appeal was placed for,
admission hearing and was admitted since the Court had fixed final hearing of main
matter. According to him, the appellant was in jail and if the appeal will not be heard for a
considerable long time, serious prejudice will be caused to the accused. On the facts and
in the circumstances, therefore, we directed the Registry to place the application for
suspension of sentence and grant of bail on Board so that an appropriate order may be
passed on the, prayer of the applicant-appellant-accused.
11. We have heard learned counsel for the parties.
12. The learned counsel for the applicant submitted that no case has been made out by the
prosecution against the appellant-accused. The trial Court, after considering the evidence
of the prosecution witnesses in its entirety, recorded an order of acquittal in favour of the
accused. He submitted that the trial Court held that P.W. 1 Deepak Bhojwani and P.W. 30
Shravan Kumar had been 'planted' by the prosecution. P.W. 2 Shyan Munshi had
expressly stated that shots were fired by two persons and appellant-accused was not one
of them. Neither P.W. 1 Deepak Bhojwani nor P.W. 2 Shyan Munshi, nor P.W. 3 Shiv
Dass Yadav, nor P.W. 4 Karan Rajput were eye-witnesses. For rejecting ocular evidence
of P.W. 6 Malini Ramani and P.W. 20 Beena Ramani, cogent and convincing reasons have
been recorded by the trial Court. It was not proved that Tata Safari was in possession of
the appellant-accused, nor was there anything to show that he used the said vehicle on
29th April, 1999. Report of ballistic expert does not support prosecution and on that
ground also, the trial Court was right in passing the order of acquittal.
13. According to the learned counsel, Beena Ramani P.W. 20, was not an eye-witness. A
statement to that effect was made by the Public Prosecutor at the trial in the Sessions
Court. It was also clear that a false excise case had been registered against the said
witness and she was pressurised to depose in favour of prosecution and as soon as her
evidence was over, she was obliged by compounding the offence on imposing fine which
went to show that it was the systematic effort of the prosecution to involve the appellant-
accused who was totally innocent. The counsel also submitted that photograph of the
accused was collected by the Police during investigation and was shown to the
prosecution witnesses and identification of the accused was meaningless. Media had
played active role and even before the conclusion of the trial, they had virtually described
the applicant not as an 'accused' but as a 'convict' or an 'offender.' According to the
learned counsel, the trial Court dispassionately and objectively considered the evidence in
its proper perspective without being influenced by extraneous factors and granted benefit
of doubt to the accused. The High Court was 'wholly' wrong in reversing the finding of
the trial Court and in convicting the applicant and in imposing sentence of imprisonment
for life. The order passed by the High Court, submitted the counsel, is not in consonance
with law and the applicant has fair and good chance of his appeal being allowed. He is in
jail since long and as the appeal is likely to take time, a reasonable prayer for suspension
of sentence and grant of bail deserves to be accepted by enlarging the applicant-accused
on bail on such terms and conditions as this Court deems fit.
14. Mr. Gopal Subramanyam, learned Addl. Solicitor General, on the other hand, strongly
opposed the prayer made by the applicant of suspension of sentence and grant of bail. He
submitted that the order of acquittal recorded by the trial Court was clearly wrong and
against the evidence on record. The High Court, as a Court of 'first appeal,' considered the
evidence and held that the trial Court was 'wholly' wrong in not believing the prosecution
witnesses. The High Court also observed that the grounds which weighed with the trial
Court for not believing prosecution witnesses, could not be said to be legal, proper or
based on evidence on record. The counsel submitted that there was no reason for the trial
Court not to believe evidence of P.W. 1 Deepak Bhojwani, P.W. 30 Shravan Kumar, P.W.
20 Beena Ramani, P.W. 6 Malini Ramani and other witnesses. The counsel submitted that
the High Court considered in detail, the reasons recorded by the trial Court and rightly
observed that to describe a particular witness as 'planted' by the prosecution is a serious
matter and normally no Court of law would proceed on that basis. Mr.
@page-SC2892
Subramanyam also submitted that from the prosecution evidence, it is clear that the
applicant along with other accused came to Tamarind Cafe on 29th April, 1999, asked for
liquor and when he was refused liquor on the ground that the bar was closed, he became
very angry, took out his 22 pistol and fired two rounds; one towards ceiling and the other
towards Jessica Lal due to which she died. This was witnessed by several persons who
were present at that time. Some of them, however, did not support the prosecution. The
learned Addl. Solicitor General submitted that the terror of the accused was clear from
the fact that about two dozen witnesses had been turned hostile. The trial Court ought to
have considered this aspect. But even otherwise, in view of the above situation, the
witnesses who were examined and supported the prosecution ought to have been believed
by the trial Court. It, however, failed to do so. The High Court was, therefore, 'fully'
justified in believing the evidence of those witnesses and in recording the order of
conviction. 15. It was also stated that according to the High Court, after the commission
of offence, the accused absconded. His farm house was raided by the police authorities
during the course of investigation. He was neither found there nor did he surrender
immediately. The High Court also recorded a finding that Tata Safari, used by the accused
at the time he visited Qutub Colonnade was recovered from NOIDA which was removed
from the place of offence. According to the High Court, the evidence on record showed
that Tata Safari was parked at Qutub Colonnade in the night of April 29-30, 1999. The
vehicle belonged to Piccadilly Agro Industries Limited of which the accused was
admittedly a Director. The vehicle was surreptitiously removed from the scene of
occurrence. The High Court noted that it was admitted by the accused that he was having
licensed pistol of .22 bore. The High Court was also aware that several witnesses turned
hostile and did not support the prosecution but from the available material, it was proved
beyond reasonable doubt that it was the applicant who had visited Qutub Colonnade on
the night of 29th/30th April, 1999 and demanded liquor and on refusal by Jessica Lal and
Shyan Munshi, he became angry and fired two shots one of which hit Jessica Lal and
proved fatal. It was, therefore, submitted by the learned Addl. Solicitor General that the
order passed by the High Court is legal, valid and in consonance with law and no error
has been committed by the High Court in setting aside the order of acquittal recorded by
the trial Court.
16. We are conscious and mindful that the main matter (appeal) is admitted and is
pending for final hearing. Observations on merits, one way or the other, therefore, are
likely to prejudice one or the other party to the appeal. We are hence not entering into the
correctness or otherwise of the evidence on record. It, however, cannot be overlooked
that as on today, the applicant has been found guilty and convicted by a competent
Criminal Court. Initial presumption of innocence in favour of the accused, therefore, is no
more available to the applicant.
17. In para 56, the High Court observed as under :
"56. In the totality of circumstances adduced from material on record, the judgment under
challenge appears to us to be an immature assessment of material on record which is self-
contradictory, based on misreading of material and unsustainable. We find that Beena
Ramani has identified Sidhartha Vashisht alias Manu Sharma, Amardeep Singh Gil, Alok
Khanna and Vikas Yadav to be the persons present at the Tamarind Cafe at the time of the
incidence. She also saw Manu Sharma firing the fatal shot which hit Jessica Lal. Her
testimony finds corroboration from the testimony of Malini Ramani and Geroge Mailhot.
There is evidence on record to show that Manu Sharma had a licensed pistol of .22 bore
which he has not produced to establish his innocence and on the contrary has taken false
plea that the pistol, its ammunition and licence had been removed by the Police on 30-4-
1999. We also find from the material on record that Manu Sharma abandoned his vehicle
while making good his escape. We also find that the ammunition used in the causing of
the firearm injury to Jessica Lal was of .22 bore which Manu Sharma admittedly
possessed and a similar live cartridge was recovered from the abandoned Tata Safari.
From this, we have no hesitation in holding that Manu Sharma is guilty of an offence
under S. 302. I.P.C. for having committed the murder of Jessica Lal on 29/30-4-1999 at
the Tamarind Cafe as also under S. 27. Arms Act."
(Emphasis supplied)
@page-SC2893
18. The High Court has also given cogent reasons for not accepting the view of the trial
Court and grounds recorded for not believing prosecution witnesses.
19. Mr. Ram Jethmalani, learned senior advocate no doubt submitted that the trial Court
was right in not relying upon the prosecution witnesses, but Mr. Gopal Subramanyam
submitted that the approach of the trial Court was incorrect and improper. According to
the High Court it was on the verge of 'perversity.'
20. It is premature to express any opinion, one way or the other at this stage but the fact
remains that the order of acquittal recorded by the trial Court has been set aside and the
applicant-accused has been convicted for an offence punishable under S. 302, I.P.C. and
ordered to undergo imprisonment for life.
21. Mr. Ram Jethmalani, learned senior advocate, invited our attention to several
decisions of this Court. Some of them relate to grant of bail at the pre-trial stage. The
Courts in such cases have considered several factors, such as, there is a presumption of
innocence in favour of an accused till it is established that he is guilty; he has to make
preparation for his defence and he must have every opportunity to look after his case; it
will be very difficult for an accused to make such preparation if he is in jail than he is out
of jail. One of the considerations which a Court of law would keep in mind at that stage is
to secure the attendance of the accused. Hence, on security being furnished, he is released
on bail if the Court is satisfied that the case on hand was fit one to grant such concession
in favour of the accused.
22. Before about eight decades, in the leading case of Emperor v. Hutchinson, AIR 1931
All 356 : 32 Cri LJ 1271 : 33 IC 842 (the Meerut Conspiracy case), Boys, J. observed :
"As to the object of keeping an accused person in detention during the trial, it has been
stated that the object is not punishment, that to keep an accused person under arrest with
the object of punishing him on the assumption that he is guilty even if eventually he is
acquitted is improper. This is most manifest. The only legitimate purposes to be served by
keeping person under trial in detention are to prevent repetition of the offence with which
he is charged where there is apparently danger of such repetition and to secure his
attendance at the trial. The first of those purposes clearly to some extent involves an
assumption of the accused's guilt but the very trial itself is based on a prima facie
assumption of the accused's guilt and it is impossible to hold that in some circumstances
it is not a proper ground to be considered. The main purpose however is manifestly to
secure the attendance of the accused."
(Emphasis supplied)
23. In concurring judgment, Mukherji, J. also stated :
"The principle to be deduced from Ss.496 and 497, Criminal P. C., therefore is that grant
of bail is the rule and refusal is the exception. That this must be so is not at all difficult to
see. An accused person is presumed under the law to be innocent till his guilt is proved.
As a presumably innocent person, he is entitled to freedom and every opportunity to look
after his own case. It goes without saying that an accused person. If he enjoys freedom,
will be in a much better position to look after his case and to properly defend himself
than if he were in custody.
(Emphasis supplied)
24. The above principle has been reiterated from time to time thereafter.
25. Section 389 of the Code expressly and specifically deals with suspension of sentence
pending appeal and release of appellant on bail. It states :
389. Suspension of sentence pending the appeal; release of appellant on bail :- (1)
Pending any appeal by a convicted person, the appellate Court may, for reasons to be
recorded by it in writing, order that the execution of the sentence or order appealed
against be suspended and, also if he is in confinement, that he be released on bail, or on
his own bond.
(2) The power conferred by this section on a appellate Court may be exercised also by the
High Court in the case of an appeal by a convicted person to a Court subordinate thereto.
(3) Where the convicted person satisfies the Court by which he is convicted that he
intends to present an appeal, the Court shall -
(i) where such person, being on bail, is sentenced to imprisonment for a term not
@page-SC2894
exceeding three years, or
(ii) where the offence of which such person has been convicted is a bailable one, and he
is on bail, order that the convicted person be released on bail, unless there are special
reasons for refusing bail, for such period as will afford sufficient time to present the
appeal and obtain the orders of the appellate Court under sub-section (1), and the
sentence of imprisonment shall, so long as he is so released on bail, be deemed to be
suspended.
(4) When the appellant is ultimately sentenced to imprisonment for a term or to
imprisonment for life, the time during which he is so released shall be excluded in
computing the term for which he is so sentenced.
26. Bare reading of the above provision makes it clear that during the pendency of
appeal, an appellate Court is empowered to suspend sentence on the appellant by
releasing him on bail. Such action, however, can be taken only after affording opportunity
to the Public Prosecutor in case of offence punishable with death or imprisonment for life
or imprisonment for ten years or more and after recording reasons in writing.
27

. Mr. Jethmalani, relying on the decisions in Kashmira Singh v. State of Punjab (1977) 4
SCC 291; Babu Singh and Ors. v. State of U.P., (1978) 1 SCC 579; Shailendra Kumar v.
State of Delhi, (2000) 4 SCC 178 : 2000 (1) JT (SC) 184, and other cases, submitted that
one of the factors which weighed with this Court in granting suspension of sentence and
releasing the applicant on bail is that in case of acquittal by the trial Court and conviction
by the appellate Court, hearing of appeal takes long time and the applicant has to remain
in jail. AIR 1977 SC 2147
AIR 1978 SC 527
2000 AIR SCW 1783

28. As observed in those cases, the practice of not releasing a person on bail who had
been sentenced for imprisonment for life under S. 302, I.P.C. was that the appeal was
likely to be heard in near future. But if such appeal would not be heard for long and not
disposed of within a 'measurable distance of time,' it would not be in the interest of
justice to keep such person in jail for a number of years and it would be appropriate if the
power under S. 389 of the Code is exercised in favour the applicant.

29

. In Kashmira Singh, this Court stated : AIR 1977 SC 2147, Para 2

"Now, the practice in this Court as also in many of the High Court has been not to release
on bail a person who has been sentenced to life imprisonment for an offence under S. 302
of the Indian Penal Code. The question is whether this practice should be departed from
and if so, in what circumstances. It is obvious that no practice howsoever sanctified by
usage and hallowed by time can be allowed to prevail if it operates to cause injustice.
Every practice of the Court must find its ultimate justification in the interest of justice.
The practice not to release on bail a person who has been sentenced to life imprisonment
was evolved in the High Courts and in this Court on the basis that once a person has been
found guilty and sentenced to life imprisonment, he should not be let loose, so long as his
conviction and sentence are not set aside, but the underlying postulate of this practice was
that the appeal of such person would be disposed of within a measurable distance of time,
so that if he is ultimately found to be innocent, he would not have to remain in jail for an
unduly long period. The rationale of this practice can have no application where the Court
is not in a position to dispose of the appeal for five or six years. It would indeed be a
travesty of justice to keep a person in jail for a period of five or six years for an offence
which is ultimately found not to have been committed by him. Can the Court ever
compensate him for his incarceration which is found to be unjustified? Would it be just at
all for the Court to tell a person : "We have admitted your appeal because we think you
have a prima facie case, but unfortunately we have no time to hear your appeal for quite a
few years and, therefore, until we hear your appeal, you must remain in jail, even though
you may be innocent?" What confidence would such administration of justice inspire in
the mind of the public? It may quite conceivably happen, and it has in fact happened in a
few cases in this Court, that a person may serve out his full term of imprisonment before
his appeal is taken up for hearing. Would a Judge not be overwhelmed with a feeling of
contrition while acquitting such a person after hearing the appeal? Would it not be an
affront to his sense of justice? Of what avail would the acquittal be to such a person who
has already served out his term
@page-SC2895
of imprisonment or at any rate a major part of it? It is therefore, absolutely essential that
the practice which this Court has been following in the past must be reconsidered and so
long as this Court is not in a position to hear the appeal of an accused within a reasonable
period of time, the Court should ordinarily, unless there are cogent grounds for acting
otherwise, release the accused on bail in cases where special leave has been granted to the
accused to appeal against his conviction and sentence."

(Emphasis supplied)
30. The other consideration, however, is equally important and relevant. When a person is
convicted by an appellate Court, he cannot be said to been 'innocent person' until the final
decision is recorded by the superior Court in his favour.
31

. Mr. Gopal Subramanyam, learned Addl. Solicitor General invited our attention to
Akhilesh Kumar Sinha v. State of Bihar, (2000) 6 SCC 461; Vinay Kumar v. Narendra
and Ors. (2002) 9 SCC 364 : JT 2002 Supp (1) SC 60; Ramji Prasad v. Rattan Kumar
Jaiswal and Anr., (2002) 9 SCC 366 : JT 2000 (7) SC 477; State of Haryana v. Hasmat,
(2004) 6 SCC 175 : JT 2004 (6) SC 6; Kishori Lal v. Rupa and Ors. (2004) 7 SCC 638 :
JT 2004 (8) SC 317 and State of Maharashtra v. Madhukar Wamanrao Smarth, (2008) 4
Scale 412 : JT 2008 (4) SC 461. 2000 AIR SCW 3602
2004 AIR SCW 4266
2004 AIR SCW 7409
2008 AIR SCW 2475

32. In the above cases, it has been observed that once a person has been convicted,
normally, ah appellate Court will proceed on the basis that such person is guilty. It is no
doubt true that even thereafter, it is open to the appellate Court to suspend the sentence in
a given case by recording reasons. But it is well settled, as observed in Vinay Kumar that
in considering the prayer for bail in a case involving a serious offence like murder
punishable under S. 302, I.P.C., the Court should consider all the relevant factors like the
nature of accusation made against the accused, the manner in which the crime is alleged
to have been committed, the gravity of the offence, the desirability of releasing the
accused on bail after he has been convicted for committing serious offence of murder, etc.
It has also been observed in some of the cases that normal practice in such cases is not to
suspend the sentence and it is only in exceptional cases that the benefit of suspension of
sentence can be granted.
33

. In Hasmat, this Court stated : 2004 AIR SCW 4266

"6. Section 389 of the Code deals with suspension of execution of sentence pending the
appeal and release of the applicant on bail. There is a distinction between bail and
suspension of sentence. One of the essential ingredients of S. 389 is the requirement for
the appellate Court to record reasons in writing for ordering suspension of execution of
the sentence or order appealed. If he is in confinement, the said Court can direct that he is
released on bail or on his own bond. The requirement of recording reasons in writing
clearly indicates that there has to be careful consideration of the relevant, aspects and the
order directing suspension of sentence and grant of bail should not be passed as a matter
of routine."
(Emphasis supplied)
34. The mere fact that during the period of trial, the accused was on bail and there was no
misuse of liberty, does not per se warrant suspension of execution of sentence and grant
of bail. What really necessary is to consider whether reasons exist to suspend execution
of the sentence and grant of bail.
35. On the facts and in the circumstances of the case, in our considered opinion, this is,
not a fit case to exercise power under Section 389 of the Code. Though the trial Court has
acquitted the applicant-accused for the offences with which he was charged, the High
Court reversed the order of acquittal and convicted him under Section 302, IPC and
ordered him to undergo rigorous imprisonment for life. Being aggrieved by the said
order, he has filed an appeal which has been admitted, is already on board and awaits
final hearing. Hence, within 'measurable distance of time' the appeal is likely to be heard.
Keeping in view the seriousness of offence, the manner in which the crime was said to
have been committed and the gravity of offence, we are of the view that no case has been
made out by the applicant-appellant for suspension of sentence and grant of bail. The
application deserves to be dismissed and is accordingly dismissed.
36. Before parting with the matter, we may clarify that we may not be understood to have
expressed any opinion on merits of
@page-SC2896
the matter one way or the other and all the observations made by us hereinabove should
be taken as confined to dealing with the prayer of the applicant-appellant under Section
389 of the Code. As and when the main matter i.e. criminal appeal will come up for
hearing, it will be decided on its own merits without being inhibited or influenced by the
observations in this order.
37. The application is accordingly disposed of.
Order accordingly. .
AIR 2008 SUPREME COURT 2896 "Airport Authority of India v. Shambhu Nath Das"
(From : Calcutta)*
Coram : 2 TARUN CHATTERJEE AND HARJIT SINGH BEDI, JJ.
Civil Appeal No. 3617 of 2008 (arising out of SLP (C) No. 10452 of 2007), D/- 16 -5
-2008.
Airport Authority of India and Ors. v. Shambhu Nath Das alias S.N. Das.
Constitution of India, Art.16 - EQUALITY IN PUBLIC EMPLOYMENT -
REINSTATEMENT - BACK WAGES - Abandonment of service - Re-instatement -
Back-wages - Respondent employee overstaying leave on medical grounds - On being
found fit, be was called upon repeatedly to resume duty - But be failed to do so and
claimed back-wages - In compliance of direction of High Court, competent authority
considered his claim sympathetically and rejected same for valid reasons - Considering
fact that respondent did not report for duty for almost 15 years, order for payment of 50%
back-wages for period he remained absent is improper.
F.M.A. No. 2375 of 2004, D/-21-03-2007 (Cal), Reversed. (Paras 6, 7)

Mrs. Rachana Joshi Issar and Ajay Thakur, for Appellants; Amlan Kumar Ghosh, for
Respondent.
* F. M. A. No. 2375 of 2004, D/- 21-3-2007 (Cal).
Judgement
1. HARJIT SINGH BEDI, J. :-Leave granted.
2. This appeal arises out of the following facts :
3. The respondent, Shambhu Nath Das, a resident of Kolkata, who was posted as an
Assistant Engineer with the appellant-authority at Kolkata was transferred to Delhi. He
reported for duty at Delhi and served for 7 days and thereafter took leave for 18 days
from 26th March 1985 to 12th April 1985 on account of his grandmother's illness. He did
not rejoin duty thereafter but made several applications for extension of leave on medical
grounds. The appellant accordingly directed him, vide order dated 12th August 1985, to
appear before a Medical Board at Kolkata at 11 a.m. on that day. The Medical Board in
its report opined that the respondent was physically fit and, therefore, capable of
resuming duty. This information was also conveyed to the respondent vide letter dated
9th September 1985 and he was advised to report for duty immediately failing which
action would be taken against him as per the rules. Despite this warning, however, the
respondent did not report for duty. A Memorandum dated 17th October 1985 was also
addressed to the respondent calling upon him yet again to resume duty on or before 30th
October 1985 failing which it would be presumed that he had voluntarily abandoned his
service with the consequence that his name would be struck off the rolls with effect from
1st November 1985. The respondent, however, still did not report for duty but challenged
the memorandum dated 17th October 1985 by filing Writ Petition No. 5715 (W) of 1986
which was ultimately disposed of by a Single Judge of the Calcutta High Court vide order
dated 10th November 1995 with a direction to the appellant to allow the respondent to
resume duty but with a further direction that he would not be entitled to any arrears of
pay and allowances or any other service benefit for the period of his absence. The
respondent, however, still did not join duty in terms of the order dated 10th November
1995 but challenged the same by filing Writ Appeal No. 3687/1995 before the Division
Bench. The Division Bench in its order dated 9th August 1996 set aside the order dated
10th November 1995 and remanded the Writ Petition to the learned Single Judge with a
direction that a reasoned order be passed after hearing the contesting parties. The matter
was, accordingly, re-heard by the learned Single Judge and it was observed that as the
writ petitioner (now respondent) had overstayed his leave, the appellant-authority would
have been justified in dismissing him from service but after having observed as such,
gave the following directions on 13th August 1999 :
@page-SC2897
"The respondent-Airports Authority of India is directed to reinstate the writ petitioner in
service at Delhi or any other Airport where a suitable post is lying vacant within six
weeks from the date of communication of this order. Insofar as the salary of the writ
petitioner is concerned during the period he stayed away from the work, the respondent,
Airports Authority of India, is directed to consider the matter sympathetically and, if it is
permissible under its rules, allow to him half of the salary and other benefits during the
period from 17th October 1985 till 10th November 1985."
4. A subsequent order of the learned Single Judge vide order dated 31st August 1999
modified the order of 13th August 1999, to the extent that the period of consideration of
salary etc. was to be limited to "the period from 17th October 1985 till 10th November
1995". The appellant accepted the judgment of the learned Single Judge and allowed the
respondent to join duty with effect from 1st November 1999 leaving the issue of payment
of back wages to be decided subsequently. The appellant thereafter gave a personal
hearing to the respondent on 31st January 2002 and passed an order dated 14th May 2002
holding that the period of unauthorized absence was to be treated as dies-non and the
claim for back-wages was accordingly disallowed on the principle of "no work no pay".
The order dated 14th May 2002 was once again challenged by the respondent by filing
Writ Petition No. 12321-W/2002 before the Calcutta High Court claiming, interalia,
back-wages for the entire period of his unauthorized absence. This Writ Petition was also
allowed by the learned Single Judge in his order dated 7th August 2003 and a direction
was issued that the question of payment of back-wages be decided afresh by the
appellant. The matter was once again taken up by the appellant and the plea for back-
wages was yet again denied by order dated 2nd/5th January 2004. This order was
challenged by the respondent by way of Writ Petition No. 4283 (W) of 2004 which was,
however, dismissed by a learned Single Judge of the High Court on 15th April 2004
observing as under :
"After hearing the learned counsel, appearing for the parties, this court is of the view that
the petition needs to be thrown in limine for the following reasons:
(a) By the order dated 13th August 1999, this court directed that "insofar as the salary of
the writ petitioner is concerned during the period he stayed away from the work, the
respondent, Airports Authority of India, is directed to consider the matter sympathetically
and, if it is permissible under its Rules, allow to him half of the salary and other
benefits."
(b) The right of the petitioner, if any, has crystallized in the order dated 13th August
1999. He cannot be allowed to reopen the matter nor is he at liberty to advance new
grounds of his entitlement. At the highest he is entitled to execute the order dated 13th
August 1999 as corrected by the order dated 31st August 1999.
(c) In the order under challenge dated 2nd/5th January, 2004, the authorities have
recorded that the petitioner could not give nay justification in order to support his claim
for back-wages. The authority concerned has recorded in his order that there is no such
rule which permits such payment. The order dated 13th August 1999 directed the
authority concerned to sympathetically consider the question of payment of back-wages
if it was permissible under the Rule. When factually there is no dispute between the
parties that the rules do not permit the payment of any such back-wages, the question
came to an end then and there.
(d) The submission advanced by Mr. Basu, the learned Senior Advocate, for the
petitioner, that if the rules are not there then the absence of Rule cannot militate against
the claim of the petitioner, in any view, cannot be entertained because that would amount
to sitting in appeal over the order dated 13th August, 1999 which I am unable to do.
(e) The submission that the Rule 31, and theory of 'dies-non' has no application to the
facts of this case and, in my view, self-annihilating because if this submissions were
given effect to then all the benefits given to the petitioner under the order dated 14th May
2002 have to be recalled. He cannot blow hot and cold.
(f) It is submitted that the petitioner is not aggrieved by the rest of the order dated 14th
May 2002 and the challenge is restricted to sub-para (i). Sub-para (i) is based on the same
principle on which the sub-paras (ii) to (vi) are based. The petitioner is happy with the
directions contained in sub-para (i) Whereon (vi) of the order dated 14th May 2002 is the
same.
@page-SC2898
In that view of the matter, the approach of the petitioner, according to this court, is
anything but bona fide. Accordingly, this petition is dismissed with costs assessed at 200
G.Ms."
5. This order of the learned Single Judge has been set aside by the Division Bench vide
the impugned order dated 21st March 2007 with the observations that the order of the
High Court in C.R.(W)No. 5715/1986 which had directed that the respondent be paid
50% of the back-wages for the period from 17th October, 1985 to 10th November, 1995
should be complied with.
6. The learned counsel for the appellant has pointed out that as the respondent had not
attended to his duties for almost 15 years despite having been called upon to do so
repeatedly, the direction of the Division Bench to grant him back-wages from 17th
October 1985 to 10th November 1995 was clearly not justified on the principle of "no
work no pay". She has pointed out that the appellant-authority would have been fully
justified even if it had dismissed the respondent from service, but on the contrary, a huge
benefit had already been given to him as he had been taken back in service despite having
remained absent for almost fifteen years. The learned counsel for the respondent has,
however, supported the judgment of the Division Bench. We are of the opinion that in the
light of the fact that the respondent did not report for duty for 15 years, there was no
justification whatsoever to grant him any back-wages on the general principle that
nobody could be directed to claim wages for the period that he remained absent without
leave or without justification. We also find that the judgment dated 13th August, 1999
which had attained finality had directed as under:
(a) "insofar as the salary of the writ petitioner is concerned during the period he stayed
away from the work, the respondent, Airports Authority of India, is directed to consider
the matter sympathetically and, if it is permissible under its Rules, allow to him half of
the salary and other benefits."
7. This claim was considered by the competent authority and rejected for valid reasons.
We are, thus, unable to endorse the High Court's order for payment of 50% back-wages
for the period from 17th October, 1985 to 10th November, 1995 which are far in excess
of the directions in the order dated 13th August, 1999. We accordingly allow this appeal
set aside the order of the Division Bench and restore the order of the learned Single Judge
dated 15 April 2004.
8. The appeal is allowed in the above terms. No order as to costs.
Appeal allowed. .
AIR 2008 SUPREME COURT 2898 "Mallavarapu Kasivisweswara Rao v. Thadikonda
Ramulu Firm"
(From : 2000 (1) Andh LT 467)
Coram : 2 TARUN CHATTERJEE AND HARJIT SINGH BEDI, JJ.
Civil Appeal No. 5597 of 2001, D/- 16 -5 -2008.
Mallavarapu Kasivisweswara Rao v. Thadikonda Ramulu Firm and Ors.
Negotiable Instruments Act (26 of 1881), S.118(a) - NEGOTIABLE INSTRUMENT -
EXECUTION - PROMISSORY NOTE - Presumption under - Suit for recovery on basis
of Pronotes - Execution of pronotes proved - Executant, appellant entitled to benefit of
presumption u/S.118(a) - Moreso, when respondents failed to discharge initial burden of
proving non-existence of consideration by direct evidence or preponderance of
probabilities.
2000 (1) Andh LT 467, Reversed.
If the defendant is proved to have discharged the initial onus of proof showing that the
existence of consideration was improbable or doubtful or the same was illegal, the onus
would shift to the plaintiff who would be obliged to prove it as a matter of fact and upon
its failure to prove would disentitle him to the grant of relief on the basis of the
negotiable instrument. If the defendant fails to discharge the initial onus of proof by
showing the non-existence of the consideration, the plaintiff would invariably be held
entitled to the benefit of presumption arising under S. 118(a) in his favour. (Para 13)
In the instant case, the plea of the respondents in their written statements was that on the
face of the pronote, no cash was paid by the appellant and, therefore, the respondents
were not liable to pay the amount because the pronotes were forged. It was a finding of
the trial Court, which was affirmed by the High Court that the pronotes were indeed
executed by the respondents. The finding as to the execution of the pronotes had become
final. The respondents had not discharged the initial burden of proving the
@page-SC2899
non-existence of consideration either by direct evidence or by preponderance of
probabilities. The mere denial, if there be any, by the respondents that no consideration
had passed would not have been sufficient and something probable had to be brought on
record to prove the non-existence of consideration. Therefore, once the execution of the
pronote has been proved, the appellant would be entitled to the benefit of the presumption
under S. 118(a) of the Negotiable Instruments Act because the respondents had failed to
discharge the initial burden and, therefore, the order of the High Court in not decreeing
the suit of the appellant in respect of the amount covered by the said pronote was not
proper.
2000 (1) Andh LT 467, Reversed. (Para 15)
Cases Referred : Chronological Paras
1999 AIR SCW 636 : AIR 1999 SC 1008 (Ref.) 8, 12, 15
Mrs. Anjani Aiyagari, for Appellant; U.A. Rana and Abhishek K. Rao (For M/s. Gagrat
and Co.), for Respondents.
Judgement
1. TARUN CHATTERJEE, J. :- This appeal is directed against the final judgment and
order dated 30th of July, 1999 passed by a Division Bench of the High Court of
Judicature of Andhra Pradesh at Hyderabad in AS No. 721/92 whereby the High Court
had affirmed the judgment and decree dated 5th of August, 1991 in OS No. 33/87 of the
1st Court of the Additional Subordinate Judge, Kakinada, E.G. District, Andhra Pradesh
decreeing the suit filed by the appellant in part for a sum of Rs. 2,33,125/-with interest @
18% from the date of the suit till realization.
2. The facts leading to the filing of this appeal as emerging from the case made out by the
appellant in the plaint are as under :
The appellant is the son-in-law of respondent No.2. The respondent Nos. 3 and 4 are the
sons of respondent No. 2 while respondent No. 1 is the firm belonging to respondent Nos.
2 to 4 whose managing partner is respondent No. 2. The appellant introduced one Pynda
Ramakumar to the respondents who agreed to advance monies to the respondents on the
understanding that the respondents would repay the amount while the appellant would
execute pronotes as surety. The appellant accordingly executed certain pronotes whose
consideration was received by the respondents. As regards repayment, the respondents
were sending monies by drafts or, otherwise in the name of one Narayan Murthy, who
was the clerk of the appellant, by depositing the same in his account. The appellant would
withdraw such amount deposited in the clerk's account by encashing the TTs or Drafts
which was then paid to Pynda Ramakumar who then got the endorsements signed by the
appellant. This continued for some time but when the respondents failed to repay the
balance amount due to Pynda Ramakumar, he pressurized the appellant for payment of
the balance amount due to him. The appellant made several demands to the respondents
for payment of the amounts due to Pynda Ramakumar but when the respondents could
not pay the amounts, the respondent no. 2 as manager of the joint family and also on
behalf of the respondent No. 1 firm executed two pronotes for sums of Rs. 2,15,000/- and
Rs. 4,72,000/- being Ex.A-20 and Ex.A-21 respectively and a Khararnama in favour of
the appellant whereby the respondent No. 1 agreed to repay amounts with interest at Rs.
2.50 ps. and Rs. 1.50 ps. respectively per annum. After execution of such pronotes, when,
despite several demands, the respondents did not pay the amounts, a notice dated 3rd of
October, 1986 was issued to them by the appellant stating that the pronotes and
khararnama were executed by respondent No. 1 in favour of the appellant which may be
discharged. The respondents vide letters dated 16th of October, 1986 and 20th of October,
1986 replied to the notice wherein they did not specifically deny the execution of the
pronotes and the Khararnama but referred to the allegations made in such notice as false
and vague.
3. In the backdrop of the above-mentioned facts, in 1987, the appellant, therefore, filed
O.S. No. 33/1987 in the 1st Court of the Additional Subordinate Judge, Kakinada for
recovery of the amounts due under the pronotes of Rs. 4,72,000/- and Rs. 2,15,000/- with
interest and costs. The respondent No. 2 contested the suit by filing written statement on
his own behalf and also on behalf of the respondent No. 1 firm denying any execution of
the pronotes in favour of the appellant and further stating that the pronotes were forged
by the appellant with the assistance of his brother-in-law and the Clerk. It was further
alleged that the appellant bore a grudge against the respondents
@page-SC2900
and Was involved in many criminal cases and since he was not looking after his wife and
children properly, the respondents had opened an account in the name of Narayanmurthy
and were sending monies regularly in that account for the maintenance of the appellants'
family and therefore, it was alleged that no money was ever borrowed from the said
Pynda Ramakumar, whom the respondent No. 2 did not know, through the appellant for
the respondent No. 1 firm. It was also alleged by the respondent No. 2 that the respondent
No. 1 firm was not carrying on any business and in fact, all its branches were closed and
the respondent Nos. 2 to 4 were partitioned in the year 1980.
4. The respondent Nos. 3 and 4 also filed separate written statements contending, inter
alia, that they had not signed any pronotes and the scribe of the pronotes in question was
the clerk of the appellant and the Attester was his brother-in-law. They also contended
that they were not aware of the alleged borrowing by the respondent No. 2 for the
respondent No. 1 firm from the said Pynda Ramakumar or the appellant and in fact, the
pronotes in question did not show that the amounts so borrowed were for the business of
the respondent No. 1 firm. It was further alleged in the written statement filed by the
respondent Nos. 3 and 4 that the pronotes were fabricated on account of family disputes
between the appellant and the respondent No. 2 and that they had no necessity to borrow
any amount from some other person. In fact, Pynda Ramakumar was a friend and an
associate of the appellant. Even otherwise, the pronotes were not binding on them as no
amounts were borrowed for the benefit of the firm and they were not signatories to the
said pronotes. It was further the case of the respondent Nos. 3 and 4 that there was no
joint family because the properties of the respondents were partitioned in the year 1980
and, therefore, the respondent No. 2 had no right or authority to borrow debts for the firm
on their behalf. Accordingly, all the respondents prayed for dismissal of the suit filed by
the appellant.
5. On the basis of the pleadings of the parties, the following issues were framed by the
trial court for consideration :-
a) Whether the two suit pronotes dated 29.08.86 and 29.08.1986 are true, valid and
binding on the defendants?
b) Whether the plaintiff is entitled to recover the suit amount with subsequent interest and
costs thereon?
c) Whether the 2nd defendant executed the suit pronotes in the capacity of Manager of
the joint family of the defendants 2 to 4 so as to bind the defendants 3 and 4?
d) Whether the defendant No. 2 executed pronotes as the Managing Partner of D. 1 firm
so as to bind its partners 3 and 4?
e) To what relief?
6. As noted herein-earlier, by the judgment dated 5th of August, 1991, the 1st Court of the
Additional Subordinate Judge, Kakinada decreed the suit of the appellant in part for a
sum of Rs. 2,33,125/- with proportionate costs and subsequent interest @ 18 % p.a. from
the date of suit till realization holding the same to be a commercial transaction (Ex.A-20).
As regards recovery of the amount due under the other pronote Ex.A-21, the trial court
held that the appellant was not entitled to recover the same because the said pronote was
not supported by consideration and accordingly, the rest of the claim of the appellant was
dismissed with proportionate costs. Feeling aggrieved by the said judgment of the trial
court, both the appellant and the respondents filed two appeals before the High Court of
Andhra Pradesh at Hyderabad being A.S. No. 721/ 87 and 1872/92 respectively. By the
impugned judgment of the High Court dated 30th of July, 1999, both these appeals were
dismissed. The appellant has filed this special leave petition before us against the
aforesaid judgment of the High Court passed in A.S. No. 721/87.
7. We have heard the learned counsel for the parties and examined the judgment of the
High Court as well as the trial Court and other materials on record including the oral and
documentary evidence. The only question that needs to be decided in this appeal is
whether in the absence of any rebuttal by the respondents to the fact that the promissory
note was for consideration as required, which gave rise to the presumption under Section
118 of the Negotiable Instruments Act, the courts below were justified in holding that
since the appellant had given evidence inconsistent with such presumption, no decree
could be passed on the basis of such presumption.
8

. The learned counsel for the appellant contended before us that the trial court had 1999
AIR SCW 636

@page-SC2901
found that the existence of both the pronotes was proved by evidence and the materials
on record. The learned counsel for the appellant accordingly contended that although it
was never the defence of the respondents that the pronotes were not supported by
consideration, nevertheless, the trial court had held that since the appellant had failed to
prove that he had borrowed those amounts from Pynda Ramakumar and lent the same to
the respondent-firm, the pronote Ex.A-21 could not be believed. The learned counsel for
the appellant, therefore, vehemently argued that the conclusion reached by the trial court
and the High Court to the effect that since the evidence adduced by the appellant was
inconsistent with the presumption, in the absence of any evidence by the respondent to
rebut the presumption about the pronote, such conclusion was contrary to law. The
learned counsel for the appellant, while elaborating her argument further also contended
before us that once the execution of the pronote Ex.A-21 was proved, the presumption
under Section 118 of the Negotiable Instruments Act came into play and after such
presumption, the initial burden was on the respondents to prove the non-existence of the
consideration by adducing direct evidence or by preponderance of probabilities showing
that the existence of such consideration was improbable, doubtful or illegal and since
they had failed to discharge such initial burden, the appellant was entitled to the benefit
of the presumption that the pronote was for consideration. In this regard, the learned
counsel for the appellant strongly relied on a decision of this court in Bharat Barrel and
Drum Company vs. Amin Chand Payrelal [(1999) 3 SCC 35).
9. These submissions of the learned counsel for the appellant were contested by the
learned counsel appearing on behalf of the respondents. The learned counsel appearing on
behalf of the respondents have contended that the appellant is not entitled to the benefit
of presumption under Section 118 of the Negotiable Instruments Act and that the
impugned judgment of the High Court was passed after considering the evidence in
extenso to hold that the pronote Ex. A-21 was not supported by any consideration. The
learned counsel for the respondents further submitted that the presumption under Section
118 is rebuttable and that the respondents had all along denied the execution of the
pronotes.
10. Having heard the learned counsel for the parties, we are of the view that this appeal
deserves to be allowed for the reasons set out hereinafter.
11. Section 118 of the Negotiable Instruments Act deals with presumptions as to
negotiable instruments. One of such presumptions appearing in Section 118(a), with
which we would be concerned in this appeal is reproduced as under :-
"that every negotiable instrument was made or drawn for consideration, and that every
such instrument, when it has been accepted, endorsed, negotiated or transferred, was
accepted, endorsed, negotiated or transferred for consideration."
12

. Under Section 118(a) of the Negotiable Instruments Act, the court is obliged to presume,
until the contrary is proved, that the promissory note was made for consideration. It is
also a settled position that the initial burden in this regard lies on the defendant to prove
the non-existence of consideration by bringing on record such facts and circumstances
which would lead the Court to believe the non-existence of the consideration either by
direct evidence or by preponderance of probabilities showing that the existence of
consideration was improbable, doubtful or illegal. In this connection, reference may be
made to a decision of this Court in the case of Bharat Barrel and Drum Manufacturing
Company vs. Amin Chand Payrelal [supra]. In paragraph 12 of the said decision, this
court observed as under :- 1999 AIR SCW 636, Para 12
"Upon consideration of various judgments as noted hereinabove, the position of law
which emerges is that once execution of the promissory note is admitted, the presumption
under Section 118(a) would arise that it is supported by a consideration. Such a
presumption is rebuttable. The defendant can prove the non-existence of a consideration
by raising a probable defence. If the defendant is proved to have discharged the initial
onus of proof showing that the existence of consideration was improbable or doubtful or
the same was illegal, the onus would shift to the plaintiff who will be obliged to prove it
as a matter of fact and upon its failure to prove would disentitle him to the grant of relief
on the basis of the negotiable instrument. The burden upon the defendant of proving the
non-existence of the consideration
@page-SC2902
can be either direct or by bringing on record the preponderance of probabilities by
reference to the circumstances upon which he relies. In such an event, the plaintiff is
entitled under law to rely upon all the evidence led in the case including that of the
plaintiff as well. In case, where the defendant fails to discharge the initial onus of proof
by showing the non-existence of the consideration, the plaintiff would invariably be held
entitled to the benefit of presumption arising under Section 118(a) in his favour. The
court may not insist upon the defendant to disprove the existence of consideration by
leading direct evidence as the existence of negative evidence is neither possible nor
contemplated and even if led, is to be seen with a doubt. The bare denial of the passing of
the consideration apparently does not appear to be any defence. Something which is
probable has to be brought on record for getting the benefit of shifting the onus of
proving to the plaintiff. To disprove the presumption, the defendant has to bring on record
such facts and circumstances upon consideration of which the court may either believe
that the consideration did not exist or its non-existence was so probable that a prudent
man would, under the circumstances of the case, shall act upon the plea that it did not
exist......"
13. From the above decision of this court, it is pellucid that if the defendant is proved to
have discharged the initial onus of proof showing that the existence of consideration was
improbable or doubtful or the same was illegal, the onus would shift to the plaintiff who
would be obliged to prove it as a matter of fact and upon its failure to prove would
disentitle him to the grant of relief on the basis of the negotiable instrument. It is also
discernible from the above decision that if the defendant fails to discharge the initial onus
of proof by showing the non-existence of the consideration, the plaintiff would invariably
be held entitled to the benefit of presumption arising under Section 118(a) in his favour.
14. Keeping the aforesaid in mind, let us now see if the respondents in this case had
discharged the initial burden, which lay on them to prove that the pronote being Ex.A-21
was not supported by consideration.
15

. The learned counsel for the appellant, as noted herein earlier, contended that the
respondents had neither taken the plea that there was no consideration for the pronote
Ex.A-21, either in the reply notice or in the written statement, nor had they adduced any
evidence to prove the non-existence of the consideration. The learned counsel for the
respondents, however, contended that the respondents had denied the very execution of
the pronotes and referred the same as forged both in the reply notice as also in the written
statement. We are unable to accept the contentions of the learned counsel for the
respondents. In the written statements, the plea of the respondents was that on the face of
the pronotes, no cash was paid by the appellant and therefore, the respondents were not
liable to pay the amount because the pronotes were forged. It was a finding of the trial
court, which was affirmed by the High Court in the impugned judgment that the pronotes
were indeed executed by the respondents. It was also a finding of the High Court that
except in the reply notice issued by the respondents, nowhere had they stated that the
consideration had not passed. It is also an admitted position that the findings of the two
courts below was that the execution of the pronotes having been proved, the presumption
under Section 118(a) must come into play and the appellant must be entitled to a decree
in the absence of evidence to the contrary. Having said this, the High Court proceeded to
observe that if there was evidence inconsistent with the presumption under Section 118(a)
of the Act, the court would not be in a position to pass a decree in favour of the appellant
on the basis of the presumption and, therefore, proceeded to examine the evidence of the
appellant in extenso. In view of the decision of this Court in Bharat Barrel and Drum
Manufacturing Company vs. Amin Chand Payrelal (supra) and also in view of the
findings arrived at by the Courts below, we are of the view that since the initial burden on
the respondents to show that the pronote being Ex.A-21 was not supported by any
consideration was not discharged by them, the High Court was not justified in not
decreeing the suit of the appellant in respect of the amount covered by the pro-note Ex.A-
21. It is an admitted position that the finding as to the execution of the pronotes had
become final. Also, we are of the view that the respondents had not discharged the initial
burden of proving the non-existence of consideration either by direct evidence or by
preponderance of probabilities. 1999 AIR SCW 636

@page-SC2903
The mere denial, if there be any, by the respondents that no consideration had passed
would not have been sufficient and something probable had to be brought on record to
prove the non-existence of consideration. In this view of the matter, we are, therefore, of
the view that once the execution of the pronote has been proved, the appellant would be
entitled to the benefit of the presumption under Section 118(a) of the Negotiable
Instruments Act because the respondents had failed to discharge the initial burden and,
therefore, the High Court was in error in appreciating the evidence of the appellant to
come to the conclusion that since such evidence was inconsistent with the pronote being
Ex.A-21, the appellant could not be given the benefit of the presumption.
16. For the foregoing reasons, the appeal is allowed and the judgments of the courts
below are, therefore, modified to the extent that the suit of the appellant must stand
decreed in its entirety. There will be no order as to costs.
Appeal allowed. .
AIR 2008 SUPREME COURT 2903 "Illa Roy Chowdhury v. Shyamali Das"
(From : Calcutta)
Coram : 2 S. B. SINHA AND Dr. MUKUNDAKAM SHARMA, JJ.
Civil Appeal No. 3638 of 2008 (arising out of SLP (C) No. 2904 of 2007), D/- 16 -5
-2008.
Illa Roy Chowdhury v. Shyamali Das and Ors.
Land Acquisition Act (1 of 1894), S.30 - ACQUISITION OF LAND - HIGH COURT -
Reference - Respondent's earlier attempt to get her impleaded to reference made by
Collector failed - Order becoming final - Conditional order passed by High Court
permitting respondent to file application - Conditions stipulated not complied with -
Respondent cannot take benefit of order - Order subsequently passed by High Court to
dispose of respondent's application - Liable to be set aside.
W. P. No. 27264 of 2006, D/-20-12-2006 (Cal), Reversed. (Paras 14, 15)
Cases Referred : Chronological Paras
2006 AIR SCW 5527 : AIR 2007 SC 215 (Ref.) 3
Bijan Kumar Ghosh, R.K. Gupta, S.K. Gupta, Arun Yadav and A.N. Baradaiyar, for
Appellant; Pradip K. Ghosh, Sr. Advocate, Chinmoy A. Khaladkar, Ms. Rukhsana
Choudhury, Ms. Anindita Gupta, Rajesh Srivastava, Tara Chandra Sharma, Ms. Neelam
Sharma, Rajeev Sharma and Rameshwar Prasad Goyal, for Respondents.
Judgement
S. B. SINHA, J. :- Leave granted.
1. This petition is directed against a judgment and order dated 20th December, 2006
passed by the High Court of Calcutta in Writ Petition No. 27264 of 2006.
By reason of the said order the High Court directed the respondent Nos. 1 and 2 as also
the other concerned respondents, added therein to dispose of the First Respondent's
application for reference in terms of Sections 30 and 31 of the Land Acquisition Act,
1894 (hereinafter referred to as 'the Act').
2. The matter relates to a property acquired under the said Act. A reference was made by
the Collector in terms of the provisions thereof. First Respondent intended to be
impleaded as the party therein. The same was rejected.
3

. Contesting parties herein claimed themselves to be the heirs and legal representatives of
Rani Rashmoni. We need not state the facts of the matter in detail as the same has been
noticed by a Bench of this Court in Shyamali Das v. Illa Chowdhry, (2006) 12 SCC 300.
2006 AIR SCW 5527

One of the questions which arose for consideration therein was as to whether the First
Respondent, in terms of an observation made by another learned Single Judge of the High
Court, had filed an application for reference under Sections 30 and 31 of the said Act. It
was noticed therein that such an application had not been filed. It was furthermore
observed :-
"21. It is one thing to say that a proceeding under Sections 30 and 31 of the Act was
maintainable at the instance of the appellant. She was given an opportunity to file the
same by the Calcutta High Court in terms of its order dated 22-9-2000. She did not avail
the said opportunity. Having not availed the opportunity, in our opinion, she was not
entitled to be impleaded as a party."
4. We would notice some of the orders passed by the Courts in the earlier rounds of
litigation.
5. From the order dated 26th September, 2005 passed in C.O. No.3447 of 2005 by a
@page-SC2904
learned Single Judge of the High Court it appears that a question arose as to whether such
an application had been filed or not. The said order reads as under :-
"Put up the matter on Friday (30.9.2005) under the heading 'For Orders' before Listed
Motion in the supplementary list.
Mr. Subroto Mukhopadhyay, Ld. Advocate appears for the opposite party No. 3. Mr.
Mukhopadhyay is requested to obtain instruction from his client as to whether the
opposite parties Nos. 1 and 2 have filed any application under sections 30 and 33 of the
Land Acquisition Act.
Smt. Shyamali Das, the opposite party No.1 appears in person. She informs this Court
that on the next date the opposite party No. 2, who is her son, shall also appear in person.
The requiring authority, viz. West Bengal Housing Board may hand over the cheque to
the Collector and the Collector is directed to retain the cheque for the present."
6. In Writ Petition No. 19298 of 2000 filed by the First Respondent a learned Single
Judge of the High Court while disposing of the same by his order dated 22nd September,
2000 directed :-
"This Court sitting in writ jurisdiction cannot determine the entitlement to the
compensation awarded. Therefore, if the petitioner is aggrieved, it is open to her to apply
before the Collector for reference under Section 30 read with Section 31 of the Land
Acquisition Act if she is so advised. Section 30 does not postulate any time-limit and as
such it can be made at any point of time and if such application is made, the Collector
may decide the same and pass appropriate order on the said application in accordance
with law. I (sic) necessary, by making reference under the provision of Section 30 and
may also resort to Section 31 if he is so advised according to his own wisdom and
discretion after having examined the dispute raised that there are prima facie disputes
existing which required to be examined. In such circumstances, the Collector is not
entitled to adjudicate the dispute which is the subject-matter of adjudication by a court; it
is only to say that there is no prima facie case raising any dispute and if prima facie case
exists then he has to make the reference under Section 30 read with Section 31. This
decision is to be taken before further disbursement is made. The Collector will also hear
the other no appear (sic) the respondents whom the petitioner will serve a copy of this
order along with a copy of the writ petition within a period of one week from date; in
default, this order will stand recalled."
7. In the aforementioned premise, the contesting respondents herein filed a writ petition
before the Calcutta High Court which was registered as Writ Petition No.27264 of 2006
resulting in passing of the impugned judgment.
8. Mr. Bijan Kumar Ghosh, learned counsel appearing on behalf of the appellant submits
that the impugned judgment cannot be sustained as it was found by this Court that no
such application had been filed.
9. Mr. Chinomy A. Kaladkhar, learned counsel appearing on behalf of respondents 1 to 3,
on the other hand, contends that filing of such an application is not disputed and in that
view of the matter the High Court cannot be said to have committed any error in passing
the impugned judgment.
10. Mr. Tara Chandra Sharma, learned counsel appearing on behalf of the State of West
Bengal, however, brought to our notice that although such an application had, in fact,
been filed, but, in view of the non-compliance of the order passed by the learned Single
Judge, no order could be passed thereupon.
11. Before this Court in the aforesaid appeal a contention was raised that no such
application was filed. It was in the aforementioned situation the abovesaid observations
were made.
12. A review application was filed thereagainst which, by reason of the order dated 14th
December, 2006 was dismissed (although allegedly the said fact was also brought to the
notice of this Court), stating :
"We have gone through the review petition and the relevant documents. In our opinion no
case for review is made out. The review petition is accordingly dismissed."
It, therefore, appears that this Court had, inter alia, proceeded on the basis that no such
application had been filed. First Respondent,
@page-SC2905
however, in her affidavit stated that such an application had been filed. Respondent Nos.
4 to 7 herein, however, in their counter-affidavit stated as under :
"16. Thereafter Smt. Shyamali Das, Respondent No. 1 submitted an application to the
District Magistrate, South 24-Parganas on 8.8.2001 stated to be the Application under
section 30 read with section 31 of the Land Acquisition Act, 1894 without any document
of ownership of the said land. Nowhere in the said Application she mentioned Plot No.
1028 of Mouja Rajapur to be her own against which award was declared. Even she had
not submitted any proof of service of writ petition and copy of order dated 22.9.2000 to
other non-appearing respondents as per order dated 22.9.2000 of Hon'ble Single Judge of
the High Court at Calcutta.
17. After receiving the Application dated 8.8.2001 of Smt. Shyamali Das - Respondent
No. 1, she was once again asked by Special Land Acquisition Officer, South 24-
Paraganas vide Memo No. W.P. No. 19298(W)/2000 L.A. 1957 dated 23.8.2001 to submit
the Land Schedule i.e. name of Mauja, Plot No., Khatian No., Area of the plots with
details of acquisition along with documents in respect of title within 15 days from the
date of receipt of the said letter so that Hon'ble Court's order can be complied with. This
letter was received by Smt. Shyamali Das - Respondent No.1 on 24.8.2001 under her own
signature.
18. As Smt. Shyamali Das - Respondent No.1 had not submitted any document in support
of her claim in her application dated 8.8.2001 as asked for vide letter dated 23.8.2001
abovementioned by the Special Land Acquisition Officer, South-24 Parganas, no further
action could be taken on her application by the Collector, South 24-Parganas, Alipore.
19. On the other hand, as per order dated 22.9.2000 of Hon'ble Single Judge of the High
Court at Calcutta dated 22.9.2000, she could not produce any proof of service of copy of
W.P. No. 19298 (W) of 2000 and copy of order dated 22.9.2000 to other non-appearing
respondents within 7 days from the date of order i.e. 22.9.2000. Therefore, the order
dated 22.9.2000 stood automatically recalled, as directed in the said order."
13. We will, therefore, proceed on the assumption that such an application indeed had
been filed, and the contention made before us in the earlier round of litigation was wrong.
14. The question, however, which arises for consideration is what would be the effect of
the order of the Calcutta High Court allowing the First Respondent to file an appropriate
application before the Collector for reference in terms of Sections 30 and 31 of the Act
which was a conditional order. It was found as of fact that the conditions precedents
therefor were not satisfied.
The consequence laid down in the said order, therefore, ensued, in terms whereof it stood
recalled. If that be so, the order of the High Court directing to dispose of the application
being innocuous was not required to be given effect to. If a conditional order was passed,
with a view to derive a benefit thereunder, it was obligatory on the part of the respondent
to satisfy the condition precedent therefor. If the condition precedent has not been
satisfied, the question of taking advantage thereof would not arise.
15. In this case, as noticed hereinbefore an attempt on the part of the First Respondent to
get herself impleaded as party in the Reference Petition did not fructify. The said order
attained finality. It does not appear that the said respondent was not sure as to whether
such an application had been filed or not. In the judgment of this Court, it will be a bare
repetition to say, that a concession has been recorded. We need not go into the effect of
such a concession as it now transpires that the same was wrongly made.
16. We would not have, therefore, interfered with the impugned judgment despite the
concession made before us but keeping in view the statement made by the State of West
Bengal, we are of the opinion that no fruitful purpose would be served in allowing the
matter to proceed pursuant to the observations made by the learned Single Judge.
17. For the reasons abovesaid, the impugned judgment is set aside. This appeal is
allowed. In the facts and circumstances of the case, there shall be no order as to costs.
Appeal allowed. .
@page-SC2906
AIR 2008 SUPREME COURT 2906 "Bank of India v. Nangia Construction (India) Pvt.
Ltd."
(From : Delhi)
Coram : 2 TARUN CHATTERJEE AND DALVEER BHANDARI, JJ.
Civil Appeal No. 1315 of 2001 with C. A. No. 3592 of 2008 (arising out of SLP (C) No.
3644 of 2007), D/- 15 -5 -2008.
Bank of India v. Nangia Construction (India) Pvt. Ltd. and Ors.
Contract Act (9 of 1872), S.126 - CONTRACT - Bank guarantee - Unconditional on
demand guarantee - Guarantee invoked within validity period - Conduct of Bank to find
excuses for refusing payment - Unfortunate - Permitting such conduct would bring entire
commercial and business transactions to a grinding halt. (Para 14)
Cases Referred : Chronological Paras
1999 AIR SCW 3452 : AIR 1999 SC 3466 (Disting.) 9, 10
AIR 1959 SC 1362 (Disting.) 9, 12
K.N. Bhat, Sr. Advocate, Ms. Swigin, Ms. Akanksha, Ms. Nina Gupta and Ms. Bina
Gupta, with him for Appellant; Mrs. Ginny Jetley, Rautray, Mrs. Kanchan Kaur Dhodi,
Ms. Sangeeta Kumar, Ashwani Garg and Ms. Shivangi Thagela, for Respondents.
Judgement
DALVEER BHANDARI, J. :- This appeal is directed against the judgment and order
dated 26.10.1999 of the High Court of Delhi at New Delhi in FAO (OS) No.81 of 1999.
2. The learned Single Judge of the High Court has taken the view that the invocation of
bank guarantee was within the validity period of the bank guarantee and the bank cannot
decline to make the payment. The Division Bench in the impugned judgment while
dismissing the appeal has clearly observed that the bank guarantee was invoked on 19th
May, 1989 within the validity period of the guarantee. The bank guarantee was merely
renewed under orders of to the court as there was a stay order against encashment of the
bank guarantee. Once the stay order was vacated there was no question of any invocation
of the bank guarantee. In the instant case, the invocation had already taken place within
the validity period. Thereafter, all that was to be done was to intimate the Bank that the
stay has been vacated and that now payment had to be made under the bank guarantee.
3. The Division Bench in great anguish has observed thus :
"It is surprising that a nationalized bank, which has given an unconditional on demand
bank guarantee takes up such a contention. No ground to refuse payment was shown to
the Lower Court or to us. It is surprising that Nationalized Bank wants to use delays of
law in order not to comply with its unconditional obligations under a bank guarantee. The
nationalized bank should know that it is such conduct which is adversely affecting the
faith of the public in banking institutions and in transaction of bank guarantee."
The Court dismissed the appeal with costs.
4. The nationalized bank despite the concurrent findings of both the courts and such a
strong observation of the Division Bench of the High Court has still chosen to file this
appeal before this Court. Even before this Court, this is not disputed that the bank
guarantee was invoked within the validity period of the bank guarantee.
5. Mr. K.N. Bhat, the learned senior counsel appearing for the appellant bank submitted
that there was a substituted agreement of contract, therefore, the invocation of the bank
guarantee by respondent No.2 on 19th May, 1989 was of no consequence.
6. It may be relevant to mention here that after the bank guarantee was invoked, an
application was filed in the court for stay of payment under the bank guarantee. To that
application, the bank was not a party. Initially, an injunction was granted by the High
Court on 29th May, 1989. This was on a condition that the bank guarantee should be kept
alive. This injunction was confirmed on 23rd April, 1990 again on the condition that the
bank guarantee should be kept renewed. The constituent who had obtained injunction and
who was to keep the bank guarantee alive, did not pay the charges of the Bank in respect
of renewals of the bank guarantees. Consequently, the appellant bank refused to renew
the bank guarantee after 26.5.1996. Thus, the beneficiary of the bank guarantee took out
an application wherein the following prayer was made :
"In the circumstances it is, therefore, most humbly and respectfully prayed that the
petitioner be directed to extend the bank guarantee for an initial period of one year and
the petitioner be directed to continue to extend the bank guarantees and furnish the same
to the respondent at least fifteen days before the expiry till the disputes are
@page-SC2907
finally adjudicated upon by arbitration and on the failure of the petitioner to renew the
bank guarantees as aforesaid the respondent may be permitted to encash the above bank
guarantee."
7. As the question was whether the bank guarantee was to be renewed, notice was issued
to the appellant bank to remain present in the court. This was in order to find out whether
they would be willing to renew the bank guarantee. The appellant bank appeared and
made it clear to the court that they were not ready to renew the bank guarantee as
according to them the charges are not being paid.
8. The appellant bank has reiterated the same argument before this court that since the
bank guarantee has not been renewed, therefore, the bank is under no obligation to pay
the amount under the bank guarantee.
9
. Learned counsel for the appellant has placed reliance on two judgments of this court, on
The Union of India v. Kishorilal Gupta and Brothers 1960 (1) SCR 493 and Makharia
Brothers v. State of Nagaland and Others (2000) 10 SCC 503. AIR 1959 SC 1362
1999 AIR SCW 3452

10. In Kishorilal Gupta (supra), this court has held that it was well settled that the parties
to an original contract could by mutual agreement enter into a new contract in
substitution of the old one.
11. There is no quarrel with this proposition. The parties are always at liberty to enter into
a fresh contract but this case has no application to the facts of the present case.
12

. In Makharia Brothers (supra), the question was : what was the State's remedy against the
contractor when the contractor failed to furnish the security deposit in cash or, in lieu
thereof, by a bank guarantee. The State could not have filed a suit requiring the contractor
to do these things for it would have tantamount to asking for a decree of specific
performance, a decree which would have been incapable of enforcement if the contractor
was unable or unwilling to pay out money or put a bank in funds to provide a bank
guarantee. When the contractor declined to extend the terms of the bank guarantee, the
proper course for the State was to terminate the contract on the ground of breach of the
terms thereof, make a claim for damages and recover on the bank guarantee, if necessary
by filing a suit. AIR 1959 SC 1362

13. We are afraid that even this case is of no help to the appellant because the facts of the
instant case are quite different. Admittedly, the bank guarantee has been invoked during
the validity period of the bank guarantee. The bank guarantee was unconditional on
demand bank guarantee. The bank was bound to honour its commitment and pay the
amount of guarantee.
14. It is unfortunate that a nationalized bank is finding excuses for refusing to make the
payment on totally untenable and frivolous grounds. The Division Bench was fully
justified in making observations regarding the conduct of the nationalized bank. The
entire trust, faith and confidence of people depend on the conduct and credibility of the
nationalized bank. In the present day world, the national and international commercial
transactions largely depend on bank guarantees. In case the banks are permitted to
dishonour their commitments by adopting such subterfuges, the entire commercial and
business transactions will come to a grinding halt. This principle has been reiterated in
large number of cases by this court. We do not deem it appropriate to burden this
judgment by reiterating all those judgments.
15. This appeal being devoid of any merit is accordingly dismissed with costs to be paid
to respondent Nos. 1 and 2.
Civil Appeal No. 3592 of 2008 (Arising out of SLP (CIVIL) No.3644 OF 2007)
16. Leave granted.
17. In view of our aforesaid decision, this appeal is also dismissed with costs.
Appeal dismissed. .
AIR 2008 SUPREME COURT 2907 "Oil and Natural Gas Corporation Ltd. v. Atwood
Oceanic International, S. A."
(From : Bombay)
Coram : 2 TARUN CHATTERJEE AND DALVEER BHANDARI, JJ.
Civil Appeal Nos. 1218 and 1219 of 2001, D/- 13 -5 -2008.
Oil and Natural Gas Corporation Ltd. v. Atwood Oceanic International, S. A.
Arbitration Act (10 of 1940), S.30 (since repealed) - ARBITRATION - AGREEMENT -
Award - Validity - Agreement entered into by appellant-Corporation with respondent
Company for carrying out drilling operations in offshore waters of India - Dispute as to
liability
@page-SC2908
to reimburse income tax dues - Employees of respondent becoming liable for income-tax
pursuant to G. O. D/- 31-3-1983 - Respondent claimed reimbursement under terms of
contract - Notification had no retrospective effect - It would not apply to assessment year
1983-84 - However, conclusion in award with regard to payment pertaining to assessment
year 1984-85 is proper. (Paras 14, 15, 16)
Cases Referred : Chronological Paras
1988 Tax LR 1413 (Bom) 13
Vivek K. Tankha, Sr. Advocate, K.R. Sasiprabhu and Ms. Bindu K. Nair, for Appellant;
Shyam Dewan, Sr. Advocate, Ms. Khooshnum R. Daviervala, Rajat Navet and Pradeep
Kumar Bakshi, for Respondent.
Judgement
DALVEER BHANDARI, J. :- These appeals are directed against the judgment of the
High Court of Judicature at Bombay delivered in Appeal Nos. 141 and 142 of 1995 dated
8th February, 2000.
2. Brief facts which are necessary to dispose of these appeals are recapitulated as under :
On 2nd March, 1983, the appellant, Oil and Natural Gas Corporation Limited entered into
an Agreement with the respondent, Atwood Oceanic International, S.A. for carrying out
drilling operations in offshore waters of India and for rendering other related services
with regard to the drilling unit Sagar Pragati belonging to the appellant on the terms and
conditions set forth in the said Agreement.
3. The said Agreement contained an Arbitration Clause 11. The said Arbitration Clause 11
reads as under :-
"Arbitration :
If any dispute, difference or question shall at any time hereafter arise between the parties
hereto or their respective representative concerning anything herein contained or arising
out of these presents or as to the rights, liabilities, or duties of the said parties hereunder
and cannot be mutually resolved the same shall be referred to arbitration, proceedings of
which shall be held at (Bombay) India. Within thirty(30) days of the receipt of the notice
of any dispute, each party shall appoint an arbitrator and such arbitrators shall appoint an
Umpire before they enter upon the reference and not later than one month from the latest
date of their respective appointments. If any of the parties fail to appoint arbitrators
within the specified period or should the two arbitrations fail to agree upon the selection
of an Umpire within the stipulated period, the Hon'ble Chief Justice of the Supreme Court
of India shall nominate the required arbitrator or the Umpire as the case may be, who
shall be a resident of India, but not a national of the country of neither of the parties. The
decision of the arbitrators and failing an agreed decision by them, the decision of the
Umpire shall be final and binding on the parties thereto.
The arbitration proceedings shall be held in accordance with the provisions of the Indian
Arbitration Act, 1940 and the rules made thereunder as amended from time to time. The
arbitrator or the Umpire, as the case may be, shall decide by whom and in what
proportion the arbitrators and Umpire's fees as well as the costs incurred in arbitration
shall borne.
The arbitrators or the Umpire may, with the consent of the parties enlarge the time, from
time, to make and publish their or his Award."
4. At the material time, when the agreement was entered into, the provisions of the Indian
Income-tax Act, 1961 (hereinafter referred to as the '1961 Act') were not applicable
beyond the territorial waters of India, i.e., beyond the limit of 12 nautical miles.
5. On 31st March, 1983 the Government of India issued a notification in exercise of
powers conferred by sections 6(6)(a) and 7(7)(a) of the Territorial Waters, Continental
Shelf, Exclusive Economic Zone, and other Maritime Zones Act, 1976 extending the
provisions of the 1961 Act to the Continental Shelf and Exclusive Economic Zone of
India with effect from 1st April, 1983 with some modifications. It is not necessary to deal
with those modifications because they are not relevant so far as the controversy involved
in the instant case is concerned.
6. The respondent on 5th March, 1985 forwarded an invoice to the appellant claiming that
pursuant to the notification dated 31st March, 1983issued by the Government of India
there was a change in the law with regard to income tax which had resulted in the
employees of the respondent becoming liable for income tax and consequently under the
employment contract, the respondent had incurred additional liability for
@page-SC2909
payment of personnel income tax which the respondent claimed under the terms of the
contract had to be reimbursed. The appellant refuted this claim by its reply dated 15th
March, 1985 and took up the stand that the appellant was not liable to reimburse the
personnel tax dues due to change of law by way of extension of the tax jurisdiction to
offshore areas. On 22nd March, 1986 the respondent sought arbitration of the dispute
between itself and the appellant on the aforesaid issues.
7. On 27th July, 1987, the dispute on the aforesaid issues was referred to arbitration of
Mr. Justice D. V. Patel (Retd.) and Mr. Justice D. M. Rege (Retd.). On 2nd March, 1989,
Mr. Justice D.V. Patel made a speaking award by which he rejected the claim of the
respondent. The other learned arbitrator Mr. Justice D.M. Rege (Retd.) made a note of
disagreement on 15th June, 1989.
8. In view of the disagreement between the two arbitrators, the dispute was referred to the
arbitration of Mr. Justice Tulzapurkar (Retd.) as Umpire. The learned Umpire made his
award on 13th October, 1989 by which the claims of the respondent were allowed.
9. The appellant aggrieved by the award of the Umpire challenged the same before the
learned Single Judge of the Bombay High Court. It was urged by the appellant that there
was error apparent on the face of the record. Reliance was placed on Clauses 5-A and 7 of
the Agreement dated 2nd March, 1983. Clauses 5-A and 7 read as under :
"Clause 5-A: Taxes.
A. Personnel - Any taxes assessed on employees of Contractor and based on income
earned in the performance of work for owner or otherwise shall be the responsibility of
the Contractor."
Clause 7 :
"In the event there occur changes in the laws of Government of India during the course of
the contract from those prevalent on 25.8.1982, which result in increase decrease to the
Contractor's cost of carrying out its duties and responsibilities under this Agreement, then
the increase/decrease in the cost shall be settled and paid/recovered after mutual
discussion."
10. It was contended that in view of the above clauses, tax assessed on the employees of
the contractor was the responsibility of the contractor/claimants. It is contended that in
view of clause 5-A, the learned Umpire erred in awarding the amount by way of
increased costs, particularly because the responsibility was that of the contractor. It was
further contended that taxes assessable on the employees of a contractor and based on the
income earned in the performance of work was one of the items. It cannot constitute
increase in the cost of carrying out responsibilities/duties on the part of the
claimants/contractor under the Agreement. It was further contended that in view of clause
5-A of the Agreement, the appellant was not liable to pay the amounts on the ground of
increased cost of the contract. The learned Single Judge observed as under :
"I do not see any merit in the said submissions advanced on behalf of ONGC. The
contractor has incurred increased costs for the accounting year ending 31st March, 1983
and 31st March, 1984. The Government of India issued Notification on 31st March, 1983
which made the salaries earned by the expatriates taxable for the accounting year ending
31st March 1983 and 31st March, 1984.
The judgment of this Court took the view that the said Notification was prospective and
not retrospective. In the above circumstances, the Arbitrator came to the conclusion that
since the Notification is dated 31st March, 1983, Clause 7 of the said Agreement would
apply. The learned Umpire came to the conclusion that the Notification constituted
change in law of the Central Government during the course of the contract. The said
change admittedly came into force after 25th August, 1982. In the above circumstances,
on reading Clause 2 read with Clause 5 read with Clause 7 of the Agreement, the learned
Umpire came to the conclusion that there was an increase in the contractors' cost under
the Agreement on account of change in the income-tax Law and which resulted in the
increase in the costs. The learned Umpire also came to the conclusion on the basis of the
evidence on record that the claimant/contractor had agreed to pay the taxes assessable on
the expatriates and in the circumstances, the claimant had incurred the increased costs,
and, therefore, the taxes have been paid by the claimants and they are entitled to that
extent to the increased cost. The learned Umpire agreed with the decision of one of
@page-SC2910
the Arbitrators Shri D.M. Rege."
After hearing learned counsel for the parties, the learned Single Judge further observed as
under :-
"In the present case, the dispute referred to the Umpire was a very narrow dispute viz.
whether the contractor was entitled to be reimbursed for the increased cost borne by him
on account of salaries of the expatriates being made eligible to income-tax pursuant to the
Notification dated 31st March, 1983. The learned Umpire, after construing the various
provisions of Clauses 2, 5A and Clause 7 of the Agreement has come to the conclusion
that since the tax law has been changed after 25th August, 1982 and since the contractor
has paid the tax on behalf of its employees/expatriates, the cost of contract had increased
and to that extent under Clause 7 he was entitled to be reimbursed. There is no merit in
the contention of ONGC that there was no increased cost of carrying out the contract on
account of taxes borne by the contractor on behalf of its employees/expatriates.
For the foregoing reasons, there is no merit in the above two Arbitration Petitions. Both
the Arbitration Petitions are accordingly dismissed with costs. Consequently, the
impugned Award dated 13 October, 1989 is made Rule of this court. Decree in terms of
the Award. Further, interest to be paid @ 12% per annum from the date of the Decree till
payment on the respective principal amounts to be calculated in terms of the said
Awards."
11. The appellant aggrieved by the said judgment of the learned Single Judge preferred an
appeal before the Division Bench of the Bombay High Court. The Division Bench heard
the learned counsel for the parties at length and examined the material documents.
12. The Division Bench carefully perused the award of the Umpire and the judgment of
the learned Single Judge and observed that the Umpire has taken one of the possible
views on a fair reading of the contractual terms and this Court cannot interfere with it.
The Court further observed that we perceive no jurisdictional error committed by the
learned Umpire.
13. Before the Division Bench it was contended on behalf of the appellant that at least
with regard to Assessment Year 1983-84, the direction in the award was clearly contrary
to law and, therefore, it ought to be interfered with. The Division Bench found substance
in this argument. The Division Bench held as under :

"Though in para 1 of the impugned Award, the Umpire granted the claims pertaining to
the two Assessment years 1983-84 and 1984-85, in para 2, he referred to the judgment of
this court in Mcdermott International Inc. vs. Union of India and Others, reported in 173
ITR 155 and noticed that the said judgment had taken the view that the Notification dated
31.3.1983 making the Indian Income-tax Act applicable to personnel working within the
Continental Shelf had no retrospective effect and that it would not apply to Assessment
Year 1983-84 (accounting year 1982-83). The Umpire thereafter proceeded to give a
direction that since the said decision was pending in Appeal before the Supreme Court
and there was a possibility of the Respondent being able to recover refund of the income
tax paid by it from the Income Tax Department, the respondent while obtaining the
decree from the appropriate Court, should give a written undertaking to the Court that in
case it recovers a refund of the concerned amount of income tax it shall refund the said
amount to the Appellant. The award states that this direction was given at the instance of
the respondent itself with a view to prevent the respondent from receiving the amount of
income tax paid by it twice over, and for protection of the interest of the Appellant.
1988 Tax LR 1413

Mr. Madon, learned Counsel for the respondent, contended that the directions with regard
to the claims were only contained in para 1 of the award which gave no reasons in
support of the said directions. Consequently, the entire award is a non-speaking award
and is immune from scrutiny of the Court. He explained away the reasons contained in
para 2 of the award as pertaining to the ancillary direction with regard to the undertaking
to be given by the respondent and not with regard to the award itself. It is not possible to
percept the contention that the award has to be read in compartments. In our view, both
paragraphs 1 and 2 of the award have to be read in conjunction. When read in
conjunction, it appears to us, the Umpire was alive to the fact that in Modermott
International (supra) this Court
@page-SC2911
had taken the view that the Notification dated 31.3.1983 had ho retrospective effect and
would not apply to Assessment Year 1983-84. If this was the law, then the respondent's
employees were not liable for making payment of income tax during the year 1983-84 for
income earned while carrying out work beyond the territorial waters of India.
Consequently, there was no question of increased cost of services within the meaning of
Clause 7 of the Contract between the parties, or was there any scope for passing on a non-
existing liability to the Appellant. At least to this extent, it appears to us that this
contention must succeed."
14. The Division Bench in the concluding para of the judgment observed that the learned
Single Judge erred in not interfering with the direction contained in the award pertaining
to assessment year 1983-84, but the conclusion of the learned Single Judge with regard to
the direction pertaining to assessment year 1984-85 is perfectly justified and needs no
interference.
15. The Division Bench partly allowed the appeal filed by the appellant and set aside the
direction contained in the Umpire's award with regard to the payment of Rs.28,26,359/-
for the year ending 31st March, 1983 (assessment year 1983-84) and uphold the rest of
the judgment of the learned Single Judge. The Division Bench further directed that the
decree is modified to the extent that there shall be a decree in accordance with the award
only pertaining to assessment year 1984-85, together with interest as directed in the
award and as granted by the learned Single Judge.
16. The appellant aggrieved by the said judgment preferred these appeals before this
Court. The appellant reiterated the same argument before this Court. The scope for
interference by this Court is extremely limited in a case of this nature. We have carefully
perused the entire material on record and analysed the impugned judgment. In our
considered opinion, no interference is called for. The appeals being devoid of any merit
are accordingly dismissed. In the facts and circumstances of the case, we direct the
parties to bear their own costs.
Appeal dismissed. .
AIR 2008 SUPREME COURT 2911 "Associated Construction, M/s. v. Pawanhans
Helicopters Pvt. Ltd."
(From : 2007 (4) Bom CR 26)
Coram : 2 TARUN CHATTERJEE AND HARJIT SINGH BEDI, JJ.
Civil Appeal Nos. 3376-3377 of 2008 (arising out of SLP (C) Nos. 17335-17336 of
2007), D/- 7 -5 -2008.
M/s. Associated Construction v. Pawanhans Helicopters Pvt. Ltd.
(A) Arbitration Act (10 of 1940), (since repealed) S.30, S.33 - ARBITRATION -
CONTRACT - Powers of Court - Award of contract - Delay in completion by contractor
attributable to respondents - Time was essence of contract - Terms of contract providing
no escalation during pendency of contract - However, such embargo would not be carried
beyond period of completion of contract - Award of arbitrator awarding price escalation
to contractor - Not so unconscionable that it required interference - Moreso, as Court
does not sit as one in appeal over award of the arbitrator.
2007 (4) Bom CR 26, Reversed. (Paras 7, 10, 11)
(B) Constitution of India, Art.14 - EQUALITY - CONTRACT - Govt. contract -
Contractor compelled to issue "No Dues Certificate" if he wanted to claim dues for works
done - Voluminous correspondence over span of almost 2 years between submission of
first final bill and second final bill - Plea by principal that submission of contractor that
"No Dues Certificate" was under duress, was afterthought not acceptable.
2007 (4) Bom CR 26, Reversed. (Para 16)
(C) Constitution of India, Art.133 - APPEAL - CONTRACT - COSTS - Costs - Govt.
contract - Non-payment of dues of contractor - Correspondence between parties showing
that contractor was compelled to give "No dues Certificate" by respondent so as to claim
payment towards completed work - Denial of payment thereafter by respondent on
ground that contractor had given "No dues Certificate" and that now he cannot turn round
and make claim - Respondent thus, taking advantage of beleguered contractor - Costs of
Rs. 10,000/- imposed on respondent. (Para 17)
Cases Referred : Chronological Paras
2006 AIR SCW 6222 (Foll.) 9
(2006) 12 Scale 149 (Foll.) 16
@page-SC2912

2004 AIR SCW 198 : AIR 2004 SC 1330 16


AIR 1989 SC 1034 (Foll.) 8
AIR 1987 SC 2316 (Foll.) 7
Shyam Divan, Sr. Advocate, Ajay Kumar, Ms. Richa Srivastava and Ms. Indu Sharma,
for Appellant; Raju Ramchandran, Sr. Advocate, J. Buther, Amit Kumar, Ms. Geeta Kalra
and Ms. Susmita Lal, for Respondent.
Judgement
1. HARJIT SINGH BEDI, J. :-Leave granted.
2. The respondent, Pawanhans Helicopters Pvt. Ltd. (hereinafter called "Pawanhans") a
Government of India undertaking, floated two tenders for allocation of work for
construction of a compound wall and a bridge over a nala. Pursuant to the aforesaid
information, several tenders were received and the tenders of the appellant (hereinafter
called the "contractor") were ultimately accepted. Pursuant to the aforesaid, two formal
agreements providing for the terms and conditions of the contract in the shape of general
conditions of the contract and special conditions of the contract governing the execution
of work were duly signed on 12th October 1999. As per the contract the work was
required to be completed within four months. It appears that on account of some delay
which was attributable to Pawanhans, the work did not proceed as per schedule and the
contractor accordingly informed Pawanhans by letters dated 15th February 1990, 23rd
February 1990, 24th March 1990, 26th June 1990 and 6th July 1990 that the work was
getting delayed as the requisite facilities for its completion had not been provided and
highlighting several factors attributable to it had supervened which had led to the delay.
The contractor also in the meanwhile vide letters dated 27th July 1990 and 6th August
1990 requested the respondent to release the outstanding bills against the work already
completed and also requested for the "Virtual Completion Certificate" vide letter dated
25th August 1990. As some work on the compound wall still remained to be completed,
the contractor agreed to take up this assignment subject to waiver of the discount of 8.2%
which was to be given to Pawanhans till then and the completed works were duly handed
over to Pawanhans on the 12th November 1990. The contractor had also submitted a bill
dated 23rd June 1991 and it was conveyed to Pawanhans that it expected compensation
on account of the Variation in the terms of the contract. Pawanhans thereupon advised the
contractor to submit a final bill which too was submitted. The bill was verified by
Pawanhans and referred to the contractor yet again with objections. The contractor vide
letter dated 21st November 1991 disputed the verification as being without any
foundation and also reserved its right to seek arbitration. After a protracted
correspondence, Pawanhans vide letter of 9th December 1991 advised the contractor to
submit a "No Claim Certificate" as a pre-condition for the release of the balance payment.
The contractor wrote to Pawanhans that it was in dire need of finances and was being
subjected to duress but nevertheless submitted a "No Dues Certificate" dated 17th
February 1992 once again specifically highlighting that the same was being issued under
duress. It appears that despite the issuance of the aforesaid certificate, Pawanhans still did
not release the payment on which the contractor wrote another letter dated 5th May 1992
and several letters thereafter but again to no effect, and on the contrary received a letter
dated 8th June 1992 from Pawanhans asking for a "No Dues Certificate" as per the
enclosed specimen without attaching any condition to the same. The contractor, now in a
desperate situation, submitted yet another "No Claim Certificate" dated 18th June 1992 as
per directions. After receiving the aforesaid document, Pawanhans in its letter dated 9th
February 1993 informed the contractor that a period of two months would be required for
the scrutiny of its bills and vide letter dated 21st May 1993 also intimated that the bills
had been submitted for verification by the Architect/Engineer as per the terms of the
contract and that in case it was willing to defray the payment, the matter could be referred
to arbitration. The contractor finally received a communication dated 8th June 1993
pointing out that as all payments due under the contract had been made and as a "No
Dues Certificate" had been furnished, no further amount was due. The contractor
accordingly served a notice dated 28th June 1993 on Pawanhans invoking the clause
relating to arbitration. The matter was referred to arbitration by two registered Architects
as per the clause. The contractor submitted its statement of claim for the outstanding
amount plus compensation and damages on 6th August 1994. The arbitrators passed two
awards on 31st December 1996, one with respect to the contract
@page-SC2913
for the compound wall and the second for the construction of the bridge awarding certain
amounts to the contractor. Aggrieved by the awards, Pawanhans filed two separate
petitions under sections 30 and 33 of the Arbitration Act, 1940 before the Bombay High
Court for a direction that the awards be set aside. The learned Single Judge in his
judgment and order dated 9th December 1998 held that clauses 18 and 34 of the contract
when read together, provided for the payment of escalation charges as the work had not
been completed within four months on account of the fault on the part of the respondent
and that the said clauses did not prohibit such a payment, more particularly as time was
the essence of the contract and as the contract was not on a fixed price, the prohibition of
escalation was if at all to be read during the period of contract only. The learned Single
Judge also repelled the arguments of the respondent that after having submitted the final
bill on 25th October 1991, it was not open to the appellant herein to submit a second final
bill on 2nd February 1993 by observing that the payment received on the 4th July 1993 as
a consequence of the bills submitted on 25th October 1991, was under duress and it is on
that account that the appellant had given the aforesaid certificate. Some objections raised
by the respondent herein were however accepted by the learned Single Judge and the
award was accordingly modified and it is the admitted case that the aforesaid
modification has been accepted and was not challenged before the Division Bench by the
contractor.
2. Two appeals were thereafter filed by Pawanhans before the Division Bench of the
Bombay High Court. The Division Bench vide its order dated 7th June 2007 allowed the
appeals and set aside the order dated 9th December 1998 of the learned Single Judge as
also the two awards dated 31st December 1996 by highlighting as a preface that it could
not be disputed that the scope for interference by the court under section 30 or 33 of the
Arbitration Act was limited as the court could not sit as a court of appeal on the decisions
arrived at by the arbitrator. The Court then applied the aforesaid principle to the facts of
the case and relied on clauses 18 and 34 ibid observed that a plain reading of the said
clauses did not visualize any claim for escalation or reduction towards the cost of the
work and again reiterated that clause 34 of the agreement prohibited the contractor from
claiming any extra amount on account of fluctuation of price. The Court further observed,
somewhat in contradiction, that a remedy towards the escalation of price had been
provided by clause 43 of the contract and clause 43-1 (E) specifically provided, the
procedure whereby such a claim could be made and as the procedure prescribed by the
clause had not been adopted, it was not open to the contractor to contend before the
arbitrator that it was entitled to some payments on account of price escalation. The Court
finally concluded that :
"Once it is clear that the respondents are not entitled to claim escalation charges and the
entire dispute, which is the subject matter of the appeals being related to the escalation
charges, the impugned orders, to the extent they confirm the award in relation to the
escalation charges, are liable to be set aside and the petitions filed by the appellants
challenging the awards in relation to the grant of the escalation charges are liable to be
allowed to that extent. Consequently, the claims for interest on the amount of damages
awarded towards the escalation are also liable to be set aside."
3. The Division Bench then examined the issues raised by the contractor as to whether
that "No Due Certificate" had been given under duress and held that there was no
evidence to show that the said certificate had been given under duress or coercion and as
the certificate itself provided a clearance of no dues, the contractor could not now turn
and say that any further payment was still due on account of the second final bill. The
Division Bench accordingly allowed the appeal. The matter is before us in these
circumstances.
4. Mr. Shyam Divan, the learned senior counsel for the contractor, has raised several
arguments before us during the course of the hearing. He has first pointed out that the
awards rendered by the arbitrator were non-speaking and in this view of the matter, the
scope for judicial interference was extremely limited and interference with the findings of
the Arbitrators was, therefore, not called for. He has also pleaded that clauses 18 and 34,
as per their plain interpretation themselves visualized a claim for escalation where the
delay had been caused by the opposite party and that in any case, the bar on the
escalation, if at all, could be
@page-SC2914
restricted only for the period of contract i.e. four months and not thereafter. He has also
submitted that clause 43-1(C) on which reliance had been placed by the Division Bench
for non-suiting the contractor, was misplaced as this clause too did not specifically or
even by implication whittle down the effects of clauses 18 and 34. It has also been argued
that the finding of the Division Bench that there was no duress on the contractor relating
to the issuance of the "No Claim Certificates" was incorrect in the light of the voluminous
evidence to the contrary on record.
5. Mr. Raju Ramachandran, the learned senior counsel appearing for Pawanhans has
fairly and at the very outset pointed out that the award in question was non-speaking and
as such the scope for interference by the court was limited. He has further contended that
it would perhaps be difficult to read into the clauses a complete bar towards escalation, as
a court would be reluctant to visualize such a bar in the light of some unforeseen
situations that might arise in the execution of a work and the gates, thus, could not for
ever be closed, but has submitted that clause 43 provided for such an opening and as this
procedure had not been adopted by the contractor, the claim under clauses 18 and 34 was
not maintainable. He has also submitted that the "No Dues Certificate" having once being
given by the contractor, it was not open to it to make a volte-face and to challenge the
said certificate on the ground that it had been given under duress and the finding of the
Division Bench on this point was, therefore, correct.
6. We have heard the learned counsel for the parties and gone through the record. As
would be apparent, the matter would rest on an interpretation of clauses 34, 43 (1) and (2)
of the General Conditions of the Contract and clause 18 of the Special Conditions of the
Contract. We reproduce hereinbelow the clauses abovementioned :
"34. The contractor shall not claim any extras for fluctuation of price and the contract
price shall not be subject to any rise or fall of prices.
43 (1) E. Architect's instructions issued in regard to the postponement of any work to be
executed under the provisions of this contract; and if the written application is made
within a reasonable time of it becoming apparent that the progress of the work or of any
part thereof has been affected as aforesaid :
Then the Architect shall ascertain the amount of such loss and/or expense. Any amount
from time to time so ascertained shall be added to the amount which would otherwise be
stated as due in such certificate.
43 (2) The provisions of this condemn are without prejudice to any other rights and
remedies which the contractor may possess.
18. It is specifically pointed out that the contractor shall not be entitled to any
compensation whatsoever on account of :
1. Any delay in supply of any material.
2. Any increase in costs of any material.
3. Any subsequent increase in cost of any material due to increase in other charges like
Railway, Steamer, freights or taxes and duties.
4. Any increase in labour costs."
7
. We have examined the arguments raised by the learned counsel in the light of the
aforesaid and other provisions. It is the admitted position that as per clause 38, the date of
the commencement of the contract was 1st November, 1989 and the date stipulated for
the completion of the work was 28th February 1990. It is also clear from sub-clause (7)
of clause 1 of the General Conditions that time would be the essence of the contract. We
also see from clause 43 aforequoted that this clause has within itself the clear indication
that the embargo placed by clauses 18 and 34 was not sacrosanct as has been found by the
Division Bench as there could be a situation where the contractor had suffered loss for
whatever reasons which was required to be reimbursed as per procedure prescribed in
clause 43. Clause 43 (2) also specifically provided that clause 43 was without prejudice
to any other rights and remedies that the contractor might possess. We find from a reading
of the judgment of the Division Bench that the contractor has been nonsuited on the plea
that it had failed to proceed under clause 43. On the contrary we believe that Clause 43 is
a clause which should be read in aid of the contractor as it clearly provides for indemnity
in case there was a delay in the completion of the work which could be attributable to
Pawanhans. We are, further, of the opinion that even assuming for a moment that there
could be no price escalation during the period of 4 AIR 1987 SC 2316, (Paras 4, 6, 7)

@page-SC2915
months i.e. during the pendency of the contract, such embargo would not be carried
beyond that period as time was the essence of the contract, The learned Division Bench
has relied upon a large number of judgments in support of its decision that in case of a
clause barring the escalation in the price, it was not open to the contractor to claim any
amount under that head. A perusal of the aforesaid judgments, however, do not show any
provision in terms of clause 43, and that in any case, these judgments pertain to a claim
of price escalation during the period of contract. It must also be borne in mind that a court
does not sit as one in appeal over the award of the arbitrator and if the view taken by the
arbitrator is permissible, no interference is called for on the premise that a different view
was also possible. We also feel that in commercial transactions all situations cannot be
visualized and the positive and unchallenged finding in the present case is that the delay
in the execution of the work was occasioned on account of reasons attributable to
Pawanhans. It cannot, therefore, be said that the award of the arbitrator was so
unconscionable that it required interference. In MCD vs. M/s. Jagan Nath Ashok Kumar
and Anr. (1987) 4 SCC 497, it was observed thus :
"In this case, there was no violation of any principles of natural justice. It is not a case
where the arbitrator has refused cogent and material factors to be taken into
consideration. The award cannot be said to be vitiated by non-reception of material or
non-consideration of the relevant aspects of the matter. Appraisement of evidence by the
arbitrator is ordinarily never a matter which the court questions and considers. The parties
have selected their own forum and the deciding forum must be conceded the power of
appraisement of the evidence. In the instant case, there was no evidence of violation of
any principle of natural justice. The arbitrator in our opinion is the sole judge of the
quality as well as quantity of evidence and it will not be for this Court to take upon itself
the task of being a judge of the evidence before the arbitrator. It may be possible that on
the same evidence the court might have arrived at a different conclusion than the one
arrived at by the arbitrator but that by itself is no ground in our view for setting aside the
award of an arbitrator."
and further concluded :
"After all an arbitrator as a judge in the words of Benjamin N. Cardozo, has to exercise a
discretion informed by tradition, methodized by analogy, disciplined by system, and
subordinated to "the primordial necessity of order in the social life".
8

. P. M.Paul vs. Union of India (1989) Supp 1 SCC 368 is a case which is almost identical
on facts. In this matter the work could not be completed during the period of the contract
and the contractor was accordingly granted extension of time to complete the same. By
an order of this Court, the dispute was referred to an Arbitrator on the reference as to who
was responsible for the delay in the completion of the work, what were to be the
repercussions of the delay and how to apportion the responsibility and the consequences.
The arbitrator made an award in favour of the contractor which was duly challenged by
the Union of India with the matter finally reaching this Court at the instance of the
contractor and this is what the Court had to say. AIR 1989 SC 1034, (Para 13)

"It was submitted that if the contract work was not completed within the stipulated time
which it appears was not done then the contractor has got a right to ask for extension of
time, and he could claim difference in price. This is precisely what he has done and has
obtained a portion of the claim in the award. It was submitted on behalf of the Union of
India that failure to complete the contract was not the case. Hence, there was no
substance in the objections raised. Furthermore, in the objections raised, it must be within
the time provided for the application under Section 30 i.e., 30 days during which the
objection was not specifically taken, we are of the opinion that there is no substance in
this objection sought to be raised in opposition to the award. Once it was found that the
arbitrator had jurisdiction to find that there was delay in execution of the contract due to
the conduct of the respondent, the respondent was liable for the consequences of the
delay, namely, increase in prices. Therefore, the arbitrator had jurisdiction to go into this
question. He has gone into that question and has awarded as he did."
9

. A similar view has been taken by this Court in K. N. Sathyapalan (D) By LRs. vs. State
of Kerala and Anr. (2006) 12 SCALE 654. It has been 2006 AIR SCW 6222, (Paras
36 and 37)

@page-SC2916
held as under :
"We have intentionally set out the background in which the Arbitrator made his award in
order to examine the genuineness and/or validity of the appellant's claim under those
heads which had been allowed by the Arbitrator. It is quite apparent that the appellant was
prevented by unforeseen circumstances from completing the work within the stipulated
period of eleven months and that such delay could have been prevented had the State
Government stepped in to maintain the law and order problem which had been created at
the work site. It is also clear that the rubble and metal, which would have been available
at the departmental quarry at Mannady, had to be obtained from quarries which were
situated at double the distance, and even more, resulting in doubling of the transportation
charges. Even the space for dumping of excess earth was not provided by the respondents
which compelled the appellant to dump the excess earth at a place which was far away
from the work site entailing extra costs for the same.
In the aforesaid circumstances, the Arbitrator appears to have acted within his jurisdiction
in allowing some of the claims on account of escalation of costs which was referable to
the execution of the work during the extended period. In our judgment, the view taken by
the High Court was on a rigid interpretation of the terms of contract and the
Supplemental Agreement executed between the parties, which was not warranted by the
turn of events."
10. We are, therefore, of the opinion in the light of the aforesaid judgments, that it was
open to the contractor to contend that it was liable to be compensated on account of the
fact that delay had been occasioned on account of reasons attributable to Pawanhans. It is
significant that the Division Bench of the High Court has been silent on this aspect of the
matter and has not referred to the finding of the learned Single Judge with regard to the
responsibility for the delay.
11. We are further of the opinion that clauses 43 and 43 (1) and (2) when read together
clearly visualize escalation of price on account of reasons beyond the control of the
contractor and attributable to the other side. Moreover clause 43 (2) clearly states that the
remedy under clause 43(1) would be in addition to such other remedy that may be open to
the contractor under the other provisions.
12. We have also gone through the record with respect to the finding of the Division
Bench that there was no duress or coercion on the contractor which had compelled it to
give a "No Dues Certificate". Mr. Raju Ramachandran has, however, submitted that the
story about duress was an afterthought in the background that the first final bill had been
submitted by the contractor on the 3rd June 1991 and the second final bill on 2nd
February 1993 i.e. almost 2 years later and that in any case, a second final bill was not
visualized under the contract. He has submitted that the observation of the arbitrator that
submission of the second final bill was sanctioned as a trade practice was without any
basis. We have gone through the record in the light of the submissions of the learned
counsel. We first refer to the letter of the contractor of 11th July 1990 to which reference
has been made by the Division Bench requesting Pawanhans to ensure a regular power
supply. The letter of 27th July 1990 by the contractor refers to the statement of accounts
submitted by it and requests for payment as per the accounts which had been cleared by
the Architect. It is to be noted that these letters are on the record and were written by the
contractor at the time when the work was in the process of completion. The desperate
tone of the contractor is however supported by the letter of 10th January 1991 in which it
was noted that though repeated requests had been made for the payment at least against
the bills certified by the Architect, a huge amount had been blocked arbitrarily over a
long period of time and a request was made for its release. The letter dated 21st
November 1991 is again a reminder to Pawanhans asking for payment and that in case
there was a dispute, the matter be referred to the arbitrator and submitting that payment
should be made at least with respect to those dues which had been certified by the
Architect. The letter dated 9th December 1991 from Pawanhans to the contractor shows
that payment could be considered provided the contractor submitted a "No Claim
Certificate". It appears that such certificate was indeed issued but with no result on which
the contractor in his letter dated 26th December 1991 in reply to the letter dated 9th
@page-SC2917
December, 1991, once again submitted that the payments be released insofar as they had
been certified by the Architects/Consultants and if there was a dispute regarding the other
payments, they should be referred to an arbitrator and in desperation further adds :
"However, if you want to hold us to economic duress by not paying what you wish to pay,
without "No Claim Certificate", we shall treat it as "Duress" and issue you such a
certificate much against our willingness as we cannot afford to liquidate our dues by such
a certificate.
Please do not hold us to a ransom and arrange to pay. In case you would still like to insist,
let us know, so that we could issue you such a certificate under duress as we have serious
financial problems."
13-14. It appears that despite the pleading tone of the aforesaid letter no payment was
made on which the contractor wrote yet another letter dated 17th February 1992 in which
it was submitted as under:
"In spite of our claim statements, you have insisted on "No Claim Certificate", we hereby
give you this certificate that we have "No Claims" and hence you pay us what you might
have worked out as our "Final Dues".
In case, you have a particular draft in which a "No Claim" Certificate need be issued to
receive our dues of our bill, please let us have the draft, or else this letter may be treated
as the certificate of "No Claim" from our side."
15. When no action was taken, another letter dated 5th May 1992 was addressed to
Pawanhans by the contractor stating that as they were facing economic duress on account
of the payment being held back, and as a "No Claim Certificate" had been issued, the
payment be defrayed as promised or else they might have to refer the matter to the
arbitrator. The letter dated 8th June 1992 is again tell-tale and we reproduce the contents
hereunder :
"Kindly let us know what is it that we have to do to get money which you say is payable
but only on your extracting "No Claim" certificate under duress. Please take note if you
fail to pay us our dues, we shall be constrained to take you to court for which you will
blame yourself if any inconvenience is caused. It is a clear 15 days notice please."
16. It appears however that no steps were taken on which the contractor addressed a letter
dated 2nd February 1993 for payment of dues and again stated that if the payment was
not made, the dispute should be referred to the arbitrator. In response to this letter,
Pawanhans in its letter dated 9th February 1993 replied that the matter was under scrutiny
and it would take about 2 months for verification and that the contractor would be
informed in due course. As no reply was received, a letter dated 21st May 1993 was
addressed by the contractor relating to the undertaking that the enquiry would be
completed within 2 months but complaining that nothing had been done and on the
contrary on 8th June 1993 the claim for any payment was rejected by Pawanhans
observing that as a "No Dues Certificate" had been submitted by the contractor, the
question of any balance payment being due did not arise. It is at this stage that the
contractor had invoked the clause for arbitration. We have reproduced the correspondence
in extenso to show that the contractor was compelled to issue a "No Dues Certificate" and
in this view of the matter, it could not be said that the contractor was bound by what he
had written. It is also clear that there is voluminous correspondence over a span of almost
2 years between the submission of the first final bill on 3rd June 1991 and the second
final bill dated 2nd February 1993 and as such the claim towards escalation or the plea of
the submission of a "No Dues Certificate" under duress being an afterthought is not
acceptable. In M/s. Ambica Construction vs. Union of India (2006) 12 SCALE 149 it was
observed as under :
"A glance at the said clause will immediately indicate that a No Claim Certificate is
required to be submitted by a contractor once the works are finally measured up. In the
instant case the work was yet to be completed and there is nothing to indicate that the
works, as undertaken by the contractor, had been finally measured and on the basis of the
same a No Objection Certificate had been issued by the appellant. On the other hand,
even the first Arbitrator, who had been appointed, had come to a finding that No Claim
Certificate had been given under coercion and duress. It is the Division Bench of the
Calcutta High Court which, for the first time, came to a conclusion that such No Claim
Certificate had not been submitted under coercion and duress.

From the submissions made on behalf of 2004 AIR SCW 198

@page-SC2918
the respective parties, and in particular from the submissions made on behalf of the
appellant, it is apparent that unless a discharge certificate is given in advance, payment of
bills are generally delayed. Although, Clause 43(2) has been included in the General
Conditions of Contract, the same is meant to be a safeguard as against frivolous claims
after final measurement. Having regard to the decision in the case of Reshmi
Constructions's (supra), it can no longer be said that such a clause in the contract would
be an absolute bar to a contractor raising claims which are genuine, even after the
submission of such No Claim Certificate."
17. We are therefore of the opinion that the judgment of the Division Bench is erroneous
and we accordingly set it aside. The judgment of the learned Single Judge is accordingly
restored. In the facts and circumstances of the case, in that Pawanhans has taken
advantage of a beleaguered contractor, and has behaved in a most unbecoming manner in
pushing it ever deeper into the chasm, the contractor will have its costs which are
computed at Rs. 10,000/-. The appeals are accordingly allowed.
Appeals allowed. .
AIR 2008 SUPREME COURT 2918 "National Capital Territory of Delhi v. Umesh
Kumar"
(From : Delhi)
Coram : 2 Dr. A. PASAYAT AND G. S. SINGHVI, JJ.
Criminal Appeal No. 699 of 2003, D/- 19 -6 -2008.
National Capital Territory of Delhi and Anr. v. Umesh Kumar.
Arms Act (54 of 1959), S.17 - Constitution of India, Art.134 - FIRE ARMS - APPEAL -
LICENSE - SUPREME COURT - Arms licence - Cancellation - Licence held by police
constable - Cancelled on his involvement in criminal case - Cancellation revoked by High
Court in view of acquittal of constable licensee in criminal case - Fact of pendency of
appeal against acquittal held irrelevant - Appeal to Supreme Court - Stay granted by
Supreme Court to judgment of High Court - In interest of justice stay directed to be
continued till disposal of appeal. (Para 8)

P.P. Malhotra, A.S.G., Ms. Varuna Bhandari Gugnani, D.S. Mahra, Avatar Singh Rawat
and Mrs. Anil Katiyar, for Appellants; K.K. Tyagi, I. Ahamad and P. Narasimhan, for
Respondent.
Judgement
Dr. ARIJIT PASAYAT, J. :- Heard learned counsel for the parties.
2. The challenge in this Appeal is to the order passed by the Division Bench of the Delhi
High Court allowing the Criminal Writ Petition No. 207/2001 by Order dated 7.12.2001.
3. The background facts in nutshell are as under :-
The respondent had been granted license for a .315 Bore Rifle. The Lt. Governor, Delhi
upheld the order passed by the Deputy Commissioner of Police (Licensing) Delhi, in
Appeal in terms of Section 18 of the Arms Act, 1959 (in short the Act). The Deputy
Commissioner had directed cancellation of license on the ground that the respondent who
was working as a Constable was involved in criminal offence and therefore, it was not in
the interest of justice to continue currency of the license granted to him. Therefore, it was
cancelled. The appeal before the Lt. Governor, as noted above, did not bring any relief to
the appellant. The High Court was moved thereafter.
4. The High Court noted factual background as follows :
The respondent's license was cancelled on the ground that that he was found involved in
case FIR No. 254/1991 for offences punishable under Sections 302, 307 read with
Section 34 of the Indian Penal Code, 1860 (in short IPC) and Sections 25, 27 and 54 of
the Arms Act. On the recommendation of the Crime Branch, notice was issued by the Dy.
Commissioner of Police (Licensing) to show cause as to why the arms license should not
be cancelled in the interest of public safety and peace as he has rendered himself to be
unsuitable to hold license. The license was cancelled in exercise of power conferred
under Section 17(3) of the Act. The Lt. Governor of Delhi as noted above dismissed the
appeal. The High Court noted that the respondent was deployed in Delhi Police and
during his involvement in the aforesaid crime was suspended and remained suspended till
he was acquitted by the learned Additional Sessions Judge, Delhi. After the acquittal,
suspension was revoked and he was reinstated in service on 19.1.2000. Before the High
Court the stand of the present respondent was that the grounds on which the licence was
cancelled
@page-SC2919
did not exist any further and there was no reason as to why the appeal should have been
dismissed.
5. It was pointed out by the High Court that on acquittal the respondent was found to be
fit enough to continue in his post. When he was found to be so fit, there was no reason as
to why he should not have a license for a gun. A reference was also made to Clause 7 of
Section 17 of the Act which provides that if the conviction is set aside on appeal or
otherwise, the suspension or revocation of the license by the court convicting the holder
of the license shall become void. According to the High Court, on the same analogy,
when the respondent's involvement was not found acceptable the licence which was
cancelled, ought to have been restored. Learned counsel for the State submitted before
the High Court that the State had already filed an appeal questioning the acquittal of the
respondent. The High Court was of the view that filing of an appeal cannot have any
effect on the judgment of the acquittal. In case acquittal is set aside it was open to the
authorities to take necessary action.
6. Learned counsel for the appellant submitted that the fact that appeal was pending
consideration, should have been given due weightage. Even otherwise, a person serving
in the Police Force and charged with serious offences should not be allowed to have a
license. According to him grant of license is discretionary and there is no right in that
sense to have a license. Learned counsel for the respondent, on the other hand, supported
the judgment of the High Court.
7. We find that while issuing notice this Court had directed stay of the impugned order by
Order dt. 2.12.2002. Subsequently, leave was granted by Order dated 2.5.2003 and the
interim order was made absolute. In other words, the High Court's Order is not operative
as of now.
8. We are of the considered view that interest of justice would be best served if the orders
passed by this Court staying operation of the High Court's order are continued till the
disposal of the Appeal by the High Court. We make it clear giving this direction, we have
not expressed any opinion on the merits of the case. However, we request the High Court
to dispose of the Appeal, if pending, as early as practicable preferably by the end of year
2008.
9. The Appeal is accordingly disposed of.
Order accordingly. .
AIR 2008 SUPREME COURT 2919 "Babulal Badriprasad Varma v. Surat Municipal
Corporation"
(From : Gujarat)
Coram : 2 S. B. SINHA AND V. S. SIRPURKAR, JJ.
Civil Appeal No. 3203 of 2008 (arising out of SLP (C) No. 568 of 2007), D/- 2 -5 -2008.
Babulal Badriprasad Varma v. Surat Municipal Corporation and Ors.
(A) Gujarat Town Planning and Urban Development Act (Presi. Act 27 of 1976), S.52,
S.65 - Gujarat Town Planning and Urban Development Rules (1979), R.26, R.33 -
TOWN PLANNING - ESTOPPEL - WAIVER - PRINCIPLES - Draft Scheme -
Appellant filed objections - However did not pursue them eventually - While proceedings
relating to allotment of final plot were in progress he even did not file any objection
thereto - Scheme attaining finality - Principle of waiver would preclude him from
claiming equitable relief of issuance of special notice in respect of final allotment of plot
- Moreso, as no such notice was mandatory under R.26.
Evidence Act (1 of 1872), S.115. (Paras 32, 33, 35)
(B) Gujarat Town Planning and Urban Development Act (Presi. Act 27 of 1976), S.52,
S.65 - Gujarat Town Planning and Urban Development Rules (1979), R.26, R.33 -
TOWN PLANNING - ALLOTMENT OF PREMISES - APPEAL - Draft Scheme -
Objection, as to - Not pursued by appellant - Appellant also not filing objection to final
allotment of plot - Scheme attaining finality and final allotment of plot made in favour of
respondent - Plea by appellant that he was ready and willing to pay some reasonable
amount to the respondent in whose favour plot has been finally allotted - Issuance of any
such direction is legally impermissible.
Constitution of India, Art.133. (Para 36)
Cases Referred : Chronological Paras
2008 AIR SCW 2446 : AIR 2008 SC 1771 (Foll.) 29
2006 AIR SCW 4988 : AIR 2006 SC 3304 : 2006 CLC 1464 (Foll.)35
2004 AIR SCW 4758 : AIR 2004 SC 4282 : 2004 Lab IC 3466 (Foll.) 34
2003 AIR SCW 313 : AIR 2003 SC 858 : 2003 Lab IC 689 (Foll.) 34
@page-SC2920

1997 AIR SCW 2719 : AIR 1997 SC 2745 (Ref.) 25


1994 Supp (2) SCC 511 (Ref.) 25
1992 AIR SCW 436 : AIR 1992 SC 752 (Ref.) 6, 8, 17
1991 AIR SCW 2515 : AIR 1991 SC 2130 (Ref.) 6, 8, 18
AIR 1975 SC 67 33
AIR 1969 SC 634 (Ref.) 27
AIR 1967 SC 1373 (Foll.) 29
AIR 1957 SC 425 (Foll.) 33
1894 AC 129 : 63 LJ PC 49 33
(1861) 30 Beav 65 : 54 ER 813 33
11 NSWLR 153 33
U.U. Lalit, Sr. Advocate, Ms. Sumita Hazarika, with him for Appellant; Prashant G.
Desai, Tejas Patel and S.C. Patel, for Respondent.
Judgement
1. S.B. SINHA, J. : - Leave granted.
2. Interpretation and/or application of the provisions of the Gujarat Town Planning and
Urban Development Act, 1976 (for short "the Act") and the Rules framed thereunder
known as the Gujarat Town Planning and Urban Development Rules, 1979 (for short "the
Rules") is in question in this appeal which arises out of a judgment and order dated
27.12.2006 passed by a Division Bench of the High Court of Gujarat at Ahmedabad in
Letters Patent Appeal No. 1611 of 2006 arising out of a judgment and order dated
23.11.2006 passed by a learned Single Judge of the said Court in SCA No. 7092 of 2001.
3. Before embarking upon the issue involved in this appeal, we may notice the admitted
fact of the matter.
The Government of Gujarat in exercise of its power conferred upon it under Section 65 of
the Act made a scheme in respect of the town of Umra, Surat on 1.06.1999.
Plot Nos. 17/7 and 17/8 were owned by Respondent No. 4 herein. Appellant was a tenant
under the said respondent in respect of Plot No. 17/8 admeasuring 1067 sq.m. He used to
run a business of marble and stone therein. A road widening project was proposed in
terms of the said scheme. Notices therefor were issued both to the appellant as also the
respondent No. 4.
4. Appellant objected thereto. He, however, did not pursue his case in regard to the
proposal for widening of the road.
For the said public purpose, viz., widening of the road, 867 sq. m. of land was taken over
leaving only 200 sq. m. of land. With a view to give effect to the provisions of the Act
and the Rules framed thereunder, proceedings were initiated for allotment of the said land
in terms of the Act. 20% of the land was taken over without payment of any
compensation. In respect of the proceedings initiated for the purpose of re-allotment of
the land, despite a public notice, the appellant did not file any objection. He did not take
any part in the proceedings therefor. Respondent No. 4 was allotted a final plot bearing
No. 157 and the said 200 sq. m. of land of plot No. 17/8 has merged in final plot No. 165
owned by the respondent No. 3.
The Scheme was notified in the year 1999. Respondent No. 1 herein which is the
statutory agency in terms of the Act for the purpose of implementation of the Scheme
issued a notice under Section 67 of the Act upon the respondent No. 4 on or about
15.01.2000. As he did not respond thereto, a notice under Section 68 of the Act was
served on him on 31.03.2000 stating :
"As per the said approved preliminary scheme the plot No. 157 is allotted to you. And, its
pole demarcations were done by the town planning officer at site. The said Final
Plot/Original Plot is allotted in lieu of your No. 17/7, 17/8 paiki land. And, the said land
is now vested in the Municipal Corporation from 1.7.1999, and is of the ownership of the
Municipal Corporation. Thereafter the notice below section 67 for the change in
occupation was issued on 15.1.2000 to you. In spite of this you have not handed over the
possession. Therefore, as per the Gujarat Town Planning and Urban Development Rules,
1979, rule 33 the undersigned in exercise of powers conferred below section 68(1) and
8(2) of the Bombay Provincial Municipalities Act and below the section 68 notice under
the Gujarat Town Planning and Urban Development Act this is to inform you that as
shown in the sketch on the reverse the premises marked should be vacated within 7 days
from receipt of the notice and hand over the possession to the Surat Municipal
Corporation. If you fail to do so then on completion of the stipulated time limit as per the
Rule 33 of the Gujarat Town Planning and Urban Development Rules, 1979 the said land
and the occupation on the same will be summarily evicted and your occupation will be
removed and if you obstruct/interfere on it after taking away the possession you trespass
then as
@page-SC2921
provided under Rule. 33 of the Gujarat Town Planning and Urban Development Rules,
1979 the action as per the section 188 of the Indian Penal Code will be initiated against
you before the Criminal Court, please take note of the same."
5. The validity and/or legality of the said notice was questioned by the appellant by filing
a writ petition in the High Court of Gujarat inter alia contending that the purported final
allotment of plot No. 165 in favour of the respondent No. 3 and allotment of final plot
No. 157 in favour of the respondent No. 4 were made without issuing any notice as
envisaged under Sections 52 and 53 of the Act.
In the said writ petition, it was prayed:
"8. On the facts and circumstances mentioned herein above, the Petitioner prays to your
Lordships that:
(A) Be pleased to issue writ of Mandamus or writ in the nature of Mandamus or
appropriate writ, order or direction, quashing and setting aside the impugned action of
acquiring and demolishing the structures available on the land in question, i.e., Original
Plot No. 17/A - R.S. No. 17/P, situated at Umra, Surat."
6. A learned Single Judge of the High Court dismissed the said writ petition inter alia
opining that the interest of landlord and tenant being common and in absence of any inter
se dispute between them even if any portion of the land which remained in possession of
the tenant was included in the Scheme, the proper remedy would be to claim
compensation to that extent, holding :
"18. It appears that in the said decision, the Apex Court while considering the scheme on
the touch-stone of the mandatory procedure to be followed by the authority under the
Bombay Town Planning Rules, has given directions to provide alternative
accommodation based on the earlier decision in case of Jaswantsingh Mathurasingh and
upheld the scheme. Such is not the issue in the present case nor there is any complaint by
the tenant that any special notice was not served or that the mandatory procedure for
finalization of the scheme is not followed. Further, it appears that if the interest of the
landlord and of the tenant is common and in absence of any inter se dispute between the
landlord and tenant, even if any portion of the land which is in possession of the tenant is
included in the scheme, the proper remedy for the tenant would be to claim for
compensation to that extent and if such compensation is not received by him, he may
resort to proper remedy available for recovery of the compensation to the extent of the
area in his occupation. At least on ground that the tenant is in occupation, it would not be
a case for interference with the scheme which is sanctioned and made a part of the
statute. Suffice it to say that the tenant will be at liberty to resort to appropriate
proceedings against the landlord for the inter se rights and also for entitlement of the
compensation. But if the area of original plot No. 17/8 is included in the final scheme and
in exchange of the original plot held by Keshav Gramini of 17/8 and 17/7, the final plot is
already allotted and as observed earlier it was even otherwise in the ownership of the
original holder and it is only on account of inter se dispute the other persons are lawfully
occupying the land, the tenant cannot insist that his landlord must be allotted the land of
final plot No. 157 simultaneously, when he is to be evicted or deprived of the portion of
the land of original plot No. 17/8. Therefore, in my view considering the peculiar facts
and circumstances of the present case, the decision of the Apex Court in case of
Mansukhlal (supra) cannot be made applicable to the present case."1991 AIR SCW 2515
1992 AIR SCW 436

7. A Division Bench of the High Court dismissed an intra-court appeal preferred


thereagainst.
8. Mr. U.U. Lalit, learned senior counsel appearing on behalf of the appellant, in support
of this appeal, inter alia would submit :
(i) The provisions of Sections 52 and 81 being imperative in character, no acquisition of
land is permissible without service of any notice upon the persons interested which would
include a tenant in occupation and carrying on business thereon.
(ii) A tenant having regard to the provisions of the Transfer of Property Act or otherwise
having an interest in the property cannot be deprived therefrom without following the
procedure established by law and without initiation of any proceedings for acquisition of
land.
(iii) The tenant's interest being distinct and separate could not have been held to be
merged with the interest of the landlord,
@page-SC2922
either for the purpose of allotment of a final plot or otherwise in favour of the landlord.
(iv) Appellant having a right over the remaining 200 sq. m. of the land of original plot
No. 17/8 should be allowed to continue thereupon and final allotment made in favour of
the respondent No. 3 to that extent should be cancelled.
Mr. Lalit in support of his contention strongly relied upon a decision of this Court in
Mansukhlal Jadavji Darji and Others v. Ahmedabad Municipal Corporation and Others
[(1992) 1 SCC 384] and Jaswantsingh Mathurasingh and Another v. Ahmedabad
Municipal Corporation and Others [1992 Supp (1) SCC 5]. 1992 AIR SCW 436
1991 AIR SCW 2515

9. Mr. Prashant G. Desai, learned counsel appearing on behalf of the respondent No. 1, on
the other hand, would submit :
(i) Public notices having been issued in terms of the Rule 26 of the Rules, an objection
which would nullify the Scheme cannot be entertained at this stage.
(ii) Respondent No. 1 Corporation merely being interested in the implementation of the
Scheme is entitled to obtain vacant possession from him so as to enable it to deliver it to
the respondent No. 3 in whose favour plot No. 165 has been finally allotted.

(iii) The Scheme in terms of Sub-section (3) of Section 65 of the Act having become a
part of the Act, validity thereof cannot be questioned at this stage as modification of the
Scheme, if any, will have to undergo the entire process once over again which is not
contemplated under the Act.
10. The Act was enacted to consolidate and amend the law relating to the making and
execution of development plans and town planning schemes in the State of Gujarat.
11. It is not necessary for us to delve deep into the statutory scheme. Suffice it to say that
Chapter IV of the Act deals with control of development and use of land included in the
development plans. Chapter V of the Act provides for town planning schemes.
Section 40 of the Act empowers the appropriate authority to make one or more schemes.
A declaration of intention to make a scheme is to be notified whereafter a draft scheme
may be published. Section 45 provides for reconstitution of the plots, sub-section (2)
whereof inter alia enables allotment of a final plot from an original plot by transfer of any
adjoining lands/Section 52 contemplates issuance of a notice in a prescribed manner and
in the prescribed form.
12. Section 52 of the. Act provides for the contents of preliminary and final scheme. It
inter alia provides for giving of a notice by the Town Planning Officer as follows :
"(1) In a preliminary scheme, the Town Planning Officer shall, -
(i) after giving notice in the prescribed manner and in the prescribed form to the persons
affected by the scheme, define and demarcate the areas allotted to, or reserved for, any
public purpose, or for a purpose of the appropriate authority and the final plots;
(ii) after giving notice as aforesaid, determine in a case in which a final plot is to be
allotted to persons in ownership in common, the shares of such persons;"
Further, sub-section (3) of Section 65, Sections 67 and 68 of the Act read as under:
"65 - Power of Government to sanction or refuse to sanction the scheme and effect of
sanction -
(3) On and after the date fixed in such notification, the preliminary scheme or the final
scheme, as the case may be, shall have effect as if it were enacted in this Act.
67 - Effect of preliminary scheme
On the day on which the preliminary scheme comes into force -
(a) all lands required by the appropriate authority shall, unless it is otherwise determined
in such scheme, vest absolutely in the appropriate authority free from all encumbrances;
(b) all rights in the original plots which have been re-constituted into final plots shall
determine and the final plots shall become subject to the rights settled by the Town
Planning Officer.
68 - Power of appropriate authority to evict summarily
On and after the date on which a preliminary scheme comes into force, any person
continuing to occupy any land which he is not entitled to occupy under the preliminary
scheme shall, in accordance with the prescribed procedure, be summarily evicted by the
appropriate authority."
@page-SC2923
13. Rules 26(1), 26(3) and 33 of the Rules read as under:
"26. Procedure to be followed by Town Planning Officer under section 51 and under sub-
section (1) of section 52 - (1) For the purpose of preparing the preliminary scheme and
final scheme the Town Planning Officer shall give notice in Form H of the date on which
he will commence his duties and shall state the time, as provided in Rule 37 within which
the owner of any properly or right which is injuriously affected by the making of a Town
Planning Scheme shall be entitled under section 82 to make a claim before him. Such
notice shall be published in the Official Gazette and in one or more Gujarati newspapers
circulated within the area of the appropriate authority and shall be pasted in prominent
places at or near the areas comprised in the scheme and at the office of the Town
Planning Officer.
(3) The Town Planning Officer shall, before proceeding to deal with the matters specified
in section 52, publish a notice in Form H in the Official Gazette and in one or more
Gujarati newspapers circulating within the area of the appropriate authority. Such notice
shall specify the matters which are proposed to be decided by the Town Planning Officer
and State that all persons who are interested in the plots or are affected by any of the
matters specified in the notice shall communicate in writing their objections to the Town
Planning Officer within a period of twenty days from the publication of notice in the
Official Gazette. Such notice shall also be posted at the office of the Town Planning
Officer and of the appropriate authority and the substance of such notice shall be pasted
at convenient places in the said locality.
33. Procedure for eviction under Section 68. - (1) For eviction under section 68, the
appropriate authority shall follow the following procedure, viz. :
(a) The appropriate authority shall in the first instance serve a notice upon a person to be
evicted requiring him, within such reasonable time as may be specified in the notice, to
vacate the land.
(b) If the person to be evicted fails to comply with the requirement of the notice, the
appropriate authority shall depute any Officer or Servant to remove him.
(c) If the person to be evicted resists or obstructs the officer or Servant deputed under
clause (b) or if he re-occupies the land after eviction, the appropriate authority shall
prosecute him under section 188 of the Indian Penal Code."
14. Before embarking upon the rival contentions, we may also notice that the provisions
of the Bombay Town Planning Rules, 1955 (for short "the Bombay Rules") are in part
materia with 'the Rules'.
Rule 21 of the Bombay Rules provides for the Procedure to be followed by the Town
Planning Officer. It makes it obligatory on the part of the officer to give notice of the date
on which he will commence his duties and shall state therein the time, within which the
owner of any property or rights which Is injuriously affected by the making of the town
planning scheme shall be advertised in one or more newspapers published in the regional
language and circulating within the jurisdiction of the local authority and shall be posted
in prominent places at or near the area comprised in the scheme and at the office of the
Town Planning Officer. Sub-rule (3) of Rule 21 of the Bombay Rules provides for serving
of a Special notice of at least three clear days' upon the person interested in any plot or in
any particular area comprised in the scheme, before the Town Planning Officer proceeds
to deal in detail with the portion of the scheme relating thereto. Sub-rule (4) makes it
Imperative upon the Town Planning Officer to "give all persons affected by any particular
(sic) of the scheme sufficient opportunity of stating their views and shall not give any
decision till he has duly considered their representations, if any". Sub-rule (5) provides
for recording a brief minute setting out the points at issue and the necessary particulars if
during the proceedings, it appears to the Town Planning Officer that there are conflicting
claims or any difference of opinion with regard to any part of the scheme.
15. Rule 26 of the Rules do not contemplate service of individual notice. It prescribes
service of notice in Form H. A copy of the notice in the said Form is kept at the office of
the Town Planning Officer during office hours. Any person affected by the proposal of
the Town Planning Scheme is entitled to inspect the Scheme in the office where
arrangements for explaining the scheme proposals are made. It furthermore provides that
any person entitled to claim
@page-SC2924
damages in terms of Section 82 of the Act should communicate the details of his claim to
the Town Planning Officer. Section 81 of the Act enables the State to transfer of right
from original to final plot or extinction of such right.
A Town Planning Scheme, therefore, envisages calling for objection from the persons
concerned for three purposes:
(i) in regard to draft scheme;
(ii) lodging of any claim for payment of compensation;
(iii) participation in the matter of allotment of final plots.
16. We may, however, notice that Rule 21 of the Bombay Rules provides for notice under
Sub-rule (3) thereof and a reasonable opportunity of hearing under Sub-rule (5) thereof.
Sub-rule (3) of Rule 21 of the Bombay Rules provides for issuance of a special notice
upon the person interested in any plot or in any particular plot comprised in the Scheme.
17

. We may also take notice of the decision of this Court in Mansukhlal Jadavji Darji
(supra) wherein this Court opined that Sub-rule (3) of Rule 21 of the Bombay Rules was
mandatory in nature, subject, of course, to the condition that on the crucial date, viz.,
when the Town Planning Scheme is notified in the official gazette, he, whether an owner
or tenant or sub-tenant, must be in possession of the property. 1992 AIR SCW 436

18

. In Jaswantsingh Mathurasingh (supra), it was reiterated that a tenant or a sub-tenant is a


person interested and is entitled to notice. In that context, it was held : 1991 AIR
SCW 2515
"8. The question is whether the tenant or a sub-tenant is a person interested and is entitled
to notice. It is obvious that under Section 105 of Transfer of Property Act, a lease creates
right or an interest in enjoyment of the demised property and a tenant or a sub-tenant is
entitled to remain in possession of the demised property until the lease is duly terminated
and eviction takes place in accordance with law. Therefore, a tenant or a sub-tenant in
possession of a tenement in the Town Planning Scheme is a person interested within the
meaning of Rules 21 (3) and (4) of the Rules. But he must be in possession of the
property on the crucial date i.e. when the Town Planning Scheme is notified in the official
Gazette. Every owner or tenant or a sub-tenant, in possession on that date alone shall be
entitled to a notice and opportunity."
19. Rule 21(3), however, of the Bombay Rules has been amended in tune with Rule 26 of
the Rules. Amended rules are in pari materia with Rule 26 of the Rules.
20. Appellant was a tenant in respect of plot No. 17/8. Plot No. 17/7 was not a plot
contiguous thereto. They were separated not only by a road but also by various other
plots.
21. It is also not in dispute that the appellant filed an objection in regard to the draft
scheme but did not eventually pursue the same. The draft scheme was approved. 867 sq.
m. of land had been acquired for public purpose out of the said plot No. 17/ 8. While the
proceedings relating to allotment of final plot were in progress, he even did not file any
objection thereto. If he intended to claim any interest in a portion of plot No. 17/8 either
for the purpose of obtaining compensation for acquisition of a part of the land or to
continue to have possession over 200 sq. m. of land in plot No. 17/8, it was obligatory on
his part to take part in the proceedings. Whether irrespective of Rule 26 of the Rules
which prescribes for issuance of a general public notice, any special notice upon the
appellant was required to be served by the State or by the authority, in our opinion,
cannot be gone into by us in these proceedings for the first time. Validity of Rule 26 of
the Rules, had never been questioned. It had also not been contended that the said Rule is
ultra vires Section 52 of the Act.
22. A person interested in continuing to keep possession over a property and/or a part of
the amount of compensation must lay his claim before the appropriate authority at the
appropriate stage. If in absence of any such claim filed by the appellant, the authorities
have proceeded to finalise allotment of final plot in favour of the respondent Nos. 3 and 4
herein, it is too late in the day to contend that the entire scheme should be re-opened.
We would consider the effect of Sub-section (3) of Section 65 of the Act a little later, but,
we may at this juncture notice that the respondent No. 3 in whose favour plot No. 165 has
been allotted which includes 200 sq. m.
@page-SC2925
of land purported to be in possession of the appellant had nothing to do with the dispute
between the appellant and his landlord the respondent No. 4. Respondent No. 4 was in
possession of a contiguous plot. Respondent No. 4 was owner of both plot Nos. 17/7 and
17/8. He was, therefore, in his own right entitled to final allotment of some plot.
23. We would, however, assume that it was obligatory on the part of the State to serve a
special notice upon the appellant. The question, however, would be : what would be the
consequence of non-compliance thereof vis-a-vis the conduct of the appellant himself ?
24. A person may waive a right either expressly or by necessary implication. He may in a
given case disentitle himself from obtaining an equitable relief particularly when he
allows a thing to come to an irreversible situation.
25. Different statutes provide for different manner of service of notice. The Bangalore
Development Authority Act, 1976 provides that every person whose name appears in the
assessment list or land revenue records shall be served with notice. [See Sureshchandra
C. Mehta v. State of Karnataka and Others 1994 Supp (2) SCC 511]

In West Bengal Housing Board etc. v. Brijendra Prasad Gupta and Others, etc. [AIR 1997
SC 2745], it was opined that the authority is not required to make a roaming enquiry as to
who is the person entitled to notice. 1997 AIR SCW 2719

26. We have referred to the said decisions only to show that the requirements in regard to
the manner of service of notice varies from statute to statute and there exists a difference
between the Bombay Rules and the Rules.
27. We are, however, not unmindful of the fact that a statute of town planning ex facie is
not a statute for acquisition of a property. An owner of a plot is asked to part therewith
only for providing for better facilities of which he would also be a beneficiary. Every step
taken by the State does not involve application of the doctrine of eminent domain.

In this case, the appellant did not oppose the draft scheme. It accepted that the State had a
right to do so. Existence of a public purpose and increase in the valuation of the property
was admitted. There exists a distinction in the action of the planning authority as regards
vesting of a property in it and one so as to enable it to create a third party interest vis-a-
vis for the purpose of re-allotment thereof. In the former case, the vesting of the land may
be held to be an act of acquisition, whereas in the latter, it would be distribution of certain
benefits having regard to the purpose sought to be achieved by a statute involving town
planning. It was on that legal principle, this Court in State of Gujarat v. Shantilal
Mangaldas and Ors. [1969 (3) SCR 341], opined that when a development is made, the
owner of the property gets much more than what would have he got, if the same remained
undeveloped in the process as by reason thereof he gets the benefit of living in a
developed town having good town planning. AIR 1969 SC 634

28. Section 67 of the Act provides that all lands required by the appropriate authority
shall, unless it is otherwise determined in such scheme, vest absolutely in the appropriate
authority free from all encumbrances with effect from the date on which the preliminary
scheme comes into force. What would be the quantum of payment of compensation
therefor is also provided in Section 82 of the Act. It is in the aforementioned situation, a
claim is to be made before the authority whenever a notice in Form H is published. If a
claim is not filed, the person, who is said to be injuriously affected, does so at its own
peril. Had such a claim been filed, the authority before making final allotment could have
considered the competing claims wherefor a large number of factors were required to be
taken into consideration, viz., the location of the land, the area of the land, the nature of
right, etc.
29. When a statute makes an elaborate provision as regards the formalities required to be
undergone at every stage by the local authority, the State Government and other
authorities concerned in preparing and making the final Town Planning Scheme, the same
should be considered to be exhaustively. [See Maneklal Chhotalal and Ors. v. M.G.
Makwana and Ors. [(1967) 3 SCR 65].

In Maneklal Chhotalal (supra), it was held : AIR 1967 SC 1373, Para 50

"49. Therefore, having due regard to the substantive and procedural aspects, we are
satisfied that the Act imposes only reasonable restrictions, in which case,
@page-SC2926
it is saved under Article 19(5), of the Constitution. The considerations referred to above
will also show that the grievance of the petitioners that Article 14 is violated, is also not
acceptable."

[See also Bhikhubhai Vithlabhai Patel and Ors. v. State of Gujarat and Anr. 2008 (4)
SCALE 278] 2008 AIR SCW 2446

30. We are, however, not oblivious that in a given situation, a question may also arise as
to whether the restrictions imposed by a statute are reasonable or not.
31. It is not a case where the State by its acts of omissions and commissions was unjustly
enriching itself. It was a dispute between two private parties as regards the right to obtain
final allotment; the principles underlying the same are not in dispute. What is in dispute is
the distribution of quantum thereof between two competing claimants, viz., landlord and
tenant. We do not mean to say that under no circumstances the appellant was entitled to
allotment of a portion of the properly or mandatory compensation in lieu thereof from the
landlord. But, we intend to emphasise that he has lost his right to enforce the same in a
public law forum. He has no enforceable claim against the State at this Juncture. He may
pursue his claim only against the respondent No. 4 in an appropriate proceedings wherein
for certain purposes the State or the authorities may also be impleaded as a party. Even if
he had a claim he would be deemed to have waived the same for the reasons stated
hereinafter.
32. It is not in dispute that :
(a) Appellant although filed an objection with regard to the draft scheme, did not choose
to pursue it.
(b) He did not file objections for re-allotment and did not participate in the proceedings
following acquisition instituted by the authorities under the Act.
In view of the above, the issue is whether it was open to him to assert his purported right
to special notice in respect of the final allotment in the instant case given the fact that he
did not pursue his objections to the draft scheme and subsequently did not
object/participate during the proceedings for re-allotment.
33. It has been noticed by us hereinbefore that under Rule 26 of the Rules applicable in
the instant case, as distinguished from the Bombay Rules (wherein special notice is
required), no special notice is mandatorily required to be served. Assuming, however, that
it was obligatory for the State to issue notice to the appellant, the question is whether the
principle of waiver precludes him from claiming equitable relief in this case due to his
earlier conduct which allowed the entire process of acquisition and allotment to become
final. We are of the opinion that even if he had any such right, he waived the same.
In Halsbury's Laws of England, Volume 16(2), 4th edition, para 907, it is stated :
"The expression "waiver' may, in law, bear different meanings. The primary meaning has
been said to be the abandonment of a right in such a way that the other parry is entitled to
plead the abandonment by way of confession and avoidance if the right is thereafter
asserted, and is either express or implied from conduct. It may arise from a party making
an election, for example whether or not to exercise a contractual right . . . Waiver may
also be by virtue of equitable or promissory estoppel; unlike waiver arising from an
election, no question arises of any particular knowledge on the part of the person making
the representation, and the estoppel may be suspensory only. . . Where the waiver is not
express, it may be implied from conduct which is inconsistent with the continuance of the
right, without the need for writing or for consideration moving from, or detriment to, the
party who benefits by the waiver, but mere acts of indulgence will not amount to waiver;
nor may a party benefit from the waiver unless he has altered his position in reliance on
it"

As early as 1957, the concept of waiver was articulated in a case involving the late
assertion of a claim regarding improper constitution of a Tribunal in Manak Lal v. Dr.
Prem Chand (AIR 1957 SC 425) in the following terms : Para 8 of AIR

"It is true that waiver cannot always and in every case be inferred merely from the failure
of the party to take the objection. Waiver can be inferred only if and after it is shown that
the party knew about the relevant facts and was aware of his right to take the objection.
As Sir Johan Romilly M. R. has observed in Vyvyan v. Vyvyan [(1861) 30 Beav 65, 74;
54 ER 813, 817] "waiver or acquiescence, like election, presupposes that the person to be
bound is fully cognizance
@page-SC2927
of his rights, and, that being so, he neglects to enforce them, or chooses One benefit
instead of another, either, but not both, of which he might claim".

In The Director of Inspection of Income Tax (investigation), New Delhi and Another v.
Pooran Mal and Sons and Another [(1975) 4 SCC 568] the issue was regarding waiver of
benefits under a statute of limitation. It was stated : AIR 1975 SC 67, Para 13

"13. We may in this connection refer to the decision in Wilson v. McIntosh. In that case
an applicant to bring lands under the Real Property Act filed his case in court under
Section 21, more than three months after a caveat had been lodged, and thereafter
obtained an order that the caveator should file her case, which she accordingly did. It was
held that he had thereby waived his right to have the caveat set aside as lapsed under
Section 23. The Privy Council held that the limitation of time contained in Section 23
was introduced for the benefit of the applicant, to enable him to obtain a speedy
determination of his right to have the land brought under the provisions of the Act and
that it was competent for the applicant to waive the limit of the three months, and that he
did waive it by stating a case and applying for and obtaining an order upon the appellant
to state her case both, which steps assumed and proceeded on the assumption of the
continued existence of the caveat. They referred with approval to the decision in Phillips
v. Martin where the Chief Justice said :
"Here there is abundant evidence of waiver, and it is quite clear that a man may by his
conduct waive a provision of an Act of Parliament intended for his benefit. The caveator
was not brought into Court in any way until the caveat had lapsed. And now the
applicant, after all these proceedings have been taken by him, after doubtless much
expense has been incurred on the part of the caveator, and after lying by and hoping to
get a judgment of the Court in his favour, asks the Court to do that which but for some
reasons known to himself he might have asked the Court to do before any other step in
the proceedings had been taken. I think he is altogether too late. It is to my mind a clear
principle of equity, and I have no doubt there are abundant authorities on the point, that
equity will interfere to prevent the machinery of an Act of Parliament being used by a
person to defeat equities which he has himself raised, and to get rid of a waiver created
by his own acts."
The legal principle emerging from these decisions is also stated in Craies on Statute Law
(6th Edn.) at page 369 as follows :
"As a general rule, the conditions imposed by statutes which authorise legal proceedings
are treated as being indispensable to giving the court jurisdiction. But if it appears that the
statutory conditions were inserted by the legislature simply for the security or benefit of
the parties to the action themselves, and that no public interests are Involved, such
conditions will not be considered as indispensable, and either party may waive them
without affecting the Jurisdiction of the court."
(Emphasis supplied)
Applying the above principles to the present case, it must be held that the benefit of
notice provided under the Act and Rules being for the benefit of the Appellant in which
no public interests are involved, he has waived the same.
34

. Significantly, a similar conclusion was reached in the case of Krishna Bahadur v. Purna
Theatre [(2004) 8 SCC 229], though the principle was stated far more precisely, in the
following terms: 2004 AIR SCW 4758, Paras 8 and 9

"9. The principle of waiver although is akin to the principle of estoppel; the difference
between the two, however, is that whereas estoppel is not a cause of action; it is a rule of
evidence; waiver is contractual and may constitute a cause of action; it is an agreement
between the parties and a party fully knowing of its rights has agreed not to assert a right
for a consideration.
10. A right can be waived by the party for whose benefit certain requirements or
conditions had been provided for by a statute subject to the condition that no public
interest is involved therein. Whenever waiver is pleaded it is for the party pleading the
same to show that an agreement waiving the right in consideration of some compromise
came into being. Statutory right, however, may also be waived by his conduct."
(Emphasis supplied)

[See also Bank of India v. O.P. Swarnakar (2003) 2 SCC 721] 2003 AIR SCW 313

35
. In Ramdev Food Products Pvt. Ltd. 2006 AIR SCW 4988, Paras 75 and 76

@page-SC2928
v. Arvindbhai Rambhai Patel and Ors. [2006 (8) SCALE 631], this Court observed :
"The matter may be considered from another angle. If the first respondent has expressly
waived his right on the trade mark registered in the name of the appellant-Company,
could he claim the said right indirectly ? The answer to the said question must be
rendered in the negative. It is well-settled that what cannot be done directly cannot be
done indirectly. The term 'Waiver' has been described in the following words : "Waiver is
the abandonment of a right in such a way that the other party is entitled to plead the
abandonment by way of confession and avoidance if the right is thereafter asserted, and is
either express or implied from conduct. A person who is entitled to rely on a stipulation,
existing for his benefit alone, in a contract or of a statutory provision may waive it, and
allow the contract or transaction to proceed as though the stipulation or provision did not
exist. Waiver of this kind depends upon consent, and the fact that the other party has
acted upon it is sufficient consideration. It seems that, in general, where one party has, by
his words or conduct, made to the other a promise or assurance which was intended to
affect the legal relations between them and to be acted on accordingly, then, once the
other party has taken him at his word and acted on it, so as to alter his position, the party
who gave the promise or assurance cannot afterwards be allowed to revert to the previous
legal relationship as if no such promise or assurance had been made by him, but he must
accept their legal relations subject to the qualification which he has himself so
introduced, even though it is not supported in point of law by any consideration. [See 16
Halsbury's Laws (4th edn) para 1471]"
In this view of the matter, it may safely be stated that the appellant, through his conduct,
has waived his right to an equitable remedy in the instant case. Such conduct precludes
and operates as estoppel against him with respect to asserting a right over a portion of the
acquired land in a situation where the scheme in question has attained finality following
as a result of the appellant's inaction.
36. Mr. Lalit submits that his client is ready and willing to pay some reasonable amount
to the respondent No. 3 in whose favour plot No. 165 has been finally allotted. Issuance
of any such direction, in our opinion, is legally impermissible.
37. We, therefore, are of the opinion that in this case, no relief can be granted to the
appellant. He may, however, take recourse to such remedy which is available with him in
law including one by filing a suit or making a representation before the State.
38. For the reasons aforementioned, the appeal is dismissed. No costs.
Appeal dismissed. .
AIR 2008 SUPREME COURT 2928 "Satna Stone and Lime Co. Ltd., M.P. v. Union of
India"
(From : AIR 2000 MP 101)
Coram : 2 TARUN CHATTERJEE AND DALVEER BHANDARI, JJ.
Civil Appeal Nos. 5456-5457 of 2000, D/- 8 -5 -2008.
Satna Stone and Lime Co. Ltd., M.P. etc. v. Union of India and Anr. etc.
(A) Arbitration Act (10 of 1940), S.30 - ARBITRATION - Setting aside of award - Error
apparent on face of record - Means error of law apparent on face of record. (Para
10)
(B) Arbitration Act (10 of 1940), S.30 - ARBITRATION - RAILWAY - SUPREME
COURT - Setting aside of award - Claim by mining company against revised
maintenance charges levied by Railways for siding provided - Arbitrator allowing claim
though barred - Decision of Supreme Court upholding power of Railways to increase
charges for services rendered also disregarded - Award liable to be set aside. (Paras
5, 6, 7, 19)
Cases Referred : Chronological Paras
1997 AIR SCW 1422 : AIR 1997 SC 1324 (Disting.) 8
1995 AIR SCW 1575 : AIR 1995 SC 2189 (Disting.) 10
1995 AIR SCW 3584 : AIR 1995 SC 2423 13
AIR 1990 SC 1426 (Rel. on) 17
AIR 1989 SC 777 (Disting.) 9
AIR 1989 SC 1263 (Disting.) 9
AIR 1988 SC 1172 (Rel. on) 5
AIR 1968 SC 22 (Rel. on) 6, 14
AIR 1967 SC 378 (Rel. on) 15
AIR 1955 SC 468 (Rel. on) 18
AIR 1923 PC 66 (Rel. on) 16
Sanjay Kr. Ghosh and Avijit Bhattacharjee, for Appellants; T.S. Doabia, Sr. Advocate,
Rakesh Garg, Ms. Kiran Bhardwaj, B.K. Prasad, Manpreet Singh
@page-SC2929
Doabia and Ms. Sushma Suri, for Respondents.
Judgement

DALVEER BHANDARI, J. :- These appeals are directed against the judgment of the
Division Bench of the High Court of Madhya Pradesh at Jabalpur dated 6.7.1999
delivered in Misc. Appeal No. 1058 of 1995 and Misc. Petition No.l of 1991.
reported in AIR 2001 MP 101

2. Brief facts relevant to dispose of these appeals are as under :-


The appellant Satna Stone and Lime Company Ltd., Satna entered into an agreement with
the General Manager, G.I.P. Railway, Government of India, on 23.3.1897, whereby a
siding was provided to the company for clearance of its goods. The agreement was
revised from time to time and the last such agreement was executed on 1.10.1942
(hereinafter called "the agreement").
3. The Railway administration on 24.8.1968 informed the appellants that the maintenance
charges would be levied at the rate of 4½% per annum instead of 2½%. The respondent
Railway administration raised the bills for the period 1.11.1963 to 31.3.1975. The
appellants paid the bill under protest on 22.6.1976. The appellants filed an application
under section 20 of the Arbitration Act, 1940. On the request of the appellants, the matter
was referred to the Arbitration. The Arbitrator decided the claim in favour of the
appellants. The objections raised by the Railways under section 30 of the Arbitration Act,
1940 were rejected by the learned District Judge and the award was made the rule of the
court.
4. The miscellaneous application was preferred by the Railways before the Division
Bench of the Madhya Pradesh High Court against the said order of the District Judge.
The respondent Railways submitted that the claim of the appellants was clearly barred by
limitation because the demand crystallized on 1.5.1975 and the claim was made on
8.8.1978.
5

. This Court in Union of India v. L.K. Ahuja and Co. (1988) 3 SCC 76 that the arbitrator
would decide unless, however, if on admitted facts a claim is found at the time of making
an Order under Section 20 of the Arbitration Act, to be barred by limitation. AIR
1988 SC 1172

6. In Union of India v. Indian Sugar Mills Association, Calcutta and Another AIR 1968
SC 22, the court dealt with this aspect and came to the conclusion that Railway
administration was competent to increase the rate chargeable for the services rendered.
7. This aspect of the matter was also ignored by the arbitrator and the High Court rightly
concluded that the award is based on wrong principles of law. If the decision of the
arbitrator is contrary to law laid down by this Court then it would be justified in
interfering with the award.
8

. The learned counsel for the appellants has placed reliance on the judgment of this Court
in B.V. Radha Krishna v. Sponge Iron India Ltd. (1997) 4 SCC 693 to demonstrate that
the High Court was not justified in substituting its own view in place of arbitrator's view
as if it was dealing with an appeal. This is forbidden by a series of judgments of this
Court. 1997 AIR SCW 1422

. Learned counsel for the appellants has also placed reliance on Food Corporation of
India v. Joginderpal Mohinderpal and Another (1989) 2 SCC 347. In this case, the court
relying on dia (1989) 1 SCC 411 reiterated the legal position that the court cannot sit in
appeal over the views of the arbitrator by reexamining and reassessing the material.
AIR 1989 SC 1263
AIR 1989 SC 777

10

. In Hindustan Construction Co. Ltd. v. Governor of Orissa and Others (1995) 3 SCC 8
the court observed as follows :- 1995 AIR SCW 1575, Para 10

"It is well known that the Court while considering the question whether the award should
be set aside, does not examine that question as an appellate court. While exercising the
said power, the court cannot reappreciate all the materials on the record for the purpose of
recording a finding whether in the facts and circumstances of a particular case the award
in question could have been made. Such award can be set aside on any of the grounds
specified in Section 30 of the Act."
11. There is no quarrel with the proposition canvassed by the learned counsel for the
appellants. This scope of interference by the court is limited and the court would not be
justified in reappreciating the material on record and substituting its own view in place of
arbitrator's view. This exercise is not permissible by the court in view of the
@page-SC2930
settled legal position.
12. In the instant case, where there is an error apparent on the face of record or where the
arbitrator has not followed the statutory legal position, the court would be justified in
interfering with the award of the arbitrator.
13

. Learned counsel for the respondents placed reliance on Trustees of the Port of Madras v.
Engineering Constructions Corporation Limited (1995) 5 SCC 531 wherein this court
observed as under :- 1995 AIR SCW 3584, (Para 20)

"In the case of a reasoned award, the court can interfere if the award is based upon a
proposition of law which is unsound in law. The error apparent on the face of the award
contemplated by Section 16(1) (c) as well as Section 30 (c) of the Arbitration Act is an
error of law apparent on the face of the award and not an error of fact. An error of law on
the face of the award means an error of law which can be discovered from the award
itself or from a document actually incorporated therein. The erroneous proposition of law
must be established to have vitiated the decision. The arbitrator being a creature of the
contract must operate within the four corners of the contract. It is not permissible to travel
beyond and consider material not incorporated in or appended to the award."
14

. The learned counsel for the respondents has also placed reliance on the case of Indian
Sugar Mills Association (supra). In this case, this court laid down that the local
administration was competent to increase the 'rates of charges'. The arbitrator ignored the
settled legal position and consequently the High Court was justified in interfering with
the award based on wrong principles of law. AIR 1968 SC 22

15

. The learned counsel for the respondents also placed reliance on Bungo Steel Furniture
Pvt. Ltd. v. Union of India (1967) 1 SCR 633. In this case, the court observed as under :-
AIR 1967 SC 378

"The arbitrator in fixing the amount of compensation had not proceeded to follow any
principles, the validity of which could be tested on the basis of laws applicable to
breaches of contract. He awarded the compensation to the extent that he considered right
in his discretion without indicating his reasons. Such a decision by an Arbitrator could
not be held to be erroneous on the face of the record."
In that case, the court further held as under :-
"It is now a well settled principle that if an arbitrator, in deciding a dispute before him,
does not record his reasons and does not indicate the principles of law on which he has
proceeded, the award is not on that account vitiated. It is only when the arbitrator
proceeds to give his reasons or to lay down principles on which he has arrived at his
decisions that the court is competent to examine whether he has proceeded contrary to
law and is entitled to interfere if such error in law is apparent on the face of the award
itself."
16. The Privy Council in Champsey Bhara and Co. v. Jivraj Balloo Spg. and Wvg. Co.
Ltd., AIR 1923 PC 66 observed as under :-
"An error in law on the face of the award means, in their Lordships' view, that you can
find in the award or a document actually incorporated thereto, as for instance, a note
appended by the arbitrator stating the reasons for his judgment, some legal proposition
which is the basis of the award and which you can then say is erroneous."
17

. In Raipur Development Authority and Others v. M/s. Chokhamal Contractors and


Others (1989) 2 SCC 721, a Constitution Bench of this Court clarified that "the ground
arising out of an error of law apparent on the face of the award prima facie appears to fall
either under Section 16(1) (c) of the Act, which empowers the Court to remit the award to
the arbitrator where an objection to the legality of the award which is apparent upon the
face of it is successfully taken, or under Section 30 (c) of the Act which empowers the
Court to set aside an award if it is 'otherwise invalid'." AIR 1990 SCW 1426

18

. From the discussion of the aforementioned cases, it is clear that the error apparent on the
face of the award contemplated by Section 16(1)(c) as well as Section 30(c) of the
Arbitration Act is an error of law apparent on the face of the award and not an error of
fact. Same principle has been reiterated in Thawardas Pherumal v. Union of India (1955)
2 SCR 48. The court reiterated the legal position that an arbitrator cannot ignore the law
or misapply it in order to do what he thinks is just and reasonable. The AIR 1955
SCW 468

@page-SC2931
legal position has been crystallized in a series of judgments of this Court that the
arbitrator has got ample power in giving an award. The arbitrator is the sole judge of the
quality as well as the quantity of evidence and it will not be for the court to take upon
itself a task of being a judge of the evidence before the arbitrator. The court should
approve the award with the desire to support it, if that is reasonably possible rather than
to destroy it, by calling it illegal. This court has very limited jurisdiction to interfere with
the reasoned award. Only when the award is based upon a proposition of law which is
unjustified in law, the error of law must appear from the award itself or from any
document or note incorporated in it or appended to it. It is not permissible to travel
beyond and consider material not incorporated in or appended to the award.
19. In view of the clear legal position which has been correctly appreciated by the High
Court in the impugned judgment, in our considered view, no interference is called for.
20. The appeals are accordingly dismissed being devoid of any merit. In the facts and
circumstances of the case, we direct the parties to bear their own costs.
Appeals dismissed. .
AIR 2008 SUPREME COURT 2931 "Cochin University of Science and Technology v.
Thomas P. John"
(From : AIR 2003 Kerala 238)
Coram : 3 B. N. AGRAWAL, HARJIT SINGH BEDI AND G. S. SINGHVI, JJ.
Civil Appeal Nos. 4159 with 6418 of 2003, D/- 6 -5 -2008.
Cochin University of Science and Technology and Anr. v. Thomas P. John and Ors.
(A) Constitution of India, Art.15, Art.226 - EQUALITY - WRITS - EDUCATION - in -
Education - Fee fixation - Administrative matter of institution - Court would not interfere
unless fee fixed is excessively high.
The matter relating to the fixation of a fee is a part of the administration of an educational
institution and it would impose a heavy onus on such an institution to be called upon to
justify the levy of a fee with mathematical precision. An educational institution chalks out
its own programme yearwise on the basis of the projected receipts and expenditure and
for the Court to interfere in this purely administrative matter would be impinging
excessively on this right. That however, does not mean that the educational institution has
a carte blanche to fix any fee that it likes but substantial autonomy must be left to it.
(Para 7)
(B) Constitution of India, Art.15, Art.226 - EQUALITY - WRITS - EDUCATION -
ESTOPPEL - Education - Fee structure - Cost-sharing course started by University - NRI
students admitted for certain academic years on particular fee structure - Fee structure for
NRI quota reduced in subsequent academic years - Students admitted earlier cannot claim
parity in fee.
AIR 2003 Ker 238, Reversed.
Evidence Act (1 of 1872), S.115.
The NRI students took admission on certain specific conditions. The University has a
right to insist that those conditions are observed. It would not be open to the students to
contend that notwithstanding that they had been admitted on a certain fee structure they
were entitled to claim as a matter of right, a reduction in fee to bring them at par with
students admitted later under a lower fee structure. The University had made its budget
estimates keeping in view the proposed receipts and if the fee levied by it and accepted
by the students was permitted to be cut down mid term on the premises that the
University had not been able to explain each and every item to justify the levy, it would
perhaps be impossible for it to function effectively.
AIR 2003 Ker 238, Reversed. (Paras 11, 12)
The argument of estoppel in such a case would, be available to an educational institution.
It is incorrect to take the view that estoppel is a plea in equity and as the right of the NRI
students under Article 14 appeared to have been violated, this plea was not available to
the University. The NRI students have not been granted admission on their overall merit
but on the basis of the 10% reservation in their favour and as such any claim based on
equity would be suspect. Secondly each set of admissions made yearwise cannot be said
to overlap the admissions made earlier or later. (Para 12)
Cases Referred : Chronological Paras
2006 AIR SCW 3460 : AIR 2006 SC 3626 : 2006 CLC 1029 (Ref.) 5
@page-SC2932
2005 AIR SCW 3923 : AIR 2005 SC 3226 (Ref.) 5, 10
2003 AIR SCW 4240 : AIR 2003 SC 3724 (Rel. on) 5, 9, 12
2002 AIR SCW 4957 : AIR 2003 SC 355 (Rel. on) 5, 6, 7, 8
AIR 1986 SC 1043 (Ref.) 5
T.L.V. Iyer, Sr. Advocate, T.G. Narayanan Nair, for the Appellants; L.N. Rao, Sr.
Advocate, Romy Chacko, Arpit Gupta and P.V. Dinesh, for the Respondents.
Judgement
1. HARJIT SINGH BEDI, J. :- These appeals by special leave arise out of the following
facts :
2. In the Undergraduate 4 years B. Tech. Cost-Sharing Engineering Course of eight
semesters started in the year 1995 by the appellant university, 10% seats were reserved
for Non-Resident Indian Students (hereinafter called "NRI students"). As per the
prospectus such students were required to make a deposit of US $5000 at the time of their
admission towards 'development charges' and to pay in addition a fee of Rs.20,000/- per
semester whereas all the other categories of students were required to pay a uniform fee
of Rs.20,000/- per semester. From the academic year 1996-97, however, the University
increased the fee for NRI students to US $4000 per annum whereas the other students
continued to pay fee at the rate of Rs.20,000 per semester. This practice was continued
for three admission years, i.e. 1996-97, 1997-98 and 1998-99, but from the year 1999-
2000 the provisions made in the year 1995-96 i.e. confining the payment of fee to a one
time payment US $5,000 and Rs.20,000/-per semester were restored. The respondents
herein who had been admitted to the course in question during the years 1997-98 and
1998-99 filed representations claiming that they had been adversely treated by the
appellant University and that they were entitled to claim parity vis-a-vis the fee structure
for NRI students as from the years 1999-2000 onwards. As the representations bore no
result, 34 of the 56 NRI students who had been admitted to the course during the two
years, filed two writ petitions before the Kerala High Court. On notice, a counter affidavit
was filed by the Registrar on behalf of the appellant University pointing out that the NRI
students had not been admitted to the course on the basis of merit and that the B. Tech.
programme conducted at the Centre was a self-financing and unaided one being run
exclusively with funds collected by way of fees. The fact as to the increase and the
changes made from time to time in the fee structure were broadly admitted but it was
pleaded that the Syndicate of the University had reduced the fee for the batch entering the
course for the year 1999-2000 before the admission process had commenced and that the
writ petitioners could not claim an automatic reduction in the fee and it was essential that
the fee structure designed for a particular batch should be allowed to continue as to make
a change midway would lead to a complete break down of the finances of the University.
The Division Bench of the High Court observed that two questions arose for
consideration :
(1) Is the action of the University in charging fee at different rates from the students on
the basis of the batches in which they were admitted arbitrary and unfair ?
(2) Are the petitioners estopped from challenging the impugned action?
and then went on to examine each point under specific heads. While dealing with the
question No. 1, the Court observed that there appeared to be no rationale for subjecting
the writ petitioners to a higher rate of fee than the rate fixed in the years 1995-96 and
1999-2000 onwards more particularly as in the written statement filed on behalf of the
University no basis for a differential treatment had been disclosed and the averment that a
reduction in the fee would lead to financial stress in the conduct of the courses had not
been substantiated by facts and figures. The Court also observed that even assuming that
the university had the right to fix the rate of fee, a duty was still cast on it to act fairly,
and being a statutory body, its decision was to be based on reasonable facts and if a
classification between the different categories of students was pleaded, it must satisfy the
test of having a rational basis.
3. On question No.2, the Court held that though the University had issued a prospectus
disclosing the fee structure, it would not bind the respondents even on the principle of
estoppel, as estoppel was a principle of equity and as it appeared that the fundamental
right of the writ petitioner under Article 14 of the Constitution had been violated, the
same could not be waived even by their own action. The ultimate direction was
accordingly rendered on 2nd April 2003 as
@page-SC2933
under :
"In view of the above, the writ petitions are allowed. The University is directed to refund
the extra fee charged from the petitioners. It may be noticed in this connection that
initially the levy of an additional fee had been stayed by this Court. However, on a
subsequent date, the order of stay was vacated. At that time an undertaking was given by
the University that in case the writ petition is allowed, the disputed amount of fee shall be
refunded. The University shall do so within two weeks from the date of receipt of a
certified copy of this order. In case of failure to refund within the time as aforesaid, it
shall be liable to pay the amount along with interest at the rate of 10% from the date of
deposit till the date of refund. The University is also directed to declare the result of the
petitioners forthwith."
4. It is against this judgment and order of the Division Bench that the present appeals
have been filed by way of Special Leave. This matter first came up for hearing on 9th
May 2003 on which date leave was granted and pending proceedings the order for refund
was stayed. We have also been told that this Court had directed the respondent students as
an interim measure to pay the entire fee as per the fee structure under which they had
been admitted and it is the conceded position that all the students have in fact deposited
the amounts in question.
5

. Mr. T. L.V. Iyer, the learned senior counsel appearing for the University has seriously
controverted the conclusions of the High Court and has pointed out that it was open to a
self financing institution to fix its own fees and interference in this exercise by the Court
was not called for. He has submitted that there was adequate material on record to show
that the University was in need of funds as the course set up was a new one and the
necessary infrastructure and facilities had yet to be developed which justified a
substantial fee on those who could best afford it. The learned counsel has placed reliance
for this submission on T.M.A.Pai Foundation vs. State of Karnataka (2002) 8 SCC 481,
Islamic Academy of Education and Anr. vs. State of Karnataka and Ors. (2003) 6 SCC
697, P.A.Inamdar and Ors. vs. State of Maharashtra and Ors. (2005) 6 SCC 537. On the
second question, posed by the High Court, Mr. Iyer has submitted that it would be a very
dangerous doctrine to lay down, that a student having accepted admission under a
particular fee structure could turn around and say at a later stage that the fee which was
called upon to pay was excessive and that he was liable to pay such fee which was
leviable on students admitted in subsequent years. It has been highlighted and in this
situation that there would be complete uncertainty in the quantum of funds available and
that it would be well nigh impossible for any educational institution to chalk out its own
parameters for development. It has finally been submitted that having taken admissions
under a certain fee regime the NRI students were estopped from challenging the same in
Court. In support of this argument, the learned counsel has relied upon Om Prakash
Shukla vs. Akhilesh Kr.Shukla and Ors. (1986) Supp. SCC 285 and Standard Chartered
Bank vs. Andhra Bank Financial Services Ltd. and Ors. (2006) 6 SCC 94. 2002 AIR
SCW 4957
2003 AIR SCW 4240
2005 AIR SCW 3923
AIR 1986 SC 1043
2006 AIR SCW 3460

. Mr. Rao, the learned senior counsel for the respondents has, however, supported the
judgment of the Division Bench and has submitted that though the right of the University
to fix a fee was undeniable, but the quantum was required to be reasonable and also
supported by relevant material to justify the levy. He has pointed out the stand of the
university had been a vacillating one, as before the High Court the plea taken was that the
funds available from the NRI students were required for infrastructure development
whereas a complete somersault had been made in the affidavit filed in this Court by
pleading that it had been observed, that during the admissions made in the year 1997-98
and 1998-99 meritorious NRI students had not sought admission on account of the high
fee and it was in that eventuality that the University had decided to re-introduce the fee
structure for the year 1995-96, so as to attract NRI students from a wider base. It has been
submitted by Mr. Rao that the quantum of the fees and the manner of its imposition
suggested that the fees was, in fact, a capitation fee, the levy of which was completely
barred by several judgments of this Court and in this connection has placed reliance on
T.M.A.Pai Foundation and Ors. (supra). It has been pleaded that as per the budget
estimates shown in the affidavit filed by the university in this Court (from the year 1996-
97 2002 AIR SCW 4957

@page-SC2934
to 1999-2000) it was clear that there were substantial reserves with the University during
the years 1997-98 (academic years) which did not warrant an increase in the fee. It has
finally been argued that in the light of the judgments aforequoted, the fee structure for the
year 1995-96, and 1996-97 had been determined by a committee and as such any
deviation therefrom by the University was unjustified. For this pleading on facts Mr. Rao
has referred us to Civil Appeal No. 6143/2003.
7
. At the very outset, it must be observed that the dispute pertains only to two years and as
of today there appears to be no difficulty, as the fee structure is now devised by
committees set up under the orders of the Supreme Court in the aftermath of the
judgment in T.M.A.Pai's case (supra) . We are also of the opinion that the matter relating
to the fixation of a fee is a part of the administration of an educational institution and it
would impose a heavy onus on such an institution to be called upon to justify the levy of
a fee with mathematical precision. The Supreme Court has laid down several broad
principles with regard to the fixation of fees and as of today, those principles are being
adopted by the committees set up for the purpose. It must be understood at the outset that
an educational institution chalks out its own programme yearwise on the basis of the
projected receipts and expenditure and for the court to interfere in this purely
administrative matter would be impinging excessively on this right. From this, however,
it should not be understood that the educational institution has a carte blanche to fix any
fee that it likes but substantial autonomy must be left to it. Mr. Rao has very candidly
admitted that it was undoubtedly open to an educational institution to fix its fee but
subject to certain broad principles. We have accordingly gone through the affidavits filed
by the appellant University and they reveal that the University had set up the new course
in the year 1995-96 for which funds were required for infrastructure development, the
development of a faculty, which would mean making provision for adequate salary for
the teaching and supporting staff so as to attract the best minds. It has also been
emphasized in the second affidavit that the fees had been first increased and subsequently
reduced as experience had shown that the amount of US $ 5000 per year was excessive
and left out of consideration a large number of NRI students who could not afford the fee
and in order to make the course available to a larger segment amongst this category, the
fee had been reduced. We are of the opinion that no contradiction or fault can be found
with the University in taking these two stances in the two affidavits as they supplement
each other and make out a justification for the initial increase in the fee and subsequent
downward revision. 2002 AIR SCW 4957

. We have also gone through the judgments cited by the learned counsel. In T.M.A.Pai
case (supra) several questions as to the rights of minority institutions to manage their own
affairs were taken up - one of the significant questions being the right to determine and
levy fee. Question 5(c) and its answer are reproduced below : 2002 AIR SCW 4957,
Para 162-G

Q.5(c) Whether the statutory provisions which regulate the facets of administration like
control over educational agencies, control over governing bodies, conditions of affiliation
including recognition/withdrawal thereof, and appointment of staff, employees, teachers
and principals including their service conditions and regulation of fees, etc. would
interfere with the right of administration of minorities?
A. So far as the statutory provisions regulating the facets of administration are concerned,
in case of an unaided minority educational institution, the regulatory measure of control
should be minimal and the conditions of recognition as well as the conditions of
affiliation to a university or board have to be complied with, but in the matter of day-to-
day management, like the appointment of staff, teaching and non-teaching, and
administrative control over them, the management should have the freedom and there
should not be any external controlling agency. However, a rational procedure for the
selection of teaching staff and for taking disciplinary action has to be evolved by the
management itself.
For redressing the grievances of employees of aided and unaided institutions who are
subjected to punishment or termination from service, a mechanism will have to be
evolved, and in our opinion, appropriate tribunals could be constituted, and till then, such
tribunals could be presided over by a judicial officer of the rank of District Judge.
@page-SC2935
The State or other controlling authorities, however, can always prescribe the minimum
qualification, experience and other conditions bearing on the merit of an individual for
being appointed as a teacher or a principal of any educational institution.
Regulations can be framed governing service conditions for teaching and other staff for
whom aid is provided by the State, without interfering with the overall administrative
control of the management over the staff.
Fees to be charged by unaided institutions cannot be regulated but no institution should
charge capitation fee."
9

. It was further held that though no capitation fee or profiteering was permissible but
"reasonable surplus to cost (sic) expansion and augmentation (sic) facilities do not,
however, amount to profiteering". This judgment came up for consideration in the Islamic
Academy case (supra) primarily at the instance of unaided professional educational
institutions, both minority and non-minority. Several questions were posed before the
Court and question No. 1 was whether the educational institutions were entitled to fix
their own fee structure. This question was answered as under : 2003 AIR SCW 4240,
Para 6

"So far as the first question is concerned, in our view the majority judgment is very clear.
There can be no fixing of a rigid fee structure by the Government. Each institute must
have the freedom to fix its own fee structure taking into consideration the need to
generate funds to run the institution and to provide facilities necessary for the benefit of
the students. They must also be able to generate surplus which must be used for the
betterment and growth of that educational institution. In paragraph 56 of the judgment it
has been categorically laid down that the decision on the fees to be charged must
necessarily be left to the private educational institutions that do not seek and which are
not dependent upon any funds from the Government. Each institute will be entitled to
have its own fee structure. The fee structure for each institute must be fixed keeping in
mind the infrastructure and facilities available, the investments made, salaries paid to the
teachers and staff, future plans for expansion and/ or betterment of the institution etc. Of
course there can be no profiteering and capitation fees cannot be charged. It thus needs to
be emphasized that as per the majority judgment imparting of education is essentially
charitable in nature. Thus the surplus/ profit that can be generated must be only for the
benefit/use of that educational institution. Profits/surplus cannot be diverted for any other
use or purpose and cannot be used for personal gain or for any other business or
enterprise."
10

. It was as a consequence of the directions issued in this case that a committee headed by
a retired Judge was set up in each State to examine the fee structure which would be
applicable both to aided and non-aided educational institutions with a further direction
that the recommendations made by the committee were to remain binding for 3 years.
Both the aforesaid judgments came up for consideration in P.A.Inamdar's case (supra)
and it was observed that though a limited number of seats not exceeding 15% may be
made available to NRIs depending upon the discretion of the management, two essential
conditions were to be kept in mind; (1) the seats would be utilized for the benefit of bona
fide NRIs and their children or wards and that within this quota merit would not be given
a complete go-by and (2) further that the amount of money "in whatever form collected
by such NRIs, should be utilized for the benefiting students such as from economically
weaker sections of the society, whom, on well-defined criteria, the educational institution
may admit on subsidized payment of fee." 2005 AIR SCW 3923

11. A reading of the aforesaid judgments would reveal that the broad principle is that an
educational institution must be left to its own devices in the matter of fixation of fee
though profiteering or the imposition of capitation fee is to be ruled out and that some
amount towards surplus funds available to an institution must be permitted and visualized
but it has also been laid down by inference that if the broad principles with regard to
fixation of fee are adopted, an educational institution cannot be called upon to explain the
receipts and the expenses as before a Chartered Accountant. We find that the observations
of the Division Bench of the High Court that no rational basis for the fixation of a higher
fee for two years had been furnished, lays down an onus on the educational institution,
which would be difficult for it to discharge with accuracy. It bears repetition that the
University had set up the
@page-SC2936
self-financing B.Tech. Course in the year 1995 and no grant in aid was available during
this period or later and it had to make arrangements for its own funds. We have also
examined the budget estimates, receipts and expenditure from the year 1996-97 to 1999-
2000. We do find that there is a surplus in the hands of institution but in the facts that a
new course was being initiated which would require huge investments, the surplus was
not unconscionable so as to require interference. Moreover, the University had made its
budget estimates keeping in view the proposed receipts and if the fee levied by it and
accepted by the students was permitted to be cut down mid term on the premise that the
University had not been able to explain each and every item to justify the levy, it would
perhaps be impossible for it to function effectively.
12

. We are also of the opinion that it would be well nigh impossible for an educational
institution to have an effective administration and to maintain high educational standards,
if a downward revision during the pendency of a course would be automatically made
applicable to students admitted earlier under a different fee structure. A periodic revision
is also visualized in the directions of the Supreme Court in Islamic Academy's case
(supra) wherein it has been provided that the fee structure fixed by a committee headed
by a retired Judge would be operable for 3 years. In the present case, we find that the NRI
students took admission on certain specific conditions and the University has a right to
insist that those conditions are observed. To our mind, therefore, it would not be open to
the students to contend that notwithstanding that they had been admitted on a certain fee
structure they were entitled to claim as a matter of right, a reduction in fee to bring them
at par with students admitted later under a lower fee structure. The argument of estoppel
in such a case would, thus, be available to an educational institution. The High Court was
influenced by the fact that estoppel was a plea in equity and as the right of the NRI
students under Article 14 appeared to have been violated, this plea was not available to
the University. We do not agree with this submission for several reasons, firstly the NRI
students have not been granted admission on their overall merit but on the basis of the
10% reservation in their favour and as such any claim based on equity would be suspect
and secondly each set of admissions made yearwise cannot be said to overlap the
admissions made earlier or later. We have also considered Mr. Rao's submission that the
fee had the trappings of a capitation fee. We find no merit in this assertion as the fee is
being levied yearwise for the course. We have also gone through the judgments cited by
Mr. Iyer. To our mind, they are not applicable to the facts of this case. 2003 AIR
SCW 4240

13. Mr. Rao has finally submitted that as the fee for the years 1995-96 and 1996-97 had
been fixed by a committee set up under the directions of the Supreme Court it was not
open to the Syndicate to suggest a higher fee thereafter. We find, however, that there
seems to be a misconception as to the facts as it is the specific case of the University that
the fee had been fixed by the Syndicate under Section 18 of the Cochin University of
Science and Technology Act, 1976 and not by any committee.
14. We therefore, find that the judgment of the Division Bench of the High Court cannot
be sustained. We accordingly set it aside and allow the appeals with no order as to costs.
Appeals allowed. .
AIR 2008 SUPREME COURT 2936 "Andhra Bank v. Andhra Bank Officers"
(From : Andhra Pradesh)*
Coram : 2 S. B. SINHA AND LOKESHWAR SINGH PANTA, JJ.
Civil Appeal No. 3405 of 2008 (arising out of SLP (C) No. 11853 of 2006), D/- 8 -5
-2008.
Andhra Bank v. Andhra Bank Officers and Anr.
Banking Companies (Acquisition and Transfer of Undertakings) Act (40 of 1980), S.19,
S.8 - Andhra Bank (Officers) Service Regulations (1982), Regn.26 - BANKING -
RESERVE BANK OF INDIA - Circular regarding bank's car for personal use - Issued by
Central Government - Prior consultation with Governor of Reserve Bank of India which
was mandatory u/S.8 - Not necessary.
W. A. No. 1334 of 2000, D/-17-04-2006 (A.P.), Reversed.
Where a Circular was issued by Central Govt. regarding use of bank's car for personal
purposes, Regn. 26 would be attracted
@page-SC2937
and not Section 8 and, therefore, prior consultation with Governor of Reserve Bank of
India, which was mandatory under Section 8, would not be necessary. Section 8 of the
Act provides for issuance of directions by the Central Government with regard to matters
of policy involving public interest which are bound to be followed by the 'New Banks' in
the discharge of its functions. A regulation framed for the purpose of laying down the
terms and conditions of service of the employees of the bank do not necessarily involve
any policy decision involving public interest. Each word used in Section 8 must be given
effect to. It is separate and distinct from the regulation making power. Moreover, use of
the Bank's cars for personal purpose is only one of the regulations. The guidelines
contemplated under sub-regulation (2) of Regulation 26 would be those issued by the
Central Government but they are not subject to any further regulation. For issuance of the
said guidelines, the procedures laid down for making Regulations were not required to be
undergone. The guidelines issued by the Government, however, would be subject to
Rules formulated by the Bank. If there is no guideline, the same by itself would not stand
in the way of the bank to make a scheme but if there is a guideline, the Rules must be
formulated in accordance therewith. Guidelines may be issued by the Government from
time to time. The expression 'from time to time' is significant. It must be given its due
meaning. It does not and cannot mean that whenever the guidelines are issued, the
Central Government must consult the Reserve Bank of India. Further, there cannot be any
doubt whatsoever where one authority is required to consult the another, such
consultation must be meaningful. It must mean conscious and effective consultation but
the same would apply where the consultation is necessary and for the purpose of issuance
of guidelines, no consultation was necessary to be made with the Reserve Bank of India
by the Central Government.
W. A. No. 1334 of 2000, D/-17-04-2006 (A.P.), Reversed. (Paras 13, 15)
Cases Referred : Chronological Paras
2004 AIR SCW 4303 : AIR 2004 SC 4007 (Ref.) 13
2002 AIR SCW 1492 : AIR 2002 SC 1589 : 2002 Lab IC 1492 (Disting.) 14
1993 AIR SCW 1135 (Disting.) 14
AIR 1991 SC 471 : 1991 Lab IC 308 (Disting.) 16
AIR 1991 SC 1362 (Disting.) 14
V.R. Reddy, Sr. Advocate, Sunil Kurarka, P.P. Singh and Abhay Yadav, with him for
Appellant; Jaideep Gupta, Sr. Advocate, D. Bharat Kumar, Anand, Azim H. Laskar, Ms.
M. Indrani and Abhijit Sengupta, with him for Respondent No. 1. Kuldeep S. Parihar and
H.S. Parihar, for RBI.
* W. A. No. 1334 of 2000, D/- 17-4-2006 (A.P.)
Judgement
1. S. B. SINHA, J. :- Leave granted.
2. Interpretation of Regulation 26 of the Andhra Bank (Officers) Service Regulations,
1982 framed under Section 19 of the Banking Companies (Acquisition and Transfer of
Undertakings) Act, 1980 (for short, 'the Act') is the question involved herein. Parliament
enacted the Act to provide for the acquisition and transfer of the undertakings of certain
banking companies, having regard to their size, resources, coverage and organization in
order to further control the heights of the economy, to meet progressively, and serve
better, the needs of the development of the economy and to promote welfare of the
people, in conformity with the policy of the State towards securing the principles laid
down in clauses (b) and (c) of Article 39 of the Constitution and for matters connected
therewith or incidental thereto. Section 3 of the Act provides for the transfer of
undertakings of the existing banks. Appellant bank is a 'New Bank' within the meaning of
the provisions of the said Act. Undertaking of the existing bank in terms of the said Act
vested in the 'New Bank'. Section 8 of the Act provides that every corresponding 'New
Bank' shall in the discharge of its functions be guided by such directions in regard to the
matters of policy involving public interest as the Central Government may, after
consultation with the Governor of the Reserve Bank, issue. Section 19 of the Act
empowers the Board of Directors to make regulations, sub-section (1) whereof is in the
following terms :
"19. Power to make regulations. - (1) The Board of Directors of a corresponding new
bank may, after consultation with the Reserve Bank and with the previous sanction of the
Central Government, by notification in the Official Gazette, make regulations, not
inconsistent with the provisions of this Act or any scheme made thereunder to provide for
all matters for which provision is expedient for the purpose of giving effect to the
@page-SC2938
provisions of this Act."
3. Indisputably, the Board of Directors, in consultation with the Reserve Bank of India
and with prior sanction of the Central Board, made regulations, Regulation 26 whereof
reads as under :
"26. Bank's car for personal purposes :
1) No officer, other than the Officers authorized by the Board, in accordance with the
guidelines of the Government, shall be allowed the use of the Bank's car for personal
purposes.
2) The use of the Bank's car for personal purposes should be subject to the rules
formulated by the Bank in accordance with the guidelines of the Government from time
to time."
4. The Board of Directors framed a scheme as regards reimbursement of the amount
expended by the officers for undertaking their journey by car from their respective
residences to the bank and back. A circular letter was issued on 20.4.1983 to the said
effect, clause (3) whereof reads as under :
3. Reimbursement towards driver's salary for the officers for whom the (sic) provides a
car.

i) At places, Bombay Delhi and Calcutta : Upto Rs. 650/- p.m.


(ii) At all other State Capitals and Area-I (above 12 lakhs population) : Upto
Rs. 450/- p.m.
(iii) At other places : Upto Rs. 400/- p.m.

This scheme comes into force with effect from 22.2.1983.


II. MAINTENANCE OF VEHICLES OWNED BY OFFICERS
Reimbursement of maintenance expenditure for cars and scooters to Chief Officers
Managers including in Administrative/Controlling Officers, Technical Officers and
Credited Officers owned by them will be as under :

Mopeds : Rs. 75/- per month


Scooters : Rs. 150/- per month.
Cars : Rs. 325/- per month.

Competent authority to sanction conveyance allowance to officers is AGM, Dy-in-charge


of BS and BD at Central Office. (Prior sanction from Central Officers necessary)."
5. In its meeting dated 20.2.1985, a scheme was formulated by the Bank which was
circulated to all concerned in terms of its letter dated 7.3.1985, the relevant part whereof
reads as under :
"1. Travel from residence to office and back is to be considered as travel for office work
(as far as reimbursement of conveyance expenses is concerned)."
The Central Government, however, by reason of a circular letter dated 25.4.1990
addressed to the Chief Executive of all public sectors banks, inter alia, stated :
"The to and fro journeys between office and residence should not be treated as official
journeys and no reimbursement for such journeys be made. The claim duly countersigned
by an officer at least one step higher than the officer claiming the reimbursement for the
entire month should be submitted only once. However, the officers in Scale IV and above
may not get their vouchers countersigned."
The said letter was circulated by the Indian Bank Association.
6. A writ petition was filed by the respondents herein before the Andhra Pradesh High
Court questioning the validity of the said purported guidelines issued by the Central
Government. During the pendency of the said writ petition, the Board of Directors issued
a circular letter dated 22.2.1991, the relevant portion whereof reads as under :
"Keeping in view the prevalent situation, we are informed by the Indian Banks'
Association vide its letter No.PD/CIR/76/E(x)/ 2208 dated 25.1.1991 that in its
discussions held with the Officers' Organisations on the above issue, the Officers'
Organisations had indicated their inclination to accept some reduction in the consumption
of petrol and subsequently, some of the senior leaders of the Officers' Organisations met
the Chairman of the Indian Banks' Association and conveyed to the IBA writing to the
banks to bring about a cut in petrol limits, it being the need of the hour. Accordingly, it
has been decided to bring about a cut in the petrol limits of the officers owning vehicles
and covered under the Scheme 'B' and bring about uniformity in all the Public Sector
Banks. In view of the above, the revised limits for reimbursement of conveyance
expenses to Officers under Scheme 'B' (for the officers owning vehicles) as against the
existing limits shall be as under with effect from 1.3.1991..."
@page-SC2939
7. A learned Single Judge of the said Court, although opining that the appellant bank was
entitled to change its policy decision, held that as prior to issuance of the guidelines, the
Central Government had not consulted the Reserve Bank of India, the same was violative
of Section 8 of the Act stating :
"The guidelines do not purport to be under the provisions of the Act. It is sought to be
argued that power to issue such guidelines is to be found in Section 8 of the Act. Section
8 only says that the Bank in question in discharge of its functions shall be guided by such
directions in regard to matters of Policy involving public interest Central Government
may after consultation with the Governor of Reserve Bank of India may give. As stated
already the impugned circular nowhere shows that any consultation took place between
respondent No. 1 and the Governor of Reserve Bank of India. It is thus clear that
respondent No. 1 cannot legitimately contend that the impugned guidelines have been
validly issued under Section 8 of the Act. There does not appear any other provision
which justifies respondent No. 1 to issue any such guidelines to the Banks.
As pointed out already, respondent No.3-Bank had formulated the scheme under its
powers conferred by the Regulations. The said scheme was implemented already. Under
the said scheme, the journeys from residence to office and vice-versa were to be treated
as 'on official duty'. This was a specific term in the scheme. Respondent No. 1 had
absolutely no power to tinker with the provisions of the scheme."
8. A Division Bench of the High Court affirmed the said view of the learned Single
Judge, stating :
"Section 8 of the Act lays down that the Bank in the discharge of functions shall be
guided by such directions in regard to matters of policy involving public interest as the
Central Government may after consultation with the Governor of Reserve Bank of India.
A perusal of the impugned guidelines makes it clear that there was no prior consultation
with the Governor of the Reserve Bank of India. The said consultation was only with the
Indian Banks' Association. Had such a consultation been there, the bank is bound by the
directions. From this it is beyond doubt that the power for issuing the impugned
guidelines cannot be traced to Section 8 of the Act."
9. With a view to ascertain as to whether, in fact, any consultative process had been
undergone with the Reserve Bank of India by the Central Government, the former was
impleaded as a party to this appeal. At our request, learned Solicitor General for India
also assisted us.
10. The only question which arises for our consideration is as to whether in the facts and
circumstances of the case, it was necessary to consult the Reserve Bank of India by the
Central Government before issuing the impugned guidelines.
11. Submission of the learned Solicitor General, as also Mr. V.R. Reddy, learned Senior
Advocate, appearing for the appellant, are :
(1) Whereas Section 8 of the Act applies when the Central Government is required to
issue a policy decision; regulation having been framed after consulting the Reserve Bank
of India in the prior sanction of the Central Government, it was not necessary to consult
the Reserve Bank of India again.
(2) Guidelines having been issued by the Central Government in terms of Regulation 26
and not in terms of Regulation 8, no consultation with the Reserve Bank of India was
necessary.
12. Mr. Jaideep Gupta, learned senior counsel appearing on behalf of the respondent, on
the other hand, would submit -
(i) Consultation with the Reserve Bank of India was imperative in nature;
(ii) In view of the fact that although bank can change its policy from time to time, if it
does so in terms of the Guidelines issued by the Central Government, the same must
subserve the legal requirements as envisaged under Section 8 of the Act;
(iii) As admittedly, the Central Government had not consulted the Reserve Bank of India,
the impugned judgments are unassailable.
13. Section 8 of the Act provides for issuance of directions by the Central Government
with regard to matters of policy involving public interest which are bound to be followed
by the 'New Banks' in the discharge of its functions. The functions of the bank are
regulated not only by the said Act but also by Banking Regulation Act, 1949 and Reserve
Bank of India Act, 1934.
@page-SC2940
A regulation framed for the purpose of laying down the terms and conditions of service of
the employees of the bank do not necessarily involve any policy decision involving
public interest. Each word used in Section 8 must be given effect to. It is separate and
distinct from the regulation making power. Section 9 and Section 19 are made for
different purposes. Whereas Section 8 postulates issuance of directions by the Central
Government which must undergo the consultative process with the Governor of the
Reserve Bank, in terms of Section 19, it is for the Board of Directors to consult the
Reserve Bank. Only thereafter the Regulations can be brought in force with the previous
sanction of the Central Government. Such Regulations must be consistent with the
provisions of the Act or the scheme made thereunder. It must provide for all matters for
which provisions have been made for the purpose of giving effect to the said Act
wherefor Section 12(2) of the Act has also a significant role to play.
The services of the employees of the existing bank were contemplated to be taken over
by reason of the provisions of the Act. They were to be governed by the same terms and
conditions and continue to have the same rights as regards pension, gratuity and other
matters subject, however, to any alternation made by the corresponding new bank with
regard to his remunerations and other terms and conditions of service. Such alterations
are required to be made only in terms of the regulations made under Section 19 and not
otherwise. Use of the Bank's cars for personal purpose is only one of the regulations. The
guidelines contemplated under sub-regulation (2) of Regulation 26 would be those issued
by the Central Government but they are not subject to any further regulation. For
issuance of the said guidelines, the procedures laid down for making Regulations were
not required to be undergone. The guidelines issued by the Government, however, would
be subject to Rules formulated by the Bank. If there is no guideline, the same by itself
would not stand in the way of the bank to make a scheme but if there is a guideline, the
Rules must be formulated in accordance therewith. Guidelines may be issued by the
Government from time to time. The expression from 'time to time' is significant. It must
be given its due meaning. It does not and cannot mean that whenever the guidelines are
issued, the Central Government must consult the Reserve Bank of India.
The question, albeit in a bit different context, came up for consideration before a Three
Judge Bench in Andhra Bank v. B. Satyanarayana and Ors. [(2004) 2 SCC 657] which we
may notice. Regulation 17 of the Regulations which was the subject matter of
consideration therein reads as :
"17. Promotions. - (1) Promotions to all grades of officers in the Bank shall be made in
accordance with the policy laid down by the Board, from time to time, having regard to
the guidelines of the Government, if any.
(2) For the avoidance of doubts, it is clarified that this Regulation shall also apply to
promotions of any category of employees to the Junior Management Grade."
It was noticed that the Government had also issued guidelines of the said Regulation.
This Court held that once the power vests in an authority by reason of the provisions of a
statute, such power can be exercised from time to time. Changes are required to be made
keeping in view the requirements of managements as also exigencies of the situation
obtaining at the relevant time. It is true, as has been contended by Mr. Gupta, that one of
the contentions raised therein was that the Regulation was arbitrary and ultra vires as it
did not contain sufficient guidelines. But this Court therein also took into consideration
the effect and purport of Section 19 of the Act in the following terms :
"12. The Regulations in terms of sub-section (2) of Section 12 read with Section 19 of the
Act were required to be framed by the Board of Directors. For amending the Regulations
each time they were not only required to consult the Reserve Bank of India and obtain
previous permission of the Central Government but also the amended Regulations were
required to be laid before both the Houses of Parliament in terms of Section 19 of the Act.
With a view to avoid the rigours of such procedural requirements, we see no reason as to
why the said power cannot be delegated to the Board of Directors keeping in view the
fact that a policy decision was required to be laid down for effecting promotions to
different grades of officers and employees at different points of time."
14. The term 'rules' used in sub-regulation (2) of Regulation 19 appeared to have
@page-SC2941
been loosely used. It did not envisage any statutory rules. The power to frame rules is
vested with the Bank. The power of the bank is required to be exercised by the Board of
Directors. The scheme in regard to reimbursement of the expenses incurred for going to
office from the residence or coming back from the office to the residence was treated to
be official. The Central Government could, thus, issue a guideline in relation thereto.
When the Central Government, in exercise of its power under Regulation 6 of the
Regulations, issues a direction, the requirements of sub- regulation (2) thereof regulating
formulation of the rule only is that it should be in accordance with the guidelines. The
words used in the provision are 'should be' and not 'must be'. The ultimate decision,
therefore, is in the bank although guidelines issued by the Government must be given due
weight. Such guidelines may be issued from time to time as the Regulations itself have
been framed in consultation with the Reserve Bank of India. The latter must be held to
have given its approval for such exercise of the power by the bank as also issuance of
guidelines by the Government. Whenever such guidelines are issued or rules are made,
fresh consultative process need not be undergone."

Furthermore, the word 'consultation' has different connotations in different contexts.


Reliance has been placed by Mr. Gupta on some decisions of this Court in Municipal
Corporation of Greater Bombay v. New Standard Engineering Co. Ltd. [(1991) 1 SCC
611]; Indian Administrative Service (S.C.S.) Association, U.P. and Ors. [1993 Supp.(1)
SCC 730]; and Gauhati High Court and Anr. v. Kuladhar Phukan and Anr. [(2002) 4 SCC
524]. In view of our findings aforementioned, the said decisions have no application in
this case. AIR 1991 SCW 1362
1993 AIR SCW 1135
2002 AIR SCW 1492

15. There cannot be any doubt whatsoever where one authority is required to consult the
another, such consultation must be meaningful. It must mean conscious and effective
consultation but the same would apply where the consultation is necessary. As for the
purpose of issuance of guidelines, no consultation was necessary to be made with the
Reserve Bank of India by the Central Government, in our opinion, the impugned
judgment cannot be sustained.
16

. Reliance has also been placed on Union of India and Ors. v. Mohd. Ramzan Khan
[(1991) 1 SCC 588], wherein it was held that the guidelines issued by the Central
Government were not relateable to any Regulation or Section 8 of the Act. The said
decision has no application in the instant case, as it was noticed that the guidelines issued
by the Central Government were not in terms of any Regulation. In this case, the
guidelines have been issued in terms of sub-regulation (2) of Regulation 26. It is,
therefore, referable to a statutory power. AIR 1991 SCW 471

17. For the reasons aforementioned, the impugned judgment cannot be sustained. It is set
aside accordingly. Appeal is allowed with costs. Counsel's fee assessed at Rs.25,000/-
(Rupees twenty five thousand only).
Appeal allowed. .
AIR 2008 SUPREME COURT 2941 "Novva ADS, M/s. v. Secy., Deptt. of Municipal
Administration and Water Supply"
(From : Madras)*
Coram : 2 Dr. A. PASAYAT AND S. H. KAPADIA, JJ.
Civil Appeal No. 2702 of 2008 (arising out of SLP (C) No. 16286 of 2006) with C. A.
Nos. 2715, 2774 and 2717 etc. etc. of 2008, D/- 9 -4 -2008.
M/s. Nowa ADS v. Secretary, Deptt. of Municipal Administration and Water Supply and
Anr.
(A) Chennai City Municipal Act (4 of 1919), S.326A - MUNICIPALITIES -
HOARDINGS - Hoardings - Regulations - Necessary even for hoardings on private
property.
Hoarding erected on a private building may obstruct public roads. When put up on private
buildings; they may be dangerous to the building and to the public; they may be
hazardous and dangerous to the smooth flow of traffic by distracting traffic, and their
content may be obscene or objectionable. It is, therefore, not correct that hoardings on
private places do not require to be regulated by licensing provisions. The fact that the
hoarding is on building or private land does not take away the regulatory measures
relating to hoardings. There can be cases where because of the size and the height, it can
be dangerous to public and also be
@page-SC2942
hazardous. There is no structural safeguard in respect of such hoardings. There has to be
regulatory measures. (Para 23)
(B) Chennal City Municipal Act (4 of 1919), S.326J, S.326H - MUNICIPALITIES -
HOARDINGS - EQUALITY - Hazardous hoarding - Prohibition to erection - Power of
District Collector - Not unguided and arbitrary.
Constitution of India, Art.14.
Section 326J empowers the District Collector to prohibit the erection of hazardous
hoardings and hoardings which are hazardous and a disturbance to the safe traffic
movement so as to adversely affect the free and safe flow of traffic. The power under
Section 326J is not arbitrary. Any action taken under S. 326J must be taken by observing
the principles of natural justice and supported by reasons. An appeal against the order of
the District Collector for action under Section 326-J lies to the State Government under
Section 326H. There cannot be a presumption of misuse of power merely because
discretion is conferred on a public authority for the exercise of the power. (Para 25)
(C) Chennal City Municipal Act (4 of 1919), S.326C- Chennal City Municipal
Corporation (Licensing of Hoardings and Levy and Collection of Advertisement Tax)
Rules (2003), R.6, R.10, R.11 - MUNICIPALITIES - HOARDINGS - LICENSE -
WRITS - Hoardings - Power to grant/refuse licence - Not unguided.
Constitution of India, Art.226.
Rule 6 put restrictions on the size of hoardings, on their height, the spacing, etc., and the
requirement of erection on steel frames. Rule 10 restricts the hoarding to be put on certain
places such as educational institutions, places of worship, hospitals, corners of roads, in
front of places of historical and aesthetic importance. The power to license is guided by
the above considerations and is not unfettered. Under Rule 11 an appeal lies to the State
Government for refusing the grant or renewal of licenses. (Paras 24, 25)
(D) Constitution of India, Art.245 - LEGISLATION - Delegated legislation - Validity
-Touchstones for testing.
A delegated legislation can be declared invalid by the Court mainly on two grounds
firstly that it violates any provision of the Constitution and secondly it is violative of the
enabling Act. If the delegate which has been given a rule making authority exceeds its
authority and makes any provision inconsistent with the Act and thus overrides it, it can
be held to be a case of violating the provisions of the enabling Act but where the enabling
Act itself permits ancillary and subsidary functions of the legislature to be performed by
the executive as its delegate, the delegated legislation cannot be held to be in violation of
the enabling Act. (Para 31)
(E) Chennai City Municipal Act (4 of 1919), S.326J - MUNICIPALITIES -
HOARDINGS - WORDS AND PHRASES - Hazardous hoardings - Prohibition to
erection - "Harzardous" - Means something that is "risky" or "dangerous".
Words and phrases - "Hazardous". (Para 27)
(F) Chennal City Municipal Act (4 of 1919), S.326C - Chennai City Municipal
Corporation (Licensing of Hoardings and Levy and Collection of Advertisement Tax)
Rules (2003), R.3(iii) - MUNICIPALITIES - HOARDINGS - LICENSE - WORDS AND
PHRASES - Hoardings - Licensing - Expression "obstruction" in R.3 - Does not only
mean physical obstruction.
Words and Phrases - Obstruction.
The expression 'obstruction' means "something that impedes or hinders". The expression,
however, has varied sets of meaning and is not necessarily confined to physical
obstructions only. It includes tangible and identifiable obstruction and even a protest is
obstructing. The expression 'obstruction' in Rule 3(iii) would, therefore, include any act
which impedes the free and safe movement of the traffic, pedestrians and vehicles. Such
an act may well be, by reason of what is displayed on the hoardings. If the subject-matter
that is displayed in such hoardings attracts attention of the drivers of vehicles and which,
in turn, impedes free and safe movement of traffic such a hoarding would clearly come
under the meaning "obstruction" contemplated under Rule 3(iii) of the Rules. (Paras
36, 37, 39)
There is certainly some difference between "hazardous" and "obstruction" though there
may be some amount of overlapping. What is hazardous cannot have definite terms. So in
that sense, Legislature had thought it wise to use the expression 'obstruction' so that it can
be brought within manageable standards. The ultimate
@page-SC2943
objective is safe traffic movement and free and safe flow of traffic. (Para 40)
(G) Constitution of India, Art.19(1)(a) - Chennai City Municipal Act (4 of 1919), S.326A
- Chennai City Municipal Corporation (Licensing of Hoardings and Levy and Collection
of Advertisement Tax) Rules (2003), R.1 - FREEDOM OF SPEECH AND
EXPRESSION - MUNICIPALITIES - LICENSE - HOARDINGS - Freedom of speech -
Act regulates putting up of hoardings - Not advertisement - Does not, therefore, offend
Art.19(1)(a).
The Act and the Advertisement Rules do not regulate advertisement. They regulate
putting of the hoarding which is found to be objectionable, destructive or obstructive in
character. It cannot be said that there is infringement of freedom of speech. The content,
effect and the purpose of statute clearly show that it is not intended to be so. (Paras
51, 52)
Cases Referred : Chronological Paras
(2004) 3 SCC 48 (Ref.) 34
2003 AIR SCW 752 : AIR 2003 SC 1191 (Rel. on) 31
2003 AIR SCW 894 : AIR 2003 SC 1533 (Ref.) 32
2001 AIR SCW 1643 : AIR 2001 SC 1736 (Rel. on) 9, 20, 25, 42
(1998) 1 SCC 363 (Rel. on) 18, 25, 48
1997 AIR SCW 4149 : AIR 1998 SC 186 (Ref.) 48
1995 AIR SCW 3602 : AIR 1995 SC 2438 (Ref.) 9, 19
AIR 1990 SC 1897 (Rel. on, Pnt. F) 38
AIR 1973 SC 106 (Ref.) 9
AIR 1966 SC 740 : 1966 Cri LJ 608 (Ref.) 9
AIR 1962 SC 305 (Ref.) 9
(1955) 1 WLR 1207 : (1955) 3 All ER 406 36
AIR 1954 SC 728 (Ref.) 22, 50
AIR 1950 SC 124 : (1950) 51 Cri LJ 1514 (Ref.) 9, 46
AIR 1950 SC 129 : (1950) 51 Cri LJ 15 25 (Ref.) 9
(1930) 283 US 697 : 75 Law Ed 1357 46

* W. P. No. 19057 of 2003, D/- 10-8-2006 (Mad.)


Judgement
1. Dr. ARIJIT PASAYAT. J. : -Delay condoned.
2. Leave granted in the Special Leave Petitions.
3. Challenge in these appeals and Writ Petitions is to the judgment delivered by a
Division Bench of the Madras High Court. In the writ petitions, challenge was to validity
of Sections 326A to 326J of the Chennai City Municipal Act, 1919 (in short the 'Act') and
the Chennai City Municipal Corporation (Licensing of Hoardings and Levy and
Collection of Advertisement Tax) Rules, 2003 (in short the 'Advertisement Rules').
4. The writ petitions were dismissed by the High Court. But a Committee was constituted
for identifying and enumerating the places of historical importance or aesthetic value and
popular places of worship in and around the city of Chennai. It was also directed to
oversee the operation of the removal of illegal and unauthorized hoardings in the city of
Chennai. The Committee was directed to be headed by a retired Judge and to consist of
several other persons. The State Government was directed to provide necessary
infrastructure and office to the Committee. The District Collector was directed to remove
and demolish all the unauthorized hoardings which were erected after the cut off date and
in respect of which no application was made to the District Collector within a period of 8
weeks. The District Collector and the Tahsildar working in their respective zones were to
be personally responsible for the removal of unauthorized hoardings in their respective
zones. The Municipal Corporation was directed to extend all necessary cooperation to the
District Collector for removal of the hoardings in the city. The Commissioner was
directed to supply to the District Collector the necessary equipment and work force for
the purpose of such removal. The Police Commissioner was also directed to provide
adequate police force to assist the demolition team. The State Government was directed
to appoint two officers not below the rank of District Collector as Special Officers vested
with the necessary powers of the District Collector to make scrutiny of the applications
pending before the Collector within a period of 4 weeks from the date of judgment. It was
pointed out that no licence was to be granted and/or renewed in respect of any hoarding
which is not in conformity with the provisions of the Act and the Advertisement Rules.
So far as the applicants who claimed to be existing hoarding owners, the District
Collector/Special Officer was required to call for the views of the Traffic police and such
views had to be communicated to the District Collector within a particular period. All the
hoardings where the applications/appeals were dismissed by the authorities were liable to
be removed forthwith and the concerned authorities to take appropriate steps for the
purpose.
5. The District Collector and the
@page-SC2944
Tahsildar were directed to take immediate steps for recovery of the advertisement tax, the
rent and the penalties from the hoarding owners whether authorized or unauthorized. The
appeals against the decision of the District Collector/Special Officer were directed to be
disposed of within 60 days as prescribed by the Advertisement Rules and for that purpose
it was suggested that the Government may consider appointment of one or more officers
at the Secretariat level, exclusively for the purpose.
6. So far as new applications are concerned, it was held that if the applicant had already
constructed a hoarding in that case hoarding was liable to be removed and demolished
and the applicant was entitled to apply only after such removal and demolition of
hoarding.
7. With reference to Rule 3(i) of the Advertisement Rules it was held that the plan of the
hoarding was to be approved by a qualified structural Engineer. In case of non removal of
unauthorized or illegal hoardings the District Collector was directed to initiate
prosecution as permissible under the Act. Direction was also given for demolition and
removal of all hoardings erected on or in front of any places of historical or aesthetical
importance, popular places of worship as enlisted by the Committee as well as on or in
front of the educational institutions and hospitals and in cases where applications were
made by any hoarding owner within the time prescribed by this Court applications were
to be decided and if the hoardings were found to be illegal, they were to be removed
without further notice. It was directed that no Civil Court shall entertain any application
against demolition or removal of the unauthorized hoardings and the writ petitions
challenging the demolition were to be placed before the bench of the Chief Justice of the
High Court.
8. It was also directed that notwithstanding any order passed by any Civil Court in the
matter the directions given in the impugned order were to prevail.
9

. In support of the appeals, various stands have been taken by the parties. Primarily it has
been submitted that the Advertisement Rules are violative of Articles 19(1)(a) and 19(1)
(b) of the Constitution of India, 1950 (in short the Constitution). It was also violative of
Article 14 because private hoardings have been treated equally with public hoardings,
thereby treating unequals with equal. With reference to the earlier Statute i.e. Tamil Nadu
Acquisition of Hoarding Act, 1985 (in short the Acquisition Act) it was submitted that the
acquisition of the public or private property was held to be illegal. With effect from
23.7.1998 amendment was made to the Act and Sections 326-A to 326-1 were introduced.
Section 326-B provides for the period of 30 days within which the owners of the
hoarding were to apply for licence. On 5.9.2000 the Act was amended and Section 326-J
was introduced. This provision permitted removal of all hoardings which are hazardous in
nature. Challenge was made to the same provision. The High Court by order dated
14.10.2001 upheld its validity. It was inter alia held that every hoarding which is adjacent
to the road is hazardous and has to be removed and the High Court judgment was
affirmed by this Court with certain modifications by this Court in P. Narayana Bhat v.
State of Tamil Nadu and Ors. (2001 (4) SCC 554). However, all the hoardings are not to
be treated as hazardous. What is hazardous is to be decided. The time period for making
application for licence was fixed. In the year 2003 the Advertisement Rules have been
enacted. It is submitted that the Rules used the expression obstruction. It was pointed out
that the obstruction refers to physical obstruction. Challenge is also made to Rule 6 which
relates to the width of the road. There is no forum available for questioning correctness of
the adjudication by the authorities. In any event it is submitted that the Rules cannot
apply to private sites. Construction of private buildings have been excluded. It is pointed
out that the concept of public order is being introduced but the same has to be relatable to
the parameters laid down in Dr. Ram Manohar Lohia v. State of Bihar and Ors. (1966 (1)
SCR 709). The public interest is relatable to Article 19(1)(g) and not Article 19(1)(a).
Hoardings are nothing but material for advertisement. Rule 9 relates to objectionable
hoardings. Placing strong reliance on Tata Press Ltd. v. M.T.N.L. (1995 (5) SCC 139) it is
contended that hoarding partake the character of commercial speech. Reference is also
made to the decisions in Sakal Papers (P) Ltd. and Ors. v. Union of India and Ors. (AIR
1962 SC 305) and Bennett 2001 AIR SCW 1643
AIR 1966 SC 740
1995 AIR SCW 3602
AIR 1973 SC 106

@page-SC2945
Coleman and Co. and Ors. v. Union of India and Ors. (1972 (2) SCC 788) to contend that
even if it is conceded for the sake of arguments that the provisions are regulatory, they
must be relatable to the parameters of Article 19(2). The regulation results in restriction
on use of private land for advertisement. It is submitted that as was noted in Sakal
Papers's case (supra) it curbs competition and in Bennett Coleman's case (supra) there
must be sufficient reason to curb the freedom of speech. Even over-burdensome levy
which affects freedom of speech was held to be unconstitutional. Reference is also made
to Romesh Thappar v. The State of Madras (AIR 1950 SC 124) and Brij Bhushan and
Anr. v. The State of Delhi (AIR 1950 SC 129) to contend that the restriction can be
relatable to public interest and not to public order. When commercial speech is protected
there is no reason to put restriction on putting hoardings. Public order relates to violence
and not law and order. The basic difference, it is submitted, between Articles 19(2) and
19(6) has not been kept in view. In essence it is submitted that display of information on
hoardings whether it is commerce, political and social is permitted by Article 19(1)(a) or
no restriction can be placed or right to disseminate information on the purported claim of
preventing obstruction or hazard to movement of traffic which is not covered by Article
19(2) as public order is not affected. The statutory rules are exhaustive of the restrictions
and restrictions do not apply to hoardings on a private land. Rules are discriminatory in
applying the same yardstick to public roads and private properties as the same treats
unequals as equals. Even if Section 326J can be used later, it has to specify reasons in the
show cause notice, has to be disposed of by the reasoned order after opportunity and the
right of appearance can lead to a decision. It is pointed out that unsustainable
discriminatory approach is adopted in permitting hoardings of political parties which are
certainly more hazardous. A different yardstick is being adopted and unguided power is
given to the authorities to adopt different norms.
10. Some of the petitioners have pointed out that there was a statutory cut off date fixed
i.e. 23-7-1998 and the enumeration was to be done to identify data as to which of the
hoardings existed prior to 23-7-1998 and the applicants may make an application before
the date extended.
11. It is submitted that Rule 6 is absolutely impracticable because most of the roads are
between 15 ft. to 50 ft. category. A statutory right is made illusory because of the size
restriction. Visibility per se is not hazardous. Rule 10 contains words which are imprecise
and flexible and the listing has not been done.
12. The statutory intention is to permit hoarding but by putting unnecessary and
unreasonable restrictions the provisions can be misused.
13. The visibility concept is important. The size as provided in Rule 6 has no basis to
ensure reasonable visibility and therefore it impeaches the Act.
14. There is no rule to operationalise Section 326B. The Act, the Advertisement Rules
and the form have to be operationalized as part of the composite scheme. It is pointed out
that right in question claimed by the appellants is a constitutional right and not statutory
right. What is objectionable is content. It is, by way of clarification, submitted, that
content is covered by Article 19(1)(a) as it covers both antecedent steps and actual
display. The content is not limited to words, colour, picture but also extent, form, size and
placement. While Rule 3(b)(ii) is a pre-censorship concept, Rule 9 is post censorship. The
freedom of speech relates both to pre and post censorship.
15. The right claimed is a preferred right. While balancing free speech against restriction,
shift of emphasis is to free speech :
16. In exercise of public power there has to be guided discretion. In the instant case there
is no guided discretion. The right to regulate being exercised in the instant case is
restrictive and not regulatory.
17. In response, learned counsel for the respondent have submitted that the appellants and
many like them have continued litigation frustrating regulation of hoardings in Chennai.
It is submitted that owners of advertisement hoardings in the city of Chennai have
persistently challenged and resisted the regulation on the erection of hoardings for the last
two decades with the result that even today city of Chennai presents the most deplorable
huge advertisement hoardings on major roads, which are not only aesthetically
objectionable but are
@page-SC2946
hazardous and dangerous to traffic. Even after continued failure to get any relief from the
Court the challenge is still continued.
18. Following the directions of this Court in M.C. Mehta v. Union of India and Ors.
(1998 (1) SCC 363), the Tamil Nadu Legislature introduced the amendment in Section
326J by Amendment Act 2000. By an amendment, the Commissioner (later amended to
District Collector) was empowered to remove the existing hoardings which were
dangerous and causing disturbance to safe traffic movement, which adversely affect free
and safe flow of traffic. The provision also empowers the District Collector to refuse the
license for such hazardous and dangerous hoardings.
19

. Earlier challenge was made to the Rules of 1998. Various contentions were raised before
the High Court including reference to Tata Press case (supra). The High Court rejected
the contention that Section 326A was arbitrary and had laid down no guidelines.
Correctness of the judgment was questioned including the alleged infringement of Article
19(1)(a). This Court rejected the appeals stating that it was ad idem with most of the
conclusions arrived at by the High Court in the impugned judgment before it. This Court
also noted that before it very same stands were reiterated. It was noted that this Court was
inclined to agree with the High Court that Section 326J was neither ultra vires Article 14
nor Article 19(1) (a) of the Constitution in view of the decision given by the High Court.
Despite this, the Advertisement Rules were challenged before the High Court. The High
Court dismissed them subject to modifications. 1995 AIR SCW 3602

20

. It is to be noted that in P. Narayan's case (supra) this Court had specifically held in
concurring with the views of the High Court that Article 19(1)(a) of the Constitution was
not violated. 2001 AIR SCW 1643

21. Section 326A defines 'hoardings' to mean any screen or board at any place whether
public or private used or intended to be used for exhibiting advertisements....." Sections
326B to 326J are provisions relating to licensing of hoardings. The Act requires licences
of hoardings; and it requires licences of hoardings in both public and private places.
22

. So far as public places are concerned, the State has a full right to regulate them, as they
vest in the State as trustees for the public. The State can impose such limitations on the
user of public places as may be necessary to protect the public generally. (See Saghir
Ahmed v. State of U.P. 1955 SCR 707). AIR 1954 SC 728

23. Hoardings erected on private places also require to be licensed and regulated as they
generally abut on and are visible on public roads and public places. Hoarding erected on a
private building may obstruct public roads when put up on private buildings; they may be
dangerous to the building and to the public; they may be hazardous and dangerous to the
smooth flow of traffic by distracting traffic, and their content may be obscene or
objectionable. It is, therefore, not correct that hoardings on private places do not require
to be regulated by licensing provisions.
24. Rule 6 of the 2003 Rules put restrictions on the size of hoardings, on their height, the
spacing, etc. and the requirement of erection on steel frames. Rule 10 restricts the
hoarding to be put on certain places such as educational institutions, places of worship,
hospitals, corners of roads, in front of places of historical and aesthetic importance.
25

. The power to license is not unfettered and is guided by the above considerations. Under
Rule 11 an appeal lies to the State Government for refusing the grant or renewal of
licenses. Section 326J of the Act empowers the District Collector to prohibit the erection
of hazardous hoardings and hoardings which are hazardous and a disturbance to the safe
traffic movement so as to adversely affect the free and safe flow of traffic. The power
under Section 326J is not arbitrary as held by the Supreme Court in M.C. Mehta v. Union
of India (1998) 1 SCC 363) on an identical provision relating to case of hoarding in New
Delhi. Any action taken under Section 326 J must be taken by observing the principles of
natural justice and supported by reasons. An appeal against the order of the District
Collector for action under Section 326J lies to the State Government under Section 326H.
There cannot be a presumption of misuse of power merely because discretion is conferred
on a public authority for the exercise-use of the power. In Narayana Bhat's case, this
Court has negatived the contention 2001 AIR SCW 1643

@page-SC2947
that the power of the licensing authorities is arbitrary and unguided.
26. Section 326A to Section 326H and the 2003 Rules are made in public interest for the
purpose of
(i) Preventing haphazard erection and proliferation of hoardings in the city.
(ii) For orderly and aesthetic appearance in the city.
(iii) For safety and prevention of hazardous and dangerous hoardings.
27. Section 326-J of the Act prohibits erection of certain hoardings which are hazardous.
The expression "hazardous" as an adjective, connotes something that is "risky" or
"dangerous" vide, Blacks Law Dictionary, Eighth Edition, page 736.
28. Section 326-J provides that where the Commissioner is satisfied that the erection of
any hoarding visible to the traffic on the road is hazardous and disturbance to the safe
traffic movement so as to adversely affect the free and safe flow of traffic, he shall not
grant any licence under Section 326-C.
29. The Commissioner is also empowered to remove any such hoarding which is erected
in contravention of the provisions thereof.
30. The provisions contained in Rule 3 do not restrict or control the scope of Section 326-
J which operates on a wider plain. While failure to obtain a no objection certificate in
terms of Rule 3(iii) itself would disentitle an applicant for the grant of a licence to erect a
hoarding, Section 326-J, prohibits erection of hazardous hoardings and also mandates the
Commissioner (now District Collector) not to grant any licence under Section 326-C in
respect of such hoardings. It also authorizes the Commissioner to order confiscation and
removal of such hoardings which are erected in contravention of the mandate therein.
31

. A delegated legislation can be declared invalid by the Court mainly on two grounds
firstly that it violates any provision of the Constitution and secondly it is violative of the
enabling Act. If the delegate which has been given a rule making authority exceeds its
authority and makes any provision inconsistent with the Act and thus overrides it, it can
be held to be a case of violating the provisions of the enabling Act but where the enabling
Act itself permits ancillary and subsidiary functions of the legislature to be performed by
the executive as its delegate, the delegated legislation cannot be held to be in violation of
the enabling Act. (See vide, State of MR and another v. Bhola alias Bhairon Prasad
Raghuvanshi (2003) 3 SCC 1). 2003 AIR SCW 752

32

. In St. Johns Teachers Training Institute v. Regional Director, National Council for
Teacher Education and Another (2003) 3 SCC 321, this Court has held that : 2003
AIR SCW 894, Para 10

"Delegated legislation permits utilization of experience and consultation with interests


affected by the practical operation of statutes. Rules and Regulations made by reason of
the specific power conferred by the Statutes to make Rules and Regulations establish the
pattern of conduct to be followed. Regulations are in aid of enforcement of the provisions
of the Statute. The process of legislation by departmental Regulations saves time and is
intended to deal with local variations and the power to legislate by statutory instrument in
the form of Rules and Regulations is conferred by Parliament. The main justification for
delegated legislation is that the legislature being over burdened and the needs of the
modern day society being complex it cannot possibly foresee every administrative
difficulty that may arise after the Statute has begun to operate. Delegated legislation fills
those needs".
33. It is well settled that a delegated legislation would have to be read in the context of
the primary statute under which it is made and, in case of any conflict, it is primary
legislation that will prevail.
34. In ITW Signode India Ltd. v. Collector of Central Excise (2004) 3 SCC 48) this Court
has held as under : "It is well settled principle of law that in case of a conflict between a
substantive Act and delegated legislation, the former shall prevail inasmuch as delegated
legislation must be read in the context of the primary/legislative Act and not vice versa".
35. The expression obstruction means "something that impedes or hinders". The
expression, however, has varied sets of meaning and is not necessarily confined to
physical obstructions only.
36. It has been held that "Obstructing" the police, includes anything which makes it more
difficult for the police to carry out their duties and is not confined to mere physical
obstructions, vide Hinchliffe v.
@page-SC2948
Sheldon,(1955) 1 WLR 1207.
37. Obstruction has a wider meaning than mere physical obstruction and it includes
tangible and identifiable obstruction and even a protest is obstructing.
38

. In Collector of Customs and Central Excise, Bhubneshwar v. Paradip Port Trust and
Another (1990 (4) SCC 250) this Court, construing the expression "obstruction"
appearing in Section 133 of the Customs Act, 1962 has been pleased to hold : AIR
1990 SC 1897, Para 11

"On the authority of Hinchliffe v. Sheldon it can be said that obstruction is not confined
to physical obstruction and it includes anything which makes it more difficult for the
police or public servant to carry out their duties."
39. The expression 'obstruction' in Rule 3(iii) would, therefore, include any act which
impedes the free and safe movement of the traffic, pedestrians and vehicles. Such an act
may well be, by reason of what is displayed on the hoardings. If the subject-matter that is
displayed in such hoardings attracts attention of the drivers of vehicles and which, in turn,
impedes free and safe movement of traffic such a hoarding would clearly come under the
meaning "obstruction" contemplated under Rule 3(iii) of the Rules.
40. It is to be noted that there is certainly some difference between hazardous and
obstruction though there may be some amount of overlapping. What is hazardous cannot
have definite terms. So in that sense, Legislature had thought it wise to use the expression
obstruction so that it can be brought within manageable standards. The ultimate objective
is safe traffic movement and free and safe flow of traffic.
41. It can be seen in applying Section 326J, the authority empowered can give No
Objection Certificate and looking at the fact situation in a given case say obstruction has
been caused. What is physical distortion or destruction can also be considered. But the
conclusions can be challenged.
42

. The problem can be looked at from another angle. Even if there is no obstruction but
there is distraction that is also to be considered. As was considered by this Court in P.
Narayan's case (supra) the provisions like appeal and the rules to bring in the principles
of natural justice can be pressed into service. That will be a right step to avoid
arbitrariness. It has been contended emphatically that private hoardings shall not cause
any physical obstruction. But this plea is, as noted above, without any substance. In our
view there may not be physical obstruction but it can be hazardous. The right to regulate
and control is inherent in exercise of power. 2001 AIR SCW 1643

43. One other thing which needs to be noted is that the authority is not examining the
contents of hoardings, size etc. The licence is for putting the hoardings. It depends upon
the size and at that stage the question of content does not come into picture. If it is
distraction, the question whether it is hazardous or creates obstruction comes later.
44. Under Rule 9 the District Collector can suo motu take action if he finds hoardings to
be objectionable. The provisions appear to be not restrictive but are regulatory. There is
no ban on advertisement hoardings but obstructive and destructive ones are to be
prohibited.
45. The apprehended arbitrariness can be well taken care of. If show cause notice is
issued, it should specify the reasons as to why the action is proposed to be taken in
respect of any hoarding or hoardings. The principles of natural justice can also be
complied with if reasons are indicated in the show cause notice and there is scope for
reply to be given. Thereafter, reasoned adjudication can be made by the authorities. It
goes without saying that objectivity has to be there, even though initially at the stage of
issuing show cause notice there is subjectivity.
46. Very narrow and stringent limits have been set to permissible legislative abridgment
of the right of free speech and expression, and this was doubtless due to the realisation
that freedom of speech and of the press lay at the foundation of all democratic
organizations, for without free political discussion no public education, so essential for
the proper functioning of the processes of popular government, is possible. A freedom of
such amplitude might involve risk of abuse. But the framers of the Constitution may well
have reflected, with Madison who was "the leading spirit in the preparation of the First
Amendment of the Federal Constitution," that "it is better to leave a few of its noxious
branches to their luxuriant growth, than, by pruning them away, to injure the
@page-SC2949
vigour of those yielding the proper fruits" :

[Quoted in Near v. Minnesotta [283 U.S. 607) (Also See Romesh Thappars case (supra).
AIR 1950 SC 124

47. So far as the question relating to enumeration in Rule 10, the High Court has taken
care of that problem by appointing a Committee to identify the places, it was submitted
that some of the directions need to be clarified. Though it is conceded that the directions
are not wrong it is submitted that they but need clarification. It is open to the appellants if
so advised to move the High Court if any clarification is necessary. But in our view the
directions cannot be faulted. It is submitted that direction No. 16 relates to forthwith
demolition. It needs no reiteration that the High Courts order is clear to the effect that
only after enumeration the demolition can be done.
48

. It is to be noted that M.C. Mehta's judgment (i.e. 1998 (1) SCC 363) resulted in
amendment of the Act. In the said case the direction given in M.C. Mehta v. Union of
India and Ors. (1997 (8) SCC 770) has been quoted. The Advertisement Rules in essence
constitute a Code for regulating erection of hoardings and do not deal with content except
where it is found to be obscene or objectionable. 1997 AIR SCW 4149

49. There are two tier arrangements in Rule 3 (b). One relating to NOC by the police and
the other the power of the District Collector to see whether the hoardings fall foul of
Section 326J and was in line with it. It needs no emphasis that the size is the yardstick
and smaller the road the bigger is the hazard because the unregulated size can lead to
chaos.
50

. In Saghir Ahmad v. State of U.P. and Ors. (1955 (11) SCR 707) it was held as follows :
AIR 1954 SC 728

"2. In view of this pronouncement of law, the State Government, which wanted to have
the exclusive right to operate Road Transport Services within its territory, sought the
assistance of the Legislature and the U.P. Road Transport Act (Act II of 1951) was passed
and became law on and from the 10th of February, 1951. It is the constitutional validity
of this enactment which is the subject-matter of contest in these present proceedings."
51. The fact that the hoarding is on building or private land does not take away the
regulatory measures relating to hoardings. There can be cases where because of the size
and the height, it can be dangerous to public and also be hazardous. There is no structural
safeguard in respect of such hoardings. There has to be regulatory measures. As has been
rightly contended by learned counsel for the respondents, the Act and the Advertisement
Rules do not regulate advertisement. They regulate putting of the hoarding which is
found to be objectionable, destructive or obstructive in character.
52. It cannot be said that there is infringement of freedom of speech. The content, effect
and the purpose of statute clearly show that it is not intended to be so.
53. The inevitable result is that the appeals and writ petitions are without merit and
deserve to be dismissed which we direct.
Order accordingly. .
AIR 2008 SUPREME COURT 2949 "State of U. P. v. Promod Kumar Shukla"
(From : Allahabad)*
Coram : 2 Dr. A. PASAYAT AND LOKESHWAR SINGH PANTA, JJ.
Civil Appeal No.2094 with 2095 of 2008 (arising out of SLP (C) Nos. 19890 of 2005
with 11752 of 2006), D/- 25 -3 -2008.
State of U.P. and Anr. v. Promod Kumar Shukla and Anr.
U.P. Cinema Regulation Act (1955), S.5, S.7 - CINEMATOGRAPH - LICENSE -
WRITS - Cinema licence - Revocation - Licence obtained by falsely representing that
applicant is owner of cinema - Grant-in-aid also received - Order cancelling licence,
grant-in-aid and for recovery of aid already received - Interference with, on ground that
question of fraud should not have been decided by authorities under Act - Improper.
C.M.W.P. No. 33291 of 2004, D/-22-02-2005 (All.), Reversed.
Constitution of India, Art.226. (Paras 4, 7)

Shall Kr. Dwivedi, A.A.G., Krishnan Venugopal, Revindra Kumar, Gunnam


Venkateswara Rao, Kuldip Singh, Dhruv Mehta, P.N. Puri, Dhiraj, Ms. Reeta Dewan
Purl, R.K. Pandey, T.P. Mishra and Kamlendra Misra, with him for the Appearing
@page-SC2950
Parties.
* C.M.W.P. No. 33291 of 2004, D/- 22-2-2005 (All).
Judgement
1. Dr. ARIJIT PASAYAT, J. :- Leave granted.
2. These two appeals arise out of a common judgment of the Allahabad High Court
allowing the Writ Petition (Civil Misc. Writ Petition No. 33291/2004) and holding that no
further order need be passed in the connected Writ Petition (Civil Misc.W.P. 37610/
2004) in view of the order of the former case.
3. Challenge in the first writ petition was to the order passed by the District Magistrate,
Allahabad dated 2-8-2004 holding that respondent-Pramod Kumar Shukla had received
grant-in-aid to the tune of Rs. 21,27,551.13 between 2-4-1990 to 1-4-1995 under
Government Order dated 21-7-1986 by concealing facts and by practising fraud. It was
pointed out that he had concealed the fact that he was not the owner and was not,
therefore, entitled to receive the grant in aid. Therefore, in exercise of powers conferred
under Section 5(3) of the U.P. Cinema Regulation Act, 1955 (in short the Cinema Act)
order of recovery was given and grant-in-aid sanctioned to him vide office order No. 299
dated 10-4-1990 was set aside. Application for renewal dated 1-7-1994 submitted by said
Pramod Kumar Shukla was rejected in exercise of powers conferred under Section 21 of
the U.P. General Clauses Act, 1904 (in short General Clauses Act). Further order was
passed under Section 12(1) of the U.P. Entertainment Tax Act, 1979 (in short
Entertainment Act) directing him to deposit the amount of entertainment tax collected by
such cheating and fraud during 2-4-1990 and 1-4-1995 amounting to Rs. 21,27,551.13.
4. Background facts which are almost undisputed run as follows :
Respondent-Pramod Kumar Shukla is the son of Shri Satya Prakash Shukla who is the
appellant in appeal arising out of SLP (C) No. 11752/2006. A Cinema Hall named Girija
Chitralaya was granted temporary permit for six months. Undisputedly, on 10-11-1986
the original owner Shri Girija Shankar Shukla had executed a Power of Attorney
appointing his grandson Pramod Kumar Shukla as the Power of Attorney holder. The said
Power of Attorney was executed on 10-11-1986 and was registered with the Sub-
Registrar, Farukkhabad on 14-11-1986. On 31-10-1988 permission was granted to
construct a permanent cinema hall. Pramod Kumar Shukla had made an application on 6-
9-1988 suppressing the fact that Girija Shankar Shukla had expired on 31-3-1987. In the
application filed, Pramod Kumar Shukla described himself as the owner of Cinema Hall
and indicated in the application that he was running a temporary cinema hall and wanted
to construct a permanent cinema building on the concerned plot of land. Interestingly, he
did not apply as a Power of Attorney holder but stated that he was the owner. Permission
was granted on certain conditions by the District Magistrate, Allahabad.
It is the stand of the appellants that Pramod Kumar Shukla falsely represented himself as
the owner and the licence was granted on the premises that Pramod Kumar Shukla was
the owner of the Cinema Hall. His father Satya Prakash Shukla made a representation
and, therefore, there was no renewal of the licence which operated from 30-3-1990 to 31-
3-1993 both days inclusive. A show cause notice was issued on 19-6-2004 alleging that
the permission granted for operating the permanent Cinema Hall was obtained by
suppressing the factual position by Pramod Kumar Shukla misrepresenting himself to be
the owner. Notice was given to show cause as to why the amount of grant-in-aid which
was obtained by fraud and by concealing the facts shall not be recovered under Section
12(1) of the Entertainment Act and the grant-in-aid sanctioned by order No. 299 dated
10-4-90 should not be cancelled and the application for renewal of licence should not be
rejected.
In grant-in-aid order dated 10-4-1990, Pramod Kumar Shukla was indicated to be the
licensee. The order passed by the District Magistrate was challenged in the Writ Petitions.
The High Court after referring to the factual scenario came to hold that the order was
passed without applying mind and with undue haste. It was noted that the authorities
should have taken appropriate legal help to understand how far such executive authority
can go to determine the issue. It was not a case of fraud between an individual and the
State by which the revenue exchequer would suffer but was a dispute between the father
and the son and without ascertaining the position either by Civil Court having appropriate
jurisdiction in respect of right, title and interest of the property and accounts or by
Criminal Court
@page-SC2951
as regards proof of fraud and determination in respect of forgery taking help of
appropriate mechanism, the order impugned was passed which was illegal.
5. Learned counsel for the appellants has submitted that the High Court completely
misconstrued the nature of the dispute. It lost sight of the fact that Pramod Kumar Shukia
had fraudulently projected himself to be the owner which admittedly he was not. The fact
that the executor of the Power of Attorney had died in 1987 much before the application
for renewal and/or application for permanent Cinema Hall was filed was not disputed.
6. Learned counsel for the respondent-Pramod Kumar Shukia on the other hand
supported the judgment of the High Court stating that in a case of this nature the
Collector should not have passed the impugned order. His claim was that there was a
family settlement and certain documents executed by his father Satya Prakash Shukia
clearly established that he was the owner of the Cinema Hall.
7. The High Court seems to have completely lost sight of the nature of the controversy
and the dispute. Whether there was any fraud practised could not have been decided in
the Writ Petition. Under Section 7 of the Cinema Act the power to revoke and cancel the
license is available to the appropriate authority. It appears that the High Court has not
examined the question as to what is the effect of Girija's death. It has also not examined
the acceptability of the claim of Pramod Kumar Shukia that he was the owner of the
Cinema Hall in which capacity he had applied for the permanent licence. These have
considerable bearing on the subject matter of dispute. The High Court has come to an
abrupt conclusion without analyzing the factual and applicable legal position. That being
so, we set aside the impugned order of the High Court and remit the matter to it for fresh
disposal in accordance with law. We request the High Court to dispose of the matter
within 4 months from today.
8. The appeals are allowed to the aforesaid extent with no order as to costs.
Appeals allowed. .
AIR 2008 SUPREME COURT 2951 "Yashwant Waman Patil v. Municipal Corporation
of Greater Mumbai"
(From : Bombay)
Coram : 2 TARUN CHATTERJEE AND H. S. BEDI, JJ.
Civil Appeal No.1398 of 2008 (arising out of SLP (C) No. 5670 of 2007), D/- 18 -2
-2008.
Yashwant Waman Patil and Ors. v. Municipal Corporation of Greater Mumbai and Ors.
Constitution of India, Art.226, Art.133 - WRITS - APPEAL - ACQUISITION OF LAND
- HIGH COURT - Interim order in writ petition - Appeal against - Acquisition of land -
Compensation - Writ petition seeking deposit of additional sums in terms of order of
High Court and for release of amount deposited in favour of appellants, landowner -
Interim order by High Court allowing appellants to withdraw said amount on condition of
their furnishing bank guarantee in respect of said sum - Supreme Court modified the said
order and directed to deposit said amount in fixed deposit in name of appellants and
interest accrued thereon permitted to be withdrawn without furnishing security. (Para
4)

Paramjit Singh Patwalia, Sr. Advocate, Gulabrao Shelke, Ms. Mita Yashvant Patil, M.Y.
Deshmukh, Ms. Neetu S. Chauhan and Rameshwar Prasad Goyal, with him for
Appellants; Pallav Shishodia, Atul Y. Chitale, Mrs. Suchitra A. Chitale and Amit Pattekar,
for Respondents.
Judgement
1. JUDGMENT :-Leave granted.
2. This appeal is directed against an interim order dated 22nd of December, 2006 passed
by a Division Bench of the High Court of Judicature at Bombay, (Ordinary Original Civil
Jurisdiction) in Notice of Motion No. 380 of 2006 arising out of Writ Petition No. 11 of
2005. On a pending writ application, a notice of motion was filed by the present
appellants, inter alia, praying against others for deposit of additional sums in terms of an
order of the Bombay High Court dated 29th of November, 2005 and also for an early
hearing of the writ petition and further to direct the release of the amount deposited in
favour of the appellants. It is not in dispute that in terms of the calculation sheet, the
municipal corporation of Greater Bombay had already deposited in the High Court a sum
of Rs. 17,23,29,933/-. The High Court by the impugned order had allowed the appellants
to withdraw the amount deposited on the condition
@page-SC2952
that the appellants shall furnish a bank guarantee in respect of the said sum on a prima
facie finding that the Municipal Corporation had enjoyed the possession of the acquired
land for more than 40 years without making payment of compensation. It is this order,
which is now under challenge in the present appeal.
3. We have heard Mr. Paramjit Singh Patwalia, learned senior counsel appearing for the
appellants and Mr. Pallav Shishodia, learned counsel appearing for the respondents. Mr.
Patwalia contended that the High Court had committed an error in imposing a condition
on the appellants to furnish bank guarantee for withdrawal of the compensated amount
when the respondents enjoyed the possession of the acquired land for more than 40 years
without making payment of compensation. According to Mr. Patwalia, since the
appellants had no source of income nor had any property to secure the withdrawal of the
amount or to furnish bank guarantee, it would be a mere impossibility to withdraw the
compensated amount and accordingly, in the facts and circumstances of the case, the
condition imposed on the appellants should be withdrawn. This submission of Mr.
Patwalia was hotly contested by Mr. Pallav Shishodia, learned counsel appearing for the
respondents who contended that the appellants have already been paid in excess under the
terms and conditions of an agreement of the year 1978 and if now they are permitted to
withdraw the amount already deposited and lying in the court and in view of the
submission of Mr. Patwalia that the appellants have no source of income nor have any
property to secure the amount that would be withdrawn, it would be a mere impossibility
to recover the amount from the appellants, if allowed to be withdrawn, in the event the
writ petition succeeds in which the award in question has been challenged.
4. Having heard the learned counsel for the parties and after considering the materials on
record and considering the submissions made on behalf of the parties, we dispose of this
appeal with the following directions :
[a] The amount deposited in the High Court shall be deposited in fixed deposit of any
nationalized bank in the name of the appellants within a period of one month from this
date initially for a period of six months which shall be renewed from time to time until
further orders of the High Court or till the disposal of the writ petition whichever is
earlier.
[b] The interest that would accrue on the aforesaid sum shall be permitted to be
withdrawn without furnishing any security or without furnishing bank guarantee by the
appellants but the principal amount to be invested as a fixed deposit in the Bank shall not
be withdrawn by the appellants till the disposal of the writ petition or until further orders
of the High Court.
[c] It is not in dispute that the amount awarded has already been deposited in the High
Court, which is now lying. We are also informed that the amount has already been
invested by the High Court. If the amount has already been deposited in a Fixed Deposit
and any interest has already been accrued, the High Court is directed to release the
interest amount out of the amount already deposited as a Fixed Deposit in favour of the
appellant.
[d] However, the High Court is requested to dispose of the pending writ petition within a
period of three months from the date of supply of copy of this order, without granting any
unnecessary adjournment to either of the parties.
5. Accordingly, we modify the interim order of the High Court in the above manner. The
appeal is thus disposed of. No order as to costs.
Order accordingly. .
AIR 2008 SUPREME COURT 2952 "B. S. N. L. v. Rajesh Kumar Saxena"
(From : 2007 (112) Fac LR 784 : 2007 (4) ALJ (DOC) 107)
Coram : 2 TARUN CHATTERJEE AND H. S. BEDI, JJ.
Civil Appeal No.1980 of 2008 (arising out of SLP (C) No. 3643 of 2007), D/- 13 -3
-2008.
B. S. N. L. and Ors. v. Rajesh Kumar Saxena.
Constitution of India, Art.16, Art.309 - Central Civil Services (Pension) Rules (1972), R.9
- EQUALITY IN PUBLIC EMPLOYMENT - SERVICE MATTERS - CIVIL SERVICE -
PENSION - PROVIDENT FUND - SUPERANNUATION - Pension - Govt. employee
attained age of superannuation during his suspension and was retired provisionally -
Held, as per rules would be entitled to payment of provisional pension and to release of
his provident fund alone. (Para 5)
@page-SC2953

K.C. Kaushik, Rahul Kaushik and Ashok Kumar Singh, for Appellants; Manoj Swarup
and Rohit S., for Respondent.
Judgement
1. HARJIT SINGH BEDI, J. :- Leave granted.
2. This appeal by way of special leave is based on the following facts.
3. The respondent Rajesh Kumar Saxena was placed under suspension vide order dated
26th June, 2004 under sub-rule(1) of Rule 10 of the Central Civil Services
(Classification, Control and Appeal) Rules 1965 on the allegation that while working as
an Officiating Chief Accounts Officer in Moradabad in the year 2003, he had failed to
maintain absolute integrity and shown lack of devotion to duty. The respondent however
attained the age of superannuation during his suspension and was retired provisionally on
31st July, 2004 as per order Annexure P-2, with the stipulation that as per the Central
Civil Services (Pension) Rules, 1972 the payments due to him would be made on
finalization of the disciplinary proceedings. The order dated 26th June, 2004 was
challenged before the Allahabad High Court by way of a Writ Petition. The Division
Bench of the High Court in its order dated 23rd November, 2006 held that as the charge
sheet had been served on the appellant more than one year after his retirement, the
proceedings against him were nonest and that as suspension itself was not a punishment
and that in any case, it would 'evaporate' on his superannuation, there was no justification
in withholding the retirement benefits such as pension, provident fund, leave salary,
group insurance, CDS amount etc. and a direction was issued to make all payments
within a period of two months from the date of communication of the order. It is in this
circumstance, the present appeal is before us by way of special leave at the instance of
the department.
4. Several arguments have been raised by the learned counsel for the appellant during the
course of the hearing and reference has been made to various rules. Special reference has
however been laid on the fact that very serious allegations of misconduct on the part of
the respondent and 30 other departmental officers who too had been suspended had been
revealed during investigation.
5. The learned counsel for the respondents has however pointed out that the question as to
whether initiation of proceedings could or could not be made after the retirement of the
respondent and as to whether the chargesheet had been served within the requisite period
had been raised in this appeal and which needed to be answered before a reversal of the
High Court's judgment. We are however of the opinion that at this stage we are not called
upon to go through these issues as to do so could have the effect of damaging the cause of
one or the other party in the departmental proceedings that are presently going on and in
particular, those amongst the suspended officials who are not before us in this matter. We
are accordingly of the opinion that as per the rules, the respondent would be entitled to
the payment of provisional pension and to the release of his provident fund alone and he
would have to await the decision on the enquiry before the release of the other retiral
benefits. We also make it clear that as we have not dealt with the basic legal issues raised
by the parties, they would be at liberty to raise them at any subsequent stage should the
need so arises. We allow the appeal to the above extent. The order of the High Court shall
be deemed to be modified in the above terms.
Order accordingly. .
AIR 2008 SUPREME COURT 2953 "Cantonment Executive Officer v. Vijay D. Wani"
(From : Bombay)
Coram : 2 A. K. MATHUR AND LOKESHWAR SINGH PANTA, JJ.
Civil Appeal No.18 of 2007, D/- 16 -4 -2008.
Cantonment Executive Officer and Anr. v. Vijay D. Wani and Ors.
Cantonment Act (2 of 1924), S.120 - Cantonment Boards Servants Rules (1937), R.12 -
CANTONMENT - DISCIPLINARY ENQUIRY - BIAS - MISCONDUCT -
REINSTATEMENT - BACK WAGES - Disciplinary enquiry - Plea of bias - Misconduct
by Junior Engineer - Three persons who conducted inquiry were also members of
Cantonment Board - Their report to be considered by Board - They would be interested to
see that report is accepted - Participation of said members in decision making process for
finding delinquent guilty and ordering his dismissal from service - Is bias which is
apparent and real - Delinquent is entitled to be reinstated with 50% backwages.
AIR 1987 SC 71; AIR 1957 SC 425;
@page-SC2954
2002 AIR SCW 295 and 1999 (1) All ER 577 (HC), Rel. on.
2005 AIR SCW 6180, 2005 AIR SCW 1040 and 2000 AIR SCW 3745, Disting. (Paras
5, 7, 9)
Cases Referred : Chronological Paras
2005 AIR SCW 1040 : AIR 2005 SC 1562 : 2005 Lab IC 1320 : 2005 All LJ 914
(Disting.) 8
2005 AIR SCW 6180 : AIR 2006 SC 531 : 2006 Lab IC 219 : 2006 (1) ALJ 496
(Disting.) 8
2002 AIR SCW 295 : AIR 2002 SC 678 : 2002 Lab IC 521 (Rel. on) 6
2000 AIR SCW 3745 : AIR 2000 SC 3243 : 2000 Lab IC 3678 (Disting.) 8
(1999) 1 All ER 577 (HC) (Rel. on) 6
AIR 1987 SC 71 (Rel. on) 3, 6
AIR 1957 SC 425 (Rel. on) 6
138 F 20 650 6
D. Rama Krishna Reddy and Mrs. D. Bharathi Reddy, for Appellants; Shivaji M. Jadhav,
for Respondents.
Judgement
1. A. K. MATHUR, J. :- This appeal is directed against the order, dated 10-1-2005 passed
in Writ Petition No. 966 of 1995 by the High Court of Judicature at Bombay whereby the
Division Bench has set aside the resolution of the Cantonment Board, Pune dated 29-10-
1991 removing the respondent from service which is completely vitiated on account of
the participation of the three members of the Enquiry Committee and the orders of the 1st
and 2nd Appellate authorities dated 8-7-1992 and 22-12-1994 and allowed the writ
petition of the Vijay D. Wani respondent (herein) and directed the Cantonment Board to
reinstate the petitioner (respondent herein) into service with 50% backwages and
continuity of service.
2. Brief facts which are necessary for disposal of this appeal are that the respondent was
appointed as Junior Engineer (Electrical) with Pune Cantonment Board with effect from
9-3-1977. Later on he was redesignated as Sectional Engineer (Electrical). In 1987, the
Cantonment Board decided to purchase N.C.T. pies for street lighting and directed the
respondent to prepare an estimate. Similarly he was also directed to prepare estimates for
electrification of S.V.P. Cantonment General Hospital, for the purpose of air conditioning
of the Operation Theater and for purchase of transformer for the same hospital. The
Contonment Board also wanted him to prepare estimates of sewerage pumps for
Ghorpadi and Wanawadi Bazar Draining Scheme and also estimates for cables and street
lights at Price of Wales Drive. The respondent as a Sectional Engineer (Electrical)
prepared all those estimates. But on 11th August, 1987, the office of the Cantonment
Board through the Chief Executive Officer served him a memorandum alleging that the
estimates prepared by the respondent suffered from total non-application of mind. The
respondent offered his explanation dated 25-8-87 to the said memorandum but that was
not accepted by the Board. A charge-sheet containing the same charges was issued to the
respondent on 13-1-1988. The respondent was put under suspension and the Cantonment
Board appointed an Enquiry Committee to enquire into the alleged misconduct of the
respondent. The Enquiry Committee found the charges proved by majority of two versus
one the third member differed on items 2 and 4. By a resolution dated 25-10-1991 the
Cantonment Board considered the Enquiry Committee's report and accepted it and passed
the order of removal of the respondent from service. The respondent filed an appeal to the
GOC-in-Chief, Southern Command, Pune and the same was dismissed on 8-7-1991. The
respondent preferred second appeal before the Government of India, Ministry of Defence,
which was also dismissed on 22-12-1994.
3

. Aggrieved against this order the respondent preferred an appeal before the High Court.
The High Court rejected the first contention of the respondent that all the three members
of the Enquiry Committee happened to be the members of the Board in which capacity
they had scrutinized, approved and accepted the estimates prepared by the respondent
when the estimates were placed before the Cantonment Board. Since they were interested
in the matter, therefore, the enquiry should have been quashed on the ground of bias.
Secondly, it was contended that the alleged misconduct of the respondent themselves
participated in the meeting of the Cantonment Board and voted in favour of the report
while considering the issue of inflicting punishment on the respondent. It was also
contended that the participation of the members of the Enquiry Committee in the Board
meeting when the report was under consideration completely vitiates the inquiry. In
support of this, the AIR 1987 SC 71

@page-SC2955
learned counsel for the respondent relied on the decision of this Court; Institute of
Chartered Accountants of India v. L.K. Ratna and Ors. reported in 1986(4) SCC 537. So
far as first contention is concerned, the High Court did not find any fault that the
petitioner/respondent (herein) had not made any specific allegation against any Board
member of the Enquiry Committee nor had imputed any mala fide or illwill to any
members of the Enquiry Committee. Therefore, the contention of the learned counsel
appearing on behalf of the petitioner/respondent (herein) of bias was rejected. So far as
second contention is concerned, it was held that there was violation of principles of
natural justice in as much as all the three members of the Enquiry Committee participated
in the Board meeting and voted in support of their Enquiry report and held the respondent
guilty of misconduct and dismissed him from service. That vitiated the decision making
process as all the three members of the Enquiry Committee was part of the decision
making process and since they were interested to see that their report be upheld by the
Committee. Therefore, there was a legitimate apprehension in the mind of the respondent
that the three members of the committee who were inquiring against the respondent and
found him guilty were interested to see that their report should be confirmed by the Board
and this seriously prejudiced and biased the process of decision making him guilty. This
contention was upheld by the Division Bench and consequently the Division Bench set
aside the order of Cantonment Board as well as the order on appeal by the GOC-in-Chief,
Southern Command, Pune and the order passed by the Secretary, Government of India,
Ministry of Defence. Aggrieved against the order passed by the Division Bench of the
High Court, this appeal was filed by the Cantonment Board.
4. We have heard learned counsel for the parties and have gone through the records.
5. The question of a bias is always the question of fact. The courts has to be vigilant
while applying the Principles of bias as it primarily depends on the facts of each case.
The court should only act on real bias not merely on likelihood of bias. In the present
case, so far as the members of the committee who conducted a disciplinary inquiry was
also the members of the Cantonment Board where the report was to be considered,
decided and whether to accept it or not and finding the respondent (herein) guilty or not.
The very fact that these three persons who conducted inquiry were also the members of
the Board and that Board was to take a decision in the matter whether the report
submitted by the Enquiry Committee should be accepted or not. Therefore, the
participation of these three members in the committee is given a real apprehension in the
mind of the respondent that he will not get a fair justice in the matter because of the three
members who submitted the report would be interested to see that their report should be
accepted. This bias in this case cannot be said to be unreal it is very much real and
substantial one that the respondent is not likely to get a fair deal by such disciplinary
committee.
6

. In this connection a reference may be made to the decision in the case of Institute of
Chartered Accountants of India (supra) in which a member, accused of misconduct is
entitled to a hearing by the Council. In this case Enquiry Committee Composed of the
President and the Vice-President and three other members of the council who constituted
as members of the disciplinary committee, was also members. Their Lordships held as
under : AIR 1987 SC 71

"Accordingly, the finding of the council holding the respondent members guilty of
misconduct was vitiated by the participation of the members of the Disciplinary
committee."

This was on the basis of the Principle of apprehension of a bias. Their Lordships
observed in the case of Manek Lal v. Prem Chand reported in AIR 1957 SC 425 wherein
it was observed : (Para 4)

It is well settled that every member of a tribunal that is called upon to try issue in judicial
or quasi-judicial proceedings must be able to act judicially; and it is of the essence of
judicial decisions and judicial administration that judges should be able to act impartially,
objectively and without any bias. In such cases the test is not whether in fact a bias has
affected the judgment the test always is and must be whether a litigant could reasonably
apprehend that a bias attributable to a member of the Tribunal might have operated
against him in the final decision of the tribunal. It is in this sense that it is often said that
justice must
@page-SC2956
not only be done but must also appear to be done."
Similarly in the judicial review of the administrative action by Professor S.A. de Smith
has also observed :
".......a report will normally include a statement of findings and recommendations, which
may be controverted before the parent body; and in such a case, the participation of
members of the sub-committee in the final decision may be of dubious validity. The
problem is not merely one of strict law; it is also one of public policy."
Similarly, in the case of Pinochit Ugarta No. 2, reported in 1999 (1) All ER 577 (HL), it
was observed that a judge is automatically disqualified from hearing a matter in which he
has a pecuniary interest in the outcome as also when the decision would lead to
promotion of a cause in which he is involved, together with one of the parties.

Similarly, in the case of Amar Nath Chowdhury v. Braithwaite and Co. Ltd. reported in
2002 (2) SCC 290 it was observed that Managing Director dismissing an employee
cannot sit in the Board of Directors to hear the employee's appeal. Doctrine of necessity
was inapplicable as the Board could have delegated its appellate power to a committee.
2002 AIR SCW 295

Similarly in Sir Bloom-Cooper's Comment on "Bias in appeal", 2005 Public Law 225 in
which he quotes at page 227 a very illuminating judgment of Judge Jerome Frank in the
case of Rt.J.P. Linhan Inc., (138 F20 650) a brief excerpt from which reads :
"Democracy must, indeed, fail unless our courts try cases fairly, and there can be no fair
trial before a judge lacking in impartiality and disinterestedness. If, however, 'bias' and
'partiality' be defined to mean the total absence of preconceptions in the mind of the
judge, then no one has ever had a fair trial and no one ever will"
It was observed in the Ninth edition of Administrative Law by H.W.R. Wade and C.F.
Forsyth that Twentieth-century judges have generally enforced the rule against bias in
administrative proceedings no less strictly than their predecessors as exemplified by the
following cases :
The mere presence of a non-member while a tribunal is deliberating is enough to
invalidate the proceedings. Thus the proceedings of a Watch Committee, hearing an
appeal by a police sergeant against his dismissal by his chief constable, were fatally
flawed by the presence of the chief constable, whose mind was made up and who was in
effect the respondent, during the committee's deliberations. For similar reasons the court
quashed the decision of a disciplinary committee which had consulted privately with the
chief fire officer who had reported a fireman for indiscipline."
7. Therefore, the ratio of all these cases is that a person cannot be a Judge in his own
case. Once the disciplinary committee finds the incumbent guilty; they cannot sit in the
judgment to punish the man on the basis of the opinion formed by them. The objectivity
is the hallmark of a judicial system in our country. The very fact is that the disciplinary
committee who found the respondent (herein) guilty participated in decision making
process for finding the respondent (herein) guilty and to dismiss him from service is bias
which is apparent and real. Consequently, the view taken by the Division Bench of the
High Court cannot be faulted.
8. However, learned counsel for appellants submitted that since the respondent did not
work, therefore, he should not be paid any salary under the Rule "no work no pay". In
this connection he invited our attention to the following cases :

1. Baldev Singh v. Union of India and Ors. reported in 2005 (8) SCC 747. 2005 AIR
SCW 6180

2. India Literacy Board and Ors. V. Veena Chaturvedi and Ors. reported in 2005 (3) SCC
79. 2005 AIR SCW 1040

3. Badrinath v. Government of Tamil Nadu and Ors. reported in 2000 (8) SCC 395.
2000 AIR SCW 3745

In the case of Baldev Singh (supra), the appellant was held in a criminal case and
thereafter on his acquittal a question arose with regard to his back wages, their Lordships
held that it did not arise as he was lawfully confined. Therefore, this case is
distinguishable.
In the case of India Literacy Board and Ors. (supra), an SLP was filed against the interim
order and their Lordships held that no opinion need to be expressed on merits of the rival
contentions and directed the High Court to hear the main writ petition and dispose of the
same on merits including the question of maintainability of the petition.
@page-SC2957
And in the case of Badrinath (supra), question was of non-communication of adverse
remarks and no question of 'no work no pay' was involved. Hence, this case also does not
support the case of the appellant.
9. So far as grant of back wages is concerned, it depends upon case to case. But in the
present case as the respondent was found guilty by the Cantonment Board but the order
of Cantonment Board was set aside because it suffered from bias and it will be unfair to
deny 50% back wages to the respondent (herein). The Division Bench also directed that
more than 13 years have passed, therefore, it did not permit the respondent to proceed
against the petition afresh. The Division Bench decided the matter on 10th January, 2005
and now more than 16 years have lapsed. Therefore, it would not be fair to permit the
respondent to proceed afresh in the matter. Consequently, we do not find any merit in this
appeal and the same is dismissed.
10. The respondent be reinstated with the benefit of 50% back wages and continuity of
service.
11. No order as to costs.
Appeal dismissed. .
AIR 2008 SUPREME COURT 2957 "Regional Provident Fund Commissioner v.
Bhavani"
Coram : 2 ALTAMAS KABIR AND V. S. SIRPURKAR, JJ.
Civil Appeal No. 6447 of 2001*, With C. A. Nos. 2916-2919 of 2008 (arising out of SLP
(C) Nos. 15469 of 2005, 16573 of 2006, 20260 and 8661 of 2004) and C. A. No. 2629 of
2006, D/- 22 -4 -2008.
Regional Provident Fund Commissioner v. Bhavani.
(A) Employees' Provident Funds and Miscellaneous Provisions Act (19 of 1952), S.5 -
Employees' Pension Scheme, 1995 - PROVIDENT FUND - PENSION - DATE OF
BIRTH - SUPERANNUATION - Pension - Eligibility - Date of birth - Different date of
birth of respondent entered in records of employer and records of Regional Provident
Fund Commissioner (Commissioner) - Respondent continued to work with Company till
date of superannuation as recorded in Company's record - Contributed to fund till year
1995 - No explanation by Commissionner as to how such contributions were received
though according to record in office of Commissioner she was superannuated earlier -
Held, respondent was eligible for 1995 Scheme. (Para 19)
(B) Consumer Protection Act (68 of 1986), S.2(1)(d)(ii), S.2(1)(o) - CONSUMER
PROTECTION - FAMILY PENSION SCHEME - PROVIDENT FUND - 'Consumer' -
Respondent, an employee of a Company becoming member of Employees' Family
Pension Scheme, 1971 and contributing to same - She is thus, availing services rendered
by Regional Provident Fund Commissioner for implementation of Scheme - Regional
Provident Fund Commissioner, who is person responsible for working of Pension
Scheme is 'service giver' and respondent is 'consumer' - More so, as it was not a case of
rendering of free service or rendering of service under contract of personal service so as
to bring relationship between Regional Provident Fund Commissioner and respondent
within concept of 'master and servant'. (Para 20)
(C) Constitution of India, Art.16 - EQUALITY IN PUBLIC EMPLOYMENT - DATE
OF BIRTH - PROVIDENT FUND - PENSION - Correction of date of birth - Date of
birth of respondent correctly recorded in records of Company including respondent's
service records - On basis of such record respondent retired from Company's service -
Wrong date of birth recorded in office record of Regional Provident Fund Commissioner
- Respondent held ineligible for benefit of Employees' Pension Scheme, 1995 - Plea that
case of respondent for correction of date should not be considered as he had applied at
fag end of his career for correction of date of birth in Provident Fund records, which
practice had been strongly discouraged by Supreme Court - Cannot be allowed.
Employees' Provident Funds and Miscellaneous Provisions Act (19 of 1952), S.5. (Para
23)
Cases Referred : Chronological Paras
1999 AIR SCW 4456 : AIR 2000 SC 331 : 2000 Lab IC 232 : 2000 CLC 578 (Foll.)
21
Dr. R.G. Padia, Sr. Advocate, Ms. Asha G. Nair, M.P.S. Tomar, Ms. Savitri Pandey,
Subhash Kaushik, D.S. Mahra, Ms. Varuna Bhandari Gugnani, B.V. Balaram Das, Mrs.
@page-SC2958
Anil Katiyar, B. Krishna Prasad, V. Prabhakar, Ramjee Prasad, Mrs. Revathy Raghavan,
Shabu Sreedharan, Noor Mohammed, Mrs. M.K. Michael, K.R. Sasiprabhu, Ms. Bindu
K. Nair, M/s. Lawyer's Knit and Co., G. Prakash, Haris Beeran, R.S. Jena, S. Marsook, R.
Sasiprabhu and Ms. Bindu K. Nair, with him for the appearing parties.
* From Judgment and Order of the National Consumer Disputes Redressal Commission,
New Delhi in Review Petn. No. 1292 of 2000, D/- 21-9-2000.
Judgement
1. ALTAMAS KABIR, J. :- Civil Appeal No. 6447 of 2001 has been taken up for hearing
along with five other matters, namely. Special Leave Petition (C) No. 15469 of 2005,
Special Leave Petition (C) No. 16573 of 2006, Special Leave Petition (C) No. 20260 of
2004, Special Leave Petition (C) No. 8661 of 2004 and Civil Appeal No. 2629 of 2006.
All the said matters are directed against orders passed by the National Consumer
Disputes Redressal Commission on Revision Petitions filed from orders passed by the
State Commission, Kerala, in appeal from orders passed by the District Forum. All of
them involve a common question of law and fact and have thus been taken up together
for hearing and final disposal.
2. Of the five matters, Special Leave Petition (C) No. 20260 of 2004 and Special Leave
Petition (C) No. 8661 of 2004 have been filed against order dated 30-7-2002 passed by
the National Consumer Disputes Redressal Commission (hereinafter referred to as 'the
National Commission') dismissing the Revision Petitions filed by the Regional Provident
Fund Commissioner for condonation of delay in filing the Review Petition. Both the
Revision Petitions were dismissed on the ground that the delay had not been sufficiently
explained. Though in these two Special Leave Petitions it is the order rejecting the
Revision Petitions on ground of delay that is involved, ultimately, it is the order of the
State Commission allowing the claim of the respondents concerned which is the subject
matter of all these five matters. Furthermore, the effect of the orders passed by the
National Commission in all these five matters is the same in respect of the different
petitioners before the District Forum and it is their claim which has been upheld right
through up to the National Commission.
3. Leave is accordingly granted in the Special Leave Petitions and all the appeals are
taken up for hearing together.
4. For the sake of convenience the facts relating to Civil Appeal No. 6447/2001 are taken
into for consideration in order to decide the common issues in these matters.
5. The respondent Bhavani was a worker in Cashew Factory No. III (Naduvathoor)
owned and managed by the Kerala State Cashew Development Corporation Limited,
Kollam and according to her she retired from service on 31-12-1995 on attaining 60 years
of age. Bhavani was a member of the Employees' Provident Fund and Family Pension
Scheme, 1971, and was making contribution to the Scheme. In her service records
maintained by the company her date of birth was shown as 31-12-1935 and though she
was eligible for pension, the same was not ordered by the Regional Provident Fund
Commissioner, the appellant herein in all these appeals. Aggrieved by the failure and/ or
the refusal of the Regional Provident Fund Authorities to release pension to her, Bhavani
filed an application before the Consumer Disputes Redressal Forum (hereinafter referred
to as 'the District Forum') Kollam, praying for a direction upon the appellant herein to
release her pensionary benefits from the date of her retirement from service i.e. 31-12-
1995. Bhavani's claim was contested by the appellant herein before the District Forum by
filing a counter affidavit wherein it was contended that the Consumer Protection Act,
1986, would have no application to a claim made under the Employees' Provident Funds
and Misc. Provisions Act, 1952 (hereinafter referred to as 'the 1952 Act'), inasmuch as,
Bhavani, the respondent herein, was not a "consumer" within the meaning of Section 2(d)
of the Act.
6. It was also contended on behalf of the appellant herein that Bhavani was a member of
the Employees' Provident Fund and Family Pension Scheme 1971 and according to the
records of the appellant she had attained the age of 60 years in 1992 before the
Employees' Pension Scheme 1995, came into operation. Before the District Forum it was
the case of the appellant herein that the said Employees' Pension Scheme, 1995,
contained a cut-off date, namely, 1st April, 1993 and those members of the Scheme who
had attained the age of 60 years prior to the cut-off date were not entitled to the benefits
thereof. It was the further case of the appellant before the District Forum that according
to the records maintained by the office of the appellant, the date of birth of Bhavani had
been shown as 24-9-1932. It was submitted that Form 2 which was required to
@page-SC2959
be filled up and filed by the concerned employee while applying for membership of the
Family Pension Fund Scheme, 1971, showed her date of birth as 24-9-1932. It was
contended that according to the information provided by Bhavani herself she had attained
the age of 60 years on 24-9-1992 and was not, therefore, entitled to the benefits of the
1995 Employees' Pension Scheme.
7. The District Forum rejected the case made out on behalf of the appellant herein on a
scrutiny of the various documents submitted on behalf of Bhavani to establish that her
date of birth in the records of the company was 31-12-1935. The District Forum came to
a finding that since Bhavani was eligible for the benefits of the 1995 Scheme, denial of
the same amounted to deficiency of service which would attract the provisions of the
Consumer Protection Act, 1986. The District Forum on considering the provisions of
Section 2(1)(d) (ii) of the Consumer Protection Act, 1986, observed that the definition of
"consumer" therein was not exhaustive and Section 2(1)(o) exempts only such services as
are rendered free of charge or under a contract of personal service.
8. The District Forum also rejected the said contention of the appellant herein upon
holding that the service rendered by the appellant herein did not fall within the exempted
categories. The District Forum categorically found that the services rendered by the
appellant herein to Bhavani came within the ambit of Section 2(1)(d)(ii) of the Consumer
Protection Act.
9. On the question of the recording of the respondent's date of birth in the records of the
appellant herein, the District Forum came to a definite finding that the respondent retired
from service only on 31-12-1995 and was, therefore, entitled to the benefits of the 1995
Employees' Pension Scheme which became operative from 1st April, 1993.
10. As indicated hereinbefore, the said order of the District Forum was challenged by the
appellant herein unsuccessfully both before the State Commission as also the National
Commission.
11. The question involved in the other matters taken up along with this appeal is identical
and have been allowed by the District Forum and thereafter by the State and National
Commissions.
12. Dr. R.G. Padia, learned senior advocate appearing in support of the appeal, repeated
the same submissions that had been advanced before the District Forum and the State
Commission. He contended that, inasmuch as, the respondent was not a 'consumer' within
the meaning of Section 2(1)(o) of the Consumer Protection Act, 1986, the said Act would
have no application particularly when from the definition itself it would be evident that
no service was being rendered to the respondent free of charge. Dr. Padia also urged that
when a master and servant relationship existed, the Consumer Protection Act would not
apply to either of them.
13. Dr. Padia also urged that the 1995 Scheme had no relation to the question of payment
of the respondent's provident fund dues.
14. Dr. Padia then went on to submit that the order of the National Commission under
challenge was very cryptic and did not indicate reasons for negating the claim of the
respondents in the appeals and deserved to be remanded to enable the National
Commission to pass a properly reasoned order. It was then urged that, in any event, this
Court had deprecated the practice of employees approaching the management at the fag
end of their careers asking that their dates of birth be altered to their advantage. Dr. Padia
submitted that this was not permissible and this Court had said so in different judgments.
It was submitted that such a claim after the attainment of superannuation was all the more
inadmissible and the order passed by the District Forum, which was upheld up to the
National Commission to direct the appellant herein to correct its records relating to the
date of birth of the respondent, was erroneous and could not be sustained.
15. In this regard, various decisions of this Court were referred to by Dr. Padia in support
of his contention that contract of personal service or a service rendered free of charge
would not attract the provisions of the Consumer Protection Act, 1986. Dr. Padia urged
that the services rendered by the appellant to the respondent amounted to personal service
which was of a free nature and would not, therefore, attract the provisions of the
Consumer Protection Act on both counts. According to Dr. Padia, the respondent could at
best have asked for pensionary benefits under the 1971 Employees' Family Pension
Scheme which had been
@page-SC2960
replaced by the Employees' Pension Scheme, 1995.
16. Dr. Padia lastly urged that the District Forum had no jurisdiction under the Consumer
Protection Act to direct alteration of the date of birth of a member which was recorded in
the records of the appellant, and, that too, without holding any inquiry in that regard.
17. On behalf of the respondents, Mr. Noor Mohammed, learned advocate for the
respondent in Special Leave Petition? No. 8661 of 2004, submitted that in a similar case
involving the same set of facts, being Special Leave Petition? No. 9667 of 2005, this
Court by order dated 26-3-2007 had dismissed the special leave petition. He, therefore,
submitted that the arguments advanced by Dr. Padia were of no consequence in view of
the order passed in Special Leave Petition? No. 9667 of 2005 wherein one K. Sarojini
was the complainant before the District Forum.
18. Mr. V. Prabhakar disputed Dr. Padia's contentions and submitted that the entries
relating to the date of birth of the respondent in the records of the company and not that
recorded in the records of the appellant were relevant for the purpose of determining the
date of superannuation of the employee concerned. It was submitted that stress had been
erroneously laid on the alleged entry in the records of the appellant to wrongfully deny
the benefits of the 1995 Employees' Pension Scheme to the respondent. It was also
submitted that various records had been produced on behalf of the respondent, including
documentary evidence from the company, in order to establish her claim that her date of
birth had been entered in her service records with the company as 31-12-1935.
19. We have carefully considered the submissions made on behalf of the respective
parties and the relevant documents which had been produced before the District Forum
and we are satisfied that the dates of birth of the respondents as recorded in their service
records with the company are the correct dates of birth of the employees and not the dates
of birth as entered in the records of the appellant. The reasoning given by the District
Forum in accepting the entries in the company's record while rejecting those in the
records of the appellant/ Regional P.F. Commissioner are based on sound logic and the
materials on record. For instance, there are certificates issued by the company to indicate
that the respondent in C.A. No. 6447/2001 had continued to work in the company till her
date of superannuation i.e. 31-12-1995 and there was no denial on the part of the
appellant that the respondent continued to contribute to the fund till the year 1995. No
explanation is forthcoming as to why and how such contributions were received, even
though according to the records of the appellant the respondent had retired on 31-12-
1992, so as to make her ineligible for the 1995 Employees' Pension Scheme which came
into operation on and from 1st April, 1993.
20. Dr. Padia's submissions regarding the non-applicability of the Consumer Protection
Act to the case of the respondent must also be rejected on account of the fact that the
Regional Provident Fund Commissioner, who is the person responsible for the working of
the 1995 Pension Scheme, must be held to be a 'service giver' within the meaning of
Section 2(1)(o) of the Consumer Protection Act. Nor is this a case of rendering of free
service or rendering of service under a contract of personal service so as to bring the
relationship between the appellant and respondent within the concept of 'master and
servant'. In our view, the respondent comes squarely within the definition of 'consumer'
within the meaning of Section 2(1)(d)(ii), inasmuch as, by becoming a member of the
Employees' Family Pension Scheme, 1971, and contributing to the same, she was
availing of the services rendered by the appellant for implementation of the Scheme. The
same is the case in the other appeals as well.
21

. In fact, the same proposition has been explained in Regional Provident Fund
Commissioner vs. Shiv Kumar Joshi [2000 (1) SCC 98], wherein in relation to the
operation of the Consumer Protection Act to the Employees' Provident Fund Schemes it
was held as follows : 1999 AIR SCW 4456, Para 12

"A perusal of the Scheme clearly and unambiguously indicates that it is a 'service' within
the meaning of Section 2(1)(o) and the member a 'consumer' within the meaning of
Section 2(1)(d) of the Act. It is, therefore, without any substance to urge that the services
under the Scheme are rendered free of charge and, therefore, the Scheme is not a 'service'
under the Act. Both the State as well as the National Commission have dealt
@page-SC2961
with this aspect in detail and rightly come to the conclusion that the Act was applicable in
the case of the Scheme on the ground that its member was a 'consumer' under Section
2(1)(d) and the Scheme was a 'service' under Section 2(1)(o)."
22. Several other earlier decisions were also referred to, where a similar view has been
expressed.
23. We are not also able to appreciate Dr. Padia's submission that the cases of the
respondents should not be considered as they had applied at the fag end of their careers
for correction of their dates of birth in the appellant's records, which practice had been
strongly discouraged by this Court. The aforesaid principle cannot apply to the case of the
respondents as their dates of birth had been correctly recorded in the records of the
company, including the respondents' service records, on the basis whereof they had
retired from the company's services.
24. We, therefore, have no hesitation in upholding the orders passed by the National
Commission. All the six appeals filed by the Regional Provident Fund Commissioner are
accordingly dismissed.
25. There will, however, be no order as to costs.
Appeals dismissed. .
AIR 2008 SUPREME COURT 2961 "Shashi Mohan v. State of M. P."
(From : Madhya Pradesh)
Coram : 2 Dr. A. PASAYAT AND P. SATHASIVAM, JJ.
Criminal Appeal No. 1093 of 2008 (arising out of SLP (Cri.) No. 7491 of 2007), D/- 15
-7 -2008.
Shashi Mohan v. State of M.P.
Penal Code (45 of 1860), S.34, S.300 - COMMON INTENTION - MURDER - Common
Intention - Murder case - Deceased shot at by main accused - Incident happening out of
family rivalry - Appellant brother of main accused - His presence at spot establised -
None of eyewitnesses alleging that he was armed - No overt act attributed to him -
Witnesses admitting that he was coming from different direction - No evidence to show
that appellant shared common intention with other accused - Conviction of appellant
u/S.300, u/S.34 liable to be set aside.
Cri. A. No.83 of 1993, D/-28-09-2007. (MP), Reversed. (Paras 6, 7, 9)
Cases Referred : Chronological Paras
1993 AIR SCW 1843 : AIR 1993 SC 1899 : 1993 Cri LJ 2246 8
Nagendra Rai, P.H. Parekh, Sr. Advocates, Jetendra Singh and S.K. Sabharwal, for
Appellant; Govind Goel, C.D. Singh, Ram Naresh Yadav, Merusagar Samanta Ray and
Sunny Choudhary, for Respondents.
* Cri. Appeal No. 83 of 1993, D/- 28-9-2007 (MP)
Judgement
Dr. ARIJIT PASAYAT, J. :- Leave granted.
2. Challenge in this appeal is to the judgment of the Division Bench of the Madhya
Pradesh High Court, Jabalpur Bench. Three persons, namely, Rameshwardayal,
Shashimohan and Revimohan hereinafter described as A1, A2 and A3 faced trial for
alleged commission of offence punishable under Section 302 read with Section 34 of the
Indian Penal Code, 1860 (in short the 'IPC'). Additional Sessions Judge, Morena, found
them guilty and sentenced each to life imprisonment. During the pendency of the appeal
before the High Court A1 died and, therefore, the appeal was held to have abated so far as
A1 is concerned. The present appeal is by A2. A1 and A2 were convicted and sentenced
under Section 302 read with Section 34 IPC while A3 was found guilty of offence
punishable under Section 302 IPC.
3. Prosecution version as unfolded during trial is as follows :
On 3-3-1992 at 9.30 a.m. on a road from Pipalwali Mata to Rui Ki Mandi and ahead of a
Chauraha in Morena one Rakesh S/o Ram Singh (hereinafter referred to as 'deceased')
was shot dead by A3 who pumped into him three gun shots resulting in instant death of
said Rakesh. There reportedly existed previous enmity between the family of A1 and of
Ram Singh father of the deceased. A1 and Ram Singh are real brother. The incident was
reported to Police at Police Station Kotwali at 9.40 a.m. by Radheyshyam (PW1), brother
of the deceased Rakesh. FIR (Ex. P/1) was recorded and the investigation was set in
motion by Registering a crime at Sr. No. 144/92 under Section 302/34 IPC. After
completion of investigation, charge sheet was filed. Accused persons abjured guilt and
claimed trial.
4. In order to establish its accusations,
@page-SC2962
the prosecution examined 12 witnesses, PWs, 1, 2 and 3 were stated to be eye-witnesses.
In order to establish its plea of false implication DW1 was examined to prove the
presence of A3 at a different place. Trial Court found the evidence to be cogent and
recorded conviction as noted above.
5. Before the High Court the primary stand of the appellant was that so far as he is
concerned, Section 34 IPC has no application. The High Court did not accept that plea.
6. In support of the appeal, learned counsel for the appellant submitted that the High
Court has categorically noted that none of the eye witnesses stated that A2, the present
appellant has pre-mediated with A1 and A3 before the offence was committed. The
witnesses admitted that the appellant was not armed with weapon and no overt act was
attributed to him. Further, he was coming from a different direction and, therefore, the
question of his sharing the common intention was not there.
7. Learned counsel for the respondent, on the other hand, submitted that though A2 was
not armed with and was coming from a different direction, his presence has been
established. He being the son of A1 and the brother of A3, the main assailant the
ingredients of Section 34 have been clearly established.
8

. Under the provisions of Section 34 IPC the essence of the liability is to be found in the
existence of a common intention animating the accused leading to the doing of a criminal
act in furtherance of such intention. As a result of the application of principles enunciated
in Section 34, when an accused is convicted under Section 302 read with Section 34, in
law it means that the accused is liable for the act which caused death of the deceased in
the same manner as if it was done by him alone. The provision is intended to meet a case
in which it may be difficult to distinguish between acts of individual members of a party
who act in furtherance of the common intention of all or to prove exactly what part was
taken by each of them. As was observed in Ch. Pulla Reddy and Ors. v. State of Andhra
Pradesh (AIR 1993 SC 1899), Section 34 is applicable even if no injury has been caused
by the particular accused himself. For applying Section 34 it is not necessary to show
some overt act on the part of the accused. 1993 AIR SCW 1843

9. When the background facts are considered in the light of legal principles set out above,
the position is clear that the accusations were not established so far as the present
appellant is concerned. No evidence was led to show sharing of common intention. The
appeal deserves to be allowed which we direct. He be set at liberty forthwith unless
required to be in custody in connection with any other case.
Appeal allowed. .
AIR 2008 SUPREME COURT 2962 "State of Punjab v. Navraj Singh"
(From : Punjab and Haryana)*
Coram : 2 Dr. A. PASAYAT AND HARJIT SINGH BEDI, JJ.
Criminal Appeal No. 1075 of 2008 (arising out of SLP (Cri.) No. 6143 of 2006), D/- 14
-7 -2008.
State of Punjab v. Navraj Singh.
Criminal P.C. (2 of 1974), S.389(1) - SENTENCE SUSPENSION - APPEAL -
EQUALITY - Suspension of conviction pending appeal - No reason indicated in order -
Order directing suspension/stay of conviction - Liable to be set aside.
Constitution of India, Art.14.
2004 AIR SCW 80; 2003 (12) SCC 434; 2004 AIR SCW 4266; 2001 AIR SCW 3339,
Rel. on.
Cri. Misc. No. 51640 of 2005 in Cri. Appeal No. 1498-SB of 2002, D/-17-01-2006 (P and
H), Reversed. (Paras 12, 13)
Cases Referred : Chronological Paras
2004 AIR SCW 80 : AIR 2004 SC 1188 : 2004 ; Cri LJ 919 (Rel. on) 8
2004 AIR SCW 4266 : AIR 2004 SC 3936 : 2004 Cri LJ 3840 (Rel. on) 11
(2003) 12 SCC 434 (Rel on) 8, 9
2001 AIR SCW 3339 : AIR 2001 SC 3320 : 2001 Cri LJ 4234 (Rel. on) 5, 8, 10
Kuldip Singh, for Appellant; Ajit Kumar, Ms. Sikha Roy Pabbi and S.K. Sabharwal, for
Respondent.
* Cri. Misc. No. 51640 of 2005 in Cri. Appeal No. 1498-SB of 2002, D/- 17-1-2006 (P
and H).
Judgement
Dr. ARIJIT PASATAT, J. :- Leave granted.
2. Challenge in this appeal is to the order passed by a learned Single Judge of the Punjab
and Haryana High Court directing that the conviction of the respondent shall
@page-SC2963
remained stayed during the pendency of Criminal Appeal No. 1498-SB of 2002.
3. Background facts in a nutshell are as follows :
4. Respondent who was working as Patwari Halqa and was convicted by learned Special
Judge, Nawanshahr, Punjab for offences punishable under Sections 7 and 13(1)(d) read
with Section 13(2) of the Prevention of Corruption Act, 1988 (in short 'P. C. Act') and
sentenced to undergo rigorous imprisonment for a period of three years and to pay a fine
of Rs. 2000/- with default stipulation. Against the judgment in question respondent filed
the aforesaid Criminal appeal which was admitted. After admission of the appeal,
respondent filed an application in terms of Section 389(1) of the Code of Criminal
Procedure, 1973 (in short the 'Code') read with Section 482 of the Code for suspension of
the judgment of learned Special Judge.
5

. The High Court by order dated 27-1-2005 stayed the conviction. According to the
appellant, the view expressed by this Court in K. C. Sareen v. CBI, Chandigarh (2001 (6)
SCC 584) was not kept in view. The High Court dismissed that application only on the
ground that the review of the order was not permissible. 2001 AIR SCW 3339
6. It is submitted by learned counsel for the appellant State that the suspension of the
conviction is clearly unsustainable. It is pointed out that the High Court noted that the
Collector, Nawanshaher had given a notice for dispensing his services as Patwari Halqa,
Musapur.
7. Learned counsel for the respondent submitted that the High Court took note of the fact
that this was a case where the prayer for suspension of the conviction was to be granted.
Unless the order of conviction was suspended, the respondent would have lost his job.
8

. In State of Maharashtra v. Gajanan and Another (2003 (12) SCC 432), it was noted as
follows : 2004 AIR SCW 80, Paras 4 and 5

Having perused the impugned order as also the judgment of this Court in K. C. Sareen's
case (2001 (6) SCC 584) we find the High Court had no room for distinguishing the law
laid down by this Court in K. C. Sareen case supra even on facts. This Court in the said
case held (SCC p. 589, para 11) :
"11. The legal position, therefore, is this; though the power to suspend an order of
conviction, apart from the order of sentence, is not alien to Section 389(1) of the Code, its
exercise should be limited to very exceptional cases. Merely because the convicted
person files an appeal in challenge of the conviction the Court should not suspend the
operation of the order of conviction. The Court has a duty to look at all aspects including
the ramifications of keeping such conviction in abeyance. It is in the light of the above
legal position that we have to examine the question as to what should be the position
when a public servant is convicted of an offence under the PC Act. No doubt when the
appellate Court admits the appeal filed in challenge of the conviction and sentence for the
offence under the PC Act, the superior Court should normally suspend the sentence of
imprisonment until disposal of the appeal, because refusal thereof would render the very
appeal otiose unless such appeal could be heard soon after the filing of the appeal. But
suspension of conviction of the offence under the PC Act, dehors the sentence of
imprisonment as a sequel thereto, is a different matter."
(Emphasis supplied)
In the said judgment of K. C. Sareen's case (supra) this Court has held that it is only in
very exceptional cases that the Court should exercise such power of stay in matters
arising out of the Act. The High Court has in the impugned order nowhere pointed out
what is the exceptional fact which in its opinion required it to stay the conviction. The
High Court also failed to note the direction of this Court that it has a duty to look at all
aspects including ramification of keeping such conviction in abeyance. The High Court,
in our opinion, has not taken into consideration any of the above factors while staying the
conviction. It should also be noted that the view expressed by this Court in K. C. Sareen
case (supra) was subsequently approved followed by the judgment of this Court in Union
of India v. Avtar Singh (2003 (12) SCC 434).
9. In Union of India v. Avtar Singh and Anr. (2003 (12) SCC 434) it was held as follows :
"This appeal is directed against the impugned order of the High Court. The respondent
accused, who has been convicted under Section 409 IPC and Section 13 of the Prevention
of Corruption Act, preferred an
@page-SC2964
appeal to the High Court, which has been entertained. On an application being filed under
Section 389 of the Code of Criminal Procedure, the High Court has suspended the
conviction solely on the ground that the non-suspension of conviction may entail removal
of the delinquent government servant from service."
10

. In K.C. Sareen's case (supra) it was noted as follows : 2001 AIR SCW 3339, Paras
10 to 13

"11. The legal position, therefore, is this : though the power to suspend an order of
conviction, apart from the order of sentence, is not alien to Section 389(1) of the Code, its
exercise should be limited to very exceptional cases. Merely because the convicted
person files an appeal in challenge of the conviction the Court should not suspend the
operation of the order of conviction. The Court has a duty to look at all aspects including
the ramifications of keeping such conviction in abeyance. It is in the light of the above
legal position that we have to examine the question as to what should be the position
when a public servant is convicted of an offence under the PC Act. No doubt when the
appellate Court admits the appeal filed in challenge of the conviction and sentence for the
offence under the PC Act, the superior Court should normally suspend the sentence of
imprisonment until disposal of the appeal because refusal thereof would render the very
appeal otiose unless such appeal could be heard soon after the filing of the appeal. But
suspension of conviction of the offence under the PC Act, dehors the sentence of
imprisonment as a sequel thereto, is a different matter.
12. Corruption by public servants has now reached a monstrous dimension in India. Its
tentacles have started grappling even the institutions created for the protection of the
republic. Unless those tentacles are intercepted and impeded from gripping the normal
and orderly functioning of the public offices, through strong legislative, executive as well
as judicial exercises the corrupt public servants could even paralyse the functioning of
such institutions and thereby hinder the democratic polity. Proliferation of corrupt public
servants could garner momentum to cripple the social order if such men are allowed to
continue to manage and operate public institutions. When a public servant is found guilty
of corruption after a judicial adjudicatory process conducted by a Court of law,
judiciousness demands that he should be treated as corrupt until he is exonerated by a
superior Court. The mere fact that an appellate or revisional forum has decided to
entertain his challenge and to go into the issues and findings made against such public
servants once again should not even temporarily absolve him from such findings. If such
a public servant becomes entitled to hold public office and to continue to do official acts
until he is judicially absolved from such findings by reason of suspension of the order of
conviction, it is public interest which suffers and sometimes, even irreparably. When a
public servant who is convicted of corruption is allowed to continue to hold public office,
it would impair the morale of the other persons manning such office, and consequently
that would erode the already shrunk confidence of the people in such public institutions
besides demoralising the other honest public servants who would either be the colleagues
or subordinates of the convicted person. If honest public servants are compelled to take
orders from proclaimed corrupt officers on account of the suspension of the conviction,
the fallout would be one of shaking the system itself. Hence it is necessary that the Court
should not aid the public servant who stands convicted for corruption charges to hold
only (sic) public office until he is exonerated after conducting a judicial adjudication at
the appellate or revisional level. It is a different matter if a corrupt public officer could
continue to hold such public office even without the help of a Court order suspending the
conviction.
13. The above policy can be acknowledged as necessary for the efficacy and proper
functioning of public offices. If so, the legal position can be laid down that when
conviction is on a corruption charge against a public servant the appellate Court or the
revisional Court should not suspend the order of conviction during the pendency of the
appeal even if the sentence of imprisonment is suspended. It would be a sublime public
policy that the convicted public servant is kept under disability of the conviction in spite
of keeping the sentence of imprisonment in abeyance till the disposal of the appeal or
revision."
11

. In State of Haryana v. Hasmat (2004 (6) SCC 175) it was noted as follows : 2004
AIR SCW 4266

"6. Section 389 of the Code deals with


@page-SC2965
suspension of execution of sentence pending the appeal and release of the appellant on
bail. There is a distinction between bail and suspension of sentence. One of the essential
ingredients of Section 389 is the requirement for the appellate court to record reasons in
writing for ordering suspension of execution of the sentence or order appealed. If he is in
confinement, the said Court can direct that he be released on bail or on his own bond. The
requirement of recording reasons in writing clearly indicates that there has to be careful
consideration of the relevant aspects and the order directing suspension of sentence and
grant of bail should not be passed as a matter of routine."
12. It is to be noted that learned single Judge while directing suspension of conviction
indicated no reasons.
13. Above being the position the order of the learned Single Judge, directing the
suspension/stay of the conviction as well as the order refusing to recall the said order
cannot stand and are set aside.
14. Appeal is allowed.
Appeal allowed. .
AIR 2008 SUPREME COURT 2965 "Mahila Vinod Kumari v. State of Madhya Pradesh"
(From : Madhya Pradesh)*
Coram : 2 Dr. A. PASAYAT AND P. SATHASIVAM, JJ.
Special Leave to Appeal (Criminal) Nos. 4950-4951 of 2008, (Cri. Misc. Petn. No. 8515-
8516 of 2008), D/- 11 -7 -2008.
Mahila Vinod Kumari v. State of M.P.
(A) Criminal P.C. (2 of 1974), S.344 - SUMMARY TRIAL - PERJURY - Prosecution for
perjury - Initiation of proceedings by Court before which perjury is committed - Pre-
conditions - Frequent use of provision by Courts essential to curb menace of perjury.
For exercising the powers under S. 344 the Court at the time of delivery of judgment or
final order must at the first instance express an opinion to the effect that the witness
before it has either intentionally given false evidence or fabricated such evidence. The
second condition is that the Court must come to the conclusion that in the interests of
justice the witness concerned should be punished summarily by it for the offence which
appears to have been committed by the witness. And the third condition is that before
commencing the summary trial for punishment the witness must be given reasonable
opportunity of showing cause why he should not be so punished. All these conditions are
mandatory.
AIR 1971 SC 1789, Foll. (Para 8)
The evil of perjury has assumed alarming propositions in cases depending on oral
evidence and in order to deal with the menace effectively it is desirable for the Courts to
use the provision more effectively and frequently than it is presently done. (Para
10)
(B) Criminal P.C. (2 of 1974), S.344 - SUMMARY TRIAL - PERJURY - FIR -
SENTENCE IMPOSITION - Prosecution for perjury - Prosecutrix filing F. I. R. that
accused persons waylaid her and raped her one after other - Accused put on trial - During
trial prosecutrix denying commission of rape and even the fact of lodging F. I. R. -
Initiation of prosecution against her for fabricating evidence - Proper - Prosecutrix
admitting guilt - Imposition of 3 months S. I. - Proper. (Para 11)
Cases Referred : Chronological Paras
AIR 1971 SC 1789 : 1971 Cri LJ 1301 (Foll) 8
Suryanarayana Singh and Ms. Pragati Neekhra, for Appellant. Cri. Appeal No. 173 of
2002, D/- 30-11-2007 (MP)
* Cri. Appeal No. 173 of 2002, D/- 30-11-2007 (MP)
Judgement
Dr. ARIJIT PASAYAT, J. :- Heard learned counsel for the petitioner.
2. Delay condoned.
3. Though, we are not inclined to entertain the special leave petitions, but we find that
there is a need for expressing views on action to be taken for maliciously setting law into
motion.
4. The petitioner lodged a report against two persons at Pichhore Police Station to the
effect that on 28.1.1993 between 6.00 to 7.00 a.m. she was waylaid by them who dragged
her and committed rape on her, one after another. She claimed to have narrated the
incident to her father and uncle and, thereafter lodged the report at the police station. On
the basis of the report, matter was investigated. The accused persons were arrested.
Charge-sheet was filed. The accused persons faced trial for alleged commission of
offence punishable under Section
@page-SC2966
376(2)(g) of the Indian Penal Code, 1860 (in short 'the IPC'). The accused persons
abjured their guilt. During trial, the petitioner stated that she had actually not been raped.
As she resiled from the statement made during investigation, she was permitted to be
cross-examined by the prosecution. She even denied to have lodged the first information
report (Exh.P-1) and to have given any statement to the police (Exh.P-2). In view of the
statement of the petitioner, the two accused persons were acquitted by judgment dated
28.11.2001. The Trial Court found that the petitioner had tendered false evidence and had
fabricated evidence against the accused persons with the intention that such evidence
shall be used in the proceedings, and, therefore, directed cognizance in terms of Section
344 of the Code of Criminal Procedure, 1973 (in short 'the Code') to be taken against the
petitioner. A show-cause notice was issued and the case was registered against the
petitioner who filed reply to the effect that being an illiterate lady, she had committed the
mistake and may be excused. The Trial Court found that the petitioner admitted her guilt
that she had lodged false report of rape against the accused. She was, accordingly,
sentenced to undergo three months' simple imprisonment. Aggrieved by the order, the
petitioner filed an appeal before the Madhya Pradesh High Court, which, by the
impugned order, was dismissed.
5. Stand before the High Court was that being an illiterate lady, she does not understand
law and the particulars of the offence were not explained to her and, therefore, the appeal
should be allowed. This was opposed by the State on the ground that the petitioner had
admitted her guilt before the Trial Court and, therefore, the conviction is well founded.
The High Court perused the records of the Trial Court and found that in the show-cause
reply she had admitted that she had told lies all through. The stand that the particulars of
the offence were not explained to her, was found to be equally untenable, because in the
show-cause notice issued, relevant details were given. In the first information report, and
the statement recorded by the police, she had clearly stated that she was raped by the
accused persons. But in Court she denied to have stated so. Learned counsel for the
petitioner submitted that the Court imposed 15 days' simple imprisonment which is harsh.
But that is not the end of the matter. The petitioner filed an application before the High
Court stating that a wrong statement was made before the High Court that she had
already suffered custody for 15 days, which weighed with the High Court to reduce the
sentence.
6. Learned counsel for the petitioner stated that being a girl of tender age, she was
pressurized by her mother and uncle to give a false report. This is at variance with the
statement made in court during trial to the effect that she had not reported anything to the
police. It is a settled position in law that so far as sexual offences are concerned, sanctity
is attached to the statement of a victim. This Court, has, in several cases, held that the
evidence of the prosecutrix alone is sufficient for the purpose of conviction if it is found
to be reliable, cogent and credible. In the present case, on the basis of the allegations
made by the petitioner, two persons were arrested and had to face trial and suffered the
ignominy of being involved in a serious offence like rape. Their acquittal, may, to a
certain extent, have washed away the stigma, but that is not enough. The purpose of
enacting Section 344, Cr.P.C. corresponding to Section 479-A of the Code of Criminal
Procedure, 1898 (hereinafter referred to as 'the Old Code') appears to be further arm the
Court with a weapon to deal with more flagrant cases and not to take away the weapon
already in its possession. The object of the legislature underlying enactment of the
provision is that the evil of perjury and fabrication of evidence has to be eradicated and
can be better achieved now as it is open to the courts to take recourse to Section 340(1)
(corresponding to Section 476 of the Old Code) in cases in which they are failed to take
action under Section 344 Cr.P.C.
7. This section introduces an additional alternative procedure to punish perjury by the
very Court before which it is committed in place of old Section 479A which did not have
the desired effect to eradicate the evils of perjury. The salient features of this new
provision are :
(1) Special powers have been conferred on two specified Courts, namely Court of Session
and Magistrate of the First Class, to take cognizance of an offence of perjury committed
by a witness in a proceeding before it instead of filing a complaint before a Magistrate
and try and punish the offender
@page-SC2967
by following the procedure of summary trials. For summary trial, see Ch. 21.
(2) This power is to be exercised after having the matter considered by the Court only at
the time of delivery of the judgment or final order.
(3) The offender shall be given a reasonable opportunity of showing cause before he is
punished.
(4) The maximum sentence that may be imposed is 3 month's imprisonment or a fine up
to Rs.500 or both.
(5) The order of the Court is appealable (vide S. 351).
(6) The procedure in this section is an alternative to one under Sections 340-343. The
Court has been given an option to proceed to punish summarily under this section or to
resort to ordinary procedure by way of complaint under Section 340 so that, as for
instance, where the Court is of opinion that perjury committed is likely to raise
complicated questions or deserves more severe punishment than that permitted under this
section or the case is otherwise of such a nature or for some reasons considered to be
such that the case should be disposed of under the ordinary procedure which would be
more appropriate, the Court may chose to do so [vide sub-section (3)].
(7) Further proceedings of any trial initiated under this section shall be stayed and thus,
any sentence imposed shall also not be executed until the disposal of an appeal or
revision against the judgment or order in the main proceedings in which the witness gave
perjured evidence or fabricated false evidence [vide sub-section (4)].
8

. For exercising the powers under the section the Court at the time of delivery of
judgment or final order must at the first instance express an opinion to the effect that the
witness before it has either intentionally given false evidence or fabricated such evidence.
The second condition is that the Court must come to the conclusion that in the interests of
justice the witness concerned should be punished summarily by it for the offence which
appears to have been committed by the witness. And the third condition is that before
commencing the summary trial for punishment the witness must be given reasonable
opportunity of showing cause why he should not be so punished. All these conditions are
mandatory. [See Narayanswamy v. State of Maharashtra, (1971) 2 SCC 182]. AIR
1971 SC 1789

9. The object of the provision is to deal with the evil perjury in a summary way.
10. The evil of perjury has assumed alarming propositions in cases depending on oral
evidence and in order to deal with the menace effectively it is desirable for the courts to
use the provision more effectively and frequently than it is presently done.
11. In the case at hand, the court has rightly taken action and we find nothing infirm in
the order of the Trial Court and the High Court to warrant interference. The special leave
petitions are, accordingly dismissed.
Petition dismissed. .
AIR 2008 SUPREME COURT 2967 "State of Madhya Pradesh v. Imrat"
(From : Madhya Pradesh)*
Coram : 2 Dr. A. PASAYAT AND P. SATHASIVAM, JJ.
Criminal Appeal No. 1059 of 2008 (arising out of SLP (Cri.) No. 2495 of 2006), D/- 11
-7 -2008.
State of M.P. v. Imrat and Anr.
(A) Penal Code (45 of 1860), S.307 - ATTEMPT TO MURDER - Attempt to murder -
Intention or knowledge of accused - Is relevant - Not nature of injury actually caused to
victim.
It is sufficient to justify a conviction under Section 307 if there is present an intent
coupled with some overt act in execution thereof. It is not essential that bodily injury
capable of causing death should have been inflicted. The Section makes a distinction
between the act of the accused and its result, if any. The Court has to see whether the act,
irrespective of its result, was done with the intention or knowledge and under
circumstances mentioned in the Section. Therefore, an accused charged under Section
307, IPC cannot be acquitted merely because the injuries inflicted on the victim were in
the nature of a simple hurt. (Para 11)
(B) Penal Code (45 of 1860), S.307 - ATTEMPT TO MURDER - Attempt to murder -
Accused assaulting complainant with lathi and Farsa - Causing amongst other, two
injuries on head of complainant - Nature of injuries were grievous and were caused by
use of sufficient force - Dying declaration of complainant was also recorded as police and
@page-SC2968
doctor felt that injuries were serious - Acquittal of accused on ground that injuries were
not sufficient in ordinary course of nature to cause death - Improper.
Crl. Appeal No. 61 of 1998, D/-28-01-2005 (M.P.), Reversed. (Para 16)
Cases Referred : Chronological Paras
2005 AIR SCW 3511 : AIR 2005 SC 3996 : 2005 Cri LJ 3435 : 2005 AIR Jhar HCR 2094
(Rel. on) 12
2004 AIR SCW 810 : AIR 2004 SC 1808 : 2004 Cri LJ 1388 : 2004 All LJ 683 (Rel. on)
12
2004 AIR SCW 815 : AIR 2004 SC 1812 : 2004 Cri LJ 1391 : 2004 AIR - Kant HCR 662
(Rel. on) 12
AIR 1983 SC 305 : 1983 Cri LJ 331 (Rel. on) 12
AIR 1965 SC 843 : 1965 (1) Cri LJ 766 (Rel. on) 13
Ms. Vibha Datta Makhija, for Appellants.
* Cri. Appeal No.61 of 1998, D/- 28-1-2005 (MP).
Judgement
1. Dr. ARIJIT PASAYAT, J. :-Leave granted.
2. Challenge in this appeal is to the judgment of a learned Single Judge of the Madhya
Pradesh High Court, Gwalior Bench partially allowing the appeal filed by the
respondents. The Additional Sessions Judge, Pichhore, District Shivpuri found the
respondents guilty of having committed offence punishable under Section 307 read with
Section 34 of the Indian Penal Code, 1860 (in short the 'IPC') and sentenced each to
undergo seven years' RI with fine of Rs.1,000/-.
3. The High Court by the impugned judgment held that the proper conviction would be
under Section 326 read with Section 34, IPC and the sentences were to be reduced to the
period already undergone.
4. Background facts in a nutshell are as follows :
On 7.2.1997 daughter of the complainant was married to Sitaram and the complainant
wanted to give his property to his daughter. Respondent-Imrat who is one of the close
relatives of the complainant, objected to this and, therefore, on 2.3.1997 accused persons
caused injuries to the complainant. At the time of the incident accused-Imrat had a lathi in
his hand and accused-Komal had a farsa with him. They caused six injuries on the
complainant. On the basis of the information lodged in the Police Station, investigation
was undertaken and challan was filed against the accused persons for committing offence
punishable under Sections 307, 324 and 506(2), IPC. However, as noted above, the trial
Court convicted the accused for commission of offence punishable under Section 307
read with Section 34, IPC. The trial Court noted that as per the evidence of the
complainant-Bhajan while he was going towards his house near the well accused persons
armed with lathi and farsa obstructed his way. Imrat told the co-accused that complainant
has to be killed. On hearing this Komal hit the head of the complainant with farsa. Imrat
gave lathi blow on the wrist of right hand and left hand and left foot. Komal hit him once
more with farsa which struck him on the head. The trial Court found the evidence of the
witnesses credible and cogent and on the evidence of the doctor found the accused
persons guilty and convicted them as afore-noted.
Before the High Court the only plea taken was that all the six injuries except injury Nos.
1 and 2 are simple in nature. Injury Nos.1 and 2 were caused by sharp edged weapons
and were grievous in nature. It was urged that there was no material to show that these
two injuries were dangerous to life or were sufficient in the ordinary course of nature to
cause death. According to them at the most the offence under Section 326, IPC was made
out. This plea found acceptance of the High Court.
5. In support of the appeal, learned counsel for the appellant-State submitted that the
injuries were on the head and were caused by sharp cutting weapons and the force with
which the blows were given can be seen from the nature of the injuries on the head.
6. No one appears for the respondents in spite of service of notice.
7. The injuries which were noticed by the doctor are as follows :
"No. 1. One cut wound on the back of head on the left side admeasuring 3.5 x 1 x 1 c.m.
No. 2. One cut wound on the right side of the head admeasuring 1 x 1 x 1.5 c.m.
No. 3. One abrasion mark on the left hand, rounded admeasuring 5 x 5 c.m.
No. 4. One crushed wound on the right elbow of 1 x 1 c.m.
@page-SC2969
No. 5. One crushed wound with swelling on the back side of right hand admeasuring 3 x
2 c.m.
No. 6. One crushed wound on the left feet measuring 4 x 5 c.m."
8. The doctor has categorically stated that injury Nos. 1 and 2 were caused by sharp
edged weapons. The dying declaration of the injured was recorded on the request of the
police.
9. It is to be noted that the alleged offences are of very serious nature. Section 307 relates
to attempt to murder. It reads as follows :
"Whoever does any act with such intention or knowledge, and under such circumstances
that, if he by that act caused death, he would be guilty of murder, shall be punished with
imprisonment of either description for a term which may extend to ten years, and shall
also be liable to fine; and, if hurt is caused to any person by such act, the offender shall be
liable either to (imprisonment for life), or to such punishment as is hereinbefore
mentioned."
10. To justify a conviction under this Section, it is not essential that bodily injury capable
of causing death should have been inflicted. Although the nature of injury actually caused
may often give considerable assistance in coming to a finding as to the intention of the
accused, such intention may also be deduced from other circumstances, and may even, in
some cases, be ascertained without any reference at all to actual wounds. The Section
makes a distinction between an act of the accused and its result, if any. Such an act may
not be attended by any result so far as the person assaulted is concerned, but still there
may be cases in which the culprit would be liable under this Section. It is not necessary
that the injury actually caused to the victim of the assault should be sufficient under
ordinary circumstances to cause the death of the person assaulted. What the Court has to
see is whether the act, irrespective of its result, was done with the intention or knowledge
and under circumstances mentioned in the Section. An attempt in order to be criminal
need not be the penultimate act. It is sufficient in law, if there is present an intent coupled
with some overt act in execution thereof.
11. It is sufficient to justify a conviction under Section 307 if there is present an intent
coupled with some overt act in execution thereof. It is not essential that bodily injury
capable of causing death should have been inflicted. The Section makes a distinction
between the act of the accused and its result, if any. The Court has to see whether the act,
irrespective of its result, was done with the intention or knowledge and under
circumstances mentioned in the Section. Therefore, an accused charged under Section
307, IPC cannot be acquitted merely because the injuries inflicted on the victim were in
the nature of a simple hurt.
12

. This position was highlighted in State of Maharashtra v. Balram Bama Patil and Ors.
(1983 (2) SCC 28); Girija Shanker v. State of Uttar Pradesh (2004 (3) SCC 793); R.
Parkash v. State of Karnataka (JT 2004 (2) SC 348) and State of M.P. v. Saleem @
Chamaru and Anr. (2005 (5) SCC 554). AIR 1983 SC 305
2004 AIR SCW 810
2004 AIR SCW 815
2005 AIR SCW 3511

13. In Sarju Prasad v. State of Bihar (AIR 1965 SC 843) it was observed in para 6 that
mere fact that the injury actually inflicted by the accused did not cut any vital organ of
the victim, is not by itself sufficient to take the act out of the purview of Section 307.
14. Whether there was intention to kill or knowledge that death will be caused is a
question of fact and would depend on the facts of a given case. The circumstances that
the injury inflicted by the accused was simple or minor will not by itself rule out
application of Section 307, IPC. The determinative question is intention or knowledge, as
the case may be, and not nature of the injury. The basic differences between Sections 333
and 325, IPC are that Section 325 gets attracted where grievous hurt is caused whereas
Section 333 gets attracted if such hurt is caused to a public servant.
15. Section 307 deals with two situations so far as the sentence is concerned. Firstly,
whoever does any act with such intention or knowledge, and under such circumstances
that, if he by that act caused death, he would be guilty of murder, shall be punished with
imprisonment of either description for a term which may extend to ten years, and shall
also be liable to fine; and secondly if hurt is caused to any person by such act the offender
shall be liable either to imprisonment for life or to such punishment as indicated in the
first part i.e.
@page-SC2970
10 years. The maximum punishment provided for Section 333 is imprisonment of either
description for a term which may extend to 10 years with a liability to pay fine.
16. It is seen that the High Court had arrived at erroneous hypothetical conclusions
ignoring the fact that the nature of injuries were grievous and were caused by use of
sufficient force by sharp edged weapons. The injuries were so serious that both the
investigating agency and the doctor felt that dying declaration was to be recorded. That
being so, the High Court's conclusion that the offence under Section 307 was not made
out is clearly indefensible. The order of the High Court is set aside and that of the trial
Court is restored.
17. The appeal is allowed.
Appeal allowed. .
AIR 2008 SUPREME COURT 2970 "Tata Industries Ltd., M/s. v. M/s. Grasim Industries
Ltd."
Coram : 1 V. S. SIRPURKAR, J.
Arbitration Petition No.5 of 2007, D/- 9 -7 -2008.
M/s. Tata Industries Ltd. and Anr. v. M/s. Grasim Industries Ltd.
(A) Arbitration and Conciliation Act (26 of 1996), S.11(6) - ARBITRATION AND
CONCILIATION - AGREEMENT - Reference to arbitrator - Shareholders agreement
between parties - Alleged breach of confidentiality Clause and improper valuation of
shares - Applicants served termination notices on non-applicant - Subsequent agreement
between parties for purchase of shares of applicant by non-applicant - Formal notice
issued for arbitration by applicant - Applicant made appointment of nominee arbitrator
under arbitration clause - Share Purchase Agreement formally executed thereafter -
Containing "without prejudice" clause and that too despite vehement claims and refusals
of those claims on part of parties - Plea that Share Purchase Agreement obliterated earlier
agreements and there was no live issue - Held, not tenable as non-applicant was already
facing arbitral notice - Application for appointment of arbitrator, entitled to be allowed.
(Paras 32, 35, 37)
(B) Arbitration and Conciliation Act (26 of 1996), S.11(6) - ARBITRATION AND
CONCILIATION - ESTOPPEL - Application for reference of dispute - Non-applicant
succeeded in stalling decision on said application before High Court on ground that one
of applicants was foreign company - Did not pursue issue of said foreign company being
not party to shareholders agreement - Said issue thus was abandoned - Cannot be
permitted to be raised before High Court.
Evidence Act (1 of 1872), S.115. (Para 36)
Cases Referred : Chronological Paras
2007 AIR SCW 2130 : 2007 CLC 745 : 2007 (3) AIR Bom R 524 (Ref.) 18, 22
(2006) 7 SCC 756 (Disting.) 20, 30
2005 AIR SCW 5932 : AIR 2006 SC 450 : 2005 CLC 1546 (Ref.) 22
2005 AIR SCW 6305 : AIR 2006 SC 577 : 2006 (1) AIR Kar R 506 : 2006 (1) AIR Jhar R
385 (Dtsting.) 20, 30
2004 AIR SCW 198 : AIR 2004 SC 1330 (Ref.) 29, 31
(1974) 4 Australian Law Reports (ALR) 257 20, 30
(1942) 1 All ER 337 19
(1926) AC 497 19
(1923) SC (HL) 37 19
Harish N. Salve, R.F. Nariman, Mukul Rohtagi, Sr. Advocate, GopalJain, Ms. Ruby
Singh Ahuja, Ms. Meenakshi Grover, Kamaldeep Dayal, Ms. Pragya Singh Baghel, Manu
Agarwal and Mrs. Manik Karanjawala, for Appellants; K.K. Venugopal, Shyam Diwan,
Sr. Advocates, Dr. Abhishek Manu Singhvi, Bharat Sangal, Gaurav Pachananda, Ankur
Talwar, Sanjeev Adhalakha and Narhari Singh, for Respondent.
Judgement
1. V. S. SIRPURKAR, J. :-Two companies, first being M/s.Tata Industries Ltd., and the
second being M/s. Apex Investments (Mauritius) Holding Private Limited (hereinafter
referred to Applicant Nos.1 and 2 respectively) have approached this Court under Section
11(6) of the Arbitration and Conciliation Act, 1996 (hereinafter referred to as "the Act")
for appointment of the Arbitrator in a commercial dispute which has arisen between them
and Grasim Industries Limited (hereinafter referred to as "the non-applicant"). Initially
the applicants had approached Bombay High Court by way of an application" under
Section 11(6) of the Act, however, a stand was taken by the non-applicant that this would
amount to an international commercial arbitration and, therefore, it would be the Chief
Justice of India alone who would have the powers to constitute the Arbitral Tribunal
under Section
@page-SC2971
11(12) of the Act. It is, therefore, that the matter has come before this Court. The parties
are ad demon this jurisdictional issue that the jurisdiction to appoint the Arbitrator lies
with the Chief Justice of India or as the case may be, his nominee.
2. There is no dispute between the parties that there is an arbitration agreement between
the parties vide Clause 12.04 of the Shareholders Agreement dated 15.12.2000 and
Clause 9 of the Share Transfer Agreement dated 1.6.2006. That issue need not, therefore,
be dilated upon.
3. The parties are also ad idem that the claims are within limitation.
4. The only question to be decided, on which the parties have extensively argued, is
whether there is a live arbitrable issue.
5. Following background facts would help to understand the controversy between the
parties.
6. M/s. Tata Industries Limited (hereinafter referred to as "TIL") is a company
incorporated under the Indian Companies Act, 1956 (Applicant No. 1) while Apex
Investments (Mauritius) Holding Private Limited (Applicant No.2) is a company
incorporated under the Laws of Mauritius. The Applicant No.2 has its registered office at
Mauritius while non-applicant M/s.Grasim is also a company incorporated under the
Indian Companies Act, 1956.
7. Tata Cellular Limited (hereinafter called the "TCL") had obtained a CMTS licence for
Andhra Pradesh Circle on 19.12.2005. Similarly, Birla AT andT Communications Ltd.
(hereinafter referred to as "BACL") which was a joint venture undertaking of A.V. Birla
Group and ATandT Wireless Group held CMTS licences for Maharashtra and Gujarat
Circles since 15.12.1995. Tata Tele services Limited (hereinafter referred to as "TTSL")
was granted a basic service licence for Andhra Pradesh Circle on 4.11.1997. A
Memorandum of Understanding was arrived at between ATandT Wireless Inc., AV Birla
Group and Tata Industries Limited on 1st March, 2000 whereby they agreed to provide
CMTS service through a single entity. As per this Memorandum of Understanding
ATandT Wireless Inc., AV Birla Group and TIL agreed to provide services through a
single entity or an alliance of entities and agreed to merge themselves to form IDEA
Cellular Limited (hereinafter referred to as "IDEA"). The Memorandum of
Understanding was entered into on 13.11.2000 by merging TCL with BACL.
8. A Shareholders Agreement came into existence on 15.12.2000 between ATandT
Wireless Inc., AV Birla Group (through Grasim Industries Limited) and Tata Group
through TIL. In this Agreement respective rights and obligations of the parties for the
merger/amalgamation of the TCL into BACL and modalities and functions of merged
entities were recorded. Under that Agreement, the applicants, the non-applicant and
ATandT Wireless Services Inc., were to hold 44,72,35,136 shares being one-third of the
subscribed and paid up Equity Share Capital of the merged entity, i.e., IDEA. Article
3.04(b) of the Shareholders Agreement provides as under:
"Each founder covenants and agrees that except as set out in Section 3.04(c) and Section
3.04(d), it will not engage in, either directly or indirectly though an affiliate, (i) any
activity that would constitute the business of the merged company within the territorial
telecom circles covered by the licences; or (ii) any opportunity outside the territorial
telecom circles covered by the licences that would constitute the business or the merged
company in India and the neighbouring territories unless the opportunity has been first
offered to the merged company to undertake such new business by placing the same
before the Board of Directors of the merged company. The Board of Directors shall
deliberate, without the participation of the India placing the opportunity before the Board
(the Opportunity Shareholder") on whether to avail of such an opportunity. If the Board
decides not to avail itself of such opportunity or does not convey the decision in respect
thereto within a period of 120 days (or such shorter period as may be necessary in the
context of the nature of the opportunity) from the date of receipt of such offer by the
merged company then;
(i) the Opportunity Shareholder shall invite the other Founders for discussions on whether
a joint venture for availing the opportunity may be undertaken;
(ii) If the other Founders do not wish to participate in a joint venture the Opportunity
Shareholder shall be free to avail of the opportunity on its own."
Article 8 is the Confidentiality Clause which reads as under:
@page-SC2972
"8.01 Confidential Information Defined : For the purposes of this Agreement,
"confidential information" shall mean all oral, written and/or tangible information created
by the merged company or disclosed by a Founder (in either case "owner") to the
receiving Founder ("Recipient") which is confidential, proprietary and/or not generally
available to the public, including but not limited to information relating in whole or in
part to present and future products, services, business plans and strategies, marketing
ideas and concepts, especially with respect of unannounced products and services,
present and future product plans, pricing, volume estimates, financial data, product
enhancement information, business plans, marketing plans, sales strategies, customer
information (including customer's applications and environment), market testing
information, development plans, specifications, customer requirements, configurations,
designs, plans, drawings, apparatus, sketches, software, hardware data, prototypes or
other technical and business information. Notwithstanding the foregoing, information
shall not be deemed confidential and Recipient shall have no obligation with respect to
any such information which:
(a) is already known to Recipient, or
(b) is or becomes publicly known, through any means including publication, inspection of
a product, or otherwise, and through no negligence or other wrongful act of Recipient, or
(c) is received by recipient from a third party without similar restriction and without
breach of this Agreement, or
(d) is independently developed by Recipient, or
(e) is furnished to a third party by owner without a similar restriction on the third party's
rights.
8.02 Treatment of Confidential Information
From the execution of this Agreement until three (3) years after the Recipient ceases to be
a shareholder, Recipient shall, and shall cause its affiliates to, keep confidential and will
not disclose, and will cause its affiliates not to disclose, to third parties, the confidential
information receive from, or made available by owner in the course of the transactions
contemplated hereby and will use and cause its affiliates to use, the same level of care
with respect to the confidential information as Recipient employees with respect to its
own proprietary and confidential information of like importance, and will not use and
will cause its affiliates not to use such confidential information for any purpose other than
the performance of its obligations under this Agreement. Promptly upon the Recipient
ceasing to be a shareholder, written confidential information will be returned to Owner or
destroyed immediately upon the request of owner, and no copies, extracts or other
reproductions shall be retained by the Recipient. All documents memoranda, notes and
other writings whatsoever prepared by recipient which contain the confidential
information shall be returned to owner or destroyed at owner's request. Confidential
information provided by owner shall remain the property of owner. For the avoidance of
doubt, the merged company shall not be deemed to be a Recipient for purposes of this
Section.
8.03 Notice prior to disclosure :
If Recipient (or its affiliate) is requested or required (by oral questions, interrogatories,
requests for information or documents, subpoena, civil investigative demand or similar
process) to disclose any confidential information, Recipient will promptly notify owner
of such request or requirement so that owner may seek an appropriate protective order or
waive compliance with the provisions of this Section 8.03. If, in the absence of a
protective order or the receipt of a waiver hereunder, Recipient (or any of its affiliates) is
in the written opinion of Recipient's counsel compelled to disclose the confidential
information or else stand liable for contempt or suffer other censure or significant
penalty, Recipient (or its affiliates) may disclose only so much of the confidential
information to the party compelling disclosure as is required by law. Recipient will
exercise (and will cause its affiliate to exercise) reasonable efforts to obtain a protective
order or other reliable assurance and confidential treatment will be accorded to
confidential information."
Article 9 of the Agreement deals with the effectiveness and termination. Article 9.02
relates to right to terminate for cause. It specifically provides that in the event of
occurrence of a material breach on the part of the Defaulting Founder, each Founder shall
have right to make the election as provided in Clause 9.02(b). However, for that purpose
the electioning Founder should have given
@page-SC2973
written notice of the alleged breach to Defaulting Founder and in terms of that notice the
Defaulting Founder has not cured, within 60 days, the said breach, if the said breach is
capable of being cured within such period, or the Defaulting Founder has not taken
substantial and appropriate steps to cure the breach. The "material breach" is defined in
this Clause as :
(i) a breach of confidentiality provisions set forth in Article VIII of the Agreement;
(ii) breach of the provisions of Section 3.04(b) relating to non-competition;
(iii) the failure to contribute capital as required under Section 2.04(a);
(iv) a breach of provisions relating to election of Directors, filling of Board vacancies,
removal of Directors and election of Chairman of the Board;
(v) a breach of the provisions under Article 10 relating to transfer of equity, capital or
voting interest;
(vi) a breach by a party of the provisions under Section 6.02.
Article 10.06 provides that if any party receives from or otherwise negotiates with third
parties a bona fide offer to purchase any of the equity capital owned or held by such party
and intends to make sale of its shares to such third party, such founder must notify the
other two parties of the Shareholders Agreement by way of a notice mentioning offer
price, the third parties making the offer and the number of shares that such third party
wants to purchase on which the offeree would have option to purchase such amount of
shares at the offer price within 45 days of the offer notice.
9. The Arbitration clause worded in Article 12.04 and Article 12.04(a), (b), (c), (d) and (e)
are as under :
"12.04 Governing law and consent to Jurisdiction: Arbitration :
(a) This Agreement and all questions of its interpretation shall be construed in accordance
with the laws of the Republic of India without regard to its principles of conflict of laws.
(b) The parties agree that they shall attempt to resolve through good faith consultation in
their behalf, disputes arising in connection with this Agreement and such consultation
shall begin promptly after a party has delivered to another party a written request for such
consultation.
(c) In the event that, after exhausting the efforts for resolution of dispute described in
paragraph (b), the parties have been unable to resolve a dispute, and if the dispute is one
which relates to an alleged breach of any representation, warranty, covenant or agreement
under or the validity or termination of this agreements, such dispute shall be finally
settled according to the procedure set forth in paragraph (d).
(d) A dispute subject to resolution under this paragraph (d) shall be finally settled by
binding arbitration in Mumbai, India before and pursuant to the Indian Arbitration and
Conciliation Act, 1996. Each party shall select an arbitrator within fifteen (15) days from
the initial arbitration request.
Promptly upon their selection, such arbitrators shall agree upon and select a third
arbitrator from the panel of arbitrators UNCITRAL. The parties shall agree in advance as
to manner in which the arbitration panel shall promptly hear witnesses and arguments,
review documents and otherwise conduct the arbitration proceedings. Should the parties
fail to reach an agreement as to the conduct of the arbitration proceeding within twenty
(20) days from the selection of the third arbitrator, the arbitration panel shall formulate its
own procedural rules and promptly commence the arbitration proceedings. The
arbitration proceedings shall be conducted as expeditiously as possible with due
consideration for the complexity of the dispute in question. The arbitration panel shall
issue its decision in writing within thirty (30) days from the hearing of final arguments by
the parties. The parties agree that the arbitrators will have the power to rule on questions
of its own jurisdiction over any dispute, to award damages and, in appropriate
circumstances, to award equitable relief but shall not be authorized to award punitive or
exemplary damages to any party. The parties specifically agree to be bound by the
decisions rendered by the arbitration panel provided for herein and agree not to submit a
dispute subject to this Section 12.04 (d) to any federal, State or local court or arbitration
association except as may be necessary to enforce the procedures of this Section 12.04(d)
or to enforce the decision rendered by the arbitrators. The parties to any dispute submitted
to arbitration here-under shall share equally the costs of the arbitral panel and shall each
bear their own attorneys' fees and other expenses incurred
@page-SC2974
in connection with any arbitration proceedings. If court proceedings to stay litigation or
compel arbitration are necessary, the party who unsuccessfully opposes such proceedings
shall pay all associated costs, expenses and attorneys' fees which are reasonably incurred
by the other party.
(e) During the pendency of a dispute/ arbitration proceedings the parties shall be bound
by the terms of this Agreement."
The parties point out that in 2004 ATandT Wireless Services Inc., which was the holding
company of ATandT Cellular Private Limited (Mauritius), merged with New Cingular
Wireless Services Inc. (hereinafter referred to as "NCW) pursuant to a global restructure
cum merger. Subsequently in September, 2005 TIL acquired the entire shareholding of
ATandT Cellular Pvt. Ltd. from NCW and the Birla Group acting through Aditya Birla
Nuvo Ltd. (hereinafter referred to as "ABNL") acquired 16.45% shares in IDEA from
ATandT Cellular Private Limited. ATandT Cellular Private Limited was subsequently
renamed as Apex Investments (Mauritius) Limited. As a result of this, the shareholding of
Birla Group in IDEA increased to 50.14% while TIL continued to have 31.69% of the
issued share capital. The balance 1.70% of IDEA was held by AIG (Mauritius) LLC.
10. On 31.1.2006, the Applicant No.1 served a notice on the non-applicant under Article
9.02 of the Shareholders Agreement in which it was stated that pursuant to an e-mail
communication received by it from one Mr. Sanjeev Aga, it was clear that Aditya Birla
Telecom Limited (hereinafter referred to "ABTL"), a subsidiary of ABNL had applied to
the Department of Telecommunication for grant of UAS license for the Mumbai Metro
Circle. It was further stated in that notice that vide Board Circular dated 29.7.2005, the
Board of Directors of IDEA had accepted proposal to apply for UAS licence for the
Mumbai Metro Circle and consequently the said application was filed on 3.8.2005. The
Applicant No. 1 thus asserted that the filing of the application for UAS licence by ABTL
for the Mumbai circle was in clear violation of Article 3.04(b) of the Shareholders
Agreement and amounting to a material breach by Aditya Birla Group under Article 9.02
of the Shareholders Agreement and accordingly it was requested to cure the said material
breach within 60 days of the receipt of the said letter by withdrawing the said application
made by ABTL for grant of UAS licence for Mumbai Circle.
11. On 27.2.2006, Applicant No.1 sent the Termination Notice under Article 9.02 (b) of
the Shareholders Agreement that ABNL, which was an affiliate company of Aditya Birla
Group, for the purposes of Shareholders Agreement had displayed confidential financial
data of IDEA including particulars of revenue, PBDIT, OPM%, PBIT, Net profit/loss,
capital employed, ROCE (annualized)% and projections, on its website. It was asserted
that this information displayed on the website was confidential information within the
meaning assigned to the said term in Article 8.01 of Shareholders Agreement. Since such
confidential information was not in the public domain and since none of the exceptions to
the protection of confidential information contained in Section 8.01 of the Shareholders
Agreement were available to ABNL, the disclosure of confidential information was a
clear breach of Article 8.02 of the Shareholders Agreement. It was also asserted that such
material breach was not capable of being cured and, therefore, the said letter was to be
treated as the Termination Notice in accordance with Article 9 of the Shareholders
Agreement and the Applicant No. 1 was proceeding to purchase the shareholding of AV
Birla Group within 90 days of the receipt of the said notice. A copy of the notice was also
endorsed to IDEA in order to take steps to facilitate access to an international firm of
auditors appointed by Applicant No.1 for computing the fair market value of the IDEA
shares.
12. These letters dated 20.2.2006 and 1.3.2006 were disputed by letters dated 31.1.2006
and 27.2.2006 respectively wherein a clear cut denial was asserted to the effect that there
was no violation of the provisions of the Shareholders Agreement relating to non-
competition and confidentiality. Then vide the subsequent correspondences dated
16.3.2006 and 27.3.2006, Applicant No.1 reaffirmed all its stand and its entitlement to
purchase the entire equity share capital of the non-applicant in IDEA in accordance with
the provisions of Shareholders Agreement. There was a further denial on the part of the
non-applicant by its letters dated 17.3.2006 and 3.4.2006.
13. However, in the meantime, the applicant received an offer for purchasing its stake as
well as the stake of M/s. Apex Investments
@page-SC2975
(Mauritius) Holding Private Limited in IDEA from Global Communication Services
Holding Limited. In such an eventuality, in terms of Clause 10.06, the applicants were
bound to offer the shares at the same price to the non-applicant. Accordingly, a notice
dated 5.4.2006 came to be served by Applicant No. 1 on its behalf and on behalf of its
subsidiary M/s. Apex Investments (Mauritius) Holding Private Limited, offering the said
shares to the non-applicant. In this notice, the material terms and conditions of the offer
were specified vide para 3. It was stated :
"By this letter, TIL and Apex are intimating the AV Birla Group of their having received a
bona fide offer for purchase of the Sale Shares and of TIL and Apex having accepted the
offer made to them, subject to this prior offer being made to the AV Birla Group. Without
prejudice to the notices of termination dated January 31, 2005 and February 27, 2006
issued by TIL to the AV Birla Group, TIL and Apex are hereby making the first offer for
purchase of the Sale Shares to the AV Birla Group at the price and on the material terms
and conditions mentioned above. This Offer Notice shall remain irrevocable for a period
of 45 days after the Notice Date (i.e., 45 days after the receipt of this Offer Notice by
yourselves).
On the very next day of this Offer, the non-applicant vide its communication dated
6.4.2006 accepted the Offer of Purchase. However, it was specified in that reply that their
acceptance of the Offer was without prejudice to their contentions that the notices of
termination referred to in the said Offer Notice were not tenable. The applicant TIL on
the very next day, i.e. 7.4.2006, conveyed that they would be shortly forwarding two
Draft Share Purchase Agreements for the sale of IDEA shares held by Tata Industries
Limited and Apex Investments (Mauritius) Holding Private Limited. However, in the
second para of this Notice, it was reiterated as under:
"We reiterate that our offer and your acceptance thereof, is without prejudice to the
notices of termination dated January 31, 2006 and February 27, 2006 issued by us, and
your rival contentions which we have not accepted."
14. On 10.4.2006, the letter dated 7.4.2006 was replied to. There again, it was reiterated :
"As regards our position regarding the notices of termination, we wish to reiterate your
position already communicated."
On 19.4.2006, by the even dated letter, the applicant reiterated that the non-applicant had
breached the Confidentiality Clause and that the claim of the non-applicant that the data
displayed on the website was not confidential under the Shareholders Agreement, is
misplaced. It was asserted in para 6 of the letter that non-applicant No. 1 was making
contradictory statements. It was lastly asserted:
"As already explained above, since the AV Birla Group is in violation of the provisions of
the Shareholders Agreement, the First Notice and the Second Notice issued by us cannot
be withdrawn. Since you have failed in agreeing upon the name of the international firm
of auditors we shall therefore proceed in accordance with the Shareholders Agreement to
do the needful in connection with determining the Fair Market Value of IDEA shares."
Again on 19.4.2006 by the letter of even date, the applicants reaffirmed the contents of
Notices of Termination dated 31.1.2006 and 27.2.2006.
15. On 24.4.2006 the Non-applicant replied to the letter dated 19.4.2006 and intimated
the applicants their readiness to make full and final payment against the delivery of
shares by TIL and the Apex. On 24.4.2006 the applicants issued a notice requesting to
commence the process of consultation with respect to the dispute which had arisen
between the parties and which was more particularly highlighted in their earlier notices.
In that notice it was said:
"We note that vide your letters dated February 28, 2006, March 1, 2006, March 17, 2006,
April 3, 2006 and April 24, 2006, you have disputed the contents of the said Notices and
have also disputed the fact of commitment of "material breaches" of Section 3.04 and 9
6f the Shareholders Agreement by you.
It is, therefore, apparent that a dispute has arisen between us in connection with the terms
of the Shareholders Agreement."
A further reference was given to Clause 12.04 (b) of the Shareholders Agreement which
provided for consultation and, therefore, by the same notice the process of consultation
was called upon by the applicants. Lastly it was straightway suggested that:
@page-SC2976
"In case you are not interested in exploring the process of consultation as aforesaid, and
would like to proceed with arbitral proceedings straightway, we would be agreeable to
that procedure also."
On 27.4.2006 the non-applicant responded to the applicants consultation notice and
asserted that there was no arbitral dispute surviving between the parties and, therefore,
there was no longer a basis for arbitration regarding the issues set forth in the termination
notices. The non-applicant also requested the applicants to withdraw the consultation
notice. It was, therefore, pointed out through the notice dated 5.5.2006 by the applicants
that the termination of the agreement on their part was made without prejudice to the
pendency of the dispute and/or arbitration proceedings and the position was clarified
from time to time. On 5.5.2006 the applicants formally served a letter by way of formal
notice for arbitration. This notice was contested by the non-applicant by its letter dated
17.5.2006 again reiterating its position that there was no arbitrable dispute surviving
between the parties. Hence on 19.5.2006, the applicants communicated to the non-
applicant that they had appointed their nominee Arbitrator under Clause 12.04 (d) of the
Shareholders Agreement.
16. So far so good. Thereafter two share purchase agreements were entered into between
the applicants and the non-applicant. There was a specific reference made to the claim of
arbitration made on behalf of the TIL in Clause (d) which runs as under:
"In relation to the notices dated January 31, 2006 and February 27, 2006 issued by TIL
(as a founder under the Shareholders Agreement as hereinafter defined) to the AV Birla
Group, TIL has pending arbitration disputes with the AV Birla Group, TIL has not
accepted the rival contentions of the AV Birla Group in respect of the said notices nor has
the AV Birla Group accepted, TIL's claim pending Arbitration. As such, the execution of
and consummation of the transaction contemplated by this Agreement shall not prejudice
or affect the pendency of continuation of the arbitration proceedings between TIL and AV
Birla Group."
In Clause 2.3 it was further specified in this Agreement that:
"TIL for itself and Apex has issued the Termination Notices. The offer made by TIL to the
AV Birla Group a pursuant to the offer notice, was made without prejudice to the said
Termination Notices, Grasim Industries Limited, as part of the AV Birla Group has
refuted the Termination Notices. Further, in order to enforce the rights accrued to TIL and
Apex on account of the Termination Notices, TIL, for itself and Apex has pursuant to
clause 12.04(d) of the Shareholders Agreement on May 5, 2006 issued a notice to
arbitrate to the AV Birla Group. Accordingly, this Agreement is being executed without
prejudice to the rival contentions of either party with reference to the Termination Notices
and the legal rights which have accrued to each party (including Apex acting through TIL
as founder) under the Shareholders Agreement."
In pursuance of this an application was filed before the Bombay High Court and as has
been stated earlier, the said application came to be withdrawn and a fresh application
came to be made before this Court for the appointment of Arbitrator.
17. Mr. Harish N. Salve, Mr. R.F. Nariman and Mr. Mukul Rohtagi, Senior Counsel on
behalf of the applicants painstakingly took the court through the Shareholders Agreement
as well as the Share Purchase Agreement. The Court was taken through the whole history
along with necessary documents and it was reiterated that the appointment of Arbitral
Tribunal was a must for resolving the live issues between the parties. Learned counsel
firstly urged that there was no dispute about there being Shareholders Agreement to
which contesting entities were the parties and further that the applicants held shares in
IDEA subject to discipline of Shareholders Agreement. Learned counsel thereafter relied
upon the Notice dated 31.1.2006 whereby a material breach was alleged on the part of the
non-applicant, i.e., to secure the licences which would necessarily involve business in
competition with IDEA. The attention of the Court was also drawn to the second notice
dated 27.2.2006 by which the confidentiality clause was alleged to have been breached,
which breach was an incurable breach. Thus the applicants asserted that under the terms
of the Shareholders Agreement, the applicants were entitled to purchase the shares of the
non-applicant in the event of the breach at a price that was heavily discounted price, i.e.,
at 25% less than the market price as per clause 9.02(b)and(c) of the Share holders
@page-SC2977
Agreement. It was, therefore, urged that the dispute had arisen which has been shown in
the Termination Notices dated 31.1.2006 and 27.2.2006, which has been disputed by the
non-applicant's reply dated 28.2.2006 and 1.3.2006. It was, therefore, urged that such
dispute clearly fell within the Arbitration Clause.
18

. It was further urged that there was a live issue between the parties and that there was a
difference between a live issue and a live claim. It was urged that whether the claim had
any merit or not was a matter which could be considered at the stage of appointing an
Arbitrator and an issue which has never been closed or treated to have been closed
remains a live issue. Heavy reliance was placed on the judgment of this Court reported in
Shree Ram Mills Ltd. v. Utility Premises (P) Ltd. [(2007) 4 SCC 599 at p.607]. It was,
therefore, urged that as per the principles laid down, the initial notices of termination
were disputed by the non-applicant; that on 27th March, 2006 an international firm of
auditors was appointed by the applicants to determine the fair value of shares; that on 5th
April, 2006, the applicant had received an offer for selling these shares to a third party
and on that very date the applicant had raised the dispute although the arbitration
proceedings had by then not yet commenced. It was pointed out by the learned counsel
that the non-applicant in its reply dated 28.2.2006 had very clearly understood this
position and has reiterated therein as under : 2007 AIR SCW 2130

"Your Second Notice is wrongful, illegal, mala fide, non est factum and designed to take
advantage of your own wrongs. The Shareholders Agreement continues to subsist under
law and contract, despite the second notice. Such notice is of no consequence under the
Shareholders Agreement or in the eye of law. You are not entitled to rely upon or take any
advantage based upon the Second Notice."
Learned counsel pointed out that in the first notice there was no termination of contract
but it was a notice to correct the material breach while the second notice purported to
terminate the Agreement. Learned counsel also pointed out that in the reply dated
28.2.2006 there was a tacit agreement that the Shareholders Agreement remained in
existence inasmuch as it was stated in its notice that :
Any breach by you of the provisions of the Shareholders Agreement relating to transfer of
shares will result in an invalid and illegal transfer, which will not be binding on either
Idea or the Birla Group. Also, such breach will be a material breach under Section 9.02 of
the Shareholders Agreement. Please be advised that the Birla Group is committed to
asserting its rights in respect of any attempted breach in accordance with law. Please also
note that any third party participating in any sale of the shares of Idea held or controlled
by the Tata Group will also be liable to the Birla Group. We also hereby put all such third
parties on notice of your obligations towards the Birla Group, and the Birla Group's
potential claims against them for inducing the breach of the Shareholders Agreement."
Learned counsel further argues that the offer which was made on 5th April, 2006 was a
"without prejudice" offer which was accepted by non-applicant on "without prejudice"
basis on 6th April, 2006 and therefore, when on 24th April, 2006 the non-applicant
informed the applicants that they were ready to make full and final payment on the same
day, the applicants requested to commence the process of consultation to resolve the
dispute which had arisen. Learned counsel also pointed out that subsequently when the
Share Purchase Agreement was executed on 1st June, 2006, it contained clauses which
made it clear that the disputes were being kept outside this arrangement and they would
be resolved separately and thus the live issue remained to be alive.
19. It was further argued that though the non-applicant had contested the notices of
termination suggesting that there was no arbitrable issue, still an agreement was entered
into on 1st June, 2006 wherein there was clear reference to the clauses keeping the
disputes arising out of the live issues alive. Learned counsel relied upon the following
observations made in Heyman and Anr. v. Darwins Ltd. [(1942) 1 All ER 337] :
"Repudiation, then, in the sense of a refusal by one of the parties to a contract to perform
his applications, thereunder, does not of itself abrogate the contract. The contract is not
rescinded. It obviously cannot be rescinded by the action of one of the parties alone. Even
if the so-called repudiation is acquiesced in or accepted by the other party, that does not
end the contract. The
@page-SC2978
wronged party has still his right of action for damages under the contract which has been
broken, and the contract provides the measures of those damages. It is inaccurate to speak
in such cases of repudiation of the contract. The contract stands, but one of the parties has
declined to fulfil his part of it. There has been what is called a total breach or a breach
going to the root of the contract, and this relieves the other party of any further obligation
to perform what he for his part has undertaken."
"I am accordingly of opinion that what is commonly called repudiation or total breach of
a contract, whether acquiesced in by the other party or not, does not abrogate a contract,
though it may relieve the injured parry of the duty of further fulfilling the obligations
which he has by a contract undertaken to the repudiating party. The contract is not put out
of existence, though all further performance of the obligations undertaken by each party
in favour of the other may cease. It survives for the purpose of measuring the claims
arising out of the breach, and the arbitration clause survives for determining the mode of
their settlement. The purpose of the contract have failed, but the arbitration clause is not
one of the purposes of the contract."
Similarly other observations by House of Lords reported in the above case were also
heavily relied upon to the effect :
"To say that the contract is rescinded or has come to an end or ceased to exit may in
individual cases convey the truth with sufficient accuracy, but the fuller expression that
the injured party is thereby absolved from future performance of his obligations under the
contract is a more exact description of the position. Strictly speaking, to say that, upon
acceptance of the renunciation of a contract, the contract is rescinded is incorrect. In such
a case the injured party may accept the renunciation as a breach going to the root of the
whole of the consideration. By that acceptance he is discharged from further performance
and may bring an action for damages, but the contract itself is not rescinded.
The injured party may therefore rely upon the contract and apply to have the action
stayed if he desires to do so."
"The same observations as apply to accepted repudiation apply, I think to frustration. The
phrase 'frustration of the contract' is as inaccurate in expression as is the phrase
'rescission of the contract by repudiation'. The contract is not frustrated. Its future
performance or the adventure is frustrated. The damages are still at large and so is the
question whether, having regard to the terms of the contract, express or implied, there has
been frustration or not. This appears to have been recognized in Scott and Sons v. Del Sel
[(1923) SC (HL) 37) which though a Scottish case was decided on the same principles as
apply in English law, and is binding upon your Lordships' House. So far as the case of
Hirji Mulji [(1926) AC 497], which is not binding, lays down a different principle, I do
not think it should be followed, despite the authority which is undoubtedly possesses."
Shortly stated the claim of the applicants is for the profits they had become entitled to by
the breach of confidentiality clause on the part of the non-applicant which profit was on
account of applicant's entitlement to purchase the shares of non-applicant at the rate 25%
less than the market price. Its reiteration is that though subsequently it agreed to sell all
its shares to the non-applicant on account of the offer having been made to it by Global
Communication Services and though subsequently the Share Purchase Agreement dated
1.6.2006 was executed between the applicants and the non-applicant, since the said
agreement was subject to the disputes raised earlier and without prejudice to its rights it
could still lay its hand on its claim.
20. As against this Shri K.K. Venugopal, Dr. Abhishek Manu Singhvi and Shri Shyam
Diwan, learned senior counsel for the Non-applicant opposed the appointment of Arbitral
Tribunal on various grounds and basically on the ground that there was no live issue
remaining pending between the parties. The major contention is that by Share Purchase
Agreement, the applicants had made an exit from the company and, therefore, they were
left with no rights under the Shareholders Agreement. A very heavy reliance was placed
by Shri Venugopal on Clauses 7.01 (a) and (b) which are as under :
"7.01 (a) For a founder to exercise its rights under this Agreement, the Founder must hold
15% of the issued, subscribed and paid up Equity Capital (the "Threshold Limit").
(b) Should the shareholding of any Founder fall below the Threshold Limit :
@page-SC2979
i) Such Founder shall not be entitled to exercise any rights under this Agreement;
ii) Such Founder shall comply with all obligations set forth in this Agreement;
iii) Such Founder shall act in accordance with the instructions of the other founders;
iv) any disposal by such Founder of any shares held by it shall be in accordance with
Article X."
Heavily relying on this clause, the learned Senior Counsel argued that once by a Share
Purchase Agreement, the applicants agreed to sell their shares to the non-applicant and
due to such act once the shareholding of the applicants fell below 15%, it had no right to
assert more particularly under the Shareholders Agreement. Learned counsel argues that
there were two options to the applicants when they received an offer from Global
Communications Services, a third party, for the sale of their shares if the applicant had
accepted the offer. First option was to sell the entire shareholding to Global
Communications and exit IDEA in which case the non-applicant could also choose to sell
its entire shareholding in IDEA to Global Communications by giving a Special Disposal
Notice and once the Tata Group allowed the Birla Group to sell all its shares to Global
Communications, it could no longer require the Birla Group to then sell the same shares
to the Tata Group for the reason that Birla Group would no longer have those shares. The
second option, according to the learned counsel was to sell the entire shareholding in
IDEA to the non-applicant and exit IDEA. According to the learned counsel in both these
cases on acceptance of the offer by the applicants from Global Communications, there
would be an inevitable result of voluntary effacement of all rights that the applicants
claim on the Birla Group shareholding in IDEA. It was pointed out that the applicants had
accepted this offer from the Global Communications much before it attempted to reserve
any rights against the non-applicant and, therefore, had irrevocably elected to exit IDEA,
electing for second option. It was, therefore, urged that there was a concluded contract
between Global Communications and the applicants even before a letter dated 5.4.2006
was sent by the applicants to Birla Group to purchase their shareholding in IDEA and
hence the applicants had elected to exit IDEA themselves. In support of this proposition,
the learned counsel relied on three cases :
(i) Sargent and ASL Developments Limited, reported at (1974) 4 Australian Law Reports
(ALR.) 257.
(ii) Jai Narain Parasrampuria (Dead) and others vs. Pushpa Devi Saraf and others,
reported at (2006) 7 SCC 756.

(iii) National Insurance Company Limited vs. Mastan and another, reported at (2006) 2
SCC 641. 2005 AIR SCW 6305

21. In short, it was contended that by accepting the offer of Global Communications the
claim made by the applicants under Termination Notices for purchase of Birla Group
shareholding in IDEA at a defaulting price got necessarily effaced with the subsequent
option given in the offer notice to Birla Group to sell their entire shareholding to Global
Communications. It was further urged that with the offer of sale to Global
Communications, the applicants recognized and acquiesced that the Birla Group's
shareholding in IDEA was not subject to any encumbrance or reservations and the
applicants, therefore, necessarily abandoned all purported claims for the purchase of Birla
Group's shareholding and there was nothing left with the applicants to reserve in the offer
notice or any documents executed thereafter. It is pointed out that the Agreement to Sell
to Global Communications took place without any notice to the Birla Group and contrary
to the alleged right of applicants to buy out Birla Group and remain invested in IDEA
with increased shareholding, the applicants themselves exited IDEA by selling their
shares to Birla Group companies and thereby reducing the Tata Group's shareholding in
IDEA to Nil. It is accordingly suggested that as a result all rights of Tata Group pursuant
to Termination Notices were effaced and there was no arbitrable dispute surviving
between the parties. It is pointed out that there was no claim for damages in the
Termination Notice and in fact such a claim was not even possible because it was
excluded by the decision to seek a buy-out under Clause 9.02 of the Shareholders
Agreement. It was then stated that the claim by the applicants was frivolous and
unsustainable because of the alleged loss of an opportunity to buy-out Birla Group's
shareholding in IDEA was not caused by any breach of contract by any Birla Group
company but it was by applicants' own voluntary decision to exit the company and
thereby lost the right to acquire
@page-SC2980
Birla Group's shareholding in IDEA. It is pointed that the purported rights of the
applicants under the Termination Notice is not possible in law also particularly because
the Tata Group's election to exit IDEA and to let Birla Group remain invested in IDEA
and as such all their claims were erased. Learned counsel also relied on Clause 7.01(b)(i)
of the Shareholders Agreement which is as under :
i) Such Founder shall not be entitled to exercise any rights under this Agreement;
It was suggested that even if the Arbitration clause remains in spite of the termination of
the Shareholders Agreement, because of the above-mentioned clause, there is complete
bar created on the exercise by the applicants of the substantive right to seek a buy out. A
live procedural mechanism of dispute resolutions through arbitration cannot be set in
motion to enforce a dead claim and, therefore, there would be no question of starting an
arbitration proceeding.
22

. As to what should be the approach of the Court in an application under Section 11 (6)
was explained by this Court in SBP Co. vs. Patel Engineering Ltd. and another, reported
in (2005) 8 SCC 618. This was a decision to resolve as to whether the Chief Justice of
India or his nominee, while dealing with the matters under Section 11(6) acts in his
administrative capacity or his judicial capacity. The Court ultimately, held that the Chief
Justice of India acts in his judicial capacity. Hon. P.K. Balasubramanyan, J., who
authored the judgment on behalf of the majority, in his conclusions, pointed out clearly in
point iv (points i, ii and iii not relevant) that :- 2005 AIR SCW 5932

"(iv) The Chief Justice or the designated Judge will have the right to decide the
preliminary aspects as indicated in the earlier part of this judgment. These will be his own
jurisdiction to entertain the request, the existence of a valid arbitration agreement, the
existence or otherwise of a live claim, the existence of the condition for the exercise of
his power and on the qualifications of the arbitrator or arbitrators..........................."

The law, thus, stands crystallised by this judgment indicating the exact scope of the Judge
in dealing with an application under Section 11(6) of the Act. The judgment was
thereafter considered in number of cases. In Shree Ram Mills Ltd. vs. Utility Premises
(P) Ltd., reported in (2007) 4 SCC 599, it was observed relying on observations made in
paragraph 39 of SBP Co. vs. Patel Engineering Ltd. and another, reported in (2005) 8
SCC 618 (cited supra) : 2007 AIR SCW 2130
2005 AIR SCW 5932

"A glance on this para would suggest the scope of the order under Section 11 to be passed
by the Chief Justice or his designate. Insofar as the issues regarding territorial jurisdiction
and the existence of the arbitration agreement are concerned, the Chief Justice or his
designate has to decide those issues because otherwise the arbitration can never proceed.
Thus, the Chief Justice has to decide about the territorial jurisdiction and also whether
there exists an arbitration agreement between the parties and whether such party has
approached the court for appointment of the arbitrator. The Chief Justice has to examine
as to whether the claim is a dead one or in the sense whether the parties have already
concluded the transaction and have recorded satisfaction of their mutual rights and
obligations or whether the parties concerned have recorded their satisfaction regarding
the financial claims. In examining this if the parties have recorded their satisfaction
regarding the financial claims, there will be no question of any issue remaining. It is in
this sense that the Chief Justice has to examine as to whether there remains anything to be
decided between the parties in respect of the agreement and whether the parties are still at
issue on any such matter. If the Chief Justice does not, in the strict sense, decide the issue,
in that event it is for him to locate such issue and record his satisfaction that such issue
exists between the parties. It is only in that sense that the finding on a live issue is given.
Even at the cost of repetition we must state that it is only for the purpose of finding out
whether the arbitral procedure has to be started that the Chief Justice has to record
satisfaction that there remains a live issue in between the parties........"
"It is for this reason that it was pointed out in the above para that it would be appropriate
sometimes to leave the question regarding the live claim to be decided by the Arbitral
Tribunal. All that he has to do is to record his satisfaction that the parties have not closed
their rights and the matter has not been barred by limitation."
@page-SC2981
In the wake of above decision, it will be now necessary to see whether a live claim or a
live issue exists in between the parties.
23. As has already been clarified in the earlier part of the judgment that there is
jurisdiction in this Court to decide the application, secondly, there is arbitration
agreement between the parties, and thirdly, the claim is not barred by limitation, the only
issue which has to be decided is whether there is a live issue in the sense as to whether
the parties have already concluded or recorded their satisfaction regarding the issues or
whether the parties are still in contest regarding certain issues.
24. In the backdrop of what has been asserted by the applicant and the stands taken by the
non-applicant, it would be first better to note certain dates, which are of extreme
importance. It will be seen there as below :

15.12.2000 : A Shareholders Agreement (SHA) came into effect between


ATandT Group, Birla Group and Tata Group.
9.11.2005 : The name of ATandT Cellular Pvt. Ltd., Mauritius was changed to
M/s. Apex Investments (Mauritius) Holding Private Limited (APEX).
31.1.2006 : The first termination notice was served by the applicant on the
non-applicant, alleging breach of Shareholders Agreement on account of the non-
applicant Company having applied for Unified Access Service Licenses (UAS Licenses),
enabling license holder to provide any kind of telecommunication service for Mumbai
Metro circle and thereby committing breach of Article 3.04 (b) of the Shareholders
Agreement and asking the non-applicant to remedy the breach within 60 days.
27.2.2006 : A second termination notice was sent by the applicant to the non-
applicant claiming that there was a breach of confidentiality clause committed by the
non-applicant and since this breach was incapable of being cured, the applicant
demanded to purchase the shareholding of the Birla Group within 90 days of the said
notice at a default price.
28.2.2006 : The non-applicant disputed the first termination notice dated
31.1.2006, while on 1.3.2006, the non-applicant replied to the second termination notice
dated 27.2.2006.
16.3.2006 : The applicants reiterated both the termination notices.
17.3.2006 : The non-applicant wrote a letter observing therein : " ..... Based
upon your abovereferenced two letters, we conclude the Shareholders Agreement
continued to remain valid, subsisting and enforceable."
27.3.2006 : The applicants reiterated the second notice of termination insisting
that they were entitled to acquire shares and shown their readiness to appoint
international firm of auditors to determine the fair market value of the shares to be
purchased by them.
3.4.2006 : The non-applicant refused the claim once again.
5.4.2006 : The applicant received the offer from Global Communication
Services for purchasing its stake in IDEA and hence, it makes offer for the sale of its
shareholding under clause 10.06 to the non-applicant. This offer notice was expressly
"without prejudice" to both the termination notices.
6.4.2006 : This offer was accepted by the non-applicant. However, it was
clarified that the acceptance of this offer was without prejudice to the contention that the
notices of termination were not tenable.
7.4.2006 : The applicant sent the letter stating therein that it would be shortly
forwarding two Draft Share Purchase Agreement for the sale of IDEA @page-SC2982
shares held by it and M/s. Apex Investments (Mauritius) Holding Private Limited. There
was again a reiteration that the offer made by it and the acceptance thereof was without
prejudice to the termination notices dated 31.1.2006 and 27.2.2006.
10.4.2006 : The non-applicant submitted the particulars of the Birla Group
Companies, which would be purchasing the shares of the applicant and M/s. Apex
Investments (Mauritius) Holding Private Limited under the two Share Purchase
Agreements with the percentage of shares to be purchased by such companies.
19.4.2006 : The applicant reaffirmed its termination notices dated 31.1.2006
and 27.2.2006 and also informed that it would proceed to appoint an international firm of
auditors for determining the fair market value of IDEA shares.
24.4.2006 : The non-applicant replied and informed that they were ready to
make full and final payment against the delivery of shares held by TIL and M/s. Apex
Investments (Mauritius) Holding Private Limited in IDEA Cellular Ltd.
24.4.2006 : The applicant made a request to commence the process of
consultation with respect to disputes which have arisen between the parties on account of
breach of Shareholders Agreement.
27.4.2006 : A letter was sent by the non-applicant that in view of the
agreement between the parties for purchase of the shares of applicant by the non-
applicant and the further stands having been taken by the parties, there was no arbitrable
dispute survived between the parties and requested the applicants to withdraw the
consultation notice.
5.5.2006 : A formal notice was issued for arbitration.
17.5.2006 : The non-applicant reiterated that there was no arbitrable dispute
survived.
19.5.2006 : The applicant conveyed the appointment of nominee arbitrator
under Clause 12.04(d).
1.6.2006 : Two formal Share Purchase Agreements came into existence. A
clause 2(D) was inserted : "2(D) In relation to the notices dated January 31, 2006 and
February 27, 2006 issued by TIL (as a founder under the Shareholders Agreement as
hereinafter defined) to the A.V. Birla Group, TIL has pending arbitration disputes with the
AV Birla Group, TIL has not accepted the rival contentions of the AV Birla Group in
respect of the said notices nor has the AV Birla Group accepted, TIL's claims pending
arbitration. As such, the execution of and consummation of the transaction contemplated
by this Agreement shall not prejudice or affect the pendency or continuation of the
arbitration proceeding between TIL and the AV Birla Group."
Another Clause 2.3 was inserted :
"2.3 TIL, for itself and Apex has issued the Termination Notices. The offer
made by TIL to the AV Birla Group pursuant to the Offer Notice, was made without
prejudice to the said Termination Notices, Grasim Industries Limited, as part of the AV
Birla Group has refuted the Termination Notices. Further, in order to enforce the rights
accrued to TIL and Apex on account of the Termination Notices, TIL, for itself and Apex
has pursuant to clause 12.04(d) of the Shareholders Agreement on May 5, 2006 issued a
notice to arbitrate to the AV Birla @page-SC2983 Group. Accordingly, this Agreement is
being executed without prejudice to the rival contentions of either party with reference to
the Termination Notices and the legal rights which have accrued to each party (including
Apex acting through TIL as founder) under the Shareholders Agreement."

25. A glance on these dates would clearly suggest that the first salvo was fired by the
applicant when it first sent the notice dated 31.1.2006. In this way, the only complaint
made was that ABTL, which was a subsidiary of ABNL, had made an application on
3.1.2006 to the Department of Telecommunication for grant of Unified Access Services
License (UASL) for Mumbai Metro circle. It was complained that this amounted to the
breach of Clause 3.04(b) of the Shareholders Agreement as the Aditya Birla Group could
not be permitted to engage in, directly or indirectly through an Affiliate, (i) any activity
that would constitute the Business of the IDEA Cellular Ltd. without complying with the
provisions of Clause 3.04(b). Noticee then was to remedy the breach within 60 days as
per the Shareholders Agreement. It is only to that extent that the dispute arose.
It would be remembered that there is a provision in the Shareholders Agreement, by
which even if the breach is committed by one parry, the other party could ask for
remedying the same within 60 days. This was not a termination of the Agreement.
However, the second salvo came to be fired on 27.2.2006, which was on account of the
breach of the Confidentiality clause. A position was taken that this breach was not such as
could be remedied and on that account, the applicants claimed all the shareholding of
Birla Group at the price, which would be 25% less than the fair market price of the
shares. In the subsequent correspondences, they also offered to appoint a firm of
Chartered Accountants for assessing the fair market value of the shares.
26. The non-applicant was not to be left behind in disputing this claim by its reply dated
1.3.2006. In this reply, it was suggested that the termination notice was only by way of a
counter action to the Birla Group's recent letters to the Department of
Telecommunications, Ministry of Communication and Information and Technology
(DoT), complaining against Tata Group's continuing breach of the relevant telecom
licenses. It was thereafter denied that the investor presentations made by ABNL, and the
further presentation dated 12.9.2005 posted on the website of ABNL could constitute a
breach on the part of Birla Group of the restrictions contained in Clause 8.01 of the
Shareholders Agreement. It was further reiterated that the said presentation could not be
deemed to be confidential under the Shareholders Agreement, and that information
conveyed in those presentations was not confidential information at all. A further stand
was taken that since ABNL was a publicly listed company, any consolidated accounts
prepared by ABNL had to be disclosed to the investors, which was consistent with its
disclosure obligations and a good corporate governance practice, and that the Tata Group
was fully aware that ABNL would be disclosing such information to the investors and the
analysts. It was further reiterated that since the information was received from IDEA
regarding consolidation and disclosure, such information was used for that purpose and
there was no impropriety or breach of Shareholders Agreement by these disclosures, and
that information was not at all confidential. An allegation was made that the Tata Group
was trying to sell its shares to the third party and that it could not do so unless the
obligations in favour of Birla Group, as contemplated in the Shareholders Agreement,
were not met.
27. In short, till this reply was given, the only defense that was raised by the non-
applicant was that there was no breach of the Confidentiality clause, as the information
itself was not confidential. There was probably an inkling to the non-applicant that the
Tata Group was interested in disposing of its shareholding in favour of some third party
and, therefore, it merely asserted its rights in case such sale to the third party of the
shareholding materializes. Therefore, at least up to this date, there was a live issue as to
whether there had been a breach of Confidentiality clause on the part of the non-
applicant.
28. However, that is not where the things stopped as merely a few weeks thereafter, the
applicants having received the offer from Global Communication Services, offered their
own shareholding to the non-applicant on 5.4.2006, reiterating therein, that this
@page-SC2984
offer was without prejudice to the dispute which has arisen between the parties. The offer
so made, was accepted by the non-applicant, however, with a caveat in the following
words :
"This is without prejudice to our contentions that the notices of termination referred to in
the said Offer Notice, are not tenable."
Therefore, even at this point of time, when the offer was accepted, the issue was very
much there as to whether there was a breach of Confidentiality clause on the part of the
non-applicant. On 7.4.2006, the applicants offered to send two Draft Share Purchase
Agreements for the sale and again reiterated that the offer and the acceptance by the non-
applicant was without prejudice to the notices of termination dated 31.1.2006 and
27.2.2006. On 10.4.2006, again the non-applicant wrote back giving the details regarding
the shares and the percentages, where again they disputed the question of termination.
There was a further reiteration on 19.4.2006, of its stand, in details, and it was
specifically reiterated in the letter dated 19.4.2006 :
"As already explained above, since the AV Birla Group is in violation of the provisions of
the Shareholders Agreement, the First Notice and the Second Notice issued by us cannot
be withdrawn. Since you have failed in agreeing upon the name of the international firm
of auditors we shall therefore proceed in accordance with the Shareholders Agreement to
do the needful in connection with determining the Fair Market Value of Idea Shares."
By their letter dated 24.4.2006, the applicants again showed their readiness to purchase
the shares. However, the earlier position was reiterated by them to the effect that the
stand taken by Tata Group was not correct. On 24.4.2006, the applicants clearly reiterated
that a dispute had arisen between the applicants and the non-applicant in connection with
the terms of Shareholders Agreement and that for that purpose, they were ready to start
the process of consolidation as an initial step. It was also conveyed that in case the non-
applicant was not interested in process of consolidation, the applicants would like to
proceed with the arbitral proceedings straight- way. Lastly, the non-applicant wrote a
letter dated 27.4.2006 in which for the first time, the position was taken that the
Agreement which was concluded between the parties consequent to the offer notice and
acceptance notice, and the further steps taken by both would be that there was no
arbitrable dispute surviving pursuant to the notices.
29

. It is here for the first time, that a stand was shifted by the non-applicant from its earlier
stand. Earlier, the contention was that there was no breach of Confidentiality clause. The
shifted stand was that because of the subsequent Agreement, the earlier issue was already
obliterated. It is on this background that ultimately the formal Agreements dated 1.6.2006
came to be entered into by the parties. But before that, a notice for the Arbitration was
already issued by the applicants by their letter dated 5.5.2006. Even on this backdrop, the
only position taken by the non-applicant by its letter dated 17.5.2006 was that Tata Group
was estopped from asserting its alleged claims against the Birla Group. In view of the
election made by Tata Group by giving the offer notice dated 5.4.2006 to the Birla Group
and the acceptance by Birla Group dated 6.4.2006. Therefore, it was concluded that there
was no arbitrable dispute surviving and yet in the Agreement dated 1.6.2006, the non-
applicant again allowed to insert the clauses regarding the Agreements being without
prejudice to the earlier rights. In fact, if the stand taken was that there was no arbitrable
issue because the offer concluded sale, there was no question of any such "without
prejudice" clause being inserted in the Agreement dated 1.6.2006. It must be remembered
that on the date when the formal Agreements were signed on 1.6.2006, the non-applicant
was already facing an arbitral notice and yet the two clauses, viz., 2(D) and 2.3 (which
we have mentioned earlier in this judgment), came to be inserted. All this would suggest
that there indeed was an issue and a live one in between the parties till then as described
in (See Chairman and MD, NTPC Ltd. vs. Reshmi Constructions, Builders and
Contractors, reported in (2004) 2 SCC 663 (Paras 35 and 36). 2004 AIR SCW 198

30

. On the other hand, Shri Venugopal, learned counsel invited my attention to a decision of
High Court of Australia in Sargent and ASL Developments Limited, reported at (1974) 4
Australian Law Reports (A.L.R.) 257 (cited supra). The decision of the Australian High
Court is on the question of Doctrine of election. In my opinion, the decision is not
applicable to the present 2005 AIR SCW 6305

@page-SC2985
controversy as there was no question of election on the part of the applicant herein. It is
merely basing its claim on account of the alleged clear breach of Shareholders Agreement
in respect of the Confidentiality clause. Even the second decision in Jai Narain
Parasrampuria (Dead) and others vs. Pushpa Devi Saraf and others, reported in (2006) 7
SCC 756 (cited supra) has no application. That is a case where this Court explained the
principles of estoppel and waiver and whether a party could be permitted to take a
different stand and the duty of the court in such matters. No such questions arise in the
present matter. The third decision in National Insurance Company Limited vs. Mastan
and another, reported in (2006) 2 SCC 641, is again on the question of Doctrine of
election, where this Court has observed that the Doctrine of election postulates that when
two remedies are available for the same relief, the aggrieved party has an option to elect
either of them, but not both. The fact-situation is not like that in the present case. This
case, therefore, has no application.
31

. The other major limb of the argument was, however, that this issue could not arise and
became a dead issue at least after the applicants sold out all their shares and their
shareholding fell below 15%. The aforementioned clause on which Shri Venugopal, Dr.
Singhvi and Shri Shyam Diwan, Advocates for the non-applicant heavily relied, being
Clause No. 7.01(a) and 7.01(b), as also 7.02 (b) provided that in case the shareholding
falls below 15%, and in this case it has actually fallen below 15%, such party would not
have any rights left with it under the Agreement (Shareholders Agreement). The argument
is obviously incorrect, as the Arbitration Agreement under Clause 12.04 would be clearly
autonomous of the Shareholders Agreement. Law is settled on this point that even if the
whole Agreement is terminated, the Arbitration Agreement would still remain (See
Chairman and MD, NTPC Ltd. vs. Reshmi Constructions, Builders and Contractors,
reported in (2004) 2 SCC 663 (Para 39). It was argued that Clause 7.01 (b) operates as a
complete bar on the exercise by the applicants of the substantive right to seek a buyout.
That may be so, however, that is only an eventuality subsequent to the crystallisation of
the live issue between the parties for which the arbitration clause would come handy to
the applicants. 2004 AIR SCW 198
32. The learned counsel for the non-applicant very vehemently argued that the issue had
become dead. The issue cannot be held to be dead for simple reason that even in the
subsequent Agreements (Shares Agreements), there is a "without prejudice" clause and
that too despite the vehement claims and refusals of those claims on the part of the
parties.
33. Whether there was a breach of Confidentiality clause and whether the applicants were
entitled to any damages on account of that clause in favour of the applicants, would be a
matter in the helm of arbitration and this Court would not go into that question.
34. It was tried to be argued that the cause of action for the arbitration based entirely on
the offer notice, which was given by the applicants to the non-applicant on account of the
applicants having received the offer from Global Communication Services. It is,
therefore, argued by the learned counsel that since the cause of action was on the offer
notice and since the Share Purchase Agreements were concluded on account of that offer
notice, any disputes arising would be solved in accordance with the Share Purchase
Agreement and not with the Shareholders Agreement and, therefore, they would be
resolved under English Law and LCIA Arbitration Rules. These clauses are to be found at
pages 286-288 as also on pages 328-332 of the main Paper Book vide Clauses 9.1, 9.4
and 9.5 in the First Agreement dated 31.1.2006, as also 9.2.1 to 9.2.5 on pages 330 and
331 of the main Paper Book.
35. The contention is clearly incorrect, as the present dispute is not based on the notice of
offer dated 5.4.2006. The contention raised by the learned counsel that the cause of action
arose only from that notice is obviously incorrect, as has been shown in the earlier parts
of the judgment. The live issue was clearly there, much before the notice was given on
05-04-2006 and the second agreement was even contemplated. That was the issue
whether there was a breach of Confidentiality clause on the part of the non-applicant and
what are the effects thereof. That issue continued to be in existence and was never given
up by the applicants. Therefore, that contention is rejected.
@page-SC2986
36. By way of last argument, it was argued that M/s. Apex Investments (Mauritius)
Holding Private Limited was never a party to the Shareholders Agreement. It was
suggested that M/s. Apex Investments (Mauritius) Holding Private Limited was originally
a member of the ATandT Wireless Group and held 32.9% shares in IDEA. It was then
known as ATandT Cellular Pvt. Ltd. It was then argued that it was required under Article
4.01(a) of the Shareholders Agreement to execute a Deed of Adherence as provided in
"Annex C" to the Shareholders Agreement in order to (i) designate ATandT Inc. as its
representative to exercise all rights and perform all obligations under the Shareholders
Agreement; (ii) agree to be bound by all the provisions of the Shareholders Agreement;
(iii) agree not to revoke the designation of the representative without prior written
consent of the AV Birla Group and the Tata Group. It was also required that the
counterpart of the executed Deed of Adherence should have been delivered to Grasim
Industries Ltd. and TIL. It was then pointed out that after TIL acquired ATandT Cellular
Pvt. Ltd. itself which held the remaining 16.45% shares in IDEA, M/s. Apex Investments
(Mauritius) Holding Private Limited became a wholly owned subsidiary of TIL. It was
then pointed out that on 28.09.2005, ATandT Cellular Pvt. Ltd., New Cingular Wireless
Service Inc. (successor-in-interest to ATandT Inc.) entered into a Sale and Purchase
Agreement with Indian Rayon and Industries Ltd. (now called ABNL), whereby the
Shareholders Agreement was specifically terminated as between the Birla Group and the
ATandT Wireless Group and all pre-existing rights/liabilities, if any, were specifically
extinguished and waived. So also on the same date, the ATandT Wireless Group (of
which ATandT Cellular Pvt. Ltd. was a part), executed a similar Sale and Purchase
Agreement with the Tata Group, whereby the Shareholders Agreement was terminated
between the Tata Group and the ATandT Wireless Group, and thus the Apex Investments
(Mauritius) Holding Private Limited (which is a fall out from the ATandT Cellular Pvt.
Ltd.) had terminated the Shareholders Agreement and extinguished and waived all
accrued rights and obligations against the Birla Group and, therefore, Apex Investments
(Mauritius) Holding Private Limited ceases to be a party to the Shareholders Agreement
and to any arbitration clause contained therein. Apex Investments (Mauritius) Holding
Private Limited, therefore, could no longer have any claim for arbitration or assert any
rights or liabilities under the Shareholders Agreement. It was further pointed out that on
15.12.2000, when the Shareholders Agreement was executed, there were only three
parties, viz., (i) ATandT Inc., which was acting on behalf of itself and ATandT Wireless
Group; (ii) Grasim, acting on behalf of itself and the Birla Group; (iii) TIL, acting on
behalf of itself and the Tata Group. The argument, therefore, is that since Apex
Investments (Mauritius) Holding Private Limited was not a party then, there could be no
privity of contract between Apex Investments (Mauritius) Holding Private Limited and
the non-applicant Birla Group. It was pointed out that since the modality drafted by
clause 4.02 of Shareholders Agreement was not followed, Apex Investments (Mauritius)
Holding Private Limited could not join the Shareholders Agreement and as such, Apex
Investments (Mauritius) Holding Private Limited not being a party to the Shareholders
Agreement, it cannot demand an arbitration through the arbitration clause. This argument
is obviously incorrect for the following reasons:
(i) It must be seen that after the offer was received from Global Communications, both
TIL on its behalf and on behalf of the Apex Investments (Mauritius) Holding Private
Limited, offered to sell their entire shareholding in IDEA in terms of the right of first
refusal, which offer has been accepted by the non-applicant.
(ii) Apex Investments (Mauritius) Holding Private Limited is now a part and parcel of the
Tata Group and is its subsidiary. Further, its interests are bound to be affected.
(iii) In raising this issue, the non-applicant is making a complete volte face, inasmuch as,
when the application had been filed before the Bombay High Court, an objection was
taken by the non-applicant that Apex Investments (Mauritius) Holding Private Limited
being a foreign company and claiming an arbitration along with Tata Group, the Bombay
High Court had no jurisdiction to entertain the application under Section 11(6). It was in
pursuance of that objection only that the Bombay High Court did not proceed further to
decide the application under Section 11(6). The learned counsel argues that this objection
regarding the Apex Investments (Mauritius) Holding Private Limited being foreign party,
arose
@page-SC2987
on the face of it, but the merits of the case did not fall for consideration in Bombay High
Court and as such the issue of Apex Investments (Mauritius) Holding Private Ltd. not
being a party to Shareholders Agreement can still be raised. The contention is not correct.
The non-applicant having raised an objection on the ground that the applicant Apex
Investments (Mauritius) Holding Private Limited was a foreign company, and, therefore,
could not have filed an application before Bombay High Court, cannot now turn around
and say that Apex Investments (Mauritius) Holding Private Limited was not a party to the
Arbitration Agreement. That will not be permissible. The learned counsel points out that
this objection was raised without prejudice, would also be of no consequence, as having
succeeded in stalling the decision of the application under Section 11(6), it cannot now
raise the argument before this Court that Apex Investments (Mauritius) Holding Private
Limited was never a party. This argument should have been addressed to the Bombay
High Court, at least in the alternative form. If in the affidavit before the Bombay High
Court filed on their behalf of the non-applicant had raised the issue and still chose not to
go into the issue whether Apex Investments (Mauritius) Holding Private Limited was or
was not a party to the Shareholders Agreement, that will not be permitted to be raised
before this Court. In fact, in restricting to the jurisdictional issue and in not perusing the
issue of Apex Investments (Mauritius) Holding Private Limited not being a party to the
Shareholders Agreement before Bombay High Court, the non-applicant abandoned that
issue. The argument is, therefore, rejected.
37. Ultimately, considering the overall situation, it is held that the application under
Section 11(6) is liable to be allowed. In that view, the following order is passed :
Hon'ble Dr. Justice A.S. Anand, former Chief Justice of India, Hon'ble Mr. Justice Arun
Kumar and Hon'ble Mr. Justice P. K. Balasubramanyan, former Judges of the Supreme
Court of India are appointed as the Arbitrators. Their terms shall be decided by
themselves.
Order accordingly. .
AIR 2008 SUPREME COURT 2987 "Jitendra Singh v. Bhanu Kumari"
(From : Rajasthan)*
Coram : 2 Dr. A. PASAYAT AND P. SATHASIVAM, JJ.
Civil Appeal No.2786 of 2008 (arising out of SLP (C) No. 4120 of 2007), D/- 11 -4
-2008.
Jitendra Singh v. Bhanu Kumari and Ors.
(A) Civil P.C. (5 of 1908), S.24 - TRANSFER OF CASE - Transfer of case -
Considerations - Fact that criminal case has been filed against party applying for transfer
- Not a ground to transfer case.
AIR 2007 (NOC) 577 (Raj), Reversed. (Paras 6, 8, 9)
(B) Civil P.C. (5 of 1908), S.24 - TRANSFER OF CASE - Transfer of case - Power
conferred on Court is discretionary - When transfer is sought by a party - Notice has to be
given to other party before ordering transfer. (Para 9)

Kailash Vasudev, Sr. Advocate, Ms. Reshma Rea Sinha, and Parijat Sinha, for Appellant;
Shyam Divan, N. Sridharan, Sr. Advocate, Hemant Sharma, Sunil Nath, Ajay Kumar, Ms.
Richa Srivastava, Ms. Indu Sharma, S. S. Rana, Mrs. Bindra Rana (for M/s. S. S. Rana
and Co.), K. Sunil, Pavan Kumar and L.R. Singh, for Respondents.
* S.B. Civil Transfer Petn. No. 41 of 2006, D/- 27-11-2006, reported in AIR 2007 (NOC)
577 : 2007 (2) Raj LW 1077.
Judgement
1. Dr. ARIJIT PASAYAT, J. :-Leave granted.
2. Challenge in this appeal is to the order passed by a learned Single Judge of the
Rajasthan High Court, Jaipur Bench. Respondent No. 1 had filed application in terms of
Section 24 of the Code of Civil Procedure, 1908 (in short the 'CPC') seeking transfer of
Civil Case No. 41/202/05 titled Jitendra Singh v. Smt. Bhanu Kumari and Ors. pending
before the District and Sessions Judge, Alwar, Rajasthan to some other Court. By the
impugned order the High Court directed that the suit in question to be transferred from
the Court of Additional District Judge No. 2 Alwar to the Court of District Judge, Jaipur
City.
3. Learned counsel for the appellant submitted that this is yet another attempt to deny the
appellant of his legitimate entitlement. It is pointed out that in an earlier petition
@page-SC2988
(Transfer Petition (C) No. 1105 of 2005 titled Maharaja Sewai Tej Singh v. Jitender Singh
and Ors.) this court declined to accept the prayer for transfer. But keeping in view the age
and the state of health of the petitioner in that case, directed appointment of a guardian ad
litem. It is pointed out that in fact as has been noted by the High Court, a learned counsel
has been appointed as guardian ad litem. It is the stand of the appellant that the reason
which weighed with the High Court to direct transfer is really of no consequence, more
particularly in view of what has been stated by this Court in the earlier Transfer Petition.
4. Learned counsel for the respondent on the other hand submitted that taking into
consideration the serious nature of the allegations made the High Court has rightly
directed transfer.
5. The High Court's conclusions based on the basis of which order of transfer has been
made, read as follows :
"Having scanned the record, I noticed that the suit was transferred by the District Judge
to the Court of Additional District Judge No. 2, Alwar in the month of December, 2005.
The petitioner although filed instant petition in this Court on August 17, 2006, did not
mention this fact that the suit stood transferred from the Court of District Judge, Alwar.
The Petitioner stated in para 3 of the petition that the respondent-Jitendra Pratap Singh is
influential person and MLA of Alwar City and he has created such a situation that there is
a strong likelihood of the matter pending before the District Judge being decided ex parte
against the petitioner and other members. This apprehension of petitioner, in my opinion,
is baseless. The Courts are not influenced by politicians and influential persons. The
petitioner should repose full confidence upon the court of justice. If ex-parte order was
passed by the learned District Judge and the petitioner was aggrieved by it, she ought to
have assailed it legally. Passing of ex parte order by the Presiding Officer of the court
cannot be a reasonable ground for transferring the case.
But looking to the fact that on January 8, 2006 respondent-Jitendra Singh lodged FIR No.
19 of 2006 with the Police Station Kotwali, Alwar against the petitioner and respondents-
Amar Raj Pal and Jaswant Singh and case under Sections 420, 467, 468 and 471, IPC has
been registered against them and considering the overall view of the nature of the case
and convenience of the parties and in the interest of justice, it would be just and
reasonable to direct transfer of suit from the court of Additional District Judge No. 2
Alwar to the Court of District Judge, Jaipur City."
6. It appears that the High Court referred to the fact that the criminal proceedings have
been instituted against the appellant. It is pointed out by learned counsel for the appellant
that the filing of the FIR is really of no consequence. In respect of the plaint averments,
in his reply before the High Court to the transfer petition it was inter alia stated as follows
:
"That the contents of sub-para (xviii) are not admitted in the manner stated. The
answering respondent lodged the FIR (Annexure-9) on the facts constituting the offences
under different Sections of the Penal Code committed by the accused persons named
therein. It is denied that the respondent No. 1 filed the FIR to exert pressure on the
petitioner and to pressurise the lawyer seeking to represent the petitioner. The allegations
are baseless and wholly without substance. It is humbly submitted that the respondent
No. 2 acted hand-in gloves with the petitioner -Smt. Bhanu Kumari and her brother
Yashwant Singh, got a purported power of attorney prepared in his favour and acting
upon that the respondent No. 2 negotiated for sale of the property of respondent No. 4
Maharaja Sawai Tej Singh Ji for a consideration of Rs.77,30,328/-. The agreement
entered into on 18.4.2005 besides being void was for inadequate consideration. The
respondent No. 2 as attorney received the part payment of the consideration in cash and
by cheque in his own name. The petitioner and the respondent Nos. 2 and 3 to make
wrongful gain colluded and connived with each other to make the respondent No. 4 sign
documents or papers who was a person under incapacity, incapable of understanding the
contents to which he was made to sign and/or that his signature was forged. The
respondent No. 2 Shri Amar Raj Lall, Advocate acted against professional ethics and
involved himself in criminal conspiracy. True and correct copy of the power of attorney,
agreement for sale dated 18.4.2005 and the receipt dated 25.4.2005 are enclosed herewith
and marked as Annexure R-1/4, R-1/5 and R-1/6 respectively. A true and correct copy of
the affidavit of respondent
@page-SC2989
No.2 Amar Raj Lall, filed in transfer petition before the Hon'ble Apex Court is filed
herewith and marked as Annexure R-1/7."
7. Though grievance is made by the respondents that no competent lawyer at Alwar is
willing to represent them, it is of significance to note that the suit has been filed by two
persons. Respondents 2 and 3 are represented by experienced lawyers and they have been
representing the respondents for very long period. The parameters for exercise in Sections
24 and 25 have been laid down by this Court in several cases. Earlier than Transfer
Petition (Civil) No.1105 of 2005 was disposed of with certain directions.
8. The reasons which weighed with the High Court to direct transfer do not appear to be
germane warranting an order of transfer.
9. The purpose of Section 24, CPC is merely to confer on the Court a discretionary
power. A court acting under Section 24, CPC may or may not in its judicial discretion
transfer a particular case. Section 24 does not prescribe any ground for ordering the
transfer of a case. In certain cases it may be ordered suo motu and it may be done for
administrative reasons. But when an application for transfer is made by a party, the court
is required to issue notice to the other side and hear the party before directing transfer. To
put it differently, the Court must act judicially in ordering a transfer on the application of
a party. In the instant case the reason which has weighed with the High Court for
directing transfer does not really make out a case for transfer.
10. Accordingly the impugned order of the High Court is set aside.
11. Appeal is allowed.
Appeal allowed. .
AIR 2008 SUPREME COURT 2989 "Bilkish v. United India Insurance Co. Ltd."
(From : 2000 AIR - Kant HCR 13)
Coram : 2 A. K. MATHUR AND ALTAMAS KABIR, JJ.
Civil Appeal No.6313 of 2001, D/- 12 -3 -2008.
Bilkish v. United India Insurance Co. Ltd. and Anr.
Motor Vehicles Act (59 of 1988), S.168 - MOTOR VEHICLES - Accident -
Compensation - Deceased was a bachelor - Carrying on business - Was income-tax
assessee - Determining loss of dependency by 50% was not correct - Loss of dependency
could be assessed to 2/3 of his total income - Compensation granted by applying
multiplier of 12 instead of 11. (Para 4)

R.S. Hegde, Chandra Prakash, J.K. Nayyar and P.P. Singh, for Appellant; Sudhir Kumar
Gupta and Mihir Kumar Chaudhary, for Respondents.
Judgement
A. K. MATHUR, J. :- This appeal is directed against the order dated 11.7.2000 passed by
the Division Bench of the Karnataka High Court whereby the Division Bench has ordered
the compensation in sum of Rs. 1,65,000/- towards dependency and Rs. 10,000/- for the
loss of estate and funeral expenses. Aggrieved against this order, the present appeal was
preferred by the appellant for suitable enhancement of the compensation. Brief facts
which are necessary for the disposal of this appeal are as under :
2. That one Hazi Mohammed Haneef died in a motor accident on 30.3.1993 when a
tempo bearing No. CAA 6591 dashed against the motorcycle (KA-01-H-7054) which he
was riding. He ultimately succumbed to injuries and died. Therefore, the claimants (the
parents of the deceased) filed MVC No. 1039/1993 claiming compensation of
Rs.15,12,000/- under the various heads. The Tribunal allowed the claim petition in part
by judgment and award dated 23.9.1996 and held that accident took place due to
negligent driving of the tempo bearing No. CAA 6591 and held that claimants were
entitled to compensation of Rs. 1,75,000/- with interest at 6% p.a. from the date of
petition to the date of realization. The compensation amount awarded in sum of Rs.
1,65,000/- towards loss of dependency and Rs. 10,000/- towards loss of estate and funeral
expenses.
3. Learned counsel for the appellant submitted that the deceased was 20 years of age and
was a bachelor. His parents were aged 47 years and 42 years respectively. The deceased
was studying in First Year B.Com. course and he was also the proprietor of a business
carried under the name and style of H.S. Traders and was an income-tax assessee. The
deceased had an income of Rs. 31,494/- in his business and had paid the
@page-SC2990
income-tax on that. The Tribunal had erroneously deducted 50% towards his personal,
living expenses and the contribution to the family/dependency worked out to Rs. 15,000/-
per annum. The Tribunal applied multiplier of 11, looking to the age of the parents and
arrived at the total loss of dependency at Rs. 1,65,000/-. Learned counsel submitted that
the assessment made by the Tribunal and affirmed by the High Court was totally
erroneous. The incumbent was a bachelor, therefore, he could not spend 50% of his
income on himself. But three-fourth of the income was contributed to the family and,
therefore, the dependency assessed by the Tribunal and by the High Court for a sum of
Rs. 15,000/- was not correct. It was also submitted that the multiplier of 11 applied by the
Tribunal was also not correct.
4. After hearing learned counsel for the parties, we are of the opinion that the view taken
by the High Court and Tribunal is not correct. The incumbent was a bachelor and he
could not have spent more than 1/3rd of his total income for personal use and rest of the
amount earned by him would certainly go to the family kitty. Therefore, determining the
loss of dependency by 50% was not correct. Therefore, we assess that he must be
spending 1/3rd towards personal use and contributing 2/3rd of his income to his family.
Therefore, we work out that Rs. 30,000/-earned by him per annum. The loss of
dependency was 2/3rd i.e. Rs. 20,000/-. The multiplier of '11' applied for loss of
dependency was also not correct and as per schedule appended to the Motor Vehicles Act,
1988 it should be '12'. Applying the multiplier of 12 the total loss of dependency will be
Rs. 20,000/-x 12 = Rs. 2,40,000/- and Rs. 10,000/- towards loss of estate and funeral
expenses, the total compensation comes to Rs. 2,50,000/- and incumbent is entitled for
interest @ 9/% per annum from the date of the petition. The appeal is allowed with the
aforesaid modification. If any amount had already been paid to the claimant then that
amount may be deducted from the total amount. Consequently, the appeal is allowed in
part with no order as to costs.
Order accordingly. .
AIR 2008 SUPREME COURT 2990 "Hakam Singh v. State of Haryana"
(From : Punjab and Haryana)
Coram : 2 TARUN CHATTERJEE AND HARJIT SINGH BEDI, JJ.
Civil Appeal Nos. 2699 with 2700 and 2701 of 2008 (arising out of SLP (C) Nos. 6347,
6427 and 6348 of 2007), D/- 9 -4 -2008.
Hakam Singh and Anr. v. State of Haryana and Ors.
WITH
Mohinder Kaur (D) by L.Rs. and Ors. v. State of Haryana and Ors.
AND
Nasib Singh and Ors. v. State of Haryana and Ors.
Civil P.C. (5 of 1908), S.96, O.41, R.27 - APPEAL - EVIDENCE - Appeal - Application
made for acceptance of additional evidence - Court dismissing appeal without
considering application - Order liable to be set aside - Matter remitted.
R. F. A. No.22 of 1989, D/-26-07-2006 (P and H), Reversed. (Paras 4, 5)

G.S. Punia, Davender Mohan Verma and Ms. Minakshi Vij, for Appellants.
Judgement
1. JUDGMENT :- Leave granted.
2. In spite of due service and opportunity to contest the appeal, no one has entered
appearance in the appeal or contested the appeal at the time of hearing.
3. These appeals, by way of Special Leave, are filed against a final judgment and order
dated 26th of July 2006 passed in Regular First Appeal Nos. 22, 185 and 217 of 1989 by
the High Court of Punjab and Haryana at Chandigarh, by which the First Appeals, filed at
the instance of the appellant, were dismissed which were filed for enhancement of
compensation for the land acquired by the respondents.
4. Without going into the facts in detail, these appeals can be disposed of on a very short
point. It is an admitted position that an application under Order 41 Rule 27 of the Code of
Civil Procedure (In short "CODE") for acceptance of additional evidence was filed before
the High Court in the aforesaid First Appeals, which were dismissed by the High Court
by the impugned order. However, the application for acceptance of additional evidence
under Order 41 Rule 27 of the CODE was not considered by the High Court
@page-SC2991
while disposing of the appeal.
5. That being the position, without going into the legality and propriety of the impugned
order of the High Court passed in the aforesaid appeals, we set aside the same and remit
back the cases to the High Court for decision of the Appeals afresh on merits and in
accordance with law along with the application for acceptance of additional evidence
under Order 41 Rule 27 of the CODE.
6. Accordingly, the impugned order is set aside. The High Court is requested to dispose of
the First Appeals in the light of the observations and directions made hereinabove within
three months from the date of supply of a copy of this Order along with application for
acceptance of additional evidence under Order 41 Rule 27 of the CODE.
7. It is expected that the parties shall not be entitled to ask for adjournments on
unsatisfactory grounds.
8. For the reasons aforesaid, the appeals are allowed to the extent indicated above. There
will be no order as to costs.
Order accordingly. .
AIR 2008 SUPREME COURT 2991 "Yogesh v. State of Maharashtra"
(From : Bombay)
Coram : 2 S. B. SINHA AND D. K. JAIN, JJ.
Criminal Appeal Nos. 744 with 745 of 2008 (arising out of SLP (Cri.) Nos. 5514 with
5515 of 2008), D/- 28 -4 -2008.
Yogesh @ Sachin Jagdish Joshi v. State of Maharashtra.
(A) Criminal P.C. (2 of 1974), S.227 - DISCHARGE - Discharge of accused - Words "not
sufficient ground for proceeding against the accused" in S.227 postulate exercise of
judicial mind by Judge - At this stage, he is not required to see as to whether trial will end
in conviction or not.
AIR 1979 SC 366; AIR 1977 SC 2018, Rel. on. (Para 15)
(B) Penal Code (45 of 1860), S.120A - CRIMINAL CONSPIRACY - Criminal
conspiracy - Meeting of minds of two or more persons for doing illegal act - Is sine qua
non - But it may not be possible to prove agreement between them by direct proof.
AIR 1980 SC 439; AIR 1981 SC 1062; AIR 1988 SC 1883; 1996 AIR SCW 1977; 2005
AIR SCW 4148; 1999 AIR SCW 1889; 1997 AIR SCW 2055, Relied on. (Para 23)
(C) Penal Code (45 of 1860), S.120A - CRIMINAL CONSPIRACY - Conspiracy and its
objective - Can be inferred from surrounding circumstances and conduct of accused.
(Para 23)
(D) Penal Code (45 of 1860), S.120A - CRIMINAL CONSPIRACY - AGREEMENT -
Criminal conspiracy - It is a substantive offence - Renders mere agreement to commit
offence punishable even if it does not take place pursuant to illegal agreement. (Para
23)
(B) Criminal P.C. (2 of 1974), S.227 - Penal Code (45 of 1860), S.120A, S.300 -
CRIMINAL CONSPIRACY - Discharge of accused - Conspiracy and murder - Deceased
had love affair with appellant's sister - Appellant and his family members allegedly
hatched conspiracy to murder deceased when they learnt about his marriage with some
other girl - Allegation based on circumstantial evidence, namely, threatening calls from
side of appellant to complainant - Discharge of mother, sister of appellant alongwith other
co-accused - Father of appellant who was stated to be mastermind behind entire
conspiracy, also discharged - No sufficient ground to proceed against appellant - He is
entitled to be discharged.
Crl. Revn. Appln. No.288 of 2005, D/-23-06-2006 (Bom.), Reversed. (Paras 25, 26,
27)
Cases Referred : Chronological Paras
2005 AIR SCW 4148 : AIR 2005 SC 3820 : 2005 Cri LJ 3950 (Rel. on) 22
1999 AIR SCW 1889 : AIR 1999 SC 2640 : 1999 Cri LJ 3124 (Rel. on) 22
1997 AIR SCW 2055 : AIR 1997 SC 2193 : 1997 Cri LJ 2535 (Rel. on) 22
1996 AIR SCW 1977 : AIR 1996 SC 1744 : 1996 Cri LJ 2448 (Rel. on) 21
AIR 1988 SC 1883 : 1989 Cri LJ 1 (Rel. on) 20
AIR 1981 SC 1062 : 1981 Cri LJ 588 (Rel. on) 19
AIR 1980 SC 439 : 1980 Cri LJ 388 (Rel. on) 18
AIR 1979 SC 366 : 1979 Cri LJ 154 (Rel. on) 11, 15
AIR 1977 SC 2018 : 1977 Cri LJ 1606 (Rel. on) 15
Ravi Shankar Prasad, Sr. Advocate, Ayyam Perumal, with him for Appellant; Ravindra
Keshavrao, Adsure, for Respondent.
@page-SC2992

Judgement
D. K. JAIN, J. :- Leave granted.
2. These two appeals are directed against orders dated 23rd June, 2006 and 19th
September, 2006 passed by the High Court of Judicature at Bombay in Criminal Revision
Application No. 288 of 2005 and in Criminal Writ Petition No.1884 of 2006 respectively.
By the first order, the High Court has repelled the challenge made to order dated 23rd
March, 2005, in Criminal Appeal No.83 of 2004, whereby the Sessions Judge, Satara had
affirmed the order passed by the Juvenile Justice Board, rejecting the application filed by
the appellant under Section 227 of the Code of Criminal Procedure, 1973 (for short "the
Code") for discharge. By the latter order, another Single Judge of the High Court has
dismissed the writ petition preferred by the appellant, seeking quashing of order dated
23rd March, 2005.
3. The facts giving rise to the present appeals lie in a narrow compass and centre around a
criminal conspiracy, allegedly hatched by the family members of the appellant to murder
the deceased, Kunal. The case of the prosecution as per the charge-sheet is that in the
month of March 1999, deceased Kunal organised an entertainment show, sponsored by
the father of the appellant. During the event, the deceased was introduced to the sister of
the appellant, Hema. The acquaintance blossomed into love between the two. Fearing that
there may be opposition to their close relationship from their family members, they
eloped and got married on 29th May, 2000. Appellant's father lodged a complaint alleging
that the deceased had kidnapped his daughter. The complainant, namely, the father of the
deceased, also reported the matter to the Pune Police.
4. Sometime in June, 2000, Kunal contacted his father and informed him that he was at
Gauhati with Hema. Thereupon, the complainant and his wife brought Kunal and Hema
to Mumbai. On persuasion by the complainant and his wife, Hema agreed to return and
stay with her parents and the marriage between Kunal and Hema is stated to have been
annulled. In December, the complainant filed complaints with the S.P., Satara and the
Additional Commissioner of Police, Pune against the father of the appellant alleging that
he had abused him over the telephone. The complainant also alleged that even thereafter,
threatening calls were made by the appellant, his sister, Hema and father. However, the
things seem to have settled down with the intervention of the Advocate of the
complainant.
5. Thereafter, Kunal was engaged to another girl and the wedding was scheduled for 30th
November, 2001. On 21st April, 2001, the deceased (Kunal) left Panchgani (where he
was living with the parents) for Mahabaleshwar in his maruti car. At about 8:30 p.m., a
taxi-driver informed the complainant that Kunal was lying in a pool of blood on
Mahabaleshwar road. The complainant rushed to the spot and took his son in an injured
condition to the hospital where he was declared brought dead. The complainant lodged an
F.I.R at Mahabaleshwar Police Station against unknown persons and an offence was
registered under Section 302 of the IPC. However, on the next day, the complainant
levelled allegation that since marriage of Kunal had been fixed with another girl, the
appellant and his family members had developed a grudge and had, therefore, hatched a
conspiracy with co-accused Umesh, Suresh, Bhavarlal Sharma, Captain Sharma to
murder Kunal.
6. On completion of investigation, charge-sheet was filed against the appellant before the
Juvenile Court, Satara, being below 18 years of age, and against fifteen other persons,
which included his father (A-1), mother (A-2), sister (A-4), a family friend (A-11),
manager of his father (A-12), in Sessions Court, Satara. All of them have been arraigned
as members to the conspiracy to murder Kunal. The appellant, herein, and accused A-1,
A-2, A-4 to A-7, A-11 and A-12 have been prosecuted for offences under Sections 302
and 120B of the Indian Penal Code, 1860 (for short "the I.P.C."), whereas accused A-7 to
A-10 and A-13 to A-16 have been prosecuted for offences under Section 302 read with
Section 120B, IPC and under the Arms Act.
7. The appellant filed an application for discharge before the Juvenile Justice Board,
under Section 227 of the Code. The Juvenile Justice Board by order dated 1st October,
2004, rejected the said application. Being aggrieved by the said order, the appellant
preferred a criminal appeal before the Sessions Judge, Satara. As noted above, the said
appeal was dismissed vide order dated 23rd March, 2005. Still aggrieved, the appellant
moved a revision application before the High Court. Vide order dated 23rd June, 2006,
the High Court dismissed the criminal revision.
@page-SC2993
8. It appears that the mother (A-2), sister (A-4), a family friend (A-11) and the manager
of appellant's father (A-12) had also moved applications under Section 227 of the Code
before the Sessions Judge, Satara for discharge, which were dismissed by virtue of orders
dated 16th January, 2006 and 7th October, 2003. Both these orders were challenged by
them by means of two Criminal Writ Petitions (Nos. 1283 and 1284 of 2006). Both the
petitions were allowed by the then Chief Justice of the High Court vide a common order
dated 7th July, 2006. Inter-alia, observing that the circumstances highlighted by the
prosecution, even if accepted in entirety, only created a suspicion of motive, these were
not sufficient to make out a case for conviction of the accused and some suspicion or
motive cannot serve as a sufficient ground for framing of charge against them.
Accordingly, all the four accused/petitioners were discharged.
9. Emboldened by the said order, on 4th August, 2006, the appellant filed a Criminal Writ
Petition (No. 1884 of 2006) under Article 227 of the Constitution read with Section 482
of the Code for quashing of aforenoted order dated 23rd March, 2005, passed by the
Sessions Judge, Satara and for discharge of the charges framed under Section 302 read
with Section 120B of the I.P.C. Expressing surprise over the fact that this petition had
been filed though the order impugned in the petition stood confirmed on dismissal of
criminal revision on 23rd June, 2006, the learned Judge rejected the plea of the appellant
that in the light of order dated 7th July, 2006, in the case of co-accused, by reason of
parity, he was also entitled to be discharged. Inter-alia, observing that earlier order dated
23rd June, 2006 in the case of the appellant, which was certainly relevant for deciding the
Criminal Writ Petitions No. 1283 and 1284 of 2006, had not been noticed in order dated
7th July, 2006 (by the Chief Justice), vide order dated 19th September, 2006, the learned
Judge dismissed the petition. As noted above, both the orders, dated 23rd June, 2006 and
19th September, 2006 are challenged in these two appeals.
10. At this juncture, two other significant subsequent developments deserve to be noted.
In the first place, a Special Leave Petition, preferred by the State against order of the
High Court dated 7th July, 2006, discharging accused Nos. 2, 4, 11 and 12, was dismissed
on 30th April, 2007. Secondly, relying on order dated 7th July, 2006, the father (A-1) of
the appellant, termed as the main accused in the charge-sheet, filed an application before
the Sessions Judge for discharge from all the charges. Taking note of the said order
passed by the High Court, and inter-alia, observing that apart from the fact that the
alleged threats are vague and are inadequate to connect the said accused with the crime,
vide order dated 14th May, 2007, the Sessions Judge came to the conclusion that there
was absolutely no material on the basis whereof a reasonable likelihood of the said
accused being convicted could be predicted. Accordingly, he has discharged the said
accused. Thus, as on date, the father (A-1), the mother (A-2), the sister (A-4) of the
appellant and his two other associates (A-11 and A-12) stand discharged of the offences
for which they were charged, namely, Sections 302 and 120B of I.P.C.
11

. Mr. Ravi Shanker Prasad, learned senior counsel appearing for the appellant, submitted
that in the charge-sheet there is no overt act attributed to the appellant regarding actual
murder and the threats allegedly given by his family members and friends to the family of
the deceased are not sufficient to infer a criminal conspiracy, particularly when, the
disputes between the two families had already been compromised much prior to the
incident. Learned counsel submitted that accepting the prosecution case as it is, there is
absolutely no material on record to frame a charge against the appellant for offences
under Sections 302 and Section 120B of the I.P.C. It was also urged that other members
of the family, namely, the mother and sister of the appellant along with two other accused
having been discharged by the High Court and similarly the father of the appellant having
been discharged by the Sessions Judge, there was no sufficient ground to proceed against
the appellant for the said offences. In support of the proposition that a mere suspicion is
not sufficient to hold that there is sufficient ground to proceed against the accused,
learned counsel placed reliance on the decision of this Court in Union of India vs.
Prafulla Kumar Samal and Anr.1 AIR 1979 SC 366
1 (1979) 3 SCC 4.
12. Mr. Ravindra Keshavrao Adsure, learned counsel appearing for the State, on the other
hand, submitted that there is
@page-SC2994
ample material on record to show a strong motive for commission of crime, namely,
Kunal's proposal to get married to another girl after an affair with Hema (A-4), which
was obviously, not liked by the appellant and his family members. It was thus, pleaded
that the High Court was justified in dismissing appellant's petitions for discharge.
13. Before adverting to the rival submissions, we may briefly notice the scope and ambit
of powers of the Trial Judge under Section 227 of the Code.
14. Chapter XVIII of the Code lays down the procedure for trial before the Court of
Sessions, pursuant to an order of commitment under Section 209 of the Code. Section
227 contemplates the circumstances whereunder there could be a discharge of an accused
at a stage anterior in point of time to framing of charge under Section 228. It provides
that upon consideration of the record of the case, the documents submitted with the police
report and after hearing the accused and the prosecution, the Court is expected, nay
bound to decide whether there is "sufficient ground" to proceed against the accused and
as a consequence thereof either discharge the accused or proceed to frame charge against
him.
15

. It is trite that the words "not sufficient ground for proceeding against the accused"
appearing in the Section postulate exercise of judicial mind on the part of the Judge to the
facts of the case in order to determine whether a case for trial has been made out by the
prosecution. However, in assessing this fact, the Judge has the power to sift and weigh
the material for the limited purpose of finding out whether or not a prima facie case
against the accused has been made out. The test to determine a prima facie case depends
upon the facts of each case and in this regard it is neither feasible nor desirable to lay
down a rule of universal application. By and large, however, if two views are equally
possible and the Judge is satisfied that the evidence produced before him gives rise to
suspicion only as distinguished from grave suspicion, he will be fully within his right to
discharge the accused. At this stage, he is not to see as to whether the trial will end in
conviction or not. The broad test to be applied is whether the materials on record, if
unrebutted, makes a conviction reasonably possible. [See: State of Bihar vs. Ramesh
Singh2and Prafulla Kumar Samal (supra)] AIR 1997 SC 2018
AIR 1979 SC 366

2 (1977) 4 SCC 39.


16. In the light of the aforenoted principles, we may now consider whether or not in the
present case the High Court was justified in declining to discharge the appellant.
However, before adverting to the circumstances, relied upon by the prosecution in
support of its primary charge that a conspiracy had been hatched to eliminate Kunal, the
essential features of the offence of conspiracy need to be noticed.
17. Section 120A of I.P.C. defines criminal conspiracy. The section reads as under :
"120A. Definition of criminal conspiracy. - When two or more persons agree to do, or
cause to be done, -
(1) an illegal act, or
(2) an act which is not illegal by illegal means, such an agreement is designated a
criminal conspiracy :
Provided that no agreement except an agreement to commit an offence shall amount to a
criminal conspiracy unless some act besides the agreement is done by one or more parties
to such agreement in pursuance thereof.
Explanation. It is immaterial whether the illegal act is the ultimate object of such
agreement, or is merely incidental to that object."
Section 120B of I.P.C. provides for punishment for an offence of criminal conspiracy.
18

. The basic ingredients of the offence of criminal conspiracy are: (i) an agreement
between two or more persons; (ii) the agreement must relate to doing or causing to be
done either (a) an illegal act; or (b) an act which is not illegal in itself but is done by
illegal means. It is, therefore, plain that meeting of minds of two or more persons for
doing or causing to be done an illegal act or an act by illegal means is sine qua non of
criminal conspiracy. Yet, as observed by this Court in Shivnarayan Laxminarayan Joshi
and Ors. vs. State of Maharashtra3, a conspiracy is always hatched in secrecy and it is
impossible to adduce direct evidence of the common intention of the conspirators.
Therefore, the meeting of minds of the conspirators can be inferred from the
circumstances proved by the prosecution, if such inference is possible. AIR 1980 SC
439

3 (1989) 2 SCC 465.


19

. In Mohammad Usman Mohammad Hussain Maniyar and Ors. vs. State of


Maharashtra4, it AIR 1981 SC 1062

@page-SC2995
was observed that for an offence under Section 120B, the prosecution need not
necessarily prove that the perpetrators expressly agree to do and/or cause to be done the
illegal act, the agreement may be proved by necessary implication.
4 (1981) 2 SCC 443.
20

. In Kehar Singh and Ors. vs. State (Delhi Administration)5, the gist of the offence of the
conspiracy has been brought out succinctly in the following words : "The gist of the
offence of conspiracy then lies, not in doing the act, or effecting the purpose for which
the conspiracy is formed, nor in attempting to do them, nor in inciting others to do them,
but in the forming of the scheme or agreement between the parties. Agreement is
essential. Mere knowledge, or even discussion, of the plan is not, per se, enough." AIR
1988 SC 1883, Para 268

5 (1988) 3 SCC 609.


21
. Again in State of Maharashtra and Ors. vs. Som Nath Thapa and Ors.6, a three-Judge
Bench of this Court held that to establish a charge of conspiracy knowledge about
indulgence in either an illegal act or a legal act by illegal means is necessary. In some
cases, intent of unlawful use being made of the goods or services in question may be
inferred from the knowledge itself. This apart, the prosecution has not to establish that a
particular unlawful use was intended, so long as the goods or service in question could
not be put to any lawful use. 1996 AIR SCW 1977

6 (1996) 4 SCC 659.


22

. More recently, in State (NCT of Delhi) vs. Navjot Sandhu @ Afsan Guru7, making
exhaustive reference to several decisions on the point, including in State through
Superintendent of Police, CBI/SIT vs. Nalini and Ors.8, Venkatarama Reddi, J. observed
thus : 2005 AIR SCW 4148, Para 12
1999 AIR SCW 1889

7 (2005) 11 SCC 600.


8 (1999) 5 SCC 253.

"Mostly, the conspiracies are proved by the circumstantial evidence, as the conspiracy is
seldom an open affair. Usually both the existence of the conspiracy and its objects have to
be inferred from the circumstances and the conduct of the accused (per Wadhwa, J. in
Nalini's case at page 516). The well known rule governing circumstantial evidence is that
each and every incriminating circumstance must be clearly established by reliable
evidence and "the circumstances proved must form a chain of events from which the only
irresistible conclusion about the guilt of the accused can be safely drawn and no other
hypothesis against the guilt is possible." (Tanviben Pankajkumar case9, SCC page 185,
para 45). G.N. Ray, J. in Tanibeert Pankajkumar observed that this Court should not allow
the suspicion to take the place of legal proof." 1997 AIR SCW 2055

9 (1997) 7 SCC 156.


23. Thus, it is manifest that the meeting of minds of two or more persons for doing an
illegal act or an act by illegal means is sine qua non of the criminal conspiracy but it may
not be possible to prove the agreement between them by direct proof. Nevertheless,
existence of the conspiracy and its objective can be inferred from the surrounding
circumstances and the conduct of the accused. But the incriminating circumstances must
form a chain of events from which a conclusion about the guilt of the accused could be
drawn. It is well settled that an offence of conspiracy is a substantive offence and renders
the mere agreement to commit an offence punishable even if an offence does not take
place pursuant to the illegal agreement.
24. Bearing in mind the essential features of the offence of criminal conspiracy,
enumerated above, we may advert to the facts of the instant case. The relevant portion of
the charge-sheet filed against all the accused reads as follows :
"Though son of the complainant Kunal Parihar got married with accused No.4 Hema
Joshi, his family again arranged for another marriage with one Meenal of Baroda on 24-
2-2001. Engagement ceremony took place and date of marriage was fixed as 30-11-2001.
This fact came to the knowledge of the accused Nos. 1, 2, 3, 4 and 5 and 11 and 12.
Therefore, in order to teach a lesson to the complainant they hatch conspiracy to kill his
only son, Kunal. Accordingly accused No. 1 contacted accused No. 6 Suresh Jhajara and
further informed him the complainant and his son should be taught a lesson as Kunal
Parihar betrayed him. Hence should be taught a lesson and further asked to carry out
future plan.Accused No. 6, contacted accused No. 7 and included him in the aforesaid
conspiracy. Accused Nos. l to 4, contacted accused No. 16, through accused Nos. 6 and 7,
@page-SC2996
accused No. 16 pending is a notorious criminal. Criminal cases are pending against him
in the District Court of Pune. In the offence regarding body, accused No. 7 contacted him
through witnesses Atul Lohar in order to carry out the aforesaid plan. Accused No. 1 gave
Rs. 80,000/- to accused No. 7 via accused No. 6. Accused No. 16, in order to cause hurt
to Kunal introduced accused Nos. 8, 9, 10, 13, 14, 15 to accused No. 7. Accused No. 7
asked accused Nos. 8, 9, 10, 13, 14, 15 to join the aforesaid conspiracy and in order to
carry out the aforesaid conspiracy successfully accused No. 7 purchased one Maruti car
No.MH-14 D-3027 from witness Afzal Khan Ibrahim Khan, resident of Dehu Road and
also motor cycle No.MH-14-M-5786. By using the aforesaid vehicles accused No.7 to 10
and 13 to 15 have committed ghastly murder of Kunal. In order to carry out the aforesaid
conspiracy successfully accused No. 7 has used revolver, khukri, sickle, sword and iron
bar and supplied it to accused Nos. 8 to 10 and from 13 to 15, by using the aforesaid
weapons the aforesaid persons have assaulted Kunal Parihar by which he sustained grave
injuries and ultimately died. Hence accused Nos. 1, 2, 3, 4, 5, 6, 7, 11 and 12 have been
charged under Sections 302, 120B IPC and accused Nos. 7, 8, 9, 10, 13, 14, 15 and 16
have charged u/S. 302 read with 120B IPC and under Arms Act, Sections 3 and 25."
[Emphasis supplied]
25. Thus, according to the prosecution version, when accused, A-1 to A-5, A-11 and A-12
learnt about the marriage of Kunal with some other girl, they hatched a conspiracy to
teach a lesson to the father of Kunal, the deceased. In furtherance thereof, accused A-1
contacted one of the assassins to kill Kunal. It is alleged that accused A-l to A-4 also
contacted accused A-16, a notorious criminal. In other words, the gravamen of the
accusation by the prosecution is that it is accused A-1 to A-5, A-11 and A-12 who had
hatched the conspiracy; acted in concert to give effect to their plan to get Kunal murdered
and in pursuance of the aforesaid criminal conspiracy, the other accused facilitated
commission of the said crime. It is common ground that the case of the prosecution is
based on the circumstantial evidence, namely, threatening calls from the side of the
accused to the complainant, his family and the earlier relationship between the deceased
and accused No.4. From the material on record, it is manifestly clear that it was the
family members of the appellant, one of their employees and a friend who allegedly had
all entered into an agreement to eliminate the deceased. However, as noted above,
accused A-l, A-2, A-4, A-11 and A-12 already stand discharged from the charges framed
against them under Sections 120B and 302 I.P.C vide orders dated 7th July, 2006 and
14th May, 2007, passed by the High Court and the Sessions Judge respectively. While
discharging the said accused, both the Courts have come to the conclusion that there is no
material on record to show that they had hatched a conspiracy to commit murder of
Kunal. Thus, the stand of the prosecution to the effect that the parents, sister and friends
of the appellants had entered into a criminal conspiracy stands rejected by virtue of the
said orders of discharge. Furthermore, in its order dated 7th July, 2006, the High Court
has opined that the circumstances, relied upon by the prosecution, even if accepted in its
entirety, only create a suspicion of motive, which is not sufficient to bring home an
offence of murder. As noted above, State's petition for special leave against the said
judgment has already been dismissed.
26. We are, therefore, of the view that in the light of the subsequent events, namely, the
orders of the High Court dated 7th July, 2006 in Criminal Writ Petitions Nos. 1283 and
1284 of 2006, discharging appellant's mother, sister and two close associates, accused
Nos. 2, 4, 11 and 12 respectively; order dated 30th April, 2007 passed by this Court
dismissing the Special Leave Petition preferred by the State against order dated 7th July,
2006 and order dated 14th May, 2007 passed by the Sessions Judge, Satara, discharging
the father (A-1) of the appellant, stated to be the mastermind behind the entire conspiracy,
for offences under Sections 120B and 302 I.P.C., on same set of circumstances and
accusations, no sufficient ground survives to proceed against the appellant for the
aforementioned offences.
27. For the reasons aforesaid, we are constrained to allow the appeals. Consequently, the
impugned orders are set aside and the appellant is discharged from the charges levelled
against him in the charge-sheet.
Appeals allowed. .
@page-SC2997
AIR 2008 SUPREME COURT 2997 "Jayendra Saraswati Swamigal v. State of Tamil
Nadu"
(From : 2006 (1) Mad LW (Cri) 122)
Coram : 3 K. G. BALAKRISHNAN, R. V. RAVEENDRAN AND Dr.
MUKUNDAKAM SHARMA, JJ.
Criminal Appeal No. 1132 of 2008 (arising out of S.L.R (Cri.) No. 1248 of 2006), D/- 22
-7 -2008.
Jayendra Saraswati Swamigal alias Subramaniamy. State of T.N.
Criminal P.C. (2 of 1974), S.24, S.406 - PUBLIC PROSECUTOR - APPOINTMENT -
TRANSFER OF CASE - Appointment of Public Prosecutor - Case transferred u/S.406
from one State to another State - It is Government of State to which case is transferred
can appoint Public Prosecutor.
2006 (1) Mad LW (Cri) 122, Reversed.
Once the case is transferred by Supreme Court as per S. 406 to another State, the
transferor-State to longer has control over the prosecution to be conducted in a Court
situated in a different State to which the case has been transferred. It is the prerogative of
the State Government to appoint a Public Prosecutor to conduct the case which is pending
in the sessions division of that State. The Supreme Court while passing order of transfer,
can give an appropriate direction as to which State should appoint the Public Prosecutor
to conduct that particular case. Such orders are passed having regard to the circumstances
of the case and the grounds on which the transfer has been effected. The Supreme Court
can certainly give directions irrespective of the provisions contained in S. 24. But in the
instant case nothing had been stated by the Supreme Court in the order of the transfer.
The provisions contained in the S. 24 shall prevail and it is for the appropriate State
Government within whose area the trial is conducted to appoint Public Prosecutor under
sub-sections (3) to (7) of S. 24.
2006 (1) Mad LW (Cri) 122, Reversed. (Para 7)
The powers under S. 24(8) are also to be exercised by the State Government of the
transferee Court where the sessions case is pending. Of course, the transferee-State can
appoint any person having qualification prescribed under sub-section (8) of S. 24,
whether he is a lawyer in the transferee-State or any other State. As it is a criminal case
registered by the transferor-State the expenses for conducting the trial are to be borne by
the transferor-State. (Paras 8, 11, 12)
Cases Referred : Chronological Paras
2005 AIR SCW 5410 : AIR 2006 SC 6 : 2005 Cri LJ 4626 (Ref.) 2
U.U. Lalit, Sr. Advocate, P.S. Narasimha, Krishna Kumar arid Pradeep Kr. Kar with him,
for Appellant; Altaf Ahmed, T.L.V. Iyer (N.P.), Sr. Advocates, V.G. Pragasam, S.J.
Aristotle, Prabhu Ramasubramanian, S.S. Khanduja and Yash Pal Dhingra with him, for
Respondent.
Judgement
K. G. BALAKRISHNAN, C.J.I. :- Leave granted.
2

. The appellant herein is one of the accused in a crime registered by Vishnu Kanchi Police
Station at Tamil Nadu. The police after investigation filed final report on 21-1-2005 and
the case was committed to the Principal Sessions Judge, Chinglepet and was registered as
Session Case No. 197/ 05. The appellant then moved this Court under S. 406 of Code of
Criminal Procedure, 1973 (hereinafter referred to as Cr. P.C.) seeking transfer of the case
to any other State. The appellant alleged in the transfer petition that he was being
unnecessarily harassed by the State of Tamil Nadu and that he would not get a fair trial.
This Court in Sri Jayendra Saraswathi Swamigal (II), T.N. v. State of Tamil Nadu (2005)
8 SCC 771 considered the matter in detail and reached the following conclusion in
paragraph 24 of the judgment :- 2005 AIR SCW 5410

"Taking into consideration the entire facts and circumstances of the case and the material
on record, we have no hesitation in holding that the petitioner and other co-accused of the
case have a reasonable apprehension that they will not get justice in the State of Tamil
Nadu. We would like to clarify here that we are casting no reflection on the district
judiciary in the State of Tamil Nadu. But it is the actions of the prosecuting agency and
the State machinery, which are responsible for creating a reasonable apprehension in the
mind of the petitioner and other co-accused that they will not get justice if the trial is held
in any place inside the State of Tamil Nadu. We are, therefore, of the opinion that the
interest of justice requires that the trial may be transferred to a place outside the State of
Tamil Nadu."
Thus, the Sessions Case No. 197/2005, pending before the Principal Sessions Judge,
@page-SC2998
Chinglepet, was transferred to the Court of Principal District and Sessions Judge,
Pondicherry and was numbered as Sessions Case No. 94/2005.
3. The Home Department of the State of Tamil Nadu on 25-11-2005 appointed one
Special Public Prosecutor and four Additional Special Public Prosecutors for conducting
the trial of the Sessions case pending before the Principal District and Sessions Judge,
Pondicherry. The appellant herein filed a petition before the Sessions Court challenging
the appointment of the Public Prosecutors by the State of Tamil Nadu and contended that
the Special Public Prosecutor appointed by State of Tamil Nadu has no right to conduct
the prosecution of the sessions case pending before the Pondicherry Court, outside the
State of Tamil Nadu. The Principal Sessions Judge, Pondicherry, by an order dated 25-1-
2005, held that under S. 24 of the Cr. P.C. the State of Tamil Nadu has the power to
appoint the Special Public Prosecutor for conducting the trial of the case and the State
had not lost its right to appoint the Public Prosecutor, merely on account of transfer of the
case to the Sessions Court at Pondicherry. The Court also noticed the fact that this Court,
while transferring the Sessions case at Pondicherry, had not specifically directed that the
State of Tamil Nadu shall not appoint a Public Prosecutor to conduct the case.
4. The appellant preferred a revision petition challenging the order passed by the
Principal District and Sessions Judge, Pondicherry. The High Court of Madras confirmed
the decision of the Sessions Court and held that the offence had been committed within
the State of Tamil Nadu, the investigation was done by the Tamil Nadu police and the
committal proceedings had also taken place in the Court at Tamil Nadu and hence the
Government of Tamil Nadu had the domain over that sessions case and unless this Court,
considering the special circumstances, directs in a particular case, appointment of a
Special Public Prosecutor by the State to which the case has been transferred in the
interest of justice, the transferee State cannot normally venture to appoint any Special
Public Prosecutor to handle the case which it received as per the orders of this Court. The
High Court was also of the view that it would be unjust to direct the transferee State
Government to open the purse stings to meet out the expenditure for the appointment of a
Special Public Prosecutor.
5. The appellant has challenged the order passed by the Sessions Court as well as the
High Court by which the Special Public Prosecutor and Additional Special Public
Prosecutors were appointed to conduct the trial of the case.
6. We heard the counsel for the appellant as well as counsel for the State of Tamil Nadu.
The counsel for the appellant contended that the appointment of the Public Prosecutor is
to be made by the State as per the procedure prescribed under S. 24 of Cr. P.C. It is
pointed out that the Government of Pondicherry has total authority to appoint a Public
Prosecutor or Additional Prosecutor or a Special Public Prosecutor to conduct a criminal
case pending before any of the Sessions divisions of the State of Pondicherry which was
formerly a Union Territory, now being a separate State and the Tamil Nadu Government
has no right to appoint any prosecutor - either a Public Prosecutor or a Special Public
Prosecutor - to conduct a trial of a case pending before the Principal Sessions Judge,
Pondicherry. The counsel for the respondent on the other hand contended that this Court
while ordering the case transfer to the State of Pondicherry had not specifically directed
that trial should be conducted by the prosecutor appointed by the Government of
Pondicherry and, therefore, State of Tamil Nadu has got the authority to appoint a Public
Prosecutor to conduct the trial of such a case. It was argued that the incident had taken
place in the State of Tamil Nadu and that being an offence committed against the State of
Tamil Nadu that State alone could appoint the Public Prosecutor to conduct the
prosecution of the case unless specifically otherwise directed by this Court while
transferring the case under S. 406 of the Cr. P.C.
7. For the purpose of understanding the scheme of appointment of a Public Prosecutor to
conduct the trial it is necessary to look into various provisions of Chapter II of the Cr.
P.C. Section 6 of Cr. P.C. prescribes that in every State there shall be following classes of
Criminal Courts : Courts of Sessions, Judicial Magistrate of the First Class (and in any
Metropolitan Area, Metropolitan Magistrate), the Judicial Magistrate of the second class
and Executive Magistrate. Section 7(1) prescribes that every State shall have a
@page-SC2999
sessions division or shall consist of several sessions divisions and every sessions division
shall, for the purposes of the Code, be a district or consist of districts. It also prescribes
that every metropolitan area shall be a separate sessions division and district. Sub-section
(2) provides that the State may alter the limits of such division and districts after
consultation with the High Court. Section 9 requires that the State Government shall
establish a Court of Session for every sessions division, and every Court of Sessions shall
be presided over by a Judge to be appointed by the High Court. Section 10 deals with the
constitution of the Assistant Sessions Judge and S. 11 deals with the constitution of the
Court of Judicial Magistrates. Section 12 deals with the appointment of Chief Judicial
Magistrate and Additional Chief Judicial Magistrate. Sections 16, 17 and 18 deal with the
constitution of the various Metropolitan Magistrates' Courts and S. 20 deals with
appointment of Executive Magistrate. Section 24 deals with the appointment of Public
Prosecutors. "Public Prosecutor" has been defined under S. 2(u) of the Cr. P.C. :-
"Public Prosecutor" means any person appointed under S. 24, and includes any person
acting under the directions of a Public Prosecutor."
Section 24(1) deals with the appointment of Public Prosecutor or Additional Public
Prosecutor for conducting any prosecution, appeal or other proceedings on behalf of the
Central Government or State Government in the High Court. Sub-section (3) of S. 24
requires that for every district, the State Government shall appoint a Public Prosecutor
and one or more Additional Public Prosecutors. Sub-sections (3) to (7) deal with
appointment of Public Prosecutor, Additional Public Prosecutor for the district. The
power of appointment is given to the State Government and such appointment should be
from a panel of names prepared by the District Magistrate in consultation with the
Sessions Judge. Sub-section (7) of S. 24 provides that a person shall be eligble to be
appointed as a Public Prosecutor or as an Additional Public Prosecutor under sub-section
(1) or sub-section (2) or sub-section (3) or sub-section (6) only if he has been in practice
as an advocate for not less than seven years. A conjoint reading of all these provisions
would clearly show that the State Government has the power of appointment of Public
Prosecutor or Additional Public Prosecutor for each district or Court of Sessions in the
session division in the State to conduct any prosecution, appeal or other proceedings
pending before the Courts in that State. The power of the State Government to appoint a
Public Prosecutor and Additional Public Prosecutor would extend only for conducting
any prosecution, appeal or other proceedings in the Courts within the State. As per the
procedure prescribed under S. 24, the State of Tamil Nadu can appoint a Public
Prosecutor to conduct criminal cases in any of the Court in that State. Such powers
cannot be exercised by the State Government to conduct cases in any other State. Once
the case is transferred as per S. 406 of the Cr. P.C. to another State, the transferor-State no
longer has control over the prosecution to be conducted in a Court situated in a different
State to which the case has been transferred. It is the prerogative of the State Government
to appoint a Public Prosecutor to conduct the case which is pending in the sessions
division of that State. Of course, this Court while passing order of transfer, can give an
appropriate direction as to which State should appoint the Public Prosecutor to conduct
that particular case. Such orders are passed having regard to the circumstanes of the case
and the grounds on which the transfer has been effected. This Court can certainly give
directions irrespective of the provisions contained in S. 24 of the Cr. P.C. But so far as
this case is concerned, nothing had been stated in the order of the transfer. The provisions
contained in the S. 24 of Cr. P.C. shall prevail and it is for the appropriate State
Government within whose area the trial is conducted to appoint Public Prosecutor under
sub-sections (3) to (7) of S. 24 of the Cr. P.C. is the Government of the State to which the
case has been transferred.
8. Sub-section (8) of S. 24 of Cr. P.C. is a special provision regarding the appointment of
a Special Prosecutor. This power can be exercised by the Central Government and the
State Government for the purpose of any case or class of cases, and a person who has
been in practice as an advocate for not less than ten years may be appointed as a Special
Public Prosecutor. These powers are also to be exercised by the State Government of the
transferee Court where the sessions case is pending. Of-course, the transferee-State can
appoint any person having
@page-SC3000
qualification prescribed under sub-section (8) of S. 24 of the Cr. P.C.
9. The purpose of transfer of the criminal case from one State to another is to ensure fair
trial to the accused. In this case, the main ground on which the transfer of the sessions
case was ordered from the Sessions Court of Chinglepet in Tamil Nadu to the Principal
District and Sessions Judge, Pondicherry, was that the action of the prosecution agency
had created a reasonable apprehension in the mind of the accused-appellant that he would
not get justice if the trial was held in the State of Tamil Nadu. The Public Prosecutor
plays a key role during trial of a Sessions case. Though the Sessions Judge has got a
supervising control over the entire trial of the case, it is the Public Prosecutor who
decides who are the witnesses to be examined on the side of the prosecution and which
witness is to be given up, or which witness is to be recalled for further examination. For
proper conduct of a criminal case the Public Prosecutor plays a vital role. It may also be
noticed herein that under S. 225 of the Cr. P.C. during every trial before the Court of
Sessions, the prosecution shall be conducted by the Public Prosecutor and as regards
withdrawal also, the Public Prosecutor in-charge of the case has to make the application
for withdrawal of prosecution as per S.231 of the Cr. P.C. In case of acquittal of the
accused the State Government may direct the Public Prosecutor to file an appeal.
10. As is evident from various provisions of the Cr. P.C., the State Government of Tamil
Nadu can only appoint a Public Prosecutor or an Additional Public Prosecutor or a
Special Public Prosecutor under S. 24 of the Cr. P.C. to conduct the prosecution and
appeal, or other proceeding in any Criminal Courts in respect of any case pending before
the Courts of Tamil Nadu and in respect of any case pending before the Courts at
Pondicherry, the State Government of Pondicherry is the appropriate Government to
appoint Public Prosecutor, Additional Public Prosecutor or Special Public Prosecutor.
11. However, we make it clear that the State of Pondicherry can appoint any counsel as
Public Prosecutor having requisite qualifications as prescribed under sub-section (8) of S.
24 of Cr. P.C. whether he is a lawyer in the State of Pondicherry or any other State. As it
is a criminal case registered by the State of Tamil Nadu the expenses for conducting the
trial are to be borne by the State of Tamil Nadu. The Advocate fees payable to the Public
Prosecutor, Additional Public Prosecutor or Special Public Prosecutor by the State of
Pondicherry shall be borne by the State of Tamil Nadu and the Home Departments of the
two States may undertake consultations with each other and an appropriate decision may
be taken by the concerned authorities in this regard.
12. We set aside the impugned order passed by the High Court and direct that the State of
Pondicherry may continue with the prosecution of the case in accordance with the law
and the Public Prosecutor or Special Public Prosecutor may be appointed by the State of
Pondicherry to conduct the criminal proceedings in respect of Sessions Case No. 94 of
2005 pending before Principal and Sessions Judge of Pondicherry.
13. The appeal is allowed to the extent indicated above.
Order accordingly. .
AIR 2008 SUPREME COURT 3000 "Eastern Coalfields Ltd. v. Dugal Kumar"
(From : Calcutta)*
Coram : 2 C. K. THAKKER AND LOKESHWAR SINGH PANTA, JJ.
Civil Appeal No.245 of 2004, D/- 28 -7 -2008.
Eastern Coalfields Ltd. v. Dugal Kumar.
(A) Constitution of India, Art.226 - WRITS - PLEA - Territorial jurisdiction - Admission
of writ petition - Counsel for respondent was present that time - Did not raise any
objection to territorial jurisdiction of Court - He expressly made statement before Court
to pass "usual order" - Accordingly order was passed by High Court - Plea before
Supreme Court that said High Court had no territorial jurisdiction to entertain writ
petition - Not tenable. (Para 13)
(B) Constitution of India, Art.133 - APPEAL - HIGH COURT - SPECIAL LEAVE
PETITION - Appeal - Appellant challenged order of High Court in review petition -
Appeal cannot be said to be not maintainable on ground that original order passed by
High Court in intra Court appeal was not challenged - Even other-wise appeal cannot
@page-SC3001
be dismissed when leave was granted in SLP after hearing the parties. (Para 14)
(C) Constitution of India, Art.226 - WRITS - Delay and laches - Claim for release of coal
to which petitioner was entitled against land acquired - Delay of about 10 years in filing
petition - Petition should not have been entertained - However, relief granted in view of
statement made by counsel appearing for respondent, Company that Court could pass
"usual order" - Further, similar orders were passed in several matters - Therefore, order
extending said benefit to petitioner, not liable to be interfered with. (Para 21)
(D) Constitution of India, Art.226 - WRITS - HIGH COURT - Modification/Clarification
of order - Power of High Court - Petition disposed on directing allotment of 1008 MTs.
coal to petitioner - After a week granting prayer for allotment of 6800 Mts. coal merely
on mentioning the matter, without any application for modification of earlier order -
Illegal. (Para 25)
Cases Referred : Chronological Paras
2004 AIR SCW 1739 : AIR 2004 SC 1950 : 2004 Lab IC 1690 20
1993 COD 344 16
(1990) 2AC 738 : (1990) 2 All ER 434 : (1990) 2 WLR 1302 17
AIR 1974 SC 259 : 1974 Lab IC 165 20
AIR 1970 SC 470 : 1970 Lab IC 402 19
AIR 1970 SC 898 19
(1874) 5 PC 221 : 22 WR 492 18
Ajit Kumar Sinha and Shivalok, for Appellant; R.K. Gupta and A.N. Bardiyar, for
Respondent.
* G. A. No. 3319 of 2000 in APOT No. 94 of 2000, D/- 28-1-2002 (Cal.).
Judgement
1. C. K. THAKKER, J. :-This appeal is filed by the Eastern Coalfields Ltd. ('Company'
for short) against an order passed by the Division Bench of the High Court of Calcutta on
January 28, 2002 dismissing Review Petition filed by the appellant herein.
2. The facts of the case are that the appellant is 'Government Company' under Section 617
of the Companies Act, 1956. A scheme was formulated by the Company to offer
employment to a person who is a land loser for 1 acre of land which has been acquired,
purchased or used by the Company. Subsequently, the policy was changed and it was also
provided that those who do not intend to get employment may be offered 800 Metric
Tons (MTs) of coal in lieu of employment of a family member whose one acre of land has
been acquired, purchased or used by the Company. The policy was again modified in
1996 and entitlement was increased to 1600 MTs.
3. It is the case of the Company that it purchased land of the respondent admeasuring
1.26 acres and registered sale deed No. 2006, dated February 17, 1989 was executed at
Dhanbad. In the light of the policy then in vogue, the respondent was offered 1008 MTs
of coal on the basis of 800 MTs per acre since the sale was for 1.26 acres of land. The
respondent accepted the said decision on February 23, 1989 and a written communication
was addressed to the General Manager stating therein that the land owner was not
interested in getting employment and he would be thankful if 1008 MTs of coal would be
given to him. The Authorities accepted the request and the appellant Company vide a
letter dated May 22, 1989, passed an order to release 1008 MTs of steam Coal, Grade-D.
It was stated that the coal would be released on fulfilling the terms and conditions
mentioned in the said communication. According to the Company, everything was over
and nothing further was required to be done in the matter. The respondent-writ-petitioner
was paid consideration for land which was sold by registered sale deed. Over and above
compensation, as per the policy in force, they offered 1008 MTs of coal which was
accepted by the respondent and quantity was also released. It was accepted by the
respondent without any protest, objection or reservation and the matter ended there.
4. After considerable delay of about a decade, a petition came to be filed being Writ
Petition No. 770 of 1999 contending therein that writ petitioner (respondent herein) was
entitled to additional quantity of 1008 MTs of coal and an appropriate direction be issued
to the Company to release the goods. The writ petition was placed for 'first' hearing on
September 6, 1999, and on the same day, it was disposed of by the Court observing inter
alia that "no affidavit in opposition has been filed", and the learned counsel for the
Company submitted that 'usual order' be passed in the matter. Accordingly, the Company
was directed to allot "balance quantity of 1008 MTs" of coal to the writ-petitioner.
5. The said order read as under;
@page-SC3002
"Mr. D.P. Majumdar, Adv. with Mr. G. Patra, Adv. appears and submits.
Mr. A.K. Mitra, Adv. with Md. Iairsh, Adv. appears and submits.
The Court : No affidavit-in-opposition has been filed. The Learned Lawyer for the
respondent submits that usual order may be passed in this matter. Accordingly, I passed
the following order :-
The respondents are directed to allot balance quantity of 1008 M.T. to the petitioner in
terms of the release order dated 25-5-1989 from Nayandanga Colliery, Mugma Area.
The writ petition is disposed of.
All parties are to act on a signed copy of the minutes of this order on the usual
undertaking."
6. It is stated by the appellant-Company that after the order dated September 6, 1999, on
September 13, 1999, again the matter was mentioned by the writ-petitioner without filing
any application and the High Court modified its earlier order dated September 6, 1999
and the balance quantity which was mentioned in the earlier order of September 6, 1999
as 1008 MTs was enhanced to 6800 MTs. Again, the matter was mentioned on September
15, 1999 and the order was corrected.
7. The Company, being aggrieved by the order passed by the learned Single Judge,
preferred an appeal being APOT No. 94 of 2004 challenging the orders passed by the
learned single Judge. The Division Bench of the High Court, however, dismissed the
appeal on February 17, 2000 observing that when the order was passed by the learned
single Judge on September 6, 1999, the counsel for the Company appeared and no reply
was filed by the Company. In the circumstances, the Company had "to blame itself". The
matter was thereafter taken up by the learned single Judge and even at that stage, no reply
was filed. According to the Court, therefore, there was no reason to interfere with the
order of the single Judge and the appeal was dismissed.
8. The Company challenged the order passed by the Division Bench of the High Court by
filing Special Leave Petition No. 8238 of 2000. When the matter came up for admission
hearing, it was withdrawn on May 12, 2000. The said order mentioned that the learned
counsel for the Company stated that the Company would file 'Review Petition' in the
High Court. The special leave petition was accordingly 'dismissed as withdrawn'.
Thereafter Review Petition was filed by the Company in the High Court and as stated
above, the Review Petition was also dismissed by the Court observing that there was "no
apparent error to review the order". The said order is challenged in the present appeal.
9. Initially when the matter was placed for admission hearing, notice was issued on
August 12, 2002. It appears that the Special Leave Petition was dismissed on February
12, 2003, but the said order was recalled by the Court on September 12, 2003. On
January 12, 2004, leave was granted, printing was dispensed with and the appeal was
ordered to be heard on SLP paper books. Parties were granted liberty to file additional
documents. Original record was requisitioned. Interim stay against the order passed by
the High Court was also granted. On March 7, 2008, a Bench of this Court presided over
by Hon'ble the Chief Justice of India directed final hearing of the matter during summer
vacation and accordingly the matter was placed before us for final disposal on May 27,
2008.
10. We have heard the learned counsel for the parties.
11. The learned counsel for the appellant-Company contended that the orders passed by
the High Court are liable to be set aside. It was submitted that admittedly, the transaction
of sale took place in Dhanbad. Both the parties - the appellant as well as respondent -
were residing at Dhanbad. The entire cause of action thus arose within the territorial
jurisdiction in the State of Bihar (now within Jharkhand area). The High Court of
Calcutta, therefore, had no territorial jurisdiction to entertain, deal with and decide the
writ petition. On that ground alone, the orders passed by the High Court of Calcutta are
liable to be set aside. It was also submitted that admittedly sale deed was executed in
February, 1989 by the respondent. The Company paid the amount of consideration and
offered 1008 MTs coal under the policy then in force over and above the amount of
consideration of property. The writ petitioner accepted the offer, release order was issued
and the goods had been delivered to him which the writ petitioner accepted without any
protest or objection. It was after about 10 years that a writ petition was filed which was
entertained and orders were passed by the High Court. According
@page-SC3003
to the appellant, there was gross delay and laches on the part of the writ petitioner in
approaching the Court and on that ground also, no order could have been passed granting
relief in favour of the writ petitioner. Moreover, there was no right-duty relationship
between the writ petitioner and the Company. The right of the writ petitioner was limited
to consideration in lieu of land sold to the Company. The said amount had already been
paid to the writ petitioner. It was only on the basis of the policy that coal was offered to
the writ-petitioner. Even if it is assumed that the writ petitioner had right to get coal as
per the policy adopted by the Company, the quantity to which the respondent was entitled
was given to him. Thereafter there was no cause for making grievance against the
Company. It is only on the basis of 1996 Policy that additional quantity was demanded by
the writ-petitioner by filing writ petition in 1999 to which there was no entitlement on the
part of the writ petitioner. Even on that ground, therefore, the petition was liable to be
dismissed. The counsel also argued that when the writ petitioner was paid consideration
for land as also coal under the policy in force and when it was accepted without any
protest, the writ petitioner was estopped under the doctrine of equitable estoppel to
challenge the said decision. By his conduct, writ petitioner made it abundantly clear that
he was satisfied as to the quantity which was offered to him and after acceptance thereof,
it was not open to challenge the said decision. It was also urged that the learned single
Judge was not right in observing that the writ petitioner was entitled to relief as prayed in
the writ petition on the ground that no counter affidavit was filed by the Company. The
record reflects that writ petition was filed by the writ petitioner on February 18, 1999. It
was placed for 'first' hearing on September 6, 1999 and on the same day, the matter was
disposed of. It was, therefore, not proper for the Court to observe that since no affidavit
was filed by the Company, the prayer of the petitioner should be granted. A grievance
was also made that even after the decision on September 6, 1999, without there being any
application, the order was modified on mentioning the matter and the quantity was
enhanced from additional 1008 MTs to 6800 MTs which was clearly illegal and without
jurisdiction. In Letters Patent Appeal also, the fact of non-filing of affidavit by the
Company weighed with the Division Bench, but as already stated, the matter was taken
up and disposed of on one and the same day at the 'first' hearing by the learned single
Judge and there was no default on the part of the Company. It was submitted by the
counsel that when the grievance was made against the order passed by the Division
Bench of the High Court in the Special Leave Petition, this Court had observed that the
counsel for the Company wanted to file Review Petition and SLP was, therefore,
dismissed as withdrawn. But even thereafter the Division Bench dismissed the Review
Petition which necessitated the Company to approach this Court again. It was, therefore,
submitted that the orders passed by the High Court may be set aside by allowing the
appeal and holding that the writ petitioner was not entitled to additional quantity of coal
and the High Court should not have ordered the Company to supply coal.
12. The learned counsel for the respondent-writ petitioner supported the orders passed by
the High Court. It was submitted that the learned single Judge was wholly right in
observing that no affidavit was filed by the Company. Moreover, the learned counsel for
the Company appeared in the Court and made a statement that 'usual order' be passed.
Accordingly, the order was passed and thereafter it was not open to the Company to raise
an objection against such order. An objection as to territorial jurisdiction of the Court also
loses its significance in the light of the statement made by the counsel appearing for the
Company. In the affidavit-in-reply, it was stated by the writ petitioner that several
similarly situated persons were granted the benefit and additional quantity of coal was
given to them. Copies of the orders in favour of all those persons were placed on record
in the counter-affidavit. It was further stated that the policy was modified in 1996 and
additional quantity of coal was given to land losers. Such benefit was also granted to
other persons. Refusal to grant similar benefit to the writ petitioner was violative of
Article 14 of the Constitution. The counsel submitted that in the circumstances, the
Division Bench was right in not entertaining intra-court appeal on the ground that if
counter was not filed by the Company, the Company had to thank itself. Again, it is not
true to say that liberty was granted by this Court to file Review Petition. Special Leave
Petition was dismissed as withdrawn but this
@page-SC3004
Court did not grant liberty to file Review Petition. Hence, the Review Petition itself was
not maintainable. The counsel also contended that even in the present proceedings, the
prayer is only to set aside an order passed in Review on January 28, 2002. The main order
passed in intra-court appeal (dismissing the appeal) has not been challenged. It was,
therefore, submitted that on all these grounds, no interference is called for and the appeal
deserves to be dismissed.
13. Having heard the learned counsel for the parties, in our opinion, the appeal deserves
to be partly allowed. So far as the technical objection raised by the Company with regard
to territorial jurisdiction of the High Court of Calcutta is concerned, in our opinion, it
would not be appropriate to set aside the order passed in favour of the writ petitioner on
that ground. It is clear from the record that the writ petition came up for admission
hearing on September 6, 1999 and the counsel for the appellant-Company was present.
Not only that he did not raise any objection as to territorial jurisdiction of the Court, he
expressly made a statement before the Court to pass "usual order". Accordingly, an order
was passed directing the Company to allot "balance quantity of 1008 MTs" of coal to the
writ petitioner. We are, therefore, unable to uphold the contention of the learned counsel
for the appellant-Company that the High Court of Calcutta had no territorial jurisdiction
to entertain the writ petition.
14. But we are also unable to uphold the contention of the writ petitioner that the appeal
is not maintainable since the Company had challenged the order passed in Review
Petition dated January 28, 2002 and not the main order dated February 17, 2000
dismissing intra-Court appeal. It was submitted by the learned counsel for the appellant
that when Review Petition was dismissed, the order passed by the Division Bench in
intra-Court appeal got merged in the order of Review Petition. But even otherwise, when
the order passed in the Review Petition is challenged, it would not be proper to dismiss
this appeal particularly when leave was granted in SLP after hearing the parties. We,
therefore, reject the objection raised by the writ petitioner.
15. As to delay and laches on the part of the writ petitioner, there is substance in the
argument of learned counsel for the appellant-Company. It is well-settled that under
Article 226 of the Constitution, the power of a High Court to issue an appropriate writ,
order or direction is discretionary. One of the grounds to refuse relief by a writ Court is
that the petitioner is guilty of delay and laches. It is imperative, where the petitioner
invokes extra-ordinary remedy under Article 226 of the Constitution, that he should come
to the Court at the earliest reasonably possible opportunity. Inordinate delay in making
the motion for a writ is indeed an adequate ground for refusing to exercise discretion in
favour of the applicant.
16. Under the English law, an application for leave for judicial review should be made
"promptly". If it is made tardily, it may be rejected. The fact that there is breach of public
law duty does not necessarily make it irrelevant to consider delay or laches on the part of
the applicant. Even if leave is granted, the question can be considered at the time of final
hearing whether relief should be granted in favour of such applicant or not. (Vide R. v.
Essex Country Council, 1993 COD 344).
17. In R. v. Dairy Produce Quota Tribunal, (1990) 2 AC 738, 749 : (1990) 2 All ER 434 :
(1990) 2 WLR 1302, the House of Lords stated;
"The public interest in good administration requires that public authorities and third
parties should not be kept in suspense as to the legal validity of a decision the authority
has reached in purported exercise of decision making powers for any longer period than
is absolutely necessary in fairness to the persons affected by the decision".
18. The underlying object of refusing to issue a writ has been succinctly explained by Sir
Barnes Peacock in Lindsay Petroleum Co. v. Prosper Armstrong, (1874) 5 PC 221 : 22
WR 492 thus;
"Now the doctrine of laches in Courts of Equity is not an arbitrary or a technical doctrine.
Where it would be practically unjust to give a remedy, either because the party has, by his
conduct, done that which might fairly be regarded as equivalent to a waiver of it, or
where by his conduct and neglect he has, though perhaps not waiving that remedy, yet put
the other party in a situation, in which it would not be reasonable to place him if the
remedy were afterwards to be asserted, in either of these cases, lapse of time and delay
are most material. But in every case, if an argument against relief,
@page-SC3005
which otherwise would be just, is founded upon mere delay, that delay of course not
amounting to a bar by any statute or limitations, the validity of that defence must be tried
upon principles substantially equitable. Two circumstances, always important in such
cases, are, the length of the delay and the nature of the acts done during the interval,
which might affect either party and cause a balance of Justice or injustice in taking the
one course or the other, so far as it relates to the remedy."
(Emphasis supplied)
19

. This Court has accepted the above principles of English law. In Tilokchand Motichand
v. H.B. Munshi, (1969) 2 SCR 824 and Rabindra Nath Bose v. Union of India, (1970) 1
SCR 697, this Court ruled that even in cases of violation or infringement of Fundamental
Rights, a writ Court may take into account delay and laches on the part of the petitioner
in approaching the Court. And if there is gross or unexplained delay, the Court may
refuse to grant relief in favour of such petitioner. AIR 1970 SC 898
AIR 1970 SC 470

20

. It is not necessary for us to refer to several decisions on this point wherein a similar
view has been taken by this Court. Suffice it to say that in Express Publications v. Union
of India, (2004) 11 SCC 526, this Court referring to Tilokchand Motichand, Rabindranath
Bose and Ramchandra Deodhar v. State of Maharashtra, (1974) 1 SCC 317, explained the
principle thus; 2004 AIR SCW 1739, (Para 26)
AIR 1974 SC 259

"No hard and fast principle can be laid down that under no circumstances delay would be
a relevant consideration in judging constitutional validity of a provision. It has to be
remembered that the constitutional remedy under Article 32 is discretionary. In one case,
this Court may decline discretionary relief if person aggrieved has slept over for long
number of years. In another case, depending upon the nature of violation, court may
ignore delay and pronounce upon the invalidity of a provision. It will depend from case to
case."
(Emphasis supplied)
21. Prima facie, we are satisfied that the learned single Judge should not have entertained
a writ petition in 1999 and in directing the Company to release balance quantity of 1008
MTs of coal to the writ petitioner. But as observed earlier, the order was passed in view of
the statement of learned counsel appearing for the Company that the Court could pass
"usual order" and accordingly the order was passed. It was also stated by the writ
petitioner in the counter-affidavit that similar orders were passed in several matters. It
would, therefore, be appropriate if we extend the benefit to the writ petitioner of the order
passed by the learned single Judge to the extent of "balance quantity of 1008 MTs of
coal", which was based on the 'statement' by the counsel for the Company.
22. In our view, however, the learned counsel for the appellant-Company is right that
after the writ petition was disposed of on September 6, 1999 wherein balance quantity of
1008 MTs of coal was directed to be allotted to the writ petitioner, the learned single
Judge was not justified in passing an order on September 13, 1999 on mentioning of the
matter without there being any application for modification/clarification of the order
dated September 6, 1999.
23. On September 13, 1999, the following order was passed by the learned single Judge;
"Mr. D.P. Majumdar, Adv. Mentions and submits.
Mr. A.K. Mitra, Adv. Appears and submits.
The Court : The last but third paragraph of the order dated 6-9-99 is corrected in the
manner as follows :-
The respondents are directed to allot balance quantity of 6800 M.T. of Steam Coke,
Grade-D quality of Coal to the petitioner in terms of the release order dated 25-5-1989
from Nayandanga Colliery, Mugma Area in terms of Annexure-'C'.
The supply is to be effected within a period of eight weeks from the date of
communication of this order.
This order is incorporating into the earlier order dated 6-9-99.
All parties are to act on a signed copy of the minutes of this order."
24. We are also of the view that in the light of the above order, the Division Bench ought
to have interfered with the direction of the learned single Judge in the order dated
September 13, 1999 and intra-Court appeal ought to have been allowed. When the intra-
Court appeal was dismissed, the appellant
@page-SC3006
approached this Court by filing Special Leave Petition. It was dismissed as withdrawn as
the Company wanted to move the Division Bench in Review Petition. To us, on the facts
and in the circumstances of the case, the Division Bench ought to have considered the
aforesaid aspect and passed an appropriate order in accordance with law.
25. From the totality of circumstances, we are of the considered view that the respondent-
writ petitioner was entitled to the price (consideration) for the land sold by him by
registered sale deed to the Company which has already been paid to him. He was also
entitled to 1008 MTs of coal which was given to him as per the Policy. He was further
entitled to 1008 MTs which has been ordered to be given to him towards "balance
quantity" on the basis of statement made by the Counsel for the Company and in terms of
'usual order' dated September 6, 1999 passed by the learned single Judge. We are,
however, convinced that the learned single Judge was not justified in granting prayer on
mentioning the matter on September 13, 1999 without any application for modification of
earlier order and direction to the Company to allot to the writ petitioner balance quantity
of 6800 MTs of steak coal Grade-D quality. To that extent, therefore, the appeal filed by
the Company deserves to be allowed.
26. For the foregoing reasons, the appeal is partly allowed and the writ petitioner is held
entitled to 1008 MTs as initially awarded to him as also 1008 MTs of coal towards
"balance quantity" as per the order dated September 6, 1999. The writ petitioner will not
be entitled to anything more. If the said quantity of coal has already been allotted, the
Company has discharged its liability and nothing more is required to be done. But if it has
not released the said quantity, the writ petitioner would be entitled to coal to the above
extent. On the facts and in the circumstances of the case, there will be no order as to
costs.
Order accordingly. .
AIR 2008 SUPREME COURT 3006 "Shail Kumari Devi v. Krishan Bhagwan Pathak"
(From : 2007 (3) Pat LJR 723)
Coram : 2 C. K. THAKKER AND D. K. JAIN, JJ.
Civil Appeal No.4666 of 2008 (arising out of SLP (C) No. 17260 of 2007), D/- 28 -7
-2008.
Shail Kumari Devi and Anr. v. Krishan Bhagwan Pathak alias Kishun B. Pathak.
(A) Criminal P.C. (2 of 1974), S.125 (as it stood before 2001 Amendment) -
MAINTENANCE - Maintenance - Interim maintenance - Court has implied power to
grant.
The jurisdiction of the Magistrate to pass Order for Maintenance to Wives, Children and
Parents is not strictly criminal in nature. Moreover, the remedy provided by Section 125
is a summary remedy for securing reasonable sum by way of maintenance subject to a
decree passed by a competent civil Court. Hence, in absence of any express bar or
prohibition, Section 125 could be interpreted as conferring power by necessary
implication to make interim order of maintenance subject to final outcome in the
application. (Paras 21, 27)
(B) Criminal P.C. (2 of 1974), S.125, S.354(6) - MAINTENANCE - JUDGMENT -
Maintenance - Award of, from date of application - Court need not record special reasons.
1983 Jab LJ 55; 1987 Cri LJ 1952 (MP); (1983) 1 Crimes 590 (MP); (1992) 1 WLC 305
(Raj); 1998 Cri LJ 2704 (Raj); 1998 Cri LJ 2803 (All); 2002 Cri LJ 4678 (Cal); 2001 Cri
LJ 2064 (All); (2000) 2 Pat LJ R 241, Overruled.
While deciding an application under Section 125 a Magistrate is required to record
reasons for granting or refusing to grant maintenance to wives, children or parents. Such
maintenance can be awarded from the date of the order, or, if so ordered, from the date of
the application for maintenance, as the case may be. For awarding maintenance from the
date of the application, express order is necessary. No special reasons, however, are
required to be recorded by the Court. No such requirement can be read in sub-section (1)
of Section 125 in absence of express provision to that effect. (Paras 44, 47)
Maintenance is a right which accrues to a wife against her husband the minute the former
gets married to the latter. It is not only a moral obligation but is also a legal duty cast
upon the husband to maintain his wife. Hence, whenever a wife does not stay with her
husband and claims maintenance, the only question which the Court is called upon to
consider is whether she was justified to live separately from her husband and still claim
maintenance from him? If the reply is in the affirmative, she is entitled to claim
maintenance. It is, therefore, open to
@page-SC3007
the Magistrate to award maintenance from the date of application and there is nothing
which requires recording of 'special reasons' though he must record reasons as envisaged
by sub-section (6) of Section 354 of the Code in support of the order passed by him.
1983 Jab LJ 55; 1987 Cri LJ 1952 (MP); (1983) 1 Crimes 590 (MP); (1992) 1 WLC 305
(Raj); 1998 Cri LJ 2704 (Raj); 1998 Cri LJ 2803 (All); 2002 Cri LJ 4768 (Cal); 2001 Cri
LJ 2064 (All); (2000) 2 Pat LJR 241, Overruled. (Para 46)
(C) Criminal P.C. (2 of 1974), S.125 (as it stood before 2001 Amendment) -
MAINTENANCE - AMENDMENT - Maintenance - Claim made by wife and daughter
before 2001 Amendment - Order granting maintenance to wife as well as daughter at rate
of Rs. 2000/- and Rs. 1000/- respectively from the date of application - Improper, being
in excess of statutory limit - Moreover, evidence showing that wife was living in house
belonging to husband - Was also in possession of husband's land and Land inherited from
father - Maintenances granted was even on facts excessive. (Para 49)
Cases Referred : Chronological Paras
2002 Cri LJ 4768 (Cal) (Overruled) 33
2001 Cri LJ 2064 : 2001 All LJ 794 (Overruled) 34, 36
2000 (2) Pat LJR 241 (Overruled) 35
1999 Cri LJ 1008 (Mad)38 1998 Cri LJ 2803 : 1998 All LJ 974 (Overruled) 38
1998 Cri LJ 2704 (Raj) (Overruled) 33
1996 Cri LJ 4476 (P and H) 39
1992 Cri LJ 1028 (MP) 40
(1992) 1 WLC 305 (Raj) (Overruled) 33
1990 Cri LJ 1880 (AP) 43, 44
AIR 1988 SC 1208 : 1988 Cri LJ 1745 38
1987 Cri LJ 1952 (MP) (Overruled) 33
AIR 1986 SC 984 : 1986 Cri LJ 41 (Foll., Pnt. A) 22, 24, 25, 27, 45
(1983) 1 Crimes 590 (MP) (Overruled) 33, 40
1983 Jab LJ 55 (Overruled) 33, 40
Abhinav Prakash and Kanhaiya Priyadarshi, for Appellants; Kumar Rajesh Singh and Ms.
Niranjana Singh, for Respondent.
Judgement
1. C. K. THAKKER, J. :-Leave granted.
2. The present appeal is filed by appellant No. 1-wife and appellant No. 2-daughter of
respondent herein - Krishan Bhagwan Pathak. The appellants have approached this Court
being aggrieved by the judgment and order passed by the High Court of Judicature at
Patna on May 3, 2007 in Criminal Revision No. 67 of 2007. By the said order, the High
Court partly allowed the revision filed by the respondent-husband and modified the order
passed by the Court of Principal Judge, Family Court, Bhojpur on October 30, 2006 in
Miscellaneous Case No. 280 of 1997, renumbered as No. 1 of 2005.
3. Shortly stated the facts of the case are that the marriage between appellant No. 1 and
the respondent was solemnized according to Hindu rites, customs and ceremonies before
more than three decades. From the said wedlock, nine children were born. Appellant No.
2-Kumari Babli is the youngest among all and she is the only child staying with her
mother-appellant No. 1. At the time of filing of the application, she was of twelve years.
4. On July 21, 1997, the appellants filed a case for maintenance in the Court of Chief
Judicial Magistrate, Bhojpur under Section 125 of the Code of Criminal Procedure, 1973
(hereinafter referred to as 'the Code') (Misc. Case No. 280 of 1997) claiming maintenance
of Rs. 500/- p.m. for appellant No. 1 and Rs. 500/- p.m. for appellant No.2. It was the
case of the appellant No. 1 that her husband had neglected to maintain his wife-appellant
No. 1 as also his legitimate daughter-appellant No. 2. On November 20, 1999, an
application was filed by the appellants requesting the Court to grant 'interim' maintenance
during the pendency of proceedings before the Court. The learned Chief Judicial
Magistrate allowed the said application, granted the prayer and fixed interim maintenance
at the rate of Rs. 300/- p.m. for each of the applicants with effect from February 12, 1998.
The parties, thereafter, led the evidence which was closed on September 3, 2001 and the
case was adjourned for final arguments. During the pendency of proceedings, however,
Family Court came to be established and the case was transferred to the Principal Judge,
Family Court, Bhojpur.
5. From the evidence, it was clear that the respondent was working as Cashier with the
State Bank of India, Bihita Branch and was getting gross salary of Rs. 18,508-98. After
deduction, his pay packet was of Rs. 9,831-76. The respondent retired from service in
January, 2006. The appellants filed a petition on September 12, 2006 with a
@page-SC3008
prayer to direct the respondent to pay arrears of maintenance which came to Rs. 11,600/-
and the Family Court on October 30, 2006, allowed the application and directed the
respondent to pay the entire amount of the arrears in lump sum by the next date of
hearing.
6. The matter was finally disposed of by the Family Court on November 29, 2006 and the
learned Principal Judge of the Family Court directed the respondent to pay maintenance
of Rs. 2,000/- p.m. to applicant-appellant No. 1-wife and Rs. 1,000/- p.m. to applicant-
appellant No. 2 - minor daughter with effect from the date of application i.e. July 21,
1997 with further order to pay arrears within three months of the order after deducting the
amount which had already been paid under the interim order passed by the Court earlier.
7. The appellant was dissatisfied with the order passed by the Principal Judge of the
Family Court and preferred Criminal Revision No. 67 of 2007 in the High Court.
8. The High Court partly allowed the Revision and modified the direction issued by the
Family Court. The High Court reduced the amount of maintenance from Rs. 2,000/ to Rs.
750/- to appellant No. 1-wife and from Rs. 1,000/- to Rs. 750/- to appellant No. 2-
daughter. The High Court also directed that the amount of maintenance would be payable
to the applicants-appellants not from the date of the application i.e. July 21, 1997 but
from the date of the order i.e. November 29, 2006. The said order is challenged by the
appellants in the present appeal.
9. On September 5, 2007, the matter was placed for admission hearing. Delay of eight
days in filing Special Leave Petition was condoned and notice was issued to the
respondent. Considering the nature of the litigation, the Registry was directed by an order
dated April 16, 2008 to place the matter for final disposal on a non-miscellaneous day and
that is how the matter is placed before us.
10. We have heard learned counsel for the parties.
11. Learned counsel for the appellants contended that the High Court was wrong in partly
allowing Revision filed by the respondent and in modifying the directions issued by the
Family Court. It was submitted that the High Court was in clear error in reducing the
amount of maintenance to appellant No. 1-wife and appellant No. 2-daughter. Similarly,
the High Court was in error in holding that the appellants were not entitled to
maintenance from the date of application but only from the date of order passed by the
Court. It was, therefore, submitted that the order passed by the High Court deserves to be
set aside by restoring the order of the Family Court.
12. The learned counsel for the respondent, on the other hand, supported the order passed
by the High Court. It was urged that the Family Court was not right in granting
maintenance to the appellants from the date of application. It was submitted that the
Family Court was again wrong in allowing maintenance of more than Rs. 500/- either to
appellant No. 1-wife or to appellant No. 2-daughter before 2001 when the relevant
provisions of law (Section 125 of the Code as it then stood), allowed Rs. 500/-p.m. as
maximum amount of maintenance. The High Court was, therefore, justified in reducing
the amount as also issuing direction to make payment, from the date of the order. It was
also urged that no 'interim' maintenance could have been awarded before the amendment
in the Code in 2001.
13. The counsel submitted that even on merits, the Family Court was not justified in
ignoring the evidence on record and in granting maintenance to wife observing that
appellant No. 1 was unable to maintain herself. The evidence clearly revealed, submitted
the counsel, that some of the properties of the respondent-husband were with the
appellant No. 1-wife. She has also inherited land from her father. Those facts, therefore,
ought to have been taken into account by the Family Court in fixing the amount of
compensation. On all these grounds, it was submitted that no interference in the order
passed by the High Court is called for in exercise of discretionary jurisdiction under
Article 136 of the Constitution and the appeal deserves to be dismissed.
14. Three questions arise for our consideration; (i) whether interim maintenance could be
awarded in absence of specific and express provision in the Code; (ii) whether the
applicant-wife and her daughter are entitled to maintenance from the date of the order
passed by the Family Court or from the date of application made by them under Section
125 of the Code; and (iii) what could be the amount of maintenance which could be
awarded by the Court.
@page-SC3009
15. Before we proceed to consider these questions, it would be appropriate if we examine
the relevant provisions of law. Subsections (1) and (2) of Section 125 of the Code, as they
were originally enacted in 1973, read thus :
125. Order for maintenance of wives, children and parents. - (1) If any person having
sufficient means neglects or refuses to maintain -
(a) his wife, unable to maintain herself, or
(b) his legitimate or illegitimate minor child, whether married or not, unable to maintain
itself, or
(c) his legitimate or illegitimate child (not being a married daughter) who has attained
majority, where such child is by reason of any physical or mental abnormality or injury
unable to maintain itself, or
(d) his father or mother, unable to maintain himself or herself,
a Magistrate of the first class may, upon proof of such neglect or refusal, order such
person to make a monthly allowance for the maintenance of his wife or such child, father
or mother, at such monthly rate not exceeding five hundred rupees in the whole as such
Magistrate thinks fit, and to pay the same to such person as the Magistrate may from time
to time direct :
Provided that the Magistrate may order the father of a minor female child referred to in
clause (b) to make such allowance, until she attains her majority, if the Magistrate is
satisfied that the husband of such minor female child, if married, is not possessed of
sufficient means.
Explanation. - For the purposes of this Chapter, -
(a) "minor" means a person who, under the provisions of the Indian Majority Act, 1875 (9
of 1875) is deemed not to have attained his majority;
(b) "wife" includes a woman who has been divorced by, or has obtained a divorce from,
her husband and has not remarried.
(2) Such allowance shall be payable from the date of the order, or if so ordered, from the
date of the application for maintenance.
(Emphasis supplied)
16. Bare reading of sub-section (1) of Section 125 leaves no room for doubt that if any
person having sufficient means, neglects or refuses to maintain his wife who is unable to
maintain herself or his legitimate (or illegitimate) child (children) unable to maintain
itself (themselves), or his father, or mother, unable to maintain himself or herself, a Court,
upon proof of negligence or refusal, order such person to pay maintenance to his wife or
child (children) or parents, as the case may be. It is also clear that maximum amount
which could be ordered to be paid was Rs. 500/- p.m. which was clear from the
expression "not exceeding Rs. 500/- in the whole".
17. It is further clear that under sub-section (2), such maintenance can be made payable
"from the date of order" or "if so ordered, from the date of the application for
maintenance".
18. By the Code of Criminal Procedure (Amendment) Act, 2001 (Act 50 of 2001), sub-
sections (1) and (2) came to be amended with effect from September 24, 2001. The
amended sub-sections now read thus :
125. Order for maintenance of wives, children and parents. - (1) If any person having
sufficient means neglects or refuses to maintain -
(a) his wife, unable to maintain herself, or
(b) his legitimate or illegitimate minor child, whether married or not, unable to maintain
itself, or
(c) his legitimate or illegitimate child (not being a married daughter) who has attained
majority, where such child is by reason of any physical or mental abnormality or injury
unable to maintain itself, or
(d) his father or mother, unable to maintain himself or herself,
a Magistrate of the first class may, upon proof of such neglect or refusal, order such
person to make a monthly allowance for the maintenance of his wife or such child, father
or mother, at such monthly rate, as such Magistrate thinks fit, and to pay the same to such
person as the Magistrate may from time to time direct :
Provided that the Magistrate may order the father of a minor female child referred to in
clause (b) to make such allowance, until she attains her majority, if the Magistrate is
satisfied that the husband of such minor female child, if married, is not possessed of
sufficient means.
Provided further that the Magistrate may,
@page-SC3010
during the pendency of the proceeding regarding monthly allowance for the maintenance
under this sub-section, order such person to make a monthly allowance for the interim
maintenance of his wife or such child, father or mother, and the expenses of such
proceeding which the Magistrate considers reasonable, and to pay the same to such
person as the Magistrate may from time to time direct :
Provided also that art application for the monthly allowance for the interim maintenance
and expenses of proceeding under the second proviso shall, as far as possible, be disposed
of within sixty days from the date of the service of notice of the application to such
person.
Explanation. - For the purposes of this Chapter, -
(a) "minor" means a person who, under the provisions of the Indian Majority Act, 1875(9
of 1875) is deemed not to have attained his majority;
(b) "wife" includes a woman who has been divorced by, or has obtained a divorce from,
her husband and has not remarried.
(2) Any such allowance for the maintenance or interim maintenance and expenses of
proceeding shall be payable from the date of the order, or, if so ordered, from the date of
the application for maintenance or interim maintenance and expenses of proceeding, as
the case may be.
(Emphasis supplied)
19. It is apparent that the ceiling which was fixed under the original enactment of 1973 of
Rs. 500/- p.m. has been removed and now it is open to a Court under the amended law to
fix such amount as it 'thinks fit'.
20. Again, there is no substantial change so far as the date of payment is concerned.
Under sub-section (2) as originally enacted, it was provided that such maintenance could
be made payable from the date of the order or if so ordered, from the date of application.
Even after the amendment of 2001, an order for payment of maintenance can be made by
a Court either from the date of the order or where an express order is made to pay
maintenance from the date of application, then the amount of maintenance can be paid
from that date, i.e. from the date of application.
21. So far as 'interim' maintenance is concerned, it is true that Section 125 of the Code as
it originally enacted did not expressly empower the Magistrate to make such order and
direct payment of interim maintenance. But the Code equally did not prohibit the
Magistrate from making such order. Now, having regard to the nature of proceedings, the
primary object to secure relief to deserted and destitute wives, discarded and neglected
children and disabled and helpless parents and to ensure that no wife, child or parent is
left beggared and destitute on the scrap-heap of society so as to be tempted to commit
crime or to tempt others to commit crime in regard to them, it was held that the
Magistrate had 'implied power' to make such order. The jurisdiction of the Magistrate
under Chapter IX (Order for Maintenance of Wives, Children and Parents) is not strictly
criminal in nature. Moreover, the remedy provided by Section 125 of the Code is a
summary remedy for securing reasonable sum by way of maintenance subject to a decree
passed by a, competent civil Court. Hence, in absence of any express bar or prohibition,
Section 123 could be interpreted as conferring power by necessary implication to make
interim order of maintenance subject to final outcome in the application.
22

. A direct question came up for consideration before this Court in Savitri v. Govind Singh
Rawat, (1985) 4 SCC 337 : 1986 Cri LJ 41. The Court considered that though there was
no specific provision for grant of interim maintenance, considering the object underlying
the provision and social purpose behind the legislation, such a power must be conceded
to the Court. AIR 1986 SC 984

23

. Speaking for the Court, Venkataramaiah, J. (as His Lordship then was) observed; Para 3
of AIR, Cri LJ
"It is true that there is no express provision in the Code which authorises a magistrate to
make an interim order directing payment of maintenance pending disposal of an
application for maintenance. The Code does not also expressly prohibit the making of
such an order. The question is whether such a power can be implied to be vested in a
magistrate having regard to the nature of the proceedings under Section 125 and other
cognate provisions found in Chapter IX of the Code which is entitled "Order For
Maintenance of Wives, Children and Parents". Section 125 of the Code confers power on
a
@page-SC3011
magistrate of the first class to direct a person having sufficient means but who neglects or
refuses to maintain (i) his wife, unable to maintain herself, or (ii) his legitimate or
illegitimate minor child, whether married or not, unable to maintain itself, or (iii) his
legitimate or illegitimate child (not being a married daughter) who has attained majority,
where such child is, by reason of any physical or mental abnormality or injury unable to
maintain itself or (iv) his father or mother, unable to maintain himself or herself, upon
proof of such neglect or refusal, to pay a monthly allowance for the maintenance of his
wife or such child, father or mothers as the case may be, at such monthly rate hot
exceeding five hundred rupees in the whole as such magistrate thinks fit. Such allowance
shall be payable from the date of the order, or, if so ordered from the date of the
application for maintenance".
24

. Interpreting the relevant provisions of the Code, putting emphasis on the duty of a
person liable to pay maintenance and applying the principle of 'social justice'. His
Lordship proceeded to state; AIR 1986 SC 984, Para 6

"In view of the foregoing it is the duty of the court to interpret the provisions in Chapter
IX of the Code in such a way that the construction placed on them would not defeat the
very object of the legislation. In the absence of any express prohibition, it is appropriate
to construe the provisions in Chapter IX as conferring an implied power on the magistrate
to direct the person against whom an application is made under Section 125 of the Code
to pay some reasonable sum by way of maintenance to the applicant pending final
disposal of the application. It is quite common that applications made under Section 125
of the Code also take several months for being disposed of finally. In order to enjoy the
fruits of the proceedings under Section 125, the applicant should be alive till the date of
the final order and that the applicant can do in a large number of cases only if an order for
payment of interim maintenance is passed by the court. Every court must be deemed to
possess by necessary intendment all such powers as are necessary to make its orders
effective. This principle is embodied in the maxim ubi aliquid conceditur, conceditur et id
sine quo res ipsa esse non potest (Where anything is conceded, there is conceded also
anything without which the thing itself cannot exist.) (Vide Earl Jowitt's Dictionary of
English Law, 1959 Edn., P. 1797). Whenever anything is required to be done by law and
it is found impossible to do that thing unless something not authorised in express terms
be also done then that something else will be supplied by necessary intendment. Such a
construction though it may not always be admissible in the present case however would
advance the object of the legislation under consideration. A contrary view is likely to
result in grave hardship to the applicant, who may have no means to subsist until the final
order is passed. There is no room for the apprehension that the recognition of such
implied power would lead to the passing of interim orders in a large number of cases
where the liability to pay maintenance may not exist. It is quite possible that such
contingency may arise in a few cases but the prejudice caused thereby to the person
against whom it is made is minimal as it can be set right quickly after hearing both the
parties. The magistrate, may, however, insist upon an affidavit being filed by or on behalf
of the applicant concerned stating the grounds in support of the claim for interim
maintenance to satisfy himself that there is a prima facie case for making such an order.
Such an order may also be made in an appropriate case ex parte pending service of notice
of the application subject to any modification or even an order of cancellation that may
be passed after the respondent is heard. If a civil court can pass such interim orders on
affidavits, there is no reason why a magistrate should not rely on them for the purpose of
issuing directions regarding payment of interim maintenance. The affidavit may be
treated as supplying prima facie proof of the case of the applicant. If the allegations in the
application or the affidavit are not true, it is always open to the person against whom such
an order is made to show that the order is unsustainable. Having regard to the nature of
the jurisdiction exercised by a magistrate under Section 125 of the Code, we feel that the
said provision should be interpreted as conferring power by necessary implication on the
magistrate to pass an order directing a person against whom an application is made under
it to pay a reasonable sum by way of interim maintenance subject to the other conditions
referred to there pending final disposal of the application".
(Emphasis supplied)
@page-SC3012
25. Parliament considered the object of the legislation, the decision of this Court in
Savitri and the fact that though the remedy is of a summary nature, the applicant who is
unable to maintain herself may have to wait for 'several years' for getting such relief. It,
therefore, amended the provision expressly authorizing the Magistrate to grant interim
maintenance.
26. In the Statement of Objects and Reasons, it was stated;
"It has been observed that an applicant, after filing application in a Court under Section
125 of the Code of Criminal Procedure, 1973, has to wait for several years for getting
relief from the Court. It is, therefore, felt that express provisions should be made in the
said Code for interim maintenance allowance to the aggrieved person under said Section
125 of the Code. Accordingly, it is proposed that during the pendency of the proceedings,
the Magistrate may order payment of interim maintenance allowance and such expenses
of the proceedings as the Magistrate considers reasonable, to the aggrieved person. It is
also proposed that the order be made ordinarily within sixty days from the date of the
service of the notice".
27

. In view of the decision of this Court in Savitri, in our opinion, the learned Magistrate
was right and wholly justified in ordering interim maintenance by an order dated
November 20, 1998. We see no infirmity in that part of the order and hold that interim
maintenance could have been granted by the learned Magistrate even before the
amendment of Section 125 in 2001. AIR 1986 SC 984
28. Regarding date from which such amount should be paid to the appellants, the Family
Court held that the appellants would be entitled to claim maintenance from the date of
application i.e. July 21, 1997.
29. The Family Court stated;
"This order will be effective from the date of application i.e. 21.7.1997. The opposite
party is directed to pay the arrears within three months of this order and shall pay the
current monthly amount of maintenance by 15th of every succeeding months."
(Emphasis supplied)
30. The Family Court thus exercised the power under sub-section (2) of Section 125
which enables the Court to make an order whether the applicant would be entitled to
maintenance from the date of the order or from the date of the application. The Family
Court ordered payment of maintenance from the date of application.
31. The High Court, however, set aside that part of the order of the Family Court. It, inter
alia, observed;
"On a consideration of the aforesaid arguments of the parties, this Court finds that the
court below has not considered the present matter in a proper manner and keeping in
view the purpose of the provisions of Section 125 of the Code. As Held in a catena of
decisions, the purpose of the said provision is to prevent vagrancy and destitution and
essentially to financially support the deserted wife or other to say that her own son has
grabbed the property and that she will sit back and will take no steps in the matter. As a
matter of fact, under Section 125 of the Code of Criminal Procedure itself, it is the duty
of the son to maintain his father and mother, if they are unable to maintain themselves;
whereas the court has not even considered the said fact. When the petitioner has raised
the issue that the opposite party has income from the land and house of her matrimonial
village, the same ought not to have been ignored by the Court in the manner, which has
been done. It raises the strong suspicion that the Court below had made up its mind to
disbelieve everything that was stated on behalf of the petitioner and believe the
contention of the opposite party, which is not the correct way of looking at the evidence
that comes in course of the said proceedings. It is for the court, in such matter, to consider
the probability of the facts and then to come to a fair conclusion as to what is the real
state of affairs. From the impugned order, it does not appear that any such attempt has
been made by the Court below and even the important admission made by the opposite
party No. 1 has been lost sight of by the Court below.
In the aforesaid view of the matter, this Court does not find that the Court below has
rightly looked into the aspect of the matter. The Court below has also not considered as to
what was the justification for passing an order for maintenance from the date of
application, which goes back to more than 9 years from the date of the order. As laid
down in the decision of this Court such an order may be necessitated if the party shows
the dire need of money for the
@page-SC3013
purpose of maintaining herself, for which she had to raise debts, during the period when
the application had been pending. There is no such material on the record, rather the
opposite party was getting interim maintenance from November, 1998 itself by order
dated 20.11.1998 although as a matter of fact the provision for interim maintenance has
been brought into existence for the first time by the Amendment Act, 2001 with effect
from 24.9.2001. However, since the said order is not under challenge, therefore, this
Court would not like to go into that issue any further. In any case, it is a relevant fact that
right from 1998, opposite party Nos. 1 and 2, have been paid interim maintenance, by
which they had managed to sustain themselves during that period and thus there is no
reason for passing the order to pay maintenance with effect from the date of application
going back more than 9 years from the date of passing of the said order."
(Emphasis supplied)
32. The above observations manifestly show that according to the High Court, there must
be justification on the part of the Court in making the order of maintenance from the date
of the application rather than from the date of the order. As there was no such reason
granting maintenance from the date of the application, the Family Court was not justified
in doing so. To that extent, therefore, the order passed by the Family Court was
vulnerable and accordingly, it was set aside by granting maintenance from the date of the
order passed by the Family Court.
33. Now, no direct decision of this Court is available on the point as to from which date a
Magistrate may order payment of maintenance to wife, children or parents. We may,
however, refer to decisions of some High Courts.
34. It seems that there is a cleavage of opinion on the question. According to one view,
since sub-section (2) of Section 125 declares that maintenance shall be payable "from the
date of the order", or, "if so ordered, from the date of application for maintenance",
normal rule is that a Magistrate should pass an order directing payment of maintenance
only from the date of the order. If he decides to deviate that course and makes an order
granting maintenance not from the date of the order but from the date of application for
maintenance, he must record reasons in support of such order [vide Mohd. Inaytullah
Khan v. Salma Bano, 1983 Jab LJ 55, Rameshwar v. Ramibai, 1987 Cri LJ 1952 (MP),
Lachhmani v. Ramu, (1983) 1 Crimes 590 (MP), Qamruddin v. Smt. Rashida, (1992) 1
WLC 305 (Raj), Shyamlal v. Mansha Bai, 1998 Cri LJ 2704 (Raj), Mohd. Ismail v.
Bilquees Bano, 1998 Cri LJ 2803 (All), Nitha Ranjan Chakraborty v. Smt. Kalpana
Chakraborty, 2002 Cri LJ 4768 (Cal), Samaydin v. State of U.P. and Anr., 2001 Cri LJ
2064 (All)].
35. The High Court, in the impugned order, also referred to a decision in Bijay Kapri v.
Smt. Kanishta Devi and Anr., (2000) 2 PLJR 241, wherein it was held that such order
could be necessitated if the party shows 'dire need' of the money for the purpose of
maintaining herself for which she had raised debts during the period when the application
had been pending. No such material had been brought on record. Rather, the applicants
were getting interim maintenance from November, 1998 by an order passed by the
Magistrate though such provision of interim maintenance had been brought in the statute
book for the first time by the Amendment Act, 2001 with effect from September 24,
2001.
36

. In Samaydin, the High Court of Allahabad observed that there may not be a discussion
of such circumstances which warranted the Court to allow it to grant maintenance from
the date of application. But, no other inference is permissible in the light of the language
of sub-section (2) of Section 125. The Court, by way of illustrative cases considered
certain situations, such as, 'dilatory tactics adopted by the husband in the disposal of the
proceeding', 'untold cruelty practised against wife', etc. In absence of special
circumstances, however, maintenance cannot be ordered from the date of application.
2001 Cri LJ 2064

37. Some other High Courts, have taken a contrary view. It was held that normally,
maintenance should be granted from the date of the application and not from the date of
the order. If the Magistrate is inclined to make an order granting maintenance from the
date of the order and not from the date of application, he should record reasons to do so.
38

. In Gnanaselvi and Ors. v. Illavarasan, (1999) 1 Crimes 22 (Mad), the High Court of
Madras observed that when the wife approaches a Court claiming maintenance by filing
1999 Cri LJ 1008
AIR 1988 SC 1208

@page-SC3014
application on the ground that she is not able to maintain herself, it is for her to prove
such inability from the date of application. Hence, when the Court ultimately decides
after conducting the inquiry that she is entitled to maintenance, the said decision must
necessarily be based upon the material showing that the wife was unable to maintain
herself when she filed an application. As a general rule, therefore, the Magistrate should
pass an order directing maintenance from the date of application. It was also observed
that the remedy is a speedy remedy and summary procedure is provided by the statute.
Despite this, usually, in such proceedings, the Court notices that the husband does not
allow the proceedings to go on by raising one objection or the other. The Court is
required to deal with all such objections, which takes time. Again, even after the order is
passed, the husband rushes to the higher forum and challenges it. Sometimes, he obtains
interim orders which results in further delay. The deserted wife and children are the
sufferers who seek shelter of the protective umbrella provided by Section 125 of the
Code. If maintenance is not granted from the date of application, the weaker sections are
sure to lose confidence in the justice delivery system. The Court noted the deep concern
expressed by this Court in P. N. Duda v. P. Shiv Shankar, (1988) 3 SCC 167, that "justice
cries in silence for long, far too long".
39. In Amarjit Kaur v. Sartaz Zingh, 1996 Cri LJ 4476 (PandH), the High Court of Punjab
and Haryana held that sub-section (2) of Section 125 does not require the Magistrate to
record special reasons for granting maintenance from the date of application. What it says
is that if the order is silent as to the date from which such maintenance is payable, it has
to be paid from the date of the order. Where, however, the maintenance is to be paid from
the date of the application itself, then there should be a specific order in that behalf by the
Court. There is nothing in the statutory provision to hold that the Magistrate must record
special reasons if he is to order that maintenance shall be payable from the date of
application.
40. In Krishna Jain v. Dharam Raj Jain, 1992 Cri LJ 1028 (MP), the Division Bench of
High Court of Madhya Pradesh considered the ambit and scope of sub-section (2) of
Section 125 in the light of other provisions of the Code. It overruled Mohd. Inaytullah
Khan, Rameshwar and Lachhmani referred to above and held that plain reading of sub-
section (2) of Section 125 makes it clear that allowance of maintenance can be awarded
from the date of the order or from the date of the application. To hold that, normally
maintenance should be made payable from the date of the order and not from the date of
the application unless such order is backed by reasons would amount to inserting
something more in the sub-section which the Legislature never intended. The Court
observed that it was unable to read in sub-section (2) laying down any rule to award
maintenance from the date of the order or that the grant from the date of the application is
an exception.
41. Regarding recording of reasons, the Bench observed that in either case i.e. grant of
maintenance from the date of the order or from the date of the application, the Court is
required to record reasons. The Court referred to sub-section (6) of Section 354 of the
Code which reads thus :
(6) Every order under Section 117 or subsection (2) of Section 138 and every final order
made under Section 125, Section 145 or Section 147 shall contain the point or points for
determination, the decision thereon and the reasons for the decision.
(Emphasis supplied)
42. It was, therefore, observed that every final order under Section 125 of the Code [and
other Sections referred to in sub-section (c) of Section 354] must contain points for
determination, the decision thereon and the reasons for such decision.
43. Our attention was also invited to a decision in K. Sivaram v. K. Mangalamba and
Ors., 1990 Cri LJ 1880 (AP). In K. Sivaram, a single Judge of the High Court of Andhra
Pradesh negatived the argument on behalf of the husband that the maintenance could be
awarded from the date of the order and such maintenance could be granted from the date
of the application only by recording special reasons. The Court held that it is the
discretion conferred on the Court by the Code to award maintenance either from the date
of the order or from the date of the petition as per the circumstances of the case. The
Code also noted that wherever Parliament wanted special reasons to be recorded for
passing a particular order, specific
@page-SC3015
provision has been made to that effect [See subsection (3) of Section 167 of the Code
(default bail), Section 361 (refusal to grant probation) etc].
44. In our considered opinion, the High Court is not right in holding that as a normal rule,
the Magistrate should grant maintenance only from the date of the order and not from the
date of the application for maintenance. And if he intents to pass such an order, he is
required to record reasons in support of such order. As observed in K. Sivaram, reasons
have to be recorded in both the eventualities. The Court was also right in observing that
wherever Parliament intended the Court to record special reasons, care had been taken to
make such provision by requiring the Court to record such reasons.
45

. Moreover, duration of litigation is not within the power or in the hands of the applicant
and entitlement to maintenance should not be left to the uncertain date of disposal of the
case. Keeping in view this hard reality, this Court in Savitri held that in absence of
prohibition to grant 'interim' maintenance such power could be read in the salutary
provision of Section 125 of the Code ensuring maintenance to wife unable to maintain
herself during the pendency of proceedings. Even Parliament took into account the reality
and by the Amendment Act, 2001 express provision has been made for the purpose.
AIR 1986 SC 984

46. Again, maintenance is a right which accrues to a wife against her husband the minute
the former gets married to the latter. It is not only a moral obligation but is also a legal
duty cast upon the husband to maintain his wife. Hence, whenever a wife does not stay
with her husband and claims maintenance, the only question which the Court is called
upon to consider is whether she was justified to live separately from her husband and still
claim maintenance from him? If the reply is in the affirmative, she is entitled to claim
maintenance. It is, therefore, open to the Magistrate to award maintenance from the date
of application and there is nothing which requires recording of 'special reasons' though he
must record reasons as envisaged by sub-section (6) of Section 354 of the Code in
support of the order passed by him.
47. We, therefore, hold that while deciding an application under Section 125 of the Code,
a Magistrate is required to record reasons for granting or refusing to grant main tenance
to wives, children or parents. Such maintenance can be awarded from the date of the
order, or, if so ordered, from the date of the application for maintenance, as the case may
be. For awarding maintenance from the date of the application, express order is
necessary. No special reasons, however, are required to be recorded by the Court. In our
Judgment, no such requirement can be read in sub- section (1) of Section 125 of the Code
in absence of express provision to that effect.
48. The last question relates to quantum of amount of maintenance. The Family Court
granted maintenance to the appellants - wife as well as daughter - at the rate of Rs. 2000/-
and Rs. 1000/- respectively from the date of application i.e. July 21, 1997. We have
reproduced the relevant part of Section 125 as originally enacted and as amended by the
Amendment Act, 2001. Before the amendment of 2001, the ceiling was Rs. 500/-. In our
opinion, therefore, the Family Court could not have granted maintenance exceeding Rs.
500/- p.m. either to appellant No. 1 or appellant No. 2 from the date of application i.e.
July 21, 1997. At the most, such an order could have been made effective from the date
the Amendment Act, 2001 came into force. To that extent, therefore, the order passed by
the Family Court was not in accordance with law.
49. But even on merits, the Family Court was not right in fixing the amount of
maintenance. The learned counsel for the respondent took us to the evidence adduced by
the parties. From the material on record, it is clear that the appellant No. 1-wife is
residing in the house belonging to the respondent-husband and such finding has been
recorded even by the Family Court. It is also in evidence that she was receiving income
from the land in her possession which belonged to her husband- respondent herein. It is
true that the respondent could not state as to the actual amount received by the wife from
the cultivation of the land. But it is also one of the considerations which is relevant and
material while fixing the amount of maintenance. Moreover, appellant No. 1 has inherited
some land from her father.
50. In view of overall facts and circumstances, in our opinion, ends of justice would be
served if we hold that both the appellants are entitled to an amount of Rs. 1000/ - each
per month as maintenance. As
@page-SC3016
already clarified, the appellants would be entitled to the said amount of maintenance from
the date the Amendment Act, 2001 came into force, i.e. September 24, 2001. So far as the
order of payment of 'interim' maintenance passed by the Magistrate is concerned, the
same was in consonance with law and no interference is called for.
51. For the foregoing reasons, the appeal deserves to be partly allowed and is accordingly
allowed to the extent indicated above.
Order accordingly.
AIR 2008 SUPREME COURT 3016 "Patel Rajnikant Dhulabhai v. Patel Chandrakant
Dhulabhai"
Coram : 2 C. K. THAKKER AND AFTAB ALAM, JJ.
Contempt Petition (C) Nos. 12-13 of 2006 in Spl. Leave Petn. (C) Nos. 7659-7660 of
2004, D/- 21 -7 -2008.
Patel Rajnikant Dhulabhai and Ors. v. Patel Chandrakant Dhulabhai and Ors.
(A) Contempt of Courts Act (70 of 1971), S.2(b) - CONTEMPT OF COURT -
DECLARATION OF TITLE - PARTITION - SUPREME COURT - Wilful disobedience
of order - Suit for declaration, partition of joint family property - Pendency - Supreme
Court by interim order totally prohibited/restrained contemners from creating any interest
in suit property in favour of third party "in any manner whatsoever" - Contemners created
interest in suit property in spite of said order - It amounts to wilful, intentional, deliberate
disobedience of order - Furthermore defence clearly found to be an afterthought and was
raised to avoid consequences of contempt proceedings. (Paras 36, 39, 45, 61, 62)
(B) Contempt of Courts Act (70 of 1971), S.2(b) - CONTEMPT OF COURT -
SUPREME COURT - Wilful disobedience of order - Interim order by Supreme Court that
any third party right which is created in suit property shall be done after notice to
petitioners - Contemners entered into transactions without issuing notice to petitioners -
Amounts to intentional disobedience and wilful breach of interim order - Defence that
Court had not directed 'prior' notice, therefore, non-issuance of notice before entering into
sale transactions would not amount to disobedience, held not tenable. (Paras 42, 44,
60, 62)
(C) Contempt of Courts Act (70 of 1971), S.12 - Civil P.C. (5 of 1908), S.94(c), O.39,
R.2A - Constitution of India, Art.129 - CONTEMPT OF COURT - INJUNCTION -
SUPREME COURT - Punishment for contempt of Court - Wilful and deliberate
disobedience of interim orders passed by Supreme Court - Can never be said to be bona
fide, honest or in good faith - Contemners held guilty u/S.12 read with S.94(c) and R.2A
of O.39 of C.P.C. and Art.129 of Constitution - So-called apology found to be not an act
of penitence, contrition or regret - Cannot be accepted - Further, in facts and
circumstances imposition of fine in lieu of imprisonment will not meet ends of justice -
Thus, contemners ordered to undergo simple imprisonment for a term of two weeks i.e.,
fourteen days. (Paras 62, 68, 69)
Cases Referred : Chronological Paras
2007 AIR SCW 1739 : AIR 2007 SC 1386 : 2007 Cri LJ 2147 (Ref.) 55
2007 AIR SCW 5577 : AIR 2007 SC 3100 : 2007 Lab IC 4133 (Ref.) 54
2006 AIR SCW 2475 : AIR 2006 SC 2007 : 2006 Cri LJ 2773 : 2006 (4) AIR Bom R 32
(Ref.) 65
2003 AIR SCW 5674 : AIR 2004 SC 105 : 2004 Cri LJ 1239 (Ref.) 50
2002 AIR SCW 1249 : AIR 2002 SC 1405 : 2002 Cri LJ 1814 (Ref.) 53
1999 AIR SCW 3182 : AIR 1999 SC 3215 : 1999 Lab IC 3234 (Ref.) 51
1997 AIR SCW 1285 : AIR 1997 SC 1240 (Ref.) 56, 58
1991 AIR SCW 2011 : AIR 1991 SC 1834 : 1991 Cri LJ 2648 (Ref.) 64
AIR 1984 SC 1374 : 1984 Cri LJ 993 (Ref.) 65
1974 AC 273 : (1973) 3 All ER 54 : (1973) 3 WLR 298 52
AIR 1960 Cal 1 63
AIR 1955 SC 19 : 1955 Cri LJ 133 (Ref.) 64
Shyam Divan, Sr. Advocate, Jay Savla, Ms. Reena Bagga and Akshit Gadnrik, for the
Contemners/Applicants; Uday Lalit, Sr. Advocate, Rameshwar Prasad Goyal, Sanjay
Kapur, Ms. Shubhra Kapur, Rajiv Kapur and Ms. Arti Singh, for Respondents.
Judgement
1. C. K. THAKKER, J. :-The present contempt petitions are filed by the petitioners
against respondent Nos. 1 to 3, alleged contemners, praying that they be held guilty of
'civil contempt' for violating orders passed by this Court on April 26, 2004 and on
January 10, 2005 and be punished accordingly. A prayer is also made
@page-SC3017
directing the contemners to deposit the amount received from third parties in
consideration of transfer of property effected by them during the period of interim orders
of this Court.
2. Shortly stated the facts of the case are that petitioners and respondent Nos. 1, 4 and 5
(in Special Leave Petitions) are real brothers and heirs and legal representatives of one
Dhulabhai Patel. It was the case of the petitioners that in 1961, one Chandulal Muljibhai
Parikh and Dhulabhai Patel (father of petitioners and respondent Nos. 1, 4 and 9)
purchased land bearing Revenue Survey Nos. 459/2, 464, 465, 466/1 and 466/2
admeasuring 6 acres and 9 gunthas of village Atladara, Taluka and District Baroda in the
State of Gujarat from one Parvatibai Ingle by a registered sale deed. According to the
petitioners, the amount of consideration was paid from the funds of Hindu Undivided
Family (HUF) of Dhulabhai Patel, but name of respondent No. 1 Chandrakant Dhulabhai
Patel was shown as the purchaser of the property being the eldest son of decdeased
Dhulabhai Patel along with Chandulal Muljibhai Parikh. Respondent Nos. 4 and 5 were
minors at that time. Respondent No. 1, Chandrakant Dhulabhai Patel was also studying in
a college and was having no source of income whatsoever. The entire amount was paid
by deceased-Dhulabhai. According to the petitioners, several documents revealed that the
property was managed by HUF of Dhulabhai Patel. In October, 1986, Dhulabhai died. No
partition by metes and bounds had been effected between the sons of deceased Dhulabhai
and the property continued to remain as HUF property. The petitioners used to manage
property after the death of Dhulabhai.
3. In 1990, a Memorandum of Understanding (MoU) was entered into between
respondent No. 1, Chandrakant Patel, being eldest son of Dhulabhai Patel family on one
hand and heirs of deceased Chandulal Parikh on the other hand whereunder it was agreed
that a portion of land towards western side would be treated as property of deceased-
Dhulabhai Patel and his family members.
4. In 1998, however, respondent No. 1 in conspiracy with third party and behind the back
of the petitioners and respondent Nos. 4 and 5 executed an agreement to sell the share of
HUF property belonged to the petitioners and other members of deceased- Dhulabhai
Patel. A collusive suit came to be filed by respondent Nos. 2 and 3 (partners of M/s. Om
Shivam Corporation) in the Court of Civil Judge (Senior Division), Baroda, being Special
Civil Suit No. 311 of 1999 for specific performance of agreement, dated January 18, 1998
against respondent No. 1 and within less than a month, a collusive and fraudulent consent
decree was passed. The petitioners were neither aware of the suit nor the decree passed
therein. It was only when a caveat was filed by M/s. Om Shivam Corporation, a
partnership firm that the petitioners suspected foul play. They, therefore, made an inquiry
and came to know about the suit and collusive decree. Immediately, they filed a suit
being Special Civil Suit No. 605 of 2002 in the Court of Civil Judge (Senior Divison),
Baroda for declaration, partition of joint family property and their share in the said
property. Along with the plaint, the petitioners filed an application (Exh. 5) under Order
XXXIX, Rules 1 and 2 read with Section 151 of the Code of Civil Procedure, 1908
(hereinafter referred to as 'the Code') for interim injunction restraining the defendants
from putting up any construction on the land admeasuring 1,43,000 sq. ft. towards
western side of the land bearing Revenue Survey Nos. 459/2, 464, 465, 466/ 1 and 466/2
and from entering into any transaction of sale, gift, mortgage or from dealing with the
property in any other manner or from handing over possession of the suit land to anyone
else. The trial Court issued notice to the defendants, but did not grant injunction as prayed
by the petitioners-plaintiffs. The petitioners challenged the said order by filing Appeal
from Order No. 140 of 2003 and the High Court of Gujarat vide its order dated May 2,
2003 directed the trial Court to dispose of the Application (Exh. 5) on merits within 15
days from the re-opening of the courts after summer vacation.
5. The trial Court thereafter heard the matter and dismissed the Application by refusing
interim injunction. The petitioners once again preferred an appeal against the said
decision in the High Court being Appeal From Order No. 241 of 2003. Along with
Memorandum of Appeal, the petitioners filed Civil Application No. 5083 of 2003 for
interim injunction. In paragraph 8 of the Civil Application, the petitioners prayed for
interim relief. The High Court by an order
@page-SC3018
dated July 23, 2003, granted ex parte ad interim relief in terms of paragraph 8(A).
6. The said paragraph read as under :
8. The applicants, therefore, pray that :
(A) Pending admission, hearing and final disposal of the aforesaid Appeal from Order,
the Hon'ble Court may be pleased to issue an injunction restraining opponents Nos. 1 to 3
herein, from putting up any construction on the suit land either themselves or through
their agents and/or servants and from disposing of the said property or creating any
interest therein in favour of a third party either by sale, mortgage, transfer, assignment,
gift or in any other manner whatsoever.
7. It is thus clear that though interim injunction was refused by the trial Court, the High
Court on July 23, 2003, granted the prayer of the petitioner and issued interim injunction
during the pendency and final disposal of Appeal from Order instituted by the petitioners-
appellants.
8

. The High Court then heard the matter and by judgment and order dated March 26, 2004
dismissed the appeal observing that considering the facts and circumstances of the case in
their entirety, the order passed by the trial Court below Application Exh. 5 called for no
interference. Interim relief which was granted earlier was ordered to be vacated.
Reported in AIR 2004 Guj 300

9. The learned counsel appearing for the petitioners-appellants in the High Court, at that
stage, made a prayer to the Court to continue interim relief granted earlier so as to enable
the petitioners-appellants to approach higher forum. After hearing the learned counsel for
the parties and noting objections raised by the other side, the Court continued interim
relief granted earlier for four weeks.
10

. Paragraphs 35 and 36 of the judgment which are relevant read as under : AIR 2004 Guj
300 at p. 309

35. Considering the entire facts and circumstances of the case and having regard to the
legal position concerning the point in issue, this Court is of the view that the impugned
order passed by the Trial Court below an application Exh. 5 does not call for any
interference in this Appeal From Order. Accordingly, this Appeal From Order is
dismissed. Interim relief granted earlier is vacated.
36. At this stage, Mr. A.J. Patel, the learned advocate appearing for the appellants
requested to continue the interim relief granted earlier by this Court for the period of six
weeks so as to enable the appellants to approach the higher forum. Mr. Bhatt and Mr.
Nanavati, learned advocate appearing for the respondent Nos. 1, 2 and 3 respectively
have strongly objected to the extension of interim relief. After considering their
submissions on this issue, the interim relief granted earlier is extended for four weeks
from today.
(Emphasis supplied)
11. Being aggrieved by the said order, the petitioners approached this Court by filing
Special Leave Petition (Civil) Nos. 7659-7660 of 2004. On April 26, 2004, this Court
issued notice and continued interim order passed by the High Court which was, as noted
earlier, in terms of prayer para 8(A). The order, dated April 26, 2004 passed by this Court
read as under :
The interim order already granted by the High Court will continue subject to further
orders of this Court."
12. Notices were served on the respondents who appeared. Affidavits and further
affidavits were thereafter filed and Special Leave Petitions were disposed of by this Court
on January 10, 2005.
13. The said order redd thus :
"The dispute in these special leave petitions pertains to the construction on certain
properties which are claimed to be the joint family properties. By the impugned interim
order, the High Court has permitted construction to be made on the suit properties. We
have considered the reasoning of the High Court. While not affirming the correctness of
the prima facie opinion expressed therein, we are of the view that the interest of the
parties will be adequately protected if a conditional order is passed. The special leave
petitions, are, accordingly, disposed of by making it clear that any construction which is
made by the respondent-purchasers on the disputed properties will be subject to the
outcome of the suit. Any third party right which is created shall be done after notice to the
petitioners. The pendency of the proceedings shall also be notified to each of the third
parties so involved. The Trial Court is directed to dispose of the suit expeditiously,
preferably within a period of six months from the date
@page-SC3019
of the communication of this order.
(Emphasis supplied)
14. The allegation of the petitioners in the present Contempt Petitions is that in spite of
interim order passed by this Court on April 26, 2004, as modified by final order dated
January 10, 2005, the respondent Nos. 1 to 3 (contemners) dealt with and transferred the
suit property by entering into agreements, accepted the amount by way of consideration,
executed sale-deeds in favour of third parties without notice to the petitioners, allowed
construction to be made and thereby committed wilful disobedience and intentional
violation of the order of the Court and they are liable to be punished in accordance with
law.
15. On these petitions, notices were issued on February 6, 2006. The respondents
appeared and filed affidavits. Having satisfied that the pleadings were complete, the
Registry was directed to place the matters for final disposal. That is how the matters have
been placed before us.
16. The learned counsel for the petitioners contended that in spite of the orders passed by
this Court, the contemners sold the property, received consideration and executed sale-
deeds. No notice was given to them about the transactions before they were entered into.
The orders of this Court were abundantly clear. The first order dated April 26, 2004,
totally restrained the respondents from taking any action during the pendency of the
proceedings. In spite of such clear-cut order, properties were sold and several other
actions were taken by the contemners and they are liable to be punished for committing
contempt of Court. It was also submitted that even under the order dated January 10,
2005, when Special Leave Petitions were disposed of and earlier interim order was
modified, the contemners had acted in violation of the directions. And on that ground
also, they are liable under the Contempt of Courts Act.
17. The learned counsel for the petitioners, in this connection, referred to agreements to
sell as well as sale-deeds and acceptance of part payment or full payment of consideration
during the intervening period between July 23, 2003 when the High Court granted interim
relief in terms of para 8(A) and also orders dated April 26, 2004 and the final order dated
January 10, 2005 by this Court. It was urged that though the suit was pending before the
trial Court and the matter was sub judice, in agreements to sell as also in sale-deeds an
express and unequivocal statement was made by the contemners that their title to the
property is 'clear' and 'marketable'. It was ordered by the Court on January 10, 2005 that
if any third party right would be created, it "shall be done after notice to the petitioners".
No such notice, however, was given to the petitioners. It was only after the properties
were sold that a 'pursis' was filed in the trial Court stating therein that certain properties
were sold by the defendants. Thus, there was clear breach of orders of this Court and the
contermners are liable to be punished for violating the interim orders.
18. The learned counsel for the respondents-contemners, on the other hand, submitted
that the Contempt Petitions are liable to be dismissed. The respondents had not willfully
and intentionally violated interim orders of this Court. No doubt, the High Court, granted
interim injunction in Civil Application in Appeal from Order on July 23, 2003 but finally
Appeal from Order itself was dismissed. Similarly, this Court on April 26, 2004
continued interim relief but ultimately, dismissed Special Leave Petitions on January 10,
2005 by modifying earlier interim order. It was also submitted that this Court never
ordered to give 'prior' notice to the petitioners before entering into any agreement or
executing sale-deed. It was, therefore, not necessary for the respondents either to issue
notice or to intimate petitioners before entering into any transaction. But in certain cases,
even 'prior' notice of transaction was given to the petitioners by the respondents; though
in some cases, such notice was given after entering into agreements/transactions but the
same could not be said to be violative of the orders passed by this Court. Alternatively, it
was submitted that even if the Court finds that there is 'technical' breach of the orders of
this Court, the respondents have tendered unconditional and unqualified apology which
may be accepted by this Court and contempt proceedings may be dropped against them
by showing magnanimity and taking lenient view.
19. We have given most anxious and thoughtful consideration to the rival contentions of
the parties. We have also gone
@page-SC3020
through the relevant record and orders passed by the High Court as also by this Court. We
have perused affidavits and further affidavits filed by the parties to these petitions. We
have carefully gone through various documents on record.
20. We have reproduced both interim orders passed by this Court; (i) order, dated April
26, 2004, and (ii) order, dated January 10, 2005. Under the first order, larger relief was
granted, obviously because at that stage, the Court on prima facie satisfaction of the case,
issued notice to the respondents. Moreover, even the High Court which had granted
interim relief on July 23, 2003, continued the said relief (though Appeal from Order was
dismissed and interim relief was vacated) so as to enable the aggrieved parties to
approach this Court. On January 10, 2005, however, this Court disposed of Special Leave
Petitions. But taking note of the pendency of the main matter (suit), the Court did not
totally vacate interim relief but modified it by imposing certain conditions. The question
before us is whether the contemners had violated both the orders or any of the two orders
and whether such violation or disobedience was wilful' or intentional as alleged by the
petitioners. If so, what punishment should be imposed on the contemners and what
should be the final order in these contempt petitions.
21. Before proceeding with the consideration of rival contentions, it may be stated for the
completion of record that Special Civil Suit No. 605 of 2002 instituted by the petitioners
herein (plaintiffs) was decreed by the IXth Additional Senior Civil Judge, Vadodara on
May 23, 2006 and it was declared that plaintiff Nos. 1 and 2 and defendant Nos. 1, 4 and
5 each has 3/ 16 share in the suit property and defendant Nos. 7, 8 and 9 each has 1/48
share in the said property. Preliminary decree was ordered to be drawn up accordingly. It
was also stated that the defendants against whom the decree is passed have challenged the
decree by filing First Appeal in the High Court and the matter is sub judice.
22. Now, it is the case of the petitioners that in spite of interim orders of this Court, dated
April 26, 2004 and January 10, 2005, in gross violation, wilful disobedience and
intentional breach thereof, the contemners had sold certain properties without notice to
the petitioners. It was stated that the contemners executed a sale-deed in favour of
Madhuben Rohit and Jasodaben Thakor on August 30, 2005 for consideration of Rs.
3,30,000/-. The contemners accepted a cheque of Rs. 2,00,000/- bearing No. 531526,
dated October 15, 2004. Similarly, an agreement to sell was executed in favour of
Hemlataben Shah for Rs. 3,00,000/- on May 26, 2005 and the entire amount was received
by three cheques; cheque No.719372, dated April 15, 2004; cheque No.719374, dated
April 22, 2004 and cheque No. 216684, dated April 29, 2004. Again, a sale deed in favour
of Smt. Ranjit Gulati was executed by accepting a cheque of Rs.51,000/- dated
September 10, 2003, a cheque of Rs. 1,00,000/- dated October 8, 2004 and a cheque of
Rs.1,39,000/- dated April 26, 2004. By producing additional documents on record, the
petitioners have stated that sale-deeds were executed by the contemners in favour of
Jayesh Natwarlal Parikh on August 18, 2005; Hemaben Jayeshbhai Parikh on August 18,
2005, Rashmikaben Navinchandra Desai on August 30, 2005 and Tolaram Radharam
Popat on April 25, 2005. All these transactions were entered into in gross violation and
breach of the orders of the Court and the contemners may, therefore, be punished for
committing contempt of Court.
23. The respondents have filed affidavits denying the allegations levelled by the
petitioners and asserting that they have not committed any act in disobedience of the
order of the Court and the contempt petitions are liable to be dismissed.
24. Respondent No. 1 (Chandrakant Dhulabhai), in his affidavit, dated June 16, 2006
(filed on August 17, 2006) stated that the contempt petitions are 'primarily' directed
against respondent Nos. 2 and 3 who are alleged to have violated the orders of this Court.
It was stated that the orders were required to be complied with by respondent Nos. 2 and
3 and were not directed against respondent No. 1. It was also stated that the respondent
No. 1 is 67 years old and has been impleaded just to harass him without making any
averment/allegation against him. It is, therefore, prayed to dismiss contempt petitions
against him.
25. A counter-affidavit is also filed by respondent Nos. 2 and 3 controverting the
averments and denying the allegations of the petitioners in the Contempt Petitions. It was
stated that during the operation of interim order dated April 26, 2004, they had
@page-SC3021
neither executed any sale deed nor created third party interest in the suit property. The
cheques said to have been accepted by them during interim orders represented the amount
paid towards consideration of the apartment booked prior to the issuance of interim
orders.
26. Regarding the allegation that the title of the suit property was described as 'clear and
marketable', it was stated that as per the order of the Court, what was required to be
mentioned was the pendency of the proceedings. In all the sale deeds, executed by the
respondents, specific recital was made that the transaction was subject to pendency of
civil suit and final decision therein.
27. Regarding the third allegation of not giving notice to the petitioners, it was stated that
no 'prior' notice was required as per the order of this Court. It was stated :
a. in order to protect the interest of the parties, this Hon'ble Court passed a conditional
order stating inter alia "any third party rights which is created shall be done after notice to
the petitioners". This Hon'ble Court has not used the word "prior" as alleged. This
Hon'ble Court had desired that as and when third party rights are created, the petitioners
should be duly informed. The purpose of such information was to enable the petitioners
to protect their interest by impleading such third parties in the pending suit.
b. Admittedly, initially, by pursis dated 1.3.2005, the petitioners were given prior
intimation about the creation of third party rights. However, subsequently it was found
that out of the 11 sale deeds which were proposed to be executed and about which prior
intimation by pursis dated 1.3.2005 was given - 7 sale deeds could not be executed for
quite some time, as the deal could not be finalized. Further, before executing the sale
deeds, 2 buyers/third parties/ allottees got the deals transferred in some other name, viz.
name of their wife/child.
28. Hence, as per legal advice obtained, the intimation was being submitted before the
Hon'ble Civil Court after taking endorsement of Advocate for the petitioner.
29. It was further stated by respondent Nos. 2 and 3 that originally land bearing Survey
Nos. 459/2, 464, 465, 466/ 1 and 466/2, admeasuring 26.608 Sq.M. was jointly purchased
by Chandrakant Dhulabhai Patel and Chandulal Muljibhai Parikh in 1961. In 1990,
however, an understanding had been arrived at between the heirs and legal
representatives of Chandulal Parikh and Chandrakant Dhulabhai Patel and as per
partition, 50% land situated on the western side (Part 'A') fell to the share of Chandrakant
Patel whereas 50% land on the eastern side (Part 'B') came to the share of Parikh family.
It was further stated that respondent Nos. 2 and 3 entered into Development Agreement
with Parikh family for 50% eastern side land (Part 'B') which came to the share of Parikh
family and also with respondent No. 1 Chandrakant Patel for western side of land (Part
'A'). The scheme was thus jointly promoted for part 'A' land and part 'B' land. In course of
time, property 'A' became disputed property, but there was no dispute as to property of
part 'B'. It was also stated that part 'B' property abuts 30 metres wide road while part 'A'
property abuts 40 meters wide T.P. Road. Thus, part 'A' property (suit property) was
having better location and higher commercial value.
30. Respondent Nos. 2 and 3 admitted that Madhuben Rohit and Jasodaben Thakor
deposited Rs. two lacs on October 15, 2004 for booking one of the shops which was to be
constructed on part 'B' property. But after the disposal of Special Leave Petitions on
January 10, 2005, the injunction came to an end regarding part 'A' property. Under the
circumstances, request of Madhuben and Jasodaben for allotment of shop in part 'A'
property instead of part 'B' property was favourably considered and accepted. Cases of
Hemlataben Shah, Rashmikaben and Ranjit Gulati were identical. Sale-deed in favour of
Dr. Tolaram Popat was executed on January 24, 2005, i.e. after final disposal of SLP and
there was no interim injunction at that time.
31. It was, therefore, submitted that respondent Nos. 2 and 3 had not created any right in
favour of third party during the operation of interim order and there was no question of
taking proceedings under the Contempt of Courts Act.
32. Regarding notice to petitioners, the respondents stated that pursis, Ex. 88 was filed in
the trial Court wherein details had been supplied in respect of 11 transfer deeds which
were to be executed. Thereafter, again pursis, Ex. 106 was filed on September 01, 2005 in
which all details were
@page-SC3022
given. Thus, they have not violated the order of this Court.
33. Finally, it was stated;
"Without prejudice to the above, the respondents tender their unconditional apology to
this Hon'ble Court."
34. In affidavit-in-rejoinder, the petitioners have stated that after examining oral and
documentary evidence, the competent Court passed a decree in favour of plaintiffs
declaring shares of plaintiffs and defendants. Meanwhile, however, the respondents
disposed of several properties. It was reiterated that illegally and with mala fide intention,
the respondents created third party interest in gross defiance of interim orders and by
making inconsistent and contradictory statements . A false statement was made by
respondents in sale deeds and in agreements to sell that the title of the respondents over
the property was 'clear and marketable'. As to properties part 'A; and 'B', it was stated that
the contention was wholly irrelevant inasmuch as the trial Court decreed the suit by
holding the plaintiffs as joint owners of the property.
35. Further affidavit was thereafter filed by respondent Nos. 2 and 3 on February 25,
2008 on the question of title deeds. It was stated that in sale deeds, a statement was made
that "third party has no right, title, interest or claim and the property (subject-matter of
sale deed) was not under attachment". But it was also stated that the suit was pending and
the sale-deeds were executed subject to final decision in the suit. Thus, the respondents
had acted bona fide They have not only notified in the sale-deeds the pendency of civil
suit but also have gone a step further and stated that the sale-deeds were being executed
subject to final decision in the pending suit. It was, therefore, submitted that the
Contempt Petitions should be dismissed.
36. From the facts stated above, it is proved that there is breach of interim orders passed
by this Court. So far as the defence as to properties falling in parts 'A' and 'B' is
concerned, in our opinion, it is clearly an afterthought and the plea has been put forward
without there being anything on record. Though it was stated that initially, Madhuben,
Hemlataben, Rashmikaben and Ranjit Gulati were given shops in part 'B' property, no
such agreements/deeds have been placed on record. It was then stated that after interim
relief was vacated and request was made by them to allot them shops in part 'A' property,
no evidence/material has been adduced by the contemners. A stereo-type affidavits have
been filed sworn on one and the same day, July 29, 2006 after contempt notice was
served upon the respondents. Even the sale-deeds do not recite part 'A' or 'B' of the
property or the fact that earlier the allottee was granted shop in part 'B' property but after
the disposal of the Special Leave Petitions, request was made by the purchaser to convert
the allotment from part 'B' property to part 'A' property. The so-called defence, therefore,
is apparently to avoid consequences of contempt proceedings.
37. Moreover, the defence does not appear to be probable. Normally, no reasonable and
prudent man/woman who purchases immovable property with a 'clear' title would request
the owner of the property to allot him/her property over which his title is not clear, which
is the subject-matter of litigation, for which a suit is pending and the matter is sub judice
in a Court of law.
38. Further, the pleas advanced by respondent Nos. 2 and 3 are inconsistent, conflicting
and irreconcilable. On the one hand, it was asserted that during the operation of the
interim order, dated April 26, 2004, respondent Nos. 2 and 3 had not created third party
interest in the suit property and the cheques said to have been accepted by them during
the pendency of proceedings 'represented the amount paid towards consideration of the
apartment booked prior to the issuance of the interim orders,' while on the other hand, it
was stated that initial booking was for shops to be constructed on part 'B' property, but
subsequently, after dismissal of Special Leave Petitions, at the request of purchasers,
allotment was converted from part 'B' property to part 'A' property. If it were so, there
was no question of raising a plea that the acceptance of cheques during the pendency of
the interim orders represented the amount of consideration booked prior to the issuance
of interim orders. If initial booking and payment was in relation to part 'B' property, it
was immaterial and altogether irrelevant whether it was prior or subsequent to filing suit
by the plaintiffs or grant of interim orders either by the High Court or by this Court.
@page-SC3023
39. We are, therefore, fully convinced that during the pendency of the proceedings and in
spite of interim orders passed by this Court, agreements have been entered into by the
contemners, cheques had been accepted and consideration had been received at least in
part. So far as the first order passed by this Court on April 26, 2004 is concerned, there
was total prohibition from creating any interest in favour of third party either by sale,
mortgage, transfer, assignment, gift or 'in any other manner whatsoever'. Hence, entering
into an agreement or acceptance of full or even part consideration would be hit by the
said order. In our considered view, it would amount to 'creation of interest' prohibited by
this Court.
40. The matter did not end there. Special Leave Petitions were filed in this Court by the
petitioners against an interim order not granting injunction below Application, Ex. 5.
Though the High Court granted such injunction, but it was vacated at the time of
dismissal of Appeal from Order. But all the parties to the suit were aware that the main
matter (suit) was pending and rights of contesting parties in the suit-property were yet to
be decided. In the circumstances, 'injunction' or 'no injunction', the title of the defendants
was 'under challenge'. It was 'cloudy' and unless and until the suit is decided, it cannot be
said that they had 'clear and marketable title'. Though the legal position is clear and
beyond controversy, we find that an express, unambiguous and unequivocal statement is
made by the contemners in all sale-deeds that their title to the property is 'clear and
marketable'. It was also stated that they were independent owners, possessors and
occupiers of the property and there was no right, interest, part share, claim of anybody
else therein.
41. It was, no doubt, submitted by the learned counsel for the contemners that it was
specifically and clearly stated that a suit was pending in the Court of Civil Judge (Senior
Division), Vadodara and it was also clarified that the documents were executed subject to
the final decision in the suit. It was further submitted that if the owner of the property
does not state that he has title over the property he seeks to transfer and that such title is
'clear and marketable', no reasonable and prudent man would come forward to purchase
such property. To this extent, the learned counsel is right. That, however, does not mean
that in such case, a person whose title is challenged and the proceedings are pending will
be permitted to say that his title is 'clear and marketable' and there is no impediment on
the contemners from transferring the property in favour of purchasers.
42. Finally, in any case, there is clear breach and violation of the order of this Court, i.e.
order, dated January 10, 2005 so far as 'notice' to the petitioners is concerned. It is not
disputed even by the contemners that in some cases, notice as to creation of third party
interest had been given after creation of such interest by filing pursis in the Court.
43. The learned counsel for the contemners half-heartedly submitted that this Court had
not ordered that 'prior' notice should be given to the petitioners regarding creation of third
party rights and hence, there was no violation of the direction of the Court.
44. We are unable to agree with the submission. This Court has stated; "Any third party
right which is created shall be done after notice to the petitioners." In our view, the
direction has only one meaning and it is that third party interest can only be created after
notice to the petitioners. Admittedly, that was not done, at least, in few cases. It is,
therefore, clear that there is violation and disobedience of the orders of the Court and the
contemners are responsible for such act.
45. From the overall considerations of the matter in the light of series of events, we hold
that the respondents-contemners have disregarded and violated the orders passed by this
Court on April 26, 2004 and January 10, 2005.
46. The next question is whether for disobedience of the order passed by this Court, the
respondents/contemners are liable to punishment? In this connection, we may refer to
some of the legal provisions. Article 129 of the Constitution declares this Court (Supreme
Court) to be "a Court of Record having all the powers of such a Court including the
power to punish for the contempt of itself". Clause (c) of Section 94 of the Code of Civil
Procedure, 1908 enacts that in order to prevent the ends of justice from being defeated,
the Court may, commit the person guilty of disobedience of an order of
@page-SC3024
interim injunction to civil prison and direct his property be attached and sold. Rule 2A of
Order XXXIX as inserted by the Code of Civil Procedure (Amendment) Act, 1976 (Act
104 of 1976) reads thus :
2A. Consequence of disobedience or breach of injunction - (1) In the case of
disobedience of any injunction granted or other order made under rule 1 or rule 2 or
breach of any of the terms on which the injunction was granted or the order made, the
Court granting the injunction or making the order, or any Court to which the suit or
proceeding is transferred, may order the property of the person guilty of such
disobedience or breach to be attached, and may also order such person to be detained in
the civil prison for a term not exceeding three months, unless in the meantime the Court
directs his release.
(2) No attachment made under this rule shall remain in force for more than one year, at
the end of which time, if the disobedience or breach continues, the property attached may
be sold and out of the proceeds, the Court may award such compensation as it thinks fit to
the injured party and shall pay the balance, if any, to the party entitled thereto.
47. The provisions of the Contempt of Courts Act, 1971 have also been invoked. Section
2 of the Act is a definition clause. Clause (a) enacts that contempt of court means 'civil
contempt or criminal contempt'. Clause (b) defines 'civil contempt' thus :
(b) 'civil contempt' means wilful disobedience to any judgment, decree, direction, order,
writ or other process of a court or wilful breach of an undertaking given to a court.
48. Reading of the above clause makes it clear that the following conditions must be
satisfied before a person can be held to have committed a civil contempt;
(i) there must be a judgment, decree, direction, order, writ or other process of a Court (or
an undertaking given to a Court);
(ii) there must be disobedience to such judgment, decree, direction, order, writ or other
process of a Court (or breach of undertaking given to a Court); and
(iii) such disobedience of judgment, decree, direction, order, writ or other process of a
Court (or breach of undertaking) must be wilful.
49. Section 12 provides punishment for contempt of Court. The relevant part of the
provision reads thus :
S. 12 - Punishment for contempt of court - (1) Save as otherwise expressly provided in
this Act or in any other law, a contempt of court may be punished with simple
imprisonment for a term which may extend to six months, or with fine which may extend
to two thousand rupees, or with both :
Provided that the accused may be discharged or the punishment awarded may be remitted
on apology being made to the satisfaction of the court.
Explanation. - An apology shall not be rejected merely on the ground that it is qualified
or conditional if the accused makes it bona fide.
(2) Notwithstanding anything contained in any law for the time being in force, no court
shall impose a sentence in excess of that specified in sub-section (1) for any Contempt
either in respect of itself or of a court subordinate to it.
(3) Notwithstanding anything contained in this section, where a person is found guilty of
a civil contempt, the court, if it considers that a fine will not meet the ends of justice and
that a sentence of imprisonment is necessary shall, instead of sentencing him to simple
imprisonment, direct that he be detained in a civil prison for such period not exceeding
six months as it may think fit.
50

. In Ashok Paper Kamgar Union v. Dharam Godha and Ors., (2003) 11 SCC 1, this Court
had an occasion to consider the concept of 'wilful disobedience' of an order of the Court.
It was stated that 'wilful' means an act or omission which is done voluntarily and with the
specific intent to do something the law forbids or with the specific intent to fail to do
something the law requires to be done, that is to say, with bad purpose either to disobey
or to disregard the law. According to the Court, it signifies the act done with evil intent or
with a bad motive for the purpose. It was observed that the act or omission has to be
judged having regard to the facts and circumstances of each case. 2003 AIR SCW 5674

51

. In Kapildeo Prasad Sah and Ors. v. State of Bihar and Ors., (1999) 7 SCC 569, it was
held 1999 AIR SCW 3182

@page-SC3025
that for holding a person to have committed contempt, it must be shown that there was
wilful disobedience of the judgment or order of the Court. But it was indicated that even
negligence and carelessness may amount to contempt. It was further observed that
issuance of notice for contempt of Court and power to punish are having far-reaching
consequences, and as such, they should be resorted to only when a clear case of wilful
disobedience of the court's order is made out. A petitioner who complains breach of
Court's order must allege deliberate or contumacious disobedience of the Court's order
and if such allegation is proved, contempt can be said to have been made out, not
otherwise. The Court noted that power to punish for contempt is intended to maintain
effective legal system. It is exercised to prevent perversion of the course of justice.
52. In the celebrated decision of Attorney General v. Times Newspaper Ltd.; 1974 AC
273 : (1973) 3 All ER 54 : (1973) 3 WLR 298; Lord Diplock stated :
"There is an element of public policy in punishing civil contempt, since the
administration of justice would be undermined if the order of any court of law could be
disregarded with impunity."
53

. In Anil Ratan Sarkar and Ors. v. Hirak Ghosh and Ors., (2002) 4 SCC 21, this Court
held that the Contempt of Courts Act has been introduced in the statute-book for securing
confidence of people in the administration of justice. If an order passed by a competent
Court is clear and unambiguous and not capable of more than one interpretation,
disobedience or breach of such order would amount to contempt of Court. There can be
no laxity in such a situation because otherwise the Court orders would become the subject
of mockery. Misunderstanding or own understanding of the Court's order would not be a
permissible defence. It was observed that power to punish a person for contempt is
undoubtedly a powerful weapon in the hands of Judiciary but that by itself operates as a
string of caution and cannot be used unless the Court is satisfied beyond doubt that the
person has deliberately and intentionally violated the order of the Court. The power under
the Act must be exercised with utmost care and caution and sparingly in the larger interest
of the society and for proper administration of justice delivery system. Mere disobedience
of an order is not enough to hold a person guilty of civil contempt. The element of
willingness is an indispensable requirement to bring home the charge within the meaning
of the Act. 2002 AIR SCW 1249

54

. In Commissioner, Karnataka Housing Board v. C. Muddaiah, (2007) 7 SCC 689, one of


us (C.K. Thakker, J.) observed that once a direction is issued by a competent Court, it has
to be obeyed and implemented without any reservation. If an order passed by a Court of
Law is not complied with or is ignored, there will be an end of Rule of Law. If a party
against whom such order is made has grievance, the only remedy available to him is to
challenge the order by taking appropriate proceedings known to law. But it cannot be
made ineffective by not complying with the directions on a specious plea that no such
directions could have been issued by the Court. Upholding of such argument would
seriously affect and impair administration of justice. 2007 AIR SCW 5577

55

. In All Bengal Excise Licensees Association v. Raghabendra Singh and Ors., (2007) 11
SCC 374, this Court considered several cases and observed that wilful and deliberate act
of violation of interim order passed by a competent Court would amount to contempt of
Court. 2007 AIR SCW 1739

56
. A reference in this connection may also be made to a decision of this Court in Tayabbhai
M. Bagasarawala v. Hind Rubber Industries (P) Ltd., (1997) 3 SCC 443. In that case, the
plaintiff-landlord filed a suit against the defendant-tenant in the City Civil Court for
permanent injunction restraining the defendant from carrying on construction in the suit
premises. Ad interim injunction was granted by the Court. Defendant's application for
vacating injunction was dismissed. The defendant, however, committed breach of
injunction. The plaintiff, hence, filed an application under Order XXXIX, Rule 2-A of the
Code. The defendant came forward and raised an objection as to jurisdiction of the Court
and power to grant injunction. The High Court, ultimately, upheld the objection and ruled
that City Civil Court had no jurisdiction to entertain the suit. It was, therefore, argued by
the defendant that he cannot be punished for disobedience of an order passed by a Court
which had no jurisdiction 1997 AIR SCW 1285

@page-SC3026
to entertain a suit or to grant injunction. The High Court upheld the contention. The
plaintiff approached this Court.
57

. This Court observed that until the question of jurisdiction had been decided, the City
Civil Court possessed power to make interim orders. The Court could also enforce them.
A subsequent decision that the Court had no jurisdiction to entertain the suit did not
render interim orders passed earlier non est or without jurisdiction. A party committing
breach of such orders could not escape the consequences of such disobedience and
violation thereof. Accordingly, the Court held the defendant guilty for intentionally and
deliberately violating interim order and convicted him under Rule 2-A of Order XXXIX
of the Code and sentenced him to one month's imprisonment. 1997 AIR SCW 1285,
Para 16

58. Speaking for the Court, Jeevan Reddy, J. stated :


"Can it be said that orders passed by the Civil Court and the High Court during this
period of six years were all non est and that it is open to the defendants to flout them
merrily, without fear of any consequence. The question is whether the said decision of the
High Court means that no person can be punished for flouting or disobeying the
interim/interlocutory orders while they were in force, i.e., for violations and disobedience
committed prior to the decision of the High Court on the question of jurisdiction. Holding
that by virtue of the said decision of the High Court (on the question of jurisdiction), no
one can be punished thereafter for disobedience or violation of the interim orders
committed prior to the said decision of the High Court, would indeed be subversive of the
Rule of Law and would seriously erode the dignity and the authority of the courts.
(Emphasis supplied)
59. From the above decisions, it is clear that punishing a person for contempt of Court is
indeed a drastic step and normally such action should not be taken. At the same time,
however, it is not only the power but the duty of the Court to uphold and maintain the
dignity of Courts and majesty of law which may call for such extreme step. If for proper
administration of justice and to ensure due compliance with the orders passed by a Court,
it is required to take strict view under the Act, it should not hesitate in wielding the potent
weapon of contempt.
60. Now, in the instant case, both the orders passed by this Court on April 26, 2004 and
January 10, 2005, were explicitly clear. The first order totally prohibited/restrained the
respondents/contemners from creating any interest whatsoever in the suit property. As
held by us, in spite of the said order, interest had been created by the contemners in the
suit property. But even otherwise there is intentional disobedience and wilful breach of
the subsequent order dated January 10, 2005 inasmuch as transactions had been entered
into without issuing notice to the petitioners. We have already held that they could not
have been entered into by the respondents before issuance of notice to the petitioners. The
respondents were clearly aware of the order. In fact, the action was sought to be defended
and justified on the ground that the Court had not directed 'prior' notice, and as such, non-
issuance of notice before entering into sale transactions would not amount to
disobedience of the order of the Court. We are unable to uphold the contention. In the
circumstances it must be held that the disobedience of the order by the contemners was
wilful, intentional and deliberate.
61. The question then is whether the case calls for imposition of punishment on the
contemners. The learned counsel for the contemners submitted that in the affidavit-in-
reply, the respondents have stated that if this Court comes to the conclusion that they had
committed contempt of Court, the Court may accept unconditional and unqualified
apology and may discharge notice. The counsel submitted that the statutory provision
itself enacts that no apology shall be rejected merely on the ground that it is qualified or
conditional [Explanation to Section 12(1)].
62. We must frankly admit our inability to agree with the learned counsel. In the light of
what is stated above, we are convinced that the contemners have intentionally and
deliberately violated the orders of the Court. We are also convinced that the orders were
clear, unambiguous and unequivocal having one and only one meaning. Wilful and
deliberate disobedience of the orders passed by the Apex Court of the country can never
be said to be bona fide, honest or in good faith. If it is so, the action calls for serious view
to ensure proper administration of justice.
63. In Hiren Bose, Re, AIR 1969 Cal 1 : 72 Cal WN 82, the High Court of Calcutta
@page-SC3027
stated :
"It is also not a matter of course that a Judge can be expected to accept any apology.
Apology cannot be a weapon of defence forged always to purge the guilty. It is intended
to be evidence of real contrition, the manly consciousness of a wrong done, of an injury
inflicted and the earnest desire to make such reparation as lies in the wrongdoer's power.
Only then is it of any avail in a Court of justice. But before it can have that effect, it
should be tendered at the earliest possible stage, not the latest. Even if wisdom dawns
only at a later stage, the apology should be tendered unreservedly and unconditionally,
before the Judge has indicated the trend of his mind. Unless that is done, not only is the
tendered apology robbed of all grace but it ceases to be an apology. It ceases to be the
full, frank and manly confession of a wrong done, which it is intended to be".
64
. It is well-settled that an apology is neither a weapon of defence to purge the guilty of
their offence; nor is it intended to operate as a universal panacea, it is intended to be
evidence of real contriteness [Vide M. Y. Shareaf v. Hon'ble Judges of the High Court of
Nagpur; (1955) 1 SCR 757; M.B. Sanghi v. High Court of Punjab and Haryana, (1991) 3
SCR 312]. AIR 1955 SC 19
1991 AIR SCW 2011

65

. In T. N. Godavarman Thirumulpad through the Amicus Curiae v. Ashok Khot and Anr.,
2006 (5) SCC 1, a three-Judge Bench of this Court had an occasion to consider the
question in the light of an 'apology' as a weapon defence by the contemner with a prayer
to drop the proceedings. The Court took note of the following observations of this Court
in L. D. Jaikwal v. State of U.P., (1984) 3 SCC 405 :2006 AIR SCW 2475
AIR 1984 SC 1374

"We are sorry to say we cannot subscribe to the 'slap - say sorry - and forget' school of
thought in administration of contempt jurisprudence. Saying 'sorry' does not make the
slipper taken the slap smart less upon the said hypocritical word being uttered. Apology
shall not be paper apology and expression of sorrow should come from the heart and not
from the pen. For it is one thing to 'say' sorry - it is another to 'feel' sorry." Para 32 of
AIR SCW

66. The Court, therefore, rejected the prayer and stated :

"Apology is an act of contrition. Unless apology is offered at the earliest opportunity and
in good grace, the apology is shorn of penitence and hence it is liable to be rejected. If the
apology is offered at the time when the contemner finds that the court is going to impose
punishment it ceases to be an apology and becomes an act of a cringing coward." Para
31 of AIR SCW

67. Similar view was taken in other cases also by this Court.
68. We are also satisfied that the so-called apology is not an act of penitence, contrition or
regret. It has been tendered as a 'tactful move' when the contemners are in the tight corner
and with a view to ward off the Court. Acceptance of such apology in the case on hand
would be allowing the contemners to go away with impunity after committing gross
contempt of Court. In our considered opinion, on the facts and in the circumstances of the
case, imposition of fine in lieu of imprisonment will not meet the ends of justice.
69. Considering the facts and circumstances in their entirety, in our opinion, ends of
justice would be served if we hold the respondents/contemners guilty under Section 12 of
the Contempt of Courts Act, 1971, read with Section 94(c) and Rule 2-A of Order
XXXIX of the Code of Civil Procedure, 1908 as amended by the Code of Civil Procedure
(Amendment) Act, 1976 and Article 129 of the Constitution and order the respondents-
contemners to undergo simple imprisonment for a term of two weeks i.e. fourteen days.
70. Ordered accordingly. The Contempt Petitions are disposed of.
Order accordingly. .
AIR 2008 SUPREME COURT 3027 "Munigadappa Meenaiah v. State of Andhra
Pradesh"
(From : Andhra Pradesh)*
Coram : 2 Dr. A. PASAYAT AND Dr. MUKUNDAKAM SHARMA, JJ.
Criminal Appeal No. 1206 of 2006, D/- 23 -7 -2008.
Munigadappa Meenaiah v. State of A.P.
(A) Penal Code (45 of 1860), S.300 - Evidence Act (1 of 1872), S.3 - MURDER -
EVIDENCE - WITNESS - Interested
@page-SC3028
witness - Credibility - Murder case - Relationship with deceased - Does not affect
credibility of witness - More often than not relation would not conceal actual culprit and
make allegations against an innocent person - For raising plea of false implication proper
foundation has to be laid. (Para 7)
(B) Penal Code (45 of 1860), S.300 - MURDER - EVIDENCE - Murder - Circumstantial
evidence - Conviction on basis of - Conditions precedent for, restated. (Para 21)
(C) Penal Code (45 of 1860), S.300 - MURDER - EVIDENCE - Murder - Circumstantial
evidence - Last seen together - Accused alleged to have invited deceased to his house for
consuming Tody and killed her by inflicting pestle blows on her head - Accused and
deceased were last seen together in night of occurrence by sons of deceased and Tody
shop owner - Body of deceased found lying in front of house of accused next morning -
Weapon of crime recovered at instance of accused - Conviction of accused for murder,
held proper. (Para 23)
Cases Referred : Chronological Paras
2003 AIR SCW 4097 : AIR 2003 SC 3601 : 2003 Cri LJ 3901 (Ref.) 22
2003 AIR SCW 5180 : AIR 2003 SC 4377 : 2003 Cri LJ 5054 (Ref.) 22
(2002) 3 SCC 76 (Rel. on) 12
1999 AIR SCW 1514 : AIR 1999 SC 1776 : 1999 Cri LJ 2588 (Rel. on) 13
1996 AIR SCW 2903 : AIR 1996 SC 3390 : 1996 Cri LJ 3461 (Ref.) 15
1992 AIR SCW 640 : 1992 Cri LJ 1104 : AIR 1992 SC 840 (Rel. on) 17
AIR 1990 SC 79 : 1990 Cri LJ 605 (Rel. on) 16
AIR 1989 SC 1890 : 1989 Cri LJ 2124 (Rel. on, Pnt B) 14
AIR 1987 SC 350 : 1987 Cri LJ 330 (Rel. on, Pnt B) 14
AIR 1985 SC 1224 : 1985 Cri LJ 1479 (Rel. on, Pnt B) 14
AIR 1984 SC 1622 : 1984 Cri LJ 1738 (Rel. on, Pnt B) 21
AIR 1983 SC 446 : 1983 Cri LJ 846 (Rel. on, Pnt B) 14
AIR 1977 SC 1063 : 1977 Cri LJ 639 (Rel. on, Pnt B) 14
AIR 1974 SC 276 : 1974 Cri LJ 331 (Rel. on, Pnt A) 9
AIR 1973 SC 2407 : 1973 Cri LJ 1589 (Rel. on) 12
AIR 1965 SC 202 : 1965 (1) Cri LJ 226 (Rel. on) 11
AIR 1957 SC 614 : 1957 Cri LJ 1000 (Rel. on, Pnt A) 9
AIR 1956 SC 316 : 1956 Cri LJ 559 (Rel. on, Pnt B) 14
AIR 1954 SC 621 : 1954 Cri LJ 1645 (Rel. on, Pnt B) 14
AIR 1953 SC 364 (Rel. on) 8, 10
AIR 1952 SC 54 : 1952 Cri LJ 547 10
AIR 1952 SC 343 : 1953 Cri LJ 129 (Rel. on) 20
Balraj Dewan, for Appellant; Mrs. D. Bharathi Reddy, for Respondent.
* Cri. Appeal No. 179 of 2003, D/- 19-10-2005 (A.P.)
Judgement
1. Dr. ARIJIT PASATAT, J. :-Challenge in this appeal is to the judgment of the Division
Bench of the Andhra Pradesh High Court upholding the conviction recorded by III
Additional District and Sessions Judge, Ranga Reddy District, holding the appellant
guilty for the offence punishable under Section 302 of the Indian Penal Code, 1860 (in
short 'IPC') and sentencing him to imprisonment for life.
2. Background facts in a nutshell are as follows :
The accused Munigadapa Meenaiah is native of Thimmapuram, Bommalaramaram of
Nalgonda District. He was doing fruit business at Medchal. Ten years back, his younger
brother Mallaiah died. After demise of Mallaiah, his wife Yellamma (hereinafter referred
to as the 'deceased'), along with her two sons took shelter at the house of the accused.
During that period, he developed illegal intimacy with the deceased and both were living
together and her sons were living separately. Suspecting the fidelity of the deceased, the
accused used to pick up quarrels with her, as a result of which, he separated and took
another portion at Medchal.
While so, the accused hatched up a plan to liquidate the deceased. On 3.6.2001 at about
9.00 a.m. the accused went to the house of deceased and invited her to house to consume
toddy and both of them went to the toddy shop of PW 2, purchased two bottle of toddy
and brought the same to his house in a tumbler and both of them consumed toddy. While
consuming toddy, the accused picked up quarrel with the deceased on the ground of her
chastity. As a consequence of which the deceased grew wild and abused him by denying
the allegations of illegal contacts with others. On that, the accused
@page-SC3029
brought a pestle and murdered the deceased by hitting the same on her head and laid the
body on the road in front of his house, cleaned the blood- stains in the room and tried to
obliterate the scientific evidence so as to throw the suspicion on other persons. On the
report given by P.W. 1, a case in Cr. No.117 of 2001 of Medchel Police Station was
registered for the offence under Section 302, IPC and after completion of investigation,
charge-sheet was filed.
Accused abjured guilt and demanded trial. The prosecution examined 10 witnesses, and
marked Exs. P 1 to P 21 and Mos. 1 to 5. On the other hand, no oral evidence was
adduced on behalf of the accused, Ex. D1 contradiction was marked.
After scrutinizing the entire material on record and after hearing the learned counsel on
both sides, the learned District Judge found the accused guilty of the offence punishable
under Section 302, IPC, convicted and sentenced him to suffer imprisonment for life.
3. The Trial Court placed reliance on the evidence of PWs-1 and 10 who spoke to have
seen the deceased last in the company of the appellant. Reference was also made to the
evidence of PW-5 relating to certain recoveries by PW-19. PW-2 also deposed to have
seen accused and deceased together when they purchased toddy and thereafter the dead
body of the deceased was found in front of the house of the accused with injuries on her
head and other parts of the body. As noted above, the learned Trial Court found the
accused guilty.
4. Before the High Court the stand was that PWs-1 and 10 are sons of the deceased and
are interested witnesses and should not have been believed. It was also submitted that the
circumstances highlighted do not make a complete chain of circumstances. The High
Court did not find any substance in the plea and dismissed the same by the impugned
judgment.
5. In support of the appeal learned counsel for the appellant reiterated the stands taken
before the High Court.
6. In response, learned counsel for the State supported the judgment of the Trial Court
and the High Court.
7. We shall first deal with the contention regarding interestedness of the witnesses for
furthering prosecution version. Relationship is not a factor to affect credibility of a
witness. It is more often than not that a relation would not conceal actual culprit and
make allegations against an innocent person. Foundation has to be laid if plea of false
implication is made. In such cases, the court has to adopt a careful approach and analyse
evidence to find out whether it is cogent and credible.
8

. In Dalip Singh and Ors. v. The State of Punjab (AIR 1953 SC 364) it has been laid down
as under :- Para 26 of AIR

"A witness is normally to be considered independent unless he or she springs from


sources which are likely to be tainted and that usually means unless the witness has
cause, such as enmity against the accused, to wish to implicate him falsely. Ordinarily a
close relation would be the last to screen the real culprit and falsely implicate an innocent
person. It is true, when feelings run high and there is personal cause for enmity, that there
is a tendency to drag in an innocent person against whom a witness has a grudge along
with the guilty, but foundation must be laid for such a criticism and the mere fact of
relationship far from being a foundation is often a sure guarantee of truth. However, we
are not attempting any sweeping generalization. Each case must be judged on its own
facts. Our observations are only made to combat what is so often put forward in cases
before us as a general rule of prudence. There is no such general rule. Each case must be
limited to and be governed by its own facts."
9

. The above decision has since been followed in Guli Chand and Ors. v. State of
Rajasthan (1974 (3) SCC 698) in which Vadivelu Thevar v. State of Madras (AIR 1957
SC 614) was also relied upon. AIR 1974 SC 276

10

. We may also observe that the ground that the witness being a close relative and
consequently being a partisan witness, should not be relied upon, has no substance. This
theory was repelled by this Court as early as in Dalip Singh's case (supra) in which
surprise was expressed over the impression which prevailed in the minds of the Members
of the Bar that relatives were not independent witnesses. Speaking through Vivian Bose,
J. it was observed : AIR 1953 SC 364, Para 25
"We are unable to agree with the learned Judges of the High Court that the testimony of
the two eye-witnesses requires corroboration.
@page-SC3030
If the foundation for such an observation is based on the fact that the witnesses are
women and that the fate of seven men hangs on their testimony, we know of no such rule.
If it is grounded on the reason that they are closely related to the deceased we are unable
to concur. This is a fallacy common to many criminal cases and one which another Bench
of this Court endeavoured to dispel in Rameshwar v. State of Rajasthan (AIR 1952 SC 54
at p.59). We find, however, that it unfortunately still persists, if not in the judgments of
the Courts, at any rate in the arguments of counsel."
11. Again in Masalti and Ors. v. State of U.P. (AIR 1965 SC 202) this Court observed (p.
209-210 para 14) :
"But it would, we think, be unreasonable to contend that evidence given by witnesses
should be discarded only on the ground that it is evidence of partisan or interested
witnesses.......The mechanical rejection of such evidence on the sole ground that it is
partisan would invariably lead to failure of justice. No hard and fast rule can be laid down
as to how much evidence should be appreciated. Judicial approach has to be cautious in
dealing with such evidence; but the plea that such evidence should be rejected because it
is partisan cannot be accepted as correct."
12. To the same effect is the decision in State of Punjab v. Jagir Singh (AIR 1973 SC
2407) and Lehna v. State of Haryana (2002 (3) SCC 76).
13

. As observed by this Court in State of Rajasthan v. Teja Ram and Ors. (AIR 1999 SC
1776) the over-insistence on witnesses having no relation with the victims often results in
criminal justice going awry. When any incident happens in a dwelling house or nearby
the most natural witnesses would be the inmates of that house. 1999 AIR SCW 1514

14. It has been consistently laid down by this Court that where a case rests squarely on
circumstantial evidence, the inference of guilt can be justified only when all the
incriminating facts and circumstances are found to be incompatible with the innocence of
the accused or the guilt of any other person. (See Hukam Singh v. State of Rajasthan
(AIR 1977 SC 1063); Eradu and Ors. v. State of Hyderabad (AIR 1956 SC 316);
Earabhadrappa v. State of Karnataka (AIR 1983 SC 446); State of U.P. v. Sukhbasi and
Ors. (AIR 1985 SC 1224); Balwinder Singh v. State of Punjab (AIR 1987 SC 350);
Ashok Kumar Chatterjee v. State of M.P. (AIR 1989 SC 1890). The circumstances from
which an inference as to the guilt of the accused is drawn have to be proved beyond
reasonable doubt and have to be shown to be closely connected with the principal fact
sought to be inferred from those circumstances. In Bhagat Ram v. State of Punjab (AIR
1954 SC 621), it was laid down that where the case depends upon the conclusion drawn
from circumstances the cumulative effect of the circumstances must be such as to
negative the innocence of the accused and bring the offences home beyond any
reasonable doubt.
15
. We may also make a reference to a decision of this Court in C. Chenga Reddy and Ors.
v. State of A.P. (1996) 10 SCC 193, wherein it has been observed thus: 1996 AIR
SCW 2903, Para 20A

"In a case based on circumstantial evidence, the settled law is that the circumstances from
which the conclusion of guilt is drawn should be fully proved and such circumstances
must be conclusive in nature. Moreover, all the circumstances should be complete and
there should be no gap left in the chain of evidence. Further the proved circumstances
must be consistent only with the hypothesis of the guilt of the accused and totally
inconsistent with his innocence....".
16. In Padala Veera Reddy v. State of A.P. and Ors. (AIR 1990 SC 79), it was laid down
that when a case rests upon circumstantial evidence, such evidence must satisfy the
following tests :
"(1) the circumstances from which an inference of guilt is sought to be drawn, must be
cogently and firmly established;
(2) those circumstances should be of a definite tendency unerringly pointing towards guilt
of the accused;
(3) the circumstances, taken cumulatively should form a chain so complete that there is
no escape from the conclusion that within all human probability the crime was committed
by the accused and none else; and
(4) the circumstantial evidence in order to sustain conviction must be complete and
incapable of explanation of any other hypothesis than that of the guilt of the accused and
such evidence should not only be consistent
@page-SC3031
with the guilt of the accused but should be inconsistent with his innocence.
17

. In State of U.P. v. Ashok Kumar Srivastava, (1992 Cri LJ 1104), it was pointed out that
great care must be taken in evaluating circumstantial evidence and if the evidence relied
on is reasonably capable of two inferences, the one in favour of the accused must be
accepted. It was also pointed out that the circumstances relied upon must be found to
have been fully established and the cumulative effect of all the facts so established must
be consistent only with the hypothesis of guilt. 1992 AIR SCW 640

18. Sir Alfred Wills in his admirable book "Wills' Circumstantial Evidence" (Chapter VI)
lays down the following rules specially to be observed in the case of circumstantial
evidence: (1) the facts alleged as the basis of any legal inference must be clearly proved
and beyond reasonable doubt connected with the factum probandum; (2) the burden of
proof is always on the party who asserts the existence of any fact, which infers legal
accountability; (3) in all cases, whether of direct or circumstantial evidence the best
evidence must be adduced which the nature of the case admits; (4) in order to justify the
inference of guilt, the inculpatory facts must be incompatible with the innocence of the
accused and incapable of explanation, upon any other reasonable hypothesis than that of
his guilt, (5) if there be any reasonable doubt of the guilt of the accused, he is entitled as
of right to be acquitted."
19. There is no doubt that conviction can be based solely on circumstantial evidence but
it should be tested by the touch-stone of law relating to circumstantial evidence laid down
by the this Court as far back as in 1952.
20

. In Hanumant Govind Nargundkar and Anr. v. State of Madhya Pradesh, (AIR 1952 SC
343), wherein it was observed thus : Para 10 of AIR

"It is well to remember that in cases where the evidence is of a circumstantial nature, the
circumstances from which the conclusion of guilt is to be drawn should be in the first
instance be fully established and all the facts so established should be consistent only
with the hypothesis of the guilt of the accused. Again, the circumstances should be of a
conclusive nature and tendency and they should be such as to exclude every hypothesis
but the one proposed to be proved. In other words, there must be a chain of evidence so
far complete as not to leave any reasonable ground for a conclusion consistent with the
innocence of the accused and it must be such as to show that within all human probability
the act must have been done by the accused."
21. A reference may be made to a later decision in Sharad Birdhichand Sarda v. State of
Maharashtra (AIR 1984 SC 1622). Therein, while dealing with circumstantial evidence, it
has been held that onus was on the prosecution to prove that the chain is complete and the
infirmity of lacuna in prosecution cannot be cured by false defence or plea. The
conditions precedent in the words of this Court, before conviction could be based on
circumstantial evidence, must be fully established. They are :
(1) the circumstances from which the conclusion of guilt is to be drawn should be fully
established. The circumstances concerned 'must' or 'should' and not 'may be' established;
(2) the facts so established should be consistent only with the hypothesis of the guilt of
the accused, that is to say, they should not be explainable on any other hypothesis except
that the accused is guilty;
(3) the circumstances should be of a conclusive nature and tendency;
(4) they should exclude every possible hypothesis except the one to be proved; and
(5) there must be a chain of evidence so complete as not to leave any reasonable ground
for the conclusion consistent with the innocence of the accused and must show that in all
human probability the act must have been done by the accused.
22

. These aspects were highlighted in State of Rajasthan v. Rajaram (2003 (8) SCC 180)
and State of Haryana v. Jagbir Singh (2003 (11) SCC 261). 2003 AIR SCW 4097
2003 AIR SCW 5180

23. In the instance PWs-1 and 10 as well as PW-2 saw the deceased and the accused
together in the night of the occurrence. In the morning, dead body of the deceased was
found in front of the house of the accused. Additionally, on the basis of information given
by the accused certain articles were recovered and one of them was the pestle used for
inflicting the injury on the head. That being so, the judgment of the Trial
@page-SC3032
Court and the High Court do not suffer from any Infirmity.
24. The appeal is without merit, deserves dismissal, which we direct.
Appeal dismissed.
AIR 2008 SUPREME COURT 3032 "Manjit Prakash v. Shobha Devi"
(From : Patna)*
Coram : 2 Dr. A. PASAYAT AND HARJIT SINGH BEDI, JJ.
Criminal Appeal No. 1113 of 2008 (arising out of SLP (Cri.) No. 4205 of 2007), D/- 18 -7
-2008.
Manjit Prakash and Ors. v. Shobha Devi and Anr.
Criminal P.C. (2 of 1974), S.439(2), S.437(5) - BAIL - Bail - Cancellation - Order must
be reasoned - Rejection of bail and cancellation of bail - Stand on different footings -
Cancellation of bail is a harsh order because it takes away liberty of an individual.
Cri. Misc. No. 48109 of 2006, D/-12-07-2007 (Pat.), Reversed. (Paras 7, 14)
Cases Referred : Chronological Paras
2004 AIR SCW 1581 : AIR 2004 SC 1866 : 2004 Cri LJ 1796 (Ref.) 8
2002 AIR SCW 1342 : AIR 2002 SC 1475 : 2002 Cri LJ 1849 : 2002 All LJ 961 8
2001 AIR SCW 1935 : AIR 2001 SC 2023 : 2001 Cri LJ 2566 (Rel. on) 8, 12, 13
(2001) Cri. Appeal No. 745 of 2001, D/- 25-7-2001 (reported in 2004 (7) SCC 539)
10
1992 AIR SCW 2621 : AIR 1993 SC 1 : 1992 Cri LJ 3712 (Rel. on) 6
AIR 1987 SC 149 : 1987 Cri LJ 157 6
AIR 1978 SC 179 : 1978 Cri LJ 129 12
Gaurav Agrawal, for Appellants; Gopal Singh, Manish Kumar, Rajnish Prasad and Amit
Pawan, for Respondents.
* Cri. Misc. No. 48109 of 2006, D/- 12-7-2007 (Pat.).
Judgement
1. Dr. ARIJIT PASAYAT, J. :-Leave granted.
2. Appellants challenge the order passed by a learned single Judge of the Patna High
Court cancelling the bail granted to them by order dated 7-9-2006 in Criminal
Miscellaneous No. 10719 of 2006. The application for cancellation of bail was filed by
the respondent No.1. Appellants 1, 2 and 3 are arrayed as accused Nos. 1, 2 and 4. Five
persons were granted bail by order dated 7-9-2006 in Criminal Misc. Case No. 10719 of
2006. By the impugned order the learned single Judge directed cancellation of bait
granted to the present appellants while holding that the two others being ladies there was
no need to cancel the bail granted to them.
3. Though various points were urged in support of the appeal primarily it was submitted
that no reasons have been given for cancelling the bail.
4. Learned counsel for the respondent No. 1 complainant submitted that though the order
cancelling bail has not elaborately dealt with the circumstances to warrant cancellation of
bail, the same is in order.
5. The appellants and the other two in respect of whom the High Court has not interfered,
were granted provisional bail by order dated 3-5-2006 which came to be confirmed on 7-
9-2006. It was stated that the husband and wife have been residing together in the
matrimonial home. Earlier there was a suit for restitution of conjugal rights filed by
appellant No.2 who withdrew the same after the provisional bail was confirmed and
instituted Matrimonial Case No. 34 of 2006 for divorce. According to the complaint on
10-10-2006, there was an incident and therefore the bail was to be cancelled. The High
Court, as rightly contended by learned counsel for the appellants, has not indicated the
reasons for directing cancellation of ball.
6

. It is trite law that the considerations for grant of bail and cancellation of bail stand on
different footings. By a majority judgment in Aslam Babalal Desai v. State of
Maharashtra, the circumstances when bail granted can be cancelled were highlighted in
the following words (SCC pp. 289-90, para 11) : 1992 AIR SCW 2621, Para 11

"11. On a conjoint reading of Sections 57 and 167 of the Code it is clear that the
legislative object was to ensure speedy investigation after a person has been taken in
custody. It expects that the investigation should be completed within 24 hours and if this
is not possible within 15 days and failing that within the time stipulated in clause (a) of
the proviso to Section 167(2) of the Code. The law expects that the investigation must be
completed with dispatch and the role of the Magistrate is to oversee the AIR 1987 SC
149

@page-SC3033
course of investigation and to prevent abuse of the law by the investigating agency. As
stated earlier, the legislative history shows that before the introduction of the proviso to
Section 167(2) the maximum time allowed to the investigating agency was 15 days under
sub-section (2) of Section 167 failing which the accused could be enlarged on bail. From
experience this was found to be insufficient particularly in complex case and hence the
proviso was added to enable the Magistrate to detain the accused in custody for a period
exceeding 15 days but not exceeding the outer limit fixed under the proviso (a) to that
sub-section. We may here mention that the period prescribed by the proviso has been
enlarged by State amendments and wherever there is such enlargement, the proviso will
have to be read accordingly. The purpose and object of providing for the release of the
accused under sub-section (2) of Section 167 on the failure of the investigating agency
completing the investigation within the extended time allowed by the proviso was to
Instil a sense of urgency in the investigating agency to complete the investigation
promptly and within the statutory time frame. The deeming fiction of correlating the
release on ball under sub-section (2) of Section 167 with Chapter XXXIII i.e. Sections
437 and 439 of the Code, was to treat the order as one passed under the latter provisions.
Once the order of release is by fiction of law an order passed under Section 437(1) or (2)
or Section 439(1) it follows as a natural consequence that the said order can be cancelled
under sub-section (5) of Section 437 or sub-section (2) of Section 439 on considerations
relevant for cancellation of an order thereunder. As stated in Raghubir Singh v. State of
Bihar the grounds for cancellation under Sections 437(5) and 439(2) are identical,
namely, bail granted under Section 437(1) or (2) or Section 439(1) can be cancelled
where (i) the accused misuses his liberty by indulging in similar criminal activity, (ii)
interferes with the course of investigation, (iii) attempts to tamper with evidence or
witnesses, (iv) thereatens witnesses or indulges in similar activities which would hamper
smooth investigation, (v) there is likelihood of his fleeing to another country, (vi)
attempts to make himself scarce by going underground or becoming unavailable to the
investigating agency, (vii) attempts to place himself beyond the reach of his surety etc.
These grounds are illustrative and not exhaustive. It must also be remembered that
rejection of bail stands on one footing but cancellation of bail is a harsh order because it
interferes with the liberty of the individual and hence it must not be lightly resorted to."
7. It is, therefore, clear that when a person to whom bail has been granted either tries to
interfere with the course of justice or attempts to tamper with evidence or witnesses or
threatnes witnesses or indulges in similar activities which would hamper smooth
investigation or trial, bail granted can be cancelled. Rejection of bail stands on one
footing, but cancellation of bail is a harsh order because it takes away the liberty of an
individual granted and is not to be lightly resorted to.
8

. In Kalyan Chandra Sarkar v. Rajesh Ranjan @ Pappu Yadav and Anr. (2004 (7) SCC
528). In para 11 it was noted as follows : 2004 AIR SCW 1581, Para 11

"11. The law in regard to grant or refusal of ball is very well settled. The Court granting
ball should exercise its discretion in a judicious manner and not as a matter of course.
Though at the stage of granting bail a detailed examination of evidence and elaborate
documentation of the merit of the case need not be undertaken, there is a need to indicate
in such orders reasons for prima facie concluding why bail was being granted particularly
where the accused is charged of having committed a serious offence. Any order devoid of
such reasons would suffer from non-application of mind. It is also necessary for the Court
granting bail to consider among other circumstances, the following factors also before
granting bail; they are :
(a) The nature of accusation and the severity of punishment in case of conviction and the
nature of supporting evidence.
(b) Reasonable apprehension of tampering with the witness or apprehension of threat to
the complainant.

(c) Prima facie satisfaction of the Court in support of the charge. (See Ram Govind
Upadhyay v. Sudarshan Singh (2002 (3) SCC 598) and Puran v. Rambilas (2001 (6) SCC
338). 2002 AIR SCW 1342
2001 AIR SCW 1935

9. It was also noted in the said case that the conditions laid down under Section 437(1)(i)
are sine qua non for granting bail even under Section 439 of the Code.
@page-SC3034
10. In para 14 it was noted as follows :
"14. We have already noticed from the arguments of learned counsel for the appellant that
the present accused had earlier made seven applications for grant of bail which were
rejected by the High Court and some such rejections have been affirmed by this Court
also. It is seen from the records that when the fifth application for grant of bail was
allowed by the High Court, the same was challenged before this Court and this Court
accepted the said challenge by allowing the appeal filed by the Union of India and
another and cancelled the bail granted by the High Court as per the order of this Court
made in Criminal Appeal No. 745 of 2001 dated 25-7-2001. While cancelling the said
bail this Court specifically held that the fact that the present accused was in custody for
more than one year (at that time) and the further fact that while rejecting an earlier
application, the High Court had given liberty to renew the bail application in future, were
not grounds envisaged under Section 437(1)(i) of the Code. This Court also in specific
terms held that the condition laid down under Section 437(1)(i) is sine qua non for
granting bail even under Section 439 of the Code. In the impugned order it is noticed that
the High Court has given the period of incarceration already undergone by the accused
and the unlikehood of trial concluding in the near future as grounds sufficient to enlarge
the accused on bail, in spite of the fact that the accused stands charged of offences
punishable with life imprisonment or even death penalty. In such cases, in our opinion,
the mere fact that the accused has undergone certain period of incarceration (three years
in this case) by itself would not entitled the accused to being enlarged on bail, nor the fact
that the trial is not likely to be concluded in the near future either by itself or coupled
with the period of incarceration would be sufficient for enlarging the appellant on bail
when the gravity of the offence alleged is severe and there are allegations of tampering
with the witnesses by the accused during the period he was on bail."
11. Even though the re-appreciation of the evidence as done by the Court granting bail is
to be avoided, the Court dealing with an application for cancellation of bail under Section
439(2) can consider whether irrelevant materials were taken into consideration. That is so
because it is not known as to what extent the irrelevant materials weighed with the Court
for accepting the prayer for bail.
12

. In Puran v. Rambilas and Anr. (2001 (6) SCC 338) it was noted as follows : 2001
AIR SCW 1935, Para 10

"11. Further, it is to be kept in mind that the concept of setting aside the unjustified illegal
or perverse order is totally different from the concept of cancelling the bail on the ground
that the accused has misconducted himself or because of some new facts requiring such
cancellation. This position is made clear by this Court in Gurcharan Singh v. State (Delhi
Admn.). In that case the Court observed as under (SCC p. 124, para 16) : AIR 1978 SC
179, Para 16

"If, however, a Court of Session had admitted an accused person to bail, the State has two
options. It may move the Sessions Judge if certain new circumstances have arisen which
were not earlier known to the State and necessarily, therefore, to that Court. The State
may as well approach the High Court being the superior Court under Section 439(2) to
commit the accused to custody. When, however, the State is aggrieved by the order of the
Sessions Judge granting bail and there are no new circumstances that have cropped up
except those already existing, it is futile for the State to move the Sessions Judge again
and it is competent in law to move the High Court for cancellation of the bail. This
position follows from the subordinate position of the Court of Session vis-a-vis the High
Court."
13. The perversity as highlighted in Puran's case (supra) can also flow from the fact that
as noted above, irrelevant materials have been taken into consideration adding
vulerability to the order granting bail. The irrelevant materials should be of a substantial
nature and not of a trivial nature.
14. Since the High Court has not indicated any reasons for directing cancellation of bail,
the impugned order cannot be maintained and is set aside. The matter is remitted to the
High Court to decide the matter afresh and dispose of the application filed. We make it
clear that we have not expressed any opinion on the merits of the case.
15. The appeal is allowed to the aforesaid extent.
Appeal allowed. .
@page-SC3035
AIR 2008 SUPREME COURT 3035 "H. U. D. A. v. Raj Singh Rana"
Coram : 2 ALTAMAS KABIR AND MARKANDEY KATJU, JJ.
Civil Appeal No. 4436 of 2008 (arising out of S. L. P. (C) No. 13644 of 2005), D/- 16 -7
-2008.
H. U. D. A. v. Raj Singh Rana.
(A) Haryana Urban Development Authority Act (13 of 1997), S.15 - Consumer Protection
Act (68 of 1986), S.21 - DEVELOPMENT AUTHORITY - CONSUMER
PROTECTION - ALLOTMENT OF PREMISES - INTEREST - Allotment of plot -
Interest on delayed payment by allottee - Interest payable on delayed payment of
tentative sale price fixed in allotment letter - No specific provision made as to rate of
interest on additional price payable by allottee - Development Authority can, therefore,
charge interest at different rates - But rate cannot be fixed arbitrarily in disregard to
Interest Act - Demand raised by Development Authority for interest payable at higher rate
on additional price - Cannot be set aside by applying rate mentioned in letter of allotment.
Interest Act (14 of 1978), S.3.(Paras 17, 18, 19)
(B) Haryana Urban Development Authority Act (13 of 1997), S.15 - DEVELOPMENT
AUTHORITY - ALLOTMENT OF PREMISES - INTEREST - Allotment of plot -
Interest for delayed payment of instalments - Rate - Cannot be fixed arbitrarily by
Development Authority - Provisions of Interest Act ought to be considered.
Interest Act (14 of 1978), S.3.(Para 17)
Cases Referred : Chronological Paras
2008 AIR SCW 962 (Rel. on) 15
2007 AIR SCW 527 : AIR 2007 SC 817 : 2007 CLC 344 : 2007 (2) AIR Kar R 169 (Rel.
on) 15
2005 AIR SCW 4533 : AIR 2005 SC 3483 : 2005 All LJ 3386 (Rel. on) 15
2004 AIR SCW 2362 : AIR 2004 SC 2141 : 2004 All LJ 1500 (Rel. on) 14, 15, 17
2004 AIR SCW 5297 (Rel. on) 15
Neeraj Kumar Jain, Sanjay Singh, Ugra Shankar Prasad, Sandeep Chaturvedi and Umang
Shankar, for Appellant; Arvind Chaudhary and Atishi Dipankar, for Respondent.
* From Order of the National Consumer Disputes Redressal Commission, New Delhi in
Revn. Petn. No. 2217 of 2004, D/- 19-11-2004.
Judgement
1. ALTAMAS KABIR, J. :-Leave granted.
2. One Baldev Singh Nagar was allotted residential plot No.718 (later on re-numbered
883) measuring 14 marlas in Sector 13 of the Urban Estate at Karnal under the provisions
of the Punjab Urban Estate (Development and Regulation) Act, 1964, which was repealed
by the Haryana Urban Development Authority Act, 1997. The said plot was subsequently
transferred to the respondent herein, Shri Raj Singh Rana, as will be evident from the
letter dated 22.3.1974 addressed to the respondent by the Estate Officer, Urban Estate,
Karnal. In the said letter various conditions have been set out in respect of the said
allotment, of which we are concerned with the condition Nos. 1, 2, 3, 4, 8 and 15, which
are reproduced hereinbelow :
"From
The Estate Officer,
Urban Estate,
Karnal.
Transferred vide Memo
No.E.O.(M)- 76/5235, Dated 01.10.1976
with condition No. 16
To
Shri R. S. Rana
S/o Shri A. S. Rana,
V.P.O. Garhi,
Distt. Sonepat.
Memo No-1664/718/14/E.O/K,
Dated : 22.3.1974
Subject : Allotment of Residential plot in the Urban Estate, Karnal.
Reference your application dated 25.9.1971 for the allotment of residential plot in the
Urban Estate at Karnal.
1. Plot No.718 measuring 14 Marias in Sector 13 of the Urban Estate at Karnal is hereby
allotted to you. The total tentative sale price of said plot is Rs. 12,250/- against which you
have already deposited Rs.6,125/-of the price mentioned in part 1 above is Rs. Nil.
2. The plot is preferential one and an additional price at the rate of 10 per cent of the price
mentioned in para 1 above is Rs. Nil.
3. The total tentative sale price of this plot (normal plus preferential cost) is Rs. Nil.
4. The above price of the plot is subject to variation with reference to the actual
measurement of the plot as well as in case of enhancement of compensation of
acquisition cost of land of this sector by the court
@page-SC3036
or otherwise and you shall have to pay this additional price of the plot, if any, as
determined by the Department within 30 days from the date of demand.
5. .......
6. .......
7. .......
8. Balance 50 per cent of the total tentative sale price shall be payable either in lumpsum
within 60 days from the date of issue of allotment letter without interest or in 2 equated
instalments with interest at the rate of 7 per cent per annum. The first and remaining
instalments of the balance amount together with interest at the rate of 7 per cent per
annum on the unpaid amount of the total tentative sale price shall fall due to payment as
under and no notice shall be served upon you to pay the same but in case in instalment is
not paid in time, you will be served with a notice to pay by same within a month together
with a sum not exceeding the amount of the instalment as may be determined by the
undersigned, by way of penalty. If the payment is not made within the said period or such
extended period as may be determined by the undersigned, not exceeding three months in
all from the date on which the instalment was originally due, the same will be recovered
as an arrear of land revenue or action will be taken under Section 10 of the Punjab Urban
Estate (Development and Regulation) Act, 1964 :-
No. of instalment
Due date on which the
Payment is to be made
First 2958.93+28.75 = 3387.68 21.3.1975
Second 3166.07+221.61 = 3387.68 21.3.1976
Third
Fourth
Fifth
Sixth:
9. ......
10. ......
11. ......
12. ......
13. ......
14. ......
15. This allotment is subject to the provisions of the Punjab Urban Estates (Development
and Regulation) Act, 1964 and the rules framed thereunder as amended from time to time
and you shall have to accept and abide by them.
16. .....
17. ......
Sd/-
Estate Officer
Urban Estate
Karnal"
3. There is no dispute that the entire amount, as initially computed as tentative sale price,
was fully paid by the respondent, together with further amounts on account of enhanced
compensation paid for the plot, on the basis of the demand notices issued to the
respondent from time to time. The problem arose when in addition to the above, the
Estate Officer, HUDA, Karnal, by his Memo dated 15.6.2001 raised an additional
demand of Rs.71,800/-by imposing simple interest @ 10 per cent per annum up to
31.3.1987, 15 per cent per annum up to 15.1.1988, compound inter -est @ 15 per cent up
to 31.8.2000 and thereafter again simple interest @ 15% per annum up to 31.8.2001.
According to the respondent, the rate of interest as indicated in the allotment letter being
7 per cent simple interest per annum, the appellant had acted illegally in demanding
interest at the higher rates, indicated hereinabove and such demand being arbitrary could
not be sustained.
4. Aggrieved by such demand, the respondent filed complaint case No.591 of 2002 before
the District Consumer Disputes Redressal Forum praying for refund of Rs.35,200/-,
which according to the respondent was the excess amount of interest charged over and
above the rate of interest at 7 per cent indicated in the allotment letter. The respondent
also prayed for interest @ 12 per cent on the refund amount from 2.11.2001, when the
interest amount was demanded and paid under protest, until repayment. The District
Forum accepted the submissions made on behalf of the respondent herein and held that
the appellants could charge interest only at the stipulated rate mentioned in the allotment
letter, namely, 7 per cent per annum and directed the appellant to calculate the interest @
7 per cent on the 3rd and 4th enhancements and to refund the extra amount charged to the
complainant/respondent with interest at the rate of 7 per cent from the date of the
complaint till its refund. The decision of the District
@page-SC3037
Forum was confirmed by the State Commission, and ultimately, the appellant herein took
the matter in revision to the National Commission in R.P.No.2217 of 2004. The National
Commission, while confirming the view taken by the District Forum and the State
Commission as to the rate of interest which could have been charged by the appellant,
considered another aspect relating to charging of compound interest @ 15 per cent per
annum from 16.1.1988 to 31.8.2000 and held that the appellant was not entitled to charge
such compound interest.
5. It is against the said order of the National Commission that this appeal has been filed
by the Haryana Urban Development Authority (hereinafter referred to as "HUDA").
6. On behalf of the HUDA it was strenuously urged that the rate of interest @ 7 per cent
per annum, as indicated in the allotment letter, was only with regard to default in payment
of instalments for the tentative sale price and not as regards the additional amounts
required to be paid in case of enhancement of compensation for acquisition cost of the
land, for which no rate of interest had been stipulated. It was submitted that on account of
default in payment of the instalments of the enhanced compensation, on account of the
low interest which was being charged, a decision was taken by HUDA on 15.1.1987 to
increase the normal rate of interest to 10 per cent per annum and interest for the delayed
payment of instalments to 18 per cent per annum, which would also include the normal
interest of 10 per cent. It was submitted that it was on account of such revised policy that
HUDA had charged interest at the rates indicated hereinbefore to ensure that instalments
were paid in time. Apart from his aforesaid submissions, learned counsel for the appellant
could not justify charging of compound interest as was done in the instant case.
7. It was urged that enhancement of rate of interest being a matter of policy to prevent
default in payment of instalments the Fora below had erred in correlating the rate of
interest mentioned in the allotment letter, which was only applicable in respect of default
payment of instalments for the tentative price initially fixed, to the defaults committed in
respect of the payment of the enhanced compensation on account of increase in the
acquisition costs. It was also submitted that since the rate of interests stipulated at 7 per
cent per annum has no application to default in payment of enhanced compensation, the
Fora below had erred in directing that interest on the latter default be also charged at the
stipulated rate of 7 per cent per annum. It is submitted that the understanding of the terms
and conditions of the allotment letter and the decision rendered by the consumer forums
on the basis thereof, was wholly erroneous and was liable to be set aside.
8. On behalf of the respondent it was contended that apart from the fact that the rate of
interest demanded was arbitrary, it was also extremely high and ought not to have been
levied from the date of allotment inasmuch as, the tentative sale price had been fully paid
and such demand could not operate retrospectively, interest on the unpaid amount could,
if at all, have been raised for periods only after the payment was made. In addition it was
submitted that it is well settled that when a contractual rate of interest has been agreed
upon by the parties, no amount by way of interest in excess thereof could be raised. It
was submitted that following the said principle, first the District Forum, and, thereafter,
the State and National Commissions had awarded interests on the delayed instalments at
the rate of 7 per cent per annum as mentioned in the allotment letter referred to above. It
was contended that condition No.8 enumerated in the letter dated 22.3.1974 written to the
respondent by the Estate Officer, Karnal, would have to be considered and understood in
such light. It is submitted that the orders of the consumer Fora was in consonance with
the provisions of the allotment letter and did not, therefore, warrant any interference by
this Court and the appeal was liable to be dismissed.
9. Having heard learned counsel for the parties and having perused the documents relied
upon by them, we are of the view that the width of the dispute is rather narrow, being
confined only to the question as to whether it was within the competence of the appellant
to charge interest on delayed payments at the rate at which it has been charged and
whether compound interest could have been charged without there being any mutual
agreement between the parties to that effect.
@page-SC3038
10. The concept of levying or allowing interest is available in almost all statutes
involving financial deals and commercial transactions, but the provision empowering
Courts to allow interest is contained in the Interest Act, 1978, which succeeded and
repealed the Interest Act, 1839. Section 3 of the said Act, inter alia, provides that in any
proceeding for the recovery of any debt or damages or in any proceeding in which a
claim for interest in respect of debt or damage already paid is made, the Court may, if it
thinks fit, allow interest to the person entitled to the debt or damages or to the person
making such claim, as the case may be, at a rate not exceeding the current rate of interest,
for the whole or part of the periods indicated in the said Section.
11. What is important is the mention of allowing the interest at a rate not exceeding the
current rate of interest. Such a provision is, however, excluded in respect of the interest
payable as of right by virtue of any agreement as indicated in sub-section(3) of Section 3.
In other words, where there is an agreement between the parties to payment of interest at
a certain stipulated rate, the same will have the precedence over the provision contained
in sub-section (1) which provides for the Court to allow interest at a rate not exceeding
the current rate of interest.
12. Yet another provision which is basic in its operation is contained in Section 34 of the
Code of Civil Procedure which also, inter alia, provides that where and insofar as a
decree is for the payment of money, the Court may in the decree order interest at such
rate as the Court deems reasonable to be paid on the principal sum adjudged, from the
date of the suit, till the date of the decree in addition to any interest adjudged on such
principal sum for any period prior to the institution of the suit, with further interest at
such rate not exceeding 6 per cent per annum as the court may deem reasonable on such
principal sum from the date of the decree till the date of payment or to such earlier date as
the court thinks fit.
13. The rates of interest charged by the appellant, purportedly in accordance with their
policy decisions, appear to have been influenced by the provisions of the Interest Act and
also the Code of Civil Procedure on the supposition that the payment of additional price
on account of enhancement of compensation was not covered by the provisions of the
allotment letter relating, to payment of interest. The view expressed by the District forum
have been accepted by the State and National Commissions.
14

. It is no doubt true that the law relating to allowing interest and the rates thereof has been
considered and settled in the case of Ghaziabad Development Authority v. Balbir Singh
(2004 (5) SCC 65), which has since been followed in various subsequent decisions. The
said decision was also one rendered under the provisions of the Consumer Protection Act,
1986, though in the said case it was a reverse situation in which the authorities were held
to be liable to compensate for misfeasance in public office. In the said case interest was
allowed @ 18% per annum which was unacceptable to this Court which observed that the
power to award compensation does not mean that irrespective of the facts of the case
compensation can be awarded in all matters at a uniform rate of 18 per cent per annum.
This Court noticed that the National Forum had been awarding interest at a flat rate of 18
per cent per annum irrespective of the facts of each case. The same was held to be
unsustainable. In the said state of facts this Court observed in para 8, as follows : 2004
AIR SCW 2362

"However, the power and duty to award compensation does not mean that irrespective of
facts of the case compensation can be awarded in all matters at a uniform rate of 18% per
annum. As seen above, what is being awarded is compensation i.e. a recompense for the
loss or injury. It therefore necessarily has to be based on a finding of loss or injury. No
hard-and-fast rule can be laid down, however, a few examples would be where an
allotment is made, price is received/paid but possession is not given within the period set
out in the brochure. The Commission/Forum would then need to determine the loss. Loss
could be determined on basis of loss of rent which could have been earned if possession
was given and the premises let out or if the consumer has had to stay in rented premises
then on basis of rent actually paid by him. Along with recompensing the loss the
Commission/Forum may also compensate for harassment/ injury, both mental and
physical. Similarly, compensation can be given if after allotment is made there has been
cancellation of
@page-SC3039
scheme without any justifiable cause."
15

. Applying the aforesaid principle laid down in the aforesaid case, it was the duty of the
Consumer Fora to consider the circumstances of the case and keep in mind the provisions
of Section 3 of the Interest Act in awarding the high rate of interest, without linking the
same to the current rate of interest. As was mentioned in Balbir Singh's case, and,
thereafter, in HUDA vs. Prem Kumar Agarwal and another (2008(1) SCALE 484); Bihar
State Housing Board vs. Arun Dakshy (2005 (7) SCC 103); Haryana Urban Development
Authority vs. Manoj Kumar (2005 (9) SCC 541) and Krishna Bhagya Jala Nigam Limited
vs. G. Harischandra Reddy and another (2007 (2) SCC 720) the rate of interest is to be
fixed in the circumstances of each case and it should not be imposed at a uniform rate
without looking into the circumstances leading to a situation where compensation was
required to be paid. 2004 AIR SCW 2362
2008 AIR SCW 962
2005 AIR SCW 4533
2004 AIR SCW 5297
2007 AIR SCW 527

16. In the instant case, the provision of the allotment letter dated 22.3.1974 appears to
have been wrongly interpreted by the Consumer Fora since the stipulated rate of interest
only takes into consideration payment of the total tentative sale price while Condition
No.4 of the allotment letter mentions that the total tentative sale price was subject to
variation in certain circumstances and that the allottee would have to pay an additional
price for the plot as a consequence thereof. It does not mention that interest at the rate of
7 per cent per annum would be payable also in respect of the additional price required to
be paid on account of increase of the acquisition cost. The said position is further
clarified by condition No.8 which also speaks of payment of the total tentative sale price
and the rate of interest at 7 per cent per annum on the instalments to be paid in respect
thereof. There is nothing further in the agreement which provides for the rate of interest
to be levied on the additional price on account of the enhancement of the acquisition cost.
17

. On such score we are inclined to agree with the learned counsel for the appellant that the
appellant was entitled, even in terms of the allotment letter to charge interest on balance
dues at a rate which was different from that stipulated in the allotment letter. At the same
time, we are in agreement with the views expressed in Balbir Singh's case (supra) which
gives an indication of the matters which are required to be considered by the Courts while
granting interest where there is no mutual understanding or agreement with regard to the
rate of interest that could be charged. While we also agree that for unpaid dues the
appellant is entitled to charge interest, such an exercise will have to be undertaken within
the parameters of circumstances and reason and the rate of interest should not be fixed
arbitrarily. In the decisions referred to hereinabove, this Court has sounded a note of
caution that rates of interest fixed by the Courts must not be arbitrary and should take into
account the current bank rates which in recent years have shown a tendency to slide
downwards. In fact, in many of the aforesaid cases, the rate of interest has been reduced
substantially. 2004 AIR SCW 2362

18. In the aforesaid circumstances, even though the rate of interest indicated in the
allotment letter dated 22.3.1974 may not have application as far as payment of the
additional price is concerned, the District Forum has erred on the site of reason and has
allowed interest at the rate of 7 per cent per annum upon holding that the demand made
by the appellant at the higher rate was contrary to the mutual agreement contained in the
allotment letter. In our view, even though a policy may have been adopted by the
appellant for imposing a deterrent rate of interest on defaults committed by allottees in
payment of their dues, such imposition has to be in keeping with the provisions of
Section 3 of the Interest Act, 1978 and not in a unreasonable manner. It may perhaps be
even more pragmatic if a condition regarding charging of interest at the prevailing
banking rates were included in the allotment letters, having regard to the provisions of
sub-section (3) of Section 3 of the said Act.
19. We, therefore, allow this appeal, set aside the orders dated 10.3.04 passed by the
District Forum, Chandigarh in Complaint Case No. 591 of 2002, as affirmed by the State
Commission, Chandigarh, on 9.7.2004 and the order passed in Revision by the National
Commission on 19.11.2004, which is the subject matter of this appeal, and quash the
additional demand of Rs.71,800 raised
@page-SC3040
on behalf of the appellant vide Memo No. EO 8682 dated 15.6.2001 and direct that the
appellant will be entitled to impose simple interest on the basis of the prevailing current
rate of interest for the purpose indicated in para 6 of the complaint filed by the respondent
(Complaint Case No.591 of 2002) before the District Forum, Chandigarh. Such a
computation is to be completed within a month from the date of receipt of this order.
Since, we have been informed at the Bar that the entire amount by way of additional
demand has been deposited upon protest, any amount which is in excess of the amount to
be computed on the basis of this order, shall be refunded to the respondent within two
weeks of such computation.
20. In the facts and circumstances of the case, the parties will bear their own costs.
Appeal allowed. .
AIR 2008 SUPREME COURT 3040 "Swamy Shraddananda v. State of Karnataka"
(From : Karnataka)
Coram : 3 B. N. AGRAWAL, G. S. SINGHVI AND AFTAB ALAM, JJ.
Criminal Appeal No. 454 of 2006, D/- 22 -7 -2008.
Swamy Shraddananda @ Murali Manohar Mishra v. State of Karnataka.
(A) Penal Code (45 of 1860), S.302 - MURDER - DEATH SENTENCE - Murder - Death
sentence - Rarest of rare case - Categories of murder cases enumerated in Machhi Singh,
AIR 1983 SC 957 - Not inflexible, absolute or immutable. (Para 28)
(B) Penal Code (45 of 1860), S.302, S.45, S.57 - MURDER - LIFE IMPRISONMENT -
SENTENCE REMISSION - SENTENCE COMMUTATION - Murder - Sentence - Case
falling just short of rarest of rare case - Court can sentence accused to imprisonment for
rest of his life or term excepting 14 years - Remission granted by executive orders to life
convicts has no legal basis.
Criminal P.C. (2 of 1974), S.432, S.433, S.28, S.354.
When a murder convict comes to Supreme Court carrying a death sentence awarded by
the trial Court and confirmed by the High Court, the Court may find that the case just
falls short of the rarest of the rare category and may feel somewhat reluctant in endorsing
the death sentence. But at the same time, having regard to the nature of the crime, the
Court may strongly feel that a sentence of life imprisonment which, subject to remission,
normally works out to a term of 14 years would be grossly disproportionate and
inadequate. If in such cases the Court's option is limited only to two punishments, one a
sentence of imprisonment, for all intents and purposes, of not more than 14 years and the
other death, the Court may feel tempted and find itself nudged into endorsing the death
penalty. Such a course would indeed be disastrous. A far more just, reasonable and proper
course would be to expand the options and to take over what, as a matter of fact, lawfully
belongs to the Court, i.e., the vast hiatus between 14 years imprisonment and death. The
Court, therefore, can substitute a death sentence by life imprisonment for rest of life of
convict or by a term in excess of fourteen years and further to direct that the convict must
not be released from the prison for the rest of his life or for the actual term as specified in
the order, as the case may be. (Paras 66, 68)
The formalisation of a special category of sentence, though for an extremely few number
of cases, shall have the great advantage of having the death penalty on the statute book
but to actually use it as little as possible, really in the rarest of the rare cases. (Para
67)
Section 45 of the Penal Code defines "life" to mean the life of the human being, unless
the contrary appears from the context. Section 53 enumerates punishments, the first of
which is death and the second, imprisonment for life. Section 57 provides that in
calculating fractions of terms of punishment, imprisonment for life shall be reckoned as
equivalent to imprisonment for twenty years. Section 57 of the Penal Code does not in
any way limit the punishment of imprisonment for life to a term of twenty years. Section
57 is only for calculating fractions of terms of punishment and provides that
imprisonment for life shall be reckoned as equivalent to imprisonment for twenty years. It
is now conclusively settled that the punishment of imprisonment for life handed down by
the Court means a sentence of imprisonment for the convict for the rest of his life. As
regards provisions for remission of sentence under Criminal P. C. and the prisons Act and
Rules they can apply only to
@page-SC3041
sentences of fixed term and life imprisonment, being for the rest of life, is by nature
indeterminate. The remission rules are, however, applied to life convicts by deemed
conversion of life imprisonment into a fixed term of 20 years. The deemed conversion of
life imprisonment into one for fixed term by executive orders issued by the State
Governments apparently flies in the face of a long line of decisions by Supreme Court
and has no statutory sanction. Life convicts are therefore, granted remission and released
from prison on completing the fourteen year term without any sound legal basis.
Moreover, remission is allowed to life convicts in the most mechanical manner without
any sociological or psychiatric appraisal of the convict and without any proper
assessment as to the effect of the early release of a particular convict on the society. The
grant of remission is the rule and remission is denied in the rarest of the rare cases.
(Paras 54, 55, 58, 63)
(C) Penal Code (45 of 1860), S.302 - MURDER - DEATH SENTENCE - LIFE
IMPRISONMENT - Murder - Death sentence - Accused committing murder of his wife
in greed of money - Deceased administered a high dose of sleeping drugs - She was put
alive in a wooden box - And hurried in a pit dug outside bedroom - Murder was
committed in a planned and cold blooded manner - But it could be said that deceased
never knew that she was betrayed - Moreover, though killing was quite ghastly it did not
cause any mental or physical pain to the victim - Accused had also confessed to his guilt -
Held, considering lack of uniformity in sentencing system and absolute irrevocability of
death penalty, sentencing accused to death would not be proper - Death sentence
substituted by life imprisonment for rest of life. (Paras 36, 37, 69)
Cases Referred : Chronological Paras
2007 AIR SCW 5836 : AIR 2007 SC 3221 : 2007 Cri LJ 4697 54
2006 (13) Scale 467 31
2005 AIR SCW 1836 : AIR 2005 SC 2132 54
2005 AIR SCW 4524 : AIR 2005 SC 3440 : 2005 Cri LJ 4124 (Rel. on) 44, 46, 54
2003 AIR SCW 5068 : AIR 2003 SC 4427 : 2003 Cri LJ 5021 (Rel. on) 47
(2003) 7 SCC 121 (Ref.) 51, 53
2002 AIR SCW 3347 : AIR 2002 SC 3006 : 2002 Cri LJ 3927 : 2002 AIR Jhar HCR 1029
(Rel. on) 43
2002 AIR SCW 3555 : AIR 2002 SC 3088 : 2002 Cri LJ 4295 (Ref.) 51
2001 AIR SCW 1139 : AIR 2001 SC 1312 : 2001 Cri LJ 1692 (Ref.) 40, 46, 54
2001 AIR SCW 1492 : AIR 2001 SC 1903 : 2001 Cri LJ 1825 (Rel. on) 40, 46
2001 AIR SCW 2189 : AIR 2001 SC 2342 : 2001 Cri LJ 2925 41, 42, 43, 46, 65
2001 AIR SCW 4717 : AIR 2002 SC 143 : 2002 Cri LJ 226 (Rel. on) 46, 50
2001 AIR SCW 5111 : AIR 2002 SC 340 : 2002 Cri LJ 928 (Rel. on) 42, 43
2000 AIR SCW 3060 : AIR 2000 SC 2762 : 2000 Cri LJ 4017 54, 65
1996 AIR SCW 3120 : AIR 1996 SC 2512 : 1996 Cri LJ 3586 (Ref.) 51
1992 AIR SCW 3658 : AIR 1993 SC 1218 : 1993 Cri LJ 314 47
1991 AIR SCW 1826 : AIR 1991 SC 1792 : 1991 Cri LJ 2483 47, 54, 55
AIR 1985 SC 1050 (Ref.) 40, 46, 62
AIR 1983 SC 855 : 1983 Cri LJ 1345 (Rel. on) 54
AIR 1983 SC 957 : 1983 Cri LJ 1457 21, 22, 27, 28, 29, 31
AIR 1981 SC 174 (2) 40, 46
AIR 1980 SC 898 : 1980 Cri LJ 636 22, 23, 24, 27, 28, 29, 30, 31, 64, 67
AIR 1980 SC 2147 : 1980 Cri LJ 1440 (Ref.) 40, 54, 65
AIR 1979 SC 916 : 1979 Cri LJ 792 38
AIR 1979 SC 1384 : 1979 Cri LJ 1058 (Ref.) 38, 42, 43, 54
AIR 1976 SC 1552 : 1976 Cri LJ 1192 (Ref.) 40, 46, 65
(1976) 428 US 153 : 49 Law Ed 2d 859 27
AIR 1973 SC 947 : 1973 Cri LJ 370 22, 23, 25, 27, 64
(1971) 402 US 183 : 28 Law Ed 2d 711 25
AIR 1961 SC 600 : 1961 (1) Cri LJ 736 (Ref.) 48, 54, 55, 65
AIR 1945 PC 64 : (1945) 46 Cri LJ 626 65
Varinder Kumar Sharma, Varun Thakur and Alok Bagrecha, for Appellant; Sanjay R.
Hegde, Amit Chawla, S. J. Aristotle, A. Rohen Singh and Vikrant Yadav, for Respondent;
U. U. Lalit, Sr. Advocate, Rana Mukherjee, Ms. Uttara Babbar, Siddharth Gautam and
Goodwill Indeevar, for Complainant.
Judgement
1. AFTAB ALAM, J. :-Death to a cold blooded murderer or life, albeit subject to severe
restrictions of personal liberty, is the vexed question that once again arises before this
court. A verdict of death would cut the matter cleanly, apart from cutting short
@page-SC3042
the life of the condemned person. But a verdict of imprisonment for life is likely to give
rise to certain questions. (Life after all is full of questions!). How would the sentence of
imprisonment for life work out in actuality? The Court may feel that the punishment
more just and proper, in the facts of the case, would be imprisonment for life with life
given its normal meaning and as defined in section 45 of the Indian Penal Code. The
Court may be of the view that the punishment of death awarded by the trial court and
confirmed by the High Court needs to be substituted by life imprisonment, literally for
life or in any case for a period far in excess of fourteen years. The Court in its judgment
may make its intent explicit and state clearly that the sentence handed over to the convict
is imprisonment till his last breath or, life permitting, imprisonment for a term not less
than twenty, twenty five or even thirty years. But once the judgment is signed and
pronounced, the execution of sentence passes into the hands of the executive and is
governed by different provisions of law. What is the surety that the sentence awarded to
the convict after painstaking and anxious deliberation would be carried out in actuality?
The sentence of imprisonment for life, literally, shall not by application of different kinds
of remission, turn out to be the ordinary run of the mill life term that works out to no
more than fourteen years. How can the sentence of imprisonment for life (till its full
natural span) given to a convict as a substitute for the death sentence be viewed
differently and segregated from the ordinary life imprisonment given as the sentence of
first choice? These are the questions that arise for consideration in this case.
2. The conviction of the appellant, Swamy Shardanannda @ Murali Manohar Mishra
under Sections 302 and 201 of the Indian Penal Code has attained finality and is no
longer open to scrutiny. The appellant was convicted by the learned XXV City Sessions
Judge, Bangalore City, under the aforesaid two sections by judgement and order dated 20
May, 2005 in SC No.212/1994. The Sessions Judge sentenced him to death for the
offence of murder and to a term of five years rigorous imprisonment and fine of rupees
ten thousand for causing disappearance of evidences of the offence; in default of payment
of fine the direction was to undergo simple imprisonment for one year. The appellant's
appeal (Criminal Appeal No. 1086 of 2005) against the judgment and order passed by the
trial court and the reference made by the Sessions Judge under section 366 of the Code of
Criminal Procedure (Criminal Referred Case No.6 of 2005) were heard together by the
Karnataka High Court. The High Court confirmed the conviction and the death sentence
awarded to the appellant and by judgment and order dated 19 September, 2005 dismissed
the appellant's appeal and accepted the reference made by the trial court without any
modification in the conviction or sentence. Against the High Court judgment the
appellant has come to this Court in this appeal. The Appeal was earlier heard by a bench
of two judges. Both the honourable judges unanimously upheld the appellant's conviction
for the two offences but they were unable to agree to the punishment meted out to the
appellant. S. B. Sinha J. felt that in the facts and circumstances of the case the
punishment of life imprisonment, rather than death would serve the ends of justice. He,
however, made it clear that the appellant would not be released from prison till the end of
his life. M. Katju J., on the other hand, took the view that the appellant deserved nothing
but death. It is thus on the limited, though very important and intractable question of
sentence that this appeal has come before us.
3. This takes us to the facts of the case that has all the elements of high drama. It has a
man's vile greed coupled with the devil's cunning; a woman's craving for a son, coupled
with extreme credulity and gullibility and a daughter's deep and abiding love for her
mother coupled with remarkable perseverance to see through the lies behind her mother's
mysterious disappearance. But a man's life cannot be decided in three sentences and we
must see the prosecution case, as established up to this court in some greater detail.
4. Shakereh, the deceased victim of the crime, came from a highly reputed and wealthy
background. She was the grand daughter of Sir Mirza Ismail, a former Dewan of the
Princely State of Mysore and the daughter of Mr. Ghulam Hussain Namaze and Mrs.
Gauhar Taj Namaze. She held vast and very valuable landed properties in her own right.
Among her various properties was a bungalow at No.81, Richmond Road, Bangalore,
constructed over nearly 38000 square foot of land that she had got in Hiba
@page-SC3043
(oral gift) from her parent's side. Another was a large piece of land measuring 40,000
square foot on Wellington Street that she had got in dowry at the time of marriage.
Shakereh was married to Mr. Akbar Khaleeli, a member of the Indian Foreign Service.
They had four daughters from the marriage. Shakereh came to know the appellant, Murali
Manohar Mishra who called himself Swamy Shraddananda, for the first time in 1983
when she and her family were visiting the erstwhile Nawab of Rampur in New Delhi. The
appellant was introduced as someone who was looking after the Rampur properties and
was said to be quite adept in managing urban landed estates. Shakereh, at that time was
facing some difficulties under the urban land ceiling law and she asked the appellant to
come over to Bangalore and help her in sorting out the problems concerning her
properties. Soon thereafter Akbar Khaleeli was posted as a diplomat to Iran. In those days
Iran was not a family-station for Indian diplomats and hence, he went alone leaving
Shakereh behind in Bangalore. The appellant then came to Bangalore and started living in
a part of her house, 81 Richmond Road, purportedly to assist in the proper management
of her properties. Apparently, more than helping in property matters he worked on her
suppressed though strong desire for a son and was able to convince her that with his
occult powers he could make her beget a son. In 1985, Shakereh and Akbar Khaleeli got
divorced. Shakereh then proceeded to marry the appellant. She paid no heed to the
opposition from family and friends and finally got married to the appellant on 17 April,
1986 under the Special Marriage Act and the marriage was registered at the Sub-
Registrar's Office, Mayo Hall, Bangalore. After marriage they lived together at 81
Richmond Road. For domestic chores they engaged a couple, a man called Raju to work
as gardener-cum handyman and his wife Josephine to work as maid servant. They lived in
the servant's quarter of the bungalow.
5. The daughters from the first marriage were most of the time staying abroad.
6. After marriage Shakereh not only showered her love and affection on the appellant but
also her material wealth. She executed a testamentary will in his favour besides a general
Power of Attorney appointing him as her agent and attorney. She opened a number of
bank accounts jointly with the appellant and also took several bank lockers in their joint
names. They also started together a private company called S. S. Housing Private Limited
of which they alone were the partners.
7. Notwithstanding her matrimonial adventures Shakereh's relations with her daughters
and her parents continued to be more or less as before. They met from time to time and
kept in touch by speaking on the telephone at regular intervals.
8. Then by the end of May 1991, Shakereh suddenly and mysteriously disappeared. She
was last met by her mother Mrs. Gauhar Namaze (examined before the trial court as PW-
25) on 13 April, 1991. Her daughter, Sabah Khaleeli (examined as PW-5) last spoke to
her on telephone on 19 April, 1991 and according to the two servants, Raju and Josephine
(PWs-18 and 19 respectively), they last saw her in the company of the appellant in the
morning of 28 May, 1991. Thereafter, Shakereh was not seen or spoken to by anyone. At
that time she was about forty years old.
9. When Sabah did not receive any call from her mother nor was she able to get through
to her on telephone she enquired about her from the appellant who said that she had gone
to Hyderabad. In June 1991, when she contacted again he told her that her mother had
gone to Kutch to attend the wedding of a wealthy diamond merchant. A week later he told
her that Shakereh was keeping a low profile due to some income tax problems.
Exasperated by the evasive and vague replies by the appellant, Sabah came down to
Bangalore but found no trace of her mother in her house. The appellant then said that
Shakereh was pregnant and she had gone to the United States of America to deliver the
child. He also said that she had got herself admitted in Roosevelt Hospital. Sabah made
enquiries and came to learn that Roosevelt Hospital records did not show admission of
anyone by the name of Shakereh or matching her description. She confronted the
appellant and accused him of giving false information about her mother. He tried to
explain that Shakereh had, in fact, gone to London but she wanted to keep her
whereabouts confidential. However all stories fabricated by the appellant about her
mother lay totally exposed to Sabah when she called on him in a hotel room in Bombay
and chanced upon the passport of her
@page-SC3044
mother lying around. A glance at the passport made it clear that its holder had not gone to
the United States or the United Kingdom or as a matter of fact anywhere out of the
country. At this stage, she came to Bangalore and lodged a written complaint at Ashok
Nagar Police Station where it was registered on 10 June 1992 simply as a woman missing
complaint bearing Cr.No.417/1992.
10. The search for the 'missing' woman started in a rather lukewarm way but the appellant
thought that the time had come to start covering his flanks. He went to the court seeking
anticipatory bail. In the bail petition he declared his total innocence and stated that
perennial litigation with close relations drove Shakereh to acute depression and in that
state, while he was away from Bangalore, she left the house in a fit of anger without
leaving any signs as to where she was headed. He was able to obtain anticipatory bail,
initially on certain condition that was later on greatly relaxed.
11. The investigation by Ashok Nagar police station did not yield any results but the
persistence of Sabah paid off. In March 1994, the Central Crime Branch (C.C.B.),
Bangalore took over the investigation of the complaint about the 'missing' Shakereh. The
case came under the charge of C. Veeraiah, CPI, CCB (PW 37) who, suspecting the role
of the appellant in the disappearance of Shakereh, subjected him to close interrogation.
Under intense interrogation the appellant broke down and owned up to having killed
Shakereh. He narrated in detail the manner of her killing and disposing of her body. He
stated that he put the body of Shakereh inside a large wooden box (that he had earlier got
made for the purpose) and got the box dropped into a pit (that he had got specially dug
up) in the grounds of 81 Richmond Road just outside their common bed-room. He then
got the pit filled up by earth and the ground-surface cemented and covered up with stone
slabs. He volunteered to take the Investigating Officer (IO) to the place and identify the
exact spot where Shakereh lay buried inside the wooden box. The appellant made the
following statement before the IO on 28 March 1994.
"If I am taken I will show the place where the wooden box was prepared and the person
who prepared it, the persons who transported the box and the people who helped in
digging out the pit and the crow bar, spade, pan used for digging pit, the cement bags and
the spot where Shakereh is buried and I exhume the dead body of the deceased and show
you. The statement what all I had earlier given to Ashok Nagar police was a false
statement given intentionally just to escape myself."
The IO then obtained an exhumation order from the Magistrate and after completing the
other legal formalities, on March 30, 1994 brought the appellant to 81 Richmond Road
along with the exhumation team. They were taken by the appellant to the rear of the
house passing through the dining hall and the kitchen. The place was open to the sky but
was enclosed on all the four sides by high walls; the floor was made of kadapa slabs
cemented at the joints. The place had no other access apart from the entry through the
kitchen. There the appellant identified the exact spot where the wooden box, with the
body of Shakereh inside it, lay buried and marked it with a piece of chalk. The
exhumation process started at 10.30 a.m. and the whole process was video graphed (as
per MO 18).
12. As pointed out by the appellant, first the stone slabs were removed and the cemented
portion below the slabs was broken up. Then the ground below was dug up and sure
enough a large wooden box was found lying deep under. The box had inside it, on top, a
foam mattress, a pillow and a bedsheet. Under the mattress was a skeleton with a sleeping
gown around it. The bones had all become disjointed. The skeleton and the long hair tufts
lying around the skull were taken out and the forensic experts rearranged the bones and
also fixed the skull and the mandibles. There was no doubt that it was a human skeleton.
Mrs. Gauhar Taj Namaze identified a red stone ring and two black rings found in the
wooden box (that must have slipped down the fingers after the flesh decayed away) as
belonging to her daughter Shakereh. The sleeping gown that was around the skeleton was
identified by the maid as belonging to her mistress Shakereh.
13. The post mortem examination was held on the same day from 4.45 to about 6 p.m.
14. The skull along with an undisputed photograph of Shakereh was sent to the Forensic
Science Laboratory for matching and identification by Photo Superimposition method.
The skeletal remains were subjected to D.N.A. fingerprinting. Both the tests gave
@page-SC3045
the same result and left no room for doubt that the skeleton was of Shakereh.
15. On 31 March, 1994 the IO once again took the appellant to 81 Richmond Road. This
time the appellant took the IO to the bedroom and showed the window that opened on the
enclosed space from where the skeleton of the deceased was recovered on the previous
day. He also explained that he had got the lower part of the room's wall broken down to
make a clearing through which the wooden box containing Shakereh's body was pushed
out of the room and into the pit. He also produced before the IO pills of eight different
kinds and the cheque books of different bank accounts.
16. The other aspect of the case is equally significant in that it provides the motive for the
murder. It came to light during investigation that after Shakereh disappeared (or, in
retrospect, was killed by the appellant) he went about selling off her properties as fast as
possible. On 30 and 31 March, 1992, in two days, the appellant sold 34 plots carved out
of Shakereh's properties to various people under registered sale-deeds using the General
Power of Attorney executed by her in his favour. The joint bank accounts were simply
used to deposit large sums being the sale proceeds of the lands sold by him and to
withdraw the amounts as soon as those were credited to the account. Needless to say that
from May 1991, it was the appellant alone who operated the joint bank accounts. He also
literally cleaned out the bank lockers that Shakereh had taken in their joint names.
17. In all the meetings of the S. S. Housing Company, he represented the presence of
Shakereh and signed the proceedings for himself and for her as holder of her General
Power of Attorney. The proceedings of the meetings were regularly sent to their
Chartered Accountant.
18. The appellant also gave regular replies to the queries of the Income Tax authorities,
one of which, of the year 1993 contains his signature and the signature of Shakereh which
is apparently forged.
19. In light of the large amount of evidences unearthed against the appellant he was
charged with the commission of murder of his wife Shakereh. As is evident, the case
against the appellant was completely based on circumstantial evidence. But the
prosecution proved its case to the hilt by examining 39 witnesses and producing before
the court a large number of exhibits, both material (MOs. 1 to 33) and documentary (P1
to P267).
20. These are, in brief, the facts of the case. On these facts, Mr. Sanjay Hegde, learned
counsel for the State of Karnataka, supported the view taken by Katju J. (as indeed by the
High Court and the trial court) and submitted that the appellant deserved nothing less
than death. In order to bring out the full horror of the crime Mr. Hegde reconstructed it
before the court. He said that after five years of marriage Shakereh's infatuation for the
appellant had worn thin. She could see through his fraud and see him for what he was, a
lowly charlatan. The appellant could sense that his game was up but he was not willing to
let go all the wealth and the lavish life style that he had gotten used to. He decided to kill
Shakereh and take over all her wealth directly. In furtherance of his aim he conceived a
terrible plan and executed it to perfection. He got a large pit dug up at a 'safe' place just
outside their bed room. The person who was to lie into it was told that it was intended for
the construction a soak- pit for the toilet. He got the bottom of one of the walls of the
bedroom knocked off making a clearing to push the wooden box through; God only
knows saying what to the person who was to pass through it. He got a large wooden box
(7x2x2 feet) made and brought to 81 Richmond Road where it was kept in the guest
house; mercifully out of sight of the person for whom it was meant. Having thus
completed all his preparations he administered a very heavy dose of sleeping drugs to her
on 28 May, 1991 when the servant couple, on receiving information in the morning
regarding a death in their family in a village in Andhra Pradesh asked permission for
leave and some money in advance. However, before giving them the money asked for
and letting them go, the appellant got the large wooden box brought from the guest house
to the bedroom by Raju (with the help of three or four other persons called for the
purpose) where, according to Raju, he saw Shakereh (for the last time) lying on the bed,
deep in sleep. After the servants had gone away and the field was clear the appellant
transferred Shakereh along with the mattress, the pillow and the bed sheet from the bed to
the box, in all probability while she was still alive. He then shut the lid of the
@page-SC3046
box and pushed it through the opening made in the wall into the pit, dug just outside the
room, got the pit filled up with earth and the surface cemented and covered with stone
slabs.
21
. What the appellant did after committing murder of Shakereh was, according to Mr.
Hegde even more shocking. He continued to live, like a ghoul, in the same house and in
the same room and started a massive game of deception. To Sabah, who desperately
wanted to meet her mother or at least to talk to her, he constantly fed lies and represented
to the world at large that Shakereh was alive and well but was simply avoiding any social
contacts. Behind the facade of deception he went on selling Shakereh's properties as
quickly as possible to convert those into cash for easy appropriation. In conclusion Mr.
Hegde submitted that it was truly a murder most foul and Katju J. was perfectly right in
holding that this case came under the first, second and the fifth of the five categories, held
by this Court as calling for the death sentence, in Machhi Singh and Ors. vs. State of
Punjab, (1983) 3 SCC 470. AIR 1983 SC 957

22. In order to properly appreciate the decision in Machhi Singh it would be necessary to
first go to its precursor, the Constitution Bench decision in Bachan Singh vs. State of
Punjab, AIR 1980 SC 898 and to an earlier Constitution Bench decision in Jagmohan
Singh vs. State of U.P., AIR 1973 SC 947, that is the precursor of Bachan Singh. The
decisions in Jagmohan Singh and Bachan Singh deal with the recurrent debate on
abolition of death penalty and are primarily concerned with the question of legitimacy of
the death sentence. Jagmohan relates to the period when the requirement for the court to
state reasons for not giving death sentence but giving the alternate sentence of life
imprisonment in a capital offence was done away with by deletion of Section 367(5) in
the Code of Criminal Procedure, 1898 and the requirement to state reasons for giving
death sentence and not the alternate of life imprisonment under Section 354(3) of the
Code of Criminal Procedure, 1973 was yet to be introduced. Bachan Singh relates to the
period after the Code of Criminal Procedure, 1973 came into force that gives to the
accused the right of pre-sentence hearing under Section 235(2) and under Section 354(3)
casts an obligation on the court to state the 'special reasons' for awarding the sentence of
death and not its alternate, the imprisonment for life or imprisonment for a term of years.
On both occasions the court upheld the Constitutional validity of death sentence for
murder and the other capital offences in the Penal Code.
23

. We are not concerned here with the issue of the Constitutionality of death sentence that
stands conclusively settled by two Constitution Bench decisions. What is of importance
for our present purpose is that both the Constitution Benches firmly declined to be drawn
into making any stand-ardisation or categorisation of cases for awarding death penalty? It
was strongly urged before the Court that in order to save the sentence of death from the
vice of arbitrariness it was imperative for the Court to lay down guidelines, to mark and
identify the types of murder that would attract the punishment of death, leaving aside the
other kinds of murder for the lesser option of the sentence of imprisonment for life. In
Jagmohan the Court turned down the submission observing (in paragraph 25 of the
judgment) as follows :AIR 1973 SC 947

"In India this onerous duty is cast upon Judges and for more than a century the Judges are
carrying out this duty under the Indian Penal Code. The impossibility of laying down
standards is at the very core of the criminal law as administered in India which invests the
Judges with a very wide discretion in the matter of fixing the degree of punishment. That
discretion in the matter of sentence is as already pointed out, liable to be corrected by
superior courts. Laying down of standards to the limited extent possible as was done in
the Model Judicial Code would not serve the purpose. The exercise of judicial discretion
on well recognized principles is in the final analysis the safest possible safeguards for the
accused.
(Emphasis added)

Barely seven years later, the same argument was advanced with even greater force before
another Constitution Bench in Bachan Singh vs. State of Punjab (supra). It was contended
that under Section 354(3) the requirement of giving special reasons' for awarding death
sentence was very loose and it left the doors open for imposition of death penalty in an
arbitrary and whimsical manner. It was further contended that for the sake of saving
AIR 1980 SC 898

@page-SC3047
the Constitutional validity of the provision the Court must step in to clearly define its
scope by unmistakably marking the types of grave murders and other capital offences that
would attract death penalty rather than the alternate punishment of imprisonment for life.
24. As on the earlier occasion, in Bachan Singh too the Court rejected the submission.
The Court did not accept the contention that asking the Court to state special reasons for
awarding death sentence amounted to leaving the Court to do something that was
essentially a legislative function. The Court held that the exercise of judicial discretion on
well established principles and on the facts of each case was not the same as to legislate.
On the contrary, the Court observed, any attempt to standardise or to identify the types of
cases for the purpose of death sentence would amount to taking up the legislative
function. The Court said that a 'standardization or sentencing discretion is a policy matter
which belongs to the sphere of legislation' and 'the Court would not by overleaping its
bounds rush to do what Parliament, in its wisdom, warily did not do.'
25. The Court also rejected the other submission that unless it precisely defined the scope
of Section 354(3) and clearly marked the types of grave murders and capital offences
there would always be the chance of imposition of death penalty in an arbitrary and
whimsical manner. In paragraph 168 of the judgment the Court observed as follows :
"Now, remains the question whether this Court can lay down standards or norms
restricting the area of the imposition of death penalty to a narrow category of murders."

It discussed the issue at length from paragraphs 169 to 195 and firmly refused to do any
categorisation or standardisation of cases for the purpose of death sentence. In the lengthy
discussion on the issue, the Court gave over half a dozen different reasons against the
argument urging for standardisation and categorisation of cases; it also cited the
American experience to show the futility of any such undertaking. A perusal of that part
of the judgment shows that a very strong plea was made before the Court for
standardisation and categorisation of cases for the purpose of death sentence. Nonetheless
the Court remained resolute in its refusal to undertake the exercise. In this regard the
court agreed with the view earlier taken in Jagmohan and observed that it was not
possible to make an exhaustive enumeration of aggravating or mitigating circumstances
which should be taken into consideration when sentencing an offender. It extracted the
passage from Jagmohan that quoted with approval the observation from an American
decision in McGautha vs. California, (1971) 402 US 183. AIR 1973 SC 947

"The infinite variety of cases and facets to each case would make general standards either
meaningless 'boiler plate' or a statement of the obvious that no Jury/Judge would need."
It also reiterated the observation in Jagmohan that such "standardisation" is well-nigh
impossible.
26. Arguing against standardisation of cases for the purpose of death sentence the Court
observed that even within a single category offence there are infinite, unpredictable and
unforeseeable variations. No two cases are exactly identical. There are countless
permutations and combinations which are beyond the anticipatory capacity of the human
calculus. The Court further observed that standardisation of the sentencing process tends
to sacrifice justice at the altar of blind uniformity.
27. It is significant to note that the Court was extremely wary of dealing with even the
question of indicating the broad criteria which should guide the Courts in sentencing a
convict of murder. It reminded itself of the observation of Stewart, J. in Greg vs. Georgia,
'while we have an obligation to ensure that the constitutional bounds are not overreached,
we may not act as judges as we might as legislatures'. Having thus cautioned itself,
though the Court recorded the suggestions of Dr. Chitale, one of the counsels appearing
in the case, as regards the 'aggravating circumstances' and the 'mitigating circumstances',
it was careful not to commit itself to Dr. Chitale's categories. In paragraph 200 the
judgment recorded the 'aggravating circumstances' suggested by Dr. Chitale, but in
paragraph 201 it observed as follows :
"Stated broadly, there can be no objection to the acceptance of these indicators but as we
have indicated already, we would prefer not to fetter judicial discretion by attempting to
make an exhaustive enumeration
@page-SC3048
one way or the other."
Similarly, in paragraph 204 the judgment recorded the 'mitigating circumstances' as
suggested by Dr. Chitale. In paragraph 205, however, it observed as follows :
"We will do no more than to say that these are undoubtedly relevant circumstances and
must be given great weight in the determination of sentence. Some of these factors like
extreme youth can instead be of compelling importance."

In the end, the Court following the decision in Jagmohan left the sentencing process
exactly as it came from the legislative, flexible and responsive to each case on its merits,
subject to the discretion of the Court and in case of any error in exercise of the discretion
subject further to correction by the Superior Court(s). The Court observed : "In
Jagmohan, this Court had held that this sentencing discretion is to be exercised judicially
on well-recognised principles, after balancing all the aggravating and mitigating
circumstances of the crime. By "well-recognised principles" the Court obviously meant
the principles crystallized by judicial decisions illustrating as to what were regarded as
aggravating or mitigating circumstances in those cases. The legislative changes since
Jagmohan - as we have discussed already-do not have the effect of abrogating or
nullifying those principles. The only effect is that the application of those principles is
now to be guided by the paramount beacons of legislative policy discernible from
Sections 354(3) and 235(2), namely : (1) The extreme penalty can be inflicted only in
gravest cases of extreme culpability; (2) In making choice of the sentence, in addition to
the circumstances of the offence, due regard must be paid to the circumstances of the
offender also." AIR 1973 SC 947

Earlier in the judgment while reaffirming Jagmohan, subject of course to certain


adjustments in view of the legislative changes (section 354(3)) the Court observed :
"The expression "special reasons" in the context of this provision, obviously means
"exceptional reasons" founded on the exceptionally grave circumstances of the particular
case relating to the crime as well as the criminal. Thus, the legislative policy now writ
large and clear on the face of Section 354(3) is that on conviction for murder and other
capital offences punishable in the alternative with death under the Penal Code, the
extreme penalty should be imposed only inextreme cases."
(Emphasis added)
In conclusion the Constitution Bench decision in Bachan Singh said :

".........It is, therefore, imperative to voice the concern that courts, aided by the broad
illustrative guidelines indicated by us, will discharge the onerous function with evermore
scrupulous care and humane concern, directed along the highroad of legislative policy
outlined in Section 354(3) viz. that for persons convicted of murder, life imprisonment is
the rule and death sentence anexception. A real and abiding concern for the dignity of
human life postulates resistance to taking a life through law's instrumentality. That ought
not to be done save in the rarest of rare cases when the alternative option is
unquestionably foreclosed." AIR 1980 SC 898, (Para 207)

(Emphasis added)

The Bachan Singh principle of 'rarest of rare cases' came up for consideration and
elaboration in the case of Machhi Singh. It was a case of extraordinary brutality. On
account of a family feud Machhi Singh the main accused in the case, along with eleven
accomplices, in course of a single night, conducted raids on a number of villages killing
seventeen people, men, women and children for no reason other than they were related to
one Amar Singh and his sister Piyaro Bai. The death sentence awarded to Machhi Singh
and two other accused by the Trial Court and affirmed by the High Court was also
confirmed by this Court. In Machhi Singh the Court put itself in the position of the
'Community' and observed that though the 'Community' revered and protected life
because 'the very humanistic edifice is constructed on the foundation of reverence for life
principle' it may yet withdraw the protection and demand death penalty, AIR 1983 SC
957

"It may do so 'in rarest of rare cases' when its collective conscience is so shocked that it
will expect the holders of the judicial power centre to inflict death penalty irrespective of
their personal opinion as regards desirability or otherwise of retaining death penalty. The
community may entertain such a sentiment when the crime is viewed from the platform
of the motive for, or the manner of commission of the crime, or the antisocial or
abhorrent nature of the crime, such
@page-SC3049
as for instance :
I. Manner of commission of murder
When the murder is committed in an extremely brutal, grotesque, diabolical, revolting or
dastardly manner so as to arouse intense and extreme indignation of the community. For
instance,
(i) when the house of the victim is set aflame with the end in view to roast him alive in
the house.
(ii) when the victim is subjected to inhuman acts of torture or cruelty in order to bring
about his or her death.
(iii) when the body of the victim is cut into pieces or his body is dismembered in a
fiendish manner.
II. Motive for commission of murder When the murder is committed for a motive which
evinces total depravity and meanness. For instance, when (a) a hired assassin commits
murder for the sake of money or reward, (b) a cold-blooded murder is committed with a
deliberate design in order to inherit property or to gain control over property of a ward or
a person under the control of the murderer or vis-a-vis whom the murderer is in a
dominating position or in a position of trust, or (c) a murder is committed in the course
for betrayal of the motherland.
III. Anti-social or socially abhorrent nature of the crime
(a) When murder of a member of a Scheduled Caste or minority community etc., is
committed not for personal reasons but in circumstances which arouse social wrath. For
instance, when such a crime is committed in order to terrorize such persons and frighten
them into fleeing from a place or in order to deprive them of, or make them surrender,
lands or benefits conferred on them with a view to reverse past injustices and in order to
restore the social balance.
(b) In cases of 'bride burning' and what are known as 'dowry deaths' or when murder is
committed in order to remarry for the sake of extracting dowry once again or to marry
another woman on account of infatuation.
IV. Magnitude of crime
When the crime is enormous in proportion. For instance when multiple murders say of all
or almost all the members of a family or a large number of persons of a particular caste,
community, or locality, are committed.
V. Personality of victim of murder When the victim of murder is (a) an innocent child
who could not have or has not provided even an excuse, much less a provocation, for
murder, (b) a helpless woman or a person rendered helpless by old age or infirmity, (c)
when the victim is a person visa-vis whom the murderer is in a position of domination or
trust, (d) when the victim is a public figure generally loved and respected by the
community for the services rendered by him and the murder is committed for political or
similar reasons other than personal reasons."
In Machhi Singh the Court held that for practical application the rarest of rare cases
principle must be read and understood in the background of the five categories of murder
cases enumerated in it. Thus the standardisation and classification of cases that the two
earlier Constitution Benches had resolutely refrained from doing finally came to be done
in Machhi Singh.
28

. In Machhi Singh the Court crafted the categories of murder in which 'the Community'
should demand death sentence for the offender with great care and thoughtfulness. But
the judgment in Machhi Singh was rendered on 20th July, 1983, nearly twenty five years
ago, that is to say a full generation earlier. A careful reading of the Machhi Singh
categories will make it clear that the classification was made looking at murder mainly as
an act of maladjusted individual criminal (s). In 1983 the country was relatively free from
organised and professional crime. Abduction for Ransom and Gang Rape and Murders
committed in course of those offences were yet to become a menace for the society
compelling the Legislature to create special slots for those offences in the Penal Code. At
the time of Machhi Singh, Delhi had not witnessed the infamous Sikh carnage. There was
no attack on the country's Parliament. There were no bombs planted by terrorists killing
completely innocent people, men, women and children in dozens with sickening
frequency. There were no private armies. There were no mafia cornering huge
government contracts purely by muscle power. There were no reports of killings of social
activists and 'whistle blowers'. There were no reports of custodial deaths and rape and
fake encounters by police or even by armed forces. These developments would
unquestionably find a more pronounced reflection AIR 1983 SC 957
AIR 1980 SC 898

@page-SC3050
in any classification if one were to be made today. Relying upon the observations in
Bachan Singh, therefore, we respectfully wish to say that even though the categories
framed in Machhi Singh provide very useful guidelines, nonetheless those cannot be
taken as inflexible, absolute or immutable. Further, even in those categories, there would
be scope for flexibility as observed in Bachan Singh itself.
29. The matter can be looked at from another angle. In Bachan Singh it was held that the
expression "special reasons" in the context of the provision of Section 354(3) obviously
means "exceptional reasons" founded on the exceptionally grave circumstances of the
particular case relating to the crime as well as the criminal. It was further said that on
conviction for murder and other capital offences punishable in the alternative with death
under the Penal Code, the extreme penalty should be imposed only in extreme cases. In
conclusion it was said that the death penalty ought not to be imposed save in the rarest of
rare cases when the alternative option is unquestionably foreclosed. Now, all these
expressions "special reasons", "exceptional reasons", "founded on the exceptional grave
circumstances", "extreme cases" and "the rarest of the rare cases" unquestionably indicate
a relative category based on comparison with other cases of murder. Machhi Singh for the
purpose of practical application sought to translate this relative category into absolute
terms by framing the five categories. (In doing so, it is held by some, Machhi Singh
considerably enlarged the scope for imposing death penalty that was greatly restricted by
Bachan Singh!).
30. But the relative category may also be viewed from the numerical angle, that is to say,
by comparing the case before the Court with other cases of murder of the same or similar
kind, or even of a graver nature and then to see what punishment, if any was awarded to
the culprits in those other cases. What we mean to say is this, if in similar cases or in
cases of murder of a far more revolting nature the culprits escaped the death sentence or
in some cases were even able to escape the criminal justice system altogether it would be
highly unreasonable and unjust to pick on the condemned person and confirm the death
penalty awarded to him/her by the courts below simply because he/she happens to be
before the Court. But to look at a case in this perspective this Court has hardly any field
of comparison. The court is in a position to judge 'the rarest of rare cases' or an
'exceptional case' or an 'extreme case' only among those cases that come to it with the
sentence of death awarded by the trial court and confirmed by the High Court. All those
cases that may qualify as the rarest of rare cases and which may warrant death sentence
but in which death penalty is actually not given due to an error of judgment by the trial
court or the High Court automatically fall out of the field of comparison. More important
are the cases of murder of the worst kind, and their number is by no means, small, in
which the culprits, though identifiable, manage to escape any punishment or are let off
very lightly. Those cases never come up for comparison with the cases this Court might
be dealing with for confirmation of death sentence. To say this is because our Criminal
Justice System, of which the court is only a part, does not work with a hundred per cent
efficiency or anywhere near it, is not to say something remarkably new or original. But
the point is, this Court, being the highest court of the Land, presiding over a Criminal
Justice System that allows culprits of the most dangerous and revolting kinds of murders
to slip away should be extremely wary in dealing with death sentence and should resort to
it, in the words of Bachan Singh, only when the other alternative is unquestionably
foreclosed. We are not unconscious of the simple logic that in case five crimes go
undetected and unpunished that is no reason not to apply the law to culprits committing
the other five crimes. But this logic does not seem to hold good in case of death penalty.
On this logic a convict of murder may be punished with imprisonment for as long as you
please. But death penalty is something entirely different. No one can undo an executed
death sentence.
31

. That is not the end of the matter. Coupled with the deficiency of the Criminal Justice
System is the lack of consistency in the sentencing process even by this Court. It is noted
above that Bachan Singh laid down the principle of the rarest of rare cases. Machhi
Singh, for practical application crystallised the principle into five definite categories of
cases of murder and in doing so also considerably AIR 1980 SC 898
AIR 1983 SC 957

@page-SC3051
enlarged the scope for imposing death penalty. But the unfortunate reality is that in later
decisions neither the rarest of rare cases principle nor the Machhi Singh categories were
followed uniformly and consistently. In Aloke Nath Dutta vs. State of West Bengal, 2006
(13) SCALE 467, Sinha J. gave some very good illustrations from a number of recent
decisions in which on similar facts this Court took contrary views on giving death penalty
to the convict (see paragraphs 154 to 182, pp. 504-510 SCALE). He finally observed that
'courts in the matter of sentencing act differently although the fact-situation may appear
to be somewhat similar' and further 'it is evident that different Benches had taken
different view in the matter'. Katju, J. in his order passed in this appeal said that he did
not agree with the decision in Aloke Nath Dutt in that it held that death sentence was not
to be awarded in a case of circumstantial evidence. Katju, J. may be right that there
cannot be an absolute rule excluding death sentence in all cases of circumstantial
evidence (though in Aloke Nath Dutta it is said 'normally' and not as an absolute rule).
But there is no denying the illustrations cited by Sinha, J. which are a matter of fact.
32. The same point is made in far greater detail in a report called, "Lethal Lottery, The
Death Penalty in India" compiled jointly by Amnesty International India and Peoples
Union For Civil Liberties, Tamil Nadu and Puducherry. The report is based on the study
of Supreme Court judgments in death penalty cases from 1950 to 2006. One of the main
points made in the report (see Chapters 2 to 4) is about the Court's lack of uniformity and
consistency in awarding death sentence.
33. The truth of the matter is that the question of death penalty is not free from the
subjective element and the confirmation of death sentence or its commutation by this
Court depends a good deal on the personal predilection of the Judges constituting the
Bench.
34. The inability of the Criminal Justice System to deal with all major crimes equally
effectively and the want of uniformity in the sentencing process by the Court lead to a
marked imbalance in the end results. On the one hand there appears a small band of cases
in which the murder convict is sent to the gallows on confirmation of his death penalty by
this Court and on the other hand there is a much wider area of cases in which the offender
committing murder of and similar or a far more revolting kind is spared his life due to
lack of consistency by the Court in giving punishments or worse the offender is allowed
to slip away unpunished on account of the deficiencies in the Criminal Justice System.
Thus the overall larger picture gets asymmetric and lop-sided and presents a poor
reflection of the system of criminal administration of justice. This situation is matter of
concern for this Court and needs to be remedied :
35. These are some of the larger issues that make us feel reluctant in confirming the death
sentence of the appellant.
36. Coming now to the facts of the case it is undeniable that the appellant killed Shakereh
in a planned and cold-blooded manner but at least this much can be said in his favour that
he devised the plan so that the victim could not know till the end and even for a moment
that she was betrayed by the one she trusted most. Further though the way of killing
appears quite ghastly it may be said that it did not cause any mental or physical pain to
the victim. Thirdly, as noted by Sinha, J. the appellant confessed his guilt at least partially
before the High Court.
37. We must not be understood to mean that the crime committed by the appellant was
not very grave or the motive behind the crime was not highly depraved. Nevertheless, in
view of the above discussion we feel hesitant in endorsing the death penalty awarded to
him by the trial court and confirmed by the High Court. The absolute irrevocability of the
death penalty renders it completely incompatible to the slightest hesitation on the part of
the court. The hangman's noose is thus taken off the appellant's neck.
38
. But this leads to a more important question about the punishment commensurate to the
appellant's crime. The sentence of imprisonment for a term of 14 years, that goes under
the euphemism of life imprisonment is equally, if not more, unacceptable. As a matter of
fact, Mr. Hegde informed us that the appellant was taken in custody on 28th March, 1994
and submitted that by virtue of the provisions relating to remission, the sentence of life
imprisonment, without any qualification or further direction would, in all likelihood, lead
to his release from jail in the first quarter of 2009 since he has already AIR 1979 SC
1384, (Para 15)

@page-SC3052
ready completed more than 14 years of incarceration. This eventuality is simply not
acceptable to this Court. What then is the answer? The answer lies in breaking this
standardisation that, in practice, renders the sentence of life imprisonment equal to
imprisonment for a period of no more than 14 years; in making it clear that the sentence
of life imprisonment when awarded as a substitute for death penalty would be carried out
strictly as directed by the Court. This Court, therefore, must lay down a good and sound
legal basis for putting the punishment of imprisonment for life, awarded as substitute for
death penalty, beyond any remission and to be carried out as directed by the Court so that
it may be followed, in appropriate cases as a uniform policy not only by this Court but
also by the High Courts, being the superior Courts in their respective States. A suggestion
to this effect was made by this Court nearly thirty years ago in Dalbir Singh and others
vs. State of Punjab, (1979) 3 SCC 745. In paragraph 14 of the judgment this Court held
and observed as follows :

" 14. The sentences of death in the present appeal are liable to be reduced to life
imprisonment. We may add a footnote to the ruling in Rajendra Prasad case. Taking the
cue from the English legislation on abolition, we may suggest that life imprisonment
which strictly means imprisonment for the whole of the men's life but in practice amounts
to incarceration for a period between 10 and 14 years may, at the option of the convicting
court, be subject to the condition that the sentence of imprisonment shall last as long as
life lasts, where there are exceptional indications of murderous recidivism and the
community cannot run the risk of the convict being at large. This takes care of judicial
apprehensions that unless physically liquidated the culprit may at some remote time
repeat murder." AIR 1979 SC 916

[Emphasis added]
We think that it is time that the course suggested in Dalbir Singh should receive a formal
recognition by the Court.
39. As a matter of fact there are sufficient precedents for the Court to take such a course.
In a number of cases this court has substituted death penalty by life imprisonment or in
some cases for a term of twenty years with the further direction that the convict would
not be released for the rest of his life or until the twenty year term was actually served
out. In this case too Sinha, J. passed exactly the same order. After declining to confirm
the death sentence given to the appellant he proceeded to give the following direction.
"However, while saying so, we (sic) direct that in a case of this nature 'life sentence' must
be meant to be 'life sentence'. Such a direction can be given, as would appear from some
precedents."
Sinha, J. then mentioned the following five cases in which this Court had passed similar
orders.
40

. In Subhash Chander vs. Krishan La and others, (2001) 4 SCC 458, five accused persons,
including Krishan Lal were put on trial for committing multiple murders. The trial court
acquitted one of the accused but convicted the rest of them and sentenced each of them to
death. In the death reference/appeals preferred by the convicted accused, the High Court
confirmed the conviction of all the four accused but commuted their death sentence to life
imprisonment. One Subhash Chander (PW-2) came to this Court in appeal. On a
consideration of the material facts this Court felt that the High Court was not justified in
commuting the sentence of death of at least one accused, Krishan Lal. But then the
counsel appearing on his behalf implored that instead of death penalty this Court might
order for imprisonment of Krishan Lal for the remaining period of his life. This Court
took note of the counsel's submission as follows : 2001 AIR SCW 1492, Paras 19 and
23

"Faced with the situation Mr. U. R. Lalit, Senior Counsel appearing for the aforesaid
respondents submitted that instead of depriving Krishan Lal (A-1) of his life, the Court
can pass appropriate order to deprive the aforesaid accused person of his liberty
throughout his life. Upon instructions, the learned Senior Counsel submitted that the said
Krishan Lal, if sentenced to life imprisonment would never claim his premature release or
commutation of his sentence on anyground. We record such a submission made on behalf
of the said accused, upon instructions."
(Emphasis added)
This Court accepted the plea made by the counsel and passed the following order :
"However, in the peculiar circumstances of the case, apprehending imminent danger
@page-SC3053
to the life of Subhash Chander and his family in future, taking on record the statement
made on behalf of Krishan Lal (A-1), we are inclined to hold that for him the
imprisonment for life shall be the imprisonment inprison for the rest of his life. He shall
not be entitled to any commutation or premature release under Section 401 of the Code of
Criminal Procedure. Prisoners Act, Jail Manual or any other statute and the rules made
for the purposes of grant of commutation and remissions."
(Emphasis added)

In Subhash Chander this court referred to an earlier judgment in State of M.P. vs. Ratan
Singh, (1976) 3 SCC 470, in which it was held that a sentence of imprisonment for life
means a sentence for the entire life of the prisoner unless the appropriate Government
chooses to exercise its discretion to remit either the whole or a part of the sentence under
Section 401 of the Code of Criminal Procedure. The Court also referred to the earlier
decisions in Sohan Lal vs. Asha Ram, (1981) 1 SCC 106 (This is a mistake since Sohan
Lal is a completely different case; apparently the reference was to Maru Ram vs. Union
of India on page 107 of the same report), Bhagirath vs. Delhi Administration, (1985) 2
SCC 580 and Zahid Hussein vs. State of West Bengal, (2001) 3 SCC 750. AIR 1976 SC
1552
AIR 1981 SC 174 (2)
AIR 1980 SC 2147
AIR 1985 SC 1050
2001 AIR SCW 1139

41

. In Shri Bhagwan vs. State of Rajasthan, (2001) 6 SCC 296, the appellant, who was 20
years old at the time of commission of the offence, had come to this Court, condemned to
death by the trial court and the High Court. According to prosecution, he had killed five
members of a family by mercilessly battering them to death. The manner of killing was
brutal and the circumstances of the crime exhibited crass ingratitude on the appellant's
part. The motive was theft of gold ornaments and other articles belonging to the victim
family. In this case, K. G. Balakrishnan, J. (as the Hon'ble the Chief Justice was at that
time) who wrote the judgment for the Court commuted the death sentence awarded to the
appellant to imprisonment for life subject to the direction that he would not be released
from the prison until he had served out at least 20 years of imprisonment including the
period already undergone by him. In this case there is also a very useful discussion with
regard to the provisions of commutation and remission in the Code of Criminal Procedure
and the prison rules to which we shall advert later on in this judgment. 2001 AIR
SCW 2189

42

. In Prakash Dhawal Khairnar (Patil) vs. State of Maharashtra, (2002) 2 SCC 35, the
condemned appellant had committed the murder of his own brother, their mother and four
members of his brother's family because the deceased brother was not partitioning the
property which the appellant claimed to be joint family property. In the totality of
circumstances this Court set aside the death sentence awarded to the appellant but
directed that for the murders committed by him, he would suffer imprisonment for life
and further that he would not be released from prison until he had served out at least 20
years of imprisonment including the period already undergone by him. For giving such a
direction, the court referred to the decisions in Shri Bhagwan (supra) and Dalbir Singh v.
The State of Punjab, (1979) 3 SCC 745.] 2001 AIR SCW 5111
2001 AIR SCW 2189
AIR 1979 SC 1384

43

. In Ram Anup Singh and others v. State of Bihar, (2002) 6 SCC 686, there were a father
and his two sons before this court. They had killed the father's brother, the brother's wife,
his daughter and his son-in-law. On conviction for the murders the father was sentenced
to life imprisonment but the two sons were given the death penalty. This Court once again
interfered and set aside the death sentence awarded by the trial court and confirmed by
the High Court to the two sons and instead sentenced them to suffer rigorous
imprisonment for life with the condition that they would not be released before
completing an actual term of 20 years including the period of imprisonment already
undergone by them. Reference was made to the decisions in Shri Bhagwan, Dalbir Singh
and Prakash Dhawal Khairnar (Patil) (supra). 2002 AIR SCW 3347

44

. The fifth decision mentioned by Sinha, J. was in Mohd. Munna vs. Union of India,
(2005) 7 SCC 417. In this case it was basically held that in the absence of an order of
remission formally passed by the appropriate government, there was no provision in the
Penal Code or in the Code of Criminal Procedure 2005 AIR SCW 4524

@page-SC3054
under which a sentence of life imprisonment could be treated as for a term of 14 years or
20 years and further that a convict undergoing imprisonment for life could not claim
remission as a matter of right.
45. To this list of five cases mentioned by Sinha, J. one could add one or two more.
46

. In Jayawant Dattatraya Suryarao vs. State of Maharashtra, (2001) 10 SCC 109, this
Court had before it a batch of five analogous cases. There were three appeals on behalf of
three of the accused convicted by the trial court; another appeal by the State in regard to
the accused who were acquitted by the trial court and a death reference in regard to one
of the appellants, Subhashsingh Shobhanathsingh Thakur (A-6) who was given sentences
of death on two counts, one under the provisions of the Terrorist and Disruptive Activities
(Prevention) Act (TADA) and the other under section 120-B of the Penal Code.
According to the prosecution case the appellants, along with a number of other co-
accused, armed with highly sophisticated weapons had raided J.J. Hospital in Mumbai
where the victim, a member of another underworld gang, was admitted for treatment. In
the hospital they made indiscriminate firing killing not only their target but also two
policemen who were on guard duty and injuring several others. The court confirmed the
conviction of appellant No. 6 but modified the sentence from death penalty to
imprisonment for life - till rest of life. For the direction given by it the court referred to
the decisions in Subhash Chander (supra), State of Madhya Pradesh (supra), Shri
Bhagwan (supra), Sohan Lal (supra), Bhagirath vs. Delhi Administration (supra) and
Zahid Hussein (supra). 2001 AIR SCW 4217

47

. In Nazir Khan and others vs. State of Delhi, (2003) 8 SCC 461, three of the appellants
before the Court were sentenced to death for committing offences punishable under
Section 364-A read with Section 120-B, IPC. They were also convicted under the
provisions of Terrorist and Disruptive Activities (Prevention) Act (TADA) with different
terms of imprisonment for those offences. This Court, however, commuted the death
sentence of the three appellants but having regard to the gravity of the offences and the
dastardly nature of their acts directed for their incarceration for a period of 20 years with
the further direction that the accused-appellants would not be entitled to any remission
from the term of 20 years. Reference was made, to the earlier decisions in Ashok Kumar
vs. Union of India, (1991) 3 SCC 498 and Sat Pal vs. State of Haryana, (1992) 4 SCC
172. 2003 AIR SCW 5068
1991 AIR SCW 1826
1992 AIR SCW 3658

48

. On a perusal of the seven decisions discussed above and the decisions referred to therein
it would appear that this Court modified the death sentence to imprisonment for life or in
some cases imprisonment for a term of twenty years with the further direction that the
convict must not be released from prison for the rest of his life or before actually serving
out the term of twenty years, as the case may be, mainly on two premises; one, an
imprisonment for life, in terms of section 53 read with section 45 of the Penal Code
meant imprisonment for the rest of life of the prisoner and two, a convict undergoing life
imprisonment has no right to claim remission. In support of the second premise reliance
is placed on the line of decisions beginning from Gopal Vinayak Godse vs. The State of
Maharashtra, 1961 (3) SCR 440, and coming down to Mohd. Munna vs. Union of India
(supra). AIR 1961 SC 600
2005 AIR SCW 4524

49. In course of hearing of the appeal before us strong doubts were raised over the
application of the second premise for putting a sentence of imprisonment beyond
remission. It was contended that to say that a convict undergoing a sentence of
imprisonment had no right to claim remission was not the same as the Court, while giving
the punishment of imprisonment, suspending the operation of the statutory provisions of
remission and restraining the appropriate Government from discharging its statutory
function.
50

. In this connection an interesting development was brought to our notice. We were


informed that Subhashsingh Shobhanathsingh Thakur whose death sentence was
modified by this Court to imprisonment for life - till rest of life by its judgment dated 5
November, 2001 in Jayawant Dattatraya Suryarao vs. State of Maharashtra, (supra) has
filed a writ petition under Article 32 of the Constitution before this Court (Writ Petition
(Criminal) No. 36 of 2008: Subhashsingh Shobhanathsingh Thakur vs. The State of
Maharashtra) challenging, on substantially the same grounds, the order of the Court,
insofar as 2001 AIR SCW 4717

@page-SC3055
it directed for the non-application of the statutory provisions of remission to his case.
51
. Our attention was also invited to a decision of this Court in State (Government of NCT
of Delhi) vs. Prem Raj, (2003) 7 SCC 121. In this case, Prem Raj, the accused-respondent
before the court was convicted by the trial court under Section 7 read with Section 13(1)
(d) and 13(2) of the Prevention of Corruption Act and was sentenced to undergo rigorous
imprisonment for two years and a fine of Rs. 500/- under Section 7. He was additionally
sentenced to undergo imprisonment for 3-1/2 years and a fine of Rs. 1,000/- under
Section 13(2) of the Act, subject to the direction that the two sentences would run
concurrently. In appeal, on a plea made on the question of sentence, a learned Single
Judge of the High Court enhanced the amount of fine to Rs. 15,000/-in lieu of the
sentences of imprisonment and directed that on deposit of the amount of fine the State
Government, being the 'appropriate Government' would formalize the matter by passing
an appropriate order under Section 433 (c) of the Code of Criminal Procedure. This
Court, on appeal by the State, held that the question of remission lay within the domain of
the appropriate Government and it was not open to the High Court to give a direction of
that kind. In the case of Prem Raj the Court referred to two earlier decisions in Delhi
Administration vs. Manohar Lal, (2002) 7 SCC 222 and State of Punjab vs. Kesar Singh,
(1996) 5 SCC 495 and in paragraph 13 of the decision observed as follows : (2002 AIR
SCW 3555) 2002 AIR SCW 3555
1996 AIR SCW 3120

"An identical question regarding exercise of power in terms of Section 433 of the Code
was considered in Delhi Admn. (now NCT of Delhi) vs. Manohar Lal. The Bench
speaking through one of us (Doraiswamy Raju, J.) was of the view that exercise of power
under Section 433 was an executive discretion. The High Court in exercise of its
revisional jurisdiction had no power conferred on it to commute the sentence imposed
where a minimum sentence was provided for the offence. In State of Punjab vs. Kesar
Singh, this Court observed as follows [though it was in the context of Section 433(b)] :
(SCC pp.595-96, para 3)"
"The mandate of Section 433, Cr. P. C. enables the Government in an appropriate case to
commute the sentence of a convict and to prematurely order his release before expiry of
the sentence as imposed by the courts.....That apart, even if the High Court could give
such a direction, it could only direct consideration of the case of premature release by the
Government and could not have ordered the premature release of the respondent itself.
The right to exercise the power under Section 433, Cr. P. C., vests in the Government and
has to be exercised by the Government in accordance with the rules and established
principles. The impugned order of the High Court cannot, therefore, be sustained and is
hereby, set aside."
Relying upon the aforesaid two decisions this Court set aside the order of the court but
left it open to the accused to move the appropriate Government for such relief as may be
available in law. It was further clarified that it would be at the sole discretion of the
Government to exercise the power conferred on it in accordance with law.
52. Before us it was submitted that just as the Court could not direct the appropriate
Government for granting remission to a convicted prisoner, it was not open to the Court
to direct the appropriate Government not to consider the case of a convict for grant of
remission in sentence. It was contended that giving punishment for an offence was indeed
a judicial function but once the judgment was pronounced and punishment awarded the
matter no longer remained in the hands of the Court. The execution of the punishment
passed into the hands of the executive and under the scheme of the statute the Court had
no control over the execution.
53. In our view, the submission is wholly misconceived and untenable and the decision in
the case of Prem Raj has no application to the issue under consideration.
54

. At this stage, it will be useful to take a very brief look at the provisions with regard to
sentencing and commutation, remission etc. of sentences. Section 45 of the Penal Code
defines "life" to mean the life of the human being, unless the contrary appears from the
context. Section 53 enumerates punishments, the first of which is death and the second,
imprisonment for life. Sections 54 and 55 give to the appropriate Government the power
of commutation of the sentence of death and the sentence of imprisonment for life
respectively. Section 55A defines "appropriate Government". Section 57 provides that in
calculating fractions of AIR 1961 SC 600
AIR 1979 SC 1384
AIR 1980 SC 2147
AIR 1983 SC 855
1991 AIR SCW 1826
2000 AIR SCW 3060
2001 AIR SCW 1139
2005 AIR SCW 1836
2005 AIR SCW 4524
2007 AIR SCW 5836

@page-SC3056
terms of punishment, imprisonment for life shall be reckoned as equivalent to
imprisonment for twenty years. It is now conclusively settled by a catena of decisions
that the punishment of imprisonment for life handed down by the Court means a sentence
of imprisonment for the convict for the rest of his life. (See the decisions of this Court in
Gopal Vinayak Godse vs. The State of Maharashtra and others, (1961) 3 SCR 440
(Constitution Bench); Dalbir Singh and others vs. State of Punjab, (1979) 3 SCC 745;
Maru Ram vs. Union of India, (1981) 1 SCC 107 (Constitution Bench); Naib Singh vs.
State of Punjab, (1983) 2 SCC 454; Ashok Kumar alias Golu vs. Union of India, (1991) 3
SCC 498; Laxman Naskar (Life Convict) vs. State of W.B., (2000) 7 SCC 626; Zahid
Hussein vs. State of West Bengal, (2001) 3 SCC 750; Kamalanantha vs. State of Tamil
Nadu, (2005) 5 SCC 194; Mohd. Munna vs. Union of India, (2005) 7 SCC 416 and C.A.
Pious vs. State of Kerala, (2007) 8 SCC 312).
55. It is equally well-settled that Section 57 of the Penal Code does not in any way limit
the punishment of imprisonment for life to a term of twenty years. Seption 57 is only for
calculating fractions of terms of punishment and provides that imprisonment for life shall
be reckoned as equivalent to imprisonment for twenty years. (See : Gopal Vinayak Godse
(supra) and Ashok Kumar alias Golu (supra). The object and purpose of Section 57 will
be clear by simply referring to Sections 65, 116, 119, 129 and 511 of the Penal Code.
56. This takes us to the issue of commutation and remission etc. of sentences. The
provisions in regard to commutation, remission, suspension etc. are to be found both in
the Constitution and in the statutes. Articles 72 and 161 of the Constitution deal with the
powers of the President and the Governors of the State respectively to grant pardons,
reprieves, respites or remissions of punishment or to suspend, remit or commute the
sentence of any person convicted of any offence. Here it needs to be made absolutely
clear that this judgment is not concerned at all with the Constitutional provisions that are
in the nature of the State's sovereign power. What is said hereinafter relates only to
provisions of. commutation, remission etc. as contained in the Code of Criminal
Procedure and the Prisons Acts and the Rules framed by the different States.
57. Section 432 of the Code of Criminal Procedure deals with the power to suspend or
remit sentences and Section 433 with the power to commute sentences. Section 433-A,
that was inserted in the Code by an amendment made in 1978, imposes restriction on
powers of remission or commutation in certain cases. It reads as follows :
"Restriction on powers of remission or commutation in certain cases - Notwithstanding
anything contained in section 432, where a sentence of imprisonment for life is imposed
on conviction of a person for an offence for which death is one of the punishments
provided by laws or where a sentence of death imposed on a person has been commuted
under section 433 into one of imprisonment for life, such person shall not be released
from prison unless he had at least fourteen years of imprisonment." Section 434 gives
concurrent power to the Central Government in case of death sentence and Section 435
provides that in certain cases the State Government must act only after consultation with
the Central Government.
58. From the Prisons Act and the Rules it appears that for good conduct and for doing
certain duties etc. inside the jail the prisoners are given some days' remission on a
monthly, quarterly or annual basis. The days of remission so earned by a prisoner are
added to the period of his actual imprisonment (including the period undergone as an
under-trial) to make up the term of sentence awarded by the Court. This being the
position, the first question that arises in mind is how remission can be applied to
imprisonment for life. The way in which remission is allowed, it can only apply to a fixed
term and life imprisonment, being for the rest of life, is by nature indeterminate.
59. Mr. U. U. Lalit, learned counsel appearing for the Informant, suggested that for
applying remission to a sentence of imprisonment for life it would be necessary to first
commute the sentence to a fixed term, say for a term of 20 years and then to apply the
remissions' earned by the prisoner to the commuted period and that would work out to 14
years of actual incarceration.
@page-SC3057
60. To throw light on the question Mr. Hegde submitted a note on remission of sentences
of imprisonment as followed in the State of Karnataka, with specific reference to the facts
of this case. The note also encloses the relevant extracts from the Karnataka Prison Rules,
1974 and the Karnataka Prison Manual, 1978. Chapter XII of the Karnataka Prison
Manual deals with the remission system; Rule 215 defines remission of sentence and
provides for three kinds of remissions, namely, ordinary remission, special remission and
remission by the State Government. But what is significant for our purpose is the
stipulation made in Rule 214(c) which reads as follows :
"The sentence of all prisoners sentenced to imprisonment for life or to more than 20
years' imprisonment in the aggregate to imprisonment for life and imprisonment for
exceeding in the aggregate 20 years, shall for the purpose of these Rules be deemed to be
sentence of imprisonment for 20 years.
(Emphasis added)
In the note submitted by the counsel it is explained that the cases of life convicts are first
considered for remission by an Advisory Board constituted under Rule 814. The
proposals for premature release of life convicts, convicted after 18 December, 1978 (the
date of introduction of Section 433-A in the Code) are placed before the Advisory Board,
as provided under Government Order No. HD 92 PRR 88, dated 17th July, 1989 on
completion of 13 years and 8 months of imprisonment including the under-trial period.
The recommendations of the Board go to the Inspector General of Prisons together with
all the records and are finally placed before the Government for considering the
premature release of the prisoners on completing 14 years of imprisonment. The State
Government considers the recommendations of the Advisory Board and gives directions
either for the forthwith release of the prisoner or that the prisoner would be released in
the ordinary course on the expiry of the sentence, less the period of remission earned. In
case of a life convict if no order of premature release is passed there can be no release by
the mere lapse of time since a life sentence is for the rest of life.
61. To the question whether any specific orders are passed by the Government to
commute the sentence of life imprisonment to imprisonment for 20 years or less, the
answer is given in the note, as follows :
"In addition to what is stated in para 3.1, it may be added that cases of life imprisonment
pass through the Advisory Board and their recommendations are examined by the Head
of the Department viz., Additional Director General of Police and Inspector General of
Prisons who later forwards them to the Government for passing final orders. That is how
the sentence of life imprisonment is commuted for a term of 20 years or less as per
provisions of Sections 54 and 55 of the IPC and Section 433-A, Cr. P. C."
It is further stated in the note as follows :
"Experience shows that in respect of life convicts an assumption can be made that the
total sentence is 20 years and if the convict earns all categories of remissions in the
normal course it may come to 6 years which is less than one-third of 20 years. This is also
in consonance with Order 214(C) of the Prisons Manual which for the purposes of the
rules deems a sentence of imprisonment for life to be a sentence of imprisonment for
twenty years."
[Emphasis added]
In the note, it is further stated that in the event the appellant's sentence is modified to life
imprisonment, his case for premature release would come up before the Advisory Board
in January 2009. The Board shall then make its recommendation in light of the
instructions contained in Chapter XLIV of the Karnataka Prisons Manual. The
recommendation of the Board will be examined by the Head of the Department and
thereafter the State Government will pass appropriate orders regarding commutation of
his sentence.
62

. We also got some enquiries made on the issue of premature release of a life convict in
the State of Bihar and came to learn that the process follows basically a similar pattern. In
Bihar too the order for early release of a convicted prisoner is passed by the State
Government in the Department of Law (Justice) on the basis of recommendations made
by the Bihar State Sentence Remission Board. But there also the significant thing is the
conversion of life imprisonment into imprisonment for a fixed term. In this regard the
Government Letter No. A/ PM-03/81-550, dated 21st January, 1984 was brought to our
notice. The letter begins by stating the Government decision that for grant of remission to
a life convict and for his release from prison, imprisonment for life will be deemed to be
imprisonment for a AIR 1985 SC 1050

@page-SC3058
term of 20 years. Then in paragraph 1 in the letter, in its original form it was stated that a
life convict would not be entitled to the benefit of set-off under Section 428 of the Code
of Criminal Procedure, 1973 for the period of incarceration as an under-trial. Paragraph 1
of the letter was, however, deleted by letter No. 3115, dated 23rd May, 1985 following
the decision of this Court in Bhagirath vs. Delhi Administration (supra). Paragraph 2 of
the letter as it originally stood stipulated that an accused who is given the punishment of
imprisonment for life in a capital offence or whose death sentence is commuted to life
imprisonment under Section 433 of the Code as well as an accused who was awarded life
sentence after 18 December, 1978 would be released from prison (a) only on completion
of 14 years of actual imprisonment; and (b) when the total period of their imprisonment
and the days of remission add up to 20 years. Paragraph 2 of this letter too was later
deleted by Government letter No. 2939, dated 29th June, 2007 that provided that the
decision to release a convict undergoing life imprisonment for a capital offence or whose
death sentence is commuted to life imprisonment would be taken by the State
Government or by the State Sentence Remission Board constituted by the Government.
63. It is thus to be seen that both in Karnataka and Bihar remission is granted to life
convicts by deemed conversion of life imprisonment into a fixed term of 20 years. The
deemed conversion of life imprisonment into one for fixed term by executive orders
issued by the State Governments apparently flies in the face of a long line of decisions by
this Court and we are afraid no provision of law was brought to our notice to sanction
such a course. It is thus to be seen that life convicts are granted remission and released
from prison on completing the fourteen years' term without any sound legal basis. One
can safely assume that the position would be no better in the other States. This Court can
also take judicial notice of the fact that remission is allowed to life convicts in the most
mechanical manner without any sociological or psychiatric appraisal of the convict and
without any proper assessment as to the effect of the early release of a particular convict
on the society. The grant of remission is the rule and remission is denied, one may say, in
the rarest of the rare cases.
64

. Here, it may be noted that this has been the position for a very long time. As far back as
in 1973, in Jagmohan Singh (supra) a Constitution Bench of this Court made the
following observation : AIR 1973 SC 947
(Para 14)

"In the context of our criminal law which punishes murderer, one cannot ignore the fact
that life imprisonment works out in most cases to a dozen years of Imprisonment and it
may be seriously questioned whether that sole alternative will be an adequate substitute
for the death penalty."
(Emphasis added)

Five years after Jagmohan, Section 433-A was inserted in the Code of Criminal
Procedure, 1973 imposing a restriction on the power of remission or commutation in
certain cases. After the introduction of Section 433-A another Constitution Bench of this
Court in Bachan Singh (supra) made the following observation : AIR 1973 SC 947
AIR 1980 SC 898
(Para 156)

"It may be recalled that in Jagmohan this Court had observed that, in practice, life
imprisonment amounts to 12 years in prison. Now, Section 433-A restricts the power of
remission and commutation conferred on the appropriate Government under Sections 432
and 433, so that a person who is sentenced to imprisonment for life or whose death
sentence is commuted to imprisonment for life must serve actual imprisonment for a
minimum of 14 years."
Thus all that is changed by Section 433-A is that before its insertion an imprisonment for
life in most cases worked out to a dozen years of imprisonment and after its introduction
it works out to fourteen years' imprisonment. But the observation in Jagmohan that this
cannot be accepted as an adequate substitute for the death penalty still holds true.
65

. Earlier in this judgment it was noted that the decision in Shri Bhagwan (supra) there is a
useful discussion on the legality of remission in the case of life convicts. The judgment in
Shri Bhagwan, in paragraph 22, refers to and quotes from the earlier decision in State of
M.P. vs. Ratan Singh (supra) which in turn quotes a passage from the Constitution 2001
AIR SCW 2189
AIR 1976 SC 1552
(Para 4)
AIR 1961 SC 600

@page-SC3059
Bench decision in Gopal Vinayak Godse (supra). It will be profitable to reproduce here
the extract from Ratan Singh :

"4. As regards the first point, namely, that the prisoner could be released automatically on
the expiry of 20 years under the Punjab Jail Manual or the Rules framed under the
Prisons Act, the matter is no longer res integra and stands concluded by a decision of this
Court in Gopal Vinayak Godse v. State of Maharashtra, (1961) 3 SCR 440, where the
Court, following a decision of the Privy Counsel in Pandit Kishori Lal v. King Emperor,
AIR 1954 PC 64 observed as follows : AIR 1961 SC 600

"Under that section a person transported for life or any other terms before the enactment
of the said section would be treated as a person sentenced to rigorous imprisonment for
life or for the said term.
If so the next question is whether there is any provision of law whereunder a sentence for
life imprisonment, without any formal remission by appropriate Government, can be
automatically treated as one for a definite period. No such provision is found in the
Indian Penal Code, Code of Criminal Procedure or the Prisons Act.
*****
A sentence of transportation for life or imprisonment for life must prima facie be treated
as transportation or imprisonment for the whole of the remaining period of the convicted
person's natural life".
The Court further observed thus :
"But the Prisons Act does not confer on any authority a power to commute or remit
sentences; it provides only for the regulation of prisons and for the treatment of prisoners
confined therein. Section 59 of the Prisons Act confers a power on the State Government
to make Rules, inter alia, for rewards for good conduct. Therefore, the Rules made under
the Act should be construed within the scope of the ambit of the Act......Under the said
Rules the order of an appropriate Government under Section 401, Criminal Procedure
Code, are a pre-requisite for a release. No other rule has been brought to our notice which
confers an indefeasible right on a prisoner sentenced to transportation for life to an
unconditional release on the expiry of a particular term including remissions. The rules
under the Prisons Act do not substitute a lesser sentence for a sentence of transportation
for life.
The question of remission is exclusively within the province of the appropriate
Government; and in this case it is admitted that, though the appropriate Government
made certain remissions under Section 401 of the Code of Criminal Procedure, it did not
remit the entire sentence. We, therefore, hold that the petitioner has not yet acquired any
right to release'.
It is, therefore, manifest from the decision of this Court that the Rules framed under the
Prisons Act or under the Jail Manual do not affect the total period which the prisoner has
to suffer but merely amount to administrative instructions regarding the various
remissions to be given to the prisoner from time to time in accordance with the Rules.
This Court further pointed out that the question of remission of the entire sentence or a
part of it lies within the exclusive domain of the appropriate Government under Section
401 of the Code of Criminal Procedure and neither Section 57 of the Indian Penal Code
nor any Rules or local Acts can stultify the effect of the sentence of life imprisonment
given by the court under the Indian Penal Code. In other words, this Court has clearly
held that a sentence for life would ensure till the lifetime of the accused as it is not
possible to fix a particular period of the prisoner's death and remissions given under the
Rules could not be regarded as a substitute for a sentence of transportation for life."

Further, in paragraph 23, the judgment in Shri Bhagwan observed as follows : 2001
AIR SCW 2189

"In Maru Ram vs. Union of India, (1981) 1 SCC 107, a Constitution Bench of this Court
reiterated the aforesaid position and observed that the inevitable conclusion is that since
in Section 433-A we deal only with life sentences, remissions lead nowhere and cannot
entitle a prisoner to release. Further, in Laxman Naskar (Life Convict) vs. State of W.B.
and Anr., (2000) 7 SCC 626, after referring to the decision of the case of Gopal Vinayak
Godse vs. State of Maharashtra, (1961) 3 SCR 440, the court reiterated that sentence for
"imprisonment for life" ordinarily means imprisonment for the whole of the remaining
period of the convicted person's natural life; that a convict undergoing such sentence may
earn remissions of his part of sentence under the Prison AIR 1980 SC 2147
2000 AIR SCW 3060
AIR 1961 SC 600

@page-SC3060
Rules but such remissions in the absence of an order of an appropriate Government
remitting the entire balance of his sentence under this section does not entitled the convict
to be released automatically before the full life term if served. It was observed that
though under the relevant Rules a sentence for imprisonment for life is equated with the
definite period of 20 years, there is no indefeasible right of such prisoner to be
unconditionally released on the expiry of such particular term, including remissions and
that is only for the purpose of working out the remissions that the said sentence is equated
with definite period and not for any other purpose."
The legal position as enunciated in Pandit Kishori Lal, Gopal Vinayak Godse, Mau Ram,
Ratan Singh and Shri Bhagwan, and the unsound way in which remission is actually
allowed in cases of life imprisonment make out a very strong case to make a special
category for the very few cases where the death penalty might be substituted by the
punishment of imprisonment for life or imprisonment for a term in excess of fourteen
years and to put that category beyond the application of remission.
66. The matter may be looked at from a slightly different angle. The issue of sentencing
has two aspects. A sentence may be excessive and unduly harsh or it may be highly
disproportionately inadequate. When an appellant comes to this court carrying a death
sentence awarded by the trial court and confirmed by the High Court, this Court may
find, as in the present appeal, that the case just falls short of the rarest of the rare category
and may feel somewhat reluctant in endorsing the death sentence. But at the same time,
having regard to the nature of the crime, the Court may strongly feel that a sentence of
life imprisonment that subject to remission normally works out to a term of 14 years
would be grossly disproportionate and inadequate. What then the Court should do? If the
Court's option is limited only to two punishments, one a sentence of imprisonment, for all
intents and purposes, of not more than 14 years and the other death, the court may feel
tempted and find itself nudged into endorsing the death penalty. Such a course would
indeed be disastrous. A far more just, reasonable and proper course would be to expand
the options and to take over what, as a matter of fact, lawfully belongs to the court, i.e.,
the vast hiatus between 14 years' imprisonment and death. It needs to be emphasized that
the Court would take recourse to the expanded option primarily because in the facts of
the case, the sentence of 14 years' imprisonment would amount to no punishment at all.
67

. Further, the formalisation of a special category of sentence, though for an extremely few
number of cases, shall have the great advantage of having the death penalty on the statute
book but to actually use it as little as possible, really in the rarest of the rare cases. This
would only be a reassertion of the Constitution Bench decision in Bachan Singh (supra)
besides being in accord with the modern trends in penology. AIR 1980 SC 898

68. In light of the discussions made above we are clearly of the view that there is a good
and strong basis for the Court to substitute a death sentence by life imprisonment or by a
term in excess of fourteen years and further to direct that the convict must not be released
from the prison for the rest of his life or for the actual term as specified in the order, as
the case may be.
69. In conclusion we agree with the view taken by Sinha, J. We accordingly substitute the
death sentence given to the appellant by the trial court and confirmed by the High Court
by imprisonment for life and direct that he shall not be released from prison till the rest of
his life.
70. This appeal stands disposed of with the aforesaid directions and observations.
Order accordingly. .
AIR 2008 SUPREME COURT 3060 "North West Karnataka Road Transport Corpn. v. H.
H. Pujar"
(From : Karnataka)*
Coram : 2 Dr. A. PASAYAT AND P. SATHASIVAM, JJ.
Civil Appeal No. 4520 of 2008 (arising out of SLP (C.) No.5120 of 2007), D/- 18 -7
-2008.
North West Karnataka Road Transport Corpn. v. H.H. Pujar.
Industrial Disputes Act (14 of 1947), S.11A - INDUSTRIAL DISPUTE - LABOUR
COURT - Labour Court - Interference with punishment - Conductor found guilty of
carrying ticketless passengers
@page-SC3061
by Inquiry Officer - Punishment of dismissal imposed - Delinquent conceded to fairness
of domestic enquiry - Also admitted that he has not issued ticket to some passengers -
Interference with punishment on ground that ticket-less passengers were not examined -
Inappropriate.
W.P. No.17519 of 2000, D/-21-10-2005 and W.A. No.3830 of 2005, D/-21-06-2006
(Kant.), Reversed. (Paras 9, 10)
Cases Referred : Chronological Paras
2004 AIR SCW 5427 : AIR 2004 SC 4761 : 2004 Lab IC 3744 : 2004 AIR Kant HCR
3130 (Rel. on) 8, 10
AIR 1977 SC 1512 : 1977 Lab IC 845 (Foll) 7, 10
R.S. Hegde, Chandra Prakash, J.K. Nayyar, Ashwani Garg, Rahul Tyagi and P. P. Singh,
for Appellant;
* W. P. No. 17519 of 2000 and W. A. No. 3830 of 2005, D/- 21-10-2005 and 21-6-2005
(Kar.) respectively.
Judgement
Dr. ARIJIT PASATAT, J. :- Leave granted.
2. Challenge in this appeal is to the judgment of a Division Bench of the Karnataka High
Court in the writ appeal No. 3830/2005 dismissing appeal against the order of learned
single Judge in Writ Petition No. 17519/2000. The writ appeal was dismissed as not
maintainable and, therefore, the challenge in the present is essential to the order of
learned single Judge.
3. Background facts in a nutshell are as follows :
Respondent-Conductor was commissioning as such in Bus No. F-16 on 15-9-1993 when
the bus was intercepted by the checking staff. It was found that the respondent had not
issued tickets to 20 out of 136 passengers. Appellant conducted domestic enquiry which
found him guilty. Consequently, he was dismissed from service vide order dated 3-4-
1995. The same was challenged by the respondent before the Labour Court invoking
Section 10(4-A) of the Industrial Disputes Act, 1947 (in short the 'Act'). The Labour
Court held that the domestic inquiry was fair and proper on the basis of the memorandum
filed by the respondent conceding to the fairness of the domestic inquiry. However, the
Labour Court set aside the order of dismissal and directed reinstatement of respondent
with full back wages, continuity of service and other consequential benefits. The basis for
this order was non checking of cash bag of the respondent and non-examination of
ticketless passengers. The order was challenged before the High Court. By order dated
21-10-2005, the learned single Judge held that the order was correct so far as setting aside
dismissal order is concerned, direction for reinstatement and continuity of service and
consequential benefits. However, the direction relating to back wages was set aside. The
writ appeal as noted above, was dismissed on the ground that the same was not
maintainable.
4. In support of the appeal learned counsel for the appellant submitted that the primary
reason indicated by the Labour Court to hold that the order of dismissal was bad, was the
alleged non-examination of the passengers to whom the respondent had not issued the
tickets. It also did not find any substance in the stand of the Corporation that earlier also
on 12 occasions for similar charges punishments were awarded but the respondent did not
improve his conduct. The High Court found that the conclusions of the Labour Court
were correct. It was noted that if more passengers were carried within the permissible
limit, it was fault of the Corporation who did not take timely reformative and remedial
measures.
5. Learned counsel for the appellant submitted that the view expressed by the High Court
is clearly contrary to the law laid down by this Court. Further, when the respondent
himself conceded to the fairness of the proceedings and the fact that he had not issued
tickets to twenty passengers, their non-examination is of no consequence.
6. There is no appearance on behalf of the respondent in spite of service of notice.
7

. In State of Haryana and Anr. v. Rattan Singh (1977 (2) SCC 491), it was, inter alia, held
as follows : AIR 1977 SC 1512

"4. It is well settled that in a domestic enquiry the strict and sophisticated rules of
evidence under the Indian Evidence Act may not apply. All materials which are logically
provbative for a prudent mind are permissible. There is no allergy to hearsay evidence
provided it has reasonable nexus and credibility. It is true that departmental authorities
and Administrative Tribunals must be careful in evaluating such material and should not
glibly swallow what is strictly speaking not relevant under the Indian Evidence Act. For
this proposition it is not necessary to cite decisions nor text books,
@page-SC3062
although we have been taken through case-law and other authorities by counsel on both
sides. The essence of a judicial approach is objectivity, exclusion of extraneous materials
or considerations and observance of rules of natural justice. Of course, fairplay is the
basis and if perversity or arbitrariness, bias or surrender of independence of judgment
vitiate the conclusions reached, such finding, even though of a domestic tribunal, cannot
be held good. However, the Courts below misdirected themselves, perhaps, in insisting
that passengers who had come in and gone out should be chased and brought before the
tribunal before a valid finding could be recorded. The 'residuum' rule to which counsel
for the respondent referred, based upon certain passages from American Jurisprudence
does not go to that extent nor does the passage from Halsbury insist on such rigid
requirement. The simple point is, was there some evidence or was there no evidence - not
in the sense of the technical rules governing regular Court proceedings but in a fair
commonsense way as men of understanding and worldly wisdom will accept. Viewed in
this way, sufficiency of evidence in proof of the finding by a domestic tribunal is beyond
scrutiny. Absence of any evidence in support of a finding is certainly available for the
Court to look into because it amounts to an error of law apparent on the record. We find,
in this case, that the evidence of Chamanlal, Inspector of the Flying Squad, is some
evidence which has relevance to the charge levelled against the respondent. Therefore, we
are unable to hold that the order is invalid on that ground.
5. Reliance was placed, as earlier stated, on the non compliance with the departmental
instruction that statements of passengers should be recorded by inspectors. These are
instructions of prudence, not rules that bind or vitiate in the violation. In this case, the
Inspector tried to get the statements but the passengers declined, the psychology of the
latter in such circumstances being understandable, although may not be approved. We
cannnot hold that merely because statements of passengers were not recorded the order
that followed was invalid. Likewise, the re-evaluation of the evidence on the strength of
co-conductors testimony is a matter not for the Court but for the Administrative Tribunal.
In conclusion, we do not think the Courts below were right in overturning the finding of
the domestic tribunal."
8

. The view was reiterated in Divisional Controller KSRTC (NWKSRTC) v. A. T. Mane


(2004 (8) SCALE 308). 2004 AIR SCW 5427

9. As rightly contended by the appellant since fairness of the proceedings was conceded
and the respondent admitted that he had not issued tickets to 20 passengers, their non
examination is really of no consequence.
10. In view of what has been stated by this Court in Rattan Singh's case (supra) and in A.
T. Mane's case (supra) award of the Labour Court and impugned order of the High Court
cannot be maintained and are set aside. The order of dismissal passed by the Corporation
is to operate.
11. The appeal is allowed without any order as to costs.
Appeal allowed. .
AIR 2008 SUPREME COURT 3062 "Kuldeep Singh v. State of Himachal Pradesh"
(From : Himachal Pradesh)*
Coram : 2 Dr. A. PASAYAT AND HARJIT SINGH BEDI, JJ.
Criminal Appeal No. 1106 of 2008 (arising out of S. L. P. (Cri.) No. 1944 of 2008), D/-
16 -7 -2008.
Kuldeep Singh v. State of H.P.
(A) Penal Code (45 of 1860), S.304A - NEGLIGENCE - EVIDENCE - WITNESS -
WORDS AND PHRASES - Rash and negligent driving - Evidence of witnesses clearly
showing that vehicle was being drive at very high speed - More than 50 persons were
there in truck and appellant was driving same at very high speed - One of witnesses even
stating that track was being driven as if it was aeroplane - Conviction as recorded cannot
be faulted.
Words and Phrases - Words "negligence" and "recklessness" - Meaning of. (Para 17)
(B) Penal Code (45 of 1860), S.304A - NEGLIGENCE - Sentence - Rash and negligent
driving - Accused cannot be dealt with leniently.
2000 AIR SCW 1623, Foll. (Paras 18, 19)
@page-SC3063
Cases Referred : Chronological Paras
2007 AIR SCW 7104 (Ref) 16
2000 AIR SCW 1653 : AIR 2000 SC 1677 : 2000 Cri LJ 2283 (Foll Prt-B) 18
(1981) 1 All ER 961 : (1981) 2 WLR 509 (Ref) 14, 15
(1981) 1 All ER 974 : (1981) 2 WLR 524 (Ref) 15
AIR 1979 SC 1848 : 1979 Cri LJ 1374 (Ref) 12
(1978) 2 All ER 168 (Ref) 11
(1977) 1 All ER 475 (Ref) 13
(1937) 2 All ER 552 : 156 LT 464 (Ref) 12
Arun K. Sinha and Vikram Kapoor, for Appellant; Naresh K. Sharma, for Respondent.
* Cri. Appeal No. 128 of 2000, D/- 17-9-2007 and 18-9-2007 (H.P.)
Judgement
1. Dr. ARIJIT PASAYAT, J. :- Leave granted.
2. Appellant faced trial for offences punishable under Sections 279, 337, 338 and 304-A
of the Indian Penal Code, 1860 (in short the 'IPC') and Section 185 of the Motor Vehicles
Act. 1988 (in short the 'MV Act'). The appellant was acquitted by learned Judicial
Magistrate, Hamirpur. State of Himachal Pradesh preferred an appeal before the
Himachal Pradesh High Court which by the impugned judgment set aside the judgment
of acquittal passed by the trial court and directed conviction of the respondent for
offences punishable under Sections 279, 337, 338 and 304-A IPC and imposed various
sentences, which were directed to run concurrently. The maximum sentence imposed was
one year.
3. Background facts in a nutshell are as follows :
On 15-4-1993 the accused-appellant was the driver of the truck HIU-3837. The said truck
was carrying a marriage party. When it reached near village Kacherha, on the public way,
at about 11 a.m., he lost its control, consequently it went off the road, rolled down in the
field, leaving Pratap Singh @ Pinku, Kuldeep Singh @ Jogi, dead on the spot and Kamal
Kishore and Manohar Lal injured persons died in the hospital at Bhoranj later, due to
accidental injuries, whereas, out of about fifty other members of the marriage-party, who
were the occupants of the said truck, Harish Kumar, Amin Chand, Rajesh Kumar, Rattan
Chand, Desh Raj, Tej Ram, Rakesh Kumar, Hem Chand, Surjit Singh, Mahant Ram,
Prakash Chand, Pawan Kumar, Shambhu Rani, Paras Ram, Ranbir Singh, Prem Chand,
Prakash Chand and Sanjay, sustained simple injuries, whereas, Anil Kumar, Joginder,
Suresh Kumar, another Joginder Singh, Ishwar Dass, Nand Lal, Lekh Ram and Lekh Raj
sustained grievous injuries. The appellant was allegedly drunk. He and the cleaner of the
truck had absconded. The case was registered.
The police took the photographs of the spot, prepared the site plan, truck was
mechanically examined, the postmortem reports and the MLCs of the injured were taken
into possession and after recording the statements of the witnesses, the challan was
presented in the Court, for the trial against the respondent.
The charges were framed and put to the appellant, to which he pleaded not guilty and
claimed trial.
Nathu Ram (PW 1) is a constable, who was on his duty to execute the processes. He is an
eye witness of the said accident. Rup Lal (PW3) was an occupant and the father of
bridegroom Kuldeep Singh (deceased), whose marriage-party was traveling in the said
truck. PW2 Dile Ram (injured), PW4 Rattan Lal, PW5 Rattan Chand (injured) son of
Salig Ram, PW6 Surjit Singh, PW 13 Rakesh Kumar, PW8 Lekh Ram, PW 10 Rattan
Chand (injured), PW16 Rakesh Kumar (injured) and PW 18 Sagar Singh were occupants
of the truck, but they broadly did not support the case of the prosecution, as alleged.
However, he admitted the accident.
4. The trial court came to the conclusion that there was no rash or negligent driving as
claimed by the prosecution. In appeal, the High Court relied on the evidence of the
witnesses particularly PWs. 1, 3 and 4 and recorded that rash and negligent driving is
clearly established. Accordingly State's appeal was allowed and conviction was recorded
and sentences imposed.
5. Learned counsel for the appellant submitted that the accident took place not because of
negligence but because the appellant tried to save the lives of children who were playing
on the road and therefore, the truck climbed on the stones by the side of the road and its
rod was broken. It was also submitted that the appellant has already suffered custody of
more than nine months and, therefore, the sentence should be restricted to the period
already undergone.
6. Learned counsel for the State on the
@page-SC3064
other hand submitted that rash and negligent driving has resulted in the death of four
persons and several others were seriously injured.
7. Section 304-A IPC applies to cases where there is no intention to cause death and no
knowledge that the act done, in all probabilities, will cause death. This provision is
directed at offences outside the range of Sections 299 and 300 IPC. Section 304-A applies
only to such acts which are rash and negligent and are directly the cause of death of
another person. Negligence and rashness are essential elements under Section 304-A.
8. Section 304-A carves out a specific offence where death is caused by doing a rash or
negligent act and that act does not amount to culpable homicide under Section 299 or
murder under Section 300. If a person willfully drives a motor vehicle into the midst of a
crowd and thereby causes death to some person, it will not be a case of mere rash and
negligent driving and the act will amount to culpable homicide. Doing an act with the
intent to kill a person or knowledge that doing an act was likely to cause a person's death
is culpable homicide. When the intent or knowledge is the direct motivating force of the
act, Section 304-A has to make room for the graver and more serious charge of culpable
homicide. The provision of this section is not limited to rash or negligent driving. Any
rash or negligent act whereby death of any person is caused becomes punishable. Two
elements either of which or both of which may be proved to establish the guilt of an
accused are rashness/negligence, a person may cause death by a rash or negligent act
which may have nothing to do with driving at all. Negligence and rashness to be
punishable in terms of Section 304-A must be attributable to a state of mind wherein the
criminality arises because of no error in judgment but of a deliberation in the mind
risking the crime as well as the life of the person who may lose his life as a result of the
crime. Section 304-A discloses that criminality may be that apart from any mens rea,
there may be no motive or intention still a person may venture or practice such rashness
or negligence which may cause the death of other. The death so caused is not the
determining factor.
9. What constitutes negligence has been analysed in Halsbury's Laws of England (4th
Edition) Volume 34 paragraph 1 (para 3) as follows :
"Negligence is a specific tort and in any given circumstances is the failure to exercise that
care which the circumstances demand. What amounts to negligence depends on the facts
of each particular case. It may consist in omitting to do something which ought to be
done or in doing something which ought to be done either in a different manner or not at
all. Where there is no duty to exercise care, negligence in the popular sense has no legal
consequence, where there is a duty to exercise care, reasonable care must be taken to
avoid acts or omissions which can be reasonably foreseen to be likely to cause physical
injury to persons or property. The degree of care required in the particular case depends
on the surrounding circumstances, and may vary according to the amount of the risk to be
encountered and to the magnitude of the prospective injury. The duty of care is owed only
to those persons who are in the area of foreseeable danger, the fact that the act of the
defendant violated his duty of care to a third person does not enable the plaintiff who is
also injured by the same act to claim unless he is also within the area of foreseeable
danger. The same act or omission may accordingly in some circumstances involve
liability as being negligent although in other circumstances it will not do so. The material
considerations are the absence of care which is on the part of the defendant owed to the
plaintiff in the circumstances of the case and damage suffered by the plaintiff, together
with a demonstrable relation of cause and effect between the two".
10. In this context the following passage from Kenny's Outlines of Criminal Law, 19th
Edition (1966) at page 38 may be usefully noted :
"Yet a man may bring about an event without having adverted to it at all, he may not have
foreseen that his actions would have this consequence and it will come to him as a
surprise. The event may be harmless or harmful, if harmful, the question rises whether
there is legal liability for it. In tort, (at common law) this is decided by considering
whether or not a reasonable man in the same circumstances would have realised the
prospect of harm and would have stopped or changed his course so as to avoid it. If a
reasonable man would not, then there is no liability and the harm must lie where
@page-SC3065
it falls. But if the reasonable man would have avoided the harm then there is liability and
the perpetrator of the harm is said to be guilty of negligence. The word 'negligence'
denotes, and should be used only to denote, such blameworthy inadvertence, and the man
who through his negligence has brought harm upon another is under a legal obligation to
make reparation for it to the victim of the injury who may sue him in tort for damages.
But it should now be recognized that at common law there is no criminal liability for
harm thus caused by inadvertence. This has been laid down authoritatively for
manslaughter again and again. There are only two states of mind which constitute mens
rea and they are intention and recklessness. The difference between recklessness and
negligence is the difference between advertence and inadvertence they are opposed and it
is a logical fallacy to suggest that recklessness is a degree of negligence The common
habit of lawyers to qualify the word "negligence" with some moral epithet such as
wicked' 'gross' or 'culpable' has been most unfortunate since it has inevitably led to great
confusion of thought and of principle. It is equally misleading to speak of criminal
negligence since this is merely to use an expression in order to explain itself."
11. "Negligence", says the Restatement of the Law of Torts published by the American
Law Institute (1934) Vol. I. Section 28 "is conduct which falls below the standard
established for the protection of others against unreasonable risk of harm". It is stated in
Law of Torts by Fleming at page 124 (Australian Publication 1957) that this standard of
conduct is ordinarily measured by what the reasonable man of ordinary prudence would
do under the circumstances. In Director of Public Prosecutions v. Camplin (1978) 2 All
ER 168 it was observed by Lord Diplock that "the reasonable man" was comparatively
late arrival in the laws of provocation. As the law of negligence emerged in the first half
of the 19th century it became the anthropomorphic embodiment of the standard of care
required by law. In order to objectify the law's abstractions like "care" "reasonableness"
or "foreseeability" the man of ordinary prudence was invented as a model of the standard
of conduct to which all men are required to conform.
12

. In Syed Akbar v. State of Karnataka, (1980) 1 SCC 30, it was held that "where
negligence is an essential ingredient of the offence, the negligence to be established by
the prosecution must be culpable or gross and not the negligence merely based upon an
error of judgment. As pointed out by Lord Atkin in Andrews v. Director of Public
Prosecutions (1937) (2) All ER 552 simple lack of care such as will constitute civil
liability, is not enough; for liability under the criminal law a very high degree of
negligence is required to be proved. Probably, of all the epithets that can be applied
'reckless' most nearly covers the case." AIR 1979 SC 1848

13. According to the dictionary meaning 'reckless' means 'careless', 'regardless' or


heedless of the possible harmful consequences of one's acts'. It presupposes that if
thought was given to the matter by the doer before the act was done, it would have been
apparent to him that there was a real risk of its having the relevant harmful consequences;
but, granted this, recklessness covers a whole range of states of mind from failing to give
any thought at all to whether or not there is any risk of those harmful consequences, to
recognizing the existence of the risk and nevertheless deciding to ignore it. In R. v.
Briggs (1977) 1 All ER 475 it was observed that a man is reckless in the sense required
when he carries out a deliberate act knowing that there is some risk of damage resulting
from the act but nevertheless continues in the performance of that act.
14. In R. v. Caldwell (1981) 1 All ER 961, it was observed that :-
"Nevertheless, to decide whether someone has been 'reckless', whether harmful
consequences of a particular kind will result from his act, as distinguished from his
actually intending such harmful consequences to follow, does call for some consideration
of how the mind of the ordinary prudent individual would have reacted to a similar
situation. If there were nothing in the circumstances that ought to have drawn the
attention of an ordinary prudent individual to the possibility of that kind of harmful
consequence, the accused would not be described as 'reckless' in the natural meaning of
that word for failing to address his mind to the possibility; nor, if the risk of the harmful
consequences was so slight that the ordinary prudent individual on due consideration of
the risk would not he deterred from treating it as negligible, could the accused
@page-SC3066
be described as reckless in its ordinary sense, if, having considered the risk, he decided to
ignore it. (In this connection the gravity of the possible harmful consequences would be
an important factor. To endanger life must be one of the most grave). So, to this extent,
even if one ascribes to 'reckless' only the restricted meaning adopted by the Court of
Appeal in Stephenson and Briggs, of foreseeing that a particular kind of harm might
happen and yet going on to take the risk of it, it involves a test that would be described in
part as 'objective' in current legal jargon. Questions of criminal liability are seldom solved
by simply asking whether the test is subjective or objective."
15. The decision of R. v. Caldwell (supra) has been cited with approval in R. v. Lawrence
(1981) 1 All ER 974 and it was observed that :
"...... Recklessness on the part of the doer of an act does presuppose that there is
something in the circumstances that would have drawn the attention of an ordinary
prudent individual to the possibility that his act was capable of causing the kind of
serious harmful consequences that the section which creates the offence was intended to
prevent, and that the risk of those harmful consequences occurring was not so slight that
an ordinary prudent individual would feel justified in treating them as negligible. It is
only when this is so that the doer of the act is acting 'recklessly' if, before doing the act,
he either fails to give any thought to the possibility of there being any such risk or, having
recognized that there was such risk, he nevertheless goes on to do it".
16

. The above position was highlighted in Naresh Giri v. State of M.P. [2008(1) SCC 791].
2007 AIR SCW 7104

17. The evidence of PWs 1, 3 and 4 clearly show that the vehicle was being driven at a
very high speed. Evidence on record show that more than 50 persons were there in the
truck and the appellant was driving the same at a very high speed. One of the witnesses
has stated that the truck was being driven as if it was an aeroplane. Therefore, the
conviction as recorded cannot be faulted.
18

. Coming to the question of sentence, in Dalbir Singh v. State of Haryana (2000(5) SCC
82) it has been stated as follows : ......While considering the quantum of sentence to be
imposed for the offence of causing death by rash or negligent driving of automobiles, one
of the prime considerations should be deterrence. A professional driver pedals the
accelerator of the automobile almost throughout his working hours. He must constantly
inform himself that he cannot afford to have a single moment of laxity or inattentiveness
when his leg is on the pedal of a vehicle in locomotion. He cannot and should not take a
chance thinking that a rash driving need not necessarily cause any accident; or even if
any accident occurs it need not necessarily result in the death of any human being; or
even if such death ensues he might not be convicted of the offence; and lastly, that even if
he is convicted he would be dealt with leniently by the court. He must always keep in his
mind the fear psyche that if he is convicted of the offence for causing death of a human
being due to his callous driving of the vehicle he cannot escape from a jail sentence. This
is the role which the courts can play, particularly at the level of trial courts, for lessening
the high rate of motor accidents due to callous driving of automobiles." 2000 AIR
SCW 1653, Para 13

19. Above being the position, we find no merit in this appeal, which is accordingly
dismissed.
Appeal dismissed. .
AIR 2008 SUPREME COURT 3066 "DLF Universal Ltd. v. Mrs. Ekta Seth"
Coram : 2 S. B. SINHA AND Dr. MUKUNDAKAM SHARMA, JJ.
Civil Appeal No.4380 of 2008 (arising out of SLP (Civ.) No. 18834 of 2006), D/- 15 -7
-2008.
DLF Universal Ltd. v. Mrs. Ekta Seth and Anr.
Monopolies and Restrictive Trade Practices Act (54 of 1969), S.12 - MONOPOLIES
AND RESTRICTIVE TRADE PRACTICES - RESTRICTIVE TRADE PRACTICE -
AGREEMENT - SUPREME COURT - Restrictive trade practice - Agreement to
purchase apartment - Cancellation of allotment and forfeiture of earnest money for non-
payment of installments of additional amount - Demand of additional price and forfeiture
of earnest money was as per terms of contract - However considering fact that
installments were regularly paid by purchaser for 5 years - Subsequent discontinuance of
payment was due to heavy price escalation and non-delivery of apartment within time
fixed - Supreme Court in exercise of power under
@page-SC3067
Art.142 to do complete justice limited forfeiture of earnest money to 50%.
Constitution of India, Art.142. (Paras 12, 13)

Ravinder Narain, Ms. Meghalee Barthakur, Akhil P. Chhabra, Subrat Deb and Rajan
Narain, for Appellant; Amar Dave, E.C. Agrawala, Mahesh Agarwal, Rishi Agrawala,
Amit Kumar Sharma, Ashutosh Garg, Ms. Neha. Aggarwal and Gaurav Goel, for
Respondents.
Judgement
Dr. MUKUNDAKAM SHARMA, J. :-Leave granted.
2. This appeal is filed by the appellant challenging the legality of the judgment and order
dated 3rd July, 2006 passed by the Monopolies and Restrictive Trade Practices
Commission, New Delhi (for short "the Commission"). By the aforesaid judgment and
order the learned Commission made certain observations and recorded findings against
the appellant herein, which are under challenge in this appeal. The Commission recorded
a finding that the action of the appellant in increasing the cost which forced the
respondent from making further payments resulting in the cancellation referred to by the
appellant was unfair trade practice and the appellant had no right to forfeit the earnest
money. Consequently a direction was issued that the appellant should return the earnest
money with interest thereon @ 9% per annum from the date of withholding the earnest
money till the date of repayment in respect of the main flat as well as the parking space in
respect of the letters dated 26th/27th May, 1999.
3. We may briefly state the facts leading to the aforesaid direction of the Commission.
The respondent herein booked a flat along with parking space in the appellant's DLF
Regency Park, Gurgaon by entering into an Apartment Buyer's Agrement on 17th June,
1993. The sale price fixed by both the parties was at Rs. 16,37,448/- payable in 42
installments spread over a period of ten years commencing from 9th March, ending on
9th March, 2003. The respondent, herein admittedly paid all the installment due upto
September 1998 which came to a sum of Rs. 9,94,836/-. But thereafter he did not make
any payment of installment and therefor was considered to be a defaulter by the
appellant. In the meantime by letter dated 19th February, 1998 the appellant demanded an
additional amount of Rs. 4,21,474.06 from respondent on account of cost of escalation,
increase in area, external electrification, fire fighting system and stand by generators. The
said amount was to be paid in four equal bi-monthly installment of Rs. 1,05,368.52/-
commencing from 15th Mare, 1998. The respondent did not honour the said demand.
Consequently, the appellant cancelled the allotment of the flat vide its letter 26th May,
1999 and forfeited the earnest money and returned the balance amount due.
4. The respondent sent a legal notice dated 23rd May, 2001 to the appellant contending
that the cancellation of the allotment of the flat was illegal and arbitrary. The appellant
sent a reply to the legal notice on 25th June, 2001 stating that the deductions made were
lawful and in accordance with the terms of the agreement.
5. The respondent, however, was not satisfied with the reply sent by the appellant and
instead filed an application under Section 12-B of the Monopolies and Restrictive Trade
Practices Act, 1969 before the Commission for payment of compensation on the ground
of unfair trade practice. It was contended inter alia, by the respondent that the appellant
was not entitled to forfeit the earnest money as they themselves were unable to give
delivery of the flat within the stipulated time and more particularly, when the appellant
has re-sold flat at a good price, therefore, as the appellant did not incur any loss, they
could not and were not, entitled to forfeit the earnest money. The aforesaid submission of
the respondent found favour with the Commission and it recorded the aforesaid finding
and allowed the application filed by the respondent, consequent to which the impugned
directions were issued which are under challenge in this appeal.
6. The issue which was raised and urged before us, therefore, clearly revolves around the
power and jurisdiction of the appellant in forfeiting the earnest money which was to the
tune of Rs. 1,80,470/-. On going through the record we, however, found that out of Rs.
1,80,470/- an amount of Rs. 1,69,012/- was forfeited as earnest money, out of which an
amount of Rs. 1,62,412/-was for the flat and an amount of Rs. 6600/- was for the parking
allotted to the respondent. The balance amount i.e. Rs. 9,571/-was forfeited by the
appellant on account of interest on the delayed payment.
7. On behalf of the learned counsel for the appellant a specific contention was raised
before us that the appellant was
@page-SC3068
entitled to forfeit the earnest money in terms of the stipulations in the agreement arrived
at between the parties with mutual consent. Learned counsel appearing for the appellant
has drawn our attention to the various clauses of the said agreement which empowered
the appellant to deduct the aforesaid earnest money. In this connection, reference was
made to the provisions contained in clauses 8 and 9 of the said agreement which read as
follows :
"8. That the Company and the Apartment Allottee hereby agree that the amounts paid on
registration to the extent of 10% of the sale price of the said premises and on allotment or
in instalments as the case may be, will collectively constitute the earnest money. Non-
fulfillment by the Apartment Allottee of the terms and conditions of application for
allotment, terms and conditions of sale and those of this Agreement as also in the event of
failure to sign this Agreement by Apartment Allottee within the time allowed may entail
the forfeiture of the earnest money.
9. That the time of payment of installments as stated in schedule of payments (Annexure-
II) is the essence of this Agreement. It shall be incumbent on the Apartment Allottee to
comply with the terms of payment and other terms and conditions of sale, failing which
he shall forfeit to the Company the entire amount of earnest money and the Agreement of
sale shall stand cancelled and the Apartment Allottee shall have no right, title, interest or
claim of whatsoever nature on the said premises. The Company shall thereafter be free to
resell and deal with the said premises in any manner whatsoever, at its sole discretion.
The amount(s), if any, paid over and above the earnest money shall be however refunded
to the Apartment Allottee by the Company without any interest."
Relying on the said provision it was contended by the learned counsel for the appellant
that the action of the appellant in forfeiting the earnest money was legal and justified.
8. Counsel for the respondent, however, refuted the aforesaid position contending, inter
alia that the possession was proposed to be given to the respondent on or before June 16,
1996 i.e. within three years from the date of booking, but the said possession was not
given even till 1998, therefore, the appellant could and would not have resorted to the
power of forfeiture of the earnest money. It was submitted on behalf of the respondent
that a sum of Rs. 4,21,474.06/-demanded towards cost of escalation, increase in area,
external electrification, fire fighting system and stand by generators was exorbitant. It
was also submitted that it is unfair on the part of the appellant to demand such a huge
amount in such a short span of six months.
9. The aforesaid submission of the respondent was also advanced before the Commission
and the same found favour with the Commission. The learned Commission observed that
substantial portion of the escalation has been attributed towards creating additional
facilities and upgrading the flats, thus putting additional and unforeseen burden upon the
allottee and that to, to be fulfilled in short span of time. The Commission further held that
the contract was one sided and the respondent was required to sign on the doted lines.
While coming to the aforesaid conclusion the Commission has relied upon one of its
earlier order dated 2nd May, 2006 in Grahak Shayak Gurgon Voluntary Consumer
Association and Ors. v. DLF Universal Ltd. and Anr. wherein in respect to the same
complex for wich the respondent filled the application for allotment, the escalation made
by the appellant has been held to be unfair trade practice.
10. The parties to the contract are governed and bound by the terms and conditions of the
agreement entered into. In the case in hand though it cannot be denied that the
respondents at the time of signing the Apartment Buyer's Agreement was well aware of
the fact that additional amount could be demanded on account of factors enumerated in
clause 4, but what would be the maximum enhancement was not prescribed in the
agreement. It seems that by inserting the words "the decision of the Company in this
regard would be final and binding on the Apartment Allottee" in clause 4 of the
agreement the company has vested in itself unrestricted power to increase the cost.
11. Coming to the second aspect as per clause 16 of the agreement it was proposed that
the possession could be given within three years from the date of booking i.e. by
@page-SC3069
16th June, 1996 but the same was not done even till September 1998 and it is evident
from letter dated 22nd February, 1999 that there was still some time and further work to
be done by the appellant to enable it to hand over the possession. As per clause 18 the
only option given was that if there is delay in delivering the possession then the allottee
would be entitled for refund of entire amount deposited with the appellant but without
any interest. In other words as per the terms of the agreement no liability will accrue
upon the appellant due to delay in handing the possession.
12. In the present case we find that the installments were duly paid for at least five years
and payment was stopped thereafter on the ground that the increase in the cost of the flat
was beyond the means of the respondent and also the fact that appellant had failed to
deliver the possession of the flat in time. On the other hand as submitted there were bona
fide reasons on the part of the appellant for their inability to handover the said possession
within the stipulated time and the increase in cost was on account of factors specifically
enumerated in clause 2 (b) and clause 4.
13. Considering the entire facts and circumstances of the case, we are of the considered
opinion that the interest of justice would be subserved if we, in exercise of our
discretionary jurisdiction under Article 142 of the Constitution of India, direct that 50%
of the amount which was forfeited be refunded by the appellant to the respondent within
three months from the date of this judgment and the balance 50% would be considered as
forfeited in terms of the provisions of the agreement. However, if the appellant fails to
pay the said amount within the stipulated period the same will carry an interest @ 8% p.
a. which will be calculated from the date when the abovementioned period expires till the
date of payment.
14. We also make it clear that this order is passed in the peculiar facts and circumstances
of this case and would not be considered as precedence in any other matter.
15. The appeal stands disposed of in terms of the aforesaid directions. There will be no
order as to costs.
Order accordingly. .
AIR 2008 SUPREME COURT 3069 "Dumpala Chandra Reddy v. Nimakayala
Balireddy"
(From : Andhra Pradesh)*
Coram : 3 Dr. A. PASAYAT, P. SATHASIVAM AND Dr. MUKUNDAKAM
SHARMA, JJ.
Criminal Appeal Nos.309 with 310 of 2001, D/- 14 -7 -2008.
Dumpala Chandra Reddy v. Nimakayala Balireddy and Ors.
Penal Code (45 of 1860), S.300, S.326, S.149 - MURDER - GRIEVOUS HURT -
UNLAWFUL ASSEMBLY - CRIMINAL PROCEEDINGS - Murder trial - Framing
charge - Non mention of S.149 - However, charge in relation to offence punishable
u/S.149 was not only implicit but also patent in charges framed - No prejudice could have
been caused to accused because of non-mention of provision - Order of High Court that
in absence of charge u/S.302 read with S.149 IPC, accused could not have been convicted
u/S.302, but that each would be liable for conviction for offence punishable u/S.326 -
Liable to be set aside.
Criminal Appeal No.69 of 1999, D/-05-07-2000, (A. P.) Reversed.
Criminal P.C. (2 of 1974), S.464.
Where in a murder trial no charge was framed under S. 149 (S. 300 read with S. 149),
However, the charges as framed made it clear that Court specified that accused persons
were members of unlawful assembly and in prosecution of the common object of such
assembly, i.e., in order to commit murder of deceased, committed offence and at that time
they were armed with daggers etc. to bring in the application of Section 148 and in
Charge No. 3, there was a specific reference to the transactions, as mentioned in the first
charge, and the object to commit murder by hacking on the body of the deceased with
daggers and causing his intentional death and thereby committing offence punishable
under Section 302 IPC, the charge in relation to offence punishable under Section 149
IPC was thus, not only implicit but also patent in the charges and no prejudice could have
been caused to accused due to non-framing of charge under S. 300 read with S. 149.
Consequently, order of High Court that in the absence of charge under Section 302 read
with Section 149 IPC, they could not have been convicted under S. 302, but that each
would be liable for conviction for offence
@page-SC3070
punishable under Section 326 would be liable to be set aside. Further, if conviction under
S. 300 read with S. 149 could not have been made in the absence of charge, conviction in
terms of S. 326 also could not be made. (Paras 9, 10)
Cases Referred : Chronological Paras
2004 AIR SCW 2119 : AIR 2004 SC 1990 : 2004 Cri LJ 2025 : 2004 All LJ 1448 (Foll)
8
1998 AIR SCW 1775 : AIR 1998 SC 2883 : 1998 Cri LJ 2558 (Disting) 10
1997 AIR SCW 3935 : AIR 1997 SC 3997 : 1998 Cri LJ 54 (Foll) 7
AIR 1958 SC 672 : 1958 Cri LJ 1251 (Foll) 8
AIR 1957 SC 623 : 1957 Cri LJ 1009 8
AIR 1956 SC 116 : 1956 Cri LJ 291 (Foll) 7, 8
R. Sundervarden, Sr. Advocate, P.S. Narasimha, Shekhar G. Devasa, L. Roshmani, V.G.
Pragasam, Mrs. D. Bharathi Reddy, Ramlal Roy, R.N. Keswani, Ms. Sumeera Raheja and
Debji Borokakti, with him for the appearing parties.
*Cri. Appeal No.69 of 1999, D/- 5-7-2000 (A.P.).
Judgement
Dr. ARIJIT PASAYAT, J.:- Heard learned counsel for the parties.
2. These two appeals, one by the complainant Dumpala Chandra Reddy and other by the
State of A.P. question the correctness of judgment of a Division Bench of the Andhra
Pradesh High Court, which, while holding that the respondents were responsible for
causing the death of one Gangireddigari Kondareddy (hereinafter referred to as the
'deceased') convicted them in terms of Section 326 of the Indian Penal Code, 1860 (in
short 'the IPC') instead of Section 302 IPC as was done by the Trial Court.
3. The respondents, along with one Nimakayala Lakshmi Reddy (A-8) s/o Obul Reddy
faced trial for alleged commission of offences punishable under Sections 148, 302 IPC
and in respect of the deceased A-8, Section 114 IPC. The Trial Court found the accused
persons guilty of offences punishable under Section 148 IPC and also under Section 302
IPC. For the former offence, they were directed to undergo sentence of two years while
for the latter offence, life imprisonment was imposed.
4. Substance of the accusations, which led to the trial, is as follows.
All the accused are residents of Gopalapuram village. The deceased was a resident of
Khajipalli village. PWs 1 and 3 are also the residents of Khajipalli village. PWs 1 and 2
are the brother-in-laws of the deceased and PW3 is the brother of the deceased.
Originally, all the accused were residents of Khajipalli village. Due to factions with the
deceased's family, the deceased left the village and migrated to Gopalapuram village.
About 18 years ago, A-8's brother's son by name Pattabhi Reddy married one Dumpala
Munemma, resident of Khajipalli village. Munemma is related to the family of the
deceased. A-2 and A-4 are brothers of Pattabhi Reddy. Pattabhi Reddy deserted his wife
Munemma, resident of Khajipalli village. Due to that, there were differences between the
accused and the deceased and also there were criminal cases filed against each other.
Accused persons were residents of Khajipalli village. After filing of the criminal cases,
the accused left Khajipalli and settled down at Gopalapuram. About 8 years prior to the
murder of the deceased, A-2, A-4 and Pattabhi Reddy had stabbed PW3 and the deceased
in a lane by the side of Sangham Lodge at Cuddapah. One year thereafter Nimakayala
Gangireddy was murdered in Khajipalli village in the fields. In that regard, a criminal
case was filed against the deceased family and that criminal case ended in acquittal. After
that incident, the deceased's family alongwith his brothers settled down at Hyderabad and
was running a motor rewinding workshop.
On the date of the incident i.e. on 25-12-1995, the deceased went to his village from
Hyderabad in the morning hours. After sometime the deceased took PW-1 and went to
Patha Cuddapah to fix up the marriage date of his younger brother. The deceased and
PW-1 took Lingamappli bus at the village at about 8.30 a.m. and reached Cuddapah at
9.30 a.m. PW-1 and the deceased went to see one Raja Reddy, who happened to be the
proprietor of Sangham Medical Stores, and they found that Raja Reddy was not available
in the medical shop and then they went to Venkateswara Cloth Stores. By that time,
Venkateswara Reddy, a resident of Balisingapalli, was also present in the cloth shop. The
deceased telephoned Raja Reddy. After sometime Raja came to the cloth shop and they
were talking to each other at the shop. After sometime, the deceased,
@page-SC3071
PW1, Venkata Subba Reddy and Raja Reddy went to Manasa Hotel situated in Madras
road. At about 2.30 p.m. the deceased along with others went to the house of Rama Subba
Reddy to fix up the marriage date of the brother of the deceased. The marriage date was
fixed as 31-01-1996. Then all of them went to Venkateswara Cloth shop and again from
there, they went to the medical shop of Raja Reddy and stayed there for some time.
Thereafter the deceased, PW-1 and Venkata Subba Reddy went to Mithen hotel for
drinking tea. At that time, i.e. around 6.00 p.m., Venkata Subba Reddy left the hotel to go
to his village. All of them consumed tea. Thereafter, the deceased and PW-1 were going
to Machupalle bus stand through Madras road with a view to catch the bus going to their
village at about 6.30 p.m.
With a view to take the cycle of Chinna Narsimha Reddy who happened to be the
proprietor of Ayyappa Electricals, situated on the northern side of Madras road, they went
to the said electrical shop. When the deceased and PW-1 were climbing the steps of the
said electrical shop, A-8 who is no more instigated the other accused and all the accused
surrounded the deceased and took out daggers from their waists and stabbed the
deceased. Finally the deceased fell in front of Lepakshi Emporium, on the road margin.
When the deceased fell down, all the accused persons again stabbed the deceased. A-5
raised cries saying that "catch hold of that fellow". PW-1 was frightened and ran to the
northern lane of the road. After sometime, PW1 came to the scene of offence and saw the
deceased lying dead with bleeding injuries all over the body. Thereafter, PW1 went to
Cuddapah I Town Police Station for giving the report.
PW-6, the S.I. of Police, Cuddapah received an oral statement of the incident from PW1
at about 7.20 p.m. He reduced the oral statement into writing and took the signature of
PW1 on the statement. On the strength of the report given by PW1, PW6 registered the
case as Cr. No. 207 of 1995 against the accused for offences punishable under Sections
147, 148 and 302 read with Section 149 IPC. The copies of first information were sent to
all concerned. At about 9.30 p.m. PW-6 visited the scene of offence and recorded the
statements of PWs 1 to 3 and others. PW6 collected MO-1 dagger and MO-2 right leg
shoe from the scene of offence.
On 26-12-1995 PW7 the CI of Police, Cuddapah, held inquest over the dead body of the
deceased in the presence of PW-5 and others. Ex. P-3 is the inquest report.
On 26-12-1995, PW-4, the Civil Assistant Surgeon, District Headquarters Hospital
Cuddapah received a requisition from the Station House Officer, I Town Police Station,
Cuddapah to conduct autopsy over the dead body of the deceased. He commenced the
autopsy on the dead body of the deceased at about 12.40 p.m. and opined that the
deceased died because of multiple injuries and the injuries to vital organs. According to
the observation, he issued post mortem certificate Ex.P-2.
PW-7, the CI of Police, Cuddapah, arrested the accused at Padagalapalli bus stop on 04-
01-1996 at 10.30 a.m. Thus, on completion of investigation, he filed the charge sheet on
12-04-1996.
The defence of the accused was of total denial. It is also suggested by the accused by way
of defence that the incident did not take place in the manner suggested by the
prosecution.
In order to substantiate the accusation, the prosecution examined PWs 1 to 7. They
produced certain documents and they were marked as Exs. P-l to P-18, whereas the
accused examined DW-1 and 2 and they produced certain documents and they were
marked as Exs.D1 and D2.
P.Ws. 1 and 2 are stated to be eye-witnesses to the occurrence. As noted above, the
evidence of the eye-witnesses was found to be credible, cogent and reliable and on that
basis, conviction was recorded. The respondents filed an appeal before the High Court.
By the impugned judgment, the High Court found that the respondents were responsible
for the death of the deceased but held that in the absence of charge under Section 302
read with Section 149 IPC, they could not have been convicted under Section 302 IPC,
but held that each would be liable for conviction for the offence punishable under Section
326 IPC. Sentence of five years RI and a fine of Rs. 1,000/- with default stipulations was
imposed.
5. Learned counsel for the appellant in each case submitted that the approach of the High
Court is clearly erroneous. It is submitted that if the accused persons could not have been
convicted under Section 302
@page-SC3072
with the aid of Section 149 IPC, they could not have also been convicted for offence
punishable under Section 326 IPC simpliciter. It is pointed out that the charges framed
clearly related to the offence punishable under Section 149, though there was no specific
mention of the provision. It is also submitted that no prejudice has been caused to the
accused because of the non-mention of the provision. In fact, the essence of Section 149
IPC was clearly spelt out while framing charge.
6. Learned counsel for the accused-respondents supported the impugned judgment of the
High Court and additionally submitted that this is a case for acquittal and the evidence of
PWs. 1 and 2 should not have been acted upon.
7

. This Court, in the oft repeated case of Willie (William) Slaney Vs. State of Madhya
Pradesh (1955 (2) SCR 1140) had highlighted the aspect of prejudice. This decision has
been referred to in a large number of subsequent cases dealing with the question of
prejudice in the background of Section 464 of the Code of Criminal Procedure, 1973 (in
short 'the Code'). In Ramkishan and Ors. Vs. State of Rajasthan (1997 (7) SCC 518), it
was noted as follows : AIR 1956 SC 116
1997 AIR SCW 3935, Para 8

"In view of the findings recorded by the learned Sessions Judge and the material on
record, we are unable to ascribe to the finding that the appellants' intention was to cause
death of Bhura deceased. The finding betrays the observation of the trial court as noticed
above. The medical evidence also does not support the ultimate finding recorded by the
trial court and upheld by the High Court. The offence in the established facts and
circumstances of the case in the case of the appellants would only fall under Section 304
Part II IPC read with Section 149 IPC and not under Section 302 IPC. Indeed no specific
charge indicating the applicability of Section 149 IPC was framed, but all the ingredients
of Section 149 IPC were clearly indicated in the charge framed against the appellants and
as held by the Constitution Bench of this Court in Willie (William) Slaney Vs. State of
M.P. the omission to mention Section 149 IPC specifically in the charge is only an
irregularity and since no prejudice is shown to have been caused to the appellants by that
omission it cannot affect their conviction." AIR 1956 SC 116

. Similar view was also taken in B.N. Srikantiah and Ors. Vs. The State of Mysore (1959
SCR 496) in the background of Section 34, vis-a-vis Section 149, IPC. In Dalbir Singh
Vs. State of U.P. (2004 (5) SCC 334), it was noted as follows. AIR 1958 SC 672
2004 AIR SCW 2119
"15. In Willie (William) Slaney Vs. State of M.P. a Constitution Bench examined the
question of absence of charge in considerable detail. The observations made in paras 6
and 7, which are of general application, are being reproduced below : (AIR P 121 6)
AIR 1956 SC 116

"6. Before we proceed to set out our answer and examine the provisions of the Code, we
will pause to observe that the Code is a Code of procedure and, like all procedural laws,
is designed to further the ends of justice and not to frustrate them by the introduction of
endless technicalities. The object of the Code is to ensure that an accused person gets a
full and fair trial along certain well established and well-understood lines that accord with
our notions of natural justice.
If he does, if he is tried by a competent court, if he is told and clearly understands the
nature of the offence for which he is being tried, if the case against him is fully and fairly
explained to him and he is afforded a full and fair opportunity of defending himself, then,
provided there is 'substantial' compliance with the outward forms of the law, mere
mistakes in procedure, mere inconsequential errors and omissions in the trial are regarded
as venal by the Code and the trial is not vitiated unless the accused can show substantial
prejudice. That, broadly speaking, is the basic principle on which the Code is based.
7. Now, here, as in all procedural laws, certain things are regarded as vital. Disregard of a
provision of that nature is fatal to the trial and at once invalidates the conviction. Others
are not vital and whatever the irregularity they can be cured; and in that event the
conviction must stand unless the Court is satisfied that there was prejudice. Some of these
matters are dealt with by the Code and wherever that is the case full effect must be given
to its provisions.
15.1 After analysing the provisions of Sections 225, 232, 535 and 537 of the Code of
Criminal Procedure, 1898 which correspond to Sections 215, 464(2), 464 and 465 of the
@page-SC3073
1973 Code, the Court held as under in para 44 of the Report : (AIR p. 128)
"44.Now, as we have said, Sections 225, 232, 535 and 537(a) between them, cover every
conceivable type of error and irregularity referable to a charge that can possibly arise,
ranging from cases in which there is a conviction with no charge at all from start to finish
down to cases in which there is a charge but with errors, irregularities and Omissions in
it. The code is emphatic that 'whatever' the irregularity it is not to be regarded as fatal
unless there is prejudice.
It is the substance that we must seek. Courts have to administer justice and justice
includes the punishment of guilt just as much as the protection of innocence. Neither can
be done if the shadow is mistaken for the substance and the goal is lost in a labyrinth of
unsubstantial technicalities. Broad vision is required, a nice balancing of the rights of the
State and the protection of society in general against protection from harassment to the
individual and the risks of unjust conviction.
Every reasonable presumption must be made in favour of an accused person; he must be
given the benefit of every reasonable doubt. The same broad principles of justice and fair
play must be brought to bear when determining a matter of prejudice as in adjudging
guilt. But when all is said and done what we are concerned to see is whether the accused
had a fair trial, whether he knew what he was being tried for, whether the main facts
sought to be established against him were explained to him fairly and clearly and whether
he was given a full and fair chance to defend himself.
If all these elements are there and no prejudice is shown the conviction must stand
whatever the irregularities whether traceable to the charge or to a want of one."

16. This question was again examined by a three Judge Bench in Gurbachan Singh Vs.
State of Punjab in which it was held as under : (AIR p.626, para 7) AIR 1957 SC 623

"In judging a question of prejudice, as of guilt, courts must act with a broad vision and
look to the substance and not to technicalities, and their main concern should be to see
whether the accused had a fair trial, whether he knew what he was being tried for,
whether the main facts sought to be established against him were explained to him fairly
and clearly and whether he was given a full and fair chance to defend himself."
17. There are a catena of decisions of this Court on the same lines and it is not necessary
to burden this judgment by making reference to each one of them. Therefore, in view of
Section 464 Cr.P.C., it is possible for the appellate or revisional court to convict an
accused for an offence for which no charge was framed unless the Court is of the opinion
that a failure of justice would in fact occasion. In order to judge whether a failure of
justice has been occasioned, it will be relevant to examine whether the accused was
aware of the basic ingredients of the offence for which he is being convicted and whether
main facts sought to be established against him were explained to him clearly and
whether he got a fair chance to defend himself."
9. The High Court, as has been rightly pointed out by learned counsel for the appellant,
lost sight of the fact that if its view is accepted in the absence of charge under Section
149, conviction in terms of Section 326 could not have been done.
10

. The High Court appears to have misconstrued the decision of this Court in Rewa Ram
Vs. Teja and Ors. (AIR 1998 SC 2883). In that case, the High Court held that the accused
persons could be held guilty only under Section 326 IPC, particularly, when it was stated
in the charge that their common object was to assault the deceased and commit rioting
with deadly weapons. The position is entirely different here. In fact, while framing charge
and combined reading of charge No. 1 and charge No.3 makes it clear, that the Court
specified that the accused persons were members of unlawful assembly and in
prosecution of the common object of such assembly, i.e, in order to commit murder of the
deceased, committed the offence and at that time they were armed with daggers etc. to
bring in the application of Section 148 IPC. In charge No.3, there is a specific reference
to the transactions, as mentioned in the first charge, and the object to commit murder by
hacking on the body of the deceased with daggers and causing his intentional death and
thereby committing offence punishable under Section 302 IPC. Therefore, the charge in
relation to offence punishable under Section 149 IPC is not only implicit but also patent
in the charges. 1998 AIR SCW 1775

@page-SC3074
11. Apart from the question of prejudice, this aspect has also been lost sight of by the
High Court.
12. The inevitable result is that the appeals deserve to be allowed, and we direct so. The
judgment of the Trial Court stands restored and that of the High Court stands set aside.
The respondents shall surrender to custody forthwith to suffer remainder of sentence, if
any.
Appeal allowed. .
AIR 2008 SUPREME COURT 3074 "Manish Jalan v. State of Karnataka"
(From : Karnataka)
Coram : 2 C. K. THAKKER AND D. K. JAIN, JJ.
Criminal Appeal No. 1066 of 2008 (arising out of SLP (Cri.) No. 1080 of 2007), D/- 11
-7 -2008.
Manish Jalan v. State of Karnataka.
(A) Criminal P.C. (2 of 1974), S.320(9) - Penal Code (45 of 1860), S.279, S.304A -
COMPOUNDING OF OFFENCE - NEGLIGENCE - Compounding of offences - Only
such offences as are included in two tables, provided u/S.320 can be compounded -
Admittedly, offences punishable u/S.279 and u/S.304A, IPC do not figure in said tables -
They are, therefore, not compoundable. (Para 6)
(B) Penal Code (45 of 1860), S.279, S.304A - Criminal P.C. (2 of 1974), S.357 -
NEGLIGENCE - JUDGMENT - SENTENCE REDUCTION - Rash driving - Death by
negligence - Sentence - Tanker driven in rash and negligent manner, causing death of
scooterist - It was case of a rash and negligent act simpliciter - Not of driving in an
inebriated condition - Sentence of imprisonment therefore reduced to period already
undergone - Amount of Rs. 1,00,000/- awarded to mother of deceased by way of
compensation. (Paras 15, 16, 17)
(C) Criminal P.C. (2 of 1974), S.357 - JUDGMENT - Compensation - Quantum of -
Determination - Nature of crime, injury suffered and capacity of convict to pay
compensation etc. have to be taken into consideration.
AIR 1988 SC 2127; AIR 1978 SC 1525; 2007 AIR SCW 2425, Rel. on. (Para 13)
Cases Referred : Chronological Paras
2007 AIR SCW 2425 : 2007 Cri LJ 2417 : 2007 CLC 982 : 2007 (3) AIR Bom R 678
(Rel. on) 14
AIR 1988 SC 2127 : 1989 Cri LJ 116 (Rel. on) 12
AIR 1978 SC 1525 : 1978 Cri LJ 1598 (Rel. on) 13
Uday U. Lalit, Sr. Advocate, Anil Kumar Tandale, with him for Appellant; Ms. Anitha
Shenoy, for Respondent.
Judgement
D. K. JAIN, J. :- Leave granted.
2. The sole appellant stands convicted under Section 279 of the Indian Penal Code, 1860
(for short 'IPC') for the offence of driving on public way so rashly or negligently as to
endanger human life and also under Section 304A, IPC for causing death by rash or
negligent act, not amounting to culpable homicide. The Trial Court sentenced him to
undergo simple imprisonment for one year and to pay fine of Rs.5000/- for both the
offences and in default to undergo simple imprisonment for two months. On appeal to the
High Court, vide its judgment dated 10th November, 2006 in Criminal Revision Petition
No. 159 of 2005, the High Court of Karnataka at Bangalore has maintained the
conviction but has reduced the sentence to simple imprisonment for one year and a fine
of Rs.5000/- for the offence under Section 279, IPC and simple imprisonment for six
months and fine of Rs.5000/- for offence under Section 304A, IPC. This judgment of the
High Court is under challenge in this appeal by special leave.
3. Since learned senior counsel for the appellant has not seriously questioned the
correctness of the conviction and has confined his arguments to the quantum of sentence,
we deem it unnecessary to refer to the accusations against the appellant in greater detail.
It would suffice to note that the appellant was charge-sheeted for driving a tanker in a
rash and negligent manner so as to endanger human life and in the process dashing
against a Kinetic Honda scooter, being driven by the deceased, who fell down and was
run over by the left wheel of the tanker. The deceased succumbed to the injuries on way
to the hospital. As noted above, on appraisal of the evidence, both the courts below have
found the appellant guilty of the offence under Sections 279 and 304A, IPC.
4. Mr. U.U. Lalit, learned senior counsel appearing for the appellant submitted that
having regard to the fact that the mother of the victim has filed an affidavit, inter alia,
stating that she does not have any
@page-SC3075
grievance against the appellant as she believes that it was an act of God and it was their
destiny that their son left them at an early age, the sentence of imprisonment awarded to
the appellant may be set aside. Learned senior counsel also pleaded that the appellant was
prepared to pay reasonable amount of compensation, which may be determined by this
Court to the mother of the victim. In support, learned senior counsel drew our attention to
the affidavit filed by the mother of the deceased, on issuance of notice to her. Para 6 of
the affidavit, on which emphasis was laid by the learned counsel, reads thus :
"I state that being the mother and class I heir of the victim, late Shri Vasant Prabhu, I am
competent and willing to compound the offence against Shri Manish Jalan. I state that I
have no objection whatsoever if this Hon'ble Court wishes to set aside the conviction and
sentence against Shri Manish Jalan. For this purpose, I am ready and willing to receive
such additional compensation which this Hon'ble Court may feel appropriate, just and
reasonable."
5. Having carefully glanced through the evidence on record and the reasoning of the
courts below, we do not find any ground to interfere with the conviction of the appellant
under the afore-mentioned provisions. Hence, we reject the challenge of the appellant
made in this appeal to his conviction.
6. On the question of compounding of the offences, as prayed for in the affidavit, Section
320 of the Code of Criminal Procedure, 1973 (for short 'CrPC') dealing with
"compounding of offences", provides that only such offences as are included in the two
tables, provided thereunder can be compounded. Sub-section (9) of Section 320 CrPC
imposes a specific bar on compounding of other offences, not included in the two tables.
Admittedly, offences punishable under Sections 279 and 304A, IPC do not figure in the
said tables and are, therefore, not compoundable. Conscious of the legal position, learned
counsel did not press for compounding of the offences. Accordingly, we reject the prayer
for compounding.
7. The next question for consideration is whether facts of the case, particularly the
supervening circumstance brought on record by way of the affidavit of the mother of the
victim, warrant interference in the quantum of sentence awarded to the appellant?
8. As noted above, pursuant to the notice issued to the heirs of the deceased, the mother
of the deceased, namely, Smt. H. Sunanda Prabhu, has filed the afore-mentioned
affidavit. Vide order dated 30th November, 2007, the District Judge, Mangalore was
directed to make necessary inquiry through his own sources whether the said affidavit
had, in fact, been sworn by Smt. H. Sunanda Prabhu and ascertain the authenticity
thereof. In his report dated 8th January, 2008, the Principal District Judge, Mangalore,
has reported that the said affidavit has been sworn by Smt. H. Sunanda Prabhu before a
Notary on 9th July, 2007 and the same is authenticated.
9. The law which enables the Court to direct payment of compensation to the dependents
of the victim is found in Section 357 CrPC (1973), corresponding to Section 545 of the
1898 Code. The relevant portion of Section 357 reads as follows :
"357. Order to pay compensation . - (1) When a court imposes a sentence of fine or a
sentence (including a sentence of death) of which fine forms a part, the court may, when
passing judgment order the whole or any part of the fine recovered to be applied -
(a) In defraying the expenses properly incurred in the prosecution;
(b) In the payment to any person of compensation for any loss or injury caused by the
offence, when compensation is, in the opinion of the Court, recoverable by such person in
a Civil Court;
(c) When, any person is convicted of any offence for having caused the death of another
person or of having abetted the commission of such an offence, in paying in,
compensation to the persons who are, under the Fatal Accidents Act, 1855 (13 of 1855)
entitled to recover damages from the person sentenced for the loss resulting to them from
such death;
(d) ... ... ...
(2) ... ... ...
(3) When a court imposes a sentence, of which fine does not form a part, the court may,
when passing judgment order the accused person to pay, by way of compensation such
amount as may be specified in the order to the person who has suffered any loss or injury
by reason of the act for which the accused person has been so sentenced.
(4) An order under this section may also
@page-SC3076
be made by all Appellate Court or by the High Court or Court of Session when exercising
its powers of revision."
10. Sub-section (1) of Section 357 clothes the Court with the power to award
compensation to a victim of the offence out of the sentence of fine imposed on the
accused. Sub-section (3) of the Section contemplates that when a Court imposes a
sentence, of which fine does not form a part, the Court may, when passing judgment,
order the accused to pay by way of compensation, such amount, as may be specified in
the order, to the person who has suffered any loss or injury by reason of the act for which
the accused person has been so sentenced. In other words, sub-section (1) provides for
application of an amount of fine as compensation when it forms part of the sentence
whereas under sub-section (3) the Court can direct the convicted person to pay
compensation even in cases where fine does not form part of the sentence. The power
vested in the Appellate Court or the High Court or the Court of Session (in revision) to
award compensation under sub-section (3) of Section 357 CrPC is wide and is in addition
to any other sentence which may be awarded on conviction of a person. Needless to add
that it is no substitute for sentence on conviction.
11. Though a comprehensive provision enabling the Court to direct payment of
compensation has been in existence all through but the experience has shown that the
provision has rarely attracted the attention of the Courts. Time and again the Courts have
been reminded that the provision is aimed at serving the social purpose and should be
exercised liberally yet the results are not very heartening. On this aspect, Law
Commission in its 42nd Report at para 3.17, inter alia, observed :
"We have a fairly comprehensive provision for payment of compensation to the injured
party under Section 545 of the Criminal Procedure Code. It is regrettable that our courts
do not exercise their salutary powers under this Section as freely and liberally as could be
desired. The Section has, no doubt, its limitations. Its application depends, in the first
instance, on whether the Court considers a substantial fine proper punishment for the
offence. In the more serious cases, the Court may think that a heavy fine in addition to
imprisonment for a long term is not justifiable, especially when the public prosecutor
ignores the plight of the victim of the offence and does not press for compensation on his
behalf."
12

. In Hari Singh Vs. Sukhbir Singh and Ors.1, while emphasising the need for making
liberal use of the provisions contained in Section 357 CrPC, this Court has observed
thus : AIR 1988 SC 2127, Para 10

1 (1988) 4 SCC 551


"It may be noted that this power of Courts to award compensation is riot ancillary to
other sentences but it is in addition thereto. This power was intended to do something to
reassure the victim that he or she is not forgotten in the criminal justice system. It is a
measure of responding appropriately to crime as well of reconciling the victim with the
offender. It is, to some extent, a constructive approach to crimes. It is indeed a step
forward in our criminal justice system."
13

. However, in awarding compensation, it is necessary for the Court to decide if the case is
a fit one in which compensation deserves to be awarded. If the Court is convinced that
compensation should be paid, then quantum of compensation is to be determined by
taking into consideration the nature of the crime, the injury suffered and the capacity of
the convict to pay compensation etc. It goes without saying that the amount of
compensation has to be reasonable, which the person concerned is able to pay. If the
accused is not in a position to pay the compensation to the injured or his dependents to
which they are held to be entitled to, there could be no reason for the Court to direct such
compensation. (See : Sarwan Singh and Ors. Vs. State of Punjab2). AIR 1978 SC 1525

2 (1978) 4 SCC 111


14
. Very recently in Dilip S. Dahanukar Vs. Kotak Mahindra Co. Ltd. and Anr.3explaining
the scope and the purpose of imposition of fine and/or grant of compensation, this Court
observed as follows : 2007 AIR SCW 2425

3 (2007) 6 SCC 528


"The purpose of imposition of fine and/ or grant of compensation to a great extent must
be considered having the relevant factors therefor in mind. It may be compensating the
person in one way or the other. The amount of compensation sought to be imposed, thus,
must be reasonable and not arbitrary. Before issuing a direction to pay compensation, the
capacity of accused to
@page-SC3077
pay the same must be judged. A fortiori, an enquiry in this behalf even in a summary way
may be necessary. Some reasons, which may not be very elaborate, may also have to be
assigned; the purpose being that whereas the power to impose fine is limited and
direction to pay compensation can be made for one or the other, factors enumerated out
of the same; but sub- section (3) of Section 357 does not impose any such limitation and
thus, power thereunder should be exercised only in appropriate cases. Such a jurisdiction
cannot be exercised at the whims and caprice of a Judge."
15. True that in the instant case the appellant has been found to be guilty of offences
punishable under Sections 279 and 304A, IPC for driving rashly and negligently on a
public street and his act unfortunately resulted in the loss of a precious human life. But it
is pertinent to note that there was no allegation against the appellant that at the time of
accident, he was under the influence of liquor or any other substance impairing his
driving skills. It was a rash and negligent act simpliciter and not a case of driving in an
inebriated condition which is, undoubtedly despicable aggravated offence warranting
stricter and harsher punishment.
16. Having regard to all these facts and bearing in mind the fact that the mother of the
victim has no grievance against the appellant and has prayed for some can be taken in the
matter and the sentence of imprisonment can be reduced. We are of the opinion that the
ends of justice would be met if the sentence of imprisonment is reduced to the period
already undergone but in addition thereto, the appellant should be directed to pay an
amount of Rs. 1,00,000/-to the mother of the deceased by way of compensation. Learned
counsel for the appellant, in fact, indicated that his client was willing to pay that much
amount. We order accordingly.
17. Accordingly, the conviction of the appellant under Sections 279 and 304A, IPC is
maintained. However, the substantive sentence of imprisonment is reduced to the period
already undergone. Imposition of fine is also affirmed. Besides, the appellant shall pay an
amount of Rs.1,00,000/- to the mother of the victim, namely, Smt. H. Sunanda Prabhu, by
way of compensation within three months from today. If the appellant fails to pay the said
amount within the stipulated time, the same shall be recovered as per the procedure
prescribed under Section 431 CrPC and be paid to Smt. H. Sunanda Prabhu.
18. The appeal is partly allowed and the order of the High Court is modified to the extent
indicated above.
Appeal partly allowed. .
AIR 2008 SUPREME COURT 3077 "Pankaj Kumar v. State of Maharashtra"
(From : Bombay)
Coram : 2 C. K. THAKKER AND D. K. JAIN, JJ.
Criminal Appeal No. 1067 of 2008 (arising out of SLP (Cri.) No. 2843 of 2006), D/- 11
-7 -2008
Pankaj Kumar v. State of Maharashtra and Ors.
(A) Criminal P.C. (2 of 1974), S.482 - Constitution of India, Art.227 - INHERENT
POWERS - HIGH COURT - Quashing of proceedings - Powers under said provisions
though very wide, should be exercised to do real and substantial justice.
2008 AIR SCW 1640; 2007 AIR SCW 733 : 2001 AIR SCW 3818, Ref. to.
1993 AIR SCW 248; AIR 1977 SC 2279; AIR 1992 SC 604, Rel. on. (Para 10)
(B) Constitution of India, Art.21 - RIGHT TO LIFE - SPEEDY TRIAL -
INVESTIGATION - Speedy trial - Right to - Extends to all criminal prosecutions -
Applicable not only to actual proceedings in Court - But also to police investigations.
Criminal P.C. (2 of 1974), S.156. (Para 17)
(C) Criminal P.C. (2 of 1974), S.482 - Constitution of India, Art.21 - INHERENT
POWERS - RIGHT TO LIFE - INVESTIGATION - CRIMINAL PROCEEDINGS -
Quashing of proceedings - Delayed investigation - Appellant and his father charged with
misappropriation of huge amounts in purchase of spare parts - Charge-sheet was
submitted after unwarranted prolonged investigation - Delay of over eight years was not
in any way attributable to appellant - Lackadaisical manner of investigation spread over a
period of four years - Valuable constitutional right of appellant to a speedy investigation
and trial infringed - Criminal proceedings initiated against him are liable to be quashed.
AIR 1978 SC 597; AIR 1979 SC 1360; 1992 AIR SCW 1872; 1996 AIR SCW 2279;
1997 AIR SCW 290; 1998 AIR SCW 3208;
@page-SC3078

1999 AIR SCW 3522 and 2002 AIR SCW 1841 Rel. on.
2005 AIR SCW 3088; 1994 AIR SCW 1480, Ref. to.
Cri. W. P. No. 149 of 1999. D/- 2 and 4-5-2006 (Bom.), Reversed. (Paras 17, 18)
Cases Referred : Chronological Paras
2008 AIR SCW 1640 : AIR 2008 SC 1528 : 2008 Cri LJ 1927 : 2008 (3) AIR Kar R 139
(Ref. to) 9, 12
2007 AIR SCW 733 : AIR 2007 SC 971 : 2007 Cri LJ 1425 : 2007 (2) AIR Bom R 253
(Ref. to) 9
2005 AIR SCW 3088 : AIR 2005 SC 2731 : 2005 Cri LJ 3091 (Ref. to) 8, 9
2002 AIR SCW 1841 : AIR 2002 SC 1856 : 2002 Cri LJ 2547 : 2002 AIR- Kant HCR
1210 (Rel. on) 16
2001 AIR SCW 3818 : AIR 2001 SC 2856 : 2001 Cri LJ 4640 (Ref. to) 9
1999 AIR SCW 3522 : AIR 1999 SC 3524 : 1999 Cri LJ 4541 (Rel. on) 16
1998 AIR SCW 3208 : AIR 1998 SC 3281 : 1998 Cri LJ 4596 (Rel. on) 16
1997 AIR SCW 290 : AIR 1997 SC 1539 : 1997 Cri LJ 195 (Rel. on) 16
1996 AIR SCW 2279 : AIR 1996 SC 1619 : 1996 Cri LJ 2380 (Rel. on) 16
1994 AIR SCW 1480 : AIR 1994 SC 1229 : 1994 Cri LJ 1975 (Ref. to) 8
1993 AIR SCW 248 : AIR 1993 SC 892 : 1993 Cri LJ 600 (Rel. on) 10
1992 AIR SCW 1872 : AIR 1992 SC 1701 : 1992 Cri LJ 2717 (Rel. on) 15, 16
AIR 1992 SC 604 : 1992 Cri LJ 527 (Rel. on) 10, 11
AIR 1979 SC 1360 : 1979 Cri LJ 1036 (Rel. on) 14, 15
AIR 1978 SC 597 (Rel. on) 14
AIR 1977 SC 2229 : 1977 Cri LJ 1900 (Rel. on) 10
Ashok Godamgaonkar, K. Maruthi Rao, Ms. K. Radha and Praveen Agrawal, for
Appellant; Ravindra Keshavrao Adsure, for Respondents.
Judgement
D. K. JAIN, J. :- Leave granted.
2. This appeal arises from the final judgment and order dated 2nd/4th May, 2006 rendered
by the High Court of Judicature at Bombay, Bench at Aurangabad, in Criminal Writ
Petition No. 149 of 1999. By the impugned judgment, the learned Single Judge has
dismissed the petition preferred by the appellant and his mother under Article 227 of the
Constitution read with Section 482 of the Code of Criminal Procedure, 1973 (for short
'CrPC'), seeking quashing of the chargesheet and the consequential proceedings initiated
against them in Special Case No.3 of 1991 pending in the court of Special Judge, Latur.
3. A few material facts, necessary for disposal of this appeal can be stated thus :
On 12th May, 1998, a First Information Report was lodged against one Sayyad
Mohammad Sayyad Ibrahim and eight other persons, inter alia alleging that during the
period from 1st October, 1980 to 22nd February, 1982, while working as District Dairy
Development Officer, Government Milk Scheme, Bhanara, Sayyad Mohammad Sayyad
Ibrahim had conspired with the appellant and his father and had committed
misappropriation of huge amounts in the purchase of spare parts etc., for the plant. The
case was referred to the Anti Corruption Bureau for investigation.
4. Investigations dragged on for over three years and ultimately on 22nd February, 1991,
a chargesheet was filed in the court of Special Judge, Latur against twelve persons for
offences punishable under Sections 120B, 409, 420, 465, 468, 471, 477 (A), 101 and 34
of the Indian Penal Code, 1860 (for short 'IPC') and Sections 5(1)(c)(d) along with
Section 5(c) of the Prevention of Corruption Act, 1947. In addition thereto, Sections
13(1)(c)(d) read with Section 13 (2) of the Prevention of Corruption Act, 1988 have also
been invoked against accused Nos. 1 to 9. The first nine accused were the employees of
the Government Milk Plant and the remaining three being the appellant and his father and
mother, arraigned as accused Nos. 11, 10 and 12 respectively.
5. As per the chargesheet, the case of the prosecution, in brief is that the said Sayyad
Mohammad Sayyad Ibrahim (accused No. 1) and one Pashubhai Narsi Shah (accused No.
10), father of the appellant, were friends since 1976. Accused No. 10 had two concerns
styled as India Trading Agency, Mumbai and Dairy Equipment Industries, Mumbai, in the
name of his wife (accused No. 12). Accused No. 1, without calling for the quotations for
purchase of spare parts for the Milk Plant, got prepared from accused Nos. 10 and 11,
bills in small amounts of Rs.10,000/- each for purchase of spare parts valued at
Rs.2,03,705; got the bills processed from the staff members (accused Nos. 2 to 9) of the
said Milk Dairy Unit and
@page-SC3079
made payments in cash and by way of demand drafts to the present appellant. The second
accusation is that for two air compressors purchased from M/s. Ingersol Rand (India),
Mumbai in the year 1978, spare parts of the total value of Rs.91,469/- were again
purchased from the concerns of accused Nos. 10 and 11 despite the fact that quotation
had been received from the original supplier. No inspection and verification of the spare
parts supplied by the said concerns was carried out; bills were got processed by accused
No.l from other staff members and payment was again made to accused No. 11 in cash
and by demand drafts. The third accusation against all the accused is that an amount of
Rs.64,100/-was paid to one M/s. Pankaj Chemicals, Mumbai, managed by accused No.
10, the father of the appellant, for cleaning of the water softening plant supplied by M/s
Ingersol Rand (India) Ltd., without actually doing any such work. The Special Judge took
cognizance of the complaint and summoned all the accused.
6. Aggrieved, the appellant and his mother (A-12) filed the afore-stated writ petition.
During the pendency of the writ petition, the mother of the appellant expired. Accused
No. 10, namely, the father of the appellant had also expired earlier.
7. Rejecting the main plea of the appellant that being born on 18th September, 1963, the
appellant was a minor at the time of transactions in question in the year 1981 and,
therefore, he could not be proceeded against and that even otherwise the chargesheet did
not disclose any offence against the appellant and his mother, by the impugned order, the
High Court dismissed the petition. The High Court has come to the conclusion that the
appellant has failed to produce any document showing his date of birth and that the
chargesheet prima facie discloses commission of offences by the appellant. Aggrieved by
the said decision, the appellant has preferred this appeal.
8

. Learned counsel appearing on behalf of the appellant submitted that there was sufficient
material on record to show that when the alleged acts of malfeasance took place, the
appellant was a minor and had nothing to do with the affairs of the concerns, which had
made supplies to the milk plant. He was neither the proprietor nor a partner in the said
concerns/firms which were managed by his father, accused No. 10. In support of the
proposition that the reckoning date for determining the age of an accused, who claims to
be a child, is the date of occurrence and not the date when the offender is produced before
the Court, reliance was placed on the decision of the Constitution Bench in Pratap Singh
Vs. State of Jharkhand and Anr.1Referring us to certain portions of the chargesheet,
learned counsel contended that except for the bald averment that the appellant had
prepared bogus bills and had received the payment, no other incriminating material has
been brought on record, to show that the appellant was looking after the affairs of the
concerns/firms owned or managed by his father and mother, namely accused No. 10 and
12 (since deceased) and, therefore, the conclusion of the High Court that a prima facie
case had been made out against the appellant is without any basis. Lastly, it was pleaded
that the appellant has been deprived of his constitutional right to have a speedy
investigation and trial, inasmuch as the FIR was registered on 12th May, 1987 for the
offences allegedly committed sometime in the year 1981; chargesheet was filed on 22nd
February, 1991 but till date not a single witness has been examined by the prosecution. In
support, reliance was placed on a decision of this Court in Santosh De v. Archna Guha
and Ors.2, wherein a delay of eight years in commencing the trial was held to be violative
of the right of the accused to a speedy trial and the High Court's decision quashing the
criminal proceedings on that ground was affirmed. 2005 AIR SCW 3088
1994 AIR SCW 1480

1 (2005) 3 SCC 551


2 AIR 1994 SC 1229
9

. Learned counsel for the State, on the other hand, submitted that in the light of clear
averment in the chargesheet, implicating the appellant, the High Court was justified in
dismissing the writ petition by applying the correct principles to be kept in view while
exercising power under Article 227 of the Constitution or under Section 482 CrPC,
recently reiterated by a three-Judge Bench of this Court in Som Mittal Vs. Government of
Karnataka3. Regarding delay in trial, learned counsel submitted that the prosecution
cannot be held responsible for delay at least from the year 1999, when the records had
2008 AIR SCW 1640
2005 AIR SCW 3088
2007 AIR SCW 733
2001 AIR SCW 3818

@page-SC3080
been summoned by the High Court. It was also submitted that even if the date of birth of
the appellant is taken as 18th September, 1963, being more than 16 years of age in
March, 1981, still he could not be treated as a juvenile under the 1986 Juvenile Justice
Act. Learned counsel also placed reliance on the decisions in Pratap Singh (supra) and
Jameel Vs. State of Maharashtra4. It was also urged that since offences, punishable under
the Prevention of Corruption Act, 1988 have been committed by the appellant, in view of
the observations of this Court in Satya Narayan Sharma Vs. State of Rajasthan5, this
Court should be loath to interfere in the matter.
3 (2008) 3 SCC 574
4 (2007) 11 SCC 420
5 (2001) 8 SCC 607
10

. The scope and ambit of powers of the High Court under Section 482, CrPC or Article
227 of the Constitution has been enunciated and reiterated by this Court in a series of
decisions and several circumstances under which the High Court can exercise jurisdiction
in quashing proceedings have been enumerated. Therefore, we consider it unnecessary to
burden the judgment by making reference to all the decisions on the point. It would
suffice to state that though the powers possessed by the High Courts under the said
provisions are very wide but these should be exercised in appropriate cases, ex debito
justitiae to do real and substantial justice for the administration of which alone the courts
exist. The inherent powers do not confer an arbitrary jurisdiction on the High Court to act
according to whim or caprice. The powers have to be exercised sparingly, with
circumspection and in the rarest of rare cases, where the court is convinced, on the basis
of material on record, that allowing the proceedings to continue would be an abuse of the
process of the court or that the ends of justice require that the proceedings ought to be
quashed. [See: Janata Dal Vs. H.S. Chowdhary and Ors.6, Kurukshetra University and
Anr. Vs. State of Haryana and Anr.7 and State of Haryana and Ors. Vs. Bhajan Lal and
Ors.8 1993 AIR SCW 248
AIR 1977 SC 2229
AIR 1992 SC 604
6 (1992) 4 SCC 305
7 (1977) 4 SCC 451
8 1992 Supp (1) SCC 335
11. Although in Bhajan Lal's case (supra), the court by way of illustration, formulated as
many as seven categories of cases, wherein the extraordinary power under the aforestated
provisions could be exercised by the High Court to prevent abuse of process of the court
yet it was clarified that it was not possible to lay down precise and inflexible guidelines
or any rigid formula or to give an exhaustive list of the circumstances in which such
power could be exercised.
12

. The purport of the expression "rarest of rare cases" has been explained very recently in
Som Mittal (supra). Speaking for the three-Judge Bench, Hon'ble the Chief Justice has
said thus : 2008 AIR SCW 1640

"When the words 'rarest of rare cases' are used after the words 'sparingly and with
circumspection' while describing the scope of Section 482, those words merely
emphasize and reiterate what is intended to be conveyed by the words 'sparingly and with
circumspection'. They mean that the power under Section 482 to quash proceedings
should not be used mechanically or routinely, but with care and caution, only when a
clear case for quashing is made out and failure to interfere would lead to a miscarriage of
justice. The expression "rarest of rare cases" is not used in the sense in which it is used
with reference to punishment for offences under Section 302 IPC, but to emphasize that
the power under Section 482 Cr.P.C. to quash the FIR or criminal proceedings should be
used sparingly and with circumspection."
13. Bearing in mind the above legal position, we are of the opinion that, for the reasons
stated hereafter, the ends of justice require that prosecution proceedings in the instant
case be quashed.
14

. Time and again this Court has emphasized the need for speedy investigations and trial as
both are mandated by the letter and spirit of the provisions of the CrPC. (In particular,
Sections 197, 173, 309, 437 (6) and 468 etc.) and the constitutional protection enshrined
in Article 21 of the Constitution. Inspired by the broad sweep and content of Article 21 as
interpreted by a seven-Judge Bench of this Court in Maneka Gandhi Vs. Union of India
and Anr.9, in Hussainara AIR 1978 SC 597
AIR 1979 SC 1360

@page-SC3081
Khatoon and Ors. Vs. Home Secretary, State of Bihar10, this Court had said that Article
21 confers a fundamental right on every person not to be deprived of his life or liberty
except according to procedure established by law; that such procedure is not some
semblance of a procedure but the procedure should be 'reasonable, fair and just'; and
therefrom flows, without doubt, the right to speedy trial. It was also observed that no
procedure which does not ensure a reasonably quick trial can be regarded as 'reasonable,
fair or just' and it would fall foul of Article 21. The Court clarified that speedy trial means
reasonably expeditious trial which is an integral and essential part of the fundamental
right to life and liberty enshrined in Article 21.
9 (1978) 1 SCC 248
10 (1980) 1 SCC 81
15

. The exposition of Article 21 in Hussainara Khatoon's case (supra) was exhaustively


considered afresh by the Constitution Bench in Abdul Rehman Antulay and Ors. Vs. R.S.
Nayak and Anr.11. Referring to a number of decisions of this Court and the American
precedents on the Sixth Amendment of their Constitution, making the right to a speedy
and public trial a constitutional guarantee, the Court formulated as many as eleven
propositions with a note of caution that these were not exhaustive and were meant only to
serve as guidelines. For the sake of brevity, we do not propose to reproduce all the said
propositions and it would suffice to note the gist thereof. These are: (i) fair, just and
reasonable procedure implicit in Article 21 of the Constitution creates a right in the
accused to be tried speedily; (ii) right to speedy trial flowing from Article 21
encompasses all the stages, namely the stage of investigation, inquiry, trial, appeal,
revision and retrial; (iii) in every case where the speedy trial is alleged to have been
infringed, the first question to be put and answered is - who is responsible for the delay?;
(iv) while determining whether undue delay has occurred (resulting in violation of right
to speedy trial) one must have regard to all the attendant circumstances, including nature
of offence, number of accused and witnesses, the work-load of the court concerned,
prevailing local conditions and so on - what is called, the systemic delays; (v) each and
every delay does not necessarily prejudice the accused. Some delays may indeed work to
his advantage. However, inordinately long delay may be taken as presumptive proof of
prejudice. In this context, the fact of incarceration of accused will also be a relevant fact.
The prosecution should not be allowed to become a persecution. But when does the
prosecution become persecution, again depends upon the facts of a given case; (vi)
ultimately, the court has to balance and weigh several relevant factors - 'balancing test' or
'balancing process' - and determine in each case whether the right to speedy trial has been
denied; (vii) Ordinarily speaking, where the court comes to the conclusion that right to
speedy trial of an accused has been infringed the charges or the conviction, as the case
may be, shall be quashed. But this is not the only course open and having regard to the
nature of offence and other circumstances when the court feels that quashing of
proceedings cannot be in the interest of justice, it is open to the court to make appropriate
orders, including fixing the period for completion of trial; (viii) it is neither advisable nor
feasible to prescribe any outer time-limit for conclusion of all criminal proceedings. In
every case of complaint of denial of right to Speedy trial, it is primarily for the
prosecution to justify and explain the delay. At the same time, it is the duty of the court to
weigh all the circumstances of a given case before pronouncing upon the complaint; (ix)
an objection based on denial of right to speedy trial and for relief on that account, should
first be addressed to the High Court. Even if the High Court entertains such a plea,
ordinarily it should not stay the proceedings, except in a case of grave and exceptional
nature. Such proceedings in High Court must, however, be disposed of on a priority basis.
AIR 1979 SC 1360
1992 AIR SCW 1872

11 (1992) 1 SCC 225


16

. Notwithstanding elaborate enunciation of Article 21 of the Constitution in Abdul


Rehman Antulay (supra), and rejection of the fervent plea of proponents of right to
speedy trial for laying down time-limits as bar beyond which a criminal trial shall not
proceed pronouncements of this Court in "Common Cause" A Registered Society Vs.
Union of India (UOI) and Ors.12, "Common Cause", A Registered Society Vs. Union of
India and Ors.13, Raj Deo Sharma Vs. 1992 AIR SCW 1872
1996 AIR SCW 2279
1997 AIR SCW 290
1998 AIR SCW 3208
1999 AIR SCW 3522
2002 AIR SCW 1841

@page-SC3082
State of Bihar14 and Raj Deo Sharma II Vs. State of Bihar15gave rise to some confusion
on the question whether an outer time limit for conclusion of criminal proceedings could
be prescribed whereafter the trial court would be obliged to terminate the proceedings
and necessarily acquit or discharge the accused. The confusion on the issue was set at rest
by a seven-Judge Bench of this court in P. Ramachandra Rao Vs. State of Karnataka16.
Speaking for the majority, R.C. Lahoti, J. (as his Lordship then was) while affirming that
the dictum in A.R. Antulay's case (supra) is correct and still holds the field and the
propositions emerging from Article 21 of the Constitution and expounding the right to
speedy trial laid down as guidelines in the said case adequately take care of right to
speedy trial, it was held that guidelines laid down in the A.R. Antulay's case (supra) are
not exhaustive but only illustrative. They are not intended to operate as hard and fast
rules or to be applied like a strait-jacket formula. Their applicability would depend on the
factsituation of each case as it is difficult to foresee all situations and no generalization
can be made. It has also been held that it is neither advisable, nor feasible, nor judicially
permissible to draw or prescribe an outer limit for conclusion of all criminal proceedings.
Nonetheless, the criminal courts should exercise their available powers such as those
under Sections 309, 311 and 258 of CrPC to effectuate the right to speedy trial. In
appropriate cases, jurisdiction of the High Court under Section 482 CrPC and Articles
226 and 227 of the Constitution can be invoked seeking appropriate relief or suitable
directions. The outer limits or power of limitation expounded in the aforenoted judgments
were held to be not in consonance with the legislative intent.
12 (1996) 4 SCC 33
13 (1996) 6 SCC 775
14 (1998) 7 SCC 507
15 (1999) 7 SCC 604
16 (2002) 4 SCC 578
17. It is, therefore, well settled that the right to speedy trial in all criminal persecutions is
an inalienable right under Article 21 of the Constitution. This right is applicable not only
to the actual proceedings in court but also includes within its sweep the preceding police
investigations as well. The right to speedy trial extends equally to all criminal
persecutions and is not confined to any particular category of cases. In every case, where
the right to speedy trial is alleged to have been infringed, the court has to perform the
balancing act upon taking into consideration all the attendant circumstances, enumerated
above, and determine in each case whether the right to speedy trial has been denied in a
given case. Where the court comes to the conclusion that the right to speedy trial of an
accused has been infringed, the charges or the conviction, as the case may be, may be
quashed unless the court feels that having regard to the nature of offence and other
relevant circumstances, quashing of proceedings may not be in the interest of justice. In
such a situation, it is open to the court to make an appropriate order as it may deem just
and equitable including fixation of time for conclusion of trial.
18. Tested on the touchstone of the broad principles, enumerated above, we are of the
opinion that in the instant case, appellant's constitutional right recognised under Article
21 of the Constitution stands violated. It is common ground that the First Information
Report was recorded on 12th May, 1987 for the offences allegedly committed in the year
1981, and after unwarranted prolonged investigations, involving aforestated three
financial irregularities; the charge-sheet was submitted in Court on 22nd February, 1991.
Nothing happened till April, 19 1999, when the appellant and his deceased mother filed
criminal writ petition seeking quashing of proceedings before the trial court. Though, it is
true that the plea with regard to inordinate delay in investigations and trial has been
raised before us for the first time but we feel that at this distant point of time, it would be
unfair to the appellant to remit the matter back to the High Court for examining the said
plea of the appellant. Apart from the fact that it would further protract the already delayed
trial, no fruitful purpose would be served as learned Counsel for the State very fairly
stated before us that he had no explanation to offer for the delay in investigations and the
reason why the trial did not commence for eight long years. Nothing, whatsoever, could
be pointed out, far from being established, to show that the delay was in any way
attributable to the appellant. Moreover, having regard to the nature of the accusations
against the appellant, briefly referred to above, who was a young boy of about eighteen
years of age in
@page-SC3083
the year 1981, when the acts of omission and commission were allegedly committed by
the concerns managed by his parents, who have since died, we feel that the extreme
mental stress and strain of prolonged investigation by the Anti Corruption Bureau and the
sword of damocles hanging perilously over his head for over fifteen years must have
wrecked his entire career. Be that as it may, the prosecution has failed to show any
exceptional circumstance, which could possibly be taken into consideration for
condoning the prolongation of investigation and the trial. The lackadaisical manner of
investigation spread over a period of four years in a case of this type and inordinate delay
of over eight years (excluding the period when the record of the trial court was in the
High Court), is manifestly clear. Thus, on facts in hand, we are convinced that the
appellant has been denied his valuable constitutional right to a speedy investigation and
trial and, therefore, criminal proceedings initiated against him in the year 1987 and
pending in the court of Special Judge, Latur, deserve to be quashed on this short ground
alone.
19. For the view we have taken, we deem it unnecessary to go into the merits of the
accusations against the appellant as also the question of his age, at the time of
commission of alleged offences.
20. Consequently, the appeal is allowed and the proceedings against the appellant in
criminal case arising out of FIR No.78 of 1987 are hereby quashed.
Appeal allowed. .
AIR 2008 SUPREME COURT 3083 "Madan v. State of Madhya Pradesh"
(From : Madhya Pradesh)*
Coram : 2 Dr. A. PASAYAT AND P. SATHASIVAM, JJ.
Criminal Appeal No. 1058 of 2008 (arising out of SLP (Cri.) No. 6777 of 2007), D/- 11
-7 -2008.
Madan and Ors. v. State of M.P.
(A) Penal Code (45 of 1860), S.96, S.99 - PRIVATE DEFENCE - Private defence - Right
of - Availability - Entire incident must be examined with care to find out whether right of
private defence is available to accused.
A plea of right of private defence cannot be based on surmises and speculation. While
considering whether the right of private defence is available to an accused, it is not
relevant whether he may have a chance to inflict severe and mortal injury on the
aggressor. In order to find whether the right of private defence is available to an accused,
the entire incident must be examined with care and viewed in its proper setting. (Para
6)
(B) Penal Code (45 of 1860), S.99 - PRIVATE DEFENCE - Private defence - Right
extending to causing of death - Burden is on accused to show that, circumstances giving
rise to reasonable grounds for apprehending that either death or grievous hurt would be
caused to him, exist. (Para 6)
(C) Penal Code (45 of 1860), S.102, S.105 - PRIVATE DEFENCE - Private defence -
Right of - It lasts so long as reasonable apprehension of danger to body continues.
Sections 102 and 105, IPC deal with commencement and continuance of the right of
private defence of body and property respectively. The right commences, as soon as a
reasonable apprehension of danger to the body arises from an attempt, or threat, or
commit the offence, although the offence may not have been committed but not until that
there is that reasonable apprehension. The right lasts so long as the reasonable
apprehension of the danger to the body continues.
AIR 1963 SC 612; 2003 AIR SCW 469; 2003 AIR SCW 3984 and 2006 (9) SCC 678,
Relied on. (Para 6)
(D) Penal Code (45 of 1860), S.300, S.304, Part I, S.96, S.100 - MURDER -
CULPABLE HOMICIDE - PRIVATE DEFENCE - Murder or culpable homicide - Proof
- Accused persons allegedly assaulted deceased by lathi - Plea by appellants-accused that
they were exercising right of private defence - Evidence is to effect that appellants were
upto some stage exercising right to protect and defend their properties - But thereafter
they exceeded right - Appellants are therefore, liable to be convicted u/S.304, Part I
instead of S.302.
Cri. App. No. 1246 of 1997, D/-18-05-2007 (M. P.), Reversed. (Paras 9, 10)
Cases Referred : Chronological Paras
(2006) 9 SCC 678 (Rel. on) 8
@page-SC3084
2003 AIR SCW 469 : AIR 2003 SC 976 : 2003 Cri LJ 1226 (Rel. on) 8
2003 AIR SCW 3984 : AIR 2003 SC 3617 : 2003 Cri LJ 3876 (Rel. on) 8
AIR 1963 SC 612 : 1963 (1) Cri LJ 495 (Rel. on) 7
Navin Kumar Singh and Aruneshwar Gupta, for Appellants; Dr. N.M. Ghatate, Sr.
Advocate, C.D. Singh, Merusagar Samantaray and Sunny Chowdhary, for Respondent.
* Cri. Appeal No. 1246 of 1997, D/- 18-5-2007 (MP)
Judgement
Dr. ARIJIT PASAYAT, J. :- Leave granted.
2. Challenge in this appeal is to the judgment of a Division Bench of Madhya Pradesh
High Court, Indore Bench upholding the conviction of the appellants for offence
punishable under Section 302 read with Section 149 and Section 323 read with Section
149 of the Indian Penal Code, 1860 (in short the 'IPC'). Each of the appellants was
sentenced to undergo RI for life and to pay a fine of Rs. 1,000/- with default stipulation.
3. The prosecution case as unfolded during trial is as follows :
In the intervening night of 3rd and 4th July, 1991 at about 12.00 in village Khandakhedi
Kishanlal (hereinafter referred to as the 'deceased'), his wife Sampatbai and daughter
Premlatabai were sleeping inside their house. At that moment appellants and deceased
accused Jalu @ Jalamsingh and juvenile accused Jeevan reached at their house. They
broke open the wooden door, while abusing the inmates and reached in the courtyard.
They told deceased Kishanlal that they would not permit him to take his she-buffallows
from their field and asked as to why deceased made a complaint in Tehsil/Revenue Court.
They also threatened to eliminate him. While saying all these, appellants Madan and
Kamal caught hold both the hands of Kishanlal and threw him near the wall, thereafter
assaulted him by lathi. Sampatbai, wife of deceased Kishanlal (PW-2) cried for help. She
and her daughter Premlata (PW-1) tried to save deceased but both were assaulted by lathi.
Umraobai (PW-3) was assaulted by the deceased accused Jalu @ Jalamsingh when she
tried to rescue the deceased. Babulal (PW-7) after hearing the cry reached over there and
he was also assaulted by accused persons. When Ramsingh (PW-8) and Premsingh (PW-
9) arrived, appellants fled away. The deceased fell unconscious and died on the way to
police station. Premlata (PW-1), Sampatbai, Umraobai, Babulal, Premsingh alongwith
village Chowkidar Anarsingh reached at the police station at 4.00 a.m. and lodged the
report (Ex.P-1) which was recorded by SHO (PW-12) Nandlal. The injured persons were
sent for medical examination and treatment. Their medical reports are Ex.P-24 to P-28.
After preparation of inquest report (Ex.P-11) dead body of Kishanlal was sent to hospital
and postmortem was conducted by Dr. A.S. Rana (PW-13) who issued postmortem report
(Ex.P-29). Investigating Officer prepared spot map (Ex.P-2) and also effected seizure of
blood stained earth, controller earth, pieces of sticks vide Ex.P-3 from the spot. Through
seizure memo (Ex.P-4) pieces of bangles, pieces of glass of watch and roof tiles were
seized. Patvari Govindram (PW-6) prepared the spot map (Ex.P-10). After arrest, on
disclosure statement of the accused persons lathis were seized and seized articles were
sent with covering letter (Ex.P-23) to FSL, Sagar. Dr. Rana also gave report (Ex.P-30)
after examination of lathis seized from the accused persons. On completion of the
investigation charge sheet was filed before the learned JMFC, Sanwer against the
appellants and deceased accused Jalu @ Jalam and juvenile accused Jeevan was produced
and charge sheeted before the juvenile Court as directed by the trial Court because he was
found below 16 years of age. During the course of trial, accused Jalu @ Jalamsingh died,
therefore, case against him was closed.
The appellants denied the charges and pleaded innocence. They examined three witnesses
in defence whereas prosecution examined 15 witnesses and adduced 31 documents in
evidence. The trial Court found the appellants guilty, convicted them as aforenoted.
Before the High Court the stand taken was to the exercise of the right of private defence.
It was pointed out that the deceased and prosecution witnesses were aggressors. In any
event, when the appellants had assaulted, then in right of private defence they are entitled
to get the benefit of exception in terms of Sections 96 and 97 IPC. The High Court turned
down the stand and upheld the conviction.
4. In support of the appeal, learned counsel for the appellants submitted that most of the
injuries were on non vital parts. It has been established that injuries have been
@page-SC3085
sustained by the appellants in the same incident. The High Court had exercised the
appellate power under Section 386 (b) (ii) of the Code of Criminal Procedure, 1973 (in
short the 'Code') and had altered the finding of the trial Court in para 27 that the
appellants were injured in the same incident in which the deceased and injured witnesses
were assaulted and it was held that as per own saying by the defence the appellants
sustained injuries at the house of the appellant-Kamal. In essence, it was pointed out that
the trial Court and the High Court should have accepted the plea of exercise of right of
private defence.
5. Learned counsel for the respondent-State on the other hand submitted that there was
injury on the head though there was no fracture and the rest were on non vital parts of the
body. Nevertheless, even according to own saying of the accused appellants, there was no
question of exercise of right of private defence.
6. A plea of right of private defence cannot be based on surmises and speculation. While
considering whether the right of private defence is available to an accused, it is not
relevant whether he may have a chance to inflict severe and mortal injury on the
aggressor. In order to find whether the right of private defence is available to an accused,
the entire incident must be examined with care and viewed in its proper setting. Section
97 IPC deals with the subject-matter of right of private defence. The plea of right
comprises the body or property (i) of the person exercising the right; or (ii) of any other
person; and the right may be exercised in the case of any offence against the body, and in
the case of offences of theft, robbery, mischief or criminal trespass, and attempts at such
offences in relation to property. Section 99 IPC lays down the limits of the right of private
defence. Sections 96 and 98 IPC give a right of private defence against certain offences
and acts. The right given under Sections 96 to 98 and 100 to 106 IPC is controlled by
Section 99 IPC. To claim a right of private defence extending to voluntary causing of
death, the accused must show that there were circumstances giving rise to reasonable
grounds for apprehending that either death or grievous hurt would be caused to him. The
burden is on the accused to show that he had a right of private defence which extended to
causing of death. Sections 100 and 101, IPC define the limit and extent of right of private
defence.
7. Sections 102 and 105, IPC deal with commencement and continuance of the right of
private defence of body and property respectively. The right commences, as soon as a
reasonable apprehension of danger to the body arises from an attempt, or threat, or
commit the offence, although the offence may not have been committed but not until that
there is that reasonable apprehension. The right lasts so long as the reasonable
apprehension of the danger to the body continues. In Jai Dev v. State of Punjab (AIR
1963 SC 612), it was observed that as soon as the cause for reasonable apprehension
disappears and the threat has either been destroyed or has been put to route, there can be
no occasion to exercise the right of private defence.
8

. The above position was highlighted in Rizan and Another vs. State of Chhattisgarh,
through the Chief Secretary, Govt. of Chhattisgarh, Raipur, Chhattisgarh (2003 (2) SCC
661), and Sucha Singh and Anr. v. State of Punjab (2003 (7) SCC 643) and Raj Pal and
Ors. v. The State of Haryana (2006 (9) SCC 678). 2003 AIR SCW 469
2003 AIR SCW 3984

9. The High Court observed that according to the appellants incident occurred in two
different places in different phases and in the incident of assault to the deceased and the
witnesses they were not present and they sustained injuries caused by the deceased and
some of the injured witnesses at the house of Kamal. The High Court has in part accepted
the stand of the appellants that they were exercising the right of private defence, but at
the same time the evidence also shows that the appellants committed criminal trespass.
Therefore, they cannot claim the benefit of exception of having acted in exercise of right
of private defence.
10. On a combined reading of the judgments of the trial Court and the High Court it is
clear that the evidence is to the effect that the accused appellants were upto some stage
exercising the right to protect and defend their properties. But thereafter they exceeded
the right. Therefore, this appears to be a case where instead of convicting the appellants
under Section 302 IPC it would be proper to convict the appellants for offence punishable
under Section 304 Part I, IPC. Custodial sentence of 10 years would meet the ends of
justice.
@page-SC3086

11. The appeal is allowed to the aforesaid extent.


Appeal allowed. .
AIR 2008 SUPREME COURT 3086 "Subodh S. Salaskar v. Jayprakash M. Shah"
(From : Bombay)
Coram : 2 S. B. SINHA AND CYRIAC JOSEPH, JJ.
Criminal Appeal No. 1190 of 2009 (arising out of SLP (Cri.) No. 541 of 2008), D/- 1 -8
-2008.
Subodh S. Salaskar v. Jayprakash M. Shah and Anr.
(A) Negotiable Instruments Act (26 of 1881), S.142(b), Proviso (as inserted in 2002),
S.138 - DISHONOUR OF CHEQUE - COMPLAINT - GENERAL CLAUSES -
Dishonour of cheque - Belated complaint - Cognizance by condoning delay - S.142(b)
Proviso conferring such power - Is substantive provision - Cannot be given retrospective
effect.
Cri. W.P. No. 330 of 2007, D/-19-10-2007 (Bom.), Reversed.
General Clauses Act (10 of 1897), S.6.
The provisions of the Act being special in nature, in terms thereof the jurisdiction of the
Court to take cognizance of an offence under Section 138 was limited to the period of
thirty days. The Parliament only with a view to obviate the aforementioned difficulties on
the part of the complainant inserted proviso to Clause (b) of Section 142 in 2002. It
confers a jurisdiction upon the Court to condone the delay. It is, therefore, a substantive
provision and not a procedural one. The matter might have been different if the
Magistrate could have exercised its jurisdiction either under Section 5 of the Limitation
Act, 1963 or Section 473 of the Criminal P.C. The provisions of the said Acts are not
applicable. If the proviso appended to Clause (b) of Section 142 contained a substantive
provision and not a procedural one, it could not be given a retrospective effect. Order
taking cognisance of belated complaint by condoning delay liable to be set aside.
Cri. W. P. No. 330 of 2007, D/-19-10-2007 (Bom), Reversed. (Paras 24, 26)
(B) Penal Code (45 of 1860), S.415, S.420 - CHEATING - Cheating - Accused alleged to
have issued post dated cheques - Cheques on presentation on a much later date
dishonoured on ground that account was not operative - Even if accused had closed
account subsequently - Intention of accused to cheat complainant right from date of
issuance of cheque cannot be inferred - S.420 therefore does not get attracted. (Para
29)
Cases Referred : Chronological Paras
2008 AIR SCW 295 : AIR 2008 SC 899 : 2008 Cri LJ 1386 : 2008 (2) AIR Kar R 144
(Rel. on) 25
2008 AIR SCW 1956 : AIR 2008 SC 1683 (Rel. on, Pt. B) 28
2008 AIR SCW 4034 (Ref.) 14
2007 AIR SCW 656 : AIR 2007 SC 912 : 2007 Cri LJ 1419 : 2007 CLC 163 (Ref.)
14
2007 AIR SCW 3578 : 2007 Cri LJ 3214 : 2007 (4) AIR Kar R 462 : 2007 CLC 966
(Ref.) 22
2007 AIR SCW 6592 : AIR 2008 SC 210 : 2008 Cri LJ 431 (Ref.) 28
(2007) 4 SCC 70 (Ref.) 14
(2007) 9 SCC 650 (Rel. on.) 25
(2006) 9 SCC 340 (Ref.) 20
2004 AIR SCW 1485 : AIR 2004 SC 3084 : 2004 Cri LJ 1774 (Rel. on, Pt. B) 27
1999 AIR SCW 737 : AIR 1999 SC 1090 : 1999 Cri LJ 1822 (Ref.) 20
1999 AIR SCW 3809 : AIR 1999 SC 3762 : 1999 Cri LJ 4606 (Ref.) 22
AIR 1984 SC 87 (Ref.) 25
Manish Mohan, Mrs. Anita Mohan and Ugra Shankar Prasad, for Appellant; Santosh
Paul, M.J. Paul, Arvind Gupta, Ms. Aanchal Jain, Manish Pitale and Ravindra Keshavrao
Adsure, for Respondents.
Judgement
1. S. B. SINHA, J. :-Leave granted.
2. Whether the proviso appended to Section 142 of the Negotiable Instruments Act, 1881
(for short "the Act") inserted by the Negotiable Instruments (Amendment and
Miscellaneous Provisions) Act, 2002, is retrospective in operation is the question
involved in this appeal which arises out of a judgment and order dated 19.10.2007 passed
by the High Court of Judicature at Bombay in Criminal Writ Petition No. 330 of 2007.
3. The relationship between the parties hereto was that of a borrower and creditor. A
financial loan of Rs. 1,70,000/- was obtained by the appellant in 1996 from the
respondent No. 1, which according to him has been paid off. Two post dated cheques, one
bearing No. 460157 dated 6.12.1996 for a sum of Rs. 26,900/- and the other bearing No.
460158 dated 28.09.2000 for a sum of Rs. 1,70,000/-, however, were handed
@page-SC3087
over to him.
4. Appellant contends that the amount of loan was repaid in cash. Admittedly, the cheques
were presented before the bank on 10.01.2001. They were returned to the respondent No.
1 by the bank alleging that no such account, in the name of the appellant was in
operation. A legal notice dated 17.01.2001 was sent by speed post asking the appellant to
pay the said amount of Rs. 1,70,000/- failing which legal action including criminal action
would be taken against him.
5. A complaint petition alleging commission of an offence under Section 138 of the Act,
however, was filed only on 20.04.2001.
6. Indisputably, the complaint petition was sought to be amended for adding Section 420
of the Indian Penal Code in the complaint petition. The said application was allowed by
an order dated 14.08.2001.
7. Appellant filed an application for discharge on 16.12.2003 inter alia on the premise that
the said complaint petition was barred by limitation. It was dismissed by an order dated
14.11.2006. The revision application filed by the appellant before the learned Additional
Sessions Judge was also dismissed. A criminal writ petition filed by the appellant marked
as Criminal Writ Petition No. 330 of 2007 before the High Court of Bombay has been
dismissed by reason of the impugned judgment holding :
(i) The question as to whether the complaint is barred by limitation is a mixed question of
law and fact. Even otherwise as a result of amendment of Clause (b) of Section 142 of the
Act even if delay has been caused in filing the complaint, the Magistrate has power to
condone the delay;
(ii) Although the Magistrate could not have allowed amendment of the complaint petition
but as it discloses sufficient averments in regard to commission of an offence under
Section 420 of Indian Penal Code, the Trial Court was justified in issuing the process in
respect of the said provision also.
8. Mr. Manish Mohan, learned counsel appearing on behalf of the appellant would submit
that the High Court committed a serious error in passing the impugned judgment insofar
as it failed to take into consideration that :
(i) the complaint petition was barred by limitation, which would be evident from the
admitted facts;
(ii) the proviso appended to Clause (b) of Section 142 being substantive in nature cannot
be held to be retrospective in operation;
(iii) allegations made in the complaint petition even if given face value and taken to be
correct in their entirety, no case has been made out for taking cognizance under Section
420 of the Indian Penal Code; (iv) in any event, as the principal complaint being for
commission of an offence under Section 138 of the Act was not maintainable, the
application for amendment to insert Section 420 of the Indian Penal Code was also not
maintainable.
9. Mr. Santosh Paul, learned counsel appearing on behalf of the respondent No. 1,
submitted that from a perusal of the complaint petition it would appear that the date of
service of notice being not fixed and the complainant having asked the post office to
disclose the date of actual service of notice, it cannot be said that the legal notice was
served upon the accused immediately after issuance thereof.
In any event, as the complaint petition disclosed commission of an offence on the part of
the appellant under Section 420 of the Indian Penal Code, the High Court's judgment is
unassailable.
10. Section 138 of the Act provides a penal provision. The object of the Parliament in
brining the same in the statute book is well-known, viz., to create an atmosphere of faith
and reliance In the banking system.
11. The Act was amended in the year 2002 whereby additional powers have been
conferred upon the court to take cognizance even after expiry of the period of limitation
by conferring on it a discretion to waive the period of one month.
12. Before embarking on the questions raised, we may notice that the proviso appended
to Section 138 of the Act limits the applicability of the main provision stating :
"138. Dishonour of cheque for insufficiency, etc., of funds in the account
*** ***
*** Provided that nothing contained in this section shall apply unless -
(a) the cheque has been presented to the bank within a period of six months from the date
on which it is drawn or within the period of its validity, whichever is earlier;
(b) the payee or the holder in due course
@page-SC3088
of the cheque, as the case may be, makes a demand for the payment of the said amount of
money by giving a notice in writing, to the drawer of the cheque, within thirty days of the
receipt of information by him from the bank regarding the return of the cheque as unpaid;
and
(c) the drawer of such cheque fails to make the payment of the said amount of money to
the payee or, as the case may be, to the holder in due course of the cheque, within fifteen
days of the receipt of the said notice."
Section 142 of the Act also puts a limitation in the power of the court to take cognizance
of the offences, which reads as under :
"142 . Cognizance of offences
Notwithstanding anything contained in the Code of Criminal Procedure, 1973 ( 2 of
1974) -
(a) no court shall take cognizance of any offence punishable under section 138 except
upon a complaint, in writing, made by the payee or, as the case may be, the holder in due
course of the cheque;
(b) such complaint is made within one month of the date on which the cause-of-action
arises under clause (c) of the proviso to section 138 :
Provided that the cognizance of a complaint may be taken by the Court after the
prescribed period, if the complainant satisfies the Court that he had sufficient cause for
not making a complaint within such period.
(c) no court inferior to that of a Metropolitan Magistrate or a Judicial Magistrate of the
first class shall try any offence punishable under section 138."
13. As noticed hereinbefore, the proviso appended to Clause (b) of Section 142 of the Act
was inserted by the Negotiable Instruments (Amendment and Miscellaneous Provisions)
Act, 2002.
14. A complaint petition alleging commission of an offence under Section 138 of the Act
must demonstrate that the following ingredients exist, i.e. :
(a) a cheque was issued;
(b) the same was presented;
(c) but, it was dishonoured;
(d) a notice in terms of the said provision was served on the person sought to be made
liable; and
(e) despite service of notice, neither any payment was made nor other obligations, if any,
were complied with within fifteen days from the date of receipt of the notice.

[See S.M.S. Pharmaceuticals Ltd. v. Neeta Bhalla and Another (2007) 4 SCC 70, Saroj
Kumar Poddar v. State (NCT of Delhi) and Another (2007) 3 SCC 693 and DCM
Financial Services Ltd. v. J.N. Sareen and Another 2008 (8) SCALE 54] 2007 AIR
SCW 656
2008 AIR SCW 4034

15. Indisputably, therefore, unless the conditions precedent for taking cognizance of an
offence under Section 138 of the Act are satisfied, the court will have no jurisdiction to
pass an order in that behalf.
16. We will have to examine the contentions raised by the learned counsel for the parties
hereto keeping in view the aforementioned legal principles in mind. Before, however, we
advert thereto, we may place on record that the averments made in the complaint petition
in regard to service of notice are in the following terms:
"8. I say that the said Bank of the Accused, returned / dishonoured Cheque No. 460158
dated 28.09.2000 of Rs. 1,70,000/-drawn on Bank of India, Maheshwari Udyan Branch,
Mumbai, under Bank remark "NO SUCH ACCOUNT WITH US". The said remark was
given in handwriting by the Branch Manager of the Bank of India, Maheshwari Udyan
Branch, Mumbai in its Bank Memo dated 10.01.2001, though in the said Bank Memo at
Sr. No. 11, it is printed at 11(b) Account closed and at 11(c) no account. This Bank Memo
was received by me on 17.01.2001. Attached herewith is Xerox copy of the said Cheque
No. 460158 dated 28.09.2000 of Bank of India, 10.01.2001 and marked thereto as Exhibit
"A" thereto which are very clear and self-explanatory. I am also attaching herewith Xerox
copy of dishonoured Cheque No. 460157 dated 06.12.1996 of Rs. 26,900/-of the Accused
drawn on Bank of India, Maheshwari Udyan Branch, Mumbai and marked it as Exhibit
"B" thereto which speak much more about the Bank account No. 1365 of the Accused
lying with his said Bank.
9. I say that immediately, vide my letter Ref. No. JMS/SSS/CRIM/01/2001 dated
17.01.2001, I sent demand notice to the Accused through Speed Post Acknowledgment
due postal services. Attached herewith
@page-SC3089
is Xerox copy of the said Demand Notice along with copy of postal speed post A.D.
receipt No. 000271184 - SSPNL 650 dated 19.01.2001 and marked it as Exhibit "C"
Colly thereto which is very clear and self-explanatory. I say that I have not yet received
Speed Post Acknowledgement Slip with due acknowledgement thereon from the Accused
as to the receipt of the said Demand notice.
10. I say that with abundant and due precautions with a view to avoid technicalities,
through my advocate, Mr. Sunil Bagwe's letter Ref. No. SSB/JMS/BOI/01/2001 dated
05.03.2001 asked for detailed information as to the reasons given by the Branch
Manager, in his Bank memo dated 10.01.2001. The Branch Manager of the said Bank
Branch of the Accused, after various my approaches, finally given acknowledgement of
the receipt of the aforesaid letter of my advocate on 14.03.2001, attached herewith is
Xerox copy of the said letter and marked it as Exhibit "D" thereto which is very clear and
self-explanatory. The Branch Manager of Bank of India, Maheshwari Udyan Branch,
Mumbai vide his letter Ref. No. MU/ADV/MNI/39/853 dated 14.03.2001, given vague,
non-cooperative, unwilling, ill-wishes reply to my advocate's letter by courier services
on 26.03.2001. Attached herewith is Xerox copy of the said letter of the Bank of India
and marked it as Exhibit "E" thereto which is very clear and self-explanatory."
17. As regards purported commission of an offence under Section 420 of the Indian Penal
Code, on the part of the petitioner, it was alleged :
"16. I say that the aforesaid Cheque which was issued by the Accused in discharge of his
debts and liability to me in full, which were dishonoured by the Bank of the accused with
reason "No such account with us". I say that the Accused failed and neglected to make
payments as per my demand notice dated 17.01.2001. The Accused has failed and
neglected to make good attempts for payment of his dishonoured cheques on receipt of
my demand notice, within the stipulated period as provided under Section 138(c) of the
N.I. Act, 1988, therefore, the Accused has committed an offence punishable under section
138 read with section 141 and section 142 of the N.I. Act 1881 (as amended) and Section
420 of the I.P.C."
18. The cause of action of filing the said complaint was stated in the following terms :
"17. I say that the aforesaid cheque of the drawer, the Accused herein was returned by the
Complainant's banker i.e. the Deccan Merchant Co-op. Bank Ltd. Ghatkopar (E) Branch,
Mumbai 400 077, which is situated within the jurisdiction of this Hon'ble Court and,
therefore, this Hon'ble Court is competent to take cognizances of this present complaint
and try the same. The demand notice to the Accused was issued within the stipulated
period and the present complaint has been filed within the prescribed period as provided
under Section 142 (b) of the Negotiable Instruments Act, 1881 (as amended) and,
therefore, the Accused has committed an offence punishable under Section 138 read with
section 141 and section 142 of the N.I. Act 1881 (as amended) and Section 420 of the
I.P.C.
18. I say that the Accused has drawn Cheque of post dated in Mumbai with intention to
cheat me. Hence, the accused must have closed his Bank Account No. 1365 of Bank of
India, Maheshwari Udyan Branch, Mumbai subsequently and now, after the receipt of my
demand notice, the accused has refused to make the payment of his dishonoured cheques
as above in Mumbai. Hence, this Hon'ble Court has jurisdiction to entertain, try and
decide this present complaint. I say that the Accused has committed criminal offences
under the Negotiable Instruments Act, 1881 (as Amended) and section 420 of the I.P.C.,
within the jurisdiction to take cognizances of the same and try and decide the said
offences."
19. A complaint petition in view of Clause (b) of Section 142 of the Act was required to
be filed within one month from the date on which the cause of action arose in terms of
clause (c) of the proviso to Section 138 of the Act which stipulates that "the drawer of
such cheque fails to make the payment of the said amount of money to the payee or as the
case may be, to the holder in due course of the cheque within fifteen days of the receipt
of the said notice".
The legal notice admittedly was issued on 17th January, 2001. It was sent by speed post.
It was supposed to be served within a couple of days. A bare perusal of the statements
made in paragraph 10 of the complaint petition, as quoted hereinbefore, clearly
demonstrate that although the actual date of service of notice was allegedly not known,
the complainant proceeded on
@page-SC3090
the basis that the same was served within a reasonable period; otherwise in absence of
service of notice or deemed service thereof, the question of non-compliance of clause (c)
of the proviso appended to Section 138 of the Act would not arise and consequently the
complaint petition would not be maintainable.
20. In Jindal Steel and Power Ltd. and Another v. Ashoka Alloy Steel Ltd. and Others
[(2006) 9 SCC 340], this Court held :

"2. By the impugned order, the High Court has quashed the prosecution under Section
138 of the Negotiable Instruments Act, 1881 (for short "the Act") and Section 420 of the
Penal Code, on the sole ground that the complaint was filed two days after the expiry of
limitation. In the present case, notice was sent under Section 138 of the Act on 4-1-1997,
which was served on the accused on 10-1-1997, giving him 15 days' time for making
payment, which expired on 25-1-1997. Cause of action to file the complaint accrued on
26-1-1997, which day has to be excluded in computing the period of limitation, as
required under Section 12(1) of the Limitation Act, 1963. Therefore, the limitation would
be counted from 27-1-1997 and the complaint was filed on 26-2-1997, within a period of
one month from that date, as such, the same was filed well within time. We find that the
point is concluded by a judgment of this Court in Saketh India Ltd. v. India Securities
Ltd. in which case taking into consideration the provisions of Section 12(1) of the
Limitation Act, it was laid down that the day on which cause of action had accrued has to
be excluded for reckoning the period of limitation for filing a complaint under Section
138 of the Act. In the present case, after excluding the day when cause of action accrued,
the complaint was filed well within time; as such the High Court was not justified in
holding that there was two days' delay in filing the complaint. For the foregoing reasons,
we are of the view that the High Court was not justified in quashing prosecution of the
respondents." 1999 AIR SCW 737

21. In terms of the provisions of the General Clauses Act, a notice must be deemed to
have been served in the ordinary course subject to the fulfillment of the conditions laid
down therein. Section 27 of the General Clauses Act reads as under :
"27. Meaning of service by post. - Where any Central Act or Regulation made after the
commencement of this. Act authorises or requires any document to be served by post,
whether the expression 'serve' or either of the expression 'give' or 'send' or any other
expression is used, then, unless a different intention appears, the service shall be deemed
to be effected by properly addressing, pre-paying and posting by registered post, a letter
containing the document, and, unless the contrary is proved, to have been effected at the
time at which the letter would be delivered in the ordinary course of post."
Thirty days' time ordinarily must be held to be sufficient for service of notice. In fact
when the service of notice is sought to be effected by Speed Post, ordinarily the service
takes place within a few days. Even under Order V, Rule 9(5) of the Code of Civil
Procedure, 1908, summons is presumed to be served if it does not come back within
thirty days. In a situation of this nature, there was no occasion for the Court to hold that
service of notice could not be effected within a period of thirty days.
22

. Presumption of service, under the statute, would arise not only when it is sent by
registered post in terms of Section 27 of the General Clauses Act but such a presumption
may be raised also under Section 114 of the Evidence Act. Even when a notice is received
back with an endorsement that the party has refused to accept, still then a presumption
can be raised as regards the valid service of notice. Such a notice, as has been held by a
Three-Judge Bench of this Court in C.C. Alavi Haji v. Palapetty Muhammed and Another
[(2007) 6 SCC 555] should be construed liberally, stating : 2007 AIR SCW 3578

"17. It is also to be borne in mind that the requirement of giving of notice is a clear
departure from the rule of criminal law, where there is no stipulation of giving of a notice
before filing a complaint. Any drawer who claims that he did not receive the notice sent
by post, can, within 15 days of receipt of summons from the court in respect of the
complaint under Section 138 of the Act, make payment of the cheque amount and submit
to the court that he had made payment within 15 days of receipt of summons (by
receiving a copy of complaint with the summons) and, therefore, the complaint is liable
to be rejected. A person who does not pay within 15 days of receipt of the summons from
the court along with the copy of 1999 AIR SCW 3809

@page-SC3091
the complaint under Section 138 of the Act, cannot obviously contend that there was no
proper service of notice as required under Section 138, by ignoring statutory presumption
to the contrary under Section 27 of the GC Act and Section 114 of the Evidence. Act. In
our view, any-other interpretation of the proviso would defeat the very object of the
legislation. As observed in Bhaskaran case if the "giving ofnotice" in the context of
Clause(b) of the proviso was the same as the "receipt of notice" a trickster cheque drawer
would get the premium to avoid receiving the notice by adopting different strategies and
escape from legal consequences of Section 138 of the Act."
[Emphasis supplied]
23. The complaint petition admittedly was filed on 20.04.2001. The notice having been
sent on 17.01.2001, if the presumption of service of notice within a reasonable time is
raised, it should be deemed to have been served at best within a period of thirty days from
the date of issuance thereof, i.e., 16.02.2001. The accused was required to make payment
in terms of the said notice within fifteen days thereafter, i.e., on or about 2.03.2001. The
complaint petition, therefore, should have been filed by 2.04.2001.
24. Ex facie, it was barred by limitation. No application for condonation of delay was
filed. No application for condonation of delay was otherwise maintainable. The
provisions of the Act being special in nature, in terms thereof the jurisdiction of the court
to take cognizance of an offence under Section 138 of the Act was limited to the period of
thirty days in terms of the proviso appended thereto. The Parliament only with a view to
obviate the aforementioned difficulties on the part of the complainant inserted proviso to
Clause (b) of Section 142 of the Act in 2002. It confers a jurisdiction upon the court to
condone the delay. It is, therefore, a substantive provision and not a procedural one. The
matter might have been different if the Magistrate could have exercised its jurisdiction
either under Section 5 of the Limitation Act, 1963 or Section 473 of the Code of Criminal
Procedure, 1976. The provisions of the said Acts are not applicable. In any event, no such
application for condonation of delay was filed. If the proviso appended to Clause (b) of
Section 142 of the Act contained a substantive provision and not a procedural one, it
could not have been given a retrospective effect. A substantive law, as it is well-settled, in
absence of an express provision, cannot be given a retrospective effect or retroactive
operation.
25. In Madishetti Bala Ramul (Dead) By LRs. v. Land Acquisition Officer [(2007) 9 SCC
650], this Court held as under :
"18. It is not the case of the appellants that the total amount of compensation stands
reduced. If it had not been, we fail to understand as to how Section 25 will have any
application in the instant case. Furthermore, Section 25 being a substantive provision will
have no retrospective effect. The original award was passed on 8-2-1981 : Section 25, as
it stands now, may, therefore, not have any application in the instant case."
The question is now covered by a judgment of this Court in Anil Kumar Goel v. Kishan
Chand Kaura [2008 AIR SCW 295] holding :
"8. All laws that affect substantive rights generally operate prospectively and there is a
presumption against their retrospectivity if they affect vested rights and obligations,
unless the legislative intent is clear and compulsive. Such retrospective effect may be
given where there are express words giving retrospective effect or where the language
used necessarily implies that such retrospective operation is intended. Hence the question
whether a statutory provision has retrospective effect or not depends primarily on the
language in which it is couched. If the language is clear and unambiguous, effect will
have to be given to the provision is question in accordance with its tenor. If the language
is not clear then the court has to decide whether, in the light of the surrounding
circumstances, retrospective effect should be given to it or not. (See : Punjab Tin Supply
Co., Chandigarh etc. etc. v. Central Government and Ors., AIR 1984 SC 87).
9. There is nothing in the amendment made to Section 142(b) by the Act 55 of 2002 that
the same was intended to operate retrospectively. In fact that was not even the stand of
the respondent. Obviously, when the complaint was filed on 28.11.1998, the respondent
could not have foreseen that in future any amendment providing for extending the period
of limitation on sufficient cause being shown would be enacted."
26. Therefore, there cannot be any doubt I
@page-SC3092
whatsoever that the courts below committed a manifest error in applying the proviso to
the fact of the instant case. If the complaint petition was barred by limitation, the learned
Magistrate had no jurisdiction to take cognizance under Section 138 of the Act. The
direction to issue summons on the appellant, therefore, being illegal and without
jurisdiction was a nullity.
27

. Section 415 of the Indian Penal Code defines "cheating". The said provision requires: (i)
deception of any person, (ii) whereby fraudulently or dishonestly inducing that person to
deliver any property to any person or to consent that any person shall retain any property,
or (iii) intentionally inducing that person to do or omit to do anything which he would not
do or omit if he were not so deceived, and which act or omission causes or is likely to
cause damage or harm to that person in body, mind, reputation or property. Deception of
any person is common to the second and third requirements of the provision. [See
Devender Kumar Singla v. Baldev Krishan Singla (2005) 9 SCC 15] 2004 AIR
SCW 1485

28

. Noticing the ingredients of cheating, this Court in Suryalakshmi Cotton Mills Ltd. v.
Rajvir Industries Ltd. and Ors., [JT 2008 (1) SC 340], held : 2008 AIR SCW 1956

"A bare perusal of Section 415 read with Section 420 of the Indian Penal Code would
clearly lead to the conclusion that fraudulent or dishonest inducement on the part of the
accused must be at the inception and not at a subsequent stage.
22. For the said purpose, we may only notice that blank cheques were handed over to the
accused during the period 2000-2004 for use thereof for business purposes but the dispute
between the parties admittedly arose much thereafter i.e. in 2005.

In B. Suresh Yadav v. Sharifa Bee 2007 (12) SCALE 364, it was held; 2007 AIR
SCW 6592

13. For the purpose of establishing the offence of cheating, the complainant is required to
show that the accused had fraudulent or dishonest intention at the time of making promise
or representation. In a case of this nature, it is permissible in law to consider the stand
taken by a party in a pending civil litigation. We do not, however, mean to lay down a law
that the liability of a person cannot be both civil and criminal at the same time. But when
a stand has been taken in a complaint petition which is contrary to or inconsistent with
the stand taken by him in a civil suit, it assumes significance. Had the fact as purported to
have been represented before us that the appellant herein got the said two rooms
demolished and concealed the said fact at the time of execution of the deed of sale, the
matter might have been different. As the deed of sale was executed on 30.9.2005 and the
purported demolition took place on 29.9.2005, it was expected that the complainant/first
respondent would come put with her real grievance in the written statement filed by her
in the aforementioned suit. She, for reasons best known to her, did not choose to do so.
No case for proceeding against the respondent under Section 420 of the Indian Penal
Code is therefore, made out.
23. Filling up of the blanks in a cheque by itself would not amount to forgery. Whereas in
the complaint petition, allegations have been made that it was respondent Nos. 2 and 3
who had entered into a conspiracy to commit the said offence as indicated hereinbefore,
in the counter affidavit, it has been alleged that the employees of the Respondent
Company did so."
29. The cheques were post dated ones. Admittedly they were issued in the year 1996.
They were presented before the bank on a much later date. They were in fact presented
only on 10.01.2001. When the cheques were issued, the accounts were operative. Even
assuming that the account was closed subsequently the same would not mean that the
appellant had an intention to cheat when the post-dated cheques were issued. Even
otherwise the allegations made in the complaint petition, even if given face value and
taken to be correct in its entirety do not disclose commission of an offence under Section
420 of the Indian Penal Code. They do not satisfy the ingredients of the suit provision. It
is, therefore, in the fact situation obtaining in the instant case, difficult to hold that the
provisions of Section 420 of the Indian Penal Code were attracted.
30. The court had no jurisdiction to allow the amendment of the complaint petition at a
later stage. Therefore, the High Court was not correct in taking the aforementioned view
in the facts and circumstances of the present case.
31. For the reasons aforementioned, the impugned judgment cannot be sustained
@page-SC3093
which is set aside accordingly. The appeal is allowed.
Appeal allowed. .
AIR 2008 SUPREME COURT 3093 "Satish Sitole v. Ganga"
(From : 2004 (1) DMC 592 (MP)).
Coram : 2 ALTAMAS KABIR AND AFTAB ALAM, JJ.
Civil Appeal No.7567 of 2004, D/- 10 -7 -2008.
Satish Sitole v. Ganga.
Hindu Marriage Act (25 of 1955), S.13(1)(ia) and S.13(1)(ib) - DIVORCE - DECREE -
SUPREME COURT - Divorce - Ground of cruelty and desertion - Husband unable to
make out grounds - Facts however showing that parties lived separately for 14 years
making acrimonious allegations against each other - Attempts at reconciliation proved
futile - Continuance of such marriage would itself amount to cruelty - Decree of divorce
granted with adequate provision of alimony in exercise of powers under Art.142.
Constitution of India, Art.142.
1995 AIR SCW 647; (2002) 10 SCC 194; 2003 AIR SCW 5841 and 2005 AIR SCW
4045, Rel on. (Paras 7, 10, 11, 12)
Cases Referred : Chronological Paras
2005 AIR SCW 4045 : AIR 2005 SC 3297 (Rel. on) 9
2003 AIR SCW 5841 : AIR 2004 SC 161 (Rel. on) 9
(2002) 10 SCC 194 (Rel. on) 9
1995 AIR SCW 647 : AIR 1995 SC 851 (Rel. on) 1, 9
Niraj Sharma, Vikrant Singh, Ms. Sadhna Sharma, and Amit Singh, for Appellant; Ms.
Lily Isabel Thomas, Ashok Panigrahi, Shaju Francis and C. P. Sharma, for Respondents.
Judgement

ALTAMAS KABIR, J. :- As far back as on 13.1.1995 two Judges of this Court in the case
of Romesh Chander V. Savitri (1995) 2 SCC 7) had occasion to pose the question as to
whether a marriage which is otherwise dead emotionally and practically should be
continued for name sake. In the instant appeal, we are also faced with the same question.
1995 AIR SCW 647

2. Marriage between the appellant and the respondent was performed on 22.5.1992
according to Hindu rites and customs. On 21.8.1994 the respondent, for whatever reason,
left her matrimonial home and went back to her parents and the couple have been living
separately ever since. Soon thereafter, the parties took recourse to the law when on
30.12.1994 the appellant sent a notice to the respondent asking her to return to her
matrimonial home. On 20.10.1995 the respondent lodged a complaint against the
appellant and his family members under Section 498-A of the Indian Penal Code alleging
demand of dowry and it is only on 2.2.2003 that they were finally acquitted after a full
trial. The appellant also moved the Court of the Sub-Divisional Magistrate for issuance of
a search warrant consequent upon which the respondent appeared before the Magistrate's
Court and agreed to return to the appellant but she did not return as agreed.
3. Ultimately, on 28.9.1998 the appellant filed Matrimonial Case No.383/1998 before the
Ninth Additional District Judge, Indore, (MP), on grounds of cruelty and desertion under
Section 13(1)(1a)(1b) of the Hindu Marriage Act for dissolution of the marriage. Despite
holding that the respondent had proved his case on grounds of cruelty and desertion, the
trial court did not grant a decree for divorce, but thought it appropriate to pass a decree of
judicial separation instead. On appeal preferred by the respondent against the decree of
judicial separation passed by the trial court and the cross appeal filed by the appellant
seeking dissolution of marriage, the High Court reversed the judgment and decree of the
trial court upon holding that it was on account of the conduct of the appellant that the
respondent was compelled to leave her matrimonial home. The learned Single Judge of
the High Court also held that he was not satisfied that the appellant had been treated with
cruelty by the respondent-wife. On such finding the High Court dismissed the appeal
filed by the appellant and his prayer for dissolution of marriage and, on the other hand,
allowed the appeal filed by the respondent-wife and set aside the judgment and decree of
the trial court.
4. The respondent is in appeal against the said judgment of the High Court.
5. Having regard to the finding of the High Court that the respondent had not treated the
appellant with cruelty and was, on the other hand, compelled to leave the matrimonial
home on account of the conduct of the appellant, a different approach was
@page-SC3094
taken on behalf of the appellant at the time of hearing of the appeal. It was sought to be
urged that even if the appellant had been unable to prove his case of cruelty and desertion
as grounds for seeking dissolution of the marriage, having regard to the irretrievable
breakdown of the marriage, technicalities should not stand in the way of this Court
granting relief to the appellant in exercise of its power under Article 142 of the
Constitution. It was submitted that out of 16 years of marriage, the parties have lived
separately for 14 years, most of which has been spent in acrimonious allegations against
each other in the litigation embarked upon by both the parties. It was submitted that there
was no possibility of retrieval of the marriage and appropriate orders should be passed to
end the agony of both the parties.
6. Since, initially on behalf of the respondent-wife it was made to appear that she was
ready and willing to go back to the appellant, subject to certain terms and conditions, we
explored the possibility of an amicable solution, but such an attempt ended in failure on
account of the rigid stance taken on behalf of the respondent. On behalf of the wife it was
submitted that certain orders had been passed by the Courts below for payment of
alimony by the appellant to the respondent but that the same had not been complied with.
At this stage it may also be mentioned that a male child (Chetan) had been born out of the
wedlock on 28.2.1993 and we had hoped that the child would act as a catalyst to an
amicable settlement, but even the existence of the child could not bring about a
reconciliation between the parties.
7. Since despite the attempts at reconciliation the Gordian Knot could not be untied and
clearly the marriage has broken down irretrievably, it was submitted on behalf of both the
parties that it would perhaps be to the best interest of the parties to have the marriage tie
dissolved with adequate provision by way of permanent alimony for the respondent.
8. It is in this background that we have to consider the appellant's prayer to set aside the
judgment of the High Court as also that of the trial court and to grant a decree for
dissolution of the marriage between the appellant and the respondents.
9

. The prayer made on behalf of the appellant and endorsed by the respondent is neither
novel nor new. At the very beginning of this Judgment we had referred to the decision of
this Court in the case of Romesh Chander (supra), where it was held that when a marriage
is dead emotionally and practically and there is no chance of its being retrieved, the
continuance of such a marriage would amount to cruelty. Accordingly, in exercise of
powers under Article 142 of the Constitution of India the marriage between the appellant
and the respondent was directed to stand dissolved, subject to the condition that the
appellant would transfer his house in the name of his wife. 1995 AIR SCW 647

10

. The power vested in this Court under Article 142 of the Constitution was also exercised
in - i) Anjana Kishore vs. Puneet Kishore, (2002) 10 SCC 194; (ii) Swati Verma vs. Rajan
Verma and Ors., (2004) 1 SCC 123; and (iii) Durga Prasanna Tripathy vs. Arundhati
Tripathy, (2005) 7 SCC 352. Of the three aforesaid cases, in the first two cases orders
passed were on Transfer Petitions where ultimately the parties agreed to divorce by
mutual consent under Section 13-B of the Hindu Marriage Act, 1955. Resorting to the
powers reserved to this Court under Article 142, decrees of divorce were granted to put a
quietus to all litigations pending between the parties on the ground that their marriages
had broken down irretrievably. In the last of the three cases, while holding that the
marriage had broken down irretrievably, this Court affirmed the decree of divorce passed
by the Family Court, but directed payment of alimony to the extent of Rs. 1,50,000.
2003 AIR SCW 5841
2005 AIR SCW 4045

11. Having dispassionately considered the materials before us and the fact that out of 16
years of marriage the appellant and the respondent had been living separately for 14
years, we are also convinced that any further attempt at reconciliation will be futile and it
would be in the interest of both the parties to sever the matrimonial ties since the
marriage has broken down irretrievably.
12

. In the said circumstances, following the decision of this Court in Romesh Chander's
case (supra) we also\ are of the view that since the marriage between the parties is dead
for all practical purposes and there is no chance of it being retrieved, the continuance
1995 AIR SCW 647

@page-SC3095
of such marriage would itself amount to cruelty, and, accordingly, in exercise of our
powers under Article 142 of the Constitution we direct that the marriage of the appellant
and the respondent shall stand dissolved, subject to the appellant paying to the respondent
a sum of Rupees Two lakhs by way of permanent alimony. In addition, the appellant shall
also pay the costs of this appeal to the respondent, assessed at Rs.25,000/-. The appeal is
disposed of accordingly.
Order accordingly. .
AIR 2008 SUPREME COURT 3095 "Deep Chandra Juneja v. Lajwanti Kathuria"
(From : Allahabad)*
Coram : 2 C. K. THAKKER AND LOKESHWAR SINGH PANTA, JJ.
Civil Appeal No.7300 of 2005, D/- 10 -7 -2008.
Deep Chandra Juneja v. Smt. Lajwanti Kathuria (Dead) through L. Rs.
U.P. Urban Buildings (Regulation of Letting, Rent and Eviction) Act (13 of 1972),
S.21(1)(a) - U.P. Urban Buildings (Regulation of Letting, Rent and Eviction) Rules
(1972), R.16 - HOUSES AND RENTS - CONCURRENT FINDINGS - APPELLATE
AUTHORITY - TENANCY - SPECIAL LEAVE APPEAL - Bona fide requirement -
Concurrent findings of Prescribed Authority and Appellate Authority in favour of Land
lady - Tenant also found to have separate accommodation in City - Comparative hardship
found to be more pressing to landlady in comparison to tenant - Findings affirmed by
High Court in writ petition - No infirmity shown in decision of High Court - No
interference called for by Supreme Court.
Constitution of India, Art.136. (Para 20)
Cases Referred : Chronological Paras
2007 AIR SCW 2306 : 2007 (3) ALJ 519 (Ref.) 19
2001 AIR SCW 598 : AIR 2001 SC 803 : 2001 All LJ 433 (Ref.) 19
(1996) 5 SCC 353 (Ref.) 19
Z.M. Naiyer and Satish Vig, for Appellant; Anil Kumar Gupta-II and K.K. Gupta, for
Respondent.
* W. P. No. 3132 of 2004, D/- 6-2-2004 (All).
Judgement
LOKESHWAR SINGH PANTA, J. :- This appeal is directed against the judgment and
order dated 06.02.2004 passed by the learned Single Judge of the High Court of
Judicature at Allahabad dismissing the writ petition filed by the appellant-tenant.
Challenge in the writ petition was to the order passed by the prescribed authority as
affirmed by the learned Additional District Judge, Court No. 1, Kanpur Nagar (Appellate
Authority) allowing the release application of Smt. Lajwanti Kathuria respondent-
landlady.
2. Facts, in brief, leading to the origin of this case, are as under :-
Smt. Lajwanti Kathuria, respondent-landlady was the owner of house No. 251 Ghaoo
Khera Post, Chakeri, Harjinder Nagar, Kanpur. The appellant herein is the tenant on the
ground floor of the demised premises consisting of one room with doo-chatti (store),
courtyard, one bathroom, one toilet and kitchen since the year 1972 on a monthly rent of
Rs. 60/-. The landlady filed release application No. 18 of 2001 under Section 21(1)(a) of
the U. P. Urban Buildings (Regulation of Letting, Rent and Eviction) Act, 1972 [for short
'the Act'] for the need of her sons, daughters-in-law and grand children. In the release
application, respondent-landlady stated that her husband Shri Chaman Lal Kathuria,
before his death, was carrying on kiryana business in small portion of the premises. She
stated that she has a large family to support comprising herself, two married sons and
their children and one daughter and her children. According to the landlady, she urgently
needed additional accommodation to provide proper and comfortable living rooms,
kitchens and bathrooms for her children who are facing tremendous inconvenience and
hardships on account of shortage of accommodation.
3. The release application was contested by the appellant-tenant by filing written
statement and denying the claim of the landlady. He, inter alia, took the stand that the
claim of the landlady was not bona fide and genuine and the release application has been
moved with the mal-intention of harassing him. It was also alleged that the sons of the
landlady were residing with her and the landlady did not need more accommodation
because other tenants who were living in the building had vacated the accommodations,
which were again let out by her to new tenants on higher rent. He also stated that the
landlady was having one more house No. 377, N-2 Road, Lal Bungalow, Harjinder
Nagar, Kanpur.
@page-SC3096
4. Both the parties were directed by the prescribed authority to submit their affidavits in
support of their claims and counterclaims.
5. The prescribed authority vide order dated 05.03.2003 allowed the release application of
the landlady and directed the appellant-tenant to hand over vacant possession of the
rented accommodation to the landlady within one month.
6. Against the said order, Rent Appeal No. 17 of 2003 was filed by the appellant-tenant
before the Appellate Authority, which was dismissed on 05.01.2004.
7. Being aggrieved against and dissatisfied with the judgment and order of the Appellate
Authority, the appellant-tenant preferred Civil Misc. Writ Petition No. 3132 of 2004
before the High Court of Allahabad, which was dismissed by the learned Single Judge by
impugned judgment. The relevant paragraph of the judgment is extracted as under :-
"The courts below after taking into consideration the entire material on record, recorded
finding of bona fide need and comparative hardship in favour of landlord. The impugned
order requires no interference in exercise of writ jurisdiction.
Accordingly, the writ petition is dismissed.
However, the petitioner is granted one year's time to vacate the premises, provided that
within one month from today, he files an undertaking before the Prescribed Authority that
on or before expiry of aforesaid period of one year he will willingly vacate and handover
the possession of the house in dispute to the landlord."
8. Now, the appellant-tenant has preferred this appeal by way of special leave challenging
the correctness and validity of the order of the High Court.
9. During the pendency of the appeal, the respondent-landlady died and was substituted
by her legal heirs.
10. Mr. Z.M. Naiyer, learned counsel appearing on behalf of the appellant-tenant,
contended that the approach of the High Court in dismissing the Writ Petition in slipshod
manner without recording reasons is erroneous. He submitted that the courts below have
committed manifest error of law and jurisdiction in entertaining and allowing second
application for release of the demised premises as her first application for the same relief
was already allowed by the Appellate Authority in Appeal No.4 of 1983 and two release
orders in respect of the same accommodation, one conditional and other blanket, cannot
co-exist and that being the position, the impugned orders are vitiated in law. He next
contended that the second application for the release of the demised premises filed by the
landlady was an abuse of process of law and the courts below should not have entertained
the second release application of the landlady to perpetrate a fraud in judicial proceedings
to achieve her nefarious designs. It was also contended that the landlady before her death
had possessed 11 cozy rooms with other allied accommodation for the need of her 5
adults and 4 minor members of the family, which are quite sufficient for their peaceful
living, but while determining the bona fide need of the landlady the courts below have
ignored the comparative hardship of the appellant-tenant.
11. Mr. Anil Kumar Gupta-II, learned counsel appearing on behalf of the respondents-
legal representative of landlady, on the other hand, contended that the prescribed
authority on proper and careful consideration of the evidence led by the parties before it,
passed a reasoned order of eviction against the appellant-tenant, the said order has been
affirmed by the Appellate Authority and also by the High Court. He submitted that this
Court ordinarily should not interfere with the concurrent findings of facts recorded by the
competent courts/authorities in exercise of its power and jurisdiction under Article 136 of
the Constitution of India.
12. We have duly considered the respective contentions of the learned counsel for the
parties and perused the entire material on record.
13. The facts narrated hereinbefore are not in dispute to the extent that the landlady
rented the demised premises to the appellant-tenant for residential purpose. It appears
from the record that on 26-11-1982/27-11-1982 an application bearing R.C. No.67 of
1982 was filed by late Smt. Lajwanti Kathuria - landlady against the appellant-tenant for
release of the demised premises. The said application was dismissed by the prescribed
authority predominantly on the ground that the premises earlier vacated by some tenants
were not
@page-SC3097
utilised by the landlady for the use and occupation of her family members, but in fact
those were let out to new tenants. However, the appeal filed by the landlady against the
impugned order of the prescribed authority was allowed by the Appellate Authority on
21.05.1983, whereby the appellant-tenant was directed to vacate the demised premises
and shift to an alternate accommodation being offered by the landlady. It was also
ordered that in case the appellant-tenant failed to vacate the premises in question within
the stipulated period, the landlady would get possession of the accommodation through
Court proceedings. The appellant-tenant submitted that the landlady did not provide the
alternative accommodation in terms of the order of the Appellate Authority and allowed
the stalemate to continue. The landlady filed second release application which was
registered Rent Case No. 18 of 2001 on the file of the prescribed authority, Kanpur
Nagar. From the pleadings of the parties, both the prescribed authority and the Appellate
Authority have noted that the landlady at the time of filing of the release application in
the year 2001 was residing in the same house with her elder son, Satish Kathuria, his
educated wife Smt. Namita and her grandsons Abhishek, Rajat and granddaughter
Karishma and younger son Joginder Kathuria, his educated wife Smt. Rajani, grand-
daughter Tanu; her married daughter Smt. Neelam Khanna, son-in-law Manish Khanna
and their daughters Kum. Kirti and Kum. Neha. She filed affidavit in support of her
release application stating inter alia that her elder son is a Contractor/ Builder, and he
needs separate room for his educated wife and children who are studying in convent
school; she needed room for her second son Joginder Kathuria, an Advocate and his
educated wife and daughter and separate accommodation for her married daughter, son-
in-law and their children who have frequently been paying visits to her house because of
her old age. She also stated that she needed one room for her old lady friends, one room
for prayer purpose and one room to be used as a Museum to preserve some articles of her
late husband. She stated that the appellant-tenant took the demised premises on rent in
July 1972 with clear understanding that being a member of Indian Air Force, he would be
transferred from his place of posting within a period of 3 to 4 years and then he would
vacate the premises forthwith. The appellant-tenant retired in the year 1984 and thereafter
he started running a small provision store in the name and style of 'Juneja Provision
Store'. Rajiv Juneja, the elder son of the appellant-tenant, is doing construction work in
the name of M/s. Juneja Engineering and also doing the business of chemical sale and
purchase in one portion of House No.229 Patel Nagar, Shanker Nagar. He is living in
rented accommodation along with his wife and children at 140-C Ramgali, Harjinder
Nagar, Kanpur, where he is in occupation of three rooms, one bathroom, latrine, kitchen
and verandah. He is also doing travel agency business in the name of Juneja Travels at
Pardemanpura, Harjinder Nagar, Kanpur. It was stated that Ajay Juneja, the second son of
the appellant-tenant, is a teacher in Virendra Swaroop Education Centre and his wife is
doing some business and they are living in House No. 124A/272, Block 11, Govind
Nagar, Kanpur, having five rooms, latrine, bathroom, kitchen, verandah and courtyard.
14. One more son of the appellant-tenant, namely, Vijay Juneja, is working as a T.V.
Engineer in Thomson Co. and his educated wife is a teacher. They are occupying two
rooms, latrine, bathroom, kitchen and verandah as tenants in premises No. 1-A Ghaoo
Khera Post Chakeri, Kanpur. The landlady also contended that the appellant-tenant had
sub-let the rented accommodation to one R. N. Singh on monthly rent of Rs.300/-, in
respect of which Civil Suit No. 1112 of 2000 titled R. N. Singh v. D.C. Juneja is pending
in the Court of Civil Judge (J/D), Kanpur Nagar. The landlady also stated that she filed
Civil Suit No.397 of 1997 against the appellant- tenant in the Court of J.S.C.C., Kanpur
Nagar, for ejectment of the appellant-tenant inter alia on the grounds of default in
payment of rent, subtenancy and damage to the demised house and the said suit is still
pending. It was also stated that the appellant-tenant has constructed a house over one plot
at Gandhi Gram and the appellant-tenant and his family members are financially very
sound. The daughter of the appellant-tenant is married and she is residing at Lucknow.
15. The appellant-tenant denied the genuine requirement of the landlady. However, he
submitted that in fact there are only nine family members of the landlady and not ten to
thirteen members as alleged by her. He stated that in the past over several
@page-SC3098
years, the landlady had evicted many tenants and thereafter she let out the
accommodations to other persons during the pendency of the present eviction application.
Looking to the strength of the family members of the landlady and accommodation
available with them, the landlady did not need any additional accommodation as per the
version of the appellant-tenant. He stated that he is living in the demised premises with
his wife, son Rajiv and his wife Gayatri, grandson Nikhil and his second son Vijay and
his wife Rinku, whereas his third son Ajay Juneja is living in a separate house No.
124A/272, Block 11, Govind Nagar, Kanpur, which was owned by Ajay's grandfather late
Shri Sant Ram Juneja. He stated that he has no other accommodation in the city of
Kanpur. He denied the allegation of sub-letting the rented accommodation to one R.N.
Singh at the rate of Rs.300/- per month and stated that the landlady got false suit
instituted against him through R.N. Singh. He submitted that as the landlady refused to
accept the rent he, therefore, has been regularly depositing the settled rent under Section
30(1) of the Act in the Court of Civil Judge, Junior Division, Kanpur Nagar. He pleaded
that if he is ordered to be evicted from the demised premises, he will suffer irreparable
injury and hardship as compared to the landlady.
16. The prescribed authority, on assessment of the entire evidence on record, came to the
conclusion that the appellant-tenant has failed to establish that the landlady had let out
some portions of the house to the new tenants on higher rent. The prescribed authority
stated in its order that the landlady has filed on record Paper No.28 in which residential
address of the appellant-tenant is mentioned as 1A, Ghaoo Khera, and other Paper
Nos.29, 30, 31 and 32 also reveal the same address of the appellant-tenant. On the basis
of the oral as well as documentary evidence on record, the prescribed authority came to
the conclusion that the appellant-tenant is residing with his son Vijay Juneja and his
family members. The averments of the landlady in her eviction application that the
appellant-tenant has got House No.140-C Ramgali, Harjinder Nagar, 220, Patel Nagar,
1/382 N-2 Road, Harjinder Nagar, has been supported by Ashok Kumar and Rakesh
Kumar in their affidavits. On comparative scrutiny of the statements pleaded by both the
parties in their pleadings and supported by their evidence, the prescribed authority has
noticed that in the Voter List Paper Nos. 45 and 46 for the year 1995-99 name of Ajay
Juneja, son of the appellant- tenant, has not been shown as occupant of premises No.
124A/272, Block 11, Govind Nagar, as contended by the appellant-tenant in his written
statement and deposition in the affidavit. In fact, Ajay Juneja at the relevant time, was
residing at house No. 251 and later on he shifted to premises No. 124A/272. The
appellant-tenant, during the pendency of the eviction proceedings, has not made any
attempt to secure alternate accommodation and he has been found living with other
members of the family in some other house and the prescribed authority as also the
Appellate Authority on scrutiny of the evidence have concluded that House No.
124A/272 was owned by the father of the appellant-tenant, which was transferred in the
name of Ajay Juneja, son of the appellant-tenant, by executing a collusive Will with clear
intention to show that the appellant-tenant does not own any house in Kanpur City and in
order to frustrate the need of the landlady. Both the courts below recorded concurrent
findings of facts that the appellant-tenant and his family members have got separate
accommodation in Kanpur City and on the basis of the evidence on record, the
requirement of the accommodation of the landlady is bona fide and genuine and the
comparative hardship also is more pressing to the landlady in comparison to the
appellant-tenant. In that view of the matter, the High Court in its jurisdiction under
Article 226 of the Constitution has rightly dismissed the writ petition of the appellant-
tenant on the premises of concurrent findings of facts.
17. In the backdrop of the facts, indisputably, the landlady could file an application
before the prescribed authority for the eviction of the appellant-tenant from the premises
under tenancy or any specified portion, thereof and the prescribed authority after
satisfying itself that the ground as stipulated in Section 21 of the Act does exist, the
eviction order has been validly passed against the appellant-tenant.
18. Rule 16 of the U. P. Urban Buildings (Regulation of Letting, Rent and Eviction)
Rules, 1972, prescribes certain factors which have also to be taken into account by the
prescribed authority while considering the application for eviction of a tenant on
@page-SC3099
the ground of bona fide need. Sub-rule (1) of Rule 16 relates to the cases of eviction from
an accommodation for the purpose of residence by the landlord or any members of his
family. Considering the facts in the light of Rule 16 pressed into service on behalf of the
appellant- tenant that taking into consideration the length of the period of tenancy, the
eviction of the appellant-tenant from the premises in question is in violation of the
provision of Rule 16, in our opinion, cannot be accepted.
19

. In Gaya Prasad v. Pradeep Srivastava [(2001) 2 SCC 604, this Court held that the need
of the landlord is to be seen on the date of application for release. In Prativa Devi v. T.V.
Krishnan [(1996) 5 SCC 353], it was held, that the landlord is the best judge of his
requirement and courts have no concern to dictate the landlord as to how and in what
manner he should live. In Rishi Kumar Govil v. Maqsoodan and Ors. [(2007) 4 SCC
465], this Court while dealing with the provisions of Section 21(1)(a) of the U. P. Urban
Buildings (Regulation of Letting, Rent and Eviction) Act, 1972 and Rule 16 of the U. P.
Urban Buildings (Regulation of Letting, Rent and Eviction) Rules, 1972, held that the
bona fide personal need of the landlord is a question of fact and should not be normally
interfered with. 2001 AIR SCW 598
2007 AIR SCW 2306

20. Having regard to the well-reasoned concurrent findings and reasoning recorded by the
prescribed authority and the Appellate Authority, which are affirmed by the High Court in
writ petition, we are of the opinion that the impugned judgment warrants no interference
inasmuch as no illegality, infirmity or error of jurisdiction could be shown before us by
the appellant-tenant.
21. In the result, for the above-stated reasons, we find no merit in this appeal and it is
dismissed, accordingly.
22. However, considering the period for which the premises in question was in the
occupation of the appellant-tenant, time is granted till 31.06.2009 to vacate the premises,
subject to filing of an affidavit by way of usual undertaking before the prescribed
authority within a period of four weeks to deliver the vacant possession on or before the
stipulated date. There will be no order as to costs.
Appeal dismissed. .
AIR 2008 SUPREME COURT 3099 "Anita Devi v. Satyendra Narain Singh"
(From : Jharkhand)*
Coram : 2 Dr. A. PASAYAT AND P. SATHASIVAM, JJ.
Civil Appeal No.4291 of 2008 (arising out of SLP (C) No. 15368 of 2006), D/- 10 -7
-2008.
Anita Devi and Ors. v. Satyendra Narain Singh and Ors.
Motor Vehicles Act (59 of 1988), S.168, S.173 - MOTOR VEHICLES - TRIBUNALS -
Accident - Compensation - Determination- Income of deceased - Taken on notional basis
on ground that no material was adduced by claimants - Record, however, showing that
documents adduced can certainly throw light on income aspect - Award remitted to
Tribunal to consider matter relating to Income of deceased and determine compensation
afresh.
M.A. No. 155 of 2003, D/-09-07-2004 (Jhar.), Reversed. (Para 6)

Deba Prasad Mukherjee, Arvind Kr. Lall and Mrs. Nandini Sen, for Appellants; A.K.
Raina and Dr. Kailash Chand, for Respondents.
* M. A. No. 155 of 2003, D/- 9-7-2004 (Jha).
Judgement
Dr. ARIJIT PASAYAT, J. :- Leave granted.
2. Challenge in this appeal is to the order passed by a Division Bench of the Jharkhand
High Court, Ranchi dismissing the Miscellaneous application filed by the appellants
under Section 173(1) of the Motor Vehicles Act, 1988 (in short the 'Act').
3. Case of the appellants, in a nutshell, is as follows :
Pramod Kumar (hereinafter referred to as the 'deceased') died in a vehicular accident in
which Maruti Van bearing registration No. ER-14P-4320 was involved. The Maruti Van
was being driven by respondent No. 1 rashly and negligently. Initially, Pramod Kumar
had sustained grievous injuries. He was first taken to the Government Hospital from
which he was referred to Bokaro General Hospital where he had expired on 18.4.2000.
The deceased was 37 years of age. The petition for compensation in terms of Section 166
of the Act was filed by the dependants of the deceased. The Motor Accidents Claims
Tribunal (in short the 'MACT) on consideration of the materials placed before it held that
claimants are entitled to compensation of Rs. 1,39,808/-. Since the vehicle was the
subject matter of insurance, the Oriental Insurance Co. Ltd. (hereinafter referred to as the
'insurer') was held liable for the compensation amount to the claimants along with interest
@ 9% per annum
@page-SC3100
from the date of filing of the application. It was found that there is no concrete material
regarding the income of the deceased. However, it was held that notional income of
Rs.15,000/- p.a. can be taken after deducting certain amounts for personal use. The
contribution was fixed at Rs.10,216/-p.a. The multiplier of 13 was applied and Rs.5,000/-
was also granted for loss of expectation of life and Rs. 2,000/- for funeral expenses. An
appeal was preferred by the claimants questioning correctness of the award, taking the
stand that the quantum fixed was very low. High Court dismissed the appeal holding that
there was no evidence of earning income.
4. In support of the appeal, learned counsel for the appellant submitted that several
documents were filed to establish the income of the deceased. This aspect was not taken
note of either by the MACT or the High Court.
5. Learned counsel for the respondents submitted that the appellants did not produce any
definite material regarding income and the MACT was justified in taking the notional
income.
6. To test the correctness of the stand of the appellants that several documents were filed
to establish the income of the deceased, the original records from MACT were called for.
It appears from the records that certain documents have been filed. It is true that there are
no copies of the income tax return or the assessment order. But the documents on record
can certainly throw light on the income aspect.
7. Above being the position, we set aside the award of the MACT as affirmed by the High
Court and remit the matter to MACT to consider the matter relating to income of the
deceased and determine the compensation afresh taking into account the documents
already on record.
8. The appeal is allowed in the aforesaid extent with no order as to costs.
Appeal allowed. .
AIR 2008 SUPREME COURT 3100 "State of Rajasthan v. Gulab Singh"
(From : 2005 Cri LJ (NOC) 232 : 2005 (2) Cri LR 1075 (Raj.)
Coram : 2 Dr. A. PASAYAT AND P. SATHASIVAM, JJ.
Criminal Appeal No. 1049 of 2008 (arising out of SLP (Cri.) No. 4118 of 2006), D/- 10
-7 -2008.
State of Rajasthan v. Gulab Singh and Ors.
Criminal P.C. (2 of 1974), S.368 - HIGH COURT - APPEAL - MURDER - POST-
MORTEM - COMMON INTENTION - ATTEMPT TO MURDER - Appeal against
conviction - Murder case - Evidence of doctor as well as post mortem report showing that
deceased had suffered 12 ante mortem injuries - Appellate Court however holding that
there were no injuries on deceased - Conviction of accused as such altered from S.300,
S.34 to S.307 - Order of High Court clearly shows non-application of mind - Order liable
to be set aside.
2005 Cri LJ (NOC) 232 : 2005 (2) Cri LR 1075 (Raj.), Reversed. (Para 8)

V. Madhukar, Rajesh Kumar and Aruneshwar Gupta, for Appellant.


Judgement
Dr. ARIJIT PASAYAT, J. :- Leave granted.
2. Challenge in this appeal is to the order passed by a Division Bench of the Rajasthan
High Court, Jodhpur, altering the conviction of the respondent for offence punishable
under Section 302 read with Section 34 of the Indian Penal Code, 1860 (in short the
'IPC') to Section 307 IPC. However, the conviction under Sections 458 and 460 IPC were
maintained. The substantive sentences in respect of the offences were reduced to the
period already undergone.
3. The trial Court i.e. learned Sessions Judge (Fast Track), Rajsamand had convicted
respondents 1 to 4 for offences punishable under Section 302 read with Section 34 and
Sections 460, 458 and 397 of IPC and various other sentences in respect of the other
offences.
4. Prosecution version in a nutshell is as follows :
Sessions case in question arose from the first information report (Exhibit P-5) which was
presented by the complainant Shri Prakash Chand (PW-4) before the police incharge of
Arakshi Kendra, Rajsamand on 11.7.1999. It was stated therein that on 11.7.1999 in the
morning at around 6.30 a commotion was taking place outside the house of Dali Chand
father of Naval Ram. The complainant went to the house of Dali Chand and saw that Smt.
Jyoti wife of Dali Chand was lying dead there, whereas her hands and mouth/face were
tied with cloth. Inside the room the hands and legs of Dali Chand were also found tied.
Thereafter, Roop Singh untied his hands and legs. Goods were lying scattered inside the
room. The children of Dali Chand live in Bombay and Dali Chand was having a shop of
controlled commodities. This incident was stated to have been committed by some
unknown persons.
After the presentation of the aforesaid complaint, case No.479/99 for offences punishable
under Sections 460/458 IPC was registered and investigation commenced.
In the course of the investigation the investigating officer recorded the statement of
@page-SC3101
the complainant Prakash Chand. The injured Dali Chand was admitted in the hospital at
Rajsamand and Udaipur for treatment. His injury report Exh.P-4 was received. His x-ray
was also conducted. After inspection of the place of the incident, spot memo Exh.P-14
was prepared. The panchayatnama memo of the dead body of the deceased Jyotibai
Exh.P-1 was prepared. The clothes which had been used to tie the hands and the mouth of
the deceased were having blood on them and, therefore, they were seized as evidence
vide Exh. P-7. After conducting the post mortem of the dead body of the deceased the
report Exh.P-3 was taken on record. Her dead body was handed over to her heirs for
cremation vide Exh.P-2. From the place of the incident bloodstained stones and control
sample stones were seized in respect whereof Exh.P-8 was prepared. Statements of the
witnesses were recorded. A list of stolen articles was prepared. Chance prints were taken
from the place of the incident. Accused Gulab Singh, Uday Singh, Nathu Singh and
Laxman Singh were arrested. Gold and silver articles along with cash of Rs. 24,400/-
were recovered at their instance. Thereafter, jewellery was recovered at the instance of
accused Dhool Singh and Shambu Singh. In this regard Exhs. P-10 to Exh.P-13 were
prepared. The place of the incident was pointed out by the accused in respect whereof
Exhs. P-49 to 52 was prepared. The accused were identified by the witness-Dali Chand
and thereafter the seized case property in the case were also identified in respect whereof
exh. P-67 to 72 were prepared. Bloodstained clothes and stones were sent for FSL
examination to Forensic Sciences Laboratory, Udaipur. The place of the incident was
photographed. After necessary investigation sufficient evidence was found against
accused Gulab Singh, Uday Singh, Nathu Singh for offences punishable under Sections
460, 458, 302 IPC and against accused Dhool Singh, Moti Singh, Shambu Singh for
offences under Sections 414, 411, 120B IPC. The station in charge, Raj Nagar filed a
charge sheet against the above named accused persons before the Court of learned Chief
Judicial Magistrate, Rajsamand. Case was committed to the Court of Session.
In order to substantiate the accusations the prosecution examined 16 witnesses. The
doctor who examined the deceased found 12 injuries on the body of the deceased. Placing
reliance on the prosecution version in the light of the evidence led the learned trial Judge
recorded the conviction and imposed sentence as afore-stated.
It is to be noted that one Shambhu Singh was sentenced to one year's rigorous
imprisonment and fine of Rs.1,000/- for the offence punishable under Section 411 IPC
was imposed. In the appeal before the High Court he was not a party. The High Court
altered the conviction primarily on the ground that there was no injury on the person of
the deceased, as allegedly accepted by learned Public Prosecutor.
5. Learned counsel for the appellant-State submitted that the reasoning of the High Court
is utterly fallacious as there was no question of the learned Prosecutor feebly agreeing
that there was no injury on the person of the deceased. In fact, the evidence of doctor to
which reference has been made by the trial Court is clear to the extent that there were 12
injuries on the body of the deceased. In the post-mortem report also injuries were
indicated. It is, therefore, submitted that the High Court was clearly in error in altering
the conviction from Section 302 read with Section 34 IPC to Section 307 IPC.
6. There is no appearance on behalf of the respondents in spite of service of notice.
7. On a bare reading of the High Court's order it is clear that it is a classic case of non-
application of mind. The only conclusion indicated by the High Court to alter the
conviction reads as follows :
"Learned PP feebly agrees that there was no injury on the person of the deceased.
Mechanical injury being absent it would be unjust if the arguments of the learned counsel
for the appellants is not given some weightage. In this background offence under Section
307 IPC would be clearly made out because in that process there was an attempt by virtue
of which one of the victims have died."
8. As rightly submitted by learned counsel for the State that from the evidence of the
doctor to which reference has been made by the trial Court and the post-mortem report, it
is clear that there were 12 injuries noticed on the body of the deceased and each one of
them was described to be antemortem. It is not clear as to how the High Court observed
that there was no injury on the body of the deceased. Still more surprising is the
observation that "mechanical" injury being absent it would be unjust, if the argument of
the learned counsel for the accused is not given some weightage. It is not understood as
to what the High Court meant by the expression 'mechanical injury'. It is unfortunate that
a Division Bench of the High Court has come to such atrocious and fallacious
conclusions. The appeal deserves to be allowed which we direct. The
@page-SC3102
judgment of the trial Court is restored and, therefore, the High Court's order so far it
relates to alteration of conviction from Section 302 read with Section 34 to Section 307
IPC stands set aside. The respondent shall surrender to Custody forthwith to serve the
remainder of sentence.
9. The appeal is allowed.
Appeal allowed. .
AIR 2008 SUPREME COURT 3102 "Ramachandran v. R. Udayakumar"
(From : Madras)*
Coram : 2 Dr. A. PASAYAT AND P. SATHASIVAM, JJ.
Criminal Appeal No. 871 of 2008 (arising out of S.L.P. (Cri.) No. 1105 of 2007), D/- 13
-5 -2008.
Ramachandran v. R. Udayakumar and Ors.
Criminal P.C. (2 of 1974), S.173(2), S.173(8) - INVESTIGATION - Investigation -
Completion of, under sub-s.(2) of S.173 - Thereafter there can be further investigation if
required under sub-s.(8) of S.173 - But not fresh investigation or re-investiagation.
(Paras 6, 7)
Cases Referred : Chronological Paras
1998 AIR SCW 1852 : AIR 1998 SC 2001 : 1998 Cri LJ 2897 (Ref.) 6
Siddarth Dave, Vijay Thakur and Senthil Jagadeesan, for Appellant; K. Ramamoorthy, Sr.
Advocate, V.G. Pragasam, S.J. Aristotle, Prabhu Ramasubramanian, A. Mariarputham,
B.K. Prasad, P. Parmeswaran, B. Balaji, K. Muthu Ganesa Pandian and Satya Mitra Garg,
for Respondents.
Judgement
1. Dr. ARIJIT PASAYAT, J. :-Leave granted.
2. Challenge in this appeal is to the order passed by a learned Single Judge of the Madras
High Court on a petition filed by respondent No. 1 under Section 482 of the Code of
Criminal Procedure, 1973 (in short 'the Code'). The prayer was to direct the respondent
No. 2 the State of Tamil Nadu represented by its Secretary, Government of Home
Department to withdraw the litigation in Crime No. 39/2004 on the file of Inspector of
Police, Palayanoor Police Station, Sivagangai District and to entrust the same to the file
of Central Bureau of Investigation (in short 'CBI'). They are respondent Nos. 5 and 6 in
the present appeal. Respondent No. 1 had filed the petition seeking for direction to re-
investigate the case by the CBI in an alleged case of murder by respondent No. 1. There
were totally 59 witnesses in the case. The High Court disposed of the petition, inter alia,
with the following directions :
"8. Under the above facts and circumstances of the case in the interest of justice, the case
in Crime No. 39/2004 on the file of the fourth respondent Stands transferred to the
Deputy Superintendent of Police, C.B.C.I.D., Madurai who shall entrust this case to a
competent and efficient Inspector of Police for the purpose of re-investigation in this
case. The Inspector of Police who is nominated by the Deputy Superintendent of Police,
C.B.C.I.D. shall afresh investigate the matter and file the final report within a period of
three months from the date of receipt of a copy of this order from this Court. The fourth
respondent shall forthwith hand over the case records in crime No. 39/2004 to the officer
to be nominated by the Deputy Superintendent of Police, C.B.C.I.D., Madurai. The
Petitioner stands ordered accordingly. Consequently/connected miscellaneous petition is
closed."
3. Learned counsel for the appellant submitted that the background facts are as follows :
On 16.7.2004 crime case was registered against respondent No. 1 and other accused
persons for alleged commission of offence punishable under Sections 147, 148, 324, 302
and 307 of the Indian Penal Code, 1860 (in short 'IPC'). On 4.8.2004 Criminal O.P. No.
444/2004 was filed by the appellant before the Madras High Court seeking transfer of
investigation in the case to some other Investigating Officer. On 24.3.2005 charge-sheet
No. 18/2005 was filed by the Inspector of Police against respondent Nos. 1 and 8 other
accused persons for commission of offence punishable under Sections 148, 302, 307 and
324 read with Section 149, IPC. On 25.4.2005 on the basis of the representation given by
the father of respondent No. 1, the Additional District Superintendent of Police,
Sivagangai filed a report in view of G.O. No. 14/ADSP/Crime/ Sivagangai/2005 and
concluded that further investigation under Section 173 (8) of the Code was necessary in
the case to find out the real culprit. Since, there was no progress even after filing of the
charge-sheet, the appellant approached the High Court seeking for a direction to the
Inspector of Police to execute the NBWs issued against 7 of the original 9 accused
including respondent No. 1 since the Inspector was colluding with the accused. The High
Court ordered S.P., Sivagangai to arrest the absconding accused persons. On 27.3.2006 an
application in terms of Section 173 (8) of the Code being Criminal M.P. No. 2227/2006
was filed by the Inspector of Police, Palayanoor Police Station, before the learned
Judicial Magistrate, Manamadurai, seeking permission to further investigate the case. The
permission
@page-SC3103
was granted. On 3.4.2006 the Inspector investigating the case wrote to the Deputy
Director, of Prosecution seeking his opinion as to whether the case was required to be
transferred to C.B.C.I.D. On 10.4.2006 legal opinion was given by the Deputy Public
Prosecutor stating that the case ought to be transferred to CBCID wing. On 20-11-2006,
Criminal O.P. No. 9175 of 2006 was filed by respondent No. 1 before the High Court
seeking for a direction for reinvestigation of the case, by the CBI. On 18.12.2006 learned
Single Judge directed the Inspector of Police nominated by the Deputy Superintendent of
Police, CBCID to investigate the matter afresh and thereafter file the final report. On
5.4.2007, Criminal M.P. No. 1/ 2007 was filed in the said Criminal O.P. by the Inspector
of police, Crime Branch, CID, Sivagangai District, Madurai Wing before the High Court
of Madras at Madurai seeking further six months' time to complete the investigation and
file the final report. The impugned order passed by the High Court is the order dated
18.12.2006.
4. It is the stand of the appellant that in an application under Section 482 the direction as
given could not have been given. It is stated that there was no scope for fresh or re-
investigation in view of what is provided in Section 173(8) of the Code.
5. Learned counsel for respondent No. 1 supported the order of the High Court.
6

. At this juncture it would be necessary to take note of Section 173 of the Code. From a
plain reading of the above section it is evident that even after completion of investigation
under sub-section (2) of Section 173 of the Code, the police has right to further
investigate under sub-section (8), but not fresh investigation or re-investigation. This was
highlighted by this Court in K. Chandrasekhar v. State of Kerala and Ors. (1998 (5) SCC
223). It was, inter alia, observed as follows : 1998 AIR SCW 1852, Para 25

"24. The dictionary meaning of "further" (when used as an adjective) is "additional;


more; supplemental". "Further" investigation therefore is the continuation of the earlier
investigation and not a fresh investigation or re-investigation to be started ab initio
wiping out the earlier investigation altogether. In drawing this conclusion we have also
drawn inspiration from the fact that sub-section (8) clearly envisages that on completion
of further investigation the investigating agency has to forward to the Magistrate a
"further" report or reports - and not fresh report or reports - regarding the "further"
evidence obtained during such investigation."
7. In view of the position of law as indicated above, the directions of the High Court for
re-investigation or fresh investigation are clearly indefensible. We, therefore, direct that
instead of fresh investigation there can be further investigation if required under Section
173 (8) of the Code. The same can be done by the CB (CID) as directed by the High
Court.
8. The appeal is allowed to the aforesaid extent.
Order accordingly. .
AIR 2008 SUPREME COURT 3103 "Skyline Contractors Pvt. Ltd. v. State of U. P."
(From : 2007 (2) UPLBEC 1783)
Coram : 2 A. K. MATHUR AND ALTAMAS KABIR, JJ.
Civil Appeal No.4272 of 2008 (arising out of SLP (C) No. 7722 of 2007), D/- 9 -7 -2008.
Skyline Contractors Pvt. Ltd. and Anr. v. State of U.P. and Ors.
Constitution of India, Art.226 - WRITS - ALLOTMENT OF PREMISES - Allotment of
plot - Cancellation - Validity - Appellant-allottee not making any deposits other than
initial deposit - Deposits ultimately made 2½ years after allotment order was passed and
was made unilaterally - Reasons given for non-payment/delayed payment not very
convincing - Thud party interests intervened and fresh allotment order made in favour of
respondent - No prayer made in writ petition for setting aside such allotment -
Cancellation of allotment not liable to be set aside. (Paras 17, 18, 19)
Cases Referred : Chronological Paras
(2004) 2 SCC 130 10
2002 AIR SCW 2489 : AIR 2002 SC 2340 9
Shall Kr. Dwivedi, A.A.G., Arun Bhardawaj, Dr. A.M. Singhvi, Ratnakar Dash, Sr.
Advocates, Mahesh Agarwal, Rishi Agarwal, Arvind Kumar, Gaurav Goel, E.C.
Agrawala, Manu Nair, Mark D'Souza, Ms. S.N. Purohit (for M/s. Suresh A. Shroff and
Co.), D.K. Goswami, Anuvrat Sharma and Ravindra Kumar, for the appearing parties.
Judgement
ALTAMAS KABIR, J. :- Leave granted.
2. The appellants herein filed a writ petition before the Allahabad High Court for
quashing an order dated 21.6.2006 issued on behalf of the New Okhla Industrial
Development Authority (hereinafter referred to as 'NOIDA') cancelling the allotment of
Plot No.A-28 in Sector 62 made in favour of the appellant.
3. Admittedly, the appellant made an application for allotment of the aforesaid
@page-SC3104
plot measuring 8000 square meters pursuant to an advertisement published on behalf of
the NOIDA inviting such applications and made an initial deposit of Rs. 13,20,000/ -
while submitting the application. On 17.4.2003 an order of allotment was issued in favour
of the appellant whereby the petitioner was required to deposit 25 per cent of the
premium amount in cash or by a bank draft in favour of NOIDA within 60 days of such
allotment. It was categorically stipulated that if the said amount was not deposited within
the time specified the depositor's earnest money would be forfeited and no extension of
time would be granted for deposit of the said amount under any circumstances. The
balance 75% of the premium amount was required to be deposited by the allottee in 10
equal half-yearly instalments along with interest at the rate of 14% per annum on
outstanding premium. Here also, it was categorically stipulated that no extension for
payment of instalments would be granted and if the allottee failed to pay the instalments
within due dates the allotment would be cancelled and the amount equivalent to 25% of
the premium would be forfeited in favour of the NOIDA. In exceptional circumstances,
however, the Chief Executive Officer of NOIDA was vested with the discretion to extend
the time for making deposits, which would be subject to payment of interest @ 17% per
annum compounded every half yearly on the defaulted amount for the defaulted period.
4. As has been noticed by the High Court in its judgment impugned in this appeal, the
appellants did not deposit any amount for a period of two and a half years after receipt of
the allotment letter. The Authority wrote to the appellant on 3.5.2005 requesting the
appellant to produce receipts of deposits, if any, made in pursuance of the allotment letter.
Three months after receiving the said letter the appellant started making deposits in
September 2005 and on 16.12.2005 wrote to the NOIDA asking for details with regard to
the deposit of stamp duty, etc. for execution of the lease deed pursuant to the allotment
made in its favour. Despite the said letter, the NOIDA cancelled the allotment made in
favour of the appellants by its order dated 21.6.2006 on the ground that the appellant had
failed to make the deposits as per clause 2(iv) of the Terms and Conditions for allotment.
As stated hereinbefore, the writ petition was filed challenging such cancellation.
5. On considering the submissions made on behalf of the parties the High Court rejected
the plea of the appellants that although the appellants had failed to deposit the premium
amount in keeping with the terms and conditions of the allotment, the said amount
subsequently deposited by the appellants had been duly accepted by the NOIDA which
had accordingly waived such terms and conditions and the allotment made in the
appellant's favour could not have been cancelled on the ground that the same had not
been deposited in time. The High Court also rejected the other submission made on
behalf of the appellants that the NOIDA had acted wrongly in re-allotting the plot in
question to the Respondent No.5 at a much cheaper rate than was demanded from the
appellants. The High Court held that having failed to make the deposits within the time
stipulated in the allotment letter the voluntary deposits subsequently made two and half
years after the issuance of the allotment letter, without the approval of the NOIDA, could
not be accepted as valid deposit and the appellants were not, therefore, entitled to any
relief. Certain judgments of this Court which have been relied upon before us by the
appellants had also been considered by the High Court which came to the conclusion that
the same were not applicable to the facts and circumstances of the instant case. The High
Court, therefore, held the writ petition to be completely misconceived and dismissed the
same.
6. The same arguments as was advanced before the High Court have also been advanced
before us with special emphasis on the letter dated 15.5.2003 written on behalf of the
appellant to the NOIDA with reference to the allotment letter of 17.4.2003. Referring to
the said letter, learned counsel for the appellant submitted that it had been mentioned
therein that a modified allotment letter would be issued to the appellants along with a
statement of account of the balance amount payable on account of typographical
discrepancy in the allotment letter, but that neither had such modified letter been given to
the appellant nor had any statement of account been issued as promised. It was also
sought to be highlighted that in the letter it had been specifically mentioned that the
officials of NOIDA had refused to accept the payment on account of some internal
inquiry and/or procedural changes being effected by NOIDA.
7. It was urged that since no reply was received to the said letter no further payments
were made in terms of the allotment letter till the appellants received the letter written on
behalf of the NOIDA on 3.5.2005 asking the appellants to produce proof of deposit of the
allotment amount which was required to be deposited by 16.2.03. It was
@page-SC3105
urged that once the said letter was received, deposits were made on 6.12.2005 making up
a total sum of Rs. 3,80,20,000/- after giving credit for deposit of the initial amount of Rs.
13,20,000/-. It was reiterated by counsel that having accepted the aforesaid deposits, the
NOIDA was estopped from cancelling the allotment by its order dated 21.6.2006.
8. In support of his submission learned senior counsel referred to several decisions of this
Court regarding the manner in which public authorities should conduct themselves while
extending benefits to private individuals by way of contracts and agreements.
9

. Learned counsel firstly referred to the decision of this Court in R.K. Saxena v. Delhi
Development Authority (AIR 2002 SC 2340) where a similar set of facts were under
consideration. In the said case, after making the initial deposit of 25 per cent of the
auctioned price, the auction purchaser prayed for extension of time to deposit the balance
of 75 per cent which was required to be paid within 60 days from the date of issuance of
the demand letter. In the said case also the Chairman, Delhi Development Authority, was
vested with discretion to extend the time for such payments up to a maximum period of
180 days, subject to payment of interest on the balance amount @ 18 per cent per annum.
The demand letter for payment of the said amount was issued on 3.1.1996 but only a part
thereof was deposited on 19.2.1996 with a prayer for further extension to make the
balance payment. Such prayer was granted and further time was granted for the said
purpose. Pursuant to said extensions certain amounts were deposited towards the balance
75 per cent., but ultimately when on 2.9.1996 further extension was sought for there was
no reply to the letter though various sums deposited thereafter were accepted by the
Authority despite the fact that such deposits were made after the stipulated time. It was
also brought to the notice of the Court that the entire balance amount had since been paid
for the plot in question. Since, despite having accepted the delayed payment the plot was
not delivered to the appellant, legal notices were issued on its behalf and subsequent
thereto the allotment was cancelled and the earnest money was forfeited. The writ
petition filed in the High Court against said cancellation of allotment was dismissed on
29.2.2000 by the High Court which held that after the expiry of the period stipulated in
the agreement the allottee could not have deposited the balance amount unilaterally
without any demand being issued to him after the extended dates and no relief could be
given to the allottee. Learned counsel pointed out that when the said matter was carried to
this Court, this Court held that the order of the High Court could not be sustained
particularly when both the delayed payments and the interest amount thereupon were
accepted by the respondent-authority. This Court observed that the moment those
payments were accepted there was deemed extension of time and that it was only one and
half years after the legal notices had been sent to the Authority that the allotment order
was cancelled. This Court held in the facts of that case that after accepting the delayed
payment the respondent-authority could not have cancelled the allotment. 2002 AIR
SCW 2489
10. Reliance was also placed on the decision of this Court in Teri Oat Estates (P) Limited
v. U.T. Chandigarh and another [(2004) 2 SCC 130] where the concept of
disproportionate action was applied in a similar case where the lessee defaulted/delayed
in payment of instalments of premium, interest thereon and ground rent in terms of the
letter of allotment but it was found that the same had been occasioned due to a situation
beyond the control of the lessee and not on account of any wilful or dishonest intention
on the part of the lessee. Keeping in mind the principles of proportionality, this Court not
only held that the lessee/appellants therein had not only shown their bona fides in making
payments before the High Court but they had also shown their willingness to make
payment on the difference amount and pursuant to the orders passed by this Court had not
only paid the entire amount due, but had also paid the ground rent upto 1998-99 and 10
per cent penalty on the forfeited amount of the entire consideration money. While
allowing the appeals, this Court observed that the land in question for all intents and
purposes had been transferred in favour of the lessee who was merely required to pay the
balance amount of 75 per cent of the consideration amount in instalments. While also
deprecating the conduct of the lessees in not making an endeavour to pay the instalments
@page-SC3106
within a reasonable period, this Court in consonance with the doctrine of proportionality
observed that after the letter of allotment had been issued in favour of the lessee/appellant
it had been put in possession of the property and had raised a six-storeyed building on the
said land. It was also observed that it had paid a part of the first instalment and had
during the pendency of the proceeding before the High Court paid a substantial amount,
together with interest @ 12 per cent., per annum, as enhanced from time to time. This
Court was, therefore, of the view that the resumption of the plot by the Estate Officer was
too drastic and such power of resumption and forfeiture should be exercised only as a last
resort. Of course, it was also indicated that such an observation did not mean that the
power of resumption and forfeiture should never be resorted to if the intention of the
allottee was dishonest or with ill-motive or the payments in terms of the allotment were
made with a dishonest view or dishonest motive.
11. Learned counsel submitted that having regard to the aforesaid decision it must also be
held in this case that cancellation of the allotment six months after the entire balance
amount had been deposited could not be sustained and the High Court had erred in
dismissing the writ petition filed by the appellant company challenging the cancellation
of the allotment made in its favour.
12. The learned counsel appearing both for the State of U.P. and NOIDA supported the
decision of the High Court and submitted that since the appellant had failed to deposit
any amount, other than the initial deposit of Rs. 13,20,000/-, within the time stipulated in
the allotment order and had unilaterally deposited the balance amount 2½ years after the
allotment order was made and, that too, after a letter had been addressed to the appellant
asking for proof of deposit of the said amounts, it was not entitled to any relief. It was
urged on behalf of the NOIDA that the deposits said to have been made by the appellant
after receipt of the said letter, had been made unilaterally and had not been accepted by
the NOIDA. Accordingly, the appellant could not derive any benefit from the decisions
cited on its behalf since in all the said cases, the deposits, though made out of time, had
subsequently been accepted by the concerned authority.
13. It was also submitted that since third party interests had intervened and the plot had
since been allotted in favour of the respondent No. 5, the relief sought for by the
appellant in the writ petition could not be granted.
14. Similar submissions were made on behalf of the respondent No. 5, in whose favour
the plot in question had been allotted after the allotment in favour of the appellant was
cancelled.
15. It was submitted that the reason sought to be given on behalf of the appellant for non
payment ,of the premium amount was extremely dubious and had been rejected by the
NOIDA in its discretion. The decisions cited on behalf of the appellant could not be
applied to the facts of this case, since in the present case, the deposits subsequently made
by the appellant had not been accepted by the NOIDA. It was lastly urged that, in any
event, no relief could be granted in favour of the appellant, since no prayer had been
made in the writ petition for cancellation of the allotment made in favour of the
respondent No. 5.
16. Having considered the submissions made on behalf of the respective parties, we are
not inclined to interfere with the order of the High Court in the present appeal.
17. There is no dispute that the appellant did not make any deposits, other than the initial
deposit of Rs. 13,20,000/-, in terms of the allotment order. There is also no dispute that
the deposits ultimately made 2½ years after the allotment order had been passed, had
been made unilaterally and only after a communication was received from the NOIDA
asking for proof of deposits made and, that too, three months after receipt of such letter.
18. We are inclined to accept the submissions made on behalf of the respondents that the
reason given for not making the deposits, as per the allotment order, is not very
convincing. We are also inclined to accept the other submissions made on behalf of the
respondents that since the deposits subsequently made by the appellant had not been
accepted by the NOIDA, the ratio of the decisions cited on behalf of the appellant would
not apply to the facts of this case, particularly, when third party interests have intervened
and a fresh allotment order had been made in favour of the respondent No. 5
@page-SC3107
and no prayer has been made in the writ petition for setting aside such allotment.
19. We, therefore, have no option but to dismiss the appeal, but without any order as to
costs. The appellant will be entitled to withdraw the deposits made by it in favour of the
respondents towards the balance of the premium amount.
Appeal dismissed. .
AIR 2008 SUPREME COURT 3107 "Kanta Devi v. State of Haryana"
(From : Punjab and Haryana)*
Coram : 2 ALTAMAS KABIR AND MARKANDEY KATJU, JJ.
Civil Appeal Nos. 1330-1332 of 2003 with C.A. Nos. 4235, 4236, 4240 and 4239 etc. etc.
of 2008 (@ SLP (C) Nos. 9486, 9380 of 2003 and 18028 of 2001 and etc. etc), D/- 8 -7
-2008.
Kanta Devi and Ors. v. State of Haryana and Anr.
Land Acquisition Act (1 of 1894), S.23 - ACQUISITION OF LAND - AGRICULTURAL
LAND - Compensation - Determination - Deduction towards development charges -
Agricultural land acquired for setting up grain market - Acquired lands adjacent to village
abadi which is already developed - Deduction of 70% towards development charges is
excessive - Reduced to 60%. (Para 30)
Cases Referred : Chronological Paras
2007 AIR SCW 7144 : AIR 2008 SC 399 : 2008 (1) ALJ 223 (Ref.)16
2005 AIR SCW 2107 : AIR 2005 SC 2214 (Ref.) 26
2005 AIR SCW 4565 : AIR 2005 SC 3467 (Ref.) 19, 22
2004 AIR SCW 5534 : AIR 2004 SC 4830 : 2004 AIR Kant HCR 3036 (Ref.) 19
2003 AIR SCW 1491 : AIR 2003 SC 1987 (Ref.) 24
(2003) 10 SCC 167 (Ref.) 21
1996 AIR SCW 633 : AIR 1996 SC 2886 (Ref.) 26
1996 AIR SCW 1854 : AIR 1996 SC 3168 (Ref.) 26
AIR 1984 SC 892 24
AIR 1977 SC 1560 24
AIR 1971 SC 2015 24
A.K. Pandey, Ms. Abha R. Sharma, Dheerendra Singh, Susheel Kr. Tomar, Sudhanshu
and Balbir Singh Gupta, for Appellants; Nidesh Gupta, Sr. Advocate, Ms. Kavita Wadia,
Manjit Singh, Harikesh Singh, T.G. George and J.P. Dhanda, with him for Respondents.
* L.P.A. No. 1019 of 2000, C. M. No. 1526 of 2000 in L.P.A. No. 1019 of 2000 and L.P.
A. No. 1020 of 2000, D/- 16-8-2000 (PandH).
Judgement
ALTAMAS KABIR, J. :- Apart from Special Leave Petition (Civil) Nos. 9488, 9499 and
9531, all of 2003, leave is granted in respect of all the other Special Leave Petitions heard
along with Civil Appeal Nos. 1330-1332 of 2003.
2. These appeals, have their genesis in a common award dated 14th March, 1989, made
by the Land Acquisition Collector, Kurukshetra, whereby he awarded compensation in
respect of the acquired lands at the rate of Rs. 60,000/- per acre for land in the form of
Chahi, Gair Mumkin Tubewell, etc. and at the rate of Rs. 40,000/- per acre in respect of
Gair Mumkin Talab Land and Rasta Land.
3. Twenty seven References were made to the Land Acquisition Judge, Kurukshetra,
under Section 18 of the Land Acquisition Act, 1894 and the same were disposed of by an
Award dated 2nd January, 1993.
4. Being dissatisfied as to the extent of compensation awarded, the claimants filed
Regular First Appeals before the High Court. Similarly, being aggrieved by the
enhancement of the market value of the lands, the State also preferred 27 Regular First
Appeals. In all, 51 Regular First Appeals, arising from the common Notification, Award,
and Judgment of the Land Acquisition Judge, were taken up for hearing together and
were disposed of by the learned Single Judge of the High Court by a common Judgment
dated 10th August, 1999.
5. Aggrieved by the judgment of the learned Single Judge, the appellants herein filed
Letters Patent Appeals (L.P.As) before the Division Bench of the High Court which were
mostly dismissed in limine or on account of technicalities. All the appellants, however,
are similarly circumstanced and having the same grievance. This batch of appeals have
been preferred against the said judgment and order of the Division Bench of the High
Court in the various Letters Patent Appeals.
6. Applications have been filed in SLP(C) Nos. 9488/2003, 9499/2003 and 9531/ 2003
for substitution and setting aside abatement and also permission to file Special
@page-SC3108
Leave Petition. As they all arise out of the said common judgment of the High Court, they
are allowed and leave is also granted in the connected Special Leave Petitions. As all the
appeals relate to the same Notification under Section 4 of the Land Acquisition Act, and
arise out of the same Award, they were taken up for hearing and final disposal together.
7. Coming back to the facts of the case, it may be indicated that on 12th June, 1986, the
State of Haryana issued a Notification under Section 4 of the Land Acquisition Act, 1894
(hereinafter, referred to as the "LA Act") for the acquisition of 265 kanals and 19 Marias
of land in the revenue estate of village Ismailabad, District Kurukshetra for the
establishment of a new grain market, construction of rest house, staff quarters and other
connected purposes of the Market Committee, Ismailabad. Notification under Section 6
was thereafter, issued on 9th June, 1987 and the Collector, Kurukshetra, by his Award
dated 14th March, 1989 awarded compensation for 250 kanals and 17 marlas, comprising
the first category of land referred to above, at the rate of Rs. 60,000/- per acre. For 15
kanals and 2 marlas of land comprising the second category of lands, the compensation
was awarded at the rate of Rs. 40,000/- per acre. Solatium and interest were also awarded
in terms of the provisions of the LA Act.
8. As indicated earlier, twenty seven References were made under Section 18 of the LA
Act to the District Judge, Kurukshetra. All the said References were heard together and
by his award dated 2nd January, 1993, the District Judge, Kurukshetra enhanced the
compensation in respect of the first category of lands to Rs. 1,28,000/- per acre and in
respect of the second category of lands to Rs. 80,000/- per acre.
9. In the Regular First Appeals filed both by the claimants and the State of Haryana, the
High Court by a common Judgment dated 10th August, 1999, enhanced the compensation
for the acquired lands to Rs. 2,88,000/- per acre and it was indicated that all the claimants
were entitled to uniform compensation as the lands had been acquired for a common
purpose. Consequently, the appeals preferred by the State of Haryana were dismissed and
those of the claimants were allowed in part.
10. It may be relevant to mention at this stage that in enhancing the compensation
payable to the claimants-appellants herein, the learned Single Judge took one exemplar
(Ex. P-6) for the purpose of comparison visa-vis the land acquired. Ex. P-6 is a sale deed,
whereby on 30th January, 1986, a plot of land measuring 148 square yards was sold for
Rs. 24,000/-, whereas the notification under Section 4 was issued six months after on
12th June, 1986. On the basis of the above, the price of the land in question works out to
Rs. 9,60,000/- per acre. The learned Single Judge, while accepting, the aforesaid
valuation, directed deduction of 70% of the value of the lands towards development
charges to make the acquired land suitable for the purpose for which it had been acquired
and also having regard to the nature of the lands on the date of publication of the
notification under Section 4 of the LA Act. On the basis of such deduction, the learned
Single Judge uniformly enhanced the compensation in respect of the lands acquired to Rs.
2,88,000/- per acre. As indicated hereinbefore, most of the Letters Patent Appeals filed by
the appellants were dismissed by the Division Bench in limine or on technical grounds.
11. These appeals have been preferred by the claimants where Letters Patent Appeals
were disposed of by cryptic orders although they were aggrieved by the rate of deduction
applied by the learned Single Judge of the High Court while disposing of the Regular
First Appeals preferred by the appellants as also the respondents.
12. On behalf of the appellants, it was contended that the rate of deduction as applied by
the learned Single Judge was highly excessive as the acquired lands were situated in an
area which was already developed. It was submitted that the acquired lands were situated
at Ambala Pehowa Road in Village Ismailabad and were adjacent to the Village abadi
where there were houses and shelters, power house, telephone exchange and a factory.
13. It was also submitted that since the acquired lands were reserved for commercial and
residential purposes, the claimants had demanded compensation at the rate of Rs.
15,000/- per marla from the Collector and in support thereof Sale Deeds in respect of
lands adjacent to the acquired land had been placed on record to show that the valuation
of the said lands were between Rs. 8,000/- to Rs. 9,000/- per marla.
14. It was submitted that this Court had
@page-SC3109
repeatedly held that in assessing the compensation payable in respect of agricultural land
or undeveloped land which had potential value for housing or commercial purposes,
normally 1/3 of the assessed value of the land is deducted depending on the nature of the
land, its location, extent of expenditure involved for development, and the land required
for roads and other civic amenities to make the land suitable for residential or commercial
purposes. However, in the instant case, despite the location of the acquired land and its
potential value, the compensation payable to the claimants was reduced drastically
without proper reason for such drastic deduction.
15. In addition to the above, it was submitted that both the learned District Judge and the
learned Single Judge of the High Court had erred in holding that the sale deed in respect
of a small plot of land was not a proper indicator for the purpose of determining the value
of a large tract of land. It was urged that even with regard to the said question, this Court
had consistently indicated that such sale deeds or such exemplars should not be discarded
in limine, but were to be taken into consideration while fixing the value of the lands
acquired.
16

. In support of the aforesaid submissions reliance was placed on the decision of this Court
in Lucknow Development Authority vs. Krishna Gopal Lahoti and Ors. [2007 (12) Scale
685] where deduction for development charges at the rate of 1/3 of the amount of
compensation was accepted to be normal. However, it was also indicated that there may
be various factors which were required to be taken into consideration while deciding the
amount of deduction to be made towards developmental charges. While in some cases, it
could be more than 1/3, in other cases it could be less, having regard to the difference
between a developed area or an area having potential value which is yet to be developed.
2007 AIR SCW 7144

17. In the same decision, while observing that where a large area is the subject matter of
acquisition the rate at which small plots are sold cannot be said to be a safe criteria, it was
also observed that it could not be laid down as an absolute proposition that the rates fixed
for small plots could not be the basis for fixation of the value of the acquired land.
However, in such cases necessary deduction/adjustments have to be made while
determining the value and in the said context it was held that a deduction of 1/3 of the
compensation amount was considered to be normal.
18. It was also sought to be urged that apart from Exh. P. 6 on which reliance had been
placed by the High Court certain other exemplars were also produced on behalf of the
claimants which were not relied upon on the ground that they had not been properly
proved. It was submitted that with the incorporation of Section 51-A in the LA Act by
way of amendment the degree of proof had been altered and although the previous legal
position was that all sale deeds on which reliance was placed by the parties were required
to be proved, after the amendment such proof was not strictly required and the various
foras up to the stage of Regular First Appeal could rely on such documents, which
included certified copies, as exemplars without having to prove the same. It was urged
that having regard to the provisions of Section 51-A of the LA Act the different foras,
including the High Court, had erred in not placing reliance on all sale deeds that had been
produced on behalf of the claimants in assessing the amount of compensation payable in
respect of the acquired land.
19

. In support of the aforesaid submission reliance was placed on a Constitution Bench


decision of this Court in Cement Corporation of India Ltd. vs. Purya and Ors. (2004) 8
SCC 270 and Ranvir Singh vs. Union of India (2005) 12 SCC 59, which supported such
contention. 2004 AIR SCW 5534
2005 AIR SCW 4565

20. Since the grievance of the appellants was only with regard to the rate of deduction on
account of the developmental charges and an attempt was made on behalf of the
appellants to assert that 70% deduction was unwarranted as the lands sought to be
acquired were already within or adjacent to a developed area, on behalf of the State-
respondent such deduction was sought to be justified.
21. It was submitted on behalf of the State-respondent that the observations made in the
Lucknow Development Authority case was more by way of caution than laying down the
general law which finds consistent expression in various other decisions of this Court,
such as Union of India vs. Ram Phool and another, (2003) 10 SCC 167 in which it had
been held that an isolated deed
@page-SC3110
of sale showing a very high price cannot be the sole basis for determining the market
value.
22

. It was submitted that the said view was reiterated in the case of Ranvir Singh (supra),
which, in fact, had been relied upon by the appellants in relation to the submissions made
with regard to Section 51-A of the LA Act. 2005 AIR SCW 4565

23. It was urged that this Court has consistently held that small tracts of land purchased
for a particular purpose may fetch fancy prices in terms of its location and the need for
acquisition by the vendee, but the same basis could not be applied to each tract of land
which were yet to be developed for public purposes such as housing or setting apart an
area for a particular purpose such as education and/or industrialization. It was submitted
that in such cases there could be no comparison with regard to the value of the lands
covered by the sale deed and those proposed to be acquired, and that the sale price of
such a small tract of land was not a safe basis for determining the value of a very large
tract of land using the comparative method.
24

. It was, however, fairly submitted that in Ravinder Narain vs. Union of India (2003) 4
SCC 481 it has been observed in paragraphs 6 and 7 as follows : 2003 AIR SCW 1491

"6. Where large area is the subject-matter of acquisition, rate at which small plots are sold
cannot be said to be a safe criterion. Reference in this context may be made to three
decisions of this Court in Collector of Lakhimpur v. Bhuban Chandra Dutta (1972) 4
SCC 236, Prithvi Raj Taneja v. State of M.P. (1977) 1 SCC 684 and Kausalya Devi Bogra
v. Land Acquisition Officer, Aurangabad (1984) 2 SCC 324. AIR 1971 SC 2015
AIR 1977 SC 1560
AIR 1984 SC 892

7. It cannot, however, be laid down as an absolute proposition that the rates fixed for the
small plots cannot be the basis for fixation of the rate. For example, where there is no
other material, it may in appropriate cases be open to the adjudicating court to make
comparison of the prices paid for small plots of land. However, in such cases necessary
deductions/adjustments have to be made while determining the prices."
25. It was also submitted that in the instant case excluding all the other exemplars, the
High Court had chosen to rely on Ex. P.6, where a small tract of land (148 sq. yards) had
been sold at the rate of Rs. 9,60,000/- per acre and the compensation had been worked
out on such basis after applying deduction of 70% of the market value towards
developmental charges, since the lands acquired were agricultural and huge investment
was required to be made by the State to make the same suitable for the purpose for which
they had been acquired, namely, the setting up of a new grain market with all the
ancillary infrastructure needed by the Market Committee, Ismailabad.
26

. It was submitted that the deduction of 70%, which had been applied by the High Court,
was quite reasonable as the sale deed relied upon by the appellants related to lands sold
for shops etc. and Ex. P.6 and other sale instances had been relied upon by the appellants
for smaller areas. It was urged that in Viluben Jhalejar Contractor (Dead) by LRs. vs.
State of Gujarat, (2005) 4 SCC 789, this Court had held that there can be different
deductions depending upon various factors. It was submitted that in various other
decisions and in particular in K. S. Shivadevamma vs. Assistant Commissioner of Land
Acquisition Officer, (1996) 2 SCC 62, it was held that although as a general rule 33½ per
cent is required to be deducted for laying of roads and other amenities, deduction to the
extent of 53% was not improper and the extent of deduction depends upon the
development need in each case. In Vasawa (Smt.) and others vs. Special Land Acquisition
Officer and others, (1996) 9 SCC 640, this Court upheld a deduction of 65%. 2005
AIR SCW 2107
1996 AIR SCW 633
1996 AIR SCW 1854

27. As an alternative argument it was urged on behalf of the State-respondent that since
the High Court had relied only on Ex. P.6 which related to the sale of only marlas of land,
the matter could be remanded to the High Court for consideration of all the various sale
deeds which were produced on behalf of the parties, to arrive at a fresh valuation for the
acquired lands.
28. It was submitted that in view of the above the submissions made on behalf the
claimants under Section 51-A of the LA Act was not relevant for determination of the
point raised in these appeals.
@page-SC3111
29. Having carefully considered the submissions made on behalf of the respective parties
we see no reason to interfere with the decision of the High Court.
30. The learned Single Judge of the High Court has taken into consideration the nature of
the land sought to be acquired in relying on Ex. P.6 in assessing the market value thereof
and has applied a deduction of 70% in arriving at the compensation to be awarded to the
claimants in respect of the said lands. The various other documents which were produced
on behalf of the claimants were in respect of the lands which were similar to the lands
forming the subject matter of Ex. P.6. The learned Single Judge has given reasons for not
relying on all the other exemplars in choosing to rely on Ex.P.6 alone. But the rate of
deduction applied appears to be on the high side in relation to the developmental work
involved in making the acquired land suitable for the purposes for which they were so
acquired. The acquired lands are adjacent to the village abadi which is already developed.
Having regard to the consistent view that a deduction of 1/3rd of the market value is
normal, though a higher deduction is permissible, we are of the view that deduction of
60% would meet the expenditure towards developmental charges considering the
proximity of the acquired lands to the areas already developed.
31. In our view, the Division Bench of the High Court while dismissing the Letters Patent
Appeal filed by the claimants could have given proper reasons before dismissing the
same in limine. However, since the decision of the Division Bench endorses that of the
learned single Judge, with which we have dealt with in detail, and with which we agree,
save for the amount of deduction applied towards developmental charges, these appeals
against the decision of the Division Bench in dismissing the appeal filed by the appellant
in C.A. Nos. 1330-1332 of 2003, and all the other connected appeals have to be allowed
in part. As far as the alternative submissions made on behalf of the State regarding
remand of these appeals to the High Court is concerned, we are not inclined to accept the
same, since we are not convinced that such a course of action needs to be adopted.
32. The appeals are accordingly allowed in part only to the extent that the deduction of
70% applied by the learned Single Judge and endorsed by the High Court is reduced to
60%.
33. Having regard to the facts of the case there will be no order as to costs.
Appeal partly allowed. .
AIR 2008 SUPREME COURT 3111 "Rahimbux v. State of M.P."
(From : Madhya Pradesh)*
Coram : 2 A. K. MATHUR AND ALTAMAS KABIR, JJ.
Criminal Appeal No. 861 of 2008 (arising out of SLP (Cri.) No. 2154 of 2007), D/- 12 -5
-2008.
Rahimbux v. State of M.P.
Penal Code (45 of 1860), S.300, S.304, Part I, Part II - MURDER - CULPABLE
HOMICIDE - Murder - Accused quarrelling with his father-in-law - 'A' intervened and
tried to dissude him - Accused got annoyed and inflicted injury on 'A' by piece of brick -
Brought sword from house and chased 'A' but failed to catch hold of him - Inflicted
abdominal injury by sword on deceased, brother of 'A', an innocent by-stander - Intention
of accused is more than apparent - It is only diversion of malice from one brother to
another brother - Thus, accused would not be entitled to benefit of S.304 Part I or S.304
Part II - His conviction for murder, was, therefore, proper. (Paras 5, 6)
Cases Referred : Chronological Paras
2004 AIR SCW 849 : AIR 2004 SC 1603 : 2004 Cri LJ 1405 : 2004 AIR Jhar HCR 1047
(Disting.) 4
2004 AIR SCW 852 : AIR 2004 SC 1605 : 2004 Cri LJ 1407 : 2004 All LJ 733 (Disting.)
4
1995 Supp (3) SCC 472 (Disting.) 4
AIR 1983 SC 463 : 1983 Cri LJ 852 (Disting.) 4
AIR 1982 SC 55 : 1982 Cri LJ 195 (Disting.) 4
AIR 1982 SC 126 (Disting.) 4
AIR 1981 SC 1441 : 1981 Cri LJ 1033 (Disting.) 4
AIR 1958 SC 465 : 1958 Cri LJ 818 (Disting.) 4
Rachana Joshi Issar, Ms. Rupal Bhatia and Ajay Thakur, for Appellant; Siddartha Dave,
Ms. Jemtiben AO, and Ms. Vibha Datta Makhija, for Respondent.
* Cri. Appeal No. 49 of 2000, D/- 2-9-2005 (MP).
Judgement
A. K. MATHUR, J. :- Leave granted.
@page-SC3112
2. This appeal is directed against the order dated 2-9-2005 passed by the Division Bench
of the Madhya Pradesh High Court whereby the Division Bench of the High Court has
affirmed the conviction of the accused appellant under Section 302 of the Indian Penal
Code and under Section 323 of the Indian Penal Code (hereinafter to be referred to as 'I.
P. C.') read with Section 25 of the Arms Act and sentence imprisonment for life and
payment of fine of Rs. 500/ - in default to suffer further imprisonment for a period of two
months under Section 302 of the I. P. C., and simple imprisonment for a period of three
months under Section 323, I. P. C. and Section 25 of the Arms Act on each count and
directed that both the sentences to run concurrently. Aggrieved against this order the
present appeal was filed by the accused. Notice was given on the limited question of the
offence.
3. Brief facts which are necessary for disposal of this appeal are that Aslam Khan, P. W. 1
lodged a report to the effect that on 8-6-1998 at about 4.00 P. M. the appellant was
quarrelling with his father-in-law. Therefore, he tried to intervene by persuading the
appellant not to beat the old person. Being annoyed, the appellant inflicted injury on him
by piece of brick. After receiving this injury, P. W. 1 ran away from that place. Thereafter,
the appellant went to his house and came out with a sword and chased him to cut him
with the sword but somehow he escaped and did not come within his reach. His brother,
Rehman Khan who was standing in front of the house was attacked by the appellant with
the sword which caused abdominal injury with intestine coming out of the wound. The
deceased Rehman Khan was immediately shifted to the Hospital along with Aslam Khan.
Dr. M. P. Garg, on examination found an incised wound in the abdominal region of
Rehman Khan and the doctor immediately shifted him to the operation theatre for
treatment by surgical specialist. Aslam Khan was also treated for his injuries sustained by
the brick. Subsequently, Rehman Khan succumbed to his injuries. A panchnama was
made and the dead body was sent for postmortem. It was found that the deceased had
incised wound ½" x 3/4" cavity deep on abdominal region near umbilical cord.
Subsequently, during the investigation the sword in which the deceased was attacked was
recovered. After completion of the investigation, challan was filed against the accused.
Learned Sessions Judge after conclusion of the trial convicted the accused-appellant
under Section 302, I. P. C., Section 323, I. P. C. and Section 25 of the Arms Act and
sentenced him as aforesaid. Thereafter, an appeal was preferred before the High Court.
The Division Bench of the High Court affirmed the conviction of the accused appellant.
Hence, the present appeal. The limited question to be examined is with regard to the
nature of offence.
4. Learned counsel for the appellant strenuously urged before us that there is only one
injury caused to the deceased. Therefore, it does not fall Under Section 302, I. P. C. but at
best it would fall under Section 304-I or Section 304-11 of the I. P. C. as the accused did
not inflict any second injury and secondly, the accused did not intend to cause any injury
to the deceased so as to cause his death. The accused in fact wanted to attack Aslam Khan
but since Aslam Khan was out of reach, therefore, he inflicted the injury to his brother,
Rehman Khan who was standing there. The accused appellant did not intend to cause
death to the deceased and as such the case does not fall within the parameters of Section
302, I. P. C. and at best it can be under Section 304-II, I. P. C. In support of this
contention, learned counsel cited the following decisions of this Court.

(i) (1981) 4 SCC 245, Kulwant Rai v. State of Punjab AIR 1982 SC 126

(ii) (1981) 4 SCC 489, Randhir Singh alias Dhire v. State of Punjab. AIR 1982 SC
55

(iii) (1983) 2 SCC 342, Jagtar Singh v. State of Punjab. AIR 1983 SC 463

(iv) (1981) 3 SCR 658, Gokul Parashram Patil v. State of Maharashtra. AIR 1981 SC
1441

(v) 2004 (2) SCALE 217, Chowa Mandal and Anr. v. State of Bihar (Now Jharkhand)
2004 AIR SCW 849
(vi) (2004) 12 SCC 250, Ramu v. State of U. P. 2004 AIR SCW 852

(vii) (1995) Supp. (3) SCC 472, Balbir Singh v. State of Punjab.

(viii) (1958) SCR 1495, Virsa Singh v. The State of Punjab. AIR 1958 SC 465

Learned counsel for the appellant has tried to seek support on the aforesaid decisions to
show that in fact the accused in the
@page-SC3113
present case did not intend to cause fatal injury to the deceased and it was only a single
blow not intended to cause the death but intended to attack the other brother, Aslam Khan
who was not out of reach. As against this, learned counsel for the State supported the
judgment of the High Court.
5. We have bestowed our best of consideration to the facts of the present case. There is no
two opinion that the present injury was inflicted on the deceased as supported by P. W. 1
and the testimony of P. Ws. 2 and 3 and the medical evidence corroborated by recovery of
sword. Therefore, so far as the voluntarily causing injury to the deceased by the accused
is concerned, there is no two opinion and the same has been fully substantiated by the
relevant evidence. In fact, the only question is whether offence under Section 302, I. P. C.
is made out of Section 304-I, or 304-II I. P. C. is made out. It is a fact that the accused
was chasing Aslam Khan and when he was cut of his reach, the accused went to his house
and brought out a sword to attack Aslam Khan out somehow, Aslam Khan managed to
escape from the spot. But unfortunately, his brother the deceased was standing in front of
his house and the accused on his failure to cause the serious harm to Aslam Khan gave
the murderous blow to the deceased which ultimately became fatal. So far as the intention
of the accused is concerned, it is more than apparent that he went to his house and
brought a sword and wanted to chase Aslam Khan. Therefore, the intention of the accused
is apparent, he intended to inflict serious injury to Aslam Khan but unfortunately on his
failure he made the deceased a victim. Therefore, from this his intention is apparent.
Nobody, chase person with sword for any benevolent purpose. It is unfortunate, instead of
causing the death of Aslam Khan the accused caused the death of his brother. It is nothing
but change of malice from one brother to another brother. From these facts we are of
opinion that it is not a case in which the benefit of Section 304-I, I. P. C. or Section 304-
II, I. P. C. can be given to the accused. One has to see the intention and the intention of
the accused in the present case is more than apparent that he chased Aslam Khan with
sword and on his failure to catch hold of Aslam Khan with sword he caused the death of
an innocent bystander who has not given any provocation to the accused for the
murderous act. It is the intention which was predominantly present in his mind when the
accused chased Aslam Khan and therefore, this intention he satisfied by inflicting the
murderous blow on the deceased on his vital part of body. Therefore, under these
circumstances, we are of opinion that the conviction of the appellant under Section 302, I.
P. C. is well founded and there is no ground to interfere in this appeal.
6. Learned counsel for the appellant has invited our attention to the various decisions
which have been cited above. Each case depends on its facts and therefore, the decisions
cited above will not render any assistance to learned counsel for the appellant. So far as
converting the case from Section 302, I. P. C. to Section 304-I or 304-II, I. P. C. is
concerned, each case has its peculiar facts and justification for converting the offence
from Section 302, I. P. C. to Section 304-I or Section 304-II, I. P. C. However, so far as
the present case is concerned, we are satisfied that the intention of the accused is more
than apparent when he chased Aslam Khan after taking out a sword from his house and
since he could not succeed to cause injury on Aslam Khan he diverted his malice on his
brother an innocent person who was present and gave no cause to this accused. Therefore,
under these circumstances, we do not find any merit to convert the offence under Section
302 I. P. C. to either Section 304-I, I. P. C. or Section 304-II, I. P. C. Consequently, there
is no merit in the present appeal and the same is dismissed.
Appeal dismissed. .
AIR 2008 SUPREME COURT 3113 "Hardeep Singh v. State of Haryana"
(From : Punjab and Haryana)*
Coram : 2 Dr. A. PASAYAT AND PRAKASH PRABHAKAR NAOLEKAR, JJ.
Criminal Appeal No. 468 of 2007, D/- 11 -5 -2008.
Hardeep Singh and Ors. v. State of Haryana.
(A) Penal Code (45 of 1860), S.34 - COMMON INTENTION - APPLICABILITY OF
AN ACT - MURDER - Applicability - Murder - Common intention - S.34 is applicable
even if no injury has been caused by particular accused himself.
S. 34 does not say "the common intention of all", nor does it say "and intention
@page-SC3114
common to all". Under the provisions of S. 34 the essence of the liability is to be found in
the existence of a common intention animating the accused leading to the doing of a
criminal act in furtherance of such intention. As a result of the application of principles
enunciated in S. 34, when an accused is convicted under S. 302 read with S. 34, in law it
means that the accused is liable for the act which caused death of the deceased in the
same manner as if it was done by him alone. The provision is intended to meet a case in
which it may be difficult to distinguish between acts of individual members of a party
who act in furtherance of the common intention of all or to prove exactly what part was
taken by each of them. S. 34 is applicable even if no injury has been caused by the
particular accused himself. For applying S. 34 it is not necessary to show some overt act
on the part of the accused. (Para 15)
(B) Penal Code (45 of 1860), S.300, S.304, Part II - MURDER - CULPABLE
HOMICIDE - EVIDENCE - WITNESS - Murder - Evidence of eyewitnesses clearly
established role played by accused 'HP' and 'HJ' - On exhortation of acquitted accused 'J'
accused 'HJ' gave blows on legs of deceased - After he fell down accused 'HP' and 'HJ'
gave blows on head - Thereafter 'J' assaulted on arm of deceased and not on vital part of
body of deceased - In circumstances conviction of accused 'HJ' and 'HP' u/S.302 read
with S.34 was proper - Conviction of 'J' altered to u/S.304 Part II. (Para 16)
Cases Referred : Chronological Paras
(2007) 1 SCC 699 (Ref.) 13
2004 AIR SCW 7376 : AIR 2005 SC 1460 (Ref.) 13
2002 AIR SCW 4271 : AIR 2002 SC 3633 : 2003 Cri LJ 41 (Ref.) 12
(2002) 3 SCC 76 (Ref.) 12
1993 AIR SCW 1843 : AIR 1993 SC 1899 : 1993 Cri LJ 2246 (Ref.) 15
AIR 1977 SC 109 : 1977 Cri LJ 164 (Ref.) 14
AIR 1974 SC 276 : 1974 Cri LJ 331 (Ref.) 9
AIR 1973 SC 2407 : 1973 Cri LJ 1589 (Ref.) 12
AIR 1965 SC 202 : 1965 (1) Cri LJ 226 (Ref.) 11
AIR 1957 SC 614 : 1957 Cri LJ 1000 (Ref.) 9
AIR 1953 SC 364 : 1953 Cri LJ 1465 (Ref.) 8, 10
AIR 1952 SC 54 : 1952 Cri LJ 547 (Ref.) 10
Ms. Kumud Lata Das, for Appellant; T.V. George, for Respondent.
* Cri. Appeal No. 198-DBA of 1997, D/- 6-1-2006 (P and H).
Judgement
Dr. ARIJIT PASAYAT, J. :- Challenge in this appeal is to the judgment of a Division
Bench of the Punjab and Haryana High Court dismissing the appeal filed by the
appellants. By a common judgment two appeals and one criminal Revision were disposed
of. Four persons faced trial for alleged commission of offence punishable under Section
302 read with Section 34 of the Indian Penal Code, 1860 (in short the 'IPC'). One of them
i.e. Gurcharan Singh was acquitted by the learned Sessions Judge, Sirsa. State questioned
his acquittal. Similarly the complainant, PW7 also filed the revision petition seeking
enhancement of the sentence of convicted accused persons to death sentence, apart from
questioning of acquittal of Gurcharan Singh. By the common judgment the High Court
dismissed the Criminal Appeal filed by the State and the Criminal Revision filed by the
complainant while dismissing the appeal filed by the appellants also.
2. Background facts in a nutshell are as follows :
Sukhdev Singh (PW-7) lodged FIR at 5.30 PM on 3.11.1994 that on 3.11.1994 at about
4.30 PM, he was going with his mother Pritam Kaur to visit the house of his father's sister
on the eve of Diwali festival. At that time, his father Amrik Singh (hereinafter referred to
as 'deceased') was going about 10 paces ahead of them on the same foot way for his
domestic work. As soon as deceased reached in front of the house of one Parlhand Singh
son of Karnail Singh, then all of a sudden, acquitted accused Gurcharan Singh, armed
with gandasi, Hardeep Singh, armed with dattar, Harjinder Singh armed with gandasi and
Jaswinder Singh also armed with gandasi, emerged from a street known as schoolwali
gali. Acquitted accused Gurcharan Singh raised a lalkara (exhortation) that "Aaj Isko
Bach Kar Jane Mat Dena". Soon thereafter, his sons appellants Harjinder Singh and
Jaswinder Singh inflicted gandasi injuries on the deceased with an intention to commit
his murder. They gave gandasi blows, which landed on the legs of the deceased. As a
result, the deceased fell down on the road. His turban also went off his head and fell on
the ground. Thereafter accused ap
@page-SC3115
Hardeep Singh gave dattar blow, hitting the deceased on his head. He was followed by
accused appellant Harjinder Singh, who also inflicted a gandasi blow on the head of the
deceased. Thereafter, acquitted respondent Gurcharan Singh and accused appellant
Jaswinder Singh caused further injuries with gandasi on the arms of the deceased.
Complainant Sukhdev Singh and his mother Pritam. Kaur raised a hue and cry saying
"Mar Dia Mar Dia". After that the deceased was dragged by all the four accused inside
the house of accused-appellant Harjinder Singh. Complainant Sukhdev Singh and his
mother Pritam Kaur followed them and raised hue and cry. On hearing their noise all the
aforesaid four accused, namely, Gurcharan Singh, Hardeep Singh, Harjinder Singh and
Jaswinder Singh, ran away with their weapons towards the village side. The complainant
and his mother looked at the deceased who had succumbed to the injuries. The
complainant, leaving behind his mother near the dead body at the spot, went to the police
post to lodge a report. His statement was recorded by Kartar Singh, Incharge, Police Post,
Kariwala, on 3.11.1994 itself. On receipt of ruqa (Ex.PB), a formal FIR (Ex.PB/1) under
Sections 302/34 IPC was registered by ASI Baljit Singh (PW3) at Police Station, Ding, at
6.50 PM. He sent a special report through Constable Bhoop Singh (PW-5) on the same
day to JMIC, Sirsa. A detailed inquest report (Ex. PD/4) was prepared on 3.11.1994 itself
by ASI Kartar Singh (PW 9). Dr. Narinder Chaudhary (PW 4) conducted the post mortem
(Ex. PD) of the deceased on 4.11.1994 at about 10.15 AM. He found as many as 4
injuries on the dead body. They are as under:
"1. A 'V' shape incised wound each limb measuring 8 cms x 2cms x brain deep over the
left parietal region 8 cms away from the pinna of the left ear. On dissection, there was
fracture of the left parietal bone. Infiltration was present underlying the injury.
2. An incised wound of 6 cms x 2 cms x scalp deep over the top of the scalp. On
dissection, infiltration was present and there was no fracture.
3. An incised wound of 8 cms x 1.5 cm x muscle deep, anterior and middle third to the
right leg. Corresponding cut was present in the kachha. On dissection, underlying bone
(tibia) was fractured and infiltration was present.
4. An incised wound of 4 cms x 1.5 cms x bone deep. Lateral aspect of the right knee
joint.
5. Two abrasions contusions measuring 3 x 2 cms, 2.5 cms x 2.5 cms over the anterior
aspect of right knee joint. On dissection of injuries Nos. 4 and 5 right patelar bone was
fractured. Infiltration was present.
6. Incised wound 11 cms in length, anterior posteriorly (through and through) from the
root of the right little finger to the root of the right thumb. Underlying bone, muscle
tendons vessels and nerves were cut.
7. Incised wound of 6 cms x 1.5 cms muscular deep over the posterior and middle to the
right fore-arm. Underlying bones were fractured. Infiltration was present.
8. Contusion of 3 cms x 2 cms over the top of the left shoulder joint. Infiltration was
present underlying the tissue.
9. Two contusions varying in size. Posterior to the left elbow.
10. Contusion of 4 cms x 2 cms, posterior and middle to the left fore-arm. Underlying
bone (ulna) was fractured and infiltration was present.
11. Abrasion contusions, four in number, varying in size, dorsum of the left hand. On
dissection infiltration was present.
12. Contusion of 3 cms x 1 cm on the left knee joint.
13. Incised wound of 5.5 cms x 1.5 cms x bone deep, anterior and middle third to the left
leg. Underlying bone was fractured. Infiltration was present.
14. Contusion of 2.5 cms x 2 cms over the lower third to the sternum. Infiltration was
present under the injury."
3. After completion of the investigation, charge sheet was filed. Since the accused
persons pleaded innocence, the trial was held. As noted above, the appellants were
convicted while Gurcharan Singh was acquitted. Appeal was filed by the State and the
present appellants and the revision was filed by the complainant.
4. Before the High Court the basic stand taken by the appellants was that the evidence of
PWs 7 and 8 do not inspire confidence. The High Court did not find any substance in the
appeal filed by the appellants and held that the evidence of PWs 7 and 8 sufficiently
established the accusations.
@page-SC3116
5. In support of the appeal, learned counsel for the appellant submitted that the evidence
of PWs 7 and 8 does not inspire confidence, particularly when they are relatives of the
deceased. Additionally, even if the prosecution version is accepted in toto, the conviction
for offence punishable under Section 302 read with Section 34 IPC cannot be maintained.
Plea that the FIR was ante timed was also taken with reference to the time of inquest. The
basic plea is regarding the applicability of Section 34 IPC. It is pointed out that the
Doctor has opined that only injury no. 1 alone could have resulted in the death.
6. Though learned counsel for the appellants submitted that the evidence of PWs 7 and 8
does not inspire confidence, no discrepancy in their evidence could be focused to discard
their evidence.
7. We shall also deal with the contention regarding interestedness of the witnesses for
furthering prosecution version. Relationship is not a factor to affect credibility of a
witness. It is more often than not that a relation would not conceal actual culprit and
make allegations against an innocent person. Foundation has to be laid if plea of false
implication is made. In such cases, the court has to adopt a careful approach and analyse
evidence to find out whether it is cogent and credible.
8

. In Dalip Singh and Ors. v. The State of Punjab (AIR 1953 SC 364) it has been laid down
as under :- (Para 26)

"A witness is normally to be considered independent unless he or she springs from


sources which are likely to be tainted and that usually means unless the witness has
cause, such as enmity against the accused, to wish to implicate him falsely. Ordinarily a
close relation would be the last to screen the real culprit and falsely implicate an innocent
person. It is true, when feelings run high and there is personal cause for enmity, that there
is a tendency to drag in an innocent person against whom a witness has a grudge along
with the guilty, but foundation must be laid for such a criticism and the mere fact of
relationship far from being a foundation is often a sure guarantee of truth. However, we
are not attempting any sweeping generalization. Each case must be judged on its own
facts. Our observations are only made to combat what is so often put forward in cases
before us as a general rule of prudence. There is no such general rule. Each case must be
limited to and be governed by its own facts."
9

. The above decision has since been followed in Guli Chand and Ors. v. State of
Rajasthan (1974 (3) SCC 698) in which Vadivelu Thevar v. State of Madras (AIR 1957
SC 614) was also relied upon. AIR 1974 SC 276

10

. We may also observe that the ground that the witness being a close relative and
consequently being a partisan witness, should not be relied upon, has no substance. This
theory was repelled by this Court as early as in Dalip Singh's case (supra) in which
surprise was expressed over the impression which prevailed in the minds of the Members
of the Bar that relatives were not independent witnesses. Speaking through Vivian Bose,
J. it was observed : AIR 1953 SC 364
(Para 25)

"We are unable to agree with the learned Judges of the High Court that the testimony of
the two eyewitnesses requires corroboration. If the foundation for such an observation is
based on the fact that the witnesses are women and that the fate of seven men hangs on
their testimony, we know of no such rule. If it is grounded on the reason that they are
closely related to the deceased we are unable to concur. This is a fallacy common to many
criminal cases and one which another Bench of this Court endeavoured to dispel in
'Rameshwar v. State of Rajasthan' (AIR 1952 SC 54 at p. 59). We find, however, that it
unfortunately still persists, if not in the judgments of the Courts, at any rate in the
arguments of counsel."
11. Again in Masalti and Ors. v. State of U.P. (AIR 1965 SC 202) this Court observed :
(pp. 209-210 para 14) :
"But it would, we think, be unreasonable to contend that evidence given by witnesses
should be discarded only on the ground that it is evidence of partisan or interested
witnesses ....... The mechanical rejection of such evidence on the sole ground that it is
partisan would invariably lead to failure of justice. No hard and fast rule can be laid down
as to how much evidence should be appreciated. Judicial approach has to be cautious in
dealing with such evidence; but the plea that such evidence should be rejected because it
is partisan cannot be accepted as correct."
12

. To the same effect is the decision in 2002 AIR SCW 4271

@page-SC3117
State of Punjab v. Jagir Singh (AIR 1973 SC 2407), Lehna v. State of Haryana (2002 (3)
SCC 76) and Gangadhar Behera and Ors. v. State of Orissa (2002 (8) SCC 381).
13

. The above position was also highlighted in Babulal Bhagwan Khandare and Anr. v. State
of Maharashtra [2005 (10) SCC 404] and in Salim Saheb v. State of M.P. (2007(1) SCC
699). 2004 AIR SCW 1376

14. Section 34 has been enacted on the principle of joint liability in the doing of a
criminal act. The Section is only a rule of evidence and does not create a substantive
offence. The distinctive feature of the Section is the element of participation in action.
The liability of one person for an offence committed by another in the course of criminal
act perpetrated by several persons arises under Section 34 if such criminal act is done in
furtherance of a common intention of the persons who join in committing the crime.
Direct proof of common intention is seldom available and, therefore, such intention can
only be inferred from the circumstances appearing from the proved facts of the case and
the proved circumstances. In order to bring home the charge of common intention, the
prosecution has to establish by evidence, whether direct or circumstantial, that there was
plan or meeting of mind of all the accused persons to commit the offence for which they
are charged with the aid of Section 34, be it prearranged or on the spur of moment; but it
must necessarily be before the commission of the crime. The true contents of the Section
is that if two or more persons intentionally do an act jointly, the position in law is just the
same as if each of them has done it individually by himself. As observed in Ashok Kumar
v. State of Punjab (AIR 1977 SC 109), the existence of a common intention amongst the
participants in a crime is the essential element for application of this Section. It is not
necessary that the acts of the several persons charged with commission of an offence
jointly must be the same or identically similar. The acts may be different in character, but
must have been actuated by one and the same common intention in order to attract the
provision.
15

. The Section does not say "the common intention of all", nor does it say "and intention
common to all". Under the provisions of Section 34 the essence of the liability is to be
found in the existence of a common intention animating the accused leading to the doing
of a criminal act in furtherance of such intention. As a result of the application of
principles enunciated in Section 34, when an accused is convicted under Section 302 read
with Section 34, in law it means that the accused is liable for the act which caused death
of the deceased in the same manner as if it was done by him alone. The provision is
intended to meet a case in which it may be difficult to distinguish between acts of
individual members of a party who act in furtherance of the common intention of all or to
prove exactly what part was taken by each of them. As was observed in Ch. Pulla Reddy
and Ors. v. State of Andhra Pradesh (AIR 1993 SC 1899), Section 34 is applicable even if
no injury has been caused by the particular accused himself. For applying Section 34 it is
not necessary to show some overt act on the part of the accused. 1993 AIR SCW 1843

16. We find that the evidence of PWs 7 and 8 clearly establishes the roles played by the
Hardeep Singh and Harjinder Singh. It is also clear from the prosecution evidence that on
hearing of the exhortation of the acquitted Gurcharan Singh, Jaswinder Singh and
Harjinder Singh had given blows on the legs of the deceased. After he fell down, Hardeep
Singh gave blow on the head. Similarly Harjinder Singh also gave blows on the head.
After the deceased fell down Jaswinder Singh did not attack on any vital part of the
deceased's body. He assaulted on the arm of the deceased. In the aforesaid circumstances,
while appeal filed by the accused appellants Hardeep and Harjinder Singh is dismissed,
the appropriate conviction of Jaswinder Singh would be under Section 304 Part II IPC.
His conviction is accordingly altered. Custodial sentence of eight years would meet the
ends of justice.
17. The appeal is allowed to the aforesaid extent.
Order accordingly. .
@page-SC3118
AIR 2008 SUPREME COURT 3118 "Bhakra Beas Management Board v. Kanta
Aggarwal"
(From : Punjab and Haryana)*
Coram : 2 Dr A. PASAYAT AND P. SATHASIVAM, JJ.
Civil Appeal No. 4216 of 2008 (arising out of SLP (C) No. 21953 of 2004), D/- 7 -7
-2008.
Bhakra Beas Management Board v. Smt. Kanta Aggarwal and Ors.
Motor Vehicles Act (59 of 1988), S.168 - MOTOR VEHICLES - SUPREME COURT -
Accident compensation - Determination - Losses and gains arising to claimant out of
accidental death of victim - Ought to be balanced - Claimant, wife of victim given
compassionate appointment - Non-Consideration of this benefit in assessing
compensation - Improper - However considering fact that accident took place before 14
years - Instead of remitting matter to Tribunal Supreme Court directed that sum of
Rupees five lakhs already deposited shall be withdrawn by claimants in full and final
settlement of his claim. (Paras 12, 13)
Cases Referred : Chronological Paras
2002 AIR SCW 2920 : AIR 2002 SC 2607 (Rel. on) 7
1998 AIR SCW 3105 : AIR 1998 SC 3191 (Rel. on) 9
1994 AIR SCW 1356 : AIR 1994 SC 1631 (Ref.) 7
(1988) 3 All ER 370 7
AIR 1971 SC 1624 (Ref.) 7
AIR 1962 SC 1 (Rel. on) 7, 8
1942 AC 601 7
Dhruv Mehta, Yashraj Singh Deora, Harshvardhan Jha and M/s. K. L. Mehta and Co., for
Appellants; Jasbir Singh Malik, Ms. Shilpi, Deepak Aggarwal, R.K. Tripathi and S.K.
Sabharwal, for Respondents.
* F.A.O. No. 3351 of 2003, D/- 11-5-2004 (PandH)
Judgement
Dr. ARIJIT PASAYAT, J. :- Leave granted.
2. Challenge in this appeal is to the judgment of a learned single Judge of the Punjab and
Haryana High Court dismissing the appeal filed against the award dated 4.2.2003 passed
by Motor Accidents Claim Tribunal, Chandigarh (in short Tribunal'). 3. Background facts
in a nutshell are as follows :
In an accident which took place on 16.11.1994 at about 10.00 p.m. K.C. Aggarwal
(hereinafter referred to as the 'deceased') who was sitting directly behind the driver lost
his life. Balbir Singh (PW-1) was an eye witness to the accident. At the relevant time, he
was standing in front of Mayur Dhaba near which the accident took place. He
categorically deposed that the jeep was coming from Bilaspur side and was going
towards Sunder Nagar at a very high speed. It was being driven by the driver in the
middle of the road. He also stated that a truck was coming from the opposite side at
normal speed. When the truck reached near the jeep, the driver of the jeep applied brakes,
but because the jeep was being driven at a very high speed, it came to halt in the middle
of the road. The back portion of the truck struck against the right side portion of the jeep.
Some persons rushed towards the jeep. In the meantime, the truck sped away from the
spot. He categorically stated that the accident occurred due to rash and negligent driving
of the jeep driver. The deceased left behind a widow and three children.
A claim petition was filed by the widow and the children under Section 166 of the Motor
Vehicles Act, 1988 (in short the 'Act'). The Tribunal awarded compensation of Rs.
8,48,160/- along with interest @ 9% per annum from the date of institution.
An appeal was filed before the High Court. It was pointed out that on the death of K.C.
Aggarwal, respondent No. 1-widow had been provided with compassionate appointment
and she was getting salary of nearly Rs. 4,700/- p.m. (basic pay of nearly Rs. 4,700/-) and
a residence was provided to her. The High Court did not accept this plea and observed
that the quantum of compensation has been rightly fixed.
4. Learned counsel for the appellant submitted that the benefits which claimant has
received on account of death of her husband have to be deducted while computing the
compensation, if any, payable. With reference to the factual aspects it is submitted that
respondent No. 1 was getting salary of nearly Rs. 4,700/- and therefore she was not
entitled to compassionate appointment. It is pointed out that the appeal filed by the
claimants is pending adjudication and without considering the relevant factors the High
Court has declined to interfere.
5. Learned counsel for the respondents on the other hand submitted that the judgment
@page-SC3119
of the High Court is in order.
6. There are several undisputed factors : (i) the husband of respondent No. 1 had received
fatal injuries in an accident; (ii) the claimants seem to be facing financial problem; (iii)
the concept of just compensation cannot be lost sight Of. The High Court does not appear
to have considered the effect of amount received on account of compassionate
appointment.
7

. In United India Insurance Co. Ltd. and Ors. v. Patricia Jean Mahajan and Ors. (2002 (6)
SCC 281) it was inter alia observed as follows : 2002 AIR SCW 2920, (Paras 23, 24,
26)

"24. Mr. Soli J., Sorabji submitted that while assessing the amount of compensation, the
benefits which have accrued to the claimants by reason of death must also be taken into
account. A kind of balancing of losses and the gains or benefit by reason of death would
be necessary. In support of the above contention he has referred to a decision reported in
Gobald Motors Service Limited v. R. M. K. Veluswami (1962 (1) SCR 929), and others.
It is a decision by a three-Judge Bench of this Court, and at SCR page 938 the
observations made by the House of Lords in Davies v. Powell Duffryn Associated
Collieries Ltd. (1942 AC 601) has been quoted which reads as follows (All ER p. 658
B) : AIR 1962 SC 1
Para 5 of AIR

"The general rule which has always prevailed in regard to the assessment of damages
under the Fatal Accidents Acts is well settled, namely, that any benefit accruing to a
dependent by reason of the relevant death must be taken into account. Under those Acts
the balance of loss and gain to a dependant by the death must be ascertained, the position
of each dependant being considered separately."
25. To further elaborate the above proposition, observations made by Lord Wright in
Davies case (supra) have also been quoted. It reads as follows :-
"The damages are to be based on the reasonable expectation of pecuniary benefit or
benefit reducible to money value. In assessing the damages all circumstances which may
be legitimately placed in diminution of the damages must be considered..... The actual
pecuniary loss of each individual entitled to sue can only be ascertained by balancing, on
the one hand, the loss to him of the future pecuniary benefit, and on the other, any
pecuniary advantage which from whatever source comes to him by reason of the death."
The learned counsel laid stress on the last part of observation made to the effect that - for
the purposes of balancing losses and gains any pecuniary advantage which from whatever
source come to them, has to be considered.

26. It is submitted in Gobald's case the principle of Davies Case was referred and taken
into consideration. Reliance has also been placed on a decision reported in M/s.
Sheikhupura Transport Co. Ltd. v. Northern India Transport Insurance Company (1971
(1) SCC 785), particularly to the observations made by the Court in paragraph 6 of the
judgment where the principle in the case of Gobald Motors (supra) has been reiterated.
AIR 1971 SC 1624
1994 AIR SCW 1356

In this connection learned counsel for the Insurance Company has also drawn our
attention to the decision in the case of Susamma Thomas (supra) particularly on
paragraph 8 of the report, where it is observed that the principle in the case of Davies v.
Powell was adopted, in the case of Gobald Motors (supra). It is thus submitted that
principle of balancing of loss and gains, so as to arrive at a just and fair amount of
compensation has been accepted by this Court as well. On behalf of the Insurance
Company Hodgson v. Trapp (1988 (3) All ER 870) has been relied on in which our
attention has particularly been drawn to the following observations made at All ER p.
873j-874b :
"........ the basic rule is that it is the net consequential loss and expense which the Court
must measure. If, in consequence of the injuries sustained, the plaintiff has enjoyed
receipts to which he would not otherwise have been entitled, prima facie, those receipts
are to be set against the aggregate of the plaintiffs losses and expenses in arriving at the
measure of his damages. All this is elementary and has been said over and over again. To
this basic rule there are, of course, certain well established, though not always precisely
defined and delineated, exceptions. But the Courts are, I think, sometimes in danger, in
seeking to explore the rationale of the exceptions, of forgetting that they are exceptions. It
is the rule which is fundamental and axiomatic and exceptions
@page-SC3120
to it which are only to be admitted on grounds which clearly justify their treatment as
such."
From the above passage it is clear that the deductions are admissible from the amount of
compensation in case the claimant receives the benefit as a consequence of injuries
sustained, which otherwise he would not have been entitled to. It does not cover cases
where the payment received is not dependent upon an injury sustained on meeting with
an accident. The other observation to which our attention has been drawn at page 876
placitum 'f' also does not help the contention raised on behalf of the Insurance Company
for deduction of amounts in the present case. The Court was considering a situation
where due to the injuries received the victim was claiming cost of care necessary in
future in respect of which statutory provision, provided for attendant's allowance. It was
found that the statutory benefit and the damages claimed were designed to meet the
identical expenses. This is however not so, at least not shown, to be so in the case in
hand."
8

. Similarly, in Gobald Motor Service Ltd. and Anr. vs. R.M.K. Veluswami and Ors. (1962
(1) SCR 929 at p. 938) it was inter alia observed as follows : "The general rule which has
always prevailed in regard to the assessment of damages under the Fatal Accidents Acts is
well settled, namely, that any benefit accruing to a dependant by reason of the relevant
death must be taken into account. Under those Acts the balance of loss and gain to a
dependant by the death must be ascertained, the position of each dependants being
considered separately." AIR 1962 SC 1 at p. 5

. In Helen C. Rebello v. Maharashtra S.R.T.C. (1999 (I) SCC 90) it was held as follows :
1998 AIR SCW 3105
Paras 33 to 35

"32. So far as the general principle of estimating damages under the common law is
concerned, it is settled that the pecuniary loss can be ascertained only by balancing on
one hand, the loss to the claimant of the future pecuniary benefits that would have
accrued to him but for the death with the "pecuniary advantage" which from whatever
source comes to him by reason of the death. In other words, it is the balancing of loss and
gain of the claimant occasioned by the death. But this has to change its colour to the
extent a statute intends to do. Thus, this has to be interpreted in the light of the provisions
of the Motor Vehicles Act, 1939. It is very clear, to which there could be no doubt that
this Act delivers compensation to the claimant only on account of accidental injury or
death, not on account of any other death. Thus, the pecuniary advantage accruing under
this Act has to be deciphered, correlating with the accidental death. The compensation
payable under the Motor Vehicles Act is on account of the pecuniary loss to the claimant
by accidental injury or death and not other forms of death. If there is natural death or
death by suicide, serious illness, including even death by accident, through train, air flight
not involving a motor vehicle, it would not be covered under the Motor Vehicles Act.
Thus, the application of the general principle under the common law of loss and gain for
the computation of compensation under this Act must correlate to this type of injury or
death, viz., accidental. If the words "pecuniary advantage" from whatever source are to be
interpreted to mean any form of death under this Act, it would dilute all possible benefits
conferred on the claimant and would be contrary to the spirit of the law. If the "pecuniary
advantage" resulting from death means pecuniary advantage coming under all forms of
death then it will include all the assets moveable, immovable, shares, bank accounts, cash
and every amount receivable under any contract. In other words, all heritable assets
including what is willed by the deceased etc. This would obliterate both, all possible
conferment of economic security to the claimant by the deceased and the intentions of the
legislature. By such an interpretation, the tortfeasor in spite of his wrongful act or
negligence, which contributes to the death, would have in many cases no liability or
meagre liability. In our considered opinion, the general principle of loss and gain takes
colour of this statute, viz., the gain has to be interpreted which is as a result of the
accidental death and the loss on account of the accidental death. Thus, under the present
Act, whatever pecuniary advantage is received by the claimant, from whatever source,
would only mean which comes to the claimant on account of the accidental death and not
other forms of death. The constitution of the Motor Accident Claims Tribunal itself under
Section 110 is, as the section states :
@page-SC3121
". . . for the purpose of adjudicating upon claims for compensation in respect of accidents
involving the death of, or bodily injury to, . . .".
33. Thus, it would not include that which the claimant receives on account of other forms
of deaths, which he would have received even apart from accidental death. Thus, such
pecuniary advantage would have no correlation to the accidental death for which
compensation is computed. Any amount received or receivable not only on account of the
accidental death but that which would have come to the claimant even otherwise, could
not be construed to be the "pecuniary advantage", liable for deduction. However, where
the employer insures his employee, as against injury or death arising out of an accident,
any amount received out of such insurance on the happening of such incident may be an
amount liable for deduction. However, our legislature has taken note of such contingency
through the proviso of Section 95. Under it the liability of the insurer is excluded in
respect of injury or death, arising out of and in the course of employee.
34. This is based on the principle that the claimant for the happening of the same
incidence may not gain twice from two sources. This, it is excluded thus, either through
the wisdom of the legislature or through the principle of loss and gain through deduction
not to give gain to the claimant twice arising from the same transaction, viz., the same
accident. It is significant to record herein both the sources, viz., either under the Motor
Vehicles Act or from the employer, the compensation receivable by the claimant is either
statutory or through the security of the employer securing for his employee but in both
cases he receives the amount without his contribution. How thus an amount earned out of
one's labour or. contribution towards one's wealth, savings, etc. either for himself or for
his family which such person knows under the law has to go to his heirs after his death
either by succession or under a Will could be said to be the "pecuniary gain" only on
account of one's accidental death. This, of course, is a pecuniary gain but how this is
equitable or could be balanced out of the amount to be received as compensation under
the Motor Vehicles Act. There is no correlation between the two amounts. Not even
remotely. How can an amount of loss and gain of one contract be made applicable to the
loss and gain of another contract. Similarly, how an amount receivable under a statute has
any correlation with an amount earned by an individual. Principle of loss and gain has to
be on the same plane within the same sphere, of course, subject to the contract to the
contrary or any provision of law."
10. It is pointed out that the award as made is extremely high and the concept of just
compensation has been lost sight of.
11. Learned counsel for the respondent supported the judgment and additionally
submitted that appeal of respondent No. 1 is pending. In normal course, when two
appeals are directed against the common judgment, both the appeals should be heard by
the same Bench of the High Court.
12. But we find that the High Court lost sight of the fact that the benefits which the
claimant receives on account of the death or injury have to be duly considered while
fixing the compensation. It is pointed out that respondent No. 1 was getting Rs. 4,700/ -
p.m. and a residence has been provided to her and actually the compassionate
appointment was given immediately after the accident.
13. In view of what has been stated above, the High Court's judgment is clearly
unsustainable. However, the accident took place more than 14 years back and it would
not be desirable to send the matter back to the Tribunal for fresh consideration. A sum of
rupees five lakhs has been deposited vide this Court's order dated 1.11.2004. We are of
the considered view that in view of the background facts, it is just and proper that the sum
of Rupees five lakhs already deposited shall be permitted to be withdrawn by the
claimants in full and final settlement of the claim relatable to the death of the deceased. It
is for the Tribunal to fix the quantum of fixed deposit and the amount to be released to the
claimants.
14. The appeal is allowed in the aforesaid extent.
Order accordingly. .
@page-SC3122
AIR 2008 SUPREME COURT 3122 "Goetze (India) Limited, M/s. v. Employees State
Insurance Corporation"
(From : Punjab and Haryana)*
Coram : 2 Dr. A. PASAYAT AND P. SATHASIVAM, JJ.
Civil Appeal No.8432 of 2001, D/- 7 -7 -2008.
M/s. Goetze (India) Limited v. Employees State Insurance Corporation.
Employees' State Insurance Act (34 of 1948), S.39 - Employees State Insurance (General)
Regulations (1950), Regn.31A - EMPLOYEES STATE INSURANCE - INTEREST -
Contribution - Delay in making payment - Liability to pay interest is statutory - There is
no power of waiver - No question of compromise or settlement arises. (Para 6)
Cases Referred : Chronological Paras
2000 AIR SCW 821 : AIR 2000 SC 1190 : 2000 Lab IC 1023 (Ref.) 2, 3
M.L. Varma, Sr. Advocate, Ms. Meera Mathur and Satya Mitra, for Appellant; C.S. Rajan,
Sr. Advocate, Anupam Mishra and V.J. Francis, for Respondent.
* CWP No. 13918 of 2000, D/- 19-12-2000 (P and H).
Judgement
Dr. ARIJIT PASAYAT, J. :- Challenge in this appeal is to the order passed by a Division
Bench of the Punjab and Haryana High Court dismissing the writ petition filed by the
appellant.
2. Controversy lies within a very narrow compass.
Employees' State Insurance Corporation (in short the 'Corporation'), the respondent
herein raised a demand for contribution under the Employees' State Insurance Act, 1948
(in short the 'Act') on the component of efficiency bonus for the period January 1988 to
September, 1989 by order dated 23.7.1992. The demand was challenged before the ESI
Court under Section 75 of the Act. Pending the proceedings before the ESI Court,
Corporation by letter dated 1.3.1997 asked for production of record for the purpose of re-
verification for the period from 1989 to 1991 and from 1992 to 1994 to determine the
amount payable.
The respondent Corporation on re-verification determined the actual amount payable as
follows :

(a) 1/88 to 3/89 asRs. 2,26,454/-


(b) 4/89 to 3/94 asRs. 5,28,071/-
_______________
Total Rs.7,54,525/-
_______________

Appellant agreed to pay the contribution and paid the same in October/November, 1997.

Appellant took the stand that there was a bona fide dispute about eligibility. Since
eligibility to "efficiency bonus" under the scheme was subject to attendance of 50 days in
a quarter is payable and paid quarterly. Appellant took the stand that it falls outside the
definition of wages under Section 2(22) of the Act. Appellant took the plea that its stand
was supported by a judgment of this Court in Whirlpool of India Ltd. v. Employees' State
Insurance Corporation [2000 (3) SCC 185]. The ESI Court disposed of the matter on
6.1.1998 taking note of the stand of the appellant that it had deposited the definite amount
after re-verification and the bank guarantee furnished by the appellant was released. On
11.1.2000 the Corporation wrote a letter to the appellant demanding payment of interest
on the amount paid to the Corporation for the period from 1988 to 1994 as covered by
order dated 6.1.1998 and directed payment of interest amounting to Rs. 4,61,825/-. The
appellant questioned the demand before the High Court by filing a writ petition. The
appellant's stand was that a compromise had been arrived at as is apparent from the order
of the ESI Court to the effect that nothing was payable by the appellant. Corporation took
the stand that the liability to pay interest was statutory and, therefore, there could not
have any compromise. In any event the submission of the appellant that nothing further
was payable as ESI contribution was noted and therefore, the bank guarantee was
released. There was no question of any compromise to waive the interest which is not
statutorily permissible. The High Court accepted the stand and dismissed the writ
petition. 2000 AIR SCW 821

3. In support of the appeal learned counsel for the appellant submitted that there was an
order of stay and therefore the question of any interest does not arise. Further when the
counsel for the Corporation himself stated that nothing further was payable, it clearly
indicated that there was a statement on the compromise for waiver of interest. It was
pointed out that with a view to
@page-SC3123
buy peace the appellant had agreed to pay the amount though this case was clearly
covered by Whirlpool's case (supra).
4. Learned counsel for the respondent on the other hand submitted that there is no
question of any compromise to waive interest because the same is statutorily payable.
There cannot be any compromise without any authority when there is no provision for
any compromise or settlement. It is therefore stated that the High Court's view is right.
5. In order to appreciate rival submissions it would be necessary to take note of few
provisions. Sections 39 and Regulations 31 and 31A reads as follows:
"Section 39 - Contributions
xx xx xx
5(a) If any contribution payable under this Act is not paid by the principal employer on
the date on which such contribution has become due he shall be liable to pay simple
interest at the rate of 12%. per annum or at such higher rate as may be specified in the
regulations till the date of its actual payment."
"Regulation 31 - Time for payment of contribution
An employer who is liable to pay contributions in respect of any employee shall pay
those contributions within 21 days of the last day of the calendar month in which the
contributions fall due :
Provided that where a factory/establishment is permanently closed, the employer shall
pay contribution on the last day of its closure."
"Regulation 31A - Interest on contribution due, but not paid in time
An employer who fails to pay contribution within the periods specified in regulation 31,
shall be liable to pay interest at the rate of 12% per annum in respect of each day of
default or delay in payment of contribution."
6. As there was delay in making the payment of the contribution the Corporation had
issued notice on 29.6.1990 at the first instance and thereafter the order was passed under
Section 45-A of the Act on 23.7.1992. The same was challenged before the ESI Court in
which an interim stay was granted on 9.10.1992. During the pendency of the matter there
was re-verification and the quantum payable by the payment was worked out. The
liability to pay interest is statutory. There is no power of waiver. The question of any
compromise or settlement does not really arise. Even otherwise the order of the ESI
Court referred to and relied upon by the appellant is of no assistance to the appellant. It
only noted statement of the appellant that he had deposited contribution payable. The
reference to "no further due" is obviously relatable to the contribution payable and
nothing beyond that.
7. Above being the position, the appeal is sans merit, deserves dismissal, which we direct.
There shall be no order as to costs.
Appeal dismissed. .
AIR 2008 SUPREME COURT 3123 "Dabur India Ltd. v. K. R. Industries"
(From : 2006 (33) PTC 348 (Delhi))
Coram : 2 S. B. SINHA AND LOKESHWAR SINGH PANTA, JJ.
Civil Appeal No.3637 of 2008 (arising out of SLP (C) No. 20941 of 2006), D/- 16 -5
-2008.
Dabur India Ltd. v. K.R. Industries.
Copyright Act (14 of 1957), S.55, S.62 - Trade and Merchandise Marks Act (43 of 1958),
S.106 (since repealed) - Civil P.C. (5 of 1908), O.7, R.11, O.2, R.3, S.20 - COPYRIGHT
- TRADE AND MERCHANDISE - PLAINT - CIVIL PROCEDURE - CIVIL COURT -
Copyright infringement - Suit for - Relief for passing off action also claimed - Suit filed
in Court where plaintiff resides - Such composite suit is not maintainable - Rejection of
plaint, proper.
A composite suit for infringement of copyright and passing off action is not maintainable
in a Court where the plaintiff resides. A composite suit would not entitle a Court to
entertain a suit in respect whereof it has no jurisdiction, territorial or otherwise. Order 2,
Rule 3 of the Code specifically states so and, thus, there is no reason as to why the same
should be ignored. The rejection of the plaint was, therefore, proper.
2006 AIR SCW 72, Expln. (Para 29)
The Parliament having inserted sub-section (2) in Section 62 of the 1957 Act, the
jurisdiction of the Court thereunder would be wider than the one under Section 20 of the
Code. The object and reasons for enactment of sub-section (2) of Section 62 would also
appear from the report of the Committee, as has been noticed by Supreme Court
@page-SC3124
being a provision which has been specially designed to confer an extra benefit upon the
authors who were not in a position to instate copyright infringement proceeding before
the Courts. If the impediment is sought to be removed by inserting an incidental
provision, there cannot be any doubt the Court could be entitled to pass an interim order,
but the same by no stretch of imagination can be extended to a cause of action which is
founded on separate set of facts as also rights and liabilities of a party under a different
Act. (Para 27, 28)
Cases Referred : Chronological Paras
2008 AIR SCW 309 : AIR 2008 SC 907 : 2008 (1) ALJ 752 (Ref.) 24
2006 AIR SCW 72 : AIR 2006 SC 730 : 2006 (1) ALJ 669 : 2006 CLC 1 (Expln.) 8, 9,
11, 17, 19, 21, 27, 28, 29
2004 AIR SCW 1087 : AIR 2004 SC 1682 : 2004 CLC 308 10, 26, 28
(2004) 11 SCC 26 (Ref.) 23
AIR 1998 All 43 : 1998 All LJ 188 19
AIR 1998 Delhi 225 19, 21
1995 AIR SCW 4065 : AIR 1996 SC 285 : 1995 Lab IC 2714 (Ref.) 24
Fali S. Nariman, Sr. Advocate, Praveen Anand, Hari Shankar K., Subhash Sharma, Sagar
Chandra, Abhishta Kumbhat, Vikas Singh Jangra, with him for Appellant; Shailen Bhatia
and Balraj Dewan, for Respondent.
Judgement
1. S. B. SINHA, J. :-Leave granted.
2. Appellant is a manufacturer of a product known as 'Dabur Red Tooth Powder' or
'Dabur Lal Dant Manjan'. In the year 1993, it had allegedly adopted a unique colour
combination and arrangement of features which was subsequently changed in December
1999.
3. Respondent herein is also said to be manufacturer of a tooth powder known as 'Sujata'.
It is said to have infringed the copyright of the appellant. A suit was filed by the appellant
against the respondent in the Delhi High Court. Paragraph 7 of the plaint reads thus :
"7. In December 1999, the plaintiff adopted a new carton while retaining the conical
shape and white cap for their product which is described hereinbelow :
•On one column has the words RED TOOTH POWDER within a yellow blurb.
• Immediately below the blurb is an oval shaped picture of a family with a yellow
background.
• Above these two features there is a legend within a blurb mentioning the fact that this is
a new pack.
• The column immediately next to it contain the same features in the Devnagri script.
• A third column sets out the details including, Composition, Weight, MRP and
Manufacturers Name etc.
• The top half of the third column contains an oval shaped device containing a
diagrammatic representation of the herbs that constitute the ingredients of Plaintiffs
product."
4. It was alleged that the said carton constituted an 'artistic work' within the meaning of
Section 25-C of the Copyright Act, 1957 (the 1957 Act). Respondent is said to have been
using an identical colour scheme lay out, arrangement of features and get up as that of the
plaintiffs, the essential features whereof are :
• "One column has the words RED TOOTH POWDER within a yellow blurb.
• A column which contains the representation of a family in an oval shape picture.
• There is a similar representation in the Devnagri script in another column.
• The details of the product are set out in another column.
• Above the details of the product there is advice of a lotus, similar to the positioning of
the plaintiffs herbs in the plaintiffs carton."
5. The reliefs claimed for by the plaintiff in the said suit, inter alia, are :
"(A) An order of permanent injunction restraining the defendant, their partners,
proprietors and/or promoters, as the case may be, their servants and agents,
representatives, dealers and all others acting for and on their behalf from reproducing any
of the artistic features of the plaintiffs DABUR RED TOOTH POWDER
container/packaging/pouch, including its colour combination, get up, layout or
arrangement of features, printing, publishing, using or otherwise reproducing any of the
artistic features thereof in any material form amounting to an infringement of copyright.
(B) An order of permanent injunction restraining the defendants, their partners,
proprietors
@page-SC3125
and/or promoters, as the case may be, their servants and agents, representatives, dealers
and all others acting for and on their behalf from manufacturing, selling, offering for sale
or otherwise directly or indirectly dealing in tooth powder packed in the impugned
packaging or any other packaging as may be a slavish imitation and/or a substantial
reproduction of the DABUR RED TOOTH POWDER container/packaging/pouch or
from committing any other act as is likely to cause confusion and deception amounting to
passing off."
6. Respondent filed an application in the suit purported to be under Order 7, Rule 11 of
the Code of Civil Procedure, 1908 (hereinafter referred to as 'the Code') contending that
as the defendant is resident of Andhra Pradesh, the Delhi High Court had no jurisdiction.
7. By reason of the impugned judgment and order dated 22.5.2006, a learned Single
Judge of the High Court accepted the said contention of the respondent.
8

. An intra-court appeal preferred there-against has been dismissed by a Division Bench of


the said Court holding that the matter is covered by the decision of this Court in Dhodha
House v. S.K. Maingi, [(2006) 9 SCC 41]. It was stated : 2006 AIR SCW 72

"The learned Single Judge has also held that so far as the aforesaid relief relating to
passing off is concerned, Delhi High Court does not have any territorial jurisdiction as the
respondent/defendant is from Andhra Pradesh and there is no documentary evidence to
show that the respondent was selling goods in Delhi. We agree with the aforesaid findings
and conclusions recorded by the learned Single Judge. Accordingly, we find no merit in
the submissions of the counsel appearing for the appellant that the composite suit of
infringement of copyright and passing off would lie in the same forum. We also do not
find any error in the judgment of the learned Single Judge as in our considered opinion so
far the relief for passing off is concerned, the same is covered by the decision of the
Supreme Court in Dhodha House case."
9

. Mr. Fali S. Nariman, learned senior counsel appearing on behalf of the appellant,
submits that the Division Bench of the High Court committed a serious error in passing
the impugned judgment in so far as it failed to take into consideration the effect of a
consolidated suit under the 1957 Act as also the Trade Marks Act, 1958 (for short
'the1958 Act'), as would be evident from the following excerpts of Dhodha House (supra)
: 2006 AIR SCW 72, Paras 22, 42 and 43

"22. We are not concerned in this case with the maintainability of a composite suit both
under the 1957 Act and the 1958 Act. Indisputably, if such a situation arises, the same
would be permissible; but the same may not be relevant for the purpose of determining
the question of a forum where such suit can be instituted. Sub-section (2) of Section 62 of
the 1957 Act provides for a non obstante clause conferring jurisdiction upon the District
Court within the local limits of whose jurisdiction, at the time of the institution of the suit
or other proceeding, the persons instituting the suit or other proceedings have been
residing. In terms of sub-section (1) of Section 62, suit can be instituted and the
proceedings can be initiated in respect of matters arising under the said chapter for
infringement of the copyright in any work or the infringement of any other right
conferred thereunder. It does not confer jurisdiction upon a District Court where the
plaintiff resides, if a cause of action arises under the 1958 Act.
xxx xxx xxx
43. The short question which arises for consideration is as to whether causes of action in
terms of both the 1957 Act and the 1958 Act although may be different, would a suit be
maintainable in a court only because it has the jurisdiction to entertain the same in terms
of Section 62(2) of the 1957 Act?
44. A cause of action in a given case both under the 1957 Act as also under the 1958 Act
may be overlapping to some extent. The territorial jurisdiction conferred upon the court
in terms of the provisions of the Code of Civil Procedure indisputably shall apply to a suit
or proceeding under the 1957 Act as also the 1958 Act. Sub-section (2) of Section 62 of
the 1957 Act provides for an additional forum. Such additional forum was provided so as
to enable the author to file a suit who may not otherwise be in a position to file a suit at
different places where his copyright was violated. Parliament while enacting the Trade
and Merchandise Marks Act in the year 1958 was aware of the provisions of the 1957
Act. It still did not choose to make a similar provision therein. Such
@page-SC3126
an omission may be held to be a conscious action on the part of Parliament. The intention
of Parliament in not providing for an additional forum in relation to the violation of the
1958 Act is, therefore, clear and explicit. Parliament while enacting the Trade Marks Act,
1999 provided for such an additional forum by enacting sub-section (2) of Section 134 of
the Trade Marks Act. The court shall not, it is well settled, readily presume the existence
of jurisdiction of a court which was not conferred by the statute. For the purpose of
attracting the jurisdiction of a court in terms of sub-section (2) of Section 62 of the 1957
Act, the conditions precedent specified therein must be fulfilled, the requisites wherefor
are that the plaintiff must actually and voluntarily reside to carry on business or
personally work for gain."
10

. Learned counsel would contend that the jurisdiction of the court to entertain a composite
suit under the 1957 Act and the 1958 Act should be determined having regard to the
provisions of Section 55 of the former. The term 'Law' within the meaning of the said
provision, it was submitted, would not only include a statute law but also the common
law and, thus, viewed from that perspective a composite suit for infringement of a
copyright as also passing of shall be maintainable. Strong reliance in this behalf has also
been placed on Exphar Sa and Anr. v. Eupharma Laboratories Ltd. and Anr. [(2004) 3
SCC 688]. 2004 AIR SCW 1087

11

. Mr. Shailen Bhatia, learned counsel appearing on behalf of the respondent, on the other
hand, would submit that this Court in Dhondha House (supra) categorically held that the
cause of action for infringement of the 1957 Act and that of the 1958 Act are distinct and
separate. 2006 AIR SCW 72

12. Order II, Rule 3 of the Code, it was submitted, deals with pecuniary jurisdiction and
not the territorial jurisdiction of the Court.
Drawing our attention to the provisions contained in Section 134 of the Trade Marks Act,
1999, the learned counsel would contend that in relation to a passing off the action, even
the Parliament did not think it expedient that any provision giving an option to the
plaintiff to file a suit where it resides and not the defendant.
13. Before adverting to the rival contentions of the parties, as noticed hereinbefore, we
may notice the provision of sub-section (1) of Section 55 and sub-section (2) of Section
62 of 1957 Act, which read :-
"Section 55. Civil remedies for infringement of copyright - (1) Where copyright in any
work has been infringed, the owner of the copyright shall, except as otherwise provided
by this Act, be entitled to all such remedies by way of injunction, damages, accounts and
otherwise as are or may be conferred by law for the infringement of a right;
Section 62 - Jurisdiction of court over matters arising under this Chapter
(1) ... ...
(2) For the purpose of sub-section (1), a "District Court having jurisdiction" shall,
notwithstanding anything contained in the Code of Civil Procedure, 1908 (5 of 1908), or
any other law for the time being in force, include a District Court within the local limits
of whose jurisdiction, at the time of the institution of the suit or other proceeding, the
person instituting the suit or other proceeding or, where there are more than one such
persons, any of them actually and voluntarily resides or carries on business or personally
works for gain."
Sub-section (2) of Section 27 of the 1958 Act reads :-
"27. No action for infringement of unregistered trade mark. -
(1) .... ....
(2) Nothing in this Act shall be deemed to affect rights of action against any person for
passing off goods as the goods of another person or the remedies in respect thereof."
Sub-section 2 of Section 106 of 1958 Act reads :-
"Section 106 - Reliefs in suits for infringement or for passing off. -
(1) .... ....
(2) Notwithstanding anything contained in sub-section (1), the court shall not grant relief
by way of damages (other than nominal damages) or an account of profits in any case -
(a) where in a suit for infringement of a trade mark, the infringement complained of is in
relation to a certification trade mark; or
(b) 'where in a suit for infringement the defendant satisfies the court -
(i) that at the time he commenced to use the trademark complained of in the suit he
@page-SC3127
was unaware and had no reasonable ground for believing that the trade mark of the
plaintiff was on the register or that the plaintiff was registered user using by way of
permitted use; and
(ii) that when he became aware of the existence and nature of the plaintiffs right in the
trade mark, he forthwith ceased to use the trade mark in relation to goods in respect of
which it was registered; or
(c) where in a suit for passing off the defendant satisfies the court -
(i) that at the time he commenced to use the trade mark complained of in the suit he was
unaware and had no reasonable ground for believing that the trade mark of the plaintiff
was in use; and
(ii) that when he became aware of the existence and nature of the plaintiffs trade mark, he
forthwith ceased to use the trade mark complained of.-
14. We may also at this stage notice the provisions of the Trade Marks Act, 1999, (for
short, '1999 Act'), Section 134 whereof reads :-
"Section 134 - Suit for infringement, etc., to be instituted before District Court
(1) No suit-
(a) for the infringement of a registered trade mark; or
(b) relating to any right in a registered trade mark; or
(c) for passing off arising out of the use by the defendant of any trade mark which is
identical with or deceptively similar to the plaintiffs trade mark, whether registered or
unregistered,
shall be instituted in any court inferior to a District Court having jurisdiction to try the
suit.
(2) For the purpose of clauses (a) and (b) of sub-section (1), a "District Court having
jurisdiction" shall, notwithstanding anything contained in the Code of Civil Procedure,
1908 (5 of 1908) or any other law for the time being in force, include a District Court
within the local limits of whose jurisdiction, at the time of the institution of the suit or
other proceeding, the person instituting the suit or proceeding, or, where there are more
than one such persons, any of them actually and voluntarily resides or carries on business
or personally works for gain.
Explanation.- For the purposes of sub-section (2), "person" includes the registered
proprietor and the registered user."
Sub-section (1) of Section 135 of the 1999 Act reads :-
"Section 135. Relief in suits for infringement or for passing off.- (1) The relief which a
court may grant in any suit for infringement or for passing off referred to in section 134
includes injunction (subject to such terms, if any, as the court thinks fit) and at the option
of the plaintiff, either damages or an account of profits, together with or without any
order for the delivery-up of the infringing labels and marks for destruction or erasure."
15. The question which was posed by the learned Single Judge is as under:-
"The next question, however, which is more important is whether the plaintiff can
combine the two causes of action one under the Copyright Act and the second under the
Act of 1958 in a situation where this court has the jurisdiction in so far as cause of action
under the Copyright Act is concerned but has no territorial jurisdiction to entertain the
cause of action relating to Act of 1958."
16. Noticing the provisions of Order II, Rules 2 and 3 of the Code of Civil Procedure
enabling the plaintiff to combine more than one cause of actions, it was opined that the
said provisions relate to pecuniary jurisdiction. The said jurisdiction, however, can be
exercised only in the event the court has otherwise jurisdiction in respect of the cause of
action wherefor the action has been brought.
17. The learned Single Judge noticed some precedents and opined :-

"13. Normally, I would have felt myself bound by the aforesaid two judgments which are
not only of this court but relate to same subject-matter, namely, joining of two causes of
action under trademark and copyright law. Even if I hold different opinion, the normal
course of action would have been to refer the matter to the Division Bench. However, this
is not necessary in the instant case in view of the fact that the controversy now stands
settled by the Supreme Court in Dhodha House (supra)." 2006 AIR SCW 72

18. The Division Bench of the High Court on the other hand while holding that the High
Court has jurisdiction to adjudicate and decide upon the relief of infringement
@page-SC3128
of copyright observed that it would be open to the appellant to file a fresh suit in the court
of competent jurisdiction in respect of its action for passing off and thus the plaint is
required to be returned for filing in the court of competent jurisdiction only in relation
thereto.
19

. In Dhodha House (supra) this Court was concerned with the correctness of judgments of
the Allahabad High Court in Surendra Kumar Maingi v. M/s. Dodha House, [AIR 1998
Allahabad 43] and the decision of the Delhi High Court in P.M. Diesels Ltd. v. M/ s. Patel
Field Marshal, [AIR 1998 Delhi 225]. 2006 AIR SCW 72

20. It was clearly held that a judgment passed by a court having no territorial jurisdiction
is a nullity. As regards the cause of action under the 1957 Act and a cause of action under
the 1958 Act and/or a passing off action, it was held that sub-section (2) of Section 62
would confer jurisdiction on a court where the plaintiff resides. The cause of action in
respect of others was stated to be where the defendant resides.
It was also noticed that in a given case the petition under the 1957 Act or 1958 Act may
be overlapping, holding :-
"44. The territorial jurisdiction conferred upon the court in terms of the provisions of the
Code of Civil Procedure indisputably shall apply to a suit or proceeding under the 1957
Act as also the 1958 Act. Sub-section (2) of Section 62 of the 1957 Act provides for an
additional forum. Such additional forum was provided so as to enable the author to file a
suit who may not otherwise be in a position to file a suit at different places where his
copyright was violated. Parliament while enacting the Trade and Merchandise Marks Act
in the year 1958 was aware of the provisions of the 1957 Act. It still did not choose to
make a similar provision therein. Such an omission may be held to be a conscious action
on the part of Parliament. The intention of Parliament in not providing for an additional
forum in relation to the violation of the 1958 Act is, therefore, clear and explicit."
21

. Noticing that whereas in Dhoda House (supra) the infringement complained of primarily
was that of 1958 Act and not under the 1957 Act, in Patel Field Marshal (supra) the thrust
was on the sale of products and/or advertisement by the appellant for registration of trade
marks in the Trade Marks Journal and other local papers. 2006 AIR SCW 72
AIR 1998 Delhi 225

The law was stated in the following terms:-


"54. For the purpose of invoking the jurisdiction of a court only because two causes of
action joined in terms of the provisions of the Code of Civil Procedure, the same would
not mean that thereby the jurisdiction can be conferred upon a court which had
jurisdiction to try only the suit in respect of one cause of action and not the other.
Recourse to the additional forum, however, in a given case, may be taken if both the
causes of action arise within the jurisdiction of the court which otherwise had the
necessary jurisdiction to decide all the issues."
22. What would, however, be the nature of composite suit, was also be taken note of. The
Court observed :-
"55. In this case we have not examined the question as to whether if a cause of action
arises under the 1957 Act and the violation of the provisions of the Trade Marks Act is
only incidental, a composite suit will lie or not, as such a question does not arise in this
case."
It is in the aforementioned context, submission of Mr. Nariman that a composite suit
would be maintainable having regard to subsection (1) of Section 55 of the 1957 Act
must be considered.
23. Sub-section (1) of Section 55 of 1957 Act provides for the remedies in terms whereof
the plaintiff shall be entitled to all reliefs by way of injunction, damages, accounts and
otherwise as are or may be conferred by law for the infringement of a right. It must be
read as 'ejusdem generis'. It must take its colour from the words, 'any proceeding' namely
the right to obtain a decree by way of injunction, decree for damages, accounts or other
incidental reliefs which can be granted by a civil court. Such a provision can be found in
the Code of Civil Procedure also, namely Order VII, Rule 7 thereof. It is, therefore, in our
opinion, would not be correct to read the words "as are or may be conferred by law" to
mean, any other law, violation whereof although would give rise to separate and distinct
cause of action. Under the Code claims arising under a statutes governing substantive or
procedural law, a number of remedies may be combined. The Court may grant an order of
injunction
@page-SC3129
even in a passing off action. It is trite that where the court has the jurisdiction/power to
adjudicate, it will necessarily have the incidental power therefor. It may, however, be
different if the Court may have exercised a power which is not provided for as a
supplemental proceeding e.g. Section 94 of the Code. {[See State of Punjab and Anr. v.
Devans Modern Brewaries Ltd. and Anr. [(2004) 11 SCC 26]}.
24

. Thus, whereas an incidental power is inherent in the court, a supplemental power may
also be exercised, keeping in view the ultimate relief which may be granted by it. We may
notice that this Court in Sakiri Vasu v. State of U.P. and others, [(2008) 2 SCC 409] held
that the Magistrate will have power to grant interim maintenance, although no such
provision existed in Section 125 of the Code of Criminal Procedure, 1973. 2008 AIR
SCW 309

Similarly in Hindustan Lever Ltd. v. Ashok Vishnu Kate, [(1995) 6 SCC 326] this Court
has held that the Labour Court will have the power to grant injunction as an incidental
power. 1995 AIR SCW 4065

25. If a person is found to be guilty of violation of copyright he will be bound to pay


damages. For the purpose of quantification of damages, taking of the accounts may be
necessary and it is in this behalf the Parliament thought it fit to use the word "otherwise".
Thus the power conferred by law within the meaning of sub-section (1) of Section 55 of
1957 Act qualifies the power of the court to grant remedies as envisaged thereunder if
any other cause of action arose under a different Act. An action for passing off is common
law right but the same does not determine the jurisdiction of the court. For exercising
such jurisdiction, the provisions of the Code would be applicable. 1957 Act being a
special law would, thus, prevail over the general law, viz., the Code.
26

. Exphar Sa (supra) cannot be said to have any application in the instant case. The
question which arose for consideration therein was as to whether the jurisdiction of a
court under sub-section (2) of Section 62 of the 1957 Act is wider than that of the court
specified under the Code of Civil Procedure and thus a person instituting a suit having
any claim on the ownership of the copyright which has been infringed, would not be a
ground for holding that he would not come within the purview of sub- section (2) Section
62 of the 1957 Act, as he had been served with a 'cease and desist' notice, opining :-
2004 AIR SCW 1087
"13. It is, therefore, clear that the object and reason for the introduction of sub-section (2)
of Section 62 was not to restrict the owners of the copyright to exercise their rights but to
remove any impediment from their doing so. Section 62(2) cannot be read as limiting the
jurisdiction of the District Court only to cases where the person instituting the suit or
other proceeding, or where there are more than one such persons, any of them actually
and voluntarily resides or carries on business or presently works for gain. It prescribes an
additional ground for attracting the jurisdiction of a court over and above the "normal"
grounds as laid down in Section 20 of the Code."
27

. There cannot be any doubt whatsoever that the Parliament having inserted sub-section
(2) in Section 62 of the 1957 Act, the jurisdiction of the Court thereunder would be wider
than the one under Section 20 of the Code. The object and reasons for enactment of sub-
section (2) of Section 62 would also appear from the report of the Committee, as has been
noticed by this Court being a provision which has been specially designed to confer an
extra benefit upon the authors who were not in a position to instate copyright
infringement proceeding before the Courts. It is in the aforementioned context the law
laid down by this Court in paragraph 13 of Dhonda House (supra) must be understood.
2006 AIR SCW 72

28

. If the impediment is sought to be removed by inserting an incidental provision, there


cannot be any doubt the court could be entitled to pass an interim order, but the same by
no stretch of imagination can be extended to a cause of action which is founded on
separate set of facts as also rights and liabilities of a party under a different Act. In Dhoda
House (supra), although Exphar Sa (supra) was not noticed, the distinction would be
apparent from the following :- 2006 AIR SCW 72, Paras 49 and 50
2004 AIR SCW 1087

"50. In this case, the Delhi High Court could not have invoked its jurisdiction in terms of
the 1957 Act. The primary ground upon which the jurisdiction of the original side of the
High Court was invoked was the violation of the 1958 Act, but in relation thereto, the
provisions of sub-section (2) of
@page-SC3130
Section 62 of the 1957 Act could not be invoked.
51. The plaintiff was not a resident of Delhi. It has not been able to establish that it
carries on any business at Delhi. For our purpose, the question as to whether the
defendant had been selling its produce in Delhi or not is wholly irrelevant (sic). It is
possible that the goods manufactured by the plaintiff are available in the market of Delhi
or they are sold in Delhi but that by itself would not mean that the plaintiff carries on any
business in Delhi."
29

. What then would be meant by a composite suit? A composite suit would not entitle a
court to entertain a suit in respect whereof it has no jurisdiction, territorial or otherwise.
Order II, Rule 3 of the Code specifically states so and, thus, there is no reason as to why
the same should be ignored. A composite suit within the provisions of the 1957 Act as
considered in Dhoda House (supra), therefore, would mean the suit which is founded on
infringement of a copyright and wherein the incidental power of the Court is required to
be invoked. A plaintiff may seek a remedy which can otherwise be granted by the court. It
was that aspect of the matter which had not been considered in Dhoda House (supra) but
it never meant that two suits having different causes of actions can be clubbed together as
a composite suit. 2006 AIR SCW 72

30. For the reasons aforementioned we do not find any merit in this appeal which fails
and is dismissed with costs. Counsel's fee assessed at Rs. 50,000/-.
Appeal dismissed. .
AIR 2008 SUPREME COURT 3130 "R. K. Shukla v. Sudhrist Narain Anand"
(From : 2004 (57) All LR 807)
Coram : 2 A. K. MATHUR AND TARUN CHATTERJEE, JJ.
Civil Appeal No.7238 of 2005, D/- 12 -5 -2008.
R.K. Shukla v. Sudhrist Narain Anand (Dead) by L.Rs.
(A) U.P. Urban Buildings (Regulation of Letting, Rent and Eviction) Act (13 of 1972),
S.12, S.16, S.34 - HOUSES AND RENTS - EVICTION - ALLOTMENT OF PREMISES
- Allotment of premises - Declaration of vacancy - Preliminary step for making allotment
order - Allotment made without declaring vacancy - Is grossly erroneous. (Para 12)
(B) U.P. Urban Buildings (Regulation of Letting, Rent and Eviction) Act (13 of 1972),
S.12, S.16 - HOUSES AND RENTS - EVICTION - Declaration of vacancy - Premises
found locked at time of inspection - Not conclusive proof of vacancy or deemed vacancy
of premises.
Simply because at time of inspection the gate of premises was locked is no ground to
conclude that the disputed premises is vacant. The fact that the gate was locked cannot be
a conclusive proof to hold that the landlord had removed his effects therefrom or that he
had allowed it to be occupied by any person who was not a member of his family or even
that he and members of his family had taken up residence elsewhere. No finding of
deemed vacancy can also be based. (Para 14)
(C) Constitution of India, Art.226, Art.136 - U.P. Urban Buildings (Regulation of Letting,
Rent and Eviction) Act (13 of 1972), S.12, S.16 - WRITS - SPECIAL LEAVE APPEAL -
HOUSES AND RENTS - CONCURRENT FINDINGS - Concurrent finding of fact -
Interference - Concurrent finding as to vacancy of premises - Interference with by writ
court on ground that sufficient opportunity was not given to landlord to object and lead
evidence to disprove vacancy - Proper - Supreme Court, moreover, refused to interfere
under Art.136 with finding of the High Court considering conduct of allottee. (Paras
11, 16, 17)
Cases Referred : Chronological Paras
(2005) 5 SCC 531 (Ref.) 15, 16
(1991) 2 Ren CJ 658 (All) 15
AIR 1985 SC 1635 : 1985 All LJ 900 (Ref.) 9, 15
AIR 1984 SC 1149 : 1984 All LJ 498 (Ref.) 5
AIR 1976 SC 1988 (Ref.) 15
J.C. Gupta, S.R. Singh, Sr. Advocates, Harish C. Kharbanda, Manoj Swaroop, Prakash
Chandra Shukla, Shrish Kumar Mishra, M.P.S. Tomar and Sandhya Goswami, for
Appellant; Ranjeet Kumar, Sr. Advocate, Mrs. Rani Chhabra, for Respondent.
Judgement
1. TARUN CHATTERJEE, J. :-One Parsuram Pandey filed an application for allotment
of a part of House No. 21, George Town, Hamilton Road, Allahabad, U.P. (in short "the
disputed premises") which had allegedly fallen vacant. There were in all, thirteen
applications for allotment of the disputed premises by various persons before the Rent
Control and Eviction Officer
@page-SC3131
(in short "the RC and EO"). On the said application of Parsuram Pandey for allotment of
the disputed premises, an order was passed by the RC and EO on 9th of September, 1980
directing the Rent Control Inspector (in short "the RCI") to inquire and report on the
issue of vacancy of the said disputed premises. Consequent to the order dated 9th of
September, 1980, the RCI, after inspecting the disputed premises, submitted his report to
the RC and EO regarding vacancy. Thereafter, the RC and EO on 18th of September,
1980 passed an order issuing notice to the landlord/respondent calling upon him to appear
on 6th of October, 1980 and directed that the matter of allotment of the disputed premises
would be considered on that date. Notices dated 15th of November, 1980 and 1st of
December, 1980 were again issued to the respondent for the aforesaid purpose. On 3rd of
January, 1981, the respondent was directed to appear before the RC and EO and
accordingly, the respondent did appear before the RC and EO but no other person was
present there. The RC and EO noted the presence of the respondent and passed the
following order: -
"Today the file was placed in presence of the landlord. None else was present."
2. The RC and EO passed an order dated 24th of January, 1981, on the question of
vacancy and also directed the matter to be put up on 31st of January, 1981 for arguments
on allotment and orders. It was the case of the respondent that by the aforesaid order
dated 24th of January, 1981, he came to know that certain applications were filed before
the RC and EO for allotment of the disputed premises although he along with his family
members was very much living in the disputed premises and there was no occasion for
anyone to make any application for allotment. Accordingly, the respondent had brought to
the notice of the RC and EO that he was occupying the disputed premises and the
question of allotment of the disputed premises to anyone else could not arise at all.
Therefore, all the applications for grant of allotment of the disputed premises must be
dismissed. It was all along the case of the respondent that he had filed his objections with
regard to the matter of allotment of the disputed premises on 24th of January, 1981 to the
extent that the disputed premises which was occupied and possessed by the respondent
was No. 21, Hamilton Road and not No. 21, Georgetown, Allahabad, with which the
respondent had no concern and the allotment applications, if they related to No. 21,
Hamilton Road, Allahabad were liable to be rejected as no part of the same was lying
vacant. At this stage, it would not be out of place to mention that the notice received by
the respondent was not indicative of the fact that the question of allotment of the disputed
premises would be considered on 3rd of January, 1981. It was also all along the case of
the respondent that the notice was served on him at his address although the notice
mentioned the address of the respondent as 103, Chowk Gangadas, Allahabad and on the
back of the notice, there was the process server's report that the respondent was residing
at No. 21, Hamilton Road, Georgetown, Allahabad. According to the respondent, without
considering the objections filed by him, the RC and EO on 24th of January, 1981
declared the vacancy particularly when the respondent himself had appeared before the
RC and EO specifically bringing to his notice that he was in physical occupation of the
disputed premises and nothing was vacant which could be said to be available for
allotment. It was also the case of the respondent that the RC and EO without considering
the objection filed by him passed the order dated 24th of January, 1981 declaring vacancy
in the following manner :-
"The file was put up. The report of RCI seen. On the spot the house was locked. No body
was living. At the main gate a Board of Shri Prasidh Narain Anand was there. Landlord
has appeared. He has made no objection. It is clear that the disputed portion, which is
western portion of the house is vacant because there is no objection from Sri S. N. Anand,
hence vacancy is being notified. To be put up on 31st January for argument on allotment
and orders."
3. A bare perusal of the aforesaid order of the RC and EO passed on 24th of January,
1981 would make it clear that the said order was passed without considering the objection
of the respondent and by even mentioning that the respondent had no objection when it
was all through his case that the objections were submitted before the RC and EO. It is
also an admitted position that the alleged report of the RCI would only show that the
disputed premises was locked at the time of inspection and it did not indicate that nobody
was residing there. Therefore,
@page-SC3132
it was the case of the respondent that the fact that the disputed premises was locked
cannot by any stretch of imagination mean that nobody was residing in the disputed
premises entitling the RC and EO to declare the same vacant for allotment.
4. On 20th of April, 1981, the respondent was heard and he was given time to file
evidence. Thereafter, on 3rd of June, 1981, an order was passed directing the respondent
to file evidence on that very date and the case was adjourned to 3rd of July, 1981 for
arguments on vacancy. On 26th of September, 1981, the respondent and the applicants
were present and were heard and on 18th of November, 1981, the RC and EO passed an
order of allotment in favour of the appellant. Against the aforesaid order of allotment, the
respondent filed a revision petition under Section 18 of the Uttar Pradesh Urban
Buildings (Regulation of Letting, Rent and Eviction) Act, 1972 before the District Judge,
Allahabad, which was, however, dismissed by order dated 4th of March, 1982. Feeling
aggrieved by the allotment order and the dismissal of the revision petition, the respondent
filed a writ petition before the High Court of Judicature at Allahabad wherein a challenge
was made to the allotment order and a prayer was made for quashing the same. By a
judgment and order dated 9th of November, 2004, the High Court had allowed the writ
petition thereby setting aside the order dated 18th of November, 1981 passed by the RC
and EO allotting the disputed premises in favour of the appellant and the order dated 4th
of March, 1982 passed by the District Judge, Allahabad dismissing the revision directed
against the said allotment order. The High Court in the impugned judgment had also
considered the validity of the order dated 24th of January, 1981, declaring vacancy
passed by the RC and EO and held the same to be invalid. It is this judgment of the High
Court, which is impugned in this appeal.
5. We have heard the learned counsel for the parties and examined the judgment of the
High Court and the District Judge as well as the order of allotment passed by the RC and
EO and the order declaring vacancy and other materials on record. Before we consider
the rival submissions made on behalf of the parties, we may, at this stage, record the
findings of the High Court while allowing the writ petition which are as follows :
(i) The report of RCI had only shown that the main gate of the disputed premises was
locked and that if found appropriate, it was the duty of the RC and EO to call the parties
to ascertain the correct position. This by itself did not amount to vacancy. There was
nothing in the report to show that there was vacancy in the house of the premises in
question.,
(ii) The order dated 24th of January, 1981 declaring vacancy did not show that on that
date, either the landlord or any applicant was present.
(iii) It was not clear from the order sheet as to whether the RCI had inspected the
disputed premises and submitted his report on the direction of the RC and EO.
(iv) The RCI report, the order sheet and any other document did not show that any notice
was given to the landlord before inspection by the RCI or that he was made aware of the
RCI Report.
(v) No order directing the landlord to file objection against vacancy was passed.
(vi) In view of sub-rule (3) of Rule 9 of the Rules framed under the U.P.Act No. 13 of
1972 and the case reported in Yogendra Tiwari vs. D.J. Gorakhpur, AIR 1984 SC 1149, it
was essential to issue notice to the landlord so that he could file release application if he
so desired.
(vii) From the orders dated 20.4.1981, 3.7.1981 and 7.8.1981 on the order sheet, it would
be clear that the RC and EO had heard the question of vacancy again.
(viii) The landlord did not file any copy of the release order of 1952.
(ix) Against the order dated 24.1.1981, declaring vacancy, although no challenge was
made independently but the same was challenged by an application for amendment
subsequently filed.
(x) The vacancy declaration order was bad in law for the following reasons :
(a) Inspection was made by the RCI without notice to the landlord.
(b) there was no material or evidence which could justify declaration of vacancy. The
RCI Report, even if it was correct, did not disclose existence of vacancy;
(c) Vacancy was declared without issuing notice to the landlord.
(d) Vacancy declaration order was reconsidered by the RC and EO but no fresh order
@page-SC3133
declaring or holding vacancy was passed by the RC and EO.
(xi) The allotment order was in violation of Section 16(9) of the Act inasmuch as while
making the allotment order, the allottee was not required to pay to the landlord advance
presumptive rent of one month.
6. On the aforesaid findings arrived at by the High Court, the writ petition was allowed.
Before we proceed further, we may also record the findings arrived at by the revisional
court which are as follows :-
(i) Subsequent to the receipt of the Rent Control Inspector, a notice was formally sent to
the landlord who had put in appearance on 3.1.1981 but he did not file any objection nor
had sought time for filing objection.
(ii) There was no objection filed by the landlord as to the vacancy before passing the
order dated 24.1.1981.
(iii) The finding of the RC and EO that the building in dispute was vacant was a finding
of fact not vitiated by any error of jurisdiction.
(iv) There was ample evidence on record to show that the landlord was residing at 103,
Chowk Gangadas, Allahabad and the disputed premises was vacant. The name of the
landlord had been entered in the electoral roll consistently from the year 1966 to year
1980.
(v) The affidavit of Smt. Prabha Shukla, wife of the appellant to the effect that the
disputed premises was let out to different University students was not contradicted by the
landlord.
(vi) All the persons who had applied for allotment had alleged that the disputed premises
was formerly in occupation of one Sri S. K. Misra but even in the objection purported to
have been filed on 24.1.1981, there was no averment that the building in dispute was not
occupied by S. K. Misra or any other person.
These were the findings made by the revisional court while rejecting the revision petition
filed by the respondent.
7. Keeping in mind the findings arrived at by the revisional court and the High Court, let
us now deal with the submissions of the learned counsel for the parties.
8. The learned senior counsel for the appellant Mr. Gupta submitted before us that the
High Court was not justified in interfering with the order dated 24th of January, 1981
declaring vacancy, in the exercise of its writ jurisdiction under Article 226 of the
Constitution. In this context, it was brought to our notice that during the pendency of the
writ petition before the High Court, after almost 20 years, on 18th of February, 2002, an
application praying for amendment of the writ petition for challenging the order dated
24th of January, 1981 by which the vacancy was declared was filed, which was allowed
by the High Court by its order dated 22nd of May, 2002. Against this order of the High
Court, the appellant had filed an application for recall of the said order but the same was
also rejected by the High Court by its order dated 14th of February, 2003. Aggrieved by
the orders of the High Court, the appellant had filed a special leave petition before this
court challenging the aforesaid orders. This court had allowed the special leave petition
by setting aside the orders dated 22nd of May, 2002 and 14th of February, 2003 in the
following manner :-
"On going through the materials on record and keeping in view the limited notice we
ordered when the special leave petition initially came up for orders relating to admission,
the fact that has to be kept into consideration is not even so much as is to what really
transpired on that day in court but how best the situation should be solved and the
interests of justice could be served. On that view of the matter, we are fully satisfied that
the orders of the High Court under challenge are to be set aside and convinced that the
interest of justice can be better served only if the orders dated 22.5.2002 and 14.02.2003
are set aside and the Civil Misc. Writ Petition No. 4621 of 1982 is restored to its file to be
disposed of afresh on merits and in accordance with law, after hearing both the parties
and giving them due opportunity.
Having regard to the further fact that the writ petition is of the year 1982, in the interest
of justice and in order to avoid any further delay, the High Court may ensure the disposal
of the matter as expeditiously as possible, at least within three months from the date of
receipt of a copy of this order.
The appeals are disposed of on the above terms. No costs."
9

. The learned senior counsel for the appellant Mr. Gupta, therefore, sought to argue before
us that by virtue of the AIR 1985 SC 1635

@page-SC3134
order passed by this court in the aforesaid special leave petition, setting aside the
aforementioned two orders of the High Court, the fact of existence of vacancy had
attained finality. The learned senior counsel thus submitted that it was not open to the
High Court to adjudicate upon the question of vacancy after the decision of this court and
also in view of the concurrent findings of fact of the RC and EO and the revisional court.
The learned senior counsel for the appellant Mr. Gupta also sought to argue that it was
not open to the High Court to reconsider the question of vacancy which had been fully
answered by the RC and EO and affirmed by the revisional court in view of the decision
of this court in Ganpat Roy and others vs. A.D.M. and others [(1985) 2 SCC 307], and
that the High Court was not justified in not following the dictum of Ganpat Roy's case
merely because it had been referred to a larger Bench.
10. These submissions of the learned senior counsel for the appellant were hotly
contested by the learned senior counsel appearing on behalf of the respondent.
11. After considering the rival submissions of the parties, we may note that the question
whether the respondent was given sufficient opportunity to object and lead evidence to
disprove the fact of vacancy was taken into consideration by the High Court and from the
materials on record and the evidence adduced by the parties, it was open to the High
Court, even in the exercise of its power under Article 226/227 of the Constitution, to
come to a finding of fact that such opportunity was not at all given to the respondent.
Even assuming that the High Court was wrong in coming to a conclusion of fact that no
opportunity was given to the respondent to file objections, then also, we are not inclined
to interfere with the judgment of the High Court in the exercise of our discretionary
power under Article 136 of the Constitution for the reasons stated hereinafter.
12. First, the finding of the High Court, as noted herein-earlier, in clause (vii) viz., that
from the orders dated 20th of April 1981, 3rd of July, 1981 and 7th of August, 1981 on
the order sheet, it was clear that the RC and EO had heard the question of vacancy again
is very crucial. Having done so, it was imperative that the RC and EO should have passed
a fresh order to the effect whether the disputed premises was vacant or not. However, in a
rather peculiar and strange manner, the RC and EO proceeded and fixed a date for
passing of the allotment order on the basis of the order dated 24th of January, 1981. We
may note at this stage that the provisions regarding allotment of vacant buildings are
governed by Sections 12, 16 and 34(8) of the U.P. Urban Buildings (Regulation of
Letting, Rent and Eviction) Act, 1972 (in short "the Act") and the Rules framed under the
said Act. The passing of the allotment order without declaring vacancy was a gross error
committed by the RC and EO because under the scheme of the provisions of the Act, the
preliminary step was to declare a vacancy, which, in our view, was not done and even if
done, the same was not in a bona fide manner. The RC and EO should have at least
conveyed their decision on that point.
13. Secondly, the RCI and the RC and EO while submitting the report and passing the
order declaring vacancy respectively did not adhere to the provisions governing the
allotment of vacant buildings, as enumerated herein-above. We find from record that no
neighbour was enquired to ascertain vacancy, much-less two neighbours as mandated by
the Rules.
14. Thirdly, the finding of the High Court that simply because the gate was locked, it was
no ground to conclude that the disputed premises was vacant cannot be ignored. The
learned senior counsel for the appellant contended that in this case, a deemed vacancy
had occurred and ingredients of Section 12 of the Act which deals with deemed vacancy
of buildings were satisfied. As rightly pointed out by the High Court in the impugned
judgment, the fact that the gate was locked cannot be a conclusive proof to hold that the
respondent had removed his effects there from or that he had allowed it to be occupied by
any person who was not a member of his family or even that he and members of his
family had taken up residence elsewhere. In our view, the question of deemed vacancy
cannot arise at all in view of the facts, which would be evidenced from the order of the
RC and EO and the report of the RCI. From the said order of the RC and EO, it does not
appear that the respondent had substantially removed his effects from the disputed
premises. As stated hereinabove, the fact of the gate being locked and the absence of the
@page-SC3135
respondent at the time of the inspection would not mean that substantial removal of
effects of the respondent had been made. In view of our discussions made hereinabove,
we are not of the view that any deemed vacancy had occurred and on this ground, we are
not inclined to interfere with the judgment of the High Court.
15

. As regards the objection raised by the learned senior counsel for the appellant to the
effect that the High Court should have followed the dictum in Ganpat Roy's case (supra)
the same is not acceptable. At that time, the matter was referred to a larger bench. The
decision was, therefore, debatable and not conclusive. But now all doubts regarding the
dictum in Ganpat Roy's case [supra] have been set at rest by a decision of this court in
Achal Mishra vs. Rama Shanker Singh and Ors. [(2005) 5 SCC 531], wherein this court
in Para 14 observed as under:- AIR 1985 SC 1635

"It is thus clear that an order notifying a vacancy which leads to the final order of
allotment can be challenged in a proceeding taken to challenge the final order, as being an
order which is a preliminary step in the process of decision-making in passing the final
order. Hence, in a revision against the final order of allotment which is provided for by
the Act, the order notifying the vacancy could be challenged. The decision in Ganpat Roy
case which has disapproved the ratio of the decision in Tirlok Singh and Co. cannot be
understood as laying down that the failure to challenge the order notifying the vacancy
then and there, would result in the loss of right to the aggrieved person of challenging the
notifying of vacancy itself, in a revision against the final order of allotment. It has only
clarified that even the order notifying the vacancy could be immediately and
independently challenged. The High Court, in our view, has misunderstood the effect of
the decision of this court in Ganpat Roy case and has not kept in mind the general
principles of law governing such a question as expounded by the Privy Council and by
this court. It is nobody's case that there is anything in the Act corresponding either to
section 97 or to section 105(2) of the Code of Civil Procedure, 1908 precluding a
challenge in respect of an order which ultimately leads to the final order. We overrule the
view taken by the Allahabad High Court in the present case and in Kunj Lata v. Xth ADJ,
that in a revision against the final order, the order notifying the vacancy could not be
challenged and that the failure to independently challenge the order notifying the vacancy
would preclude a successful challenge to the allotment order itself. In fact, the person
aggrieved by the order notifying the vacancy can be said to have two options available.
Either to challenge the order notifying the vacancy then and there by way of a writ
petition or to make the statutory challenge after a final order of allotment has been made
and if he is aggrieved even thereafter, to approach the High Court. It would really be a
case of election of remedies." AIR 1985 SC 1635
AIR 1976 SC 1988

16. In the present case, the High Court had permitted the respondent to amend the writ
petition whereby he sought to challenge the order dated 24th of January, 1981 declaring
vacancy. Such order of the High Court allowing the amendment was challenged before
this court and this court had remanded the matter to the High Court setting aside such
order requesting the High Court to decide the writ petition afresh. Since this Court had
remanded the matter to the High Court for a fresh decision on the question whether the
amendment should be allowed or not along with the merits of the writ petition, it cannot
be said that the High Court was in error after the order of this court to allow the
application for amendment on facts as this court did not decide the merits as to whether
the application for amendment should be allowed or not. We have already quoted
hereinearlier the substantial portion of the order of this court in that special leave petition
and from the same, it is clear that it was passed without going into the merits of the
orders allowing the application for amendment of the writ petition and this court had
simply set aside the said orders of the High Court remanding the matter to the High Court
for disposal of the same afresh and in accordance with law after hearing both the parties
and after giving them due opportunity. The High Court by the impugned judgment had
simply followed the directions made by this court in the order passed in that special leave
petition, as quoted hereinearlier, and came to a conclusion that the order dated 24th of
January, 1981 declaring vacancy was bad in law. That apart, it is clear from the decision
of this court in Achal Mishra's case [supra] that it was open to the
@page-SC3136
respondent to challenge the order declaring vacancy in the writ petition against the
allotment order even though the said order was not challenged independently there and
then. Therefore, the High Court was fully justified in considering the validity of the
vacancy declaration order while hearing the writ petition against the allotment order. In
view of our discussions made hereinabove, we are, therefore, of the view that since this
court had not decided that special leave petition on merits, it cannot be said that the
vacancy declaration order had attained finality. Therefore, the High Court was fully
justified in considering the question of vacancy, which was a core issue in the writ
petition because if the vacancy declaration itself was bad in law, the consequent allotment
order which was passed cannot be said to be not in violation of Section 16 of the Act. In
any view of the matter, the question regarding vacancy was a core issue in the writ
petition and in our view, the High Court, on consideration of the materials on record was
entitled to look into it by invoking its writ jurisdiction under Article 226 of the
Constitution. Since the order passed by the High Court was based on consideration of
facts, which cannot be interfered with except in exceptional cases, we do not find any
reason to interfere with the same under Article 136 of the Constitution of India.
17. There is another aspect of this matter for which, in the facts and circumstances of this
case, we would not exercise our discretionary power under Article 136 of the
Constitution. The vacancy declaration order and the consequent allotment in favour of the
appellant was made in the manner indicated herein earlier and the appellant stormed into
the disputed premises more than two decades back and started enjoying the same without
paying a single penny in respect of the same. It was only after the judgment of the High
Court that he had deposited the amount as directed by the High Court. Therefore, we do
not find any reason to interfere with the impugned judgment of the High Court under
Article 136 of the Constitution in the facts and circumstances of the present case.
18. For the foregoing reasons, we do not find any merit in this appeal. The appeal is thus
dismissed. There will be no order as to costs. The appellant is, however, granted time to
vacate the disputed premises by 30th of November, 2008 subject to filing an usual
undertaking before this court within one month from this date.
Appeal dismissed.
AIR 2008 SUPREME COURT 3136 "Girish Singh v. State of Uttaranchal"
(From : 2007 Cri LJ 4466 (Uttarakhand))
Coram : 2 Dr. A. PASAYAT AND P. SATHASIVAM, JJ.
Criminal Appeal No. 849 of 2008 (arising outof SLP (Cri.) No. 4212 of 2007), D/- 9 -5
-2008.
Girish Singh v. State of Uttaranchal.
(A) Penal Code (45 of 1860), S.304, Part II, S.304A - CULPABLE HOMICIDE -
NEGLIGENCE - EVIDENCE - WITNESS - Culpable homicide not amounting to murder
- Accused pushed down deceased from hill - Death of deceased - Credible evidence of
eye-witnesses, brother and son of deceased - Conviction of accused u/S.304 Part II - Was
proper - S.304-A not attracted. (Paras 9, 10)
(B) Penal Code (45 of 1860), S.304A - NEGLIGENCE - Protection under - Availability -
There should be neither intention nor knowledge to cause death - Presence of any of these
two elements - Would not attract provisions u/S.304-A. (Paras 8, 9)

Radhe Shyam Sharma, (A.C.) for Appellant; Sunil Kumar Singh, Anil Kumar Singh and
Jatinder Kumar Bhatia, for Respondent.
Judgement
Dr. ARIJIT PASAYAT, J. :- Leave granted.
2. Challenge in this appeal is to the judgment of a learned Single Judge of the Uttaranchal
High Court dismissing the appeal filed by the appellant who was convicted for offence
punishable under Section 304 Part II of the Indian Penal Code, 1860 (in short 'IPC') and
was sentenced to undergo imprisonment for five years and pay a fine of Rs. 5,000/- with
default stipulation.
3. Background facts in a nutshell are as follows:
Sageer Ansari (hereinafter referred to as the 'deceased') was a carpenter, who used to live
in Hotel Hari Om in Uttarkashi. On 27.3.2005, he was coming from Hari Om Hotel
towards Uttarkashi town. Accused/ appellant Girish Singh was coming from opposite
direction towards Sageer Ansari-deceased. When both of them reached near Tambakhani
they had some altercations
@page-SC3137
between them. Suddenly, accused-appellant Girish Singh pushed deceased Sageer Ansari
from the road. Consequently, Sageer Ansari fell down from the hill and suffered injuries
due to the fall from Uttarkashi - Tehri Road. The incident took place at 1.00 p.m. PW3
Israil Mian, brother of the deceased, and PW4 Mazhar Ansari, son of the deceased, who
were following Sageer Ansari (deceased), witnessed the incident. The two rushed to the
place of incident and took the injured to the hospital where he succumbed to the injuries
suffered by him in the incident. PW3 Israil Mian, brother of the deceased, lodged first
information report (Ext. A-3) with the police station. Investigation was undertaken and on
completion of investigation charge sheet was filed. As accused abjured guilt, trial was
held.
4. Placing reliance on the evidence of two eye witnesses i.e. Israil Mian (PW3) and
Mazhar Ansari (PW4) (brother and son of the deceased respectively), the Trial Court
found the accused-appellant guilty and convicted him and imposed sentence as noted
above.
5. In appeal before the High Court the stand of the accused was that this is not a case
where Section 304 Part II IPC is applicable. On the other hand, this is a case where even
if the prosecution version is accepted in toto, it would, at the most, an offence punishable
under Section 304A IPC. Another plea related to acceptance of the evidence of PWs 3
and 4 on the ground that they are related to the deceased. Both the pleas were rejected
and appeal was dismissed. The stand taken before the High Court was reiterated by the
learned counsel for the appellant.
6. In response, learned counsel for the respondent-State supported the judgments of the
Trial Court as upheld by the High Court.
7. The plea relating to relative's evidence has no substance, when such evidence has
credence it can be acted upon.
8. Coming to the plea of the applicability of Section 304-A, it is to be noted that the said
provision relates to death caused by negligence. Section 304-A applies to cases where
there is no intention to cause death and no knowledge that the act done in all probability
will cause death. The provision relates to offences outside the range of Sections 299 and
300 IPC. It applies only to such acts which are rash and negligent and are directly the
cause of death of another person. Rashness and negligence are essential elements under
Section 304-A. It carves out a specific offence where death is caused by doing a rash or
negligent act and that act does not amount to culpable homicide under Section 299 or
murder in Section 300 IPC. Doing an act with the intent to kill a person or knowledge
that doing an act was likely to cause a person's death is culpable homicide. When the
intent or knowledge is the direct motivating force of the act, Section 304-A IPC has to
make room for the graver and more serious charge of culpable homicide.
9. In order to be encompassed by the protection under Section 304-A there should be
neither intention nor knowledge to cause death. When any of these two elements is found
to be present, Section 304-A has no application.
10. When the background facts are considered in the light of the legal principles set out
above, the inevitable conclusion is that stand of the appellant is clearly unsustainable.
11. The appeal is without merit, deserves dismissal, which we direct.
Appeal dismissed.
@page-SC3137
AIR 2008 SUPREME COURT 3137 "FCS Software Solutions Ltd. v. LA Medical
Devices Ltd."
(From : Punjab and Haryana)
Coram : 2 C. K. THAKKER AND D. K. JAIN, JJ.
Civil Appeal No. 4271 of 2008 (arising out of SLP (C) No. 21123 of 2007), D/- 9 -5
-2008.
FCS Software Solutions Ltd. v. LA Medical Devices Ltd. and Ors.
Companies Act (1 of 1956), S.457, S.454 - COMPANY - AUCTION SALE - Auction
sale of property of Com pany - Highest bid of appellant accepted and sale confirmed -
However, certain facts related to valuation of movable and immovable properties,
fixation of reserve price, non-inventory of Plant and Machin ery - Found to be not set out
in procla mation of sale nor disclosed at time of sale notice - Order by Company Judge
directing fresh auction - No complaint can be made against it - Moreso, when appellant
failed to avail opportunity to participate in fresh auction.
In the instant case the sale of movable and immovable properties was ordered. The bid of
the appellant of Rs. 1.47 Crores was
@page-SC3138
highest and was accepted by the official liq uidator. Sale confirmed in his favour on
making entire payment. However, certain facts related to valuation of movable and
immovable properties, fixation of reserve price, non-inventory of plant and machin ery,
etc., were found to be brought to the notice of intending purchasers as these were not set
out in the proclamation of sale nor were disclosed at the time of sale notice. The
Company Judge considered the objec tions of the other bidders and having prima facie
satisfied, ordered fresh auction. When fresh bids were received, it was found that the
highest offer was found that the high est offer was of respondent Society which was of
Rs. 3.5 crores. The Company Judge extend an opportunity to the appellant to raise its bid.
However, the appellant was adamant to get the property for Rs. 1.47 crores on the ground
that the said offer was highest and all the proceedings taken by the Official Liquidator
and Company Judge thereafter were totally illegal and unlawful.
Held, the approach of the Company Judge should be to get highest price so as to sat isfy
maximum claims against the Company in liquidation. Thus the procedure followed by the
Company Judge, therefore, cannot be said to be illegal. (Paras 21, 34)
When the highest bid of Rs. 3.5 crores made by the respondent was accepted, op
portunity was afforded to the appellant. It, however, did not avail such opportunity. Af ter
the fresh auction, sale deed was ex ecuted, possession was handed over to re spondent, it
has incurred expenses. At this stage, if the sale is set aside, serious preju dice will be
caused to respondent-Society. Since the appellant had deposited the en tire amount, the
respondent directed to pay an amount of Rs. 30 lacs to the appellant which would serve
the ends of justice. Pay ment of Rs. 30 lacs will serve as a "solatium to the purchaser for
his trouble and disap pointment for the loss of that which is per haps a good bargain".
(Paras 35, 36)
Cases Referred : Chronological Paras
2004 AIR SCW 3263 : AIR 2004 SC 3392 : 2004 AIR - Jhar HCR 2005 (Ref.) 33
2000 AIR SCW 4225 : AIR 2000 SC 3642 : 2000 CLC 2176 (Ref.) 27
2000 AIR SCW 2465 : AIR 2000 SC 2346 : 2000 CLC 1647 (Ref.) 28, 30
(1996) 85 Comp Cas 788 (SC) (Ref.) 29
(1996) 85 Comp Cas 792 (SC) (Ref.) 29
AIR 1974 SC 1331 (Ref.) 26
AIR 1970 SC 2037 (Ref.) 25
(1899) ILR 26 Cal 449 (FB) 36
Dr. A. M. Singhvi, Sr. Advocate, S. K. Dubey, Sunil Sharma and K. V. Mohan, for
Appellant; Ranjit Kumar, Sr. Advocate, Rajendra Singhvi, Mrs. Madhur Dadlani, Ms.
Maitreyi Singhvi, Brij Bhusan, Arun Kathpalia, Rakesh Kumar and Ms. Shipra Ghose,
for Respondents.
Judgement
C. K. THAKKER, J. :- Leave granted.
2. The present appeal is filed by the appellant against an order passed by the Division
Bench of the High Court of Punjab and Haryana on October 15, 2007 in Com pany
Appeal No. 10 of 2006. By the said order, the Division Bench of the High Court
dismissed the appeal filed by the appellant-herein and confirmed the order passed by the
Company Judge on February 16, 2006 in Company Application No. 178 of 2005 in
Company Petition No. 42 of 1999.
3. Few facts of the case are that La Medical Devices Ltd. respondent No. 1 went into
liquidation. Official Liquidator was ap pointed by the Court who is joined as Re spondent
No. 1 in the present proceed ings. In view of the liquidation of the Com pany and dues to
be paid by it, proceedings were initiated for sale of property of the Com pany. Sale notice
was issued by the Com pany Court on October 19, 2004 which was published in various
newspapers inviting sealed tenders for the sale of property of the Company situated at
NOIDA (U.P.). Twelve bids were received which were opened on November 16, 2004.
The bid of the appel lant for Rs. 1.47 crores for immovable as well as movable property
was the highest. One of the creditors, namely, Pradeshiya Industrial and Investment
Corporation of U.P. Ltd. (PICUP) granted 'no objection' to the sale-price. Since the
appellant was the highest bidder, it wrote a letter to the Offi cial Liquidator on December
19, 2004 fol lowed by a reminder dated January 20, 2005 requesting him to expedite the
process and issue letter of acceptance of the offer of the appellant so that possession of
the Unit can be given to the appellant and the property could be made ready for
commencement of production. It is the case of the appellant that there was no reply by
the Official Liquidator to the appellant. The appellant,
@page-SC3139
therefore, wrote a letter to the Com pany Judge on January 27, 2005, complain ing that
though it was the duty of the Offi cial Liquidator to accept the highest offer submitted by
the appellant, no action had been taken by the Official Liquidator and there was delay in
the process of finaliza tion of acceptance of bid. The appellant also complained about the
threat administered by the Official Liquidator. According to the appellant, thereafter on
February 15, 2005, the Official Liquidator accepted the bid of the appellant for Rs. 1.47
crores for immovable property. The appellant on its own had forgone its claim of leased
ma chinery. The Official Liquidator also in structed the appellant to deposit 25% of the
bid amount for immovable property within fifteen days. The appellant, however, depos
ited the said amount on the same day, i.e. February 15, 2005. According to the ap pellant,
the Company Judge having found the auction in accordance with law and for adequate
price and there being no other objection, confirmed the auction sale in favour of the
appellant-Company on March 24, 2005. The Company Judge also di rected the Official
Liquidator to hand over possession of the Unit by executing sale deed in favour of the
appellant after receiving full and final payment within one month. The Official Liquidator
conveyed the appellant vide his letter dated April 4, 2005 that an order was passed by the
Company Judge in favour of the appellant. The appellant was also asked to deposit the
rest of the amount immediately. On receiving the letter dated April 4, 2005 from the
Official Liquidator, the appellant deposited the remaining amount on April 12, 2005. The
appellant thereby became entitled to receive posses sion and execution of sale deed in
respect of immovable property of the Company. By a communication dated April 21,
2005, the Official Liquidator informed the appellant that possession of the property
would be handed over to the appellant on May 6, 2005 at 11.30 a.m.
4. According to the appellant, thereaf ter the Official Liquidator did not act legally and in
accordance with law. On May 6, 2005, though the officers of the appellant waited at the
site for getting possession of the prop erty, neither the Official Liquidator nor his
representative turned up to hand over possession of the property to the appellant. The
appellant-Company, hence, sent a telegraphic notice to the Official Liquidator and
requested him to immediately comply with the order of the Company Judge confirming
sale and handing over possession to the ap pellant. Instead of complying with the or der
of the Court and handing over pos session of the property, the Official Liqui dator sent a
letter purported to have been written on May 5, 2005, stating therein that possession
would not be given to the ap pellant on May 06, 2005 as higher bid of Rs. 1.55 crores had
been received and order for handing over possession to the appel lant had been stayed by
the Hon'ble Court. The appellant made enquiries and it was found that it was at the
instance of the Offi cial Liquidator himself that Company Ap plication No. 178 of 2005
before the Com pany Judge was moved and he created ob struction in delivery of
possession of prop erty to the appellant on the alleged ground that he had received higher
offer. The ap pellant stated that other two persons also offered more amount. The
appellant, in the circumstances, filed Company Applica tion No. 407 of 2005 under Rule
9 of the Company Court Rules, 1959 read with Sec tion 151 of the Code of Civil
Procedure, 1908 stating therein that it had paid full amount, sale was confirmed and in
spite of the order by the Company Judge, possession had not been handed over to the
appellant. A prayer was made to direct Official Liquidator to hand over possession of
property to the appellant. Meanwhile other offers were also received by Official
Liquidator. One Satish Choudhary offered an amount of Rs.2.10 crores. Keeping in view
all the facts, the learned Company Judge, vide his order dated February 16, 2006 directed
re-sale of property by issuing fresh advertisement in the newspapers mentioned in the
order. It was also observed that the reserve price would be fixed at Rs.2.10 crores.
Tenders should reach in the office of Official Liqui dator latest on March 22, 2006, and
would be opened in Court at 1.45 p.m. on March 23, 2006 and inter-se bidding would be
per mitted at that time.
5. In pursuance of the above direction, advertisement was issued. Official Liquida tor
received offers. The highest offer was by Nice Society-respondent No. 3 herein for Rs.
3.5 crores and in the circumstances, bid of respondent No. 3 was accepted. The appellant
in the meanwhile challenged the order of the Company Judge and, as observed
@page-SC3140
earlier, the appeal was dismissed by the Division Bench. It is this order which is
challenged in the present appeal.
6. Notice was issued by this Court on February 23, 2007. Affidavits and further affidavits
were filed thereafter. Considering the nature of litigation, the Office was directed to place
the matter for final hear ing on a non-miscellaneous day and that is how the matter has
been placed before us.
7. We have heard the learned counsel for the parties.
8. The learned counsel for the appellant contended that the Company Judge as well as the
Division Bench of the High Court were wholly wrong in setting aside the auction sale in
favour of the appellant. It was submitted that pursuant to sale notice, tenders were
invited, twelve persons offered their bids. The bid of the appellant was high est. In
consonance with law, therefore, the said bid was accepted and the appellant deposited
amount of 25% as required by law. It also paid the remaining amount of 75%. Sale was
confirmed in favour of the appellant and direction was issued by the Company Judge to
the Official Liquidator to hand over possession of the property to the appellant. The
Official Liquidator, how ever, with mala fide intention and oblique motive, refused to do
so.
9. According to the counsel, once the bid was accepted and sale was confirmed, it could
not be set aside except on the grounds of fraud, material irregularity, etc. It is not even the
case of the Official Liq uidator, submitted the counsel, that there was fraud or material
irregularity in sale and hence, sale could not have been set aside. It was urged that the
only ground put for ward by the Official Liquidator was that he had received a higher
offer of Rs.1.55 crores from another person. The said offer was received after about seven
months. The difference in price was 5.44%.
10. According to the counsel, judicial notice can be taken that price of real estate
increases day by day and increase of 5% after seven months could not justify the Court in
setting aside auction sale which was conducted in consonance with law. On that ground
alone, therefore, both the or ders are liable to be set aside by ordering handing over
possession of property to the appellant.
11. It was also submitted that mala fide action on the part of the Official Liquidator was
apparent and from the records and proceedings, it was clearly established. On May 6,
2005, the officers of the appellant remained present at the site to accept pos session, but
the Official Liquidator did not come. A letter, said to have been written on May 5, 2005
was received by the appellant belatedly wherein the only ground given by the Official
Liquidator was that he had re ceived a higher bid of Rs. 1.55 crores and the Company
Judge had issued stay order. The so-called order passed by the Company Judge was also
not sent along with the let ter.
12. It was alleged that the Official Liquidator did not hand over possession since the
appellant did not oblige him by succumbing to his demands. It was asserted that against
the said Official Liquidator, the Central Bureau of Investigation (CBI) had instituted
proceedings alleging corrup tion and he was also arrested in that con nection.
13. Regarding bid of respondent No.3-Society for Rs.3.5 crores, it was submit ted that it
is well settled that auction sales cannot be set aside on the ground that higher offers are
received from other bid ders after confirmation of sale. If this is al lowed and sales are set
aside, there is no end to it. In absence of illegalities or material irregularities, credibility
of court sales cannot be doubted nor such sales be set aside. If price is the only
consideration, today the property is worth Rs.5.5 crores. In that case, sale in favour of
respondent No.3 should also be set aside and fresh auc tion must be ordered.
14. It was stated that the appellant was a bona fide purchaser. It was not even the
allegation of the Official Liquidator or any other bidder that highest bid of the ap pellant
for Rs.1.47 crores was in any way improper, insufficient or inadequate. The appellant,
hence, cannot be deprived of the fruits to which it was otherwise entitled to. On all these
grounds it was submitted that the order passed by the courts below deserve to be set
aside.
15. On behalf of the Official Liquidator, an affidavit is filed. It may, however, be stated
that the present incumbent is differ ent than the one who was in the office at the relevant
time. The deponent denied the allegations levelled by the appellant but stated that in view
of the higher offer
@page-SC3141
received by the then Official Liquidator and stay granted by the Company Judge, pos
session was not handed over to the appel lant and the said action could not be said to be
illegal or contrary to law. It was stated that as per the order of the Company Judge, fresh
tenders were invited and in the said process, respondent No. 3-Society offered Rs.3.5
crores which was accepted and no interference is called for.
16. A counter-affidavit is also filed by respondent No. 3 stating that the appellant had not
approached the Court with true and full facts. Sale notice which was issued on October
19, 2004 was incomplete and in valid. It did not state valuation of movable and
immovable properties. Reserve price was also not fixed. No inventory of plant and
machinery was made. There was no full description of movable properties. Neces sary
material was not made available at the site nor in the office of the Official Liquida tor. In
view of those irregularities, the property could not fetch fair price. When all the above
defects were brought to the notice of the Company Judge, the Company Judge was
satisfied that the process under taken was not proper and hence fresh bids were invited.
17. It was also submitted that no formal order of confirmation in favour of the appellant
was at any time made by the Court. But even if it is assumed for the sake of argument
that the order of the Com pany Judge issuing direction to the Official Liquidator to hand
over possession to the appellant can be said to be an order of con firmation, it would not
take away the power of the Company Judge to invite fresh ten ders if sale was not held in
accordance with law. Moreover, if auction sale was confirmed by the Court on March 24,
2005, the appel lant was bound to pay the remaining amount of 75% before 15th day of
confirmation. Admittedly, the appellant failed to pay the remaining amount within the
stipulated period. It, therefore, cannot claim any ben efit on the basis of such auction sale.
18. The counsel urged that even other wise this Court has held in several cases that the
Court has power to set aside even confirmed sale if it is satisfied that the prop erty would
have fetched higher price. The approach of the Court in such matters is to ensure that the
property must fetch maxi mum price which would benefit the Com pany in clearing its
dues and liabilities towards its creditors, contributors and work ers. In the final analysis,
the auction sale in favour of respondent No. 3 for an amount of Rs.3.5 crores which was
substan tially higher than the bid of the appellant for Rs.1.47 crores may not be interfered
with.
19. It was further stated that when re spondent No. 3 made an offer of Rs.3.5 crores, the
Court, in fairness, extended op portunity to the appellant if it wanted to participate in the
proceedings and willing to offer higher price. The appellant, how ever, refused to take
part as also refused to pay anything more than what was offered earlier. It is, therefore,
not open to the ap pellant to complain and insist to get owner ship rights and possession
of property for Rs.1.47 crores.
20. It was stated that respondent No. 3 has paid full amount, got sale deed executed in its
favour and had spent substantial amount thereafter and on that ground also, the Court
may not exercise discretionary and equitable jurisdiction under Article 136 of the
Constitution.
21. Having heard the learned counsel for the parties, in our opinion, no case has been
made out by the appellant against the order passed by the High Court. From the facts
stated above, it is clear that in Novem ber, 2004, the bid of the appellant was highest and
was accepted by the Official Liquidator. But it is also clear that certain facts which were
necessary to be brought to the notice of intending purchasers were not set out in the
proclamation of sale nor were disclosed at the time of sale notice. They related to
valuation of movable and immovable properties, fixation of reserve price, non-inventory
of plant and machin ery, etc. The attention of the Company Judge was invited by other
bidders by filing Com pany Applications. The Company Judge considered the objections
and having prima facie satisfied, ordered fresh auction. We find no illegality in the said
approach. When fresh bids were received, it was found that the highest offer was of
respondent No. 3-Soci ety which was of Rs.3.5 crores. The Com pany Judge extended an
opportunity to the appellant to raise its bid. It, however, appears that the appellant was
adamant to get the property for Rs.1.47 crores on the ground that the said offer was
highest and all the proceedings taken by the Offi cial Liquidator and Company Judge
thereafter
@page-SC3142
were totally illegal and unlawful. In our opinion, the respondents are right that in such
cases, the approach of the Company Judge should be to get highest price so as to satisfy
maximum claims against the Com pany in liquidation. The procedure followed by the
Company Judge, therefore, cannot be said to be illegal.
22. It may be observed at this stage that even before the Division Bench, such
opportunity was afforded to the appellant to raise its bid but it was not availed of by the
appellant.
23. The Division Bench, in the im pugned order, noted;
"Even during the course of proceedings in this appeal, we had specifically asked the
learned counsel for the appellant as to whether the appellant was willing to go for inter-se
bidding to which he flatly declined".
24. In this connection, we may refer to some of the decisions of this Court to which our
attention has been invited.
25
. In M/s. Navalkha and Sons v. Sri Ramanya Das and Ors., 1969 (3) SCC 537, it was held
by this Court that the principles which should govern confirmation of sale are well-
settled. Where the acceptance of the offer by the Commission is subject to confirma tion
of the Court, mere acceptance of offer by the Commission would not confer vested right
to the property in favour of the bidder. Condition of confirmation by Court operates as a
safeguard against the property being sold at an inadequate price, whether or not it is a
consequence of any irregularity or fraud in the conduct of the sale. It is the duty of the
Court to satisfy itself about the proper valuation. But once the Court comes to the
conclusion that the price offered is adequate, no subsequent higher offer can constitute a
valid ground for refusing con firmation of the sale or offer already received. AIR
1970 SC 2037

26

. In Kayjay Industries (P) Ltd. v. M/s. Asnew Drums (P) Ltd. and Ors., (1974) 2 SCC
213, this Court held that it is the duty of the Court to accept the highest bid and the Court
is not bound to go on adjourning the sale on the basis of valuation report. Referring to
and relying on Navalkha, the Court stated that in public sales, the authority must protect
interest of the parties keeping in view the fact that a Court sale is a forced sale and,
notwithstanding the competitive element of public auction, the best price is not often
forthcoming. AIR 1974 SC 1331

27

. In Union Bank of India v. Official Liquidator, High Court of Calcutta and Ors., (2000) 5
SCC 274, this Court observed that in auc tion sale of the property of the Company which
is ordered to be wound up, the Com pany Court acts as a custodian for the in terest of the
Company and its creditors. It is the duty of the Company Court to satisfy itself as to
reasonableness of price by dis closing valuation report to secured creditors of the
Company and other interested per sons. It was further held that the Court should exercise
judicial discretion to en sure that sale of property should fetch adequate price. For
deciding what would be reasonable price, valuation report of an expert is essential. The
Company Judge himself must apply his mind to the valua tion report. The Court observed
that the High Court did not interfere with the auction sale on the ground of sympathy for
the workers which was not proper. The auction sale was, therefore, set aside by this Court
and Offi cial Liquidator was directed to re-sell the property after obtaining fresh valuation
re port and after furnishing copy of such re port to secured creditors. 2000 AIR
SCW 4225

28

. In Divya Manufacturing Company (P) Ltd. v. Union Bank of India and Ors., (2000) 6
SCC 69, this Court held that even con firmed sale can be set aside. In that case, highest
bid by a party was accepted by the Court and the sale was confirmed, but be fore
possession was delivered to the auc tion purchaser and execution of sale deed, other
parties offered much higher price. The High Court required the subsequent bidders to
deposit an amount of 25% which was done. Considering the facts in their entirety the
High Court set aside the confirmation of past highest bid. The said action was challenged
in this Court. 2000 AIR SCW 2465

29. This Court held that in an appropri ate case, even confirmed sale can be set aside. The
Court in this connection, relied upon earlier two decisions in LICA (P) Ltd. (1) v. Official
Liquidator, (1996) 85 Comp Cas 788 (SC) and LICA (P) Ltd. (2) v. Of ficial Liquidator,
(1996) 85 Comp Cas 792 (SC).
30. The learned counsel for the appel lant is no doubt right in submitting that in Divya,
there was a specific condition (Clause 11) which empowered the Court to
@page-SC3143
set aside confirmed sale "in the interest of creditors, contributors and all concerned and/or
public interests".
31. But the Court put the matter on principle and stated;
"It is the duty of the Court to see that the price fetched at the auction is an adequate price
even though there is no suggestion of irregularity or fraud".
(Emphasis supplied)
32. It proceeded to observe; "Confirmation of the sale by a Court atgrossly inadequate
price, whether or not it is a consequence of any irregularity or fraud in the conduct of
sale, could be set aside on the ground that it was not just and proper exercise of judicial
discretion. In such cases, a meaningful intervention by the Court may prevent, to some
extent, un derbidding at the time of auction through Court".
(Emphasis supplied)
33

. In Gajraj Jain v. State of Bihar and Ors., (2004) 7 SCC 151, this Court reiterated that in
ab sence of valuation report and reserve price, the auction sale becomes only a pretence.
If there is no proper mechanism and if the intending purchasers are not able to know
details of the assets or itemised valuation, auction sale cannot be said to be in accor dance
with law. If publicity and maximum participation is to be attained, all bidders must know
the details of the assets and the valuation thereof. 2004 AIR SCW 3263

34. In the present case, it was alleged that there were several irregularities in the first
auction. The tender notice did not state valuation of movable and immovable prop erty;
reserve price was not fixed, inventory of plant and machinery was not made avail able,
etc. If on consideration of these facts, the Company Judge ordered fresh auction, in our
considered opinion, no complaint can be made against such action.
35. In our opinion, the submission of the learned counsel for respondent No. 3 is also
well-founded that when its highest bid of Rs.3.5 crores was accepted, oppor tunity was
afforded to the appellant. It, however, did not avail such opportunity. The counsel is also
right in referring to sub sequent events that after the fresh auction, sale deed was
executed, possession was handed over to respondent No.3, it had in curred expenses. If at
this stage, the sale is set aside, serious prejudice will be caused to respondent No. 3-
Society.
36. At the same time, however, from the facts it is clear that the appellant's bid was
accepted in November, 2004. Immediately, it had deposited 25% amount. The appel lant
also deposited remaining amount of 75% on April 12/13, 2005. It would, there fore, be
appropriate if we direct respondent No. 3 to pay an amount of Rs.30 lacs to the appellant
which in our opinion would serve the ends of justice. Payment of Rs.30 lacs will serve as
a "solatium to the purchaser for his trouble and disappointment for the loss of that which
is perhaps a good bar gain". [Vide Chundi Charan v. Bankey Behary, (1899) ILR 26 Cal
449 (FB)].
37. Before parting, we may clarify that serious allegations have been levelled by the
appellant against the then Official Liquidator. It was also stated that Central Bureau of
Investigation (CBI) has insti tuted criminal proceedings alleging cor ruption against the
Official Liquidator and he was arrested. We are disposing of the present appeal as in our
opinion, the order passed by the Company Jude and con firmed by the Division Bench of
the High Court are in consonance with law. But we may not be understood to have
expressed any opinion on the allegations levelled by the appellant against the Official
Liquida tor. As and when the matter comes up for consideration before an appropriate
Court/ Authority, it will be decided on its own mer its irrespective of the disposal of this
appeal by us.
38. For the foregoing reason, the appeal is partly allowed to the extent indicated above.
Order accordingly.
AIR 2008 SUPREME COURT 3143 "Southern Steel Ltd. v. Jindal Vijayanagar Steel
Ltd."
(From : Karnataka)
Coram : 2 TARUN CHATTERJEE AND DALVEER BHANDARI, JJ.
Criminal Appeal Nos. 845-846 of 2008 (arising out of SLP (Cri) Nos. 2351-2352 of
2005), D/- 8 -5 -2008.
Southern Steel Ltd. and Others v. Jindal Vijayanagar Steel Ltd.
Criminal P.C. (2 of 1974), S.482 - Negotiable Instruments Act (26 of 1881), S.138 -
INHERENT POWERS - DISHONOUR OF CHEQUE - Quashing of proceedings -
Complainant
@page-SC3144
of dishonour of cheque - Of fence by company - Quashing sought on ground that accused
company is sick - Purchase orders were issued after com pany was declared sick - Such
fact not disclosed to complainant-Supplier - Cheque for payment of price of material
supplied - Issued by Director of accused-Company with full knowledge of debt - Conduct
of accused indicates that right from inception, accused had no intention of paying amount
for purchases made - High Court refusing to quash proceedings - No interference by S.C.
(Para 9)
Cases Referred : Chronological Paras
2000 AIR SCW 521 : AIR 2000 SC 926 : 2000 Cri LJ 1424 : 2000 CLC 453 (Disting)
6, 7
2000 AIR SCW 609 : AIR 2000 SC 954 : 2000 Cri LJ 1464 : 2000 CLC 459 (Disting)
6, 7
M. L. Varma, Sr. Advocate, P. Niroop, T. Anil Kumar, Venkateswara Rao Anumolu and
Satya Mitra, for Appellants; Rajiv Shakdher, Sr. Advocate, U. A. Rana, Abhishek Rao for
Gyaltsen Barfungapa, (for M/s. Gagrat and Co.), for Respondents.
Judgement
1. DALVEER BHANDARI, J. :- Leave granted in both these Special Leave Peti tions.
2. These appeals are directed against the judgment dated 3.1.2005 of the High Court of
Karnataka at Bangalore in Criminal Peti tion Nos. 948 and 949 of 2003.
3. The brief facts of these appeals are recapitulated as under :
On the request of the appellants (South ern Steel Ltd.), the respondent company (Jindal
Vijayanagar Steel Ltd.) had supplied HR Coils to the appellants. The terms of pay ment
under the purchase order dated 25.4.1998 granted 45 days interest free credit to the
appellants for the goods sold and delivered by the 1st respondent. The appellants had
issued the cheques in ques tion in favour of the respondent company. The said cheques
were dishonoured on pre sentation. In response to the legal notice sent by the respondent
company, the appellants, through two substantially identical replies dated 6.4.2000, for
the first time, contended that the appellants had been declared a sick company on
8.4.1997 under the provisions of the Sick Industrial Companies (Special Provisions) Act,
1985 (for short "SICA") and, therefore, no legal proceedings of recovery of the
outstanding amount could be initi ated against the appellant company.
4. It may be pertinent to mention that the purchases were made by the appellants from the
respondent company after the ap pellant company was declared sick under the provisions
of SICA. The appellants could not dispute the fact that the purchases were made after the
appellant company was de clared sick under the SICA. The purchases were made holding
out clear representation that the goods will be paid for. Ultimately, on non-payment of the
outstanding amount, the respondent company initiated criminal proceedings against the
appellant company by filing a criminal complaint under section 138 of the Negotiable
Instruments Act, 1881.
5. The appellant company, aggrieved by the said proceedings, filed Criminal Petitions
Nos.3225-3226 of 2000 under section 482 Cr.P.C. for quashing the proceedings under
section 138 of the Negotiable Instruments Act. The High Court dismissed both these
petitions holding that it was premature to analyze the entire documentary evidence as put
forth by both sides to give a finding one way or the other. Thereafter, the appellants,
subsequent to the directions given by the High Court, approached the trial court and
produced the documents including the or der passed by the Board for Industrial Fi nancial
Reconstruction (for short "BIFR") under section 22-A of SICA. They sought discharge of
the accused under section 258 of the Code of Criminal Procedure. However, the trial
court dismissed those applications. Thereafter, the appellants again approached the High
Court by filing two criminal peti tions for quashing the criminal proceedings.
6

. The appellants placed reliance on the two judgments of this court in the matter of BSI
Ltd. and Another v. Gift Hold ings Pvt. Ltd. and Another (2000) 2 SCC 737 and Kusum
Ingots and Alloys Ltd. v. Pennar Peterson Se curities Ltd. and Others (2000) 2 SCC 745.
In the impugned judgment, the High Court has dealt with these two judg ments along
with other judgments. The High Court also accepted the legal position that the court has
the power to discharge the ac cused or quash the proceedings at mid-stage, but having
regard to the peculiar facts and circumstances of the case, according 2000 AIR
SCW 521
2000 AIR SCW 609
@page-SC3145
to the impugned judgment of the High Court, it would not be appropriate to interfere.
Con sequently, both these petitions filed by the appellants before the High Court were dis
missed.
7. The appellants, aggrieved by the im pugned order of the High Court, preferred these
appeals and submitted that the ap pellants' case is squarely covered by the
aforementioned cases of Kusum Ingots and BSI Ltd.
8. We have carefully analyzed the prin ciples of law enunciated in both these cases, along
with other cases, cited by Mr. M. L. Verma, learned senior Advocate appearing for the
appellant company. There is no quar rel with the legal proposition which has been laid
down in both the aforementioned cases, but we are in complete agreement with the view
taken by the High Court in the im pugned judgment that in view of the pecu liar facts and
circumstances of these cases, the aforesaid judgments have no application to these cases.
9. According to the High Court, admit tedly the purchase orders in question were entered
into and the purchases were made by the appellants with full knowledge of the
proceedings that the company was declared sick under the SICA, the appellants clearly
all through gave the impression to the 6 re spondent company that the outstanding
amount towards the purchase of the goods would be shortly cleared. The fact that the
purchases were made with the clear prom ise to repay could not be disputed by the
appellants. The Directors had in fact issued the cheques for discharging their liability
with the full knowledge, would not only clearly show that there was an undisputed debt,
but would also show that, right from the inception, the appellants in fact had no intention
of paying the amount for the pur chases made by them. The intention of the appellants
can be gathered by their subse quent acts, conduct and behaviour of tak ing a shelter
under the provisions of SICA. Hence, the appellants are not entitled to any indulgence of
this court under its extraor dinary jurisdiction under Article 136 of the Constitution. The
appellants had lost their total credibility because of their conduct. When the appellant
company was declared sick, then without disclosing this fact the appellants ought not to
have made huge pur chases from the respondent company. Ultimately, the appellant
company did not pay for the purchases. This clearly indicates that the appellants had no
intention of making payment of the purchases made by it.
10. The High Court, in the impugned or der, has directed the trial court to dispose of the
cases of the appellants as early as possible, but not later than six months from the date of
its order. The appellants by ap proaching this Court have caused avoidable delay in
disposal of these cases before the trial court.
11. In the facts and circumstances of the case, we deem it appropriate to request the trial
court now to conclude the trial of these cases as expeditiously as possible and, in any
event, within six months from the date of this judgment. We direct the parties to appear
before the trial Court on 1st July, 2008.
12. The trial court is directed to decide these cases without being influenced by any
observations passed by this court or the High Court in the impugned judgment. These
appeals, being devoid of any merit, are accordingly dismissed.
Appeals dismissed.
AIR 2008 SUPREME COURT 3145 "National Insurance Co. Ltd. v. Yellamma"
(From : AIR 2006 Kant 240)
Coram : 2 S. B. SINHA AND LOKESHWAR SINGH PANTA, JJ.
Civil Appeal No. 3317 of 2008 (arising out of SLP (C) No. 16359 of 2006), D/- 6 -5
-2008.
National Insurance Co. Ltd. v. Yellamma and Anr.
Motor Vehicles Act (59 of 1988), S.147 - Insurance Act (4 of 1938), S.64VB - MOTOR
VEHICLES - INSURANCE - SUPREME COURT - Compensation - Liability of insurer -
Owner of vehicle after getting vehicle in sured issued third party cheque towards
payment of premium - Development Of ficer of insurer by inadvertence issued a cover
note - When said mistake came to his notice owner was asked to pay amount of premium
- Amount was not tendered and instead owner returned original cover note and took back
cheque - As a result covernote was cancelled - It could be said that no privity of contract
came into being between owner and insurer - In surer not liable to pay compensation for
accident occurred - Supreme Court un der Art.142 directed insurer to make
@page-SC3146
payment and to recover it from owner. (Paras 9, 10, 11)
Cases Referred : Chronological Paras
2007 AIR SCW 7948 :AIR 2008 SC 767 : 2008 (1) AIR Kar R 485 (Foll.) 7
2001 AIR SCW 902 : AIR 2001 SC 1197 (Ref.) 7
2000 AIR SCW 788 : AIR 2000 SC 1082 (Ref.) 7
M. K. Dua, Kishore Rawat and Dhiraj, for Appellant; Uday Umesh Lalit, Sr. Advocate,
(A. C.) for Respondents.
Judgement
S. B. SINHA, J. :- Leave granted.
2. Respondent No.2 was the owner of a Mini Bus. An insurance policy in respect of the
said vehicle was sought to be taken by him. For the said purpose, the second re spondent
issued a third party cheque to wards payment of insurance premium.
The Development Officer of the appellant by inadvertence issued a cover note. How ever,
when the said mistake came to his notice, the respondent No.2 was contacted by the
Development Officer. He was asked to pay the amount of premium. It was not tendered
and instead the respondent No.2 is said to have returned the original cover note and took
back the cheque. The original cover note as also all the duplicate copies thereof was
cancelled.
The said insurance cover was issued for the period 3.9.1991 to 2.9.1992. On or about
12.9.1991, the said vehicle met with an ac cident. First respondent who suffered an injury
therein filed a claim petition in terms of the provisions contained in Section 166 of the
Motor Vehicles Act, 1988 (the Act). An award for a sum of Rs.43,000/- was made. The
Tribunal, in its award, categorically held :
"The petitioners have produced Ex.P.7 the Xerox copy of the cover note Ex.R. 1. There is
all the chances of the owner of the vehicle having taken Xerox copy of the cover note
Ex.R. 1 and returning the original cover note Ex.R. 1 to the insurance company as
deposed by RW. 1. If really the cover note was not cancelled, the original cover note
should have been with the insured and respondent No.3 could not have produced the
original cover note Ex.R.1. Hence, the case of 3rd respondent that the owner of the
vehicle had given third party cheque and that later he had taken back the cheque and
returned the original cover note Ex.R. 1 to the insur ance company and that the insurance
company has cancelled cover note is more prob able. As Ex.P.7 is the Xerox copy of the
origi nal cover note Ex.R.1 and as the original cover note Ex.R. 1 and its copies Exs.R.2
to R.4 have been produced by the insurance company, the argument of the learned coun
sel for the petitioners that respondent No.3 is liable to pay the compensation cannot be
accepted. Hence, from the above discussion, I hold that there was no valid insurance
policy as on the date of the accident and as such the respondent No.3 is not liable to pay
any compensation to the petitioners."
3. Second Respondent did not prefer any appeal thereagainst. First Respondent only
preferred an appeal questioning the quan tum of compensation.
The High Court, by reason of the im pugned judgment, while enhancing the amount of
compensation to a sum of Rs.1,50,000/-, held :
"The above provision disclose that a policy can be issued against the issuance of cheque
and the liability commences from the date of issuance of cheque and not from the date of
its encashment. There is no provision in law that the consideration for policy should flow
only from the insured and not from the third party. The development officer has acted in a
hasty manner. No attempt was made to present the cheque for encashment. If the cheque
was encashed it was well and good for the insurer otherwise steps could have been taken
for cancellation of the policy Ex.R.1. The reason that the cheque is not issued by the
insured is not a ground for valid cancellation. The endorsement of can cellation is vague,
it does not bear the date. The officer who has made endorsement of cancellation is not
examined. The endorse ment of the insured is not taken on the policy to substantiate that
the cancellation was with due notice and knowledge by the in sured. Therefore, under the
above circum stances, the very cancellation of the policy for untenable reason is bad in
law. The ac cident has occurred within 15 days from the date of issue of cover note.
Hence, the in surer is liable to pay the compensation."
4. Mr. Dua, learned counsel appearing on behalf of the appellant, would submit that
keeping in view the provisions contained in Section 65(v)(b) of the Insurance Act, 1938
and furthermore in view of the finding of fact arrived at by learned Motor Vehicles Acci
dents Claims Tribunal which was not ques tioned by the insured, the impugned judgment
@page-SC3147
cannot be sustained.
As nobody had appeared despite service of notice on behalf of the respondent, we
requested Mr. U.U. Lalit, senior counsel to assist us.
5. It is neither in doubt nor in dispute that all the copies including the insurance cover
which were marked as Ex.R.1 to R.4 had been produced before the Tribunal to show that
original insurance cover had been taken back by the Development Officer con cerned for
one reason or the other.
The Administrative Officer of the appel lant not only examined himself before the
Tribunal but also proved the note prepared by the Divisional Manager of Ludhiana which
was marked as Ex.R.5. The Tribunal, as noticed hereinbefore, on appreciation of the
evidence produced before it, held that the vehicle was not legally insured.
6. The High Court, however, wrongly pro ceeded on the premise that a cheque could be
issued by a third party.
A contract of insurance like any other contract, is a contract between the insured and the
insurer. The amount of premium is required to be paid as a consideration for arriving at a
concluded contract. If the in surer insists that a cheque should be issued only by the
insured and not by a third party, no exception thereto can be taken. The fact remains that
the cheque was not encashed. Concededly, the insured did not make any payment.
Section 64VB of the Insurance Act man dates that before a contract of insurance comes
into being, the premium should be received by the insurer in advance, stating :
"Section 64VB - No risk to be assumed unless premium is received in advance- (1) No
insurer shall assume any risk in India in respect of any insurance business on which
premium is not ordinarily payable outside India unless and until the premium payable is
received by him or is guaranteed to be paid by such person in such manner and within
such time as may be prescribed or unless and until deposit of such amount as may be
prescribed, is made in advance in the prescribed manner.
(2) For the purposes of this section, in the case of risks for which premium can be
ascertained in advance, the risk may be as sumed not earlier than the date on which the
premium has been paid in cash or by cheque to the insurer.
Explanation. Where the premium is tendered by postal money order or cheque sent by
post, the risk may be assumed on the date on which the money order is booked or the
cheque is posted, as the case may be."
7

. The question came up for consider ation recently before this Court in Deddaooa and
Ors. v. Branch Manager, National Insurance Co. Ltd. [(2008) 2 SCC 595], wherein upon
notic ing the precedents which were operating in the field, it was clearly held : 2007
AIR SCW 7948

"18. The ratio of the said decision was, however, noticed by this Court in New India
Assurance Co. Ltd. v. Rula and Ors. [(2000) 3 SCC 195]. It was held that ordinarily a
liability under the contract of insurance would arise only on payment of premium, if such
pay ment was made a condition precedent for taking effect of the insurance policy but
such a condition which is intended for the ben efit of the insurer can be waived by it. It
was opined : 2000 AIR SCW 788

'13...If, on the date of accident, there was a policy of insurance in respect of the ve hicle
in question, the third party would have a claim against the Insurance Company and the
owner of the vehicle would have to be indemnified in respect of the claim of that party.
Subsequent cancellation of the insur ance policy on the ground of non-payment of
premium would not affect the rights al ready accrued in favour of the third party.' The
dicta laid down therein clarifies that if on the date of accident the policy subsists, then
only the third party would be entitled to avail the benefit thereof.

19. Almost an identical question again came up for consideration before this Court in
National Insurance Co. Ltd. v. Seema Malhotra and Ors. [(2001) 3 SCC 151], a Division
Bench no ticed both the aforementioned decisions and analysed the same in the light of
Section 64-VB of the 1938 Act. It was held : 2001 AIR SCW 902

'17. In a contract of insurance when the insured gives a cheque towards payment of
premium or part of the premium, such a contract consists of reciprocal promise. The
drawer of the cheque promises the insurer that the cheque, on presentation, would yield
the amount in cash. It cannot be for gotten that a cheque is a bill of exchange drawn on a
specified banker. A bill of exchange
@page-SC3148
is an instrument in writing contain ing an unconditional order directing a cer tain person
to pay a certain sum of money to a certain person. It involves a promise that such money
would be paid.
18. Thus, when the insured fails to pay the premium promised, or when the cheque issued
by him towards the premium is re turned dishonoured by the bank concerned the insurer
need not perform his part of the promise. The corollary is that the insured cannot claim
performance from the insurer in such a situation.
19. Under Section 25 of the Contract Act an agreement made without consideration is
void. Section 65 of the Contract Act says that when a contract becomes void any per son
who has received any advantage under such contract is bound to restore it to the person
from whom he received it. So, even if the insurer has disbursed the amount covered by
the policy to the insured before the cheque was returned dishonoured, the insurer is
entitled to get the money back.
20. However, if the insured makes up the premium even after the cheque was
dishonoured but before the date of accident it would be a different case as payment of
consideration can be treated as paid in the order in which the nature of transaction re
quired it. As such an event did not happen in this case, the Insurance Company is le gally
justified in refusing to pay the amount claimed by the respondents.'
20. A contract is based on reciprocal promise. Reciprocal promises by the par ties are
condition precedents for a valid con tract. A contract furthermore must be for
consideration."
8. In today's world payment by cheque is ordinarily accepted as valid tender but the same
would be subject to its encashment. A distinction, however, exists between the statutory
liability of the insurance company vis-a-vis the third party in terms of Sections 147 and
149 of the Motor Vehicles Act and its liability in other cases but it is clear that if the
contract of insurance had been can celled and all concerned had been intimated
thereabout, the insurance company would not be liable to satisfy the claim.
9. In this case, there cannot be any doubt or dispute whatsoever that no privity of con
tract came into being between the appellant and the second respondent and as such the
question of enforcing the purported contract of insurance while taking recourse to Sec
tion 147 of the Motor Vehicles Act did not arise.
Second respondent did not contest the case at any stage. It did not adduce any evi dence
before the Tribunal. It does not ap peal from the judgments of the High Court. No
argument in the appeal was advanced in his behalf. Before us also, no appearance has
been made on behalf of the respondent No.2 despite service of notice.
10. The accident took place in the State of Karnataka. Respondent No.2 is a resident of
Ludhiana. The transaction in question was purported to have been entered in Ludhiana.
First respondent, therefore, in our opinion, may not be in a position to enforce the award
as against the respondent No.2.
11. In the peculiar facts and circum stances of this case, we are, therefore, of the opinion
that the interest of justice would be subserved if we, in exercise of our juris diction under
Article 142 of the Constitu tion of India, direct that the awarded amount be paid by the
appellant to the first respon dent with liberty to it to recover the same from the second
respondent by initiating an appropriate proceeding in this behalf.
12. This appeal is allowed to the afore mentioned extent and with the aforemen tioned
directions. As the respondents have not appeared before us, there shall be no order as to
costs.
Order accordingly.
AIR 2008 SUPREME COURT 3148 "Satyawati Sharma v. Union of India"
(From : AIR 2002 Delhi 509 (FB))
Coram : 2 B. N. AGRAWAL AND G. S. SINGHVI, JJ.
Civil Appeal No. 1897 with 1898 of 2003 and 5622 of 2006, D/- 16 -4 -2008.
Satyawati Sharma (Dead) by L.Rs. v. Union of India and Anr.
Delhi Rent Control Act (59 of 1958), S.14(1)(e) - HOUSES AND RENTS - EVICTION -
EQUALITY - Eviction - Bona fide require ment - Restricting landlord's right to evict
tenants only of residential accom modation - Violates Art.14 - Classifi cation of premises
into residential and non-residential for purpose of S.14(1)(e) - Has by passage of time
become irratio nal - S.14(1)(e) partly struck down.
AIR 2002 Delhi 509 (FB), Reversed.
ILR (1973) 1 Delhi 90, Overruled.
Constitution of India, Art.14.
@page-SC3149

Section 14(1)(e) is violative of the doctrine of equality embodied in Article 14 of the Con
stitution of India insofar as it discriminates between the premises let for residential and
non-residential purposes when the same are required bona fide by the landlord for occu
pation for himself or for any member of his family dependent on him and restricts the
latter's right to seek eviction of the tenant from the premises let for residential pur poses
only. Discriminatory portion of S. 14 (1)(e) only struck down.
AIR 2002 Delhi 509 (FB), Reversed.
ILR (1973) 1 Delhi 90, Overruled. (Paras 38, 39, 40)
The main reason for classification of pre mises into residential and non-residential was
that by imposing restriction on the evic tion of tenants of premises let for non-resi dential
purposes, the Government wanted to solve the acute problem of housing cre ated due to
partition of the country in 1947. As an aftermath of partition many hundred-thousands of
people had been uprooted from the area which now forms part of Pakistan; that they were
forced to leave their homes and abandon their business establishments, industries,
occupation and trade and the Government was very much anxious to en sure resettlement
of such persons. It was felt that if the landlords are readily allowed to evict the tenants,
those who came from West Pakistan will never be able to settle in their life. Therefore, in
the 1947 and 1958 Acts, the legislature did not provide for evic tion of tenants from the
premises let for non-residential purposes on the ground that the same are required by the
landlords for their bona fide use and occupation. As of now a period of almost 50 years
has elapsed from the enactment of the 1958 Act. During this long span of time much
water has flown down the Ganges. Those who came from West Pakistan as refugees and
even their next generations have settled down in dif ferent parts of the country, more
particu larly in Punjab, Haryana, Delhi and sur rounding areas. They are occupying prime
positions in political and bureaucratic set up of the Government and have earned huge
wealth in different trades, occupation, busi ness and similar ventures. Not only this, the
availability of buildings and premises which can be let for non-residential or commercial
purposes has substantially increased. Therefore, the reason/cause which prompt ed the
Court to sustain the differentiation/classification of the premises with reference to the
purpose of their user, is no longer available for negating the challenge to Sec tion 14(1)
(e) on the ground of violation of Article 14 of the Constitution. The discrimi nation which
was latent in Section 14 (1)(e) at the time of enactment of 1958 Act has, with the passage
of time become so pro nounced that the impugned provision can not be treated intra vires
Article 14 of the Constitution by applying any rational crite ria. (Paras 26, 28)
Cases Referred : Chronological Paras
2006 AIR SCW 1098 : AIR 2006 SC 1471 24
2005 AIR SCW 4820 (Rel. on) 6, 19, 23, 28
2003 AIR SCW 5193 : AIR 2003 SC 4493 (Ref.) 8, 9
2002 AIR SCW 1156 : AIR 2002 SC 1334 7
2002 AIR SCW 2374 : AIR 2002 SC 2256 (Rel. on) 12, 28
(1999) 1 Rent LR 689 (P and H) 24
1998 AIR SCW 202 : AIR 1998 SC 602 (Rel. on) 6, 11, 34
(1998) 8 SCC 275 (Ref.) 11
1996 AIR SCW 238 : AIR 1996 SC 857 (Rel. on) 5, 6, 7, 10, 19, 21, 23, 24, 28
(1996) 3 SCC 105 (Ref.) 39
1995 AIR SCW 2834 : AIR 1995 SC 1811 17
1994 AIR SCW 5011 (Ref.) 11
AIR 1987 SC 741 (Disting.) 4, 7, 27, 35
AIR 1987 SC 2117 (Ref.) 6, 8, 11
AIR 1986 SC 244 11
AIR 1986 SC 1444 (Rel. on) 6, 20, 33
AIR 1985 SC 796 18, 22, 28
AIR 1984 SC 121 (Rel. on) 20, 32, 33
AIR 1982 SC 33 37
AIR 1981 SC 1711 (Ref.) 35
AIR 1980 SC 1 31
AIR 1979 SC 478 7
AIR 1978 SC 597 17
AIR 1975 SC 1409 (Ref.) 35
AIR 1974 SC 1 : 1974 Lab IC 1 16
AIR 1974 SC 1631 : 1974 Lab IC 1103 16
ILR (1973) 1 Delhi 90 (Overruled) 4, 6, 25, 27, 28
AIR 1966 AP 51 32
AIR 1964 SC 1179 (Rel. on) 29
AIR 1964 SC 1590 30
AIR 1958 SC 538 15
AIR 1957 SC 628 (Ref.) 39
C. S. Rajan, T. S. Doabia, Sr. Advocates, A. C. Gambhir, P. D. Sharma, Mohan Pandey,
Shish Pal Laler, Balbir Singh Gupta, Ms. Sadhana Sandhu, R. C. Kathtam, P.
@page-SC3150
Parmeswaran, D. S. Mahra, B. V. Balram Das, Manu Nayar, K. K. Sharma, Hameed S.
Shaikh, A. L. Trehan, Praveen Swarup and Ashwani Kumar, for the Appearing Par ties.
Judgement
G. S. SINGHVI, J. :- Whether Section 14(1)(e) of the Delhi Rent Control Act, 1958 (for
short 'the 1958 Act') is ultra vires the doctrine of equality enshrined in Article 14 of the
Constitution of India is the question which arises for determination in these ap peals.
2. For the sake of convenience, we have noted the facts from Civil Appeal No.1897 of
2003 :
(i) On August 18, 1953, Delhi Improve ment Trust leased out a plot of land mea suring
184 sq. yards situated at Basti Reghar, Block 'R', Khasra Nos. 2942/1820 to 2943/1820 to
Shri Jagat Singh son of Pt. Ram Kishan. In terms of Clause 4(c) of the lease deed, the
lessee was prohibited from using the land and building (to be con structed over it) for any
purpose other than residence, with a stipulation that in case of breach of this condition,
the lease shall be come void.
(ii) After constructing the building, the lessee inducted Shri Jai Narain Sharma and Dr.
Ms. Tara Motihar, as tenants in two por tions of the building, who started using the rented
premises for running watch shop and clinic respectively.
(iii) Smt. Saryawati Sharma (appellant herein), who is now represented by her LRs.
purchased property i.e. house bearing No.3395-3397, Ward No. XVI, Block R, Gali No.
1, Reghar Pura, New Delhi from legal heirs of the lessee.
(iv) After purchasing the property, the appellant filed Petition Nos. 184 of 1980 and 187
of 1980 for eviction of the tenants by claiming that she needed the house for her own
bona fide need and also for the use and occupation of the family members depen dant
upon her. The appellant further pleaded that she wanted to demolish the building and
reconstruct the same. She also alleged that tenants have been using the premises in
violation of the conditions of lease and, therefore, they are liable to be evicted.
(v) The tenants contested the eviction petitions by asserting that the so-called need of the
landlord was not bona fide; that there were no valid grounds for permitting the landlord
to demolish the building and reconstruct the same and that they had not violated the
conditions of lease. They fur ther pleaded that the previous owner let out the premises for
non-residential purposes; that the appellant was also issuing rent re ceipts by describing
the rented portions as shop/clinic and that in view of order dated 11.12.1978 issued by the
Government of India, Ministry of Housing and Urban De velopment, Delhi Development
Authority was condoning violations of the lease con ditions.
(vi) By an order dated 17.5.1991, Addi tional Rent Controller, Delhi dismissed the
eviction petitions. He held, that the appel lant is, owner and landlady of the suit pre
mises, but she has not been able to prove that portions thereof were let for residential
purposes; that the appellant and her depen dent family members do not have suitable
alternative accommodation except the one occupied by her elder son, who was under the
threat of eviction and that the need of the appellant is bona fide. The Additional Rent
Controller further held that the ten ants are guilty of violating Clause 4(c) of deed dated
August 18, 1953. He, however, de clined to pass order for recovery of posses sion by
observing that under Section 14(1)(e) of the Act, such an order can be passed only in
respect of premises let for residential purposes. The Additional Rent Controller also
rejected other grounds of eviction put forward by the appellant.
3. The appeal preferred by the appel lant was dismissed by Rent Control Tribu nal, Delhi
vide its judgment dated 10.11.1998. The Tribunal agreed with the Additional Rent
Controller that an order of eviction of the tenant can be passed under Section 14(1)(e)
only if the premises were let for residential purposes. The Tribunal then held that the
portions given to the tenants were being used for non-residential purposes and, therefore,
they cannot be evicted on the ground of bona fide need of the landlord.
4

. The appellant challenged the orders of the Additional Rent Controller and Rent Control
Tribunal in Civil Writ Petition No. 1093 of 1999. She filed another petition, which was
registered as Civil Writ Petition No-1092 of 1999, with the prayer that Sec tion 14(1) (e)
of the Act be declared ultra vires of Article 14 of the Constitution insofar as it does not
provide for eviction of the tenant from the premises let for non-residential purposes. Both
the writ petitions were heard AIR 1987 SC 741

@page-SC3151
by the Full Bench of Delhi High Court along with other writ petitions involving challenge
to the vires of Section 14(1)(e) and were dis missed by the order under challenge. The
Full Bench referred to an earlier judgment of the Division Bench in H. C. Sharma vs. Life
Insurance Corporation of India and Anr. [ILR 1973 (1) Delhi 90] and large number of
judgments of this Court including Amarjit Singh vs. Smt. Khatoon Qua marin [1986 (4)
SCC 736] and held :-
i) Tenants of non-residential premises are a class by themselves. The Parliament in its
legislative wisdom did not think it fit to make any provision for eviction of a tenant from
such premises on the ground of bona fide requirement of the landlord for residential
purpose. Referenced to Section 29(2)(r) of the 1995 Act, in our opinion, cannot be said to
have any relevance whatsoever for the purpose of determining. Admittedly, the 1995 Act
is yet to come into force. If the said Act is yet to come into force, the question of taking
recourse to the provisions of the said Act would not arise moreso because this court in
exercise of its jurisdiction under Ar ticle 226 of the Constitution of India would not be in
a position to direct the Govern ment to do so which is a legislative func tion. On the other
hand, the very fact that said Act is yet to come into force in an indi cia to the fact that the
Central Government does not in its wisdom consider that the said benefit should be
extended to non-residen tial premises also.
ii) Judicial review of legislation is permis sible only on limited grounds, namely when a
statute is enacted by a legislature which had no authority therefor or when it inter alia
violates any of the provisions contained in Part III of the Constitution. Once it is held, as
we are bound to, that the non-residen tial premises having regard to the interpre tation
clause, forms a separate class, such classification, having a reasonable nexus with the
ground of eviction, cannot be said to be discriminatory in nature. Article 14 of the
Constitution would apply only to per sons similarly situated. Owners of residen tial and
non-residential premises stand on different footings. In the event, the legisla ture in its
wisdom thinks it fit to extend its protective wing to a class of tenants from being evicted
on a particular ground, the same by itself cannot be said to be discrimi natory so as to
attract the wrath of Article 14 of the Constitution of India. The court in a situation of this
nature is only entitled to see as to whether such classification is valid and rational. Once
the rationality in such legislation is found, the court will put its hands off.
iii) Furthermore, the provisions of the said Act had been declared intra vires by the Apex
Court in Amarjit Singh v. Khatoon Quamarain (supra). In that case, an argu ment was
advanced that unless the second limb of Section 14(1)(e) of the Act is read in such a way
that it was in consonance with Articles 14 and 21 of the Constitution of India, the same
would be void as being un constitutional. The question raised therein has been dealt with
the Apex Court.
(iv) In the instant case, the Statute it self has indicated the persons or things to whom its
provisions are recommended to apply. The said Act is a beneficial legisla tion. It seeks to
protect the tenants. Ten ants are broadly classified into three cat egories-residential, non-
residential and/or other tenant. Such a classification as re gards premises or tenancy
cannot per se be said to be unreasonable.
(v) In the instant case, so far as Sections 14(1)(e) and 14(1)(k) are concerned, the stat ute
itself has indicated the persons to whom the provisions would apply. The provision is
absolutely clear and unambiguous. In such a case the Court is only required to examine
whether the classification is based upon reasonable differentia, distinguishing the person,
group from those left out and whether such differentia has reasonable nexus with the
objects to be achieved. The impugned provision indisputably was in tended to
beneficially apply to landlords and of one class of tenancy viz. tenancy in re spect of the
residential premises and not non-residential premises.
5

. The Full Bench also noticed the judgment in Harbilas Rai Bansal vs. State of Punjab and
Anr. [1996 (1) SCC 1] whereby Section 13(3)(a) of the East Punjab Urban Rent
Restriction Act, 1949, as amended by Punjab Act No. 29 of 1956, was struck down but
distinguished the same by making the following observa tions :- 1996 AIR SCW 238

"The objects and reasons of the said Act, thus, were considered having regard to the
provisions made at the time of commence ment of the said Act. Such a contingency
@page-SC3152
does not arise in the instant case. Reason able nexus to the objects to be achieved of the
said Act having regard to the perfor mance for which the building is being used must be
found out from the legislative in tent. Legislative intent may change from State to State."
6

. Learned counsel for the appellants relied on the judgment of this Court in Harbilas Rai
Bansal v. State of Punjab and Anr. (supra) and argued that the classification made be
tween the premises let for residential pur poses and non-residential purposes in the matter
of eviction of tenant on the ground of bona fide need of the landlord is irratio nal,
arbitrary and violative of Article 14 of the Constitution. Shri A.C. Gambhir sub mitted
that even though the constitutional validity of Section 14(1)(e) of the Act was up held by
the Division Bench of the High Court in H. C. Sharma vs. Life Insurance Corpo ration of
India and Anr. (supra), that decision cannot, in the changed circumstances and in view of
the later judgments of this Court in Rattan Arya vs. State of Tamil Nadu [(1986) 3 SCC
385], Harbilas Rai 1996 AIR SCW 238
AIR 1986 SC 1444
2005 AIR SCW 4820
1998 AIR SCW 202
AIR 1987 SC 2117

Bansal vs. State of Punjab (supra), Rakesh Vij vs. Dr. Raminder Pal Singh Sethi [(2005) 8
SCC 504] be treated as good law. He argued that the reason which prompted the
legislature to exclude the pre mises let for non-residential purposes from the purview of
Section 14(1)(e) of the 1958 Act and which found approval of the Divi sion Bench of the
High Court has, with the passage of time, become non-existent and the classification of
the premises into resi dential and non-residential with reference to the purpose of lease
has become totally arbitrary and irrational warranting a decla ration of invalidity qua the
impugned sec tion. In support of this argument, the learned counsel relied on the
judgment of this Court in Malpe Vishwanath Acharya and others vs. State of Maharashtra
and an other [1998 (2) SCC 1]. Shri Gambhir pointed out that in the Delhi Rent Control
Act, 1995 (for short 'the 1995 Act'), which was enacted by the Parliament in the light of
the National Housing Policy, 1992 and observations made by this Court in Prabhakaran
Nair vs. State of Tamil Nadu [1987 (4) SCC 238], no distinction has been made between
the premises let for resi dential and non-residential purposes in the matter of eviction of
the tenant on the grounds of landlord's bona fide need and argued that even though that
Act has not been enforced, the Court can take cogni zance of the legislative changes and
declare the implicit restriction contained in Section 14(1)(e) on the eviction of tenant
from the premises let for non-residential purposes as unconstitutional.
7

. Shri C. S. Rajan, learned senior coun sel appearing for the Union of India empha sized
that the purpose of the Act is to pro tect the tenants against arbitrary eviction by the
landlord and argued that the classi fication of the premises with reference to the purpose
of lease should be treated as based on rational grounds because the same is meant to
further the object of the enactment. Shri Rajan referred to the judg ment of Amarjit Singh
v. Smt. Khatoon Quamarin (supra) to show that challenge to the constitutionality of the
Section 14(1)(e) on the ground of vio lation of Article 14 has already been nega tived and
argued that the vires of that pro vision cannot be re-examined merely be cause a similar
provision contained in the 'Punjab Act' has been declared unconstitu tional in Harbilas
Rai Bansal v. State of Punjab (supra). Learned senior counsel relied on the judg ments of
this Court In Re, The Special Courts Bill, 1978 [1979 (1) SCC 380] and Padma Sundra
Rao (Dead) and others v. State of Tamil Nadu and others [2002 (3) SCC 533] and argued
that the Court should not attempt to rewrite Section 14(1)(e) so as to facilitate eviction of
the tenants from the premises let for non-residential pur poses. Shri S.P. Laler, learned
counsel ap pearing for the respondents in Civil Appeal Nos.1897 of 2003 and 1898 of
2003 sup ported the judgment of the Full Bench of the High Court and argued that the
distinc tion made by the legislature between the pre mises let for residential and non-
residential purposes is based on rational ground i.e. acute shortage of non-residential
premises/ buildings and, therefore, the same cannot be treated as unconstitutional. AIR
1987 SC 741
1996 AIR SCW 238
AIR 1979 SC 478
2002 AIR SCW 1156

8. We have considered the respective arguments/submissions. For deciding the question


raised in these appeals, it will be useful to notice the salient features of rent
@page-SC3153
control legislations, which were made ap plicable to Delhi from time to time. These are :-
i) In exercise of the power vested in it un der Rule 81 of the Defence of India Rules, the
Government of India promulgated New Delhi House Rent Control Order, 1939. This
order was made applicable only to residen tial premises. Section 11 thereof provided that
a tenant in possession of a house shall not be evicted therefrom whether in execu tion of a
decree or otherwise and whether before or after the termination of the ten ancy except on
the grounds mentioned therein. Clause (iv) of sub-section (2) of Sec tion 11-A was as
under :
"that the landlord was at no time during the twelve months immediately preceding the
date of his application residing within the limits of the Delhi or New Delhi Munici pality
or the Notified Areas of the Civil Sta tion, Delhi or Delhi Fort, that it is essential in the
public interest that he should take up residence in that area and that he is unable to secure
other suitable accommo dation, the Controller shall make an order directing the tenant to
put the landlord in possession of the house, and if the Control ler is not so satisfied, he
shall make an or der rejecting the application."
(ii) On 15th October, 1942, the Punjab Urban Rent Restriction Act, 1941 was made
applicable to the Province of Delhi, except the areas to which the New Delhi House Rent
Control Order was applicable. The definition of the expression "premises" in the Punjab
Urban Rent Restriction Act made no distinc tion Between "residential" and non-residen
tial" premises. Section 10(1) of that Act pro vided that no order for recovery of posses
sion of any premises shall be made so long as the tenant pays or is ready and willing to
pay rent to the full extent allowable by this Act and perform other conditions of the ten
ancy. However, in terms of proviso to Sec tion 19(1), the Court could make an order for
recovery of possession if the landlord satisfied that the prescribed notice had been served
on the tenant. Sub-section (2) of Sec tion 10 provided that where any order men tioned in
sub-section (1) has been made on or after the First day of January, 1939 but not executed
before the commencement of the Act, the Court by which the order was made may if it is
of opinion that the order would not have been made if the Act had been in operation on
the date the order was made, rescind or vary the order. The pro viso to Section 10(2)
enumerated the other grounds for eviction of the tenants. One of the grounds was that the
premises are rea sonably and bona fide required by the land lord for his own occupation.
(iii) In 1944, the Delhi Rent Control Or dinance (XXV), 1944 was promulgated. In this
Ordinance, the word 'premises' was defined to mean any building which is let separately
for use as a residence or for com mercial use or for any other purpose. Clauses (a) to (e)
of Section 9 of the Ordi nance specified the grounds on which the landlord could recover
possession of the premises. One of the grounds was that the landlord requires the
premises for his use as residence. This means the landlord could not recover possession
of the premises if he needed the same for commercial use.
(iv) In 1947, the Delhi and Ajmer-Merwara Rent Control Act was enacted and was made
applicable to all the parts of Delhi. Section 2(b) of the 1947 Act which contained the
definition of the word 'premises' read as under :-
"premises" means any building which is, or is intended to be, let separately for use as a
residence or for commercial use or for any other purpose..................."
Section 9(e) which provided for eviction of the tenant on the ground of bona fide re
quirement of the landlord was as under :-
"that purely residential premises are re quired bona fide by the landlord who is the owner
of such premises for occupation as a residence for himself or his family, that he neither
has nor is able to secure other suit able accommodation, and that he has ac quired his
interest in the premises at a date prior to the beginning of the tenancy or the 2nd day of
June, 1944, whichever is later, or if the interest has devolved on him by inheritance or
succession, his predecessor had acquired the interest at a date prior to the beginning of
the tenancy or the 2nd day of June, 1944, whichever is later......;"
(v) The 1947 Act was replaced by the Delhi and Ajmer Rent Control Act, 1952. Section
13 of that Act enumerated various grounds on which a tenant could be evicted. Clause (c)
of Section 13(1) was as under :-
"that the premises let for residential pur poses are required bona fide by the land lord who
is the owner of such premises for occupation as a residence for himself or his
@page-SC3154
family and that he has no other suitable accommodation;
Explanation :- For the purposes of this clause, "residential premises" include any
premises which having been let for use as a residence are, without the consent of the
landlord, used incidentally for commercial or other purposes:................"
(vi) After 6 years, the Delhi Rent Con trol Act, 1958 was enacted. The Preamble of this
Act shows that it is a legislation for the control of rents and evictions and of rates of
hotels and lodging houses, and for the lease of vacant premises to Government, in cer tain
areas in the Union Territory of Delhi. Section 2(i) of that Act defines the premises to
mean any building or part of a building which is intended to be or is let for use as a
residence or for commercial use or for any other purpose. The definition of the term
"standard rent" contained in Section 2(k) refers to the premises irrespective of its use.
Section 3 which exempts certain premises from the operation of the Act also does not
make any distinction between residential and non-residential premises. Clause (c) of that
section which provides for exemption in the context of monthly rent speaks of resi dential
as well as non-residential premises. Section 6 relates to standard rent. It deals with
residential as well as non-residential premises. Para A of Section 6(1) specifies the
standard rent for residential premises and para B specifies such rent for premises other
than residential premises. Sub-section (2) of Section 6 which provides for fixation of
standard rent refers to premises irrespec tive of their user. The limitation prescribed
(Section 12) for filing application for fixa tion of standard rent does not make any dis
tinction between the premises let for resi dential, commercial and other purposes. Section
14(1) which contains prohibition against passing of an order or decree by any Court or
Controller for recovery of posses sion of any premises does not make any dis tinction
between the premises let for resi dential, commercial or other purposes. Clauses (a), (b),
(c), (f), (g), (j), (k) and (1) of proviso to Section 14(1) specify different grounds for
recovery of possession of the premises irrespective of its user. Only clauses (d) and (e)
speak of premises let for use as residence or residential purposes.
Sections 2(i) and 14(l)(d) and (e) of the 1958 Act which have bearing on the deci sion of
the appeals, read as under :-
2. In this Act, unless the context other wise requires-
(i) "premises" means any building or part of a building which is, or is intended to be, let
separately for use as a residence or for commercial use or for any other purpose, arid
includes,
(i) the garden, grounds and outhouses, if any, appertaining to such building or part of the
building'
(ii) any furniture supplied by the land lord for use in such building or part of the building;
but does not include a room in a hotel or lodging house.
14. Protection of tenant against evic tion.- (1) Notwithstanding anything to the contrary
contained in any other law or con tract, no order or decree for the recovery of possession
of any premises shall be made by any court or Controller in favour of the landlord against
a tenant :
Provided that the Controller may, on an application made to him in the prescribed
manner, make an order for the recovery of possession of the premises on one or more of
the following grounds only, namely :-
(a) to (c)............ . . ...........
(d) that the premises were let for use as a residence and neither the tenant nor any
member of his family has been residing therein for a period of six months immedi ately
before the date of the filing of the ap plication from the recovery of possession thereof;
(e) that the premises let for residential purposes are required bona fide by the land lord
for occupation as a residence for him self or for any member of his family depen dent on
him, if he is the owner thereof, or for any person for whose benefit the pre mises are held
and that the landlord or such person has no other reasonably suitable residential
accommodation.
Explanation. - For the purposes of this clause, "premises let for residential pur poses"
include any premises which having been let for use as a residence are, without the
consent of the landlord, used inciden tally for commercial or other purposes.

(vii) The 1958 Act was amended five times between 1960 to 1988, but demands
continued to be made by the landlords and the tenants for its further amendment to suit
their respective causes. In 1992 National 2003 AIR SCW 5193

@page-SC3155
Housing Policy was notified. One of the im portant features of that Policy was to remove
legal impediments to the growth of housing in general and rental housing in particular.
Both the Houses of Parliament adopted the Policy. Thereafter, the 1995 Act was enacted.
Though the new Act has not been enforced so far and in Common Cause v. Union of
India and others [2003 (8) SCC 250], this Court declined to issue a writ of mandamus to
Central Government to notify the same, it will be useful to take cognizance of the state
ment of objects and reasons and Section 22(r) of the 1995 Act to which reference was
made by the learned counsel during the course of hearing. The same reads as un der :-
Statement of Objects and Reasons:
The relations between landlords and ten ants in the National Capital Territory of Delhi are
presently governed by the Delhi Rent Control Act, 1958. This Act came into force on the
9th February, 1959. It was amended thereafter in 1960, 1963, 1976, 1984 and 1988. The
amendments made in 1988 were based on the recommendations of the Eco nomic
Administration Reforms Commission and the National Commission on Urbanisation.
Although they were quite ex tensive in nature, it was felt that they did not go far enough
in the matter of removal of disincentives to the growth of rental hous ing and left many
questions unanswered and problems unaddressed. Numerous represen tations for further
amendments to the Act were received from groups of tenants and landlords and others.

2. The demand for further amendments to the Delhi Rent Control Act, 1958 received
fresh impetus with the tabling of the Na tional Housing Policy in both Houses of Par
liament in 1992. The Policy has since been considered and adopted by Parliament. One of
its major concerns is to remove legal im pediments to the growth of housing in gen eral
and rental housing in particular. Para graph 4.6.2 of the National Housing Policy
specifically provides for the stimulation of investment in rental housing especially for the
lower and middle income groups by suit able amendments to rent control laws by State
Governments. The Supreme Court of India has also suggested changes in rent control
laws. In its judgment in the case of Prabhakaran Nair v. State of Tamil Nadu, the Court
observed that the laws of landlords and ten ants must be made rational, humane, cer tain
and capable of being quickly imple mented. In this context, a Model Rent Con trol
Legislation was formulated by the Cen tral Government and sent to the States to enable
them to carry out necessary amend ments to the prevailing rent control laws. Moreover,
the Constitution (Seventy-Fifth Amendment) Act, 1994 was passed to en able the State
Governments to set up State-level Rent Tribunals for speedy disposal of rent cases by
excluding the jurisdiction of all courts except the Supreme Court. AIR 1987 SC 2117

3. In the light of the representations and developments referred to above, it has been
decided to amend the rent control law pre vailing in Delhi. As the amendments are
extensive and substantial in nature, instead of making changes in the Delhi Rent Con trol
Act, 1958, it is proposed to repeal and replace the said Act by enacting a fresh leg
islation.
4. To achieve the above purposes, the present Bill, inter alia, seeks to provide for the
following, namely :-
(a) exemption of certain categories of pre mises and tenancies from the purview of the
proposed legislation;
(b) creation of tenancy compulsorily to be written agreement;
(c) compulsory registration of all written agreements of tenancies except in certain
circumstances;
(d) limit the inheritability of tenancies;
(e) redefine the concept of rent payable and provide for its determination, enhance ment
and revision;
(f) ensure adequate maintenance and re pairs of tenanted premises and facilitate further
improvement and additions and al terations of such premises;
(g) balance the interests of landlords and tenants in the matter of eviction in speci fied
circumstances;
(h) provide for limited period tenancy and automatic eviction of tenants upon expiry of
such tenancy;
(i) provide for the fixing and revision of fair rate and recovery of possession in re spect of
hotels and lodging houses;
(j) provide for a simpler and speedier sys tem of disposal of rent cases through Rent
Authorities and Rent Tribunal and by bar ring the jurisdiction of all courts except the
@page-SC3156
Supreme Court; and
(k) enhance the penalties for infringement of the provisions of the legislation by land
lords and tenants.
5. On enactment, the Bill will minimize distortion in the rental housing market and
encourage the supply of rental housing both from the existing housing stock and from
new housing stock.
6. The Notes on clauses appended to the Bill explain the various provisions of the Bill."
22. Protection of tenant against evic tion.-
(r) that the premises let for residential or non-residential purposes are required, whether
in the same form or after re-con struction or re-building, by the landlord for occupation
for residential or non-residen tial purpose for himself or for any member of his family if
he is the owner thereof, or for any person for whose benefit the premises are held and that
the landlord or such per son has no other reasonably suitable accom modation.
9

. An analysis of the above-noted provi sions would show that till 1947 no tangible
distinction was made between the premises let for residential and non-residential pur
poses. The implicit restriction on the landlord's right to recover possession of the non-
residential premises was introduced in the Delhi and Ajmer-Marwara Rent Control Act,
1947 and was continued under the 1958 Act. However, the 1995 Act does not make any
distinction between the premises let for residential and non-residential purposes in the
matter of eviction of tenant on the ground that the same are required by the landlord for
his/her bona fide use or occupation. Even though, the 1995 Act is yet to be en forced and
in Common Cause v. Union of India (supra) this Court declined to issue a writ of man
damus to the Central Government, for that purpose, we can take judicial notice of the fact
that the legislature has, after taking note of the developments which have taken place in
the last 37 years i.e. substantial increase in the availability of the commercial and non-
residential premises or the premises which can be let for commercial or non-resi dential
purposes and meteoric rise in the prices of land and rentals of residential as well as non-
residential premises, removed the implicit embargo on the landlord's right to recover
possession of the premises if the same are bona fide required by him/her. 2003 AIR
SCW 5193

10

. Section 13 (3) (a) of the Punjab Act (unamended and amended), which came up for
consideration in Harbilas Rai Bansal vs. State of Punjab (supra) reads as under :- 1996
AIR SCW 238
Unamended Section 13(3) (a) of the Punjab Act.
13(3)(a). A landlord may apply to the Con troller for an order directing tenant to put the
landlord in possession -
(i) in the case of a residential or a sched uled building if-
(a) he requires it for his own occupation;
(b) he is not occupying another residen tial or a scheduled building, as the case may be, in
the urban area concerned; and
(c) he has not vacated such a building without sufficient cause after the commence ment
of this Act, in the said urban area;
(ii) in the case of a non-residential build ing or rented land, if-
(a) he requires it for his own use;
(b) he is not occupying in the urban area concerned for the purpose of his business any
other such building or rented land, as the case may be; and
(c) he has not vacated such a building or rented land without sufficient cause af ter the
commencement of this Act, in the urban area concerned;
(iii) in the case of any building, if he re quires it for the re-erection of that building, or for
its replacement by another building, or for the erection of other building;
(iv) in the case of any building, if he re quires it for use as an office or consulting room
by his son who intends to start prac tice as a lawyer or as a "registered practitio ner"
within the meaning of that expression as used in the Punjab Medical Registration Act,
1916 (II of 1916), or for the residence of his son who is married, if--
(a) his son as aforesaid is not occupy ing in the urban area concerned any other building
for use as office, consulting room or residence, as the case may be; and
(b) his son as aforesaid has not vacated such a building without sufficient cause af ter the
commencement of this Act, in the urban area concerned :
Provided that where the tenancy is for a specified period agreed upon between the
@page-SC3157
landlord and the tenant, the landlord shall not be entitled to apply under this sub-sec tion
before the expiry of such period :
Provided further that where that landlord has obtained possession of a residential, a
scheduled or non-residential building or rented land under the provisions of sub-
paragraph (i) or sub-paragraph (ii) he shall not be entitled to apply again under the said
sub-paragraphs for the possession of any other building, of the same class or rented land:
Provided further that where a landlord has obtained possession of any building under the
provisions of sub-paragraph (iv) he shall not be entitled to apply again un der the said
sub-paragraph for the posses sion of any other building for the use of or, as the case may
be, for the residence of the same son.
(b) The Controller shall, if he is satisfied that the claim of the landlord is bona fide make
an order directing the tenant to put the landlord in possession of the building or rented
land on such date as may be speci fied by the Controller and if the Controller is not so
satisfied, he shall make an order rejecting the application :
Provided that the Controller may give the tenant a reasonable time for putting the
landlord in possession of the building or rented land and may extend such time so as not
to exceed three months in the aggre gate.
Amended Section 13(3)(a) of the Punjab Act.
13. Eviction of tenants.- (1) A tenant in possession of a building or rented land shall not
be evicted therefrom in execution of a decree passed before or after the commence ment
of this Act or otherwise and whether before or after the termination of the ten ancy,
except in accordance with the provi sions of this section, or in pursuance of an order
made under Section 13 of the Punjab Urban Rent Restriction Act, 1949, as sub sequently
amended.
(2) * * * *
(3) (a) A landlord may apply to the Con troller for an order directing the tenant to put the
landlord in possession-
(i) in the case of a residential building, if (omitted as not relevant)
(ii) in the case of rented land, if-
(a) he requires it for his own use;
(b) he is not occupying in the urban area concerned for the purpose of his business any
other such rented land, and
(c) he has not vacated such rented land without sufficient cause after the commence ment
of this Act, in the urban area con cerned.
11

. Before proceeding further we con sider it necessary to observe that there has been a
definite shift in the Court's approach while interpreting the rent control legisla tions. An
analysis of the judgments of 1950s' to early 1990s' would indicate that in ma jority of
cases the courts heavily leaned in favour of an interpretation which would ben efit the
tenant - Mohinder Kumar and oth ers vs. State of Haryana and an other [1985 (4) SCC
221]; Prabhakaran Nair and others vs. State of Tamil Nadu and others (supra), D.C.
Bhatia and others vs. Union of India and another [1995 (1) SCC 104] and C.N.
Rudramurthy vs. K. Barkathulla Khan [1998 (8) SCC 275]. In these and other cases, the
Court consistently held that the paramount object of every Rent Control Leg islation is to
provide safeguards for tenants against exploitation by landlords who seek to take undue
advantage of the pressing need for accommodation of a large number of people looking
for a house on rent for resi dence or business in the background of acute scarcity thereof.
However, a different trend is clearly discernible in the latter judg ments. In Malpe
Vishwanath Acharya and others v. State of Maharashtra and another (supra), this Court
considered the ques tion whether determination and fixation of rent under the Bombay
Rents, Hotel and Lodging House, Rates Control Act, 1947, by freezing or pegging down
of rent as on 1.9.1940 or as on the date of first letting was arbitrary, unreasonable and
violative of Article 14 of the Constitution. The three-Judge Bench answered the question
in af firmative but declined to strike down the concerned provisions on the ground that
the same were to lapse on 31.3.1998. Some of the observations made in that judgment are
worth noticing. These are : AIR 1986 SC 244
AIR 1987 SC 2117
1994 AIR SCW 5011
1998 AIR SCW 202, Paras 28 and 29

"Insofar as social legislation, like the Rent Control Act is concerned, the law must strike a
balance between rival interests and it should try to be just to all. The law ought not to be
unjust to one and give a disproportionate
@page-SC3158
benefit or protection to another section of the society. When there is short age of
accommodation it is desirable, nay, necessary that some protection should be given to the
tenants in order to ensure that they are not exploited. At the same time such a law has to
be revised periodically so as to ensure that a disproportionately larger ben efit than the
one which was intended is not given to the tenants. It is not as if the Gov ernment does
not take remedial measures to try and off-set the effects of inflation. In order to provide
fair wage to the salaried employees the Government provides for pay ment of dearness
and other allowances from time to time. Surprisingly this principle is lost sight of while
providing for increase in the standard rent - the increases made even in 1987 are not
adequate, fair or just and the provisions continue to be arbitrary in today's context."
"When enacting socially progressive leg islation the need is greater to approach the
problem from a holistic perspective and not to have narrow or short sighted parochial
approach. Giving a greater than due empha sis to a vocal section of society results not
merely in the miscarriage of justice but in the abdication of responsibility of the legis
lative authority. Social Legislation is treated with deference by the Courts not merely
because the Legislature represents the people but also because in representing them the
entire spectrum of views is ex pected to be taken into account. The Legis lature is not
shackled by the same con straints as the courts of law. But its power is coupled with a
responsibility. It is also the responsibility of the courts to look at legislation from the altar
of Article 14 of the Constitution. This Article is intended, as is obvious from its words, to
check this ten dency; giving undue preference to some over others."
12

. In Joginder Pal vs. Naval Kishore Behal [2002 (5) SCC 397], the Court after noticing
sev eral judicial precedents on the subject observed as under : 2002 AIR SCW 2374,
Para 8

"The rent control legislations are heavily loaded in favour of the tenants treating them as
weaker sections of the society requiring legislative protection against exploitation and
unscrupulous devices of greedy land lords. The legislative intent has to be re spected by
the courts while interpreting the laws. But it is being uncharitable to legislatures if they
are attributed with an inten tion that they lean only in favour of the ten ants and while
being fair to the tenants, go to the extent of being unfair to the landlords. The legislature
is fair to the tenants and to the landlords - both. The courts have to adopt a reasonable and
balanced approach while interpreting rent control legislations starting with an assumption
that an equal treatment has been meted out to both the sections of the society. In spite of
the over all balance tilting in favour of the tenants, while interpreting such of the
provisions as take care of the interest of the landlord the court should not hesitate in
leaning in favour of the landlords. Such provisions are engrafted in rent control
legislations to take care of those situations where the landlords too are weak and feeble
and feel humble.
[Emphasis added]
13. We shall now deal with the core question whether Section 14(l)(e) of the 1958 Act
can be treated as violative of equal ity clause embodied in Article 14 of the Con stitution
insofar as it differentiates between the premises let for residential and non-resi dential
purposes in the matter of eviction on the ground of bona fide requirement of the landlord
and restricts the landlord's right only to the residential premises.
14. Article 14 declares that the State shall not deny to any person equality before the law
or the equal protection of the laws. The concept of equality embodied in Article 14 is also
described as doctrine of equality. Broadly speaking, the doctrine of equality means that
there should be no discrimina tion between one person and another, if having regard to
the subject-matter of leg islation, their position is the same. The plain language of Article
14 may suggest that all are equal before the law and the State can not discriminate
between similarly situated persons. However, application of the doctrine of equality
embodied in that Article has not been that simple. The debate which started in 1950s on
the true scope of equality clause is still continuing. In last 58 years, the courts have been
repeatedly called upon to adjudi cate on the constitutionality of various leg islative
instruments including those meant for giving effect to the Directive Principles of State
Policy on the ground that same vio late the equality clause. It has been the con stant
refrain of the courts that Article 14 does not prohibit the legislature from clas sifying
apparently similarly situated persons,
@page-SC3159
things or goods into different groups provided that there is rational basis for do ing so.
The theory of reasonable classifica tion has been invoked in large number of cases for
repelling challenge to the consti tutionality of different legislations.
15

. In Ram Krishna Dalmia and Ors. vs. Shri Justice S.R. Tendolkar and Ors., [AIR 1958
SC 538], this Court considered the inter-play of the doctrines of equality and classifica
tion and held :- Para 11 of AIR

"It is now well established that while Ar ticle 14 forbids class legislation, it does not
forbid reasonable classification for the pur poses of legislation. In order, however, to pass
the test of permissible classification two conditions must be fulfilled, namely (i) that the
classification must be founded on an in telligible differentia which distinguishes per sons
or things that are grouped together from others left out of the group, and (ii) that that
differentia must have a rational relation to the object sought to be achieved by the statute
in question. The classifica tion may be founded on different bases, namely, geographical,
or according to ob jects or occupations or the like. What is nec essary is that there must
be a nexus be tween the basis of classification and the object of the Act under
consideration. It is also well established by the decisions of Supreme Court that Article
14 condemns discrimination not only by a substantive law but also by a law of
procedure."
Speaking for the Court, Chief Justice S.R. Das enunciated some principles, which have
been referred to and relied in all subsequent judgments. These are :
"(a) that a law may be constitutional even though it relates to a single individual if, on
account of some special circumstances or reasons applicable to him and not applicable to
others, that single individual may be treated as a class by himself;
(b) that there is always a presumption in favour of the constitutionality of an enact ment
and the burden is upon him who at tacks it to show that there has been a clear
transgression of the constitutional prin ciples;
(c) that it must be presumed that the leg islature understands and correctly appreci ates
the need of its own people, that its laws are directed to problems made manifest by
experience and that its discriminations are based on adequate grounds;
(d) that the legislature is free to recog nize degrees of harm and may confine its
restrictions to those cases where the need is deemed to be the clearest;
(e) that in order to sustain the presump tion of constitutionality the Court may take into
consideration matters of common knowledge, matters of common report, the history of
times and may assume every state of facts which can be conceived existing at the time of
legislation; and
(f) that while good faith and knowledge of the existing conditions on the part of a leg
islature are to be resumed, if there is noth ing on the face of the law or the surround ing
circumstances brought to the notice of the court on which the classification may
reasonably be regarded as based, the pre sumption of constitutionality cannot be car ried
to the extent of always holding that there must be some undisclosed and unknown reasons
for subjecting certain individuals or corporations to hostile or discriminating leg islation."
16

. In Mohd. Shujat Ali v. Union of India [1975 (3) SCC 76], the Court observed that
Article 14 ensures to every person equality before law and equal protection of the laws.
However, the constitutional code of equality and equal opportunity does not mean that
the same laws must be applicable to all persons. It does not compel the State to run "all
its laws in the channels of general legislation". It recognises that having regard to
differences and disparities which exist among men and things, they cannot all be treated
alike by the application of the same laws. "To recognise marked differences that exist in
fact is living law; to disregard practical dif ferences and concentrate on some abstract
identities is lifeless logic." The Legislature must necessarily, if it is to be effective at all in
solving the manifold problems which con tinually come before it, enact special legis
lation directed towards specific ends limited in its application to special classes of per
sons or things. "Indeed, the greater part of all legislation is special, either in the extent to
which it operates, or the objects sought to be attained by it." At the same time, the Court
cautioned against the readymade in voking of the doctrine of classification to ward off
every challenge to the legislative instruments on the ground of violation of equality
clause and observed : AIR 1974 SC 1631

@page-SC3160
"The equal protection of the laws is a "pledge of the protection of equal laws". But laws
may classify. And, as pointed out by Justice Brawer, "the very idea of classifica tion is
that of inequality". The Court has tackled this paradox over the years and in doing so, it
has neither abandoned the de mand for equality nor denied the legislative right to classify.
It has adopted a middle course of realistic reconciliation. It has re solved the
contradictory demands of legis lative specialization and constitutional gen erality by a
doctrine of reasonable classifi cation. This doctrine recognises that the leg islature may
classify for the purpose of leg islation but requires that the classification must be
reasonable. It should ensure that persons or things similarly situated are all similarly
treated. The measure of reason ableness of a classification is the degree of its success in
treating similarly those simi larly situated."
"A reasonable classification is one which includes all persons or things similarly situ ated
with respect to the purpose of the law. There should be no discrimination between one
person or thing and another, if as re gards the subject-matter of the legislation their
position is substantially the same. This is sometimes epigrammatically described by
saying that what the constitutional code of equality and equal opportunity requires is that
among equals, the law should be equal and that like should be treated alike. But the basic
principle underlying the doctrine is that the Legislature should have the right to classify
and impose special burdens upon or grant special benefits to persons or things grouped
together under the classification, so long as the classification is of persons or things
similarly situated with respect to the purpose of the legislation, so that all per sons or
things similarly situated are treated alike by law. The test which has been evolved for this
purpose is - and this test has been consistently applied by this Court in all de cided cases
since the commencement of the Constitution - that the classification must be founded on
an intelligible differentia which distinguishes certain persons or things that are grouped
together from others and that differentia must have a ratio nal relation to the object sought
to be achieved by the legislation."

"We have to be constantly on our guard to see that this test which has been evolved as a
matter of practical necessity with a view to reconciling the demand for equality with the
need for special legislation directed to wards specific ends necessitated by the com plex
and varied problems which require so lution at the hands of the Legislature, does not
degenerate into rigid formula to be blindly and mechanically applied whenever the
validity of any legislation is called in question. The fundamental guarantee is of equal
protection of the laws and the doc trine of classification is only a subsidiary rule evolved
by courts to give a practical content to that guarantee by accommodat ing it with the
practical needs of the society and it should not be allowed to submerge and drown the
precious guarantee of equal ity. The doctrine of classification should not be carried to a
point where instead of being a useful servant, it becomes a dangerous master, for
otherwise, as pointed out by Chandrachud, J., in State of Jammu and Kashmir v. Triloki
Nath Khosa, the guaran tee of equality will be submerged in class legislation
masquerading as laws meant to govern well-marked classes characterised by different and
distinct attainments". Overem phasis on the doctrine of classification or an anxious and
sustained attempt to dis cover some basis for classification may gradually and
imperceptibly deprive the guarantee of equality of its spacious con tent. That process
would inevitably end in substituting the doctrine of classification for the doctrine of
equality: the fundamental right to equality before the law and equal protection of the laws
may be replaced by the overworked methodology of classifica tion. Our approach to the
equal protection clause must, therefore, be guided by the words of caution uttered by
Krishna Iyer, J. in State of Jammu and Kashmir v. Triloki Nath Khosa (at SCC p. 42) :
AIR 1974 SC 1, Para 5

"Mini-classifications based on micro-dis tinctions are false to our egalitarian faith and
only substantial and straightforward classifications plainly promoting relevant goals" can
have constitutional validity. To overdo classification is to undo equality."
[Emphasis added]
17
. In L.I.C. of India and another vs. Consumer Education and Research Centre and others
[1995 (5) SCC 482], the Court reiterated the above-noted principle in the following
words :- 1995 AIR SCW 2834, Para 29

"The doctrine of classification is only a AIR 1978 SC 597

@page-SC3161
subsidiary rule evolved by the courts to give practical content to the doctrine of equality,
overemphasis on the doctrine of classifica tion or anxious or sustained attempt to dis
cover some basis for classification may gradually and imperceptibly erode the pro found
potency of the glorious content of equality enshrined in Article 14 of the Con stitution.
The overemphasis on classifica tion would inevitably result in substitution of the doctrine
of classification to the doc trine of equality and the Preamble of the Constitution which is
an integral part and scheme of the Constitution. Maneka Gandhi v. Union of India [1978
(1) SCC 248] ratio extricated it from this moribund and put its elasticity for egalitar ian
path finder lest the classification would deny equality to the larger segments of the
society. The classification based on employ ment in Government, semi-Government and
reputed commercial firms has the insidious and inevitable effect of excluding lives in vast
rural and urban areas engaged in unorga nized or self-employed sectors to have life
insurance offending Article 14 of the Con stitution and socio-economic justice."
18

. In Gian Devi Anand vs. Jeevan Kumar and Ors. [1985 (2) SCC 683] the Supreme Court
considered the question whether the statutory tenancy in respect of commercial premises
is heri table. The facts of that case were that one Wasti Ram was tenant in respect of Shop
No. 20, New Market, West Patel Nagar of the respondents at a monthly rental of Rs. 110/.
The tenancy commenced from September 1, 1959. In April, 1970, the respondent-land
lord determined the tenancy by serving a notice to quit. In September, 1970 he filed a
petition under Section 14 of the Act for evic tion of Wasti Ram on the grounds of non
payment of rent, bona fide requirement, change of user from residential to commer cial,
substantial damage to the property and sub-letting. He also impleaded one Ashok Kumar
Sethi, as defendant No. 2 by alleg ing that he had been unlawfully inducting a sub-tenant.
The Rent Controller negatived all the grounds of challenge except the non payment of
rent. He held that the premises had been let out for commercial purpose and as such the
ground of bona fide requirement was not available to the landlord for seek ing eviction of
the tenant. On the issue of non-payment of rent, the Rent Controller held that the tenant
was liable to pay a sum of Rs. 24/- by way of arrears for the period from March 1, 1969
to February 28, 1970 after taking into consideration all payments made and a further sum
of Rs. 90/- on ac count of such arrears for the month of Sep tember, 1970. He,
accordingly, directed evic tion of the tenant. The landlord challenged the order of the
Rent Controller by filing an appeal. The tenant, namely Wasti Ram, filed cross-objection
on the findings recorded by the Rent Controller on the issue of default. The Rent Control
Tribunal allowed the cross-objection of the tenant and held that there was no default in
the matter of payment of rent. The Tribunal rejected the landlord's plea regarding damage
to the property but remanded the matter to the Rent Controller for deciding the question
of sub-letting afresh after affording opportunity to the parties to lead evidence. Smt. Gian
Devi Anand, the widow of the deceased-tenant appealed against the order of the Tribunal.
The landlord filed cross-objections to ques tion the finding recorded by the Tribunal on
the issue of default by the tenant in pay ment of rent. The High Court held that after the
demise of the statutory tenant, his heirs do not have the right to remain in posses sion
because the statutory tenancy was not heritable and the protection afforded to the
statutory tenant was not avail able to the heirs. This Court re versed the order of the High
Court and held : AIR 1985 SC 796
at Pp. 803-804 of AIR

"We find it difficult to appreciate how in this country we can proceed on the basis that a
tenant whose contractual tenancy has been determined but who is protected against
eviction by the statute, has no right of property but only a personal right to re main in
occupation, without ascertaining what his rights are under the statute. The concept of a
statutory tenant having no es tate or property in the premises, which he occupies is
derived from the provisions of the English Rent Acts. But it is not clear how it can be
assumed that the position is the same in this country without any refer ence to the
provisions of the relevant stat ute. Tenancy has its origin in contract. There is no dispute
that a contractual tenant has an estate or property in the subject- matter of tenancy, and
heritability is an incident of the tenancy. It cannot be assumed, however, that with the
determination of the tenancy the estate must necessarily disappear and the statute can
only preserve his status of
@page-SC3162
irremovability and not the estate he had in the premises in his occupation. It is not
possible to claim that the "sanctity" of con tract cannot be touched by legislation. It is,
therefore, necessary to examine the provi sions of the Madhya Pradesh Accommoda tion
Control Act, 1961 to find out whether the respondent's predecessors-in-interest retained a
heritable interest in the disputed premises even after the termination of their tenancy."

In paragraph 34 of the judgment, the Court highlighted difference between the residen tial
and commercial tenancies and con cluded that the legislature could never have intended
that the landlord would be entitled to recover possession of the premises or the building
let for commercial purposes on the death of the tenant of the commercial ten ancies, even
if no ground for eviction as pre scribed in the Rent Act is made out. In the concluding
part of the judgment, the Court took cognizance of the absence of provision for eviction
of the tenant of non-residential premises even when the same are bona fide required by
the land lord for his use or occupation and observed : AIR 1985 SC 796, Para 39

"Before concluding, there is one aspect on which we consider it desirable to make certain
observations. The owner of any pre mises, whether residential or commercial, let out to
any tenant, is permitted by the Rent Control Acts to seek eviction of the ten ant only on
the grounds specified in the Act, entitling the landlord to evict the tenant from the
premises. The restrictions on the power of the landlords in the matter of recovery of
possession of the premises let out by him to a tenant have been imposed for the benefit of
the tenants. In spite of various restric tions put on the landlord's right to recover
possession of the premises from a tenant, the right of the landlord to recover posses sion
of the premises from the tenant for the bona fide need of the premises by the land lord is
recognised by the Act, in case of resi dential premises. A landlord may let out the
premises under various circumstances. Usually a landlord lets out the premises when he
does not need it for own use. Circum stances may change and a situation may arise when
the landlord may require the premises let out by him for his own use. It is just and proper
that when the landlord requires the premises bona fide for his own use and occupation,
the landlord should be entitled to recover the possession of the pre mises which continues
to be his property in spite of his letting out the same to a tenant. The legislature in its
wisdom did recognise this fact and the Legislature has provided that bona fide
requirement of the landlord for his own use will be a legitimate ground under the Act for
the eviction of his tenant from any residential premises. This ground is. however,
confined to residential premises and is not made available In case of com mercial
premises. A landlord who lets out commercial premises to a tenant under cer tain
circumstances may need bona fide the premises for his own use under changed conditions
on some future date should not in fairness be deprived of his right to re cover the
commercial premises. Bona fide need of the landlord will stand very much on the same
footing in regard to either class of premises, residential or commercial. We therefore,
suggest that Legislature may con sider the advisability of making the bona fide
requirement of the landlord a ground of evic tion in respect of commercial premises as
well."
[Emphasis added]
19

. What is significant to be noted is that in para 34 of the aforementioned judg ment, the
distinction between residential and non-residential tenancies was made in the context of
the rights of the heirs of the tenant to continue to enjoy the protection envisaged under
Section 14(1). The Court was of the view that the heirs of the tenants of the commercial
premises cannot be de prived of the protection else the family of the tenant may be
brought on road or de prived of the only source of livelihood. The Court also opinioned
that if the heirs of the individual tenants of commercial tenancies are deprived of the
protection, extremely anomalous consequences will ensue be cause the companies,
corporations and ju ridical entities carrying on business or com mercial activities in
rented premises will continue to enjoy the protection even after the change of
management, but the heirs of individual tenants will be denuded of simi lar protection. At
the same time, the Court noted that the landlord of a premises let for residential purpose
may bona fide require the same for his own use or the use of his dependent family
members and observed that the legislature should remove appar ent discrimination
between residential and non-residential tenancies when the landlord 1996 AIR
SCW 238
2005 AIR SCW 4820

@page-SC3163
bona fide requires the same. If the observa tions contained in para 34 are read in any
other manner, the same would become to tally incompatible with the observation con
tained in the penultimate paragraph of the judgment and we do not see any reason for
adopting such course, moreso, because the later part of the judgment has been relied in
Harbilas Rai Bansal v. State of Punjab (supra) and Rakesh Vij v. Dr. Raminder Pal Singh
Sethi (supra).
20

. In Rattan Arya vs. State of Tamil Nadu (supra), the Court considered challenge to the
con stitutionality of Section 30(ii) of the Tamil Nadu Buildings (Lease and Rent Control)
Act, 1960 under which residential buildings or part thereof occupied by any tenant pay
ing monthly rent of more than Rs. 400/-were exempted from operation of the Act. It was
urged on behalf of the appellant that distinction made between the residential and non-
residential buildings in the matter of applicability of the Act was unreasonable, irrational
and arbitrary. The Court referred to different rent control legislations appli cable to the
State of Tamil Nadu and ob served that the scheme of the Act does not make any
distinction between residential and non-residential buildings insofar as the rights of the
tenant's and obligations of the landlord's are concerned and there are no special rights
attached to the tenancies of the non-residential buildings as against the tenancies of
residential buildings so as to warrant exemption only to residential build ings. The Court
also took cognizance of enor mous increase of rents throughout the coun try, referred to
the judgment in Motor General Traders vs. State of Andhra Pradesh [1984 (1) SCC 222]
and struck down Section 30(ii) of the Tamil Nadu Act on the ground that the same is
violative of Article 14 of the Constitution. AIR 1986 SC 1444
AIR 1984 SC 121

21

. In Harbilas Rai Bansal vs. State of (supra), the Su preme Court examined the con
stitutionality of the amendment made in the Punjab Act, whereby the landlord was
deprived of his right to seek eviction of tenant from non-residential build ing on the
ground of bona fide requirement for his own use. This Court referred to the unamended
and amended Section 13(l)(a) of the Punjab Act and observed : 1996 AIR SCW 238,
Paras 12 and 13

"The Scheme of the Act, unmistakably aims at regulating the conditions of tenancy,
controlling the rents and preventing unrea sonable and mala fide eviction of tenants of the
residential and non-residential build ings. For the advancement of these objects, tenants
are invested with certain rights and landlords are subjected to certain obliga tions. These
rights and obligations are at tached to the tenants and the landlords of all buildings,
residential or non-residential. None of the main provisions of the Act, to which we have
referred, make any serious distinction between residential and non-resi dential buildings."
The provisions of the Act, prior to the amendment, were uniformly applicable to the
residential and non-residential build ings. The amendment, in the year 1956, cre ated the
impugned classification. The ob jects and reasons of the Act indicate that it was enacted
with a view to restrict the in crease of rents and to safeguard against the mala fide
eviction of tenants. The Act, there fore, initially provided - conforming to its objects and
reasons - bona fide require ment of the premises by the landlord, whether residential or
non-residential, as a ground of eviction of the tenant. The classi fication created by the
amendment has no nexus with the object sought to be achieved by the Act. To vacate a
premises for the bona fide requirement of the landlord would not cause any hardships to
the tenant. Statu tory protection to a tenant cannot be ex tended to such an extent that the
landlord is precluded from evicting the tenant for the rest of his life even when he bona
fide re quires the premises for his personal use and occupation. It is not the tenants but
the landlords who are suffering great hardships because of the amendment. A landlord
may genuinely like to let out a shop till the time he bona fide needs the same. Visualise a
case of a shopkeeper (owner) dying young. There may not be a member in the family to
continue the business and the widow may not need the shop for quite some time may like
to let out the shop till the time per children grow up and need the premises for their
personal use. It would be wholly arbi trary in a situation like this to deny her the right to
evict the tenant. The amendment has created a situation where a tenant can continue in
possession of a non-residential premises for life and even after the tenant's death his heirs
may continue the tenancy. We have no doubt in our mind that the objects,-
@page-SC3164
reasons and the scheme of the Act could not have envisaged the type of situa tion created
by the amendment which is patently harsh and grossly unjust for the landlord of a non-
residential premises."
22

. For taking the aforesaid view, the Court drew support from the observations contained
in the concluding portion of the judgment in Gian Devi Anand vs. Jeevan Kumar and Ors.
(supra). This is evident from paragraph 17 of the judgment, which is extracted be low :-
AIR 1985 SC 796

"In Gian Devi case the question for con sideration before the Constitution Bench was
whether under the Delhi Rent Control Act, 1958, the statutory tenancy in respect of
commercial premises was heritable or not. The Bench answered the question in the af
firmative. The above-quoted observations were made by the Bench keeping in view that
hardship being caused to the landlords of commercial premises who cannot evict their
tenants even on the ground of bona fide re quirement for personal use. The observa tions
of the Constitution Bench that "bona fide need of the landlord will stand very much on
the same footing in regard to ei ther class of premises, residential or com mercial" fully
support the view we have taken that the classification created by the amend ment has no
reasonable nexus with the ob ject sought to be achieved by the Act. We, therefore, hold
that the provisions of the amendment, quoted in earlier part of the judgment, are violative
of Article 14 of the Constitution of India and are liable to be struck down."
23

. The ratio of Harbilas Rai Bansal vs. State of Punjab (su pra) was noted and approved in
Rakesh Vij vs. Dr. Ravinder Pal Singh Sethi (supra), in the back drop of the argument that
the amendment made to the Punjab Act, 1956 was not applicable to the Union Territory
of Chandigarh. While rejecting the argument, the three Judge Bench referred to Article
13(2) of the Constitution, some of the judg ments in which that Article was considered
and observed : 1996 AIR SCW 238
2005 AIR SCW 4820, Para 11
"We find sufficient force in the conten tion raised by the learned counsel for the
respondent-landlord. In Harbilas Rai Bansal this Court held in very clear terms that the
classification created by the Amendment Act, 1956. by which the words "a non-
residential building or" occurring in Section 13(3) (a) (ii) were deleted and certain other
amendments had been made, had no rea sonable nexus with the object sought to be
achieved by the Act and consequently the provisions of the Amendment Act were vio
lative of Article 14 of the Constitution."
24

. The judgment in Harbilas Rai Bansal vs. State of Punjab (su pra) was recently noticed in
Mohinder Prasad Jain vs. Manohar Lal Jain [(2006) 2 SCC 724]. The respondent in that
case applied for eviction of the tenant (appellant) from the shop in question on the ground
of bona fide personal requirement i.e. for the purpose of running wholesale business in
Ayurvedic medicines. The Rent Controller dismissed the application on the ground that
bona fide requirement of the landlord has not been proved. The Appellate Authority re
versed the order of the Rent Controller and returned a finding that the landlord has been
able to prove his bona fide requirement. In the revision filed by the appellant, reliance
was placed on the judgment of the Full Bench of Delhi High Court in Satyawati Sharma
v. Union of India and Ors. (that judgment is under challenge in these appeals) and it was
urged that an application for eviction of the tenant on the ground of bona fide require
ment of the landlord is not maintainable in respect of non-residential premises." The
learned Single Judge of Punjab and Haryana High Court referred to an earlier judgment
of the Division Bench of that Court in State of Haryana vs. Ved Prakash Gupta [(1999) 1
Rent Law Reporter 689], wherein the restric tion imposed on the landlord's right to evict
the tenant under the Haryana Urban (Con trol of Rent and Eviction) Act, 1973, was struck
down and held that the judgment of the Full Bench of Delhi High Court cannot be relied
for granting relief to the appellant. This Court noted that a similar provision had been
declared unconstitutional in Harbilas Rai Bansal vs. State of Punjab (supra), which was
approved by three Judge Bench in Rakesh Vij vs. Dr. Ravinder Pal Singh Sethi (supra)
and held that the tenant cannot question the landlord's right to seek evic tion of the tenant
from non-residential pre mises. 1996 AIR SCW 238
2006 AIR SCW 1098
AIR 2002 Delhi 509

25. We may now advert to the judgment of Delhi High Court in H. C. Sharma vs. Life
Insurance Corporation of India and Ann (supra)
@page-SC3165
and the one under challenge. The facts of H. C. Sharma's case were that the peti tioner
had leased out Flat No.28-E, Connaught Place, New Delhi to National In surance
Company Limited for non-residen tial use. Subsequently, the National Insur ance
Company Limited became Life Insur ance Corporation of India. The petitioner made
efforts to convince the Corporation that the premises are required for his bona fide use
and occupation but could not con vince the concerned authorities. He, there fore, filed an
application for recovery of pos session. The same was dismissed by the High Court. He
then filed Writ Petition ques tioning the constitutionality of Section 14(l)(e) on the
ground that the classifica tion of the premises into residential and non-residential is
arbitrary and violative of Ar ticle 14 of the Constitution. The Division Bench of Delhi
High Court traced the his tory of rent control legislation applicable to Delhi, the
background in which protection was extended to the tenants generally and the limited
right given to the landlord to seek eviction of the tenants only from the pre mises let for
residential purposes and ob served : "In judging whether the restriction imposed by the
impugned provisions is rea sonable, the court can look into the circum stances under
which the restriction came to be imposed. Judicial notice can be taken of the fact that in
1947 there was a large influx of refugees into Delhi. A large num ber of people who were
uprooted from their hearths and homes in West Pakistan settled in Delhi. This resulted in
acute shortage of house accommodation and business pre mises with the result that rents
soared to a high level which necessitated the regulation of relations between landlords
and ten ants..."
The object in not providing for the evic tion of a tenant from a non-residential pre mises
on the ground specified in sub-clause (e) was to give security of tenure to a tenant of such
premises. If a tenant of a non-resi dential premises was allowed to be evicted on the
ground of personal requirement by the landlord, it would have had the effect of
completely dislocating the business of the tenant and this in turn could have grave
consequences on the social and economic fabric of the country, besides causing untold
misery to the tenant."
[Emphasis added]
The Division Bench rejected the plea of discrimination and observed :-
The grievance of the petitioner is that the discrimination between the two classes of
landlords is without any rational basis. World War II broke out in 1939 and an acute
shortage of housing accommodation devel oped. To control the rents and eviction of
tenants, the Rent Control Order of 1939 was issued. A study of the relevant provisions of
the rent control legislation discussed in the earlier part of the judgment would show that
the restrictions imposed on the landlords to recover possession of residential premises
were very stringent up to 1952. Under the Rent Control Order of 1939 and the Delhi Rent
Control Ordinance, 1944 a landlord could recover possession of residential pre mises
only when he had not resided within the limits of Delhi or New Delhi during the twelve
months immediately preceding the date of the application and further satisfied the
conditions that it was essential in the public interest that he should take up resi dence in
that area and that he was unable to secure other suitable accommodation. Under the Rent
Control Act of 1947, a land lord could recover possession of residential premises only if
he did not possess other suitable accommodation and further, that he had acquired his
interest in the premises at a date prior to the beginning of the ten ancy or the 2nd day of
June, 1944, which ever was later. The rigour of the restrictions qua residential premises
was relaxed in the Act of 1952 and a landlord could recover possession of residential
premises if he re quired it bona fide for occupation as a resi dence for himself or his
family and he had no other suitable accommodation. In com parison to this the Rent
Control Order, 1939 was not applied to non-residential premises. The Delhi Rent Control
Ordinance did not place any bar on the right of the landlord to recover possession of non-
residential pre mises. The only restriction placed was that the landlord could recover
possession of the premises for his residential use. The bar against the eviction of tenants
from non-resi dential premises was introduced in the Rent Control Act, 1947 and it has
continued since then. A landlord cannot recover possession of non-residential premises on
the ground of his personal need. There is a clear object behind classification of the
premises into "residential" and "non-residential". We have earlier observed that in 1947.
on partition of the country, there was a large influx of
@page-SC3166
refugees into Delhi. The Government was faced with the problem of resettling the refu
gees. This necessitated the imposition of restrictions on the right to evict tenants from
residential and non-residential premises. The legislature keening in view the needs of the
people and other circumstances allowed the landlord to evict tenants from residen tial
premises for his personal use in case he did not have any other suitable accommo dation,
but restricted the right of the land lord to recover possession of non-residential premises
on the ground of personal need. The necessity behind this discrimination is to assure the
security of tenure to the tenants of non-residential premises so that they can settle in their
business without the fear of being ejected.
Owners of residential buildings and non-residential buildings each stand out as a class by
themselves. The impugned provi sions make no distinction inter se between the two
classes of properties or their land lords. The impugned provisions take within their fold
all the persons similarly situate. So long as there is equality under similar conditions and
among persons similarly situated, there is no infringement of Ar ticle 14."
[Emphasis added]
26. A critical analysis of the above noted judgment makes it clear that the main rea son
which weighed with the High Court for approving the classification of premises into
residential and non-residential was that by imposing restriction on the eviction of ten ants
of premises let for non-residential pur poses, the government wanted to solve the acute
problem of housing created due to partition of the country in 1947. The Court took
cognizance of the fact that as an after math of partition many hundred-thousands of
people had been uprooted from the area which now forms part of Pakistan; that they were
forced to leave their homes and aban don their business establishments, indus tries,
occupation and trade and the Govern ment was very much anxious to ensure re settlement
of such persons. It was felt that if the landlords are readily allowed to evict the tenants,
those who came from West Pa kistan will never be able to settle in their life. Therefore, in
the 1947 and 1958 Acts, the legislature did not provide for eviction of tenants from the
premises let for non-resi dential purposes on the ground that the same are required by the
landlords for their bona fide use and occupation.
27

. Insofar as the judgment under chal lenge is concerned, we find that the Full Bench
upheld the validity of Section 14(l)(e) mainly by relying upon the judgment of the
Division Bench in H. C. Sharma vs. Life Insurance Corporation of India and Anr. (supra)
and of this Court in Amarjit Singh vs. Smt. Khatoon Quamarin (supra) and by observing
that leg islature has the right to classify persons, things, and goods into different groups
and that the Court will not sit over the judgment of the legislature. It is significant to note
that the Full Bench did not, at all, advert to the question whether the reason/cause which
supplied rationale to the classification con tinued to subsist even after lapse of 44 years
and whether the tenants of premises let for non-residential purposes should continue to
avail the benefit of implicit exemption from eviction in the case of bona fide requirement
of the landlord despite sea saw change in the housing scenario in Delhi and substan tial
increase in the availability of buildings and premises which could be let for non-
residential or commercial purposes. AIR 1987 SC 741

28

. In our opinion, the reasons which weighed with the High Court in H.C. Sharma vs. Life
Insurance Corporation of India and Anr. (supra) and the impugned judgment cannot in the
changed scenario and in the light of the ratio of Harbilas Rai Bansal vs. State of Punjab
(su pra), which was approved by three-Judge Bench in Rakesh Vij v. Dr. Raminder Pal
Singh Sethi of the judgment in Gian Devi Anand vs. Jeevan Kumar and Ors. (supra), now
be made basis for justifying the classification of premises into residential and non-
residential in the context of landlord's right to recover possession thereof for his bona fide
requirement. At the cost of repetition, we deem it proper to mention that in the rent
control legislations made appli cable to Delhi from time to time residential and non-
residential premises were treated at par for all purposes. The scheme of the 1958 Act also
does not make any substan tial distinction between residential and non-residential
premises. Even in the grounds of eviction set out in proviso to Section 14(1), no such
distinction has been made except 1996 AIR SCW 238
2005 AIR SCW 4820
AIR 1985 SC 796
1996 AIR SCW 238
2002 AIR SCW 2734
2005 AIR SCW 4820

@page-SC3167
in Clauses (d) and (e). In H. C. Sharma vs. Life Insurance. Corporation of India (supra),
the Division Bench of the High Court, after taking cognizance of the acute problem of
housing created due to partition of the coun try, upheld the classification by observing
that the Government could legitimately re strict the right of the landlord to recover pos
session of only, those premises which were let for residential purposes. The Court felt
that if such restriction was not imposed, those up-rooted from Pakistan may not get
settled in their life. As of now a period of almost 50 years has elapsed from the en
actment of the 1958 Act. During this long span of time much water has flown down the
Ganges. Those who came from West Pa kistan as refugees and even their next gen
erations have settled down in different parts of the country, more particularly in Punjab,
Haryana, Delhi and surrounding areas. They are occupying prime positions in po litical
and bureaucratic set up of the Government and have earned huge wealth in different
trades, occupation, business and similar ventures. Not only this, the avail ability of
buildings and premises which can be let for non-residential or commercial pur poses has
substantially increased. There fore, the reason/cause which prompted the Division Bench
of the High Court to sustain the differentiation/classification of the pre mises with
reference to the purpose of their user, is no longer available for negating the challenge to
Section 14(l)(e) on the ground of violation of Article 14 of the Constitution, and we
cannot uphold such arbitrary clas sification ignoring the ratio of Harbilas Rai Bansal vs.
State of Punjab (supra), which was reit erated in Joginder Pal vs. Naval Kishore Behal
(supra) and ap proved by three-Judges Bench in Rakesh Vij vs. Dr. Raminder Pal Singh
Sethi (supra). In our con sidered view, the discrimination which was latent in Section
14(l)(e) at the time of enactment of 1958 Act has, with the passage of time (almost 50
years) has be come so pronounced that the impugned pro vision cannot be treated intra
vires Article 14 of the Constitution by applying any ra tional criteria.
29

. It is trite to say that legislation which may be quite reasonable and rational at the time of
its enactment may with the lapse of time and/or due to change of circumstances become
arbitrary, unreasonable and viola tive of the doctrine of equity and even if the validity of
such legislation may have been upheld at a given point of time, the Court may, in
subsequent litigation, strike down the same if it is found that the rationale of
classification has become non-existent. In State of Madhya Pradesh vs. Bhopal Sugar
Industries [AIR 1964 SC 1179], this Court while dealing with a question whether
geographical classification due to historical reasons could be sustained for all times and
observed : At p. 1182, Para 6 of AIR

"Differential treatment arising out of the application of the laws so continued in dif ferent
regions of the same reorganised, State, did not therefore immediately attract the clause of
the Constitution prohibiting discrimination. But by the passage of time, considerations of
necessity and expediency would be obliterated, and the grounds which justified
classification of geographical re gions for historical reason may cease to be valid. A
purely temporary provision which because of compelling forces justified dif ferential
treatment when the Reorganisation Act was enacted cannot obviously be per mitted to
assume permanency, so as to per petuate that treatment without a rational ba sis to
support it after the initial expediency and necessity have disappeared.
30

. In Narottam Kishore Dev Verma vs. Union of India [AIR 1964 SC 15901 the chal lenge
was to the validity of Section 87-B of the Code of Civil Procedure which granted
exemption to the rulers of former Indian States from being sued except with the consent
of the Central Government. In the course of judgment, it was observed as under : Para 11
of AIR

"If under the Constitution all citizens are equal, it may be desirable to confine the
operation of Section 87-B to past transac tions and nor to perpetuate the anomaly of the
distinction between the rest of the citi zens and Rulers of former Indian States. With the
passage of time, the validity of his torical considerations on which Section 87-B is
founded will wear out and the continu ance of the said section in the Code of Civil
Procedure may later be open to serious chal lenge."
31

. In H. H. Shri Swamiji Shri Admar Mutt Etc. v. The Commissioner, Hindu Religious
AIR 1980 SC 1, Para 29

@page-SC3168
and Charitable Endowments Depart ment [1979 (4) SCC 642] this Court was called upon
to consider the validity of the continued application of the provisions of the Madras
Hindu Religious Endowment Act, 1951 in the area which had formerly been part of State
of Madras and which had latter become part of the new State of Mysore (now Karnataka)
as a result of the State Re organisation Act, 1956. While declining to strike down the
legislation on the ground of violation of Article 14 of the Constitution, the Court
observed :
"An indefinite extension and application of unequal laws for all time to come will mili
tate against their true character as tempo rary measures taken in order to serve a tem
porary purpose. Thereby, the very founda tion of their constitutionality shall have been
destroyed the foundation being that Section 119 of the State Reorganisation Act serves
the significant purpose of giving reasonable time to the new units to consider the spe cial
circumstances obtaining in respect of diverse units. The decision to withdraw the
application of unequal laws to equals can not be delayed unreasonably because of the
relevance of historical reasons which jus tify the application of unequal laws is bound to
wear out with the passage of time. In Broom's Legal Maxim (1939 Edition, page 97) can
be found a useful principle "Cessante Ratione Legis Cessat Ipsa Lex", that is to say,
"Reason is the sour of the law, and when the reason of any particular law Ceases, so does
the law itself."
32

. In Motor General Traders vs. State of Andhra Pradesh (su pra), validity of Section
32(b)of the A.P. Buildings (Lease, Rent and Eviction) Control, Act, 1960 was considered.
By that Section it was declared that the provisions of the main Act will not apply to the
build ings constructed after 25th August, 1957. The Court noted that exemption had con
tinued for nearly a quarter century and struck down the same despite the fact that validity
thereon had been upheld by the High Court in Chintapalli Achaiah v. P. Gopala Krishna
Reddy [AIR 1966 AP 51]. Some of the observations made in the judgment are worth
noticing. These are : AIR 1984 SC 121

"What may be unobjectionable as a tran sitional or temporary measure at an initial stage


can still become discriminatory and hence violative of Article 14 of the Constitution if it
is persisted in over a long period without any justification."
"What was justifiable during a short pe riod has turned out to be a case of hostile
discrimination by lapse of nearly a quarter of century....We are constrained to pro nounce
upon the validity of the impugned provision at this late stage because of garb of
Constitution which it may have possessed earlier has become worn out and its uncon
stitutionality is now brought to a success ful challenge".
"As already observed, the landlords of the buildings constructed subsequent to August 26,
1957 are given undue preference over the landlords of buildings constructed prior to that
date in that the former are free from the shackles of the Act while the latter are subjected
to the restrictions imposed by it. What should have been just an incentive has become a
permanent bonanza in favour of those who constructed buildings subsequent to August
26, 1957. There being no justifi cation for the continuance of the benefit to a class of
persons without any rational ba sis whatsoever, the evil effects flowing from the
impugned exemption have caused more harm to the society than one could antici pate.
What was justifiable during a short period has turned out to be a case of hostile
discrimination by lapse of nearly a quarter of century. The second answer to the above
contention is that mere lapse of time does hot lend constitutionality to a provision which
is otherwise bad. "Time does not run in favour of legislation. If it is ultra vires, it cannot
gain legal strength from long failure on the part of lawyers to perceive and set up its
invalidity. Albeit, lateness in an at tack upon the constitutionality of a statute is but a
reason for exercising special cau tion in examining the arguments by which the attack is
supported."
33

. In Rattan Arya and Ors. vs. State of Tamil Nadu and Anr. (supra) the Court relied on the
ratio of Motor General Traders vs. State of Andhra Pradesh (supra) and struck down
Section 30(ii) of the Tamil Nadu Buildings (Lease and Rent) Control Act, 1960 by
observing that there was no ratio nal basis in picking out the class of tenants of residential
buildings paying a rent of more than Rs.400/- per month and to deny simi lar right to
tenants of other buildings and residential or non-residential premises. AIR 1986 SC
1444
AIR 1984 SC 121

@page-SC3169
34

. In Malpe Vishwanath Acharya and Others vs. State of Maharashtra and Another (supra),
the Court found that the criteria for deter mination and fixation of rent by freezing or by
pegging down of rent as on 1.9.1940 or as on first date of letting, had, with the pas sage
of time become irrational and arbitrary but did not strike down the same on the ground
that extended period of Bombay Rent Act was coming to an end on 31.3.1998. 1998
AIR SCW 202

35

. Before parting with this aspect of the case, we may refer to the judg ment of Amarjit
Singh vs. Smt. Khatoon Quamarin (supra), on which reliance has been placed by the Full
Bench of the High Court for negating the appellant's challenge to Section 14(1)(e). In that
case, the respondent sought eviction of the tenant from the first floor of the pre mises
situated at Maharani Bagh, New Delhi on the ground of personal and bona fide ne cessity.
The suit filed by the landlady was decreed by the learned Single Judge of the Delhi High
Court and a direction was issued for eviction of the tenant (appellant). This Court referred
to the earlier judgments in Pasupuleti Venkateswarlu vs. Motor and General Traders
[1975 (1) SCC 770], Hasmat Rai vs. Raghunath Prasad [1981 (3) SCC 103] and held that
in view of the availability of alternative accommodation to the landlady, the High Court
was not justi fied in ordering eviction of the tenant. AIR 1987 SC 741
AIR 1975 SC 1409
AIR 1981 SC 1711
36. A careful reading of the aforemen tioned judgment shows that the plea of
unconstitutionality of Section 14(1)(e) of the 1958 Act was neither raised nor debated
with any seriousness and the observation made by the Court in that regard cannot be
treated as the true ratio of the judgment, which as mentioned above, mainly rested on the
interpretation of the expression "rea sonably suitable residential accommoda tion". The
bedrock of the respondent's claim was that she had a right to comfortable liv ing and
availability of alternative accommo dation, by itself not sufficient for declining eviction
of the tenant. While rejecting this argument, the Court observed :
"17. .............The logic of the argument of Shri Kacker is attractive, but the legality of the
said submission is unsustainable. Rent restriction laws are both beneficial and re strictive,
beneficial for those who want pro tection from eviction and rack-renting but restrictive so
far as the landlord's right or claim for eviction is concerned. Rent restric tion laws would
provide a habitat for the landlord or landlady if need be, but not to seek comforts other
than habitat that right the landlord must seek elsewhere."
37. Another contention raised on behalf of the landlady was that Section 14(1)(e) of the
1958 Act should be read in a manner which will make it in conformity with Ar ticles 14
and 16 of the Constitution. This is evinced from para 18 of the judgment which is
extracted below :-

"18. Our attention was drawn to the de cision in the case of Bishambhar Dayal Chandra
Mohan v. State of U.P.[1982 (1) SCC 39] and our attention was drawn to the observations
at p. 66 and 67 of the said case in aid of the submission that right to property is still a
constitutional right and therefore in exercise of that right if a landlord or an owner of a
house lets out a premises in ques tion there was nothing wrong. Shri Kacker submitted
that the second limb of Section 14(1)(e) of the Act should be read in such a way that it
was in consonance with Article 14 and Article 21 of the Constitution. Oth erwise it would
be void as being unconsti tutional. As a general proposition of law this is acceptable."
AIR 1982 SC 33

The Court rejected the argument and ob served :


"The Act in question has the authority of law. There is no denial of equality nor any
arbitrariness in the second limb of Section 14(1)(e) of the Act, read in the manner
contended for by the appellant. Article 21 is not violated so far as the landlord is
concerned. The rent restricting Acts are beneficial leg islations for the protection of the
weaker party in the bargains of letting very often. These must be so read that these
balance harmoniously the rights of the landlords and the obligations of the tenants. The
Rent Re striction Acts deal with the problem of rack renting and shortage of
accommodation. It is in consonance with the recognition of the right of both the landlord
and the tenant that a harmony is sought to be struck whereby the bona fide requirements
of the landlords and the tenants in the expanding explosion of need and population and
short age of accommodation are sought to be harmonised and the conditions imposed to
evict a tenant are that the landlord must have bona fide need. That is satisfied in this
@page-SC3170
case. That position is not disputed. The sec ond condition is that landlord should not have
in his or her possession any other rea sonably suitable accommodation. This does not
violate either Article 14 or Article 21 of the Constitution."
38. In view of the above discussion, we hold that Section 14(1)(e) of the 1958 Act is
violative of the doctrine of equality embod ied in Article 14 of the Constitution of India
insofar as it discriminates between the pre mises let for residential and non-residential
purposes when the same are required bona fide by the landlord for occupation for him
self or for any member of his family depen dent on him and restricts the latter's right to
seek eviction of the tenant from the pre mises let for residential purposes only.
39. However, the aforesaid declaration should not be misunderstood as total strik ing
down of Section 14(1)(e) of the 1958 Act because it is neither the pleaded case of the
parties nor the learned counsel argued that Section 14(1)(e) is unconstitutional in its
entirety and we feel that ends of justice will be met by striking down the discriminatory
portion of Section 14(1)(e) so that the re maining part thereof may read as under :-
"that the premises are required bona fide by the landlord for himself or for any mem ber
of his family dependent on him, if he is the owner thereof, or for any person for whose
benefit the premises are held and that the landlord or such person has no other reasonably
suitable accommodation."
While adopting this course, we have kept in view well recognized rule that if the of
fending portion of a statute can be severed without doing violence to the remaining part
thereof, then such a course is permissible R.M.D. Chamarbaugwalla vs. Union of In dia
(AIR 1957 SC 628) and Bhawani Singh vs. State of Rajasthan [1996 (3) SCC 105].
As a sequel to the above, the explanation appearing below Section 14(1)(e) of the 1958
Act will have to be treated as redundant.
40. In the result, the appeals are al lowed. The impugned judgment is set aside and
Section 14(1)(e) of the 1958 Act is partly struck down. Section 14(1)(e) shall now read as
indicated in para 39 above. Conse quently, the writ petitions filed by the ap pellants shall
stand allowed and the orders impugned therein shall stand quashed. The parties are left to
bear their own costs.
Appeals dismissed.
AIR 2008 SUPREME COURT 3170 "Govt. of A. P. v. K. Brahmanandam"
(From : Andhra Pradesh)
Coram : 2 S. B. SINHA AND LOKESHWAR SINGH PANTA, JJ.
Civil Appeal No. 3043 of 2008 (arising out of SLP (C.) No. 20561 of 2006), D/- 29 -4
-2008.
Govt. of A. P. and Ors. v. K. Brahmanandam and Ors.
(A) A.P. Education Act (1 of 1982), S.12 - Constitution of India, Art.16 - EDUCATION -
SALARIES - STATE - Salary - Liability of State - Right of teacher of recognised school
to claim salary - Must arise under a con tract or under a Statute - If such a right arises
under contract between appointee and institution, only the latter would be liable therefor -
If State is not liable in terms of Statute, no legal right accrues in favour of those who had
been appointed in violation of Rules. (Para 11)
(B) Constitution of India, Art.16, Art.14, Art.226 - EQUALITY IN PUBLIC
EMPLOYMENT - WRITS - REGULARISATION OF SERVICE - APPOINTMENT -
Regularisation of service - Appointments made in violation of manda tory provision of a
statute - Court ordinarily would not issue writ of mandamus for regularisation of service.
(Paras 12, 13)
(C) A.P. Education Act (1 of 1982), S.99, S.20, S.83 - A.P. Educational Institutions
(Establishment, Recognition, Administration and Control of Schools under Private
Managements) Rules (1993), R.12 - EDUCATION - RECRUITMENT - CONTRACT -
DOCTRINES - Salary - Claim by teachers - Recruitment of respondents as teachers -
Provisions of Recruitment Rules had not followed - Even order of approval from District
Education Officer was not obtained - Question of regularising services of respondents
does not arise - However, in view of continu ous service they are entitled to salary from
school authorities - Doctrine of quasi contract cannot be applied as against State.
Contract Act (9 of 1872), S.70.
2006 AIR SCW 1991; 2007 AIR SCW 1018; 2006 AIR SCW 3865; 2007 AIR SCW
6301; 2007 AIR SCW 589 and 2007 AIR
@page-SC3171
SCW 7569, Rel. on.
AIR 1987 SC 454, Ref.
W. A. No. 1321 of 2001, D/-25-08-2005 (AP), Reversed. (Para 18)
Cases Referred : Chronological Paras
2007 AIR SCW 589 : AIR 2007 SC 893 (Rel. on) 15
2007 AIR SCW 1018 : AIR 2007 SC 1082 (Rel. on) 15
2007 AIR SCW 6301 : AIR 2007 SC (Supp) 995 (Rel. on) 15
2007 AIR SCW 7569 (Rel. on) 15
2006 AIR SCW 1991 : AIR 2006 SC 1806 : 2006 (3) AIR Kar R 320 (Rel. on) 9, 13,
14, 16, 18, 19
2006 AIR SCW 3865 : 2006 (5) AIR Bom R 449 (Rel. on) 14
AIR 1987 SC 454 : 1986 Lab IC 1417 (Ref.) 7
AIR 1979 SC 1676 : 1976 Lab IC 1206 13
AIR 1972 SC 1767 : 1972 Lab IC 618 13
AIR 1967 SC 1071 13
R. Sundraravardhan, Sr. Advocate, Mrs. D. Bharathi Reddy, Ms. Altaf Fatima and Debojit
Barkakoti, for Appellants; G. Ramakrishna Prasad, Suyodhan Byra paneni, Siddarth
Patnaik and G. Arun, for Respondents.
Judgement
S. B. SINHA, J. :- Leave granted.
2. Whether the State or the Educational Institution is liable to bear the financial burden
for payment of wages to the con cerned respondents herein is the question involved in
this appeal which arises out of a judgment and order dated 25.08.2005 passed by the High
Court of Andhra Pradesh in Writ Appeal No. 1321 of 2001.
3. Respondents, seven in number, were appointed as Secondary Grade Teachers in
Church of South India, UP Elementary School. Allegedly, the provisions of the rules had
not been followed in recruiting the teachers. Indisputably, such rules of recruit ment had
been laid down by G.O.Ms. No. 1 dated 1.01.1994. The said rules were framed by the
State in exercise of its power con ferred upon it under Section 99 read with Sections 20,
21, 79, 80 and 83 of the Andhra Pradesh Education Act, 1982 known as the Andhra
Pradesh Educational Institutions (Establishment, Recognition, Administration and
Control of Schools Under Private Man agements) Rules, 1993 (for short "the Rules").
4. The Rules categorized several schools; Upper Primary Schools being one of them. The
Rules defines the "Educational Agency" in Rule 2(1)(b) to mean "the Society/ Trust/
Association including Endowment, Board/ Wakf Board and Christian Mission (Church/
Diocese or Congregation) and the like, spon soring/ managing/ running the schools".
"Minority Educational Institution" has been defined in Rule 2(f) of the Rules to mean
"any educational agency of which at least 2/3rd members belong to a religious/linguistic
minority".
5. Rule 7 of the Rules provides for scru tiny and grant of permission on an applica tion
filed therefor by the institution in ques tion. Rule 9 provides for the manner in which
recognition can be granted. Rule 10 provides for the conditions for grant of permission
and recognition. Rule 12 provides for the appointment of staff. Sub-rule (3) of Rule 12
mandates that advertisement for recruit ment shall be made at least in two newspa pers
having large circulation. The Employ ment Exchange is also required to be noti fied in
regard to the vacancies. A Staff Se lection Committee constituted for undertak ing the
recruitment process is to consist of a nominee of the District Educational Of ficer not
below the rank of Deputy Educa tional Officer. Sub-rule (8) of Rule 12 pro vides that all
appointments should be sub ject to the approval of the competent au thority.
6. It is stated that the management of the institution, before the recruitment of the
respondents, neither obtained any prior per mission from the District Educational Of ficer
nor made advertisement in two news papers nor notified the vacancies to the Em
ployment Exchange. Even no order of ap proval as regards the said appointments was
obtained from the District Education Officer.
The State contends that the selection pro cess had been undergone hurriedly, which
created a lot of suspicion.
Respondents, however, contend that they were appointed as Secondary Grade Teach ers
at different places through due selec tion process and they had been performing their
duties to the utmost satisfaction of the authorities of the concerned schools. Indis putably,
their salaries had not been paid. They made representations therefor. Their representations
were rejected by the District Education Officer by an order dated 10.12.1999.
@page-SC3172
7. Respondents thereafter filed writ peti tions before the High Court. The State filed a
counter affidavit wherein it was inter alia contended that the writ petitioners- respon dents
had been appointed through side door(s) by the then Correspondent Rev. Prasad Rao in
collusion with the teachers concerned as also the then Education Of ficer.
A learned Single Judge of the High Court relying on the principles laid down by this
Court in Ashok Kumar Yadav v. State of Haryana [AIR 1987 SC 454] as also on the
premise that the said respondents have been working for several years and as further
more they possessed minimum qualification held that only because the procedural as
pects had not been followed as per the said GOMs No. 1 dated 1.01.1994 and other di
rections from time to time, the same would not be a bar for grant of relief in their favour,
stating :
"In my considered view, the same prin ciple will also apply to the facts of this case.
Admittedly, the petitioners are continuing in service for more than 8 years and it would
be inequitable to disturb them at this dis tance of time."
8. On an intra court appeal having been filed, a Division Bench of the said Court dis
missed the appeal, stating :
"The main grievance of the respondents is that though they were appointed as Sec ondary
Grade Teachers, through due selec tion process, neither their appointments were approved
nor they were paid any sal ary till date. In earlier round of litigation, in W.P. No. 9616 of
1995, this court directed the authorities concerned to consider the proposals sent by the
Management on 1.4.1996 and take appropriate decision. In pursuance thereof, the fourth
appellant passed orders on 10.12.1999 rejecting the cases for approval. The case of the
respon dents is that they have put up sufficient length of service. The learned Single
Judge placed reliance on the Judgment cited su pra and held that it would be unjust to dis
turb the respondents after eight years of service and accordingly set aside the im pugned
order passed by the fourth appel lant. In the above background of the case, we are of the
opinion that the learned Single Judge has arrived at a just conclusion and the same, in our
considered opinion, de serves no interference."
9

. Mr. R. Sundraravardhan, learned se nior counsel appearing on behalf of the ap pellants,


would submit that the State has no liability to pay the salary of the concerned teachers
keeping in view the fact that their services had not been approved. The learned counsel
would further contend that it is not even a case where paragraph 53 of the Con stitution
Bench decision of this Court in Sec retary, State of Karnataka and Others v. Umadevi (3)
and Oth ers [(2006) 4 SCC 1] would ap ply. 2006 AIR SCW 1991

10. Mr. G. Ramakrishna Prasad, learned counsel appearing on behalf of the respon dents,
on the other hand, would contend that in view of passage of time and particu larly in view
of the fact that the respondents had been continuing to work for a long time, this Court
should not interfere with the im pugned judgment.
11. The liability of the State to pay salary to a teacher appointed in the recognized schools
would arise provided the provisions of the statutory rules are complied with, subject to
just exception. The right to claim salary must arise under a contract or under a statute. If
such a right arises under a contract between the appointee and the in stitution, only the
latter would be liable therefor. Its right in certain situation to claim reimbursement of
such salary from the State would only arise in terms of the law as was prevailing at the
relevant time. If the State in terms of the statute is not liable to pay the salary to the
teachers, no legal right ac crues in favour of those who had been ap pointed in violation
of mandatory provisions of the statute or statutory rules.
12. The equality clause contained in Ar ticles 14 and 16 of the Constitution of In dia, it is
trite, must be scrupulously followed. The court ordinarily would not issue a writ of or in
the nature of mandamus for regu larization of the service of the employee which would
be violative of the constitutional scheme.
13. Appointments made in violation of the mandatory provisions of a statute would be
illegal and, thus, void. Illegality cannot be ratified. Illegality cannot be regularized, only
an irregularity can be.

The said legal principle has been enunciated by a Constitution Bench of this Court in
Umadevi (3) (supra), para 53 2006 AIR SCW 1991, (Para 44)

@page-SC3173
whereof reads as under :

"53. One aspect needs to be clarified. There may be cases where irregular appointments
(not illegal appointments) as ex plained in S.V. Narayanappa, R.N. Nanjundappa and
B.N. Nagarajan and referred to in para 15 above, of duly qualified per sons in duly
sanctioned vacant posts might have been made and the employees have continued to
work for ten years or more but without the intervention of orders of the courts or of
tribunals. The question of regularisation of the services of such em ployees may have to
be considered on mer its in the light of the principles settled by this Court in the cases
abovereferred to and in the light of this judgment. In that con text, the Union of India, the
State Govern ments and their instrumentalities should take steps to regularise as a one-
time mea sure, the services of such irregularly ap pointed, who have worked for ten years
or more in duly sanctioned posts but not un der cover of orders of the courts or of tribu
nals and should further ensure that regular recruitments are undertaken to fill those vacant
sanctioned posts that require to be filled up, in cases where temporary employ ees or
daily wagers are being now employed. The process must be set in motion within six
months from this date. We also clarify that regularisation, if any already made, but not
sub judice, need not be reopened based on this judgment, but there should be no further
bypassing of the constitutional re quirement and regularising or making per manent, those
not duly appointed as per the constitutional scheme." AIR 1967 SC 1071
AIR 1972 SC 1767
AIR 1979 SC 1676

[Emphasis supplied]
14

. We are not unmindful of the fact that the said paragraph has been interpreted differently
by different Benches. Some benches have remitted the matter back to the
tribunal/authorities for consideration of the mat ter afresh in the light of the said
observations, e.g., in Mineral Exploration Corpora tion Employees' Union v. Mineral
Exploration Corporation. Ltd. and Ann [(2006) 6 SCC 310], it was directed : 2006
AIR SCW 3865 (Para 39)

"We, therefore, direct the Tribunal to de cide the claim of the workmen of the Union
strictly in accordance with and in compli ance with all the directions given in the judg
ment by the Constitution Bench in Secy., State of Karnataka v. Umadevi (3) and in
particular, paras 53 and 12 relied on by the learned Senior Counsel appearing for the
Union. The Tribunal is di rected to dispose of the matter afresh within 9 months from the
date of receipt of this judgment without being influenced by any of the observations made
by us in this judg ment. Both the parties are at liberty to sub mit and furnish the details in
regard to the names of the workmen, nature of the work, pay scales and the wages drawn
by them from time to time and the transfers of the workmen made from time to time,
from place to place and other necessary and requisite details. The above details shall be
submit ted within two months from the date of the receipt of this judgment before the
Tribu nal." 2006 AIR SCW 1991

15
. On the other hand, in some of the cases, the said paragraph, for example, in the decision
of this Court in Municipal Cor poration, Jabalpur v. Om Prakash Dubey [(2007) 1 SCC
373] had been applied to the fol lowing effect : 2007 AIR SCW 589

"The question which, thus, arises for con sideration, would be : Is there any distinc tion
between 'irregular appointment' and 'il legal appointment'? The distinction between the
two terms is apparent. In the event the appointment is made in total disregard of the
constitutional scheme as also the re cruitment rules framed by the employer, which is
State within the meaning of Article 12 of the Constitution of India, the recruit ment would
be an illegal one; whereas there may be cases where, although, substantial compliance of
the constitutional scheme as also the rules have been made, the appoint ment may be
irregular in the sense that some provisions of the rules might not have been strictly
adhered to."

[See also Punjab Water Sup ply and Sewerage Board v. Ranjodh Singh and Others etc.
(2007) 2 SCC 491, Punjab State Ware housing Corp., Chandigarh v. Manmohan Singh
and Anr., 2007 (3) SCALE 401 and Post Master General, Kolkata and Others v. Tutu Das
(Dutta) 2007 (6) SCALE 453] 2007 AIR SCW 1018
2007 AIR SCW 6301
2007 AIR SCW 7569

16

. In the light of the decision of this Court in Umadevi (3) (su pra), para 53 thereof would
be applicable subject to the condi tion that the matter had not been 2006 AIR SCW 1991,
(Para 44)

@page-SC3174
pending before any court or tribunal. Indis putably, the litigation between the parties was
pending since January, 2000. The institution's application for approval of the said
appointments had been rejected. There fore, para 53 of Umadevi (3) (supra) has no
application.
17. Even in relation to application of the concept of equal pay for equal work, the
Constitution Bench held :
"44. The concept of "equal pay for equal work" is different from the concept of con
ferring permanency on those who have been appointed on ad hoc basis, temporary ba sis,
or based on no process of selection as envisaged by the rules. This Court has in various
decisions applied the principle of equal pay for equal work and has laid down the
parameters for the application of that principle. The decisions are rested on the concept of
equality enshrined in our Con stitution in the light of the directive prin ciples in that
behalf. But the acceptance of that principle cannot lead to a position where the court
could direct that appointments made without following the due procedure established by
law, be deemed permanent or issue directions to treat them as perma nent. Doing so,
would be negation of the principle of equality of opportunity. The power to make an
order as is necessary for doing complete justice in any cause or mat ter pending before
this Court, would not normally be used for giving the go-by to the procedure established
by law in the matter of public employment. Take the situation arising in the cases before
us from the State of Karnataka. Therein, after Dharwad deci sion the Government had
issued repeated directions and mandatory orders that no temporary or ad hoc employment
or engage ment be given. Some of the authorities and departments had ignored those
directions or defied those directions and had contin ued to give employment, specifically
inter dicted by the orders issued by the execu tive. Some of the appointing officers have
even been punished for their defiance. It would not be just or proper to pass an order in
exercise of jurisdiction under Article 226 or 32 of the Constitution or in exercise of power
under Article 142 of the Constitution permitting those persons engaged, to be absorbed or
to be made permanent, based on their appointments or engagements. Complete justice
would be justice accord ing to law and though it would be open to this Court to mould the
relief, this Court would not grant a relief which would amount to perpetuating an
illegality."
18

. In view of the decision in Umadevi (3) (supra), we are of the opinion that the question
of regularizing the services of the respondents does not arise. Respondents-writ
petitioners (teachers), however, are entitled to salary from the school authorities as they
have worked even if no valid contract had come into being. The salary amount would be
payable in terms of Section 70 of the Indian Contract Act. The principles of quasi-
contract, however, must apply keeping in view the relationship be tween the parties. The
doctrine of quasi-contract cannot be applied in a situation of this nature as against the
State. 2006 AIR SCW 1991

19. For the reasons aforementioned, the appeal is allowed to the aforementioned ex tent.
It would, however, be open to the school authorities to take such action, as it may deem
fit and proper, in the light of the deci sion of this Court in Umadevi (3) (supra). No costs.
Appeal allowed.
AIR 2008 SUPREME COURT 3174 "Kamala v. K. T. Eshwara Sa"
(From : Karnataka)*
Coram : 2 S. B. SINHA AND V. S. SIRPURKAR, JJ.
Civil Appeal No. 3038 of 2008 (arising out of SLP (C) No. 9222 of 2007), D/- 29 -4
-2008.
Kamala and Ors. v. K. T. Eshwara Sa and Ors.
(A) Civil P.C. (5 of 1908), O.7, R.11(d) - PLAINT - Rejection of plaint - R.11(d) of O.7
has limited application - Conclusion that suit is barred under any Law must be drawn
from averments made in plaint. (Para 15)
(B) Civil P.C. (5 of 1908), O.7, R.11(a), R.11(b), R.11(c), R.11(d) - PLAINT - Rejection
of plaint - Different clauses in R.11 of O.7 should not be mixed up. (Para 15)
(C) Civil P.C. (5 of 1908), O.7, R.11, O.14, R.2 - PLAINT - CIVIL PROCEDURE -
OBJECT OF AN ACT - Scope - Absence of juris diction on part of Court can be invoked
at different stages - O.7, R.11 is one and O.14, R.2 is another. (Para 15)
(D) Civil P.C. (5 of 1908), O.7, R.11(d) - PLAINT - Rejection of plaint - For invoking
R.11(d) of O.7, no amount of evidence can
@page-SC3175
be looked into - At that stage issues on merit of matter would not be within realm of
Court. (Para 16)
(E) Civil P.C. (5 of 1908), O.7, R.11(d), S.12 - PLAINT - PARTITION - Rejection of
plaint - Partition suit - Shares of parties were defined - There was partition amongst
parties in sense that they could transfer their un divided share - What would be effect of
partition suit which had not been taken to its logical conclusion by getting prop erties
partitioned by metes and bounds - Is a question which cannot be gone in proceeding
under O.7, R.11(d).
R. F. A. No. 1761 of 2005, D/-13-02-2007 (Kant), Reversed.
A suit was filed claiming partition in the properties. After passing of the preliminary
decree, no property was available for parti tion. The properties were possessed by the co-
sharers independently in accordance with the respective shares held by the co-sharer.
There had been a division of the joint family properties by metes and bounds resulting in
complete severance of status. What would, however, be the effect of a partition suit which
had not been taken to its logical con clusion by getting the properties partitioned by metes
and bounds is a question which, cannot be gone into in a proceeding under Order VII,
Rule 11(d) of the Code. Whether any property is available for partition is it self a
question of fact. Identity of properties which were subject-matter of the earlier suit vis-a-
vis properties which were subsequently acquired and effect thereof is beyond the purview
of Order VII, Rule 11(d).
1994 AIR SCW 243; (2004) 9 SCC 512; 2007 AIR SCW 6953, Ref. to.
R. F. A. No. 1761 of 2005, D/-13-02-2007 (Kant), Reversed. (Paras 21, 24, 33)
Cases Referred : Chronological Paras
2007 AIR SCW 6953 :AIR 2008 SC 363 (Ref. to) 26
(2006) 5 SCC 658 28
(2006) 5 SCC 662 28
(2005) 7 SCC 510 26, 28
(2004) 9 SCC 512 (Ref. to) 26
1994 AIR SCW 243 : AIR 1994 SC 853 (Ref. to) 25
AIR 1986 SC 1253 : 1986 All LJ 62526
AIR 1982 SC 760 31
AIR 1977 SC 2421 30
AIR 1961 SC 1077 32
AIR 1952 Cal 579 29
(1936) 1 All ER 287 : 154 LT 423 28
S. N. Bhat, for Appellants; R. Venkata ramani, Sr. Advocate, Shanmukhappa, S. Balaji,
Mallikarjun S., Ms. E. R. Sumathy, S. R. Sharma, S. Srinivasan, Ms. Madhumita Bora, S.
Balaji, P. R. Ramasesh, G. C. Bharuka, A. Mohan Sinha, Mrs. Hansa Bharuka, H. S.
Somnath and Devashish Bharuka, for Respondents.
* R. F. A. No. 1761 of 2005, D/- 13-2-2007 (Kant.)
Judgement
1. S. B. SINHA, J. :- Leave granted.
2. Application of Order VII, Rule 11(d) of the Code of Civil Procedure (for short "the
Code") in the facts and circumstances of this case, is involved in this appeal which arises
out of a judgment and order dated 13.02.2007 passed by a Division Bench of the High
Court of Karnataka at Bangalore.
3. The relationship between the par ties is not in dispute, as would appear from the
genealogical tree :

Allegedly, the eldest son of Kabadi Gopalsa went out of the joint family by ex ecuting a
registered Deed of Release upon taking his share in the ancestral property on or about
10.03.1918.
4. A partition is said to have taken place between two sons of Chinnusa, i.e., Kabadi
Giddusa and Kabadi Gopalsa on or about 1.05.1926. Kabadi Gopalsa died in 1947.
5. There exists a dispute as to whether the properties in suit were divided amongst the
four sons of Kabadi Gopalsa. However, admittedly, a suit was filed by Ramusa (son of
Gopalsa) against his mother and three brothers in respect of three house proper ties being
Item Nos. 1, 2 and 3 and the Rev enue land (Item No. 4). Defendant No. 3 in the said suit
was the grand-father of the deceased-husband of the appellant No. 1 in the present case.
6. It is not in dispute that on or about 11.11.1952, the properties which allegedly fell to
the share of Chikka Chinnusa was auction sold in favour of one Moolchand Sharma in
execution of a decree passed against him in OS No. 311 of 1948-49 being Execution No.
421 of 1950-51.
7. A preliminary decree was passed by the Trial Court declaring 2/9th share of the
plaintiff. It is, however, conceded at the Bar that the said decree was rectified declaring
the share of the plaintiff to be 1/4th in the
@page-SC3176
joint family property. A final decree pro ceedings was initiated. During the pendency of
the said proceedings, Moolchand Sharma sold his land in Survey Nos. 22 and 23 ad
measuring 1 acre 0.38 guntas, Survey No. 48/2 admeasuring 0.32 guntas and Survey No.
48/5 admeasuring 0.13½ guntas to Munimarappa.
A final decree was said to have been passed on 11.06.1955. Yet again, Ramusa
@page-SC3177
executed a registered deed of sale on 30.08.1956 in favour of R. Vittal Sa in re spect of 2
acres and 1 3/4 guntas in Survey Nos. 22 and 23, 0.29 3/4 guntas in Survey No. 47/2 and
0.13 ½ guntas in Survey No. 48/ 5. Dodda Chinnusa executed a registered deed of sale
on 2.09.1956 in favour of K.G. Daktappa in respect of 2 acres and 1 3/4 guntas in Survey
Nos. 22 and 23, 0.29 3/4 guntas in Survey No. 47/2 and 0.13 3/4 guntas in Survey No.
48/5.
8. By an order dated 18.06.1956, the Trial Court directed the Commissioner to demar cate
the lands falling in the share of the plaintiff and allot to him. Various interlocu tory
proceedings were initiated and several orders were passed thereupon. As noticed
hereinbefore, the preliminary decree was amended declaring 1/4th share of the plain tiff
and the defendant Nos. 1 to 3 with re spect to all the properties by reason of an order
dated 27.02.1963. Whereas accord ing to the respondents, the parties had taken
possession of the properties fallen in their respective shares and had been enjoying and
even alienating them to the third parties, the appellant strenuously denied and dis puted
the same.
An order of injunction was passed in the said suit being OS No. 15 of 1953 by an or der
dated 20.03.1963 restraining the defen dant No. 2 from transferring the suit sched ule
properties on the premise that the joint family property had not been divided by metes
and bounds. However, while setting aside the said interim order of injunction, the learned
Court by an order dated 7.07.1967 observed as under :
(i) On 27.02.1963, the preliminary decree was amended and 1/4th share of Plaintiff and
Defendants 1 to 3 was defined.
(ii) Item No. 4 of the suit property is rev enue property.
(iii) Defendant No. 3 (grandfather of de ceased-husband of Plaintiff-Appellant herein) has
sold its share in Item No. 4 of the plaint schedule property.
(iv) 'The suit is pending till the final de cree is passed. No final decree as such has been
passed in this suit concerning the 4th item of the plaint schedule. It is true that the Civil
Court has to simply forward the preliminary decree to the Collector for pur poses of
partitioning the same and that the Civil Court has no jurisdiction to correct or review the
partition that may be made by the Collector'.
The final decree proceeding was, however, dismissed for default on or about 03.09.1974.
9. Respondent No. 1 thereafter filed a partition suit against Respondent No. 2 in the Court
of City Civil Judge at Bangalore which was marked as OS No. 6180 of 2003. The said
suit was dismissed as not pressed.
10. Appellant has filed a suit which was marked as OS No. 6352 of 2004 claiming
partition in the properties, being the same as were described as Item Nos. 1, 2, 3 and 4 of
the schedule appended to the plaint in OS No. 15 of 1953.
In the said suit, an application for rejec tion of the plaint was filed by the respon dents
which has been allowed by the learned trial Judge and affirmed by the High Court by
reason of the impugned judgment.
11. Mr. S. N. Bhat, learned counsel ap pearing on behalf of the appellants, inter alia
would submit that as in the preliminary decree passed in OS No. 15 of 1953 only the
share of Ramusa, plaintiff therein, namely, his 2/9th share, which was amended as 1/ 4th
share, was declared and furthermore in view of the fact that no decree was passed in the
final decree proceedings, the suit for partition was maintainable.
The subject-matter of the said suit, it was urged, was three houses and the properties
which have been alienated. Whereas the house properties are said to have been di vided,
the alienated properties were not, as would appear from the order dated 11.06.1955 and in
that view of the matter, the impugned judgments cannot be sus tained.
Mr. P.R. Ramasesh, learned counsel adopted the submission of Mr. Bhat.
12. Mr. G.C. Bharuka, and Mr. R. Venkataramani, learned senior counsel ap pearing on
behalf of the respondents, on the other hand, would submit :
(i) After passing of the preliminary decree, no property was available for partition. The
properties were possessed by the co-sharers independently in accordance with the
respective shares held by the co-sharer.
(ii) There had been a division of the joint family properties by metes and bounds re
sulting in complete severance of status, which having been admitted in the plaint, no
cause of action survives for grant of a
@page-SC3178
decree for partition.
(iii) Defendant No. 3 Chikka Chinnusa, who remained ex parte, unsuccessfully tried to
reopen the proceedings and obtained an order of injunction pursuant to the sale ef fected
by the court in execution of a decree passed against him, but, in the year 1967, the said
proceedings were dropped and thus, he is bound thereby.
(iv) As would appear from the order dated 3.09.1974, severance of joint status being not
vitiated by any fraud, which has resulted in complete division of the properties should not
be permitted to be reopened at this stage.
(v) In any event, sale deeds having been executed by the co-sharers from the years 1954
to 1956 and their validity having not been assailed directly, the same cannot be done in an
indirect manner, the suit for par tition is not maintainable.
(vi) In a proceeding under Order VII, Rule 11(d) of the Code, the court would be en titled
to look into the documents which have been annexed to the plaint and in that view of the
matter, recitals made therein may also be looked into for the purpose of determin ing the
question as to whether there had been a complete severance of joint status.
(vii) As none of the properties are avail able in an original undivided condition, the
impugned order should not be interfered with.
13. Order VII, Rule 11 of the Code pro vides for rejection of plaint, clause (d) whereof
specifies "where the suit appears from the statement in the plaint to be barred by any
law".
14. The learned Trial Judge as also the High Court proceeded to pass the impugned order
relying on or on the basis of the pre liminary decree dated 20.03.1963 and the appellate
orders. The High Court opined that the conclusion of the learned Trial Judge directing
rejection of plaint was correct hav ing regard to the provisions contained in Section 12 of
the Code read with Order II, Rule 2 thereof. It was held that no cause of action was
disclosed in the suit.
15. Order VII, Rule 11(d) of the Code has limited application. It must be shown that the
suit is barred under any law. Such a conclusion must be drawn from the aver ments made
in the plaint. Different clauses in Order VII, Rule 11, in our opinion, should not be mixed
up. Whereas in a given case, an application for rejection of the plaint may be filed on
more than one ground specified in various sub-clauses thereof, a clear find ing to that
effect must be arrived at. What would be relevant for invoking clause (d) of Order VII,
Rule 11 of the Code is the aver ments made in the plaint. For that pur pose, there cannot
be any addition or sub traction. Absence of jurisdiction on the part of a court can be
invoked at different stages and under different provisions of the Code. Order VII, Rule 11
of the Code is one, Order XIV, Rule 2 is another.
16. For the purpose of invoking Order VII, Rule 11 (d) of the Code, no amount of evi
dence can be looked into. The issues on merit of the matter which may arise between the
parties would not be within the realm of the court at that stage. All issues shall not be the
subject-matter of an order under the said provision.
The principles of res judicata, when at tracted, would bar another suit in view of Section
12 of the Code. The question involv ing a mixed question of law and fact which may
require not only examination of the plaint but also other evidence and the order passed in
the earlier suit may be taken up either as a preliminary issue or at the final hearing, but,
the said question cannot be determined at that stage.
It is one thing to say that the averments made in the plaint on their face discloses no cause
of action, but it is another thing to say that although the same discloses a cause of action,
the same is barred by a law.
The decisions rendered by this Court as also by various High Courts are not uniform in
this behalf. But, then the broad prin ciple which can be culled out therefrom is that the
court at that stage would not con sider any evidence or enter into a disputed question of
fact of law. In the event, the jurisdiction of the court is found to be barred by any law,
meaning thereby, the subject-matter thereof, the application for registra tion of plaint
should be entertained.
17. The preliminary decree which was passed in OS No. 15 of 1953 reads as un der :
"Its order and decree except against de fendant Nos. 5 and 6 declaring the plain tiffs right
to 2/9th share in the entire joint family properties. There shall be equitable division by
metes and bounds of the plain tiff 2/9th share. The defendant Nos. 1 to 4
@page-SC3179
and 8 and 7 to deliver the plaintiff posses sion of 2/9th share in the said properties. 3rd
defendant shall render proper accounts for the declaration of profits and rents made by
him on enquiry require under Order 20 Rule XII regarding future amounts prof its........"
18. The said decree, however, was amended on 27.02.1963, as would appear from the
order dated 07.06.1967, to which we have adverted to heretobefore.
19. It is, however, beyond any doubt or dispute that a final decree proceedings was
initiated. An Advocate-Commissioner was appointed. Directions were issued therein from
time to time. But, indisputably, there had been no partition by metes and bounds. The
landed property was not partitioned. In its order dated 20.03.1963, the court no ticed that
separate sale deeds were executed by the defendants but despite the same, an order of
injunction was passed to the fol lowing effect :
"1) They should not remove the earth for the purpose of making bricks; and
2) They should not construct anything, on the suit property. I.A. 22 is allowed. No order
as to costs."
20. The final decree proceedings were ultimately dropped by an order dated 3.09.1974.
Neither the Trial Court nor the High Court had taken into consideration the effect and
purport thereof. In the aforemen tioned context, the plaint filed by the appel lants herein
whether deserved outright re jection is the question.
21. Dr. Bharuka and Mr. Venkataramani have taken great pains to read the entire plaint
before us as well as a large number of documents to contend that no cause of ac tion was
disclosed and in any event, the suit was barred by the principle of res judicata.
The other limbs of arguments which have been advanced before us, viz., keeping in view
the deeds of sale executed by the re spondents and the court auction sale which had taken
place in respect of the appellants' share, had not been raised before the learned Trial
Judge.
We may proceed on the assumption that the shares of the parties were defined. There was
a partition amongst the parties in the sense that they could transfer their undi vided share.
What would, however, be the effect of a partition suit which had not been taken to its
logical conclusion by getting the properties partitioned by metes and bounds is a question
which, in our opinion, cannot be gone into in a proceeding under Order VII, Rule 11(d) of
the Code. Whether any property is available for partition is itself a question of fact.
Whether the suit would be maintainable, if the plaintiff had not questioned the valid ity of
deeds of sale, is not the question which can be answered by us at this stage.
The only contention raised before the learned Trial Judge was the applicability of the
principles of res judicata. Even for the said purpose, questions of fact cannot be gone
into. What can only be seen are the averments made in the plaint. What inter alia would
be relevant is as to whether for the said purpose the properties were sold by reason of any
arrangement entered into by and between the parties out of court; whether they had
accepted the partition or whether separate possession preceded the actual sale; or whether
the contention that a presumption must be drawn that for all practical purposes the parties
were in sepa rate possession, are again matters which would not fall for consideration of
the court at this stage.
22. The plaintiff-appellant might not have prayed for any decree for setting aside the
deeds of sale but they have raised a legal plea that by reason thereof the rights of the
coparceners have not been taken away. Their status might not be of the coparceners, after
the preliminary decree for partition was passed but as we have indicated hereinbe fore the
same cannot be a subject-matter of consideration in terms of Order VII, Rule 11(d) of the
Code.
23. One of the grounds taken in the counter-affidavit of the respondent Nos. 10, 11, 13
and 17 under Order VII, Rule 11(d) of the Code is as under :
"16. So far as item No. 8 of the Sched ule-A, the subsequent purchases have made flats
and 80% have been sold to third party and the third-party interest have been cre ated and
third parties are not made parties before the Court. Hence, the suit is bad in law for
misjoinder and non-joinder of nec essary parties. Moreover, third party's in terest has
been created and separate khatas have been issued."
24. What would be its effect is again a question which cannot fall for determination
@page-SC3180
under Order VII, Rule 11(d) of the Code. These facts require adjudication. The iden tity
of the properties which were the sub ject-matter of the earlier suit vis-a-vis the properties
which were subsequently ac quired and the effect thereof is beyond the purview of Order
VII, Rule 11(d) of the Code.
25. Whether the properties mentioned in the plaint are available for partition is es
sentially a question of fact. Whether an or der of injunction was obtained on the basis of a
misleading statement in the earlier suit or whether they were entitled therefor are not the
questions which, in our opinion, can be gone into at this stage. Moreover, it is contended
that some lands have been ac quired by the Bangalore Development Au thority. But, we
do not know in whose favour the awards were made and even if some body has received
the awarded amount, what would be the effect thereof.

We may place on record that the plain tiffs are said to be guilty of suppression of facts, as
would appear from para 2 of the application filed under Order VII, Rule 11(d) of the
Code, but then what would be the ef fect of such suppression has to be deter mined. [See
S.P. Chengalvaraya Naidu (dead) by L.Rs. v. Jagannath (dead) by L.Rs. and others, AIR
1994 SC 853] What would be the effect of non availability of the property vis-a vis the
contentions of the respondents in regard to Item No. 8 is a question which re quires
further probe. 1994 AIR SCW 243

26. Order VII, Rule 11(d) of the Code serves a broad purpose as has been noted in
Liverpool and London S.P. and I. Association Ltd. v. M.V. Sea Success I and Anr. [(2004)
9 SCC 5121 in the following terms :

"The idea underlying Order 7, Rule 11(a) is that when no cause of action is disclosed, the
courts will not unnecessarily protract the hearing of a suit. Having regard to the changes
in the legislative policy as adum brated by the amendments carried out in the Code of
Civil Procedure, the courts would interpret the provisions in such a manner so as to save
expenses, achieve expedition and avoid the court's resources being used up on cases
which will serve no useful pur pose. A litigation which in the opinion of the court is
doomed to fail would not fur ther be allowed to be used as a device to harass a litigant.
[See Azhar Hussain v. Rajiv Gandhi (1986) Supp SCC 315 at pp. 324-35]" But therein
itself, it was held : AIR 1986 SC 1253

"Whether a plaint discloses a cause of action or not is essentially a question of fact. But
whether it does or does not must be found out from reading the plaint itself. For the said
purpose the averments made in the plaint in their entirety must be held to be correct. The
test is as to whether if the aver ments made in the plaint are taken to be correct in their
entirety, a decree would be passed."

In C. Natrajan v. Ashim Bai and Anr. [2007 (12) SCALE 163], this Court held : 2007
AIR SCW 6953

"An application for rejection of the plaint can be filed if the allegations made in the plaint
even if given face value and taken to be correct in their entirety appear to be barred by
any law. The question as to whether a suit is barred by limitation or not would, therefore,
depend upon the facts and circumstances of each case. For the said purpose, only the
averments made in the plaint are relevant. At this stage, the court would not be entitled to
consider the case of the defence. (See Popat and Kotecha Prop erty v. State Bank of India
Staff Association [(2005) 7 SCC 510]]"
27. Dr. Bharuka as also Mr. Venkataramani have relied upon a large number of decisions.
We do not say that they are wholly irrelevant but what we in tend to say is they are not
relevant for our purpose at this stage. Relevance of the said decisions must be noticed by
the court at an appropriate stage. If we make any com ment thereupon, the same may
affect the rights of the parties at a later stage. We, therefore, refrain from doing so.
28. We may, however, notice only a few decisions of this Court.
In Popat and Kotecha Property v. State Bank of India Staff Association [(2005) 7 SCC
510], the question which arose for consid eration was as to whether the suit was barred by
limitation.
It was held :
"22. There is distinction between "mate rial facts" and "particulars". The words "ma terial
facts" show that the facts necessary to formulate a complete cause of action must be
stated. Omission of a single material fact leads to an incomplete cause of action and the
statement or plaint becomes bad. The
@page-SC3181
distinction which has been made between "material facts" and "particulars" was brought
by Scott, L.J. in Bruce v. Odhams Press Ltd.
23. Rule 11 of Order 7 lays down an in dependent remedy made available to the
defendant to challenge the maintainability of the suit itself, irrespective of his right to
contest the same on merits. The law osten sibly does not contemplate at any stage when
the objections can be raised, and also does not say in express terms about the filing of a
written statement. Instead, the word "shall" is used clearly implying thereby that it casts a
duty on the court to perform its obligations in rejecting the plaint when the same is hit by
any of the infirmities pro vided in the four clauses of Rule 11, even without intervention
of the defendant. In any event, rejection of the plaint under Rule 11 does not preclude the
plaintiffs from pre senting a fresh plaint in terms of Rule 13." This Court opined that
therein questions of fact were to be determined.
The matter, however, was referred to a three-Judge Bench of this Court in Balasaria
Construction (P) Ltd. v. Hanuman Seva Trust and others [(2006) 5 SCC 662]. However,
as no conflict of decisions of this Court was found, it was referred back to the two-Judge
Bench again. A two-Judge Bench of this Court in Balasaria Construction (P) Ltd. v.
Hanuman Seva Trust and Others [(2006) 5 SCC 658] held :
"8. After hearing counsel for the parties, going through the plaint, application under
Order 7, Rule 11(d), CPC and the judgments of the trial court and the High Court, we are
of the opinion that the present suit could not be dismissed as barred by limitation without
proper pleadings, framing of an is sue of limitation and taking of evidence. Question of
limitation is a mixed question of law and fact. Ex facie in the present case on the reading
of the plaint it cannot be held that the suit is barred by time. The findings recorded by the
High Court touching upon the merits of the dispute are set aside but the conclusion
arrived at by the High Court is affirmed. We agree with the view taken by the trial court
that a plaint cannot be re jected under Order 7, Rule 11(d) of the Code of Civil
Procedure."
29. Reliance has been placed on Tara Pada Ray v. Shyama Pada Ray and others [AIR
1952 Calcutta 579] wherein the averments made in the deed of sale had been taken into
consideration. Therein, however, the Calcutta High Court noticed that the fi nal decree
proceedings need not be resorted to where the directions contained in a pre liminary
decree had been acted upon by the parties. Even such a question is required to be gone
into.
30

. Reliance has also been placed on T. Arivandandam v. T.V. Satyapal and another [(1977)
4 SCC 467], wherein it has been held : AIR 1977 SC 2421

"5. We have not the slightest hesitation in condemning the petitioner for the gross abuse
of the process of the court repeatedly and unrepentently resorted to. From the statement
of the facts found in the judgment of the High Court, it is perfectly plain that the suit now
pending before the First Munsif's Court, Bangalore, is a flagrant mis use of the mercies of
the law in receiving plaints. The learned Munsif must remem ber that if on a meaningful -
not formal -reading of the plaint it is manifestly vexa tious, and meritless, in the sense of
not dis closing a clear right to sue, he should exer cise his power under Order 7, Rule 11,
CPC taking care to see that the ground mentioned therein is fulfilled. And, if clever
drafting has created the illusion of a cause of action, nip it in the bud at the first hearing
by examin ing the party searchingly under Order 10, CPC. An activist Judge is the answer
to ir responsible law suits. The trial courts would insist imperatively on examining the
party at the first hearing so that bogus litigation can be shot down at the earliest stage.
The Penal Code is also resourceful enough to meet such men, (Cr. XI) and must be trig
gered against them. In this case, the learned Judge to his cost realised what George Ber
nard Shaw remarked on the assassination of Mahatma Gandhi :
"It is dangerous to be too good."
Each case, however, must be considered on its own facts.
31

. Mr. Venkataramani has also placed reliance upon a decision of this Court in M/s.
Kalloomal Tapeswari, Prasad (HUF), Kanpur v. Commissioner of Income Tax, Kanpur
[(1982) 1 SCC 447] to contend that even partial partition is permissible. No exception
thereto can be taken but the effect thereof vis-a-vis another suit, it is trite, cannot be
determined under Order VII, Rule 11 of the AIR 1982 SC 760

@page-SC3182
Code.
32. We may, however, notice that in Kashinathsa Yamosa Kabadi, etc. v. Narsingsa
Bhaskarsa Kabadi, etc. [AIR 1961 SC 1077], this Court stated the law, thus :
"26. To sum up: on a consideration of the materials placed before the court, the refer ence
to Panchas is proved to be made vol untarily by all the parties, that the Panchas had in the
first instance decided that each branch was to get a fourth share in the prop erties and that
decision was accepted by the parties, that division of properties made from time to time
was also accepted by the par ties, and subsequently, when the Panchas were unable to
proceed with the division, the matter was referred by consent of the parties to Godkhindi
and Godkhindi divided with the consent of the parties the outstandings, but he was unable
to divide the remaining properties. For reasons we have already stated, the division made
by the Panchas and by Godkhindi is binding upon the parties. Such properties as are not
partitioned must, of course, be ordered to be divided and the division will be made
consistently with the rules of Hindu Law. To the division of such properties which have
not been divided, the decision of the Panchas dated 23-9-1946, will not apply."
33. For the reasons aforementioned, the impugned order cannot be sustained. The appeal
is allowed. We, however, must make it clear that the parties would be at liberty to raise all
contentions before the learned Trial Judge at appropriate stage(s). The parties shall, in the
facts and circumstances of the case, bear their own costs of this ap peal.
Appeal allowed.
AIR 2008 SUPREME COURT 3182 "Chairman, Punjab National Bank v. Astamija
Dash"
(From : 2006 (2) Cur LR 706 (Orissa))
Coram : 2 S. B. SINHA AND V. S. SIRPURKAR, JJ.
Civil Appeal No. 3125 with 3126 of 2008 (arising out of SLP (C) No. 18997 with 23155
of 2005), D/- 30 -4 -2008.
Punjab National Bank by Chairman and Anr. v. Astamija Dash
WITH
Astamija Dash v. Punjab National Bank and Anr.
(A) Banking Companies (Acquisition and Transfer of Undertakings) Act (5 of 1970),
S.19 - Punjab National Bank (Of ficers) Service Regulations (1979), Regn.16 -
BANKING - SERVICE MATTERS - Probationer - Confirmation - Opinion that Officer is
not fit for confiration - To be formed by Authority only within period of probation
including pe riod of extension and not beyond the same.
1995 All LJ 864, Overruled. (Para 31)
(B) Banking Companies (Acquisition and Transfer of Undertakings) Act (5 of 1970), S.19
- Punjab National Bank (Of ficers) Service Regulations (1979), Regn.16 - BANKING -
SERVICE MATTERS - EQUALITY - Probationer - Confirmation test - Number of
chances to be given for pass ing, not mentioned in appointment let ter - Executive
Committee of Bank decided to give maximum 3 chances to Man agement Trainee - Bank
given opportu nity to one woman employee to appear at test for 4th time on ground of her
mother's illness - Denial of such oppor tunity to petitioner whose case stand on better
footing as she could not appear at time of earlier tests because of her physi cal condition -
Would be arbitrary.
Constitution of India, Art.14.
Executive Committee of the Bank decided that maximum number of 3 chances should be
given to the Management Trainee for qualifying in the confirmation test. Regula tion does
not speak of any confirmation test. The offer of appointment does not speak about
number of chances to be given for passing the confirmation test. In the instant case the
petitioner, Management Trainee on probation underwent a mis-carriage of her conception
and she had to remain alone at the place of posting and could not prepare for the
examination when called for first time. At the time when she was called upon to appear in
the examination for the second time, she was in the advance stage of preg nancy and she
was medically advised not to move, as she had miscarriages at two previ ous occasions.
That is the reason why she could not appear in the test. When she was called upon to
appear for the third exami nation, she having undergone caesarian delivery, she was
advised rest by the doctor. At the time of her examination, her son was only six months'
old and was not keeping good health. She appeared in the said test, but could not succeed.
In the meanwhile the probation period expired. It was extended
@page-SC3183
by another year. Indisputably, she had oth erwise completed her period of probation. She
also fulfilled the other conditions of ser vice. Her services, however, were terminated.
However, she preferred an appeal there against before the appellate authority ask ing for
another chance to clear the confir mation test stating that similarly situated employee
including one 'I' had been given another chance to appear at the examina tion, not on the
ground of her own illness but on her mother's illness.
Held, the action of the Bank was arbi trary. The Executive Committee or for that matter
the Appellate Authority cannot ex ercise the power of relaxation in a discrimi natory
manner. It was expected to act judi ciously, assuming that the employer had a discretion
in this behalf. Discretion cannot be equated with whims and caprices. The petitioner thus,
directed to be reinstated in service forthwith. She, however, may be paid only 50% of the
back-wages. (Paras 32, 47, 52, 53, 55)
Cases Referred : Chronological Paras
2008 AIR SCW 725 :AIR 2008 SC 1322 (Ref.) 46
2008 AIR SCW 1190 : 2008 Lab IC 2022 (Ref.) 48
2007 AIR SCW 2111 (Ref.) 51
2007 AIR SCW 3981 : AIR 2007 SC 2291 : 2007 Lab IC 2812 : 2007 (4) ALJ 718 (Ref.)
50
2007 AIR SCW 4604 : AIR 2007 SC 2588 (Ref.) 51
2006 AIR SCW 1392 : AIR 2006 SC 1489 (Ref.) 45
2006 (11) Scale 108 (Ref.) 46
1997 AIR SCW 3918 : AIR 1997 SC 3981 : 1998 Lab IC 129 (Ref.) 27
1995 All LJ 864 (Overruled) 11
1994 AIR SCW 3840 (Ref.) 41
1994 AIR SCW 4367 (Ref.) 40
1993 AIR SCW 1509 : AIR 1993 SC 1601 (Ref.) 41
1993 AIR SCW 2098 : AIR 1993 SC 1947 (Ref.) 41
1991 AIR SCW 1241 : AIR 1991 SC 1402 : 1991 Lab IC 1266 (Ref.) 18, 21, 22, 28
AIR 1991 SC 101 : 1991 Lab IC 91 (Ref.) 42
AIR 1986 SC 1844 : 1986 Lab IC 1198 (Ref.) 21, 22, 26, 29, 30
AIR 1983 SC 473 : 1983 Cri LJ 811 (Ref.) 39
AIR 1974 SC 1 : 1974 Lab IC 1 (Ref.) 40
AIR 1974 SC 555 : 1974 Lab IC 427 (Ref.) 38
AIR 1974 SC 2192 : 1974 Lab IC 1380 (Ref.) 25
AIR 1968 SC 1210 : 1968 Lab IC 1409 (Ref.) 21, 22, 23, 24, 25, 29, 30
S. B. Upadhyay, Sr. Advocate, Dhruv Mehta, Harshvardhan Jha, Yashraj Singh Deora,
Gulshan Sharma (for M/s. K.L. Mehta and Co.), Santosh Mishra and Mrs. Sharmila
Upadhyay, for the Appearing Par ties.
Judgement
1.S. B. SINHA, J. :- Leave granted in both the matters.
2. These two appeals arise out of a judg ment and order dated 20th May, 2005 ren dered
by the Division Bench of the Orissa High Court at Cuttack in W.P. No. 2333 of 1991.
3. Writ Petitioner (Respondent in appeal arising out of SLP (C) No. 18997 of 2005 and
Appellant in the connected appeal) was appointed as a Management Trainee in the Punjab
National Bank (Bank). She was duly selected by the Banking Service Recruitment Board,
Delhi. An offer of appointment was made to her favour on or about 28th July, 1986 inter
alia on the following terms and conditions :-
"2. TRAINING/PROBATION/CONFIRMA TION
"You will be on training/probation for a period of 2 years from the date of your join ing
the Bank and you will be considered for confirmation in the service, subject to your
satisfactory report on your training, pass ing Bank's confirmation test and receipt of
satisfactory report from the Police authori ties about your character and antecedents. You
may also be required to pass a test in a language other than your mother tongue before
confirmation.
3. During the period of probation your services can be terminated by giving one month's
notice or payment of one-month's emoluments in lieu thereof. The Bank may, however, in
its discretion extend your pro bation by a further period not exceeding one year. If you
desire to leave the services of the Bank at any time during the period of probation,
including the extended period thereof, if any, you shall give a month's no tice or in lieu
thereof, you shall pay a month's emoluments to the bank."
4. She joined the services' of the Bank
@page-SC3184
immediately thereafter.
Pursuant to the condition that she must pass the confirmation test, she appeared in the
said examination on 29th May, 1988. She did not pass the said examination. She was,
however, again asked to appear in the examination on 1st October, 1989. By a let ter
dated 5th September, 1989 she ex pressed her inability to do so, stating :-
"I invite a kind reference to your letter dated 18th August, 1989 wherein I was ad vised to
appear in the confirmation test scheduled to be held on 1 October, 1989 in Central Staff
College, Delhi. In this con text, I would like to inform you that I am passing through the
period of pregnancy and am advised by the doctor not to undertake long journey during
the period of pregnancy till delivery as a result of Which I will not be able to appear the
test.
I, therefore, request you to kindly con sider my case sympathetically and permit me to
appear the test on the next schedule date, in future. I enclose a medical certifi cate for
your information and favourable consideration."
5. She had two miscarriages. She was asked again to appear in the examination on 19th
August, 1990. She appeared in the said test, but could not succeed. In the meanwhile the
probation period expired on 28th July, 1988. It was extended by an other year i.e. till 28th
July, 1989.
6. Indisputably, she had otherwise com pleted her period of probation. She also ful filled
the other conditions of service. Her services, however, were terminated by an order dated
9th November, 1990 stating :-
"You were appointed as a Management Trainee on 25.8.1986. One of the terms of your
appointment stipulates that :
"3. CONFIRMATION
You will be considered for confirmation in the Bank service after two years on :
a) Satisfactory report of your training.
b) Passing Bank's confirmation Test.
c) You may also be required to pass a test in language other than your mother tongue
before confirmation.
In the event of your not satisfactorily com pleting the training referred to in sub para 3(a)
or failure to qualify the Bank's confir mation test within the training/probation period of
two years or to pass the test in a language other than your mother tongue, your probation
may be extended by a fur ther period not exceeding one year. If dur ing the period of
probation, including the period of extension, if any, the competent authority is of the
opinion that you are not fit for confirmation to be retained in the bank service, your
services shall be liable to be terminated by one month's notice or pay ment of one month's
emoluments in lieu thereof.
You had appeared in the confirmation test held on 29.5.1988 but you did not qualify the
same. In the confirmation test held on 1.10.1989, you have been advised to appear for the
test but you had remained absent. Although the Bank would have been justi fied in
terminating your services in accor dance with your terms of appointment, a lenient view
was taken and you were given a last and final opportunity for qualifying the confirmation
test in August, 1990. You appeared in the Bank's confirmation test held on 19.8.1990 but
did not qualify the test."
7. She preferred an appeal thereagainst before the appellate authority asking for another
chance to clear the confirmation test stating that similarly situated employees including
one Indubala had been given an other chance to appear at the examination. However, the
said appeal was dismissed by an order dated 28th November, 1990.
8. She filed a writ petition before the High Court, inter alia, contending :-
i) In terms of the Punjab National Bank (Officers) Service Regulations, 1979 insis tence
of passing the confirmation test was illegal.
ii) As the extended period of probation expired on 28th July, 1989 she should be deemed
to have been confirmed in service.
iii) She had been subjected to discrimi natory treatment vis-a-vis Indubala who was
granted an opportunity to appear in the con firmation test for the 4th time on the ground
of illness of her mother.
9. The High Court rejected the first two contentions raised by the writ petitioner herein
but accepted the third. The writ pe tition was allowed on the said premise.
10. Both the parties, being aggrieved, are before us.
11. Mr. Dhruv Mehta, learned counsel ap pearing on behalf of the Bank would sub mit :-
@page-SC3185
i) Although Regulations do not provide for a confirmation test, the writ petitioner hav ing
accepted the conditions of appointment, she cannot be permitted to approbate or
reprobate.

ii) Her services having not been expressly confirmed, the doctrine of implied confir
mation is not applicable in view of the deci sion of the Division Bench of the Allahabad
High Court in General Manager, Punjab National Bank and oth ers vs. Khar Bhan Ram
(1995) II LLJ 93 All. 1995 All LJ 864

iii) Reliance placed by the High Court on the case of Indubala was misplaced as an
affidavit explaining the situation had not been taken into consideration and in any event
no legal right can be claimed on the basis of an illegality committed by the em ployer as
Article 14 of the Constitution of India speaks of a positive right.
12. Mr. S.S. Upadhyay, learned Senior Counsel, appearing on behalf of the writ pe
titioner, on the other hand, would submit :-
i) The order of termination being not based on unsatisfactorily completion of the in-house
training or the non-passing of a language test, which only have been pro vided for in the
Regulations, the impugned order of termination could not have been passed on the
premise that the writ peti tioner did not pass the confirmation test.
ii) Having regard to Regulations 15 and 16 of the Regulations, her services having been
terminated on 9th November, 1990 i.e., after the extended period of the period of
probation upto 28th July, 1989 expired, she would be deemed to have been confirmed in
service.
iii) In any event she had clearly been dis criminated against inasmuch as an em ployee
who was similarly situated had been given a fourth opportunity to clear the said
confirmation test, whereas the writ peti tioner was not.
13. The Bank is a scheduled Bank within the meaning of the provisions of the Bank ing
Companies (Acquisition and Transfer of Undertakings) Act, 1970. It has various statutory
powers. It has inter alia power to frame Regulations. In exercise of its powers conferred
upon it by Section 19 read with sub-section (2) of Section 12 of the Banking Companies
(Acquisition and Transfer of Undertakings) Act, 1970, the Board of Di rectors of the
Punjab National Bank, in consultation with the Reserve Bank of India and with the
previous sanction of the Cen tral Government made Regulations known as the Punjab
National Bank (Officers') Ser vice Regulations, 1979 (for short Regula tions).
Applicability of the said Regulations to the case of the writ petitioner is not in dispute.
14. Regulations 15, 16 and 36, which are relevant for our purposes may be noticed.
In terms of sub-regulation (1) of Regula tion 15, the period of probation is two years.
Sub-regulation (1) of Regulation 16 provides for confirmation of service, if in the opinion
of the competent authority, the officer has satisfactorily completed the training in any
institution to which he might have been de puted for training and in the in-service train
ing in the bank. The proviso appended thereto provides for passing a test in a lan guage
other than the mother tongue of the office. Sub-regulation (2) of Regulation 16 provides
for extension of the period of pro bation only in the event the officer does not
satisfactorily complete either or both the trainings referred to in sub-regulation (1) or fails
to pass the test referred to therein. Ex tension of the period of probation, however, could
not exceed a further period of one year. Sub-regulation (3) of Regulation 16 provides that
service of an employee can be termi nated in the case of a direct appointee, by one
month's notice or payment of one month's emoluments in lieu thereof only when during
the period of probation, includ ing the period of extension, if any, the com petent
authority is of the opinion that the officer is not fit for promotion.
Regulation 36 provides for maternity regulation in terms whereof leave upto a period of
six months can be granted.
15-16. Indisputably, the Regulations do not provide for passing of any confirmation test.
Such a confirmation test had been pre scribed only in the letter of appointment.
Ordinarily, although when conflict occurs between an executive order and a statutory
Regulation, the latter will prevail; we will proceed on the premise that such a condi tion
could be imposed by the competent authority.
17. We, for the time being, would also assume that having regard to the doctrine of
approbation and reprobation as also the doctrine of election, the writ petitioner could not
question the validity or otherwise of the said executive action.
@page-SC3186
18

. While saying so, however, we are not unmindful of the observations made by this Court
in Municipal Corporation, Raipur vs. Ashok Kumar Misra (1991) 3 SCC 325. 1991
AIR SCW 1241

"6. Exercise of the power to extend the probation is hedged with the existence of the rule
in that regard followed by positive act of either confirmation of the probation or
discharge from service or reversion to the substantive post within a reasonable time after
the expiry of the period of probation. If the rules do not empower the appointing authority
to extend the probation beyond the prescribed period, or where the rules are absent about
confirmation or passing of the prescribed test for confirmation of proba tion then inaction
for a very long time may lead to an indication of the satisfactory completion of
probation."
19. The period of probation is governed by a statutory provision. The appointing
authority is bound thereby. The initial pe riod of probation is two years, subject only to
non compliance of the conditions laid down under sub-regulation (1) of Regulation 16,
namely failure to complete satisfacto rily either or both the trainings referred to therein or
passing of a language test, the period of probation can be extended. The statute mandates
that it can be extended for a period not exceeding one year. The to tal period of probation,
therefore, can be three years and not more. No doubt for con firming the services of an
officer of the bank, the competent authority must satisfactorily form an opinion that the
officer had com pleted the trainings in any institution to which the officer had been
deputed as also the in-service training in the bank. It is not the case of the appellant-bank
that the pro viso appended thereto is applicable in the case of the writ petitioner.
20. Extension of the period of probation limited to one year, however, is circum scribed
by the conditions specified in sub-regulation (2). What is apparent, has been made
explicit by sub-regulation (3) as the competent authority has to form an opinion that the
officer is not fit for confirmation only within the period of probation including the period
of extension and not beyond the same.
21. The High Court, as noticed hereinbe fore, has relied upon the decision of the Di
vision Bench of the Allahabad High Court in the case of appellant bank itself.

Chief Justice S.S. Sodhi, speaking for the Division Bench, distinguished the cases of
State of Punjab vs. Dharam Singh 1968 (3) SCR 1 and Om Pakash Maurya v. U.P. Co-
operative Sugar Federation, Lucknow 1986 Suppl SCC 95 to hold :- AIR 1968 SC
1210
AIR 1986 SC 1844

"It may be mentioned here that both Dharam Singh and Om Prakash's cases (supra) were
later distinguished by the Su preme Court in Municipal Corporation, Raipur v. Ashok
Kumar Misra (1991-II-LLJ-343), where the facts were somewhat simi lar as herein that
the relevant service rules, besides requiring successful completion of probation for
confirmation in service, also laid down, as an essential pre-condition, the passing of the
departmental test. It was held that mere expiry of the period of probation did not
automatically have the effect of deemed confirmation. Both the earlier judg ments of the
Supreme Court in Dharam Singh and Om Prakash cases (supra) were noticed and
distinguished. Seen in this light, there can be no escape from the conclusion that in the
circumstances here, mere expiry of the period of probation, without the re spondent
having qualified in the confirma tion test, did not entitle him to confirma tion in service.
1991 AIR SCW 1241

22

. The decisions of this Court in Dharam Singh (supra) and Om Prakash Maurya (supra),
on the one hand, and Municipal Corpo ration, Raipur vs. Ashok Kumar Misra (1991) 3
SCC 325, on the other, as would appear from the discussions made hereinafter had set the
legal principles differently. AIR 1968 SC 1210
AIR 1986 SC 1844
1991 AIR SCW 1241

23

. In Dharam Singh, a Con stitution Bench of this Court cat egorically held that :- AIR
1968 SC 1210, (Para 3)
"Where on the completion of the speci fied period of probation the employee is al lowed
to continue in the post without an order of confirmation, the only possible view to take in
the absence of anything to the contrary in the original order of appointment or promotion
or the service rules, is that the initial period of probation has been extended by necessary
implication."
What is, therefore, evident is that the mat ter must be governed by the statutory rules.
@page-SC3187
24

. The Rule in question which was applicable in Dharam Singh (supra) was sub-rule (3) of
Rule 6 of the Punjab Educational Service (Provincialised Cadre) Class III Rules, 1961
which was in the following terms :- AIR 1968 SC 1210 (Para 1)

"6(3) On the completion of the period of probation the authority competent to make
appointment may confirm the member in his appointment or if his work or conduct dur
ing the period of probation has been in his opinion unsatisfactory he may dispense with
his services or may extend his period of pro bation by such period as he may deem fit or
revert him to his former post if he was pro moted from some lower post.
Provided that the total period of proba tion including extensions, if any, shall not exceed
three years."

In view of the said Rule it was held :- Para 3 of AIR

"This Court has consistently held that when a first appointment or promotion is made on
probation for a specific period and the employee is allowed to continue in the post after
the expiry of the period without any specific order of confirmation, he should be deemed
to continue in his post as a pro bationer only, in the absence of any indica tion to the
contrary in the original order of appointment or protection or the service rules. In such a
case, an express order of confirmation is necessary to give the em ployee a substantive
right to the post, and from the mere fact that he is allowed to con tinue in the post after
the expiry of the speci fied period of probation it is not possible to hold that he should be
deemed to have been confirmed."

It was further held :- Para 5 of AIR

"In the present case, R. 6(3) forbids extension of the period of probation beyond three
years. Where, as in the present case, the service rules fix a certain period of time beyond
which the probationary period cannot be extended, and an employee ap pointed or
promoted to a post on probation is allowed to continue in that post after completion of the
maximum period of pro bation without an express order of confir mation, he cannot be
deemed to continue in that post as a probationer by implication. The reason is that such
an implication is negatived by the service rule forbidding ex tension of the probationary
period beyond the maximum period fixed by it. In such a case, it is permissible to draw
the inference that the employee allowed to continue in the post on completion of the
maximum period of probation has been confirmed in the post by implication."
25

. The said principle was reiterated by a seven Judge Bench of this Court in Samsher Singh
vs. State of Punjab (1974) 2 SCC 831 stating : AIR 1974 SC 2192

"71. Any confirmation by implication is negatived in the present case because be fore the
completion of three years the High Court found prima facie that the work as well as the
conduct of the appellant was unsatisfactory and a notice was given to the appellant on
October 4, 1968 to show cause as to why his services should not be termi nated.
Furthermore, Rule 9 shows that the employment of a probationer can be pro posed to be
terminated whether during or at the end of the period of probation. This indicates that
where the notice is given at the end of the probation the period of pro bation gets
extended till the inquiry proceed ings commenced by the notice under Rule 9 come to an
end. In this background the ex planation to Rule 7(1) shows that the period of probation
shall be deemed to have been extended impliedly if a Subordinate Judge is not confirmed
on the expiry of this period of probation. This implied extension where a Subordinate
Judge is not confirmed on the expiry of the period of proba tion is not found in Dharam
Singh case. This explanation in the present case does not mean that the implied extension
of the probationary period is only between two and three years. The explanation on the
contrary means that the provision regarding the maximum period of probation for three
years is directory and not mandatory unlike in Dharam Singh case and that a probationer
is not in fact con firmed till an order of confirmation is made." AIR 1968 SC 1210

(Emphasis supplied)
26

. In Om Parkash Maurya vs. U.P. Coop. Sugar Factories Federation (1986) Suppl. SCC 95
following Dharam Singh, this Court held :- AIR 1986 SC 1844

"4. In the instant case the order of ap pointment promoting the appellant to the post of
Commercial Officer merely indicated that his probationary period could be ex tended and
he could be reverted to the post
@page-SC3188
of Office Superintendent without any notice. Stipulation for extension of probationary
period in the appointment order must be considered in accordance with the proviso to
Regulation 17(1) which means that the probationary period could be extended for a
period of one year more. Indisputably on the expiry of the appellant's initial probation ary
period of one year, the appointing au thority extended the same for another pe riod of one
year which also expired on Sep tember 4, 1982. During the period of pro bation
appellant's services were neither ter minated nor was he reverted to his substan tive post
instead he was allowed to continue on the post of Commercial Officer. On the expiry of
the maximum probationary period of two years, the appellant could not be deemed to
continue on probation, instead he stood confirmed in the post by implication. The
appellant acquired the status of a confirmed employee on the post of Commer cial Officer
and the appointing authority could not legally revert him to the lower post of
Superintendent."
(Emphasis supplied)
27

. The said principle, we may notice, was again reiterated in Chief G.M., State Bank of
India vs. Bijoy Kumar Mishra : (1997) 7 SCC 550 wherein this Court had the occasion to
con sider a pari materia rule, stating :- 1997 AIR SCW 3918

"10. There can thus be no doubt that the deemed confirmation which is inferred from the
employer's conduct is permissible only when it follows from the positive act of the
employer permitting the employee to con tinue to work on the post even after comple tion
of the maximum period of probation permitted under the Service Rules since no other
inference is possible in such a situa tion from the employer's conduct of continu ing to
take work from the employee after that period."
28

. We may, having noticed the legal prin ciples enunciated by this Court, consider the
decision of this Court in Ashok Kumar Misra (supra). The rel evant Rule which was
involved therein was Rule 8 of Madhya Pradesh Gov ernment Servants' General
Conditions of Service Rules, 1961 which was in the fol lowing terms :- 1991 AIR
SCW 1241

"8. Probation. (.1) A person appointed to a service or post by direct recruitment shall
ordinarily be placed on probation for such period as may be prescribed.
(2) The appointing authority may, for suf ficient reasons, extend the period of proba tion
by a further period not exceeding one year.
Note.- A probationer whose period of probation is not extended, under this sub-rule, but
who has neither been confirmed nor discharged from service at the end of the period of
probation shall be deemed to have been continued in service, subject to the condition of
his service being terminable on the expiry of a notice of one calendar month given in
writing by either side.
(3) A probationer shall undergo such training and pass such departmental exami nations
during the period of his probation as may be prescribed.
(4) and (5) are not relevant, hence omit ted.
(6) On the successful completion of pro bation and the passing Of the prescribed
departmental examinations, the probationer shall be confirmed in the services or post to
which he has been appointed."
29. The note appended to sub-rule (2) of Rule 8 as also sub-rule (6) thereof made all the
difference. In terms of note appended to sub-rule (2) a legal fiction was created in terms
whereof upon completion of the ex tended period of probation the employee would have
been continued in service, sub ject to the condition that the same would be terminable on
the expiry of a notice of one calendar month and furthermore an express order confirming
the service would be nec essary.
In the aforementioned situa tion Dharam Singh (supra) and Om Prakash Maurya (supra)
were distinguished opining :- AIR 1968 SC 1210
AIR 1986 SC 1844

"6. Exercise of the power to extend the probation is hedged with the existence of the rule
in that regard followed by positive act of either confirmation of the probation or
discharge from service or reversion to the substantive post within a reasonable time after
the expiry of the period of probation. If the rules do not empower the appointing authority
to extend the probation beyond the prescribed period : or where the rules are absent about
confirmation or passing of the prescribed test for confirmation of proba tion then inaction
for a very long time may lead to an indication of the satisfactory
@page-SC3189
completion of probation. But in this case Rule 8 expressly postulates otherwise. The
period of probation is subject to extension by order in writing for another period of one
year. Passing the prescribed examinations and successful completion of probation and to
make an order of confirmation are condi tion precedent. Mere expiry of the initial period
of probation does not automatically have the effect of deemed confirmation and the status
of a deemed confirmation of the probation. An express order in that regard only confers
the status of an approved pro bationer. We are of the view that note to sub-rule (2) read
with sub-rule (6) of Rule 8 mani fests the legislative intent that confirmation of the
probation of the respondent would be made only on successful completion of the
probation and the passing of the prescribed examinations. It is not the respondent's case
that he passed all the examinations. He shall be deemed to be continued on probation.
Before confirmation the appointing author ity is empowered to terminate the service of
the probationer by issuing one calendar month's notice in writing and on expiry thereof
the service stands terminated with out any further notice. Within three months from the
date of expiry of original two years period of probation and within one year's period, the
order of termination was made. In this view the question of conducting an inquiry under
the Classification, Control and Appeal (Rules) after giving an opportunity and that too for
specific charges does not arise."
30. The order of termination in that case was, therefore passed within the extended period
of probation in service. Ashok Kumar Misra (supra), therefore, in our opinion did not
speak in a different tone and is in con formity with the legal principles laid down in
Dharam Singh (supra) and others.
31. The High Court, therefore, in our opinion was not correct in relying upon the decision
of the Division Bench of the Allahabad High Court in the case of the ap pellant bank. It,
with utmost respect, did not lay down the correct law and is, there fore, liable to be
overruled.
32. So far as the question of discrimina tion meted out to the writ petitioner is con cerned,
we may notice that the High Court had come down heavily on the bank and, in our
opinion, rightly so.
33. In the affidavit affirmed by one Rabi Shankar Sabat (Manager Personnel) of the Bank
the Indubala case was sought to be distinguished stating :-
"5. That as regards the case of petitioner vis-a-vis case of one Ms. Indubala is con cerned
it is submitted that both the cases stand absolutely on different footing. Ms. Indubala is
not a batchmate of petitioner. Ms. Indubala joined the bank as Manage ment Trainee on
12.11.1984 and she was called for confirmation test of Management Trainee held by the
bank on 13.7.1986. She did not appear in the said test. She was given another chance for
confirmation test held on 26.4.1987 who appeared in the said test but failed. Ms. Indubala
was again called for confirmation test held on 29.5.1988, as per the decision of the
Executive Commit tee of the bank, however, she could not take the confirmation test this
time due to her illness and accordingly her services were terminated by the Bank on
29.5.1988. On appeal by Ms. Indubala the matter was put up before the Executive
Committee of the bank and the Executive Committee in its meeting held on 16.8.89
decided that in view of the fact that Ms. Indubala could not avail of the last i.e. 3rd
chance due to her illness, she may be afforded another opportunity to appear in the
confirmation test and this is how Ms. Indubala was given one more chance. Copy of
decision of the Committee dated 16.8.89 is filed herewith as Annex ure-F. On the other
hand the petitioner joined the Bank as Management Trainee on 25.8.1986 and she failed
to appear in the confirmation test held in April, 1988 and October, 1989. She was also
given the 3rd chance the last chance where she did ap pear and failed to qualify: It may
be noted here that Ms. Indubala was given the 3rd chance but she could not appear in the
3rd and last chance due to her illness. So both the cases stand on a different footing as
such petitioner cannot claim any benefit on the basis of Ms. Indubala's case. Besides, that
was a solitary case and as the Principle that one wrong cannot justify another wrong the
petitioner cannot claim any advantage rely ing on the same."
34. At once we may notice that the said statement contained a factual error which has
been very fairly conceded by Mr. Mehta as the ground for giving another opportu nity to
Indubala to appear at the confirma tion test for the 4th time was not on the ground of her
own illness but on her
@page-SC3190
mother's illness.
35. Was the fact situation in Indubala's case is different from that of the writ peti tioner?
The answer must be found in the judgment of the High Court, wherein it was held that the
writ petitioner's case stands on a much better footing. As she could not appear in the third
chance on account of the illness of her mother which did not de pict the correct state of
affairs whereas the writ petitioner underwent a miscarriage of her conception in the
month of April, 1988, she had to remain alone at the place of post ing and could not
prepare for the examina tion. At the time when she was called upon to appear in the
examination for the second time, she was in the advance stage of preg nancy and she was
medically advised not to move, as she had miscarriages at two previ ous occasions. That
is the reason why she could not appear in the test. When she was called upon to appear
for the third exami nation, she having undergone caesarian delivery, she was advised rest
by the doctor. At the time of her examination, her son was only six month's old and was
not keeping good health.
36. Submission of the Bank is that the competent authority had formulated a policy of not
permitting anybody to appear the in the test more than thrice. Strong reliance in this
behalf is placed on the decision of the Executive Committee of the Bank dated 19th June,
1990 deciding that maximum number of 3 chances should be given to the Management
Trainee for qualifying in the confirmation test failing which services of the probationer be
terminated.
37. Regulation does not speak of any con firmation test. The offer of appointment does
not speak about the number of chances to be given for passing the confirmation test. A
decision was taken in this behalf when the writ petitioner had already appeared on one
occasion. The decision even otherwise was to give at least three opportunities to a
candidate must be real and effective one. Such a contention must be considered hav ing
regard to the doctrine of reasonableness and fairness, which the Bank is required to
comply with keeping in view its status of a State within the meaning of Article 12 of the
Constitution of India. As a 'State' the Bank was bound to follow the equity clause con
tained in Articles 14 and 16 of the Consti tution of India. Its action even in relation to its
own employees is expected to be not only fair but also non-arbitrary.
38

. In E.P. Royappa v. State of Tamil Nadu arid another (1974) 4 SCC 3, a Constitutional
Bench of this court "as regards the argument that the petitioner was ap pointed to a post
that was inferior to the status and office of the Chief Secretary, thus offending Articles 14
and 16 of the Consti tution, opined :- AIR 1974 SC 555, Paras 85 and 86

"Article 16 embodies the fundamental guarantee that there shall be equality of op


portunity for all citizens in matters relating to employment or appointment to any office
under the State. Though enacted as a dis tinct and independent fundamental right because
of its great importance as a prin ciple ensuring equality of opportunity in public
employment which is so vital to the building up of the new classless egalitarian society
envisaged in the Constitution, Ar ticle 16 is only an instance of the applica tion of the
concept of equality enshrined in Article 14. In other words, Article 14 is the genus while
Article 16 is a species, Article 16 gives effect to the doctrine or equality in all matters
relating to public employment. The basic principle which therefore, informs both Articles
14 and 16 is equality and in hibition against discrimination. Now, what is the content and
reach of this great equalising principle? It is a founding faith, to use the words of Bose, J.,
"a way of life", and it must not be subjected to a narrow pedantic or lexicographic
approach. We can not countenance any attempt to truncate its all-embracing scope and
meaning, for to do so would be to violate its activist magnitude. Equality is a dynamic
concept with many aspects and dimensions and it cannot be "cribbed, cabined and
confined" within tra ditional and doctrinaire limits. From a posi tivistic point of view,
equality is antithetic to arbitrariness. In fact equality and arbi trariness are sworn enemies;
one belongs to the rule of law in a republic while the other, to the whim and caprice of an
abso lute monarch. Where an act is arbitrary it is implicit in it that it is unequal both
accord ing to political logic and Constitutional law and is therefore violative of Article
14, and if it affects any matter relating to public em ployment, it is also violative of
Article 16. Articles 14 and 16 strike at arbitrariness in State action and ensure fairness
and equal ity of treatment. They require that State action
@page-SC3191
must be based on equivalent relevant principles applicable alike to all similarly situate
and it must not be guided by any extraneous or irrelevant considerations be cause that
would be denial of equality. Where the operative reason for State action as distinguished
from motive inducing from the antechamber of the mind, is not legiti mate and relevant
but is extraneous and outside the area of permissible consider ations, it would amount to
mala fide exer cise of power and that is hit by Articles 14 and 16. Mala fide exercise of
power and ar bitrariness are different lethal radiations emanating from the same vice : in
fact the latter comprehends the former. Both are inhibited by Articles 14 and 16."
It was further held:
"It is also necessary to point out that the ambit and reach of Articles 14 and 16 are not
limited to cases where the public ser vant affected has a right to a post. Even if a public
servant is in an officiating position, he can complain of violation of Articles 14 and 16 if
he has been arbitrarily or unfairly treated or subjected to mala fide exercise of power by
the State machine."
39

. In the Constitution Bench decision of this court in Mithu vs. State of Punjab 1983) 2
SCC 277, the issue to be decided was the vires of Section 303 of the Indian penal Code
vis -a-vis Article 21 of the Constitution. It was held that the same was unconstitutional as
it violated the guarantee of equality clause contained in Articles 14 and 21 since a per son
who is sentenced to life imprisonment incurs the mandatory penalty of death un der
Section 303 if he commits a murder while he is under the sentence of life im prisonment
defied logic. AIR 1983 SC 473

40

. InT.R. Kothandaraman and Ors. vs. Tamil Nadu Water Supply and Drainage Bd. and
Ors. (1994) 6 SCC 282, the issue be fore this court was as regards what Article 16 has to
say when the right to be consid ered for promotion is either barred or re stricted on the
basis of educational qualifi cations. Thus, the validity of Rule 2(b), which prescribed the
ratio of 3:2 for direct recruits and promotees, the former being degree-holders and latter
diploma-holders was chal lenged as being violatiye of the guarantee of equality embodied
in Article 16. Citing with approval Justice Krishna Iyer in State of Jammu and Kashmir v.
Trilokinath Khosa (1974) 1 SCC 19, it was thus stated : 1994 AIR SCW 4367, Para 3
AIR 1974 SC 1

"Krishna Iyer, J., stated that the social meaning of Articles 14 and 16 is neither dull
uniformity nor specious 'telentism'. Further, the soul of Article 16 is the promotion of the
common man's capabilities, opening up full opportunities to develop without succumb
ing to the sophistic argument of the elite that talent is the privilege of the few and they
must rule. But then, personnel policy does require an eye on efficiency; and so, though
'chill penury" should not 'repress their noble rage', technical proficiency cannot be sacri
ficed at the altar of wooden equality. All these call for a striking of balance between the
long hunger for equal chance of the lowlier and the disturbing concern of the community
for higher standards of performance. Even so, mini-classifications based on micro-distinc
tions are false to our egalitarian faith; and over-doing of classification would be undo ing
of equality. The Court has to function always as a sentinel on the qui vive."
Thus, dismissing the writ petition, it was held that a harmony would thus be struck, by
maintaining reasonableness in the ratio, between the call of social justice and the need for
higher education, without in any way jeopardising the principal object of clas sification,
by the impugned rule.
41

. In T. Sham Bhat vs. Union of India (UOI) and Anr. 1994 Supp (3) SCC 340, the vires of
Regulation 2 of the Indian Admin istrative Service (Appointment by Selection) Second
Amendment Regulations, 1989 - the IAS Second Amendment Regulations was
challenged before this court. 1994 AIR SCW 3840

It inter alia referred to the following judg ments :

"16. Venkatashwara Theatre vs. State of Andhra Pradesh and Ors. (1993) 3 SCC 677, is a
de cision of this Court which points out, as to how discrimination can arise, if persons
who are unequals are treated as equals, thus : 1993 AIR SCW 2098

"Just as a difference in the treatment of persons similarly situate leads to discrimi nation,
so also discrimination can arise if persons who are unequals, i.e. differently placed, are
treated similarly.... A law pro viding for equal treatment of unequal ob jects, transactions
or persons would be con demned as discriminatory if there is absence
@page-SC3192
of rational relation to the object intended to be achieved by the law."

Food Corporation of India vs. Kamdhenu Cattle Feed Indus tries (1993) 1 SCC 71, is a
deci sion of this Court where it is pointed out that requirement of non-arbitrariness in a
State action, if ought, to conform to Article 14 of the Constitution, due weight must be
given to reasonable or legitimate expecta tions of the persons likely to be affected by such
action, thus : 1993 AIR SCW 1509

"To satisfy this requirement of non arbi trariness in a State action, it is, therefore,
necessary to consider and give due weight to the reasonable or legitimate expectations of
the persons likely to be affected by the decision or else that unfairness in the exer cise of
the power may amount to an abuse or excess of power apart from affecting the bona fides
of the decision in a given case. The decision so made would be exposed to challenge on
the ground of arbitrariness."
Thus, holding the classification between the increase in number of years of continu ous
service of non-State Civil Service Class-I officers to make them eligible for selection to
the Indian Administrative service which deprived them of the right to be considered for
selection under the IAS Selection Regu lations which held the field for over 33 years, as
unjust, arbitrary, unreasonable and that which arbitrarily affected the legitimate and
normal expectations of non-State Civil Ser vice Class-I officers and was inhibited by
Article 14 of the Constitution, the regula tion was struck down as unconstitutional.
42

. Delhi Transport Corpora tion vs. D.T.C. Mazdoor Con gress and others 1991 Supp (1)
SCC 600, dealt with the question of constitutional validity of the right of the employer to
terminate the services of per manent employees without holding any in quiry in certain
circumstances by reason able notice or pay in lieu of notice. After re ferring to a plethora
of decisions of this court on the application of Articles 14 and 16 in cases of public
employment, it was opined : AIR 1991 SC 101, Para 264

"Thus it could be hold that Article 14 read with 16(1) accords right to an equality or an
equal treatment consistent with the prin ciples of natural justice. Any law made or action
taken by the employer, corporate statutory or instrumentality under Article 12 must act
fairly, justly and reasonably. Right to fair treatment is an essential in built of natural
justice. Exercise of unbridled and uncanalised discretionary power im pinges upon the
right of the citizen; vesting of discretion is no wrong provided it is exer cised purposively,
judiciously and without prejudice. Wider the discretion, the greater the chances of abuse.
Absolute discretion is destructive of freedom than of man's other inventions. Absolute
discretion marks the beginning of the end of the liberty. The con ferment of absolute
power to dismiss a per manent employee is antithesis to justness or fair treatment. The
exercise of discretion ary power wide of mark would breed arbi trary, unreasonable or
unfair actions and would not be consistent with reason and justice. The provisions of a
statute, regula tions or rules that empower an employer or the management to dismiss,
remove or re duce in rank of an employee, must be con sistent with just, reasonable and
fair proce dure. It would, further, be held that right to public employment which includes
right to continued public employment till the em ployee is superannuated as per rules or
com pulsorily retired or duly terminated in ac cordance with the procedure established by
law is an integral part of right to livelihood which in turn is an integral facet of right to
life assured by Article 21 of the Constitu tion. Any procedure prescribed to deprive such a
right to livelihood or continued em ployment must be just, fair and reasonable procedure.
In other words an employee in a public employment also must not be arbi trarily unjustly
and unreasonably be de prived of his/her livelihood which is ensured in continued
employment till it is terminated in accordance with just, fair and reasonable procedure.
Otherwise any law or rule in vio lation thereof is void."
43. We have noticed hereinbefore the plight of the writ petitioner as to why she could not
prepare well or appear at the sec ond test. In such a situation an employee in certain
establishments would be governed by the Maternity Benefit Act, 1961. All shops and
establishments were brought within the purview of the said Act by Act No. 61 of 1988
w.e.f. 10th January, 1989. In terms of the provisions of the said Act, a woman is
prohibited from working in an establishment during the period of six weeks from
immediately following the day of her delivery, miscarriage or medical termination of
pregnancy. She, if a request is made by
@page-SC3193
her in this behalf, even would not be asked to work for the period specified in sub-sec
tion (4) of Section 4. Apart from the right to payment of maternity benefits, she would be
entitled to the benefits of Sections 6 and 9 thereof. Section 9 reads as under :-
"9. Leave for miscarriage, etc. In case of miscarriage or medical termination of preg
nancy, a woman shall, on production of such proof as may be prescribed, be entitled to
leave with wages at the rate of maternity benefit, for a period of six weeks immedi ately
following the day of her miscarriage, or, as the case may be, her medical termi nation of
pregnancy."
44. Mr. Mehta has, however, drawn our attention to Regulation 36 of the Regulations to
contend that maternity leave was admis sible up to a period of 6 months only, on and
from the 1st day of April, 2000. A statu tory Regulation, as is well known, is subject to
the provisions of a Parliamenttary Act. Regulations framed by the Board of Direc tors of
the Bank fail to provide for grant of maternity leave and other benefits to which a woman
employee would be entitled to in terms of the Maternity Benefit Act, 1961. A subordinate
legislation, as is well known, must be made in conformity with the Par liamentary Act.
45

. In Bombay Dyeing and Mfg. Co. Ltd. vs. Bombay Envi ronmental Action Group and
Ors. (2006) 3 SCC 434, this court opined : 2006 AIR SCW 1392, Para 103

"By reason of any legislation whether en acted by the legislature or by way of subor
dinate legislation, the State gives effect to its legislative policy. Such legislation, how
ever, must not be ultra vires the Constitu tion. A subordinate legislation apart from being
intra vires the Constitution, should not also be ultra vires the parent Act under which it
has been made. A subordinate leg islation, it is trite, must be reasonable and in
consonance with the legislative policy as also give effect to the purport and object of the
Act and in good faith."
46. Similarly, in Vasu Dev Singh and Ors. vs. Union of India and Ors. 2006 (11) SCALE
108, wherein the Validity of Section 3 of the East Punjab Urban Rent Restriction Act,
1949 was challenged, this court after refer ring to a large number of decisions on sub
ordinate legislation, held :
"A statute can be amended, partially repealed or wholly repealed by the legislature only.
The philosophy underlying a statute or the legislative policy, with the passage of time,
may be altered but therefore only the legislature has the requisite power and not the
executive. The delegated legislation must be exercised, it is trite, within the param eters
of essential legislative policy. The ques tion must be considered from another angle.
Delegation of essential legislative function is impermissible. It is essential for the leg
islature to declare its legislative policy which can be gathered from the express words
used in the statute or by necessary implication, having regard to the attending circum
stances. It is impermissible for the legisla ture to abdicate its essential legislative func
tions. The legislature cannot delegate its power to repeal the law or modify its essen tial
features."

[See also Employees' State In surance Corporation vs. H.M.T. Ltd. and another 2008 (1)
SCALE 341.] 2008 AIR SCW 725

47. We, however, are not oblivious of the fact that the contention as regards the ap
plicability of the Maternity Benefit Act, 1961 had not been raised before the High Court.
We will assume for the sake of arguments that the said Act is not applicable. How ever,
we intend to emphasize that the atti tude on the part of the State in exercise of its power
of discretion should otherwise be commensurate with the doctrine of reason ableness. A
State, even for applying the con stitutional scheme of equality would not enforce its
decision only upon taking into consideration the cases of the different par ties before it. A
woman who had undergone miscarriages, in our opinion, was entitled to a different
treatment. Article 14 indis putably is a positive concept. Applicability of the doctrine of
equality as a positive con cept, therefore, should have been the premise that as a woman
having regard to the state of affairs in which the writ peti tioner was placed, she was
entitled to ob tain a different treatment from the employer. Article 14 does not apply in a
vacuum. Whereas persons absolutely similarly situ ated, should be treated equally, equal
treat ment to the persons dissimilarly situated would also attract the wrath of Article 14. It
is from that point of view that the writ petitioner's case ought to have been consid ered
vis-a-vis indubala. If the appellate authority was entitled to exercise its power
@page-SC3194
of relaxation, which in the absence of any statutory interdict (presumably it was en titled
to), it should have considered the case of the petitioner vis-a-vis Indubala, whose case,
was rightly been found by the High Court stood at a much weaker foundation. Thus,
Article 14 must be held to have been violated; the power of relaxation having been
conceded to the appellate authority.
48

. This aspect of the matter has been considered by this Court in Nehru Yuva Kendra
Sangathan vs. Mehbub Alam Laskar 2008 (1) SCALE 590. 2008 AIR SCW 1190

"A 'State' within the meaning of Article 12 of the Constitution of India should have
placed full facts before the High Court. Only in its anxiety to show that the case of Ajay
Kumar Gupta was different from that of the respondent, it came out with the truth that the
respondent was guilty of a serious mis conduct."
49. We are not unmindful that as a posi tive concept, Article 14 would not apply in
illegality.
50

. In Nagar Mahapalika, Kanpur vs. Smt. Vibha Shukla and Ors. 2007 (8) SCALE 361, the
court on the issue of regularization of services opined: "Equality is a positive con cept.
Therefore, it cannot be invoked where any illegality has been committed or where no
legal right is established." 2007 AIR SCW 3981

51

. Similar opinion was expressed in State of Orissa and Ors. vs. Prasana Kumar Sahoo
2007 (6) SCALE 236 at paragraphs 23 and 24 and in Vice Chancellor, M.D. University,
Rohtak vs. Jahan Singh 2007 (4) SCALE 226 at paragraph 28. 2007 AIR SCW 4604
2007 AIR SCW 2111

52. The Executive Committee of the Bank had fixed the number of chances to be given to
an employee in the confirmation test. If it is enforced against the writ petitioner hav ing
regard to her physical position, to ap pear in the second examination, the provi sions
thereof, keeping in mind the principle underlying the statutory provisions of Ma ternity
Benefit Act, may not be held to be applicable. She was, thus, entitled to an other
opportunity to appear at the exami nation. The Executive Committee or for that matter the
appellate authority cannot exer cise the power of relaxation in a discrimina tory manner.
It was expected to act judiciously, assuming that the employer had a discretion in this
behalf. Discretion cannot be equated with whims and caprices.
53. We, for the reasons abovementioned, are not in a position to accept the submis sion of
Mr. Mehta that it was for the em ployer to decide as to how many chances have to be
given to each employee and the Bank cannot be deprived of such discretion ary
jurisdiction.
54. For the views we have taken we need not deal with the question as to whether the
insistence of confirmation test is not in ac cordance with the Regulations.
55. For the reasons abovementioned ap peal filed by the Bank is dismissed and that of the
writ petitioner is allowed. The writ petitioner shall be reinstated in service forth with.
She, however, may be paid only 50 % of the back wages. This order we are pass ing
keeping in view that her services had been terminated on 9th November, 1990. The writ
petitioner is also entitled to costs. Counsel's fee assessed at Rs.50,000/-.
Order accordingly.
AIR 2008 SUPREME COURT 3194 "A. K. Jain v. Prem Kapoor"
(From : Punjab and Haryana)
Coram : 2 TARUN CHATTERJEE AND AFTAB ALAM, JJ.
Civil Appeal No. 4680 of 2008 (arising out of S.L.P. (C) No. 8530 of 2007), D/- 28 -7
-2008.
A. K. Jain v. Prem Kapoor.
(A) Haryana Urban (Control of Rent and Eviction) Act (11 of 1973), S.13(3)(a)(i)(b) and
S.13(3)(a)(i)(b)(c) - HOUSES AND RENTS - EVICTION - PLEADINGS - Eviction
petition - Omission in pleadings alleged - Pleadings fully complied with requirements of
S.13(3)(a)(i)(b) and (c) - No material to sub stantiate plea that landlord failed to make
necessary declaration under said provi sion - Order rejecting eviction petition is liable to
be set aside. (Para 11)
(B) Haryana Urban (Control of Rent and Eviction) Act (11 of 1973), S.13(3)(a) -
HOUSES AND RENTS - EVICTION - PLEA - TENANCY - Eviction - Personal
necessity - Plea that event of landlord's retirement from ser vice was never brought on
record in ac cordance with law - Fact of retirement was admitted by tenant - No prejudice
was caused to him on ground that land-lord did not make any formal amendment in
pleading - Plea raised is calculated
@page-SC3195
only to frustrate proceeding and, there fore, liable to be rejected. (Paras 13, 14)
(C) Haryana Urban (Control of Rent and Eviction) Act (11 of 1973), S.13(3)(a)(i) and
S.13(3)(a)(ii) - HOUSES AND RENTS - EVICTION - Eviction - Personal necessity -
Landlord's son, his wife and children are part of family - All of them were liv ing together
- Accommodation of son, his wife and their children is part of landlord's personal
necessity - Case of landlord falls u/S.13(3)(a)(i) and not u/S.13(3)(a)(ii).
Civil Revn. No. 2344 of 2005, D/-03-04-2007 (P and H), Reversed.
2002 AIR SCW 2374, Rel. on.
2002 AIR SCW 278; 2007 AIR SCW 3250, Disting. (Para 16)
Cases Referred : Chronological Paras
2007 AIR SCW 3250 : 2007 (4) ALJ 297 (Disting.) 12
2002 AIR SCW 278 : AIR 2002 SC 665 (Disting.) 12
2002 AIR SCW 2374 : AIR 2002 SC 2256 (Rel. on) 16
Manoj Swarup, Rohit Sohgaura, Tejvir S. Bhatia, for Appellant; D. S. Chadha and Mrs.
Amita Gupta, for Respondent.
Judgement
AFTAB ALAM, J. :- Heard counsel for the parties.
2. Leave granted.
3. This appeal arises for an eviction pro ceeding instituted by the appellant, the land lord,
in terms of S. 13 of the Haryana Ur ban (Control of Rent" and Eviction) Act, 1973. The
eviction of the respondent, the tenant, was sought on a number of grounds of which only
personal necessity now remains rel evant, and we, accordingly, propose to take note of
facts germane to that ground. The Rent Controller, Faridabad rejected the Evic tion
Petition (24 of 1998) filed by the appel lant by order dated 26th August, 2004. The
appellate authority, however, allowed the appellant's appeal'(Rent Appeal No. 6 of 2004)
and ordered the respondent's eviction by judgment dated 28th March, 2005. The High
Court in turn, set aside the appellate order and restored the judgment and order passed by
the Rent Controller vide, the judg ment and order dated 3rd April, 2007 in Civil Revision
No. 2344 of 2005 (O and M), preferred by the respondent. The High Court allowed the
respondent's revision and dismissed the eviction petition filed by the appellant pri marily
on the ground that the appellant had failed to make the necessary averments in the
eviction petition as required by S. 13(3)(a)(i) of the Act. The finding of the High Court
that led to the dismissal of the appellant's case is admittedly based on an error of record
and the order coming under appeal is therefore liable to be set aside on that score alone.
But before proceeding fur ther we must recount the relevant facts and the respective
views taken by the Courts below.
4. The appellant, in August, 1995, in ducted the respondent as a tenant in a por tion of his
dwelling house at No. 1007 in Sector 14, Faridabad. The let out portion consisted of one
garage-room and one bed room with attached bath and toilet. The por tion of the house
that remained with the landlord consisted of two bed rooms, baths and toilets and a
kitchen.
5. On 13 April, 1998 the appellant filed the petition for eviction of the respondent from
the tenanted portion of the house in ter alia on grounds of personal necessity. In the
eviction petition, it was pleaded that the appellant needed the entire house, includ ing the
part tenanted by the respondent for himself and his family. It was stated that the
appellant's family consisted of himself, his wife and a son and a daughter, both of whom
were of marriageable age and were likely to be married soon. The respondent resisted the
eviction proceeding and contro verted the plea of personal necessity by pointing out that
the appellant was an Ex ecutive Engineer in the Haryana Electricity Board and he was
posted in Hisar where he lived in an official residence. His daughter stayed in Sholapur in
connection with her studies. It was contended that the plea of personal necessity was false
and unfounded.
6. The eviction proceeding remained pending before the Rent Controller for over six
years in course of which a number of developments took place that, according to the
appellant, further aggravated his need for a larger space and he required, more than
before, the tenanted portion of his house. The appellant's son was married in Janu ary,
1999 and his wife (the appellant's-daughter-in-law) also came to live with them in that
house. In 2004, he had one daugh ter who was about two years old at that time. (It was
stated before us that the appellant's son has now two female children aged 8
@page-SC3196
years and 6 years respectively). During the pendency of the proceeding before the Rent
Controller, the daughter of the appellant was also married and though she lieved with her
husband, both of them frequently came to visit and stayed over-night with her parents.
The appellant, therefore, needed a separate room not only for her son and daughter-in-law
but also for her daughter and son-in-law. Another significant development was that the
appellant's retirement from service had come very close and he had no other place to live
than his own house, a portion of which was the subject-matter of the pro ceeding.
7. The Rent Controller did not accept the appellant's case and rejected the eviction
petition by order dated 26th August, 2004.
8. Against the order passed by the Rent Controller the appellant filed appeal before the
appellate authority on 27th September, 2004. It is significant to note here that five days
after the order was passed by the Rent Controller the appellant retired from service on
31st August, 2004 and before the appel late authority he produced his retirement order
dated 25th August, 2004 which was marked as 'X.'
9. The appellate authority allowed the appellant's appeal, accepted the case of per sonal
necessity and ordered the respondent's eviction. It took into consider ation all the
developments that took place since the institution of the proceeding and which were
amply borne out from the evi dences led before the Rent Controller. As regards the
appellant's retirement from ser vice the appellate authority made the fol lowing
observations :
"Admittedly. A. K. Jain, petitioner has since retired from service with effect from 31-8-
2004 vide order dated 25-8-2004 mark X and now he has shifted in his house at
Faridabad with his wife in which the respon dent is a tenant in some portion of the house.
The petitioner is not having any otherwise in Faridabad where he along with his wife start
living after his retirement from service. The petitioner has the right to live with dig nity
after his retirement in his own house which was constructed by him while he was in
service."
(Emphasis added)
The Appellate Authority further observed as follows :-
"The petitioner has constructed his house during his service career. He has retired from
his service. He wants to reside in his house constructed by him during his service ca reers
but the respondent is bent upon not to allow the petitioner to reside in the same house
which was constructed by him on expenditure of huge amount while in ser vice."
(Emphasis added)
The Appellate Authority accordingly found and held :-
".........that the petitioner was en titled to get possession of the rented por tion of the house
on account of personal necessity and the respondent was liable to be evicted on account
of the personal ne cessity of the landlord."
10. The respondent-tenant filed revision before the High Court against the order of
eviction passed by the Appellate Authority. As noted above, the High Court allowed the
revision, set aside the order passed by the Appellate Authority and restored the order of
Rent Controller.
11. The High Court order coming under appeal before us is based on the premise that in
the eviction petition filed by him the appellant-landlord had failed to make the necessary
declarations as required under S. 13(3)(a)(i)(b) and (c) of the Act. In order to show the
omissions in the pleadings the High Court extracted certain paragraphs from what it
supposed to be the eviction petition filed by the appellant. Unfortunately, the High Court
committed ah error of record and the extracts reproduced in the High Court judgment are
not from Rent Petition No. 24/1998, from which the revision arose but those are from a
different petition filed later by the appellant. Learned counsel appear ing for the
respondent-tenant fairly accepted that the statements quoted in the High Court judgment
are not from the eviction petition filed by the appellant and in that regard the High Court
has committed an error of record. For our satisfaction we also referred to Rent Petition
No. 24 of 1998 which is a part of Annexure P-1 to the S.L.P. On a perusal of paragraphs
5(1) to (iv) we are satisfied that the pleadings fully comply with the require ments of
S.13(3)(a)(i)(b) and (c) of the Act. The High Court order is thus liable to be set aside on
this score alone.
12

. Though conceding that the High Court suffered from a fatal error of record, learned
counsel appearing for the 2002 AIR SCW 278
2007 AIR SCW 3250

@page-SC3197
respondent-tenant sought to resist the appellant's appeal on two other grounds. The
counsel first submitted that in the year 1998 when the eviction proceeding was instituted
the appellant was very much in service. He re tired from service a few days after the evic
tion petition was dismissed by the Rent Con troller. However, the event of the appellant's
retirement from service, arising subsequent to the dismissal of the proceeding by the first
Court was never brought on record in ac cordance with law either by making any
amendment in the pleadings or by a peti tion for bringing on record any additional
evidence. All that was done was to simply file the retirement order before the Appel late
Authority where it was marked as 'X.' The Appellate Authority had, therefore, com mitted
an error in taking it into consider ation in support of the appellant's plea of personal
necessity. In support of the sub mission he relied upon the decisions of this Court in (i)
Om Prakash Gupta v. Ranbir B. Goyal (2002) 2 SCC 256 and (ii) Ram Kumar Barnwal v.
Ram Lakhan (2007) 5 SCC 660.
13. We are unable to accept the submis sion. It needs to be clarified that the respon dent
tenant does not deny the fact that the appellant retired from service on 31 August, 2004.
As matter of fact, when asked point edly, learned counsel for the Respondent was not in a
position to deny that the appel lant had in fact retired from service on 31 Au gust 2004.
He, however, contended that the fact of the appellant's retirement had not come before the
Court in accordance with law. It is noted above that the Appellate Authority observed that
the retirement of the appellant was an admitted fact. It may fur ther be noted that before
the High Court a petition under Order 41 Rule 27 was filed on behalf of the appellant-
landlord to pro duce the retirement order as an additional piece of evidence. The High
Court, however, rejected the prayer observing as follows :
"It is not in dispute that the respondent-landlord had retired during the pendency of the
petition before the Rent Controller and the learned Appellate Authority has already taken
note of the subsequent event i.e. re tirement of the landlord and, therefore, the present
application under Order 41 Rule 27 of the CPC cannot be said to be competent.
Accordingly, the same is dismissed being un necessary."
(Emphasis added)
14. Even before us it was admitted that the appellant had retired from service on the date
stated by him. The appellant's retire ment from service on the date as stated by him being
admitted by the respondent the Appellate Authority was fully justified in tak ing that
development into consideration. No prejudice was caused to the respondent be cause the
appellant did not make any for mal amendments in the pleadings or be cause the
retirement order filed before the Authority was not accompanied with a for mal petition
under Order 41 Rule 27. The contention raised on behalf of the respon dent is not highly
technical and it is calcu lated only to frustrate the proceeding com ing to a just conclusion
by making it quite incidental and completely subservient to the Court's procedures. We
also fail to see how the two decisions relied upon by the coun sel can support the case of
respondent. We thus find no merit in the submission.
15. Learned counsel next submitted that the order of eviction was also based on the
growing needs of the appellant's son but in that, connection there was no pleading as
required under Section 13(3) (a) (ii) of the Act. The provision referred by the counsel
reads as follows :
"(3) A landlord may apply to the Control ler for an order directing the tenant to put the
landlord in possession-
(a) in the case of a residential building, if -
(i) xxx xxx xxx
(ii) he requires it for use as an office or consulting room by his son who intends to start
practice as a lawyer, qualified archi tect or chartered accountant or as a "regis tered
practitioner" within the meaning of that expression used in the Punjab Medical
Registration Act, 1916, the Punjab Ayurvedic and Unani Practitioners Act, 1963, or the
Punjab Homoeopathic Practitioners Act, 1965, or for the residence of his son who is
married :
Provided that such son is not occupying in the urban area concerned any other build ing
for use as office, consulting room or resi dence, as the case may be, and has not va cated
it without sufficient cause after the commencement of the 1949 Act."
16

. We are unable to accept this submission 2002 AIR SCW 2374, Para 32

@page-SC3198
because we see no application of the quoted provision to the facts of this case. Section
13(3)(a)(ii) will have application only in case the eviction is sought for the son's
requirement independently and separately from the landlord. In this case, the son and his
wife and children are part of the landlord's family and all of them are living together. The
accommodation of the son, his wife and their children is part of the land lords personal
necessity. The case of the appellant clearly falls under Section 13(3)(a)(i) that deals with
the situation where the landlord requires the tenanted premises for his own occupation
and it does not at tract Section 13(3)(a)(ii) that deals with the requirements of the son of
the landlord. In taking the view we are supported by the de cision of this Court in
Joginder Pal v. Nawal Kishore Behal, (2002)5 SCC 397. In Paragraph 33 of the decision
it was held and observed as follows :
"Our conclusions are crystallized as un der :
(i) The words "for his own use" as occur ring in Section 13(3)(a)(ii) of the East Punjab
Urban Rent Restriction Act. 1949 must re ceive a wide, liberal and useful meaning rather
than a strict or narrow construction.
(ii) The expression - landlord requires for "his own use", is not confined in its mean ing
to actual physical user by the landlord personally. The requirement not only of the
landlord himself but also of the normal "emanations" of the landlord is included therein.
All the cases and circumstances in which actual physical occupation or user by someone
else, would amount to occupation or user by the landlord himself, cannot be exhaustively
enumerated. It will depend on a variety of factors such as interrelationship and
interdependence - economic or other wise, between the landlord and such per son in the
background of social, socio-reli gious and local customs and obligations of the society or
region to which they belong.
(iii) The tests to be applied are : (i) whether the requirement pleaded and proved may
properly be regarded as the landlord's own requirement; and, (ii) whether on the facts and
in the circumstances of a given case, actual occupation and user by a per son other than
the landlord would be deemed by the landlord as "his own" occupation or user. The
answer would, in its turn, depend on (i) the nature and degree of rela tionship and/or
dependence between the landlord pleading the requirement as "his own" and the person
who would actually use the premises; (ii) the circumstances in which the claim arises and
is put forward; and (iii) the intrinsic tenability of the claim. The Court on being satisfied
of the reasonability and genuineness of claim, as distinguished from a mere ruse to get rid
of the tenant, will uphold the landlord's claim.
(iv) While casting its judicial verdict, the Court shall adopt a practical and meaning ful
approach guided by the realities of life.
(v) In the present case, the requirement of the landlord of the suit premises for user as
office of his chartered accountant son is the requirement of landlord "for his own use"
within the meaning of Section 13(3)(a)(ii)."
17. For the reasons discussed above we find no merit in this respondent's pleas.
Accordingly, the appeal succeeds and it is allowed. The order of the High Court is set
aside and the order of the Appellate Author ity is restored.
18. In the facts and circumstances of the case, however, the respondent tenant is allowed
six months time to vacate the ten anted premises on condition of filing the usual affidavit
before this Court within four weeks from today.
Appeal allowed.
AIR 2008 SUPREME COURT 3198 "Nasiruddin Khan v. State of Bihar"
(From : 2000 (2) BLJR 970) (Patna)
Coram : 2 Dr. A. PASAYAT AND Dr. MUKUNDAKAM SHARMA, JJ.
Criminal Appeal No. 3 of 2001, D/- 29 -7 -2008.
Nasiruddin Khan and Ors. v. State of Bihar.
Penal Code (45 of 1860), S.304, Part II, S.223, S.96 - CULPABLE HOMICIDE - HURT -
PRIVATE DEFENCE - SENTENCE IMPOSITION - Dispute over agricultural 2 land
leading to incident - On refusal by informant to stop fixing poles in field - Accused
bringing spears and lathis and assaulting brothers of informant - Appellant caused head
injury with spear to one brother - Other accused assaulting other brothers with lathis -
Accused al leged to have suffered injuries in incident raising plea of self defence -
Injuries on accused found to be superficial - Plea of self defence held, not tenable -
Convic tion of appellant u/S.304 Part II and other accused u/S.323 - Justified
@page-SC3199
- Sentence of 5 years imposed on appel lant - Maintained - Other accused, how ever,
given benefit of probation in view of long passage of time. (Para 5)

P. S. Mishra, Sr. Advocate, Harshvardhan Thatagat, Ms. Shefali Jain, Mrs. Sunita R.
Singh and Rajesh Prasad Singh, with him for Appellants; Gopal Singh, Manish Kumar
and Ms. Vimla Sinha, for Respondent.
Judgement
Dr. ARIJIT PASAYAT, J. :- Challenge in this appeal is to the judgment of a learned Single
Judge of the Patna High Court up holding the conviction of the appellant No. 1 for
offence punishable under Section 304 Part II of the Indian Penal Code, 1860 (in short the
'IPC') and sentence of 5 years and the other two appellants who were convicted for
offence punishable under Section 323 IPC and were sentenced to undergo rigor ous
imprisonment for one year.
2. Prosecution version as unfolded dur ing trial is as follows :
As per the Fardbeyan of Murtaza Khan (Ext.3), on 6.10.1981 at about 11.30 the
informant was fixing pegs on his own sahan land to keep maize crops. Suddenly accused
persons Nasiruddin Khan, Lajim Khan and Mehmood Khan came there and protested to
the act of fixing of the pole. The infor mant replied that he was fixing the pegs on his own
sahan and, therefore, there was no question of any protest. He also asked the accused to
get the land measured to ascer tain whether the peg was being fixed on the land of the
latter or on his own land. But Nasiruddin Khan, Lajim Khan and Mehmood Khan
announced that the informant will not heed to reason and, therefore, he should be
assaulted. Thereafter, accused Majiruddhin Khan brought a bhala and Qayamuddin Khan
and Sadruddin Khan brought lathis. Majiruddin handed over the bhala to Nasiruddin khan
and went back to bring another bhala. Lajim Khan also brought a lathi from his home.
Thereafter, informant's brothers Salam Khan, Farman Khan and Kalam Khan sons of
Munshi Khan appeared there and asked the accused persons not to indulge in assault. In
the meantime, informant's brother Kalam Khan was sub jected to assault with bhala by
Nasiruddin Khan hitting him on the right temporal re gion. Subsequently, Nasiruddin
Khan stood there with bhala in his hand and Sadruddin Khan, Gayamuddin Khan and
Lajim Khan assaulted Kalam Khan, Salam Khan and Farman Khan as also Munshi Khan.
When the villagers gathered, the accused persons made good their escape. The injured
Kalam Khan was brought to the hospital, where the informant gave his fardbeyan.
The trial Court on the basis of the evi dence of nine witnesses found the accused guilty
and sentenced them as attire-noted. In the appeal, the views of the Trial Court were
affirmed.
3. Learned counsel for the appellants submitted that the appellants exercised the right of
private defence and, therefore, no offence was committed. Additionally it is submitted
that occurrence took place on 6.10.1981 and, therefore, in case of appel lant No. 1 the
sentence should be reduced to the period already undergone. In case of other two
appellants the provisions of Sec tion 360 of the Code of Criminal Procedure, 1973 (in
short 'Cr.P.C.') should be applied.
4. Learned counsel for the State on the other hand supported the judgments of the trial
Court and the High Court.
5. The trial Court and the High Court found that this is not a case where it can be said that
the appellants were exercising the right of private defence. The plea taken was that the
accused persons also suffered in juries and, therefore, the defence version of false
implication and exercise of right of pri vate defence should be accepted. The High Court
noticed that the injuries on the ac cused persons were superficial in nature. After
analyzing the evidence, the trial Court and the High Court have found that there was no
question of exercising the right of private defence. The trial Court and the High Court
with reference to the evidence on record found that the so-called injury on Nasiruddin has
to be viewed with suspicion. Although Nasiruddin claimed and alleged that he had
sustained bhala injury, the in jury report (Ex.B) discloses that the injury found on him by
the Doctor indicated that the accused had suffered injury by hard blunt substance and the
injuries were simple. Therefore, the High Court's judg ment so far as the conviction and
sentence of appellant No. 1 is concerned is affirmed. However, considering the long
passage of time and the period of sentence imposed, we think it appropriate to extend to
appel lants Nos.2 and 3 the benefits of Section 360 Cr.P.C. on entering into bonds of such
@page-SC3200
amount as may be fixed by the learned trial Judge. Appellant No. 1 shall surrender to
custody forthwith to serve remainder of sen tence.
6. The appeal is disposed of accordingly.
Order accordingly.
AIR 2008 SUPREME COURT 3200 "Shambhoo Singh v. State of Rajasthan"
(From : Rajasthan)*
Coram : 2 Dr. A. PASAYAT AND Dr. MUKUNDAKAM SHARMA, JJ.
Criminal Appeal No. 1134 of 2008 (aris ing out of SLP (Cri.) No. 1050 of 2008), D/- 22
-7 -2008.
Shambhoo Singh v. State of Rajasthan.
(A) Penal Code (45 of 1860), S.300, Exception 4 - MURDER - Murder - Sudden quarrel
- Incident taking place out of land dis pute - Accused party challenging de ceased and his
family members while they were ploughing field - Quarrel and ex change of hot words
ensued - Accused thereupon inflicting knief blows on de ceased and others - Conviction
of ac cused for murder liable to be altered to conviction u/S.304, Part I.
D. B. Cri. A. No. 531 of 2003, D/-03-01-2007 (Raj). Reversed. (Para 8)
(B) Penal Code (45 of 1860), S.300, Exception 4 - MURDER - Sudden fight - Implies
mutual provocation and blows on each side - Whole blame in such case cannot be placed
on one side - Apportionment of share of blame which attaches to each fighter difficult -
Accused to get benefit of Exception 4 has to further prove that he has not taken undue
advantage. (Para 7)

Shiv Kumar Suri, for Appellant; Ansar Ahmad Chaudhary, for Respondent.
* D. B. Cri. Appeal No. 531 of 2003, D/- 3-1-2007 (Raj).
Judgement
1. Dr. ARIJIT PASAYAT, J. :- Leave granted.
2. The appellant questions legality of the judgment rendered by a Division Bench of the
High Court of Rajasthan at Jodhpur Bench. The learned Additional Sessions Judge No. 2,
Udaipur found the accused guilty of offence punishable under Section 302 of the Indian
Penal Code, 1860 (in short the 'IPC') and sentenced him to undergo RI for life and to pay
a fine with default stipulation. He was also convicted for offence punishable under
Section 447 IPC and sen tenced to undergo 15 days' RI. Additionally, he was convicted
for offence punishable under Section 307 IPC and sentenced to undergo 10 years RI and
pay a fine of Rs.100/-. Similarly, in respect of offence punishable under Section 324 IPC
he was sentenced to undergo RI for one year. In appeal, by the impugned judgment, High
Court confirmed the judgment of conviction and sentence.
3. Prosecution version as unfolded dur ing trial is as follows :
On 3.8.1999, Vaje Singh (PW-1) lodged a First Information Report at Police Station
Pahara stating, inter-alia that in the morn ing at about 9.00 a.m. his brother Jawan Singh
(hereinafter referred to as the 'de ceased') was ploughing the field. He along with his
father Gulab Singh and elder brother Ram Singh was working in the field. At that time,
his neighbour appellant Shambhoo Singh, his father Som Singh and mother Smt. Jeevi
arrived there abusing them. Appellant Shambhoo Singh was car rying knife in his hand.
Som Singh and Smt. Jeevi were carrying lathis. They challenged them and questioned as
to how they were ploughing the field of their possession. There ensued a quarrel and
exchange of hot words. Appellant Shambhoo Singh stabbed the knife on the chest of
Jawan Singh. He caused another injury by knife on the stom ach. On intervention by his
father, appel lant Shambhoo Singh caused injury by knife. He also caused injuries to his
mother Smt. Shanta and elder brother Ram Singh. Appellant Shambhoo Singh also
caused in juries to him. Jawan Singh succumbed to the injuries on the spot. It was stated
that there was a land dispute between them, which led to the unfortunate incident.
On this information, police registered a case and proceeded with the investigation. The
post-mortem of the dead body was con ducted by Dr. Mahendra (PW-17) on the spot vide
Ex.P-42. He noticed the following inju ries on his person :
1. An incised stab wound - 2.0 cm x 1.0 cm x perforating up to chest cavity placed in 6th
intercostal space below left Nipple place obliquely.
On exploration - There is a wound of 1.55 cm x 1.0 cm x 2 cm deep left ventricle of the
Heart. Cavity full of Blood.
@page-SC3201
2. An incised stab wound - 1.5 cm x 1.0 cm x thoracic cavity deep 5 cm lateral to injury
No. 1 placed obliquely.
On exploration of wound - There is a wound of 1.0 cm x 2 cm lung tissue deep placed on
the left lung. Thoracic cavity was full of blood.
3.An incised stab wound - 2.0 cm x 1.0 cm x abdominal cavity deep. On exploration of
wound -There was no injury to any Ab dominal Viscera. Intestinal loops are pro truding
through this wound.
4.Abrasion- 2.5 cm x 1.0 cm placed on upper 1/3rd on medial side of right leg.
The cause of death was shock due to severe bleeding following stab wound to chest and
abdomen. The injured persons namely PW-1 Vaje Singh, PW-10 Gulab Singh and PW-3
Smt. Shanta were sent to the hospital. Their injuries were examined by PW-11, Dr. B.P.
Verma. He examined the injuries of PW-1 Vaje Singh vide Ex. P-12 and noticed the
following injury on his person :
Incised wound 4 x 2 x 1 1/2 cm on left gluteus.
He also examined the injuries of PW-10 Gulab Singh vide Ex.P-11 and noticed the
following injuries :
Stab wound transversely with bleeding on left intra mammary region 4 x 1 x plural cavity
deep surgical empug sema left side.
He also examined the injuries of PW-3 Smt. Shanta vide Ex. P-13 and noticed the
following injuries :
Incised wound 1½ x ½ x ½ cm on Right arm M/3rd Ant.
After usual investigation, the police laid charge sheet against appellant Shambhoo Singh,
his father Som Singh and mother Smt. Jeevi for offence under Sections 302, 307, 326,
324, 447/34 IPC. The accusedpersons pleaded not guilty of the charges levelledagainst
them and claimed trial.
The trial Court placing reliance on the evidence of PWs 1, 2, 3 and 10 found the evidence
to be credible and cogent. It found the evidence of the injured witness to be without any
blemish. Accordingly, the trial Court recorded the conviction and sentence as afore-noted.
In appeal before the High Court, the primary stand taken was that the ocular evi dence
does not inspire corroboration. It was submitted that in any event offence pun ishable
under Section 302 IPC is not made out as the occurrence occurred in course of
suddenquarrel. The High Court did not find any substance and dismissed the appeal.
4. In support of the appeal, learned coun sel for the appellant submitted that the oc
currence took place during the course of sudden quarrel and, therefore, Section 302 IPC
has no application.
5. Learned counsel for the respondent-State supported the judgments of the trial Court
and the High Court.
6. For bringing in operation of Exception 4 to Section 300 IPC, it has to be established
that the act was committed without premedi tation, in a sudden fight in the heat of pas
sion upon a sudden quarrel without the of fender having taken undue advantage and not
having acted in a cruel or unusual man ner.
7. The Fourth Exception to Section 300 IPC covers acts done in a sudden fight. The said
Exception deals with a case of pros ecution not covered by the First Exception, after
which its place would have been more appropriate. The Exception is founded upon the
same principle, for in both there is ab sence of premeditation. But, while in the case of
Exception 1 there is total depriva tion of self-control, in case of Exception 4, there is only
that heat of passion which clouds men's sober reasons and urges them to deeds which
they would not otherwise do. There is provocation in Exception 4 as in Exception 1; but
the injury done is not the direct consequence of that provocation. In fact Exception 4
deals with cases in which notwithstanding that a blow may have been struck, or some
provocation given in the origin of the dispute or in whatever way the quarrel may have
originated, yet the subse quent conduct of both parties puts them in respect of guilt upon
equal footing. A "sud den fight" implies mutual provocation and blows on each side. The
homicide commit ted is then clearly not traceable to unilat eral provocation, nor in such
cases could the whole blame be placed on one side. For if it were so, the Exception more
appropri ately applicable would be Exception 1. There is no previous deliberation or
determination
@page-SC3202
to fight. A fight suddenly takes place, for which both parties are more or less to be
blamed. It may be that one of them starts it, but if the other had not aggravated it by his
own conduct it would hot have taken the serious turn it did. There is then mutual
provocation and aggravation, and it is diffi cult to apportion the share of blame which
attaches to each fighter. The help of Excep tion 4 can be invoked if death is caused (a)
without premeditation; (b) in a sudden fight; (c) without the offender having taken un due
advantage or acted in a cruel or un usual manner; and (d) the fight must have been with
the person killed. To bring a case within Exception 4 all the ingredients men tioned in it
must be found, It is to be noted that the "fight" occurring in Exception 4 to Section 300
IPC is not defined in IPC. It takes two to make a fight. Heat of passion requires that there
must be no time for the passions to cool down and in this case, the parties have worked
themselves into a fury on ac count of the verbal altercation in the begin ning. A fight is a
combat between two or more persons whether with or without weapons. It is not possible
to enunciate any general rule as to what shall be deemed to be a sud den quarrel. It is a
question of fact and whether a quarrel is sudden or not must necessarily depend upon the
proved facts of each case. For the application of Excep tion 4, it is not sufficient to show
that there was a sudden quarrel and there was no pre meditation. It must further be shown
that the offender has not taken undue advan tage or acted in cruel or unusual manner. The
expression "undue advantage" as used in the provision means "unfair advantage".
8. In the background facts as considered in the light of evidence the inevitable con
clusion is that the appropriate conviction would be under Section 304 Part I, IPC.
Custodial sentence of 10 years would meet the ends of justice. The conviction in respect
of other offences and the sentences imposed do not suffer from any infirmity to warrant
interference. The sentences shall run con currently.
9. The appeal is allowed to the aforesaid extent.
Order accordingly.
AIR 2008 SUPREME COURT 3202 "D. Purushotama Reddy. v. K. Sateesh"
(From : Karnataka)*
Coram : 2 S. B. SINHA AND CYRIAC JOSEPH, JJ.
Civil Appeal No.4751 of 2008 (arising out of SLP (Civ.) No. 8520 of 2007), D/- 1 -8
-2008.
D. Purushotama Reddy and Anr. v. K. Sateesh.
Civil P.C. (5 of 1908), O.37, R.1 - NEGOTIABLE INSTRUMENT - RECOVERY OF
DUES - JUDGMENT - DECREE - Suit for recovery of money due under dishonoured
cheque - Compensation paid by defendant u/S.357, Crimi nal P.C. in criminal
proceedings - Has to be adjusted while passing decree - Specific duty has been cast on
Civil Courts by S.357(5) of Code to take into account sum paid as compensation - Decree
directed to be amended.
Criminal P.C. (2 of 1974), S.357. (Para 13)
Cases Referred : Chronological Paras
2008 AIR SCW 208 :AIR 2008 SC 876 : 2008 (2) AIR Bom R 45 (Ref.) 15
2008 AIR SCW 5160 (Ref.) 14
2007 AIR SCW 2425 : 2007 Cri LJ 2417 : 2007 (3) AIR Bom R 678 : 2007 CLC 982
(Rel. on) 14
AIR 1991 SC 855 (Ref.) 15
S. N. Bhat, for Appellants; Jana Kalyan Das, for Respondent.
* F. A. No. 1171 of 2006, D/- 22-12-2006 (Kant).
Judgement
1. S. B. SINHA, J. :- Leave granted.
2. Whether in a suit for recovery of money on a cheque issued by the defendant but
dishonoured, the amount received by the plaintiff-creditor in a criminal proceeding
should be adjusted, is the core question in volved herein.
3. Plaintiff - Respondent filed a suit against the appellants, which was marked as O.S. No.
1844 of 2004, for recovery of a sum of Rs. 3, 09,000/- with interest. In the plaint, it was
averred that Shri K. Balasubramanyam (father of the respondent) andDefendant No. 1
(Appellant No. 1 herein) were good friends. Defendant Nos. 1 and 2 had been carrying on
business. They ap proached the plaintiff through Shri K. Balasubramanyam for financial
assistance and obtained a loan of Rs. 2,00,000/- (Rs. l,00,000/- on 15.03.2001 and Rs.
1,00,000/- on 25.03.2001). Two promissory notes were also executed therefor.
@page-SC3203
4. Defendants -Appellants purported to be in discharge of the said debt issued two
cheques bearing Nos. 3960 dated 15.03.2003 and 3959 dated 31.05.2003 drawn on Bank
of India, which on presen tation, were returned dishonoured. Indisput ably, a complaint
under Section 200 of the Code of Criminal Procedure, 1973 read with Sections 138 and
142 of the Negotiable In struments Act, 1881 (for short "the Act"), marked as C.C. No.
19337 of 2003, was filed.
A judgment, of conviction and sentence against the appellant was passed therein by an
order dated 15.12.2005 sentencing him to pay a sum of Rs. 2,10,000 by way of fine and
in default thereof to undergo simple imprisonment for a period of three months. It was
also directed that out of the said amount of fine, a sum of Rs. 2,00,000/-would be paid to
the complainant by way of compensation in terms of Section 357 of the Code of Criminal
Procedure (for short "the Code") and the remaining amount was to be payable to the
State. In the said criminal proceedings, the appellants deposited a sum of Rs. 31,500/- on
7.02.2006, Rs. 68,500/-on 21.07.2006 and Rs. 1,10,000/- on 13.12.2006.
5. O.S. No. 1844 of 2004 was decreed by the Trial Court by a judgment and order dated
23.01.2006, ordering :
"This suit is hereby decreed for a sum of Rs. 3,09,000/- (Rupees three lakhs nine
thousand only) with court costs and cur rent interest at 6% p.a. on the principal amount of
Rs. 2,00,000/-from the date of suit till realization. The defendants are jointly and
severally, liable to pay the decre tal amount."
6. In the civil proceedings also, the ap pellants admittedly have deposited a sum of Rs.
1,90,000/-. An appeal was preferred thereagainst before the High Court of Karnataka at
Bangalore marked as R.F.A. No. 1171 of 2006, which by reason of the impugned
judgment has been dismissed.
7. The principal contention raised herein is that the Trial Court and consequently the High
Court committed a serious error in decreeing the suit in its entirety, i.e., for a sum of Rs.
3,09,000/- with interest without taking into consideration the fact that an amount of Rs.
2,10,000/- had already been deposited by the appellants in the said crimi nal proceedings.
8. Contention of the respondent, however, is that as the said question was not and could
not have been raised before the Trial Court, the impugned judgment is sus tainable. It was
furthermore urged that in view of the well-settled principle of law that pendency of a
criminal matter would not be an impediment in proceeding with a civil suit, the impugned
judgment should not be interfered with.
9. A suit for recovery of money due from a borrower indisputably is maintainable at the
instance of the creditor. It is further more beyond any doubt or dispute that for the same
cause of action a complaint peti tion under terms of Section 138 of the Act would also be
maintainable.
10. The question, however, is as to whether the courts in one proceeding can issue
directions to deposit amount in favour of the plaintiff without taking into consid eration
the amount deposited by the defen dant in the other.
11. We have noticed hereinbefore that whereas the judgment of conviction and sen tence
was passed on 15.12.2005, the suit was decreed by the civil court on 23.01.2006. Deposit
of a sum of Rs. 2,00,000/- by the appellants in favour of the respondent herein, was
directed by the Criminal Court. Such an order should have been taken into consideration
by the trial Court.
An appeal from a decree, furthermore, is a continuation of suit. The limitation of power
on a civil court should also be borne in mind by the appellate court. Was any duty cast
upon the civil court to consider the amount of compensation deposited in terms of Section
357 of the Code is the question. In terms of sub-section (1) of Section 357 of the Code, a
criminal court is empowered to direct that out of the amount recovered from an accused
by way of fine, compensation of a specified amount may be directed to be paid for any
loss or injury caused by the offence, when compensation is, in the opin ion of the Court,
recoverable by a person in a Civil Court. It is, therefore, evident that the amount of
compensation could have been directed to be paid by the Criminal Court as the same was
recoverable by the respondent as against the appellants in a civil court also. Such an order
can also be passed by the Appellate Court or by the High Court or by the Court of Session
when ex ercising its power of revision.
@page-SC3204
12. Sub-section (5) of Section 357 of the Code, which is relevant for our purpose, reads
as under :
"357. Order to pay compensation -
*** *** ***
(5) At the time of awarding compensation in any subsequent civil suit relating to the same
matter, the Court shall take into ac count any sum paid or recovered as com pensation
under this section."
13. Evidently, a duty has been cast upon the civil courts to take into account the sum paid
or recovered as compensation in terms of Section 357 of the Code. It is futile to urge that
on the date on which the Civil Court passed the decree the appellants were not convicted.
As noticed hereinbefore, the ap peal is a continuation of the suit and in that view of the
matter as the appellants had in total deposited a sum of Rs. 4,00,000/-, i.e., Rs. 2,10,000/-
in the criminal proceeding and Rs. 1,90,000/- in the civil proceedings, out of which a sum
of Rs. 3,09,000/- has been withdrawn by the respondent, the High Court was obligated to
take the same into consideration. In other words, having regard to the provisions of sub-
section (5) of Sec tion 357 of the Code, a duty was cast upon the High Court to take into
account the fact that a sum of Rs. 2,00,000/- had already been paid by the appellants to
the respon dent.Concededly, both the proceedings were maintainable. Law recognizes the
same. The Parliament must have the situation of this nature in mind while enacting
Clause (b) of sub-section (1) of Section 357 of the Code and sub-section (5) thereof.
14

. In Dilip S. Dahanukar v. Kotak Mahindra Co. Ltd. and Another [(2007) 6 SCC 528],
while con sidering a question as to what should be the reasonable amount in the mat ter
of grant of compensation vis-a-vis the power of the appellate court to issue an in terim
direction in relation thereto, this Court held : 2007 AIR SCW 2425

"38. The purpose of imposition of fine and/or grant of compensation to a great extent
must be considered having the rel evant factors therefor in mind. It may be compensating
the person in one way or the other. The amount of compensation sought to be imposed,
thus, must be reasonable and not arbitrary. Before issuing a direction to pay
compensation, the capacity of the accused to pay the same must be judged. A fortiori, an
enquiry in this behalf even in a summary way, may be necessary. Some reasons, which
may not be very elaborate, may also have to be assigned; the purpose being that whereas
the power to impose fine is limited and direction to pay compensa tion can be made for
one or the other fac tors enumerated out of the same; but sub section (3) of Section 357
does not impose any such limitation and thus, power there under should be exercised only
in appropri ate cases. Such a jurisdiction cannot be ex ercised at the whims and caprice of
a judge.
39. If a fine is to be imposed under the Act, the amount of which in the opinion of
Parliament would be more than sufficient to compensate the complainant; can it be said,
that an unreasonable amount should be directed to be paid by the court while exercising
its power under sub-section (3) of Section 357? The answer thereto must be rendered in
the negative. Sub-section (5) of Section 357 also provides for some guide lines.
Ordinarily, it should be lesser than the amount which can be granted by a civil court upon
appreciation of the evidence brought before it for losses which might have reasonably
been suffered by the plaintiff. Jurisdiction of the civil court, in this behalf, for realisation
of the amount in question must also be borne in mind. A criminal case is not a
substitution for a civil suit, far less execution of a decree which may be passed.
40 Prosecution under the Act may be contemplated as a measure of deterrence, but the
same is never meant to be a perse cution.
41. Even in a case where violation of fun damental right guaranteed under Article 21 is
alleged, the amount of compensation can not be arbitrary or unreasonable even un der
public law."

[See also Manish Jalan v. State of Karnataka JT 2008 (7)SC 643] 2008 AIR SCW 5160

This Court therein adopted the doctrine of purposive construction. It was opined that
compensation directed to be paid should be a reasonable one.
15

. In New India Assurance Co. v. Nusli Neville Wadia and Anr. [2007 (14) SCALE 556], it
was held : 2008 AIR SCW 208
"50. Except in the first category of cases, as has been noticed by us hereinbefore, Sec
tions 4 and 5 of the Act, in our opinion, may AIR 1991 SC 855

@page-SC3205
have to be construed differently in view of the decisions rendered by this Court. If the
landlord being a State within the meaning of Article 12 of the Constitution of India is
required to prove fairness and reasonable ness on its part in initiating a proceeding, it is
for it to show how its prayer meets the constitutional requirements of Article 14 of the
Constitution of India. For proper inter pretation not only the basic principles of natural
justice have to be borne in mind, but also principles of constitutionalism in volved
therein. With a view to read the pro visions of the Act in a proper and effective manner,
we are of the opinion that literal interpretation, if given, may give rise to an anomaly or
absurdity which must be avoided. So as to enable a superior court to interpret a statute in
a reasonable manner, the court must place itself in the chair of a reasonable
legislator/author. So done, the rules of purposive construction have to be resorted to
which would require the con struction of the Act in such a manner so as to see that the
object of the Act fulfilled; which in turn would lead the beneficiary under the statutory
scheme to fulfill its con stitutional obligations as held by the Court inter alia in Ashoka
Marketing Ltd. (supra).
51. Barak in his exhaustive work on 'Pur posive Construction' explains various mean ings
attributed to the term "purpose". It would be in the fitness of discussion to re fer to
Purposive Construction in Barak's words :
"Hart and Sachs also appear to treat "pur pose" as a subjective concept. I say "appear"
because, although Hart and Sachs claim that the interpreter, should imagine himself or
herself in the legislator's shoes, they in troduce two elements of objectivity: First, the
interpreter should assume that the legisla ture is composed of reasonable people seek ing
to achieve reasonable goals in a reason able manner; and second, the interpreter should
accept the non-rebuttable presump tion that members of the legislative body sought to
fulfill their constitutional duties in good faith. This formulation allows the interpreter to
inquire not into the subjec tive intent of the author, but rather the in tent the author would
have had, had he or she acted reasonably."
16. Submission of the learned counsel for the respondent that the said question was not
raised before the learned Trial Judge or before the High Court is of no moment. Sub-
section (5) of Section 357 of the Code casts a duty upon the court. It was for the Trial
Court/High Court to take the same into consideration. Such consideration was re quired
to be bestowed despite the fact that the said provision was not brought to its notice.
17. Actus curiae neminem gravabit (no person shall be prejudiced by an act of court) is a
well-known maxim. In any event, this Court in exercise of its jurisdiction under Article
136 as also under Article 142 of the Constitution of India can direct rectification of a
mistake committed by the courts be low :
18. We, therefore, are of the opinion that the impugned judgment should be modified and
is directed to be modified accordingly. The matter is remitted to the learned Trial Judge.
The learned Trial Judge is directed to take into consideration the amount of compensation
deposited by the appellants in the criminal case and for the said pur pose, the learned
Trial Judge should draw up a fresh decree while correcting the de cree in terms of the
order of this Court. The learned Trial Judge shall, while preparing a fresh decree, take
into consideration the various dates on which the diverse amounts had been deposited by
the appellants and calculate the interest payable thereupon.
19. The appeal is allowed to the afore mentioned extent. In the facts and circum stances
of the case, there shall be no order as to costs.
Order accordingly.
AIR 2008 SUPREME COURT 3205 "Samundra Devi v. Narendra Kaur"
(From : Madhya Pradesh)*
Coram : 2 S. B. SINHA AND CYRIAC JOSEPH, JJ.
Civil Appeal No.4748 of 2008 (arising out of SLP (C) No. 26125 of 2005), D/- 1 -8
-2008.
Samundra Devi and Ors. v. Narendra Kaur and Ors.
(A) Motor Vehicles Act (59 of 1988), S.147, S.149 - MOTOR VEHICLES - Third party
risk - Liabil ity fastened on insurer - Insurer would be bound to indemnify insured unless
exceptions provided in S.149 are at tracted. (Para 14)
@page-SC3206

(B) Motor Vehicles Act (59 of 1988), S.147, S.168 - MOTOR VEHICLES - WRITS -
Liability of insurer - In certain situations, insurer would not be liable to reimburse insured
- However, Court can direct insurer to pay compen sation to third party and recover
amount from owner of vehicle.
Constitution of India, Art.226. (Para 15)
(C) Motor Vehicles Act (59 of 1988), S.149(2), S.170 - Civil P.C. (5 of 1908), O.41, R.33
- MOTOR VEHICLES - APPEAL - INSURANCE - Quantum of compensa tion -
Interference with at instance of insurer - Permissibility - Award - In surer challenging
quantum of compensa tion on ground that driver was not hold ing valid licence - No
appeal preferred either by owner or driver of vehicle - In absence of any leave granted in
terms of S.170, insurer cannot question quantum of compensation by taking umbrage un
der O.41, R.33, C.P.C. as no indepen dent appeal by it was maintainable - Di rection
issued by High Court to insurer to pay reduced amount of compensation and recover
amount from owner - Is im proper.
Misc. Civ. App. No.621 of 2001, D/-16-05-2005 (M.P.), Reversed.
2007 AIR SCW 766, Relied on. (Paras 16, 17, 18)
Cases Referred : Chronological Paras
2007 AIR SCW 766 : AIR 2007 SC 989 : 2007(2) AIR Kar R 443 (Rel on.) 19
Ms. Kamini Jaiswal, for Appellants; M. K. Dua, for Respondents.
* Misc. Civ. Appeal No. 621 of 2001, D/- 16-5-2005 (M.P.)
Judgement
S. B. SINHA, J. :- Leave granted.
2. This appeal is directed against a judg ment and order dated 16.05.2005 passed by the
High Court of Madhya Pradesh at Jabalpur reducing the amount of compen sation
awarded in favour of the appellants herein by the Tribunal from Rs. 13,32,000/-to Rs.
6,96,000/- in an appeal preferred by the claimants in terms of Section 173 of the Motor
Vehicles Act, 1988.
3. One Shiv Shakti Singh while proceed ing in a car on 9.4.1998 met with an acci dent
having been hit by a truck bearing reg istration No.MP-09-KA-6899. The said truck was
owned by the first respondent herein and was being driven at the relevant time by the
second respondent. The said truck was insured with the respondent No.3. As a result of
the injuries sustained in the said accident, Shiv Shakti Singh died. The car was also
damaged.
4. Appellants herein filed an applica tion under Section 166 of the Motor Vehicles Act,
1988 claiming compensation for a sum of Rs. 31,89,000/-. One of the contentions raised
by respondent No. 1 was that the ac cident took place owing to the acts of negli gence on
the part of the deceased himself. Respondent No.3 furthermore contended that the
respondent No.2 did not possess a valid and effective driving licence.
Several issues were framed by the Tribu nal, inter alia, as regards, the breach of policy
conditions as also the quantum of compen sation.
By an award dated 8.2.2001, the Motor Vehicles Accident Claims, Tribunal, held :
(a) The accident took place due to the negligence of the driver of the truck;
(b) The deceased having been earning a sum of Rs. 10,000/-, the amount of contri bution
to his family was about 8,000/- per month and, thus, on application of the multiplier of
16, the total loss of dependency would be a sum of Rs. 12,80,000/-. A sum of Rs. 15,000
as loss of companionship for applicant No.2, a sum of Rs.2,000/- towards funeral
expenses and Rs. 35,000/- towards costs of repairing of the Maruti Car, were also
granted.
(c) The driver of the truck did not pos sess a valid driving licence, and, therefore, breach
of policy of the contract of insurance was established as a result whereof the re spondent
No.3 was not liable to reimburse the owner of the vehicle any such amount payable by
him by way of compensation payable.
5. Appellants preferred an appeal there-against, contending :
a) The insurer ought to have been found to be liable to pay the amount of compensa tion
along with the owner and driver; and
b) The appropriate multiplier adopted should have been 20 instead of 16.
6. Admittedly, no appeal was preferred against the said judgment and award by the owner
and driver of the vehicle as also the insurer thereof. Before the High Court, the
respondent No.3, inter alia, contended that in the event it be held that it was liable to
reimburse the owner of the vehicle, it was entitled to contest the quantum of compen
sation as being excessive.
@page-SC3207
In view of the said contention, the High Court formulated the following questions for its
consideration :
"(i) Whether the insurer is liable to in demnify the owner of the vehicle and there fore
liable to pay the compensation?
(ii) If so, what will be the just compen sation?"
7. The High Court, on perusal of the driv ing licence, the contract of insurance as also the
testimonies of witnesses examined on behalf of the parties, held :
8. We accordingly hold that the Insurer having established that the driver was not "duly
licenced' to drive the truck in ques tion and also having established want of care on the
part of the insured in allowing the insured truck to be driven by a driver who possessed
only a LMV (Private) licence, on paying the compensation amount to the
Appellants/Claimants and recover the same from the Insured (Respondent No. 1)."
8. Despite noticing the fact that the ap peal was by the claimants, the High Court took
into consideration the contention raised by the respondent No.3 that no documen tary
evidence having been produced to es tablish the income of the deceased, it should be
reduced to 49,000/- for the period end ing 31.3.1996 and Rs. 53,000/- for the year
31.3.1997. As regards to the finding of the learned Tribunal that the deceased had in come
from the agricultural lands, it was opined that as agricultural land continued to be owned
by the family; what was lost was only the valuable supervision of the deceased. On the
said findings, it was held :
"14. On an overall consideration of the evidence, we are of the view that the income of
the deceased from the profession and the value (cost) of supervision of agriculture land
should be as Rs. 5,000/- per month or Rs. 60,000/- per annum. If one-third is de ducted
towards personal and living expenses of the deceased, the contribution to the fam ily
would be Rs. 40,000/- per annum. As the deceased was 39 years, the appropri ate
multiplier is 16. Therefore, the total loss of dependency will be Rs. 6,40,000/-......
15. Normally, we would not have inter fered by reducing the compensation in an appeal
filed by the claimants. But as no ticed above, the insurer was exonerated and when it is
sought to be made liable, it can point out that the compensation is exces sive. Though we
pointed to the Appellants' counsel that by not pressing the appeal, the claimants may have
the advantage of an award for Rs. 13,32,000/- against the owner as the owner has not
challenged the award, he submitted that unless the insurer is made liable, the chances of
recovery are difficult and therefore the appellants would like to pursue the appeal."
9. The appeal was, thus, allowed, direct ing :
"(a) The insurer (Third Respondent) is also jointly and severally made liable to pay the
compensation to Appellants.
(b) The compensation is reduced from Rs. 13,32,000/- to Rs. 6,96,000/- with inter est @
12% per annum from the date of peti tion till date of deposit.
(c) On payment by the Insurer, it shall be entitled to recover the amount paid from the
owner of the vehicle (insured), namely the First Respondent, by executing the award."
"Normally, we would not have interfered by reducing the compensation in an appeal filed
by the claimants. But as noticed above, the insurer was exonerated and when it is sought
to be made liable, it can point out that the compensation is excessive. Though we pointed
to the Appellants' counsel that by not pressing the appeal, the claimants may have the
advantage of an award for Rs. 13,32,000/-against the owner as the owner has not
challenged the award, he sub mitted that unless the insurer is made li able, the chances of
recovery are difficult and therefore the appellants would like to pursue the appeal."
10. The appellant is, thus, before us.
11. Ms. Kamini Jaiswal, learned coun sel appearing on behalf of the appellant, would
submit that the High Court commit ted a serious error in reducing the amount of
compensation from Rs. 13,32,000/- to Rs. 6,96,000/-. It was urged that the rea sonings of
the High Court are not legally sustainable as the owner and driver as also the insurer had
not preferred any appeal against the award dated 8.2.2001 passed by the learned Motor
Vehicles Accident Claims Tribunal.
12. Mr. M.K. Dua, learned counsel ap pearing on behalf of the respondent, on the other
hand, would support the judgment.
@page-SC3208
13. The claimants/appellants filed an application for grant of compensation on the
premise that they suffered damages owing to the acts of rashness and negligence in
driving on the part of the respondent No.2. The owner of the vehicle as also the driver
thereof were, thus, principally liable to pay compensation to the dependents of the de
ceased.
14. A contract of Insurance as is well known is a contract of indemnity. In a case of
accident, the primary liability under law for payment of compensation is that of the
driver. The owner of the vehicle also becomes vicariously liable therefor. In a case involv
ing a third-party to the contract of insur ance in terms of Section 147 of the Motor
Vehicles Act, 1988 providing for a compul sory insurance, the insurer becomes statu
torily liable to indemnify the owner. Indis putably, the insurance company would be
liable to indemnify the insured in respect of loss suffered by a third party or in respect of
damages of property. In a case, therefore, where the liability is fastened upon the in surer,
the insurer would be bound to indem nify the insured unless the exceptions con tained in
Section 149 of the Act are attracted.
15. It has not been disputed before us that in certain situations while opining that the
insurance company would not be liable to reimburse the insured, a direction upon the
insurance company to pay the amount of compensation to a third party and recover the
same from the owner of the vehicle is permissible. Such a direction has been is sued by
the High Court. The said directions are not under challenge.
Keeping in view the aforementioned prin ciple in mind, the question which arises for our
consideration is as to whether it was permissible for the High Court to interfere with the
quantum of compensation as awarded by the learned Tribunal, although no appeal was
preferred either by the owner or the driver of the vehicle nor any appeal was preferred by
the insurance company.
16. Indisputably, in relation to a third party, the grounds upon which the insurer can deny
its liability are contained sub-sec tion (2) of Section 149 of the Act. Ordinarily and
subject to just exceptions, the insur ance company would have no right to ques tion the
quantum of compensation in ab sence of any leave having been granted in its favour in
terms of Section 170 of the Act. The High Court, with respect, failed to con sider this
aspect of the matter. Appellants preferred appeals before it on limited grounds. Their
contentions could have been rejected or accepted. The High Court, how ever, could not
have considered the conten tion raised on behalf of the respondent No.3 which was not
available to them in law. It was legally impermissible for the respondent No.3 to question
a finding of fact arrived at by the Tribunal, taking umbrage under Or der 41 Rule 33 of
the Code of Civil Proce dure or otherwise. It could not have been permitted to do so. It is
well settled that what cannot be permitted to be done directly, cannot be permitted to be
done indirectly. Indisputably, no leave was obtained in terms of Section 170 of the Act.
The quantum of compensation awarded by the learned Tri bunal was accepted by the
owner. Only in some exceptional cases and that too when the liability to pay the amount
of compen sation is fastened upon the insurance com pany and insured, it can be heard on
issues relating to the quantum of compensation and not otherwise.
17. In this case, the respondent No.3 has been given liberty to recover the amount of
compensation from the owner of the ve hicle. The insurance company has been held to
have no statutory liability as one of its contentions that the driver was not holding a valid
and effective driving licence has been upheld.
18. In the aforementioned situation, we are of the opinion that even Order 41 Rule 33 of
the Code of Civil Procedure was not applicable as in a situation of this nature, the
respondent No.3 ordinarily could not have maintained an independent appeal on the
quantum of compensation having regard to the fact situation obtaining herein, and, thus,
in our opinion, the High Court com mitted a serious error in issuing the im pugned
directions, despite noticing that even no appeal has been preferred by the owner or driver
of the vehicle as also respondent No.3.
19

. Order 41 Rule 33 of the Code of Civil Procedure has limited application. When there
exists a legal interdict, the same would not apply. It was so held in S. Nazeer Ahmed v.
State Bank of Mysore and Ors. [(2007) 11 SCC 75], stating : 2007 AIR SCW 766

@page-SC3209
"8. We also see considerable force in the submission of learned Counsel for the ap
pellants that the High Court has miscon ceived the object of Order XLI Rule 33 of the
Code and has erred in invoking it for the purpose of granting the plaintiff Bank a de cree.
This is a case where the suit filed by the plaintiff Bank had been dismissed by the trial
court. The plaintiff Bank had come up in appeal. It was entitled to challenge all the
findings rendered against it by the trial court and seek a decree as prayed for in the plaint,
from the appellate court. Once it is found entitled to a decree on the basis of the reasoning
of the appellate court, the suit could be decreed by reversing the appropri ate findings of
the trial court on which the dismissal of the suit was based. For this, no recourse to Order
XLI Rule 33 is necessary. Order XLI Rule 33 enables the appellate court to pass any
decree that ought to have been passed by the trial court or grant any further decree as the
case may require and the power could be exercised notwithstand ing that the appeal was
only against a part of the decree and could even be exercised in favour of the
respondents, though the re spondents might not have filed any appeal or objection against
what has been decreed. There is no need to have recourse to Order XLI Rule 33 of the
Code, in a case where the suit of the plaintiff has been dismissed and the plaintiff has
come up in appeal claiming a decree as prayed for by him in the suit. Then, it will be a
question of entertaining the appeal considering the relevant ques tions and granting the
plaintiff the relief he had sought for if he is found entitled to it. In the case on hand
therefore there was no occasion for applying Order XLI Rule 33 of the Code. If the view
of the High Court was that the suit was barred by Order II Rule 2 of the Code, it is
difficult to see how it could have resorted to Order XLI Rule 33 of the Code to grant a
decree to the plaintiff in such a suit. In that case, a decree has to be de clined. That part of
the reasoning of the High Court is therefore unsustainable."
20. In view of our findings aforemen tioned, the impugned judgment cannot be sustained.
It is set aside accordingly. The appeal is allowed. However, in the facts and circumstances
of the case, there shall be no order as to costs.
Appeal allowed.
AIR 2008 SUPREME COURT 3209 "Ponnam Chandraiah v. State of A. P."
(From : Andhra Pradesh)*
Coram : 2 Dr. A. PASAYAT AND P. SATHASIVAM, JJ.
Criminal Appeal No. 1182 with 1183 of 2008 (arising out of SLP (Cri.) Nos. 6715 with
6792 of 2006), D/- 30 -7 -2008.
Ponnam Chandraiah v. State of A.P.
(A) Evidence Act (1 of 1872), S.3 - EVIDENCE - WITNESS - MURDER - Interested
witness - Relation with de ceased - Does not affect credibility - More often than not a
relation would not conceal actual culprit and inculpate an innocent person.
Penal Code (45 of 1860), S.300. (Para 8)
(B) Evidence Act (1 of 1872), S.3 - EVIDENCE - MURDER - CULPABLE HOMICIDE
- APPEAL - Appreciation of evidence - Falsus in uno falsus in omnibus - Principle of, has
no application - Murder case - Plea that since evidence of witnesses has not been
accepted as regards some accused entire prosecution case has to be rejected - Untenable -
In appeal by co-accused, considering evidence, their conviction al tered from S.300 to
S.304. Part I - Simi lar benefit extended to appellant.
Penal Code (45 of 1860), S.300, S.304, Part I. (Para 13)
Cases Referred : Chronological Paras
2006 AIR SCW 3680 : AIR 2006 SC 2716 : 2006 Cri LJ 4033 15
2004 AIR SCW 6916 : AIR 2005 SC 249 : 2005 All LJ 201 (Rel. on, Pnt B) 14
2003 AIR SCW 3984 : AIR 2003 SC 3617 : 2003 Cri LJ 3876 (Rel. on, Pnt B) 14
(2002) 3 SCC 76 (Rel. on) 13
AIR 1974 SC 276 : 1974 Cri LJ 331 (Rel. on) 10
AIR 1973 SC 2407 : 1973 Cri LJ 1589 (Rel. on) 13
AIR 1965 SC 202 : 1965 (1) Cri LJ 226 (Rel. on) 12
AIR 1957 SC 366 : 1957 Cri LJ 550 (Rel. on, Pnt B) 13
AIR 1957 SC 614 : 1957 Cri LJ 1000 (Rel. on) 10
AIR 1953 SC 364 (Rel. on) 9
AIR 1952 SC 54 : 1952 Cri LJ 547 (Ref.) 11
Ms. K. Amareswari, V. R. Reddy, Sr. Ad vocates; P. Venkat Reddy, Anil Kumar Tandale,
V. G. Pragasam, S.J. Aristotle and
@page-SC3210
Prabu Ramasubramanian, for Appellant; Mrs. D. Bharathi Reddy, for Respondent.
* Cri. Appeal No. 1130 of 2005, D/- 27-7-2006 (A.P.)
Judgement
1. Dr. ARIJIT PASAYAT, J. :- Leave granted.
2. Appellants question correctness of the judgment rendered by a Division Bench of the
Andhra Pradesh High Court.
3. Sixteen persons including the appel lants faced trial for alleged commission of offences
punishable under Sections 147, 148, 448 read with Sections 149, 302 read with Section
149 and Section 324 read with Section 149 of the Indian Penal Code, 1860 (in short the
'IPC'). Learned III Addi tional Sessions Judge, Karimnagar for each one of them guilty. In
appeal, High Court upheld the conviction of A1, A3, A7 to A9, A12 and A13 and sentence
of imprisonment for life as imposed by the trial court. Rest of the accused persons were
acquitted. The present appeals are by A7 to A9 and A13.
4. Background facts in a nutshell are as follows :
PW-1 is the wife,. PW-2 is the father, PW-3 is the mother, PW-4 is the brother and PW-5
is the sister-in-law of the deceased. The accused, deceased and the material wit nesses are
residents of Neerukulla village. The deceased purchased an Auto and was plying in
between Sulthanabad and Neerukulla. On 02-07-2003 at about 9-00 p.m., the deceased
returned to his house from Sulthanabad and informed PWs-1 to 3 that when he requested
A-1 and A-2 to travel in his Auto as per the serial number, they refused to travel in his
Auto and beat him. On 03-07-2003 morning, PW-1 and the de ceased went to the house
of the Sarpanch and raised a dispute. The Sarpanch called A-1-and informed about the
incident. A-l admitted his guilt in the presence of PWs-9 and 10. On the same day at
about 6-00 p.m., A-l to A-16 came to the house of the de ceased and attacked him. A-l
beat the de ceased with a stick. The deceased ran into the house and bolted the door. In
the mean while, when PW-2 intervened to rescue the deceased, A-l beat him with a stick.
A-3 broke the doors and all the accused entered the house and beat the deceased. Some of
the accused were armed with iron rods and axes. They beat the deceased indiscrimi
nately. Then the deceased ran out from the house. The accused chased and beat him
indiscriminately. Finally, the deceased fell down at the Gram Panchayat, office on re ceipt
of the injuries. Later, the deceased was taken in an Auto to the Government Hospi tal,
Sulthanabad. On the advise of the Doc tor, they went to the Police Station and gave Ex.
P-1 report. On the basis of Ex.P-1, the police registered a crime for the offences under
Sections 147, 148, 448, 307, 327 read with 149 of I.P.C. Thereafter, the deceased and
PW-2, who received injuries, were re ferred to the Government Hospital, Karimnagar.
The deceased, while under going treatment, succumbed to the injuries. After the death of
the deceased, the Sections of law were altered in the crime through the alteration memo.
The Inspector of Police took up investigation, prepared the rough sketch, observed the
scene of offence, held inquest over the dead body of the deceased, seized M.Os.1 and 2
and later sent the dead body for post-mortem examination. The accused were arrested and
weapons were recovered. After completion of the investigation, the police laid the
charge-sheet. The accused denied the charges and claimed for trial.
The prosecution, in order to prove the guilt of the accused, examined PWs.1 to 22 and
marked Exs.P-1 to P-39. On behalf of the defence, no oral evidence was adduced, but
Ex.D-1, a portion of Section 161, Cr.P.C. statement of PW-3 was marked.
5. High Court by a common judgment disposed of four appeals numbered as Crimi nal
Appeal Nos. 1114, 1128, 1130 and 1155 of 2005.
6. In support of the appeals learned counsel for the accused persons submitted that the
conviction is based primarily on the evidence of witnesses who were related to the
deceased. Further the accusations even if accepted in toto do not make out a case
relatable to Section 302, IPC.
7. Learned counsel for the respondent-State on the other hand supported the judg ments
of the Courts below.
8. In regard to the interestedness of the witnesses for furthering the prosecution ver sion,
relationship is not a factor to affect the credibility of a witness. It is more often than not
that a relation would not conceal the actual culprit and make allegations against an
innocent person. Foundation has to be laid if a plea of false implication is made. In such
cases, the court has to adopt a careful approach and analyse evidence to
@page-SC3211
find out whether it is cogent and credible.
9
. In Dalip Singh and Ors. v. The State of Punjab (AIR 1953 SC 364) it has been laid down
as under :- Para 26 of AIR

"A witness is normally to be considered independent unless he or she springs from


sources which are likely to be tainted and that usually means unless the witness has
cause, such as enmity against the accused, to wish to implicate him falsely. Ordinarily a
close relation would be the last to screen the real culprit and falsely implicate an in
nocent person. It is true, when feelings run high and there is personal cause for enmity,
that there is a tendency to drag in an inno cent person against whom a witness has a
grudge along with the guilty, but founda tion must be laid for such a criticism and the
mere fact of relationship far from being a foundation is often a sure guarantee of truth.
However, we are not attempting any sweeping generalization. Each case must be judged
on its own facts. Our observations are only made to combat what is so often put forward
in cases before us as a general role of prudence. There is no such general rule. Each case
must be limited to and be governed by its own facts."
10

. The above decision has since been followed in Gull Chand and Ors. v. State of
Rajasthan (1974 (3) SCC 698) in which Vadivelu Thevar v. State of Madras (AIR 1957
SC 614) was also relied upon. AIR 1974 SC 276

11

. We may also observe that the ground that the witness being a close relative and
consequently being a partisan witness, should not be relied upon, has no substance. This
theory was repelled by this Court as early as in Dalip Singh's case (supra) in which
surprise was expressed over the impres sion which prevailed in the minds of the Members
of the Bar that relatives were not independent witnesses. Speaking through Vivian Bose,
J. it was observed : AIR 1953 SC 364, Para 25

"We are unable to agree with the learned Judges of the High Court that the testimony of
the two eye-witnesses requires corrobo ration. If the foundation for such an obser vation
is based on the fact that the witnesses are women and that the fate of seven men hangs on
their testimony, we know of no such rule. If it is grounded on the reason that they are
closely related to the deceased we are unable to concur. This is a fallacy common to many
criminal cases and one which another Bench of this Court endeav oured to dispel in -
'Rameshwar v. State of Rajasthan' (AIR 1952 SC 54 at p.59). We find, however, that it
unfortunately still persists, if not in the judgments of the Courts, at any rate in the
arguments of counsel."
12. Again in Masalti and Ors. v. State of U.P. (AIR 1965 SC 202) this Court observed (p.
209-210 para 14) :
"But it would, we think, be unreasonable to contend that evidence given by witnesses
should be discarded only on the ground that it is evidence of partisan or interested
witnesses.......The mechanical rejection of such evidence on the sole ground that it is
partisan would invariably lead to failure of justice. No hard and fast rule can be laid down
as to how much evidence should be appreciated. Judicial approach has to be cautious in
dealing with such evidence; but the plea that such evidence should be re jected because it
is partisan cannot be ac cepted as correct."
13. To the same effect is the decision in State of Punjab v. Jagir Singh (AIR 1973 SC
2407) and Lehna v. State of Haryana (2002 (3) SCC 76). Stress was laid by the accused-
appellants on the non-acceptance of evi dence tendered by some witnesses to con tend
about desirability to throw out entire prosecution case. In essence prayer is to apply the
principle of "falsus in uno falsus in omnibus" (false in one thing, false in ev erything).
This plea is clearly untenable. Even if major portion of evidence is found to be deficient,
in case residue is sufficient to prove guilt of an accused, notwith standing acquittal of
number of other co-accused persons, his conviction can be maintained. It is the duty of
Court to sepa rate grain from chaff. Where chaff can be separated from grain, it would be
open to the Court to convict an accused notwith standing the fact that evidence has been
found to be deficient to prove guilt of other accused persons. Falsity of particular ma
terial witness or material particular would not ruin it from the beginning to end. The
maxim "falsus in uno falsus in omnibus" has no application in India and the witnesses
cannot be branded as liar. The maxim "falsus in uno falsus in omnibus" has not received
general acceptance nor has this maxim come to occupy the status of rule of law. It is
merely a rule of caution. All that it amounts to, is that in such cases testimony may be
@page-SC3212
disregarded, and not that it must be disre garded. The doctrine merely involves the
question of weight of evidence which a Court may apply in a given set of circumstances,
but it is not what may be called 'a manda tory rule of evidence'. (See Nisar Ali v. The
State of Uttar Pradesh, (AIR 1957 SC 366).
14

. The above position was elaborately discussed in Sucha Singh and Anr. v. State of Punjab
(2003 (6) JT SC 348), and Israr v. State of U.P. (2005 (9) SCC 616). 2003 AIR
SCW 3984
2004 AIR SCW 6916
2006 AIR SCW 3680

15. In S. Sudershan Reddy v. State of A.P. (AIR 2006 SC 2716), it was observed :
Relationship is not a factor to affect credibility of a witness. It is more often than not that
a relation would not conceal actual culprit and make allegations against an innocent
person. Foundation has to be laid if plea of false im plication is made. In such cases, the
court has to adopt a careful approach and analyse evidence to find out whether it is
cogent and credible.
16. In Criminal Appeal No. 222 of 2007, this Court has occasioned to deal with the cases
of some of the co-accused persons. In that case it was concluded as follows :
"If the evidence on record is considered on the touchstone principles set out above the
inevitable conclusion is that the proper conviction would be Section 304, Part I, IPC
instead of Section 302, IPC. The conviction of the appellants is accordingly altered from
Section 302 read with Section 149 to Sec tion 304, Part I read with Section 149, IPC.
Custodial sentence of 10 years would meet the ends of justice. The findings of the guilt in
respect of other offences and the sen tences imposed do not warrant interference. The
sentence shall run concurrently."
17. In view of what has been stated in the aforesaid Criminal Appeal, the appeals are
allowed to the aforesaid extent.
18. The appeals are partly allowed.
Order accordingly.
AIR 2008 SUPREME COURT 3212 "Rajbabu v. State of M. P."
(From : 2003 (3) MPLJ 179)
Coram : 2 R. V. RAVEENDRAN AND Dr. MUKUNDAKAM SHARMA, JJ.
Criminal Appeal No.895 of 2003, D/- 24 -7 -2008.
Rajbabu and Anr. v. State of M.P.
(A) Evidence Act (1 of 1872), S.113A - EVIDENCE - ABETMENT TO SUICIDE -
Abetment of suicide - Presumption as to - Mere facts of suicide by woman within 7 years
and cruelty by husband does not automatically give rise to pre sumption - Court is
required to look into all other circumstances of case.
2001 AIR SCW 4282; 1994 AIR SCW 844, Relied on. (Para 15)
(B) Penal Code (45 of 1860), S.306, S.498A - Evidence Act (1 of 1872), S.113A -
ABETMENT TO SUICIDE - CRUELTY BY HUSBAND OR HIS RELATIVE -
EVIDENCE - Abetment of suicide and cruelty - Proof - Deceased set herself on fire
within two years of marriage - Evidence on record disclosed that deceased was unhappy
as her husband was illiterate and poor - And mother-in-law of deceased used to ask her to
run hand-driven flour mill - No direct evidence or other cir cumstances to establish that
mother-in-law either aided or instigated her to com mit suicide - Presumption u/S.113-A
of Evidence Act cannot be drawn - Though letter allegedly written by de ceased mentions
attitude of family was not good towards deceased, no such Inci dent was mentioned -
Mother-in-law is, therefore, entitled to benefit of doubt.
2003 (3) MPLJ 179, Reversed. (Paras 16, 17, 18, 19, 20)
Cases Referred : Chronological Paras
2001 AIR SCW 4282 : AIR 2001 SC 3837 : 2001 Cri LJ 4724 (Rel. on) 15
1994 AIR SCW 844 : AIR 1994 SC 1418 : 1994 Cri LJ 2104 (Rel. on) 15
(1950) 2 All ER 458 : 66 TLR (P2) 589 (CA) 15
Shiv Sagar Tiwari, for Appellants; Ms. Vibha Datta Makhija, for Respondent.
Judgement
1. Dr. MUKUNDAKAM SHARMA, J. :- By this Judgment and Order we propose to
dispose of the appeal filed by the appel lants against the judgment and order dated 23-9-
2002 of the High Court of Madhya Pradesh at Jabalpur whereby the Learned Single
Judge dismissed the appeal filed by the appellants against the judgment and order dated
17-6-1989 of the Learned Addi tional Sessions Judge, Khurai, convicting the appellants
under the provisions of Sec tions 306 and 498-A of the Indian Penal Code (for short 'the
IPC') and sentencing each of them to undergo rigorous imprison ment for three years on
each count.
2. The deceased, Shanti Bai, daughter
@page-SC3213
of Janak Rani (PW-1) and Gyan Das (PW-2) was married to Rajbabu-appellant No.l two
years prior to the date of occurrence. On 17.7.1988 Shanti Bai set herself on fire in her
matrimonial home and she died be cause of burn injuries received by her. At the time
when the occurrence took place the Appellant No.2, Smt. Munnibai (mother-in-law of the
deceased) had gone to fetch water from the well. The husband of Appellant No. 2, Shri
Jagat Bandhu (father-in- law of the deceased), who was acquitted by the trial court, was
away to some other place, whereas Appellant No. 1 had gone for cut ting wood front the
forest. Appellant No. 1 immediately coming to know about the in cident came back and
lodged the first infor mation report at police station Bhangarh which was recorded by the
Head Constable Narbada Prasad, who was examined as PW-9 during the trial. The said
report which was lodged by the appellant No. 1 was exhibited during the trial and was
marked as Ex. P. 16. The deceased was carried to the railway sta tion Karonda for being
taken to the Govern ment Hospital at Bina for treatment. The police station Incharge, Mr.
Ashok Chourasia (PW-8), also arrived at the railway station and recorded the dying
declaration, wherein it was noted that Shanti Bai died in the ac cidental fire while
cooking food in the house. In the said dying declaration the deceased exculpated all the
members of her matrimo nial home. Immediately thereafter she died at the railway station
itself. The police there after sent the body for post-mortem exami nation which was
conducted and exhibited as Ex. P.20 during the trial. According to the said post-mortem
report the deceased suffered 90% burns which were found to be ante mortem. The police
thereafter started investigation and on completion thereof, submitted a charge-sheet
against Rajbabu-appellant No. 1, Smt. Munnibai-appellant No. 2, who is mother of
appellant No. l and Jagat Bandhu, father of the appellant No.1 under Sections 306 and
498-A of the IPC. On the basis of the aforesaid charge-sheet, charges were framed against
all the three accused-appellants under Sections 306 and 498-A of the IPC for treating the
deceased with cruelty and abetting her to commit suicide as a result of which allegedly
she committed suicide.
3. During the course of the trial, alto gether eight witnesses were examined in order to
prove the charges against the accused persons. Trial court after hearing the arguments and
on appreciation of the evi dence on record acquitted accused No.3, the father of the
Appellant No. 1, whereas an order was passed convicting appellant No.1 and appellant
No.2 under Sections 306 and 498-A of the IPC after holding that offences against both of
them are proved beyond rea sonable doubt. The learned trial court there after passed an
order of sentence, sentenc ing both the appellants to undergo rigor ous imprisonment for
three years on each count. Both the sentences were to run con currently. No fine was
imposed. Against the aforesaid judgment both the appellants filed an appeal before the
High Court which was dismissed by its judgment on 23rd Septem ber, 2002. Being
aggrieved by the said judg ment this appeal has been filed by the ap pellants. During the
course of the trial they were granted bail. In the present appeal or der was passed by this
Court enlarging them on bail.
4. We have heard the learned counsel appearing for the appellants. However, coun sel for
the respondent-State did not appear in the hearing of the appeal although her name was
shown in the daily cause list. Counsel appearing for the appellants at the very outset
brought to our notice that Ap pellant No. 1, namely, Rajbabu son of Jagat Bandhu had
died on 27th September, 2005 at village Sabgah. The said appellant hav ing died, the
appeal filed by him stands dis posed of having been abated and therefore having been
rendered infructuous. This appeal, therefore, survives only so far as accused/appellant
No.2, namely, Smt. Munni Bai is concerned.
5. Learned counsel appearing for the appellant, Smt. Munni Bai submitted that the
deceased had died of burn injuries re ceived by her while she was cooking food in the
kitchen in her matrimonial home and that it is clearly established from the records that all
the other members of the family, at the time of occurrence were not present. It was also
submitted that the prosecution case itself indicates that appellant No. 2 had gone out of
the house for fetching water and, therefore, she could not have been held guilty for an
offence either under Section 306 or 498-A of the IPC. He further submit ted that the only
incriminating evidence that could be said to be available against her is the letter which
was allegedly written by the deceased and was exhibited as Ex. P. 1 and
@page-SC3214
a dying declaration which was recorded by Shri Ashok Choursia, the Investigating Of
ficer who was examined as PW-8. It was submitted by him that none of the aforesaid
documents pinpoint the guilt of the appel lant in the act of commission of suicide by the
deceased.
6. We have considered the aforesaid sub missions in the light of the record including the
evidence adduced on behalf of the pros ecution. There is no eye-witness to the oc
currence of the act of suicide committed by the deceased who was the daughter-in-law as
she was the only person available at the relevant time in the matrimonialhome. At that
point of time she was cooking food for all the members of the family who had gone out
of the house. Her husband, appellant No. 1 had gone to the forest for collecting wood
whereas her father-in-law, who was original accused No.3 had gone out of the house for
some other work and whereas the sole appellant before us, had gone out of the house to
fetch water. The only evidence that has been produced and was used for levelling
accusations against the present appellant was the dying declaration and the contents of
Ex. P. 1 which is stated to be a letter written by the deceased. Some of the witnesses like
PW-1 and PW-3, the family members of the parental home of the de ceased have stated in
their deposition about the alleged ill- treatment meted out to the deceased by the in-laws
family. Let us there fore analyse and appreciate the said pieces of evidence as available
on record.
7. The dying declaration was recorded on 17.7.1988 at about 12.45 hrs. by the In
vestigating Officer, PW-8 at the railway sta tion from where the deceased was to be taken
to the hospital for medical treatment. The incident had taken place at about 10.30 a.m.
Deceased had stated in the said dying declaration which was recorded in the pres ence of
some of the villagers that while pour ing kerosene oil, one end of her sari caught fire as
she was cooking food and that in the aforesaid manner she got burnt. It is also stated by
her in the said dying declaration that she did not set fire on her own and nobody set fire
on her and that while pre paring meal her sari caught fire accidentally. She has
categorically stated in the said dy ing declaration that no quarrel had taken place and that
there was no problem in her matrimonial home. The said statement was read over to her
and her thumb impression was put as she could not sign because of the burn injuries
received by her.
8. The courts below have questioned the evidentiary value of the said dying declara tion.
A perusal of the said dying declaration would prove and establish that there is noth ing
incriminating in the said statement against the appellant and, therefore, the said dying
declaration, which was exculpatory in nature, so far as the prosecution is con cerned is of
no relevance and would rather help the accused-appellants. As there is nothing
incriminating in the said document against the appellants, neither are we in clined nor are
we required to go into the question of evidentiary value of the said document.
9. The other incriminating document against the accused-appellants is the un dated letter
exhibited as exhibit P. 1. The said letter appears to have been written by the deceased,
addressing to father, mother and the younger brothers of the husband. In the said
statement the deceased has stated that she is unable to tolerate the atmosphere in the
family in her matrimonial home. She also stated that she prefer to live in hell because in-
laws have done such acts with her which are of no use to mention. She has also stated
that whatever has been done was all-right. In her letter she has stated that she had always
considered her father-in-law and mother-in- law more than her parents and that even then
they have treated her in such a manner which she never expected. It is mentioned therein
that the matrimonial house was ruined after her arrival and that she was treated like an
enemy. She has stated that her mother-in-law had told that if she (Shanti Bai) is kept in
their house then nothing will remain. In that view of the mat ter she did not want to
become burden on herself nor on her in laws and that moment was the last time of her
life. Of course, in the letter there is no date written but to wards the end of the letter it
was mentioned that it was the last day of her life. She also stated that she had a long life
but the hard words had made her life incomplete and she has no further time to write
further. The said letter appears to have been written on the date of occurrence and in the
said letter she had given vent to all her expressions, feel ings and contempt for the family.
The said letter was found in the box seized from the room where the incident Occurred.
10. The issue, therefore, that falls for
@page-SC3215
our consideration is whether the conviction can be based against the appellant No. 2 on
the basis of the said letter alone.
11. The prosecution has examined the mother of the deceased as PW-1. She had stated in
her deposition that her daughter told her that in her in-laws house, her mother-in-law used
to ask her to run hand flour-mill. She also stated that her son-in-law Rajbabu also used to
quarrel with her daughter and used to beat her. She has also stated that her daughter told
her that her mother-in-law used to use filthy language for her father and brothers. It is
further stated by her that once her husband had gone to bring Shanti Bai, at that time her
mother-in-law did not send her rather she was beaten by her in his presence for not
cleaning the utensils. Thereafter her hus band came back. In her cross-examination she
stated that her daughter wanted to be come educated and wanted to go for employ ment.
Her daughter told her after coming back from the matrimonial home that her husband is
not educated and the family is poor for which she had expressed pain. Her daughter told
her that her life would be spoiled in that house and on that issue she was very unhappy. It
was also stated by her that her daughter never sent any letter from her in-law's house. She
further replied in her cross-examination that the deceased never told anything to her
relatives and members of the society regarding her troubles because she never wanted to
make her life public.
12. We have also on record the deposi tion of the sister-in-law of the deceased Smt.
Kamla Rani who was examined as PW-3. She has also deposed that when Shanti Bai
came back from her in-law's house for the first time she told her that her husband and
mother-in-law are troubling her very much. She is forced to run hand driven flour-mill
which she was not habitual and when she was not able to run the flour-mill, her mother-
in- law and husband used to beat her. It has also been stated in her deposi tion that after
coming back from her in-law's house Shanti Bai told her that once there had been a
quarrel between her and her mother-in-law and on the said issue her husband wanted to
burn her but at that moment her mother-in-law stopped her husband not to do so at that
time. It was further stated in her deposition that Shanti Bai told her not to narrate this
story to any of her brothers. The contents of Exhibit P. 1 were approved by PW-3, stating
that the said letter was written by the deceased Shanti Bai.
13. It appears from the statement of PW-1 and also corroborated by the statement of PW-
3 (sister-in-law of the deceased) that the deceased studied upto XI standard and wanted to
study further and wanted to be employed but since her husband was not literate and since
the family was poor, they could not make arrangements for her fur ther studies and they
could not have even allowed her to go for employment, for which she was upset. In her
statement PW-1 had, of course, brought in someallegations about the mother-in-law but
only from that state ment it cannot be said that she had directly any hand in the act of
commission of sui cide. So far as the evidence of PW-1 and PW-3 are concerned, there is
only evidence to the extent that at times the deceased was not treated well by the
appellant.
14. Of course, reliance is placed by the learned courts below on the provisions of Section
113-A of the Indian Evidence Act, 1872 (for short 'the Evidence Act'). Any per son who
abets the commission of suicide is liable to be punished under Section 306, IPC. Section
107, IPC lays down the ingredients of abetment which includes instigating any person to
do a thing or engaging with one or more persons in any conspiracy for the doing of a
thing, if an act or illegal omission takes place in pursuance of that conspiracy and in order
to the doing of that thing, or intentional aid by any act or illegal omis sion to the doing of
that thing.
15

. In the instant case there is no direct evidence to establish that the appellant ei ther aided
or instigated the deceased to com mit suicide or entered into any conspiracy to aid her in
committing suicide. In the ab sence of direct evidence the prosecution has relied upon
Section 113-A of the Evidence Act, under which the court may presume on proof of
circumstances enumerated therein, and having regard to all the other circumstances of the
case, that the suicide had been abetted by the accused. The Ex planation to Section 113-A
further clarifies that cruelty shall have the same meaning as in Section 498-A of the IPC.
Under Sec tion 113-A of the Evidence Act, the pros ecution has first to establish that the
woman concerned committed suicide within a pe riod of seven years from the date of her
2001 AIR SCW 4282

@page-SC3216
marriage and that her husband or any rela tive of her husband had subjected her to
cruelty. Sec. 113-A gives a discretion to the court to raise such a presumption, having
regard to all the other circumstances of the case, which means that where the allega tion
is of cruelty it must consider the na ture of cruelty to which the woman was sub jected,
having regard to the meaning of the word "cruelty" in Section 498-A, IPC. The mere fact
that a woman committed suicide within seven years of her marriage and that she had been
subjected to cruelty by her husband or any relative of her husband, does not automatically
give rise to the pre sumption that the suicide had been abetted by her husband or any
relative of her hus band. The court is required to look into all the other circumstances of
the case. One of the circumstances which has to be consid ered by the court is whether
the alleged cru elty was of such a nature as was likely to drive the woman to commit
suicide or to cause grave injury or danger to life, limb or health of the woman. The law
has been suc cinctly stated in Ramesh Kumar v. State of Chhattisgarh, reported in (2001)
9 SCC 618 wherein this Court ob served :
"12. This provision was introduced by the Criminal Law (Second) Amendment Act, 1983
with effect from 26-12-1983 to meet a social demand to resolve difficulty of proof where
helpless married women were elimi nated by being forced to commit suicide by the-
husband or in-laws and incriminating evidence was usually available within the four
corners of the matrimonial home and hence was not available to anyone outside the
occupants of the house. However, still it cannot be lost sight of that the presump tion is
intended to operate against the ac cused in the field of criminal law. Before the
presumption may be raised, the foundation thereof must exist. A bare reading of Sec tion
113-A shows that to attract applicabil ity of Section 113-A, it must be shown that (i) the
woman has committed suicide, (ii) such suicide has been committed within a period of
seven years from the date of her marriage, (iii) the husband or his relatives, who are
charged had subjected her to cru elty. On existence and availability of the abovesaid
circumstances, the court may pre sume that such suicide had been abetted by her husband
or by such relatives of her husband. Parliament has chosen to sound a note of caution.
Firstly, the presumption is not mandatory; it is only permissive as the employment of
expression 'may presume' suggests. Secondly, the existence and avail-ability of the
abovesaid three circumstances shall not, like a formula, enable the pre sumption being
drawn; before the presump tion may be drawn the court shall have to have regard to 'all
the other circumstances of the case'. A consideration of all the other circumstances of the
case may strengthen the presumption or may dictate the con science of the court to
abstain from draw ing the presumption. The expression - 'the other circumstances of the
case' used in Section 113-A suggests the need to reach a cause-and-effect relationship
between the cruelty and the suicide for the purpose of raising a presumption. Last but not
the least, the presumption is not an irrebuttable one. In spite of a presumption having
been raised the evidence adduced in defence or the facts and circumstances otherwise
available on record may destroy the presumption. The phrase 'may presume', used in
Section 113-A is defined in Section 4 of the Evidence Act, which says - 'Whenever it is
provided by this Act that the court may presume a fact, it may either regard such fact as
proved, unless and until it is disproved, or may call for proof of it.' "

In State of W.B. v. Orilal Jaiswal, reported in (1994) 1 SCC 73, this Court observed :
1994 AIR SCW 844, Para 14

"15. We are not oblivious that in a crimi nal trial the degree of proof is stricter than what
is required in a civil proceedings. In a criminal trial however intriguing may be facts and
circumstances of the case, the charges made against the accused must be proved beyond
all reasonable doubts and the requirement of proof cannot lie in the realm of surmises and
conjectures. The require ment of proof beyond reasonable doubt does not stand altered
even after the introduc tion of Section 498-A, IPC and Section 113-A of the Indian
Evidence Act. Although, the court's conscience must be satisfied that the accused is not
held guilty when there are reasonable doubts about the complicity of the accused in
respect of the offences alleged, it should be borne in mind that there is no, absolute
standard for proof in a crimi nal trial and the question whether the charges made against
the accused have been proved" beyond all reasonable doubts must depend upon the facts
and circumstances
@page-SC3217
of the case and the quality of the evidences adduced in the case and the materials placed
on record. Lord Denning in Bater v. Bater (1950) 2 All ER 458 (All ER at p. 459) has
observed that the doubt must be of a rea sonable man and the standard adopted must be a
standard adopted by a reasonable and just man for coming to a conclusion consid ering
the particular subject-matter."
16. Having regard to the principles afore said, we may now advert to the fact of the
present case. Here is a case where the evi dence on record discloses that the deceased
wanted to be married in a literate family. She was not happy with the fact that her
husband was illiterate and also with the sta tus and condition of the family of her hus
band. She was also required to do some domestic work as the family was poor, for which
she was not happy. The deceased was of the view point that her life has been spoiled by
marrying appellant No. 1. The letter reflects the attitude of the in-laws of the deceased
towards the deceased. In the said letter there was no reference of any act or incident
whereby the appellants were alleged to have committed any wilful act or omission or
intentionally aided or instigated the deceased to commit suicide.
17. On such slender evidence, therefore, we are not persuaded to invoke the presump tion
under Section 113-A of the Evidence Act to find the appellant guilty of the offence under
Section 306 IPC.
18. The next question which remains for our consideration is whether an offence is made
out under section 498A of IPC. Though the letter allegedly written by the deceased
mentions the fact that the attitude of the family was not good towards the deceased and
she was not treated well but there is no mention about any of such incident. PW1 and
PW3 in their statements have empha sized that the mother-in-law of the deceased used to
ask the deceased to run hand driven flourmill to which she was not habitual. In the year
1988 when the abovementioned incident occurred, the hand driven flourmills were
generally used by women in the poor families in the villages and even till today one may
find use of the same in some vil lages in the country. Thus asking one to run the same at
that point of time may not amount to an act of cruelty.
19. In the said statements there is also a mention of an incident where the deceased had
been beaten by her husband. The mother-in-law (appellant No. 2) cannot be held liable
for the said act; rather there is evidence on record of PW3 who had stated that appellant
No. 2 had once restrained her son. Though in the statement of PW 1 there is mention of
one or two incident when the present appellant had beaten the deceased but there appears
to be possibility of em bellishment. The father of the deceased (PW2), in his statement
has not made any statement regarding cruelty being commit ted on his daughter in her in-
laws house. After analyzing the said evidence and the statements made by PW1 and PW3
we are of the opinion that the benefit of doubt should be granted to appellant No. 2.
20. We, therefore, set aside the convic tion under Sections 306 and 498A of the IPC
passed against the appellant No. 2 and acquit her granting her benefit of doubt. The
appeal is allowed in so far as appellant No. 2 is concerned. The appeal has abated in so
far as appellant No. 1 is concerned. The appellant No. 2 is already on bail. She is released
from the terms of her bail bonds.
Appeal allowed.
AIR 2008 SUPREME COURT 3217 "Raj Rajendra Singh Seth v. State of Jharkhand"
(From : Jharkhand)*
Coram : 2 Dr. A. PASAYAT AND S. H. KAPADIA, JJ.
Criminal Appeal No. 1135 of 2008 (aris ing out of SLP (Cri.) No. 1728 of 2004), D/- 22
-7 -2008.
Raj Rajendra Singh Seth v. State of Jharkhand and Anr.
Prevention of Corruption Act (2 of 1947), S.5 - CORRUPTION - EVIDENCE - Demand
and acceptance of bribe - Evidence clearly showing that appellant, accused had asked to
pass money to "N" - In presence of indepen dent witnesses money was delivered to 'N' -
'N' in turn handed over money to accused - Currency notes recovered from accused after
following requisite formali ties - Thus, all requisites for proving demand and acceptance
of bribe by ac cused established - Conviction of ac cused - No interference. (Paras
10, 11)
Cases Referred : Chronological Paras
(2006) 12 SCC 277 (Ref.) 9
@page-SC3218

2000 AIR SCW 4018 : AIR 2001 SC 147 : 2001 Cri LJ 175 (Ref.) 9
2000 AIR SCW 4427 : AIR 2001 SC 318 : 2001 Cri LJ 515 (Ref.) 9
(2000) 9 SCC 752 (Ref.) 9
C.D. Singh and Merusagar Samantaray, for Appellant; Rajiv Dutta, Sr. Advocate, Saket
Singh and P. Parmeswaran, with him (for No.2) and B.B. Singh, (for No. 1) for Re
spondents.
* Cri. Appeal No. 7 of 1998 (R), D/- 10-9-2003 (Jhar)
Judgement
Dr. ARIJIT PASATAT, J. :- Leave granted.
2. Challenge in this appeal is to the judg ment rendered by a learned Single Judge of the
Jharkhand High Court. Two appeals were filed by the present appellant and one Nag
Narain who was accused No.2 question ing correctness of the judgment dated 4th
December, 1997 and order of sentence dated 16.12.1997 passed by the Special Judge,
CBI, Ranchi in R.C. case No. 15 of 1998. Learned Special Judge held the appellants
guilty of offence punishable under Sections 120B and 161 of the Indian Penal Code, 1860
(in short the 'IPC') and also under Sec tion 5(2) read with Section 5(1)(d) of the Pre
vention of Corruption Act, 1947 (in short the 'Act'). Each of them was sentenced to un
dergo RI for one year and to pay a fine of Rs.5,00/- with default stipulation.
3. Prosecution version as unfolded dur ing trial is as follows:
A written complaint was made to S.P. CBI, Dhanbad on 1.9.1985 by one Raju Hadi,
Safayi Mazdoor of Pathological Laboratory Area-9, BCCL, Dhanbad alleging therein that
he had visited Chamodih Dispensary in connection with treatment of his father Sri Hublal
Hadi who was examined by Dr. L.B. Sah, who referred him to Central Hospital,
Dhanbad. Hublal Hadi was admitted in Bed No. 16 ENT Department of Central Hospital
on 29.8.1985. Raju Hadi had been to the hospital on 31.8.1985 to see his ailing fa ther
and his ailing father complained of lack of proper treatment and he requested him to meet
the concerned doctor, Raju Hadi as certained that his father was under the treat ment of
Dr. R.R.S. Seth, the appellant. He requested Nag Narain to allow him to meet Dr. R.R.S.
Seth and met Dr. R.R.S. Seth, who demanded a sum of Rs.500/- from him for giving
proper medical treatment to his fa ther and also insisted that the amount be paid on
1.9.1985. The doctor also told Raju Hadi that in case he was not available in the hospital,
he would pay the amount to his ward boy Nag Narain, who would pass the amount to
him. Since Raju Hadi was not willing to make the payment of bribe amount to the doctor
and ward boy, he lodged a com plaint to the S.P. CBI, Dhanbad for taking necessary
action. On the basis of complaint, verification was made and on getting con firmation
report, Sri R.C. Choudhary, In spector, registered the complaint on 1.9.1985 and took up
the investigation. The I.O. obtained the services of the two inde pendent witnesses Devraj
Prasad Sinha (PW-2) and Ved Prakash Pahuja (PW-1). These two independent witnesses
reported before Shri R.C. Choudhary in the office of the CBI. Thereafter members of the
CBI formed a raiding party and this party also assembled before him. After formal
introduction of each other, the purpose of assembly was ex plained and practical
demonstration regard ing the purpose and use of phenolphthalein powder and chemical
reaction with sodium carbonate was given in the immediate pres ence of two independent
witnesses and the members of raiding party. After demonstra tion was over, the informant
Raju Hadi was asked to produce a sum of Rs.500/- (five G.C.C. notes of rupees one
hundred denomi nation each) and he produced the same and numbers of these notes were
noted down and these. G.C.C. notes were tainted with phenolphthalein powder and
handed over to Raju Hadi. The informant was directed to pay the amount to the accused
only on de mand. Instructions were also issued to the witnesses and the members of the
raiding parry to play their respective parts before and after trap. All these practical demon
strations were noted down and demonstra tion chart was prepared on which all the
members of the raiding party made their respective signatures. After pre-trap for malities,
PW3 and other members of the team including independent witnesses proceeded towards
Central Hospital and PW2 was directed to shadow PW3 and to hear conversation in
between the PW3 and the appellants. When they reached Central Hos pital and went to
the chamber of appellant Dr. Seth, chamber was found locked but informant met other
appellant Nag Narain and PW3 paid the tainted money amount ing to Rs.500/- to Nag
Narain who kept the same in his right pocket of his shirt and asked PW3 to proceed with
him to the resi dence of Dr. Seth as he will give money in
@page-SC3219
his presence and PW3 appellant Nag Narain proceeded from Central Hospital to the
residence of Dr. Seth and PW2 and other mem bers of the team were following them.
When PW3 remained near the gate, other persons of the team remained outside the gate.
On reaching burand of the house, appellant Nag Narain pressed call bell whereupon appel
lant Dr. Seth opened the door and came out and he gave money to him. In the mean time,
PW2 who saw this came out of the gate and gave signal and thereafter members of the
team pounced upon them introducing themselves as CBI officials and they caught Dr.
Seth and recovered money from his pos session, Nag Narain was also caught. There after
right hand of Dr. Seth was dipped in a solution which turned pink and this solu tion was
kept in a bottle and sealed. Simi larly, left hand of Dr. Seth was also dipped in another
solution which also turned pink and this solution was also kept in a sepa rate bottle and
sealed. At the same time, right hand of appellant Nag Narain was dipped in similar
solution which also turned pink and was kept in a bottle and sealed. Similarly left hand of
Nag Narain was also dipped in another solution which also turned pink and this solution
was also kept in a bottle and sealed. His shirt was also dipped in a solution and that
solution turned pink and that solution was kept in a bottle and sealed. Thereafter members
of the team put their respective signatures on all the bottles which were sealed. Thereafter
post-trap formalities were carried out at the pre mises of Dr. Seth, upon which all the
mem bers of the team put their respective signa tures. Both Nag Narain and Dr. Seth were
later arrested soon after recovery of money. After investigation of the case charge sheet in
the case was submitted and cognizance of the case was taken and learned court below in
course of trial recorded evidence of witnesses of both sides and marked exhib its of
documents produced on behalf of both sides and ultimately came to a conclusion and held
both the appellants guilty and accordingly, convicted them and sentenced them.
4. The two accused persons filed appeals before the High Court. Their stand was that
there are a lot of contradictions in the evi dence of witnesses. It was submitted that
everything was pre-planned and conspiracy was hatched to falsely implicate the appel
lant. It was highlighted that so much preparations were made before trap, but it is not
clear as to who recovered the money from the hands of the appellant. It was stated that
PW8 was the brain behind the so-called trap.
5. After considering the rival stands the High Court held that the accusations were
established but considering passage of time reduced the sentence to the period already
undergone.
6. In support of the appeal learned coun sel for the appellant submitted that both the Trial
Court as well as the High Court lost sight of the following features:
(1) No demand was established;
(2) Role of PW-8 is highly suspicious and his evidence lacks of
(3) There were no independent witnesses;
(4) There was no positive evidence as to who had recovered the money as claimed by the
prosecution from the appellant.
7. Learned counsel for the respondent, on the other hand submitted that both the Trial
Court as well as the High Court have analyzed the evidence in great detail and there is no
infirmity in the impugned judg ment.
8. Much has been made of the fact that most of the witnesses were in the same of fice.
The evidence is to the effect that the appellant had asked PW-3 to pay money to co-
accused Nag Narain who was to pass the money to him. PW-2 in his evidence has
categorically stated that the decision was taken in CBI office that money is to be paid to
Nag Narain who has made payment to the accused. Similarly, PW-10 while mak ing
verification about the genuineness of the allegations made by PW-3 has stated that he
went to the residence of the appellant and he hid himself behind the bush and from there
he heard talks between PW-3 and ap pellant. He has stated that the appellant asked PW-3
to make payment to Nag Narain. PW-3 corroborated this part of the statement of PW-10
who is a constable. He was en trusted with the job to verify the genuine ness of the
allegations made by PW-3. He went to his chamber and Nag Narain was present there.
PWs 1 and 2 were indepen dent witnesses and in their presence money was delivered to
Nag Narain by PW-3. This was done because when PW-3 and others reached at the
hospital, the chamber was found locked. PW-3 met Nag Narain and paid money to him
and proceeded to residence
@page-SC3220
of the appellant. After reaching there PW-3 and Nag Narain went inside the gate and PW-
2 and others remained at the gate. It is clear from the evidence that the appellant came out
after the call bell was pressed and Nag Narain passed the money to him. PW-2 who saw
passing of money to the appellant, gave a signal and immediately thereafter Nag Narain
and the appellant were arrested and money was recovered from the right hand of the
appellant and both the hands of the accused persons were washed in separate solution and
they turned pink. The currency notes were also recovered and the requisite formalities
were followed. The plea that there is no demand made by the appellant is clearly belied
by the evidence on record. The evidence clearly establishes that the appellant had asked
the money to be passed on to Nag Narain who in turn handed over the money to the
appellant.
9. In B. Noha v. State of Kerala and Anr. (2006 (12) SCC 277) it was, inter alia, ob served
by this Court as follows:

"10. The evidence shows that when PW-1 told the accused that he had brought the money
as directed by the accused, the ac cused asked PW-1 to take out and give the same to him.
When it is proved that there was voluntary and conscious acceptance of the money, there
is no further burden cast on the prosecution to prove by direct evi dence, the demand or
motive. It has only to be deduced from the facts and circum stances obtained in the
particular case. It was held by this Court in Madhukar Bhaskarrao Joshi v. State of
Maharashtra (2000 (8) SCC 571) as follows : 2000 AIR SCW 4018

"12. The premise to be established on the facts for drawing the presumption is that there
was payment or acceptance of gratifi cation. Once the said premise is established the
inference to be drawn is that the said gratification was accepted 'as motive or re ward' for
doing or forbearing to do any offi cial act. So the word 'gratification' need not be
stretched to mean reward because reward is the outcome of the presumption which the
court has to draw on the factual pre mises that there was payment of gratifica tion. This
will again be fortified by looking at the collocation of two expressions adja cent to each
other like 'gratification or any valuable thing'. If acceptance of any valu able thing can
help to draw the presump tion that it was accepted as motive or reward for doing or
forbearing to do an official act, the word 'gratification' must be treated in the context to
mean any payment for giv ing satisfaction to the public servant who received it."

11. This decision was followed by this Court in M. Narsinga Rao v. State of A.P. (2001
(1) SCC 691). There is no case of the accused that the said amount was received by him
as the amount which he was legally entitled to re ceive or collect from PW-1. It was held
in the decision in State of A.P. v. Kommaraju Gopala Krishna Murthy (2000 (9) SCC
752), that when amount is found to have been passed to the public servant the burden is
on public servant to establish that it was not by way of illegal gratification. That bur den
was not discharged by the accused." 2000 AIR SCW 4427

10. In the case at hand all the requisites for proving the demand and acceptance of bribe
have been established.
11. There is, therefore, no merit in this appeal which is accordingly dismissed.
Appeal dismissed.
AIR 2008 SUPREME COURT 3220 "Banwar Lal v. G. Kalavathi"
(From : Andhra Pradesh)
Coram : 3 B. N. AGRAWAL, ALTAMAS KABIR AND G. S. SINGHVI, JJ.
Civil Appeal Nos. 2200-2201 with 2202 of 2001, D/- 16 -4 -2008.
Banwar Lal and Ors. v. G. Kalavathi (Dead) by L. Rs and Ors.
And
G. Kalavathi v. Banwar Lal and Ors.
(A) Contempt of Courts Act (70 of 1971), S.2(b) - CONTEMPT OF COURT -
Disobedience of order - Direction to maintain status quo for three months passed by
Court - Nothing to indicate that status quo order was ex tended - Fact that construction
work was being carried on land in question in violation of status quo order, during pe riod
when status quo order was in force - Not established - Contempt petition filed almost four
years after order of sta tus quo ceased to operate - Appellants not guilty of contempt.
Contempt Appln. No. 1563 of 1999 and Contempt Case No. 1819 of 1999, D/-19-01-
2001 (A.P.), Reversed. (Para 26)
(B) A.P. Land Encroachment Act (3 of 1905), S.6 - ENCROACHMENT - HIGH COURT
-
@page-SC3221
Notice to vacate premises - Issued by Revenue Officers to respondent alleging that land
was Govt. land - Notice quashed in writ petition - Order of status quo in writ appeal -
However, land utilised for construction of minister's quarter by appellant - Direc tion for
giving vacant possession to respondents cannot be given to appellants - Appellants
themselves assessing value of land to be more than Rs. 20,000/- per square yard - Order
of High Court grant ing compensation to respondents @ Rs. 5000/- per sq. yard not
proper - Compensation enhanced to Rs. 15,000/- per sq. yard. (Paras 29, 32)

Anoop G. Chaudhary and R.F. Nariman, Sr. Advocates; Manoj Saxena, Rajneesh Kumar,
Singh, Rahul Shukla, T.V. George and A. Subba Rao, with them for the Ap pearing
Parties.
Judgement
ALTAMAS KABIR, J. :- These appeals by way of special leave are directed against the
judgment and order dated 19th Janu ary, 2001, passed by the Andhra Pradesh High Court
in Contempt Application No.1563 of 1999 and Contempt Case No. 1819 of 1999, holding
that deliberate and wilful acts of disobedience of the orders passed by the High Court, by
the Mandal Revenue Of ficer arid also by the other superior authori ties had been amply
proved and as such they had committed contempt of Court. The High Court then went on
to observe as follows :-
"Viewed from any angle the respondents clearly disobeyed the orders of this Court and
they committed contempt of Court. It is no doubt true that construction over the land in
question has been taken place. But, what is the relief that has to be granted to the
petitioner? No purpose would be served to the petitioner if there is a direction to
handover vacant possession of the property of the petitioner. The order of demolition of
the buildings so constructed over the land in question and restituting the said land to the
petitioner would cause great hardships to the respondents.
To put an end to the litigation once for all and since the respondents have already made
constructions over the land in ques tion by flouting the orders passed by this Court and no
purpose will be served if the respondents are directed to be punished as some of them as
on today either transferred or retired from service, we feel it appropriate to direct the
respondent-authorities to pay compensation to the petitioner in the contempt case at the
rate of Rs.5,000/- per sq. yard as valued by the State itself in the application filed u/S.8 of
the A.P. Land Grab bing (Prohibition) Act and numbered as L.G.C. No. 25 of 2000 or any
other reason able amount in lieu of directing the respon dents to redeliver possession of
the land in question to the petitioner, in case the pro ceedings in L.G.C. No. 25 of 2000
go in favour of the petitioner. The question of maintainability, jurisdiction, propriety or
competency to continue the case in L.G.C. No. 25 of 2000 in view of the earlier pro
ceedings and observations made therein in cluding the directions to institute a civil suit to
adjudicate the title to the property is kept open to be considered by the Special Court.
However, implementation of this order is stayed for a period of six weeks from today.
Subject to the observations indicated above, the contempt case and the contempt
applications are closed. There shall be no order as to costs."
2. It is clear from the aforesaid order that instead of directing demolition of the
construction already made and/or punish ing the contemnors, the High Court felt it more
appropriate to pass an equitable or der directing payment of compensation to the
petitioner in the Contempt Case at the rate of Rs.5,000/- per sq. yard as valued by the
State itself in an application filed under Section 8 of the Andhra Pradesh Land Grab bing
(Prohibition) Act, against the predeces sor-in-interest of the respondent herein. The said
direction was, however, given along with a rider that such compensation would be paid
only in the event the land grabbing case was dismissed in favour of the predecessor-in-
interest of the respondent herein. Cer tain other directions were also given regard ing the
question of maintainability, jurisdic tion, propriety or competence of the petition ers in
the land grabbing case to continue the same in view of earlier directions given by the
High Court, including the direction to institute a civil suit to adjudicate the title to the
property. The implementation of the order was stayed for a period of six weeks from the
date of the order presumably with the intention of allowing the parties to con test the
order before a higher forum.
3. It is the said order of the High Court which is the subject matter of challenge in these
appeals.
@page-SC3222
4. While Civil Appeal Nos.2200-2201 of 2001 have been filed by the officials found by
the High Court to have committed con tempt of Court, Civil Appeal No.2202 of 2001 has
been filed by the writ petitioner, Smt. G. Kalavathi, being aggrieved by the amount of
compensation fixed by the High Court at the rate of Rs.5,000/- per square yard.
5. In order to appreciate the submissions advanced on behalf of respective parties and the
circumstances in which the impugned order came to be passed, a brief outline of the facts
leading to the passing of the said order is set out hereinbelow.
6. The predecessor-in-interest of the present respondents, one Smt. G. Kalavathi, came to
be the absolute owner and possessor of 10 acres of land in Survey No. 129/36/1, being
part of Survey No.403 of Shaikpet Vil lage, Road No. 13, Banjara Hills, Hyderabad. It
was her case before the High Court that she had purchased the said property under a
registered sale deed dated 11th February, 1971, from one Mr. Papaiah, who had pur
chased the same by a registered sale deed dated 28th February, 1963, from one Venkat
Swamy. Venkat Swamy is said to have ac quired the property by a patta issued by the
concerned authorities in the year 1340 Fasli, equivalent to the year 1930 of the Christian
era. From the description of the property contained in the order impugned in this appeal,
the said property is bounded on the North by a Nala (Government land); on the South by
Road No. 13; on the East by land in Survey No. 129/75 belonging to M/s. Hyderabad
Industries Limited and in the West by the land belonging to Kazi Mohd. Hussain in
Survey No. 129/36.
7. The respondent No. 1 herein leased out the said land measuring ten acres to M/s.
Hyderabad Industries Limited by a regis tered lease deed dated 27th June, 1971. The
Mandal Revenue Officer, Golconda Mandal, issued a notice to the respondent No. 1 on
20th June, 1986 under Section 6 of the A.P. Land Encroachment Act, 1905, calling upon
the said respondent to vacate the land within 3 days from the date of receipt of the notice.
Challenging the said notice, the respondent filed Writ Petition No. 11714 of 1986 for a
declaration that the action of the Mandal Revenue Officer in issuing the notice was
Illegal, arbitrary and unconstitutional and also praying that the said officer be re strained
from interfering with her posses sion over the land. The said writ petition was allowed
and the notices issued by the Mandal Revenue Officer under Sections 6 and 7 of the A.P.
Land Encroachment Act was quashed with leave to the Government to file a suit to get its
title established.
8. The writ appeal filed by the said Gov ernment through the Mandal Revenue Of ficer,
Golconda, Hyderabad, was also dis missed and status quo was directed to be maintained
for a period of three months from the date of the judgment, namely, 14th Sep tember,
1995, to enable the Government to take appropriate steps. In spite of the time granted to
the State Government to take appropriate steps in the matter, no steps were taken by the
Government to establish its title over the land in question by filing a civil suit. On the
other hand, it has been alleged on behalf of the respondent No. 1 that as soon as she came
to learn about the con struction being carried on by the appellants herein over the land in
question, she caused telegraphic notices to be sent to the appel lant on 20th November,
1999 and 23rd No vember, 1999, together with a registered le gal notice issued on 26th
November, 1999, calling upon the appellants herein to stop the construction work on the
land. As the appellant did not stop the construction work purportedly in disobedience of
the orders passed by the High Court, the respondent No. 1 was compelled to file a
contempt peti tion. On this aspect of the matter, it has been observed in the impugned
judgment that the State initiated proceedings against the re spondent No. 1 under Section
8 of the A.P. Land Grabbing (Prohibition) Act, 1982, in December, 1999, as a counter-
blast to the contempt case alleging that the land in ques tion which was under the
occupation of the respondent No.1 was Government land.
9. It is on the basis of the aforesaid facts as disclosed before it that the Division Bench
came to the conclusion that the authorities started construction even after the orders
passed by the High Court in the writ peti tion without first approaching the civil court for
declaration of its title. The Division Bench also observed that the said act of the appel
lants herein amply prove that there was a deliberate and wilful act of disobedience of the
orders of the said Court by the Mandal Revenue Officer and other superior authori ties
and as such they had committed con tempt of court. It is thereafter that the ob servations
are made relating to payment of compensation, which have been set out
@page-SC3223
hereinbefore.
10. Appearing in support of the appeal, Mr. Anoop Chaudhary, Senior Advocate, urged
that both the Single Judge and also the Division Bench of the High Court had completely
failed to take notice of the fact that the land on which the construction work was carried
on was not the land which the respondent No. 1 had purportedly acquired from Papaiah
and that the same had been leased out to M/s.. Hyderabad Industries Limited. Since the
said land was under the occupation of M/s. Hyderabad Industries Limited, the question of
making construc tions on the said land did not arise.
11. Mr. Chaudhary then submitted that in the contempt proceedings, the High Court had
appointed a Commission consisting of high-level officers of the Survey and Land
Records Department, Hyderabad, to conduct a survey of the land in the presence of the
Advocate-Commissioners representing the respondent No. 1, who was the petitioner in
the contempt case, along with his juniors. The Commissioners were directed to visit the
disputed property and to report on the fol lowing items:-
(1) They shall ascertain the total extent of land covered by the map.
(2) They shall measure the total extent of land occupied and the extent of land occu pied
by Birla Guest House belonging to M/ s.. Hyderabad Industries Limited.
(3) They shall measure the extent of land that is shown in the map with brown tines
indicating boundaries and whether any part thereof form part of Birla Guest House.
(4) What is the extent of land that is un der the occupation of the Government and being
used for construction of Minister's quarters.
12. In the report submitted by the Com missioners, it has been stated that the ref erence
map furnished by the respondent No. 1 herein to the Advocate Commissioners and passed
on by them to the team of Sur vey Officers, does not tally with the record of measurement
of Block 'S' in ward No. 11 and that having regard to the said factor, the Commissioners
had chosen to follow the actual state of things on the ground. After conducting a survey
as directed by the High Court in the presence of the Advocate Com missioners of the
respondent No. 1 herein, the Commissioners observed that the total extent of land
covered by the map was 31 acres and 20 guntas. It was also pointed out that the land
covered by the green-coloured boundary in the reference map was identi cal with
reference to Road No. 13, Banjara Hills and the compound walls and it was found that the
area consists of Ministers' quarters constructed by the Government and also the land
under the occupation of Birla Guest House which has a separate compound wall. As far
as the land covered by Birla Guest House built by Hyderabad Industries Limited is
concerned, the same measured 13 acres and 35 guntas and had a pucca compound wall on
all the sides. It is also stated that on verification of the cop ies of the title deeds of M/s..
Hyderabad In dustries Limited it was found that the com pany had purchased a total
extent of 13 acres and 13 guntas under three documents, which, in fact, meant that the
said company was in excess possession of land to an ex tent of 22 guntas of land, which
could not be demarcated exactly because the position on the ground was different from
the bound ary shown in the plan annexed to the title deeds.
On examination of the sale deeds belong ing to the petitioners and the lease deed
executed by Smt. G. Kalavathi in favour of M/s. Hyderabad Industries Limited and the
plan annexed to the said documents, it was observed that the same were found to be
inconsistent and not in conformity with the area shown in the reference map with brown
lines. However, although, it was not possible to locate exactly the said area on the
ground, on actual measurement the area in the ref erence map was found to consist of 9
acres 32 guntas.
The total extent of land under the occu pation of the Government and being used entirely
for construction of Ministers' quar ters was found to be measuring 17 acres and 25 guntas
and it was also indicated that it was bounded on all sides by a pucca wall, except on the
north, where in a portion there was a rubble wall.
Mr. Chaudhary submitted that having regard to the report of the Commissioners, it would
be evident that the land of the re spondent No. 1 could not be correctly iden tified in
relation to the map of the site plan annexed to her title deeds and also the lease deed
executed by her. In any event, the land under the occupation of M/s. Hyderabad
Industries Limited, measuring 13 acres and 35 guntas was bounded on all sides by a
@page-SC3224
pucca compound wall.
13. According to Mr. Chaudhary, the land leased by the respondent No. 1 to the company
would be within the compound wall of the company, and, accordingly, the question of
making any construction there upon, which forms the basis of the complaint petition, did
not arise and both the Single Judge and the Division Bench of the High Court erred in
holding otherwise.
14. In addition to the above, Mr. Chaudhary submitted that the High Court in its contempt
jurisdiction had adopted the mantle of Collector within the meaning of the Land
Acquisition Act, 1894, and had thereby exceeded its jurisdiction in assess ing
compensation to be paid in respect of the lands belonging to the respondent No. 1. Mr.
Chaudhary, submitted that the orders both of the Single Judge and the Division Bench of
the High Court stood vitiated when it travelled beyond the scope of its contempt
jurisdiction.
15. Mr. Chaudhary lastly submitted that there was no subsisting order of re straint or any
direction given by the High Court, which if violated, would have at tracted the provisions
relating to contempt of Court. However, in the absence of such injunction or direction,
the High Court had wrongly exercised its contempt jurisdiction and had also travelled
beyond the same, which renders the orders of the High Court vulnerable and liable to be
set aside.
16. Appearing in support of the appeal preferred by Smt. G. Kalavathi and oppos ing
those filed by the contemnors, Mr. R.F. Nariman, learned senior counsel, submit ted that
the basic premise of the case made out on behalf of the appellants in Civil Ap peal
Nos.2200-2201 of 2001 was fallacious on account of mistaken identity of the plot
belonging to the appellants in Civil Appeal No. 2202 of 2001. Mr. Nariman submitted
that the said position would, in fact, be very clear from the counter affidavit filed by the
Mandal Revenue Officer, Golconda, in Writ Petition No. 11714 of 1986 which had been
filed by Smt. G. Kalavathi and out of which the contempt proceedings had arisen. The
learned counsel pointed out that in paragraph 2 of the counter-affidavit the Mandal
Revenue Officer, Golconda, had stated that since Survey No.403 covered the entire area
popularly known as Banjara Hills and Ju bilee Hills area had become a paradise for
speculators, the Government had appointed a Special Gr. Dy. Collector, in 1982, for iden
tification and demarcation of Government and private lands in the Banjara Hills area. It
was also stated that during the survey it had come to light that M/s. Hyderabad As bestos
Limited, who were the successors to M/s. Hyderabad Industries Limited, had occupied a
portion of Survey No.403 of Shaikpet village and had constructed a com pound wall. It
was also stated that the in quiry conducted by the aforesaid official in respect of the
Banjara Hills area reveal that the area occupied by M/s. Hyderabad As bestos Limited,
comprises of the following lands:-
1. Land purchased by M/s. Hyderabad Asbestos Limited, from Sri Md. Azam in Sy. No.
129/75 of Shaikpet Village vide docu ment No.55/62, dated 11.1.1962.
2. Land purchased by M/s. Hyderabad Asbestos Limited from Fazal Nawaz Jung in
Jubilee Hills Municipality Plot No.129/75/ D1 vide document No.93, dated 29.5.1963.
3. Land purchased by M/s. Hyderabad Asbestos Limited from Fazal Nawaz Jung in
Jubilee Hills Municipality Plot No.129/75/ D1 under as agreement of Sale, dated
17.9.1963 (outside compound).
4. Land occupied by M/s. Hyderabad Asbestos Limited from Smt. G. Kalavathi, W/o
Venkataswamy through a Lease Deed No.2021/72, dated 28.6.1971 in Sy. No. 129/36/1.
5. Land occupied by M/s. Hyderabad Asbestos Limited without any authority in Sy. No.
403.
17. In relation to the above it was also indicated that the purchases made in re spect of
serial Nos. 1 to 3 indicated above were from the original owners of the land, whereas the
land measuring 19 acres and 3 guntas, indicated in items 4 and 5, which had been
occupied by the company, was Government land in Survey No.403 of Shaikpet village
which had been occupied by the said company without any authority in law. It was sought
to be justified that since there were no certified records nor any rev enue records to show
the right, title and in terest of the petitioner in Sub-Division No. 129/36/1 in the record of
Shaikpet vil lage, and the same was found to be non existent under the provisions of the
A.P. Land Encroachment Act, 1905, resumption proceedings were initiated.
@page-SC3225
18. Mr. Nariman pointed out that as would appear from the Report of the Survey
Commissioner appointed by the Court an area measuring 13 acres 13 guntas was found to
have been purchased by M/s. Hyderabad Industries Limited from others, which area had
been enclosed by a com pound wall on all sides. However, from the counter affidavit
filed by the Mandal Rev enue Officer in the writ petition it was evi dent that apart from
the above land which was within their boundary walls the com pany was also in
occupation of 10 acres of land which had been leased to it by Smt. G.Kalavathi by a lease
deed dated 28.6.1971 in Survey No. 129/36/1. Another plot of land measuring 9 acres and
3 guntas were also shown to be in the occupation of M/s. Hyderabad Industries Limited
without any authority to possess the same. Mr. Nariman pointed out that the
measurements dis closed would indicate that the area covered by the three sale deeds in
favour of the com pany measuring about 13 acres and 13 guntas had been encircled by a
boundary wall. Apart from the above, a further 19 acres was also under the occupation of
the said company which included the 10 acres which had been leased by Smt. G.
Kalavathi in 1971. Mr. Nariman submitted that it was this 19 acres which was the subject
matter of the notice issued by the Mandal Revenue Officer, Golconda, on 20.6.1986
under Sec tion 6 of the A.P. Land Encroachment Act, 1905, which had been challenged in
Writ Petition No. 11714 of 1986 and had been quashed in the said proceeding.
19. Mr. Nariman urged that the said af fidavit of the Mandal Revenue Officer in jux
taposition with the report of the Survey Com missioner appointed by the High Court,
would at once reveal that the land which was leased by Smt. G. Kalavathi to the com
pany was outside the boundary wall which had been erected by the company around the
lands acquired by it through 3 separate sale deeds. It was urged that the High Court had
rightly found that the construction work which was being undertaken for construc tion of
Ministers' quarters, was also being carried on in the 10 acres of land which had been
leased by Smt. G. Kalavathi to M/s. Hyderabad Industries Limited.
20. Mr. Nariman submitted that when the land in question had been settled with Vankata
Swamy by a patta, the said docu ment contained a condition that Revenue for the said
land would have to be paid @ Rs. 5 per acre before construction of houses and thereafter
at Rs.10 per acre after construc tion of houses. According to Mr. Nariman, although the
claim of Papaiah in respect of the title over the land had been rejected in 1971, the
Government did not take any fur ther steps in the matter and from 1971 Smt. G.
Kalavathi remained in open possession directly and through M/s. Hyderabad Indus tries
Limited as her lessee. Mr. Nariman pointed out that it is only after the order was passed
under Section 6 of the A.P. Land Encroachment Act, 1905, that the Govern ment, in
accordance with its own case, took a decision in 1986. In this regard Mr. Nariman also
pointed out that although Papaiah's claim was purported to have been rejected by the
Government in 1971 there were demand notices in 1974, 1975 and 1976 which had been
complied with by the writ petitioners. Mr. Nariman submitted that having regard to the
aforesaid facts there was a genuine dispute regarding title and the writ petitioner could
not have been evicted from the land except in due process of law. Mr. Nariman also urged
that once the notice under Section 6 referred to above was quashed by the High Court the
peti tioner continued to enjoy absolute rights as the owner of the said property. Mr.
Nariman submitted that having failed in their at tempts to evict the writ petitioner from
the land by the aforesaid means, the authori ties filed a complaint against the writ peti
tioner under the provisions of the A.P. Land Grabbing (Prohibition) Act, 1981. Mr.
Nariman urged that the High Court had quite rightly decided the contempt petition, but
had erroneously directed that compensation be paid to the petitioner, in the event the land
grabbing case was dismissed, at the rate of Rs.5,000/- per square yard, when the
appellants themselves had admitted in their affidavit that the value of the lands in
question would be more than Rs. 20,000 per square yard at the relevant point of time. Mr.
Nariman also submitted that the High Court had erred in restricting payment of
compensation till after the decision in the land grabbing case, since it had found that the
same had been filed only as a counter blast to the contempt petition filed by the writ
petitioner.
21. Mr. Nariman submitted that there was no ground to interfere with the findings of the
High Court or the decision arrived at
@page-SC3226
therein in the contempt proceedings, but the compensation computed was liable to be
reconsidered in the light of the admission made by the appellants themselves in Civil
Appeal Nos. 2200-01 of 2001.
22. Since this appeal arises out of or ders passed by the High Court on the con tempt
petition filed on behalf of respondents in Contempt Application No. 1563 of 1999 and CC
No. 1819/1999, it would be in the fitness of things to first of all consider whether the
High Court was justified in hold ing that the appellants had violated the or der dated
14.9.1995 passed by the Division Bench in Writ Appeal No.742 of 1989 filed by the
Mandal Revenue Officer, Golconda Mandal, Hyderabad.
23. As has been indicated hereinbefore the respondents herein challenged the no tice
dated 20.6.1986 issued by the Mandal Revenue Officer, Golconda Mandal, Hyderabad,
under Section 6 of the A.P. Land Encroachment Act, 1905 and the learned single Judge
quashed the same for the rea sons indicated in his judgment and order dated 14.10.1988.
24. In the Writ Appeal filed by the Mandal Revenue Officer, Golconda Mandal,
Hyderabad, the Division Bench of the High Court did not interfere with the judgment and
order of the learned Single Judge upon holding that the learned Single Judge had rightly
allowed the writ petition. However, while dismissing the writ appeal the Divi sion Bench
directed the status quo to be maintained for a period of three months from the date of the
order to enable the Govern ment to take appropriate steps in the mat ter.
25. It is in respect of this order that the contempt petition was filed and the Division
Bench also arrived at a finding that the ap pellant herein had committed contempt of
Court by violating the said order.
26. We are unable to agree with the rea soning of the Division Bench in the im pugned
judgment in holding that the appel lants had committed contempt of the said order dated
14.9.1995. There is no ambigu ity that while the direction to maintain sta tus quo for three
months were given on 14.9.1995, the contempt petition was filed only in October, 1999.
The period for which status quo was directed to be maintained came to an end on
14.12.1995 and there is nothing on record to indicate that the order of status quo was ever
extended. The fact that construction was going on in the land in question came to the
knowledge of the respondents herein on 19.11.1999 when Smt. G. Kalavathi visited the
property and thereafter steps were set in motion for filing of the contempt petition. Since
there is noth ing to indicate whether the construction work was being carried on during
the pe riod when the order of status quo was in force and the contempt petition was filed
almost four years after the order of status quo ceased to operate, it is difficult for us to
appreciate as to how the Division Bench of the High Court could have found the appel
lants herein to have committed contempt of Court. The findings of the High Court in re
spect of the same must be held to be con trary to the materials on record.
27. This now brings us to the other limb of the order impugned in these appeals.
28. From the facts as stated herein-above it is clear that M/s. Hyderabad In dustries
Limited was in possession of five different plots of land, out of which three plots
measuring 13 acres and 13 guntas had been purchased by the said company. Of the
remaining two plots, the said company was also in occupation of 10 acres of land which
had been leased to it by Smt. G. Kalavathi. It is also apparent that the lands enclosed
within the boundary wall of the guest house of M/s. Hyderabad Industries Limited were
the lands forming the subject matter of the three sale deeds. In other words, the land
leased to the company by Smt. G. Kalavathi was part of the 19 acres of land which was
the subject matter of the notice issued by the Mandal Revenue Officer, Golconda Mandal,
Hyderabad, on 20.6.1986 under Section 6 of the Andhra Pradesh Land En croachment
Act, 1905, which was quashed in Writ Petition No. 11714 of 1986 and had been utilised
for constructing the ministers' quarters.
29. From the submissions made and also from the findings in the impugned judg ment it
is also clear that the said 19 acres of land have already been utilised for con struction of
ministers' quarters and as rightly pointed out by the High Court would cause great
hardship to the respondents in the event a direction was given to them to restore the land
to the respondents.
30. In the other appeal, being Civil Ap peal No.2202 of 2001, filed by Smt. G. Kalavathi,
one of the questions raised is whether the High Court was justified in directing
@page-SC3227
payment of compensation at the rate of Rs.5000 per square yard subject to the condition
that the same be paid after the determination of the land grabbing case filed by the State
against the respondents under the provisions of Andhra Pradesh Land Grabbing
Prohibition Act, 1982. It has been urged by Mr. Nariman that the appellants had
themselves assessed the value of the lands in question to be more than Rs.20,000/- per
square yard in Ground 'S' of their Special Leave Petition filed in 2001 and hence the
amount decided as compensation by the High Court was not commen surate with the
value assessed by the appellants themselves, and should therefore, be increased.
31. Considering the ground realities and the assessment made by the State authori ties
themselves, there appears to be some substance in Mr. Nariman's submissions. We,
therefore, allow the appeals in part and set aside the findings of the Courts below
regarding contempt of Court alleged to have been committed by the appellants herein. We
hold that the appellants are not guilty of contempt of Court as has been alleged.
32. We also direct that the compensa tion awarded by the Division Bench in the judgment
under appeal, should be suitably increased in view of the assessment made by the State
authorities themselves. We ac cordingly, enhance the compensation awarded by the
Division Bench of the High Court from Rs.5,000/- per square yard to Rs.15,000/- per
square yard. We make it clear that we are not interfering with the other orders and
directions given in the im pugned judgment regarding payment of such compensation.
33. The appeals are disposed of accord ingly. There will be no order as to costs.
Order accordingly.
AIR 2008 SUPREME COURT 3227 "Bathula Nagamalleswara Rao v. State rep. by
Public Prosecutor"
(From : Andhra Pradesh)
Coram : 2 PRAKASH PRABHAKAR NAOLEKAR AND LOKESHWAR SINGH
PANTA, JJ.
Criminal Appeal No. 1097 of 2006, D/- 22 -4 -2008.
Bathula Nagamalleswar Rao and Ors. v. State, Rep. by Public Prosecutor.
(A) Penal Code (45 of 1860), S.302 - MURDER - SENTENCE REDUCTION - Murder -
Sentence - One of accused found guilty for being member of unlaw ful assembly and
sharing common intention - Plea for reduction of his sentence to period already
undergone by him - Ground that he was 87 years old and suf fering from multiple
ailments - Held, cannot be accepted. (Paras 14, 35, 36)
(B) Penal Code (45 of 1860), S.300 - MURDER - EVIDENCE - UNLAWFUL
ASSEMBLY - Murder - Appreciation of evidence - Accused persons inflicted number of
in juries - Caused death of deceased per sons - F.I.R. found to be not tainted and
antetimed - Clear and consistent evidence by eye-witness, wife of one of deceased
persons - Corroborated by medical evidence - Also corroborated by another eye-witness
whose presence at place of incident was fully established - Fact that said witnesses were
related to deceased - Not sufficient to discard their testimony - Conviction of accused
persons u/S.302, u/S.149 - Not liable to be interfered with. (Paras 14, 36)
(C) Criminal P.C. (2 of 1974), S.154, S.157 - FIR - INVESTIGATION - F.I.R. - Three
persons murdered in a village on same evening - Alleged delay of about 16 hours in
sending F.I.R. to Magistrate - Explanation that, ma jority of police personnel were
deputed in village for maintaining law and order situation which was too tense in view of
said murders - Held, correctness and truthfulness of F.I.R. which was promptly lodged in
Police Station cannot be doubted. (Para 21)
(D) Evidence Act (1 of 1872), S.3 - EVIDENCE - WITNESS - Evidence of eye-witness -
Credibility - Three persons murdered in village in two incidents on same evening - Son of
one of accused in later case was murdered in first incident - Deceased No. 1 and deceased
No. 2 were murdered in second incident - Fact that name of eye-wit ness in later case
finds mention in FIR for earlier murder - Cannot be found to discard cogent and credible
testimony of said eye-witness holding other accused persons responsible for murder in
later case. (Para 34)
Cases Referred : Chronological Paras
(2007) 1 SCC 699 (Ref.) 33
2006 AIR SCW 3680 : AIR 2006 SC 2716 : 2006 Cri LJ 4033 (Ref.) 32, 34
2006 AIR SCW 5043 : 2007 (1) AIR Jhar R 340 (Ref.) 31
@page-SC3228

2004 AIR SCW 2857 : AIR 2004 SC 2294 : 2004 Cri LJ 2527 (Ref.) 31
(2004) 9 SCC 193 (Ref.) 15, 20
2002 AIR SCW 4271 : AIR 2002 SC 3633 : 2003 Cri LJ 41 (Ref.) 33
(2002) 3 SCC 76 (Ref.) 33
2001 AIR SCW 4763 : AIR 2002 SC 175 : 2002 Cri LJ 254 (Ref.) 15, 19
1994 AIR SCW 2210 : 1995 Cri LJ 457 : 994 All LJ 1032 (Ref.) 15, 18, 30
1994 Supp (2) SCC 372 (Ref.) 15, 17, 29
AIR 1973 SC 501 : 1972 Cri LJ 1296 (Ref.) 15, 16
AIR 1973 SC 2407 : 1973 Cri LJ 1589 (Ref.) 33
P.S. Patwalia, Sr. Advocate, G. Ramakrishna Prasad, Siddharth Patnaik and G. Arun, for
Appellants; R. Sundaravardhan, Sr. Advocate, Ms. Altaf Fathima (for Mrs. D. Bharathi
Reddy), for Respondent; Siddhartha Chowdhury, for Applicant.
Judgement
1. LOKESHWAR SINGH PANTA, J. :-Bathula Nagamalleswara Rao, Bathula
Laxminarayana, Bathula Seshaiah, Narne Subba Rao and Morakonda Bapa Rao have
filed this appeal against the judgment and order dated 10th April, 2006 passed by the
Division Bench of the High Court of Judica ture, Andhra Pradesh at Hyderabad in
Criminal Appeal No. 942 of 2004 confirm ing their conviction and sentences imposed
upon them by the trial court.
2. In all, 12 accused, namely, B. Nagamalleswara Rao (A-1), B. Laxmi narayana, (A-2),
B. Gopalan, (A-3), B. Subba Rao (A-4), B. Sreenu (A-5), B. Venkateswara Rao (A-6),
Jonnalgadda Ramu (A-7), Jonnalagadda Suresh (A-8), B Kotaiah (A-9), B. Seshaiah (A-
10), Narne Subba Rao (A-11) and Morakonda Baparao (A-12) were charged and tried by
Additional Sessions Judge, Guntur, for offences under Sections 148/449/302/302 r/w
149/302 /302 r/w 149 and 427 of the Indian Penal Code [for short 'IPC'] for committing
murder of two persons namely, Marchavarapu Venkatarao (deceased No. 1) and Thalluri
Sambasiva Rao (deceased No. 2) in village Uddandarayuni palem.
3. The VII Additional Sessions Judge, Guntur, by judgment dated 31st May, 2004
convicted all the accused (except A-3) and sentenced them as under:-

Accused Nos. Conviction by Trial Court Sentence Imposed by Trial Court


A-1 U/s. 302 r/w 149, 302, 148 and 449 Life Imprisonment
A-2 302 r/w 149, 302, 148 and 449 Life Imprisonment
A-4 302 r/w 149 and 449 Life Imprisonment
A-5 302 r/w 149 and 449 Life Imprisonment
A-6 302 r/w 149, 302 and 449 Life Imprisonment
A-7 302 r/w 149 and 449 Life Imprisonment
A-8 302 r/w 149 and 449 Life Imprisonment
A-9 302 r/w 149 and 449 Life Imprisonment
A-10 302 r/w 149, 302 and 449 Life Imprisonment
A-11 302 r/w 149, 302 and 449 Life Imprisonment
A-12 302 r/w 149, 302 and 449 Life Imprisonment

4. The accused were also imposed a fine of Rs. 500/- each on each count and in de fault
of payment of fine, to suffer S.I. for six months and they were also sentenced to suffer
R.I. for a period of five years and to pay a fine of Rs. 1000/- each for the offence under
Section 449 of the IPC and in default of payment of fine to undergo S.I. for a pe riod of
six months. All the substantive sen tences were ordered to run concurrently.
5. The trial court, however, acquitted A-4, A-5, A-7 to A-9 for the offences under Section
427, IPC. A-3 died during the pen dency of the trial and trial against him, ac cordingly,
stood abated.
6. Brief facts necessary for disposal of this appeal are as follows:-
All the accused, deceased No. 1, deceased
@page-SC3229
No. 2 and other material witnesses are all residents of village Uddandarayunipalem
within the jurisdiction of Thulluru Police Station. In the last panchayat elections held in
the village area, deceased No. 1 was elected as Sarpanch of the village as a can didate of
Telugu Desam Party by defeating his rival Congress Party candidate namely, Puli Babu
who had the support of Bathula Venkateswara Rao (A-6). It was alleged that one
Tadikoda Venkateswara Rao s/o Pedalakshmi Narayana approached de ceased No. 1 for
settlement of the pending land dispute between him and A-6. Deceased No. 1 alleged to
have advised both the par ties to approach the village elders and if their dispute still
remained unsettled, then they could approach the competent court of law. It was alleged
that A-6 bore grudge against deceased No. 1 as the latter was trying to support Tadikolana
Venkateswara Rao in the said matter.
7. On the day of occurrence i.e. 18th July, 1996 around 6:30 p.m. Rama Mohan Rao, son
of A-6 was murdered at the out skirts of the same village. The accused per sons believed
that Tadikolana Venkateswara Rao might have murdered the son of A-6. On the same day
at about 7:30 p.m. M. Shakuntala (PW-1), wife of deceased No.1, was standing on the
verandah of their house, while deceased No. 1 and deceased No.2 were chit-chatting with
M. Srihari (PW-2) and B. Sathyanarayana (PW-3) - both friends of deceased No.1, in
front of the house of deceased No. 1, they noticed a group of men armed with dangerous
weapons moving towards the house of deceased No. 1. On the advice of PW-3, deceased
No. 1 and his wife PW-1 rushed inside the house and bolted its door from inside. In the
mean while, all the twelve accused trespassed into the house and committed mischief by
break ing the telephone and other valuable articles lying in the house of deceased No. 1
and in the process, the accused noticed deceased No. 1 concealing himself in the
bedroom. The accused attacked deceased No. 1 indiscrimi nately with lethal weapons. It
was the case of the prosecution that A-3 stabbed deceased No.1 with a crow bar thrice
below stomach and in left side of his abdomen. A-1 as saulted deceased No. 1 with a
knife on both sides of the neck and chest. A-2 hacked on the right upper arm and A-3 cut
the right wrist of deceased No. 1 with a crow bar. Thereafter A-1 to A-3 continued
inflicting injuries on other parts of the body of de ceased No. 1 in the presence of his wife
PW-1 and as a result of severe injuries, Venkateswara Rao deceased No. 1 died at the
spot. It was alleged that the remaining ac cused chased Thalluri Sambasiva Rao de ceased
No. 2 who tried to escape from the scene of occurrence when A-6, A-10, A-l 1 and A-12
attacked deceased No.2 with axe, spears and crow bars. As a result of mul tiple injuries
suffered by deceased No. 2 upon his body, he died in the bathroom of the house of
deceased No. 1. The whole inci dent was witnessed by P.Ws. 1, 2 and 3. On seeing the
accused persons mercilessly at tacking and assaulting deceased No. 1 and deceased No. 2
with dangerous weapons, both PWs-2 and 3 got frightened and they escaped from the
scene of occurrence by jumping over the wall of the house. A-4, A-5, A-7, A-8 and A-9 in
the process damaged the doors, window glass panes of the house, motorcycle, television,
etc. of deceased No.1. All the accused thereafter left the scene of occurrence.
8. On receiving the information about the incident, P. Maqbool Khan (PW-11) Sub-
Inspector of Thulluru Police Station, rushed to the scene of occurrence and noticed the
dead bodies of deceased No. 1 and deceased No. 2 lying in the house of deceased No. 1.
He recorded the statement of P.W. 1 at about 9:00 p.m (Exhibit P-1). On the basis of Ex
hibit P.1, PW-8 Head Constable P. Mallikarjuna Rao of Thulluru Police Station registered
First Information Report (F.I.R.) in Crime No. 63/1996 under Exhibit P-5. Thereafter, K.
Babu Rao (PW-13), Circle In spector of Police, took up investigation and conducted
Panchnama (Exhibit P-2) of the scene of occurrence in the presence of B. Subba Rao
(PW-5), held inquest over the dead bodies of both the deceased in the pres ence of M.
Subba Rao (PW-6) and Puli Sambayya (PW-7) and prepared a joint In quest Report
(Exhibit P-4). Then, P.W. 13 got the scene of occurrence photographed through J.
Rambabu (PW-9). On receipt of requisition from the Police on 19th July, 1996, Dr. K.
Mahipal Reddy (PW-12) held autopsy over the dead bodies of deceased Nos. 1 and 2 and
issued post-mortem cer tificates [Exhibits P9 and P10] respectively. He opined that both
the deceased appeared to have died due to multiple injuries. On 6th August, 1996 at about
8:00 a.m. P.W. 11 arrested A-7, A-8 and A-9. On 23rd August,
@page-SC3230
1996, he arrested A-2 and A-11 whereas A-1, A-3, A-4, A-6 and A-10 were arrested by
PW-11 on 8th September, 1996.
9. After completion of the investigation and after receipt of the post-mortem report,
charge-sheet was filed against the accused persons in the Court of Additional Munsif
Magistrate, Mangalagiri. Additional Munsif Magistrate committed trial to the learned
Sessions Judge, Guntur who, in turn, made over the trial of the case to the Court of 2nd
Additional Sessions Judge, Guntur. The learned Additional Sessions Judge, Guntur
framed charges under Section 148, IPC against A-1, A-2, A-4 to A-12 and under Section
449 of IPC against A-6, A-10 to A-12 and under Section 302 read with Sec tion 149, IPC
against A-4, A-5, A-7 to A-9. All the accused persons pleaded not guilty to the charges
and claimed to be tried. Sub sequently, the trial of the case was trans ferred to VII
Additional Sessions Judge, Guntur.
10. The prosecution in support of its case examined as many as 13 witnesses. In their
statements recorded under Section 313 of the Code of Criminal Procedure [for short
'Cr.P.C.'], the accused denied their involve ment in the crime and pleaded false impli
cation on suspension. In defence, the ac cused have examined Thummala Veeraiah
Chowdary (D.W.1), Machiraju Koti Surya Prakasharao (D.W. 2), Yethirajula Srinivasarao
(D.W.3) and Garji Rambabu (D.W. 4). The trial court, on scrutiny of the evidence on
record, held the accused guilty of the aforesaid offences and, accordingly, convicted and
sentenced them.
11. Being aggrieved, the accused filed the appeal under Section 374, Cr.P.C. be fore the
High Court and the High Court con firmed the conviction and sentence imposed upon A-
1, A-2, A-10, A-11 and A-12 by the trial court. The High Court, however, has acquitted
A-4, A-5, A-6, A-7, A-8 and A-9 of all the charges levelled against them. Now, A-l, A-2,
A-10, A-11 and A-12 are appel lants before us in appeal.
12. We have heard the learned counsel for the parties and with their assistance ex amined
the judgment of the High Court and re-appraised the entire material evidence placed on
record. Mr. P. S. Patwalia, learned Senior Advocate appearing on behalf of A-1, A-2, A-9,
A-11 and A-12, assailed the judg ment of the High Court inter alia contend ing :-
(i) that the High Court erred in not con sidering the fact that there is unconscionable delay
of 16 hours in sending the FIR (Ex. P5) to the Magistrate especially when the Police had
taken the statement of PW-1 at 7.30 p.m. itself regarding the commission of the offences
on the basis of which the FIR was recorded at 9.00 p.m. in the police Sta tion. According
to him, the unexplained delay in sending the FIR to the Magistrate created a serious doubt
about the correct ness and truthfulness of the prosecution case. He emphasized that the
FIR on the face of it indicates that the same was pre pared after due deliberation and
consulta tion between PW-1, and other interested witnesses in connivance of the police.
He submitted that if FIR was in existence at 11:00 p.m. on 18.07.1996 in which the
names of A-8 and A-9 were mentioned as assailants, they could have been arrested by the
Investigating Officer on the spot it self as the evidence shows that they were present at
the scene of occurrence when police reached the place and this circum stance would also
prove that the FIR was a tainted, ante-timed and ante-dated docu ment prepared by the
police around 3:00 p.m. on 19.07.1996.
(ii) that the testimony of PW-1, wife of deceased No. 1 is wholly unreliable, unbe lievable
and inherently improbable and has been wrongly believed by the Courts below. If she
was shocked into silence at the time of incident of murder it could not have been possible
for her in the normal course to have given precise details of the injuries being inflicted by
each of the accused with a par ticular weapon on different parts of the body of deceased
No. 1. According to the learned counsel, PW -1 is not an eye-witness to the incident of
murder and she has been pro jected later on by the Investigating Officer after receipt of
the injuries statements from the doctor who conducted the post-mortem over the dead
bodies of both the deceased, otherwise PW-1 could not have accurately detailed the
precise injuries on the body of deceased No.1 in her statement (Ex. P1).
(iii) that the High Court has disbelieved the testimony of PW-3 in regard to the pres ence
of A-4 to A-9 at the scene of occur rence and their participation in the commis sion of
crime and, therefore, they were given benefit of doubt. The conviction of A-1, A-2, A-10,
A-11 and A-12 relying upon the same set of evidence of P.W. 3 being one of the
@page-SC3231
accused in FIR No. 64/1996 for the murder of A-6, therefore, is not proper and sustain
able as PW-3 is positively an interested wit ness to frame the appellants in a false case.
13. Mr. R. Sundaravardhan, learned se nior counsel for the respondent-State on the other
hand contended that the reasons given by the trial court as well as by the High Court for
recording the order of conviction of the appellants are based upon proper apprecia tion of
evidence led by the prosecution in the case. According to the learned counsel, the courts
below have accepted the expla nation of the prosecution for causing delay in submitting
the FIR to the Magistrate and, therefore, this Court shall not be obliged to disturb the
finding of facts recorded by Courts below. He then submitted that merely because P.W. 1
and P.W. 3 the two eye-wit nesses are relative and friend respectively of deceased No. 1,
their testimony cannot be disbelieved and discarded on this premise only as their
evidence corroborated by other evidence is cogent, clear and satisfactory with the
hypothesis of the guilt of the ap pellants and this Court should be slow to interfere in the
well-reasoned and well-mer ited judgments of the courts below.
14. We have given our anxious consid eration to the rival contentions of the learned
counsel for the parties. The arguments put forward by Mr. Patwalia although are ex
tremely attractive, yet we find ourselves unable to agree with the same. It is no doubt true
that FIR (Ex. P5) came to be recorded at 11:00 p.m. on 18.07.1996 in the Police Station
by Head Constable P. Mallikajunarao (PW-8), who was posted at the relevant time at
Police Station, Thulluru. The incident of murder of deceased No. 1 and deceased No. 2
took place at 7:30 p.m. on 18.07.1996 as per Complaint (Ex. P1) made by PW-1 to Sub-
Inspector Maqbool Khan (PW-11), which was sent to Police Station, Thulluru, through
PC No. 2896 for registration of the FIR. It has come in the cross-examination of PW-8
that the distance between Police Station, Thulluru, and Mangalagiri where the Magistrate
holds court is about 20 kms. PW-8 categorically stated that FIR (Ex. P5) was registered
by him at 11:00 p.m. on 18.07.1996 and even if the copy of the FIR was to be sent to the
Magistrate during the midnight, it was not possible for the Police Constable to take the
FIR and deliver the same to the Magistrate at Mangalagiri on the intervening night of
18/19.07.1996 as there was no bus service available during the night time between the
two places. The trial court in its judgment observed that the FIR was received by the
Magistrate at 4.00 p.m. on 19.07.1996. The trial court has no ticed but not accepted the
contention of the learned counsel for the accused persons that in these circumstances the
statement of PW-1 (Ex. P1) might have been recorded at 3.00 p.m. on 19.07.1996 at
village Uddandayuni palem and from the village itself the state ment might have been
sent to Mangalagiri in a police vehicle and, therefore, the same was received by the
Magistrate at 4.00 p.m. on 19.07.1996. We have independently scru tinized the evidence
of PW-1 and PW-11, the Investigating Officer. It is their clear and consistent testimony
that statement (Ex. P1) was recorded at 9.00 p.m. on the night of 18.07.1996 after the
incident of murder had occurred at 7.30 p.m. It is undisputed fact that one
Ramamohanarao, son of A-6, was murdered on the same evening at about 6.00 or 6.30
p.m. at the outskirts of village Uddandayunipalem. It is the evidence of PW-11, Sub-
Inspector of Police, that about 7.15 p.m. M. Venkatarao informed him that his rival group
of men armed with deadly weap ons was moving towards their village and on receiving
the information he along with other police personnel had immediately pro ceeded to
village Uddandayunipalem in a private jeep for maintaining law and order. PW, Head
Constable, in cross-examination, corroborated the version of PW, Sub-Inspec tor of
Police, that on 18.07.1996 around 7.15 p.m. on receipt of the information in regard to
some incident of violence in the village, P.W. 11 along with five staff members left the
Police Station for maintaining law and order in the village. PW-8 sent FIR (Ex. P5) to the
Magistrate through PC No.2896 who brought statement of PW-1 [Ex. P-1] at 11:00 p.m.
to the Police Station. PW-13, Circle In spector of Police, stated that on the mid night of
18/19.07.1996 at about 00.15 hours he received a copy of express FIR through PC No.
2896 disclosing the inci dents of murder of three persons in village Uddandayunipalem.
The endorsement made on FIR (Ex. P5) by the Magistrate on its bare perusal would
reveal that he received a copy of the F.I.R. through PC No. 1293 and not through PC No.
2896 as deposed by P.W. 8. It appears from the record that PC No. 2896 handed over a
copy of express FIR to PW-13, Circle Inspector of Police, at village
@page-SC3232
Uddandayunipalem at 00.15 hours on the intervening night of 18/19.07.1996 and then
constable PC No. 1293 might have been de puted to deliver the FIR (Ex. P5) to the
Magistrate at Mangalagiri. The trial court in its judgment observed that keeping in view
the serious and tense situation in the village because of the murder of three persons on the
same evening, the entire staff of Police Station was deputed to maintain law and order
problem there. Out of the victims, one was the son of A-6, ex-Sarpanch of the vil lage,
whereas deceased No. 1 was the sitting Sarpanch of the same village. Taking these
circumstances into consideration, the trial court held that there was no delay in lodg ing
the FIR with the police and delay in send ing a copy of the FIR to the Magistrate was a
result of shortage of police personnel who could not be deputed to deliver the same to the
Magistrate during the night of 18.07.1996 or in the early hours of 19.07.1996. The High
Court has reexam ined the evidence on record and held that the delay in sending FIR to
the Magistrate was not deliberate or intentional, but be cause of some technical errors
committed by the Investigating Officer during the course of investigation of the case
which could not be found fatal to the case of the prosecu tion, especially when the
Investigation Of ficer was not cross-examined on this point.
15

. Mr. Patwalia, in support of the sub mission that the FIR (Ex. P5) was ante-timed and
ante-dated and delay of 16 hours in sending the FIR to the Magistrate would cast a
serious doubt of its correctness, has re lied upon the decisions of this Court in Thulia Kali
v. The State of Tamil Nadu [(1972) 3 SCC 393]; Arjun Marik and Ors. v. State of Bihar
[(1994) Supp (2) SCC 372]; Meharaj Singh (L/Nk.) v. State of U. P. [(1994) 5 SCC 188];
Thanedar Singh v. State of M. P. [(2002) 1 SCC 487] and Kunju Muhammed alias
Khumani and Anr. v. State of Kerala [(2004) 9 SCC 193]. AIR 1973 SC 501
1994 AIR SCW 2210
2001 AIR SCW 4763

16
. It is not in dispute that this Court in series of judgments has repeatedly and consistently
emphasized that First Information Report in a criminal case is an extremely vital and
valuable piece of evidence for the purpose of corroborating the oral evidence adduced at
the time of trial. The importance of the report can hardly be over-estimated from the
standpoint of the accused. The object of insisting upon prompt lodging of the report to the
police in respect of com mission of an offence is to obtain early in formation regarding
the circumstances in which the crime was committed, the names of the actual culprits and
the part played by them as well as the names of the eye-wit nesses present at the scene of
occurrence and requirement of sending the FIR to the Magistrate forthwith is to enable
the Magis trate concerned to have a watch on the progress of the investigation. In Thulia
Kali v. State of Tamil Nadu (supra), it is held that delay in lodging the First Information
Report often results in embellishment which is a crea ture of after-thought. On account of
delay the report not only gets benefits of the ad vantage of spontaneity, danger creeps in
of the introduction of coloured version, exag gerated account of connected story as a re
sult of deliberation and consultation. It is held that in order to avoid all criticisms of the
defence, it is therefore essential that the delay in lodging of the First Information Report
should be satisfactorily explained. AIR 1973 SC 501

17. In Arjun Marik and Ors. v. State of Bihar (supra), this Court held that Section 157 of
Cr.P.C. mandates that if, from infor mation received or otherwise, an officer-in-charge of
Police Station has reason to sus pect the commission of an offence which he is
empowered under Section 156 to investi gate, he shall forthwith send a report of the same
to the Magistrate empowered to take cognizance of such offence upon a police report.
Section 157, Cr.P.C. thus, in other words, directs the sending of the report forthwith, i.e.
without any delay and imme diately. Further, Section 159, Cr. P.C. en visages that on
receiving such report, the Magistrate may direct an investigation or, if he thinks fit, to
proceed at once or depute any other Magistrate subordinate to him to proceed to hold a
preliminary inquiry into the case in the manner provided in the Code of Criminal
Procedure. The forwarding of the occurrence report is indispensable and ab solute and it
has to be forwarded with earli est despatch which intention is implicit with the use of the
word "forthwith" occurring in Section 157, which means promptly and without any
undue delay. The purpose and object is so obvious which is spelt out from the combined
reading of Sections 157 and 159, Cr.P.C. It has the dual purpose, firstly to avoid the
possibility of improvement in
@page-SC3233
the prosecution story and introduction of any distorted version by deliberations and
consultation and secondly to enable the Magistrate concerned to have a watch on the
progress of the investigation.
18

. In Meharaj Singh (L/Nk.) v. State of U. P. (supra), this Court held that delay in sending
special report to the Magistrate or failure to send copy of the FIR to the Medical Officer
along with dead body for post-mortem and absence of its reference in inquest report can
give rise to an inference that the FIR had been ante-timed and had not been recorded till
the inquest proceedings were over at the spot by the Investigating Officer. 1994 AIR
SCW 2210

19

. Again in Thanedar Singh v. State of M. P. (supra), this Court held on facts that failure of
the prosecution to clear doubt regarding the date and time of recording of the FIR and
delay in sending FIR to Magistrate cast a serious doubt on the correctness of the FIR.
2001 AIR SCW 4763

20. In Kunju Muhammed alias Khumani and Anr. v. State of Kerala [(2004) 9 SCC 193],
this Court reiterated that the delay in send ing the FIR to the Magistrate without giving
satisfactory explanation creates a serious doubt about the prosecution case. In that case it
was found : (a) the statement of PW-1 that the complaint was signed on the mid night of
3.11.1991; (b) FIR reaching the Ju risdictional Magistrate more than 36 hours after the
incident in question though the court is situated in the same town; (c) the evidence of the
doctor as to the presence of rigor mortis on the body of the deceased in dicating death
must have occurred much earlier than 8.15 to 8.30 a.m. on 03.11.1991, and (d) recording
in the inquest report (Ex. P6) that the body of the deceased when ex amined was found to
be cold and frozen. In the circumstances noticed above, the Court held that, delay of more
than 36 hours in sending the FIR to the Magistrate created a serious doubt about the
prosecution case and had found the accused persons entitled for benefit of doubt.
21. We have earlier pointed out that in the present case there was a delay of about 16
hours in sending the FIR (Ex. P5) to the Magistrate, but the explanation as recorded by
the trial court that the majority of the police personnel were deputed in village
Uddandarayunipalem for maintaining the law and order situation which was too tense in
view of the murder of three men of the village on the same evening, we do not find any
cogent and convincing reason for doubt ing the correctness and truthfulness of the FIR
which was promptly lodged in the Po lice Station at 9:00 p.m. in relation to the murder of
deceased No. 1 and deceased No. 2 at about 7:30 p.m. The Investigating Of ficer in cross-
examination denied the sug gestion of the defence that Ex. P1 was brought into existence
around 2:35 p.m. or 3:00 p.m. on the next day, i.e. 19.07.1996, after a good deal of
consultations and con fabulations with the leaders of rival group of the accused and
particularly after observ ing the injuries on the dead body of deceased No. 1. He
categorically stated that he re corded the statement (Ex. P1) made by PW-1 between 9:00
p.m. to 10:00 p.m. on 18.07.1996 and on completion thereof, the same was sent through
P.C. No. 2896 to the Police Station for registration of the FIR and after registration of the
same constable brought the copy of the FIR to the scene of occurrence at about 12:00
mid-night and thereafter PW-11 took up further investiga tion of the case. A suggestion of
the defence that his superiors also aided in fabrication of Ex. P1 and under their
instructions he had not only ante-timed but also ante-dated the FIR and planted witnesses
and having done so he deposed falsely against the accused persons, has categorically
been de nied by him.
22. The evidence of PW-1 was attacked by A-1, A-2, A-3, A-10, A-11 and A-12 on the
ground that her presence at the relevant time on the spot on the day of occurrence was
doubtful as in the morning she had gone to the house of her daughter at Vijayawada and
was summoned after the incident of murder for making the statement (Ex.P1) on the basis
of which FIR came to be registered. We have independently scrutinized the evi dence of
PW-1, wife of deceased No. 1. It is her clear and consistent evidence that on 18.07.1996
at about 7:30 p.m. while she was standing at the verandah of her house and her husband
along with Talluri Sambasivarao (deceased No. 2), M. Srihari (P.W. 2), B. Satyanarayana
(P.W. 3) and some more persons were chit-chatting in front of their house, she saw about
40 persons armed with dangerous weapons rushing to their house and on seeing them,
P.W. 3 asked her and her husband Venkatarao to
@page-SC3234
conceal themselves inside the house other wise they would be killed by the rival group
men. According to her, A-1, A-2, A-4 to A-12 along with A-3 came to the verandah of
their house and after breaking open the doors, they caused damage to the T.V. and other
articles lying in the rooms. A-3 was armed with crow-bar, A-1 was armed with knife, A-2
was having an axe in his hand. They entered into their bedroom through the bottom
portion of the door. When her hus band was standing at the corner of the bed room, she
requested A-1 to A-3 not to harm her husband. A-1 pulled her aside and A-3 stabbed her
husband thrice with the crow bar. A-2 assaulted him on the right arm. When her husband
fell down on the floor, A-1 to A-3 indiscriminately inflicted more injuries on his body.
Thereafter, her mother-in-law knocked at the door of the house and on opening the door
she was informed by her mother-in-law that A-6, A-10, A-11 and A-12 had murdered
Talluri Sambasivarao (deceased No. 2) in their bathroom with crow-bars and axes. The
trial court as also the High Court both have accepted the evi dence of PW-1 in its entirety
without any suspicion and embellishment. The deceased No.1 sustained as many as 26
injuries on his body as noticed by Dr. K. Mahipal Reddy (PW-12) in post mortem
certificate (Ex. P10). The cause of death of deceased No. 1 in the opinion of the doctor
was due to multiple injuries sustained by him with sharp-edged weapon. Ex. P1, which
was the earliest ver sion of the incident of murder narrated by PW-1 to the police would
reveal that she had categorically named A-1, A-2, A-3 (A-3 died during the pendency of
the trial), A-10, A-11 and A-12 as assailants who mercilessly inflicted multiple injuries on
almost every part of the body of her husband with lethal weapons resulting in his death.
The evidence of PW-1 finds corroboration from the medi cal evidence. Despite lengthy
cross-exami nation, nothing has been elicited to discredit and discard her testimony,
which has re mained unshattered and consistent. A sug gestion of the defence that A-10
was unable to walk without assistance of any person as he, at the relevant time, was aged
about 70 years, has been denied by her. She categori cally reasserted that firstly A-1 to A-
3 en circled her husband and then A-1 assaulted him thrice with knife, while A-2
assaulted him on the right fore-arm with an axe and again A-1 and A-3 one after the other
inflicted more severe injuries on the body of her husband with the weapons they were
holding in their hands. A series of sugges tions were put to her by the defence: (a) that on
the morning of the day of the incident of murder of her husband she had gone to
Vijayawada to look after her daughter Vasavi who was sick, (b) she was not present in the
village and that about 10:00 p.m. or 11:00 p.m. she was called from Vijayawada, (c) after
due deliberation arid consultation among her well-wishers and relatives, she got
complaint (Ex. P1) fabricated at about 2 O' clock in the intervening night of 18/
19.07.1996, (d) the contents of Ex. P1 were not stated by her and she only subscribed her
signatures thereon, and (e) the police also contributed their role in fabricating her
statement (Ex. P1) have emphatically and categorically been denied by her. She, how
ever, admitted that A-6 is a leader of one group in her village, but she denied that on
account of ill-feelings between two rival groups of the people in the village, a false case
was foisted against the appellants by her. She also denied the further suggestion that none
of the appellants was responsible for causing death of her husband or the death of
Sambasivarao, deceased No. 2.
23. The testimony of this witness has been corroborated by PW-3, another eye witness of
the occurrence and PW-11, the Investigating Officer. The presence of PW-3 on the scene
of murder has fully been es tablished. It is the evidence of PW-3 that on 18.07 1996 at
about 7.30 p.m. he along with PW-2, deceased No. 2 and few more people was present in
the house of deceased No. 1 and when they were chit-chatting in front of the house, all
the accused persons armed with crow-bars, spears, axes and knives rushed to the house of
deceased No. 1 and on seeing them, he advised deceased No.1 and his wife PW-1 to
conceal themselves inside their house as he apprehended some danger to the life of
deceased No. 1. PW-1 and deceased No. 1 went inside their house and bolted the door
from inside. He saw that out of twelve accused persons half of them had surrounded the
house of deceased No. 1 and the remaining accused chased deceased No. 2, who was
running to his house, which is at a distance of 10 yards from the house of deceased No.1.
He saw A-6, A-10, A-11 and A-12 breaking open the door of the bath room of deceased
No. 1 where they killed deceased No. 2 with crow-bars, axes and
@page-SC3235
spears. In his presence, the police conducted inquest over the dead body of both the
deceased persons. The High Court found that deceased No. 2 sustained more than 22
injuries on his body as noticed by doctor in post-mortem certificate (Ex. P9). According
to the opinion of the doctor, the cause of death of deceased No. 2 was due to multiple
injuries. The High Court has given the de tails of the injuries sustained by both the
deceased at pages 8 to 14 of the judgment. We do not think it necessary to reproduce the
details of those injuries again in this judgment for unnecessarily burdening the record as
the death of both the deceased due to the injuries sustained by them on their bodies is not
in dispute. PW-3 also stated that inquest on the dead bodies of deceased No. 1 and
deceased No. 2 was conducted by the police in his presence. In spite of lengthy cross-
examination, his testimony has not been shattered or impeached by the defence. A
suggestion of the defence that for the past four days from the date of record ing of his
statement in the court, he was tutored to give evidence against the accused persons has
categorically been denied by him. He has admitted that he was framed as an accused in
the murder of the son of A-6.
24. PW-2 in his deposition stated that at about 7.30 p.m. on 18.07.1996 he along with
PW-3, deceased No.1, deceased No. 2 and some persons were chit-chatting in front of the
house of deceased No. 1. PW-1, wife of deceased No. 1, was sitting on a cot in the
verandah. At that time, a group of men had gathered on the road side raising slogans near
the house of deceased No. 1. PW-3 re quested deceased No. 1 and his wife PW-1 to
conceal themselves inside the house. They both went inside the house and bolted the door
of their house from inside. On seeing the accused persons armed with dangerous
weapons, he along with PW-3 made good their escape from the scene of occurrence by
jumping over the northern side com pound wall of the house of deceased No. 1. On
coming to know about the arrival of po lice, he went to the house of deceased No. 1 and
found deceased No. 1 and deceased No. 2 lying dead in a pool of blood. The inquest
report of the dead bodies of deceased No.1 and deceased No. 2 was conducted and pre
pared by the Investigating Officer in his pres ence.
25. The evidence of PWs-6 and 7 would go to show that they were present on 19.07.1996
at about 3.00 a.m. when the police conducted inquest on the dead body of deceased No. 1
under Ex. P3 and on the same day at about 5.30 a.m., inquest on the dead body of
deceased No. 2 was conducted under Ex. P4. Both these witnesses are mediators. They
stated that deceased No.1 and deceased No. 2 died as a result of fatal injuries sustained by
them. Both these wit nesses have, categorically stated about the descriptive particulars of
the scene of oc currence at the house of deceased No. 1.
26. D. Sivanagendramma (PW-4) is the wife of deceased No. 2. It is her evidence that on
receiving the information of her husband's death, she went to the house of deceased No.1
where she spotted her hus band lying in a pool of blood in the bath room. Nothing
substantial has been elicited from her cross-examination by the defence to disbelieve and
discredit her evidence.
27. PW-11 is the Investigating Officer. It is his evidence that on 18.07.1996 around 7:15
p.m. he was informed on telephone by M. Venkatarao (deceased No. 1), the then
Sarpanch of village Uddandarayunipalayem that a group of men raising slogans and car
rying dangerous weapons in their hands was moving towards his house with clear inten
tion to commit offence and requesting the police to come to the village to defuse the tense
situation. He, in turn, informed the Inspector of Police, Amaravathi, through VHF and
himself along with staff members proceeded to the village to maintain law and order
problem. The Inspector of Police, Amaravathi, later on came to the village. When he
reached at the house of M. Venkatarao, he found his dead body lying in a room and also
spotted the dead body of deceased No. 2 in the bath room of the house of deceased No. 1.
On receiving necessary in structions from Inspector of Police, P.W. 11 recorded the
statement (Ex. P-1) of P.W.1 at about 9.00 p.m. and obtained her signatures thereon. He
sent the said statement through PC No. 2896 to Police Station, Thulluru, and instructed
PW-8, who was in charge of Po lice Station, to register the case. He handed over the
investigation to the Inspector of Police. He arrested some of the accused per sons on
different dates. On examination of the evidence of this witness, it stands proved that he
read over the statement (Ex. P1) to PW-1, who admitted the contents thereof to be
correct.
@page-SC3236
28. the High Court. We find not an iota of evidence on record to prove The High Court,
on re-examination and re-appraisal of the evidence of PW-3, PW-11 and other evidence
on record, held that the presence of A-4 to A-9 on the scene of occurrence has not been
proved by the prosecution by leading cogent, satisfactory and convincing evidence to
prove that the said accused persons were the members of unlawful assembly and shared
common ob ject with the other accused with an inten tion to eliminate deceased No. 1 and
de ceased No. 2. The evidence of P.W. 10 the then SDPO, Guntur, who partly conducted
the investigation of the case, would prove that on the day of incident of murder, A-5
attended to the work of general laboratory of Water Treatment Unit of VTPS situated at a
distance of 40 kms. from the scene of oc currence. His evidence would also disclose that
A-5 was present in the Thermal Power Station from 4.30 p.m. to 8.30 p.m. on the day of
occurrence which, according to the prosecution, took place at about 7.30 p.m. In these
circumstances, the possibility of A-5 participating in the commission of the crime has
been rightly ruled out by that A-7 and A-8 had shared the common intention to eliminate
deceased No. 1 and deceased No.2. Nothing has been brought on record to show that they
were inimical against deceased No. 1 and deceased No.2 so as to cause their death. The
evi dence against A-5 to A-9 was not satisfac tory and convincing to connect them both to
the commission of the crime and, there fore, we have no hesitation to accept the
reasoning recorded and finding arrived at by the High Court in acquitting A-4 to A-9 by
giving them benefit of doubt.
29. It is by now well-settled that mere relationship of the witnesses cannot be the sole
basis to discard or disbelieve their evi dence if it is otherwise found to be believ able and
trustworthy. However, when the Court has to appreciate the evidence of any interested
witness it has to be very careful in weighing their evidence. In other words, the evidence
of an interested witness re quires greater care and caution while scru tinizing his
evidence. The Court has to ad dress to itself whether there are any infir mities in the
evidence of such a witness; whether the evidence is reliable and trust worthy and whether
the genesis of the crime unfolded by such an incident is probable or not. If the evidence
of any interested witness or a relative on a careful scrutiny is found to be consistent and
trustworthy, free from infirmities or any embellishment there is no reason not to place
reliance on the same (see Arjun Marik and Ors. v. State of Bihar [(1994) Supp (2) SCC
372].
30

. In Meharaj Singh v. State of U. P. (supra), this Court held that testimony of interested
witness cannot be rejected on the sole ground of interestedness but should be subjected to
close scrutiny. 1994 AIR SCW 2210

31

. In Anil Sharma and Ors. v. State of Jharkhand [(2004) 5 SCC 679], this Court reitered
and re-emphasised that the testimony of related wit ness, if after deep scrutiny found to
be oth erwise truthful and credible, cannot be dis carded on the sole ground of interested
wit nesses. Again, in Bhimappa Chandappa Hosamani and Ors. v. State of Karnataka
[(2006 (11) SCC 323), it has been held that credibility of the wit ness is to be tested by
reference to the qual ity of his evidence which must be free of any blemish or suspicion,
must impress the court as wholly truthful, must appear to be natural and so convincing
that the court has no hesitation in recording a conviction solely on that basis. 2004
AIR SCW 2857
2006 AIR SCW 5043

32

. In a recent decision, this Court in S. Sudershan Reddy and Ors. v. State of A. P. [(2006)
10 SCC 163] held that relationship is not a factor to affect the credibility of a witness. It
is more often than not that a relation would not conceal the actual culprit and make al
legations against an innocent person. Foun dation has to be laid if plea of false implica
tion is made. In such cases, the court has to adopt a careful approach and analyse the
evidence to find out whether it is cogent and credible. 2006 AIR SCW 3680

33

. Again this Court in Salim Sahab v. State of M. P. [(2007) 1 SCC 699] held that mere
relationship is not a factor to affect credibility of a witness. To the same effect are the
decisions in State of Punjab v. Jagir Singh [(1974) 3 SCC 277]; Lehna v. State of Haryana
[(2002) 3 SCC 76] and Gangadhar Behera v. State of Orissa [(2002) 8 SCC 381]. AIR
1973 SC 2407
2002 AIR SCW 4271

34

. The High Court no doubt has disbe lieved some portion of the evidence of P.W. 3 in
regard to the presence of A-4 to A-9 at 2006 AIR SCW 3680

@page-SC3237
the scene of occurrence on the grounds that A-5 was present in the Thermal Power Sta
tion from 4.30 p.m. to 8.30 p.m. on the day of occurrence, therefore the possibility of A-5
participating in the commission of the of fence could not be ruled out. The High Court
has found no evidence against A-7 and A-8 on record to hold them guilty for the mur der
of deceased No.1 and deceased No. 2. It is well-settled that the Court can rely upon that
part of the statement of the witness which is cogent, trustworthy, consistent and
believable for the purpose of holding the accused guilty of the offence. Simply because
the name of P.W. 3 finds mentioned in FIR for the murder of son of A-6, the cogent and
credible testimony of PW-3 holding A-1, A-2, A-10, A-11 and A-12 responsible for the
murder of deceased No.2 cannot be lost sight of on that ground alone. The presence of
PW-3 on the scene of occurrence has been fully established by the evidence of PW-1 and
PW-2 and his name also finds mentioned as an eye- witness in the FIR which was lodged
by P.W. 1 immediately after the incident of murder. The High Court has found a por tion
of the evidence of P.W. 3 deficient in regard to the involvement of A-1, A-2, A-10, A-11
and A-12 for committing the offences levelled against them and accordingly given them
benefit of doubt. It is settled that even if a major portion of the evidence is found to be
deficient, in case the residue is sufficient to prove guilt of an accused, conviction can be
maintained. It is the duty of the court to separate grain from chaff. Where chaff can be
separated from grain, it would be open to the court to convict an accused notwith
standing the fact that evidence of some of the witnesses has been found to be deficient.
Falsity of a particular material witness or material particular would not ruin it from the
beginning to the end. The maxim "falsus in uno falsus in omnibus" has no applica tion in
India and the witnesses cannot be branded as liars [See S. Sudershan Reddy and Ors. v.
State of A. P. [(2006) 10 SCC 163].
35. Lastly, it was urged by Mr. Patwalia that the case; of B. Seshaiah (A-10) is an
extremely hard case, who is now aged about 87 years and is suffering from Parkinson's
disease Hypertension, Diabetes with severe Calcific AV Stenosis, Mild AR, Moderate
MR and Anemia of some degree. This apart, A-10 has already undergone jail suffering for
about three years and, therefore, taking all these factors into consideration, his sentence
may be reduced to the period already undergone by him. We are afraid to accept this
submission of the learned counsel, be cause A-10 has been held guilty for being a
member of unlawful assembly and sharing common intention with A-l, A-2, A-10, A-11
and A-12 to commit the murder of de ceased No. 2.
36. Having given our careful consider ation to the submissions made by the learned
counsel for the parties and in the light of the evidence discussed hereinabove and tested
in the light of the principles of law highlighted above, it must be held that the evaluation
of the findings recorded by the High Court do not suffer from any mani fest error and
misappreciation of evidence on record. Hence, we agree with the finding of the High
Court that the A-1, A-2, A-10, A-11 and A-12 are the real culprits and their conviction
and sentences for the offences punishable under Section 302 read with Section 149,
Section 302 and Section 449 of the IPC are wholly sustainable and justi fiable.
37. In the result, there is no merit in this appeal and it is, accordingly, dismissed.
38. B. Seshaiah (A-10) is on bail pursu ant to the order of this Court passed on
30.10.2006. His bail bonds are cancelled and surety is discharged. He is directed to
surrender before the trial court within one month from the date of this order. The learned
trial court shall remand him to jail for serving out the remainder part of the sentences. On
failure of B. Seshaiah to ap pear before the trial court within the stipu lated period, the
learned trial court shall take appropriate steps against A-10 in accor dance with law.
Appeal dismissed.
AIR 2008 SUPREME COURT 3237 "Correspondent, St. Michael's T. T. I. v. V. N.
Karpaga Mary"
(From : Madras)*
Coram : 2 S. B. SINHA AND LOKESHWAR SINGH PANTA, JJ.
Civil Appeal No.2960 of 2008 (arising out of SLP (C) No. 19520 of 2006), D/- 24 -4
-2008.
Correspondent, St. Michael's T. T. I. v. V. N. Karpaga Mary and Ors.
T.N. Recognised Private Schools (Regulation) Act (20 of 1974), S.22 - EDUCATION -
TERMINATION OF SERVICE - REINSTATEMENT - BACK WAGES - Termination of
service - Teacher having
@page-SC3238
requisite qualifications - Appointed on regu lar basis - Termination of service on ba sis of
subsequent G. O. raising requisite qualifications - G. O. not given any retrospective
application - Termination of service is improper - Teacher entitled to reinstatement with
back-wages - Backwages, however, confined to 75% in facts and circumstances of case.
(Paras 11, 12, 13, 16)
Cases Referred : Chronological Paras
2006 AIR SCW 2952 : AIR 2006 SC 2304 : 2006 (4) ALJ 578 10
2006 AIR SCW 5779 : 2007 Lab IC 262 : 2007 (1) AIR Jhar R 677 (Ref.) 10, 14
2006 AIR SCW 6414 : 2007 Lab IC 590 (Ref.) 10, 14
2005 AIR SCW 4634 : AIR 2005 SC 3966 : 2005 AIR - Jhar HCR 2447 14
2004 AIR SCW 273 : AIR 2004 SC 1373 : 2004 AIR - Kant HCR 373 (Disting.) 9, 13,
15
Romy Chacko, for Appellant; L.N. Rao, Sr. Advocate, Jayanth Muthraj, C.K. Sasi and
V.G. Pragasam, for Respondents.
* W. A. No. 2167 of 2005, D/- 24-2-2006 (Mad).
Judgement
1. S. B. SINHA, J. :- Leave granted.
2. Appellant is an aided institution. It is aided by the State of Tamil Nadu. Terms and
conditions of its teachers are governed by Tamil Nadu Recognised Private Schools
(Regulation) Rules, 1974 framed under Tamil Nadu Recognised Private Schools (Regula
tion) Act, 1973.
3. Respondent was appointed in the said school on or about 11.7.1977. He was a graduate
in Master of Education as also in Master of Science. He was having the requi site
qualification for recruitment to the said post.
4. The State, however, issued a Govern ment Order dated 16.9.1994 raising the
qualification of a teacher, inter alia, stating :
"XV Staff Requirements :

Teaching Staff Qualifications etc.

(b) Subject Teacher A Post-Graduate Degree in the relevant subject and M.Ed. Degree
with teaching experience preferably in recognized schools. There shall be four subject
teachers to handle Tamil, English, Mathematics, Science and Social Science. The
Headmaster shall handle one of the five subjects."

On the plea that the respondent did not hold the requisite qualifications in terms of the
said GOMs dated 16.9.1994, his services were terminated by the appellant by an or der
dated 6.1.1995 with effect from 22.12.1994.
5. Respondent filed a writ petition be fore the High Court questioning the said order of
termination, inter alia, stating that the said purported G.O.Ms. dated 16.9.1994 could not
have been given a retrospective effect.
6. A learned Single Judge of the said Court, by an order dated 5.9.2005 quashed the said
order of termination opining that once the appointment was made in a lawful manner and
the teachers were found to have the requisite qualifications as prescribed at the time of
such appointment, a revision in qualification so as to be applied retrospec tively and
affect the career of an appointee would not be permissible.
Appellant-institution was held to be ame nable to writ jurisdiction. It was also found that
the State had not issued any direction to remove the respondent from service. On the said
findings, it was directed :
Therefore, it is not open to the respon dents to adopt an erroneous interpretation and to
deny employment to the petitioner who has been serving the school from 1977. It is
pertinent to note that in W.P.M.P. No. 9628 of 1995, the learned Single Judge has issued
interim orders on 21.4.1995 itself that no appointment shall be made for a period of two
weeks if the third respondent had not appointed any one in the place of the Petitioner.
There is nothing on record to show that the said interim order had been vacated
subsequently, though the respon dent, in their counter affidavit, has stated that a qualified
teacher had been appointed on 23.12.1994."
7. An intra court appeal was preferred thereagainst before the Division Bench of the said
Court. The question which was raised before the said Bench was limited to the question
of payment of the back wages
@page-SC3239
to the respondent and as to whether the State is liable therefor or the appellant in stitution.
The contention of the parties were noted by the Division Bench as under :
"According to the appellant, namely the Teacher's Training Institute, the teachers services
were terminated only because the Government repeatedly wrote letters that the
Government Order has to be strictly adhered to and that it is only at their instructions that
the teacher was terminated. The learned Special Government Pleader would submit that
while it is true that the Government insisted on all Institutes to strictly comply with the
conditions stipulated in the Gov ernment Order, no instructions had been specifically
issued in this case to terminate the services of the teacher, nor had any in structions been,
issued to comply with the Government Order retrospectively in respect of any person
who has already been in ser vice. According to them, since they are bear ing the
expenditure arising out of the ap pointment of the new teacher, they cannot be saddled
with the burden twice. It is also the case of the Government that it was never the
Institute's case that any specific instruc tions were issued by the Government to ter minate
the services of the teacher; and that if the Institute had taken a decision which is not
supportable in law, then it is the Institute which has to bear the financial bur den and not
the State."
While refraining itself from interfering with the order of the learned Single Judge, the
Division Bench observed :
"However, it is open to the Institute to apply to the Government, if so advised, for
reimbursement of the wages paid to the teacher in compliance of the orders of the learned
Single Judge and thereafter, it is for the Government to take a decision in this regard. No
costs. Consequently, W.A.M.P. No.4015 of 2005 is closed."
8. Appellant is, thus, before us.
9

. Mr. Romy Chacko, learned counsel appearing on behalf of the appellant, would submit
that the petitioner institution being not a State within the meaning of Article 12 of the
Constitution of India, the writ peti tion was not maintainable. It was further more
contended that in any event, as the order of termination of the respondent was not mala
fide, the High Court should have held that she was not entitled to back wages and no
legal right vested in her in obtaining the relief of reinstatement. Reliance in this behalf
has been placed on Pearlite Liners (P) Ltd. v. Manorama Sirsi [(2004) 3 SCC 172].
2004 AIR SCW 273

10

. Mr. L.N. Rao, learned senior counsel appearing on behalf of the respondent, on the other
hand, would urge that the appel lant herein having not questioned the juris diction of the
learned Single Judge to inter fere with the matter, the contention that the writ petition was
not maintainable should not be allowed to be raised before this Court for the first time. It
was furthermore urged that keeping in view the respective conten tions made before the
Division Bench, it is evident that the question of the respondent's having any alternate
employment did not and could not arise. Strong reliance in this behalf has been placed on
Jasbir Singh v. Punjab and Sind Bank and Ors. [(2007) 1 SCC 566]; Gangadhar Pillai v.
Siemens Ltd. [(2007 (1) SC 533); and Vice Chancellor, Banaras Hindu Uni versity and
Ors. v. Shrikant [(2006) 11 SCC 42). 2006 AIR SCW 5779
2006 AIR SCW 6414
2006 AIR SCW 2952

11. It is neither in doubt nor in dispute that the appellant possessed the requisite
qualification at the time of his entry in the service. The educational qualification for a
teacher was sought to be raised by the State much later, namely, in the year 1994.
Respondent, indisputably, was appointed on a permanent basis. She was a regular teacher.
If she was a regular teacher, the question of termination of her services rely ing on or on
the basis of the purported GOMs dated 16.9.1994 did not arise as the same had not been
given retrospective effect. The State never said that in terms of the said GOMs, the
services of the employees who had validly been appointed, should be ter minated.
12. The contention of the appellant that there was some apprehension that recogni tion, as
granted by the State to the said in stitution, may be withdrawn should have been taken up
by it with the State at the first instance. It having failed to do so, in our opinion, no legal
infirmity can be found in the judgment.
13

. Reliance placed by the learned coun sel on Pearlite Liners (P) Ltd. v. Manorama Sirsi
[(2004) 3 SCC 142] is not apposite. The courts 2004 AIR SCW 273

@page-SC3240
exercise different jurisdictions while enter taining applications filed under different
statutes. While entertaining a suit, the court's jurisdiction would be governed by the
Specific Relief Act, 1963. Although principles laid down therein may be found to be appli
cable, the said provisions by themselves need not be strictly applied by the High Court
while exercising its jurisdiction un der Article 226 of the Constitution of India.
The question that the appellant was ame nable to writ jurisdiction is not in dispute. If it
was amenable to writ jurisdiction, the High Court was not only entitled to set aside an
order of termination of service on an in terpretation that neither the GOMs had any
retrospective application nor, in any event, had any application to the case of appoint
ment of the respondent but also to grant back wages. On the said premise, the High Court
had the jurisdiction to set aside the order of termination. Once the order of ter mination
was set aside, the logical corollary therefor should ordinarily ensue, subject of course to
denial of the benefit either in to tality or in part.
It was in the aforementioned situation, the question of grant of back wages would arise.
14

. In Jasbir Singh v. Punjab and Sind Bank and Ors. [(2007) 1 SCC 566], this Court
directed reinstatement in service with back wages, con tinuity of service and other
consequential benefits. {See also Gangadhar Pillai v. Siemens Ltd. [(2007) 1 SCC 533]}.
2006 AIR SCW 5779
2006 AIR SCW 6414
In General Manager, Haryana Roadways v. Rudhan Singh [(2005) 5 SCC 591], this Court
stated : 2005 AIR SCW 4634

"8. There is no rule of thumb that in ev ery case where the Industrial Tribunal gives a
finding that the termination of service was in violation of Section 25-F of the Act, en tire
back wages should be awarded. A host of factors like the manner and method of selection
and appointment i.e. whether af ter proper advertisement of the vacancy or inviting
applications from the employment exchange, nature of appointment, namely, whether ad
hoc, short term, daily wage, tem porary or permanent in character, any spe cial
qualification required for the job and the like should be weighed and balanced in tak ing a
decision regarding award of back wages. One of the important factors, which has to be
taken into consideration is the length of service, which the workman had rendered with
the employer. If the workman has rendered a considerable period of service and his
service are wrongfully termi nated he may be awarded full or partial back wages keeping
in view the fact that at his age and the qualification possessed by him he may not be in a
position to get another employment. However, where the total length of service rendered
by a workman is very small, the award of back wages for the com plete period i.e. from
the date of termina tion till the date of the award, which our experience shows is often
quite large, would be wholly inappropriate. Another important factor, which requires to
be taken into con sideration, is the nature of employment. A regular service of permanent
character can not be compared to short or intermittent daily-wage employment though it
may be for 240 days in a calendar year."
15

. The said decision as also the deci sion in Pearlite Liners (P) Ltd. v. Manorama Sirsi
[(2004) 3 SCC 172] have been rendered in a dif ferent fact situation, namely, the jurisdic
tion of the Labour Court under Section 11-A of the Industrial Disputes Act. The ques tion
as to what would be the relevant fac tors for the industrial court to grant the said relief
need not be the same for the writ court. For grant of back wages, this Court has laid down
several principles therefor. 2004 AIR SCW 273

16. However, in the facts and circum stances of this case, we are of the opinion that the
interest of justice will be subserved if the quantum of back wages is confined to Rs.75%
for the total period the respondent remained out of service.
17. Appeal is allowed to the aforemen tioned extent. This order is being passed keeping in
view the fact that the respondent has already been reinstated in service. In the facts and
circumstances of the case, there shall be no order as to costs.
Order accordingly.
AIR 2008 SUPREME COURT 3240 "Abdul Ghaffar v. State of Bihar"
(From : Patna)*
Coram : 2 S. B. SINHA AND PRAKASH PRABHAKAR NAOLEKAR, JJ.
Criminal Appeal No. 603 of 2008 (arising out of SLP (Cri.) No. 479 of 2007), D/- 4 -4
-2008.
Abdul Ghaffar and Anr., etc. v. State of Bihar.
Criminal P.C. (2 of 1974), S.439 - BAIL - Bail - Incident taking place out of dis pute as to
ownership and possession of land - Complainant party immediately
@page-SC3241
after incident lodging F.I.R. - Accused-appellants named therein as persons who fatally
assaulted members of complain ant party - Counter case pleaded by appellants - Refusal
of bail to appellants who were named in F.I.R. by Trial Court and High Court - Proper.
(Para 5)

D.N. Goburdhan, Ms. Pinky Anand and Ms. Geeta Luthra, for Appellants; Nagendra Rai,
Sr. Advocate, M.K. Choudhary, S.K. Verma and Gopal Singh, with him for Respondents.
* Cri. Misc. Nos. 18884 and 19226 of 2006, D/- 11-8-2006 (Pat).
Judgement
1. P. P. NAOLEKAR, J. :- Leave granted.
2. On 12.12.2005, an FIR was registered at Turkaulia P.S., District East Champaran,
Bihar on the basis of a complaint lodged by Vinay Kumar Sharma. As per the said FIR, at
about 1 o'clock in the afternoon he along with his brothers and nephew Ajay Sharma
reached the land which belongs to his fam ily. After some time, the accused-appel lants,
namely, Abdul Gaffar, Abdul Khair, Banaras Rai with other accused persons named in the
FIR along with 150 people reached the fields where the complainant party was present,
with arms and ammuni tion. Manan Dewan and Alauddin Dewan ordered their men to
kill the complainant and his family members. On this, Azhar Sah fired with a country-
made pistol, which struck on the head of the complainant. Then Abdul Gaffar, Abdul
Khair and Banaras Rai gave beatings to Ajay Sharma, nephew of the complainant with an
iron rod and a lathi, which ultimately resulted in his succumb ing to the injuries sustained
by him. The FIR further alleged the participation of Abdul Khair of having caused injury
to Loknath Sharma, brother of the complainant, with an iron rod.
3. As per the appellants, they are the owners of the lands measuring 75 bighas in village
Chilrawan, District East Champaran, Bihar, as these lands were part of the land settled by
Betia Raj with the ancestors of the appellants and they had mortgaged the property in the
year 1915. After the period of 60 years, under the Bihar Money Lend ers Act, the said
lands would revert back to the appellants and their family members. It is a bone of
contention of the appellantsthat on the relevant date they were in pos session of the
property and it was the com plainant party which had tried to take forc ible possession
over the land and that is how the incident took place in which five members of the
appellants' family were shot dead and several others, namely, Alauddin, Mohd. Siraj,
Banaras Rai, Zainuddin Sah, Abdul Ghaffar and Narayan Roy were badly injured by use
of firearms. The appellants contend that they had informed the police on 12.12.2005 itself
that the Sharma brothers including the District Judge who is one of the brothers had
planned to take over pos session of the property and had threatened to kill the appellants
like birds.
4. It is contended by the learned coun sel for the appellants that the appellants have been
falsely implicated as the other party was the attackers and in the process of their attack
five persons died and a num ber of other persons were seriously injured.

5. On a plain reading of the FIR which was lodged on the same day, i.e. on 12.12.2005, it
is apparent that the accused-appellants were named as the persons who had caused
serious injuries to Ajay Sharma, nephew of the complainant, who later on succumbed to
injuries. When the appel lants have been named in the FIR which was lodged
immediately after the incident, at this stage we do not find any infirmity in the im pugned
order of the High Court and that of the Sessions Court whereunder the accused-appellants
were not released on bail.
6. For the aforesaid reasons, we do not find any merit in the appeal. It is, accord ingly,
dismissed.
Appeal dismissed.
AIR 2008 SUPREME COURT 3241 "Poonam Kumari v. Jai Prakash Pandey"
(From : Patna)*
Coram : 2 Dr. A. PASAYAT AND P. SATHASIVAM, JJ.
Civil Appeal No.2871 of 2008 (arising out of SLP (C) No. 14039 of 2004), D/- 21 -4
-2008.
Poonam Kumari v. Jai Prakash Pandey and Ors.
Constitution of India, Art.14 - EQUALITY - ALLOTMENT OF PETROL PUMPS -
SUPREME COURT - Allot ment of Petrol Pump - Selection of deal ers by Dealer
Selection Board (DSB) - Allotment made - Procedural irregulari ties subsequently found
to be committed by DSB - Cancellation of allotment - Supreme Court refused to interfere
- Since DSB has ceased to be in existence direction issued that fresh consideration shall
be made by Selection Committee
@page-SC3242
nominated by the General Manager, IOC, Bihar. (Para 10)

Himanshu Shekhar Jha and Rameshwar Prasad Goya, for the Appellants; Dr. R.G. Padia,
Sr. Advocate, Ranjan Mukherjee, S.C. Ghosh, H.K. Puri, Ms. Priya Puri, S.K. Puri, V.M.
Chauhan, Tufali A. Khan, B.K. Prasad, M.P. Parmeshwaran and Shiv Sagar Tiwary, for
Respondents.
* L.P.A. No. 409 of 2004, D/- 7-4-2004 (Pat).
Judgement
1. Dr. ARIJIT PASAYAT, J. :- Leave granted. Challenge in this appeal is to the order of a
Division Bench of the Patna High Court dismissing the Letters Patent appeal filed by the
appellant.
2. Background facts in a nutshell are as follows :
On 1.9.2000, the Indian Oil Corporation Ltd. (in short the 'IOC') issued advertisement
inviting applications for appointment of a dealer in respect of certain retail outlets (petrol
pumps) in various places including one in Brahampur in State of Bihar. The appellant was
one of the applicants. The applications were verified by IOC and the applications of all
the eligible candidates were forwarded to the Dealer Selection Board (in short the 'DSB')
for making selection. The DSB issued interview letters to all those can didates who were
found eligible. It consid ered the materials placed before it by the applicants and
produced during interviews, and on the basis of the interview allegedly prepared a select
list on merits in the fol lowing order :
1. Smt. Poonam Kumari-Appellant,
2. Shri Dinesh Kumar Singh; and
3. Shri Anil Kumar.
On being placed at No. 1 in the Select List, a letter of Intent was issued on 8.11.2001 and
the necessary order was handed over to the appellant. She claims to have made
substantial investments in making the Re tail Outlet operational. The entire infrastruc ture
was put up by IOC including the ar rangement of the land, the oil tanks were installed and
certain persons were employed as members of staff and with effect from 12.11.2001,
appellant started operating the Retail Outlet.
One J. P. Pandey (Respondent No. 1), who was also one of the applicants and whose
name did not figure in the select list, filed a writ petition in the High Court challenging
the selection made by the DSB. There the main allegation was that even though his father
had made the land available to IOC, he was not given a preference in the matter of
allotment and appointment as a dealer. In the Writ petition appellant was also im pleaded
as a party. However, no notice was served on her. By a judgment dated 15.1.2004 the
Writ Petition was allowed and the selection made by the DSB was quashed. Since
appellant claimed that no notice was served on her and she was not in a position to place
her case before learned Single Judge who heard and allowed the Writ Petition, she filed
LPA No.93 of 2004. On 3.2.2004 after hearing the parties, the Division Bench dis posed
of the LPA observing as follows :
"On record it is clear and apparent and some of the Respondents were not before the Writ
Court to make a submission for the simple reason that they were without no tice.
This Court is of the opinion that it would be expedient and appropriate in the inter est of
justice that the appellant (respondent No. 6 in the Writ petition) is granted a lib erty to
apply for having the matter consid ered upon her case so that the Hon'ble Court may pass
such orders as the Court may deem fit and just on her application.
Regard being had to the circumstances of this case if an application-is filed for con
sideration of the writ court within a week, this matter will be placed as a fresh case."
Pursuant to the said order, an applica tion (MJC No.256 of 2004) was filed praying that
the order dated 15.1.2004 in the Writ Petition (C.W.J.C. No. 14506 of 2001) be recalled.
3. Learned Single Judge took up the matter on 3.3.2004 and after noticing the grievance
of the appellant held that though she was not afforded the opportunity of hearing before
the Writ Petition was allowed, there was no necessity for changing the ul timate decision.
The Appellant filed the LPA 401 of 2004 questioning the order passed. By the impugned
order the Division Bench of the Patna High Court held that since the matter has been
remitted to the DSB for fresh consideration, there was no illegality in the order.
4. Learned counsel for the appellant sub mitted that learned Single Judge accepted that
the appellant had not got the opportu nity of being heard. The earlier reasoning could not
have been repeated to dismiss the
@page-SC3243
application.
5. It is pointed out that the appellant was placed at serial No. 1 of the select list and had
been given permission to operate retail outlet and had made huge investments and
therefore her selection could not have been nullified by learned Single Judge. It is
therefore submitted that the LPA should have been allowed.
6. Learned counsel for the appellant has further pointed out that in view of the order
passed by this Court, and on account of the fact that the appellant has made huge in
vestments and had made the retail outlet operational and it was functioning, without any
reason the facility has been withdrawn. It was, therefore, prayed that pending dis posal of
the matter she should be permitted to operate.
7. On the other hand, learned counsel for the respondents submitted that noticing that
there were several irregularities, the DSB was asked to reconsider the matter.
8. It is pointed out that the writ peti tioner had brought to the notice, in the writ petition,
that his father had given land to IOC on lease for about thirty years with an option of
further renewal for thirty years. It was, therefore, pleaded that preference ought to have
been given to the writ-peti tioner, but had not been really given. It ap pears from the order
of the learned Single judge that he found that there were certain procedural irregularities
committed by the DSB and therefore a fresh consideration was warranted.
9. It is pointed out by learned counsel of the IOC that the DSB is not in existence since
9.5.2002. It is further pointed out that in another case, the Court directed that the matter
should be considered by high offi cials of IOC in its zonal office.
10. While declining to interfere in the matter, because of the procedural lapses noticed by
learned Single Judge, we direct that instead of DSB, which is no longer in existence, in
the line of what has been di rected by this Court in another case, we di rect that
consideration shall be made by the Selection Committee nominated by the Gen eral
Manager, IOC, Bihar State Office, Patna, who is stated to be the State Head. Let the
Committee deal with the matter expedi tiously. Since the matter is pending long, we
direct the Committee to consider the matter in its proper perspective by taking into
account all the materials already on record and to be placed by the parties. Let the
exercise be completed within a period of four months from today. We make it clear that
we have not expressed any opinion on the merits of the case.
11. Appeal is disposed of accordingly. No costs.
Order accordingly.
AIR 2008 SUPREME COURT 3243 "State of U. P. v. Ram Adhar"
(From : Allahabad)*
Coram : 2 H. K. SEMA AND MARKANDEY KATJU, JJ.
Civil Appeal No.5691 of 2002, D/- 10 -4 -2008.
State of U.P. and Anr. v. Ram Adhar.
(A) Constitution of India, Art.16 - EQUALITY IN PUBLIC EMPLOYMENT - Adhoc
appointee - Appointed for fixed pe riod - Continuance in service till regu lar appointment
is made - Not permis sible in law.
S. A. No. 185 of 2001, D/-14-08-2001 (All), Reversed.
1991 AIR SCW 793, Foll. (Paras 5, 6)
(B) Constitution of India, Art.16 - EQUALITY IN PUBLIC EMPLOYMENT -
APPOINTMENT - Appointment - Post requiring special skills - Criteria for appointment
- Should only be merit. (Para 7)
Cases Referred : Chronological Paras
1991 AIR SCW 793 (Foll.) 5
Fuzail Khan, Sehdev Singh, Anil Kumar Jha and Ravi Prakash Mehrotra, for Appel lants;
K. Sharda Devi, for the Respondents.
* S. A. No. 185 of 2001, D/- 14-8-2001 (All) (L.B.)
Judgement
JUDGMENT :- This appeal filed by the State is directed against the judgment and order
dated 14.08.2001 passed by the Divi sion Bench of the Allahabad High Court.
2. Heard the parties.
3. The respondent herein was appointed on ad hoc basis on the post of Stenographer for a
period of three months. The time was extended twice and ultimately, the respon dent also
appeared in the test but failed. The respondent preferred writ petition be fore the learned
Single Judge. The same was
@page-SC3244
disposed by the learned Single Judge allow ing the respondent to continue till the regu
larly selected Stenographer joins the post. The same was confirmed by the Division
Bench of the High Court.
4. While issuing notice on 15.10.2001 this Court stayed both the orders of the Di vision
Bench and the learned Single Judge. In view of the interim order, the respondent is no
more in service today. Even otherwise an ad hoc appointee appointed for a period of three
months as Stenographer, whose term is further extended, should not be al lowed to
continue in the public interest when he failed in the test.
5

. It may be mentioned that there is no principle of law that a person appointed in a


temporary capacity has a right to continue till a regular selection. Rather, the legal po
sition is just the reverse, that is, that a tem porary employee has no right to the post vide
State of U.P. v. Kaushal Kishore, (1991) 1 SCC 691. Hence, he has no right to continue
even for a day as of right, far from having a right to continue till a regu lar appointment.
1991 AIR SCW 793

6. On this sole ground we set aside both the orders of the learned Single Judge and the
Division Bench of the High Court. This appeal is allowed. No costs.
7. Before parting with this case we would like to mention that very often selection and
appointments are made on posts requiring special skills like that of a stenographer. On
such posts the only criterion should be merit. However, very often such appoint ments are
not made on merit but on some recommendations, and such appointees are very often
incompetent.
8. If an incompetent stenographer is appointed for the Court the result will be that the
correct order passed by the Judge will not be recorded, and this will create many
problems. Much of the time of the Judge will be spent on making corrections. Hence
great care must be taken by the se lection committee for selecting persons to be appointed
on posts requiring special skills like that of a stenographer purely on merit disregarding
any recommendation made by anyone, howsoever high.
Appeal allowed.
AIR 2008 SUPREME COURT 3244 "Mg. Dir. Bangalore Metropolitan Tpt. Corpn. v.
Sarojamma"
(From : Karnataka)*
Coram : 2 S. B. SINHA AND V. S. SIRPURKAR, JJ.
Civil Appeal No.2897 of 2008 (arising out of SLP (Civ.) No. 17647 of 2006), D/- 22 -4
-2008.
Mg. Dir., Bangalore Metropolitan Tpt. Corpn. v. Sarojamma and Anr.
(A) Motor Vehicles Act (59 of 1988), S.163A, Sch.2 - MOTOR VEHICLES - No fault
liability - Compensation - To be determined on struc tural formula - Reduction of income
of deceased towards personal expenses - To be generally by 1/3rd and not be 50%.
(Paras 7, 8)
(B) Motor Vehicles Act (59 of 1988), S.163A - MOTOR VEHICLES - Accident
compensation - Determination - Income of victim - Victim tutor in Army Teachers
Training Institute - Having qualification to become teacher - Assessing his income as Rs.
3000/-p.m. - Not excessive. (Para 8)
(C) Motor Vehicles Act (59 of 1988), S.168, S.173 - MOTOR VEHICLES - INTEREST -
Compensation - Award of interest - Mental agony suffered by claiment-mother - Not
ground to grant higher interest - Interest rate brought down from 10% to 7%. (Para
14)
Cases Referred : Chronological Paras
2004 AIR SCW 7475 (Ref.) 12
(1996) 4 SCC 362 (Ref.) 10
1994 AIR SCW 1356 : AIR 1994 SC 1631 (Ref.) 9
AIR 1962 SC 1 10
R. S. Hegde, Chandra Prakash, J.K. Nayyar and P.P. Singh, for Appellant; Ms. Kiran Suri,
for Respondents.
* MFA No. 5584 of 2003 (MV) C/W Cr. OB No. 236 of 2004 (MV), D/-28-03-2006
(Kar).
Judgement
1. S. B. SINHA, J :- Leave granted.
2. One Ravi Kumar (deceased) son of Respondent No. 1 was travelling in a bus
belonging to the appellant on 25.11.1998. It met with an accident. The deceased sus
tained injuries. He subsequently succumbed thereto. He was unmarried. He was aged
about 18 years. He left behind the respon dent No. 1 as his only heir and legal repre
sentative.
A claim petition was filed in terms of Sec tion 163-A of the Motor Vehicles Act, 1988 (for
short "the Act"). The Tribunal calculated
@page-SC3245
the loss of dependency at Rs.3,84,000/-, wherefor the multiplier of 16 was applied. The
Tribunal estimated the income of the deceased at Rs.3,000/- p.m. One-third was deducted
from the said amount towards his personal expenses. An appeal was preferred
thereagainst by the appellant. By reason of the impugned judgment, the High Court while
allowing the multiplier of 15 instead of 16 increased the rate of interest from 7% to 10%.
Respondent No. 1 was held to be entitled to a total sum of Rs. 3,64,500/- (Rs. 3,60,000 +
2,000 + 2,500).
3. Mr. R.S. Hegde, learned counsel ap pearing on behalf of the appellant would submit :
(i) There was no evidence to show that the deceased was earning a sum of Rs. 3,000/-
p.m.
(ii) The age of the respondent No. 1 being 45 as on the date of accident, the High Court
committed a serious error in applying the multiplier of 15; as the deceased was a bach
elor
(iii) The claimant being his mother, the Tribunal as also the High Court should have
deducted 50% of the amount from his in come.
(iv) The High Court committed a serious error in enhancing the rate of interest from 7%
to 10% wherefor no justification has been shown.
4. Ms. Kiran Suri, learned counsel ap pearing on behalf of the respondents, on the other
hand, would urge :
(i) It is not a fit case where this Court should exercise its discretionary jurisdiction under
Article 136 of the Constitution of In dia.
(ii) Keeping in view the fact that the mother has lost her only son, the Tribunal should
have awarded compensation towards loss of estate and loss of love and affection.
(iii) As deduction of one-third towards personal expenses is applied in all cases, the
impugned judgment should not be in terfered with.
(iv) Keeping in view the fact that the acci dent had taken place in the year 1998, grant of
10% interest was wholly justified.
5. Section 163-A of the Act was inserted by Act No. 54 of 1994 with effect from
14.11.1994. For invoking the said provision, it is not necessary for a claimant to estab lish
any act of negligence on the part of the driver. It is not necessary even to plead that the
death had occurred owing to any wrong ful act or neglect or default of owner of the
vehicle.
6. Quantum of compensation is to be determined in terms of the Schedule II ap pended
thereto. In terms thereof, apart from the amount of compensation as provided for therein
only funeral expenses, loss of con sortium (if beneficiary is the spouse), loss of estate,
medical expenses, would be payable.
7. As the Schedule II provides for a struc tured formula, ordinarily, the same has to be
adhered to. The structured formula it self stipulates reduction of income of the de ceased
by one-third in consideration of the expenses which he would have incurred to wards
maintaining himself, had he been alive.
8. Whereas in determining an applica tion for grant of compensation under Sec tion 166
of the Act, the Tribunal may be entitled to find out actual loss of damages suffered by the
claimants, the formula hav ing not envisaged such a contingency, we are of the opinion
that ordinarily one-third should be deducted from the income of the deceased and not the
half thereof.
For determining the amount of compen sation, the most relevant factor, therefore, is the
income of the deceased. He was a tu tor. He was admitted in the Army Teachers Training
institute. He had the requisite po tential of becoming a teacher. His income, thus, having
been estimated at Rs. 3,000/-p.m. cannot be said to be on a very high side.
9

. This Court in General Manager, Kerala State Road Transport Corpora tion, Trivandrum
v. Susamma Thomas (Mrs.) and Others [(1994) 2 SCC 176) held as under :1994 AIR
SCW 1356, (Para 7)

"9. The assessment of damages to com pensate the dependants is beset with diffi culties
because from the nature of things, it has to take into account many impon derables, e.g.,
the life expectancy of the de ceased and the dependants, the amount that the deceased
would have earned during the remainder of his life, the amount that he would have
contributed to the dependants during that period, the chances that the deceased may not
have lived or the dependants may not live up to the estimated remaining period of their
life expectancy, the chances that the deceased might have got
@page-SC3246
better employment or income or might have lost his employment or income altogether."
10
. This aspect of the matter has also been considered in U.P. State Road Trans port
Corporation arid Others v. Trilok Chandra and Others [(1996) 4 SCC 362] by a Three-
Judge Bench of this Court in the following terms : AIR 1962 SC 1

"9. The compensation to be awarded has two elements. One is the pecuniary loss to the
estate of the deceased resulting from the accident, the other is the pecuniary loss sus
tained by the members of his family for his death. The Court referred to these two ele
ments in the Gobald Motor Service's case. These two elements were to be awarded un der
Section 1 and Section 2 of the Fatal Ac cidents Act, 1855 under which the claim in that
case arose. The Court in that case cau tioned that while making the calculations no part of
the claim under the first or the second element should be included twice. The Court gave
a very lucid illustration, which can be quoted with profit :
An illustration may clarify the position. X is the income of the estate of the deceased, Y is
the yearly expenditure incurred by him on his dependents (we will ignore the other
expenditure incurred by him). X-Y i.e. Z, is the amount he saves every year. The capit
alised value of the income spent on the de pendents, subject to relevant deductions, is the
pecuniary loss sustained by the mem bers of his family through his death. The capitalised
value of his income, subject to relevant deductions, would be the loss caused to the estate
by his death. If the claimants under both the heads are the same, and if they get
compensation for the entire loss caused to the estate, they can not claim again under the
head of personal loss the capitalised income that might have been spent on them if the
deceased were alive. Conversely, if they got compensation under Section 1, representing
the amount that the deceased would have spent on them, if alive, to that extent there
should be de duction in their claim under Section 2 of the Act in respect of compensation
for the loss caused to the estate. To put it differ ently if under Section 1 they got
capitalised value of Y, under Section 2 they could get only the capitalised value of Z, for
the capi talised value Y + Z = X would be the capital ised value of his entire income."
11. What should be the legal principle on which the principle of just compensation should
be worked out had been the subject matter of various decisions of this Court. This court
in cases after cases noticed that the principles on which the multiplier method was
developed has been given a go by. In many cases, a hybrid method based on the
subjectivity of the Tribunal has been noticed. Guidelines provided for by the stat utes as
also the Superior Court have not been applied. The courts have also noticed several
defects in the schedule. It was opined that ordinarily the multiplier should not exceed 16.
12

. Our attention has also been drawn to a decision of this Court in Fakeerappa and Another
v. Karnataka Ce ment Pipe Factory and Others [(2004) 2 SCC 473] wherein it was held :
2004 AIR SCW 7475

"7. What would be the percentage of de duction for personal expenditure cannot be
governed by any rigid rule or formula of universal application. It would depend upon
circumstances of each case. The deceased undisputedly was a bachelor. Stand of the
insurer is that after marriage, the contribu tion to the parents would have been lesser and,
therefore, taking an overall view the Tribunal and the High Court were justified in fixing
the deduction.
8. It has to be noted that the ages of the parents as disclosed in the Claim Petition were
totally unbelievable. If the deceased was aged about 27 years as found at the time of post
mortem and about which there is no dispute, the father and mother could not have been
aged 38 years and 35 years respectively as claimed by them in the Claim Petition. Be that
as it may, taking into ac count special features of the case we feel it would be appropriate
to restrict the deduc tion for personal expenses to one-third of the monthly income.
Though the multiplier adopted appears to be slightly on the higher side, the plea taken by
the insurer cannot be accepted as there was no challenge by the insurer to the fixation of
the multiplier before the High Court and even in the ap peal filed by the appellants before
the High Court the plea was not taken."
13. No finding has been arrived at by the Tribunal that the age of the claimant was 45 or
below. Why the multiplier of 16 had been applied by the Tribunal was not stated. The
High Court has also not laid down the legal premise upon which it had
@page-SC3247
applied the multiplier of 15. It, however, appears that the learned counsel for the
appellant himself stated that the correct multiplier would be 15 and not 16 which has
been accepted by the High Court. We do not, therefore intend to interfere with the said
finding in the instant case.
14. The High Court, however, took into consideration an irrelevant factor, viz., that the
claimant must have been suffering from a mental agony in determining the rate of interest
as also the age of the deceased. We do not see any justification for increase in the rate of
interest. We, therefore, are of the opinion that the interest of justice would be subserved if
the rate of interest payable on the awarded amount is brought down to 7%, as was
directed by the Tribunal.
15. The appeal is allowed only to the aforementioned extent. No costs.
Order accordingly.
AIR 2008 SUPREME COURT 3247 "Udai Raj Singh v. Hari Ram"
(From : Allahabad)*
Coram : 3 Dr. A. PASAYAT, P. SATHASIVAM AND AFTAB ALAM, JJ.
Civil Appeal Nos. 6602-6603 of 2001, D/- 21 -4 -2008.
Udai Raj Singh and Ors. v. Hari Ram and Ors.
Civil P.C. (5 of 1908), O.9, R.9 - DECREE - DISMISSAL - INJUNCTION - Dis missal
of suit for default - Consequences - Not any way less than dismissal of suit on merits -
Entitlement of respondents to Sirdari right - Injunction suit by ap pellants against
respondents dismissed for default - Order of consolidation au thorities and High Court yet
holding that respondent was not entitled to sirdari rights - Improper.
C.M.P. No. 4413 of 1978, D/-25-11-1997 (All) and C.M.R.P. No. 6816 of 1998, D/-06-
08-1998 (All), Reversed. (Paras 4, 5, 6)

Manoj Swarup and Ms. Lalita Kohli (for M/s. Manoj Swarup and Co.), for Appellants;
B.S. Banthia, for Respondents.
* CMP No. 4413 of 1978 and CM Rev. Petn. No.6816 of 1998, D/- 25-11-1997 and D/-
6-8-1998 (All.) respectively.
Judgement
1. AFTAB ALAM, J. :-These two ap peals, one against the judgment of the Allahabad
High Court in the main Writ Peti tion (Civil Misc. Writ Petition No.4413/78) and the
other against the order in the Review Petition (Civil Misc. Review Petition No.6816 of
1998) filed by the appellants arise from the proceedings under the U.P. Con solidation of
Holdings Act. The subject-mat ter of the dispute are three plots bearing Nos.641, 642 and
803 situate at village Barhat, Pargana Shakiabad, District Ghazipur. The contest over the
disputed land had begun between Kedar and Badri sons of Sarju Ram and Ghurahu
respectively on the one side and Baij Nath son of Daroga Singh and others on the other.
In course of the proceedings the original contestants died and the litigation was carried on
by their respective heirs and legal representatives. The appellants before this Court are
the sons of Daroga Singh and Baij Nath and the re spondents are the sons of Kedar, Badri
and Rupu.
2. In the basic year the disputed land was undeniably recorded in the names of Sarju and
Ghurahu i.e. predecessors-in-in terest of the respondents. The village where the land is
situated was brought under con solidation operations by notification, dated October 2,
1956. In the consolidation pro ceedings the Assistant Consolidation Officer by his order
dated June 7, 1959, found Baij Nath and others in possession of the dis puted land. He
held them as Sirdars and directed for entering their names in respect of the disputed plots.
No appeal was filed against the order passed by the Assistant Consolidation Officer and
the matter was allowed to rest at that stage. Later on, Kedar and Badri filed an objection
under Section 12 of the Act claiming that they were the bhumidars, in possession of the
disputed plots and the names of Daroga Singh and others were wrongly recorded in
respect of those plots in the revenue records. The Con solidation Officer found that the
dispute in volved the question of title and by his order, dated October 3, 1961 referred the
matter to the Civil Judge. The Civil Judge by his order, dated October 5, 1962 referred
the matter for arbitration. The Arbitrator by his Award, dated August 12, 1964 held that
Badri Ghurahu and others were the bhumidars of the disputed land and further found that
Baij Nath and others were not the Sirdars of the plots in dispute. The Civil Judge
confirmed the Award by order, dated January 8, 1965. Against the order of the Civil
Judge, Baij Nath and others filed a re vision before the High Court and the High Court
remanded the matter to the
@page-SC3248
Consolidation Officer for deciding the question of Sirdari rights claimed by them.
3. The Consolidation Officer framed is sues in light of the remand order and on a
consideration of the materials produced by the two sides passed the order, dated April 13,
1973 holding that Baij Nath was Sirdar of the plots in dispute and the entry of his name
should be maintained. He gave direc tions accordingly. Against the order passed by the
Consolidation Officer the side of Kedar and Badri took the matter in appeal before the
Settlement Officer, Consolidation, Ghazipur. The Settlement Officer by order, dated
December 1, 1973 allowed the appeal and reversed the finding of the Consolida tion
Officer. The order of the Settlement Of ficer was based mainly on the revenue records.
Against the Appellate order the side of Baij Nath filed a revision before the Deputy
Consolidation Officer, Ghazipur. In the re vision one of the grounds raised by them was
that earlier the other side had filed Suit No.658 of 1955 seeking a decree of perma nent
injunction against the defendants (the side of Baij Nath) in regard to the disputed plots.
The suit was dismissed by the Civil Court and hence, it was no longer open to the other
side to question or challenge their possession of the disputed land.
4. The revisional authority did not con sider the aforesaid point raised by the ap pellants
or for that matter any other points raised on their behalf and disposed of the revision in a
somewhat curious manner. It noted the facts of the case and the points urged on behalf of
the two sides and then abruptly gave the operative direction under the heading 'order'.
There is no indication in the order why the revisional authority deemed fit to uphold the
appellate order or why he accepted the case of Kedar and Badri in preference to the pleas
raised by the side of Baij Nath. On a plain reading the revisional order appears to us to be
quite unsustainable. The appellants then took the matter to the High Court. But before the
High Court too the writ petition was dis missed without any proper consideration of the
pleas raised on their behalf. The High Court simply referred to some of the obserÂvations
made by the Settlement Officer and dismissed the writ petition. In review, the point with
regard to the earlier suit, being title Suit No.658 of 1955 was specifically pressed but the
High Court brushed it aside by observing that since the suit was dismissed for default
there was no decision on merits deciding the rights of the parties.
5. We are afraid, that can hardly be ac cepted as a proper consideration of the con
sequences of the dismissal of the suit be tween the same parties in regard to the dis puted
land.
6. On hearing counsel for the appellant and on going through the materials on record,
including the orders coming under challenge we are satisfied that the revisional order and
the two orders passed by the High Court are unsustainable in law. We, accord ingly, set
aside the order dated February 10, 1978 passed by the ADM/Deputy Consoli dation
Officer, Ghazipur in Revision No. 16 as well as the orders of the High Court, dated
November 25, 1997 in Civil Misc. Writ Peti tion No.4413 of 1978 and the other, dated
August 6, 1998 in Civil Misc. Review Peti tion No.6816 of 1998 and remit the matter to
the revisional authority.
7. As the matter has become very old it is expected that the revisional authority will
finally dispose of the matter after hearing both the sides in accordance with law within
four months from the date of the receipt of the order.
8. In the result the appeals are allowed but with no order as to costs.
Appeals allowed.
AIR 2008 SUPREME COURT 3248 "Raj Kumar v. State of Uttaranchal"
(From : Uttaranchal)*
Coram : 2 C. K. THAKKER AND D. K. JAIN, JJ.
Criminal Appeal No.855 of 2007, D/- 7 -4 -2008.
Raj Kumar alias Raju v. State of Uttaranchal (now Uttarakhand)
Penal Code (45 of 1860), S.396 - DACOITY - MURDER - Dacoity with murder - Six
persons al leged to have committed offence - Two of them simply acquitted - Remaining
four accused could not be convicted for offence of dacoity, being less than five which is
essential ingredient for commis sion of dacoity - Moreover all accused were acquitted for
commission of offence of criminal conspiracy as also of receiv ing stolen property and
said acquittal has attained finality - Conviction of accused u/S.396, liable to be set aside.
@page-SC3249

Cri. Appeal No. 315 of 2001, D/-16 10-2006 (Utr.). Reversed. (Paras 34, 35)
Cases Referred : Chronological Paras
2003 AIR SCW 6717 : AIR 2004 SC 1043 : 2004 Cri LJ 635 (Ref.) 28
(1994) 4 SCC 381 : 1993 (3) JT (SC) 324 (Disting.) 33
AIR 1990 SC 1180 : 1990 Cri LJ 1183 (Disting.) 32
AIR 1983 SC 352 (1) : 1983 Cri LJ 691 (1) (Ref.) 16
AIR 1976 SC 1084 : 1976 Cri LJ 835 (Ref.) 26
AIR 1973 SC 760 :1973 Cri LJ 599 (Ref.) 17
1964 (1) Cri LJ 573 (SC) (Ref.) 25
AIR 1963 SC 174 : 1963 (1) Cri LJ 100 (Ref.) 23
AIR 1963 SC 1413 : 1963 (2) Cri LJ 351 (Ref.) 24
AIR 1957 SC 320 : 1957 Cri LJ 416 (Ref.) 31
AIR 1956 SC 441 : 1956 Cri LJ 822 (Ref.) 29
AIR 1953 SC 364 : 1953 Cri LJ 1465 (Ref.) 21, 22, 23
Sushil Kumar, Sr. Advocate, Praveen Kumar Rai, Vinay Arora and Abhisth Kumar, with
him for Appellant; B.N. Jha and J.K. Bhatia, for Respondent.
* Cri. Appeal No. 315 of 2001, D/- 16-10-2006 (Utr).
Judgement
C. K. THAKKER, J. :- This appeal is filed by Raj Kumar @ Raju-accused No. 1 in
Sessions Trial No. 14 of 1983. He was con victed for an offence punishable under Sec
tion 396 of the Indian Penal Code (IPC) along with three other accused and ordered to un
dergo rigorous imprisonment for ten years and to pay fine of Rs.2,000, in default of
payment of fine, to undergo additional im prisonment for one year. The said order was
passed by the II Addl. Sessions Judge, Nainital on August 9, 1985. The appellant
preferred an appeal against the said judg ment and order which was registered as Crimial
Appeal No. 2128 of 1985 in the High Court of Allahabad and was re-numbered as
Criminal Appeal No. 315 of 2001 in the High Court of Uttaranchal at Nainital and was
dismissed by the High Court on Decem ber 16, 2006.
2. The case of the prosecution was that on September 24, 1982 one Tilak Raj-PW1
submitted a written report at Police Station, Khatima stating therein that when he reached
his house for taking lunch at about 12.30 p.m., he saw Raj Kumar @ Raju, Pushpendra
Singh, Swadesh Chandra @ Gappu and Nirankar in his house. All the accused were
dragging his wife who was soaked in blood. According to Tilak Raj, on seeing him, the
accused fled away. Though he tried to catch hold of them, Pushpendra Singh pushed at
his chest due to which he fell down. He raised hue and cry. According to the complainant
Raj Kumar, Swadesh Chandra @ Gappu and Nirankar were hold ing knives. All the
accused committed loot of articles from his house and also commit ted murder of his wife
Kamlesh Kumari. Rajendra Kumar-PW6, Purshottam Lal PW3, Prem Kumar-PW5 and
other persons who came there, also saw the accused run ning away. The first information
was re corded and investigation was conducted. Accused were arrested and certain
articles were also recovered from them. On comple tion of investigation, charge-sheet
was sub mitted by the Investigating Officer against four accused referred to above and
two oth ers, namely, Mohan Lal and Balram Singh for offences punishable under Sections
396 and 412 read with Section 120B, IPC. The case was committed by the Magistrate to
learned II Addl. Sessions Judge. Charge was framed and the accused were asked whether
they pleaded guilty upon which they denied to have committed any offence and claimed
to be tried.
3. In order to prove the case against the accused, the prosecution examined 13 wit nesses.
Out of them Tilak Raj-PW1, infor mant and husband of deceased Kamlesh Kumari was
an eye-witness who supported the case of the prosecution. According to him, he saw all
the accused dragging dead body of his wife Kamlesh Kumari from kitchen. He wanted to
catch hold of the ac cused but could not do so as Pushpendra Singh pushed him and
resultantly he fell down. It was also his case that on hearing his cries, PW3-Purshottam
Lal, PW5-Prem Kumar and PW6-Rajendra Kumar reached at the scene of offence and
saw all the ac cused running away. After the arrest of ac cused, looted property and
weapons of of fence, namely, knives were recovered.
4. It was also the case of the prosecu tion that there was a conspiracy to commit dacoity
by all the accused on the previous day i.e. September 23, 1982. To prove con spiracy, the
prosecution examined PW7 Kishan Lal-real brother of PW1-Tilak Raj.
@page-SC3250
PW11-Dr. J.K. Goel who performed post mortem of deceased Kamlesh Kumari on
September 25, 1982 was examined to prove injuries sustained by the deceased and to
establish that she died due to those inju ries.
5. The case of the accused under Sec tion 313 of the Code of Criminal Procedure, 1973
was of total denial. In support of their case, the accused examined DW1-Dr. Joshi and
DW2-Laxmi Datt.
6. The trial Court, after appreciating the evidence on record, held that it was not proved
by the prosecution that there was conspiracy between the accused to commit dacoity. So
far as charge for committing an offence punishable under Section 120B, IPC is
concerned, the trial Court acquitted them. Likewise, the trial Court held that since re
covery effected by the prosecution was not in consonance with law, it could not be said
that stolen articles of dacoity were found from the accused and consequently charge for
an offence punishable under Section 412, IPC also could not be said to be established. All
the accused were, therefore, acquitted.
7. The Court, however, held that as far as commission of an offence punishable under
Section 396 was concerned, from the evidence of PW1-Tilak Raj and PW3 Purshottam
Lal, PW5-Prem Kumar and PW6-Rajendra Kumar, it was clearly estab lished. According
to the Court, however, an offence punishable under Section 396, IPC was established
against accused Nos. 1 to 4 Raj Kumar @ Raju-A1, Pushpendra Singh A2, Swadesh
Chandra @ Gappu-A3 and Nirankar-A4. It was not established that Mohan Lal-A5 and
Balram Singh-A6 were also present and party to the crime. They were, therefore, ordered
to be acquitted.
8. All the four convicted accused pre ferred appeal and the High Court, as stated above,
confirmed the order of conviction and sentence recorded by the trial Court and dis missed
the appeal.
9. Notice was issued by this Court in the present appeal instituted by accused No. 1 Raj
Kumar @ Raju on February 9, 2007. On July 9, 2007, leave was granted. Bail, how ever,
was refused. In view of the said fact, the matter was placed for final hearing and that is
how the matter is before us.
10. We have heard learned counsel for the parties.
11. The learned counsel for the appellant contended that both the Courts have committed
an error in convicting the appel lant for an offence punishable under Sec tion 396, IPC. It
was submitted that charge was framed for an offence punishable un der Section 396, IPC
against the appellant along with other five accused. When the trial Court acquitted two of
them, no conviction could have been recorded of the remaining four accused for an
offence punishable un der Section 396, IPC. It was submitted that for conviction of
accused under Section 396, IPC, there must be five or'more persons and in view of
acquittal of two out of six per sons, both the Courts were wrong in invok ing and
applying Section 396, IPC. On that ground alone, the order of conviction and sentence
deserves to be set aside. It was also submitted that all the accused were charged for
criminal conspiracy but they were ac quitted for an offence punishable under Sec tion
120B, IPC and the said order was not challenged by the State. Again, all the ac cused
were charged for an offence of dis honestly receiving property stolen in the commission
of dacoity, punishable under Section 412, IPC, but even on that count, the trial Court
acquitted them and the said order also has attained finality. In the light of the above facts,
conviction of the appel lant under Section 396, IPC is clearly illegal and requires to be set
aside. It was stated by the learned counsel that the appellant was never charged for
commission of an of fence of murder of deceased Kamlesh Kumari, punishable under
Section 302, IPC and the Courts went wrong in convicting him under Section 396, IPC.
The counsel sub mitted that the appeal deserves to be allowed by setting aside the order
of conviction and sentence recorded by the trial Court and confirmed by the High Court.
12. The learned Government Pleader, on the other hand, supported the order of con
viction and sentence by both the Courts. He submitted that it is true that all the accused
were acquitted by the trial Court for com mitting an offence punishable under Sec tion
120B, IPC (criminal conspiracy). It is also true that they were acquitted of an of fence
punishable under Section 412, IPC (for dishonestly receiving or retaining property stolen
in commission of dacoity). It is equally true that out of six accused, two were ac quitted
by the trial Court by giving benefit of doubt and hence there were less than five accused
before the Court. But from the prosecution
@page-SC3251
evidence and particularly from the evidence of PW1-Tilak Raj, PW3-Purshottam Lal,
PW5-Prem Kumar and PW6-Rajendra Kumar, the Courts below were convinced that
there was dacoity in the house of PW1 Tialk Raj and during the course of commit ting
dacoity, the accused killed Kamlesh Kumari, wife of PW1-Tilak Raj and convicted the
appellant under Section 396, IPC. That order cannot be said to be illegal or unlaw ful.
Alternatively, the learned counsel con tended that the accused had committed murder of
Kamlesh Kumari and they can independently be convicted for an offence punishable
under Section 302, IPC simplic iter. He, therefore, submitted that the ap peal deserves to
be dismissed.
13. Having heard learned counsel for the parties, in our opinion, the appeal deserves to be
allowed. It is not in dispute that charges were framed against six persons but even the trial
Court was not convinced with the prosecution evidence about complicity of all the
accused and granted benefit of doubt to two of them. It may be stated that the trial Court
did not record a finding that there were six persons who committed dacoity and out of
them, two accused could not be iden tified but the remaining four were identified and
came to the conclusion that it was proved that there were six accused and all of them
committed the offence of dacoity but in view pf insufficient evidence as to iden tity, two
of them were required to be acquit ted. In such case, conviction of less than five accused
can be sustained as in accor dance with law. But, once the Court doubts the presence and
participation of two out of six accused and grants benefit to them, there are less than five
persons and no conviction can be recorded for an offence under Sec tion 396, IPC against
them. Let us consider the legal position on this aspect.
14. Chapter XVII (Sections 378 to 462) deals with offences against property. Sec tions
378 to 382 relate to theft. Sections 383 to 389 concern offences of extortion. Sec tions
390 to 402 deal with robbery and dacoity. Section 391 defines dacoity and it reads thus :
391. Dacoity
When five or more persons conjointly commit or attempt to commit a robbery, or where
the whole number of persons con jointly committing or attempting to commit a robbery,
and persons present and aiding such commission or attempt, amount to five or more,
every person so committing, at tempting or aiding, is said to commit "dacoity".
15. Whereas Section 395 provides pun ishment for dacoity, Section 396 prescribes
penalty for an offence of dacoity with mur der. The said section reads thus :
396. Dacoity with murder
If any one of five or more persons, who are conjointly committing dacoity, commits
murder in so committing dacoity, every one of those persons shall be punished with
death, or imprisonment for life, or rigorous imprisonment for a term which may extend to
ten years, and shall also be liable to fine.
16

. In Ram Lakhan v. State of Uttar Pradesh, (1983) 2 SCC 65, this Court held that convic
tion for an offence of dacoity of less than five persons is not sustainable. In that case, the
appellant was convicted for an offence punishable under Section 395, IPC and sen tenced
to seven years rigorous imprison ment. FIR was registered against nine per sons. The trial
Court, however, acquitted five persons and convicted four. On appeal, the High Court
acquitted three persons out of four and conviction of one of the accused, appellant before
this Court, was upheld. This Court, while allowing the appeal and acquit ting the accused,
held that before an offence under Section 395 can be made out there must be an assembly
of five or more per sons. On the findings of the courts below, it was manifest that only
one person was left, who could not be convicted for an offence under Section 395.AIR
1983 SC 352 (1)

17

. In Saktu and Ann v. State of U.P., (1973) 1 SCC 202, the case of the prosecution was
that 15-16 persons entered the house of one Jwala Prasad and looted the property. First
Information Report was lodged by the infor mant-Jwala Prasad. All the accused were
charged for offences punishable under Sec tions 395, 397 and 412, IPC. The trial Court
acquitted one of the accused. In appeal, the High Court of Allahabad acquitted some other
accused but convicted three accused (Nos. 1, 6and 7). AIR 1973 SC 760

18. It was contended before this Court that as the High Court found that only three
persons had participated in the occurrence, there was an error in convicting them for
dacoity, since the offence of dacoity could
@page-SC3252
not be committed by less than five persons. This Court, however, negatived the conten
tion observing as under:
"The charge in the instant case is that apart from the named seven or eight per sons, there
were five or six others who had taken part in the commission of the dacoity. The
circumstance therefore that all except the three accused, have been acquitted by the High
Court will not militate against the conviction of those three for dacoity. It is important
that it was at no time disputed that more than thirteen or fourteen persons had taken part
in the robbery. The High Court acquitted a large number of the accused because their
identity could not be established. The High Court, however, did not find that the group
which committed robbery in the house of Jwala Prasad con sisted of less than five
persons".
(Emphasis supplied)
19. It is thus clear that for recording conviction of an offence of robbery, there must be
five or more persons. In absence of such finding, an accused cannot be con victed for an
offence of dacoity. In a given case, however, it may happen that there may be five or
more persons and the factum of five or more persons is either not disputed or is clearly
established, but the Court may not be able to record a finding as to identity of all the
persons said to have committed dacoity and may not be able to convict them and order
their acquittal observing that their identity is not established. In such case, conviction of
less than five persons - or even one - can stand. But in absence of such finding, less than
five persons cannot be convicted for an offence of dacoity.
20. A similar situation arises in dealing with cases of 'unlawful assembly' as defined in
Section 141, IPC and the liability of every member of such unlawful assembly for an
offence committed in prosecution of com mon object under Section 149, IPC. Section
141 indicates that an assembly of five or more persons can be said to be 'unlawful
assembly', if common object of the persons comprising such assembly is as mentioned in
the said section. Section 149 declares that if an offence is committed by any member of
unlawful assembly in prosecution of com mon object of that assembly, every member of
such assembly is guilty of that offence.
21

. In Dalip Singh and Ors. v. State of Punjab, 1954 SCR 145, it was held that if the
prosecution fails to establish that the appellants were five or more in number. Section
149, IPC cannot be applied. But the Court held that it is not essential that five persons
must always be convicted for invocation of the said provi sion. Where it is possible to
conclude that though five or more persons were 'unques tionably' at the place of offence
and the iden tity of one or more was in doubt, conviction of less than five persons with
the aid of Sec tion 149, IPC would be legal and lawful. AIR 1953 SC 364

22

. Speaking for the Court, Bose, J. stated : AIR 1953 SC 364, Paras 19 and 20

"Before section 149 can be called in aid, the court must find with certainty that there were
at least five persons sharing the common object. A find ing that three of them "may or
may not have been there" betrays uncertainty on this vi tal point and it consequently
becomes im possible to allow the conviction to rest on this uncertain foundation.
This is not to say that five persons must always be convicted before Section 149 can be
applied. There are cases and cases. It is possible in some cases for Judges to con clude
that though five were unquestionably there the identity of one or more is in doubt. In that
case, a conviction of the rest with the aid of Section 149 would be good. But if that is the
conclusion it behoves a court, particularly in a murder case where sen tences of
transportation in no less than four cases have been enhanced to death, to say so with
unerring certainty. Men cannot be hanged on vacillating and vaguely uncer tain
conclusions".
(Emphasis supplied)
23

. Again, in Mohan Singh v. State of Rajasthan, (1962) Supp 3 SCR 848, two of the five
per sons who were tried together for offences punishable under Section 302 read with
Sections 147 and 149, IPC were con victed. In the charge, those five accused per sons and
none others were mentioned as forming unlawful assembly and the evidence led in the
case was confined to them. The question was whether two persons could be convicted by
applying Section 149, IPC. The Court, referring to Dalip Singh, stated : AIR 1963 SC
174, Para 9
AIR 1953 SC 364

"Cases may also arise where in the charge, the prosecution names five or more persons
and alleges that they constituted
@page-SC3253
an unlawful assembly. In such cases, if both the charge and the evidence are confined to
the persons named in the charge and out of the persons so named two or more are
acquitted leaving before the court less than five persons to be tried, then Section 149
cannot be invoked. Even in such cases, it is possible that though the charge names five or
more persons as composing an unlawful assembly, evidence may nevertheless show that
the unlawful assembly consisted of some other persons as well who were not identified
and so not named. In such cases, either the trial court or even the High Court in appeal
may be able to come to the con clusion that the acquittal of some of the per sons named
in the charge and tried will not necessarily displace the charge under sec tion 149 because
along with the two or three persons convicted were others who com posed the unlawful
assembly but who have not been identified and so have not been named. In such cases,
the acquittal of one or more persons named in the charge does not affect the validity of
the charge under section 149 because on the evidence the court of facts is able to reach
the conclu sion that the persons composing the unlaw ful assembly nevertheless were five
or more than five".
24

. Similarly, in Krishna Govind Patil v. State of Maharashtra, (1964) 1 SCR 678, after
referring to Mohan Singh, the Court observed : AIR 1963 SC 1413 at P. 1416, Para 7
"It may be that the charge discloses only named persons; it may also be that the
prosecution witnesses named only the said ac cused; but there may be other evidence,
such as that given by the court witnesses, defence witnesses or circumstantial pieces of
evidence, which may disclose the exist ence of named or unnamed persons, other than
those charged or deposed to by the prosecution witnesses, and the court, on the basis of
the said evidence, may come to the conclusion that others, named or unnamed, acted
conjointly along with one of the ac cused charged. But such a conclusion is re ally based
on evidence".
25

. In yet another decision in Ram Bilas Singh v. State of Bihar, (1964) 1 SCR 775, this
Court said : 1964 (1) Cri LJ 573

"The decisions of this Court quoted above thus make it clear that where the prosecu tion
case as set out in the charge and as supported by the evidence is to the effect that the
alleged unlawful assembly consists of five or more named persons and no oth ers, and
there is no question of any partici pation by other persons not identified or identifiable it
is not open to the court to hold that there was an unlawful assembly un less it comes to
the definite conclusion that five or more of the named persons were members thereof.
Where, however, the case of the prosecution and the evidence adduced indicates that a
number in excess of five persons participated in the incident and some of them could not
be identified, it would be open to the court to convict less than five of the offence of
being members of the unlawful assembly or convict them of the offence committed by
the unlawful as sembly with the aid of Section 149, I.P.C. provided it comes to the
conclusion that five or more persons participated in the inci dent".
26

. In Maina Singh v. State of Rajasthan, (1976) 2 SCC 827, the appellant was charged
along with four others under Section 302 read with Section 149, IPC. Only the appellant
was convicted and the rest were acquitted. He was convicted under Section 302 read with
Section 34, IPC. There was no indication either in the FIR or in the evidence that any
other person unnamed or unidentified other than the five charged, to have participated in
the crime. The appellant challenged his conviction. AIR 1976 SC 1084

27. Setting aside the conviction for an offence punishable under Section 302 read with
Section 34, IPC, this Court held that if in a given case, the charge discloses only the
named persons as co-accused and the prosecution witnesses confine their testi mony to
them, even then it would be per missible to come to a conclusion that oth ers, named or
unnamed, besides those men tioned in the charge or the evidence of the prosecution
witnesses, acted conjointly with one of the charged accused if there is other evidence to
lead to that conclusion, but not otherwise.
28

. In Ram Dular Rai v. State of Bihar, (2003) 12 SCC 352 : JT 2003 (9) SC 301, this Court
said; 2003 AIR SCW 6717, Para 7
"Coming to the question whether Section 149 has application when presence of more
than five persons is established, but only four are identified, Section 149 does not require
@page-SC3254
that all the five persons must be iden tified what is required to be established is the
presence of five persons with a common intention of doing an act. If that is estab lished
merely because the other persons present are not identified that does not in any way affect
applicability of Section 149, IPC".
29. The learned counsel appearing for the State, however, referred to Ram Shankar Singh
and Ors. v. State of Uttar Pradesh, AIR 1956 SC 441. In that case, six accused were
placed on trial for an offence of dacoity. Three of them belonged to complainant's village
whereas remaining three belonged to adjoining village. The trial Court convicted all the
six accused. The High Court, how ever, acquitted three accused and convicted the
remaining three under Section 395, IPC. This Court held that the High Court erred in
making a distinction between the three accused belonging to the complainant's vil lage
while the remaining three belonged to an adjoining village. This Court observed that the
High Court, having come to the conclusion that three out of six accused were not guilty,
should have gone into the ques tion whether there was satisfactory evidence to show that
the three remaining accused could be convicted under Section 395, IPC on the charge as
framed. This Court further held that the charge was framed against six persons and they
were placed on trial. It did not indicate that those six persons along with other unknown
persons committed dacoity. On the finding arrived at by the trial Court that all the six
persons committed the offence of dacoity punishable under Section 395, IPC, nothing
more was necessary. When the High Court set aside conviction of three accused and
acquitted them out of six persons jointly tried, it was left only with three appellants as the
persons concerned with the crime. The High Court, in the cir cumstances, according to
this Court, ought to have considered whether there was sat isfactory evidence to show
that the three ap pellants could be convicted of the lesser offence of robbery under
Section 392, IPC if there was evidence to show that they had committed acts of theft and
used violence while committing the theft.
30. In the case on hand, both the Courts below have considered the case of the pros
ecution and acquitted two accused com pletely. Moreover, all the accused were ac quitted
for commission of offence of criminal conspiracy as also of receiving stolen property in
commission of dacoity and the said acquittal has attained finality.
31. Shyam Behari v. State of Uttar Pradesh, AIR 1957 SC 320 also does not carry the
matter further. There, a finding was recorded that the accused and his compan ions, who
were more than five, attempted to commit dacoity but they failed in their at tempt as the
villagers raised hue and cry. Residents of village reached at the place and the miscreants
ran away without collecting booty. They were chased by some persons and caught one of
the dacoits. He fired a pistol shot which hit a villager who subse quently died. This Court
held that the of fence of dacoity was complete and it ended the moment the dacoits took
to their heels and another and a separate transaction took place when one of the accused
shot at a vil lager. Hence, even though he could not be convicted of having committed an
offence under Section 396, IPC, he could be con victed for an offence under Sections 395
and 302, IPC.
32
. Ramdeo Rai Yadav v. State of Bihar, (1990) 2 SCC 675 : JT 1990 (1) SC 356 is clearly
distin guishable. In that case, charge was framed against the accused for commission of
of fence punishable under Section 396, IPC but alternative charge was also framed for an
offence punishable under Section 302, IPC. In the light of framing of alternative charge,
this Court held that conviction of the appel lant-accused for an offence punishable un der
Section 302, IPC can be sustained. AIR 1990 SC 1180

33. Similar is the ratio in Anshad and Ors. v. State of Karnataka, (1994) 4 SCC 381 : JT
1993 (3) SC 324. There five accused were tried for offences punishable under Sections
396, 449, 395 and 307, IPC and were con victed. In the light of the factual position, the
Court held that conviction of accused Nos. 1, 2 and 3 could be altered to one un der
Section 302 read with Section 34, IPC, Section 394 read with Section 34, IPC and
Section 379 read with Section 34, IPC.
34. In the instant case, as observed ear lier, there were six accused. Out of those six
accused, two were acquitted by the trial Court without recording a finding that though
offence of dacoity was committed by six persons, identity of two accused could not be
established. They were simply acquit ted by the Court. In our opinion, therefore,
@page-SC3255
as per settled law, four persons could not be convicted for an offence of dacoity, being
less than five which is an essential ingredi ent for commission of dacoity. Moreover, all
of them were acquitted for an offence of criminal conspiracy punishable under Sec tion
120B, IPC as also for receiving stolen property in the commission of dacoity pun ishable
under Section 412, IPC. The convic tion of the appellant herein for an offence punishable
under Section 396, IPC, therer fore, cannot stand and must be set aside,
35. For the foregoing reasons, the ap peal is allowed. The conviction of the appel lant for
an offence punishable under Sec tion 396, IPC is set aside and he is ordered to be
acquitted. Since the appellant is in jail, he is ordered to be released forthwith if his
presence is not required in any other case.
36. The appeal is accordingly allowed.
Appeal allowed.
AIR 2008 SUPREME COURT 3255 "Chandrakant Shankarrao Machale v. Parubai
Bhairu Mohite"
(From : Bombay)*
Coram : 2 S. B. SINHA AND V. S. SIRPURKAR, JJ.
Civil Appeal No. 1957 of 2008 (arising out of SLP (C) No. 2491 of 2007), D/- 13 -3
-2008.
Chandrakant Shankarrao Machale v. Smt. Parubai Bhairu Mohite (Dead) through L.Rs.
Transfer of Property Act (4 of 1882), S.60 - MORTGAGE DEED - DEED - LEASE -
Redemption of mortgage - Suit for-Registered mortgage deed executed by predecessor of
plaintiff in favour of defendant - Plaintiff claiming redemption of mortgage by
reconveyance of property - Plea by defendant that parties had by their conduct agreed to
treat transaction as lease - Defendant was in possession as mortgagee - He could not
prove his possession as tenant - His status could not be changed from mortgagee to that
of lessee by unregistered deed - Plain tiff is entitled to decree.
2000 AIR SCW 2849; 1996 AIR SCW 1726, Rel. on. (Paras 11, 13, 15)
Cases Referred : Chronological Paras
2000 AIR SCW 2849 : AIR 2000 SC 2633 (Rel. on.) 13
1996 AIR SCW 1726 : AIR 1996 SC 1659 (Rel. on.) 14, 15
S. N. Bhat, N.P.S. Panwar and D.P. Chaturvedi, for Appellant; H.L. Tiku, Sr. Ad vocate,
Mrs. Yeshmeet Kaur and Ashok K. Mahajan, with him for Respondent.
*S. A. No. 722 of 2000, D/- 28-11-2006 (Bom).
Judgement
S. B. SINHA, J. :- Leave granted.
2. Appellant was the owner of a house property situated in Taluka Gandhinglaj, District
Kolhapur. On 28.2.1983, Bhairu Rama Mohite and Krishna Rama Mohite (the
predecessor of the first respondent herein) executed a registered deed of mortgage in
favour of the appellant. The possession of the said property was delivered in favour of the
mortgagee. The period prescribed in the said Deed of Mortgage was seven years. The
amount of mortgage was Rs.20,000/-. It was agreed that upon expiry of the said period,
the property would revert back to the mort gagor.
3. Allegedly, an unregistered agreement was entered into by and between the par ties
herein, stating :
"This agreement in writing executed by us in respect of land C.S. No. 1943 admea suring
252 Sq. meters situated Mouje Gadhinglj which includes house and open space belongs
to us absolutely. Earlier the house admeasuring East West 39 feet i.e. 11 meters 89
centimeters and South-North 49 feet i.e. 14 meters 94 centimeters, to tally adm. 177.63
59 meters house as also the open space in front of the house East West 21.89 meters and
South-North 6.10 meters totally adm.72.52 sq. meters from out of which house and the
open space on the Western side admeasuring 36.26 meters was given to you by way of
mortgage by con ditional sale for Rs.20,000/- under regis tered document No.229 dated
1.3.1983 and the same is recorded in your name."
Some other terms and conditions were also laid down therein.
4. Respondents, being the predecessors in interest of the mortgagor filed a suit for
redemption of the mortgage. Krishna died during the pendency of the suit. As the pe riod
specified in the said Deed of Mortgage was to expire on 28.2.1990, the plaintiff served
with a notice dated 17.2.1990 for redeeming the suit property. As the same was not acted
upon, a suit for redemption of mortgage was filed.
5. Contention of the appellant, inter alia, was that he has been put in possession of the
said property as a tenant. It was urged that by reason of the said mortgage, his right
@page-SC3256
to occupy the premises as a tenant was not extinguished.
In view of the aforementioned rival con tentions of the parties, the learned Trial Judge
framed the following issues :
"1. Do the plaintiff prove that Bhairu Rama Mohite and the defendant No. 10 Mort gaged
the suit property to the defendant by conditional sale on 1.3.1983?
2. Does the defendant prove that he is in possession of the suit property as a monthly
tenant?
3. Are the plaintiffs entitled to redeem the mortgage by obtaining reconveyance of the suit
property from the defendant?
4. Are they entitled to possession of the suit property?
5. What order and decree?"
6. On issue No.1, learned Trial Court opined :
"Now we have to see what was intended by the parties to these three documents. It may
be noted that parties to these docu ments are the same. The defendant claims tenancy
rights by virtue of agreement dated 24.2.1983 (Exh.52). On the other hand ac cording to
the plaintiff the property compris ing this agreement is not the subject matter of the.
mortgage the controversy has to be solved by going through the contents of the
document. From the recitals it appears that the Municipal House No. 1440 was agreed to
be let for the period of 7 years on lease by accepting Rs.20,000/-. It also appears from the
recitals that the plaintiff received Rs. 1.OOO/- on the day of agreement. I do not come
across recitals of the defendant having been put in possession of the prop erty. On the
contrary, the recitals do show that the agreement was executor (sic) in nature and the
intended transaction was to be completed within 15 days. Therefore, I do not subscribe to
the submission of the defendant that the lease was created by Exh.52 on 24.2.1983. It is
true that if we peruse the description of the property given in the mortgage deed Exh.62,
it does not corre spond with the description given in Exh.52. However, the plaintiff has
failed to prove that on the day of execution of the mortgage deed there were two
Municipal House numbers viz., 1440 and 1440-A. However, since I have already
observed document Exh.52 to be an agreement of the lease to be created within 15 days, I
do not wish to rely strongly on that document since the fate of the suit is rest on
subsequent document Exh.53 and Exh.62."
The suit, however, was dismissed opin ing that the plaintiff has leased the suit prop erty
in favour of the defendant.
7. An appeal preferred thereagainst by the plaintiff was allowed by the Court of Appeal,
holding :
"After going through terms and conditions of deed of lease styled as Kararpatra (Exh.53)
it reveals that it is not fresh contract of lease but, under that document, the right created
by plaintiffs, in favour of the defendant in the immovable property for Rs.20,000/- as a
mortgagee, has been extinguished. By the terms and conditions of this agreement, par
ties, intended to change the right created in favour of defendant, under registered deed of
mortgage, which was admittedly executed for the mortgage amount of Rs.20,000/-
Therefore, u/S. 17(l)(b) of the Indian Regis tration Act, this second agreement dated
1.3.1983 (Exh.53) requires registration. Admittedly, this document is not registered
document. Therefore, under this second agreement (Exh.53) the relations created in
between the plaintiffs and defendant as mortgagor and mortgagee, cannot be extin
guished. I hold that, the Kararpatra (Exh.53) cannot extinguish the right created in favour
of the defendant as mortgagee in the suit property. So also when under deed of mort
gaged deed (Exh.62) in lieu of amount of Rs.20,000/- interest has been created in favour
of the defendant, after execution of this deed of mortgage, plaintiffs had no right to lease
out the same property to the defen dant by cancelling this registered deed of mortgage
under unregistered agreement of lease (Exh.53) I hold that, the learned trial court, totally
ignored the legal position that, by unregistered agreement (Exh.53) the con tract of
mortgage (Exh.62) cannot be extin guished or cancelled. Therefore, the agree ment
(Exh.53) is of no help to the defendant to prove that under that document he ac quired
interest in the suit property as a ten ant of the plaintiff."
8. As regards the contention that the appellant became a tenant under the plain tiffs, the
Court of First Appeal held that the relationship between the parties were that of a
mortgagor and mortgagee and the de fendant-appellant had failed to prove that he was in
possession of the suit property as a tenant. The appeal was, thus, allowed and the suit was
decreed.
@page-SC3257
9. A second appeal preferred by the ap pellant herein has been dismissed by the High
Court opining that no substantial ques tion of law arose for its consideration.
10. Mr. S.N. Bhat, learned counsel ap pearing on behalf of the appellant, would, in
support of the appeal, contend that the transactions of mortgage and the lease were
separate and independent transactions.
The Court of First Appeal as also the High Court, thus, committed a serious error in
passing the impugned judgment. The learned counsel furthermore urged that al though a
decree for redemption could be granted but in execution of the said decree, only
symbolical possession could be directed to be issued.
11. The Deed of Mortgage dated 28.2.1983 was a registered document. The terms of a
registered document could be varied or altered only by another registered document. A
finding of fact has been arrived at that the appellant could not prove his possession as a
tenant. We have noticed hereinbefore that the appellant was put in possession as a
mortgagee. It was, there fore, in our opinion, impermissible in law to change his status
from a mortgagee to that of a lessee by reason of an unregistered deed of lease (even if
we assume that the same had been executed).
The learned Court of Appeal may not be entirely correct in taking recourse to Sec tion 92
of the Indian Contract Act or hold ing that the deed of lease required registra tion even for
the purpose of month to month tenancy, but, as indicated hereinbefore, we have
considered the question from a differ ent angle.
12. Furthermore, the only question of law which was pressed before the High Court was :
"The lower appellate court ought to have held that the respondents and appellant executed
an agreement dated 28.2.1983 i.e. Exh.62 and immediately on the next day, i.e., on
1.3.1983 executed the agreement for tenancy which is a subsequent agreement. Hence it
ought to have been held that the parties have by their conduct agreed to treat the
transaction as a lease and hence suit filed by respondents for redemption of mort gage is
not maintainable in law and ought to have been dismissed with costs."
No substantial question of law, thus, had been raised.
13

. The deed of mortgage was a regis tered one. It fulfilled the conditions of a valid
mortgage. Its terms could not have been var ied or altered by reason of an unregistered
document so as to change the status of the parties from mortgagee to a lessee. [See S.
Saktivel (dead) by L.Rs. v. M. Venugopal Piliai and Ors. [AIR 2000 SC 2633 para 67]
2000 AIR SCW 2849

14

. Our attention has been drawn to a decision of this Court in Gopalan Krishnakutty v.
Kunjamma Pilliai Sarojini Amma and Ors. [(1996) 3 SCC 424] wherein upon taking into
consideration some of its earlier decisions, this Court held : 1996 AIR SCW 1726, Para 6
The High Court, in the present case, pro ceeded on the erroneous assumption in law that
surrender of the lease by the lessee (de fendant) must be implied from the fact of
execution of the usufructuary mortgage in his favour by the lessor (plaintiff). As indi
cated, this is an erroneous assumption in law. This question has to be decided on the
contents of the deed since there is no other evidence of surrender of the lease by the
defendant on execution of the mortgage. We find nothing in the mortgage deed (Annex
ure A-1) dated 18.7.1974 read with the re lease deed of the same date to prove either an
express or an implied surrender of the lease by the defendant in favour of the plain tiff on
execution of the mortgage deed. Since there is no automatic merger of the interest of a
lessee with that of a mortgagee when the same person is the lessee as well as the
mortgagee, in absence of proof of surrender of the lease by the defendant, on redemp tion
of the mortgage, the plaintiff is not en titled automatically to recover possession of the
leased premises. The defendant's right to continue in possession as a lessee, there fore,
continues to subsist."
15

. We are concerned here with a con verse case. The case as to whether the in terest of a
lessee merged with the interest of a mortgagee would depend upon facts and
circumstances of each case, as indicated in Gopalan Krishna Murti. There cannot be any
hard and 'fast rule for arriving at only one deci sion as the decision thereupon will depend
upon the terms of the document. 1996 AIR SCW 1726

16. For the aforementioned reasons, there is no merit in the appeal. The same is dismissed
accordingly. In the facts of the case, there shall, however, be no order as to costs.
Appeal dismissed.
@page-SC3258
AIR 2008 SUPREME COURT 3258 "M. A. Sattar v. State of A. P."
(From : Andhra Pradesh)*
Coram : 2 S. B. SINHA AND HARJIT SINGH BEDI, JJ.
Criminal Appeal Nos. 1227-1228 of 2003, D/- 26 -3 -2008.
M. A. Sattar and Ors. v. State of A. P.
Penal Code (45 of 1860), S.300 - MURDER - IDENTIFICATION PARADE - WITNESS
- Murder - Enmity between deceased and ac cused group - Accused put chilly pow der in
eyes of deceased and prosecution witness - Inflicted multiple stab injuries on deceased
thereafter - Said prosecu tion witness lodged F.I.R. promptly - No evidence to show that
accused had been shown to eye-witnesses prior to test iden tification parade - There was
no reason to disregard test identification parade proceedings regarding three accused -
Presence of said eye-witnesses on spot, not doubtful - Accused whose presence had been
perceived as doubtful, already acquitted - Conviction of remaining accused - No
interference. (Para 8)

Rohit Minocha, for Appellants; Mrs. D. Bharathi Reddy, for Respondent.


* Cri. A. Nos. 313 and 1161 of 2000, D/- 5-9-2002 (A. P.)
Judgement
1. HARJIT SINGH BEDI, J. :-These appeals arise from the following facts :
2. The deceased, Y. Penchal Reddy and accused No. 1. P. Yellaiah along with several
other persons had occupied some govern ment land in Bhagat Singh Nagar and were
residing in tenements that had been raised by them. A dispute had however arisen be
tween the group represented by the deceased and that by P. Yellaiah with regard to the
sale of plots. In the elections to the associa tion, the deceased was elected as President
and the group represented by P. Yellaiah was defeated. It appears that on account of the
bitter relations between the two groups, one Mallaiah belonging to the group of the ac
cused was killed in 1993 wherein the de ceased was named as the main accused. He was
however acquitted by the trial court. A few days prior to the present incident some
violence had taken place and a meeting be ing held by the deceased was disturbed by the
members belonging to the accused party. On 18th March, 1997 at 10.00 A.M., PW 1
Mohd. Hussain accompanied by the de ceased was going to Chinthal and when they
reached near Omkar Rice Mill, a group of four persons, aged about 25 years, came from
the opposite direction and after put ting chilly powder in the eyes of the deceased and PW
1, inflicted multiple stab injuries on the deceased. Mohd. Hussain PW 1 rushed the
injured to the C.D.R. Hospital immediately but he was declared dead on arrival. He then
went to, the police station which was adjoining the hospital and gave a statement at about
1.1 A.M. which was recorded by PW 11 for offences punishable under Sections 341 and
302 of the I.P.C. On completion of the investigation, a charge under Section 302 read
with 34 IPC was framed against the accused G. Raju (A-4), Hari Lal (A-5), M.A. Sathar
(A-6) and Shivaji (A-7) and under Section 302 read with 109 was framed against accused
P. Yellaiah (A 1), K. Satyanarayana (A-2), and M. Ashok (A-3),
3. The prosecution in support of its case placed reliance on the evidence of 11 wit nesses
and several other pieces of incrimi nating evidence. The trial court held that PW 1 and
PW 4, Mohd. Hussain and Sathi Reddy respectively were eye witnesses to the incident as
the latter too had been walking just ahead of the deceased at the time of the attack. The
Court observed that the test identification parade had been conducted wherein PW 1 had
identified four of the ac cused i.e. A-4 to A-7 whereas PW 4 had iden tified only two of
them i.e. A-4 and A-7 in Court PW 1 had identified only A-3 to A-6 meaning thereby that
A-3 had been identi fied for the first time in court and that A-7 who had been identified at
the time of the test identification parade had not been iden tified in the court. The Court
also observed that PW 4 had identified only A-4 and A-7 at the test identification parade
but in the witness box he had identified A-4 to A-7 as the culprits. The trial court
accordingly held that the evidence as to the involvement of A-3 to A-7 was doubtful and
that there was no evidence whatsoever to, connect A-1 and A-2 to the offence. The trial
court accord ingly ordered that :
"A4 to A6 are found guilty for the offence under Section 302 read with 34 I.P.C and they
are convicted under Section 235(2) Cr.P.C. The other accused are not guilty for the
offences charged against them and they are acquitted under Section 235 (1) Cr.P.C. The
accused Nos.2 and 7 shall be released forthwith, if they are not required in any
@page-SC3259
other case. "
4. Two appeals were thereafter taken to the High Court by the three convicted ac cused.
The court opined that both the eye witnesses were consistent insofar as the in volvement
of A-4 was concerned. The court then held that insofar as A-5 and A-6 were concerned,
the eye witnesses account at the test identification parade did incriminate them and PW 1
had asserted that he had been able to see the accused as he had been wearing spectacles at
the time when the chilly powder had been thrown in his face. The court also held that
there was no rea son whatsoever to disregard the test identi fication parade proceedings
insofar as these accused were concerned, as they had been conducted in the presence of a
Magistrate and the suggestion by the defence that the accused had been shown to the eye
wit nesses before the parade could not be be lieved. It was also found that the presence of
the two witnesses at the spot at the rel evant time had to be accepted, more par ticularly
as PW 4 had no substantial con nection with either of the parties and was truly an
independent witnesses. The appeal was accordingly dismissed and the judgment and
order of the trial court was maintained. It is in these circumstances that the present
appeals are before" us by way of special leave.
5. Mr. Rohit Minocha, the learned coun sel for the appellants has repeated the ar guments
addressed before the two courts below. It has been argued that there were material
discrepancies between the state ments of the two witnesses with regard to the
involvement of the appellants and that the trial court itself had found that the in
volvement of four of the accused had not been proved. It has also been argued that there
was considerable delay in the conduct of the test identification parade and as such no
sanctity could be attached to its proceed ings.
6. The learned Government counsel has, however, supported the impugned judg ments
and has argued that the requisite sift ing had already been done and benefit given to such
of the accused whose presence had been perceived as doubtful.
7. We have considered the submissions of the learned counsel for the parties. It must first
and foremost, be noted that the First Information Report had been recorded within a very
short time of the incident in which full details of the incident had been given. As per the
record, the incident had taken place at about 10.00 A.M. on 18th March, 1997 and the
First Information Re port had been recorded within an hour or so at the police station
itself. We also find no reason whatsoever to doubt the presence of the eye witnesses. PW
1 is the first infor mant and PW 4 figures in the report. We are also of the opinion that
there is no merit in the stand of the learned counsel for ap pellants that an adverse
inference ought to be drawn from the fact that though the ac cused had been arrested on
2nd April, 1997 the test identification parade had been con ducted in the presence of the
Magistrate af ter a long delay on 26th April, 1997. We find that except for a bare
accusation that the accused had been shown to the witnesses during the interregnum,
there is no evidence to show that this had in fact happened and even the suggestion put to
the witnesses that they had seen the accused either in the po lice station or at the I.D.P.L.
Colony, shows that the defence was probing in the dark and trying to make out a case for
the de fence where none existed.
8. It is also clear from the record that there appeared to be some enmity between the
deceased and the group headed by P. Yellaiah (A-1) and though it has not been possible
for the prosecution to bring evi dence to show his involvement, but it has come in the
evidence of PW 1 that three days prior to the incident, a meeting of all the colony
members had been held wherein a decision had been taken that about 20 plots were to be
sold to further develop the colony. It appears that this is the factor which pre cipitated the
incident. We are, therefore, of the opinion that there is no merit in the appeals. They are
accordingly dismissed.
Appeal dismissed.
AIR 2008 SUPREME COURT 3259 "Dinesh Kumar v. State of Rajasthan"
(From : Rajasthan)*
Coram : 2 Dr. A. PASAYAT AND Dr. MUKUNDAKAM SHARMA, JJ.
Criminal Appeal No. 1215 of 2008 (aris ing out of SLP (Cri.) No. 5603 of 2007), D/- 4 -8
-2008.
Dinesh Kumar v. State of Rajasthan.
(A) Penal Code (45 of 1860), S.300 - MURDER - EVIDENCE - WITNESS - MAXIMS -
Murder case - Appreciation of evidence
@page-SC3260
Falsus in uno falsus in omnibus - Evi dence of eye-witnesses - Found to be exaggerated
against co-accused - But cogent, truthful and credible against accused-appellant -
Appellant can be con victed on basis of eye-witnesses' evidence.
Evidence Act (1 of 1872), S.3. (Para 6)
(B) Penal Code (45 of 1860), S.300 - MURDER - EVIDENCE - WITNESS - Murder -
Interested and inimically de posed eye-witnesses - Conclusion that they would shield real
culprit and rope in innocent person - Cannot be drawn - Court has to analyse evidence-
Evidence on analysis if found to be clear; cogent and credible, there is no reason to dis
card same. (Para 6)
(C) Penal Code (45 of 1860), S.300 - MURDER - WITNESS - Murder case - Interested
witness - Relation with deceased - No ground to dis card evidence of witnesses. (Para
6)

Vijay Singh Charak, Advocate (A.C.), for Appellant; Aruneshwar Gupta, Naveen Kumar
Singh and Shashwat Gupta, for Re spondent.
* Cri. Appeal No. 176 of 2002, D/- 14-5-2007 (Raj.)
Judgement
1. Dr. ARIJIT PASAYAT, J. :-Heard learned counsel for the parties.
2. Leave granted.
3. Challenge in this appeal is to the judg ment of a Division Bench of the Rajasthan High
Court, Jaipur Bench, upholding con viction recorded by learned Additional Ses sions
Judge No. 2, Fast Track Court, for of fences punishable under Sections 302, 307, 324,
148, 452 and Section 323 read with Section 149 of the Indian Penal Code, 1860 (in short
the 'IPC'). In all, five persons faced trial. The appeal filed by four others was allowed and
conviction in respect of each one of them was set aside. They were con victed in terms of
Section 302 read with 149, IPC, 307 read with 149, 324/149, 148/452 and 323, IPC. The
'appellant was sentenced to imprisonment for life and to pay a fine with default
stipulations and to suffer 10 years, 1 year, 2 years and 6 months' impris onment in respect
of other offences.
4. Background facts giving rise to the trial are as follows :
Informant-Suresh Kumar (PW-10) sub mitted a written report (Ex.P-5) at Police Station
Mahaveer Nagar, Kota, on 8.4.2001 at about 3.00 a.m. stating that his house is situated at
Keshave Nagar. Kota, and Babu Lal Nai was also residing in front of his house. There
had been long standing en mity between them and many a times they had altercations in
the past. On 7.4.2001 at about 11 O'clock, while the informant and his family members
were sleeping in the house after bolting the door from inside, the accused Babu Lal Nai
along with his wife Smt. Geeta, and sons Dinesh, Sattu @ Satyanarayan and Sonu @
Sunil and Smt. Nirmala w/o Shri Dinesh armed with deadly weapons entered into his
house. Chittar Lal, father of the informant (hereinafter referred to as the 'deceased')
enquired from the ac cused as to why they had entered into the house. On this, Babu Lal
Nai and his wife Smt. Geeta exhorted the accused to finish them. Dinesh with the sword
which he was carrying pierced abdomen of Chittar Lal. Babu Lal Nai inflicted injury on
the abdo men of Chittar Lal. Uma Shanker and the informant Suresh Kumar rushed to
rescue Chittar Lal. Satyanarayan inflicted blow with an iron rod on the head of Chittar
Lal. Dinesh and Satyanarayan inflicted knife injury to Uma Shankar and Vinod; while
Nirmala W/o Dinesh and Smt. Geeta w/o Babu Lal and Sonu @ Sunil inflicted inju ries
to his father and brother with lathies and iron rod. Chittar Lal and Uma Shankar became
unconscious on the spot. On hear ing hue and cry, neighbours also gathered there. Injured
Uma Shankar and Chittar Lal were taken to the hospital. Chittar Lal succumbed to the
injuries while injured Uma Shankar was admitted to the hospital. On the basis of the
aforesaid report, a case un der Sections 147, 148, 149, 302, 307, 452 and 323, IPC was
registered and investiga tion commenced. After investigation, charge sheet was filed. In
due course, the case came up for trial before the learned Additional Sessions Judge No. 2
(Fast Track), Kota. Charges under Sections 148, 452, 302 or 302/149, 307 or 307/149,
324 or 324/149 and 323 or 323/149, IPC were framed against the appellant and other co-
accused who denied the charges and claimed trial. The prosecution in support of its case
ex amined 18 witnesses. In their explanation under Section 313 of the Code of Criminal
Procedure, 1973 (in short 'Cr.P.C.') the ap pellant and other co-accused claimed inno
cence. Appellant-Dinesh got himself exam ined as DW-1 under Section 315, Cr. P.C.
@page-SC3261
Before the Trial Court, the primary stand was that there was long standing enmity
between the parties and, therefore, there was false implication. It was also submitted that
appellant was, in fact, assaulted by the com plainant party had also suffered injuries and
had lodged a cross case.
Learned Trial Judge on hearing final sub missions convicted and sentenced the appellants
as indicated hereinabove.
In appeal, the stand was reiterated. The High Court found that though there was some
amount of exaggerations so far as the others are concerned, the evidence of the eye-
witnesses PWs-7, 10 and 13 was cred ible and cogent and, therefore, dismissed the appeal
so far as the appellant is concerned.
5. In support of the appeal, learned coun sel for the appellant submitted that when four
persons have, been acquitted by the High Court, the conviction of the appellant should
not have been maintained, more par ticularly, when they are related. Learned counsel for
the respondent-State, on the other hand, supported the judgment of the High Court.
6. It is to be noted that PWs-7 and 13 were the injured witnesses and PW-10 was another
eye-witness and was the informant. Law is fairly well settled that even if acquit tal is
recorded in respect of co-accused on the ground that there were exaggerations and
embellishments, yet conviction can be recorded if the evidence is found cogent, credible
and truthful in respect of another accused. The mere fact that the witnesses were related
to the deceased cannot be a ground to discard their evidence. In law tes timony of an
injured witness is given im portance. When the eye-witnesses are stated to be interested
and inimically deposed to wards the accused, it has to be noted that it would not be
proper to conclude that they would shield the real culprit and rope in innocent persons.
The truth or otherwise of the evidence has to be weighed pragmati cally. The Court would
be required to analyse the evidence of related witnesses and those witnesses who are
inimically de posed towards the accused. But if after care ful analysis and scrutiny of
their evidence, the version given by the witness appears to be clear, cogent and credible,
there is no reason to discard the same. Conviction can be made on the basis of such
evidence. In the instant case, the Trial Court and the High Court have analysed the
testimony of PWs-7, 10 and 13 in great detail. It is re vealed that the appellant had
inflicted the first sword blow to the deceased in his ab domen and he fell on the ground.
The High Court, however, found that the role ascribed to the others was not fully
satisfied. The sword used in the offence was recovered at the instance of the appellant
and the same was found to be stained with same group of human blood, as that of the
deceased, as per the FSL report, Exh.P-28. PW-7 stated that when he tried to save his
father, the accused also inflicted blows on him and he sustained injury by sharp edged
weapon i.e. the sword. According to him, the accused inflicted the blow by the sword on
his neck and he fell down. Though, the appellant stated that he had suffered injuries at the
hands of the deceased and his sons, as rightly noted by the Trial Court and the High
Court, they were superficial injuries and as the doctor opined, could be self-inflicted.
7. Above being the position, we find no merit in this appeal, which is, accordingly
dismissed.
Appeal dismissed.
AIR 2008 SUPREME COURT 3261 "K. Narayanan v. S. Murali"
(From : Madras)
Coram : 2 TARUN CHATTERJEE AND HARJIT SINGH BEDI, JJ.
Civil Appeal Nos. 4480-4481 of 2002, D/- 5 -8 -2008.
K. Narayanan and Anr. v. S. Murali.
Trade and Merchandise Marks Act (43 of 1958), S.29, S.18 - TRADE AND
MERCHANDISE - TRADE MARK - Action for passing off - Cause of action - Mere
filing of application by respondent for registration of similar Trade Mark - Does not give
rise to cause of action for passing off ac tion. (Paras 13, 15)
Cases Referred : Chronological Paras
2006 AIR SCW 72 : AIR 2006 SC 730 : 2006 (1) ALJ 669 : 2006 CLC 1 (Rel. on) 14
2001 (3) CTC 652 (Mad) 3, 14
(1995) OSA Nos. 53 and 82 of 1995, D/-13-3-1995 and 29-3-1995 (Mad) 5
1990 (Supp) SCC 727 (Ref.) 13
1990 PTC 240 (Mad) 5
1983 PTC 207 (Del) 5, 10
Ms. Gladys Daniel, Anup Kumar and K. V. Vijayakumar, for Appellants; Dr. A. Francis
Julian, Sr. Advocate, Sumit Kumar
@page-SC3262
for M/s. Arputham, Aruna and Co.), for Re spondent.
Judgement
1. TARUN CHATTERJEE, J. :-The present appeals are filed at the instance of the
appellants against the Judgment and fi nal order dated 18th of April, 2002 passed by the
High Court of Madras in O.S.A. Nos. 149 and 150 of 2002 whereby the Division Bench
of the High Court had dismissed the appeals of the appellants.
2. The brief facts leading to the filing of these appeals may be narrated as under :
The appellants are engaged in the busi ness of manufacturing and selling Banana Chips
and had adopted the trade mark A ONE with respect to the said Banana Chips in 1986.
The appellants had applied for an application for registration of the trade mark A-ONE
before the Trade Mark Registry at Chennai on 6th of December, 1999 with re spect to the
said Banana Chips. The appli cation of the appellants for registration of the trademark is
still pending.
On 7th of February, 2000, the respon dent filed O.S.No. 1 of 2000 on the file of the
District Judge at Coimbatore against the appellants, seeking an injunction restrain ing the
appellants from passing off their goods using the trade mark A-ONE. The said suit was
dismissed by the District Judge at Coimbatore on 23rd of December, 2001.
The respondent filed three trade mark applications numbered as 899359, 899360 and
899361 on 24th of January, 2000 be fore the Trade Mark Registry at Chennai seeking
registration as user of the mark A ONE throughout India since 1995.
Thereafter the appellants filed C.S.No. 482 of 2001 on 22nd of May, 2001 on the file of
the High Court of Madras, seeking an injunction to restrain the respondent from passing
off his goods using the trade mark A-ONE.
The appellants filed an application before the High Court for leave to institute the suit
and by order dated 11th of June, 2001, the High Court granted leave.
On 6th of March, 2002, the learned Single Judge of the High Court dismissed the in
junction application and also revoked the leave to sue, granted by it to the appellants.
The appellants, being aggrieved by the aforesaid order, preferred appeals before the
Division Bench of the High Court, which was dismissed by the Division Bench by an
order dated 18th of April, 2002.
Being aggrieved and dissatisfied with the aforesaid judgment of the Division Bench, the
appellants have filed these Special Leave Petitions in this Court which, on grant of leave,
were heard by us in presence of learned counsel for the parties.
3. We have heard the learned counsel for the parties and examined the impugned
judgment of the Division Bench of the High Court as also of the learned Single Judge and
other materials on record and we deem it appropriate to reproduce the findings of the
Division Bench while dismissing the appeals, which are as under :-
"The point raised in the appeals is one which was already decided against the ap pellant
by our considered judgment in the case of Premier Distilleries Pvt. Ltd. vs. Sushi
Distilleries 2001 (3) CTC 652.
Learned counsel sought to contend that there is an earlier view of this Court which
conflicts the view which we have taken. Hav ing perused that order, we find that it was
merely a summary order which does not address itself pointedly to the question. Mere
filing of the application for registration of the trade mark in the Registry situated at
Madras would not suffice to confine the ju risdiction of this Court. That question was
specifically addressed, and dealt with in our reasoned order in the case of Premier Dis
tilleries Pvt. Ltd. (supra). In that order, we have pointed out that the Very term "cause of
action" would clearly imply that the ac tion viz., the institution of the suit must fol low
the cause, and not precede it. Even be fore the registration is granted for the trade mark,
there is no right in the person to as sert that the mark has been infringed. A proposed
registration which may, or may not be granted will not confer a cause of action to the
plaintiff, whether the application for registration is filed by the plaintiff, or the
defendant."
4. Before we look at the submissions of the parties before us, we deem it expedient at this
stage to reproduce the relevant pro visions of the Trade and Merchandise Marks Act,
1958 (in short, the 'Act'), which would be required by us for a proper appreciation of the
controversy involved.
Section 18(1) of the Act may be repro duced as under:-
" Any person claiming to be the propri etor of a trade mark used or proposed to be
@page-SC3263
used by him, who is desirous of registering it, shall apply in writing to the Registrar in the
prescribed manner for the registration of his trade mark either in Part A or in Part B of the
register."
Section 28 of the Act may be reproduced as under:-
"Subject to the other provisions of this Act, the registration of a trade mark in Part A or
Part B of the register shall, if valid, give to the registered proprietor of the trade mark the
exclusive right to the use of the trade mark in relation to the goods in respect of which the
trade mark is registered and to obtain relief in respect of infringement of the trade mark in
the manner provided by this Act."
5. Let us now consider the submissions of the learned counsel for the parties. The learned
counsel for the appellants argued before us that the Division Bench of the High Court in
its, impugned judgment had taken a contrary view from the Judgment of the Division
Bench of the High Court of Delhi in M/s. Jawahar Engineering Com pany and others,
Ghaziabad vs. M/s. Jawahar Engineers Pvt. Ltd., Sri Rampur, Distt. Ahmednagar,
Maharashtra [1983 PTC 207], which has held that the real point which confers
jurisdiction to the Court is not the place where the advertisement has published but the
place for which the trade mark is sought for sale. It has also held that when an injunction
is sought, it is not necessary that the threat should have be come a reality before the
injunction is granted or refused and it can even be sought for a threat that is still to
materialize.
The learned counsel for the appellants further submitted that the view taken by the
Division Bench of the High Court of Delhi, reported in 1983 PTC 207, was followed by
the learned Single Judge of the High Court of Madras in the Judgment, reported in 1990
PTC 240.
The learned counsel for the appellants further submitted that a similar view was followed
by the Division Bench of the High Court of Madras in its unreported judgments dated
13th of March, 1995 and 29th of March, 1995 in O.S.A. No. 53/1995 and O.S.A. No.
82/1995 respectively.
The learned counsel for the appellants submitted that when the respondent filed a trade
mark application at the Trade Mark Registry at Chennai, a threat was communicated
regarding the use of the trade mark in Chennai, and it was immaterial whether there was
actual use or not and the appellants would be entitled to an injunc tion (being a
prohibitive remedy) against the said mark.
The learned counsel for the appellants finally argued that the respondent had based its
application for registration of the trade mark on use of the mark throughout India without
any geographical limitation from 1st of April, 1995, which included the city of Chennai,
which thus entitled the appellants to file the suit at the High Court of Madras based on the
claims made in the trade mark application.
6. These submissions of the learned counsel for the appellants were contested by the
learned counsel appearing on behalf of the respondent. The learned counsel for the
respondent contended that mere filing of an application for registration of trade mark by
the respondent in Chennai would not confer any territorial jurisdiction for the High Court
at Chennai to entertain the present suit filed by the appellants, when admittedly both the
parties to the suit re sided in Coimbatore, had their place of busi ness in Coimbatore and
the goods were sold only in Coimbatore.
7. The learned counsel appearing on behalf of the respondent further contended that since
according to Section 18 of the Act, an application for registration could be filed by both
proprietor of a trade mark used and proposed to be used by him, there fore mere filing of
an application for regis tration would not result in creating a cause of action for filing a
suit for passing off.
8. The learned counsel appearing on behalf of the respondent also contended that since
according to Section 28 of the Act, the registration of a trade mark gave a person,
exclusive ownership of the trade mark and right to take action against the infringement of
the trade mark, therefore an action against infringement of trade mark could not be made
in the court merely on the basis of an application for registration of trade mark.
9. It was further argued that actual sale of goods was necessary to be proved in the case of
passing off action and therefore the Court within whose jurisdiction the commer cial sale
of goods took place, had jurisdic tion to entertain a suit for passing off.
10. It was further argued that the decision
@page-SC3264
of the Division Bench of the Delhi High Court in M/s. Jawahar Engineering Com pany
and others, Ghaziabad (supra) was not applicable to the present case because in that case
the plaintiff was a registered owner of the trade mark and the action was for injunction
regarding a threatened breach of registered trade mark, whereas in the present case, the
appellants were not regis tered owners.
11. It was finally argued by the learned counsel appearing on behalf of the respon dent
before us, that, by merely filing a trade mark application, the respondent did not
misrepresent in the course of trade that his goods were the goods of the appellants and
therefore there was no cause of action for filing a suit for passing off, which necessar ily
required sale of one's goods deceptively as though it were the goods of another.
12. Having heard the learned counsel for the parties and after carefully examin ing the
aforementioned judgment of the High Court and also of the learned Single Judge, we do
not find any infirmity in the judgment of Division Bench of the High Court holding that,
before registration is granted for the trade mark, there is no right in the person to assert
that the mark has been infringed and that a proposed regis tration which may, or may not
be granted will not confer a cause of action to the plain tiff, whether the application for
registration is filed by the plaintiff, or the defendant.
13. In this connection, the following de cisions of this Court may be strongly relied
upon :-
In Wander Ltd. and another vs. Antox India P. Ltd., [1990 (Supp) SCC 727] (para 16), it
has been observed as follows :-
"Passing-off is said to be a species of un fair trade competition or of actionable un fair
trading by which one person, through deception, attempts to obtain an economic benefit
of the reputation which another has established for himself in a particular trade or
business. The action is regarded as an action for deceit. The tort of passing-off in volves a
misrepresentation made by a trader to his prospective customers calculated to injure, as a
reasonably foreseeable conse quence, the business or goodwill of another which actually
or probably, causes damages to the business or goodwill of the other trader."
In the present case, mere filing of a trade mark application cannot be regarded as a cause
of action for filing a suit for passing off since filing of an application for registra tion of
trade mark does not indicate any deception on the part of the respondent to injure
business or goodwill of the appellants.
14

. In Dhodha House vs. S.K. Maingi, [(2006) 9 SCC 41] (para 31), it has been observed as
follows :- 2006 AIR SCW 72

"A cause of action will arise only when a registered trade mark is used and not when an
application is filed for registration of the trade mark. In a given case, an application for
grant of registration certificate may or may not be allowed. The person in whose favour a
registration certificate has already been granted (sic) indisputably will have an
opportunity to oppose the same by filing an application before the Registrar, who has the
requisite jurisdiction to determine the said question. In other words, a suit may lie where
an infringement of trade mark -or copyright takes place but a cause of action for filing the
suit would not arise within the jurisdiction of the court only because an ad vertisement
has been issued in the Trade Marks Journal or any other journal, notify ing the factum
filing of such an application."
In the aforesaid decision, this Court has expressed its concurrence to the views ob served
by the Division Bench of the High Court of Madras in Premier Distilleries Pvt. Ltd. vs.
Sushi Distilleries [2001(3) CTC 652], which observed as under :-
"The cause of action in a suit for passing off, on the other hand and as already ob served,
has nothing at all to do with the lo cation of the Registrar's office or the factum of
applying or not applying for registration. It is wholly unnecessary for the plaintiff to
prove that he had applied for registration. The fact that the plaintiff had not applied for
registration will not improve the case of the defendant either. Filing of an applica tion for
registration of a trade mark, there fore, does not constitute a part of cause of action where
the suit is one for passing off."
(Emphasis supplied)
15. In this view of the matter, we are, therefore, of the opinion that filing of an
application for registration of a trade mark does not constitute a part of cause of action in
a suit for passing off. The appellants can not file the suit in the High Court of Madras
seeking an injunction to restrain the respondent
@page-SC3265
from passing off his goods using the trade mark A-ONE, based only on the claims made
in the trade mark application of re spondent filed before the Trade Mark Regis try, since
the necessary requirements of an action for passing off are absent.
16. Accordingly, there is no ground to interfere with the impugned judgment of Division
Bench of the High Court of Madras. For the reasons aforesaid, the appeals stand
dismissed with no order as to costs.
Appeal dismissed.
AIR 2008 SUPREME COURT 3265 "Kerala State Electricity Board v. Chinamma
Antony"
(From : Kerala)*
Coram : 2 Dr. A. PASAYAT AND HARJIT SINGH BEDI, JJ.
Civil Appeal No.4381 of 2008 (arising out of SLP (C) No. 18788 of 2006), D/- 15 -7
-2008.
Kerala State Electricity Board v. Chinamma Antony.
Electricity Act (9 of 1910), S.51 - Telegraph Act (13 of 1885), S.10(2), S.18 -
ELECTRICITY - TELEGRAPH - SUPREME COURT - Drawal of electricity line over
property - Diminution in land value - Compensation for loss - Claim for - Dismissal by
High Court by relying on its Full Bench decision - Said Pull Bench decision set aside in
another case by Supreme Court and matter remitted back - Thus, im pugned order set
aside and matter remit ted for fresh consideration in view of de cisions of Supreme Court
in2007 AIR SCW 4104 and 2008 AIR SCW 2633. (Para 5)
Cases Referred : Chronological Paras
2008 AIR SCW 2633 (Ref.) 5
2007 AIR SCW 1145 : AIR 2007 SC 1142 (Ref.) 3
2007 AIR SCW 4104 (Ref.) 3
2002 AIR SCW 1272 : AIR 2002 SC 1423 (Ref.) 3
AIR 2000 Ker 215 : 2000 AIHC 2850(FB) 2 1995 AIR SCW 3784 : AIR 1996 SC 106
3
1995 Supp (2) SCC 635 3
M.T. George, for Appellant.
* C.R.P. No. 960 of 2005, D/- 16-11-2005 (Ker).
Judgement
1. Dr. ARIJIT PASAYAT, J. :-Leave granted.
2

. Challenge in this appeal is to the judg ment of a learned single Judge of the Kerala High
Court dismissing the Civil Revision Petition filed by the appellant the Kerala State
Electricity Board (in short the 'Board'). Challenge in the Civil Revision was to the order
passed by learned Additional District Judge, Thodupuzha, granting the enhanced
compensation for alleged loss suffered by the respondent (hereinafter referred to as the
'claimant') on account of drawal of electric ity line over her property. The dispute re lated
to the compensation awarded for dimi nution in land value and the grant of inter est.
Relying on a full Bench decision of a Kerala High Court in Kumba Amma v. K.S. E.B.
(2002 (1) KLT 542), the High Court dismissed the Civil Revi sion Petition.AIR 2000 Ker
215

. In support of the appeal learned coun sel for the appellant-Board submitted that the High
Court's judgment is clearly unsus tainable as the Full Bench decision in Kamba Amma's
case (supra) was set aside by this Court in The Kerala State Electricity Board v. Livisha
etc. etc. (2007 (6) SCC 792) by the common judgment in Civil Appeal No. 289 of 2006
and other Civil Appeals. This Court set aside the impugned order in each case and
remitted the matter back to the High Court for a fresh consideration. It was inter-alia
observed as follows : 2007 AIR SCW 4104

"10. The situs of the land, the distance between the high voltage electricity line laid
thereover, the extent of the line thereon as also the fact as to whether the high voltage line
passes over a small tract of land or through the middle of the land and other similar
relevant factors in our opinion would be determinative. The value of the land would also
be a relevant factor. The owner of the land furthermore, in a given situation may lose his
substantive right to use the property for the purpose for which the same was meant to be
used.

11. So far as the compensation in rela tion to fruit bearing trees are concerned the same
would also depend upon the facts and circumstances of each case. We may, inci dentally,
refer to a recent decision of this Court in Land Acquisition Officer v. Kamadana
Ramakrishna Rao (2007(3) SCC 526) wherein claim on yield basis has been held to be
relevant for determining the amount of compensa tion payable under the Land
Acquisition Act; same principle has been reiterated in Kapur Singh Mistri v. Financial
Commr. and Revenue Secy. to Govt. of Punjab (1995 Supp (2) SCC 2007 AIR
SCW 1145
2002 AIR SCW 1272, Para 14

@page-SC3266
635), State of Haryana v. Gurcharan Singh (1995 Supp (2) SCC 637), Para 4 and Air
ports Authority of India v. Satyagopal Roy (2002 (3) SCC 527). In Airports Authority's
case (supra) it was held : (SCC p. 533, para 14)

"14. Hence, in our view, there was no reason for the High Court not to follow the
decision rendered by this Court in Gurucharan Singh's case (su pra) and determine the
compensation payable to the respondents on the basis of the yield from the trees by
applying 8 years' multiplier. In this view of the mat ter, in our view, the High Court
committed error apparent in awarding compensation adopting the multiplier of 18".
1995 AIR SCW 3784

12. We are, therefore, of the opinion that the High Court should consider the matter afresh
on the merit of each matter haying regard to the fact situation obtaining therein. The
impugned judgments, therefore, cannot be sustained. These are set aside accord ingly.
The matters are remitted to the High Court for consideration thereon afresh. The appeals
are allowed. In the facts and cir cumstances of the case, there shall be no order as to
costs.
4. There is no appearance on behalf of the respondent though notice has been served.
5

. Following the view expressed by this Court in the decision referred to above, and in The
Kerala State Electricity Board v. B. Sreekumari (2008 (5) SCC 398), we set aside the im
pugned order of the High Court and remit the matter to it for fresh consideration keep ing
in view the principles set out in the de cisions referred to above. 2008 AIR SCW 2633
6. The appeal is allowed without any or der as to costs.
Appeal allowed.
AIR 2008 SUPREME COURT 3266 "Regional Manager, Central Bank of India v.
Madhulika Guru Prasad Dahir"
(From : Bombay)
Coram : 2 C. K. THAKKER AND D. K. JAIN, JJ.
Civil Appral No.4636 of 2008 (arising out of S.L.P. (C) No. 9781 of 2005), D/- 25 -7
-2008.
Regional Manager, Central Bank of India v. Madhulika Guru Prasad Dahir and Ors.
Banking Companies (Acquisition and Transfer of Undertakings) Act (5 of 1976), S.19 -
BANKING - TERMINATION OF SERVICE - APPOINTMENT - Termination of service
- Bank employee - Appointment secured on false caste claim - Termination of ser vice is
valid - Facts that scrutiny of caste certificate was done after long delay, Scrutiny
Committee took long time and appointee had put in long service - In consequential -
Equity, sympathy or generosity has no place where original ap pointment rests on a false
caste certificate.
W.P. No. 2558 of 2003, D/-06-04-2005 (Bom), Reversed. (Paras 13, 16, 18)
Cases Referred : ChronologicalParas
2007 AIR SCW 3111 : AIR 2007 SC 2048 : 2007 (3) AIR Jhar R 380 : 2007 (4) AIR Bom
R 180 : 2007 Lab IC 2482 (Rel. on) 8, 9, 13
2007 AIR SCW 5822 : AIR 2007 SC 3211 : 2007 Lab IC 4455 : 2007 (6) ALJ 353
(Disting.) 9
2007 (8) Scale 108 (Disting.) 9
2006 AIR SCW 6093 : AIR 2007 SC 295 : 2007 (1) AIR Bom R 557 (Foll.) 15
2005 AIR SCW 4094 : AIR 2005 SC 3330 (Rel. on) 13
2005 AIR SCW 4477 : AIR 2005 SC 3395 : 2005 Lab IC 4164 8
2004 AIR SCW 419 : AIR 2004 SC 1469 : 2004 Lab IC 556 (Rel. on) 13
2003 AIR SCW 4912 : AIR 2003 SC 4268 : 2003 All LJ 2486 (Rel. on) 13
(2003) 8 SCC 319 (Foll.) 14
1994 AIR SCW 4116 : AIR : 1995 SC 94 12
(1889) AC 337 13
Dinesh Mathur, Sunil Murarka, Saurav Jain and Rameshwar Prasad Goyal, for Ap
pellant; Ashok Srivastava, Sr. Advocate, V. N. Raghupathy, Satyajit A. Desai, Ms. Anagha
S. Desai and Venkateswara Rao Anumolu, for Respondents.
Judgement
D. K. JAIN, J. :- Leave granted.
2. This appeal, by special leave, has been preferred by the Central Bank of India, a public
sector undertaking, against the judg ment and order dated 6th April, 2005, passed by the
High Court of Judicature at Bombay, Nagpur Bench, in Writ Petition No.2558 of 2003.
By the impugned judg ment, the writ petition filed by the first re spondent, hereinafter
referred to as 'the employee', challenging the order of termi nation of her services has
been allowed with a direction to reinstate her with continuity in service but without back-
wages.
3. The employee was appointed to the post of Clerk in the appellant-Bank with effect
@page-SC3267
from 18th March, 1981, against a post reserved for "Scheduled Tribes", as she had
claimed to be belonging to "Thakur Sched uled Tribe", on the basis of a caste certifi cate
issued on 4th December, 1979. In March, 1991, she was promoted as Junior Officer. It
appears that pursuant to the in structions issued, by the Government of In dia, Ministry of
Finance, Department of Eco nomic Affairs on, 23rd March, 1990, direct ing all the public
sector banks/financial institutions to get the caste certificates of the existing employees,
belonging to vari ous scheduled tribes, verified, the caste cer tificate filed by the
employee was referred to the Committee for Scrutiny and Verification of Tribe Claims,
Nagpur (for short the Scrutiny Committee), respondent No.2 herein.
4. The Scrutiny Committee granted an opportunity of hearing to the employee on 11th
October, 1999; made its own inquiries and found that the employee was not origi nally
belonging to "Thakur Scheduled Tribe" but had obtained the Scheduled Tribe cer tificate
from the issuing authority to take advantage of various concessions given to the tribal
communities. The Scrutiny Com mittee discovered that the employee had studied in
Jeevan Shikshan Vidyalaya, Nagpur and by blotting out the original caste "Rajput
Dahayat" as mentioned in the school leaving certificate, noted down the caste as
"Thakur", and thus, took admission in col lege on the basis of caste certificate "Thakur",
which caste also got mentioned in the college leaving certificate of the em ployee. In the
absence of any documentary evidence adduced by the employee to prove her claim, the
Scrutiny Committee relied on the affinity test and found that the employee did not show
any affinity towards "Thakur Scheduled Tribe". Accordingly, vide its or der dated 2nd
May, 2000, the Scrutiny Com mittee cancelled the caste certificate dated 4th December,
1979, issued to the employee.
5. The aforesaid order of the Scrutiny Committee was challenged by the employee by
way of a writ petition. The High Court allowed the writ petition on the ground that there
was non application of mind on the part of the Scrutiny Committee; set aside its order
dated 2nd May, 2000 and re manded the matter to the Scrutiny Com mittee for
reconsideration of the caste claim of the employee.
6. Pursuant thereto, the Scrutiny Com mittee, reconsidered the caste claim of the
employee. Concurring with its earlier find ings, vide order dated 29th May, 2003, the
Committee again rejected the claim of the employee and cancelled the caste certificate
dated 4th December, 1979. Consequently, by order dated 28th June, 2003, the ser vices of
the employee were terminated by the appellant on the ground that her tribe claim had
been invalidated. Aggrieved, the employee preferred a writ petition challeng ing both the
said orders.
7. It is pertinent, however, to note that at the hearing of the writ petition before the High
Court, the employee did not challenge the correctness of the order of the Scrutiny
Committee, dated 29th May, 2003 and the same attained finality, which manifestly shows
that the employee accepted the find ing of the Scrutiny Committee that she did not belong
to "Thakur Scheduled Tribe", as recorded on the caste certificate filed for obtaining
employment. The order of termi nation of service was challenged only on the ground of
inordinate delay of more than 10 years in making a reference to the Scrutiny Committee
for verification of the caste cer tificate. Accepting the said plea of the em ployee, while
upholding the order of the Scrutiny Committee dated 29th May, 2003, invalidating the
caste certificate, the High Court has quashed the order of termination dated 28th June,
2003. The High Court has observed that since the services of the em ployee have been
terminated after a period of twenty two years on the basis that she does not belong to the
Scheduled Tribe and the employee having herself given up the claim of belonging to the
Scheduled Tribe, the interest of justice demandsthatthe employeebe continuedinservice.
Ag grieved thereby, the appellant has come up before us in this appeal.
8

. Learned counsel appearing on behalf of the appellant submitted that having found that
the findings of the Scrutiny Committee were correct, the High Court should not have
quashed the order of termination of service, as admittedly the employee had obtained
appointment by fraudulent means by produc ing a false caste certificate. It was asserted
that neither the delay in reference to the Scrutiny Committee nor the time taken by the
Committee in scrutinizing the certificate can be used as a ground to validate a fraudu lent
action. It was also urged that having entered the service by deceitful means, ren dering of
long period of services does not 2007 AIR SCW 3111
2005 AIR SCW 4447

@page-SC3268
entitle the appellant to any relief in equity. In support of the proposition, reliance was
placed on the decisions of this Court in Addl. General Manager - Human Re source,
Bharat Heavy Electricals Ltd. vs. Suresh Ramkrishna Burde1and Bank of India and Anr.
vs. Avinash D. Mandivikar and Ors.2.
1. (2007) 5 SCC 336.
2. (2005) 7 SCC 690.
9

. Per contra, learned senior counsel ap pearing on behalf of the employee, while sup
porting the decision of the High Court, sub mitted that in the absence of any finding by
the Scrutiny Committee that the caste cer tificate had been obtained fraudulently, the
High Court was justified in directing her reinstatement. Learned counsel contended that it
has not been evolved as a general principle that in every case where the caste claim is
rejected, the equitable jurisdiction of the Court is ousted. In support of the submission
that the parameters for consid eration of claim for employment are differ ent in a case of
fraud as compared to a mere rejection of such a claim, learned counsel referred us to the
decisions of this Court in Suresh Ramkrishna Burde (supra), Manjula Sircar and Ors. vs.
Harendra Bahadur Singh and Ors.3and Punjab National Bank and Anr. Vs. Vilas, S/O
Govindrao Bokade and Anr.4.2007 AIR SCW 5822

3. AIR 2007 SC 3211.


4. 2007 (8) SCALE 108.
10. The sequence and the narration of facts above leaves little doubt in our mind that the
caste certificate, on the basis whereof the employee got employment, was false to her
knowledge. Based on that the Scrutiny Committee, on reconsideration af ter remand by
the High Court, vide order dated 29th May, 2003, again invalidated employee's caste
certificate, resulting in ter mination of the services by order dated 28th June, 2003
(supra). As noted above, the said order of the Scrutiny Committee having not been
challenged, has attained finality and remains in operation. It is, thus, not a case of mere
rejection of a claim and the cited authorities are inapplicable.
11. In the above background, the ques tions forour consideration would be : (i) whether
delay in making reference to the Scrutiny Committee for verification of the caste
certificate as also the delay on the part of the Scrutiny Committee in such verifica tion per
se vitiates the order of termination of services of an employee, even when the certificate
is ultimately found to be false and (ii) whether because of the employee having rendered
services to the employer for over twenty years, would it be equitable to can cel her
appointment, when admittedly in the first instance the employee was not eligible for such
appointment?
12

. Both the issues are no longer res Integra. The implications of misuse of the benefits
conferred by the Constitution on a particular section or sections of the citizenry were
highlightedby this Court in Kumari Madhuri Patil and Anr. vs. Addl. Commissioner,
Tribal Develop ment and Ors.5. It was said that the admission wrongly gained or
appointment wrongly obtained on the basis of false social status certificate necessarily
has the effect of depriving the genuine Scheduled Castes or Scheduled Tribes or OBC
candidates as enjoined in the Constitution of the benefits conferred on them by the
Constitution. The genuine candidates are also denied ad mission to educational
institutions or ap pointments to office or posts under a State for want of social status
certificate. There fore, with a view to streamline the proce dure for the issuance of social
status cer tificates, their scrutiny and their approval, the Court issued as many as fifteen
direc tions. One of the directions so issued, was that as soon as the finding is recorded by
the Scrutiny Committee, holding that the certificate obtained was false, on its can
cellation and confiscation simultaneously, it should be communicated to the educa tional
institution concerned, or the appoint ing authority with a request to cancel the admission
or the appointment. Thereupon, the admission or the appointment shall be cancelled
without any further notice to the candidate and the candidate shall be de barred from
further study or continue in office in a post. 1994 AIR SCW 4116

5. (1994) 6 SCC 241.


13

.Similarly, the plea regarding render ing of services for a long period has been considered
and rejected in a series of deci sions of this Court and we deem it unneces sary to launch
on exhaustive dissertation on principles in this context. It would suf fice to state that
except in a few decisions, where the admission/appointment was not cancelled because of
peculiar factual matrix 2004 AIR SCW 419
2007 AIR SCW 3111
2003 AIR SCW 4912
2005 AIR SCW 4094

@page-SC3269
obtaining therein, the consensus of ju dicial opinion is that equity, sympathy or generosity
has no place where the original appointment rests on a false caste certifi cate. A person
who enters the service by producing a false caste certificate and ob tains appointment for
the post meant for a Scheduled Caste or Scheduled Tribe or OBC, as the case may be,
deprives a genuine can didate falling in either of the said catego ries, of appointment to
that post, does not deserve any sympathy or indulgence of this Court. He who comes to
the Court with a claim based on falsity and deception can not plead equity nor the Court
would be jus tified to exercise equity jurisdiction in his favour. An act of deliberate
deception with a design to secure something, which is other wise not due, tantamounts to
fraud. Fraud is a conduct either by letter or words, which induces the other person or
authority to take a definite determinative stand as a response to the conduct of the former
either by words or letter, [See : R. Vishwanatha Pillai vs. State of Kerala and Ors.6Bank
of India (supra), Addl. Gen eral Manager (supra), Deny vs. Peek7, Ram Preeti Yadav vs.
U.P. Board of High School and Inter mediate Education and Ors.8and Bhaurao Dagdu
Paralkar vs. State of Maharashtra and Ors.9]
6. (2004) 2 SCC 105.
7. (2003) 8 SCC 311.
8. (2003) 8 SCC 311.
9. (2005) 7 SCC 605.
14. In Ram Chandra Singh vs. Savitri Devi and Ors.10, this Court had observed that fraud
is anathema to all equitable principles and any affair tainted with fraud cannot be
perpetuated or saved by the application of any equitable doctrine.
10. (2003) 8 SCC 319.
15

. Recently, in State of Maharashtra and Ors. v. Ravi Prakash Babulalsing Parmar and
Anr.11dealing with a similar situ ation, this Court observed thus : 2006 AIR SCW 6093,
(Para 23)

11. (2007) 1 SCC 80.


"The makers of the Constitution laid em phasis on equality amongst citizens. The
Constitution of India provides for protective discrimination and reservation so as to en
able the disadvantaged group to come on the same platform as that of the forward
community. If and when a person takes an undue advantage of the said beneficent pro
vision of the Constitution by obtaining the benefits of reservation and other benefits
provided under the Presidential Order al though he is not entitled thereto, he not only
plays a fraud on the society but in effect and substance plays a fraud on the Consti tution.
When, therefore, a certificate is granted to a person who is not otherwise entitled thereto,
it is entirely incorrect to contend that the State shall be helpless spectator in the matter."
16. Having considered the matter in the light of the aforestated legal position, in our
judgment, the decision of the High Court is untenable. As noted supra, the employee
having accepted the finding of the Scrutiny Committee, holding that the caste certificate
furnished by the employee was false, the very foundation of her appointment vanished
and her appointment was rendered illegal. Her conduct renders her unfit to be contin ued
in service and must necessarily entail termination of her service. Under these cir
cumstances, there is absolutely no justifi cation for her claim in respect of the post merely
on the ground that she had worked on the post for over twenty years. The post was meant
for a reserved candidate but she usurped the same by misrepresentation and deception. In
our opinion, the fact that caste certificate was referred to the Scrutiny Com mittee for
verification after ten years of her joining the service and a long time was taken by the
Scrutiny Committee to verify the same is of no consequence inasmuch as delay on both
the counts does not validate the caste certificate and the consequent illegal ap pointment.
17. We are also unable to persuade our selves to agree with learned counsel for the
employee that in the absence of any finding of fraud having been played by the
employee, the order of the High Court is equitable and should not be interfered with. As
noted above, the selection of the employee was con ceived in deceit and, therefore, could
not be saved by equitable considerations.
18. In view of the aforegoing discussion, the impugned judgment and order quash ing the
order of termination of service of the employee and directing her reinstatement cannot be
sustained. The order of termina tion based on the report of the Scrutiny Committee does
not suffer from any infir mity and the High Court should not have interfered with the
same.
@page-SC3270
19. Accordingly, the appeal is allowed, the judgment and order of the High Court dated
6th April, 2005 is set aside but with no order as to costs.
Appeal allowed.
AIR 2008 SUPREME COURT 3270 "Arjun Mahto v. State of Bihar"
(From : Patna)*
Coram : 2 Dr. A. PASAYAT AND Dr. MUKUNDAKAM SHARMA, JJ.
Criminal Appeal No.753 with 1179 and 754 of 2001, D/- 13 -8 -2008.
Arjun Mahto v. State of Bihar.
Penal Code (45 of 1860), S.396, S.395 - DACOITY - MURDER - SENTENCE
REDUCTION - Dacoity with murder-Accused armed with deadly weapons entering into
house of informant - Informant and his father seriously injured - Fatal injury caused to
one of informants' brother - Accused thereafter looted articles in house - Wit nesses
examined by prosecution were related to deceased - Out of 4 eye-wit nesses evidence of 2
found to be cogent and credible - Their evidence cannot be disbelieved because of
relationship or because other eye-witnesses were disbe lieved - Conviction of accused
proper - Passage of time held not sufficient to re duce sentence. (Paras 5, 8, 12)
Cases Referred : Chronological Paras
(2007) 1 SCC 699 (Ref.) 11
2004 AIR SCW 7376 : AIR 2005 SC 1460 (Ref.) 11
2002 AIR SCW 4271 : AIR 2002 SC 3633 : 2003 Cri LJ 41 (Ref.) 10
(2002) 3 SCC 76 (Ref.) 10
AIR 1974 SC 276 : 1974 Cri LJ 331 (Rel. on) 7
AIR 1973 SC 2407 : 1973 Cri LJ 1589 (Ref.) 10
AIR 1965 SC 202 : 1965 (1) Cri LJ 226 (Rel. on) 9
AIR 1957 SC 614 : 1957 Cri LJ 1000 (Rel. on) 7
AIR 1953 SC 364 : 1953 Cri LJ 1465 (Rel. on) 6, 8
AIR 1952 SC 54 : 1952 Cri LJ 547 (Rel. on) 8
H. L. Agrawal, Sr. Advocate, Gaurav Agrawal, Prashant Kumar, K.K. Gupta and Ms.
Naresh Bakshi, for Appellant; Gopal Singh, Manish Kumar Singh and Ms. Shweta Km.
Singh, for Respondents.
* Cri. Appeal No. 371 of 1987, D/- 18-8-2000 (Pat.)
Judgement
1. Dr. ARIJIT PASAYAT, J. :-These three appeals have a common matrix and judg ment
of a Division Bench of the Patna High Court. By the impugned judgment, the State's
appeal was dismissed while in the case of present appellants their conviction was altered
from Section 396 of the Indian Penal Code, 1860 (in short the 'IPC') to Sec tion 395 IPC.
The sentence of Rigorous Im prisonment for life was altered to seven years rigorous
imprisonment.
2. Factual position in a nutshell is as fol lows:
The case of the prosecution, briefly stated, as contained in the Fardbayan (Ext. 2) of the
informant Yusuf Ali Khan son of Khalil Bux Khan (P.W.4) is as follows :
The Fardbayan was recorded at the P.O. Village Khaira, P.S. Auras, District Gaya at 8
p.m. relating to the alleged occurrence of 7 p.m. on the same day on 15.03.1983. The
Fardbayan was recorded by Mohd. Asfaque Ali (P.W.5), the officer incharge of Bankey
Bazar outpost. It is alleged that the infor mant was at his house at about 7 p.m. Sud denly
10 armed persons entered into the house through the open door. One of them fired twice
from the gun making the infor mant injured. He fled inside the room. He was dragged
into the courtyard where his father Khalil Bux Khan (P.W. 3), and his brother-Nausher Ali
Khan (hereinafter re ferred to as the 'deceased) were sitting. On the deceased Nausher Ali
Khan injuries with Pharsa and gunshot was caused while Khalil Bux Khan (P.W.3)
suffered injuries with lathi and Pharsa. It is mentioned in the fardbeyan that the man who
caused injuries to the in formant with gun shot was of fair complex ion and a tall man
whose name he did not know. In the meantime 20-25 more persons entered through the
door and inside the house and began looting away the house hold properties. Among
them the informant identified as many as 12 persons namely, (1) Bindeshwari Sap (2)
Suraj Pasi (3) Ram Lal Chamar, (4) Baijnath Mishra, (5) Bishaum Singh, (6) Arjun
Mahato (7) Basudeo Yadav, (8) a teacher of Bankey Bazar High School resident of
village Barka Jamuara, (9) Ram Swarup Ram (10) Lalu Khan and (11) Vijoy Yadav and
Sammid Ahmed Khan. The source of identification was the light of lantern burning in the
house.
@page-SC3271
The value and the list of the articles was to be furnished subsequently. Altogether 70-80
dacoits were alleged to have participated in the dacoity. While retreating the dacoits were
raising slogans "M.C.C. Zindabad". After the dacoits had left, the informant came to
know that dacoits had also entered into the house of his uncle and grandfather and after
causing injuries to them they had also looted away a Licensee gun from their house.
It is said that during investigation after recording the Fardbayan and formal F.I.R. (Ext.3)
the I.O. sent the injured persons to the hospital for their medical treatment. Some of the
accused persons were also placed in the T.I. Parade and some of the accused were
identified by the witnesses in the T.I. Parade. One of the injured named Nausher Ali Khan
was removed to Calcutta hospital, where he died, from Magadh Hos pital Gaya for
further treatment whose P.M. Report was procured by the I.O. from Park Street Police
Calcutta.
After investigation charge-sheet was sub mitted by the officer against some of the ac
cused persons named in the F.I.R and also against some of the accused who were iden
tified in the T.I. Parade and also against some of these who were named subsequently by
the witnesses before the investigating police officer whose names neither figured in the
F.I.R. nor were said to have been iden tified in the test identification parade. Some of the
accused named in the F.I.R. were not sent up by the police for trial. However, some of
them were made accused by the orders of the trial court under Section 319 of the Code of
Criminal Procedure, 1973 (in short 'Cr.P.C.'). Some of the P.Ws were already ex amined
before charge was ordered to be framed against four of the accused under Section 319 Cr.
P.C. which resulted in the de novo trial of all the accused persons.
Altogether eight witnesses were examined on behalf of the prosecution. Three of them
i.e. Ganzaffar Ali Khan (PW 1), Zahid Ali Khan (PW 2) and Yusuf Ali Khan (PW 4) were
three sons of Khalil (PW 2) in whose house the alleged dacoity took place. PW 4 the in
formant was also seriously injured and ac cording to the prosecution sustained eleven
injuries.
The trial court placed reliance on the evi dence of all the four witnesses who claimed to
be eye witnesses. Placing reliance on their evidence, the trial court held the five
appellants in the three appeals before this Court to be guilty of offence punishable under
Sec tion 396 IPC. However, he directed acquit tal of the eleven co-accused persons. The
convicted accused persons preferred two ap peals before the High Court while the State
preferred an appeal questioning acquittal of Dr. Shamin Ahmad Khan alias Samman
Khan. The High Court held that though the evidence of PWs 1 and 3 cannot be said to be
cogent, the evidence of PWs 2 and 4 were with out blemish. Accordingly it upheld the
judg ment of the trial court, so far as finding the appellant's guilt is concerned. It found
that occurrence took place on 15.3.1983 and the deceased breathed his last long after
about a month. That being so the appropriate con viction would be under Section 395
IPC. Ac cordingly the conviction was altered and sen tence imposed was also altered.
3. In support of the appeal learned coun sel for the appellant submitted that even in the
FIR there was no consistency. Though certain persons were named and overt acts were
attributed to them the evidence in Court was at variance with the statement. So far as the
appellant Bindeshwari Rao is concerned, it is submitted that though he was named in the
FIR, the role attributed to him was not specific. It was also submitted that most of the
appellants have suffered sentence of nearly 4l/2 years and consider ing the long passage
of time, the sentence should be suitably reduced. In the case of accused appellant
Bindeshwari Rao, it is submitted that though he has suffered cus tody for about one year,
considering the unclear role attributed to him by the pros ecution his case deserves
sympathetic consideration. Witnesses were related to the deceased and their evidence
should not be accepted particularly even two of them were dis believed and the role of
Dr. Shamim Ahmad Khan which was so eloquently stated by the witnesses has been
discarded.
4. Learned counsel for the State sup ported the judgment of the High Court.
5. Merely because the eye-witnesses are family members their evidence cannot per se be
discarded. When there is allegation of interestedness, the same has to be estab lished.
Mere statement that being relatives of the deceased they are likely to falsely im plicate
the accused cannot be a ground to discard the evidence which is otherwise co gent and
credible. We shall also deal with the contention regarding interestedness of
@page-SC3272
the witnesses for furthering prosecution ver sion. Relationship is not a factor to affect
credibility of a witness. It is more often than not that a relation would not conceal actual
culprit and make allegations against ah in nocent person. Foundation has to be laid if plea
of false implication is made. In such cases, the court has to adopt a careful ap proach and
analyse evidence to find out whether it is cogent and credible.
6

. In Dalip Singh and Ors. v. The State of Punjab (AIR 1953 SC 364) it has been laid down
as under :- Para 26 of AIR

"A witness is normally to be considered independent unless he or she springs from


sources which are likely to be tainted and that usually means unless the witness has
cause, such as enmity against the accused, to wish to implicate him falsely. Ordinarily a
close relation would be the last to screen the real culprit and falsely implicate an in
nocent person. It is true, when feelings run high and there is personal cause for enimity,
that there is a tendency to drag in an inno cent person against whom a witness has a
grudge along with the guilty, but founda tion must be laid for such a criticism and the
mere fact of relationship far from being a foundation is often a sure guarantee of truth.
However, we are not attempting any sweeping generalization. Each case must be judged
on its own facts. Our observations are only made to combat what is so often put forward
in cases before us as a general rule of prudence. There is no such general rule. Each case
must be limited to and be governed by its own facts."
7

. The above decision has since been followed in Guli Chand and Ors. v. State of
Rajasthan (1974 (3) SCC 698) in which Vadivelu Thevar v. State of Madras (AIR 1957
SC 614) was also relied upon. AIR 1974 SC 276

. We may also observe that the ground that the witness being a close relative and
consequently being a partisan witness, should not be relied upon, has no substance. This
theory was repelled by this Court as early as in Dalip Singh's case (supra) in which
surprise was expressed over the impression which prevailed in the minds of the Members
of the Bar that relatives were not independent witnesses. Speaking through Vivian Bose,
J. it was observed : AIR 1953 SC 364, (Para 25)

"We are unable to agree with the learned Judges of the High Court that the testimony of
the two eyewitnesses requires corrobora tion. If the foundation for such an observa tion is
based on the fact that the witnesses are women and that the fate of seven men hangs on
their testimony, we know of no such rule. If it is grounded on the reason that they are
closely related to the deceased we are unable to concur. This is a fallacy common to many
criminal cases and one which another Bench of this Court endeav oured to dispel in,
'Rameshwar v. State of Rajasthan' (AIR 1952 SC 54 at p.59). We find, however, that it
unfortunately still persists, if not in the judgments of the Courts, at any rate in the
arguments of counsel."
9. Again in Masalti and Ors. v. State of U.P. (AIR 1965 SC 202) this Court observed : (p.
209-210 para 14) :
"But it would, we think, be unreasonable to contend that evidence given by witnesses
should be discarded only on the ground that it is evidence of partisan or interested
witnesses.......The mechanical rejection of such evidence on the sole ground that it is
partisan would invariably lead to failure of justice. No hard and fast rule can be laid down
as to how much evidence should be appreciated. Judicial approach has to be cautious in
dealing with such evidence; but the plea that such evidence should be re jected because it
is partisan cannot be ac cepted as correct."
10

. To the same effect is the decisions in State of Punjab v. Jagir Singh (AIR 1973 SC
2407), Lehna v. State of Haryana (2002 (3) SCC 76) and Gangadhar Behera and Ors. v.
State of Orissa (2002 (8) SCC 381). 2002 AIR SCW 4271

11

. The above position was also high lighted in Babulal Bhagwan Khandare and Anr. v.
State of Maharashtra [2005 (10) SCC 404] and in Salim Sahab v. State of M.P. (2007(1)
SCC 699). 2005 AIR SCW 7376

12. In the instant case, the evidence of the injured witness PW 4 is of vital impor tance.
Similar is the case of PW 2 even though PWs 1 and 3 have not been fully be lieved yet
the credibility of the evidence of PWs 2 and 4 cannot be doubted on that score. So far as
the acquittal of Dr. Shamim Ahmad Khan is concerned, the trial court and the High Court
doubted prosecution version so far as he is concerned because he was not named in the
FIR. It is a serious case of
@page-SC3273
dacoity, any leniency in sentence would not only be undesirable but also would be im
proper. The passage of time cannot wash away gravity of offence. Therefore; judged from
any angle these appeals have no sub stance and are dismissed.
13. The accused persons who are on bail, shall surrender to custody forthwith to serve
remainder of their sentence.
14. The appeals fail and are dismissed.
Appeal dismissed.
AIR 2008 SUPREME COURT 3273 "Malwa Cotton and Spinning Mills Ltd. v. Virsa
Singh Sidhu"
(From : Punjab and Haryana)*
Coram : 2 Dr. A. PASAYAT AND Dr. MUKUNDAKAM SHARMA, JJ.
Criminal Appeal No. 1265 with 1266 to 1272 of 2008 (arising out of SLP (Cri.) No. 6049
of 2005 with 408 to 414 of 2006), D/- 13 -8 -2008.
Malwa Cotton and Spinning Mills Ltd. v. Virsa Singh Sidhu and Ors.
(A) Negotiable Instruments Act (26 of 1881), S.138, S.141 - Criminal P.C. (2 of 1974),
S.482 - NEGOTIABLE INSTRUMENT - INHERENT POWERS - DISHONOUR OF
CHEQUE - Dishonour of cheque - Offence by Company - Quashing of proceedings -
Accused, one of Directors - Plea that he had intimated Company of his desire to retire
before cheque in ques tion was issued - Cannot be ground to quash proceedings against
him.
Cri. Misc. No.52153-M of 2002, D/-10-02-2005 (PandH), Reversed.
In case of prosecution of Directors of Com pany for dishonour of cheque the plea raised
by one of the Directors that he had intimated the Company of his desire to resign much
before the cheque was issued, cannot be ground to quash the proceedings initiated against
him. The plea raised, gives rise to questions of facts, such as, whether the ac cused-
Director had intimated the Company and whether there was any resolution ac cepting his
desire and what was the effect of delay by Company in submitting requisite form before
Registrar of Companies. Resolution of such questions require lead ing of evidence and
can only be decided in Trial.
Cri. Misc. No.52153-M of 2002, D/-10-02-2005 (PandH), Reversed. (Paras 6, 11)
(B) Negotiable Instruments Act (26 of 1881), S.138, S.141 - DISHONOUR OF CHEQUE
- INHERENT POWERS - Dishonour of cheque - Offence by Company - Pros ecution of
all Directors - Fact that alle gations against Directors are general - Not ground to quash
proceedings.
Criminal P.C. (2 of 1974), S.482. (Para 12)
Cases Referred : Chronological Paras
2007 AIR SCW 2591 : AIR 2007 SC 1682 : 2007 Cri LJ 2448 : 2007 CLC 860 (Rel. on,
Pnt. B) 10
2005 AIR SCW 2364 : AIR 2005 SC 2436 : 2005 Cri LJ 2566 (Rel. on, Pnt. B) 9
1995 (2) AC 500 (PC) 10
1915 AC 705 (HL) 10
Sanjay Kapur, Ms. Shubhra Kapur, Rajiv Kapur and Ms. Arti Singh, for Appellant; P.P.
Singh, for Respondents.
* Cri. Misc. No. 52153-M of 2002, D/- 10-2-2005 (PandH).
Judgement
Dr. ARIJIT PASAYAT, J. :- SLP (Crl.) 6049/2005
1. Leave granted.
2. Challenge in this appeal is to the or der passed by a learned single Judge of the Punjab
and Haryana High Court accepting the prayer of respondent No. 1 for quashing the
proceedings pending before the Judicial Magistrate, First Class, Ludhiana. The pro
ceedings related to the complaint filed by the appellant alleging commission of offence
punishable under S. 138 of the Negotiable Instruments Act, 1881 (in short the 'Act'). In
all, 8 petitions were filed which were dis posed of by the common judgment.
3. The present appeals relate to Criminal Miscellaneous No. 52153 of 2002 and con
nected cases. The High Court quashed the proceedings primarily on the ground that re
spondent No. 1-Virsa Singh Sidhu in the first case had resigned from the Directorship be
fore the cheques were issued. The other pe titions were allowed on the ground that there
were some general allegations that all Di rectors were responsible.
4. Learned counsel for the appellant sub mitted that the High Court's judgment is clearly
unsustainable. So far as respondent No. 1 is concerned he claims to have re signed on 2-
4-1999 whereas cheques were issued on various dates viz. in December, 2000 and
February, 2001. It is pointed out that the Form No. 32 which was required to be filed with
Registrar of Companies was filed on 5-7-2001 i.e. much after the cheques were issued.
Whether in fact the respondent
@page-SC3274
No. 1's claim to have resigned was factually correct would have been established in trial
and the High Court could not have passed the impugned judgment while dealing with the
application under S. 482 of the Code of Criminal Procedure, 1973 (in short the 'Code')? It
is further pointed out that the High Court was not justified in holding that there was no
specific allegation against other accused persons. With reference to the com plaint it was
pointed out that specific alle gation is to the effect that the accused per sons were in-
charge of day-to-day manage ment work. In any event, this is not a ques tion which could
have been gone into in a proceeding under S. 482 of Code. It is a matter of trial.
5. Learned counsel for respondent No. 1 on the other hand submitted that the High Court
was justified in its view that respon dent No. 1 had intimated the company about his
desire to resign. If the company delayed in submitting the requisite Form before the
Registrar of Companies, he cannot be made to suffer.
6. As rightly contended by learned coun sel for the appellant factual disputes are
involved. What was the effect of delayed pre sentation before the Registrar of Companies
is essentially a matter of trial. Whether re spondent No. 1 had intimated the company and
whether there was any resolution ac cepting his resolution are matters in respect of which
evidence has to be led. Therefore, the High Court was not justified in its view.
7. So far as allegations against the Di rectors are concerned about their position in the
company the complaint specifically contained the averments regarding the po sition of
the accused-Directors in the com pany.
8. At this juncture, it would be relevant to take note of certain observations made by this
Court in various cases.
9

. In S.V. Muzumdar v. Gujarat State Fertilizer Co. Ltd. and Anr. (2005 (4) SCC 173), it
was inter alia ob served as follows : 2005 AIR SCW 2364

"3. The facts as projected by the respon dents in the complaint were to the effect that the
respondent No. 1 (hereinafter referred to as the 'complainant') supplied goods on credit to
M/s. Garware Nylons Ltd. (herein after referred to as the 'Company') (accused No. 14).
Cheques issued by the company were not honoured by the drawee Bank on the ground of
insufficient funds. Payments were not made even after legal notices. There were 1.4
accused persons including the com pany named in the complaint. Some of the accused
persons were Directors and while others were employees. Learned Chief Ju dicial
Magistrate, Vadodara after recording statement of marketing manager who had filed the
complaint for himself and on be half of the complainant-company, issued summons to all
the accused persons for fac ing trial for alleged commission of offences punishable under
S. 138 of the Act read with Ss. 420 and 114 of the Indian Penal Code, 1860 (in short the
'IPC'). The order issuing summons was challenged by filing criminal revision applications
which were dismissed by order dated 21-3-1996. Said common judgment and order was
challenged before the High Court by filing special criminal ap plications and these
applications were permitted to be withdrawn to enable the appel lants to move
applications before the learned Chief Judicial Magistrate- as stated by the petitioners.
Application was filed with prayer to drop proceedings. That application was rejected by
order dated '21-8-1997. Same was questioned before the High Court. The challenge
before the High Court was prima rily on the ground that there was no mate rial to show
that the accused persons at the time of offence as allegedly committed were in-charge
and/or responsible to the com pany for the conduct of the business as re quired under S.
141(1) of the Act. It was also submitted that the deeming provision un der sub-section (2)
of S. 141 which covers persons with whose consent or connivance or any attributable
negligence for commis sion of the offence by the company was also not applicable. The
High Court did not ac cept the pleas and held that the controversy was to be adjudicated
at the trial. It consid ered the petition to be unacceptable attempt to stall the criminal
proceedings at the threshold.
xx xx xx xx xx
8. We find that the prayers before the Courts below essentially were to drop the
proceedings on the ground that the allega tions would not constitute a foundation for
action in terms of S. 141 of the Act. These questions have to be adjudicated at the trial.
@page-SC3275
Whether a person is in-charge of or is re sponsible to the company for conduct of
business is to be adjudicated on the basis of materials to be placed by the parties. Sub
section (2) of S. 141 is a deeming provision which as noted supra operates in certain
specified circumstances. Whether the re quirements for the application of the deem ing
provision exist or not is again a matter for adjudication during trial. Similarly, whether
the allegations contained are suffi cient to attract culpability is a matter for adjudication at
the trial.
9. Under scheme of the Act, if the person committing an offence under S. 138 of the Act
is a company, by application of S. 141 it is deemed that every person who is in-charge of
and responsible to the company for con duct of the business of the company as well as
the company are guilty of the offence. A person who proves that the offence was com
mitted without his knowledge or that he had exercised all due diligence is exempted from
becoming liable by operation of the proviso to sub-section (1). The burden in this re gard
has to be discharged by the accused.
10. The three categories of persons cov ered by S. 141 are as follows :
(1) The company who committed the of fence.
(2) Everyone who was in-charge of and was responsible for the business of the com pany.
(3) Any other person who is a director or a manager or a secretary or officer of the
commpany with whose-connivance or due to whose neglect the Company has commit ted
the offence.
11. Whether or not the evidence to be led would establish the accusations is a matter for
trial. It needs no reiteration that proviso to sub-section (1) of S. 141 enables the ac cused
to prove his innocence by discharg ing the burden which lies on him."
10

. In N. Rangachari v. Bharat Sanchar Nigam Ltd. (2007 (5) SCC 108), it was observed as
fol lows : 2007 AIR SCW 2591, Para 13

"19. Therefore, a person in the commer cial world having a" transaction with a com pany
is entitled to presume that the Direc tors of the company are in-charge of the af fairs of
the company. If any restrictions on theif powers are placed by the memoran dum or
articles of the company, it is for the Directors to establish it at the trial. It is in that
context that S. 141 of the Negotiable Instruments Act provides that when the of fender is
a company, every person, who at the time when the offence was committed was in-charge
of and was responsible to the company for the conduct of the business of the company,
shall also be deemed to be guilty of the offence along with the company. It appears to us
that an allegation in the complaint that the named accused are Di rectors of the company
itself would usher in the element of their acting for and on be half of the company and of
their being in charge of the company. In Gower and Davies' Principles of Modern
Company Law (17th Edn.), the theory behind the idea of identifi cation is traced as
follows :
"It is possible to find in the cases varying formulations in the underlying principle, and
the most recent definitions suggest that the Courts are prepared today to give the rule of
attribution based on identification a somewhat broader scope. In the original for mulation
in Lennard's Carrying Company case (1915 AC 705 (HL) Lord Haldane based
identification on a person 'who is really the directing mind and will of the Corporation,
the very ego and centre of the personality of the Corporation.' Recently, however, such an
approach has been castigated by the Privy Council through Lord Hoffmann in Merid ian
Global case (1995 (2) AC 500 (PC)) as a misleading 'general metaphysic of compa nies.'
The true question in each case was who as a matter of construction of the stat ute in
question, or presumably other rule of law, is to be regarded as the controller of the
company for the purpose of the identifi cation rule."
11. Therefore, the High Court was not justified in quashing the proceedings so far as
respondent No. 1 in the first case is con cerned. The appeal is allowed.
12. In view of the order passed in crimi nal appeal arising out of S.L.P (Cri.) No.
6049/2005, where details have been indi cated, other appeals deserved to be allowed. The
impugned order of the High Court in each case is set aside.
Appeal allowed.
@page-SC3276
AIR 2008 SUPREME COURT 3276 "Vinay Kumar Rai v. State of Bihar"
(From : 2005 Cri LJ 3929 (Patna))
Coram : 2 Dr. A. PASAYAT AND P. SATHASIVAM, JJ.
Criminal Appeal No.371 with 382 of 2006, D/- 18 -8 -2008.
Vinay Kumar Rai and Anr. v. State of Bihar.
(A) Evidence Act (1 of 1872), S.3 - EVIDENCE - MURDER - WITNESS - Murder case-
Eye-witnesses - Relation ship is not a factor to affect credibility of witness - Allegation of
interestedness of witness has to be established.
AIR 1953 SC 364; ADR 1974 SC 276; AIR 1957 SC 614; AIR 1952 SC 54; AIR 1965
SC 202; AIR 1973 SC 2407; 2002 (3) SCC 76; 2002 AIR SCW 4271: 2004 AIR SCW
7376; 2007 (1) SCC 699, Rel. on. (Paras 6, 9)
(B) Evidence Act (1 of 1872), S.3 - EVIDENCE - MURDER - WITNESS - Murder case -
Natural witnesses - Over insistence on outsider witnesses who have not seen anything as
against natu ral witnesses often results in criminal injustice.
1999 AIR SCW 1914, Rel. on. (Para 13)
(C) Penal Code (45 of 1860), S.300, S.34 - MURDER - COMMON INTENTION -
EVIDENCE - WITNESS - Murder - Proof - Appellants alleg edly fired on deceased with
pistol - Evi dence of eye-witness established that ac cused persons caught hold of
deceased and on their exhortation appellant shot at deceased - Medical evidence was
clearly in line with what has been stated by eye-witnesses - There was no discrep ancy
between ocular evidence and medi cal evidence as alleged - Conviction of appellant and
others is proper. (Paras 14, 15)
Cases Referred : Chronological Paras
(2007) 1 SCC 699 (Rel. on) 12
2004 AIR SCW 7376 : AIR 2005 SC 1460 (Rel. on) 12
2002 AIR SCW 4271 : AIR 2002 SC 3633 : 2003 Cri LJ 41 (Rel. on) 11
(2002) 3 SCC 76 (Rel. on) 11
1999 AIR SCW 1514 : AIR 1999 SC 1776 : 1999 Cri LJ 2588 (Rel. on) 13
AIR 1974 SC 276 : 1974 Cri LJ 331 (Rel. on) 8
55 AIR 1973 SC 2407 : 1973 Cri LJ 1589 (Rel. on) 11
AIR 1965 SC 202 : 1965 (1) Cri LJ 226 (Rel. on) 10
AIR 1957 SC 614 : 1957 Cri LJ 1000 (Rel. on) 8
AIR 1953 SC 364 : 1953 Cri LJ 1465 (Rel. on) 7, 9
AIR 1952 SC 54 : 1952 Cri LJ 547 (Rel. on) 9
Nagendra Rai, Rajeev Singh, Anshul Raj and Rajesh Prasad Singh, for Appellants; Gopal
Singh and Anukul Raj, for Respon dent.
Judgement
1. Dr. ARIJIT PASAYAT, J. :- Challenge in these appeals is to the judgment of a Di vision
Bench of the Patna High Court up holding the conviction of the appellants for offence
punishable under Section 302 read with Section 34 of the Indian Penal Code, 1860 (in
short 'IPC') so far as accused Ajeet Kumar Rai alias Ajeet Narayan Rai and Vinay Kumar
Rai, appellant No. 1 in Criminal Ap peal No. 371 of 2006 and Ashutosh Kumar Rai alias
Sanjay Kumar Rai, appellant in other Criminal Appeal. Ashutosh Kumar Rai was further
charged for committing the murder of Nanda Kumar Singh punishable under Section 302,
IPC and Section 27 of the Arms Act, 1959 (in short 'Arms Act'). The High Court
dismissed the appeals. The present appeals had been filed by Vinay Kumar Rai (A-3) and
Ajeet Kumar Rai alias Ajeet Narayan Rai (A-1) and Ashutosh Kumar Rai (A-2). The
Presiding Officer, Additional Court No. 1, Fast Track Court in Sessions Trial Nos. 578/96
and 1/2001 held Ajeet Kumar Rai and Vinay Kumar Rai guilty under Section 302 read
with Section 34, IPC and accused Ashutosh Kumar Rai under Section 302, IPC and
sentenced to undergo RI for life. He was also found guilty of of fence under Section 27 of
the Arms Act and sentenced to undergo Rl for three years. Two appeals were filed before
the High Court which by the impugned judgment dismissed the same. All accused were
put on trial for committing the murder of Nanda Kumar Singh (hereinafter referred to as
the 'de ceased') in furtherance of their common in tention for offence punishable under
Sec tion 302 read with Section 34, IPC.
2. Prosecution version in a nutshell is as follows :
According to the first information report given by Vishwanath Singh (PW-7) before the
police on 26-7-1996 at 1.10 p.m., at about 12 noon, while he was sitting on the veran dah
of the house and his son Nand Kumar Singh, the deceased had gone to the field to
@page-SC3277
inquire as to whether the land has been ploughed or not, he did not find tractor there and
while he was returning he saw the ap pellants and started shouting. Hearing the alarm, the
informant along with Sachida Nand Singh (PW-4) rushed there and found that appellants
Vinay Kumar Rai and Ajeet Kumar Rai alias Ajeet Narayan Rai had caught hold of his
son and appellant Ashutosh Kumar Rai alias Sanjay Kumar Rai had put pistol on his right
temple. The mo ment they saw him and Sachinda Nand Singh, appellant Vinay Kumar
and Ajeet Kumar Rai alias Ajeet Narayan Rai exhorted to fire at which appellant
Ashutosh Kumar Rai alias Sanjay Kumar Rai fired at his son on the temple. Sustaining
the injuries his son fell down and all the appellants fled away brandishing the pistol.
When the informant and his nephew Sachidanand Singh reached there, they found injury
above the temple and immediately put him on a rickshaw and brought to the Government
Hospital, Sasaram where the doctor declared him brought dead. On the basis of the
aforesaid information, Sasaram (M) P.S. Case No. 386 of 1996 was registered under
Section 302 / 34 of IPC and 27 of the Arms Act.
According to the first information report, the motive for the occurrence is the pen dency
of litigation before the Director of Consolidation.
The police, after Investigation, submitted charge-sheet against the appellants and they
were ultimately committed to the Court of Session where all the appellants were charged
for offence under Section 302/34 of IPC whereas, appellant-Ashutosh Kumar Rai alias
Sanjay Kumar Rai was further charged for offence under Section 302 of IPC and Sec tion
27 of the Arms Act.
The appellants denied to have commit ted any offence and pleaded false implica tion on
account of previous enmity and their further defence was that the deceased was killed on
the same day at about 12 noon by fire-arm by some unknown persons near the house of
Ram Nagina Singh.
Prosecution in order to substantiate the accusations examined nine witnesses out of which
Sachidanand (PW-4), Sunil Kumar Singh (PW-5), Srikant Singh (PW-6) and Vishwa
Nath Singh (PW-7) claimed to be eye witnesses. The last named person was the
informant. In order to prove their innocence, the accused persons" examined four wit-
nesses. The trial Court believed the evidence of the eye-witnesses and found the accused
guilty.
3. In appeal, it was submitted that there was discrepancy between the medical evi dence
and the oral evidence and; therefore, the prosecution version should not have been
accepted. The primary stands were regarding the alleged discrepancy between the
medical evidence and the ocular evidence and the eye-witnesses being related to the
deceased. The High Court did not find any substance in any of these stands and dis
missed the appeals.
4. The stands taken before the High Court were reiterated in these appeals. It was high
lighted by learned counsel for the appellants that it is unusual that eye-witnesses who are
closely related to the deceased did not try to intervene to save the deceased from the
assailants.
5. Learned counsel for the State on the other hand with reference to the conclusions of the
High Court pointed out that the wit nesses have said that they saw the incident from a
distance of about 15 to 20 yards. They stated that the deceased was shot dead even before
they reached the place of occurrence. Therefore, there was no question of inter vening to
save the life of the deceased.
6. Merely because the eye-witnesses are family members their evidence cannot per se be
discarded. When there is allegation of interestedness, the same has to be estab lished.
Mere statement that being relatives of the deceased they are likely to falsely im plicate
the accused cannot be a ground to discard the evidence which is otherwise co gent and
credible. We shall also deal with the contention regarding interestedness of the witnesses
for furthering prosecution ver sion. Relationship is not a factor to affect credibility of a
witness. It is more often than not that a relation would not conceal actual culprit and
make allegations against an in nocent person. Foundation has to be laid if plea of false
implication is made. In such cases, the court has to adopt a careful ap proach and analyse
evidence to find out whether it is cogent and credible.
7

. In Dalip Singh and Ors. v. the State of Punjab, (AIR 1953 SC 364) it has been laid down
as under :- Para 26 of AIR

"A witness is normally to be considered independent unless he or she springs from


sources which are likely to be tainted and
@page-SC3278
that usually means unless the witness has cause, such as enmity against the accused to
wish to implicate him falsely. Ordinarily a close relation would be the last to screen the
real culprit and falsely implicate an in nocent person. It is true, when feelings run high
and there is personal cause for enmity, that there is a tendency to drag in an inno cent
person against whom a witness has a grudge along with the guilty, but founda tion must
be laid for such a criticism" and the mere fact of relationship far from being a foundation
is offer a sure guarantee of truth. However, we are not attempting any sweeping
generalization. Each case must be judged on its own facts. Our observations are only
made to combat what is so often put forward in cases before us as a general rule of
prudence. There is no such general rule. Each case must be limited to and be governed by
its own facts."
8

. The above decision has been followed in Guli Chand and Ors. v. State of Rajasthan
(1974 (3) SCC 698) in which Vadivelu Thevar v. State of Madras (AIR 1957 SC 614) was
also re lied upon. AIR 1974 SC 276

. We may also observe that the ground that the witness being a close relative and
consequently being a partisan witness, should not be relied upon, has no substance. This
theory was repelled by this Court as early as in Dalip Singh's case (supra) in which
surprise was expressed over the impression which prevailed in the minds of the Members
of the Bar that relatives were not independent witnesses. Speaking through Vivian Bose,
J. it was observed : AIR 1953 SC 364, Para 25

"We are unable to agree with the learned Judges of the a High Court that the testi mony of
the two eye-witnesses requires cor roboration. If the foundation for such ah ob servation
is based on the fact that the wit nesses are women and that the fate of seven men hangs on
their testimony, we know of no such rule. If it is grounded on the reason that they are
closely related to the deceased we are unable to concur. This is a fallacy common to many
criminal cases and one which another Bench of this Court endeav oured to dispel in -
'Rameshwar v. State of Rajasthan', (AIR 1952 SC 54 at p. 59). We find, however, that it
unfortunately still per sists, if not in the judgments of the Courts, at any rate in the
arguments of counsel."
10. Again in Masalti and Ors. v. State of U. P., (AIR 1965 SC 202) this Court observed
(Pp. 209-210, Para 14) :
"But it would, we think, be unreasonable to contend that evidence given by witnesses
should be discarded only on the ground that it is evidence of partisan or interested
witnesses ........... The mechanical rejection of such evidence on the stole ground that it is
partisan would invariably lead to failure of justice. No hard arid fast rule can be laid
down as to how much evidence should be appreciated. Judicial approach has to be
cautious in dealing with such evidence; but the plea that such evidence should be re
jected because it is partisan cannot be ac cepted as correct."
11

. To the same effect is the decisions in State of Punjab v. Jagir Singh, (AIR 1973 SC
2407); Lehna v. State of Haryana, (2002 (3) SCC 76) and Gangadhar Behera and Ors. v.
State of Orissa, (2002 (8) SCC 381). 2002 AIR SCW 4271

12

. The above position was also high lighted in Babulal Bhagwan Khandare and Anr. v.
State of Maharashtra, (2005 (10) SCC 404) and in Salim Sahab v. State of M. P. (2007 (1)
SCC 699). 2004 AIR SCW 7376

13

. The over insistence on witnesses hav ing no relation with the victims often results in
criminal justice going away. When any incident happens in a dwelling house the most
natural witnesses would be the in mates of that house, It is unpragmatic to ignore such
natural witnesses and insist on outsiders who would no thave even seen any thing. If the
Court has discerned from the evidence or even from the investigation records that some
other Independent per son has witnessed any event connecting the incident in question
then there is justifica tion for making adverse Comments against non-examination of such
person as prosecu tion witness. Otherwise, merely on surmises the Court should not
castigate a prosecu tion for not examining other persons of the locality as prosecution
witnesses. Prosecu tion can be expected examine only those who have witnessed the
events and not those whohavenot seen it though the neighbourhood may be replete with
other residents also. [See; State of Rajasthan v. Teja Ram and Ors., (AIR 1999 SC 1776)].
1999 AIR SCW 1514

14. It is to be noted that PWs-4 and 7


@page-SC3279
have stated in their evidence about the pres ence of PWs-5 and 6. Interestingly, Raj
Kumar Singh (PW-1) in his statement also stated about their presence at the place of
occurence. Though it was pleaded by learned counsel for the appellants that PW-1's evi
dence cast a doubt about the prosecution version, it is to be noted that PW-1 never
claimed to be an eye-witness. The evidence of PWs and eye-witnesses clearly established
that accused Ajeet Kumar Rai and Vinay Kumar Rai caught hold of deceased and on their
exhortation appellant Ashutosh shot at the deceased. Therefore, Section 34 has
application.
15. Coming to the alleged discrepancy between the ocular evidence and the medi cal
evidence as rightly noted by the High Court there was no discrepancy. The medi cal
evidence was clearly in line with what has been stated by eye-witnesses. The High Court
has noted that the expression used by the witnesses cannot be analysed in hy pothetical
manner. According to the eye-wit nesses gunshot injury was caused on the right temple
but the injury was found on the upper eyelid arid everted wound on the right oricle
margin. Therefore, it can never be said that medical evidence is contrary to the ocular
evidence.
16. Looked at from any angle, the appeals are without merit and deserve dismissal which
we direct.
Appeals dismissed.
AIR 2008 SUPREME COURT 3279 "State of Punjab v. Kulwant Singh"
(From : 2000 (2) Chand Cri C 116 (Punj and Har))
Coram : 3 Dr. A. PASAYAT, P. SATHASIVAM AND Dr. MUKUNDAKAM
SHARMA, JJ.
Criminal Appeal No.493 of 2001, D/- 16 -7 -2008.
State of Punjab v. Kulwant Singh alias Kanta.
(A) Criminal P.C. (2 of 1974), S.386 - APPEAL - HIGH COURT - MURDER - Appeal
against acquittal - Powers of High Court - Murder trial - Witness clamming to be eye-
witness examined after a month of incident - Non-examination of Inves tigating Officer
on that aspect - No rea son indicated for such non-examination - High Court finding that
there was ab normal delay in lodging F.I.R. - High Court also taking note of fact of non-
examination of person who took injured to hospital, to be a vulnerable factor - Non-
explanation for delay in sending accused to hospital-Non-explanation of injuries on
accused - Order of acquittal by High Court on taking note of above aspects - Cannot be
faulted with.
Penal Code (45 of 1860), S.300. (Para 14)
(B) Penal Code (45 of 1860), S.300 - MURDER - EVIDENCE - Murder-Injuries on
accused - Evidence of doctor that injuries were on account of dragging him on surface
littered with broken glasses - Such injuries being se rious should be explained by
prosecution.
Where the evidence of the doctor who examined the accused clearly stated that the
injuries were on account of dragging him on surface littered with broken glasses, such
injuries being of serious nature had to be explained. Though non-explanation of the minor
injuries could not be a factor to make the prosecution version vulnerable in all cases, but
if the injuries were of serious nature the prosecution has to explain that aspect. (Paras
5, 6)
Cases Referred : Chronological Paras
2007 AIR SCW 1850 : 2007 Cri LJ 2136 : 2007 (3) AIR Kar R 235 (Ref.) 13
2006 AIR SCW 177 : AIR 2006 SC 831 : 2006 Cri LJ 799 (Ref.) 13
2002 AIR SCW 1532 : AIR 2002 SC 1621 : 2002 Cri LJ 2024 (Ref.) 13
2000 AIR SCW 1430 : AIR 2000 SC 1833 : 2000 Cri LJ 2212 (Ref.) 13
1998 AIR SCW 1255 : AIR 1998 SC 1376 : 1998 Cri LJ 2034 (Ref.) 13
1996 AIR SCW 2438 : AIR 1996 SC 2035 : 1996 Cri LJ 2867 (Ref.) 13
AIR 1979 SC 387 : 1980 Cri LJ 812 : 1979 All LJ 137 (Ref.) 12
AIR 1973 SC 2622 : 1973 Cri LJ 1783 (Ref.) 11
AIR 1963 SC 200 : 1963 (1) Cri LJ 235 (Ref.) 10
AIR 1961 SC 715 : 1961 (1) Cri LJ 766 (Ref.) 8, 10
AIR 1956 SC 217 : 1954 Cri LJ 421 (Ref.) 8
AIR 1954 SC 36 : 1954 Cri LJ 331 (Ref.) 8
AIR 1945 PC 151 (Ref.) 8
AIR 1934 PC 227 (2) (Ref.) 7, 9, 10, 12
Ajay Pal, Rohit Walecha and Kuldip Singh, for Appellant; Mrs. Kawaljit Kochar, Ms.
Sweta and Ms. Kusum Chaudhary, for Re spondent.
@page-SC3280

Judgement
1. Dr. ARIJIT PASAYAT, J. :-Challenge by the State of Punjab in this appeal is to the
acquittal of the respondent from the charge of commission of offence punishable under
Section 302 of the Indian Penal Code, 1860 (in short the 'IPC'). The learned Ses sions
Judge, Faridkot, had convicted the respondent for the said offence and awarded death
sentence. In view of the award of the death sentence, reference was made to the High
Court under Section 366 of the Code of Criminal Procedure, 1973 (in short the 'Code').
2. Prosecution version as unfolded dur ing trial is as follows :
Law was set on motion on the basis of the statement made by Parminder Singh (PW4)
who stated that he is running a mar riage palace known as Chahal Marriage Pal ace on
Malout Road, at Muktsar. Ashok Kumar, Lalji Tiwari and Sham Sunder sons of Sager
Ram, residents of Gangoli Khurd, Distt. Gaunda [U.P.] and Kulwant Singh alias Kanta
son of Mohinder Singh were employed by him as servants in that marriage palace. Gurpal
Singh son of Mukhtiar Singh, was employed as Chowkidar in the said marriage palace.
There was a function in the mar riage palace on the evening of 26.8.1996. Manjit Singh
Mistri, resident of Malout along with other labourers was constructing sheds in that
marriage palace for the last many days. After the conclusion of the function he went to
his house and told his servants to look after the property of the marriage palace. On
27.8.1996 at about 7.00 a.m. Manjit Singh Mistri came to his house and told him that
smell was coming out of the rooms of the marriage palace. Hearing this he along with his
brother Parminder Singh and Mistri Manjit Singh went to the mar riage palace and saw
that smoke was com ing out of the marriage palace. He along with Raminder Singh and
Manjit Singh Mistri saw from the back side door of the marriage pal ace that the dead
bodies of Ashok Kumar and Sham Sunder (hereinafter referred to as deceased by name)
were burning there. They also saw that pieces of glass were scat tered in the lobby of the
marriage palace and Lalji Tiwari (hereinafter referred to as de ceased by name) was lying
dead in the ad joining room and there were mark 'of injuries on his head and the blood
was coming out of the injuries. When they came out, they saw that Gurpal Singh
Chowkidar was ly ing unconscious in the grassy ground of the marriage palace and his
head was stained with blood. A vehicle was arranged and Gurpal Singh was sent to the
Civil Hospital, Muktsar, with Manjit Singh. He along with his brother Raminder Singh
went inside and when Kulwant Singh moved a little, think ing that he was alive, they
picked him up and got him admitted in the Civil Hospital. He left his brother Raminder
Singh with the dead bodies. He suspected that the offence was committed by respondent-
Kulwant Singh alias Kanta due to some grievance. There was no injury on the body of
Kulwant Singh and three persons had been murdered and the fourth was lying in serious
condi tion, as such it appeared that it was an act of Kulwant Singh. After the registration
of the case S.I. Beant Singh, who was posted as S.H.O. in Police Station City Muktsar, at
that time, along with ASI Ravel Singh, ASI Gurmel Singh and other officials and
Parminder Singh went to the place of oc currence. Raminder Singh (PW5) was found
present near the dead bodies : Inquest re ports of Lalji Tiwari, Ashok Kumar and Sham
Sunder were prepared. There were hair in the right hand of the dead body and piece of
cloth in the left hand of Lalji Tiwari. The dead bodies along with the requests were sent
for post-mortem examination through ASI Gurmel Singh. S.I. Beant Singh in spected the
place of occurrence. One bed sheet stained with blood, blood stained earth were picked
up from the bed room where the dead body of Lalji Tiwari was lying. These were made
into parcels and sealed with the seal bearing mark 'BS' and the parcels were taken into
possession. Pieces of glass were picked up from the lobby and they were made into parcel
and were taken into pos session. Ashes were picked up from the room where the dead
bodies of Sham Sunder and Ashok Kumar were lying. These were made into parcel and
sealed with the seal bearing impression 'BS' and were taken into pos session. Burnt mat
was picked up and made into parcel and taken into possession. Blood stained earth was
picked up from the place where the Chowkidar was lying. The same was made into parcel
and sealed with the seal bearing mark 'BS' and taken into pos session. Blood-stained
ashes were also picked up and made into parcel and sealed with the seal bearing mark
'BS'. Rough site plan of the place of occurrence was prepared. Statements of the PWs
were recorded. Then Beant Singh S.I. went to the Hospital. Accused
@page-SC3281
Kulwant Singh, who was admitted in the hospital was interrogated and on interrogation
he disclosed that he had kept concealed one iron rod, his shirt and pant stained with blood
underneath the empty cement bags lying in the store of marriage palace and he could get
the same recovered. The disclosure statement of the accused was recorded. Then the
accused was got discharged from the Hospital. After that the accused in accordance with
his disclosure statement got recovered an iron rod, pant and shirt stained with blood. A
separate parcel of the iron rod and another parcel of the pant and shirt were prepared and
the same were taken into possession. Before making the parcel of the shirt a piece was
taken out of the same and was taken into possession. After the post-mortem examination
ASI Gurmel Singh produced the belongings of the deceased and these were taken into
possession after making into parcel. On return to the Police Station the case property was
deposited with the M.H.C. On 30.8.1996 the accused was taken to the Hospital where a
sample of his hair was taken by Dr. Meena Jagga, made into a parcel and sealed by the
doctor and the said parcel was taken into possession. On return to the Police Station the
case property was deposited with the M.H.C. The parcels of the hair, pant, shirt of the
accused, chadar, blood which were picked up from near the bed, burnt pieces of cloth and
the soil picked up from near the place where Gurpal Singh was lying and pieces of cloth
of the shirt and the parcel of iron rod were sent to the Forensic Science Laboratory and
the reports regarding the same were received. Statements of the PWs were recorded and
after the completion of the investigation the accused was sent up for trial.
The charge against the accused was for the alleged commission of offence punishable
under Section 302, I.P.C. for having committed the murders of Sham Sunder, Lalji Tiwari
and Ashok Kumar and under Section 307, I.P.C. for causing injuries to Gurpal Singh was
framed on 16.1.1997. The accused pleaded innocence and claimed trial.
The prosecution to prove its case against the accused examined Dr. P.N. Girdhar (PW1),
Dr. Meena Jagga (PW2), Dr. Madan Gopal Sharma (PW3), Parminder Singh (PW4),
Raminder Singh (PW5), Gurpal Singh (PW6), Baljit Singh (PW7), Beant Singh S.I.
(PWS), Kulwant Chand M.H.C. (PW9), Constable Harbans Lal as (PW10), Constable
Balwindar Singh (PW11) and ASI Gurmel Singh (PW12). The reports of the Forensic
Science Laboratory Ex.P.GG, Ex.P.HH. Ex.P.JJ and Ex.P.KK have also been tendered in
evidence.
The trial Court found that the evidence on record which was substantial in nature clearly
established the guilt of the accused as there were three persons who had lost their lives
and, therefore, the death sentence was awarded. In appeal, the High Court upset the
conviction and directed acquittal.
3. Learned counsel for the appellant-State submitted that this was a case where the
accusations were established against the accused and, therefore, the High Court was not
justified in interfering with the well-reasoned and elaborate judgment of the trial Court.
4. In response, learned counsel for the accused-respondent supported the judgment of the
High Court.
5. It is to be noted that the pivotal witness was PW-6. He claimed to be an eyewitness. In
that sense, this was not a case where prosecution relied on circumstantial evidence. The
High Court found that this witness was examined after about a month of the incident. The
only explanation offered was that the witness was hospitalized and was lying in
unconscious state for about one week. Even if that be so, no explanation was offered as to
why after PW-6 was released from the hospital he was not examined for about three
weeks. The Investigating Officer who could have thrown light on this aspect was not
examined. No reason was indicated for such non-examination. The other relevant factor
is that the alleged incident took place around 12 midnight. The information was lodged at
about 8.30 a.m. PW-4 stated that he learnt about the incident from Manjit Singh Mistri at
about 7.00 a.m.and when he went to the place of occurrence he found dead bodies of two
persons.From the back side of the hall he found that dead bodies of the two deceased
persons were burnt. He noticed this aspect along with his brother Raminder Singh and
Manjit Singh Mistri. The respondent-accused was lying on scattered pieces of glass in the
lobby in front of the adjoining bed room. Blood was oozing from his injured head. On
coming out he found Gurpal Singh (PW-6) lying unconscious
@page-SC3282
on the grassy ground. He made arrangements for sending Gurpal Singh to the Civil
Hospital, Muktsar alongwith Manjit Singh Mistri. He went inside and when he saw that
the accused-respondent was alive, he was also sent to the Civil Hospital, Muktsar. After
that he claimed to have gone to the Police Station to lodge the report. The High Court
found that there was abnormal delay in lodging the FIR. Learned counsel for the
appellant-State submitted that the informant Parminder Singh (PW4) was told by Manjit
Singh around 7.00 a.m. and some time was spent for taking the injured person to the
hospital and thereafter the FIR was lodged and, therefore, there was in fact no delay. It is
to be noted that the High Court found that the aforesaid Manjit Singh was not examined
as a witness. His evidence would have thrown considerable light as to whether and when
he informed the informant as claimed. He is supposed to have taken Gurpal Singh to the
hospital. His non-examination has been rightly taken note of to be a vulnerable factor by
the High Court. There are certain other aspects which need to be noted. PW-6 was taken
to the hospital. The doctor attending to him sent information to the SHO of the concerned
Police Station at 7.15 a.m. According to the evidence of the doctor, he was taken to the
hospital at 7.15 a.m. and immediately thereafter the information was sent to the Police
Station. Interestingly, the respondent was taken to the hospital at 8.30 a.m. It has not been
explained by the prosecution as to why there was delay in sending the respondent-
accused to the hospital. Here again, doctor attending to him sent information to the Police
Station. It was accepted before the High Court that before lodging of the FIR by the
informant, the intimations given by the police had reached the police station. The effect
of these informations was not considered by the trial Court. Whether they constituted FIR
or not is another question. The third factor which has weighed with the High Court to
direct acquittal was the non-explanation of the injuries on the accused. Though, non-
explanation of the minor injuries could not be a factor to make the prosecution version
vulnerable in all cases, but if the injuries were of serious nature the prosecution has to
explain that aspect.
6. In the instant case, the evidence of the doctor who examined the accused clearly stated
that the injuries were on account of dragging him on surface littered with broken glasses.
In that background the injuries on the accused had to be explained.
7. Considering the scope of Section 417, Cr.P.C. (old) (corresponding to Section 378 of
present Cr.P.C.), the Privy Council in Sheo Swarup and others v.. King Emperor, AIR
1934 PC 227, held that the Court gives full power to the High Court to review at large the
evidence upon which order of acquittal is founded and to reach the conclusion whether an
order of acquittal needs to be reversed or not upon that evidence, hence no limitation
should be placed on that power unless found expressly stated in the Code. However, the
Privy Council put certain principles as a matter of caution to be observed by the appellate
court stating that the High Court should and will always give proper weight and
consideration to such matters before reaching its conclusion upon facts, namely- (1) the
view of the trial court as to the credibility of the witnesses; (2) the presumption of
innocence in favour of the accused and that presumption is not weakened by the fact that
he has been acquitted at his trial; (3) the right of the accused to the benefit of any doubt;
and (4) the slowness of an appellate court in disturbing a finding of fact arrived at by a
Judge who had the advantage of seeing the witnesses. To summarize the Privy Council
observed :
"......To state this however is only to say that the High Court in its conduct of the appeal
should and will act in accordance with rules and principles well known and recognized in
the administration of justice."
8

. The aforesaid view was reiterated by the Privy Council in Nur Mohammad v. Emperor,
AIR 1945 PC 151, and affirmed by this Court also in Prandas v. State, AIR 1954 SC 36.
The judgment of this Court was rendered by a Bench of six-Hon'ble Judges of this Court.
A three-Judge Bench of this Court in Aher Raja Khima v. State of Saurashtra (1955 (2)
SCR AIR 1956 SC 217
AIR 1961 SC 715, Para 8

1285) observed that it is not enough for the High Court to take a different view of the
evidence; there must also be "substantial and compelling reasons" for holding that the
trial Court was wrong. The words "substantial and compelling reasons" observed, in
Khima's case (supra) sought to be interpreted subsequently as if only in exceptional cases
High Court can interfere. The matter came up before this Court in Sanwat
@page-SC3283
Singh v. State of Rajasthan (1961 (3,) SCR 120), where a three-Judge Bench of this Court
noted that the words "substantial and compelling reasons" used in certain decisions have
created some difficulty in understanding the scope of the said words. Explaining the same
this Court held as under :
"This Court obviously did not and could not add a condition to S. 417 of the Criminal
Procedure Code. The words were intended to convey the idea that an appellate court not
only shall bear in mind the principles laid down by the Privy Council but also must give
its clear reasons for coming to the conclusion that the order of acquittal was wrong."
9. It concluded as under;

"The foregoing discussion yields the following results (1) an appellate court has full
power to review the evidence upon which the order of acquittal is founded; (2) the
principles laid down in Sheo Swarup's case afford a correct guide for the appellate court's
approach to a case in disposing of such an appeal; and (3) the different phraseology used
in the judgments of this Court, such as : (i) "substantial and compelling reasons", (ii)
"good and sufficiently cogent reasons", and (iii) "strong reasons" are not intended to
curtail the undoubted power of an appellate court in an appeal against acquittal to review
the entire evidence and to come to its own conclusion; but in doing so it should not only
consider every matter on record having a bearing on the questions of fact and the reasons
given by the court below in support of its order of acquittal in its arriving at a conclusion
on those facts; but should also express those reasons in its judgment which lead it to hold
that the acquittal was not justified." AIR 1934 PC 227 (2)

10

. A Constitution Bench of this Court again reviewed all the aforesaid judgments in M.G.
Agarwal vs. State of Maharashtra (1963 (2) SCR 405) and reiterating the principles laid
down in Sheo Swarup (supra), it affirmed the view taken by the this Court in Sanwat
Singh (supra) and held "it is not necessary that before reversing a judgment of acquittal,
the High Court must necessarily characterised the findings recorded therein as perverse."
AIR 1963 SC 200
AIR 1934 PC 227 (2)
AIR 1961 SC 715

11

. In Shivaji Sahabrao Bobade v. State of Maharashtra (1973) 2 SCC 793, it was held that
"in law there are no fetters on the plenary power of the appellate Court to review the
whole evidence on which the order of acquittal is founded and, indeed, it has a duty to
scrutinize the probative material de novo, informed, however, by the weighty thought that
the rebuttable innocence attributed to the accused having been converted into an acquittal
the homage our jurisprudence owes to individual liberty constrains the higher court not to
upset the holding without very connincing reasons and comprehensive considerations."
AIR 1973 SC 2622

12

. In K. Gopal Reddy v. State of Andhra Pradesh, 1979 (2) SCR 363, reiterating the
principles as mentioned in Sheo Swarup (supra), this Court observed as under: AIR
1979 SC 387, Para 9
AIR 1934 PC 227 (2)

"............... Occasionally phrases like 'manifestly illegal', 'grossly unjust', have been used
to describe the orders of acquittal which warrant interference. But, such expressions have
been used more, as flourishes of language, to emphasise the reluctance of the Appellate
Court to interfere with an order of acquittal than to curtail the power of the Appellate
Court to review the entire evidence and to come to its own conclusion .....If two
reasonably probable and evenly balanced views of the evidence are possible, one must
necessarily concede the existence of a reasonable doubt. But, fanciful and remote
possibilities must be left out of account. To entitle an accused person to the benefit of a
doubt arising from the possibility of a duality of views, the possible view in favour of the
accused must be as nearly reasonably probable as that against him...."
13

. There has not been any change and in many subsequent decisions, i.e., Ramesh Babu
Lal Doshi v. State of Gujarat, JT 1996 (6) SC 79; George v. State of Kerala, AIR 1998 SC
1376; Jaswant Singh v. State of Haryana, (2000) 4 SCC 484; Bhagwan Singh and others
v. State, of M.P. JT 2002 (3) SC 387 and Kallu v. State of 1996 AIR SCW 2438
1998 AIR SCW 1255
2000 AIR SCW 1430
2002 AIR SCW 1532
2006 AIR SCW 177
2007 AIR SCW 1850

@page-SC3284
M.P., JT 2006 (12) SC 586, the aforesaid views have been reiterated. This Court in
Chandrappa and Ors. v. State of Karnataka (2007 (4) SCC 415), having a complete
retrospect on all the earlier judgments, has culled down, in para 41, the following
principles regarding the power of the appellate court while dealing with an appeal against
an order of acquittal:
(1) An appellate Court has full power to review, appreciate and reconsider the evidence
upon which the order of acquittal is founded.
(2) The Code of Criminal Procedure, 1973 puts no limitation, restriction or condition on
exercise of such power and an appellate Court on the evidence before it may reach its
own conclusion, both on questions of fact and of law;
(3) Various expressions, such as, 'substantial and compelling reasons', 'good and sufficient
grounds', 'very strong circumstances', 'distorted conclusions', 'glaring mistakes', etc. are
not intended to curtail extensive powers of an appellate Court in an appeal against
acquittal. Such phraseologies are more in the nature of 'flourishes of language' to
emphasize the reluctant of an appellate Court to interfere with acquittal than to curtail the
power of the Court to review the evidence and to come to its own conclusion.
(4) An appellate court, however, must bear in mind that in case of acquittal, there is
double presumption in favour of the accused. Firstly, the presumption of innocence
available to him under the fundamental principle of criminal jurisprudence that every
person should be presumed to be innocent unless he is proved guilty by a competent court
of law. Secondly, the accused having secured his acquittal, the presumption of his
innocence is further reinforced, reaffirmed and strengthened by the trial court.
(5) If two reasonable conclusions are possible on the basis of evidence on record, the
appellate court should not disturb the finding of acquittal recorded by the trial court."
14. In our considered view the acquittal as directed by the High Court cannot be faulted.
Even though it may be possible hypothetically to take a different view on the evidence,
we are not inclined to, interfere with the view of the High Court. The appeal is, therefore,
dismissed.
Appeal dismissed.
AIR 2008 SUPREME COURT 3284 "Satya Narain Yadav v. Gajanand"
(From : 2000 WLC (UC) 682 (Rajasthan))
Coram : 2 Dr. A. PASAYAT AND Dr. MUKUNDAKAM SHARMA, JJ.
Criminal Appeal No. 305 with 999 of 2001, D/- 1 -8 -2008.
Satya Narain Yadav v. Gajanand and Anr.
(A) Penal Code (45 of 1860), S.96 - PRIVATE DEFENCE - Right of private defence -
Availability - Abstract test for determination - Cannot be laid down - It being question of
fact Court has to consider all surrounding circumstances for determining its availability.
(Paras 9, 10)
(B) Penal Code (45 of 1860), S.96 - PRIVATE DEFENCE - Right of private defence -
Accused need not raise specific plea - Court can consider its availability even in absence
of plea by accused. (Para 9)
(C) Penal Code (45 of 1860), S.96 - PRIVATE DEFENCE - Right of private defence -
Burden of proof - Lies on accused - However, burden of proof is not as onerous as on
prosecution - Preponderance of probability in favour of plea is sufficient. (Paras 9, 10)
(D) Penal Code (45 of 1860), S.96 - PRIVATE DEFENCE - Right of private defence of
body - Availability - Relevant factors for determination.
In order to find whether right of private defence is available or not, the injuries received
by the accused, the imminence of threat to his safety, the injuries caused by the accused
and the circumstances whether the accused had time to have recourse to public authorities
are all relevant factors to be considered. (Para 13)
(E) Penal Code (45 of 1860), S.99 - PRIVATE DEFENCE - Right of private defence of
body - Question of exceeding or overstepping of right - Court has to consider situation
which accused has faced pragmatically - Technical approach has to be avoided. (Paras
14, 15)
(F) Penal Code (45 of 1860), S.96 - PRIVATE DEFENCE - Right of private defence - Is
a defensive
@page-SC3285
right - Not to used as weapon of retribution or aggression. (Para 17)
(G) Criminal P.C. (2 of 1974), S.378 - APPEAL - EVIDENCE - Appeal against acquittal -
Powers of Court - Not restricted in any way - Court can reappreciate evidence - Both
questions of fact and of law are open to determination by High Court - Court, however,
must be bear in mind that accused has double presumption of innocence in his favour -
Court not to interfere if two views are possible. (Paras 22, 48)
(H) Criminal P.C. (2 of 1974), S.378 - APPEAL - MURDER - APPELLATE COURT -
Appeal against acquittal - Interference - Murder case - Accused convicted on allegation
that he brutally hit deceased by axe - Incident alleged to have happened when accused
grazed his goats in field of deceased - Accused alleging that he and his associate was
assaulted with lathi by deceased while they were driving away goats - Plea of right of
private defence raised - Interference by Appellate Court by accepting plea - Held, in
circumstances, not liable to be interfered with. (Para 53)
Cases Referred : Chronological Paras
2008 AIR SCW 4863 (Rel. on) 18
2007 AIR SCW 1850 : 2007 Cri LJ 2136 : 2007 (3) AIR Kar R 235 : AIR 2007 (Supp)
SC 111 (Rel. on) 48
2006 AIR SCW 177 : AIR 2006 SC 831 : 2006 Cri LJ 799 (Rel. on) 47
2005 AIR SCW 1311 : AIR 2005 SC 1983 : 2005 Cri LJ 1727 (Rel. on) 18
2003 AIR SCW 3688 : AIR 2003 SC 2978 : 2003 Cri LJ 3705 (Rel. on) 52
2003 AIR SCW 6731 : AIR 2004 SC 1053 : 2004 Cri LJ 640 (Rel. on) 46
2002 AIR SCW 781 : AIR 2002 SC 1051 : 2002 Cri LJ 1489 (Rel. on) 43
2002 AIR SCW 1532 : AIR 2002 SC 1621 : 2002 Cri LJ 2024 (Rel. on) 44
2002 AIR SCW 3199 : AIR 2002 SC 2821 : 2002 Cri LJ 3751 (Rel. on) 45
2002 AIR SCW 4315 : AIR 2002 SC 3667 : 2003 Cri LJ 53 (Rel. on, Pnt. D) 13
1996 AIR SCW 2438 : AIR 1996 SC 2035 : 1996 Cri LJ 2867 (Rel. on) 42
1995 AIR SCW 4625 : 1996 Cri LJ 878 (Rel. on, Pnt. D) 13
1991 AIR SCW 1022 : AIR 1991 SC 1316 : 1991 Cri LJ 1464 (Rel. on, Pnt. E) 14
AIR 1988 SC 2154 : 1989 Cri LJ 288 (Rel. on) 51
AIR 1979 SC 387 : 1980 Cri LJ 812 : 1979 Cri LJ 137 (Rel. on) 41
AIR 1979 SC 391 : 1979 Cri LJ 323 (Rel. on, Pnt. C) 9
AIR 1979 SC 577 : 1979 Cri LJ 584 (Rel. on, Pnt. C) 9
AIR 1977 SC 2226 : 1977 Cri LJ 1897 (Rel. on, Pnt. C) 9
AIR 1975 SC 87 : 1975 Cri LJ 44 (Rel. on, Pnt. D) 13
AIR 1975 SC 1478 : 1975 Cri LJ 1079 (Rel. on, Pnt. C) 9
AIR 1974 SC 286 : 1974 Cri LJ 337 (Rel. on) 41
AIR 1973 SC 2622 : 1973 Cri LJ 1783 (Rel. on) 39
AIR 1971 SC 460 : 1971 Cri LJ 422 (Rel. on) 41
AIR 1971 SC 1857 : 1971 Cri LJ 1296 (Ref) 15
AIR 1968 SC 702 : 1968 Cri LJ 806 (Rel. on, Pnt. C) 9
AIR 1963 SC 200 : 1963 (1) Cri LJ 235 (Rel. on) 38
AIR 1963 SC 612 : 1963 (1) Cri LJ 495 (Ref.) 12
AIR 1962 SC 439 : 1962 (1) Cri LJ 479 (Rel. on) 38
AIR 1961 SC 715 : 1961 (1) Cri LJ 766 (Rel. on) 37, 41
AIR 1956 SC 217 : 1956 Cri LJ 426 (Rel. on) 36
AIR 1955 SC 807 : 1955 Cri LJ 1653 (Rel. on) 35
AIR 1954 SC 36 : 1954 Cri LJ 331 (Rel. on) 30
AIR 1953 SC 76 : 1953 Cri LJ 521 (Rel. on) 33, 38
AIR 1952 SC 52 : 1952 Cri LJ 331 (Rel. on) 32, 38
AIR 1945 PC 151 : (1946) 47 Cri LJ 1 29
AIR 1934 PC 227 (2) : (1935) 36 Cri LJ 786 25, 29, 31, 37, 41
Anurag Sharma, Joseph Pookatt, Prashant Kumar, Aruneshwar Gupta, Kumar Kartikay,
Ranvijay and Ms. Pratibha Jain, for the appearing parties.
Judgement
Dr. ARIJIT PASAYAT, J. :- These two appeals are directed against the judgment of a
Division Bench of the Rajasthan High Court directing acquittal of respondent Gajanand.
One appeal has been filed by the informant while the other has been filed by the State.
The trial Court i.e. learned Additional Sessions Judge, No.2, Bundi, Rajasthan, had
convicted Gajanand for offence punishable under Section 302 of the Indian Penal Code,
1860 (in short the 'IPC') and sentenced to undergo rigorous imprisonment for life and to
pay a fine of Rs. 2,000/-
@page-SC3286
with default stipulation. Co-accused Mahavir was, however, acquitted.
2. Background facts in a nutshell are as follows :
Kana (PW-13) lodged a First Information Report (Ext.P-2) that while he and his brother
Devi Lal (hereinafter referred to as 'deceased') were working in the field, respondent No.
1-Gajanand and Mahavir objected to it and caused injury to him. It was stated that
Gajanand was having an axe in his hand and he caused injuries to Devi Lal on his head.
Earlier, a case under Section 307 IPC was registered but on the death of Devi Lal charge
under Section 302 IPC was added. After completion of investigation, challans were
submitted against Gajanand and Mahavir.
3. As appears from the first information report the informant had stated that while they
were harvesting the wheat at the agricultural land in the morning two cow boys were
grazing their goats. When loss was caused to the crops, Devi Lal asked them not to do so.
They insisted that goats would be grazed and their names were Gajanand and Mahavir.
When Devi Lal further asked them not to do so, Gajanand gave a blow of axe on the left
side of his head and the deceased fell down. Thereafter, Mahavir assaulted with fists and
the handle of the axe. Thereafter both of them fled away.
4. Charge sheet under Section 302 as well as under Section 302 read with Section 149
was filed separately against accused persons. The accused persons pleaded innocence.
Sixteen witnesses were examined to further the prosecution version. Kana (PW-13) was
stated to be an eye-witness. The Trial Court found Gajanand guilty of offence punishable
under Section 302 IPC. Mahavir, as noted above, was acquitted. Present respondent
preferred an appeal before the High Court which by the impugned judgment set aside the
conviction.
5. The accused had taken the stand during trial that when Mahavir was trying to drive out
the goat which entered into the field of Devi Lal, he was given beatings by the deceased
and two lathi blows were inflicted on him. When Gajanand came to his rescue, he was
also addressed in contemptuous manner and was given two lathi blows.
6. It was stated that by exercising the right of private defence, two blows were given by
Gajanand and, therefore, the accused person cannot be convicted. The High Court has
accepted this stand of the accused and held that the right of private defence is available.
7. In suppprt of the appeal filed by the State and the informant it is submitted that the trial
Court has erroneously held that the accused were exercising the right of private defence.
8. Learned counsel for the accused-Gajanand submitted that when injury was caused by
lathi not only on Mahavir but also to Gajanand-respondent, it cannot be said that they
were not exercising the right of private defence. In any event, according to him, if
parameters of appeal against acquittal are kept in view, no interference is called for.
9

. Only question which needs to be considered is the alleged exercise of right of private
defence. Section 96, IPC provides that nothing is an offence which is done in the exercise
of the right of private defence. The Section does not define the expression 'right of private
defence'. It merely indicates that nothing is an offence which is done in the exercise of
such right. Whether in a particular set of circumstances, a person legitimately acted in the
exercise of the right of private defence is a question of fact to be determined on the facts
and circumstances of each case. No test in the abstract for determining such a question
can be laid down. In determining this question of fact, the Court must consider all the
surrounding circumstances. It is not necessary for the accused to plead in so many words
that he acted in self-defence. If the circumstances show that the right of private defence
was legitimately exercised, it is open to the Court to consider such a plea. In a given case
the Court can consider it even if the accused has not taken it, if the same is available to be
considered from the material on record. Under Section 105 of the Indian Evidence Act,
1872 (in short 'the Evidence Act'), the burden of proof is on the accused; who sets up the
plea of self-defence, and, in the absence of proof, it is riot possible for the Court to
presume the truth of the plea of self-defence. The Court shall presume the absence of
such circumstances. It is for the accused to place necessary material on record either by
himself adducing positive evidence or by eliciting necessary facts from the witnesses
examined for the prosecution. An accused taking the plea of the right of private defence is
not necessarily required to call evidence; he can establish his plea by reference Para 9
of AIR

@page-SC3287
to circumstances transpiring from the prosecution evidence itself. The question in such a
case would be a question of assessing the true effect of the prosecution evidence, and not
a question of the accused discharging any burden. Where the right of private defence is
pleaded, the defence must be a reasonable and probable version satisfying the Court that
the harm caused by the accused was necessary for either warding off the attack or for
forestalling the further reasonable apprehension from the side of the accused. The burden
of establishing the plea of self-defence is on the accused and the burden stands
discharged by showing preponderance of probabilities in favour of that plea on the basis
of the material on record. (See Munshi Ram and Ors. v. Delhi Administration (AIR 1968
SC 702), State of Gujarat v. Bai Fatima (AIR 1975 SC 1478), State of U.P. v. Mohd.
Musheer Khan (AIR 1977 SC 2226), and Mohinder Pal Jolly v. State of Punjab (AIR
1979 SC 577). Sections 100 to 101 define the extent of the right of private defence of
body. If a person has a right of private defence of body under Section 97, that right
extends under Section 100 to causing death if there is reasonable apprehension that death
or grievous hurt would be the consequence of the assault. The oft quoted observation of
this Court in Salim Zia v. State of U. P. (AIR 1979 SC 391), runs as follows:
"It is true that the burden on an accused person to establish the plea of self-defence is not
as onerous as the one which lies on the prosecution and that, while the prosecution is
required to prove its case beyond reasonable doubt, the accused need not establish the
plea to the hilt and may discharge his onus by establishing a mere preponderance of
probabilities either by laying basis for that plea in the cross-examination of the
prosecution witnesses or by adducing defence evidence."
10. The accused need not prove the existence of the right of private defence beyond
reasonable doubt. It is enough for him to show as in a civil case that the preponderance of
probabilities is in favour of his plea.
11. A plea of right of private defence cannot be based on surmises and speculation. While
considering whether the right of private defence is available to an accused, it is not
relevant whether he may have a chance to inflict severe and mortal injury on the
aggressor. In order to find whether the right of private defence is available to an accused,
the entire incident must be examined with care and viewed in its proper setting. Section
97 deals with the subject matter of right of private defence. The plea of right comprises
the body or property (i) of the person exercising the right; or (ii) of any other person; and
the right may be exercised in the case of any offence against the body, and in the case of
offences of theft, robbery, mischief or criminal trespass, and attempts at such offences in
relation to property. Section 99 lays down the limits of the right of private defence.
Sections 96 and 98 give a right of private defence against certain offences and acts. The
right given under Sections 96 to 98 and 100 to 106 is controlled by Section 99. To claim a
right of private defence extending to voluntary causing of death, the accused must show
that there were circumstances giving rise to reasonable grounds for apprehending that
either death or grievous hurt would be caused to him. The burden is on the accused to
show that he had a right of private defence which extended to causing of death. Sections
100 and 101, IPC define the limit and extent of right of private defence.
12. Sections 102 and 105, IPC deal with commencement and continuance of the right of
private defence of body and property respectively. The right commences, as soon as a
reasonable apprehension of danger to the body arises from an attempt, or threat, to
commit the offence, although the offence may not have been committed but not until
there is that reasonable apprehension. The right lasts so long as the reasonable
apprehension of the danger to the body continues. In Jai Dev. v. State of Punjab (AIR
1963 SC 612), it was observed that as soon as the cause for reasonable apprehension
disappears and the threat has either been destroyed or has been put to route, there can be
no occasion to exercise the right of private defence.
13

. In order to find whether right of private defence is available or not, the injuries received
by the accused, the imminence of threat to his safety, the injuries caused by the accused
and the circumstances whether the accused had time to have recourse to public authorities
are all relevant factors to be considered. Similar view was expressed by this Court in
Biran Singh v. State of Bihar (AIR 1975 SC 87). (See : Wassan Singh v. State of Punjab
(1996) 1 1995 AIR SCW 4625
2002 AIR SCW 4315

@page-SC3288
SCC 458, Sekar alias Raja Sekharan v. State represented by Inspector of Police, T.N.,
2002 (8) SCC 354).
14

. As noted in Butta Singh v. The State of Punjab (AIR 1991 SC 1316), a person who is
apprehending death or bodily injury cannot weigh in golden scales in the spur of moment
and in the heat of circumstances, the number of injuries required to disarm the assailants
who were armed with weapons. In moments of excitement and disturbed mental
equilibrium it is often difficult to expect the parties to preserve composure and use
exactly only so much force in retaliation commensurate with the danger apprehended to
him where assault is imminent by use of force, it would be lawful to repel the force in
self-defence and the right of private-defence commences, as soon as the threat becomes
so imminent. Such situations have to be pragmatically viewed and not with high-powered
spectacles or microscopes to detect slight or even marginal overstepping. Due weightage
has to be given to, and hyper technical approach has to be avoided in considering what
happens on the spur of the moment on the spot and keeping in view normal human
reaction and conduct, where self-preservation is the paramount consideration. But, if the
fact situation shows that in the guise of self-preservation, what really has been done is to
assault the original aggressor, even after the cause of reasonable apprehension has
disappeared, the plea of right of private-defence can legitimately be negatived. The Court
dealing with the plea has to weigh the material to conclude whether the plea is
acceptable. It is essentially, as noted above, a finding of fact. 1991 AIR SCW 1022

15. The right of self-defence is a very valuable right, serving a social purpose and should
not be construed narrowly. (See Vidhya Singh v. State of M.P. (AIR 1971 SC 1857)).
Situations have to be judged from the subjective point of view of the accused concerned
in the surrounding excitement and confusion of the moment, confronted with a situation
of peril and not by any microscopic and pedantic scrutiny. In adjudging the question as to
whether more force than was necessary was used in the prevailing circumstances on the
spot it would be inappropriate, as held by this Court, to adopt tests by detached
objectivity which would be so natural in a Court room, or that which would seem
absolutely necessary to a perfectly cool bystander. The person facing a reasonable
apprehension of threat to himself cannot be expected to modulate his defence step by step
with any arithmetical exactitude of only that much which is required in the thinking of a
man in ordinary times or under normal circumstances.
16. In the illuminating words of Russel (Russel on Crime, 11th Edition Volume I at page
49):
"....a man is justified in resisting by force anyone who manifestly intends and endeavours
by violence or surprise to commit a known felony against either his person, habitation or
property. In these cases, he is not obliged to retreat, and may not merely resist the attack
where he stands but may indeed pursue his adversary until the danger is ended and if in a
conflict between them he happens to kill his attacker, such killing is justifiable."
17. The right of private defence is essentially a defensive right circumscribed by the
governing statute i.e. the IPC, available only when the circumstances clearly justify it. It
should not be allowed to be pleaded or availed as a pretext for a vindictive, aggressive or
retributive purpose of offence. It is a right of defence, not of retribution, expected to repel
unlawful aggression and hot as retaliatory measure. While providing for exercise of the
right, care has been taken in IPC not to provide and has not devised a mechanism
whereby an attack may be a pretence for killing. A right to defend does not Include a right
to launch an offensive, particularly when the need to defend no longer survived.
18

. The above position was highlighted in V. Subramani and Anr. vs. State of Tamil Nadu
(2005 (10) SCC 358) and Genda Singh and Ors. v. State of U.P. (Criminal Appeal arising
out of SLP (Crl.) No. 1029/07 disposed of on July 9, 2008). 2005 AIR SCW 1311
Reported in 2008 AIR SCW 4863

19. It also would be necessary and proper to consider and clarify the legal position first.
Chapter XXIX (Sections 372-394) of the Code of Criminal Procedure, 1973 (hereinafter
referred to as "the present Code") deals with appeals. Section 372 expressly declares that
no appeal shall lie from any judgment or order of a criminal court except as provided by
the Code or by any other law for the time being in force. Section 373 provides for filing
of appeals in certain cases. Section 374 allows appeals from convictions. Section 375
bars appeals in cases where the accused
@page-SC3289
pleads guilty. Likewise, no appeal is maintainable in petty cases (Section 376). Section
377 permits appeals by the State for enhancement of sentence. Section 378 confers power
on the State to present an appeal to the High Court from an order of acquittal. The said
section is material and may be quoted in extenso:
"378. Appeal in case of acquittal.-(1) Save as otherwise provided in sub-section (2) and
subject to the provisions of sub-sections (3) and (5), the State Government may, in any
case, direct the Public Prosecutor to present an appeal to the High Court from an original
or appellate order of acquittal passed by any court other than a High Court, or an order of
acquittal passed by the Court of Session in revision.
(2) If such an order of acquittal is passed in any case in which the offence has been
investigated by the Delhi Special Police Establishment constituted under the Delhi
Special Police Establishment Act, 1946 (25 of 1946), or by any other agency empowered
to make investigation into an offence under any Central Act other than this Code, the
Central Government may also direct the Public Prosecutor to present an appeal, subject to
the provisions of sub-section (3), to the High Court from the order of acquittal.
(3) No appeal under sub-section (1) or sub-section (2) shall be entertained except with the
leave of the High Court.
(4) If such an order of acquittal is passed in any case instituted upon complaint and the
High Court, on an application made to it by the complainant in this behalf, grants special
leave to appeal from the order of acquittal, the complainant may present such an appeal to
the High Court.
(5) No application under sub-section (4) for the grant of special leave to appeal from an
order of acquittal shall be entertained by the High Court after the expiry of six months,
where the complainant is a public servant, and sixty days in every other case, computed
from the date of that order of acquittal.
(6) If, in any case, the application under sub-section (4) for the grant of special leave to
appeal from an order of acquittal is refused, no appeal from that order of acquittal shall
lie under sub-section (1) or under sub-section (2).
20. Whereas Sections 379-380 cover special cases of appeals, other sections lay down
procedure to be followed by appellate courts.
21. It may be stated that more or less similar provisions were found in the Code of
Criminal Procedure, 1898 (hereinafter referred to as "the old Code") which came up for
consideration before various High Courts, Judicial Committee of the Privy Council as
also before this Court. Since in the present appeal, we have been called upon to decide
the ambit and scope of the power of an appellate court in an appeal against an order of
acquittal, we have confined ourselves to one aspect only i.e. an appeal against an order of
acquittal.
22. Bare reading of Section 378 of the present Code (appeal in case of acquittal) quoted
above, makes it clear that no restrictions have been imposed by the legislature on the
powers of the appellate court in dealing with appeals against acquittal. When such an
appeal is filed, the High Court has full power to reappreciate, review and reconsider the
evidence at large, the material on which the order of acquittal is founded and to reach its
own conclusions on such evidence. Both questions of fact and of law are open to
determination by the High Court in an appeal against an order of acquittal.
23. It cannot, however, be forgotten that in case of acquittal, there is a double
presumption in favour of the accused. Firstly, the presumption of innocence is available
to him under the fundamental principle of criminal jurisprudence that every person
should be presumed to be innocent unless he is proved to be guilty by a competent court
of law. Secondly, the accused having secured an acquittal, the presumption of his inno
cence is certainly not weakened but reinforced, reaffirrned and strengthened by the trial
court.
24. Though the above principles are well established, a different note was struck in
several decisions by various High Courts and even by this Court. It is, therefore,
appropriate if we consider some of the leading decisions on the point.
25

. The first important decision was rendered by the Judicial Committee of the Privy
Council in Sheo Swarup v. R Emperor ((1934) 61 IA 398). In Sheo Swarup the accused
were acquitted by the trial court and the local Government directed the Public Prosecutor
to present an appeal to the High Court from an order of acquittal under Section 417 of the
old Code (similar to Section 378 of the present Code). At the time of hearing of appeal
before the High Court, it was contended AIR 1934 PC 227 (2)

@page-SC3290
on behalf of the accused that in an appeal from an order of acquittal, it was not open to
the appellate court to interfere with the findings of fact recorded by the trial Judge unless
such findings could not have been reached by him had there not been some perversity or
incompetence on his part. The High Court, however, declined to accept the said view. It
held that no condition was imposed on the High Court in such appeal. It accordingly
reviewed all the evidence in the case and having formed an opinion of its weight and
reliability different from that of the trial Judge, recorded an order of conviction. A
petition was presented to His Majesty in Council for leave to appeal on the ground that
conflicting views had been expressed by the High Courts in different parts of India upon
the question whether in an appeal from an order of acquittal, an appellate court had the
power to interfere with the findings of fact recorded by the trial Judge. Their Lordships
thought it fit to clarify the legal position and accordingly upon the "humble advice of
their Lordships", leave was granted by His Majesty. The case was, thereafter, argued. The
Committee considered the scheme and interpreting Section 417 of the Code (old Code)
observed that there was no indication in the Code of any limitation or restriction on the
High Court in exercise of powers as an Appellate Tribunal. The Code also made no
distinction as regards powers of the High Court in dealing with an appeal against
acquittal and an appeal against conviction. Though several authorities were cited
revealing different views by the High Courts dealing with an appeal from an order of
acquittal, the Committee did not think it proper to discuss all the cases.
26. Lord Russel summed up the legal position thus:
"There is, in their opinion, no foundation for the view, apparently supported by the
judgments of some courts in India, that the High Court has no power or jurisdiction to
reverse an order of acquittal on a matter of fact, except in cases in which the lower court
has 'obstinately blundered', or has 'through incompetence, stupidity or perversity' reached
such 'distorted conclusions as to produce a positive miscarriage of justice', or has in some
other way so conducted or misconducted itself as to produce a glaring miscarriage of
justice, or has been tricked by the defence so as to produce a similar result."
27. His Lordship, then proceeded to observe: (IA p.404)
"Sections 417, 418 and 423 pf the Code give to the High Court full power to review at
large the evidence upon which the order of acquittal was founded, and to reach the
conclusion that upon that evidence the order of acquittal should be reversed. No
limitation should be placed upon that power, unless it be found expressly stated in the
Code."
28. The Committee, however, cautioned appellate courts and stated: (IA p.404)
"But in exercising the power conferred by the Code and before reaching its conclusions
upon fact, the High Court should and will always give proper weight and consideration to
such matters as (1) the views of the trial Judge as to the credibility of the witnesses; (2)
the presumption of innocence in favour of the accused, a presumption certainly not
weakened by the fact that he has been acquitted at his trial; (3) the right of the accused to
the benefit of any doubt; and (4) the slowness of an appellate court in disturbing a finding
of fact arrived at by a judge who had, the advantage of seeing the witnesses. To state this,
however, is only to say that the High Court in its conduct of the appeal should and will
act in accordance with rules and principles well known and recognised ni the
administration of justice."
(Emphasis supplied)
29
. In Nur Mohd. v. Emperor (AIR 1945 PC 151), the Committee reiterated the above view
in Sheo Swarup (Supra) and held that in an appeal against acquittal, the High Court has
full powers to review and to reverse acquittal. AIR 1934 PC 227 (2)

30. So far as this Court is concerned, probably the first decision on the point was Prandas
v. State (AIR 1954 SC 36) (though the case was decided on 14-3-1950, it was reported
only in 1954). In that case, the accused was acquitted by the trial court. The Provincial
Government preferred an appeal which was allowed and the accused was convicted for
offences punishable under Sections 302 and 323 IPC. The High Court, for convicting the
accused, placed reliance on certain eye-witnesses.
31

. Upholding the decision of the High Court and following the proposition of law in Sheo
Swarup (supra), a six-Judge Bench held as follows: AIR 1934 PC 227 (2)

"6. It must be observed at the very outset


@page-SC3291
that we cannot support the view which has been expressed in several cases that the High
Court has no power under Section 417. Criminal Procedure Code, to reverse a judgment
of acquittal, unless the judgment is perverse or the subordinate court has in some way or
other misdirected itself so as to produce a miscarriage of justice.'
(Emphasis supplied)
32

. In Surajpal Singh v. State (1952 SCR 193), a two-Judge Bench observed that it was well
established that in an appeal Under Section 417 of the (old) Code, the High Court had full
power to review the evidence upon which the order of acquittal was founded. But it was
equally well settled that the presumption of innocence of the accused was further
reinforced by his acquittal by the trial court, and the findings of the trial court which had
the advantage of seeing the witnesses and hearing their evidence could be reversed old
for very substantial and compelling reasons. AIR 1952 SC 52

33

. In Ajmer Singh v. State of Punjab (1953 SCR 418) the accused was acquitted by the trial
court but was convicted by the High Court in an appeal against acquittal filed by the
State. The aggrieved accused approached this Court. It was contended by him that there
were "no compelling reasons" for setting aside the order of acquittal and due and proper
weight had not been given by the High Court to the opinion of the trial court as regards
the credibility of witnesses seen and examined. It was also commented that the High
Court committed an error of law in observing that "when a strong 'prima facie' case is
made out against an accused person it is his duty to explain the circumstances appearing
in evidence against him and he cannot take shelter behind the presumption of innocence
and cannot state that the law entitles him to keep his lips sealed". AIR 1953 SC 76

34. Upholding the contention, this Court said:


"We think this criticism is well founded. After an order of acquittal has been made the
presumption of innocence is further reinforced by that order, and that being so, the trial
court's decision can be reversed not on the ground that the accused had failed to explain
the circumstances appearing against him but only for very substantial and compelling
reasqns."
(Emphasis supplied)
35

. In Atley v. State of U.P. (AIR 1955 SC 807) this Court said: Para 5 of AIR

"In our opinion, it is not correct to say that unless the appellate court in an appeal Under
Section 417, Criminal Procedure Code came to the conclusion that the judgment of
acquittal under appeal was perverse it could not set aside that order.
It has been laid down by this Court that it is open to the High Court on an appeal against
an order of acquittal to review the entire evidence and to come to its own conclusion, of
course, keeping in view the well-established rule that the presumption of innocence of the
accused is not weakened but strengthened by the judgment of acquittal passed by the trial
court which had the advantage of observing the demeanour of witnesses whose evidence
have been recorded in its presence.
It is also well settled that the court of appeal has as wide powers of appreciation of
evidence in an appeal against an order of acquittal as in the case of an appeal against an
order of conviction, subject to the riders that the presumption of innocence with which
the accused person starts in the trial court continues even up to the appellate stage and
that the appellate court should attach due weight to the opinion of the trial court which
recorded the order of acquittal. If the appellate court reviews the evidence, keeping those
principles in mind, and comes to a contrary conclusion, the judgment cannot be said to
have been vitiated."
(Emphasis supplied)
36

. In Aher Raja Khima v. State of Saurashtra ((1955) 2 SCR 1285) the accused was
prosecuted under Sections 302 and 447, IPC. He was acquitted by the trial court but
convicted by the High Court. Dealing with the power of the High Court against an order
of acquittal, Bose, J. speaking for the majority (2:1) stated: (AIR p. 220, para 1) "It is, in
our opinion, well settled that it is not enough for the High Court to take a different view
of the evidence; there must also be substantial and compelling reasons for holding that
the trial court was wrong." AIR 1956 SC 217

(Emphasis supplied)
37

. In Sanwat Singh v. State of Rajasthan (1961) 3 SCR 120, a three-Judge Bench


considered almost all leading decisions on the point and observed that there was no
difficulty in applying AIR 1961 SC 715

@page-SC3292
the principles laid down by the Privy Council and accepted by the Supreme Court. The
Court, however, noted that appellate courts found considerable difficulty in understanding
the scope of the words "substantial and compelling reasons" used in certain decisions. It
was observed inter-alia as follows :
"This Court obviously did not and could not add a condition to Section 417 of the
Criminal Procedure Code. The words Were intended to convey the idea that an appellate
court not only shall bear in mind the principles laid down by the Privy Council but also
must give its clear reasons for coming to the conclusion that the order of acquittal was
wrong."
The Court concluded as follows :

"9. The foregoing discussion yields the following results: (1) an appellate court has full
power to review the evidence upon which the order of acquittal is founded; (2) the
principles laid down in Sheo Swarup case afford a correct guide for the appellate court's
approach to a case in disposing of such an appeal; and (3) the different phraseology used
in the judgments of this Court, such as, (1) 'substantial and compelling reasons', (ii) 'good
and sufficiently cogent reasons', and (iii) 'strong reasons' are not intended to curtail the
undoubted power of an appellate court in an appeal against acquittal to review the entire
evidence and to come to its own conclusion; but in doing so it should not only consider
every matter on record having a bearing on the questions of fact and the reasons given by
the court below in support of its order of acquittal in its arriving at a conclusion on those
facts, but should also express those reasons in its judgment, which lead it to hold that the
acquittal was not justified." AIR 1934 PC 227 (2)

38

. Again, in M. G. Agarwal v. State of Maharashtra (1963) 2 SCR 405, the point was raised
before a Constitution Bench of this Court. Taking note of earlier decisions, it was
observed as follows: AIR 1963 SC 200

"17. In some of the earlier decisions of this Court, however, in emphasising the
importance of adopting a cautious approach in dealing with appeals against acquittals, it
was observed that the presumption of innocence is reinforced by the order of acquittal
and so, 'the findings of the trial court which had the advantage of seeing the witnesses
and hearing their evidence can be reversed only for very substantial and compelling
reasons': vide Surajpal Singh v. State (1952 SCR 193). Similarly in Ajmer Singh v. State
of Punjab (1953 SCR 418), it was observed that the interference of the High Court in an
appeal against the order of acquittal would be justified only if there are Very substantial
and compelling reasons to do so'. In some other decisions, it has been stated that an order
of acquittal can be reversed only for 'good and sufficiently cogent reasons' or for 'strong
reasons'. In appreciating the effect of these observations, it must be remembered that
these observations were not intended to lay down a rigid or inflexible rule which should
govern the decision of the High Court in appeals against acquittals. They were not
intended, and should not be read to have intended to introduce an additional condition in
clause (a) of Section 423(1) of the Code. All that the said observations are intended to
emphasize is that the approach of the High Court in dealing with an appeal against
acquittal ought to be Cautious because as Lord Russell observed in Sheo Swarup the
presumption of innocence in favour of the accused 'is not certainly weakened by the fact
that he has been acquitted at his trial'. Therefore, the test suggested by the expression
'substantial and compelling reasons' should not be construed as a formula which has to be
rigidly applied in every case. That is the effect of the recent decisions of this Court, for
instance, in Sanwat Singh v. State of Rajasthan and Harbans Singh v. State of Punjab
(1962 Supp 1 SCR 104) and so, It Is not necessary that before reversing a judgment of
acquittal, the High Court must necessarily characterise the findings recorded therein as
perverse." AIR 1952 SC 52
AIR 1953 SC 76
AIR 1934 PC 227 (2)
AIR 1952 SC 52
AIR 1962 SC 439

(Emphasis supplied)
39

. Yet in another leading decision in Shivaji Sahabrao Bobade v. State of Maharashtra


(1973 (2) SCC 793) this Court held that in India, there is no jurisdictional limitation on
the powers of appellate court. "In law there are no fetters on the plenary power of the
appellate court to review the whole evidence on which the order of acquittal is founded
and, indeed, it has a duty to scrutinise the probative material de novo, informed, however,
by the weighty thought that the rebuttable AIR 1973 SC 2622

@page-SC3293
innocence attributed to the accused having been converted into an acquittal the homage
our jurisprudence owes to individual liberty constrains the higher court not to upset the
holding without very convincing reasons and comprehensive consideration."
40. Putting emphasis on balance between importance of individual liberty and evil of
acquitting guilty persons, this Court observed as follows:
"6. Even at this stage we may remind ourselves of a necessary social perspective in
criminal cases which suffers from insufficient forensic appreciation. The dangers of
exaggerated devotion to the rule of benefit of doubt at the expense of social defence and
to the soothing sentiment that all acquittals are always good regardless of justice to the
victim and the community, demand especial emphasis, in the contemporary context of
escalating crime and escape. The judicial instrument has a public accountability. The
cherished principles or golden thread of proof beyond reasonable doubt which runs thro'
the web of our law should not be stretched morbidly to embrace every hunch, hesitancy
and degree of doubt. The excessive solicitude reflected in the attitude that a thousand
guilty men may go but one innocent martyr shall not suffer is a false dilemma. Only
reasonable doubts belong to the accused. Otherwise any practical system of justice will
then breakdown and lose credibility with the community. The evil of acquitting a guilty
person light-heartedly, as a learned author (Glanville Williams in Proof of Guilt) has
saliently observed, goes much beyond the simple fact that just one guilty person has gone
unpunished. If unmerited acquittals become general, they tend to lead to a cynical
disregard of the law, and this in turn leads to a public demand for harsher legal
presumptions against indicted 'persons' and more severe punishment of those who are
found guilty. Thus, too frequent acquittals of the guilty may lead to a ferocious penal law,
eventually eroding the judicial protection of the guiltless. For all these reasons it is true to
say, with Viscount Simon, that 'a miscarriage of justice may arise from the acquittal of the
guilty no less than from the conviction of the innocent....' In short our jurisprudential
enthusiasm for presumed innocence must be moderated by the pragmatic need to make
criminal justice potent and realistic. A balance has to be struck between chasing chance
possibilities as good enough to set the delinquent free and chopping the logic of
preponderant probability to punish marginal innocents."
(Emphasis supplied)
41

. In K. Gopal Reddy v. State of A.P (1979) 1 SCC 355, the Court was considering the
power of the High Court against an order of acquittal under Section 378 of the present
Code. After considering the relevant decisions on the point it was stated as follows:
AIR 1979 SC 387

"9. The principles are now well settled. At one time it was thought that an order of
acquittal could be set aside for 'substantial and compelling reasons' only and courts used
to launch on a search to discover those 'substantial and compelling reasons'. However, the
'formulae' of 'substantial and compelling reasons', 'good and sufficiently cogent reasons'
and 'strong reasons' and the search for them were abandoned as a result of the
pronouncement of this Court in Sanwat Singh v. State of Rajasthan (1961) 3 SCR 120. In
Sanwat Singh case this Court harked back to the principles enunciated by the Privy
Council in Sheo Swarup v. R. Emperor and reaffirmed those principles. After Sanwat
Singh v. State of Rajasthan this Court has consistently recognised the right of the
appellate court to review the entire evidence and to come to its own conclusion bearing in
mind the considerations mentioned by the Privy Council in Sheo Swarup case.
Occasionally phrases like 'manifestly illegal', 'grossly unjust', have been used to describe
the orders of acquittal which warrant interference. But, such expressions have been used
more as flourishes of language, to emphasise the reluctance of the appellate court to
interfere with an order of acquittal than to curtail the power of the appellate court to
review the entire evidence and to come to its own conclusion. In some cases
(Ramaphupala Reddy v. State of A.P., (AIR 1971 SC 460); Bhim Singh Rup Singh v.
State of Maharashtra (AIR 1974 SC 286), it has been said that to the principles laid down
in Sanwat Singh case may be added the further principle that 'if two reasonable
conclusions can be reached on the basis of the evidence on record, the appellate court
should not disturb the finding of the trial court'. This, of course, is not a new principle. It
stems out of the fundamental principle of our criminal jurisprudence that the accused is
entitled to the benefit of any AIR 1961 SC 715
AIR 1934 PC 22(2)

@page-SC3294
reasonable doubt. If two reasonably probable and evenly balanced views of the evidence
are possible, one must necessarily concede the existence of a reasonable doubt. But,
fanciful and remote possibilities must be left out of account. To entitle an accused person
to the benefit of a doubt arising from the possibility of a duality of views, the possible
view in favour of the accused must be as nearly reasonably probable as that against him.
If the preponderance of probability is all one way, a bare possibility of another view will
not entitle the accused to claim the benefit Of any doubt. It is, therefore, essential that any
view of the evidence in favour of the accused must be reasonable even as any doubt, the
benefit of which an accused person may claim, must be reasonable."
(Emphasis supplied)
42

. In Ramesh Babulal Doshi v. State of Gujarat (1996) 9 SCC 225, this Court said: 1996
AIR SCW 2438, Para 7

"While sitting in judgment over an acquittal the appellate court is first required to seek an
answer to the question whether the findings of the trial court are palpably wrong,
manifestly erroneous or demonstrably unsustainable. If the appellate court answers the
above question in the negative the order of acquittal is not to be disturbed. Conversely, if
the appellate court holds, for reasons to be recorded, that the order of acquittal cannot at
all be sustained in view of any of the above infirmities it can then-and then only-
reappraise the evidence to arrive at its own conclusions."
43

. In Allarakha K. Mansuri v. State of Gujarat (2002) 3 SCC 57, referring to earlier


decisions, the Court state : 2002 AIR SCW 781

"7. The paramount consideration of the court should be to avoid miscarriage of justice. A
miscarriage of justice which may arise from the acquittal of guilty is no less than from the
conviction of an innocent. In a case where the trial court has taken a view based upon
conjectures and hypothesis and not on the legal evidence, a duty is cast upon the High
Court to reappreciate the evidence in acquittal appeal for the purposes of ascertaining as
to whether the accused has committed any offence or not. Probable view taken by the
trial court which may not be disturbed in the appeal is such a view which is based upon
legal and admissible evidence. Only because the accused has been acquitted by the trial
court, cannot be made a basis to urge that the High Court under all circumstances should
not disturb such a finding."
44

. In Bhagwan Singh v. State of M.P. (2002) 4 SCC 85, the trial court acquitted the
accused but the High Court convicted them. Negativing the contention of the appellants
that the High Court could not have disturbed the findings of fact of the trial court even if
that view was not correct, this Court observed: "7. We do not agree with the submissions
of the learned counsel for the appellants that under Section 378 of the Code of Criminal
Procedure the High Court could not disturb the finding of facts of the trial court even if it
found that the view taken by the trial court was not proper. On the basis of the
pronouncements of this Court, the settled position of law regarding the powers of the
High Court in an appeal against an order of acquittal is that the Court has full powers to
review the evidence upon which an order of acquittal is based and generally it will not
interfere with the order of acquittal because by passing an order of acquittal the
presumption of innocence in favour of the accused is reinforced. The golden thread which
runs through the web of administration of justice in criminal case is that if two views are
possible on the evidence adduced in the case, one pointing to the guilt of the accused and
the other to his innocence, the view which is favourable to the accused should be
adopted. Such is not a jurisdiction limitation on the appellate court but judge-made
guidelines for circumspection. The paramount consideration of the court is to ensure that
miscarriage of justice is avoided. A miscarriage of justice which may arise from the
acquittal of the guilty, is no less than from the conviction of an innocent. In a case where
the trial court had taken a view ignoring the admissible evidence, a duty is cast upon the
High Court to reappreciate the evidence in acquittal appeal for the purposes of
ascertaining as to whether all or any of the accused has committed any offence or not".
2002 AIR SCW 1532

45

. In Harijana Thirupala v. Public Prosecutor, High Court of A.P. (2002) 6 SCC 470, this
Court said : 2000 AIR SCW 3199

"12. Doubtless the High Court in appeal either against an order of acquittal or conviction
as a court of first appeal has full power to review the evidence to reach its own
independent conclusion. However, it will not
@page-SC3295
interfere with an order of acquittal lightly or merely because one other view is possible,
because with the passing of an order of acquittal presumption of innocence in favour of
the accused gets reinforced and strengthened. The High Court would not be justified to
interfere with order of acquittal merely because it feels that sitting as a trial court it would
have proceeded to record a conviction; a duty is cast on the High Court while reversing
an order of acquittal to examine and discuss the reasons given by the trial court to acquit
the accused and then to dispel those reasons. If the High Court fails to make such an
exercise the judgment will suffer from serious infirmity."
46

. In Ramanand Yadav v. prabhu Nath Jha (2008) 12 SCC 606, this Court observed :
2003 AIR SCW 6731

"21. There is no embargo on the appellate court reviewing the evidence upon which an
order of acquittal is based. Generally, the order of acquittal shall not be interfered with
because the presumption of innocence of the accused is further strengthened by acquittal.
The golden thread which runs through the web of administration of justice in criminal
cases is that if two views are possible on the evidence adduced in the case, one pointing
to the guilt of the accused and the other to his innocence, the view which is favourable to
the accused should be adopted. The paramount consideration of the court is to ensure that
miscarriage of justice is prevented. A miscarriage of justice which may arise from
acquittal of the guilty is no less than from the conviction of an innocent. In a case where
admissible evidence is ignored, a duty is cast upon the appellate court to reappreciate the
evidence in a case where the accused has been acquitted, for the purpose of ascertaining
as to whether any of the accused committed any offence or not".
47

. Again in Kallu v. State of M.P. (2006) 10 SCC 313, this Court stated: 2006 AIR
SCW 177

"8. While deciding an appeal against acquittal, the power of the appellate court is no less
than the power exercised while hearing appeals against conviction. In both types of
appeals, the power exists to review the entire evidence. However, one significant
difference is that an order of acquittal will not be interfered with, by an appellate court,
where the judgment of the trial court is based on evidence and the view taken is
reasonable and plausible. It will not reverse the decision of the trial court merely because
a different view is possible. The appellate court will also bear in mind that there is a
presumption of innocence in favour of the accused and the accused is entitled to get the
benefit of any doubt. Further if it decides to interfere, it should assign reasons for
differing with the decision of the trial court."
(Emphasis supplied)
48

. From the above decisions, in Chandrappa and Ors. v. State of Karnataka (2007 (4) SCC
415), the following general principles regarding powers of the appellate court while
dealing with an appeal against an order of acquittal were culled out: 2007 AIR
SCW 1850

(1) An appellate court has full power to review, reappreciate and reconsider the evidence
upon which the order of acquittal is founded.
(2) The Code of Criminal Procedure, 1973 puts no limitation, restriction or condition on
exercise of such power and an appellate court on the evidence before it may reach its own
conclusion, both on questions of fact and of law.
(3) Various expressions, such as, "substantial and compelling reasons", "good and
sufficient grounds", "very strong circumstances", "distorted conclusions", "glaring
mistakes", etc. are not intended to curtail extensive powers of an appellate court in an
appeal against acquittal. Such phraseologies are more in the nature of "flourishes of
language" to emphasise the reluctance of an appellate court to interfere with acquittal
than to curtail the power of the court to review the evidence and to come to its own
conclusion.
(4) An appellate court, however, must bear in mind that in case of acquittal, there is
double presumption in favour of the accused. Firstly, the presumption of innocence is
available to him under the fundamental principle of criminal jurisprudence that every
person shall be presumed to be innocent unless, he is proved guilty by a competent court
of law. Secondly, the accused having secured his acquittal, the presumption of his
innocence is further reinforced, reaffirmed and strengthened by the trial court.
(5) If two reasonable conclusions are possible on the basis of the evidence on record, the
appellate court should not disturb the finding of acquittal recorded by the trial court.
@page-SC3296
49. A person has, no doubt, a profound right not to be convicted of an offence which is
not established by the evidential standard of proof beyond reasonable doubt. Though this
standard is a higher standard, there is, however, no absolute standard. What degree of
probability amounts to "proof" is an exercise particular to each case. Referring to the
interdependence of evidence and the confirmation of one piece of evidence by another, a
learned author says [see "The Mathematics of Proof II": Glanville Williams, Criminal
Law Review, 1979, by Sweet and Maxwell, p.340 (342)]:
"The simple multiplication rule does not apply if the separate pieces of evidence are
dependent. Two events are dependent when they tend to occur together, and the evidence
of such events may also be said to be dependent. In a criminal case, different pieces of
evidence directed to establishing that the defendant did the prohibited act with the
specified state of mind are generally dependent. A junior may feel doubt whether to credit
an alleged confession, and doubt whether to infer guilt from the fact that the defendant
fled from justice. But since it is generally guilty rather than innocent people who make
confessions, and guilty rather than innocent people who run away, the two doubts are riot
to be multiplied together. The one piece of evidence may confirm the other."
50. Doubts would be called reasonable if they are free from a zest for abstract
speculation. Law cannot afford any favourite other than truth. To constitute reasonable
doubt, it must be free from an overemotional response. Doubts must be actual and
substantial doubts as to the guilt of the accused persons arising from the evidence, or
from the lack of it, as opposed to mere vague apprehensions. A reasonable doubt is not an
imaginary, trivial or a merely possible doubt, but a fair doubt based upon reason and
common sense. It must grow out of the evidence in the case.
51

. The concepts of probability, and the degrees of it, cannot obviously be expressed in
terms of units to be mathematically enumerated as to how many of such units constitute
proof beyond reasonable doubt. There is an unmistakable subjective element in the
evaluation of the degrees of probability and the quantum of proof. Forensic probability
must, in the last analysis, rest on a robust common sense and, ultimately, on the trained
intuitions of the Judge. While the protection given by the criminal process to the accused
persons is not to be eroded, at the same time, uninformed legitimization of trivialities
would make a mockery of administration of criminal justice. This position was
illuminatingly stated by Venkatachaliah, J. (as His Lordship then was) in State of U.P. v.
Krishna Gopal (1988 (4) SCC 302). AIR 1988 SC 2154

52

. The above position was highlighted in Krishnan and Anr. v. State represented by
Inspector of Police (2003 (7) SCC 56). 2003 AIR SCW 3688
53. Considering the background facts we are of the view that the High Court has rightly
held that Gajanand was exercising the right of private defence. Therefore, there is no
merit in these appeals which are dismissed accordingly.
Appeal dismissed.

You might also like